Está en la página 1de 523

Machine Translated by Google

Machine Translated by Google

1600  John  F.  Kennedy  Blvd.
Ste  1800  
Filadelfia,  PA  19103–2899

LOS  SECRETOS  QUIRÚRGICOS  DE  ABERNATHY,  SEXTA  EDICIÓN ISBN:  978­0­323­05711­0

Copyright  Q  2009,  2005,  2003,  2000,  1996,  1991,  1986  de  Mosby,  Inc.,  una  filial  de  Elsevier  Inc.

Reservados  todos  los  derechos.  Ninguna  parte  de  esta  publicación  puede  ser  reproducida  o  transmitida  de  
ninguna  forma  o  por  ningún  medio,  electrónico  o  mecánico,  incluyendo  fotocopias,  grabaciones  o  cualquier  
sistema  de  almacenamiento  y  recuperación  de  información,  sin  el  permiso  por  escrito  del  editor.
Los  permisos  se  pueden  solicitar  directamente  al  Departamento  de  derechos  de  Elsevier:  teléfono:  (þ1)  215  
239  3804  (EE.  UU.)  o  (þ44)  1865  843830  (RU);  fax:  (þ44)  1865  853333;  correo  electrónico:  
healthpermissions@elsevier.com.  También  puede  completar  su  solicitud  en  línea  a  través  del  sitio  web  de  
Elsevier  en  http://www.elsevier.com/permissions.

AVISO

El  conocimiento  y  las  mejores  prácticas  en  este  campo  cambian  constantemente.  A  medida  que  las  nuevas  
investigaciones  y  la  experiencia  amplíen  nuestro  conocimiento,  pueden  ser  necesarios  o  apropiados  
cambios  en  la  práctica,  el  tratamiento  y  la  farmacoterapia.  Se  recomienda  a  los  lectores  que  consulten  la  
información  más  actualizada  proporcionada  (i)  sobre  los  procedimientos  presentados  o  (ii)  por  el  fabricante  
de  cada  producto  que  se  administrará,  para  verificar  la  dosis  o  fórmula  recomendada,  el  método  y  la  
duración  de  la  administración  y  las  contraindicaciones.  Es  responsabilidad  del  médico,  basándose  en  su  
propia  experiencia  y  conocimiento  del  paciente,  hacer  diagnósticos,  determinar  las  dosis  y  el  mejor  
tratamiento  para  cada  paciente  individual,  y  tomar  todas  las  precauciones  de  seguridad  apropiadas.  En  la  
máxima  medida  de  la  ley,  ni  el  editor  ni  el  editor  asumen  ninguna  responsabilidad  por  cualquier  lesión  y/o  
daño  a  personas  o  propiedad  que  surja  o  esté  relacionado  con  cualquier  uso  del  material  contenido  en  este  
libro.
El  editor

Datos  de  catalogación  en  publicación  de  la  Biblioteca  del  Congreso

Los  secretos  quirúrgicos  de  Abernathy.  –  6ª  ed. /  [editado  por]  Alden  H.  Harken,  Ernest  E.  Moore.
pag. ;  cm.  –  (La  serie  de  los  secretos)
Incluye  referencias  bibliográficas  e  indice.
ISBN  978­0­323­05711­0  1.  
Cirugía:  exámenes,  preguntas,  etc.  I.  Abernathy,  Charles.  II.
Harken,  Alden  H.  III.  Moore,  Ernest  Eugenio.  IV.  Título:  Secretos  quirúrgicos.
Serie  V.
[DNLM:  1.  Procedimientos  Quirúrgicos,  Preguntas  Operativas­Examen.  WO  18.2  A146  2009]

RD37.2.S975  2009  
617.0076–dc22 2008031054

Editor  de  adquisiciones:  Jim  Merritt
Editora  de  desarrollo:  Christine  Abshire
Gerente  de  Proyecto:  María  Stermel
Gerente  de  marketing:  Alyson  Sherby

Impreso  en  China
Machine Translated by Google

DEDICACIÓN

Charles  M.  Abernathy,  MD  
1941–1994

en
Machine Translated by Google

CONTRIBUYENTES

Brett  B.  Abernathy,  MD  Instructor  
clínico,  División  de  Urología,  Departamento  de  Cirugía,  Centro  de  Ciencias  de  la  Salud  de  la  Universidad  de  Colorado,  
Denver,  Colorado;  El  Centro  de  Urología  de  Colorado,  Denver,  Colorado

Jason  Q.  Alexander,  MD  Jefe,  
Cirugía  Vascular,  Centro  Médico  Kaiser  Oakland,  Oakland,  California;  Director  del  Sitio  de  Residencia  de  Cirugía  
General,  División  de  Terapia  Vascular  y  Endovascular,  Universidad  de  California,  San  Francisco–East  Bay,  Oakland,  
California

David  Altman,  MD,  MBA
Director  médico,  Centro  médico  del  condado  de  Alameda,  Oakland,  California

Nancy  C.  Andersen,  MD  
Residente,  Departamento  de  Cirugía  General,  Hospitales  de  la  Universidad  de  Carolina  del  Norte,  Chapel  Hill,  Carolina  del  Norte

Benjamin  O.  Anderson,  MD  Director,  
Centro  de  Salud  Mamaria,  Profesor,  Departamento  de  Cirugía,  Facultad  de  Medicina  de  la  Universidad  de  Washington,  
Seattle,  Washington

Ashok  N.  Babu,  MD  
Residente  de  Cirugía  General,  Departamento  de  Cirugía,  Universidad  de  Colorado,  Denver,  Colorado

Thomas  E.  Bak,  MD  
Departamento  de  Cirugía  de  Trasplante,  Universidad  de  Colorado  Denver,  Aurora,  Colorado;  Hospital  de  la  Universidad  
de  Colorado,  Denver,  Colorado

Carlton  C.  Barnett,  Jr.,  MD  Profesor  
asistente,  Departamento  de  Cirugía,  Centro  Médico  Southwestern  de  la  Universidad  de  Texas,  Dallas,  Texas

Dr.  Joel  Baumgartner
Residente  de  Cirugía,  Departamento  de  Cirugía,  Universidad  de  Colorado,  Denver,  Colorado

Bernard  Timothy  Baxter,  MD
Profesor,  Departamento  de  Cirugía,  Centro  Médico  de  la  Universidad  de  Nebraska,  Omaha,  Nebraska;  cirujano  de  plantilla,
Departamento  de  Cirugía,  Hospital  Metodista,  Omaha,  Nebraska

Dra.  Kathryn  Beauchamp
Profesor  Asistente,  Departamento  de  Neurocirugía,  Universidad  de  Colorado  en  Denver,  Denver,  Colorado;
Neurocirujano,  Departamento  de  Neurocirugía,  Denver  Health  Medical  Center,  Denver,  Colorado

Allen  T.  Belshaw,  MD  Profesor  
Clínico  Asistente,  Departamento  de  Cirugía,  Centro  de  Ciencias  de  la  Salud  de  la  Universidad  de  Colorado,  Denver,  
Colorado;  Cirujano  general,  Yampa  Valley  Medical  Center,  Steamboat  Springs,  Colorado

Denis  D.  Bensard,  MD  Profesor  
de  Cirugía,  Departamento  de  Cirugía  Pediátrica,  Universidad  de  Cincinnati  y  Centro  Médico  del  Hospital  Infantil  de  
Cincinnati,  Cincinnati,  Ohio;  Director,  Departamento  de  Cirugía  Pediátrica,  The  Peyton  Manning  Children's  Hospital  at  St.  
Vincent,  Indianápolis,  Indiana

XV
Machine Translated by Google
xvi  COLABORADORES

Walter  L.  Biffl,  MD,  FACS  Profesor  
Asistente  de  Cirugía,  Departamento  de  Cirugía,  Facultad  de  Medicina  de  la  Universidad  de  Colorado  en  Denver,  Denver,  
Colorado;  Cirujano  de  cuidados  intensivos,  Subdirector  de  Calidad  y  Seguridad  del  Paciente,  Departamento  de  Cirugía,  Denver  
Health  Medical  Center,  Denver,  Colorado

Natasha  D.  Bir,  MD,  Residente  de  
MHS ,  Departamento  de  Cirugía,  Universidad  de  California,  San  Francisco–East  Bay,  Oakland,  California  Elizabeth  

C.  Brew,  MD  Práctica  privada,  Cirugía  general,  Wheat  Ridge,  Colorado

Laurence  H.  Brinckerhoff,  MD  Jefe,  
Cirugía  Torácica  General,  Profesor  Asistente,  Departamento  de  Cirugía,  Centro  Médico  Tufts,  Boston,  Massachusetts

Jamie  M.  Brown,  MD  Profesor  
Asociado  de  Cirugía,  Departamento  de  Cirugía,  Universidad  de  Maryland,  Baltimore,  Maryland;  Profesor  Asociado  de  Cirugía,  
Departamento  de  Cirugía,  Centro  Médico  de  la  Universidad  de  Maryland,  Baltimore,  Maryland

Mark  P.  Cain,  MD  
Profesor,  Departamento  de  Urología,  Universidad  de  Indiana,  Indianápolis,  Indiana;  Riley  Hospital  for  Children,  
Indianápolis,  Indiana

Kristine  E.  Calhoun,  MD  Profesora  
Asistente,  Departamento  de  Cirugía,  Facultad  de  Medicina  de  la  Universidad  de  Washington,  Seattle,  Washington

Brian  P.  Callahan,  MD  Jefe  de  
residentes,  Departamento  de  Neurocirugía,  Centro  de  Ciencias  de  la  Salud  de  la  Universidad  de  Colorado,  Denver,  Colorado  

Jeffrey  Campsen,  MD  Becario  de  trasplante  quirúrgico,  Departamento  de  Trasplantes,  Centro  de  Ciencias  de  la  Salud  de  la  
Universidad  de  Colorado,  Aurora,  Colorado

Anne  Cannon,  RN,  BSN
Coordinador  de  dispositivos  de  asistencia  ventricular,  Departamento  de  Cirugía  Cardiotorácica,  Hospital  de  la  Universidad  de  Colorado,
Aurora,  Colorado  

Mario  F.  Chammas,  Jr.,  MD  Urólogo,  
Departamento  de  Urología,  Universidad  de  Colorado  Denver,  Aurora,  Colorado;  Urólogo,  Departamento  de  Urología,  
Centro  Médico  de  Salud  de  Denver,  Denver,  Colorado  David  J.  Ciesla,  MD,  MS  Profesor  Asociado,  Departamento  de  

Cirugía,  Universidad  del  Sur  de  Florida,  Tampa,  Florida;  Director  de  Trauma  y  Cuidados  Intensivos  Quirúrgicos,  
Departamento  de  Cirugía,  Hospital  General  de  Tampa,  Tampa,  Florida

Joseph  C.  Cleveland,  Jr.,  MD  Profesor  
Asociado  de  Cirugía,  Director  Quirúrgico,  Trasplante  Cardiaco  y  MCS,  Departamento  de  Cirugía,  Universidad  de  Colorado  en  
Denver,  Aurora,  Colorado;  Profesor  Asociado,  Departamento  de  Cirugía,  Hospital  de  la  Universidad  de  Colorado,  Aurora,  
Colorado;  Jefe,  Cirugía  CT,  Departamento  de  Cirugía,  Centro  Médico  de  Asuntos  de  Veteranos  de  Denver,  Denver,  Colorado

C.  Clay  Cothren,  MD  
Profesor  Asistente,  Departamento  de  Cirugía,  Facultad  de  Medicina  de  la  Universidad  de  Colorado,  Denver,  Colorado;  
Director  del  programa,  Cuidados  intensivos  quirúrgicos  y  becas  TACS,  Departamento  de  cirugía,  Denver  Health  Medical  
Center,  Denver,  Colorado

Paul  R.  Crisostomo,  MD  Research  
Fellow,  Departamento  de  Cirugía,  Universidad  de  Indiana,  Indianápolis,  Indiana;  Residente  en  Cirugía,  Departamento  de  
Cirugía,  Universidad  de  Indiana,  Indianápolis,  Indiana
Machine Translated by Google
COLABORADORES  xvii

Elizabeth  L.  Cureton,  MD  Residente  
en  Cirugía  General,  Departamento  de  Cirugía,  Universidad  de  California,  San  Francisco–East  Bay,  Centro  Médico  del  
Condado  de  Alameda,  Oakland,  California

Laura  DiMatteo,  MD
Residente  de  Cirugía  Ortopédica,  Departamento  de  Ortopedia,  Centro  de  Ciencias  de  la  Salud  de  la  Universidad  de  Colorado,
Denver,  Colorado

Alexander  Q.  Ereso,  MD  Jefe  de  
residentes,  Departamento  de  Cirugía,  Universidad  de  California,  San  Francisco–East  Bay,  Centro  Médico  del  Condado  de  
Alameda,  Oakland,  California

Michael  E.  Fenoglio,  MD  Cirujano  
general,  Departamento  de  Cirugía  General,  Hospital  Presbiteriano  St.  Luke's,  Denver,  Colorado;  Cirujano  General,  
Departamento  de  Cirugía  General,  Hospital  St.  Joseph,  Denver,  Colorado;  Cirujano  general,  Departamento  de  Cirugía  
General,  Centro  de  Ciencias  de  la  Salud  de  la  Universidad  de  Colorado,  Denver,  Colorado

Christina  A.  Finlayson,  MD  Profesora  
adjunta,  Departamento  de  Cirugía,  Universidad  de  Colorado  Denver,  Aurora,  Colorado;  Directora,  Diane  O'Connor  
Thompson  Breast  Center,  Hospital  de  la  Universidad  de  Colorado,  Aurora,  Colorado

David  A.  Fullerton,  MD  Profesor  
y  Jefe  de  División,  Departamento  de  Cirugía  Cardiotorácica,  Universidad  de  Colorado  Denver,  Aurora,  Colorado;  
Cirujano  de  la  Facultad,  Departamento  de  Cirugía,  División  de  Cirugía  Cardiotorácica,  Hospital  Universitario,  Aurora,  
Colorado;  Cirujano  de  la  Facultad,  Departamento  de  Cirugía  Cardiotorácica,  The  Children's  Hospital,  Aurora,  Colorado;  
Cirujano  docente,  Departamento  de  Cirugía  Cardiotorácica,  Centro  Médico  de  la  Administración  de  Veteranos,  Denver,  
Colorado

Glenn  W.  Geelhoed,  MD,  MPH,  MA,  DTMH,  ScD  (Hon),  MA,  MPhil,  EdD,  FACS  Profesor  de  Cirugía,  Educación  
Médica  Internacional,  Microbiología,  Inmunología  y  Medicina  Tropical,  Departamentos  de  Cirugía  y  Microbiología,  
Inmunología  y  Medicina  Tropical,  Oficina  del  Decano,  Centro  Médico  de  la  Universidad  George  Washington,  Washington,  
DC;  Distinguido  Profesor  Global  de  Medicina  Internacional,  Centro  para  el  Aprendizaje  Creativo,  Centro  de  Ciencias  Médicas  
de  la  Universidad  de  Toledo,  Toledo,  Ohio;  Profesor  Distinguido  de  Obstetricia  y  Ginecología,  Departamento  de  Obstetricia  y  
Ginecología  y  Medicina  Materno  Fetal,  Universidad  Estatal  de  Nueva  York  Upstate,  Syracuse,  Nueva  York

Ricardo  J.  Gonzalez,  MD  Profesor  
Asistente  de  Cirugía,  Departamento  de  Cirugía,  Universidad  de  Colorado,  Aurora,  Colorado

Raffi  Gurunluoglu,  MD,  PhD
Profesor  Asociado,  Departamento  de  Cirugía  Plástica,  Centro  de  Ciencias  de  la  Salud  de  la  Universidad  de  Colorado,  Denver,
Colorado;  Jefe  y  Profesor  Asociado,  Departamento  de  Cirugía  Plástica,  Denver  Health  Medical  Center,  Denver,
Colorado

Richard­Tien  V.  Ha,  MD  
Residente  de  Cirugía,  Departamento  de  Cirugía,  Universidad  de  California,  San  Francisco–East  Bay,  Oakland,  California

James  B.  Haenel,  Especialista  
en  Cuidados  Intensivos  Quirúrgicos  RRT,  Departamento  de  Cirugía,  Denver  Health  Medical  Center,  Denver,  Colorado

Alden  H.  Harken,  MD  Profesor  
y  Presidente  del  Departamento  de  Cirugía  de  la  Universidad  de  California,  San  Francisco–East  Bay,  Oakland,  California;  Jefe  
de  Cirugía,  Departamento  de  Cirugía,  Centro  Médico  del  Condado  de  Alameda,  Oakland,  California

Tabetha  R.  Harken,  MD,  MPH  Fellow  en  
Obstetricia  y  Ginecología,  Universidad  de  California  en  San  Francisco,  San  Francisco,  California
Machine Translated by Google
xviii  COLABORADORES

Richard  J.  Hendrickson,  MD  Cirujano  
pediátrico,  Indianápolis,  Indiana

Laurel  R.  Imhoff,  MD,  MPH  Residente  
de  cirugía,  Departamento  de  cirugía,  Universidad  de  California,  San  Francisco–East  Bay,  Centro  médico  del  condado  de  
Alameda,  Oakland,  California

Dr.  Ramin  Jamshidi
Residente,  Departamento  de  Cirugía,  Universidad  de  California,  San  Francisco,  San  Francisco,  California

Jeffrey  L.  Johnson,  MD  Director,  
SICU,  Departamento  de  Cirugía,  Denver  Health  Medical  Center,  Denver,  Colorado;  Profesor  Asistente  de  Cirugía,  Facultad  de  
Medicina  de  la  Universidad  de  Colorado  en  Denver,  Aurora,  Colorado

Darrell  N.  Jones,  PhD  
Administrador,  Cirugía  Vascular,  Departamento  de  Cirugía,  Universidad  de  Colorado  Denver,  Aurora,  Colorado;  Director,  
Departamento  de  Diagnóstico  Vascular,  Hospital  de  la  Universidad  de  Colorado,  Aurora,  Colorado

Janeen  R.  Jordan,  MD  
Residente,  Departamento  de  Cirugía,  Programa  de  Ciencias  de  la  Salud  de  la  Universidad  de  Colorado  Denver,  Denver,  
Colorado;  Residente,  Departamento  de  Cirugía,  Centro  Médico  de  Salud  de  Denver,  Denver,  Colorado;  Departamento  de  
Cirugía,  Hospital  de  la  Universidad  de  Colorado,  Denver,  Colorado

Dra.  Sarah  Judkins
Residente  de  Cirugía  General,  Departamento  de  Cirugía,  Centro  de  Ciencias  de  la  Salud  de  la  Universidad  de  Colorado,  Denver,
Colorado

Frederick  M.  Karrer,  MD  Profesor  
de  Cirugía  y  Pediatría,  Facultad  de  Medicina  de  la  Universidad  de  Colorado  en  Denver;  Profesor,  Departamento  de  Cirugía,  The  
Children's  Hospital,  Denver,  Colorado

Jeffry  L.  Kashuk,  MD,  Profesor  Asistente  
de  Cirugía  de  FACS,  Departamento  de  Cirugía  Trauma,  Denver  Health  Rocky  Mountain  Trauma  Center,  Denver,  Colorado;  
Profesor  Asistente,  Departamento  de  Cirugía,  Centro  de  Ciencias  de  la  Salud  de  la  Universidad  de  Colorado,  Denver,  Colorado

Jarrod  N.  Keith,  MD  
Residente  de  Cirugía  General,  Departamento  de  Cirugía,  Centro  de  Ciencias  de  la  Salud  de  la  Universidad  de  Colorado,  
Denver,  Colorado

Fernando  J.  Kim,  MD,  Profesor  Asociado  
de  FACS,  Director  de  Oncología  Urológica  Mínimamente  Invasiva,  Centro  de  Ciencias  de  la  Salud  de  la  Universidad  de  Colorado,  
Centro  de  Cáncer  Tony  Grampsas,  Denver,  Colorado;  Jefe  de  Urología,  Departamento  de  Cirugía,  Centro  Médico  de  Denver,  
Denver,  Colorado

G.  Edward  Kimm,  Jr.,  MD  Profesor  
Clínico  Asistente,  Departamento  de  Cirugía,  Centro  de  Ciencias  de  la  Salud  de  la  Universidad  de  Colorado,  Denver,  
Colorado;  Cirujano  adjunto,  Departamento  de  Cirugía,  Denver  Health  Medical  Center,  Denver,  Colorado

Dra.  Ann  Marie  Kulungowski
Residente  de  Cirugía  General,  Departamento  de  Cirugía  General,  Hospital  de  la  Universidad  de  Colorado,  Aurora,  Colorado

Adam  H.  Lackey,  MD  
Residente,  Departamento  de  Cirugía,  Centro  de  Ciencias  de  la  Salud  de  la  Universidad  de  Colorado,  Denver,  Colorado

Michael  L.  Lepore,  MD,  Profesor  FACS ,  
Departamento  de  Otorrinolaringología–Cirugía  de  Cabeza  y  Cuello,  Facultad  de  Medicina  de  la  Universidad  de  Colorado,  
Denver,  Colorado;  Director  de  Otorrinolaringología:  Cirugía  de  Cabeza  y  Cuello,  Departamento  de  Cirugía,  Denver  Health  Medical
Machine Translated by Google
COLABORADORES  xix

Centro,  Denver,  Colorado;  Profesor,  Departamento  de  Cirugía,  Departamento  de  Asuntos  de  Veteranos,  Denver,  Colorado;
Profesor,  Departamento  de  Otorrinolaringología–Cirugía  de  Cabeza  y  Cuello,  Hospital  Universitario,  Denver,  Colorado

Kathleen  R.  Liscum,  MD  Profesora  
adjunta,  Departamento  de  Cirugía,  Facultad  de  Medicina  de  Baylor,  Houston,  Texas;  Jefe  de  Cirugía  General,  Departamento  de  
Cirugía,  Hospital  General  Ben  Taub,  Houston,  Texas

Andrew  E.  Luckey,  MD  Práctica  
privada,  cirugía  general  y  laparoscópica,  Los  Ángeles,  California

Joyce  A.  Majure,  MD  
Departamento  de  Cirugía,  Centro  Médico  Regional  St.  Joseph,  Lewiston,  Idaho

Martin  D.  McCarter,  MD  Profesor  
asociado,  División  de  Tumores  GI  y  Cirugía  Endocrina,  Facultad  de  Medicina  de  la  Universidad  de  Colorado  en  Denver,  
Denver,  Colorado;  Hospital  de  la  Universidad  de  Colorado,  Aurora,  Colorado

Robert  C.  McIntyre,  Jr.,  MD  Profesor  
asociado,  Departamento  de  Cirugía,  Facultad  de  Medicina  de  la  Universidad  de  Colorado  en  Denver,  Aurora,  Colorado

Nadia  McMillan
Estudiante,  Universidad  Johns  Hopkins,  Baltimore,  Maryland

Margaret  M.  McQuiggan,  MS,  RD,  CNSD  Especialista  en  
investigación  clínica,  Departamento  de  cirugía,  Instituto  de  investigación  del  Hospital  Metodista,  Houston,  Texas

Randall  B.  Meacham,  MD  Profesor,  
Jefe  de  División,  Director  del  Programa  de  Residencia,  División  de  Urología,  Facultad  de  Medicina  de  la  Universidad  de  Colorado  
en  Denver,  Aurora,  Colorado;  Director  de  Práctica,  Departamento  de  Urología,  Hospital  de  la  Universidad  de  Colorado,  Aurora,  
Colorado;  Personal,  Departamento  de  Cirugía/Urología,  Centro  Médico  de  la  Administración  de  Veteranos,  Denver,  Colorado;  
Personal,  Departamento  de  Urología,  Centro  Médico  de  Denver,  Denver,  Colorado

Daniel  R.  Meldrum,  MD,  FACS  Director  de  
Investigación,  Profesor  Asociado,  Departamento  de  Cirugía  Cardiotorácica,  Universidad  de  Indiana,  Indianápolis,  Indiana;  Cirujano  
cardiotorácico  del  personal,  Departamento  de  Cirugía  Cardiotorácica,  Centro  Médico  VA,  Indianápolis,  Indiana;  Cirujano  
cardiotorácico  del  personal,  Departamento  de  Cirugía  Cardiotorácica,  Centro  Médico  de  la  Universidad  de  Indiana,  Indianápolis,  
Indiana

Kirstan  K.  Meldrum,  MD  Profesora  
adjunta,  Departamento  de  Urología  Pediátrica,  Escuela  de  Medicina  de  la  Universidad  de  Indiana,  Indianápolis,  Indiana

Ryan  P.  Merkow,  MD  
Departamento  de  Cirugía,  Facultad  de  Medicina  de  la  Universidad  de  Colorado  en  Denver,  Denver,  Colorado;  Departamento  de  
Cirugía,  Hospital  de  la  Universidad  de  Colorado,  Denver,  Colorado

Ernest  E.  Moore,  MD  Profesor  
y  Vicepresidente,  Departamento  de  Cirugía,  Centro  de  Ciencias  de  la  Salud  de  la  Universidad  de  Colorado,  Denver,  Colorado;  
Jefe  de  Servicios  de  Cirugía  y  Trauma,  Departamento  de  Cirugía,  Denver  Health  Medical  Center,  Denver,  Colorado

Frederick  A.  Moore,  MD,  Profesor  FACS,  
Departamento  de  Cirugía,  Weill  Cornell  Medical  College,  Nueva  York,  Nueva  York;  Jefe,  División  de  Cuidados  Intensivos  
Quirúrgicos  y  Cirugía  de  Cuidados  Intensivos,  Departamento  de  Cirugía,  The  Methodist  Hospital,  Houston,  Texas

Steven  J.  Morgan,  MD,  Profesor  Asociado  
de  FACS,  Departamento  de  Ortopedia,  Facultad  de  Medicina  de  la  Universidad  de  Colorado,  Denver,  Colorado;  Director  asociado,  
Departamento  de  ortopedia,  Denver  Health  Medical  Center,  Denver,  Colorado
Machine Translated by Google
xx  COLABORADORES

Mark  R.  Nehler,  MD  Director  
de  Programa,  Departamento  de  Cirugía,  Centro  de  Ciencias  de  la  Salud  de  la  Universidad  de  Colorado,  Denver,  Colorado;  
Profesor  Asociado  de  Cirugía,  Departamento  de  Cirugía,  Denver  VA  Medical  Center,  Denver,  Colorado

Tony  T.  Nguyen,  DO  Jefe  de  
Cirugía  Residente,  Departamento  de  Cirugía,  Universidad  de  California,  Programa  de  Residencia  Quirúrgica  de  San  
Francisco–East  Bay,  Oakland,  California

Lawrence  W.  Norton,  MD  Profesor  
emérito,  Departamento  de  Cirugía,  Facultad  de  Medicina  de  la  Universidad  de  Colorado,  Aurora,  Colorado

Trevor  L.  Nydam,  MD  Residente,  
Departamento  de  Cirugía,  Facultad  de  Medicina  de  la  Universidad  de  Colorado  en  Denver,  Aurora,  Colorado

Dr.  Kagan  Ozer
Profesor  Asociado,  Departamento  de  Cirugía  Ortopédica,  Universidad  de  Colorado,  Denver,  Colorado;  jefe  de  mano
Cirugía,  Departamento  de  Cirugía  Ortopédica,  Denver  Health  Medical  Center,  Denver,  Colorado

Cyrus  J.  Parsa,  MD  Jefe  de  
residentes,  Departamento  de  Cirugía  Torácica,  Universidad  de  Duke,  Durham,  Carolina  del  Norte

David  A.  Partrick,  MD  Profesor  
asociado,  Departamento  de  Cirugía  Pediátrica,  Universidad  de  Colorado,  Denver,  Colorado;  Jefe  de  Cirugía  Pediátrica,  Centro  
Médico  de  Salud  de  Denver,  Denver,  Colorado;  Profesor  asociado,  Pediatría,  The  Children's  Hospital,  Denver,  Colorado

Nathan  W.  Pearlman,  MD  Profesor,  
Departamento  de  Cirugía,  Centro  de  Ciencias  de  la  Salud  de  la  Universidad  de  Colorado,  Denver,  Colorado;  Cirujano  adjunto,  
Departamento  de  Cirugía,  Denver  VA  Medical  Center,  Denver,  Colorado

Erik  D.  Peltz,  DO  
Residente  Quirúrgico,  Departamento  de  Cirugía,  Universidad  de  Colorado  en  Denver,  Denver,  Colorado;  Residente  quirúrgico,  
becario  de  investigación  quirúrgica,  Departamento  de  Cirugía/Centro  de  Investigación  de  Trauma,  Centro  Médico  de  Salud  de  
Denver,  Denver,  Colorado

Steven  L.  Peterson,  DVM,  MD  Profesor  
asociado,  Departamento  de  Cirugía,  Universidad  de  Ciencias  de  la  Salud  de  Oregón,  Portland,  Oregón;  Servicio  de  Cirugía  Plástica  
y  de  Mano,  División  de  Cirugía  Plástica,  Departamento  de  Cirugía,  Administración  de  Veteranos  de  Portland,  Portland,  Oregón

Marvin  Pomerantz,  MD
Profesor  de  Cirugía  y  Director  del  Centro  para  el  Tratamiento  Quirúrgico  de  Infecciones  Pulmonares,  Departamento  de
Cirugía,  División  de  Cirugía  Cardiotorácica,  Universidad  de  Colorado  Denver,  Aurora,  Colorado;  Profesor  de
Cirugía,  Departamento  de  Cirugía,  Hospital  de  la  Universidad  de  Colorado,  Aurora,  Colorado

Craig  H.  Rabb,  MD  Profesor  
asociado,  Departamento  de  Neurocirugía,  Facultad  de  Medicina  de  la  Universidad  de  Colorado,  Denver,  Colorado;  Jefe,  
Neurocirugía,  Departamento  de  Neurocirugía,  Denver  Health  Medical  Center,  Denver,  Colorado

Christopher  D.  Raeburn,  MD  Profesor  
asistente,  Departamento  de  Cirugía,  Facultad  de  Medicina  de  la  Universidad  de  Colorado  en  Denver,  Aurora,  Colorado

T.  Brett  Reece,  MD  
Residente,  PGY  IX,  División  de  Cirugía  Cardiotorácica,  Universidad  de  Colorado  Denver,  Aurora,  Colorado;  Residente,  PGY  IX,  
Departamento  de  Cirugía,  División  de  Cirugía  Cardiotorácica,  Hospital  Universitario,  Aurora,  Colorado
Machine Translated by Google
COLABORADORES  xxi

Thomas  F.  Rehring,  MD,  Profesor  Clínico  
Asociado  de  Cirugía  de  la  FACS,  Sección  de  Cirugía  Vascular,  Centro  de  Ciencias  de  la  Salud  de  la  Universidad  de  Colorado  
en  Denver,  Denver,  Colorado;  Director,  Departamento  de  Terapia  Vascular,  Jefe,  Cirugía  Vascular  y  Endovascular,  Grupo  
Médico  Permanente  de  Colorado,  Denver,  Colorado

John  A.  Ridge,  MD,  PhD  Jefe,  
Sección  de  Cirugía  de  Cabeza  y  Cuello,  Departamento  de  Oncología  Quirúrgica,  Centro  Oncológico  Fox  Chase,  Filadelfia,  
Pensilvania

Jonathan  P.  Roach,  MD  Residente,  
Departamento  de  Cirugía,  Universidad  de  Colorado  Denver,  Denver,  Colorado

Thomas  N.  Robinson,  MD  Profesor  
Asistente,  Departamento  de  Cirugía,  Universidad  de  Colorado  en  el  Centro  de  Ciencias  de  la  Salud  de  Denver,  Aurora,  
Colorado;  Departamento  de  Cirugía,  Hospital  de  la  Universidad  de  Colorado,  Aurora,  Colorado

Christina  L.  Roland,  MD  
Investigadora,  Departamento  de  Cirugía,  Centro  Médico  Southwestern  de  la  Universidad  de  Texas,  Dallas,  Texas

Carlos  A.  Rueda,  MD  
Residente  de  Cirugía  General,  Departamento  de  Cirugía,  Universidad  de  Colorado  Denver,  Denver,  Colorado;  Residente  
de  Cirugía  General,  Departamento  de  Cirugía,  Hospital  de  Denver  de  la  Universidad  de  Colorado,  Denver,  Colorado;  
Residente  de  Cirugía  General,  Departamento  de  Cirugía,  Denver  Health  Medical  Center,  Denver,  Colorado

Craig  H.  Selzman,  MD  Profesor  
asociado,  Departamento  de  Cirugía,  División  de  Cirugía  Cardiotorácica,  Facultad  de  Medicina  de  la  Universidad  de  Utah,  
Salt  Lake  City,  Utah

Amandeep  Singh,  MD
Profesor  Clínico  Asistente  de  Medicina,  División  de  Medicina  de  Emergencia,  Departamento  de  Medicina,  Universidad  de
California,  San  Francisco,  San  Francisco,  California;  Médico  de  cabecera,  Departamento  de  Medicina  de  Emergencia,
Centro  médico  del  condado  de  Alameda–Hospital  general  de  Highland,  Oakland,  California

Wade  R.  Smith,  MD  Profesor,  
Departamento  de  Ortopedia,  Facultad  de  Medicina  de  la  Universidad  de  Colorado,  Aurora,  Colorado;  Director,  Departamento  
de  Ortopedia,  Centro  Médico  de  Salud  de  Denver,  Denver,  Colorado;  Departamento  de  Ortopedia,  Centro  Médico  de  Asuntos  
de  Veteranos,  Denver,  Colorado

David  E.  Stein,  MD  
Profesor  Asistente,  Departamento  de  Cirugía,  Facultad  de  Medicina  de  la  Universidad  de  Drexel,  Filadelfia,  Pensilvania;  Jefe,  
División  de  Cirugía  Colorrectal;  Departamento  de  Cirugía,  Hospital  Universitario  Hahnemann,  Filadelfia,  Pensilvania

Gregory  V.  Stiegmann,  MD  Profesor  
de  Cirugía,  Departamento  de  Cirugía,  Facultad  de  Medicina  de  la  Universidad  de  Colorado  en  Denver,  Denver,  
Colorado;  Vicepresidente  de  Asuntos  Clínicos,  Hospital  de  la  Universidad  de  Colorado,  Aurora,  Colorado;  Cirujano  de  
planta,  Departamento  de  Cirugía,  Hospital  de  Asuntos  de  Veteranos  de  Denver,  Denver,  Colorado

Karyn  Stitzenberg,  MD,  MPH
Oncología  quirúrgica,  Fox  Chase  Cancer  Center,  Filadelfia,  Pensilvania

U.  Mini  B.  Swift,  MD  Asesor  
médico,  Centro  médico  del  condado  de  Alameda,  Oakland,  California

Alex  J.  Vanni,  MD  
Residente,  Departamento  de  Urología,  Clínica  Lahey,  Burlington,  Massachusetts
Machine Translated by Google
xxii  COLABORADORES

Gregory  P.  Victorino,  MD  Profesor  
asociado,  Departamento  de  Cirugía,  Universidad  de  California,  San  Francisco–East  Bay,  Centro  Médico  del  
Condado  de  Alameda,  Oakland,  California;  Jefe,  División  de  Trauma,  Departamento  de  Cirugía,  Centro  Médico  
del  Condado  de  Alameda,  Oakland,  California

Thomas  A.  Whitehill,  MD  Profesor  
asociado,  Departamento  de  Cirugía,  Universidad  de  Colorado,  Denver,  Colorado;  Jefe  de  Servicios  Quirúrgicos,  
Centro  Médico  VA,  Denver,  Colorado

Jennifer  M.  Worth,  MD  
Residente  de  Cirugía  General,  Departamento  de  Cirugía  General,  Centro  Médico  de  la  Universidad  de  Nebraska,  
Omaha,  Nebraska

Franklin  L.  Wright,  MD  
Residente,  Departamento  de  Cirugía,  Universidad  de  Colorado  Denver,  Denver,  Colorado

Dr.  Michael  Zimmerman
Profesor  Asistente,  División  de  Cirugía  de  Trasplante,  Centro  de  Ciencias  de  la  Salud  de  la  Universidad  de  Colorado,  Aurora,
Colorado
Machine Translated by Google

PREFACIO

Cuando  nos  referimos  a  una  obra  de  arte,  música  o  literatura  como  un  "clásico",  una  de  las  observaciones  que
lo  que  hacemos  es  que  la  obra  ha  estimulado  una  amplia  variedad  de  tratamientos  e  interpretaciones.
La  imitación  es,  por  supuesto,  la  forma  más  visible  y  creíble  de  adulación.  Cuando  Charlie  Abernathy  inicialmente  
asaltó  nuestra  zona  de  comodidad  clínica  quirúrgica  con  un  aluvión  de  preguntas,  ni  él  ni  nosotros  predijimos  que  
sus  irritantes  esfuerzos  generarían  toda  una  "Serie  de  secretos"  de  desafiantes  abernatismos  en  casi  todas  las  
disciplinas  médicas.
Pero,  característicamente,  Charlie  tenía  sus  dedos  hábilmente  colocados  en  el  pulso  del  progreso.  Casey
Stengel  señaló:  "En  el  béisbol,  se  pierden  más  juegos  de  los  que  se  ganan".  Si  no  investigas,  aprendes  o  
cuestionas,  estás  perdiendo.  En  medicina,  y  ciertamente  en  cirugía,  no  puedes  quedarte  quieto.  Alfred  North  
Whitehead,  el  filósofo  estadounidense,  observó:  ''Ningún  hombre  de  ciencia  podría  suscribir  sin  calificación...  todas  
sus  propias  creencias  científicas  de  hace  diez  años''.  Debemos  ser  flexibles,  evolucionar,  cuestionar.  Afortunadamente,  
los  cirujanos  son  casi  únicos  en  nuestra  capacidad  de  autocrítica.  Nunca  debemos  marchar,  como  una  legión  de  
lemmings,  hacia  un  mar  de  aceptación  intelectual.
Esta  sexta  edición  de  está  
Quirúrgico Misterios
nuevamente  dedicada  a  la  irritantemente  penetrante  serie  de  preguntas  de  Abernathy.  
Charlie  nunca  le  dio  mucha  importancia  a  la  respuesta  pesadamente  tradicional.
Los  cirujanos  intelectualmente  activos  nunca  deben  sentirse  demasiado  cómodos.  Desafiar  el  dogma  es  bueno;  la  
comodidad  es  mala.  Los  dinosaurios  eran  inflexibles  y  están  extintos.  Los  cirujanos  nunca  lo  serán  tampoco.

Alden  H.  Harken,  MD  Ernest  
E.  Moore,  MD  Abril  de  2008

XXIII
Machine Translated by Google

LOS  100  PRINCIPALES  SECRETOS

Andrew  E.  Luckey,  MD  y  Cyrus  J.  Parsa,  MD

Estos  secretos  son  100  de  las  principales  alertas  del  tablero.  Resume  los  conceptos,  principios  y  detalles  más  
destacados  de  la  práctica  quirúrgica.

1.  El  objetivo  principal  en  el  tratamiento  de  las  arritmias  cardíacas  es  lograr  una  frecuencia  ventricular  entre  60  y
100  latidos  por  minuto;  objetivo  secundario  es  mantener  el  ritmo  sinusal.

2.  Los  determinantes  clínicos  de  muerte  encefálica  son  la  pérdida  de  los  tejidos  papilar,  corneal,  oculovestibular,
Reflejos  oculocefálicos,  orofaríngeos  y  respiratorios  durante  más  de  6  horas.  El  paciente  también  debe  someterse  
a  una  prueba  de  apnea,  en  la  que  se  permite  que  la  PCO2  aumente  hasta  al  menos  60  mmHg  sin  hipoxia  
coexistente.  El  paciente  debe  ser  observado  por  la  ausencia  de  respiración  espontánea.

3.  Los  riesgos  estimados  del  virus  de  la  hepatitis  B  (VHB),  el  virus  de  la  hepatitis  C  (VHC)  y
la  transmisión  del  virus  de  la  inmunodeficiencia  humana  (VIH)  por  transfusión  de  sangre  en  los  Estados  Unidos  es  
de  1  en  205  000  para  el  VHB;  1  en  1.935.000  para  VHC;  y  1  en  2.135.000  para  el  VIH.

4.  La  ubicación  más  común  de  un  testículo  no  descendido  es  el  canal  inguinal.

5.  La  masa  renal  sólida  más  común  en  la  infancia  es  un  nefroma  mesoblástico  congénito,  y  en
infancia,  es  un  tumor  de  Wilms.

6.  El  síndrome  de  Ogilvie  es  una  dilatación  masiva  aguda  del  ciego  y  de  las  vías  ascendente  y  descendente
colon  transverso  sin  obstrucción  orgánica.

7.  El  mejor  método  de  detección  para  el  cáncer  de  próstata  es  el  examen  rectal  digital  combinado  con  antígeno  
prostático  específico  (PSA)  en  suero.

8.  El  tipo  histológico  más  común  de  cáncer  de  vejiga  es  el  carcinoma  de  células  de  transición.

9.  El  carcinoma  in  situ  de  vejiga  se  trata  con  inmunoterapia  con  bacilo  intravesical
Calmette­Guérin.

10.  La  causa  más  común  de  infertilidad  masculina  es  el  varicocele.

11.  La  causa  no  bacteriana  más  común  de  neumonía  en  pacientes  trasplantados  es
citomegalovirus  (CMV).

12.  El  quimerismo  es  el  intercambio  de  leucocitos  entre  el  injerto  y  el  receptor  para  que  el  injerto  se  convierta  en
un  compuesto  genético  tanto  del  donante  como  del  receptor.

13.  OKT3  es  un  anticuerpo  monoclonal  de  ratón  que  se  une  y  bloquea  el  receptor  CD3  de  células  T.

14.  La  enfermedad  más  común  que  requiere  trasplante  de  hígado  es  la  hepatitis  C.

1
Machine Translated by Google
2  100  PRINCIPALES  SECRETOS

15.  El  higroma  quístico  es  una  malformación  congénita  con  predilección  por  el  cuello.  es  un  benigno
lesión  que  suele  presentarse  como  una  masa  blanda  en  la  parte  lateral  del  cuello.

16.  En  los  neuroblastomas,  la  edad  de  presentación  es  el  principal  factor  pronóstico.  Los  niños  menores  de  1  año  tienen  una  tasa  
de  supervivencia  general  >70  %,  mientras  que  la  tasa  de  supervivencia  de  los  niños  mayores  de  1  año  es  <35  %.

17.  La  complicación  más  temida  de  la  hernia  diafragmática  es  la  circulación  fetal  persistente.

18.  Las  tres  variantes  más  comunes  de  fístula  traqueoesofágica  son  (1)  atresia  esofágica  proximal  con  fístula  traqueoesofágica  
distal,  (2)  atresia  esofágica  aislada  y  (3)  fístula  traqueoesofágica  con  atresia  esofágica.

19.  La  atresia  puede  ocurrir  en  cualquier  parte  del  tracto  gastrointestinal  (GI):  duodenal  (50  %),  yeyunoileal  (45  %)  o  colónico  (5  %).  
La  atresia  duodenal  surge  por  falla  en  la  recanalización  durante  la  octava  a  décima  semana  de  gestación;  Las  atresias  
yeyunoileal  y  colónica  son  causadas  por  un  accidente  vascular  mesentérico  intrauterino.

20.  Los  dos  tipos  de  disección  aórtica  son  la  disección  ascendente  (tipo  A),  que  comienza  en  la  aorta  ascendente  y  puede  
continuar  en  la  aorta  descendente,  y  la  disección  descendente  (tipo  B),  que  afecta  solo  a  la  aorta  descendente.

21.  La  probabilidad  de  que  un  nódulo  pulmonar  solitario  sea  cáncer  es  la  misma  que  la  edad  del  paciente;  de  este  modo,
el  nódulo  de  un  paciente  de  60  años  tiene  un  60%  de  probabilidades  de  ser  cáncer.

22.  La  estadificación  mediastínica  (mediastinoscopia)  está  indicada  si:  (1)  el  nódulo  pulmonar  mide  >2  cm;  (2)  el
el  mediastino  está  "lleno"  como  se  ve  en  una  tomografía  computarizada  (TC);  y  (3)  el  nódulo  se  "besa"  contra  el  mediastino.  
Una  resección  pulmonar  está  contraindicada  si:  (1)  los  ganglios  paratraqueales  ipsolaterales  "altos"  son  positivos;  (2)  los  
ganglios  contralaterales  son  positivos;  o  (3)  se  identifica  una  histología  no  diferenciada  ("células  de  avena").

23.  Actualmente,  las  causas  más  comunes  de  estenosis  aórtica  son  las  anomalías  congénitas  y  la  enfermedad  calcificada  
(degenerativa).

24.  En  la  regurgitación  mitral,  el  ventrículo  izquierdo  expulsa  sangre  por  dos  vías:  (1)  anterógrada  a  través  de  la  válvula  aórtica  
o  (2)  retrógrada  a  través  de  la  válvula  mitral.  La  cantidad  de  cada  volumen  sistólico  expulsado  retrógradamente  hacia  la  
aurícula  izquierda  es  la  fracción  regurgitante.  Para  compensar  la  fracción  regurgitante,  el  ventrículo  izquierdo  debe  
aumentar  su  volumen  sistólico  total.
Esto  finalmente  produce  una  sobrecarga  de  volumen  del  ventrículo  izquierdo  y  conduce  a  una  disfunción  ventricular.

25.  Las  indicaciones  para  el  injerto  de  derivación  de  la  arteria  coronaria  (CABG)  son  (1)  la  arteria  coronaria  principal  izquierda
estenosis;  (2)  enfermedad  de  la  arteria  coronaria  de  tres  vasos  (estenosis  del  70  %)  con  función  ventricular  izquierda  (LV)  
deprimida  o  enfermedad  de  la  arteria  coronaria  (CAD)  de  dos  vasos  con  compromiso  de  la  descendente  anterior  izquierda  
proximal  (LAD);  y  (3)  angina  a  pesar  de  la  terapia  médica  agresiva.

26.  La  CABG  mejora  el  miocardio  en  hibernación.  La  hibernación  miocárdica  se  refiere  a  la  función  contráctil  miocárdica  reversible  
asociada  con  una  disminución  del  flujo  coronario  en  el  contexto  de  viabilidad  miocárdica  preservada.  Algunos  pacientes  con  
disfunción  sistólica  global  exhiben  una  mejoría  dramática  en  la  contractilidad  miocárdica  después  de  CABG.

27.  El  tratamiento  quirúrgico  de  la  colitis  ulcerosa  es  la  colectomía  total  con  anastomosis  del  reservorio  ileoanal.
Machine Translated by Google
LOS  100  PRINCIPALES  SECRETOS  3

28.  La  úlcera  de  Dieulafoy  es  una  malformación  vascular  gástrica  con  una  arteria  submucosa  expuesta,  generalmente  dentro  de  los  
2  a  5  cm  de  la  unión  gastroesofágica.  Se  presenta  con  hematemesis  indolora,  a  menudo  masiva.

29.  El  papel  de  la  colectomía  subtotal  a  ciegas  en  el  tratamiento  de  la  hemorragia  digestiva  baja  masiva  se  limita  a  un  pequeño  grupo  
de  pacientes  en  los  que  no  se  puede  identificar  una  fuente  de  hemorragia  específica.  El  procedimiento  se  asocia  con  una  tasa  
de  mortalidad  del  16%.

30.  Los  pólipos  colorrectales  <2  cm  tienen  un  2%  de  riesgo  de  contener  cáncer;  Los  pólipos  de  2  cm  tienen  un  riesgo  del  10  %;  y  los  
pólipos  >2  cm  tienen  un  riesgo  de  cáncer  del  40%.  El  60%  de  los  pólipos  vellosos  miden  >2  cm  y  el  77%  de  los  pólipos  
tubulares  miden  <1  cm  en  el  momento  del  descubrimiento.

31.  Los  pacientes  con  cáncer  colorrectal  con  afectación  de  los  ganglios  linfáticos  (clasificación  de  Dukes)  deben
recibir  quimioterapia  después  de  la  operación  para  tratar  las  micrometástasis.

32.  La  regla  de  Goodsall  establece  que  la  ubicación  de  la  abertura  interna  de  una  fístula  anorrectal  se  basa  en  la  posición  de  la  
abertura  externa.  Una  abertura  externa  posterior  a  una  línea  trazada  transversalmente  a  través  del  perineo  se  origina  en  una  
abertura  interna  en  la  línea  media  posterior.  Una  abertura  externa  anterior  a  esta  línea  se  origina  en  la  cripta  anal  más  cercana  
en  dirección  radial.

33.  Hernia  inguinal  encarcelada:  las  estructuras  en  el  saco  herniario  todavía  tienen  un  buen  suministro  de  sangre  pero  están  
atrapadas  en  el  saco  debido  a  las  adherencias  o  al  cuello  estrecho  del  saco  herniario.  La  hernia  inguinal  estrangulada  ocurre  
cuando  las  estructuras  de  la  hernia  tienen  un  suministro  de  sangre  comprometido  debido  a  la  constricción  anatómica  en  el  
cuello  de  la  hernia.

34.  El  signo  de  Chvostek  es  un  espasmo  de  los  músculos  faciales  causado  por  un  golpeteo  en  el  tronco  del  nervio  facial.
El  signo  de  Trousseau  es  un  espasmo  del  carpo  provocado  por  la  oclusión  de  la  arteria  braquial  durante  3  minutos  con  un  
manguito  de  presión  arterial.  Ambos  signos  indican  hipocalcemia.

35.  Las  dos  opciones  quirúrgicas  para  la  enfermedad  de  Graves  son  la  tiroidectomía  subtotal  o  la  tiroidectomía  casi  total.
tiroidectomía.

36.  La  única  prueba  bioquímica  que  se  necesita  rutinariamente  para  identificar  a  los  pacientes  con  hipertiroidismo  
insospechado  es  la  concentración  sérica  de  la  hormona  estimulante  de  la  tiroides  (TSH).

37.  Las  causas  de  hipertensión  corregibles  quirúrgicamente  son  la  hipertensión  renovascular,
feocromocitoma,  síndrome  de  Cushing,  hiperaldosteronismo  primario,  coartación  de  la  aorta  y  enfermedad  del  parénquima  
renal  unilateral.

38.  La  "prueba  negativa  triple"  o  "tríada  de  diagnóstico"  para  diagnosticar  una  masa  mamaria  palpable  incluye
exploración  física,  imagenología  mamaria  y  biopsia.

39.  La  radiación  de  la  pared  torácica  está  indicada  después  de  la  mastectomía  en  pacientes  con  cánceres  primarios  de  más  de  5  
cm,  márgenes  de  mastectomía  positivos  o  más  de  cuatro  ganglios  linfáticos  positivos,  todos  los  cuales  están  asociados  con  
índices  elevados  de  recurrencia  locorregional.

40.  Los  ganglios  linfáticos  centinela  son  la  primera  parada  para  que  las  células  tumorales  hagan  metástasis  a  través  de  los  vasos  
linfáticos  del  tumor  primario.

41.  El  sitio  de  origen  más  común  de  los  melanomas  subungueales  es  el  dedo  gordo  del  pie.  Se  recomienda  la  amputación  en  
la  articulación  metatarsiano­falángica  o  proximal  a  ella  y  la  biopsia  del  ganglio  linfático  centinela  regional.
Machine Translated by Google
4  100  PRINCIPALES  SECRETOS

42.  Ramus  marginalis  mandibularis,  la  rama  más  baja  del  nervio  que  inerva  los  músculos  depresores  del  labio  inferior,  es  la  
rama  del  nervio  facial  que  se  lesiona  con  mayor  frecuencia  durante  la  parotidectomía.

43.  El  anillo  de  Waldeyer  es  la  mucosa  de  la  orofaringe  posterior  que  cubre  un  lecho  de  tejido  linfático  que  se  agrega  
para  formar  las  amígdalas  palatina,  lingual,  faríngea  y  tubárica.  Estas  estructuras  forman  un  anillo  alrededor  de  
la  pared  faríngea.  Este  puede  ser  el  sitio  del  tumor  primario  o  metastásico.

44.  Un  paciente  en  el  que  el  examen  de  cabeza  y  cuello  es  completamente  normal  pero  con  aguja  fina
la  aspiración  (PAAF)  de  un  ganglio  cervical  revela  cáncer  escamoso,  se  debe  examinar  la  boca,  la  faringe,  la  
laringe,  el  esófago  y  el  árbol  traqueobronquial  bajo  anestesia  (endoscopia  triple).  Si  no  se  ve  nada,  se  debe  realizar  
una  biopsia  a  ciegas  de  la  nasofaringe,  las  amígdalas,  la  base  de  la  lengua  y  los  senos  piriformes  en  la  misma  
sesión.

45.  Los  microorganismos  implicados  en  la  aterosclerosis  incluyen  Chlamydia  pneumoniae,  Helicobacter  pylori,  estreptococos  y  
Bacillus  typhosus.

46.  La  tasa  acumulativa  de  amputaciones  por  claudicación  en  10  años  es  del  10  %.  La  enfermedad  vascular  es  sistémica,
por  lo  tanto,  muchos  de  estos  pacientes  mueren  antes  de  la  amputación.

47.  La  reducción  absoluta  del  riesgo  de  accidente  cerebrovascular  es  del  6  %  durante  un  período  de  5  años  en  
pacientes  asintomáticos  con  >60  %  de  estenosis  que  se  someten  a  endarterectomía  carotídea  (CEA)  más  
aspirina  versus  pacientes  tratados  con  aspirina  sola  (5,1  %;  cirugía  versus  11  %  médica).  Rx).  Esto  es  del  estudio  
del  Estudio  de  aterosclerosis  carotídea  asintomática  (ACAS,  por  sus  siglas  en  inglés)  (consulte  el  capítulo  1  de  
Lecturas  obligatorias).

48.  La  tasa  de  expansión  promedio  de  un  aneurisma  de  aorta  abdominal  es  de  0,4  cm/año.

49.  La  heparina  se  une  a  la  antitrombina  III,  haciéndola  más  activa.

50.  El  paciente  con  sospecha  de  claudicación  intermitente  debe  evaluarse  inicialmente  mediante  la  obtención  del  índice  tobillo­
brazo  (ITB)  o  presiones  segmentarias  de  las  extremidades  en  reposo.  Por  lo  general,  un  ABI  de  0,6  refleja  claudicación  
y  un  ABI  de  <0,3  refleja  amenaza  en  las  extremidades.

51.  El  shock  es  el  consumo  subóptimo  de  oxígeno  (O2)  y  la  excreción  de  dióxido  de  carbono  (CO2)  en  el
nivel  celular.

52.  El  óxido  nítrico  se  sintetiza  en  las  células  endoteliales  vasculares  mediante  la  óxido  nítrico  sintasa  constitutiva  (NOS)  y  
la  NOS  inducible,  utilizando  arginina  como  sustrato.

53.  La  saliva  tiene  la  mayor  concentración  de  potasio  (20  mEq),  seguida  de  las  secreciones  gástricas  (10  mEq)  y  luego  
las  secreciones  pancreáticas  y  duodenales  (5  mEq).

54.  Gasto  calórico  basal  igual  a  25  kilocalorías  por  kilogramo  al  día  con  un  requerimiento  de
aproximadamente  1  g  de  proteína  por  kilogramo  por  día.

55.  Seis  gramos  y  un  cuarto  de  proteína  contienen  1  g  de  nitrógeno.

56.  La  dextrosa  tiene  3,4  kcal/g;  la  proteína  tiene  4  kcal/g;  y  grasa  9  kcal/g  (la  solución  de  lípidos  al  20%  proporciona
2kcal/ml).
Machine Translated by Google
LOS  100  PRINCIPALES  SECRETOS  5

57.  Las  tasas  máximas  de  infusión  de  glucosa  en  fórmulas  parenterales  no  deben  exceder  los  5  miligramos  por  kilogramo  
por  minuto.

58.  El  síndrome  de  realimentación  ocurre  en  pacientes  con  desnutrición  moderada  a  severa  (p.  ej.,  alcoholismo  crónico  o  
anorexia  nerviosa)  quienes,  con  una  gran  carga  de  nutrientes,  desarrollan  disminuciones  clínicamente  significativas  en  
los  niveles  séricos  de  fósforo,  potasio,  calcio  y  magnesio.  La  hiperglucemia  es  común  secundaria  a  la  secreción  de  insulina  
embotada.  La  producción  de  trifosfato  de  adenosina  (ATP)  se  mitiga  y  la  insuficiencia  respiratoria  es  común.

59.  La  glutamina  es  el  aminoácido  más  común  que  se  encuentra  en  el  músculo  y  el  plasma.  Los  niveles  disminuyen  después  de  
la  cirugía  y  el  estrés  fisiológico.  La  glutamina  sirve  como  sustrato  para  las  células  que  se  replican  rápidamente  
(curiosamente,  también  es  el  sustrato  metabólico  número  uno  para  las  células  neoplásicas),  mantiene  la  integridad  y  la  
función  de  la  barrera  intestinal  y  protege  contra  el  daño  de  los  radicales  libres  al  mantener  los  niveles  de  glutatión  (GSH).  
La  glutamina  es  inestable  en  forma  intravenosa  (IV)  a  menos  que  se  vincule  como  un  dipéptido.

60.  La  fiebre  es  causada  por  macrófagos  activados  que  liberan  interleucina­1,  factor  de  necrosis  tumoral  (TNF)  e  interferón  
en  respuesta  a  bacterias  y  endotoxinas.  El  resultado  es  un  restablecimiento  del  centro  termorregulador  hipotalámico.

61.  El  gasto  cardíaco  (CO)  es  igual  a  la  frecuencia  cardíaca  multiplicada  por  el  volumen  sistólico;  el  CO  normal  es  de  5  a  6  L/min
y  el  índice  cardíaco  es  de  2,4  a  3,0  litros  por  minuto  por  metro  cuadrado.

62.  La  resistencia  vascular  sistémica  (RVS)  es  igual  a  la  presión  arterial  media  (PAM)  menos
presión  venosa  central  (PVC)  dividida  por  CO  multiplicado  por  80;  y  se  escribe  como:  SVR  ¼  a  [(MAP  CVP)/CO]  80.  La  
SVR  normal  es  de  800  a  1200  dinasec/cm5 .

63.  Los  signos  de  shock  hipovolémico  son  CVP  y  presión  de  enclavamiento  capilar  pulmonar  (PCWP)  bajos,  GC  bajo  y  
saturación  de  oxígeno  venoso  mixto  (SVO2)  y  RVS  alto.

64.  Los  signos  de  shock  cardiogénico  son  CVP  y  PCWP  altos,  CO  y  SVO2  bajos,  y  SVR  variable.

65.  Los  signos  de  shock  séptico  son  CVP  y  PCWP  bajos  o  normales,  CO  alto  inicialmente,  SVO2  alto,
y  baja  RVS.

66.  El  signo  de  Kehr  es  dolor  simultáneo  en  el  cuadrante  superior  izquierdo  (LUQ)  y  en  el  hombro  izquierdo,  lo  que  indica
Irritación  diafragmática  por  ruptura  del  bazo  o  absceso  subdiafragmático.  Anatómicamente,  el  diafragma  y  la  parte  
posterior  del  hombro  izquierdo  disfrutan  de  una  inervación  paralela.

67.  La  sensibilidad  de  rebote  (roce  de  las  superficies  peritoneales  entre  sí)  implica  inflamación  peritoneal  (peritonitis).

68.  Las  cinco  W  de  la  fiebre  posoperatoria  son  herida  (infección),  agua  (infección  del  tracto  urinario,  UTI),  viento  (atelectasia,  
neumonía),  caminar  (tromboflebitis)  y  drogas  maravillosas  (fiebres  por  drogas).

69.  La  cricotiroidotomía  no  debe  realizarse  en  pacientes  menores  de  12  años  ni  en  ningún  paciente  con  sospecha  de  
traumatismo  laríngeo  directo  o  rotura  traqueal.

70.  El  pulso  radial  palpable  (muñeca)  refleja  presión  arterial  sistólica  (PAS)  >80  mm  Hg;  el  pulso  femoral  (ingle)  palpable  refleja  
PAS  >70  mm  Hg;  y  el  pulso  carotídeo  (cuello)  palpable  refleja  PAS  >60  mm  Hg.
Machine Translated by Google
6  100  PRINCIPALES  SECRETOS

71.  Una  regla  general  para  la  infusión  de  cristaloides  para  reemplazar  la  pérdida  de  sangre  es  una  proporción  de  3:1  de  cristaloides  isotónicos.
a  la  sangre

72.  Los  ojos  de  mapache  (equimosis  periorbitaria)  y  el  signo  de  Battle  (equimosis  mastoidea)  son  síntomas  clínicos.
indicadores  de  fractura  basilar  del  cráneo.

73.  La  presión  de  perfusión  cerebral  (CPP)  es  igual  a  MAP  menos  la  presión  intracraneal  (ICP);  y  se  escribe  como  CPP  ¼  
MAP  ICP.  Existe  cierto  debate  sobre  el  CPP  mínimo  permitido,  pero  el  consenso  indica  que  es  necesario  un  CPP  de  50  
a  70  mm  Hg.

74.  La  violación  del  platisma  define  una  herida  penetrante  en  el  cuello.

75.  El  neumotórax  a  tensión  es  la  acumulación  de  aire  en  el  espacio  pleural  que  provoca  un  aumento  intratorácico
presión  y  resultando  en  una  disminución  en  el  retorno  venoso  al  corazón.

76.  El  sitio  más  común  de  lesión  de  la  aorta  torácica  en  un  traumatismo  cerrado  es  justo  distal  a  la  salida  de  la  arteria  subclavia  
izquierda.

77.  La  manifestación  más  común  de  lesión  miocárdica  contusa  es  la  arritmia.

78.  Las  indicaciones  para  toracotomía  en  un  paciente  estable  con  hemotórax  incluyen  un  tubo  inmediato
gasto  por  toracostomía  >  1500  ml  y  sangrado  continuo  de  250  ml/h  durante  4  horas  consecutivas.

79.  La  tríada  de  Beck  es  hipotensión,  distensión  de  las  venas  del  cuello  y  ruidos  cardíacos  apagados  (piense  en  el  pericardio).
taponamiento).

80.  La  arteria  hepática  aporta  aproximadamente  el  30  %  del  flujo  sanguíneo  al  hígado  y  la  vena  porta  aporta  el  70  %  
restante.  El  suministro  de  oxígeno,  sin  embargo,  es  similar  para  ambos  al  50%.

81.  La  maniobra  de  Pringle,  que  se  utiliza  para  reducir  la  hemorragia  hepática,  es  una  oclusión  manual  del  ligamento  
hepatoduodenal  para  interrumpir  el  flujo  sanguíneo  al  hígado.

82.  La  esplenectomía  reduce  significativamente  los  niveles  de  inmunoglobulina  M  (IgM).

83.  El  noventa  por  ciento  de  las  muertes  por  traumatismos  resultantes  de  fracturas  pélvicas  son  el  resultado  de  
hemorragia  venosa  y  exudación  ósea;  sólo  el  10%  de  la  hemorragia  pélvica  letal  por  traumatismo  cerrado  es  
arterial  (el  sitio  más  común  es  la  arteria  glútea  superior).

84.  El  protocolo  para  la  rotura  vesical  intraperitoneal  por  traumatismo  cerrado  es  el  tratamiento  operatorio,  mientras  que  el  
protocolo  para  la  rotura  extraperitoneal  es  el  tratamiento  observador.

85.  El  pseudoaneurisma  es  una  ruptura  de  la  pared  arterial  que  conduce  a  un  hematoma  pulsátil  contenido  por  adventicia  
vascular  y  tejido  conjuntivo  fibroso  (pero  no  las  tres  capas  de  la  pared  arterial,  que  es  lo  que  define  un  verdadero  
aneurisma).

86.  El  signo  más  temprano  del  síndrome  compartimental  de  las  extremidades  inferiores  es  neurológico  en  la  distribución  de
el  nervio  peroneo  con  entumecimiento  en  el  primer  espacio  web  dorsal  y  dorsiflexión  débil.

87.  Las  luxaciones  posteriores  de  rodilla  se  asocian  con  lesiones  de  la  arteria  poplítea  y  son  una  indicación  para
angiografía.

88.  El  tratamiento  de  la  sospecha  de  fractura  del  navicular  a  pesar  de  la  radiografía  negativa  es  un  yeso  de  brazo  corto  y  
repetir  la  radiografía  en  2  semanas;  estas  fracturas  también  tienen  un  alto  riesgo  de  necrosis  avascular.
Machine Translated by Google
LOS  100  PRINCIPALES  SECRETOS  7

89.  La  fórmula  de  Parkland  es  Ringer  lactato  a  4  ml/kg  de  porcentaje  del  área  de  superficie  corporal  total  (TBSA)  quemada  
(solo  de  segundo  y  tercer  grado).  Infundir  el  50  %  del  volumen  en  las  primeras  8  horas  y  el  50  %  restante  durante  las  16  
horas  siguientes.

90.  La  tasa  metabólica  alcanza  un  máximo  de  2,5  veces  la  tasa  metabólica  basal  en  quemaduras  graves  >50  %  TBSA.

91.  Los  cálculos  biliares  y  el  abuso  de  alcohol  son  las  dos  causas  principales  de  pancreatitis  aguda.

92.  El  abuso  de  alcohol  representa  el  75%  de  los  casos  de  pancreatitis  crónica.

93.  Las  várices  gástricas  aisladas  con  hiperesplenismo  indican  trombosis  de  la  vena  esplénica  y  son  una
indicación  de  esplenectomía.

94.  El  tratamiento  de  la  pancreatitis  por  cálculos  biliares  es  la  colecistectomía  y  la  colangiografía  intraoperatoria
durante  la  misma  estancia  hospitalaria  una  vez  remitida  la  pancreatitis.

95.  Los  inhibidores  de  la  bomba  de  protones  (IBP)  inhiben  de  forma  irreversible  la  bomba  de  iones  de  hidrógeno  de  las  células  parietales.

96.  El  tratamiento  definitivo  de  la  gastritis  por  reflujo  alcalino  tras  un  Billroth  II  incluye  una  gastroenteritis  en  Y  de  Roux
yeyunostomía  de  una  rama  yeyunal  eferente  de  40  cm.

97.  La  úlcera  de  Cushing  es  una  úlcera  por  estrés  que  se  encuentra  en  pacientes  críticos  con  el  sistema  nervioso  central  (SNC)
lesión.  Por  lo  general,  es  único  y  profundo,  con  tendencia  a  perforarse.

98.  La  úlcera  de  Curling  es  una  úlcera  por  estrés  que  se  encuentra  en  pacientes  en  estado  crítico  con  lesiones  por  quemaduras.

99.  La  úlcera  marginal  es  una  úlcera  que  se  encuentra  cerca  del  margen  de  la  anastomosis  gastroentérica,  generalmente  en  el
lado  del  intestino  delgado.

100.  La  causa  más  frecuente  de  obstrucción  del  intestino  delgado  es  la  enfermedad  adhesiva;  el  segundo  mas
causa  común  es  una  hernia.
Machine Translated by Google

I.  TEMAS  GENERALES

¿ESTÁS  LISTO  PARA  TU  ROTACIÓN  
QUIRÚRGICA? CAPÍTULO  
1

Tabetha  R.  Harken,  MD,  MPH,  U.  Mini  B.  Swift,  MD,  Alden  H.  Harken,  MD

La  cirugía  es  un  deporte  participativo,  de  equipo  y  de  contacto.  Preséntese  a  los  pacientes,  residentes  y  
asistentes  con  entusiasmo  (que  cubre  una  multitud  de  pecados),  puntualidad  (a  las  personas  tipo  A  no  les  gusta  
esperar)  y  limpieza  (debe  verse,  actuar  y  oler  como  un  médico).

1.  ¿Por  qué  debe  presentarse  a  cada  paciente  y  preguntar  por  su  principal
¿queja?
Los  síntomas  son  percepción,  y  la  percepción  es  más  importante  que  la  realidad.  Para  un  paciente,  la  
queja  principal  no  es  simplemente  una  cuestión  de  vida  o  muerte;  es  mucho  más  importante.  Los  pacientes  
se  encuentran  rutinariamente  en  situaciones  comprometedoras,  incómodas,  vergonzosas  e  indignas.  Sin  
embargo,  los  pacientes  son  personas  y  tienen  intereses,  preocupaciones,  ansiedades  y  una  historia.  Como  
estudiante,  tiene  la  oportunidad  de  ubicar  la  queja  principal  de  su  paciente  en  el  contexto  del  resto  de  su  vida.  
Esta  habilidad  es  importante,  y  el  paciente  siempre  estará  agradecido.
Puede  cumplir  un  propósito  real  como  oyente  y  traductor  para  el  paciente  y  su  familia.
Los  pacientes  quieren  confiar  en  usted  y  amarlo.  Esta  confianza  en  la  terapia  quirúrgica  es  una  herramienta  
formidable.  Cuanto  más  comprenda  un  paciente  acerca  de  su  enfermedad,  más  podrá  participar  para  mejorar.  
La  recuperación  es  más  rápida  si  el  paciente  ayuda.
Del  mismo  modo,  cuanto  más  comprenda  el  paciente  acerca  de  su  terapia  (incluidos  los  efectos  secundarios
y  posibles  complicaciones),  más  efectiva  es  la  terapia  (este  principio  no  está  en  los  libros  de  texto).  
Puede  ser  el  intérprete  de  su  paciente.  Esta  es  la  diversión  de  la  cirugía  (y  la  medicina).

2.  ¿Cuál  es  la  respuesta  correcta  a  casi  todas  las  preguntas?
Gracias.  La  gratitud  es  una  herramienta  invaluable  en  las  salas.

3.  ¿Existen  reglas  simples  desde  las  trincheras?
1.  Llevarse  bien  con  las  enfermeras.  Las  enfermeras  saben  más  que  el  resto  de  nosotros  sobre  los  códigos,  
rutinas  y  rituales  para  hacer  que  las  salas  funcionen  sin  problemas.  Es  posible  que  no  sepan  tanto  
sobre  los  feocromocitomas  y  los  filamentos  intermedios,  pero  sobre  las  cosas  que  importan,  saben  
mucho.  ¡Reconoce  eso,  y  te  tomarán  bajo  sus  alas  y  te  enseñarán  una  tonelada!

2.  Ayudar.  Si  sus  residentes  parecen  ocupados,  probablemente  lo  estén.  Por  lo  tanto,  si  pregunta  cómo  puede  ayudar  y  
están  demasiado  ocupados  incluso  para  responder,  preguntar  de  nuevo  probablemente  no  produzca  mucho.
Siempre  aproveche  la  oportunidad  de  tomar  radiografías,  buscar  resultados  de  laboratorio  y  recuperar  
una  bolsa  de  sangre  del  banco.  El  equipo  reconocerá  su  entusiasmo  y  recompensará  sus  contribuciones.
3.  Ser  aplastado.  A  todos  nos  gustaría  una  secretaria,  pero  no  se  va  a  proporcionar  una  en  este
rotación.  Tus  residentes  hacen  mucho  de  su  propio  trabajo  sin  que  tú  lo  sepas.  Entonces,  si  siente  que  
el  trabajo  sucio  está  por  debajo  de  usted,  tal  vez  debería  pensar  en  otra  profesión.
4.  Trabajando  duro.  Esta  rotación  es  un  aprendizaje.  Si  te  esfuerzas,  obtendrás  una  idea  realista  de  lo  que  
significa  ser  residente  (e  incluso  médico  en  ejercicio)  en  esta  especialidad.  (Esto  tiene  grandes  ventajas  
cuando  selecciona  un  tipo  de  pasantía).
5.  Mantenerse  informado.  Al  principio,  puede  sentir  que  no  es  una  parte  real  del  equipo.
Sin  embargo,  si  es  persistente  y  confiable,  pronto  sus  residentes  le  confiarán  trabajos  más  importantes.

9
Machine Translated by Google
10  CAPÍTULO  1  ¿ESTÁS  LISTO  PARA  TU  ROTACIÓN  QUIRÚRGICA?

6.  Educarse  a  sí  mismo  y  luego  educar  a  sus  pacientes.  Aquí  está  uno  de  los  gratificantes
lugares  (como  se  indica  en  la  pregunta  1)  donde  puedes  llegar  a  lo  más  alto  del  equipo.  Hable  con  sus  pacientes  
sobre  todo  (incluyendo  su  enfermedad  y  terapia),  y  lo  amarán  por  ello.

7.  Mantener  una  actitud  positiva.  Como  estudiante  de  medicina,  puede  sentir  que  no  es  una  parte  crucial  del  equipo.  
Incluso  si  es  increíblemente  inteligente,  es  poco  probable  que  tome  las  decisiones  administrativas  cruciales.  Entonces,  
¿qué  deja  eso:  actitud.  Si  está  entusiasmado  e  interesado,  sus  residentes  disfrutarán  tenerlo  cerca  y  trabajarán  para  
mantenerlo  involucrado  y  satisfecho.  Un  quejoso  deslumbrantemente  inteligente  pero  malhumorado  es  más  adecuado  
para  una  rotación  en  la  morgue.  Recuerde,  es  probable  que  su  residente  esté  siguiendo  a  15  pacientes  enfermos,  le  
paguen  menos  de  $2  por  hora  y  no  haya  dormido  más  de  5  horas  en  los  últimos  3  días.

Las  cosas  simples  como  sonreír  y  decir  gracias  (cuando  alguien  te  enseña)  son  increíblemente  útiles  y  son  
recompensadas  en  todas  las  rotaciones  clínicas  con  experiencia  y  buenas  calificaciones.

8.  ¡Divertirse!  Esta  es  la  profesión  más  emocionante,  gratificante,  gratificante  y  divertida  y  es  ligera
años  mejor  que  lo  que  sea  el  segundo  mejor  (esta  no  es  solo  nuestra  opinión).

4.  ¿Cuál  es  el  mejor  enfoque  para  las  notas  quirúrgicas?
Las  notas  quirúrgicas  deben  ser  sucintas.  La  mayoría  de  los  cirujanos  todavía  mueven  los  labios  cuando  leen.  Consulte  
la  Tabla  1­1.

TABLA  1­1.  BESTAP  PRO  A  CH  A  NOTAS  QUIRÚRGICAS

Órdenes  de  Admisión

Admitir  a  5  West  (nombre  del  asistente)

Condición: Estable

Diagnóstico: Dolor  abdominal;  r/o  apendicitis

Signos  vitales: cada  4  horas

Parámetros: Por  favor  llame  HO  para:

T  >38  C

160  <  PA  <  90

120  <  FC  <  60

Dieta: OSFL

Fluidos: 1000  LR  w  20  mEq  KCl  @  100  ml/h  ASA  650  

Medicamentos:   mg  PR  prn  para  T  >38,5  C

Gracias.

Firme  con  su  nombre/deje  espacio  para  la  firma  del  residente  (su  número  de  localizador)

Historia  y  Examen  Físico  (H  &  P)

La  Sra.  O'Flaherty  es  una  ♀  [mujer  blanca]  de  55  años  que  ingresó  con  un  cc  [dolencia  principal]:  ''me  duele  el  
estómago''.  al  ingreso]  cuando  notó  un  inicio  gradual  de  dolor  tipo  cólico  en  la  parte  media  del  epigastrio.  El  dolor  
ahora  es  severo  (7/10;  7  en  una  escala  de  10)  y  recurrente  cada  5  minutos.  El  paciente  describió  þ  vómitos  (þ  bilis,  
sangre)  [con  bilis,  sin  sangre].

PMH  [historial  médico  pasado]

Hosp[italizaciones]: Neumonía  (1991)

Parto  (1970,  1972)
Machine Translated by Google
CAPÍTULO  1  ¿ESTÁS  LISTO  PARA  TU  ROTACIÓN  QUIRÚRGICA?  11

TABLA  1­1.  BESTAP  PRO  A  CH  A  NOTAS  QUIRÚRGICAS—CON  T  'D

Cirugía]: esplenectomía  por  trauma  (1967)

Alergias: Codeína,  mariscos

Social: ETOH  [alcohol]  1  

Tabaco: ppd  [paquete  por  día]  x  25  años

ROS  [revisión  de  sistemas]

Respiratorio]: tos  productiva

Cardíaco: o¯  dolor  torácico  [o  ¼  no  observado,  no  
contribuyente  o  no  aquí]

o¯  MI  [infarto  de  miocardio]

Renal: o¯  disuria

o¯  frecuencia

Neuro  [lógica]: WNL  [dentro  de  los  límites  normales]

Examen  físico  (PE)

PA: 140/90

HORA: 100  (normal)  16  

FR  [frecuencia  respiratoria]: respiraciones/min

Temperatura: 38.2C

WD  [bien  desarrollado],  WN  [bien  nutrido],  levemente  obeso,  55  años  ♀  con  malestar  abdominal  moderado

HEENT  [cabeza,  ojos,  oídos,  nariz  y  garganta]:  WNL

Resp: Limpie  los  pulmones  bilat[eralmente]

o¯  sibilancias

Corazón: o¯  m  [murmullo]

RSR  [ritmo  sinusal  regular]

Abdomen: Levemente  distendido

Ataques  agudos  que  coinciden  con  dolor  tipo  cólico

Sensibilidad  a  la  palpación  (no  es  necesario  lastimar  
al  paciente  para  averiguarlo)

o¯  Rebote

Rectal: (Siempre  hazlo;  nunca  pospongas  el  examen  rectal  
en  tu  rotación  quirúrgica)

Hematest:  negativo  para  sangre

Sin  masas,  sin  ternura

Pélvico: sin  masas

Sin  sensibilidad  anexial

Sin  sensibilidad  al  movimiento  cervical  ni  signo  de  

candelabro;  ¡si  el  movimiento  rápido  del  cuello  uterino  
hace  que  su  paciente  golpee  el  candelabro!  signo  
peritoneal  no  específico,  posiblemente  enfermedad  
pélvica  inflamatoria  (EPI;  gonorrea)

(Continuado)
Machine Translated by Google
12  CAPÍTULO  1  ¿ESTÁS  LISTO  PARA  TU  ROTACIÓN  QUIRÚRGICA?

TABLA  1­1.  BESTAP  PRO  A  CH  A  NOTAS  QUIRÚRGICAS—CON  T  'D
Extremidades: ROM  completo  [rango  de  movimiento]

o¯  edema

Pulsos  saltantes  (3þ)

Impresión]: Dolor  abdominal

r/o  SB  [intestino  delgado]  obstrucción  2  
[secundaria]  a  adherencias

prescripción:
sonda  OF  [nasogástrica]

líquidos  intravenosos

Consentimiento  op[erativo]

Escriba  y  mantenga

[Firma]

Notas  sobre  el  H&P  quirúrgico

  Una  H&P  quirúrgica  debe  ser  sucinta  y  centrada  en  el  problema  del  paciente.

&  Comience  con  la  queja  principal  (en  palabras  del  paciente).

&  ¿El  problema  es  nuevo  o  crónico?

&  PMH:  siempre  incluya  hospitalizaciones  previas  y  medicamentos.

&  ROS:  restrinja  la  revisión  a  los  sistemas  de  órganos  (pulmón,  corazón,  riñones  y  sistema  nervioso)  que  pueden  
afectar  esta  admisión.

&  PE:  siempre  comience  con  los  signos  vitales  (incluidas  la  respiración  y  la  temperatura);  por  eso  estos  signos  son  
vitales.

&  Rebote  significa  irritación  peritoneal  inflamatoria  o  peritonitis.

Nota  preoperatoria  La  nota  

preoperatoria  es  una  lista  de  verificación  que  confirma  que  usted  y  el  paciente  están  listos  para  el  procedimiento  quirúrgico  
planificado.  Coloque  esta  nota  en  las  Notas  de  progreso:

Preop  dx  [diagnóstico]: Obstrucción  SB  2  a  adherencias

CXR  [radiografía  de  tórax]: Claro

ECG  [electrocardiograma]: NSR  con  cambios  de  onda  ST­T

Sangre: Tipo  y  cross­match  x  2  u

Consentir: En  la  tabla

Nota  operativa

La  nota  operativa  debe  proporcionar  a  cualquier  persona  que  se  encuentre  con  el  paciente  después  de  la  cirugía  toda  la  
información  necesaria:

Diagnóstico  preoperatorio:
Obstrucción  SB

Diagnóstico  postoperatorio: Igual,  todo  el  intestino  viable

Procedimiento: Lap[arotomía]  exploratoria  con  lisis  de  adherencias

Cirujano: Nombre  él  o  ella

Asistentes: Ponlos  en  una  lista

Anestesia: GEA  [anestesia  general  endotraqueal]
Machine Translated by Google
CAPÍTULO  1  ¿ESTÁS  LISTO  PARA  TU  ROTACIÓN  QUIRÚRGICA?  13

TABLA  1­1.  BESTAP  PRO  A  CH  A  NOTAS  QUIRÚRGICAS—CON  T  'D

I&O  [entrada  y  salida]: En:  1200  ml  Lactato  de  Ringer  (R/L)

Salida:  400  ml  de  orina

EBL  [pérdida  de  sangre  estimada]: 50ml

Muestra: Ninguno

Drenajes: Ninguno

[Firma  con  tu  nombre]

AAS,  aspirina;  PA,  presión  arterial  sistólica;  BRP,  privilegios  de  baño;  hora  cero;  HO,  oficial  de  la  casa;  FC,  frecuencia  cardiaca;  NPO,  nada  por  
la  boca  (esto  incluye  agua  y  pastillas);  OOB,  fuera  de  la  cama;  PR,  por  recto;  PRN,  según  sea  necesario;  q,  cada;  r/o,  descartar;  T,  temperatura.

Nota:  No  puede  ser  demasiado  cortés  o  demasiado  agradecido  con  los  pacientes  o  las  enfermeras.

ALTA  HOSPITALARIA

5.  ¿Qué  es  una  transición  de  cuidados?

Es  una  palabra  elegante  para  cualquier  cambio  en  un  entorno  de  atención  clínica.  Los  ejemplos  incluyen:  del  hospital  
al  hogar,  del  hogar  al  departamento  de  emergencias  (ED)  y  del  asilo  de  ancianos  al  hogar.

6.  ¿Cuál  es  una  de  las  cosas  más  peligrosas  que  puede  hacerle  a  su  paciente?
Darles  de  alta  del  hospital.

7.  ¿Por  qué  el  alta  hospitalaria  es  un  procedimiento  peligroso?
Los  hospitales  están  diseñados  para  brindar  el  máximo  apoyo.  Se  gestionan  los  procedimientos;  la  dieta  está  
controlada;  e  incluso  la  polifarmacia  cada  vez  más  obligada  está  orquestada  de  tal  manera  que  cada  pastilla  se  
traga  con  precisión  metronómica.  Entonces,  al  igual  que  un  aguilucho  bebé,  el  paciente  es  "empujado"  sin  
contemplaciones  de  este  nido  de  pacientes  hospitalizados  regulado  por  el  gobierno  federal.  Y  nuevamente,  como  el  
iglet  bebé,  esperamos  que  ese  paciente  tome  vuelo  en  casa.

8.  ¿Qué  mejoraría  la  seguridad  al  alta?
Siga  hasta  el  "último  cierre  de  sesión".  Cierre  la  sesión  con  su  paciente,  sus  familiares  y  el  próximo  médico  que  los  
atenderá  en  el  hogar  de  ancianos  o  la  clínica.

9.  ¿Cuáles  son  los  elementos  más  importantes  del  cierre  de  sesión  final  (descarga
resumen)?
Los  resúmenes  de  alta  deben  incluir:  
Diagnósticos  primarios  y  de  otro  tipo  Historial  
médico  pertinente  y  hallazgos  físicos  Fechas  en  que  fueron  
hospitalizados  y  curso  breve  en  el  hospital  (suponga  que  el  médico  externo  sabe  cómo  tratar  la  hiperpotasemia)

Resultados  de  procedimientos  
Pruebas  de  laboratorio  

anormales  Recomendaciones  de  los  especialistas  que  consultó  
Información  que  le  dio  al  paciente  y  a  la  familia  Medicamentos  de  alta:  
detalles  de  los  arreglos  de  seguimiento  Lista  de  citas,  pruebas  pendientes  
o  procedimientos  para  programar  o  verificar  Nombre  e  información  de  
contacto  del  medico  internado
Machine Translated by Google
14  CAPÍTULO  1  ¿ESTÁS  LISTO  PARA  TU  ROTACIÓN  QUIRÚRGICA?

La  idea  de  que  un  alta  hospitalaria  es  un  asunto  riesgoso,  pero  que  un  médico  o  un  estudiante  de  
medicina  consciente  puede  reducir  el  riesgo  proviene  de:
Kripalani  S,  LeFevre  F,  Phillips  CO  et  al.:  Déficits  en  la  comunicación  y  la  transferencia  de  información  entre  los  médicos  de  
atención  primaria  y  de  hospital,  JAMA  297:831­841,  2007.

APÉNDICE:  LECTURAS  OBLIGATORIAS

Kristin  Kanka,  DO  y  Terrence  H.  Liu,  MD

A  diferencia  de  las  rondas  médicas,  en  las  que  para  mantenerse  al  día  es  necesario  citar  un  artículo  de  
revista  actual  (preferiblemente  del  día  anterior),  en  cirugía,  puede  prosperar  conociendo  las  siguientes  
referencias,  pero  necesita  conocerlas  en  frío.

1.  Mangano  DT,  Goldman  L:  Evaluación  preoperatoria  de  pacientes  con  enfermedad  coronaria  conocida  o  sospechada,
N  Engl  J  Med  333:1750­1756,  1995.

Esta  es  una  actualización  del  artículo  original  de  Goldman  (N  Engl  J  Med,  1977)  en  el  que  fue  pionero  en  el  concepto  de
''resultado  quirúrgico  ajustado  al  riesgo''.  Debe  copiar  la  Tabla  2,  Tres  índices  de  riesgo  cardíaco  comúnmente  utilizados,  y  llevarla  
siempre  consigo.  Intuitivamente,  un  triatleta  superará  el  estrés  quirúrgico  mejor  que  un  juez  de  la  Corte  Suprema,  pero  este  artículo  
proporciona  un  sistema  de  puntos  con  el  que  puede  calcular  el  riesgo  perioperatorio  objetivo.

2.  Veronesi  U,  Cascinelli  N,  Mariani  L  et  al.:  Veinte  años  de  seguimiento  de  un  estudio  aleatorizado  que  comparó  la  cirugía  conservadora  
de  la  mama  con  la  mastectomía  radical  para  el  cáncer  de  mama  temprano,  N  Engl  J  Med  347:1227­1232,  2002.
Setecientas  mujeres  con  cáncer  de  mama  <2  cm  se  asignaron  al  azar  a  mastectomía  radical  o  cuadrantectomía  y  radioterapia.  
Después  de  1976,  las  pacientes  con  ganglios  axilares  positivos  también  recibieron  ciclofosfamida,  metotrexato  y  5­fluorouracilo  
(CMF)  adyuvantes.  Después  de  20  años,  30  mujeres  en  el  grupo  de  tratamiento  conservador  y  8  mujeres  en  el  grupo  de  
mastectomía  radical  sufrieron  recurrencia  local  (p  =  0,01).
Por  el  contrario,  la  incidencia  de  muertes  por  todas  las  causas  a  los  20  años  fue  idéntica  en  41%.  Los  autores  concluyen  que  la  
terapia  de  conservación  de  la  mama  es  el  "tratamiento  de  elección"  para  las  mujeres  con  "cánceres  de  mama  relativamente  pequeños".

3.  Fisher  B,  Anderson  S,  Bryant  J  et  al.:  Veinte  años  de  seguimiento  de  un  ensayo  aleatorizado  que  compara
mastectomía,  lumpectomía  y  lumpectomía  más  irradiación  para  el  tratamiento  del  cáncer  de  mama  invasivo,  N  Engl  J  Med  
347:1223­1241,  2002.

La  investigación  clínica  es  difícil  de  hacer.  Los  ensayos  del  National  Surgical  Adjuvant  Breast  and  Bowel  Project  (NSABP),  iniciados  
hace  25  años,  continúan  sirviendo  como  punto  de  referencia  para  excelentes  investigaciones  prospectivas  aleatorias.  En  este  
estudio,  1851  mujeres  fueron  aleatorizadas  después  de  que  se  extirpó  el  tumor  de  mama  y  se  documentó  el  estado  de  los  
ganglios.  Los  autores  concluyen  que  la  lumpectomía  seguida  de  irradiación  mamaria  es  el  tratamiento  adecuado.  Para  apreciar  
los  enormes  problemas  de  interpretación  de  los  ensayos  clínicos,  debe  leer  este  artículo  detenidamente.  La  radiación  disminuyó  
la  muerte  por  cáncer  de  mama,  pero  esta  reducción  fue  parcialmente  contrarrestada  por  un  aumento  en  las  muertes  por  otras  
causas.

4.  Barnett  HJ,  Taylor  DW,  Eliasziw  M  et  al.:  Beneficio  de  la  endarterectomía  carotídea  en  pacientes  con
estenosis  moderada  o  grave,  N  Engl  J  Med  339:1415­1425,  1998.

Este  es  el  Ensayo  de  endarterectomía  carotídea  sintomática  de  América  del  Norte  (NASCET)  iniciado  en  1987.  NASCET  asignó  al  
azar  a  pacientes  con  estenosis  carotídea  grave  (70  %  a  99  %)  y  estenosis  moderada  (<70  %)  a  tratamiento  médico  estándar  o  
endarterectomía  carotídea  (CEA).  Para  1991,  la  clara  ventaja  de  la  cirugía  en  pacientes  sintomáticos  con  estenosis  severa  era  
tan  clara  que  se  detuvo  el  estudio  para  este  grupo.  Este  manuscrito  informa  una  reducción  de  5  años  en  el  accidente  
cerebrovascular  ipsilateral  del  22,2  %  (médico)  al  15,7  %  (quirúrgico)  (p  ¼  0,045)  en  pacientes  con  estenosis  moderada  (50  %  a  
69  %).  Una  vez  que  un  paciente  con  enfermedad  de  la  carótida  se  vuelve  sintomático,  eso  es  siniestro.  Mientras  presencia  varias  
enfermedades,  compila  inconscientemente  una  lista  de  enfermedades  que  no  desea.  Una  gran  quemadura  y  un  gran  accidente  
cerebrovascular  están  en  la  parte  superior  de  la  lista  de  todos.

5.  Endarterectomía  por  estenosis  de  la  arteria  carótida  asintomática.  Comité  Ejecutivo  para  los  Asintomáticos
Estudio  de  aterosclerosis  carotídea,  JAMA  273:1421­1428,  1995.

El  Estudio  de  aterosclerosis  carotídea  asintomática  (ACAS)  aleatorizó  a  1662  pacientes  asintomáticos  con
Estenosis  de  la  arteria  carótida  >60%  a  prescripción  médica  (una  aspirina  al  día  más  modificación  de  factores  de  riesgo)  o  CEA.  
Después  de  solo  2,7  años,  el  riesgo  proyectado  de  ictus  homolateral  y  muerte  a  5  años  fue  del  5,1  %  en  el  grupo  quirúrgico  y  del  
11  %  en  el  grupo  médico.  Este  es  un  riesgo  agregado  (incluido  el  problema  perioperatorio)
Machine Translated by Google
CAPÍTULO  1  ¿ESTÁS  LISTO  PARA  TU  ROTACIÓN  QUIRÚRGICA?  15

reducción  del  53%.  Este  artículo  concluye  que  un  paciente  asintomático  con  una  lesión  de  la  arteria  carótida  del  60%  o  más,  
que  tiene  un  riesgo  aceptable  (la  aterosclerosis  es  una  enfermedad  sistémica)  para  una  cirugía  electiva,  disfrutará  de  una  
reducción  en  el  riesgo  de  accidente  cerebrovascular  ipsilateral  a  los  5  años  si  la  cirugía  se  puede  realizar.  con  menos  del  3%  de  
morbilidad  o  mortalidad  agregada.

6.  Selzman  CH,  Miller  SA,  Zimmerman  MA  et  al.:  El  caso  del  bloqueo  beta­adrenérgico  como  profilaxis  contra  la  morbilidad  y  mortalidad  
cardiovascular  perioperatoria,  Arch  Surg  136:286­290,  2001.
Cuando  los  pacientes  sufren  morbilidad  y  mortalidad  perioperatorias,  el  sistema  cardiovascular  suele  ser  el  culpable.
Los  pacientes  con  enfermedad  de  las  arterias  coronarias  (CAD)  no  pueden  aumentar  el  flujo  sanguíneo  coronario  para  satisfacer  
la  mayor  demanda  de  oxígeno  asociada  con  el  estrés  quirúrgico.  El  bloqueo  beta­adrenérgico  disminuye  el  consumo  de  oxígeno  
del  miocardio  y  los  betabloqueantes  cardioselectivos  no  exacerban  el  broncoespasmo  en  pacientes  con  enfermedad  pulmonar  
obstructiva  crónica  (EPOC).  Estos  autores  argumentan  que  todos  los  pacientes  mayores  de  40  años  se  beneficiarán  del  bloqueo  beta­
adrenérgico  iniciado  2  semanas  antes  de  la  cirugía  electiva.

7.  Van  den  Berghe  G,  Wouters  P,  Weekers  F  et  al.:  Terapia  intensiva  de  insulina  en  pacientes  críticamente  enfermos,  N  Engl
J  Med  345:1359­1367,  2001.

Tanto  la  hiperglucemia  como  la  resistencia  a  la  insulina  son  características  de  los  pacientes  críticos.  Estos  autores  asignaron  al  azar  a  
1548  pacientes  de  la  unidad  de  cuidados  intensivos  quirúrgicos  (UCIC)  a  un  control  intensivo  de  la  glucosa  en  sangre  (mantenido  en  
80  a  110  mg/100  dl)  o  a  una  terapia  convencional  (administrar  insulina  solo  si  la  glucosa  en  sangre  supera  los  215  mg/100  dl).
El  control  intensivo  de  la  glucosa  disminuyó  la  mortalidad  en  la  unidad  de  cuidados  intensivos  (UCI)  del  8  %  al  4,6  %  (p  ¼  0,04),  con  el  
mayor  impacto  en  pacientes  con  insuficiencia  multiorgánica  por  un  foco  séptico.
En  cirugía,  la  atención  al  detalle  cuenta  mucho:  &  
Mantenga  el  nivel  de  azúcar  en  la  sangre  entre  80  y  110  mg/100  dl.

&  Administre  antibióticos  profilácticos  de  0  a  2  horas  antes  de  la  operación  para  que  el  paciente  tenga  un  buen  nivel  de  antibióticos  
en  sangre  en  el  momento  de  la  incisión.

&  Mantenga  a  su  paciente  abrigado  (37  C).
&  La  hiperoxia  reduce  la  infección.

8.  Van  De  Vijver  MJ,  He  YD,  van't  Veer  LJ  et  al.:  Una  firma  de  expresión  génica  como  predictor  de  supervivencia  en  el  cáncer  de  mama,  N  
Engl  J  Med  347:1999­2009,  2002.

Los  autores  postulan  que  70  de  nuestros  35.000  genes  dictan  el  carácter  del  cáncer  de  mama.  Entonces,  el  cáncer,  a  diferencia  de  la  
fibrosis  quística  y  la  enfermedad  de  células  falciformes,  requiere  una  constelación  de  mutaciones  genéticas,  no  solo  una.  Siguieron  
a  295  pacientes  durante  12  años  e  informaron  que  esta  "firma  genética  70"  predice  la  supervivencia  mejor  que  los  indicadores  clásicos  
de  la  edad  del  paciente,  el  tamaño  del  tumor,  la  histología  del  tumor,  el  grado  patológico,  el  estado  del  receptor  hormonal  e  incluso  la  
enfermedad  de  los  ganglios  linfáticos.  Este  último  es  la  sorpresa.  Los  autores  observan  que  la  metástasis  a  distancia  te  mata,  los  
ganglios  linfáticos  positivos  no.  En  pacientes  con  ganglios  linfáticos  positivos  o  negativos,  el  perfil  genético  determina  la  supervivencia.  
Cada  cáncer  no  adquiere  la  capacidad  de  hacer  metástasis  a  medida  que  crece,  esa  capacidad  está  programada  en  la  primera  célula  
neoplásica  que  establece  residencia  en  su  paciente.

9.  Sandham  JD,  Hull  RD,  Brant  RF  et  al.:  Un  ensayo  controlado  aleatorizado  del  uso  de  la  arteria  pulmonar
catéteres  en  pacientes  quirúrgicos  de  alto  riesgo,  N  Engl  J  Med  348:5­14,  2003.

Este  es  un  excelente  estudio  en  el  que  1994  pacientes  quirúrgicos  de  la  UCI  fueron  asignados  al  azar  a  una  terapia  dirigida  por  objetivos  
guiada  por  un  catéter  de  arteria  pulmonar  (AP)  o  atención  estándar  sin  un  catéter  de  AP.  Los  pacientes  estaban  enfermos  y,  para  ser  
incluidos  en  la  aleatorización,  debían  tener  más  de  60  años,  tener  un  riesgo  estimado  de  clase  III  o  IV  ASA  (enfermedad  mayor)  y  estar  
programados  para  cirugía  electiva  o  urgente.  La  mortalidad  hospitalaria  y  la  supervivencia  a  los  6  y  12  meses  fueron  esencialmente  
idénticas.  Tras  años  de  apasionado  debate,  la  utilidad  de  un  catéter  PA,  incluso  en  pacientes  quirúrgicos  enfermos,  ya  no  puede  
justificarse.  Por  el  contrario,  si  después  de  haberle  administrado  fluidos  y  agentes  cardiotónicos  en  dosis  bajas,  su  paciente  no  mejora  
o  aún  presenta  un  cuadro  confuso,  coloque  un  catéter  PA  y  obtenga  más  información.  Cuando  su  paciente  mejore,  sáquelo.

10.  Harken  AH:  Ya  es  suficiente,  Arch  Surg  134:1061­1063,  1999.

Este  artículo  explora  la  responsabilidad  del  cirujano  de  evaluar  el  riesgo  quirúrgico,  relacionar  el  riesgo  con  el  beneficio  fisiológico  y  
psicológico  anticipado  y  desarrollar  estrategias  de  sentido  común  para  apreciar  la  felicidad  individual  del  paciente.  Cuando  los  beneficios  
superan  los  riesgos  operativos  anticipados,  esto  es  fácil,  proceda  con  la  cirugía.  Cuando  los  riesgos  exceden  los  beneficios,  esto  puede  
ser  incómodo,  pero  el  reconocimiento  sensible  de  este  problema  relativamente  común  por  parte  del  cirujano  puede  limitar  la  extensión  
del  duelo  del  paciente  y  la  familia,  evitar  el  despilfarro  de  recursos  limitados  y  desviar  apropiadamente  la  culpa  de  la  toma  de  decisiones  
de  la  familia  hacia  la  familia.  cirujano.

11.  Eatock  FC,  Chong,  P,  Menezes  N  et  al.:  Un  estudio  aleatorizado  de  alimentación  nasogástrica  temprana  versus  nasoyeyunal  en  pancreatitis  
aguda  grave,  Am  J  Gastroenterol  100:432­439,  2005.

La  alimentación  temprana  en  algunos  pacientes  con  pancreatitis  aguda  (PA)  causa  dolor  y  tradicionalmente  se  cree  que  es  el  resultado  
del  empeoramiento  de  la  enfermedad  producido  por  la  estimulación  prematura  del  páncreas.  Evidencia  científica  reciente
Machine Translated by Google
16  CAPÍTULO  1  ¿ESTÁS  LISTO  PARA  TU  ROTACIÓN  QUIRÚRGICA?

sugiere  que  la  sobreestimulación  de  las  células  acinares  pancreáticas  puede  no  ser  la  causa  subyacente  de  la  PA,  lo  que  lleva  a  
los  médicos  a  cuestionar  los  beneficios  de  descansar  el  páncreas.  Se  ha  demostrado  que  el  suministro  de  nutrientes  al  intestino  
delgado  distal  es  beneficioso  durante  la  PA  grave.  Este  estudio  es  el  primer  estudio  prospectivo  aleatorizado  en  el  que  50  
pacientes  adultos  con  PA  grave  fueron  aleatorizados  para  recibir  alimentación  temprana  mediante  sondas  nasogástricas  o  
nasoyeyunales.  Los  criterios  de  valoración  medidos  incluyeron  la  gravedad  de  la  enfermedad  medida  por  lo  enfermo  que  está  el  
paciente  (puntajes  APACHE  II),  la  magnitud  de  la  inflamación  sistémica  (niveles  de  proteína  C  reactiva  [PCR]),  la  progresión  
clínica  y  el  dolor.  Se  observó  una  mortalidad  global  del  24,5%,  sin  diferencias  en  la  mortalidad  entre  los  grupos.  No  se  observaron  
diferencias  en  las  tasas  de  complicaciones,  cambios  en  la  PCR,  cambios  en  APACHE  II  o  cambios  en  el  nivel  de  dolor.  Este  
estudio  es  significativo  porque  desafía  científicamente  el  sesgo  quirúrgico  de  que  descansar  el  páncreas  ayuda  a  los  pacientes  
con  PA  a  recuperarse  más  rápido.

12.  McFalls  EO,  Ward  HB,  Moritz  TE  et  al.:  Revascularización  de  la  arteria  coronaria  antes  de  la  cirugía  vascular  mayor  electiva,  N  Engl  
J  Med  351:2795­2804,  2004.
Este  es  un  ensayo  aleatorizado  prospectivo  de  la  Administración  de  Veteranos  que  se  realizó  para  evaluar  los  beneficios  de  la  
revascularización  coronaria  preoperatoria  en  pacientes  sometidos  a  cirugía  vascular  mayor.  Quinientos  diez  pacientes  fueron  
aleatorizados  para  revascularización  coronaria  mediante  injerto  de  derivación  de  arteria  coronaria  (CABG),  abordaje  percutáneo  
o  tratamiento  médico  estándar.  Las  características  de  los  pacientes  fueron  similares  en  ambos  grupos;  El  40%  eran  diabéticos,  
el  45%  eran  fumadores,  el  40%  con  antecedentes  de  infarto  de  miocardio  (IM),  el  30%  con  enfermedad  coronaria  de  tres  
vasos  y  el  20%  con  antecedentes  de  accidente  cerebrovascular  (ACV)  o  accidente  isquémico  transitorio  (AIT).  El  resultado  del  
paciente  se  evaluó  durante  la  hospitalización  y  en  el  seguimiento  a  largo  plazo.  Los  resultados  no  mostraron  diferencias  en  las  
complicaciones  posoperatorias  ni  en  las  tasas  de  mortalidad  hospitalaria  entre  los  grupos  tratados.  A  los  2,7  años  después  de  la  
aleatorización,  no  se  observaron  diferencias  en  la  mortalidad  entre  los  grupos.
Se  produjeron  retrasos  significativos  en  el  tratamiento  en  los  pacientes  de  revascularización  preoperatoria  (54  días  frente  a  
18  días).  Estos  resultados  demostraron  que,  a  menos  que  los  pacientes  presenten  síndrome  coronario  agudo  (SCA),  no  hay  
beneficios  claros  a  corto  o  largo  plazo  en  la  revascularización  coronaria  de  rutina  antes  de  los  procedimientos  quirúrgicos  
vasculares  mayores.

13.  Andre  T,  Boni  C,  Mounedji­Boudiaf  L  et  al.:  Oxaliplatino,  fluorouracilo  y  leucovorina  como  tratamiento  adyuvante  para  el  cáncer  de  
colon,  N  Engl  J  Med  2343­2351,  2004.
Aproximadamente  la  mitad  de  los  pacientes  que  se  someten  a  cirugía  curativa  por  cáncer  colorrectal  recae  y  muere  a  causa  
de  la  enfermedad  metastásica.  La  presencia  o  ausencia  de  metástasis  en  los  ganglios  linfáticos  es  uno  de  los  factores  
pronósticos  más  importantes  para  la  supervivencia.  Estudios  previos  han  demostrado  que  los  pacientes  con  cáncer  de  colon  en  
etapa  III  (ganglio  positivo)  han  mejorado  la  supervivencia  con  la  terapia  adyuvante  con  5­FU  y  leucovorina  (FL)  en  comparación  
con  la  cirugía  sola.  Este  ensayo  de  control  aleatorizado  comparó  FL  con  FL  más  oxaliplatino  durante  6  meses  en  pacientes  con  
cáncer  de  colon  en  estadio  II  y  estadio  III.  El  punto  final  primario  fue  la  supervivencia  libre  de  enfermedad.  Se  aleatorizaron  más  
de  1100  pacientes  a  cada  brazo  del  estudio  y,  después  de  una  mediana  de  seguimiento  de  40  meses,  se  observó  una  diferencia  
estadísticamente  significativa  en  la  supervivencia  entre  los  grupos  (26,1  %  frente  a  21,1  %;  p  =  0,002).  La  supervivencia  libre  de  
enfermedad  para  los  grupos  fue  significativamente  diferente  en  78,2  %  versus  72,9  %.  Las  complicaciones  relacionadas  con  el  
tratamiento,  incluidos  los  síntomas  gastrointestinales,  la  neuropatía  sensorial  y  la  fiebre,  ocurrieron  con  más  frecuencia  en  los  
pacientes  con  FL  +  oxaliplatino.  Los  análisis  de  subgrupos  mostraron  el  mayor  beneficio  entre  los  pacientes  en  etapa  III.  Este  
estudio  se  denomina  ensayo  MOSAIC  y  es  responsable  de  los  estándares  actuales  de  tratamiento  de  quimioterapia  para  pacientes  
con  cáncer  de  colon  en  etapa  III.

14.  Fitzgibbons  RJ  Jr,  Giobbie­Hurder  A,  Gibbs  JO  et  al.:  Espera  vigilante  frente  a  reparación  de  hernia  inguinal  en  hombres  con  
síntomas  mínimos:  un  ensayo  clínico  aleatorizado,  JAMA  295:285­292,  2006.

Decidir  si  operar  y  cuándo  es  una  de  las  decisiones  más  importantes  que  tomará  como  cirujano.  Este  ensayo  puso  a  
prueba  esa  decisión  en  lo  que  respecta  a  los  hombres  con  hernias  inguinales  mínimamente  sintomáticas.  Fitzgibbons  
es  un  experto  reconocido  a  nivel  nacional  en  el  campo  de  la  cirugía  de  hernias  y  presentó  los  resultados  de  este  
estudio  prospectivo,  aleatorizado  y  multicéntrico  en  la  Serie  Grand  Round  Master  de  la  Society  of  American  
Gastrointestinal  and  Endoscopic  Surgeons  (SAGES)  (para  ver  el  video,  visite:  www.  medscape.com/viewarticle/553466).
En  este  ensayo,  720  hombres  con  hernias  inguinales  levemente  sintomáticas  fueron  aleatorizados  en  dos  grupos:  conducta  
expectante  versus  reparación  sin  tensión.  Fueron  seguidos  durante  2  a  4,5  años.  No  se  encontraron  diferencias  significativas  
entre  los  dos  grupos  en  función  de  los  resultados  principales  del  ensayo,  el  dolor  y  la  incomodidad  que  interfieren  con  la  actividad,  
y  los  cambios  desde  el  inicio  en  la  puntuación  del  componente  físico  (PCS)  de  la  calidad  de  vida  relacionada  con  la  salud  del  
formulario  corto  36.  encuesta.  Por  lo  tanto,  la  conducta  expectante  en  este  subgrupo  de  pacientes  es  admisible  porque  el  riesgo  
de  encarcelamiento  es  poco  frecuente  (1,8/1000  pacientes­año).

15.  Neumayer  L,  Giobbie­Hurder  A,  Jonasson  O  et  al.:  Reparación  de  hernia  inguinal  con  malla  abierta  frente  a  malla  laparoscópica,  N  
Engl  J  Med  350:1819­1827,  2004.
Con  el  desarrollo  de  la  cirugía  mínimamente  invasiva  a  fines  de  la  década  de  1980,  muchas  operaciones,  incluida  la  reparación  
de  hernias  inguinales,  se  adaptaron  al  abordaje  laparoscópico.  Las  ventajas  de  la  reparación  laparoscópica  incluyen
Machine Translated by Google
CAPÍTULO  1  ¿ESTÁS  LISTO  PARA  TU  ROTACIÓN  QUIRÚRGICA?  17

significativamente  menos  dolor  postoperatorio  y  un  retorno  más  rápido  a  las  actividades  habituales.  Sin  embargo,  la  laparoscopia  
conlleva  riesgos.  El  mejor  enfoque  para  la  reparación  de  las  hernias  inguinales  ha  sido  controvertido,  multifactorial  y  poco  concluyente.
Las  operaciones  laparoscópicas  deben  realizarse  bajo  anestesia  general,  y  existe  un  mayor  potencial  de  complicaciones  
graves,  que  incluyen,  entre  otras,  perforación  intestinal  y  lesión  de  vasos  importantes.  Muchos  estudios  han  demostrado  una  
ventaja  general  de  las  técnicas  laparoscópicas  sobre  las  abiertas  sin  tensión,  pero  la  mayoría  de  estos  estudios  se  realizaron  en  
centros  especializados.  Este  gran  ensayo  multicéntrico,  prospectivo  y  aleatorizado,  realizado  en  la  Administración  de  Veteranos,  
es  la  excepción  notable  y  puede  ser  más  representativo  de  la  población  general.  Dos  mil  ciento  sesenta  y  cuatro  pacientes  fueron  
asignados  al  azar  a  reparación  laparoscópica  versus  Lichtenstein  o  abierta  sin  tensión  de  hernias  inguinales.  Aunque  los  pacientes  
en  el  grupo  laparoscópico  tuvieron  menos  dolor  y  regresaron  al  trabajo  antes,  la  recurrencia  fue  significativamente  más  común  (10,1  
%  versus  4,9  %).  Con  base  en  la  recurrencia  y  la  seguridad,  se  encontró  que  la  reparación  abierta  sin  tensión  era  superior  a  la  
reparación  laparoscópica.  Este  estudio  y  el  editorial  subsiguiente  del  Dr.  Jacobs  plantean  muchas  preguntas  con  respecto  a  las  
curvas  de  aprendizaje  para  los  procedimientos  laparoscópicos,  la  habilidad  del  cirujano  y  la  futura  capacitación  de  los  residentes.

16.  Poldermans  D,  Boersma  E,  Bax  J  et  al.:  El  efecto  del  bisoprolol  sobre  la  mortalidad  perioperatoria  y  el  infarto  de  miocardio  en  pacientes  
de  alto  riesgo  sometidos  a  cirugía  vascular,  N  Engl  J  Med  341:1789­1794,  1999.
Este  estudio  es  un  buen  seguimiento  del  estudio  de  Mangano  y  colegas  que  evaluó  los  efectos  cardioprotectores  del  bloqueo  beta  
en  pacientes  sometidos  a  cirugía  mayor  (N  Eng  J  Med  335:1713­1720,  1996).  La  población  de  pacientes  en  el  estudio  de  
Mangano  tenía  o  estaba  en  riesgo  de  CAD  y  se  sometió  a  varios  procedimientos  quirúrgicos.  En  un  seguimiento  de  2  años,  
encontraron  que  el  bloqueo  beta  no  redujo  significativamente  la  incidencia  de  infarto  de  miocardio  perioperatorio  o  muerte  por  
causas  cardíacas  durante  la  hospitalización.  La  población  de  pacientes  estudiada  no  tenía  un  alto  riesgo  de  complicaciones  
cardíacas  perioperatorias  y,  por  lo  tanto,  no  pudieron  mostrar  un  beneficio  significativo  del  bloqueo  beta  perioperatorio.  Para  
demostrar  la  ventaja  del  bloqueo  beta  perioperatorio,  Poldermans  y  sus  colegas  seleccionaron  pacientes  con  alto  riesgo  de  
complicaciones  cardíacas  según  las  pruebas  preoperatorias,  incluida  la  ecocardiografía  con  dobutamina  positiva.  También  eligieron  
pacientes  que  se  sometían  específicamente  a  procedimientos  vasculares.  En  esta  población  de  alto  riesgo,  el  bloqueo  beta  de  hecho  
redujo  significativamente  la  mortalidad  perioperatoria  por  causas  cardíacas  y  el  infarto  de  miocardio  no  fatal  en  un  34%.  Este  es  un  
gran  ejemplo  de  la  importancia  de  estratificar  el  riesgo  de  los  pacientes  con  precisión.  En  el  futuro,  todas  las  "boletas  de  
calificaciones"  de  los  cirujanos  serán  de  conocimiento  público.  Es  fundamental  para  nosotros  estratificar  con  precisión  el  riesgo  de  
nuestros  pacientes.
Las  recomendaciones  para  el  uso  de  betabloqueantes  perioperatorios  en  pacientes  quirúrgicos  de  alto  riesgo  son:
&  El  bloqueo  beta  debe  iniciarse  1  a  2  semanas  antes  de  la  operación
&  Ritmo  cardíaco  objetivo  preoperatorio  inferior  a  70  latidos  por  minuto
&  Frecuencia  cardíaca  posoperatoria  inmediata  inferior  a  80  latidos  por  minuto

17.  Giger  UF,  Michel  JM,  Opitz  I  et  al.:  Factores  de  riesgo  de  complicaciones  perioperatorias  en  pacientes  sometidos  a  colecistectomía  
laparoscópica:  análisis  de  22  953  casos  consecutivos  de  la  base  de  datos  de  la  Asociación  Suiza  de  Cirugía  Laparoscópica  y  
Toracoscópica,  J  Am  Coll  Surg  203:723­728 ,  2006.
Utilizando  la  base  de  datos  suiza,  los  autores  identificaron  una  serie  de  factores  de  riesgo  de  complicaciones  locales  y  sistémicas  en  
pacientes  sometidos  a  colecistectomía  laparoscópica  (CL).  No  se  reportan  sorpresas  durante  esta  investigación;  sin  embargo,  los  
hallazgos  parecen  ser  útiles  para  que  todos  los  reconozcamos  de  modo  que  podamos  ajustar  y  controlar  las  variables  relacionadas  
con  el  cirujano,  incluidos  los  niveles  de  habilidad  del  aprendiz  y  el  supervisor  involucrado  en  los  casos  complejos  y  el  momento  de  
la  cirugía  para  los  pacientes  complejos.

18.  Hebert  P,  Wells  G,  Blajchman  M  et  al.:  Un  ensayo  clínico  multicéntrico,  aleatorizado  y  controlado  de  transfusión
requisitos  en  cuidados  críticos,  N  Eng  J  Med  340:409­417,  1999.
Los  glóbulos  rojos  son  responsables  del  suministro  de  oxígeno  a  los  tejidos,  y  el  aumento  del  suministro  de  oxígeno  es
generalmente  se  presume  que  es  beneficioso  en  pacientes  críticamente  enfermos;  por  lo  tanto,  un  umbral  de  transfusión  de  
(hemoglobina)  10,0  g  a  menudo  se  había  considerado  aceptable  en  el  entorno  de  cuidados  intensivos.  Tanto  los  riesgos  como  los  
beneficios  de  las  transfusiones  de  sangre  pueden  ser  significativos.  Dado  que  las  transfusiones  de  sangre  se  asocian  con  infusión  
de  volumen  excesivo,  inmunosupresión  y  transmisión  de  infecciones,  los  beneficios  de  una  estrategia  de  transfusión  liberal  no  se  
habían  establecido  claramente  y  exponían  potencialmente  a  muchos  pacientes  que  no  necesariamente  necesitaban  una  transfusión  
a  riesgos  evitables.  Este  ensayo  multicéntrico,  aleatorizado  y  controlado  asignó  al  azar  a  838  pacientes  euvolémicos  de  cuidados  
intensivos  a  una  estrategia  de  transfusión  "restrictiva"  o  "liberal".  En  el  grupo  restrictivo,  los  pacientes  recibieron  glóbulos  rojos  
cuando  su  hemoglobina  cayó  por  debajo  de  7  g/dl.  En  el  grupo  liberal,  los  pacientes  recibieron  transfusión  de  hemoglobina  de  10  g/
dl.  Los  pacientes  que  estaban  menos  gravemente  enfermos  y  eran  menores  de  55  años,  tenían  una  mortalidad  a  los  30  días  mucho  
más  baja  en  el  brazo  restrictivo  del  estudio  que  los  del  grupo  liberal  (8,7%  a  16,1%  y  5,7%  a  13%,  respectivamente).  Los  pacientes  
del  grupo  restrictivo  también  recibieron  menos  transfusiones  (media  de  2,6  unidades  frente  a  5,6  unidades)  y  experimentaron  una  
menor  mortalidad  hospitalaria  (22,2  %  frente  a  28,1  %;  p  =  0,05).  Los  eventos  cardíacos,  incluidos  el  edema  pulmonar  y  el  infarto  
de  miocardio,  ocurrieron  con  mayor  frecuencia  entre  los  pacientes  que  recibieron  transfusiones  liberales  durante  su  estadía  en  la  
UCI.  Estos  hallazgos  sugieren  que  una  estrategia  de  transfusión  restrictiva
Machine Translated by Google
18  CAPÍTULO  1  ¿ESTÁS  LISTO  PARA  TU  ROTACIÓN  QUIRÚRGICA?

con  valores  de  hemoglobina  de  7,0  a  9,0  se  puede  aplicar  con  seguridad  a  la  mayoría  de  los  pacientes  en  estado  crítico,  con  la  excepción  
de  los  pacientes  con  SCA.  Al  arrojar  luz  sobre  los  factores  desencadenantes  tradicionales  de  las  transfusiones,  este  ensayo  alienta  a  los  
médicos  a  justificar  el  uso  y  evaluar  los  riesgos  y  los  beneficios  de  las  transfusiones  de  sangre.

19.  The  Clinical  Outcomes  of  Surgical  Therapy  Study  Group.  Una  comparación  de  laparoscopia  asistida  y  abierta
colectomía  por  cáncer  de  colon,  N  Engl  J  Med  350:2050­2059,  2004.

Los  estudios  que  comparan  operaciones  abdominales  abiertas  y  laparoscópicas  generalmente  han  demostrado  tiempos  más  cortos
hospitalización  y  recuperación  de  pacientes  tratados  por  laparoscopia;  sin  embargo,  como  resultado  de  preocupaciones  con  
resecciones  oncológicas  inadecuadas  y  un  posible  compromiso  en  la  supervivencia  del  paciente,  la  colectomía  laparoscópica  
no  había  sido  ampliamente  aceptada  para  el  tratamiento  del  cáncer  de  colon.  Este  ensayo  de  control  aleatorio  fue  diseñado  
para  evaluar  los  resultados  en  pacientes  sometidos  a  colectomía  laparoscópica  por  cáncer  de  colon.  Un  total  de  872  pacientes  
fueron  aleatorizados  a  colectomía  abierta  o  colectomía  laparoscópica,  con  características  demográficas  de  pacientes  y  
distribuciones  de  ubicaciones  de  tumores  similares  en  ambos  brazos  de  tratamiento.  Los  hallazgos  del  estudio  no  indicaron  
diferencias  en  las  tasas  de  complicaciones,  la  mortalidad  a  los  30  días  y  el  estado  del  margen  quirúrgico  entre  los  brazos  de  
tratamiento.  Sin  embargo,  la  recuperación  perioperatoria  fue  más  rápida  entre  los  pacientes  tratados  por  laparoscopia  con  una  
estadía  hospitalaria  significativamente  más  corta  y  una  duración  reducida  del  uso  de  analgésicos  narcóticos  informados.  A  los  3  
años  de  seguimiento,  no  hubo  diferencias  en  las  tasas  de  recurrencia,  la  supervivencia  general  y  la  supervivencia  libre  de  
enfermedad.  Estos  resultados,  junto  con  hallazgos  similares  informados  en  un  ensayo  europeo  (Lancet  Oncol  6:477­484,  2005),  
han  establecido  claramente  la  colectomía  laparoscópica  como  un  tratamiento  quirúrgico  aceptable  para  el  cáncer  de  colon.  Para  
ver  una  colectomía  laparoscópica  por  cáncer,  visite  www.websurg.com.

20.  Lee  T,  Marcantonio  E,  Mangione  C  et  al.:  Derivación  y  validación  prospectiva  de  un  índice  simple  para  la  predicción  del  riesgo  cardíaco  
de  una  cirugía  mayor  no  cardíaca,  Circulation  100:1043­1049,  1999.

Durante  la  evaluación  preoperatoria  se  deben  establecer  y  discutir  con  el  paciente  los  riesgos  y  beneficios  de  la  operación.  El  sistema  
cardiovascular  se  ve  desafiado  durante  el  período  perioperatorio  y  las  complicaciones  cardíacas  conllevan  una  morbilidad  significativa.  
Por  lo  tanto,  la  estratificación  del  riesgo  de  complicaciones  cardíacas  es  fundamental  para  cada  paciente.  Históricamente,  se  diseñaron  
pautas  que  incluían  los  criterios  de  Goldman  y  el  índice  de  riesgo  cardíaco  para  determinar  el  riesgo  cardíaco.  El  uso  de  estos  sistemas  
se  ha  visto  limitado  por  su  complejidad.  Este  estudio  propuso  un  índice  de  riesgo  cardíaco  revisado  (RCRI)  mucho  más  simple  para  
predecir  el  riesgo  de  complicaciones  cardíacas  en  los  principales  procedimientos  no  cardíacos  electivos.  El  estudio  se  realizó  en  un  
hospital  académico  de  gran  reputación  e  incluyó  a  4315  pacientes.  Las  principales  medidas  de  resultado  fueron  las  complicaciones  
cardíacas.  Se  identificaron  seis  predictores  independientes  e  iguales  de  complicaciones,  entre  ellos:  tipo  de  cirugía  de  alto  riesgo,  
antecedentes  de  cardiopatía  isquémica,  antecedentes  de  insuficiencia  cardíaca  congestiva  (ICC),  antecedentes  de  enfermedad  
cerebrovascular,  tratamiento  preoperatorio  con  insulina  y  una  creatinina  sérica  preoperatoria  >  2,0  mg/dl.  El  RCRI  se  puede  calcular  
rápidamente  y  es  una  herramienta  valiosa  que  se  utiliza  actualmente  para  estratificar  con  precisión  el  riesgo  de  pacientes  con  
complicaciones  cardíacas  en  procedimientos  no  cardíacos  electivos  importantes.

21.  Gurm  HS,  Yadav  JS,  Fayad  P  et  al.,  para  los  investigadores  SAPPHIRE:  resultados  a  largo  plazo  de  la  colocación  de  stent  carotídeo
versus  endarterectomía  en  pacientes  de  alto  riesgo,  N  Engl  J  Med  358:1572­1579,  2008.

Los  autores  señalan  que  existe  una  relación  directa  entre  el  grado  de  estenosis  carotídea  y  el  accidente  cerebrovascular  ipsilateral.
En  manos  de  cirujanos  vasculares  e  intervencionistas  experimentados,  esta  enfermedad  puede  ser  manejada  y  los  pacientes  pueden  
anticipar  resultados  extraordinariamente  buenos.  La  morbilidad  y  mortalidad  de  un  CEA  quirúrgico,  incluso  en  pacientes  debilitados,  es  
bastante  baja.  Cuando  se  infla  un  catéter  de  angioplastia  en  la  circulación  cerebral,  existe  el  riesgo  de  que  una  pequeña  parte  de  la  placa  
desmenuzada  flote  hacia  el  norte  y  provoque  pérdida  de  memoria.  Así,  se  desarrolló  una  red  de  pesca  que  se  coloca  distal  al  despliegue  
del  balón  de  angioplastia  y  el  stent  (los  dos  primeros  autores  reconocen  que  son  los  inventores  y  tienen  las  patentes  del  dispositivo  de  
protección  contra  embolias).  En  un  ensayo  prospectivo  aleatorizado  de  260  pacientes,  los  autores  concluyen  que  la  colocación  de  stents  
en  la  arteria  carótida  con  protección  por  el  dispositivo  de  protección  contra  émbolos  "no  es  inferior"  a  la  CEA  al  mes,  al  año  y  a  los  3  años.
Machine Translated by Google

REANIMACIÓN  CARDIOPULMONAR CAPITULO  
2

Amandeep  Singh,  MD

1.  ¿Qué  es  el  paro  cardíaco  y  la  muerte  súbita  cardíaca?

El  paro  cardíaco  es  el  cese  repentino  de  la  función  de  bombeo  cardíaco  eficaz  como  resultado  de  asistolia  ventricular  
(eléctrica  o  mecánica)  o  taquicardia  ventricular  sin  pulso  o  fibrilación  ventricular.  La  muerte  cardíaca  súbita  es  la  
muerte  natural  inesperada  por  una  causa  cardíaca  dentro  de  la  primera  hora  del  inicio  de  los  síntomas;  en  una  persona  
sin  una  condición  previa,  eso  parecería  fatal.

2.  ¿Cuál  es  la  arritmia  más  común  que  se  encuentra  durante  la  muerte  súbita  cardíaca?

La  fibrilación  ventricular  (FV)  es  el  ritmo  predominante  que  se  encuentra  en  los  primeros  3  a  5  minutos  después  de  un  
paro  cardíaco  repentino.  La  FV  se  caracteriza  por  despolarizaciones  y  repolarizaciones  rápidas  y  caóticas  que  hacen  
que  el  corazón  se  estremezca  y  no  pueda  bombear  sangre  de  manera  eficaz.

3.  ¿Cuál  es  el  tratamiento  inicial  para  un  paciente  que  se  encuentra  en  fibrilación  ventricular?
La  terapia  inmediata  con  desfibrilación  es  el  único  tratamiento  efectivo  para  la  FV  y  es  más  efectivo  si  se  realiza  dentro  de  
los  5  minutos  posteriores  al  colapso.  El  inicio  de  la  reanimación  cardiopulmonar  (RCP)  con  compresiones  torácicas  y  
ventilación  proporciona  una  pequeña  pero  crítica  cantidad  de  sangre  al  corazón  y  al  cerebro  mientras  se  espera  que  llegue  
un  desfibrilador.

4.  ¿Es  obligatoria  la  intubación  endotraqueal  durante  la  reanimación  cardiopulmonar?
No.  Se  puede  lograr  una  ventilación  adecuada  con  el  posicionamiento  adecuado  de  las  vías  respiratorias,  una  vía  
respiratoria  orofaríngea  o  nasofaríngea  y  una  máscara  con  válvula  de  bolsa  conectada  a  una  fuente  de  oxígeno.  La  
inserción  de  un  tubo  endotraqueal  se  puede  diferir  hasta  que  el  paciente  no  responda  a  la  RCP  inicial  y  la  desfibrilación.

5.  ¿Cómo  se  posiciona  la  vía  aérea  durante  un  intento  de  reanimación?
En  un  paciente  inconsciente,  la  obstrucción  de  las  vías  respiratorias  más  común  es  la  lengua  del  paciente,  que  vuelve  a  
caer  en  la  garganta  cuando  los  músculos  de  la  garganta  y  la  lengua  se  relajan.  La  apertura  de  las  vías  respiratorias  para  
liberar  la  lengua  de  la  obstrucción  se  puede  realizar  mediante  la  maniobra  de  inclinación  de  la  cabeza  y  elevación  del  
mentón,  o  en  el  paciente  con  sospecha  de  lesión  de  la  columna  cervical,  la  maniobra  de  tracción  mandibular.  Si  está  
disponible,  se  debe  insertar  una  vía  aérea  oral  o  una  trompeta  nasal.

6.  Describir  las  maniobras  de  inclinación  de  la  cabeza,  elevación  del  mentón  y  tracción  de  la  mandíbula.
La  maniobra  de  inclinación  de  la  cabeza  y  elevación  del  mentón  consta  de  dos  maniobras  separadas.  Primero,  se  coloca  una  
mano  en  la  frente  y  se  usa  para  rotar  la  cabeza  a  una  posición  de  "olfateo" (es  decir,  el  cuello  completamente  extendido  y  la  
cabeza  inclinada  hacia  atrás).  En  segundo  lugar,  la  otra  mano  se  usa  para  levantar  el  mentón  hacia  adelante  y  hacia  arriba.
En  la  maniobra  de  tracción  mandibular,  el  rescatador  coloca  ambas  manos  a  los  lados  de  la  cara  de  la  víctima,  agarra  
la  mandíbula  en  su  ángulo  y  levanta  la  mandíbula  hacia  adelante.

7.  ¿Cuál  es  el  método  adecuado  de  compresiones  torácicas  en  niños  y  adultos?
La  posición  adecuada  para  las  manos  durante  las  compresiones  torácicas  en  niños  y  adultos  (alrededor  de  1  año  de  
edad  y  mayores)  es  en  el  centro  del  pecho  en  la  línea  de  los  pezones.  Usando  el  talón  de  ambos

19
Machine Translated by Google
20  CAPÍTULO  2  REANIMACIÓN  CARDIOPULMONAR

manos,  el  rescatador  debe  comprimir  el  pecho  aproximadamente  de  1½  a  2  pulgadas  para  adultos.
El  mismo  método  se  usa  para  los  niños,  sin  embargo,  una  mano  suele  ser  adecuada  para  comprimir  el  tórax  y  la  
profundidad  de  la  compresión  debe  ser  de  un  tercio  a  la  mitad  de  la  profundidad  del  tórax.
Los  rescatistas  deben  empujar  fuerte,  empujar  rápido  (frecuencia  de  100  compresiones  por  minuto),  permitir  que  
el  tórax  retroceda  completamente  entre  las  compresiones  y  minimizar  las  interrupciones  en  las  compresiones  para  
todas  las  víctimas.

8.  ¿Qué  es  la  técnica  de  reanimación  cardiopulmonar  con  compresión  abdominal  interpuesta?

La  técnica  de  RCP  con  compresión  abdominal  interpuesta  utiliza  un  socorrista  dedicado  para  proporcionar  compresión  
manual  del  abdomen  (a  mitad  de  camino  entre  el  xifoides  y  el  ombligo)  durante  la  fase  de  relajación  de  la  compresión  
torácica.  Se  cree  que  esta  técnica  mejora  el  retorno  venoso  durante  la  RCP  y  se  ha  demostrado  que  aumenta  el  retorno  
de  la  circulación  espontánea  y  la  supervivencia  a  corto  plazo  para  las  reanimaciones  intrahospitalarias.

9.  ¿Qué  frecuencia  respiratoria  debe  lograrse  durante  un  intento  de  reanimación?
Los  rescatistas  deben  administrar  de  8  a  10  respiraciones  por  minuto  durante  la  RCP,  con  cada  respiración  administrada  
durante  1  segundo  a  un  volumen  corriente  suficiente  para  producir  una  elevación  del  tórax  (aproximadamente  6  a  7  ml/
kg  o  500  a  600  ml).  Se  pueden  agregar  una  serie  de  dispositivos  disponibles  comercialmente  en  línea  con  un  dispositivo  
de  máscara  de  válvula  de  bolsa  para  ayudar  a  administrar  la  cantidad  adecuada  de  respiraciones  por  minuto.
Debe  evitarse  la  hiperventilación.

10.  ¿Cuáles  son  las  ventajas  de  la  inserción  de  una  vía  central  durante  la  reanimación  cardíaca?
Las  ventajas  de  la  inserción  de  una  vía  central  
y  la  administración  de  grandes  volúmenes  de  líquido  se  pueden  facilitar  con  un  catéter  central  de  gran  calibre.
  Las  concentraciones  máximas  del  fármaco  son  mayores  y  los  tiempos  de  circulación  son  más  breves  con  los  catéteres  centrales.
&  La  inserción  supraclavicular  en  la  vena  subclavia  requiere  una  interrupción  mínima  en  el  tórax.
compresiones
&  Puede  ser  más  rápido  obtener  acceso  central  con  guía  de  ultrasonido  en  comparación  con  acceso  periférico
acceso  en  un  paciente  severamente  hipotenso.
Aunque  existen  ciertas  ventajas  con  la  inserción  de  una  vía  central,  no  hay  datos  que  sugieran  mejores  resultados  
con  la  colocación  de  una  vía  central.  En  la  mayoría  de  los  casos,  una  canulación  intravenosa  (IV)  o  intraósea  (IO)  
periférica  de  gran  calibre  es  adecuada  para  la  reanimación  cardíaca.

11.  ¿Qué  medicamentos  de  soporte  vital  cardíaco  avanzado  han  demostrado  una  mejor  supervivencia  de  pacientes  
neurológicamente  intactos  al  alta  hospitalaria?
No  existen  medicamentos  de  soporte  vital  cardíaco  avanzado  (ACLS,  por  sus  siglas  en  inglés)  que  hayan  demostrado  ser  
útiles  en  este  sentido.  Hasta  la  fecha,  ningún  ensayo  controlado  con  placebo  ha  demostrado  que  la  administración  de  
cualquier  agente  vasopresor  en  cualquier  etapa  de  taquicardia  ventricular  (TV)  sin  pulso,  FV,  actividad  eléctrica  sin  pulso  
(AESP)  o  asistolia  aumente  la  tasa  de  supervivencia  neurológicamente  intacta  hasta  el  alta  hospitalaria.

12.  ¿Cuál  es  la  secuencia  para  el  tratamiento  de  la  fibrilación  ventricular  o  sin  pulso?
¿taquicardia  ventricular?
Tan  pronto  como  sea  posible,  se  deben  administrar  120  a  200  J  de  electricidad  a  través  de  un  desfibrilador  
bifásico  (o  360  J  a  través  de  un  desfibrilador  monofásico).  Un  período  de  RCP  debe  preceder  a  la  desfibrilación  si  el  
paro  no  fue  presenciado.  Inmediatamente  después  de  la  desfibrilación,  se  continúa  con  la  RCP  durante  2  minutos,  
momento  en  el  que  se  realiza  una  breve  verificación  del  pulso  y  el  ritmo  mientras  se  recarga  el  desfibrilador.  En  pacientes  
con  FV/TV  persistentes,  se  debe  reanudar  la  RCP  hasta  que  se  complete  la  carga,  y  solo  detenerse  cuando  se  administra  
la  descarga.  Esta  secuencia  de  CPR­SHOCK­CPR  RHYTHM  CHECK  debe  continuarse  mientras  el  paciente  permanezca  
en  FV/TV.  Cuando  IV  o  IO
Machine Translated by Google
CAPÍTULO  2  REANIMACIÓN  CARDIOPULMONAR  21

acceso  está  disponible,  se  debe  repetir  1  mg  de  epinefrina  IV  o  IO  cada  3  a  5  minutos  o  se  debe  administrar  una  
dosis  única  de  40  U  de  vasopresina  IV  o  IO.  Considere  amiodarona  300  mg  IV  o  IO  (con  150  mg  IV  o  IO  adicionales  
administrados  para  pacientes  con  FV  o  TV  refractaria).  Las  compresiones  torácicas  no  deben  interrumpirse  para  la  
administración  de  medicamentos.

13.  ¿Cuál  es  la  secuencia  para  el  tratamiento  de  la  asistolia/actividad  eléctrica  sin  pulso?
La  RCP  debe  iniciarse  inmediatamente  y  continuarse  durante  2  minutos  antes  de  una  breve  verificación  
del  pulso  y  el  ritmo.  La  RCP  continúa  después  de  la  verificación  del  pulso  y  el  ritmo.  Esta  secuencia  de  CPR­
RHYTHM  CHECK  debe  continuarse  mientras  el  paciente  permanezca  en  asistolia/AESP.
Cuando  se  dispone  de  acceso  IV  o  IO,  se  debe  administrar  1  mg  de  epinefrina  IV  o  IO  repetida  cada  3  a  5  minutos  
o  una  dosis  única  de  40  U  de  vasopresina  IV  o  IO;  Se  administra  1  mg  de  atropina  IV  repetido  cada  3  a  5  minutos  
(hasta  un  máximo  de  tres  dosis)  para  la  asistolia  o  la  tasa  PEA  lenta.  Las  compresiones  torácicas  no  deben  
interrumpirse  para  la  administración  de  medicamentos.

14.  ¿Cuáles  son  los  factores  comunes  tratables  que  contribuyen  al  paro  cardíaco?
Hipovolemia  
Hipoxia  Iones  
de  hidrógeno  (acidosis)
Hipopotasemia  o  hiperpotasemia  
Hipoglucemia  Hipotermia  Toxinas  
Taponamiento  cardíaco  Neumotórax  
a  tensión  Trombosis  coronaria  
Trombosis  pulmonar  Traumatismo  
Una  manera  fácil  de  recordar  la  
etiología  de  un  paro  cardíaco  
tratable  es  recordar  que  hay  seis  
causas  "H"  y  seis  causas  "T".

15.  ¿Tiene  algún  papel  la  fibrinolisis  de  rutina  en  pacientes  con  actividad  eléctrica  sin  pulso?
¿paro  cardiaco?
No.  Los  resultados  de  un  gran  ensayo  clínico  reciente  no  mostraron  ningún  efecto  significativo  del  tratamiento  
cuando  se  administró  un  agente  fibrinolítico  (tPA)  a  pacientes  extrahospitalarios  con  paro  cardíaco  indiferenciado  
que  no  respondieron  a  las  intervenciones  iniciales.  Los  pacientes  individuales  en  paro  cardíaco  en  los  que  existe  
una  fuerte  sospecha  de  PEA  (p.  ej.,  paciente  inmovilizada,  periparto,  trombosis  venosa  profunda  [TVP]  por  
antecedentes  o  sugerida  por  el  examen  físico)  pueden  beneficiarse  del  uso  de  fibrinolíticos  como  último  recurso  de  
por  vida.  intervención  salvadora.

16.  ¿Cuáles  son  los  objetivos  iniciales  del  soporte  posresucitación?
Optimizar  la  función  cardiopulmonar  y  la  perfusión  sistémica,  especialmente  la  perfusión  al  cerebro.
Trate  de  identificar  la  causa  precipitante  del  arresto  e  instituya  medidas  para  evitar  que  vuelva  a  ocurrir.
Instituir  medidas  que  puedan  mejorar  la  supervivencia  neurológicamente  intacta  a  largo  plazo.

17.  ¿Qué  es  la  hipotermia  terapéutica  posterior  a  la  reanimación?
Se  ha  demostrado  que  la  inducción  de  hipotermia  posterior  a  la  reanimación  (enfriamiento  a  32  °C  a  34  °C  
durante  12  a  24  horas)  para  pacientes  comatosos  con  retorno  de  la  circulación  espontánea  conduce  a  un  mejor  
resultado  neurológico  en  pacientes  con  paro  cardíaco.  Aunque  la  mayoría  de  los  estudios  clínicos  de  enfriamiento  
han  utilizado  técnicas  de  enfriamiento  externo  (p.  ej.,  mantas  de  enfriamiento  y  aplicación  frecuente  de  bolsas  de  
hielo),  los  estudios  más  recientes  sugieren  que  las  técnicas  de  enfriamiento  interno  (p.  ej.,  solución  salina  fría,  
catéter  de  enfriamiento  endovascular)  también  pueden  usarse  para  inducir  hipotermia.
Machine Translated by Google
22  CAPÍTULO  2  REANIMACIÓN  CARDIOPULMONAR

18.  ¿Cuál  es  el  papel  del  monitoreo  de  CO2  al  final  de  la  marea ?
La  monitorización  del  dióxido  de  carbono  (CO2)  al  final  de  la  espiración  es  un  indicador  no  invasivo  seguro  y  
eficaz  del  gasto  cardíaco  (CO)  durante  la  RCP.  Durante  un  paro  cardíaco,  se  sigue  generando  CO2  en  todo  el  
cuerpo.  El  principal  determinante  de  la  excreción  de  CO2  es  su  velocidad  de  entrega  desde  los  sitios  de  producción  
periféricos  a  los  pulmones.  En  el  estado  de  flujo  bajo  durante  la  RCP,  la  ventilación  es  relativamente  alta  en  
comparación  con  el  flujo  sanguíneo,  por  lo  que  la  concentración  de  CO2  al  final  de  la  espiración  es  baja.  Si  la  
ventilación  es  razonablemente  constante,  los  cambios  en  las  concentraciones  de  CO2  al  final  de  la  espiración  
reflejan  cambios  en  el  CO.

19.  ¿Qué  anomalías  electrolíticas  pueden  provocar  un  paro  cardíaco?
Las  arritmias  ventriculares  malignas  pueden  resultar  de  niveles  de  potasio  significativamente  elevados  o  
reducidos  y  de  hipomagnesemia.  El  paro  cardiorrespiratorio  puede  resultar  de  hipermagnesemia  severa.

20.  ¿Qué  modificaciones  de  soporte  vital  cardíaco  avanzado  se  requieren  en  pacientes  con  anomalías  electrolíticas  graves?

El  paro  cardíaco  hiperpotasémico  se  puede  observar  en  pacientes  con  insuficiencia  renal,  acidosis  metabólica,  
hemólisis,  lisis  tumoral  por  quimioterapia  y  rabdomiolisis.  También  se  ve  en  pacientes  que  reciben  múltiples  
transfusiones  de  sangre  y  en  aquellos  que  toman  ciertos  medicamentos.  La  muerte  cardíaca  súbita  en  pacientes  
con  hiperpotasemia  grave  documentada  o  sospechada  (>7  mEq/L  con  cambios  tóxicos  en  el  electrocardiograma  [ECG])  
obliga  al  tratamiento  inmediato  con  dosis  IV  de  1000  mg  de  cloruro  de  calcio,  50  mEq  de  bicarbonato  de  sodio,  25  g  de  
glucosa  y  10  unidades  de  insulina  regular.

El  paro  cardíaco  hipopotasémico  se  observa  en  pacientes  con  hipopotasemia  grave  (<2,5  mEq/L).
Se  sospecha  hipopotasemia  en  pacientes  con  alcoholismo,  uso  de  diuréticos,  diarrea  severa  y  diabetes  mellitus  
(DM).  El  tratamiento  inmediato  con  10  mEq  de  potasio  IV  administrado  durante  5  minutos  está  indicado  para  pacientes  
con  arritmia  ventricular  maligna.  La  hipomagnesemia  concurrente  es  común  en  pacientes  con  hipopotasemia.

La  hipomagnesemia  se  observa  en  pacientes  con  alcoholismo,  uso  de  diuréticos,  diarrea  severa,
cetoacidosis  diabética  (CAD)  y  quemaduras  graves.  El  paro  cardíaco  resultante  de  una  hipomagnesemia  
grave  a  menudo  está  precedido  por  torsades  de  pointes  en  un  monitor  cardíaco.  Estos  pacientes  requieren  2  g  
de  MgSO4  IV  durante  5  minutos.
La  parada  cardiorrespiratoria  por  hipermagnesemia  se  trata  con  1000  mg  de  calcio
cloruro  IV  durante  2  minutos,  junto  con  volumen  agresivo  y  soporte  respiratorio.

21.  ¿Cuáles  son  las  causas  comunes  de  paro  cardíaco  por  anafilaxia?
La  anafilaxia  potencialmente  mortal  se  observa  con  reacciones  a  antibióticos  (especialmente  penicilinas  
parenterales  y  otros  betalactámicos),  aspirina  y  fármacos  antiinflamatorios  no  esteroideos,  y  agentes  de  
contraste  intravenosos.  Ciertos  alimentos,  incluidos  los  frutos  secos,  los  mariscos  y  el  trigo,  se  asocian  con  
anafilaxia  potencialmente  mortal  por  broncoespasmo  y  asfixia.

22.  ¿Qué  modificaciones  de  soporte  vital  cardíaco  avanzado  se  requieren  en  pacientes  con  paro  cardíaco  como  
resultado  de  anafilaxia?
El  paro  cardíaco  por  anafilaxia  es  el  resultado  de  una  obstrucción  aguda  de  las  vías  respiratorias  junto  con  una  
profunda  vasodilatación  venosa  que  conduce  al  colapso  cardiovascular.  La  intubación  endotraqueal  temprana,  la  RCP  
prolongada,  la  administración  intensiva  de  volumen  (típicamente  entre  4  y  8  L  de  cristaloides  isotónicos)  y  los  fármacos  
adrenérgicos  son  los  pilares  del  tratamiento.  Los  pacientes  con  paro  cardíaco  completo  pueden  recibir  dosis  altas  de  
epinefrina  (es  decir,  aumentando  de  1  mg  a  3  mg  a  5  mg  durante  5  minutos).  La  cricotiroidotomía  quirúrgica  o  con  aguja  
está  indicada  si  el  edema  de  las  vías  respiratorias  impide  la  intubación  endotraqueal.
Machine Translated by Google
CAPÍTULO  2  REANIMACIÓN  CARDIOPULMONAR  23

23.  ¿Qué  modificaciones  de  soporte  vital  cardíaco  avanzado  se  requieren  en  pacientes  con
paro  cardíaco  asociado  con  trauma?
El  soporte  vital  básico  y  avanzado  para  el  paciente  traumatizado  es  fundamentalmente  el  mismo  que  para  el  paciente  
con  un  paro  cardíaco  primario.  La  hipovolemia,  el  neumotórax  a  tensión  y  el  taponamiento  pericárdico  deben  evaluarse  
y  abordarse  rápidamente  durante  la  reanimación.

24.  ¿Todos  los  pacientes  en  paro  cardíaco  deben  recibir  reanimación  cardiopulmonar?
No.  Las  razones  legítimas  para  suspender  la  RCP  incluyen:  
El  paciente  tiene  una  orden  válida  de  no  reanimación  (DNR).
El  paciente  tiene  signos  de  muerte  irreversible  (p.  ej.,  rigor  mortis,  decapitación,  descompensación  o  lividez  dependiente).

No  se  puede  esperar  ningún  beneficio  fisiológico  porque  las  funciones  vitales  se  han  deteriorado  a  pesar  de  la  
terapia  máxima  (p.  ej.,  shock  séptico  progresivo  o  cardiogénico).

25.  ¿Cuándo  deben  terminar  los  esfuerzos  de  reanimación?
La  decisión  de  terminar  los  esfuerzos  de  reanimación  recae  en  el  médico  tratante  en  el  hospital  y  se  basa  en  la  
consideración  de  muchos  factores,  incluido  el  tiempo  hasta  la  RCP,  el  tiempo  hasta  la  desfibrilación,  la  enfermedad  
comórbida,  el  estado  previo  al  paro  y  el  ritmo  inicial  del  paro.  Ninguno  de  estos  factores  solo  o  en  combinación  es  
claramente  predictivo  del  resultado.  Los  informes  indicaron  que  la  RCP  prolongada  podría  ser  efectiva  en  el  paro  
cardíaco  resultante  de  hipotermia,  sobredosis  de  drogas  y  anafilaxia.

26.  ¿Cuándo  se  debe  iniciar  un  "código  lento"?
Nunca.  La  práctica  de  brindar  reanimación  ineficaz  a  sabiendas  compromete  la  integridad  ética  de  los  
proveedores  de  atención  médica  y  socava  la  relación  médico­paciente  o  enfermera­paciente.

27.  ¿Pueden  estar  presentes  los  miembros  de  la  familia  durante  la  reanimación  de  un  ser  querido?
Sí.  La  mayoría  de  los  miembros  de  la  familia  encuestados  antes  de  una  reanimación  no  solo  afirman  que  les  gustaría  
estar  presentes  durante  un  intento  de  reanimación,  sino  que  muchos  miembros  de  la  familia  dicen  que  es  reconfortante  
estar  al  lado  de  su  ser  querido  y  alivia  el  dolor  asociado  con  un  pérdida  repentina  o  esperada.

28.  ¿Cuáles  son  las  causas  más  comunes  de  paro  cardíaco  perioperatorio  en  niños?
Alrededor  del  50%  de  los  casos  están  relacionados  con  la  anestesia,  el  25%  son  el  resultado  de  la  falta  de  
destete  del  bypass  cardiopulmonar  y  el  20%  están  relacionados  con  una  hemorragia  quirúrgica  no  controlada.  
Entre  los  casos  relacionados  con  la  anestesia,  las  causas  cardiovasculares  fueron  las  más  comunes  (41%  de  todos  
los  paros),  siendo  la  hipovolemia  por  pérdida  de  sangre  y  la  hiperpotasemia  por  transfusión  de  sangre  almacenada  las  
causas  cardiovasculares  identificables  más  comunes.  Entre  las  causas  respiratorias  de  paro  (27%  de  todos  los  paro),  
la  obstrucción  de  las  vías  respiratorias  por  laringoespasmo  fue  la  causa  más  común.  El  paro  cardíaco  relacionado  con  
medicamentos  representó  el  18%  de  todos  los  paros.  La  lesión  vascular  sufrida  durante  la  colocación  de  catéteres  
venosos  centrales  fue  la  causa  de  paro  más  común  relacionada  con  el  equipo.

29.  ¿Qué  es  el  índice  de  riesgo  cardíaco  revisado?

El  índice  de  riesgo  cardíaco  revisado  (RCRI)  predice  con  precisión  eventos  cardíacos  importantes  (p.  ej.,  infarto  de  
miocardio  [IM],  embolia  pulmonar  [EP],  FV,  bloqueo  cardíaco  o  paro  cardíaco)  en  adultos  sometidos  a  cirugía  mayor  
no  cardíaca.  A  cada  factor  de  riesgo  se  le  asigna  un  único  punto:  procedimiento  quirúrgico  de  alto  riesgo,  antecedentes  
de  cardiopatía  isquémica,  antecedentes  de  insuficiencia  cardiaca  congestiva  (ICC),  antecedentes  de  enfermedad  
cerebrovascular,  tratamiento  preoperatorio  con  insulina,  creatinina  sérica  preoperatoria  >  2,0  mg/dl.  El  riesgo  de  un  
evento  cardíaco  mayor  es  <1%  si  no  hay  ningún  factor  de  riesgo  o  uno  está  presente.
Se  produce  un  riesgo  de  evento  cardíaco  mayor  del  6,6  %  si  están  presentes  dos  factores  de  riesgo,  y  aumenta  al  11  
%  si  están  presentes  tres  o  más  factores  de  riesgo.
Machine Translated by Google
24  CAPÍTULO  2  REANIMACIÓN  CARDIOPULMONAR

BIBLIOGRAFÍA

1.  Ali  B,  Zafari  AM:  Revisión  narrativa:  reanimación  cardiopulmonar  y  atención  cardiovascular  de  emergencia:  revisión  de
las  directrices  actuales,  Ann  Intern  Med  147:171­179,  2007.

2.  Asociación  Estadounidense  del  Corazón:  Pautas  de  la  Asociación  Estadounidense  del  Corazón  para  reanimación  cardiopulmonar  y
atención  cardiovascular  de  emergencia,  Circulación  112  (suplemento),  2005.

3.  Bhananker  SM,  Ramamoorthy  C,  Geiduschek  JM  et  al.:  Paro  cardíaco  relacionado  con  la  anestesia  en  niños:  actualización
del  registro  de  paro  cardíaco  perioperatorio  pediátrico,  Anesth  Analg  105:344­350,  2007.

4.  Ibrahim  WH:  Avances  recientes  y  controversias  en  reanimación  cardiopulmonar  de  adultos,  Postgrad  Med  J
83:649­654,  2007.

5.  Lee  TH,  Marcantonio  ER,  Mangione  CM  et  al.:  Derivación  y  validación  prospectiva  de  un  índice  simple  para
predicción  del  riesgo  cardíaco  de  la  cirugía  mayor  no  cardíaca,  Circulation  100:1043­1049,  1999.

6.  Greene  RS,  Howes  D:  Modulación  hipotérmica  de  lesión  cerebral  anóxica  en  sobrevivientes  adultos  de  paro  cardíaco:  Una  revisión  
de  la  literatura  y  un  algoritmo  para  médicos  de  emergencia,  Can  J  Emerg  Med  7:  42­47,  2005.
Machine Translated by Google

EVALUACIÓN  Y  TRATAMIENTO  DE
ARRITMIAS  CARDÍACAS CAPÍTULO  
3

Laurel  R.  Imhoff,  MD,  MPH,  y  Alden  H.  Harken,  MD

1.  ¿Son  lo  mismo  las  arritmias  cardíacas  y  las  arritmias  cardíacas?
Sí.  Algunos  puristas  le  dirán  que  una  arritmia  puede  ser  solo  la  ausencia  de  un  ritmo  cardíaco.  Pero  estos  
son  los  mismos  puristas  que  usan  la  palabra  iatrogénico  para  referirse  a  "causado  por  un  médico",  cuando,  
por  supuesto,  lo  único  que  realmente  puede  ser  "iatrogénico"  son  los  padres  de  un  médico.

2.  ¿Todas  las  arritmias  cardíacas  son  clínicamente  importantes?
La  mayoría  no  lo  son.  Muchos  de  nosotros  tenemos  contracciones  ventriculares  prematuras  (PVC)  aisladas  o  
despolarizaciones  ventriculares  prematuras  (PVD)  todo  el  tiempo.  Los  atletas  excelentemente  acondicionados  
frecuentemente  exhiben  frecuencias  cardíacas  en  reposo  en  los  años  30.  Una  arritmia  cardíaca  clínicamente  importante  
es  un  ritmo  que  molesta  al  paciente.  Como  regla  general,  si  la  frecuencia  ventricular  del  paciente  es  de  60  a  100  latidos  
por  minuto  (independientemente  del  mecanismo),  el  ritmo  cardíaco  no  es  un  problema.

3.  Enunciar  los  objetivos  en  el  tratamiento  de  las  arritmias  cardíacas.
El  objetivo  principal  es  controlar  la  frecuencia  ventricular  entre  60  y  100  latidos  por  minuto,  y  el  objetivo  
secundario  es  mantener  el  ritmo  sinusal.

4.  ¿Qué  importancia  tiene  el  ritmo  sinusal?
Depende  de  la  función  ventricular  del  paciente.  La  inducción  de  fibrilación  auricular  en  un  estudiante  de  
medicina  voluntario  no  provoca  ningún  efecto  hemodinámico  medible.  Su  distensibilidad  ventricular  es  tan  
buena  que  no  necesita  un  "impulso"  auricular  para  llenar  el  ventrículo  por  completo.
Por  el  contrario,  cuanto  peor  (más  rígido)  es  el  corazón  del  paciente,  más  debe  intentar  mantener  el  ritmo  
sinusal.  Observamos  a  un  paciente  con  una  fracción  de  eyección  (FE)  del  ventrículo  izquierdo  (VI)  del  7%  
cuyo  gasto  cardíaco  (GC)  disminuyó  un  40%  cuando  desarrolló  fibrilación  auricular  espontáneamente.

5.  ¿Necesita  estar  hasta  los  tobillos  en  papel  de  electrocardiograma  y  personalmente
familiarizado  con  los  Dres.  Mobitz,  Lown  y  Ganong  para  tratar  las  arritmias  cardíacas  en  la  unidad  de  
cuidados  intensivos  (UCI)?
No.

6.  Cuando  lo  llama  la  enfermera  de  la  UCI  para  ver  a  un  paciente  con  arritmia,  ¿qué  preguntas  se  hace?

1.  ¿Presenta  realmente  el  paciente  una  arritmia?  ¿Qué  está  haciendo  el  paciente?  son  las  cosas
que  se  parece  a  la  fibrilación  ventricular  (FV)  en  realidad  es  solo  el  paciente  que  se  cepilla  los  dientes?
¿O  la  tira  de  ritmo  que  parece  asistolia  es  realmente  solo  una  pista  suelta?  Si  el  paciente  
presenta  una  arritmia,  hágase  las  siguientes  preguntas.

25
Machine Translated by Google
26  CAPÍTULO  3  EVALUACIÓN  Y  TRATAMIENTO  DE  LAS  DISRITMIAS  CARDÍACAS

2.  ¿La  arritmia  requiere  intervención?  Los  PVC  aislados  generalmente  se  pueden  ignorar  de  manera  segura.
Del  mismo  modo,  una  bradicardia  en  reposo  en  un  triatleta  es  normal.  Esta  es  la  ocasión  para  lanzarse  
a  su  "examen  físico  de  2  segundos".  ¿Está  el  paciente  sudoroso  y  confundido  o  alerta  y  feliz?

3.  ¿Qué  es  un  examen  físico  de  2  segundos?  Miras  a  los  ojos  del  paciente,  con  la  esperanza  de
determinar  si  él  o  ella  está  perfundiendo  su  cerebro.  Si  el  paciente  le  devuelve  la  mirada,  tiene  algo  de  
tiempo.  Si  el  paciente  requiere  terapia,  hágase  las  siguientes  preguntas.

4.  ¿Qué  tan  pronto  se  requiere  la  terapia?  En  este  punto,  el  paciente  se  vuelve  (paradójicamente)
irrelevante.  El  indicador  más  sólido  que  dicta  la  velocidad  de  la  intervención  no  es  qué  tan  enfermo  está  el  
paciente,  sino  qué  tan  asustado  está  usted.  Debe  determinar  rápidamente  si  es  probable  que  la  demora  en  
la  terapia  ponga  en  riesgo  al  paciente.  Si  es  probable  que  la  arritmia  cardíaca  inflija  consecuencias  
psicopatológicas  (hipoxémicas)  no  solo  en  el  paciente,  sino  también,  por  extensión,  en  su  familia  extensa  
(sociedad),  debe  tener  miedo.  Si  tiene  miedo,  debe  preguntarse:  5.  ¿Cuál  es  la  terapia  más  segura  y  
efectiva?

7.  Si  el  paciente  requiere  terapia  antiarrítmica,  ¿cuál  es  la  más  segura  y  más
estrategia  efectiva?
La  terapia  para  las  arritmias  cardíacas  es  simple  y  comprende  tres  conceptos  comprensibles:
1.  Si  el  paciente  está  hemodinámicamente  inestable  (el  único  determinante  de  la  inestabilidad  es  si  usted  está  
asustado),  cardiovierta  con  360  J.  (Para  energía  más  baja,  consulte  el  Capítulo  2).
2.  Si  el  paciente  tiene  una  taquicardia  de  complejo  ancho,  cardiovertir  con  360  J.
3.  Si  el  paciente  tiene  una  taquicardia  de  complejo  angosto,  infundir  un  bloqueador  del  nódulo  
auriculoventricular  (AV)  por  vía  intravenosa  (IV).  Si  en  algún  momento  el  paciente  se  vuelve  inestable,  
proceda  con  la  cardioversión.

8.  Al  evaluar  un  impulso  cardíaco,  ¿cómo  distingue  supraventricular  de
origen  ventricular?
Origen  supraventricular:  cuando  un  impulso  se  origina  por  encima  del  nódulo  AV  (supraventricular),  puede  
acceder  a  los  ventrículos  solo  a  través  del  nódulo  AV.  El  nódulo  AV  se  conecta  con  el  sistema  de  Purkinje  
endocárdico,  que  conduce  los  impulsos  rápidamente  (2  a  3  m/seg).  Un  impulso  supraventricular  activa  los  
ventrículos  rápidamente  (<0,08  s,  80  ms,  o  dos  pequeños  cuadros  en  el  papel  del  electrocardiograma  [ECG]),  
produciendo  un  latido  de  complejo  estrecho.
Origen  ventricular:  cuando  un  impulso  se  origina  directamente  desde  un  sitio  ectópico  en  el  ventrículo,  lleva  
más  tiempo  acceder  al  sistema  de  Purkinje  de  alta  velocidad.  Un  impulso  ventricular  activa  lentamente  toda  la  
masa  ventricular  (<0,08  s,  80  ms,  o  dos  pequeños  cuadros  en  el  papel  de  ECG),  produciendo  un  latido  de  
complejo  ancho  (ver  Fig.  3­1).

Figura  3­1.  Los  latidos  de  complejo  ancho  son  de  origen  ventricular.  Los  latidos  de  complejo  estrecho  
son  de  origen  supraventricular.
Machine Translated by Google
CAPÍTULO  3  EVALUACIÓN  Y  TRATAMIENTO  DE  LAS  DISRITMIAS  CARDÍACAS  27

9.  Crédito  adicional:  Correlacione  el  ECG  con  el  flujo  de  iones  de  la  membrana  de  los  cardiomiocitos.
Consulte  la  figura  3­2.

Figura  3­2.  Potencial  de  acción  típico  de  un  miocito  cardíaco,  los  desplazamientos  iónicos  
responsables  de  cada  fase  y  correlación  con  el  ECG  de  superficie.  A,  Fase  0  =  despolarización  
rápida,  caracterizada  por  la  entrada  rápida  de  sodio  (Na+)  a  través  de  los  canales  de  Na+  
dependientes  de  voltaje.  B,  Fase  1  ¼  breve  repolarización,  caracterizada  por  la  entrada  transitoria  
de  cloruro  (Cl­).  C,  Fase  2  ¼  fase  de  meseta,  caracterizada  por  un  rápido  aumento  en  la  permeabilidad  
del  calcio  (Ca2+)  a  través  de  los  canales  de  Ca2+  tipo  L.  Fase  3  ¼  repolarización  con  potasio  (K+)  
saliendo  de  la  célula.  D,  Despolarización  lenta  de  las  células  del  marcapasos  causada  por  la  entrada  
lenta  de  Na+.  (De  Meldrum  DR,  Cleveland  JC,  Sheridan  BC  et  al.:  implicaciones  quirúrgicas  cardíacas  
de  la  dishomeostasis  del  calcio  en  el  corazón,  Ann  Thorac  Surg  61:1273­1280,  1996,  con  autorización).

10.  ¿Todos  los  latidos  de  complejo  ancho  derivan  de  los  ventrículos?
No,  pero  la  mayoría  lo  hace.  Un  impulso  de  origen  supraventricular  que  se  conduce  con  aberración  a  través  del  
ventrículo  puede  tardar  bastante  tiempo  en  convertirse  en  un  latido  de  complejo  ancho.  En  un  estudio,  el  89  %  
de  100  pacientes  que  acudieron  a  un  servicio  de  urgencias  (SU)  con  taquicardia  de  complejo  ancho  finalmente  
demostraron  presentar  taquicardia  ventricular,  mientras  que  al  11  %  se  les  diagnosticó  taquicardia  supraventricular  
con  aberración.
Machine Translated by Google
28  CAPÍTULO  3  EVALUACIÓN  Y  TRATAMIENTO  DE  LAS  DISRITMIAS  CARDÍACAS

11.  ¿Qué  hace  si  no  puede  saber  si  un  complejo  ventricular  es  ancho  o  estrecho?
De  manera  aguda  y  transitoria  (durante  5  segundos)  bloquee  el  nódulo  AV  administrando  6  mg  de  adenosina  IV;  si  el  
complejo  ventricular  persiste,  es  ventricular.  Si  el  complejo  ventricular  se  detiene,  era  supraventricular.

12.  Para  evitar  que  lleguen  muchos  impulsos  supraventriculares  a  los  ventrículos,  ¿cómo  se  bloquea  farmacológicamente  el  nódulo  
auriculoventricular?
En  segundos,  administre  6  mg  de  adenosina  IV.
En  minutos,  extraiga  20  mg  de  Diltiazem  (bloqueador  de  los  canales  de  calcio),  infundir  por  vía  IV  durante  2  minutos.
Si  es  necesario,  inicie  una  infusión  IV  continua  de  5­10  mg/h  inmediatamente  después  del  bolo  IV.  (Para  la  infusión  IV,  
no  exceda  los  15  mg/h  y  el  fármaco  no  debe  infundirse  durante  más  de  24  horas).

En  horas,  poner  0,5  mg  de  digoxina  en  100  ml  de  lactato  de  Ringer  e  infundir  por  goteo  intravenoso  durante  30  minutos.

PUNTOS  CLAVE:  CARACTERIZACIÓN  DEL  CARDIACO
DISRITMIAS
1.  Origen  supraventricular:  cuando  un  impulso  se  origina  por  encima  del  nódulo  AV,  puede  acceder  a  los  ventrículos  
solo  a  través  del  nódulo  AV  para  llegar  al  sistema  de  Purkinje,  que  conduce  y  activa  los  ventrículos  rápidamente,  
produciendo  un  latido  de  complejo  estrecho  (<  2  cajas  pequeñas  en  electrocardiograma).

2.  Origen  ventricular:  cuando  un  impulso  se  origina  en  un  sitio  ectópico  del  ventrículo,  tarda  más  en  acceder  al  sistema  
de  Purkinje  de  alta  velocidad.  Un  impulso  ventricular  activa  toda  la  masa,  produciendo  lentamente  un  latido  de  
complejo  amplio  (>2  cuadros  pequeños  en  el  ECG).

3.  No  todos  los  latidos  de  complejo  ancho  son  de  origen  ventricular.

4.  Para  distinguir  la  taquicardia  ventricular  de  la  supraventricular,  bloquee  transitoriamente  el  nódulo  AV  con  una  inyección  
intravenosa  de  adenosina.  Si  persiste  el  complejo  ventricular,  es  taquicardia  ventricular;  si  el  complejo  se  detiene,  es  
taquicardia  supraventricular.

13.  ¿Por  qué  dar  digoxina?
La  digoxina  es  un  bloqueador  del  nódulo  AV  eficaz,  pero  aumenta  la  excitabilidad  de  los  cardiomiocitos.  Al  administrar  
digoxina,  aumenta  la  probabilidad  de  impulsos  supraventriculares;  pero  al  bloquear  el  nodo  AV,  hace  que  estos  impulsos  
sean  menos  peligrosos.

14.  ¿Por  qué  infundir  digoxina  durante  30  a  60  minutos  por  vía  intravenosa?
Los  estudios  indican  que  un  gran  pulso  de  digoxina  (empuje  IV)  se  concentra  en  el  miocardio,  lo  que  hace  que  los  miocitos  
se  hiperexciten.  La  digoxina  infundida  más  lentamente  evita  este  problema.

15.  Enumere  los  pasos  para  llamar  a  una  arritmia  por  su  nombre.
Bradicardia:  <60  latidos  por  minuto  Taquicardia:  
100  a  250  latidos  por  minuto  Aleteo:  frecuencia  auricular  
o  ventricular  250  a  400  latidos  por  minuto  Fibrilación:  frecuencia  auricular  o  
ventricular  >400  latidos  por  minuto

SITIOS  WEB

www.blaufuss.org/

www.americanheart.org/presenter.jhtml?identifier=10000056#P
Machine Translated by Google
CAPÍTULO  3  EVALUACIÓN  Y  TRATAMIENTO  DE  LAS  DISRITMIAS  CARDÍACAS  29

BIBLIOGRAFÍA

1.  Echahidi  N,  Pibarot  P,  O'Hara  G  et  al.:  Mecanismos,  prevención  y  tratamiento  de  la  fibrilación  auricular  después  de
cirugía,  J  Am  Coll  Cardiol  51:793­801,  2008.

2.  Harken  AH:  Arritmias  cardíacas.  En  Wilmore  DW,  Cheung  L,  Harken  AH  et  al.,  editores:  Scientific  American  Surgery,  Nueva  
York,  1999,  Scientific  American.

3.  Meldrum  DR,  Cleveland  JC,  Sheridan  BC  et  al .:  implicaciones  quirúrgicas  cardíacas  de  la  dishomeostasis  del  calcio  en  el
corazón,  Ann  Thorac  Surg  61:1273­1280,  1996.

4.  Vukanovic­Criley  JM,  Criley  S,  Warde  CM  et  al .:  Competencia  en  habilidades  de  examen  cardíaco  en  estudiantes  de  medicina,
aprendices,  médicos  y  profesores:  un  estudio  multicéntrico,  Arch  Intern  Med  166:  610­616,  2006.

5.  Walsh  SR,  Tang  T,  Wijewardena  C  et  al.:  Arritmias  postoperatorias  en  pacientes  quirúrgicos  generales,  Ann  R  Coll  Surg
Inglés  89:91,  2007.
Machine Translated by Google

CAPÍTULO  
4

CÓMO  PENSAR  EN  EL  CHOQUE
Laurel  R.  Imhoff,  MD,  MPH,  y  Alden  H.  Harken,  MD

1.  Defina  choque.
El  shock  

es:  &  No  solo  presión  arterial  baja.
&  No  sólo  disminución  de  la  perfusión  periférica.
&  No  solo  el  suministro  de  oxígeno  sistémico  limitado.
En  última  instancia,  el  shock  es  una  disminución  de  la  respiración  tisular.  El  shock  es  el  consumo  subóptimo  de  oxígeno  
y  la  excreción  de  dióxido  de  carbono  (CO2)  a  nivel  celular.

2.  ¿El  choque  está  relacionado  con  el  gasto  cardíaco?
Sí.  Un  estudiante  de  medicina  saludable  puede  redistribuir  el  flujo  de  sangre  preferentemente  a  los  órganos  vitales.  
Después  de  una  hemorragia  de  3  a  4  unidades,  el  pistolero  joven  típico  todavía  puede  pensar  y  decirle:  "cuatro  tipos  
me  asaltaron".  todavía  perfundiendo  su  cerebro.

3.  ¿Es  democrática  la  perfusión  de  órganos?
No.  El  flujo  sanguíneo  limitado  siempre  se  redirige  hacia  las  arterias  carótida  y  coronaria.  La  vasoconstricción  periférica  
roba  sangre  inicialmente  del  mesenterio,  luego  del  músculo  esquelético  y  luego  de  los  riñones  y  el  hígado.

4.  ¿Esta  capacidad  de  autorregulación  vascular  es  uniforme  en  todos  los  pacientes?
No.  Con  la  edad  y  la  aterosclerosis,  los  pacientes  pierden  su  capacidad  de  redistribuir  el  flujo  sanguíneo  limitado.
Una  disminución  del  20  %  en  el  gasto  cardíaco  (GC),  o  una  caída  de  la  presión  arterial  a  90  mm  Hg,  puede  
poner  en  peligro  la  vida  de  un  juez  de  la  Corte  Suprema,  mientras  que  puede  ser  indetectable  en  un  triatleta.

5.  Para  fines  diagnósticos  y  terapéuticos  prácticos,  ¿se  puede  clasificar  el  shock?
Sí.

1.  El  shock  hipovolémico  obliga  a  la  reanimación  con  volumen.
2.  El  shock  cardiogénico  requiere  estimulación  cardíaca  (farmacológica  y  eventualmente
mecánico).
3.  El  choque  por  colapso  vascular  periférico  exige  la  manipulación  farmacológica  del
tono  vascular  periférico  (y  atención  directa  a  la  causa  de  la  vasodilatación,  típicamente  sepsis).

6.  ¿Es  recomendable  tratar  todos  los  shocks  de  la  misma  forma  secuencial?
En  última  instancia,  sí.  Ya  sea  que  un  banquero  que  muerde  un  cigarro  presente  una  gran  hemorragia  
gastrointestinal  (GI)  (choque  hipovolémico)  o  un  dolor  opresivo  en  el  pecho  retroesternal  (choque  cardiogénico),  el  
cirujano  debe  seguir  los  siguientes  pasos  en  orden:
1.  Optimizar  el  estado  del  volumen;  administre  volumen  hasta  que  un  mayor  aumento  en  la  precarga  del  lado  
derecho  (presión  venosa  central  [CVP])  y  del  lado  izquierdo  (presión  de  enclavamiento  capilar  pulmonar  [PCWP])  
no  confiera  ningún  beneficio  adicional  para  el  CO  o  la  presión  arterial  (PA).  (Este  paso  es  la  ley  de  Starling;  
coloque  el  corazón  del  paciente  en  la  parte  superior  de  la  curva  de  Starling).

30
Machine Translated by Google
CAPÍTULO  4  CÓMO  PENSAR  EN  EL  CHOQUE  31

2.  Si  el  CO,  la  PA  y  la  perfusión  tisular  siguen  siendo  inadecuados  a  pesar  de  la  precarga  adecuada,  
el  paciente  tiene  un  problema  de  bomba  (shock  cardiogénico).  Infundir  fármacos  inotrópicos  
cardíacos  (agonistas  b)  hasta  el  punto  de  toxicidad  (típicamente  ectopia  cardíaca),  lo  que  provocará  
muchas  contracciones  ventriculares  prematuras  aterradoras.  Para  shock  cardiogénico  
farmacológicamente  refractario,  inserte  una  bomba  de  balón  intraaórtico  (IABP).
3.  Si  el  paciente  muestra  un  GC  sorprendentemente  alto  y  una  PA  paradójicamente  baja  (esta  pérdida  inusual  del  
control  de  la  autorregulación  vascular  se  asocia  típicamente,  pero  no  siempre,  con  sepsis),  infundir  un  fármaco  
vasoconstrictor  periférico  (agonista  alfa).

7.  ¿Cuál  es  la  ruta  de  acceso  preferida  para  la  infusión  de  volumen?
El  flujo  depende  de  la  longitud  y  el  radio  del  catéter.  El  volumen  se  puede  infundir  al  doble  de  la  velocidad  
a  través  de  un  catéter  periférico  de  calibre  14  de  5  cm  que  a  través  de  una  vía  central  de  calibre  16  de  20  cm.
La  evaluación  de  la  PVC  (y  la  presión  de  llenado  del  lado  izquierdo)  es  necesaria  si  el  paciente  no  responde  a  la  
reanimación  con  volumen  inicial.

8.  ¿Se  debe  infundir  cristaloides,  coloides  o  sangre?
Si  el  objetivo  es  solo  mejorar  la  precarga  y  reparar  el  CO  y  la  PA,  la  solución  cristaloide  debería  ser  suficiente.  
Es  controvertido  si  el  coloide  infundido  permanece  en  el  compartimento  vascular.
Si  el  objetivo  es  aumentar  el  suministro  de  oxígeno  sistémico,  los  glóbulos  rojos  fijan  mucho  más  oxígeno  que  el  
plasma  (consulte  el  Capítulo  7).  Los  cristaloides  deben  mejorar  el  flujo  y  la  sangre  debe  aumentar  el  suministro  
de  oxígeno.

9.  Cuando  la  precarga  cardíaca  es  adecuada,  ¿qué  agentes  inotrópicos  son  útiles?
La  dobutamina,  la  epinefrina  y  la  norepinefrina  son  el  chocolate,  la  vainilla  y  la  fresa  de  los  32  sabores  de  las  drogas  
cardiogénicas.  Estos  tres  medicamentos  son  todo  lo  que  el  cirujano  realmente  necesita.

10.  ¿Es  lo  mismo  dopamina  que  dobutamina?
No.  La  dopamina  estimula  los  receptores  dopaminérgicos  renales  y  puede  ser  útil  en  dosis  bajas  (2  
miligramos  por  kilogramo  por  minuto)  para  contrarrestar  la  vasoconstricción  arteriolar  renal  que  acompaña  al  shock.  
La  dopamina  no  tiene  cabida  como  agente  inotrópico  cardíaco  primario.

11.  Discuta  el  uso  de  dobutamina,  epinefrina  y  norepinefrina.
Consulte  la  Tabla  4­1.

TABLA  4­1.  EE.  UU.  EOF  DOBU  T  AMIN  E , E  PIN  E  RARO  E , Y  NO  UN  PIN  Y  UN  PHRIN

La  dobutamina  es  un  agonista  b1  (inotrópico  cardíaco),  pero  también  tiene  algunos  efectos  b2  (vasodilatación  
periférica).

Comience  en:  5  microgramos  por  kilogramo  por  minuto  y  aumente  hasta  el  punto  de  toxicidad  (ectopia  
cardíaca).

Nota:  Infundir  al  efecto  deseado  (no  ceñirse  rígidamente  a  una  dosis  preconcebida).  Debido  a  que  la  
dobutamina  tiene  algunos  efectos  vasodilatadores,  puede  ser  atemorizante  infundirla  en  pacientes  
típicamente  hipotensos  en  estado  de  shock.

La  epinefrina  es  un  agonista  adrenérgico  b  y  a  combinado,  con  los  efectos  b  predominando  en  dosis  más  bajas  
y  la  vasoconstricción  progresiva  que  acompaña  a  dosis  mayores.

Comience  en:  0,05  microgramos  por  kilogramo  por  minuto  y  aumente  hasta  el  punto  de  toxicidad  (ectopia  
cardiaca).

Nota:  Al  igual  que  con  la  dobutamina,  infundir  hasta  lograr  el  efecto  deseado.

(Continuado)
Machine Translated by Google
32  CAPÍTULO  4  CÓMO  PENSAR  SOBRE  EL  CHOQUE

TABLA  4­1.  EE.  UU.  EOF  DOBU  T  AMIN  E , E  PIN  E  RARO  E , Y  NOR  E  PIN  E  PHRIN  E—  CON  T  'D

La  norepinefrina  es  un  agonista  adrenérgico  alfa  y  beta  combinado,  y  los  efectos  alfa  predominan  
en  todas  las  dosis.

Comience  en:  0,05  miligramos  por  kilogramo  por  minuto  y  aumente  hasta  el  punto  de  toxicidad  (ectopia  cardíaca).

Nota:  la  vasoconstricción  periférica  relativamente  pura  rara  vez  está  indicada  y  debe  usarse  solo  para  modular  el  
tono  vascular  periférico  en  el  shock  por  colapso  vascular  periférico.

12.  ¿Cuándo  está  indicado  un  balón  de  contrapulsación  intraaórtico?
El  soporte  circulatorio  mecánico  está  indicado  cuando  la  precarga  de  ambos  ventrículos  (CVP  y  PCWP)  se  ha  
optimizado  y  los  fármacos  estimuladores  cardíacos  adicionales  están  limitados  por  series  alarmantes  de  contracciones  
ventriculares  prematuras.  No  tengas  miedo  de  recurrir  al  soporte  mecánico.

PUNTOS  CLAVE:  RESUMEN  DE  AGENTES  ADRENÉRGICOS
1.  Dobutamina:  agonista  b1  (inotrópico  cardíaco)  con  efectos  b2  de  leves  a  moderados
vasodilatación).

2.  Epinefrina:  agente  adrenérgico  b  y  a  combinado,  con  los  efectos  b  predominando  en  la  parte  inferior
dosis  y  la  vasoconstricción  progresiva  que  acompaña  al  aumento  de  las  dosis.

3.  Norepinefrina:  agonista  adrenérgico  b  y  a  combinado,  con  los  efectos  a  predominando  en
todas  las  dosis.

13.  ¿Qué  hace  un  balón  de  contrapulsación  intraaórtico?
Proporciona  aumento  diastólico  y  descarga  sistólica.

14.  ¿Qué  es  el  aumento  diastólico?
Se  inserta  percutáneamente  un  balón  blando  de  40  ml  a  través  de  la  arteria  femoral  común  hasta  la  aorta  
torácica  descendente.  El  globo  no  es  oclusivo  (no  debe  tocar  las  paredes  aórticas).
Cuando  está  inflado,  desplaza  40  ml  de  sangre  y  es  exactamente  como  una  transfusión  aguda  de  40  ml  de  sangre  
a  la  aorta,  aumentando  cada  volumen  sistólico  del  VI  en  40  ml.  La  infusión  del  balón  se  activa  a  partir  del  complejo  
QRS  a  partir  de  un  ECG  de  superficie  (cualquier  derivación).  El  globo  siempre  se  infla  durante  la  diástole  para  
aumentar  la  presión  arterial  diastólica  (PAD)  y  aumentar  el  flujo  sanguíneo  coronario  (FSC).  El  ochenta  por  ciento  
del  CBF  ocurre  durante  la  diástole.

PUNTOS  CLAVE:  BOMBA  DE  BALÓN  INTRAAÓRTICO
1.  Indicado  para  shock  cardiogénico  refractario  a  manipulación  farmacológica.

2.  Desencadenada  por  el  complejo  QRS  del  ECG  de  superficie;  se  infla  durante  la  diástole  (onda  T)  y  se  desinfla  en
sístole  (onda  R  o  en  la  muesca  dicrótica  en  la  curva  de  presión  aórtica).

3.  El  ochenta  por  ciento  del  CBF  ocurre  durante  la  diástole.

4.  Mecánicamente  da  como  resultado  un  aumento  diastólico  y  una  descarga  sistólica  (postcarga).
reducción).
Machine Translated by Google
CAPÍTULO  4  CÓMO  PENSAR  EN  EL  CHOQUE  33

15.  ¿Qué  es  la  descarga  sistólica?
El  desinflado  del  globo  es  un  proceso  activo  (no  pasivo).  El  helio  se  succiona  bruscamente  del  globo,  dejando  
un  espacio  vacío  de  40  ml  en  la  aorta.  El  ventrículo  izquierdo  puede  expulsar  los  primeros  40  ml  de  su  volumen  
sistólico  en  este  espacio  vacío  con  una  carga  de  trabajo  drásticamente  reducida.  Un  balón  intraaórtico  aumenta  
el  FSC  durante  la  diástole,  mientras  que  disminuye  el  consumo  cardíaco  de  oxígeno  justo  antes  de  la  presístole.

16.  Nombre  las  contraindicaciones  del  balón  de  contrapulsación  intraaórtico.
Las  dos  contraindicaciones  principales  para  una  IABP  son  la  insuficiencia  aórtica  y  la  fibrilación  auricular.
La  insuficiencia  aórtica  es  cuando  el  aumento  diastólico  distiende  y  lesiona  el  ventrículo  izquierdo.
La  fibrilación  auricular  se  produce  cuando  el  inflado  y  el  desinflado  del  balón  no  pueden  sincronizarse  adecuadamente.

SITIOS  WEB

www.aic.cuhk.edu.hk/web8/IABP.htm

www.ccmtutorials.com/index.htm
www.acid­base.com/

BIBLIOGRAFÍA

1.  Harken  AH:  Arritmias  cardíacas.  En  Wilmore  DW,  Cheung  L,  Harken  AH  et  al.,  editores:  Scientific  American  Surgery,  
Nueva  York,  1999,  Scientific  American.
2.  Hirshberg  A,  Hoyt  DB,  Mattox  KL:  De  ''baldes  con  fugas''  a  lesiones  vasculares:  modelos  de  comprensión  de
hemorragia  incontrolable.  J  Am  Coll  Surg  204:665­672,  2007.
3.  Holcroft  JW:  Conmoción.  En  Wilmore  DW,  Cheung  L,  Harken  AH  et  al.,  editores:  American  College  of  Surgeons  Surgery,
Nueva  York,  2002,  WebMD  Corporation.
4.  Peters  MJ,  Brierley  J:  Volver  a  lo  básico  en  el  shock  séptico.  Cuidados  Intensivos  Med  34(6):991­993.  Epub  2008.
5.  Vincent  JL,  Weil  MH:  Revisión  del  reto  de  los  fluidos.  Crit  Care  Med  34:1333­1337,  2006.
Machine Translated by Google

CAPÍTULO  
5
¿QUÉ  ES  LA  INSUFICIENCIA  PULMONAR?
Dr.  Alden  H.  Harken

1.  ¿Qué  es  la  insuficiencia  pulmonar?
La  superficie  alvéolo­capilar  del  pulmón  es  del  tamaño  de  una  cancha  de  tenis  individual.  El  propósito  del  pulmón  es  hacer  
coincidir  la  ventilación  alveolar  (V)  con  el  flujo  sanguíneo  (Q).  El  desajuste  V/Q  conduce  a  insuficiencia  pulmonar.

2.  ¿Cómo  se  caracteriza  el  desajuste  entre  la  ventilación  alveolar  y  el  flujo  sanguíneo?
Shunt:  disminución  de  la  ventilación  en  relación  con  el  flujo  sanguíneo  regional;  "derivaciones"  de  
sangre  arterial  pulmonar  (no  oxigenada)  por  alvéolos  hipoventilados.
Espacio  muerto:  disminución  del  flujo  sanguíneo  regional  pulmonar  en  relación  con  la  ventilación.
Por  lo  tanto,  tanto  la  derivación  (menos  ventilación  que  flujo)  como  el  espacio  muerto  (zonas  ventiladas  que  no  están  
perfundidas)  califican  como  desajuste  V/Q.

3.  ¿Cuánta  energía  se  gasta  en  el  trabajo  de  respiración?
Un  estudiante  de  medicina  saludable  gasta  alrededor  del  3%  del  consumo  total  de  oxígeno  (uso  de  energía)  en  el  trabajo  de  
respiración.  Después  de  una  lesión,  particularmente  una  gran  quemadura,  los  pacientes  pueden  aumentar  el  gasto  energético  
fraccional  de  la  respiración  hasta  el  20%  de  su  uso  total  de  energía.

4.  ¿Qué  incisiones  quirúrgicas  comprometen  más  significativamente  la  salud  vital  de  un  paciente?
¿capacidad?
Intuitivamente,  una  incisión  o  lesión  en  una  extremidad  influye  menos  en  la  capacidad  vital,  seguida  secuencialmente  por  
una  incisión  en  el  abdomen  inferior,  una  esternotomía  media,  una  toracotomía  y  una  incisión  en  el  abdomen  superior.
¡Una  incisión  abdominal  superior  es  peor  que  una  toracotomía!

5.  ¿Es  útil  una  radiografía  de  tórax  para  evaluar  la  insuficiencia  respiratoria?
Sí,  pero  la  radiografía  debe  interpretarse  con  cuidado.  Puede  ser  difícil  estandarizar  la  técnica  de  rayos  X,  especialmente  
en  una  unidad  de  cuidados  intensivos  (UCI).

6.  ¿Qué  debe  buscar  en  la  radiografía  de  tórax  de  un  paciente  con
¿insuficiencia  respiratoria?
1.  ¿Están  ambos  pulmones  completamente  expandidos?

2.  ¿Hay  áreas  localizadas  de  infiltrado,  atelectasia  o  consolidación?
3.  ¿Hay  áreas  generalizadas  de  infiltrado,  atelectasia  o  consolidación?
4.  ¿Están  los  tubos  endotraqueales  y  otros  en  la  posición  correcta?
5.  ¿Por  qué  es  importante  la  distinción  local  versus  generalizada  al  evaluar  la  insuficiencia  respiratoria?
Un  proceso  local  puede  ser  producido  por  tumor  o  aspiración,  y  ambos  son  diagnosticados  y  tratados  mediante  
broncoscopia.  Es  más  probable  que  los  infiltrados  multilobulares  generalizados  representen  un  síndrome  de  fuga  alveolo­
capilar  difusa,  como  el  síndrome  de  dificultad  respiratoria  del  adulto  (SDRA).

7.  ¿Qué  es  el  síndrome  de  dificultad  respiratoria  del  adulto?
El  ARDS  es  una  transudación  capilar  difusa  y  multilobular  de  líquido  hacia  el  intersticio  pulmonar  que  disocia  la  concordancia  
normal  de  V  con  Q.

34
Machine Translated by Google
CAPÍTULO  5  ¿QUÉ  ES  LA  INSUFICIENCIA  PULMONAR?  35

8.  Qué  gobierna  el  flujo  de  líquido  a  través  de  los  capilares  pulmonares  hacia  el  intersticio
del  pulmon?
Starling  describió  inicialmente  el  equilibrio  entre  la  presión  hidrostática  intravascular  (Pc),  que  tiende  a  empujar  
el  líquido  fuera  de  los  capilares,  y  la  presión  oncótica  coloidal  (COP),  que  succiona  el  líquido  a  través  de  la  barrera  
endotelial  capilar  (K):

Flujo  de  fluido  ¼  5  KðPc  COPÞ

9.  ¿Qué  causa  el  SDRA?

Cualquier  cosa  que  aumente  la  disfunción  pulmonar  al  promover  el  pulmón  húmedo:
1.  La  insuficiencia  cardíaca  retrocede  a  la  Pc  intravascular  pulmonar,  forzando  el  líquido  en  el  pulmonar
intersticio.

2.  La  desnutrición  y  la  insuficiencia  hepática  disminuyen  las  proteínas  plasmáticas  y,  por  tanto,  el  COP.  El  fluido  no  es
succionado  fuera  del  pulmón  (si  la  proteína  total  y  la  albúmina  son  bajas).
3.  La  sepsis  puede  romper  la  barrera  endotelial  capilar  (K),  permitiendo  que  el  agua  y
proteína  se  filtre  al  pulmón.

PUNTOS  CLAVE:  CARACTERÍSTICAS  CLÍNICAS  DE  AGUDO
SÍNDROME  DE  DIFICULTAD  RESPIRATORIA

1.  Hipoxemia  grave  refractaria  al  aumento  de  la  concentración  de  oxígeno  inspirado

2.  Infiltrados  pulmonares  difusos

3.  Distensibilidad  pulmonar  baja

4.  Gran  desajuste  de  ventilación/perfusión  (V/Q)

10.  Explique  el  SDRA  de  alta  presión  versus  el  de  baja  presión.
Los  puristas  señalan  apropiadamente  que  la  congestión  pulmonar  resultante  de  la  presión  hidrostática  de  Pc  intravascular  
alta  secundaria  a  insuficiencia  cardíaca  en  realidad  no  es  un  síndrome  de  dificultad  respiratoria  primario.
Si  la  presión  de  enclavamiento  capilar  pulmonar  (PCWP)  es  >18  mm  Hg,  el  diagnóstico  es  edema  pulmonar  de  
alta  presión  (no  SDRA).  Un  paciente  con  estenosis  mitral  pura  puede  tener  congestión  pulmonar  (presión  alta),  
mientras  que  un  paciente  desnutrido  puede  desarrollar  congestión  pulmonar  (presión  baja);  ninguno  de  estos  es,  
estrictamente  hablando,  ARDS,  aunque  los  pacientes  con  ARDS  frecuentemente  tienen  componentes  de  ambos.

11.  ¿Qué  es  una  presión  oncótica  coloidal  (COP)  normal?
es  de  22  mm  Hg.

12.  ¿Cómo  se  calcula  el  COP?

De  COP,  el  75%  normalmente  es  creado  por  albúmina  sérica  junto  con  globulinas  y  fibrinógeno:

COP  ¼  2:1  ðproteína  totalÞ

Si  se  infunde  una  molécula  osmóticamente  activa  como  el  hetastarch,  este  cálculo  se  estropea.
Machine Translated by Google
36  CAPÍTULO  5  ¿QUÉ  ES  LA  INSUFICIENCIA  PULMONAR?

13.  Defina  ARDS  de  baja  presión.
ARDS  de  baja  presión  es  un  término  redundante.  Para  hacer  el  diagnóstico  de  ARDS,  la  PCWP  debe  ser  <18  mm  Hg.  
SDRA  puro  existe  solo  si  la  PCWP  es  >4  mm  Hg  menos  que  la  COP.

14.  ¿Cómo  pueden  filtrarse  los  capilares  pulmonares  si  el  COP  supera  el  PCWP?
El  concepto  actual  implica  una  expresión  séptica  de  los  receptores  de  adhesión  CD11  y  CD18  de  los  neutrófilos,  
que  se  adhieren  a  las  moléculas  de  adhesión  intercelular  del  endotelio  vascular  pulmonar.
Los  estímulos  sépticos  provocan  que  los  neutrófilos  adherentes  liberen  proteasas  intravasculares  y  radicales  
de  oxígeno.  El  daño  endovascular  resultante  descompone  la  K,  lo  que  permite  la  fuga  pulmonar,  incluso  a  una  
Pc  baja.

15.  ¿Qué  es  un  sándwich  Lasix?
Muchos  cirujanos,  cuando  tienen  la  espalda  contra  la  pared,  administran  25  g  de  albúmina  seguidos  en  20  minutos  por  
20  mg  de  furosemida  (Lasix)  por  vía  intravenosa  (IV).  Razonan  que  la  albúmina  extrae  líquido  del  pulmón  saturado  de  
agua  y  que  el  Lasix  promueve  la  diuresis  para  librar  al  paciente  del  exceso  de  agua.
Este  concepto  terapéutico  probablemente  solo  funcione  en  pacientes  que  no  están  muy  enfermos.  Cuanto  más  
enfermo  está  el  paciente,  más  rápido  se  filtra  la  albúmina  infundida  y  se  equilibra  a  través  de  la  barrera  endotelial  
endovascular  dañada.  Se  succiona  poca  agua  del  pulmón  enfermo  en  preparación  para  la  diuresis.

16.  Enumere  los  objetivos  de  la  terapia  para  ARDS.
1.  Reducir  el  edema  pulmonar  (típicamente  con  un  diurético).
2.  Reducir  la  toxicidad  del  oxígeno  (la  concentración  de  oxígeno  inspirado  <60  %  es  segura).
3.  Limite  el  barotrauma  pulmonar  (evite  la  presión  inspiratoria  máxima  en  >40  cm  H2O).
4.  Promover  la  coincidencia  de  V  y  Q;  con  frecuencia  presión  positiva  al  final  de  la  espiración  (PEEP)
es  útil.

5.  Mantener  el  suministro  de  oxígeno  sistémico  (gasto  cardíaco  contenido  de  oxígeno  arterial  [CO]).

17.  ¿Qué  gobierna  la  distribución  de  la  perfusión  pulmonar?
Se  rige  principalmente  por  la  gravedad.  Las  porciones  dependientes  del  pulmón  siempre  están  mejor  perfundidas.

18.  Analice  la  vasoconstricción  pulmonar  hipóxica  (VPH).
La  mayoría  de  los  estudiantes  cree  que  después  de  dedicar  todo  el  segundo  año  de  la  facultad  de  medicina  
al  feocromocitoma  y  la  vasoconstricción  pulmonar  hipóxica  (VPH),  ambas  entidades  pueden  olvidarse  con  seguridad.  
Al  menos  en  el  caso  del  VPH,  esto  no  es  cierto.  Un  paciente  que  acaba  de  someterse  a  una  endarterectomía  carotídea  
(CEA)  ilustra  la  relevancia  del  VPH.  Cuando  el  paciente  se  despierta  de  la  anestesia,  la  presión  arterial  (PA)  es  de  
220/120  mm  Hg  y  la  presión  arterial  parcial  de  oxígeno  (PO2)  con  oxígeno  al  100%  es  de  500  mm  Hg.  Para  que  el  
paciente  no  sople  la  anastomosis  carotídea,  el  cirujano  infunde  urgentemente  nitroprusiato.  ¡ En  20  minutos,  la  presión  
arterial  es  de  120/80  mm  Hg,  pero  la  PO2  (todavía  con  100  %  de  oxígeno)  ha  bajado  a  125  mm  Hg!

¿El  técnico  de  laboratorio  arruinó  el  análisis  de  gases  en  sangre?  No;  este  es  un  ejemplo  de  la  importancia  
clínica  del  VPH,  que  dirige  el  suministro  arteriolar  pulmonar  de  sangre  desoxigenada  hacia  los  alvéolos  ventilados  y  
lejos  de  las  regiones  pulmonares  mal  ventiladas.  El  paciente  estaba  usando  VPH  para  alcanzar  una  PO2  de  500  mm  
Hg.  Todos  los  agentes  antihipertensivos  (p.  ej.,  nitroprusiato)  y  la  mayoría  de  los  anestésicos  generales  bloquean  el  
VPH.  El  incremento  de  PO2  de  125  a  500  mm  Hg  se  debe  al  VPH.  El  VPH  dirigió  la  perfusión  hacia  áreas  ventiladas  
del  pulmón.

19.  ¿Qué  gobierna  la  distribución  de  la  ventilación  en  los  pulmones?
Un  gran  gradiente  de  presión  pleural  (más  negativo  en  la  parte  superior  del  pulmón  por  20  cm  H2O)  expulsa  el  gas  
principalmente  del  pulmón  dependiente  durante  cada  respiración  exhalada.  La  distensibilidad  regional  del  pulmón  
dependiente  es  mucho  mejor  que  la  del  vértice  pulmonar,  que  todavía  está  distendido  con  gas  al  final  de  la  exhalación.  
El  enfoque  habitual  es  perfundir  y  ventilar  preferentemente  el  pulmón  dependiente.
Machine Translated by Google
CAPÍTULO  5  ¿QUÉ  ES  LA  INSUFICIENCIA  PULMONAR?  37

PUNTOS  CLAVE:  OBJETIVOS  TERAPÉUTICOS  EN  AGUDOS
SÍNDROME  DE  DIFICULTAD  RESPIRATORIA

1.  Reducir  el  edema  pulmonar

2.  Reducir  la  toxicidad  del  oxígeno  (FiO2  <60%)

3.  Minimizar  los  barotraumas  (mantener  la  presión  inspiratoria  máxima  <40  cm  H2O)

4.  PEEP  para  promover  la  coincidencia  V/Q

5.  Mantener  el  suministro  de  oxígeno  sistémico  (contenido  de  oxígeno  arterial  CO)

20.  ¿Cómo  compromete  el  SDRA  la  función  pulmonar?
La  tráquea  se  mantiene  abierta  con  anillos  cartilaginosos,  pero  los  bronquiolos  terminales  no.  El  pulmón  húmedo  colapsa  
los  bronquiolos  terminales,  atrapando  el  gas  alveolar  distal.  La  perfusión  persistente  de  estas  regiones  mal  ventiladas  es  
una  derivación  que  produce  hipoxia.

21.  ¿Cuánto  tarda  la  sangre  arterial  pulmonar  (desoxigenada)  en  equilibrarse  por  completo  con  el  gas  alveolar  
atrapado  (pobremente  oxigenado)?
Tarda  unos  tres  cuartos  de  segundo.  Después  de  eso,  no  se  agrega  más  oxígeno  y  no  se  elimina  más  dióxido  de  
carbono  (CO2)  de  la  sangre  perfundida.  El  cierre  bronquiolar  terminal  que  produce  gas  alveolar  atrapado  es  malo.

22.  ¿Cuál  es  la  terapia  para  el  cierre  de  las  vías  respiratorias  terminales  y  la  derivación  resultante?
secundaria  al  pulmón  húmedo  del  SDRA?
La  PEEP  debe  mantener  abiertos  los  bronquiolos  terminales,  promoviendo  la  ventilación  de  los  alvéolos  previamente  
atrapados  y  minimizando  la  derivación.

23.  ¿Cuándo  puede  el  paciente  salir  de  la  ventilación  mecánica  y  ser  extubado?
¿sin  peligro?
El  paciente  debe  estar  lo  suficientemente  alerta  para  proteger  sus  vías  respiratorias,  requerir  una  concentración  
de  oxígeno  inspirado  (FiO2)  no  superior  a  0,4  y  estar  cómodo  respirando  con  una  pieza  en  T  (sin  ventilación  mecánica)  
durante  60  minutos  a  una  frecuencia  respiratoria  <20  y  una  ventilación  minuto  <10  L/min.  El  paciente  debe  poder  generar  
una  fuerza  inspiratoria  negativa  >20  cm  H2O.  Finalmente,  después  de  1  hora  en  la  pieza  en  T,  la  oxigenación  debe  
proporcionar  una  saturación  de  hemoglobina  >85%  sin  acidosis  respiratoria  (ver  Capítulo  7).

24.  ¿Qué  es  el  óxido  nítrico?

El  óxido  nítrico  (NO)  se  sintetiza  en  las  células  endoteliales  vasculares  por  la  óxido  nítrico  sintasa  constitutiva  (cNOS)  y  la  
NOS  inducible  (iNOS).  Intuitivamente,  el  NO  inhalado  debe  difundirse  a  través  de  los  alvéolos  ventilados  para  aumentar  la  
perfusión  regional  y  mejorar  la  coincidencia  de  V  y  Q.

25.  ¿El  NO  inhalado  funciona  en  el  SDRA?

Casi  24  ensayos  clínicos  controlados  aleatorios  han  evaluado  la  eficacia  terapéutica  del  NO  inhalado.  Aunque  la  
oxigenación  sistémica  y  la  hipertensión  pulmonar  mejoran  transitoriamente,  el  tiempo  de  ventilación  y  la  supervivencia  final  
no  se  ven  afectados.
Machine Translated by Google
38  CAPÍTULO  5  ¿QUÉ  ES  LA  INSUFICIENCIA  PULMONAR?

SITIOS  WEB

www.ardsnet.org

www.nlm.nih.gov/medlineplus/ency/article/000103.htm

BIBLIOGRAFÍA

1.  Bartlett  R:  Insuficiencia  pulmonar,  Nueva  York,  2006,  Colegio  Americano  de  Cirujanos,  Cirugía  WebMd
Corporación.

2.  Chetta  A,  Tzani  P,  Marangio  E  et  al .:  Efectos  respiratorios  de  la  cirugía  y  pruebas  de  función  pulmonar  en  el
evaluación  preoperatoria.  Acta  Biomed  77:69­74,  2006.

3.  Davidson  TA,  Caldwell  ES,  Curtis  JR  et  al.:  Calidad  de  vida  reducida  en  sobrevivientes  del  síndrome  de  dificultad  respiratoria  
aguda  en  comparación  con  pacientes  de  control  en  estado  crítico.  JAMA  281:354­360,  1999.

4.  Gust  R,  McCarthy  TJ,  Kozlowski  J  et  al.:  La  respuesta  al  óxido  nítrico  inhalado  en  la  lesión  pulmonar  aguda  depende  de
distribución  del  flujo  sanguíneo  pulmonar  antes  de  su  administración.  Am  J  Respir  Crit  Care  Med  159:563­570,  1999.

5.  Pesenti  A,  Fumagalli  R:  PEEP:  ¿cosméticos  para  gases  en  sangre  o  una  terapia  para  el  SDRA?  Crit  Care  Med  27:253­254,  1999.

6.  Wang  T,  Tagayun  A,  Bogardus  A  et  al.:  ¿Con  qué  precisión  podemos  predecir  el  volumen  espiratorio  forzado  en  un  segundo?
después  de  una  resección  pulmonar  mayor?  Am  Surg  73:1047­1051,  2007.

7.  Westwood  K,  Griffin  M,  Roberts  K  et  al.:  La  espirometría  de  incentivo  disminuye  las  complicaciones  respiratorias  después  de  
una  cirugía  abdominal  mayor.  Cirujano  5:339­342,  2007.
Machine Translated by Google

VENTILACION  MECANICA CAPÍTULO  
6

Jeffrey  L.  Johnson,  MD  y  James  B.  Haenel,  RRT

1.  ¿Por  qué  los  pacientes  necesitan  ventilación  mecánica?
Hay  tres  categorías  básicas  de  necesidad  cuando  se  trata  de  ventilación  mecánica  (MV):  (1)  
impulso  respiratorio  inadecuado;  (2)  incapacidad  para  mantener  una  ventilación  alveolar  adecuada;  y  
(3)  hipoxia.  La  decisión  de  proporcionar  VM  debe  basarse  en  el  examen  clínico  y  la  evaluación  del  intercambio  
de  gases  mediante  análisis  de  gases  en  sangre  arterial  (ABG),  según  sea  necesario.  Es  una  decisión  
individualizada  porque  los  valores  de  corte  arbitrarios  para  la  presión  parcial  de  oxígeno  (PO2),  la  presión  
parcial  de  dióxido  de  carbono  (pCO2)  o  el  equilibrio  ácido­base  (pH)  pueden  no  ser  pertinentes  para  todos  
los  pacientes.  Los  trastornos  comunes  que  requieren  la  necesidad  de  VM  incluyen  trastornos  parenquimatosos  
primarios,  como  neumonía,  edema  pulmonar  o  contusión  pulmonar,  y  enfermedades  sistémicas  que  
comprometen  indirectamente  la  función  pulmonar,  como  sepsis  o  disfunción  del  sistema  nervioso  central  (SNC).

2.  ¿La  ventilación  mecánica  mejora  el  pulmón?
No  precisamente.  En  el  marco  de  la  insuficiencia  respiratoria,  el  objetivo  es  apoyar  el  intercambio  de  gases  mientras  
se  invierte  el  proceso  de  la  enfermedad  subyacente.  Se  pueden  usar  ciertas  técnicas  para  reclutar  más  espacio  
aéreo  para  el  intercambio  de  gases,  pero  en  general  es  mucho  más  fácil  dañar  el  pulmón  con  un  ventilador  (es  
decir,  lesión  pulmonar  inducida  por  ventilador  [VILI])  que  repararlo.

3.  ¿Cuántos  modos  de  ventilación  puedes  nombrar?
Los  modos  comunes  incluyen  ventilación  mecánica  controlada  (CMV),  ventilación  con  control  asistido  
(ACV),  ventilación  obligatoria  intermitente  (IMV),  IMV  sincronizada  (SIMV),  ventilación  controlada  por  presión  
(PCV),  ventilación  con  soporte  de  presión  (PSV),  ventilación  de  relación  inversa  (IRV),  ventilación  con  
liberación  de  presión  de  las  vías  respiratorias  (APRV),  ventilación  por  minuto  obligatoria  (MMV),  ventilación  
de  alta  frecuencia  (HFV)  y  modos  de  control  dual,  como  el  control  de  volumen  regulado  por  presión  (PRVC).
La  diferencia  más  básica  entre  estos  modos  de  ventilación  se  basa  en  si  administran  respiraciones  
mandatorias  (ya  sea  que  la  inspiración  sea  activada  o  ciclada  por  la  máquina)  versus  respiraciones  
espontáneas  (ya  sea  que  la  inspiración  sea  activada  y  ciclada  por  el  paciente).

4.  ¿Qué  tres  elementos  pueden  caracterizar  todas  las  mecánicas  antes  mencionadas?
Modos  de  ventilación?
Cada  modo  se  puede  describir  por  cómo  se  activa  una  respiración  (por  el  paciente  o  por  la  máquina),  cómo  
se  cicla  (cambia  de  inhalación  a  exhalación)  y  cómo  se  limita  (por  ejemplo,  por  tiempo,  por  presión  o  por  
flujo). ).

5.  ¿Cuáles  son  los  modos  de  ventilación  con  presión  positiva  más  utilizados?
ACV,  IMV,  SIMV  y  PSV,  que  difieren  principalmente  en  cómo  se  activan  y  ciclan  las  respiraciones.

6.  ¿Cómo  funciona  la  ventilación  con  control  asistido?
El  modo  ACV  proporciona  un  número  mínimo  establecido  de  respiraciones  activadas  por  la  máquina  y  también  permite  
que  el  paciente  active  una  respiración.  Cada  respiración  (obligatoria  o  espontánea  activada  por  el  paciente)  se  cicla  
cuando  se  ha  administrado  un  volumen  preestablecido  a  una  velocidad  de  flujo  preestablecida.  Debido  a  que  el  
paciente  recibe  un  volumen  tidal  completo  con  cada  respiración,  incluso  cuando  está  taquipneico,  el  ACV  puede  
provocar  alcalosis  respiratoria  con  más  frecuencia  y  puede  promover  una  presión  espiratoria  final  autopositiva  (PEEP).

39
Machine Translated by Google
40  CAPÍTULO  6  VENTILACIÓN  MECÁNICA

7.  ¿En  qué  se  diferencia  la  ventilación  mecánica  intermitente  de  la  asistencia­control?
¿ventilación?
Al  igual  que  ACV,  el  ventilador  proporciona  un  número  preestablecido  de  respiraciones  activadas  por  la  máquina  a  un  volumen  
tidal  y  velocidad  de  flujo  preestablecidos  en  el  modo  IMV.  Sin  embargo,  a  diferencia  del  ACV,  las  respiraciones  activadas  por  el  
paciente  no  se  ciclan  a  un  volumen  preestablecido.  Más  bien,  las  respiraciones  espontáneas  se  ciclan  en  función  de  los  propios  
esfuerzos  respiratorios  del  paciente  (generalmente  al  detectar  el  final  del  esfuerzo  del  paciente  a  medida  que  cae  la  tasa  de  flujo).
La  IMV  puede  permitir  una  disminución  de  la  presión  media  en  las  vías  respiratorias  (Paw)  y  posiblemente  menos  barotrauma,  porque  no  
todas  las  respiraciones  son  respiraciones  con  presión  positiva  completa.

8.  Comparar  la  ventilación  mecánica  intermitente  con  la  ventilación  mecánica  intermitente  sincronizada  SIMV  evita  el  
"acumulación"  de  respiraciones  al  aplazar  una  respiración  activada  por  una  máquina  si  se  produce  en  medio  de  
una  respiración  espontánea.  Por  lo  tanto,  es  más  fácil  sincronizar  el  esfuerzo  del  paciente  con  el  ventilador  en  el  modo  
SIMV.  En  la  práctica,  la  mayoría  de  IMV  se  entrega  como  SIMV.  Ambos  modos  implican  un  trabajo  respiratorio  adicional  
por  parte  del  paciente.  Se  puede  agregar  presión  de  soporte  durante  las  respiraciones  espontáneas  para  aliviar  este  
trabajo.  Puede  ser  ventajoso  aliviar  tanto  trabajo  respiratorio  como  sea  posible  en  la  etapa  inicial  de  la  insuficiencia  
respiratoria.

9.  ¿Cuáles  son  los  tipos  de  ventilación  con  presión  limitada?
PSV,  PRVC,  HFV  y  PCV.  La  PSV  es  un  modo  de  ventilación  utilizado  en  pacientes  que  respiran  espontáneamente  
para  disminuir  el  trabajo  de  respiración  impuesto  desde  el  tubo  endotraqueal  y  para  vencer  la  resistencia  en  el  circuito  
de  respiración.  A  menudo  se  utiliza  para  "destetar"  o  determinar  si  un  paciente  está  preparado  para  interrumpir  la  VM.  
PSV  es  una  forma  pura  de  ventilación  asistida.  El  paciente  siempre  debe  disparar  la  respiración  (no  la  máquina).  Esto  
hace  que  el  ventilador  suministre  una  presión  preestablecida  determinada  por  el  médico,  lo  que  aumenta  el  volumen  
tidal  (VT).  Se  puede  agregar  PEEP  o  presión  positiva  continua  en  las  vías  respiratorias  (CPAP).  PCV  es  un  modo  
activado  por  la  máquina  (basado  en  el  tiempo)  en  el  que  se  aplica  una  presión  establecida  (límite).  Se  cicla  en  función  
de  una  cantidad  de  tiempo  preestablecida,  independientemente  del  tamaño  de  la  respiración  que  se  administró  durante  
ese  tiempo.

10.  Resuma  las  ventajas  y  limitaciones  de  la  ventilación  controlada  por  presión.
Las  ventajas  incluyen  (1)  la  limitación  de  la  presión  máxima  y  la  prevención  teórica  de  la  sobredistensión  y  (2)  una  
mejor  coincidencia  con  los  requisitos  de  flujo  del  paciente  que  con  un  caudal  establecido.  Las  posibles  limitaciones  
incluyen  la  variación  en  los  volúmenes  administrados  como  resultado  de  una  mayor  resistencia  de  las  vías  
respiratorias,  una  menor  distensibilidad  pulmonar  y  un  menor  esfuerzo  del  paciente.

11.  ¿Qué  son  las  variables  de  fase?
Hay  cuatro  variables  de  fase  básicas:  presión,  volumen,  flujo  y  tiempo.  Estas  son  las  mismas  variables  incorporadas  
por  el  ventilador  para  detectar  el  final  de  la  fase  de  flujo  inspiratorio  de  una  respiración  (es  decir,  ciclado  por  presión,  
ciclado  por  volumen,  ciclado  por  flujo  y  ciclado  por  tiempo).  Las  variables  de  fase  pueden  ser  controladas  por  el  
paciente  o  por  el  ventilador.

12.  ¿Qué  son  las  variables  desencadenantes?
Las  variables  de  disparo  describen  cómo  un  ventilador  inicia  la  inspiración.  Se  pueden  usar  una  o  más  de  las  
variables  de  fase:  tiempo,  presión,  flujo  o  volumen.  Por  ejemplo,  en  el  modo  CMV,  el  tiempo  es  la  única  opción  de  
activación  disponible;  no  importa  lo  que  haga  el  paciente,  no  pueden  activar  una  respiración.  En  el  modo  ACV  o  
SIMV,  el  paciente  puede  recibir  una  respiración  basada  en  el  tiempo;  sin  embargo,  una  disminución  de  la  presión  inicial  
generada  por  el  paciente  (activada  por  presión)  o  del  flujo  de  gas  generado  por  el  paciente  (activada  por  flujo)  también  
puede  iniciar  una  respiración.

13.  ¿Qué  son  las  variables  límite?

Las  variables  límite  (presión,  volumen  y  flujo)  son  parámetros  que  no  se  pueden  superar  durante  la  inspiración.  Durante  
una  inspiración,  la  presión,  el  flujo  y  el  volumen  aumentarán  y,  si  no  superan  un  valor  preestablecido,  se  dice  que  la  
respiración  está  limitada  por  la  variable  principal.
Machine Translated by Google
CAPÍTULO  6  VENTILACIÓN  MECÁNICA  41

14.  ¿Cuáles  son  los  objetivos  de  la  ventilación  mecánica  en  pacientes  con  insuficiencia  respiratoria  aguda?
¿falla?

En  pacientes  con  insuficiencia  respiratoria  aguda  (IRA),  los  objetivos  son  preservar  o  mejorar  la  oxigenación  arterial  
y  la  ventilación,  optimizar  la  mecánica  pulmonar  y  promover  la  comodidad  del  paciente  mientras  se  previene  la  
indagación  pulmonar  inducida  por  el  ventilador.  Las  complicaciones  pueden  surgir  de  presiones  alveolares  elevadas  o  
concentraciones  de  oxígeno  inspiradas  persistentemente  altas  (FiO2).

15.  ¿Cuáles  son  los  ajustes  iniciales  del  ventilador  en  caso  de  insuficiencia  respiratoria  aguda?
Hay  muchas  posibilidades  y  estilos,  pero  generalmente  se  desea  un  modo  que  brinde  soporte  completo.  La  evidencia  
del  ensayo  ARDS­Net  se  centra  en  ACV,  que  garantiza  la  entrega  de  un  volumen  preestablecido;  este  modo  es  
simplemente  el  más  estudiado.  Los  modos  ciclados  por  presión  son  aceptables,  pero  probablemente  solo  ofrezcan  una  
ventaja  teórica.  En  cualquier  modo,  la  FiO2  comienza  en  1,0  y  se  valora  hacia  abajo  según  se  tolere.  Una  FiO2  alta  ante  
una  lesión  pulmonar  aguda  provoca  un  empeoramiento  del  cortocircuito  intrapulmonar,  posiblemente  como  resultado  de  
una  atelectasia  por  absorción.  El  volumen  corriente  se  basa  en  el  peso  corporal  ideal  (IBW)  y  la  fisiopatología  de  la  lesión  
pulmonar.  Volúmenes  de  8  a  10  ml/kg  de  IBW  probablemente  sean  aceptables  si  la  presión  meseta  está  dentro  de  un  
rango  seguro.  Sin  embargo,  en  el  contexto  del  síndrome  de  dificultad  respiratoria  aguda  (SDRA)  o  lesión  pulmonar  aguda  
(ALI),  las  presiones  o  los  volúmenes  elevados  pueden  exacerbar  la  lesión  pulmonar  subyacente.  Por  lo  tanto,  los  
volúmenes  más  pequeños  (6  ml/kg  IBW)  se  administran  con  mayor  frecuencia.

16.  ¿Qué  variables  del  ventilador  controlan  la  relación  inspiratoria/espiratoria  (I/E)?
La  relación  inspiratoria/espiratoria  (I/E)  es  el  efecto  neto  de  cuatro  configuraciones  del  ventilador:  la  frecuencia  respiratoria  
(RR),  el  Vt,  el  flujo  máximo  y  la  configuración  de  la  forma  de  onda.  El  índice  de  flujo  máximo  es  el  índice  de  flujo  máximo  
entregado  por  el  ventilador  durante  la  parte  inspiratoria  del  ciclo  respiratorio.  Un  caudal  inicial  de  50  a  80  L/min  suele  ser  
satisfactorio.  En  un  modo  ciclado  por  volumen  (p.  ej.,  ACV),  una  tasa  de  flujo  más  alta  significa  un  tiempo  inspiratorio  
más  corto  y  una  relación  I/E  más  baja.  Una  relación  I/E  de  1:2  a  1:3  es  razonable  en  la  mayoría  de  las  situaciones.  Los  
pacientes  con  enfermedad  pulmonar  obstructiva  crónica  (EPOC)  pueden  necesitar  tiempos  espiratorios  más  prolongados  
para  permitir  una  exhalación  adecuada.  Esto  se  puede  lograr  aumentando  el  flujo,  disminuyendo  así  la  relación  I/E.  Las  
tasas  de  flujo  altas  pueden  aumentar  las  presiones  en  las  vías  respiratorias  y  empeorar  la  distribución  de  gas  en  algunos  
casos;  velocidades  de  flujo  más  lentas  pueden  reducir  las  presiones  en  las  vías  respiratorias  y  mejorar  la  distribución  del  
gas  al  aumentar  la  relación  I/E.  La  forma  de  onda  del  ventilador  (p.  ej.,  cuadrada  versus  desacelerada)  también  afectará  
la  relación  I/E.  Sin  cambiar  la  configuración  del  flujo  máximo,  la  forma  de  onda  cuadrada  da  como  resultado  una  presión  
máxima  más  alta  en  las  vías  respiratorias  y  un  tiempo  E  más  prolongado  que  la  selección  de  una  forma  de  onda  de  
desaceleración.

17.  ¿Qué  es  la  presión  positiva  al  final  de  la  espiración?
PEEP  es  una  elevación  de  la  presión  de  referencia  por  encima  de  la  presión  atmosférica  al  final  de  la  exhalación.

18.  ¿Qué  hace  la  presión  positiva  al  final  de  la  espiración?
La  PEEP  previene  el  colapso  alveolar,  recluta  alvéolos  atelectásicos,  aumenta  la  capacidad  residual  funcional  y  
revierte  la  hipoxemia.  En  todos  los  pacientes  al  principio  del  curso  de  su  insuficiencia  respiratoria,  es  probable  que  la  
PEEP  deba  manipularse  en  respuesta  a  los  períodos  de  desaturación  (después  de  descartar  las  causas  comunes  de  
hipoxemia,  como  el  taponamiento  mucoso  y  el  barotrauma)  para  evaluar  el  potencial  de  reclutamiento.

19.  ¿Qué  es  la  presión  espiratoria  final  intrínseca  o  autopositiva?
La  PEEP  intrínseca  (PEEPi)  es  el  desarrollo  de  presión  positiva  y  flujo  continuo  dentro  de  los  alvéolos  al  final  de  la  
espiración  sin  aplicación  de  PEEP  extrínseca  (PEEPe).  Los  pacientes  con  altos  requisitos  de  ventilación  por  minuto  o  
pacientes  que  reciben  relaciones  I/E  altas  tienen  riesgo  de  PEEPi.  En  pulmones  sanos  durante  la  VM,  si  la  frecuencia  
respiratoria  es  demasiado  rápida  o  el  tiempo  de  espiración  es  demasiado  corto,  no  hay  tiempo  suficiente  para  una  
exhalación  completa,  lo  que  da  como  resultado  una  acumulación  de  respiraciones  y  la  generación  de  presión  positiva  en  
las  vías  respiratorias  al  final  de  la  exhalación.  Los  tubos  endotraqueales  de  diámetro  pequeño  también  pueden  limitar  la  
exhalación  y  contribuir  a  la  PEEPi.  Pacientes  con  aumento  de  la  resistencia  de  las  vías  respiratorias  y
Machine Translated by Google
42  CAPÍTULO  6  VENTILACIÓN  MECÁNICA

distensibilidad  pulmonar  disminuida  tienen  un  alto  riesgo  de  PEEPi.  Dichos  pacientes  tienen  dificultad  para  
exhalar  gas  debido  a  la  obstrucción  o  colapso  de  las  vías  respiratorias  pequeñas  y  son  propensos  a  desarrollar  
PEEPi  durante  la  ventilación  espontánea  y  la  VM.  PEEPi  tiene  los  mismos  efectos  secundarios  que  PEEPe,  pero  
detectarlo  requiere  más  vigilancia.
Si  no  se  reconoce  la  presencia  de  auto­PEEP,  se  pueden  producir  cambios  de  ventilador  inadecuados.
La  única  forma  de  detectar  y  medir  la  PEEPi  es  ocluir  el  puerto  espiratorio  al  final  de  la  espiración  
mientras  se  controla  la  presión  de  las  vías  respiratorias.  La  disminución  de  la  frecuencia  o  el  aumento  del  flujo  
inspiratorio  (para  disminuir  la  relación  I/E)  pueden  dar  tiempo  para  una  exhalación  completa.  Considere  administrar  
terapia  broncodilatadora  en  caso  de  broncoespasmo.

20.  ¿Cuáles  son  los  efectos  secundarios  de  la  presión  positiva  al  final  de  la  espiración?
a.  El  barotrauma  puede  deberse  a  la  sobredistensión  de  los  alvéolos.  
b.  El  gasto  cardíaco  (GC)  puede  disminuir  como  resultado  de  la  presión  intratorácica,  produciendo
un  aumento  de  la  presión  transmural  de  la  aurícula  derecha  y  una  disminución  del  retorno  venoso.  La  PEEP  
también  aumenta  la  presión  de  la  arteria  pulmonar,  lo  que  puede  disminuir  el  gasto  del  ventrículo  derecho.
La  dilatación  del  ventrículo  derecho  puede  hacer  que  el  tabique  interventricular  se  incline  hacia  el  ventrículo  
izquierdo,  lo  que  dificulta  el  llenado  del  ventrículo  izquierdo  y  disminuye  el  GC,  especialmente  si  el  paciente  
está  hipovolémico.
C.  Interpretación  incorrecta  de  las  presiones  de  llenado  cardíaco:  la  presión  transmitida  desde  el  alvéolo  a  la  
vasculatura  pulmonar  puede  elevar  falsamente  las  lecturas.  Una  regla  general  es  restar  la  mitad  de  la  PEEP  
aplicada  durante  cinco  de  la  presión  de  oclusión  de  la  arteria  pulmonar.  d.  La  sobredistensión  de  los  alvéolos  
por  exceso  de  PEEP  disminuye  el  flujo  sanguíneo  a  estas  áreas,
aumento  del  volumen  del  espacio  muerto  (VD;  VD/
VT).  mi.  El  trabajo  respiratorio  puede  aumentar  con  la  PEEP  porque  se  requiere  que  el  paciente  genere
una  presión  negativa  mayor  para  activar  el  flujo  del  ventilador.
F.  Aumento  de  la  presión  intracraneal  (PIC)  y  retención  de  líquidos.  gramo.  
Aumento  del  agua  pulmonar.

21.  ¿Qué  es  un  paquete  de  ventilador?
El  término  paquete  de  ventilador  abarca  varias  medidas  preventivas  que  prácticamente  todos  los  pacientes  
ventilados  deben  recibir.  El  uso  de  dichos  paquetes  puede  minimizar  la  incidencia  de  neumonía  asociada  al  ventilador  
(NAV)  y  otras  complicaciones.  Se  ha  demostrado  que  las  técnicas  sencillas  junto  a  la  cama,  como  el  cuidado  bucal  y  
la  elevación  de  la  cabecera  de  la  cama,  por  ejemplo,  reducen  la  VAP.  La  necesidad  de  VM  prolongada  también  pone  
al  paciente  en  riesgo  de  hemorragia  gastrointestinal  (GI)  y  trombosis  venosa  profunda  (TVP)  y,  por  lo  tanto,  la  
profilaxis  de  la  úlcera  por  estrés  y  la  TVP  debe  iniciarse  como  parte  del  paquete  de  ventilación.

22.  ¿Qué  es  la  hipoventilación  controlada  con  hipercapnia  permisiva?
La  hipoventilación  controlada  (o  hipercapnia  permisiva)  es  una  estrategia  de  protección  pulmonar  que  limita  la  
presión  o  el  volumen  mediante  la  cual  se  permite  que  aumente  la  pCO2,  dando  más  importancia  a  la  protección  del  
pulmón  que  al  mantenimiento  de  la  eucapnia.  El  VT  establecido  se  reduce  a  un  rango  de  aproximadamente  4  a  6  ml/
kg  de  IBW  en  un  intento  de  mantener  la  presión  alveolar  (presión  estática)  por  debajo  de  30  cm  H2O.  Varios  estudios  
en  ARDS  y  estado  asmático  han  demostrado  una  disminución  en  el  barotrauma,  los  días  de  cuidados  intensivos  y  la  
mortalidad  con  este  enfoque.  Se  permite  que  la  pCO2  aumente  lentamente  hasta  un  nivel  de  80  a  100  mm  Hg.  Si  se  
produce  inestabilidad  cardiovascular  a  medida  que  cae  el  pH,  entonces  puede  ser  necesaria  la  adición  de  bicarbonato  
de  sodio  intravenoso.  Alternativamente,  se  puede  esperar  a  que  el  riñón  normal  retenga  bicarbonato  en  respuesta  a  
la  hipercapnia.  La  hipercapnia  permisiva  suele  ser  bien  tolerada.  Los  posibles  efectos  adversos  incluyen  vasodilatación  
cerebral  que  conduce  a  un  aumento  de  la  PIC  y  la  hipertensión  intracraneal  es  la  única  contraindicación  absoluta  
para  la  hipercapnia  permisiva.  Pueden  ocurrir  aumento  de  la  actividad  simpática,  vasoconstricción  pulmonar  y  
arritmias  cardíacas,  pero  rara  vez  son  significativos.  La  depresión  de  la  contractilidad  cardíaca  puede  ser  un  problema  
en  pacientes  con  disfunciones  ventriculares  subyacentes.  En  casos  de  ARF,  el  empeoramiento  del  equilibrio  ácido­
base  puede  impedir  o  limitar  la  hipercapnia  permisiva  agresiva.
Machine Translated by Google
CAPÍTULO  6  VENTILACIÓN  MECÁNICA  43

23.  ¿Qué  es  el  cumplimiento?  ¿Cómo  se  determina?
La  distensibilidad  es  una  medida  de  la  distensibilidad  y  se  expresa  como  el  cambio  de  volumen  para  un  cambio  de  
presión  dado.  La  determinación  del  cumplimiento  implica  la  interrelación  entre  la  presión,  el  volumen  y  la  resistencia  al  
flujo  de  aire.  Las  dos  presiones  relevantes  que  deben  monitorearse  durante  la  VM  son  las  presiones  máxima  y  estática.

24.  ¿Cómo  se  mide  la  presión  máxima?
La  presión  máxima  se  mide  durante  el  suministro  de  flujo  de  aire  al  final  de  la  inspiración.  Está  influenciado  por  el  
volumen  de  insuflación,  la  resistencia  de  las  vías  respiratorias  y  el  retroceso  elástico  de  los  pulmones  y  la  pared  torácica  
y  refleja  la  distensibilidad  dinámica  del  sistema  respiratorio  total.

25.  ¿Cómo  se  mide  la  presión  estática?
La  presión  estática  o  meseta  se  mide  durante  una  pausa  al  final  de  la  inspiración,  durante  una  condición  sin  flujo,  
y  refleja  la  distensibilidad  estática  del  sistema  respiratorio,  incluido  el  parénquima  pulmonar,  la  pared  torácica  y  
el  abdomen.

26.  ¿Cómo  se  calcula  el  cumplimiento?
Tanto  la  distensibilidad  dinámica  como  la  estática  deben  calcularse  como  parte  rutinaria  de  la  monitorización  
del  ventilador.  El  cumplimiento  dinámico  se  calcula  como  VT  (PEEP  total  de  la  pata),  y  el  cumplimiento  de  
meseta  o  estático  es  VT  (PEEP  total  de  presión  de  meseta).  Los  valores  normales  para  la  distensibilidad  dinámica  y  
estática  son  de  60  a  100  ml/cm  H2O.  Una  disminución  en  la  distensibilidad  dinámica  sin  un  cambio  en  la  distensibilidad  
estática  sugiere  un  aumento  agudo  en  la  resistencia  de  las  vías  respiratorias  y  puede  evaluarse  aún  más  comparando  
la  presión  pico  y  la  presión  meseta.  El  gradiente  normal  es  de  aproximadamente  10  cm  H2O.  Un  gradiente  >10  cm  
H2O  puede  ser  secundario  a  obstrucción  del  tubo  endotraqueal,  taponamiento  mucoso  o  broncoespasmo.  Si  el  
volumen  es  constante,  los  cambios  agudos  en  la  distensibilidad  tanto  dinámica  como  estática  sugieren  una  disminución  
en  la  distensibilidad  del  sistema  respiratorio  que  puede  ser  causada  por  el  empeoramiento  de  la  neumonía,  ARDS,  
atelectasia  o  aumento  de  las  presiones  abdominales.
El  cumplimiento  es  un  valor  global  y  no  describe  lo  que  sucede  regionalmente  en  los  pulmones  con  ARDS,  en  
el  que  las  regiones  enfermas  se  intercalan  con  regiones  relativamente  sanas.
Los  valores  de  cumplimiento  de  20  a  40  cm  H2O  son  comunes  en  ARDS  avanzado.  La  distensibilidad  pulmonar  
disminuida  refleja  la  distensibilidad  del  pulmón  que  participa  en  el  intercambio  de  gases,  no  los  alvéolos  colapsados  
o  llenos  de  líquido.  Como  regla  general,  cuando  la  distensibilidad  estática  es  <25  ml/cm  H2O ,  la  desconexión  del  
ventilador  puede  ser  difícil  debido  a  la  taquipnea  durante  las  pruebas  de  respiración  espontánea.

27.  ¿La  ventilación  en  decúbito  prono  es  una  opción  para  pacientes  que  son  difíciles  de
¿oxigenar?
¡Absolutamente!  La  PaO2  mejora  significativamente  en  aproximadamente  dos  tercios  de  los  pacientes  con  ARDS  
cuando  se  colocan  en  decúbito  prono.  Los  mecanismos  incluyen  (1)  reclutamiento  de  campos  pulmonares  dorsales  
colapsados  por  redistribución  del  edema  pulmonar  a  las  regiones  ventrales;  (2)  aumento  de  la  ventilación  que  mejora  el  
movimiento  del  diafragma;  (3)  eliminación  de  los  efectos  compresivos  del  corazón  sobre  los  campos  pulmonares  
inferiores  inferiores,  mejorando  así  la  ventilación  regional;  (4)  mantenimiento  de  la  perfusión  pulmonar  dorsal  frente  a  
una  ventilación  dorsal  mejorada,  lo  que  conduce  a  una  mejor  combinación  de  ventilación/perfusión  (V/Q);  y  (5)  un  
cambio  en  el  gradiente  de  presión  pleural  desde  la  región  ventral  a  la  dorsal  del  pulmón.

28.  ¿Cuáles  son  las  indicaciones  de  la  ventilación  en  decúbito  prono?
Las  indicaciones  para  la  ventilación  en  decúbito  prono  no  están  claramente  establecidas.  Iniciamos  una  prueba  
prono  en  cualquier  paciente  que  permanece  hipoxémico  o  requiere  concentraciones  altas  de  FiO2  después  de  la  
realización  o  maniobras  de  reclutamiento/PEEP.  El  mejor  predictor  de  un  mejor  resultado  durante  la  ventilación  prono  
puede  ser  una  disminución  de  la  PaCO2  y  no  una  mejor  oxigenación.

29.  Junior  O'Flaherty  está  ''luchando  contra  el  ventilador''.  ¿Qué  debo  hacer?
Inicialmente,  las  posibles  causas  se  separan  en  problemas  del  ventilador  (máquina,  circuito  y  vías  respiratorias)  
y  problemas  relacionados  con  el  paciente.  Las  causas  relacionadas  con  el  paciente  incluyen  hipoxemia,  secreciones  o
Machine Translated by Google
44  CAPÍTULO  6  VENTILACIÓN  MECÁNICA

taponamiento  mucoso,  neumotórax,  broncoespasmo,  infección  (neumonía  o  sepsis),  embolia  pulmonar,  
isquemia  miocárdica,  hemorragia  gastrointestinal,  empeoramiento  de  la  PEEPi  y  ansiedad.  Los  problemas  
relacionados  con  el  ventilador  incluyen  fugas  o  desconexión  del  sistema;  soporte  inadecuado  del  ventilador  o  FiO2  
suministrado;  problemas  relacionados  con  las  vías  respiratorias,  como  extubación,  tubo  endotraqueal  obstruido,  hernia  o  
ruptura  del  manguito;  y  sensibilidad  o  flujos  de  activación  inadecuados.  Hasta  que  se  identifique  el  problema,  el  paciente  
debe  ser  ventilado  manualmente  con  O2  al  100%.  Los  ruidos  respiratorios  y  los  signos  vitales  deben  comprobarse  
inmediatamente.  El  análisis  de  ABG  y  la  radiografía  de  tórax  portátil  son  valiosos,  pero  si  se  sospecha  un  neumotórax  a  
tensión,  la  descompresión  inmediata  precede  a  la  radiografía  de  tórax.

30.  ¿Se  debe  utilizar  el  bloqueo  neuromuscular  para  facilitar  la  ventilación  mecánica?
Los  bloqueadores  neuromusculares  (NMBA,  por  sus  siglas  en  inglés)  se  usan  comúnmente  para  facilitar  la  VM  durante  
el  ARDS,  pero  a  pesar  de  la  amplia  aceptación,  hay  pocos  datos  y  aún  no  hay  consenso  disponible  sobre  cuándo  se  
deben  usar  estos  agentes.  Gainnier  et  al.  (2004)  fueron  los  primeros  en  informar  los  efectos  de  una  infusión  de  NMBA  de  
48  horas  sobre  el  intercambio  de  gases  en  pacientes  con  ARDS  temprano.  Todos  los  pacientes  fueron  ventilados  según  el  
protocolo  ARDSNet.  Se  produjeron  mejoras  significativas  en  la  oxigenación  y  la  capacidad  para  reducir  la  PEEP  en  el  grupo  
de  NMBA  y  se  mantuvieron  más  allá  del  período  de  infusión  de  48  horas.  Aunque  queda  por  dilucidar  por  qué  la  parálisis  
muscular  mejora  la  oxigenación,  se  cree  que  los  NMBA  disminuyen  el  consumo  de  oxígeno,  promueven  la  interfaz  paciente­
ventilador  y  aumentan  la  distensibilidad  de  la  pared  torácica.

La  parálisis  muscular  también  puede  ser  beneficiosa  en  situaciones  específicas,  como  hipertensión  
intracraneal  o  modos  de  ventilación  no  convencionales  (p.  ej.,  IRV  o  técnicas  extracorpóreas).
Los  inconvenientes  del  uso  de  estos  medicamentos  incluyen  la  pérdida  del  examen  neurológico,  la  tos  abolida,  la  
posibilidad  de  parálisis  prolongada,  atrofia  diafragmática  y  muerte  asociada  con  desconexiones  inadvertidas  del  
ventilador.  El  uso  de  NMBA  no  debe  tomarse  a  la  ligera.  Primero  se  debe  intentar  una  sedación  adecuada;  si  se  
considera  absolutamente  necesario,  el  uso  de  NMBA  debe  limitarse  a  24  a  48  horas  para  evitar  posibles  complicaciones.

PUNTOS  CLAVE

1.  La  ventilación  alveolar  inadecuada,  la  hipoxia  y  el  impulso  respiratorio  alterado  son  los  tres
razones  por  las  que  los  pacientes  pueden  necesitar  VM.

2.  Todos  los  modos  de  ventilación  se  pueden  describir  en  función  de  cómo  se  activa,  cicla  y  limita  una  respiración.

3.  La  configuración  inicial  del  ventilador  para  ARF  debe  proporcionar  soporte  completo.  El  modo  ACV  es  el  más  estudiado.

4.  La  hipoventilación  y  dejar  que  aumente  la  pCO2  es  permisible,  y  beneficioso,  si  permite  al  médico  limitar  la  
presión  y  el  estiramiento  alveolar  (hipercapnia  permisiva).

5.  Cuando  un  paciente  parece  estar  "luchando"  con  el  ventilador,  el  primer  paso  es  retirar  el
ventilador  y  ventilar  manualmente  (''bolsa'')  al  paciente.  Esto  le  permite  eliminar  las  variables  del  ventilador  como  
causa  y  evaluar  las  variables  del  paciente  que  pueden  necesitar  un  tratamiento  urgente  (p.  ej.,  neumotórax  a  tensión).

BIBLIOGRAFÍA

1.  Abroug  F,  Ouanes­Besbes  L,  Elatrous  S  et  al .:  El  efecto  de  la  posición  propensa  en  la  dificultad  respiratoria  aguda
síndrome  o  lesión  pulmonar  aguda:  un  metanálisis.  Áreas  de  incertidumbre  y  recomendaciones  para  la  investigación.  Int  Care  
Med,  en  prensa  2008.

2.  Burger  CD,  Resar  RK:  enfoque  de  "paquete  de  ventilación"  para  la  prevención  de  la  neumonía  asociada  a  la  ventilación.  Mayonesa
ClinProc  81(6):849­850,  2006.
Machine Translated by Google
CAPÍTULO  6  VENTILACIÓN  MECÁNICA  45

3.  Campbell  RS,  Davis  BR:  Ventilación  controlada  por  presión  versus  ventilación  controlada  por  volumen:  ¿importa?  cuidado  de  la  respiración
47(4):416­424,  2002.

4.  Gainnier  M,  Roch  A,  Forel  JM  et  al .:  Efecto  de  los  agentes  bloqueadores  neuromusculares  en  el  intercambio  de  gases  en  pacientes
presentando  síndrome  de  dificultad  respiratoria  aguda.  Crit  Care  Med  32:113­119,  2004.

5.  Levine  S,  Nguyen  T,  Taylor  N  et  al .:  Atrofia  rápida  por  desuso  de  las  fibras  del  diafragma  en  humanos  con  ventilación  mecánica.  N  
Engl  J  Med  358(13):1327­1335,  2008.

6.  Meade  MO,  Cook  DJ,  Guyatt  GH  et  al.:  Estrategia  de  ventilación  con  volúmenes  corrientes  bajos,  maniobras  de  reclutamiento  y  presiones  
positivas  altas  al  final  de  la  espiración  para  la  lesión  pulmonar  aguda  y  el  síndrome  de  dificultad  respiratoria  aguda:  un  ensayo  
controlado  aleatorizado.  JAMA  299(6):637­645,  2008.

7.  Mercat  A,  Richard  JM,  Vielle,  B  et  al.:  Ajuste  de  presión  positiva  al  final  de  la  espiración  en  adultos  con  lesión  pulmonar  aguda  y  síndrome  
de  dificultad  respiratoria  aguda.  JAMA  299(6):646­655,  2008.

8.  Pierson,  DJ:  Indicaciones  para  ventilación  mecánica  en  adultos  con  insuficiencia  respiratoria  aguda.  cuidado  de  la  respiración
47(3):249­262,  2002.
Machine Translated by Google

CAPÍTULO  
7
¿POR  QUÉ  RECIBIR  GASES  EN  LA  SANGRE  ARTERIAL?

Dr.  Alden  H.  Harken

1.  ¿La  respiración  está  realmente  sobrevalorada?
Puede  ser.  Un  maestro  de  yoga  japonés  sobrevivió  respirando  bien  una  vez  por  minuto  durante  1  hora  (ver  referencia  3).

2.  El  Sr.  O'Flaherty  acaba  de  someterse  a  una  herniorrafia  inguinal  bajo  control  local.
anestesia.  La  enfermera  de  la  sala  de  recuperación  pide  permiso  para  sedarlo.  Ella  dice  que  él  está  confundido  e  ingobernable  
y  sigue  tratando  de  levantarse  de  la  cama.  ¿Es  seguro  sedar  al  Sr.  O'Flaherty?

No.  Un  paciente  confundido  y  agitado  en  la  sala  de  recuperación  o  en  la  unidad  de  cuidados  intensivos  quirúrgicos  (UCI)  
debe  ser  reconocido  como  hipoxémico  agudo  hasta  que  se  demuestre  lo  contrario.

3.  El  Sr.  O'Flaherty  es  trasladado  a  la  SICU  y,  a  las  2:00  a .  m.,  la  enfermera  de  la  SICU  llama  para  informar  que  tiene  una  presión  
parcial  de  oxígeno  (PO2)  de  148  mm  Hg  en  la  mascarilla  de  oxígeno.  ¿Está  bien  darse  la  vuelta  y  volver  a  dormir?

No.  Se  necesita  más  información.

4.  Echas  un  vistazo  a  la  taza  de  café  abandonada  sobre  tu  gastada  copia  de  Surgical  Secrets.  ¿Cuál  es  el  PO2  de  esa  taza  de  
café?
Es  de  148  mm  Hg.

5.  ¿Cómo  es  posible  que  el  Sr.  O'Flaherty  y  el  café  tengan  el  mismo  PO2?
El  café  abandonado  presumiblemente  ha  tenido  tiempo  de  equilibrarse  completamente  con  el  gas  atmosférico.
A  nivel  del  mar,  la  presión  barométrica  es  de  760  mm  Hg.  Para  obtener  la  presión  parcial  de  oxígeno  (PO2)  en  el  café,  
reste  la  presión  de  vapor  de  agua  (47  mm  Hg)  y  multiplíquela  por  la  concentración  de  oxígeno  (20,8%)  en  la  atmósfera:

PO2  ¼  ð760  47Þ  20:8%  ¼  148  mm  Hg

6.  ¿Cuál  es  la  diferencia  entre  el  PO2  del  Sr.  O'Flaherty  y  el  del  café?
Nada.  Ambos  representan  la  presión  parcial  de  oxígeno  en  un  fluido.  Es  necesario  un  conjunto  completo  de  gases  en  sangre.

7.  ¿Qué  constituye  un  conjunto  completo  de  gases  sanguíneos?
PO2  
pCO2  
pH
saturación  de  hemoglobina
concentración  de  hemoglobina

8.  Si  el  Sr.  O'Flaherty  y  el  café  tienen  el  mismo  PO2,  ¿cómo  haría  el  Sr.  O'Flaherty?
si  le  hicieran  una  transfusión  de  intercambio  con  café?
Gravemente.

46
Machine Translated by Google
CAPÍTULO  7  ¿POR  QUÉ  RECIBIR  GASES  EN  LA  SANGRE  ARTERIAL?  47

9.  ¿Por  qué?
Aunque  las  tensiones  de  oxígeno  son  las  mismas,  la  cantidad  de  oxígeno  en  la  sangre  es  mucho  mayor.

10.  ¿Cómo  se  cuantifica  la  cantidad  de  oxígeno  en  la  sangre?
El  contenido  de  oxígeno  arterial  (CaO2)  se  cuantifica  como  ml  de  oxígeno/100  ml  de  sangre.  (Cuidado:  casi  todas  las  
demás  concentraciones  tradicionalmente  se  proporcionan  por  ml  o  por  L  y  no  por  100  ml).  Debido  a  que  ml  de  oxígeno  
es  un  volumen  en  100  ml  de  sangre,  estas  unidades  con  frecuencia  se  abrevian  como  vol  %.

11.  ¿Por  qué  la  sangre  es  más  espesa  que  el  café  (o  el  vino)?
Porque  la  hemoglobina  se  une  a  una  gran  cantidad  de  oxígeno.  Un  total  de  10  g  de  hemoglobina  completamente  
saturada  (hematocrito  alrededor  del  30  %)  se  une  a  13,4  ml  de  oxígeno,  mientras  que  100  ml  de  plasma  a  una  PO2  de  
100  mm  Hg  contiene  sólo  0,3  ml  de  oxígeno.

12.  ¿La  posición  de  la  curva  de  disociación  de  la  oxihemoglobina  hace
¿diferencia?

&  Un  aumento  en  pCO2  &  
Un  aumento  en  las  concentraciones  de  iones  de  hidrógeno  (no  pH)
&  Un  aumento  de  la  temperatura
Todos  desplazan  la  curva  de  oxihemoglobina  hacia  la  derecha;  es  decir,  el  oxígeno  se  libera  más  fácilmente  en  
los  tejidos.  Sin  embargo,  dentro  de  los  límites  fisiológicos,  Mae  West  probablemente  lo  dijo  mejor:  "Hay  menos  en  esto  
de  lo  que  parece".

PUNTOS  CLAVE:  MEDIADORES  DE  LA  OXIHEMOGLOBINA
CURVA  DE  DISOCIACIÓN
Giro  a  la  derecha Shift  izquierdo

1.  Aumento  de  pCO2  2.   1.  Disminución  de  [Hþþ],  pH  más  alto  2.  

Aumento  de  [H++],  pH  más  bajo  3.   Altitudes/elevaciones  más  altas  3.  

Aumento  de  temperatura  4.  Aumento   Disminución  de  2,3­DPG  (p.  ej.,  a  las  4  semanas  de  

de  2,3­DPG almacenamiento,  la  sangre  no  mantiene  2,3­DPG)

13.  Si  el  contenido  de  oxígeno  arterial  (Cao2)  o,  en  última  instancia,  el  suministro  de  oxígeno  sistémico  (gasto  cardíaco  Cao2)  
es  lo  que  el  cirujano  realmente  quiere  saber,  ¿por  qué  la  enfermera  informa  la  PO2  del  Sr.  O'Flaherty  en  lugar  de  su  
Cao2  a  las  2:00  a .  m .?
Nadie  sabe.

14.  ¿Cuál  es  el  método  más  rápido  y  práctico  para  aumentar  el  valor  del  Sr.  O'Flaherty?
¿Cao2?
Transfusión  de  glóbulos  rojos.  El  CaO2  del  paciente  aumenta  en  un  25%  con  la  transfusión  desde  una  concentración  
de  hemoglobina  de  8  a  10  g/dl.  El  CaO2  del  paciente  se  ve  afectado  de  forma  insignificante  por  un  aumento  de  la  PO2  
arterial  de  100  a  200  mm  Hg  (la  hemoglobina  está  totalmente  saturada  en  ambos  casos).

15.  ¿Qué  es  un  disparador  de  transfusión?
El  hematocrito  al  que  se  transfunde  automáticamente  a  un  paciente.  Este  no  es  un  concepto  útil.
La  Conferencia  de  Consenso  del  NIH,  extrayendo  datos  de  los  Testigos  de  Jehová,  pacientes  con  insuficiencia  renal  y  
monos,  concluyó  que  no  es  necesario  transfundir  a  un  paciente  hasta  que  el
Machine Translated by Google
48  CAPÍTULO  7  ¿POR  QUÉ  RECIBIR  GASES  EN  LA  SANGRE  ARTERIAL?

el  hematocrito  es  del  21%.  El  dogma  quirúrgico  tradicional  exige  un  hematocrito  >30%.  Sin  embargo,  cuando  
el  paciente  tiene  problemas,  las  autoridades  en  cuidados  intensivos  quirúrgicos  fomentan  la  transfusión  a  un  
hematocrito  del  45%  para  optimizar  el  suministro  de  oxígeno  sistémico.

16.  ¿Qué  gobierna  el  impulso  respiratorio?
pCO2  y  el  pH  están  inextricablemente  entrelazados  por  la  ecuación  de  Henderson­Hasselbalch.  Al  hacer  
malabarismos  con  esta  ecuación  en  el  líquido  cefalorraquídeo  (LCR)  de  las  cabras,  está  claro  que  la  concentración  
de  iones  de  hidrógeno  del  LCR  (no  la  pCO2)  controla  el  impulso  respiratorio.  Sin  embargo,  esta  distinción  no  es  
clínicamente  importante.  Lo  que  es  importante  es  que  si  una  persona  se  vuelve  acidótica  ya  sea  con  cetoacidosis  
diabética  (CAD)  o  al  subir  corriendo  un  tramo  de  escaleras,  aumenta  la  ventilación  por  minuto  (VE) .

17.  ¿Qué  tan  estricto  es  el  control  respiratorio?  O,  si  aguantas  la  respiración  durante  1  minuto,  ¿cuánto  quieres  respirar?

Mucho  (a  menos  que  seas  un  maestro  de  yoga  acercándose  al  nirvana).

18.  Después  de  60  segundos  de  apnea,  ¿qué  sucede  con  la  PaCO2?
Aumenta  solo  de  40  a  47  mm  Hg.  Pequeños  cambios  en  pCO2  (y  pH)  se  traducen  en  un  gran  estímulo  respiratorio.  
Normalmente,  la  compensación  respiratoria  de  la  acidosis  metabólica  es  escasa.

19.  Defina  el  exceso  de  base.
El  exceso  de  base  es  un  indicador  pobre  del  componente  metabólico  de  los  trastornos  acidobásicos.
Después  de  corregir  la  pCO2  a  40  mm  Hg,  el  exceso  o  déficit  de  base  se  promociona  como  una  medida  indirecta  del  
lactato  sérico.  Aunque  muchos  parámetros  que  dirigen  la  reanimación  con  volumen  en  estado  de  shock  son  más  
prácticos  y  directos  (véase  el  capítulo  4),  se  ha  anunciado  que  el  déficit  de  base  es  útil.
El  exceso  o  déficit  de  base  se  calcula  a  partir  del  nomograma  de  Sigaard­Anderson  en  el  laboratorio  de  gases  en  
sangre.  Normalmente,  no  hay  exceso  o  déficit  de  base.  El  estado  ácido­base  es  "correcto".

BIBLIOGRAFÍA

1.  Catheline  JM,  Bihan  H,  Le  Quang  T  et  al.:  Evaluación  cardiaca  y  pulmonar  preoperatoria  en  cirugía  bariátrica.
Obes  Surg  18:271­277,  2008.

2.  Dekerle  J,  Baron  B,  Dupont  L  et  al .:  estado  estacionario  de  lactato  máximo,  umbral  de  compensación  respiratoria  y  poder  
crítico.  Eur  J  Appl  Physiol  89:280­288,  2003.

3.  Miyamura  M,  Nishimura  K,  Ishida  K  et  al.:  ¿Puede  un  hombre  respirar  una  vez  por  minuto  durante  una  hora?  El  efecto  del  yoga
Ejercicios  de  gases  en  sangre.  Jpn  J  Physiol  52:313,  2002.

4.  Tada  T,  Hashimoto  F,  Matsushita  Y  et  al.:  Estudio  de  satisfacción  vital  y  calidad  de  vida  de  pacientes  que  reciben
terapia  de  oxigeno.  J  Med  Invest  50:55­63,  2003.
Machine Translated by Google

LÍQUIDOS,  ELECTROLITOS,  GATORADE,
Y  SUDOR CAPÍTULO  
8

Dr.  Alden  H.  Harken

1.  ¿Qué  es  la  solución  salina  hipertónica?
La  solución  salina  normal  es  cloruro  de  sodio  al  0,9%.  La  solución  salina  hipertónica  es  cloruro  de  sodio  al  7,5%  (8  veces  más  
concentrada  que  la  solución  salina  normal).

PUNTOS  CLAVE:  CONCENTRACIONES  DE  IONES  EN  CRISTALOIDE
SOLUCIONES

1.  La  mitad  de  solución  salina  normal  o  NaCl  al  0,45  %:  77  mEq  de  Na+,  77  mEq  de  Cl

2.  Solución  salina  normal  o  NaCl  al  0,9  %:  154  mEq  de  Na+,  154  mEq  de  Cl

3.  Solución  salina  normal  hipertónica  o  NaCl  al  7,5%:  1283  mEq  de  Na+,  1283  mEq  de  Cl

4.  Ringer  lactato:  130  mEq  de  Na+,  110  mEq  de  Cl,  38  mEq  de  lactato,  4  mEq  de  K+  y
3  mEq  Cas

2.  ¿Para  qué  sirve  la  solución  salina  hipertónica?
Resucitación.  La  hipótesis  inicial  era  que  un  poco  de  solución  salina  hipertónica  atraería  agua  extravascular  
hacia  el  compartimento  intravascular,  restaurando  rápidamente  el  volumen.  Ahora  parece  que  una  sacudida  
osmótica  (incluso  un  salto  transitorio  de  140  a  180  mOsm)  pacificaría  a  los  neutrófilos  circulantes  para  que  no  se  
adhieran  a  la  endovasculatura  y  provoquen  inflamación  postraumática.

3.  ¿La  solución  salina  hipertónica  sirve  para  algo  más?
La  pacificación  de  los  neutrófilos  "cebados"  debería  disminuir  el  riesgo  de  insuficiencia  multiorgánica  
postraumática.

4.  ¿Cómo  se  convierte  1  g  de  sodio  en  miliequivalentes  (mEq)?
Divida  por  el  peso  atómico  del  sodio:

1  g  ð1000  mgÞ  de  sodio  23  ¼  43:5  mEq

5.  ¿Cuántos  mEq  de  sodio  hay  en  1  cucharadita  de  sal?  104  mEq  (o  2400  mg).

6.  ¿Cuántos  mEq  de  sodio  hay  en  una  botella  de  Gatorade  de  8  onzas?
5  mEq.

7.  ¿Cuánto  cuesta  un  bloque  de  sal  de  40  libras?  $3.40  en  la  
tienda  de  alimentos.

49
Machine Translated by Google
50  CAPÍTULO  8  LÍQUIDOS,  ELECTROLITOS,  GATORADE  Y  SUDOR

8.  ¿Cuál  es  el  contenido  de  electrolitos  de  los  líquidos  intravenosos?
Consulte  la  Tabla  8­1.

TABLA  8­1.  CONTENIDO  DE  ELEC  TROLITO  EN  EL  INTERIOR  DE  SUFICIENTES  LÍQUIDOS  SF

Solución  (mEq/L) Sodio  Potasio  Cloruro  Bicarbonato/Lactato

Solución  salina  normal  (0,9%  NaCl) 154  — 154  —

Solución  de  lactato  de  Ringer   130 4 109 28*


dextrosa  al  5%  ½  solución  salina  normal 77  — 77  —

*El  lactato  se  convierte  inmediatamente  en  bicarbonato.

9.  ¿Cómo  se  relacionan  estas  concentraciones  con  los  compartimentos  de  electrolitos  y  fluidos  corporales?
Consulte  la  Tabla  8­2.

TABLA  8­2.  CONCENTRACIONES  DE  ELECTROLITO  EN  CUERPO  LÍQUIDO

Compartimento  (mEq/L) Sodio Potasio Cloruro Bicarbonato/Lactato

Plasma 142 4 103 27


Líquido  intersticial 144 4 114 30
fluido  intracelular 10 150 — 10

10.  ¿Cuáles  son  los  volúmenes  diarios  (mL/24  h)  y  el  contenido  de  electrolitos  (mEq/L)  de  las  
secreciones  corporales  de  un  estudiante  de  medicina  de  70  kg?
Consulte  la  Tabla  8­3.

TABLA  8­3.  VOLÚMENES  DIARIOS  Y  CONTENIDO  DE  ELECTROLITO  DEL  CUERPO

SECRECIONES

ml/24  horas Sodio Potasio Cloruro Bicarbonato

Saliva þ1500 10 25 10 30
Estómago þ1500 50 10 130 —

Duodeno þ1000 140 5 80 —

Íleon þ3000 140 5 104 30


Colon 6000 60 30 40 —

Páncreas þ500 140 5 75 100

biliar þ500 140 5 100 30


Sudor* þ1000 50 — — —

Gatorade  — 21 — 21 —

*Ver  pregunta  6.
Machine Translated by Google
CAPÍTULO  8  LÍQUIDOS,  ELECTROLITOS,  GATORADE  Y  SUDOR  51

11.  ¿Las  glándulas  sudoríparas  responden  a  la  aldosterona?  ¿Se  pueden  entrenar?
Si  y  si.  El  sudor  de  Archie  Bunker  contiene  100  mEq/L  de  sodio,  mientras  que  un  corredor  de  maratón  
olímpico  retiene  sodio  (el  sodio  del  sudor  puede  ser  tan  bajo  como  25  mEq/L).

12.  ¿Es  Gatorade  realmente  solo  sudor  de  atleta  con  sabor?
Sí.

13.  ¿Cuáles  son  los  requisitos  diarios  de  fluidos  y  electrolitos  para  el  mantenimiento  de  un  camión  de  70  kg?
¿estudiante  de  medicina?
Volumen  total  de   2500  ml  
líquido  Sodio 70  mEq  (1  mEq/kg)  
Potasio 35  mEq  (0,5  mEq/kg)

14.  ¿El  paciente  postoperatorio  de  rutina  requiere  suplementos  de  sodio  o  potasio  por  vía  
intravenosa?  ¿Prueba  rutinaria  de  electrolitos  en  suero?
No  y  no.

15.  ¿Puede  un  paciente  con  un  buen  corazón  y  riñones  superar  todos  menos  los  más  lamentables?
manejo  incompetente  de  fluidos  y  electrolitos?
Sí.

16.  ¿Se  puede  arrojar  a  un  estudiante  de  medicina  sano  a  una  insuficiencia  cardíaca  congestiva
infusión  intravenosa  de  100  ml  de  dextrosa  al  5%  en  solución  salina  por  kilogramo  por  hora?

No.  Uno  simplemente  tendrá  orina  hasta  los  tobillos.

17.  ¿Qué  es  la  alcalosis  por  sustracción?
La  succión  nasogástrica  vigorosa  de  un  paciente  con  mucho  ácido  gástrico  elimina  el  ácido  clorhídrico,  
dejando  al  paciente  alcalótico.

18.  ¿Qué  electrolito  es  más  útil  para  reparar  una  alcalosis  metabólica  hipopotasémica?
Cloruro.

19.  Enumere  los  mejores  indicadores  del  estado  de  volumen  de  un  paciente.
Ritmo  cardíaco  

Presión  arterial  Gasto  

urinario  Temperatura  

del  dedo  gordo  del  pie

20.  ¿Un  dedo  gordo  del  pie  caliente  indica  un  paciente  hemodinámicamente  estable?
Más  probable.  La  capacidad  de  autorregulación  vascular  de  un  paciente  joven  y  sano  es  enorme.  Las  
circulaciones  carotídea  y  coronaria  se  mantienen  hasta  el  amargo  final.  Por  el  contrario,  si  el  dedo  gordo  del  pie  
del  paciente  está  caliente  y  perfundido,  el  paciente  está  estable.

21.  ¿Cuál  es  la  diuresis  postoperatoria  mínima  adecuada?
Es  de  0,5  mililitros  por  kilogramo  por  hora.

22.  ¿Qué  es  un  sodio  en  orina  postoperatorio  típico?
Es  <20  mEq/L.
Machine Translated by Google
52  CAPÍTULO  8  LÍQUIDOS,  ELECTROLITOS,  GATORADE  Y  SUDOR

23.  ¿Por  qué?
El  estrés  quirúrgico  provoca  la  secreción  de  mineralocorticoides  (aldosterona)  para  que  el  riñón  normal  
retenga  sodio.

24.  Explique  la  aciduria  paradójica.
Los  pacientes  posoperatorios,  en  virtud  de  la  succión  nasogástrica  (pérdida  de  ácido  gástrico),  las  transfusiones  
de  sangre  (el  citrato  de  la  sangre  se  convierte  en  bicarbonato)  y  la  hiperventilación  (disminución  de  la  pCO2),  
suelen  ser  alcalóticos.  Los  pacientes  también  están  estresados  y  sus  riñones  retienen  sodio  y  agua.
Los  túbulos  renales  deben  intercambiar  algunos  otros  cationes  por  el  sodio  retenido.  El  riñón  elige  
intercambiar  iones  de  potasio  e  hidrógeno.  Incluso  ante  la  alcalosis  sistémica,  el  riñón  postoperatorio  absorbe  
sodio  y  excreta  iones  de  hidrógeno,  produciendo  una  aciduria  paradójica.

25.  ¿Qué  es  el  tercer  espacio?
La  hipotensión  y  la  infección  estimulan  los  neutrófilos  (complejos  de  receptores  CD11  y  CD18),  promoviendo  
la  adherencia  a  las  células  endoteliales  vasculares.  La  activación  subsiguiente  de  los  neutrófilos  adherentes  
arroja  proteasas  y  radicales  superóxido  tóxicos,  abriendo  grandes  agujeros  en  el  revestimiento  vascular.  El  
agua  y  la  albúmina  plasmática  se  filtran  a  través  de  los  agujeros.  El  volumen  extraído  del  espacio  vascular  
hacia  el  tercer  espacio  de  la  víscera  intersticial  y  hueca  (intestino)  crea  una  hipovolemia  relativa  y  requiere  
reposición  adicional  de  líquidos.

26.  ¿Qué  es  un  sándwich  Lasix?
Son  25  g  de  albúmina  seguidos  de  20  mg  de  furosemida  (Lasix)  por  vía  intravenosa  (IV).  Si  el  paciente  está  
edematoso,  la  albúmina  intravenosa  teóricamente  succiona  agua  osmóticamente  fuera  del  tercer  espacio  
intersticial.  A  medida  que  el  exceso  de  agua  ingresa  al  compartimento  vascular,  Lasix  produce  una  diuresis  
saludable.  Sin  embargo,  en  la  mayoría  de  los  pacientes  en  la  unidad  de  cuidados  intensivos  (UCI),  la  albúmina  
infundida  se  equilibra  rápidamente  a  través  del  endotelio  vascular  dañado.  No  se  introduce  agua  adicional  en  
el  volumen  de  sangre.  Aunque  los  cirujanos  a  menudo  piden  sándwiches  de  Lasix,  probablemente  solo  
funcionen  en  pacientes  sanos  que  no  los  necesitan.

BIBLIOGRAFÍA

1.  Brown  MD:  Medicina  de  emergencia  basada  en  la  evidencia:  cristaloide  hipertónico  versus  isotónico  para  la  reanimación  con  líquidos  en
pacientes  críticamente  enfermos.  Ann  Emerg  Med  40:113­114,  2002.

2.  Bunn  F,  Roberts  I,  Tasker  R  et  al .:  Cristaloide  hipertónico  versus  isotónico  para  la  reanimación  con  líquidos  en  pacientes  en  estado  crítico
pacientes  Cochrane  Database  Syst  Rev  (1):CD002045,  2002.

3.  Dellinger  RP,  Levy  MM,  Carlet  JM  et  al.:  Surviving  Sepsis  Campaign:  directrices  internacionales  para  el  manejo  de  la  sepsis  grave  y  el  shock  séptico:  
2008.  Crit  Care  Med  36:296­327,  2008.

4.  Greaves  I,  Porter  KM,  Revell  MP:  Reanimación  con  líquidos  en  la  atención  traumatológica  prehospitalaria:  una  visión  de  consenso.  JR  Coll
Surg  Edinb  47:451­457,  2002.

5.  Perel  P,  Roberts  I:  Coloides  versus  cristaloides  para  la  reanimación  con  líquidos  en  pacientes  en  estado  crítico.  Base  de  datos  Cochrane
Revisión  del  sistema  (4):CD000567,  2004.

6.  Traber  DL:  Reanimación  con  líquidos  después  de  hipovolemia.  Crit  Care  Med  30:1922,  2002.
Machine Translated by Google

VALORACIÓN  NUTRICIONAL,  PARENTERAL,
Y  NUTRICIÓN  ENTERAL CAPÍTULO  
9

Margaret  M.  McQuiggan,  MS,  RD,  CNSD  y  Frederick  A.  Moore,  MD,  FACS

EVALUACIÓN  NUTRICIONAL

1.  ¿Qué  incluye  una  valoración  nutricional?
Los  antecedentes  médicos  y  quirúrgicos  determinan  condiciones  preexistentes,  estrés  metabólico  y  
alteraciones  en  la  función  de  órganos  que  influyen  en  el  soporte  nutricional.  El  examen  físico  evalúa  la  masa  
muscular,  las  reservas  adiposas,  la  integridad  de  la  piel  y  el  estado  de  hidratación.  Los  datos  de  laboratorio  
incluyen  sodio  sérico  (Na),  potasio  (K),  dióxido  de  carbono  (CO2),  cloruro  (Cl),  nitrógeno  ureico  en  sangre  
(BUN),  creatinina,  glucosa,  calcio  ionizado  (Ca),  fosfato  sérico  (PO4),  magnesio  ( Mg)  y  conteo  sanguíneo  
completo  (CBC)  con  diferencial.  Son  útiles  los  gases  en  sangre  arterial  (GSA)  para  evaluar  el  estado  ácido­base  
y  la  retención  de  CO2,  la  albúmina,  la  transferrina,  la  prealbúmina  y  el  nitrógeno  urinario.  La  hemoglobina  
glicosilada  (HgbA1C),  el  perfil  de  lípidos,  la  proteína  C  reactiva  (CRP),  la  25­OH  vitamina  D,  los  elementos  traza  
y  las  pruebas  de  función  hepática  (LFT)  también  pueden  ser  valiosos.  El  perfil  de  fármacos  revela  agentes  que  
afectan  el  metabolismo  de  los  nutrientes  (insulina,  levotiroxina,  corticosteroides),  alteran  el  gasto  de  energía  
(bloqueadores  beta,  Diprivan)  o  afectan  la  función  gastrointestinal  (GI)  (agentes  procinéticos,  antibióticos).  Los  
datos  antropométricos  incluyen  altura,  peso,  circunferencia  de  cintura  y  cadera.  La  prueba  de  pliegues  cutáneos  
con  calibradores  es  útil  una  vez  que  se  ha  resuelto  el  edema,  pero  rara  vez  se  usa  en  el  entorno  de  cuidados  
agudos  sin  grasa.  El  análisis  de  impedancia  bioeléctrica  (BIA)  cuantifica  la  reserva  adiposa,  el  agua  intracelular  
y  extracelular  y  el  líquido  del  tercer  espacio  en  pacientes  quirúrgicos  estables.  La  absorciometría  de  rayos  X  de  
energía  dual  (DEXA)  es  eficaz  para  rastrear  la  densidad  mineral  ósea  que  puede  verse  comprometida  con  la  
edad,  el  estado  hormonal,  la  terapia  con  medicamentos  y  las  enfermedades  crónicas.  La  historia  nutricional  
revela  información  sobre  las  prácticas  nutricionales  del  individuo.  La  historia  social  explora  los  datos  económicos,  
la  red  de  apoyo  social  o  las  conductas  de  abuso  de  sustancias  y  puede  predecir  la  probabilidad  de  una  atención  
domiciliaria  adecuada  y  el  cumplimiento  del  tratamiento  por  parte  del  paciente,  una  vez  dado  de  alta.

2.  ¿Qué  son  la  desnutrición  primaria  y  la  desnutrición  secundaria?
La  desnutrición  primaria  se  produce  cuando  el  individuo  consume  calorías,  proteínas,  vitaminas  o  minerales  
inadecuados.  Puede  ocurrir  como  resultado  de  malas  elecciones  de  alimentos,  anorexia,  pobreza,  
alcoholismo,  regímenes  de  apoyo  subóptimos  o  después  de  una  cirugía  bariátrica.  La  desnutrición  secundaria  
ocurre  incluso  cuando  se  infunden  o  consumen  alimentos  adecuados.  Puede  ser  el  resultado  de  disfunción  
orgánica  (hipoalbuminemia  con  cirrosis),  malabsorción  (enfermedad  de  Crohn),  inmovilidad  (pérdida  de  masa  
muscular),  tratamiento  farmacológico  (resistencia  a  la  insulina  con  corticosteroides)  o  el  estado  inflamatorio  
(respuesta  de  fase  aguda).

3.  ¿Cuál  es  la  importancia  de  las  proteínas  séricas  en  la  evaluación  nutricional?
Las  proteínas  citadas  con  más  frecuencia  y  fácilmente  disponibles  para  la  valoración  nutricional  son  la  
albúmina,  la  transferrina  y  la  prealbúmina,  que  se  producen  en  el  hígado  ( cuadro  9­1).  Las  tres  proteínas  
constitutivas  caen  en  picado  poco  después  de  una  lesión  o  cirugía  porque  el  hígado  vuelve  a  priorizar  la  
producción  de  proteínas  de  fase  aguda.  Luego,  a  medida  que  se  resuelve  la  respuesta  al  estrés,  el  hígado  reanuda

53
ENT
PAR
CAPÍ
NUT
EVA
54  
Y  
9  
Machine Translated by Google

TABLA  9­1.  PROTEINAS  SÉRICAS

Proteína

Albúmina
Sintético
Sitio

Hígado
Significación  clínica

Se  relaciona  con  los  resultados;  se  relaciona  
con  el  edema
Media  vida

20–21  días
Limitaciones

En  el  mejor  de  los  casos  para  la  
producción  hepática:  12  a  25  g/24  horas;  
efectos  de  dilución;  larga  vida  media;  
usado  solo,  sensibilidad  pobre
Interpretación

Normal  <3,5  g/dl  
Depleción  leve  2,8–3,5  g/dl  
Moderada  2,2–2,8  g/dl  Severa  <2,2  
g/dl

prealbúmina Hígado Indica  déficits  nutricionales  antes   2–4  días Vida  media  corta Normales  >18  mg/dl


que  la  albúmina. Depleción  leve  10–18  mg/dl
Moderado  5­10  mg/dl
Grave  <5  mg/dl

transferrina Hígado Más  sensible  que  la  albúmina;   8–10  días Pobre  marcador  de  reposición  temprana;   Depleción  leve  150–200  mg/dl


parámetro  relativamente  útil  en  la   sensible  a  los  cambios  en  el  hierro  corporal Moderado  100–150  mg/dl
enfermedad  hepática  en  comparación   Grave  <100  mg/dl
con  la  albúmina;  puede  calcular  de
TIBC

C­reactivo Hígado Aumenta  abruptamente  después   48–72  horas Puede  aumentar  con  la  obesidad  y  otros   Normal  basal  <3  mg/dl


Proteína  (PCR) de  la  lesión.  Indicador  temprano   estados  inflamatorios  crónicos. Infección  bacteriana  30­35  mg/dl
y  confiable  de  la  gravedad  de  la   Infección  viral  <20  mg/dl
enfermedad  o  lesión. Picos  48–72  horas  después  del  trauma  
hasta  35  mg/dl

TIBC,  capacidad  total  de  fijación  de  hierro.
Machine Translated by Google
CAPÍTULO  9  EVALUACIÓN  NUTRICIONAL,  NUTRICIÓN  PARENTERAL  Y  ENTERAL  55

producción  de  proteínas  constitutivas.  Las  kilocalorías  y  las  proteínas  adecuadas  facilitan  este  proceso.  Como  
resultado  de  vidas  medias  más  cortas,  la  prealbúmina  y  la  transferrina  son  más  útiles  en  la  unidad  de  cuidados  
intensivos  (UCI)  y  deben  limitarse  a  pacientes  con  depuración  de  creatinina  >50  ml/min.  La  prealbúmina  viaja  en  la  
circulación  unida  a  la  proteína  fijadora  de  retinol  (RBP)  y  la  vitamina  A.  Los  niveles  de  prealbúmina  pueden  estar  
elevados  en  la  insuficiencia  renal  a  pesar  del  compromiso  nutricional  resultante  de  la  disminución  del  catabolismo  y  la  
excreción  de  RBP.  La  transferrina  se  eleva  con  la  depleción  de  hierro,  independientemente  de  los  efectos  de  la  
nutrición.

4.  ¿Cómo  se  determinan  los  requerimientos  de  proteínas?
La  necesidad  de  proteínas  se  determina  en  función  del  peso  del  paciente,  los  factores  de  estrés  actuales,  las  
pérdidas  extraordinarias  de  piel  y  la  función  de  los  órganos.  Aunque  la  ingesta  diaria  recomendada  (IDR)  de  proteína  
para  personas  sanas  es  de  solo  0,8  g  de  proteína/kg  de  peso  corporal,  se  pueden  utilizar  las  siguientes  pautas  en  el  
paciente  quirúrgico:  Nivel  de  lesión  Lesión/estrés  leve  Lesión/estrés  moderado  Lesión/estrés  grave
Requisito  de  proteína  1,2  
a  1,4  g  de  proteína/kg  1,5  a  
1,7  g  de  proteína/kg  1,8  a  2,5  
g  de  proteína/kg

5.  ¿Cuál  es  la  importancia  del  nitrógeno  urinario  en  la  evaluación  nutricional?
El  nitrógeno  urinario  total  (TUN)  es  el  indicador  más  confiable  del  uso  y  excreción  de  nitrógeno  en  el  paciente  
que  se  encuentra  en  la  unidad  de  cuidados  intensivos  quirúrgicos  (UCI).  Sin  embargo,  el  nitrógeno  ureico  urinario  
(UUN)  está  más  disponible  en  la  mayoría  de  los  laboratorios  hospitalarios.  Aunque  TUN  y  UUN  son  casi  iguales  
en  pacientes  ambulatorios  sanos  con  función  renal  y  hepática  normales,  los  pacientes  en  estado  crítico  tienen  una  
correlación  pobre  entre  los  dos.  Una  recolección  de  orina  de  12  horas  se  compara  bien  con  una  recolección  de  24  
horas  (Graves).  El  apoyo  nutricional  óptimo  promueve  un  balance  de  nitrógeno  de  þ3  a  þ5.  Estime  las  necesidades  
proteicas  del  paciente  añadiendo:

½24h  UUN  ðgÞ  þ  2  g  N  pérdidas  insensibles  þ  3  6:25  ¼  cantidad  requerida  de  proteína  ðgÞ

Recuerda  que  6,25  g  de  proteína  rinden  1  g  de  nitrógeno.  Las  pérdidas  insensibles  se  incrementan  con  
quemaduras,  decúbito,  heridas  por  vacío  y  grandes  heridas.  La  UUN  no  es  útil  como  guía  de  prescripción  
nutricional  en  insuficiencia  hepática,  disfunción  renal  (aclaramiento  de  creatinina  <  50  ml/min)  o  lesión  medular  
reciente.

6.  ¿Debe  restringirse  severamente  la  proteína  en  el  paciente  quirúrgico  con  insuficiencia  hepática?
insuficiencia  o  insuficiencia  renal?
Limite  las  proteínas  a  0,6  a  0,8  g/kg  en  el  paciente  con  encefalopatía  hepática,  si  la  encefalopatía  produce  
consecuencias  clínicas  significativas.  Sin  embargo,  solo  alrededor  del  10%  de  los  pacientes  con  enfermedad  hepática  
crónica  son  sensibles  a  las  proteínas;  por  lo  tanto,  deben  explorarse  otras  causas  de  encefalopatía,  como  infección,  
estreñimiento  y  alteraciones  electrolíticas.  De  lo  contrario,  dé  una  carga  de  proteína  posquirúrgica  más  típica  (1,3  a  
1,5  g/kg).
En  pacientes  lesionados  y  gravemente  enfermos  con  insuficiencia  renal,  equilibre  la  necesidad  de  aumentar  las  
proteínas  con  la  necesidad  de  aumentar  la  diálisis.  Dar  la  proteína  adecuada  puede  requerir  una  diálisis  más  frecuente.
Las  pérdidas  y  los  requerimientos  de  aminoácidos  aumentan  con  la  hemodiálisis  (HD)  más  intensiva  (10  a  12  g  de  
aminoácidos  eliminados  con  cada  HD,  o  5  a  12  g  de  aminoácidos  al  día  con  hemodiálisis  venovenosa  continua  
[CVVHD]).

7.  ¿Cómo  se  determinan  las  necesidades  de  kilocalorías?

Existen  numerosos  métodos  para  establecer  objetivos  de  kilocalorías  en  el  paciente  quirúrgico:  (a)  ecuaciones  de  
predicción,  (b)  estimaciones  de  kcal/kg  o,  (c)  calorimetría  indirecta.  Una  ecuación  de  predicción  común,  la  Harris  
Benedict  (HBE),  se  desarrolló  en  1919  para  su  uso  en  personas  sanas,  en  ayunas  y  ambulatorias,  pero  tiene  una  
utilidad  limitada  en  pacientes  hospitalizados.
Se  han  desarrollado  varias  ecuaciones  de  predicción,  pero  la  mayoría  de  los  médicos  emplean  un  objetivo  total  
de  kcal/kg  como  se  muestra  en  la  tabla  9­2.
Machine Translated by Google
56  CAPÍTULO  9  EVALUACIÓN  NUTRICIONAL,  NUTRICIÓN  PARENTERAL  Y  ENTERAL

TABLA  9­2.  KI  LO  CA  LORI  Y  METAS  EN  QUIRÚRGICA  CALPA  TI  EN  T

Nivel  de   Nivel  por  Indirecto
Paciente alimentación  (kcal/kg) Calorimetría

Pacientes  con  peso  normal 25–30 REE*  1.0

Pacientes  con  bajo  peso 30–35   REE  1.2

Pacientes  obesos 20–25{ REE  0.85

Obesidad  mórbida 10­20{ REE  0.75

*El  gasto  de  energía  basal  (BEE)  es  el  número  de  kilocalorías  gastadas  en  reposo,  en  ayunas.  El  gasto  
energético  en  reposo  (REE)  se  mide  en  estado  alimentado  y  es  entre  un  5  %  y  un  10  %  superior  al  BEE.
{
Peso  ajustado  ¼  [(Peso  real  Peso  ideal)  0,25]  þ  Peso  ideal

8.  ¿Qué  es  la  calorimetría  indirecta  y  cuándo  es  útil?
La  calorimetría  indirecta  es  una  prueba  respiratoria  que  mide  la  producción  de  CO2  y  el  consumo  de  oxígeno  
del  paciente  durante  aproximadamente  30  minutos,  hasta  alcanzar  el  estado  estacionario.  Los  resultados  se  
integran  en  la  ecuación  de  Weir  modificada:

REE  ¼  ½ð3:796  VO2Þþð1:214  VCO2Þ  1440  min=día

Donde:  
REE  ¼  gasto  energético  en  reposo  (kcal/día)
VO2  ¼  consumo  de  oxígeno  (L/min)
VCO2  ¼  CO2  exhalado  (L/min)
El  informe  indica  el  número  de  kilocalorías  que  consume  el  paciente  en  24  horas  y  el  cociente  respiratorio  
(RQ).  RQ  ¼  VCO2/VO2  y  proporciona  información  sobre  el  tipo  de  sustrato  que  se  está  utilizando.  Los  RQ  para  
el  metabolismo  de  grasas,  proteínas  y  carbohidratos  son  0,7,  0,83  y  1,0,  respectivamente.  La  sobrealimentación  
dará  como  resultado  un  RQ  >1.0,  como  resultado  de  una  mayor  producción  de  CO2  asociada  con  la  lipogénesis.

La  prueba  es  útil  en  el  paciente  con  ventilación  mecánica  (VM)  una  vez  que  el  paciente  está  relativamente
estable,  con  una  concentración  fraccional  de  oxígeno  en  el  gas  inspirado  (FiO2)  <60%  y  una  presión  
máxima  al  final  de  la  espiración  (PEEP)  <10.  Los  estudios  son  útiles:
a.  Cuando  la  sobrealimentación  (diabetes  mellitus,  enfermedad  pulmonar  obstructiva  crónica,  obesidad)  es
indeseable
b.  Cuando  la  subalimentación  (insuficiencia  renal,  heridas  grandes)  es  perjudicial.  
C.  En  pacientes  cuyos  factores  físicos  o  clínicos  promuevan  alteraciones  en  el  gasto  energético  (lesión  
medular).
d.  Cuando  se  utilizan  fármacos  que  alteran  significativamente  el  gasto  energético  (paralizantes,
bloqueadores  beta).

mi.  En  pacientes  que  no  responden  como  se  esperaba  a  los  regímenes  calculados.

NUTRICIÓN  ENTERAL

9.  ¿Cuándo  se  debe  considerar  la  nutrición  enteral?
Siempre,  pero  especialmente  cuando  es  improbable  que  un  paciente  satisfaga  >70%  de  las  necesidades  nutricionales  por  vía  oral.
Los  pacientes  que  han  sufrido  una  lesión  importante  en  la  cabeza  (escala  de  coma  de  Glasgow  <8),  un  traumatismo  
importante  en  el  torso,  un  traumatismo  importante  en  la  pelvis  y  los  huesos  largos  o  un  traumatismo  torácico  importante  se  benefician  

de  la  nutrición  enteral.  Aproximadamente  el  85%  de  los  pacientes  (incluso  los  que  se  someten  a  cirugía  GI)  toleran  la  alimentación  
enteral  temprana  dentro  de  las  24  horas  posteriores  a  la  operación.
Machine Translated by Google
CAPÍTULO  9  EVALUACIÓN  NUTRICIONAL,  NUTRICIÓN  PARENTERAL  Y  ENTERAL  57

10.  ¿Cómo  accede  al  tracto  gastrointestinal  para  alimentarse?
Busque  el  acceso  mediante  la  colocación  a  ciegas  de  una  sonda  nasogástrica  (NG)  o  una  sonda  nasoduodenal.  
Colocar  una  sonda  nasoyeyunal  (NJ)  a  ciegas,  endoscópica  o  fluoroscópicamente.  Logre  la  descompresión  gástrica  
con  alimentaciones  nasoyeyunales  concurrentes  con  una  gastrostomía/yeyunostomía  percutánea  endoscópica  
(PEG/PEJ).  Como  alternativa,  coloque  una  gastrostomía  o  una  yeyunostomía  de  alimentación  intraoperatoriamente.

11.  ¿Qué  tipos  de  fórmulas  enterales  están  disponibles?
Los  alimentos  enterales  poliméricos  son  productos  sin  lactosa  a  base  de  soya  que  contienen  proteínas,  
carbohidratos  y  grasas  intactos.  La  mayoría  ofrece  1  kcal/ml  y  de  37  a  62  g  de  proteína  por  litro.  Las  
modificaciones  especiales  de  las  fórmulas  estándar  incluyen  fibra  dietética  o  agentes  que  mejoran  el  sistema  
inmunitario,  como  aceite  de  pescado,  arginina,  glutamina  y  nucleótidos.  Las  fórmulas  "elementales"  contienen  
aminoácidos,  di­,  tri­  y  quatra­péptidos,  dextrosa  y  un  mínimo  de  grasa.  Varias  fórmulas  concentradas  (2  kcal/ml)  
están  disponibles  para  uso  en  pacientes  con  insuficiencia  cardíaca  congestiva  (ICC),  insuficiencia  renal  e  insuficiencia  
hepática.  En  general,  los  productos  que  son  "específicos  para  una  enfermedad"  o  que  contienen  nutrientes  en  forma  
elemental  son  más  caros  que  los  productos  estándar.

12.  ¿Son  necesarias  fórmulas  especializadas  para  el  paciente  con  diabetes  mellitus  en  estado  crítico?

No.  Las  fórmulas  con  carbohidratos  reducidos  y  cargas  de  grasas  aumentadas  se  comercializan  como  
superiores  para  mantener  el  control  glucémico.  Estos  productos  no  han  mostrado  resultados  superiores  clínicamente  
significativos  en  pacientes  hospitalizados  en  ensayos  controlados  aleatorios.  El  uso  de  fórmulas  estándar  hiperproteicas  
en  carga  isocalórica  o  hipocalórica,  combinadas  con  una  adecuada  insulinoterapia,  es  el  tratamiento  más  eficaz  de  la  
hiperglucemia  en  el  paciente  estresado  con  DM  tipo  2.  El  nivel  de  control  glucémico  asociado  con  un  mejor  resultado  
se  logra  mejor  con  insulina,  a  diferencia  de  la  restricción  de  carbohidratos.  Además,  la  alimentación  gástrica  con  
fórmulas  ricas  en  grasas  en  el  paciente  diabético  con  gastroparesia  puede  retrasar  el  vaciamiento  gástrico  y  aumentar  
el  riesgo  de  aspiración.

13.  ¿Se  deben  usar  fórmulas  "pulmonares"  especializadas  en  todos  los  pacientes  en
ventiladores?
No.  Se  han  comercializado  fórmulas  especializadas  con  alto  contenido  de  grasas  omega­6  para  reducir  la  
producción  de  CO2  en  pacientes  con  EPOC  que  retienen  CO2.  En  teoría,  estas  fórmulas  minimizan  la  producción  
de  CO2  y  facilitan  el  destete.  Sin  embargo,  evitar  la  sobrealimentación  es  más  importante  para  reducir  la  
producción  de  CO2  que  proporcionar  fórmula  rica  en  grasas.  La  alimentación  gástrica  con  estos  productos  aumenta  
el  riesgo  de  aspiración.

14.  ¿Qué  complicaciones  se  relacionan  con  el  soporte  enteral?
La  alimentación  enteral  puede  producir  anomalías  electrolíticas,  hiperglucemia,  intolerancia  GI,  aspiración  
pulmonar  y  erosiones  nasofaríngeas.  Las  complicaciones  quirúrgicas  del  acceso  enteral  incluyen  fugas,  desalojo  del  
tubo,  vólvulo,  infección  de  tejidos  blandos  y  necrosis  intestinal.

15.  ¿Se  debe  esperar  a  los  ruidos  intestinales  o  flatos  antes  de  comenzar  la  enteral?
alimentaciones?
No.

16.  ¿Se  debería  retrasar  más  el  soporte  nutricional  en  pacientes  obesos  asumiendo  que
han  aumentado  las  reservas?
No.  Los  pacientes  obesos  tienen  más  grasa,  pero  durante  el  estrés  todos  los  pacientes  se  vuelven  hipermetabólicos  
y  descomponen  las  reservas  de  proteínas  endógenas  para  movilizar  los  aminoácidos  para  la  gluconeogénesis,  la  
producción  de  proteínas  y  el  trifosfato  de  adenosina.  Al  igual  que  con  los  pacientes  que  tienen  un  peso  normal,  los  
pacientes  obesos  requieren  suplementos  nutricionales  ricos  en  proteínas  para  satisfacer  las  mayores  demandas  
de  nitrógeno.
Machine Translated by Google
58  CAPÍTULO  9  EVALUACIÓN  NUTRICIONAL,  NUTRICIÓN  PARENTERAL  Y  ENTERAL

17.  ¿Se  deben  diluir  las  fórmulas  enterales  para  la  presentación  inicial?
No.  La  dilución  retrasa  el  logro  de  los  objetivos  de  alimentación  y  aumenta  la  probabilidad  de  contaminación  bacteriana.  
La  osmolaridad  de  la  solución  es  un  culpable  relativamente  menor  en  la  producción  de  diarrea.

18.  ¿Cómo  se  debe  manejar  la  diarrea  relacionada  con  la  alimentación  enteral?
La  diarrea  leve  generalmente  no  requiere  tratamiento.  Con  diarrea  moderada  a  severa,  considere  la  reducción  de  la  
alimentación,  agentes  antidiarreicos  y  estudios  de  heces  para  Clostridium  difficile.  Evaluar  el  perfil  de  medicación  para  elixires,  
laxantes,  ablandadores  de  heces  y  agentes  procinéticos  que  contienen  sorbitol.  Supervisar  los  problemas  de  saneamiento  
relacionados  con  el  manejo  de  la  fórmula.  Se  ha  informado  cierto  éxito  con  fibra  soluble  o  lactobacillus  (yogur)  en  la  diarrea  
asociada  con  antibióticos.

19.  Durante  la  alimentación  gástrica,  ¿a  qué  nivel  de  volumen  residual  gástrico  (GRV)  se  debe  mantener  la  alimentación?

Utilice  siempre  mediciones  del  volumen  residual  gástrico  (GRV)  junto  con  la  evaluación  clínica.
Aumente  la  vigilancia  con  200  a  500  ml  de  GRV  y  comience  con  agentes  procinéticos.  Suspender  las  alimentaciones  si  
GRV  >  500  ml.

20.  ¿La  alimentación  enteral  contiene  suficiente  agua  para  satisfacer  todas  las  necesidades  de  líquidos?
La  mayoría  de  las  fórmulas  de  1  kcal/ml  (estándar)  contienen  un  85  %  de  agua  por  volumen,  mientras  que  las  fórmulas  de  2  
kcal/ml  contienen  un  70  %  de  agua.  El  agua  generalmente  no  es  un  problema  en  el  paciente  en  la  UCI  que  recibe  múltiples  
líquidos  y  medicamentos  por  vía  intravenosa  (IV).  Sin  embargo,  después  de  la  UCI,  o  en  pacientes  destinados  al  hogar  o  a  
centros  de  cuidados  prolongados,  es  esencial  escribir  una  receta  de  agua  con  la  orden  de  alimentación  por  sonda.  Las  pautas  
generales  para  las  necesidades  totales  de  agua  se  muestran  en  la  Tabla  9­3.
Por  ejemplo,  si  la  necesidad  total  calculada  de  líquido  es  de  2400  ml  para  un  paciente  de  60  kg  y  2400  cc  de  alimentación  
por  sonda  proporcionan  aproximadamente  2000  ml  de  agua  gratis,  escriba  una  orden  para  dar  200  ml  de  agua  al  paciente  dos  
veces  al  día.

TABLA  9­3.  AGUA  DIARIA  Y  DE  S  EN  RELACIÓN  CON  LA  EDAD
Paciente Edad Necesidades  diarias  de  agua

Adulto  promedio 25–55  años 35ml/kg

Años  adultos  jóvenes  y  activos 16–35 40ml/kg

Adulto >55–65  años 30ml/kg

Anciano >65  años 25ml/kg

21.  ¿Cómo  se  infunde  la  nutrición  enteral?

La  nutrición  enteral  se  infunde  de  forma  continua,  en  forma  de  bolo  o  cíclicamente.  La  infusión  continua  es  mejor  en  el  
paciente  gravemente  enfermo  que  requiere  alimentación  pospilórica.  La  alimentación  en  bolo  se  utiliza  en  pacientes  más  
estables  con  alimentación  gástrica.  Las  alimentaciones  cíclicas  o  nocturnas  son  útiles  para  el  paciente  que  recibe  ingesta  oral  
concurrente  y  está  en  transición  al  apoyo  oral  completo,  o  para  aquellos  que  requieren  períodos  sin  alimentación  para  
fisioterapia  o  actividades  de  la  vida  diaria.

22.  ¿Es  mejor  la  nutrición  enteral  que  la  nutrición  parenteral  total?
Sí.  Los  sustratos  administrados  por  vía  enteral  se  toleran  mejor,  se  asocian  con  menos  complicaciones  metabólicas  y  
hepáticas  y  ayudan  a  preservar  la  integridad  normal  de  la  mucosa.  El  ochenta  por  ciento  del  tejido  inmunológico  del  cuerpo  
está  en  el  intestino  y  necesita  nutrición  local  y  sistémica.  Una  revisión  de  13  estudios  que  describieron  un  total  de  856  
pacientes  que  estaban  en  estado  crítico,  contrastando  la  nutrición  parenteral  total  (TPN)  con  la  nutrición  enteral,  concluyó  que  
la  nutrición  enteral  reduce  las  complicaciones  infecciosas  y,  por  lo  general,  es  más  rentable  que  la  nutrición  parenteral.
Machine Translated by Google
CAPÍTULO  9  EVALUACIÓN  NUTRICIONAL,  NUTRICIÓN  PARENTERAL  Y  ENTERAL  59

23.  ¿Se  debe  interrumpir  la  alimentación  enteral  a  medianoche  en  todos  los  pacientes  sometidos  a  cirugía  
electiva  con  anestesia  general?
No.  La  Sociedad  Estadounidense  de  Anestesiólogos  recomienda  que  los  adultos  sanos  dejen  de  ingerir  sólidos  durante  al  
menos  6  horas  y  líquidos  durante  2  horas  antes  de  someterse  a  procedimientos  electivos.  Es  posible  que  sea  necesario  
modificar  las  pautas  para  pacientes  con  enfermedades  coexistentes  que  pueden  afectar  el  vaciamiento  gástrico:  embarazo,  
obesidad,  DM,  hernia  de  hiato,  enfermedad  por  reflujo  gastroesofágico  (ERGE),  íleo  u  obstrucción  intestinal,  atención  de  
emergencia  o  alimentación  por  sonda  enteral.  La  investigación  clínica  reciente  muestra  una  mejor  recuperación  
inmunológica  y  funcional  del  tracto  GI  con  períodos  de  ayuno  perioperatorios  reducidos.

24.  ¿Es  obligatoria  la  dieta  de  líquidos  claros  después  de  la  cirugía?
No.  Los  resultados  clínicos  son  similares  cuando  los  pacientes  ayunan  o  reciben  líquidos  claros  hasta  la  aparición  
de  flatos  o  evacuaciones  intestinales  en  lugar  de  recibir  una  dieta  regular  que  comienza  un  día  después  de  la  
operación.

25.  ¿La  nutrición  preoperatoria  con  dietas  inmunoestimulantes  mejora  la
¿resultado?

Sí.  Las  dietas  inmunoestimulantes  perioperatorias  (IED,  por  sus  siglas  en  inglés)  pueden  reducir  las  complicaciones  e  
infecciones  posoperatorias  y  mejorar  la  inmunocompetencia  posoperatoria  en  pacientes  adecuadamente  seleccionados.  
Las  recomendaciones  consensuadas  de  la  Cumbre  de  EE.  UU.  sobre  terapia  enteral  inmunoestimulante  (2001)  aconsejan  
administrar  IED  a  pacientes  gravemente  desnutridos  que  se  someten  a  cirugía  GI  inferior  de  5  a  7  días  antes  de  la  cirugía.  
Los  datos  emergentes  sugieren  un  beneficio  para  los  pacientes  que  no  están  clínicamente  desnutridos.

26.  ¿Se  debe  utilizar  el  peso  corporal  real,  ideal  o  ajustado  en  los  cálculos  de  nutrición  para  el  paciente  con  
obesidad?
Los  estudios  que  utilizan  un  peso  ajustado  por  obesidad  en  los  cálculos  de  kilocalorías  [peso  corporal  ideal  (PCI)  +  0,25  
(PCI  real)]  se  correlacionan  mejor  con  el  gasto  de  energía  medido  que  cuando  se  utiliza  el  peso  real.

CONTROVERSIAS  ENTERALES

27.  ¿Qué  son  los  probióticos  y  cuándo  son  útiles?
Los  probióticos  son  microbios  vivos  que  benefician  al  huésped.  Los  estudios  clínicos  muestran  el  uso  terapéutico  
o  preventivo  de  diversas  cepas  de  probióticos  para  la  diarrea  asociada  a  antibióticos,  la  diarrea  asociada  a  rotavirus  y  la  
reservoritis.  Los  resultados  son  prometedores  para  el  síndrome  del  intestino  irritable,  la  colitis  ulcerosa  y  para  la  reducción  
de  los  efectos  secundarios  en  la  terapia  con  antibióticos  para  Helicobacter  pylori.

28.  ¿Qué  es  más  importante:  el  nitrógeno  o  el  balance  calórico?
En  última  instancia,  mantener  un  balance  positivo  de  nitrógeno  puede  ser  más  importante  que  lograr  un  balance  positivo  
de  kilocalorías.

29.  ¿Son  superiores  las  alimentaciones  pospilóricas  a  las  alimentaciones  gástricas?
Después  de  una  cirugía  mayor  o  una  lesión,  el  vaciado  gástrico  se  ve  afectado  durante  varios  días.  La  alimentación  
enteral  temprana,  con  sus  beneficios  conocidos,  puede  no  ocurrir  fácilmente  a  través  de  una  alimentación  gástrica  en  las  
primeras  etapas  de  la  lesión.  Las  alimentaciones  pospilóricas  entregan  más  kilocalorías,  un  retorno  más  oportuno  al  
anabolismo  y  promueven  una  tasa  más  baja  de  complicaciones  infecciosas  que  la  alimentación  gástrica  continua.

30.  ¿Cuándo  se  deben  usar  fórmulas  para  mejorar  el  sistema  inmunológico?
Casi  nunca.  Los  pacientes  en  ensayos  controlados  aleatorios  han  demostrado  que  los  IED  mejoran  el  resultado  y  reducen  
la  morbilidad  séptica  en  pacientes  propensos  a  la  sepsis  intraabdominal  después  de  un  traumatismo  torácico  mayor  y  
después  de  una  resección  quirúrgica  mayor  de  cánceres  del  tracto  gastrointestinal  superior.  El  uso  de  IED  debe  ser
Machine Translated by Google
60  CAPÍTULO  9  EVALUACIÓN  NUTRICIONAL,  NUTRICIÓN  PARENTERAL  Y  ENTERAL

restringido  a  estos  pacientes,  y  la  duración  debe  ser  limitada  debido  al  aumento  de  los  gastos.
Los  IED  no  se  han  probado  adecuadamente  en  otros  tipos  de  pacientes  y,  cuando  se  probaron  en  una  variedad  de  
pacientes  en  la  UCI,  hay  algunas  pruebas  que  sugieren  que  podrían  ser  dañinos.
Existe  la  preocupación  de  que  la  administración  de  arginina  en  pacientes  con  sepsis,  que  tienen  una  expresión  
aumentada  de  la  óxido  nitroso  sintasa  inducible  (iNos),  dará  como  resultado  una  producción  excesiva  de  óxido  nitroso  
y  una  vasodilatación  exagerada  resultante  y  estrés  oxidativo.

31.  ¿Se  debe  usar  fórmula  con  mayor  contenido  de  aceite  de  pescado  en  pacientes  que  van  a  tener  síndrome  de  dificultad  
respiratoria  aguda?
Dos  ensayos  controlados  aleatorios  financiados  por  la  industria  demuestran  un  resultado  superior  en  pacientes  
con  síndrome  de  dificultad  respiratoria  aguda  (SDRA)  cuando  se  les  proporciona  un  producto  enteral  con  alto  contenido  de  
ácidos  grasos  omega­3  en  comparación  con  una  fórmula  "pulmonar"  con  alto  contenido  de  omega­6.  Desafortunadamente,  
la  dieta  de  control,  una  fórmula  rica  en  ácidos  grasos  omega­6,  no  es  el  estándar  de  atención  y  puede  empeorar  el  SDRA.  
Los  ácidos  grasos  omega­6  altos  aumentan  la  inflamación  y  producen  mediadores  lipídicos  que  empeoran  el  desajuste  de  
ventilación/perfusión  (V/Q)  en  el  pulmón  y  empeoran  la  oxigenación  en  el  SDRA.
Se  necesita  un  ensayo  controlado  aleatorizado  que  compare  la  fórmula  polimérica  estándar  con  contenido  moderado  de  
grasas  y  una  fórmula  rica  en  omega­3.

NUTRICIÓN  PARENTERAL

32.  ¿Qué  es  la  nutrición  parenteral?
La  nutrición  parenteral  es  el  aporte  de  proteínas  en  forma  de  aminoácidos  (4  kcal/g),  dextrosa  (3,4  kcal/g)  y  grasas  (la  
solución  de  lípidos  al  20  %  aporta  2  kcal/ml),  vitaminas,  minerales,  oligoelementos,  líquidos  y,  a  veces,  insulina  a  través  de  
una  infusión  IV.

33.  ¿Cuáles  son  las  indicaciones  de  nutrición  parenteral?
Use  nutrición  parenteral  cuando  el  tracto  gastrointestinal  no  funcione  en  absoluto,  por  ejemplo,  resección  intestinal  
importante,  "intestino  corto",  peritonitis,  hemorragia  intestinal,  íleo  paralítico,  fístulas  enterocutáneas  de  alto  volumen,  íleo  
y  diarrea  intratable  grave  (>  1  litro/día) .

34.  ¿Qué  tipos  de  acceso  están  disponibles  para  la  entrega  de  nutrición  parenteral?
Las  soluciones  parenterales  centrales  tienen  osmolaridades  de  hasta  3000  mOsm/L.  Éstos  requieren  la  entrega  en  una  
vena  de  lumen  grande  (p.  ej.,  subclavia  o,  con  menos  frecuencia,  una  vena  femoral).  Si  se  utiliza  un  catéter  de  puertos  
múltiples,  debe  reservarse  un  «puerto  virgen»  exclusivamente  para  la  infusión  de  nutrientes.  Cuando  sea  necesaria  una  
infusión  de  nutrición  parenteral  prolongada  en  el  entorno  postagudo,  considere  un  dispositivo  de  acceso  a  largo  plazo,  
como  un  catéter  Hickman  o  Broviac.  Sin  embargo,  esto  puede  no  ser  necesario  cuando  el  catéter  venoso  central  se  coloca  
en  condiciones  estériles  y  el  paciente  y  los  cuidadores  brindan  un  cuidado  meticuloso.

35.  ¿Los  pacientes  con  pancreatitis  deben  ser  alimentados  exclusivamente  por  vía  parenteral?
Aunque  los  pacientes  con  pancreatitis  tradicionalmente  han  recibido  "descanso  intestinal"  y  TPN,  los  estudios  
demuestran  mejores  resultados  con  la  alimentación  enteral  más  allá  del  ligamento  de  Treitz.  La  alimentación  enteral  
administrada  en  el  yeyuno  promueve  una  menor  morbilidad  infecciosa,  una  estancia  hospitalaria  más  corta,  menos  
complicaciones,  una  resolución  más  rápida  del  síndrome  de  respuesta  inflamatoria  sistémica  (SIRS)  y  un  curso  de  la  
enfermedad  más  breve  que  la  nutrición  parenteral.  Si  no  se  tolera  la  nutrición  enteral,  se  debe  considerar  la  nutrición  
parenteral  no  antes  de  los  5  días  de  hospitalización.

36.  ¿Están  contraindicados  los  lípidos  intravenosos  en  la  pancreatitis?
En  casos  extremadamente  raros  de  pancreatitis  causada  por  hiperlipidemia  congénita,  se  deben  suspender  los  lípidos.  
Sin  embargo,  en  la  mayoría  de  los  casos  de  pancreatitis  grave  en  los  que  no  se  tolera  la  nutrición  enteral,  es  mejor  evitar  
las  emulsiones  de  lípidos  IV  hasta  que  la  respuesta  inflamatoria  haya  disminuido.
Machine Translated by Google
CAPÍTULO  9  EVALUACIÓN  NUTRICIONAL,  NUTRICIÓN  PARENTERAL  Y  ENTERAL  61

37.  ¿Qué  complicaciones  se  asocian  con  la  nutrición  parenteral?
El  desequilibrio  de  líquidos  y  electrolitos,  el  metabolismo  alterado  de  la  glucosa,  el  aumento  de  las  LFT,  la  esteatosis  
hepática,  la  candidiasis  sistémica,  las  infecciones  del  sitio  y  la  atrofia  intestinal  están  asociados  con  la  TPN.  Puede  ocurrir  
hemotórax  o  neumotórax  durante  la  colocación  de  la  vía  central.  Aunque  es  raro,  se  producen  embolias  de  aire  o  colocación  
extravascular  de  vías  centrales.

38.  ¿Por  qué  los  pacientes  alimentados  por  vía  parenteral  a  menudo  desarrollan  hiperglucemia?
Los  pacientes  que  se  alimentan  por  vía  parenteral  pueden  desarrollar  hiperglucemia  como  resultado  del  aumento  del  estrés  
y  la  respuesta  inflamatoria,  la  movilidad  limitada,  la  terapia  concomitante  con  esteroides  y  la  ingesta  excesiva  de  kilocalorías.

39.  ¿Cómo  se  debe  manejar  la  hiperglucemia?
Evaluar  la  información  sobre  el  régimen  de  control  de  la  glucosa  en  el  hogar  a  partir  del  historial  de  medicamentos.
A  menudo  es  necesaria  una  infusión  continua  de  insulina  durante  una  enfermedad  crítica  para  lograr  un  control  
glucémico  adecuado.  Cuando  los  requisitos  de  insulina  son  predecibles  y  el  paciente  se  vuelve  más  estable  
metabólicamente  y  sale  de  la  UCI,  se  agrega  insulina  a  la  TPN.  La  insulina  NPH  está  dirigida  a  pacientes  que  consumen  
comidas  a  intervalos  regulares  y,  por  lo  tanto,  no  es  apropiada  con  alimentaciones  intravenosas  continuas.  Las  tasas  de  
infusión  de  glucosa  no  deben  exceder  los  5  miligramos  por  kilogramo  por  minuto.

40.  ¿Por  qué  se  utilizan  las  emulsiones  grasas  intravenosas  y  cuándo  están  contraindicadas?
Teóricamente,  las  emulsiones  grasas  se  emplean  para  prevenir  la  deficiencia  de  ácidos  grasos  esenciales.  En  realidad,  
esta  afección  es  rara,  tarda  varias  semanas  en  desarrollarse  y  requiere  solo  del  3%  al  4%  de  kilocalorías  como  ácido  
linoleico  (o  10%  de  kilocalorías  como  emulsión  de  grasa  estándar).  Las  emulsiones  grasas  también  se  utilizan  para  
proporcionar  kilocalorías  adicionales  una  vez  que  el  suministro  de  glucosa  supera  las  5  kilocalorías  por  kilogramo  por  
minuto.  Cuando  se  entregan  en  mezclas  de  nutrientes  totales  (soluciones  3  en  1),  las  emulsiones  lipídicas  son  estables  
durante  24  horas.  Cuando  se  infunde  como  único  nutriente,  limite  los  tiempos  de  colgado  a  <12  horas  para  evitar  el  
crecimiento  bacteriano.  Evite  las  emulsiones  grasas  con  pancreatitis  inducida  por  hiperlipidemia  y  cuando  los  triglicéridos  
séricos  estén  significativamente  elevados  (p.  ej.,  >500  mg/dl).  Debido  a  que  están  asociados  con  una  mayor  mortalidad  
temprana  después  del  trauma  y  un  aumento  de  infecciones  en  enfermedades  críticas,  el  riesgo  de  emulsión  de  grasa  IV  
supera  el  beneficio  durante  la  respuesta  de  fase  aguda  temprana.

41.  ¿Qué  es  el  síndrome  de  realimentación  y  cómo  se  controla  o  previene?
El  síndrome  de  realimentación  ocurre  cuando  un  paciente  está  desnutrido  de  moderado  a  severo  y  tiene  reservas  de  
sustrato  limitadas,  generalmente  como  resultado  de  alcoholismo  crónico,  anorexia  nerviosa,  cirugía  posbariátrica  o  
inanición  crónica.  Cuando  se  le  presenta  una  gran  carga  de  nutrientes,  el  paciente  desarrolla  rápidamente  una  disminución  
clínicamente  significativa  en  el  suero  K,  fósforo  (P),  Ca  y  Mg  debido  a  los  cambios  de  compartimento  o  al  aumento  de  la  
utilización  de  estos  iones.  La  hiperglucemia  es  común  como  resultado  de  la  disminución  de  la  secreción  de  insulina  basal  
(ver  Kraft).  Proporcionar  amplias  cantidades  de  K,  P,  Ca  y  Mg  con  la  mezcla  parenteral  inicial,  dentro  de  los  límites  de  
solubilidad  de  la  solución.  Reducir  la  carga  inicial  de  kilocalorías  en  un  25  %  del  objetivo  limitando  las  kilocalorías  de  
dextrosa.  Controle  la  glucosa  en  sangre  4  veces  al  día  y  el  K,  P,  Ca  y  Mg  en  suero  diariamente  durante  5  días  después  de  
iniciar  la  alimentación,  mientras  aumenta  las  kilocalorías  hasta  los  niveles  objetivo.

42.  ¿Cómo  se  debe  monitorear  la  nutrición  parenteral?
La  nutrición  parenteral  debe  monitorearse  diariamente  con  química  sérica  (Na,  K,  Cl,  CO2,  glucosa,  Mg,  P  y  Ca)  
durante  los  primeros  días  de  terapia  en  el  entorno  de  cuidados  intensivos.  Controle  la  glucosa  en  sangre  cada  6  horas.  
Con  un  equilibrio  aceptable  de  líquidos  y  electrolitos,  reduzca  la  frecuencia  a  1  o  2  veces  por  semana.  La  idoneidad  del  
régimen  de  nutrición  puede  evaluarse  mediante  la  evidencia  de  una  cicatrización  adecuada  de  la  herida,  el  mantenimiento  
del  estado  de  hidratación,  la  preservación  de  la  masa  celular  corporal  y  la  reposición  oportuna  de  los  niveles  de  proteínas  
constitutivas.  La  sobrealimentación  puede  presentarse  como  resistencia  a  la  insulina,  hipertrigliceridemia,  aumento  de  LFT  
e  hipercapnia.
Machine Translated by Google
62  CAPÍTULO  9  EVALUACIÓN  NUTRICIONAL,  NUTRICIÓN  PARENTERAL  Y  ENTERAL

43.  ¿Qué  programas  de  infusión  se  utilizan  para  la  TPN?

TPN  generalmente  se  infunde  continuamente.  En  pacientes  más  ambulatorios  y  en  terapia  domiciliaria,  un  programa  de  
infusión  cíclica  o  nocturna  (ciclo  de  12  a  18  horas)  aumenta  la  libertad  del  paciente.

44.  ¿Cómo  se  debe  suspender  la  TPN?

Para  suspender  la  TPN,  reduzca  la  velocidad  de  infusión  a  la  mitad  durante  2  horas,  vuelva  a  reducirla  a  la  mitad  
durante  2  horas  y  luego  apáguela.  Esta  "descenso  gradual"  previene  la  hipoglucemia  reactiva.

45.  ¿Cuál  es  el  costo  de  la  nutrición  parenteral?
Los  costos  de  la  solución  parenteral  varían  ampliamente  dependiendo  de  los  constituyentes.  El  costo  de  los  componentes  
de  la  solución  de  TPN,  la  preparación,  los  dispositivos  de  acceso  y  el  monitoreo  de  laboratorio  cuesta  hasta  10  veces  
más  que  una  alimentación  enteral  estándar.  Muchos  terceros  pagadores  no  brindan  más  reembolso  por  la  terapia  
parenteral  que  por  la  enteral  en  el  entorno  hospitalario.

46.  ¿Cuánto  intestino  es  necesario  para  evitar  la  dependencia  de  la  TPN  después  del  intestino  delgado?
¿resección?

El  intestino  delgado  adulto  normal  tiene  una  longitud  de  300  a  800  cm.  La  pérdida  de  más  de  dos  tercios  
se  considera  síndrome  de  intestino  corto.  La  condición  del  intestino  delgado  remanente  es  importante.

CONTROVERSIAS  DE  LOS  PADRES

47.  ¿Las  soluciones  de  TPN  deben  contener  el  mismo  porcentaje  de  kilocalorías  grasas  que  se  recomienda  en  la  dieta  
de  los  estadounidenses  sanos  (es  decir,  30  %  del  total  de  kilocalorías)?

Las  recomendaciones  de  la  American  Heart  Association  (AHA)  para  el  30  %  del  total  de  kilocalorías  como  grasa  
están  orientadas  a  la  prevención  de  enfermedades  cardiovasculares  en  personas  sanas  y  nunca  fueron  pensadas  
para  la  alimentación  intravenosa  en  personas  gravemente  enfermas.  Además,  la  AHA  sugiere  que  esas  kilocalorías  se  
distribuyan  entre  grasas  saturadas,  monoinsaturadas  y  poliinsaturadas,  incluidos  los  ácidos  grasos  de  la  serie  omega­3.  
Las  formulaciones  de  lípidos  actuales  disponibles  en  los  Estados  Unidos  están  hechas  de  aceite  de  soja  o  de  una  mezcla  
de  aceite  de  soja  y  cártamo;  por  lo  tanto,  son  predominantemente  grasas  poliinsaturadas  (omega­6).  Las  kilocalorías  de  
glucosa  son  las  kilocalorías  más  rentables,  seguidas  de  las  kilocalorías  de  aminoácidos  estándar  y  luego  las  calorías  de  
lípidos.  Las  infusiones  de  lípidos  que  superan  1  g/kg  de  peso  corporal  se  asocian  con  una  inmunocompetencia  disminuida  
y  una  oxigenación  alterada  en  pacientes  críticamente  enfermos.

48.  ¿La  glutamina  suplementaria  mejora  el  resultado  en  pacientes  quirúrgicos?
La  glutamina,  el  aminoácido  que  se  encuentra  en  mayor  concentración  en  el  músculo  y  el  plasma,  disminuye  después  
de  una  cirugía,  lesión  o  estrés.  Por  lo  tanto,  es  un  aminoácido  condicionalmente  esencial.  Desempeña  un  papel  como  
sustrato  metabólico  para  las  células  que  se  replican  rápidamente,  mantiene  la  integridad  y  la  función  de  la  barrera  
intestinal  y  protege  al  enterocito  del  daño  de  los  radicales  libres.  La  glutamina  no  se  incluye  en  las  soluciones  estándar  
de  aminoácidos  debido  a  su  solubilidad  y  estabilidad  limitadas.
La  suplementación  puede  reducir  las  tasas  de  complicaciones  infecciosas  y  disminuir  la  estancia  hospitalaria  en  
pacientes  quirúrgicos.

49.  ¿Deberían  usarse  rutinariamente  hormona  de  crecimiento  recombinante,  glutamina  y  una  dieta  modificada  para  maximizar  
la  adaptación  intestinal  después  de  la  resección  intestinal?
Cinco  ensayos  clínicos  han  aparecido  en  la  última  década.  Tres  mostraron  resultados  negativos,  mientras  que  dos  
mostraron  resultados  positivos.  Los  resultados  positivos  son  de  corta  duración.  Hasta  que  se  realicen  más  investigaciones,  
esta  costosa  terapia  no  debe  ser  rutinaria,  y  la  nutrición  intensiva  y  el  manejo  farmacológico  deben  seguir  siendo  el  pilar  
de  la  atención.
Machine Translated by Google
CAPÍTULO  9  EVALUACIÓN  NUTRICIONAL,  NUTRICIÓN  PARENTERAL  Y  ENTERAL  63

BIBLIOGRAFÍA

1.  Brady  M,  Kinn  S,  Stuart  P:  Ayuno  preoperatorio  para  adultos  para  prevenir  complicaciones  perioperatorias,  Cochrane
Base  de  datos  Syst  Rev  CD004423,  2003.

2.  Heyland  DK,  Dhaliwal  R,  Day  A  et  al.:  Comité  Canadiense  de  Pautas  de  Práctica  Clínica  de  Cuidados  Intensivos.  Pautas  de  práctica  
clínica  canadienses  para  el  apoyo  nutricional  en  pacientes  adultos  críticamente  enfermos  con  ventilación  mecánica,  JPEN  27:355­373,  
2003.

3.  Frankenfield  D,  Hise  M,  Malone  A  et  al .:  Predicción  de  la  tasa  metabólica  en  reposo  en  pacientes  adultos  en  estado  crítico:  resultados
de  una  revisión  sistemática  de  la  evidencia.  J  Am  Diet  Assoc  107:1552­1561,  2007.

4.  Gadek  JE,  DeMichele  SJ,  Karlstad  MD  et  al.:  Efecto  de  la  alimentación  enteral  con  ácido  eicopentaenoico,  ácido  gamma­linolénico  y  
antioxidantes  en  pacientes  con  síndrome  de  dificultad  respiratoria  aguda.  Crit  Care  Med  27:1409­1420,  1999.

5.  Graves  C,  Saffle  J,  Morris  S:  Comparación  de  los  tiempos  de  recolección  de  nitrógeno  ureico  en  orina  en  pacientes  en  estado  crítico.  
Nutr  Clin  Pract  20:271­275,  2005.

6.  Haugen  HA,  Chan  LN,  Li  F:  Calorimetría  indirecta:  una  guía  práctica  para  médicos.  Nutr  Clin  Pract  22:377­388,  2007.

7.  Guías  de  práctica  clínica  KDOQI  para  nutrición  en  insuficiencia  renal  crónica.  Am  J  Kidney  Dis  35  (6  suplemento  2);  S1­S140,
2000.

8.  Konstantinides  FN,  Konstantinides  NN,  Li  JC  et  al .:  Nitrógeno  de  urea  urinaria:  demasiado  sensible  para  calcular  los  estudios  de  balance  
de  nitrógeno  en  nutrición  clínica  quirúrgica.  JPEN  15:189­193,  1991.

9.  Kozar  R,  McQuiggan  M,  Moore  F:  Apoyo  nutricional  de  pacientes  con  trauma.  En  Shikora  S,  Martindale  RG,
Schwaitzburg  S,  editores:  Consideraciones  nutricionales  en  la  unidad  de  cuidados  intensivos.  Silver  Springs,  MD,  2002,  ASPEN  
Publishers.

10.  Kraft  MD,  Btaiche  IF,  Sacks  GS:  Revisión  del  síndrome  de  realimentación.  Nutr  Clin  Pract  20:625­633,  2005.

11.  Matarese  LE,  O'Keefe  SJ,  Kandil  HM  et  al .:  Síndrome  del  intestino  corto:  pautas  clínicas  para  el  manejo  de  la  nutrición.
Nutr  Clin  Pract  20:493­502,  2005.

12.  McClave  SA:  Apoyo  nutricional  en  pancreatitis  aguda.  Gastroenterol  Clin  North  Am  36:65­74,  2007.

13.  Novak  F,  Heyland  DK,  Avenell  A  et  al .:  suplementos  de  glutamina  en  enfermedades  graves:  una  revisión  sistemática  de  la  evidencia.  Crit  
Care  Med  30:2022­2029,  2002.

14.  Pontes­Arruda  A,  Araga˜o  AM,  Albuquerque  JD:  Efectos  de  la  alimentación  enteral  con  ácido  eicosapentaenoico,
ácido  gamma­linolénico  y  antioxidantes  en  pacientes  ventilados  mecánicamente  con  sepsis  grave  y  shock  séptico.
Crit  Care  Med  34:  2325­2333,  2006.

15.  Van  den  Berghe  G,  Wouters  P,  Weekers  F  et  al.:  Terapia  intensiva  de  insulina  en  pacientes  críticos.  N  Inglés  J  Med
345:1359­1367,  2001.
Machine Translated by Google

¿QUÉ  SIGNIFICA  FIEBRE  
CAPÍTULO  
10

POSTOPERATORIA?
Dr.  Alden  H.  Harken

1.  ¿Qué  es  la  fiebre?

La  temperatura  central  normal  varía  entre  36  C  y  38  C.  Debido  a  que  los  humanos  hibernan  un  poco  durante  la  noche,  
estamos  frescos  (36  C)  justo  antes  de  levantarnos  por  la  mañana;  después  de  acelerar  nuestros  motores  todo  el  día,  
tenemos  calor  por  la  noche  (38  C).  La  fiebre  es  un  estado  patológico  que  refleja  un  proceso  inflamatorio  sistémico.  La  
temperatura  central  es  >38  C  pero  rara  vez  >40  C.

2.  ¿Qué  es  la  hipertermia  maligna?
Una  respuesta  rara  y  potencialmente  mortal  a  los  anestésicos  inhalados  o  a  algunos  relajantes  musculares.  La  
temperatura  central  aumenta  >40  C.  El  metabolismo  anormal  del  calcio  en  el  músculo  esquelético  produce  calor,  
acidosis,  hipopotasemia,  rigidez  muscular,  coagulopatía  y  colapso  circulatorio.

3.  ¿Cómo  se  trata  la  hipertermia  maligna?
&  Detener  la  anestesia.
  Administre  bicarbonato  de  sodio  (2  mEq/kg  por  vía  intravenosa  [IV]).
  Administre  dantroleno  (bloqueador  de  los  canales  de  calcio  a  2,5  mg/kg  IV).
&  Continúe  con  dantroleno  (1  mg/kg  cada  6  horas  durante  48  horas).
&  Refresque  al  paciente  con  esponjas  con  alcohol  y  hielo.

PUNTOS  CLAVE:  HIPERTERMIA  MALIGNA
1.  Respuesta  catastrófica  rara,  familiar  (autosómica  dominante  con  penetrancia  variable)  a  anestésicos  inhalados  
o  relajantes  musculares.

2.  Mecanismo:  metabolismo  anormal  del  calcio  en  el  músculo  esquelético.

3.  Manifestaciones  clínicas:  temperatura  central  >40  C,  trismo,  hipercapnia,  taquicardia,  taquipnea,  hipertensión,  
arritmias  cardíacas,  acidosis  metabólica,  hipoxemia,  mioglobinuria  o  coagulopatía.

4.  Manejo:  detener  la  anestesia;  administrar  dantroleno  durante  48  horas,  sodio  suplementario
bicarbonato;  enfriar  activamente  al  paciente.

4.  ¿Qué  causa  la  fiebre?

Los  macrófagos  son  activados  por  bacterias  y  endotoxinas.  Los  macrófagos  activados  liberan  interleucina­1,  
factor  de  necrosis  tumoral  (TNF)  e  interferón,  que  restablecen  el  centro  termorregulador  hipotalámico.

64
Machine Translated by Google
CAPÍTULO  10  ¿QUÉ  SIGNIFICA  FIEBRE  POSTOPERATORIA?  sesenta  y  cinco

5.  ¿Se  puede  tratar  la  fiebre?
Sí.  La  aspirina,  el  acetaminofén  y  el  ibuprofeno  son  inhibidores  de  la  ciclooxigenasa  que  bloquean  la  
formación  de  prostaglandina  E2  en  el  hipotálamo  y  controlan  eficazmente  la  fiebre.

6.  ¿Debe  tratarse  la  fiebre?
Esto  es  controvertido.  Ninguna  evidencia  sugiere  que  la  supresión  de  la  fiebre  mejore  el  resultado  del  
paciente.  Sin  embargo,  los  pacientes  se  sienten  más  cómodos  y  el  cirujano  recibe  menos  llamadas  de  las  
enfermeras.

7.  ¿Debe  investigarse  la  fiebre?
Sí.  La  fiebre  indica  que  algo  (frecuentemente  tratable)  está  pasando.  El  umbral  para  la  consulta  depende  del  
paciente.  Un  paciente  trasplantado  con  una  temperatura  de  38  C  requiere  escrutinio,  mientras  que  un  estudiante  
de  medicina  saludable  con  una  temperatura  idéntica  de  38  C  24  horas  después  de  una  apendicectomía  puede  ser  
ignorado.

8.  Resuma  un  examen  de  fiebre.
  Solicitar  hemocultivos,  tinción  de  Gram  y  cultivo  de  orina  y  tinción  de  Gram  y  cultivo  de  esputo.
&  Mire  las  incisiones  quirúrgicas.
  Examine  los  sitios  intravenosos  antiguos  y  actuales  en  busca  de  evidencia  de  tromboflebitis  séptica.
  Si  los  ruidos  respiratorios  son  preocupantes,  obtenga  una  radiografía  de  tórax.

9.  ¿Cuál  es  la  causa  más  común  de  fiebre  durante  el  período  postoperatorio  temprano  (1  a  3  días)?

La  respuesta  tradicional  es  atelectasia.  Sin  embargo,  un  neumotórax  total  no  causa  fiebre.
¿Por  qué  una  pequeña  atelectasia  causa  fiebre,  mientras  que  mucha  atelectasia  (neumotórax)  no?
La  explicación  más  probable  es  que  la  atelectasia  estéril  (y  el  colapso  pulmonar  posoperatorio  temprano  
generalmente  no  está  infectado)  no  tiene  nada  que  ver  con  la  fiebre.

10.  ¿Las  incisiones  quirúrgicas  comprometen  los  patrones  de  respiración  espontánea?
Sí.  La  capacidad  vital  se  midió  en  un  gran  grupo  de  pacientes  24  horas  después  de  varios  procedimientos  
quirúrgicos.  Una  incisión  en  el  abdomen  superior  fue  la  peor,  seguida  de  una  incisión  en  el  abdomen  inferior,  luego  
(contradictoriamente)  una  toracotomía,  una  esternotomía  media  y  una  incisión  en  las  extremidades.

11.  ¿Se  debe  tratar  la  atelectasia  con  espirometría  de  incentivo?
Sí,  pero  no  para  evitar  la  fiebre.

12.  Defina  una  infección  de  herida.
Una  herida  infectada  contiene  >105  organismos  por  gramo  de  tejido.  Una  incisión  infectada  aparece  eritematosa  
(roja),  edematosa  (hinchada)  y  sensible.

13.  ¿Algunas  heridas  son  propensas  a  la  infección?
Cada  mililitro  de  saliva  humana  contiene  108  bacterias  aeróbicas  y  anaeróbicas,  grampositivas  y  
gramnegativas.  Todas  las  heridas  por  mordedura  humana  deben  considerarse  contaminadas.
Las  heridas  por  mordedura  de  animales  suelen  estar  menos  contaminadas.  (Es  más  seguro  besar  a  tu  perro  que  a  tu  
prometido´[e].)

14.  ¿Se  infectan  las  incisiones  pronto  después  de  la  cirugía?
La  incisión  debe  examinarse  en  un  paciente  con  fiebre  (39  C)  <12  horas  después  de  la  cirugía.
Busque  una  secreción  serosa  maloliente  en  una  herida  particularmente  dolorosa  (todas  las  incisiones  duelen)  con  
o  sin  crepitación.  La  tinción  de  Gram  de  la  secreción  serosa  para  bacilos  grampositivos  confirma  o  excluye  el  
diagnóstico  de  infección  por  clostridios.
Machine Translated by Google
66  CAPÍTULO  10  ¿QUÉ  SIGNIFICA  FIEBRE  POSTOPERATORIA?

15.  Resuma  el  tratamiento  de  la  gangrena  gaseosa  por  clostridios.
  La  herida  debe  abrirse  de  inmediato,  con  reanimación  con  fluidos  del  paciente.  El
El  pilar  de  la  terapia  es  el  desbridamiento  quirúrgico  agresivo  del  tejido  necrótico  (piel,  músculo  y  fascia).  
Haz  un  gran  agujero  y  no  te  preocupes  por  cerrarlo.
&  Administre  penicilina,  12  millones  de  U/día  IV  durante  1  semana.
  El  oxígeno  hiperbárico  no  es  útil.

16.  ¿Son  motivo  de  preocupación  las  infecciones  de  heridas  necrosantes  no  clostridiales?
La  gangrena  estreptocócica  hemolítica,  la  gangrena  escrotal  idiopática  y  la  celulitis  necrosante  sinérgica  
gramnegativa  son  entidades  distintas,  pero  se  han  agrupado  en  la  categoría  única  de  fascitis  necrosante.  Todos  
requieren  el  mismo  enfoque  inicial:
1.  Reanimación  con  líquidos  y  electrolitos.
2.  Antibióticos  de  amplio  espectro  (''triples'').
3.  Desbridamiento  quirúrgico  agresivo  de  todo  el  tejido  necrótico.

17.  ¿Qué  son  los  antibióticos  triples?
Un  enfoque  rápido  para  infecciones  potencialmente  mortales  cuando  el  paciente  está  gravemente  enfermo  
y  el  cirujano  está  seriamente  preocupado:
1.  Cobertura  de  grampositivos  (p.  ej.,  ampicilina)
2.  Cobertura  contra  gramnegativos  (p.  ej.,  gentamicina)
3.  Cobertura  anaeróbica  (p.  ej.,  metronidazol  [Flagyl])
Para  evitar  el  crecimiento  excesivo  de  levaduras  y  bacterias  resistentes,  concéntrese  en  la  bacteria  culpable  tan  pronto  como  
los  cultivos  la  definan.

PUNTOS  CLAVE:  CLOSTRIDIAL  VERSUS  NOCLOSTRIDIAL
INFECCIONES  DE  HERIDA  NECROTIZANTE

1.  La  infección  por  clostridios  afecta  al  músculo  subyacente  y  provoca  mionecrosis  o  gangrena  gaseosa.

2.  La  infección  no  clostridial  involucra  la  fascia  subcutánea  (también  conocida  como  fascitis  necrosante).

3.  Manejo  similar:  reanimación  con  líquidos  y  electrolitos,  antibióticos  (penicilina  en  dosis  altas  para  la  infección  
por  clostridios,  triples  de  amplio  espectro  para  la  fascitis  necrosante)  y  desbridamiento  quirúrgico  agresivo  del  
tejido  necrótico.

18.  Dar  las  dosis  de  antibióticos  triples.
Ampicilina  1  g  cada  6  horas  IV  en  adultos  40  mg/
kg  cada  6  horas  IV  en  niños  Gentamicina  7  mg/
kg  IV  cada  24  horas  (esta  dosis  única  diaria  es  menos  nefrotóxica  que  2  mg/kg  IV  cada  8  horas)

Metronidazol  500  mg  IV  cada  6  horas  en  adultos  7,5  mg/kg  
IV  cada  6  horas  en  niños

19.  ¿Qué  procedimientos  quirúrgicos  están  predispuestos  a  infecciones  de  heridas?
Procedimientos  gastrointestinales  (GI),  especialmente  cuando  se  abre  el  colon.

20.  ¿Cuándo  suelen  ocurrir  las  infecciones  de  heridas?
Ocurren  entre  12  horas  y  7  días  después  de  la  operación.

21.  ¿Cómo  se  trata  la  infección  de  una  herida?
La  herida  debe  abrirse  y  drenarse  por  completo.
Machine Translated by Google
CAPÍTULO  10  ¿QUÉ  SIGNIFICA  FIEBRE  POSTOPERATORIA?  67

22.  ¿Es  necesario  irrigar  una  herida  infectada?
El  riego  con  agua  del  grifo  disminuye  la  carga  bacteriana  y  favorece  la  cicatrización.  El  alcohol  es  tóxico  para  los  
tejidos.  El  clorhidrato  de  sodio  (solución  de  Dakin)  y  el  peróxido  de  hidrógeno  matan  los  fibroblastos  y  retardan  la  
epitelización.  Como  regla  general,  no  pongas  nada  en  una  herida  que  no  te  pondrías  en  el  ojo.

23.  ¿Cuándo  ocurren  las  infecciones  del  tracto  urinario?
Cuanto  más  tiempo  esté  colocada  la  sonda  uretral  (Foley),  más  probable  es  que  tenga  una  infección  del  tracto  
urinario  (ITU).  La  instrumentación  urológica  en  el  momento  de  la  cirugía  puede  acelerar  considerablemente  el  proceso.
Los  gérmenes  se  arrastran  por  el  exterior  del  catéter  uretral  y  entre  5  y  7  días  después  de  la  cirugía,  la  mayoría  de  
los  pacientes  albergan  orina  infectada.

24.  ¿Cómo  se  diagnostica  una  infección  del  tracto  urinario?
Una  UTI  tiene  un  urocultivo  con  >105  bacterias/ml.  Los  glóbulos  blancos  en  el  análisis  de  orina  son  muy  
sospechosos.

25.  Mencione  las  causas  tardías  más  frecuentes  de  fiebre  posoperatoria.
La  tromboflebitis  séptica  (de  una  línea  intravenosa)  y  los  abscesos  ocultos  (generalmente  intraabdominales)  
tienden  a  presentarse  alrededor  de  2  semanas  después  de  la  cirugía.

SITIOS  WEB

www.mhacanada.org

www.anes.ucla.edu/dept/mh.html

BIBLIOGRAFÍA

1.  Bansal  BC,  Wiebe  RA,  Perkins  SD  et  al.:  Agua  del  grifo  para  irrigación  de  laceraciones.  Am  J  Emerg  Med  20:469­472,
2002.

2.  da  Luz  Moreira  A,  Vogel  JD,  Kalady  MF  et  al.:  Evaluaciones  de  fiebre  después  de  cirugía  colorrectal:  identificación  de  riesgo
factores  que  aumentan  el  rendimiento  y  disminuyen  el  costo.  Dis  Colon  Recto  51:  1202­1207,  2008.

3.  Helmer  KS,  Robinson  EK,  Lally  KP  et  al.:  Las  pautas  estandarizadas  de  atención  al  paciente  reducen  la  morbilidad  infecciosa  en
pacientes  de  apendicectomía.  Am  J  Surg  183:608­613,  2002.

4.  Lewis  RT:  profilaxis  antibiótica  oral  versus  sistémica  en  cirugía  de  colon  electiva:  un  estudio  aleatorizado  y  un  metanálisis  envían  un  
mensaje  desde  la  década  de  1990.  Can  J  Surg  45:173­180,  2002.

5.  Singer  AJ,  Quinn  JV,  Thode  HC  Jr  et  al .:  Determinantes  de  un  resultado  deficiente  después  de  la  laceración  y  la  reparación  de  la  
incisión  quirúrgica.  Plast  Reconstr  Surg  110:429­435,  2002.
Machine Translated by Google

INFECCIÓN  DE  HERIDA  QUIRÚRGICA
CAPÍTULO  
11

Steven  L.  Peterson,  DVM,  MD

1.  ¿Por  qué  debemos  preocuparnos  por  la  infección  de  la  herida  quirúrgica?
Aproximadamente  30  millones  de  pacientes  se  someten  a  cirugía  cada  año  en  los  Estados  Unidos  y  el  20%  de  estos  
pacientes  adquieren  al  menos  una  infección  nosocomial  en  el  período  posoperatorio.  Las  infecciones  en  los  sitios  quirúrgicos  
son  la  tercera  forma  más  común  de  estas  infecciones  y  complican  del  1%  al  12%  de  todas  las  operaciones.  El  riesgo  de  muerte  
es  4  veces  mayor  en  pacientes  que  desarrollan  infecciones  en  las  heridas,  y  el  tratamiento  de  cada  infección  cuesta  entre  
$12,000  y  $30,000.
Las  tasas  reportadas  comúnmente  para  operaciones  específicas  son:  
Colecistectomía  3%  Herniorrafia  inguinal  2Toracotomía  
%  Apendicectomía  
6%  Colectomía  
5%   12%

2.  ¿Qué  comprende  una  infección  de  herida  quirúrgica?
Las  infecciones  de  la  herida  quirúrgica  se  denominan  más  apropiadamente  infecciones  del  sitio  quirúrgico  (ISQ)  y  deben  
ocurrir  dentro  de  los  30  días  posteriores  a  la  cirugía,  a  menos  que  se  deje  un  cuerpo  extraño  in  situ.  En  el  caso  de  material  
extraño  implantado,  debe  transcurrir  1  año  antes  de  que  se  pueda  descartar  la  cirugía  como  causal.
Las  SSI  se  subdividen  según  la  profundidad  de  la  afectación  del  tejido  en  tres  categorías  clínicamente  relevantes.

1.  Las  ISQ  incisionales  superficiales  involucran  solo  la  piel  y  el  tejido  subcutáneo.
2.  Las  ISQ  incisionales  profundas  involucran  capas  profundas  de  tejido  blando,  como  las  capas  fasciales  o  musculares  de  la
incisión.

3.  Las  ISQ  de  espacio  de  órganos  implican  cualquier  estructura  anatómica  abierta  o  manipulada  durante  la
procedimiento  operativo.

3.  Enumere  los  signos  clásicos  de  las  infecciones  del  sitio  quirúrgico  de  la  incisión  superficial,  de  la  incisión  profunda  y  del  espacio  de  
órganos.
ISQ  incisionales  superficiales  y  profundas:  &  
Calor  (calor)
&  rubor  (enrojecimiento)
&  Tumor  (hinchazón)
&  Dolor  (dolor)
  Drenaje  purulento  En  
presencia  de  signos  y  síntomas  sistémicos,  debe  sospecharse  una  ISQ  en  el  espacio  de  los  órganos:
y  fiebre
e  íleo
y  choque

El  diagnóstico  definitivo  de  las  ISQ  del  espacio  de  órganos  puede  requerir  estudios  de  imagen.

4.  ¿Por  qué  ocurren  estas  infecciones?
Muchos  factores  contribuyen,  sin  embargo,  el  principio  fundamental  es  que  las  heridas  traumáticas  y  
quirúrgicas  violan  la  capa  protectora  normal  de  la  piel.  La  importancia  de  un  intacto

68
Machine Translated by Google
CAPÍTULO  11  INFECCIÓN  DE  LA  HERIDA  QUIRÚRGICA  69

tegumento  se  ha  demostrado  experimentalmente  en  el  que  se  determinó  que  se  requiere  un  inóculo  de  8  
millones  de  bacterias  para  la  infección  de  la  piel  intacta,  se  requiere  1  millón  para  la  piel  lesionada  y  solo  se  
requieren  100  cuando  hay  material  extraño  presente.

5.  La  cirugía  siempre  viola  la  piel  y  muchas  veces  dejamos  material  extraño.  como  puedo
evitamos  las  SSI?
Si  bien  es  cierto  que  el  acto  básico  de  la  cirugía  compromete  las  defensas  de  los  pacientes,  podemos  tomar  
medidas  para  prevenir  la  infección  de  la  herida.  Estos  pasos  involucran  al  cirujano  y  al  paciente.

6.  ¿Qué  puede  hacer  el  cirujano  para  disminuir  las  ISQ?
El  primer  paso  que  puede  tomar  el  cirujano  es  lavarse  las  manos  adecuadamente.  El  lavado  quirúrgico  clásico  
consiste  en  3  minutos  de  cepillado  con  povidona  yodada  o  gluconato  de  clorhexidina.  Se  ha  demostrado  que  
este  protocolo  tiene  una  alta  tasa  de  incumplimiento,  lo  que  puede  contribuir  a  las  SSI.
Los  datos  indican  un  mejor  cumplimiento  con  tasas  comparables  de  SSI  utilizando  un  protocolo  mucho  
más  simple  que  consiste  en  un  lavado  de  manos  de  1  minuto  con  jabón  no  antiséptico  seguido  de  frotamiento  de  
manos  con  una  solución  alcohólica  acuosa  líquida.  Se  desconoce  si  estos  protocolos  de  lavado  más  simples  
también  se  pueden  aplicar  en  el  futuro  a  la  preparación  del  paciente.

7.  ¿Qué  más  puede  hacer  el  cirujano  para  controlar  las  ISQ?
El  cirujano  puede  limitar  la  duración  de  la  cirugía  y  seguir  buenos  principios  quirúrgicos  eliminando  el  espacio  
muerto,  controlando  la  hemorragia,  minimizando  la  colocación  de  material  extraño  (incluida  la  sutura  excesiva)  y  
exhibiendo  un  manejo  suave  del  tejido.  El  cirujano  debe  asegurarse  de  que  el  paciente  permanezca  caliente  
durante  el  período  perioperatorio.  Este  simple  acto  de  calentamiento  se  demostró  en  dos  estudios  prospectivos  
para  disminuir  significativamente  la  incidencia  de  ISQ.

8.  ¿No  puede  el  cirujano  predecir  quién  se  va  a  infectar  y  simplemente  darle  muchos  antibióticos  para  evitar  que  ocurra  
la  infección?
Hasta  cierto  punto,  las  SSI  se  pueden  anticipar.  Los  factores  que  han  demostrado  tener  cierto  valor  predictivo  
para  el  cirujano  son  el  estado  físico  del  paciente  según  la  clasificación  de  la  Sociedad  Estadounidense  de  
Anestesiólogos,  los  resultados  de  los  cultivos  intraoperatorios  y  la  duración  de  la  estancia  hospitalaria  
preoperatoria.  La  suficiencia  del  suministro  de  sangre  regional  también  es  importante,  como  lo  demuestra  la  baja  
tasa  de  infección  en  las  heridas  faciales.  La  descripción  clásica  de  heridas  basada  en  el  grado  de  contaminación  
macroscópica  también  puede  ser  valiosa.  Este  esquema  clasifica  las  heridas  en  una  de  cuatro  categorías:  1.  Las  
heridas  limpias  son  heridas  atraumáticas  en  las  que  no  se  encuentra  inflamación,  no  se  producen  interrupciones  
en  la  técnica  estéril  y  no  se  penetra  en  una  víscera  hueca.
2.  Las  heridas  limpias­contaminadas  son  idénticas  excepto  que  se  introduce  una  víscera  hueca.
3.  Las  heridas  contaminadas  son  causadas  por  un  traumatismo  de  una  fuente  limpia  o  por  un  derrame  menor  de
materiales  infectados.
4.  Las  heridas  sucias  e  infectadas  son  causadas  por  un  traumatismo  de  una  fuente  contaminada  o
derrame  de  material  infectado  en  una  incisión.
Las  tasas  de  infección  informadas  para  cada  categoría  son  2,1  %,  3,3  %,  6,4  %  y  7,1  %,  respectivamente.
Los  antibióticos  pueden  ayudar,  pero  solo  cuando  se  usan  apropiadamente.

9.  ¿Cómo  uso  correctamente  los  antibióticos  para  prevenir  las  ISQ?
Primero,  sabiendo  a  qué  organismo  se  dirige,  luego  eligiendo  un  antibiótico  apropiado  y  administrándolo  en  el  
momento  apropiado  a  través  de  la  ruta  apropiada.  Debido  a  que  generalmente  no  tendrá  un  cultivo  preoperatorio  
para  guiar  la  terapia,  debe  basar  su  elección  de  antibiótico  en  los  organismos  previstos.  Los  estafilococos  son  el  
organismo  cutáneo  más  común  y  el  agente  etiológico  más  común  en  las  ISQ.  La  cefazolina,  una  cefalosporina  de  
primera  generación,  suele  ser  el  antibiótico  recomendado  para  la  profilaxis  en  procedimientos  quirúrgicos  limpios.  
En  circunstancias  en  las  que  se  sabe  que  se  ha  producido  una  contaminación,  los  antibióticos  iniciales  deben  
adaptarse  en  función  de  la  flora  común  del  órgano  lesionado.  Si  se  ingresó  al  intestino,  son  comunes  las  
enterobacterias  y  los  anaerobios;  las  incisiones  del  tracto  biliar  y  del  esófago  producen  estos  organismos  más  
enterococos.  Las  vías  urinarias  o  la  vagina  pueden  contener  estreptococos  del  grupo  D,  especies  de  Pseudomonas  
y  Proteus.
Machine Translated by Google
70  CAPÍTULO  11  INFECCIÓN  DE  LA  HERIDA  QUIRÚRGICA

10.  Si  se  utilizan  antibióticos,  ¿cómo  y  cuándo  se  deben  administrar?
El  beneficio  máximo  se  obtiene  cuando  las  concentraciones  tisulares  son  terapéuticas  en  el  momento  de  la  
contaminación.  La  eficacia  aumenta  cuando  se  administran  antibióticos  profilácticos  por  vía  intravenosa  (IV)  menos  
de  1  hora  antes  de  la  incisión  quirúrgica;  la  administración  tardía  es  similar  a  la  ausencia  de  administración.  Los  regímenes  de  
dosis  múltiples  no  tienen  ningún  beneficio  comprobado  sobre  los  regímenes  de  dosis  única.
La  selección  indiscriminada  de  antibióticos  fuera  de  los  protocolos  hospitalarios  recomendados  puede  aumentar  la  incidencia  
de  ISQ.  En  circunstancias  especiales,  pueden  estar  indicadas  vías  de  administración  distintas  de  la  intravenosa.

PUNTOS  CLAVE:  CLASIFICACIÓN  DE  HERIDAS  Y
TASA  DE  INFECCIÓN

1.  La  herida  limpia  es  atraumática,  sin  interrupciones  en  la  técnica  estéril,  sin  entrada  en  las  vías  respiratorias,
tracto  alimentario  o  genitourinario.  La  incidencia  es  del  2,1%.

2.  La  herida  limpia­contaminada  es  lo  mismo  que  la  herida  limpia  excepto  por  la  entrada  en  el  tracto  respiratorio,  
alimentario  o  genitourinario.  La  incidencia  es  del  3,3%.

3.  La  herida  contaminada  tiene  un  trauma  de  una  fuente  limpia  o  un  derrame  menor  de  sangre  infectada.
materiales  La  incidencia  es  del  6,4%.

4.  La  herida  sucia  es  un  traumatismo  de  una  fuente  contaminada  o  derrame  de  materiales  infectados.
La  incidencia  es  del  7,1%.

11.  Mencione  otras  vías  que  usaría  para  la  administración  profiláctica  de  antibióticos.
En  pacientes  portadores  nasales  de  Staphylococcus  aureus,  la  administración  intranasal  de  pomada  de  mupirocina  puede  
tener  cierta  eficacia  para  disminuir  las  ISQ  nosocomiales  y  nosocomiales.  En  la  cirugía  de  colon  electiva,  un  metanálisis  de  
estudios  publicados  indicó  que  los  antibióticos  administrados  por  vía  oral  combinados  con  antibióticos  intravenosos  son  
superiores  a  los  antibióticos  intravenosos  solos  en  la  prevención  de  infecciones  en  el  sitio  quirúrgico.

12.  ¿Realmente  sirve  de  algo  todo  ese  lavado  pulsátil  que  utiliza  el  cirujano  en  el  quirófano?

Sí.  El  lavado  pulsátil  a  alta  presión  se  ha  evaluado  ampliamente  en  la  contaminación  de  tejidos  blandos  y  ha  demostrado  ser  7  
veces  más  efectivo  para  reducir  la  carga  bacteriana  que  el  lavado  con  jeringa  de  pera.
El  retroceso  elástico  inherente  de  los  tejidos  blandos  permite  que  las  partículas  escapen  entre  pulsos  de  fluido.  La  
presión  y  la  frecuencia  de  pulso  óptimas  parecen  ser  de  50  a  70  lb/in.2  y  800  pulsos/min.  Agregar  antibióticos  a  las  
soluciones  de  lavado,  aunque  se  practica  comúnmente,  no  ha  demostrado  definitivamente  que  mejore  el  resultado.

13.  ¿Qué  puede  hacer  el  paciente  para  ayudar  a  disminuir  las  ISQ?
Deja  de  fumar.  Aunque  la  obesidad,  el  estado  nutricional  deficiente,  la  edad  avanzada  y  la  diabetes  son  factores  de  riesgo  
para  las  ISQ,  fumar  cigarrillos  es  probablemente  el  principal  factor  prevenible  de  las  ISQ  en  el  paciente,  al  igual  que  es  la  
principal  causa  prevenible  de  muerte  y  discapacidad  en  los  Estados  Unidos.  La  mitad  de  todas  las  personas  que  fuman  
finalmente  mueren  a  causa  de  una  enfermedad  relacionada  con  el  tabaquismo.  Fumar  no  solo  mata,  sino  que  también  triplica  
con  creces  el  riesgo  de  ruptura  de  la  herida  por  incisión.  En  un  estudio,  fumar  aumentó  seis  veces  la  incidencia  de  ISQ  en  
procedimientos  quirúrgicos  limpios,  del  0,6  %  al  3,6  %.
El  consumo  de  tabaco  da  como  resultado  una  disminución  del  flujo  sanguíneo  y  una  disminución  del  suministro  de  oxígeno  a  la  herida.

Los  subproductos  tóxicos  del  tabaco  también  impiden  directamente  todas  las  etapas  de  la  cicatrización  de  heridas.  A  
pesar  de  este  conocimiento,  los  cirujanos  continúan  operando  de  manera  electiva  a  los  fumadores,  y  la  mayoría  de  los  
fumadores  continúan  fumando  hasta  el  día  de  la  cirugía.
Machine Translated by Google
CAPÍTULO  11  INFECCIÓN  DE  LA  HERIDA  QUIRÚRGICA  71

14.  Cuando  falla  la  prevención,  ¿qué  hace  usted  por  las  SSI?
La  primera  línea  de  tratamiento  en  las  ISQ  es  el  drenaje.  Esto  se  establece  mediante  la  reapertura  de  la  herida  o,  
en  el  caso  de  infecciones  del  espacio  profundo,  utilizando  técnicas  guiadas  por  tomografía  computarizada  (TC)  o  
ultrasonido  para  la  colocación  de  drenajes  o  la  planificación  prequirúrgica.  La  terapia  con  antibióticos  se  usa  para  
controlar  la  celulitis  asociada  y  la  sepsis  generalizada.

15.  ¿Qué  puede  suceder  con  las  ISQ  incisionales  superficiales  o  profundas  no  tratadas?
Localmente,  la  herida  se  rompe  y  la  infección  diseca  los  planos  tisulares  y  continúa  avanzando.  Si  la  infección  
progresa  rápidamente,  se  puede  desarrollar  fascitis  necrosante.  Finalmente,  las  capas  de  resistencia  del  cierre  de  
la  herida  se  abren  (dehiscencia).

16.  Definir  dehiscencia  de  herida.
La  ruptura  parcial  o  total  de  cualquiera  o  todas  las  capas  de  la  herida  operatoria.

17.  Defina  evisceración.
Rotura  de  la  pared  abdominal  y  extrusión  de  las  vísceras  abdominales.

18.  ¿Qué  factores  predisponen  a  la  dehiscencia?
La  edad  >60  años,  la  obesidad,  el  aumento  de  la  presión  intraabdominal,  la  desnutrición,  la  
insuficiencia  renal  o  hepática,  la  diabetes  mellitus  (DM),  el  uso  de  corticosteroides  o  fármacos  citotóxicos  y  
la  radiación  han  sido  implicados  en  la  dehiscencia  de  la  herida.  La  infección  también  juega  un  papel  
importante;  se  identifica  un  agente  infeccioso  en  más  de  la  mitad  de  las  heridas  que  sufren  dehiscencia.  A  pesar  
de  estas  excusas,  el  factor  más  importante  en  la  dehiscencia  de  la  herida  es  la  idoneidad  del  cierre.  Los  bordes  
fasciales  no  deben  desvitalizarse.  Idealmente,  las  suturas  de  la  línea  alba  no  deben  colocarse  ni  demasiado  lateral  
ni  demasiado  medialmente.  La  colocación  lateral  excesiva  puede  incorporar  el  riego  sanguíneo  variable  del  
músculo  recto  del  abdomen  y  comprometer  la  circulación  fascial.  La  colocación  medial  excesiva  pierde  el  punto  
de  fuerza  máxima  en  la  zona  de  transición  entre  la  línea  alba  y  la  vaina  del  recto  abdominal.  Además,  las  suturas  
deben  anudarse  correctamente  sin  tensión  excesiva  y  debe  elegirse  un  material  de  sutura  con  una  resistencia  a  
la  tracción  adecuada.

19.  ¿Cuándo  ocurre  la  dehiscencia  de  la  herida?
Puede  ocurrir  en  cualquier  momento  después  de  la  cirugía;  sin  embargo,  es  más  común  entre  el  quinto  y  décimo  
día  postoperatorio,  cuando  la  fuerza  de  la  herida  es  mínima.

20.  ¿Cuáles  son  los  signos  y  síntomas  de  la  dehiscencia  de  la  herida?
Normalmente,  una  cresta  de  engrosamiento  palpable  (cresta  de  cicatrización)  se  extiende  alrededor  de  0,5  cm  
a  cada  lado  de  la  incisión  en  1  semana.  La  ausencia  de  esta  cresta  puede  ser  un  fuerte  predictor  de  ruptura  
inminente  de  la  herida.  Más  comúnmente,  la  fuga  de  líquido  serosanguinolento  de  la  herida  es  el  primer  signo.  
En  algunos  casos,  la  evisceración  repentina  puede  ser  el  primer  indicio  de  dehiscencia  de  la  herida  abdominal.  
El  paciente  también  puede  describir  una  sensación  de  lagrimeo  o  estallido  asociado  con  tos  o  arcadas.

21.  Describir  el  tratamiento  adecuado  de  la  dehiscencia  de  la  herida.
Si  la  dehiscencia  no  está  asociada  con  infección,  el  cierre  electivo  puede  ser  el  curso  terapéutico  
apropiado.  Sin  embargo,  si  el  estado  del  paciente  o  la  herida  hacen  que  el  cierre  sea  inaceptable,  se  debe  permitir  
que  la  herida  cicatrice  por  segunda  intención.  Una  cicatriz  inestable  o  una  hernia  incisional  pueden  tratarse  en  un  
momento  posterior  más  seguro.  La  dehiscencia  de  una  herida  de  laparotomía  con  evisceración  es  una  emergencia  
quirúrgica  con  una  mortalidad  reportada  del  10%  al  20%.  El  tratamiento  inicial  en  este  caso  consiste  en  una  
reanimación  adecuada  mientras  se  protegen  los  órganos  eviscerados  con  toallas  húmedas;  el  siguiente  paso  es  
el  cierre  quirúrgico  rápido.  El  intestino  o  el  epiplón  expuestos  deben  lavarse  minuciosamente  y  devolverse  al  
abdomen;  la  pared  abdominal  debe  estar  cerrada;  y  la  herida  de  la  piel  debe  estar  abierta.  El  cierre  de  heridas  
asistido  por  vacío  puede  ser  valioso  en  casos  seleccionados.
Machine Translated by Google
72  CAPÍTULO  11  INFECCIÓN  DE  LA  HERIDA  QUIRÚRGICA

SITIO  WEB

www.acssurgery.com/abstracts/acs/acs0102.htm

BIBLIOGRAFÍA

1.  Barie  PS:  profilaxis  y  terapia  antibiótica  quirúrgica  moderna:  menos  es  más.  Surg  Infect  1:23­29,  2000.

2.  Garner  GB,  Ware  DN,  Cocanour  CS  et  al.:  El  cierre  de  heridas  asistido  por  vacío  proporciona  una  reaproximación  
fascial  temprana  en  pacientes  traumatizados  con  abdomen  abierto.  Am  J  Surg  182:630­638,  2001.

3.  Harbarth  S,  Fankhauser  C,  Schrenzel  J  et  al .:  Detección  universal  de  Staphylococcus  aureus  resistente  a  la  meticilina  al  
ingreso  hospitalario  e  infección  nosocomial  en  pacientes  quirúrgicos.  JAMA  299:1149­1157,  2008.

4.  Kluytmans  J,  Voss  A:  Prevención  de  infecciones  posquirúrgicas:  a  algunos  les  gusta  el  calor.  Curr  Opin  Infect  Dis  15:427­432,
2002.

5.  Krueger  JK,  Rohrich  RJ:  Despejar  el  humo:  la  justificación  científica  para  la  abstención  del  tabaco  con  cirugía  plástica.
Plast  Reconstr  Surg  108:1063­1073,  2001.

6.  Morange­Saussier  V,  Giraudeau  B,  van  der  Mee  N  et  al .:  Transporte  nasal  de  Staphylococcus  resistente  a  la  meticilina
aureus  en  cirugía  vascular.  Ann  Vasc  Surg  20:767­772,  2006.

7.  Myles  PS,  Iacono  GA,  Hunt  JO  et  al.:  Riesgo  de  complicaciones  respiratorias  e  infección  de  heridas  en  pacientes  sometidos  
a  un  servicio  ambulatorio.  Anestesiología  97:842­847,  2002.

8.  Parienti  JJ,  Thibon  P,  Heller  R  et  al .:  Frotado  de  manos  con  una  solución  alcohólica  acuosa  versus  lavado  de  manos  quirúrgico  
tradicional  y  tasas  de  infección  del  sitio  quirúrgico  de  30  días:  un  estudio  de  equivalencia  aleatorio.  JAMA  288:722­727,  2002.

9.  Perl  TM,  Cullen  JJ,  Wenzel  RP  et  al.:  mupirocina  intranasal  para  prevenir  infecciones  postoperatorias  por  Staphylococcus  
aureus.  N  Engl  J  Med  346:1871­1877,  2002.

10.  Seltzer  J,  McGraw  K,  Horsman  A  et  al .:  Conciencia  de  las  infecciones  del  sitio  quirúrgico  para  enfermeras  de  práctica  avanzada.
ACCN  Clin  Iss  13:  398­409,  2002.
Machine Translated by Google

PRIORIDADES  EN  LA  EVALUACIÓN  DE  LA
ABDOMEN  AGUDO
CAPÍTULO  
12

Dr.  Alden  H.  Harken

1.  ¿Cuál  es  la  responsabilidad  del  cirujano  ante  un  paciente  con  abdomen  agudo?

1.  Identificar  qué  tan  enfermo  está  el  paciente  (tratar  primero  al  paciente  y  luego  a  la  enfermedad).
2.  Para  determinar  si  el  paciente  (a)  necesita  ir  directamente  a  la  sala  de  operaciones,  (b)  debe  ser  admitido  para  
reanimación  u  observación,  o  (c)  puede  ser  enviado  de  manera  segura  a  casa.

2.  ¿Cuál  es  el  curso  más  peligroso  en  un  paciente  con  abdomen  agudo?
Para  enviar  al  paciente  a  casa.

3.  ¿Es  importante  realizar  el  diagnóstico  en  urgencias?
No.  Con  frecuencia,  el  tiempo  dedicado  a  confirmar  un  diagnóstico  en  el  departamento  de  emergencias  (ED)  se  
pierde  debido  a  la  reanimación  en  el  hospital  o  al  tratamiento  en  la  sala  de  operaciones.  El  único  paciente  que  
necesita  un  diagnóstico  relativamente  firme  es  el  que  va  a  ser  enviado  a  casa.

4.  Si  el  objetivo  esencial  no  es  realizar  el  diagnóstico,  ¿qué  debe  hacer  el  cirujano?

1.  Reanimar  al  paciente.  La  mayoría  de  los  pacientes  no  comen  ni  beben  cuando  se  enferman.
La  mayoría  de  los  pacientes  pierden  al  menos  varios  litros  de  líquido.  La  depleción  de  líquidos  es  peor  en  
pacientes  con  diarrea  o  vómitos.
2.  Inicie  una  vía  intravenosa  (IV)  grande.
3.  Reemplace  los  electrolitos  perdidos  (vea  el  Capítulo  8).
4.  Inserte  un  catéter  de  Foley.
5.  Examinar  al  paciente  (frecuentemente).

5.  ¿Son  los  síntomas  y  signos  únicamente  engañosos  en  algún  grupo  de  pacientes?
Sí.  Tenga  cuidado  con  los  siguientes  grupos:  &  
Los  muy  jóvenes,  que  no  pueden  hablar.
  Diabéticos  por  neuropatía  visceral.
  Los  muy  ancianos,  en  quienes,  como  en  los  diabéticos,  la  inervación  abdominal  está  embotada.
&  Pacientes  que  toman  esteroides,  que  deprimen  la  inflamación  y  enmascaran  todo.
  Pacientes  con  inmunosupresión  (un  paciente  trasplantado  de  corazón  o  riñón  puede  actuar  alegre  incluso  
con  el  intestino  muerto  o  gangrenado).

6.  Resuma  la  historia  necesaria.
1.  La  edad  del  paciente.  Neonatos  presentes  con  intususcepción;  mujeres  jóvenes  presentes  con
embarazo  ectópico,  enfermedad  pélvica  inflamatoria  y  apendicitis;  los  ancianos  presentan  cáncer  de  colon,  
diverticulitis  y  apendicitis.
2.  Problemas  asociados.  ¿Hospitalizaciones  previas,  cirugía  abdominal  previa,  medicamentos,  enfermedad  
cardíaca  y  pulmonar?  Una  historia  ginecológica  extensa  es  valiosa;  sin  embargo,  probablemente  sea  más  
seguro  asumir  que  todas  las  mujeres  entre  12  y  40  años  están  embarazadas.

73
Machine Translated by Google
74  CAPÍTULO  12  PRIORIDADES  EN  LA  EVALUACIÓN  DEL  ABDOMEN  AGUDO

3.  Localización  del  dolor  abdominal.  Cuadrante  superior  derecho:  vesícula  biliar  o  enfermedad  biliar,
úlcera  duodenal.  Flanco  derecho:  pielonefritis,  hepatitis.  Midepigastrium:  úlcera  duodenal  o  gástrica,  
pancreatitis,  gastritis.  Cuadrante  superior  izquierdo:  bazo  roto,  absceso  subdiafragmático.  Cuadrante  
inferior  derecho:  apendicitis  (capítulo  37),  embarazo  ectópico,  hernia  incarcerada,  hematoma  del  recto.  
Cuadrante  inferior  izquierdo:  diverticulitis,  hernia  incarcerada,  hematoma  del  recto.  Nota:  el  cáncer,  a  
menos  que  obstruya  (cáncer  de  colon),  y  el  sangrado  (diverticulosis)  generalmente  no  duelen.

4.  Duración  del  dolor.  El  dolor  de  una  úlcera  duodenal  perforada  o  sigmoide  perforado
el  divertículo  es  repentino,  mientras  que  el  dolor  de  la  pielonefritis  es  gradual  y  persistente.  El  dolor  de  
la  obstrucción  intestinal  es  intermitente  y  cólico.  Nota:  Aunque  el  cirujano  rota  a  través  de  un  servicio  
gastrointestinal  (GI),  es  posible  que  la  paciente  no  sepa  esto  y  presente  una  patología  urológica,  
ginecológica  o  vascular.

EXAMEN  FÍSICO

7.  ¿Son  importantes  los  signos  vitales?
Sí.  Son  vitales.  Si  la  frecuencia  cardíaca  (FC)  y  la  presión  arterial  (PA)  están  en  el  lado  equivocado  de  100  
(frecuencia  cardíaca  >100  latidos/min,  presión  arterial  sistólica  <100  mm  Hg),  ¡cuidado!  La  taquipnea  
(frecuencia  respiratoria  >16)  refleja  dolor  o  acidosis  sistémica.  La  fiebre  puede  desarrollarse  tardíamente,  
particularmente  en  el  paciente  inmunodeprimido  que  puede  estar  afebril  frente  a  una  peritonitis  florida.

8.  ¿Qué  es  el  rebote?
El  peritoneo  está  bien  inervado  y  es  exquisitamente  sensible.  No  es  necesario  lastimar  al  paciente  para  provocar  
signos  peritoneales.  Presione  suavemente  el  abdomen  y  suelte.  Si  el  paciente  se  estremece,  el  peritoneo  está  
inflamado  (dolor  de  rebote).

9.  ¿Qué  es  el  dolor  medio?
Mittelschmerz  es  dolor  en  medio  del  ciclo  menstrual.  La  ovulación  se  asocia  frecuentemente  con  sangrado  
intraperitoneal.  La  sangre  irrita  el  sensible  peritoneo  y  duele.

10.  ¿Qué  significan  los  ruidos  intestinales?
Si  algo  duele  (p.  ej.,  un  tobillo  torcido),  el  paciente  tiende  a  no  usarlo.  El  intestino  inflamado  está  tranquilo.  
El  contenido  intestinal  exprimido  a  través  de  una  obstrucción  parcial  produce  un  tintineo  agudo.
Sin  embargo,  los  sonidos  intestinales  son  notoriamente  poco  fiables.

11.  Explique  la  importancia  de  la  distensión  abdominal.
La  distensión  puede  derivar  de  gas  o  líquido  intraentérico  o  extraentérico  (el  peor  de  todos,  sangre).
La  distensión  abdominal  siempre  es  significativa  y  mala.

12.  ¿Es  importante  la  palpación  abdominal?
Sí.  Recuerde,  el  paciente  es  (o  debería  ser)  el  amigo  del  cirujano.  No  hay  necesidad  de  causar  dolor.  La  
palpación  guía  al  cirujano  hacia  la  zona  anatómica  de  mayor  sensibilidad  (por  lo  general,  el  área  enferma).  Lo  
mejor  es  comenzar  la  palpación  en  un  área  que  no  duela.  Los  exámenes  rectales  (prueba  de  sangre  en  las  
heces)  y  pélvicos  localizan  aún  más  la  patología.

13.  ¿Qué  es  el  signo  de  Kehr?
El  diafragma  y  la  parte  posterior  del  hombro  izquierdo  disfrutan  de  inervación  paralela.  El  dolor  simultáneo  en  
el  cuadrante  superior  izquierdo  y  en  el  hombro  izquierdo  indica  irritación  diafragmática  por  ruptura  del  bazo  o  
absceso  subdiafragmático.

14.  ¿Qué  es  un  signo  del  psoas?
La  irritación  del  músculo  psoas  retroperitoneal  por  un  apéndice  retrocecal  inflamado  causa  dolor  con  la  flexión  
de  la  cadera  derecha  o  la  extensión  del  muslo.
Machine Translated by Google
CAPÍTULO  12  PRIORIDADES  EN  LA  EVALUACIÓN  DEL  ABDOMEN  AGUDO  75

ESTUDIOS  DE  LABORATORIO

15.  ¿Cómo  es  útil  un  hemograma  completo?
1.  Hematocrito.  Si  el  hematocrito  es  alto  (>45%),  lo  más  probable  es  que  el  paciente  esté  seco  o  que  tenga  
enfermedad  pulmonar  obstructiva  crónica  (EPOC).  Si  es  bajo  (<30%),  el  paciente  probablemente  tenga  una  
enfermedad  más  crónica  (asociada  con  la  pérdida  de  sangre;  siempre  haga  un  recto  y  analice  las  heces  para  
detectar  sangre).
2.  Recuento  de  glóbulos  blancos.  La  inflamación  tarda  horas  en  liberar  citocinas  y  elevar  el  recuento  de  glóbulos  
blancos.  Un  recuento  normal  de  glóbulos  blancos  es  totalmente  consistente  con  un  problema  abdominal  significativo.

16.  ¿Es  necesario  el  análisis  de  orina?
Sí.  Los  glóbulos  blancos  en  la  orina  pueden  redirigir  la  atención  al  diagnóstico  de  pielonefritis  o  cistitis.  La  hematuria  apunta  
a  cálculos  renales  o  ureterales.  Debido  a  que  un  apéndice  inflamado  puede  estar  directamente  sobre  el  uréter  derecho,  se  
pueden  encontrar  glóbulos  rojos  y  blancos  en  la  orina  de  pacientes  con  apendicitis.

17.  ¿Qué  es  un  ''trío  del  abdomen''?
1.  Radiografía  de  tórax  en  posición  vertical:  busque  aire  libre  debajo  del  diafragma  (víscera  perforada)  y
neumonía  o  neumotórax.
2.  Abdomen  erguido:  busque  aire  libre  debajo  del  diafragma  y  niveles  hidroaéreos  (obstrucción  intestinal).  Recuerde  
buscar  sigmoides  o  aire  rectal  (obstrucción  parcial).
3.  Abdomen  en  decúbito  supino:  esta  radiografía  no  dice  nada.
La  mayoría  de  los  cálculos  ureterales  se  pueden  visualizar.  Sólo  el  10%  de  los  cálculos  biliares  son  radiopacos  y
rara  vez  se  observan  fecalitos  apendiculares.
Honores:  aire  en  el  sistema  biliar  indica  una  fístula  bilioentérica;  esto,  en  asociación  con  los  niveles  hidroaéreos  
intestinales,  hace  el  diagnóstico  de  íleo  biliar.

PUNTOS  CLAVE:  EVALUACIÓN  RADIOGRÁFICA
PARA  EL  ABDOMEN  AGUDO
1.  Puede  ayudar  en  la  evaluación  diagnóstica  pero  no  debe  suplantar  el  examen  físico  en
Evaluación  de  un  abdomen  agudo.

2.  Tres  vías  del  abdomen:  busque  aire  libre  debajo  del  diafragma,  patología  intratorácica,
niveles  hidroaéreos,  canal  alimentario  dilatado  y  aire  distal  en  el  recto.

3.  Ultrasonido:  útil  para  evaluaciones  biliares,  obstétricas  y  ginecológicas  y  vasculares;  puede  notar  intraperitoneal
o  colecciones  de  líquido  retroperitoneal.

4.  TC:  creciente  uso  en  el  ámbito  clínico,  con  excelente  visualización  de  las  estructuras  abdominales.
Inconvenientes:  costo,  exposición  a  la  radiación.

18.  ¿Qué  es  un  bucle  centinela?
Excepto  en  los  niños  (que  tragan  todo,  incluido  el  aire),  los  gases  del  intestino  delgado  siempre  son  patológicos.  
Un  solo  asa  de  gas  del  intestino  delgado  adyacente  a  un  órgano  inflamado  (p.  ej.,  el  páncreas)  puede  apuntar  al  órgano  
enfermo.

19.  ¿Es  valioso  el  ultrasonido?

Sí,  si  el  diagnóstico  de  trabajo  es  colecistitis,  cálculos  biliares,  embarazo  ectópico,  quiste  ovárico,  aneurisma  aórtico  
abdominal  o  líquido  intraperitoneal/retroperitoneal.
Machine Translated by Google
76  CAPÍTULO  12  PRIORIDADES  EN  LA  EVALUACIÓN  DEL  ABDOMEN  AGUDO

20.  ¿Es  valiosa  la  tomografía  computarizada  abdominal?
Sí,  si  el  diagnóstico  de  trabajo  es  un  absceso  intraabdominal  (diverticulitis  sigmoidea),  pancreatitis,  sangrado  
retroperitoneal  (fuga  de  aneurisma  de  aorta  abdominal;  este  paciente  debería  haber  ido  directamente  a  la  sala  
de  operaciones),  o  patología  intrahepática  o  esplénica.

21.  ¿Qué  es  una  tomografía  computarizada  de  doble  contraste?
El  intestino  se  delinea  con  bario  o  Gastrografin.  Los  vasos  sanguíneos  se  delinean  con  un  tinte  vascular  
yodado.  La  tomografía  computarizada  muestra  con  precisión  el  contenido  abdominal  en  relación  con  los  puntos  
de  referencia  vasculares  e  intestinales.  La  TC  con  contraste  de  pancreatitis  es  valiosa  para  evaluar  las  zonas  
de  perfusión  o  necrosis.

TRATAMIENTO  QUIRÚRGICO

22.  Si  el  paciente  está  enfermo  (y  no  mejora),  ¿qué  se  debe  hacer?
Después  de  la  reanimación  con  líquidos,  se  debe  explorar  el  abdomen  del  paciente.  Se  ha  promocionado  una  
laparotomía  exploradora  como  la  conclusión  lógica  de  un  examen  físico  completo.

23.  ¿Es  dañina  una  laparotomía  negativa?
Sí,  pero  los  pacientes  pueden  sobrevivir  incómodamente  a  una  laparotomía  negativa,  mientras  que  un  infarto  
intestinal  no  detectado  (o  apendicitis)  puede  poner  en  peligro  la  vida.

24.  Nombre  el  problema  más  desafiante  en  toda  la  medicina.
Un  abdomen  agudo.

SITIO  WEB

www.acssurgery.com/abstracts/acs/acs0301.htm

BIBLIOGRAFÍA

1.  D'Agostino  J:  Emergencias  abdominales  comunes  en  niños.  Emerg  Med  Clin  N  Am  20:139­153,  2002.

2.  Dhillon  S,  Halligan  S,  Goh  V  et  al.:  El  impacto  terapéutico  de  la  ecografía  abdominal  en  pacientes  con  síntomas  abdominales  
agudos.  Clin  Radiol  57:268­271,  2002.

3.  Forster  MJ,  Akoh  JA:  Apendicitis  perforada  disfrazada  de  pancreatitis  aguda  en  un  paciente  con  obesidad  mórbida.
World  J  Gastroenterol  14:1795­1796,  2008.

4.  Gajic  O,  Urrutia  LE,  Sewani  H  et  al.:  Abdomen  agudo  en  la  unidad  de  cuidados  intensivos  médicos.  Crit  Care  Med
30:1187­1190,  2002.

5.  Rozycki  GS,  Tremblay  L,  Feliciano  DV  et  al.:  Trescientas  celiotomías  emergentes  consecutivas  en  general
pacientes  de  cirugía:  influencia  de  técnicas  y  procedimientos  avanzados  de  diagnóstico  por  imágenes  en  el  diagnóstico,  Ann  
Surg  235:681­689,  2002.
Machine Translated by Google

ENFERMEDAD  INFECCIOSA  QUIRÚRGICA
CAPÍTULO  
13

Glenn  W.  Geelhoed,  MD,  MPH,  MA,  DTMH,  ScD  (Hon),  MA,  MPhil,  EdD,  FACS

1.  ¿Han  controlado  los  avances  en  antibióticos  modernos  muchos,  si  no  la  mayoría,  de  los  problemas  de  infección  
quirúrgica?
No.  En  pacientes  quirúrgicos  gravemente  enfermos  en  entornos  de  unidades  de  cuidados  intensivos  (UCI),  
los  problemas  de  sepsis  han  aumentado  y  siguen  estando  entre  las  principales  causas  de  muerte  en  pacientes  
de  la  UCI,  especialmente  aquellos  con  insuficiencia  orgánica  múltiple  (MOF)  y  deficiencias  en  las  defensas  del  
huésped.  El  tratamiento  con  antibióticos  puede  cambiar  el  perfil  biográfico  de  la  flora  asociada  con  la  muerte  de  los  
pacientes,  pero  no  puede  superar  las  múltiples  causas  de  la  falla  de  la  resistencia  del  huésped  a  la  infección  que  
acompañan  a  las  brechas  de  la  barrera  a  la  invasión  microbiana  y  las  respuestas  inflamatorias  e  inmunológicas  a  los  
"sospechosos  habituales".

2.  ¿Qué  tipo  de  barrera  de  nalgas  permite  la  invasión  microbiana  que  puede  establecerse?
infección  del  sitio  quirúrgico?
La  piel  y  los  revestimientos  mucosos  del  cuerpo  mantienen  una  barrera  entre  el  mundo  exterior  multifloral  y  el  
medio  interior  estéril  de  los  tejidos  y  órganos  (incluso  cuando  el  mundo  exterior  es  un  tubo  de  flora  densamente  
poblada  a  través  del  medio  de  las  cavidades  corporales  generalmente  estériles,  como  el  tracto  gastrointestinal  [GI]).  
Es  fácil  ver  la  brecha  de  la  barrera  cuando  un  cuchillo  penetra  la  piel,  transportando  flora  exterior  debajo  de  la  piel,  
o  cuando  ese  cuchillo  perfora  y  derrama  el  contenido  contaminado  del  intestino  en  el  abdomen.  Es  menos  obvio  
cuando  la  presentación  de  nalgas  es  causada  por  un  estado  de  flujo  bajo  o  cuando  la  nutrición  inadecuada  o  las  
toxinas  alteran  las  inmunoglobulinas  de  la  mucosa,  lo  que  hace  que  la  "barrera  del  cuerpo  del  insecto"  sea  permeable.  
Estas  comunidades  polimicrobianas  de  organismos  pueden  comenzar  a  invadir  a  través  de  la  brecha  en  tales  
barreras,  particularmente  si  hay  más  fallas  en  la  tercera  línea  de  defensa  en  la  resistencia  humoral  y  celular.

3.  ¿Cuál  es  la  diferencia  entre  contaminación  e  infección?
¡La  presencia  de  microorganismos  no  hace  una  infección!
Las  comunidades  de  flora  residentes  en  las  superficies  del  cuerpo  hacen  poco  daño,  y  la  flora  intestinal  
es  incluso  beneficiosa  cuando  está  contenida  en  el  intestino.  Incluso  es  posible  que  las  bacterias  estén  presentes  
de  forma  transitoria  fuera  de  sus  residencias  comensales  habituales  sin  constituir  una  infección  en  el  huésped  
normalmente  intacto.  Por  ejemplo,  al  cepillarse  los  dientes  enérgicamente,  las  bacterias  gramnegativas  de  varios  
tipos  que  residen  en  la  cavidad  oral  se  introducen  en  el  torrente  sanguíneo,  pero  probablemente  se  eliminen  
rápidamente  mediante  los  mecanismos  de  defensa  normales,  a  menos  que  encuentren  una  resistencia  reducida  del  
huésped  o  sembraron  una  válvula  cardíaca  protésica. .

4.  ¿Cómo  puede  la  enorme  carga  de  bacterias  en  el  tracto  gastrointestinal  inferior
ser  beneficioso?
Los  insectos  pueden  ser  hermosos.  Estas  son  las  mismas  bacterias  que  han  vivido  simbióticamente  con  y  
en  humanos  durante  milenios.  Sintetizan  vitamina  K,  algo  de  lo  que  literalmente  no  podemos  prescindir,  o  
desplazan  a  los  organismos  patógenos  en  cantidades  abrumadoras.  También  ayudan  a  metabolizar  las  sales  biliares  
y  desempeñan  un  papel  en  la  desintoxicación  de  algunos  peligros  ambientales,  de  forma  similar  a  los  sistemas  
sépticos.

77
Machine Translated by Google
78  CAPÍTULO  13  ENFERMEDADES  INFECCIOSAS  QUIRÚRGICAS

5.  Siempre  que  se  encuentre  un  derrame  intestinal  intraabdominal,  ¿es  obligatorio  realizar  un  cultivo  de  la  contaminación  
fecal  y  obtener  la  sensibilidad  de  todos  los  organismos  identificados?

No.  Hay  una  diferencia  entre  contaminación  e  infección.  Por  lo  tanto,  los  cultivos  de  derrame  fecal  en  el  peritoneo  no  
proporcionarán  información  útil.  El  contaminante,  solo  por  su  cambio  de  posición  con  respecto  a  la  pared  intestinal,  no  es  
probable  que  sea  estéril.  ¿Cuándo  le  gustaría  que  se  cerrara  el  laboratorio?  ¿Se  contentará  con  escuchar  un  informe  de  
Escherichia  coli  y  bacteroides,  dos  de  las  más  de  800  especies  que  incluso  el  laboratorio  más  compulsivo  difícilmente  
puede  ser  competente  para  identificar,  dada  la  exposición  al  aire  y  el  tiempo  transcurrido  hasta  el  procesamiento  en  
diferentes  medios?  ¿Cómo  cambiará  su  terapia  la  información  de  un  error  de  muestreo  de  contaminantes  mixtos  adquiridos  
en  la  comunidad?  Si,  por  ejemplo,  no  se  identifican  anaerobios  en  la  muestra  fecal,  ¿estará  tan  seguro  de  que  no  están  
presentes  como  para  excluir  estas  especies  de  la  cobertura?

La  lección  que  se  debe  aprender  es  que  la  cultura  de  los  contaminantes  adquiridos  en  la  comunidad  es  costosa,
incompleto  y  poco  edificante;  el  cultivo  de  microbios  invasores  en  infecciones,  en  particular  microbios  adquiridos  
en  hospitales  que  persisten  después  del  tratamiento,  puede  brindar  información  crítica  y  es  un  uso  más  apropiado  de  los  
recursos  microbiológicos.

6.  ¿Qué  son  las  preparaciones  (p.  ej.,  preparaciones  intestinales)?
Las  preparaciones  son  procedimientos  de  descontaminación,  diseñados  para  reducir  la  flora  residente  antes  de  un  
procedimiento  invasivo  electivo.  Las  preparaciones  pueden  tomar  la  forma  de  un  proceso  simple,  como  una  torunda  
con  alcohol  que  se  unta  sobre  la  piel  antes  de  un  pinchazo  rápido  de  la  inyección  subcutánea,  o  puede  implicar  la  
preparación  de  un  área  más  grande  de  la  superficie  de  la  piel  para  el  campo  quirúrgico  de  la  incisión  (consulte  la  
pregunta  7).
Una  preparación  intestinal  está  diseñada  de  manera  similar  para  reducir  la  flora  residente  en  el  intestino  
a  través  de  (1)  catarsis  mecánica  (es  decir,  purga);  (2)  dilución  osmótica  o  de  volumen  con  grandes  volúmenes  de  
solución  salina,  otras  soluciones  electrolíticas  o  manitol;  o  (3)  administración  oral  de  antibióticos  no  absorbidos.  De  
estos  métodos,  el  más  importante  es  claramente  la  catarsis  mecánica  porque  purga  grandes  cantidades  de  flora,  que  
pueden  representar  hasta  dos  tercios  del  peso  seco  del  contenido  del  colon.  Una  de  las  razones  más  convincentes  para  
la  elección  de  ciertos  antibióticos  orales  en  las  preparaciones  intestinales  (ver  pregunta  9)  es  su  vigorosa  acción  catártica.

7.  ¿Cómo  se  esteriliza  la  piel  o  las  cavidades  mucosas  de  un  paciente  para  preparar  un
campo  para  la  incisión  quirúrgica?
Hay  una  manera,  apenas  recomendada,  por  la  cual  los  pacientes  pueden  ser  "esterilizados";  al  igual  que  los  
instrumentos  y  las  cortinas,  se  pueden  colocar  en  un  autoclave.  Pero  aparte  de  este  ejemplo  absurdo,  la  piel  
nunca  es  estéril.  Los  procesos  de  descontaminación  nunca  son  perfectos,  especialmente  en  un  tejido  tan  complejo  con  
grietas  y  estructuras  cutáneas  accesorias  en  las  que  residen  las  bacterias.  Descansar  las  manos  enguantadas  sobre  
un  "campo  estéril"  no  incluye  la  piel  ni  las  superficies  mucosas.
En  el  mejor  de  los  casos,  simplemente  reducimos  la  flora  al  inóculo  de  bajo  nivel  que  puede  ser  manejado  
por  la  mayoría  de  los  sistemas  de  defensa  del  huésped  intactos,  como  en  el  ejemplo  de  cepillarse  los  dientes,  pero  las  
superficies  de  los  tejidos  vivos  nunca  son  estériles.  Un  método  que  elimine  todos  los  organismos  microbianos  de  dichas  
superficies  también  desvitalizaría  las  células  de  los  mamíferos  y  las  volvería  más  susceptibles  a  los  inóculos  microbianos  
de  nivel  inferior.

8.  ¿Qué  medios  se  pueden  utilizar  para  reducir  la  flora  residente  en  la  superficie  sin  más
lesionar  la  piel  o  las  mucosas?
&  Volumen  de  lavado  (solo  para  valor  mnemotécnico:  la  dilución  es  la  solución  a  la  contaminación)
&  Desengrasante,  que  solubiliza  los  aceites  sebáceos  que  pueden  atrapar  la  flora
&  Matanza  microbicida  con  un  agente  bacteriostático
Machine Translated by Google
CAPÍTULO  13  ENFERMEDAD  INFECCIOSA  QUIRÚRGICA  79

En  gran  medida,  un  fluido  sencillo  y  barato  que  puede  servir  como  diluyente,  disolvente  de  grasas,
y  el  antimicrobiano  es  el  alcohol.  El  alcohol  es  casi  ideal  como  solución  de  preparación,  con  las  
desventajas  menores  de  que  deshidrata  y  es  mínimamente  inflamable.  Debido  a  que  se  vaporiza  y  desaparece,  
la  flora  puede  propagarse  desde  los  intersticios,  fuera  del  campo,  o  incluso  a  través  de  la  lluvia  radiactiva  en  
aerosol  sobre  el  campo,  lo  que  requiere  la  adición  de  bacteriostasis  de  duración  prolongada  a  la  preparación  
con  alcohol.
El  yodo  también  mata  las  bacterias,  pero  presenta  un  peligro  mayor  para  las  células  sensibles  de  los  mamíferos  
(oxida  las  paredes  celulares  de  las  plantas  pequeñas).  Se  puede  lograr  una  concentración  inicial  más  baja  de  yodo  
y  una  acción  de  mayor  duración  mediante  la  incorporación  de  un  yodóforo,  una  sustancia  de  uso  casi  universal  en  
las  preparaciones.  La  aplicación  de  "paños  de  incisión"  permeables  a  la  humedad  y  al  vapor  o  "paños  de  anillo"  
que  previenen  la  desecación  puede  retrasar  aún  más  la  repoblación  de  la  flora  sobre  el  campo  preparado  (pero  
aún  no  estéril).

9.  ¿Qué  son  los  antibióticos  "limpiapipas"?
Los  limpiapipas  son  regímenes  de  antibióticos  administrados  por  vía  oral  que  reducen  la  flora  en  el  tracto  GI,  del  
cual  no  se  absorben  bien.  Son  un  componente  casi  ideal  de  las  preparaciones  intestinales  porque  son  agentes  
catárticos  potentes  y  logran  la  gran  mayoría  de  su  "limpieza  de  tuberías"  mediante  una  acción  purgante  mecánica.  
Los  limpiapipas  más  populares  incluyen  una  base  de  neomicina  o  eritromicina.

10.  ¿Qué  es  la  descontaminación  intestinal  selectiva?  ¿Como  funciona?
No  funciona.  Este  método  utilizó  limpiadores  de  pipa  en  pacientes  con  alto  riesgo  de  desarrollar  sepsis  por  MOF  
con  el  objetivo  teórico  de  reducir  el  riesgo  involucrado  en  la  ruptura  de  la  barrera  del  tracto  GI  y  la  inoculación  con  
flora  intestinal.  Buena  evidencia  experimental  indicó  que  este  método  debería  reducir  la  alta  tasa  de  mortalidad  en  
pacientes  gravemente  enfermos  con  alto  riesgo  de  sepsis  quirúrgica.  Sin  embargo,  después  de  prolongados  
ensayos  clínicos,  no  logró  demostrar  un  beneficio  en  la  supervivencia  del  paciente.  La  razón  probable  es  que,  
mientras  que  los  estudios  de  laboratorio  se  realizaron  en  modelos  animales  intactos  con  sistemas  de  defensa  del  
huésped  en  funcionamiento,  las  fallas  de  defensa  más  allá  de  la  recámara  de  la  barrera  pueden  explicar  por  qué  la  
descontaminación  intestinal  selectiva  no  benefició  a  los  pacientes  gravemente  enfermos.
Además,  la  flora  del  hospital  residente  repobló  el  intestino  purgado  con  el  tiempo,  pero  con  formas  virulentas  de  
microbios  seleccionados  por  su  resistencia  a  los  antibióticos  de  amplio  espectro.  El  método  todavía  tiene  algún  uso  
en  pacientes  sometidos  a  procedimientos  como  quimioterapia  de  dosis  alta  o  trasplante  de  médula  ósea  y  en  
algunos  pacientes  aislados  en  "islas  de  vida" (p.  ej.,  pacientes  con  enfermedades  de  inmunodeficiencia  o  
quemaduras).

ANTIBIÓTICOS

11.  ¿Son  los  antibióticos  los  fármacos  milagrosos  clásicos?
Solo  porque  te  preguntas  si  van  a  funcionar,  si  van  a  causar  más  daño  que  bien  y  si  la  próxima  generación  
será  inasequible  o  tóxica.
El  escepticismo  es  saludable  con  respecto  a  cualquier  procedimiento  o  agente  en  el  cuidado  de  la  salud,  
pero  especialmente  con  respecto  a  los  antibióticos,  que  se  aceptan  casi  universalmente  como  agentes  que  
previenen  y  curan  infecciones.  Se  pasan  por  alto  la  primacía  de  la  defensa  del  huésped  en  este  proceso  vital  y  la  
posible  interferencia  de  los  mismos  fármacos  a  los  que  se  atribuye  el  control  de  infecciones.  Debemos  mirar  
críticamente  el  papel  limitado  que  deben  jugar  los  antibióticos  en  el  cuidado  de  la  salud  y  frenar  su  uso  excesivo,  
que  genera  aún  más  daño  que  gasto  innecesario.

12.  ¿Qué  se  entiende  por  generaciones  de  antibióticos,  como  en  las  cefalosporinas  de  tercera  
generación?
Los  primeros  antibióticos  eran  bacteriostáticos,  en  gran  parte  a  través  de  la  interferencia  en  la  síntesis  de  
proteínas,  por  lo  que  podían  evitar  que  un  microorganismo  se  reprodujera  incluso  si  no  lo  mataban.  La  diferencia  
entre  infestación  (presencia  de  microbios  vivos  en  el  huésped)  e  infección
Machine Translated by Google
80  CAPÍTULO  13  ENFERMEDADES  INFECCIOSAS  QUIRÚRGICAS

(replicación  y  propagación  de  microorganismos  en  el  huésped)  puede  ser  útil  para  comprender  cómo  los  fármacos  
anteriores  posiblemente  controlaban  la  infección  pero  eran  menos  capaces  de  eliminar  los  organismos  en  cualquier  
período  breve  de  terapia.
La  penicilina  cambió  todo  eso.  Puede  ser  el  primer  antibiótico  con  un  reclamo  legítimo  al  título  de  
"medicamento  maravilloso"  porque  tiene  la  capacidad  microbicida  de  erradicar  organismos  sensibles.
La  penicilina  fue  la  primera  generación  de  antibióticos  betalactámicos,  junto  con  las  cefalosporinas  
congéneres  de  primera  generación  (p.  ej.,  cefazolina).  Compartían  la  estructura  de  betalactámicos  y  tenían  una  
buena  cobertura  de  grampositivos  con  menor  rango  en  cualquier  efecto  sobre  los  microbios  gramnegativos.
Los  antibióticos  betalactámicos  de  segunda  generación  (p.  ej.,  cefoxitina)  cubrieron  nuevas  clases  de  
microbios  más  allá  de  los  aerobios  grampositivos,  como  muchas  de  las  especies  de  Bacteroides,  pero  tuvieron  
poco  efecto  sobre  los  microbios  aerobios  gramnegativos.  Debido  a  que  las  cefalosporinas  de  tercera  generación  
cubrían  algunos  de  los  últimos  microbios,  se  promocionaron  como  una  terapia  de  agente  único  para  todas  las  
floras  de  riesgo  principal.
Al  igual  que  con  la  penicilina,  el  fármaco  maravilloso  original,  el  asombro  se  desvaneció  con  los  fracasos  
de  los  nuevos  agentes  debido  a  la  resistencia  antimicrobiana  inducida  rápidamente.  La  diferencia  más  fácil  de  
medir  y  calcular  en  las  generaciones  es  el  costo:  los  valores  mayoristas  son  de  aproximadamente  $2,00/g  para  
la  primera  generación,  $5,00/g  para  la  segunda  y  $30,00/g  para  la  tercera.  A  pesar  de  este  aumento  gradual  en  el  
costo,  las  generaciones  superiores  pierden  parte  de  su  potencia  contra  los  organismos  grampositivos  originales  para  
los  cuales  los  agentes  de  primera  generación  fueron  realmente  maravillosos.  Por  lo  tanto,  se  necesitan  2  g  de  
moxalactama  para  ser  la  mitad  de  buenos  que  1  g  de  cefazolina  para  la  cobertura  de  grampositivos.  No  es  necesario  
que  un  farmacoeconomista  pregunte:  "¿Qué  he  obtenido  a  cambio  de  este  recargo  de  sesenta  veces?"

13.  ¿Cuál  es  el  papel  de  las  cefalosporinas  de  tercera  generación  en  la  profilaxis  quirúrgica?
Ninguno  (¡no  más  preguntas  aquí!).  Si  la  flora  de  riesgo  principal  es  grampositiva,  es  mejor  la  primera  
generación;  si  el  riesgo  anaeróbico  es  considerable,  la  segunda  generación  es  mejor.  Y  cualquiera  de  las  clases  es  
mucho  más  barata  y  parece  haber  generado  menos  resistencia  que  las  cefalosporinas  de  tercera  generación,  que  
son  desmesuradamente  caras  para  su  uso  en  la  profilaxis  y  rara  vez  tan  efectivas  como  otras  terapias  de  agente  
único  para  la  infección  del  sitio  quirúrgico  establecida  (SSI).  Las  indicaciones  específicas,  como  la  meningitis  
pediátrica,  la  neumonía  adquirida  en  el  hospital  u  otras  infecciones  específicas  fuera  de  las  indicaciones  de  predominio  
quirúrgico,  pueden  usar  o  excluir  estos  agentes.

14.  ¿Cómo  mejoran  los  inhibidores  enzimáticos  combinados  con  antibióticos  su  espectro  antimicrobiano?

Los  microorganismos  tienen  sus  propios  mecanismos  de  defensa,  y  las  cepas  que  tienen  la  capacidad  de  producir  
enzimas  que  degradan  los  antibióticos  logran  una  ventaja  de  selección  antinatural  con  el  uso  generalizado  de  
antibióticos.  Esto  es  lo  que  le  pasó  a  la  penicilina:  surgieron  las  penicilinasas.
Pero  los  fabricantes  farmacéuticos  astutos  cerraron  esa  escapatoria  para  el  ingenio  bacteriano  en  la  
degradación  de  la  penicilina  mediante  la  colocación  estratégica  de  un  grupo  metilo  para  arruinar  la  capacidad  de  
supervivencia  de  los  productores  de  penicilinasa.  El  resultado  fue  la  meticilina,  pero  la  persistencia  de  los  microbios  
significa  que  ahora  tenemos  una  plaga  de  Staphylococcus  aureus  resistente  a  la  meticilina  (MRSA).  Además,  los  
microbios  superan  en  número  a  los  fabricantes  de  productos  farmacéuticos  y  tienen  un  tiempo  de  respuesta  más  
corto  que  el  proceso  de  aprobación  de  la  Administración  de  Alimentos  y  Medicamentos  (FDA).  Los  microbios  siempre  
estarán  por  delante  de  nosotros  en  ingenio,  aunque  solo  sea  por  su  número.
Las  estrategias  más  nuevas  de  la  bacteria  incluían  la  producción  de  betalactamasas.  La  respuesta
de  la  industria  farmacéutica  fue  un  grupo  de  inhibidores  de  beta­lactamasa,  como  el  ácido  clavulánico  o  
sulbactam.  La  combinación  de  un  inhibidor  de  betalactamasa  con  una  penicilina  modificada  como  la  ampicilina  
debería  tener  una  mayor  actividad  contra  las  bacterias  que  producen  betalactamasa,  siempre  que  fueran  sensibles  
a  la  ampicilina  en  primer  lugar.  Las  dosis  más  altas  del  agente  original  durante  un  tiempo  más  corto  pueden  lograr  el  
mismo  efecto,  a  menudo  a  un  costo  menor,  porque  los  medicamentos  combinados  se  desarrollaron  mucho  más  
recientemente  y  están  protegidos  por  patentes.
Machine Translated by Google
CAPÍTULO  13  ENFERMEDAD  INFECCIOSA  QUIRÚRGICA  81

15.  ¿Cuáles  son  los  tipos  de  terapia  con  antibióticos  más  costosos?
&  Medicamentos  que  se  administran  cuando  no  se  necesitan.
&  Medicamentos  que  se  necesitan  con  urgencia  pero  que  no  funcionan.

&  Medicamentos  que  causan  más  daño  que  bien  debido  a  la  toxicidad  del  huésped,  cualquiera  que  sea  
su  potencial  antibiótico.

16.  ¿Se  pueden  administrar  antibióticos  orales  en  lugar  de  antibióticos  intravenosos  en  pacientes  quirúrgicos  gravemente  
enfermos?
¡Sí,  si  pudieran  tomarlos!  Estos  pacientes  casi  invariablemente  no  pueden  tomar  nada  por  la  boca  (NPO),  a  menudo  
están  inconscientes  y  es  probable  que  no  estén  conectados  a  un  ventilador.  Además,  el  intestino  ha  quedado  fuera  de  
servicio  debido  a  las  sondas  de  succión  nasogástrica  (NG),  la  laparotomía  y  el  íleo,  y  los  problemas  intraabdominales  
primarios  a  menudo  asociados  con  la  necesidad  de  antibióticos,  como  la  sepsis  intraabdominal  y  la  pancreatitis.  Por  
lo  general,  estos  pacientes  están  en  reposo  intestinal  completo  y  es  probable  que  también  reciban  nutrición  parenteral.

El  intento  de  utilizar  algún  tipo  de  antibiótico  administrado  por  vía  intestinal  se  basa  en  la  farmacocinética  
y  el  espectro  favorables  de  las  quinolonas,  que  se  pueden  iniciar  por  vía  intravenosa  y  cambiar  tan  pronto  
como  sea  posible  a  la  forma  oral  cuando  se  haya  reanudado  la  alimentación.  Casi  todos  estos  pacientes  
comienzan  con  algún  tipo  de  programa  de  antibióticos  intravenosos  (IV)  y  el  inicio  del  régimen  de  antibióticos  es  
más  importante  que  la  forma  en  que  se  reducen  los  pacientes  antes  de  interrumpir  el  tratamiento.

PROFILAXIS

17.  ¿Se  debe  usar  profilaxis  antibiótica  sistémica  en  colon  electivo?
¿resección?
Sí,  más  allá  de  cualquier  sombra  estadística  de  duda.  Se  han  realizado  al  menos  dos  docenas  de  ensayos  
clínicos  con  controles  de  placebo  contra  una  variedad  de  antibióticos,  principalmente  aquellos  activos  contra  al  
menos  la  flora  predominantemente  anaeróbica,  y  casi  todos  han  mostrado  una  reducción  de  las  complicaciones  
infecciosas  en  el  grupo  de  antibióticos.  Nunca  más  debe  repetirse  este  punto,  y  ningún  paciente  debe  correr  
riesgo  cuando  la  profilaxis  con  antibióticos  sistémicos  se  ha  establecido  como  el  estándar  de  atención.  No  se  
pueden  realizar  éticamente  nuevos  ensayos  clínicos  contra  placebo  en  este  grupo  de  pacientes  con  riesgo  
conocido  dada  la  reducción  de  riesgo  confirmada.

Otros  grupos  de  riesgo  (p.  ej.,  cesárea  después  de  la  ruptura  de  membranas)  además  de  los  
pacientes  sometidos  a  resección  de  colon  han  sido  estandarizados  por  ensayos  en  grandes  poblaciones  
de  pacientes  y  han  mostrado  una  reducción  de  riesgo  similar.  Se  ha  demostrado  el  beneficio  de  la  profilaxis.  
En  otros  grupos  de  pacientes  que  no  pueden  estandarizarse  debido  a  factores  de  contaminación  inusuales  o  
factores  únicos  de  deterioro  de  la  resistencia  del  huésped,  las  pautas  para  la  profilaxis  racional  deben  seguir  principios  
similares.

18.  ¿Dos  dosis  profilácticas  son  mejores  que  una  para  prevenir  la  infección?
¿Son  tres  dosis  mejor  aún?

Sólo  una  dosis  (la  dosis  en  la  circulación  sistémica  en  el  momento  del  inóculo)  de  antibiótico  profiláctico  puede  
demostrar,  más  allá  de  toda  duda  clínica  o  estadística,  que  es  eficaz.  La  necesidad  de  repetir  la  dosis  una  o  más  
veces  durante  las  24  horas  posteriores  al  inóculo  depende  de  los  niveles  sanguíneos  del  fármaco,  que  dependen  
en  gran  medida  de  la  unión  a  proteínas  y  la  tasa  de  eliminación.  También  sabemos  con  certeza  que  10  días  del  
mismo  fármaco  profiláctico  que  es  eficaz  si  se  administra  inmediatamente  antes  del  inóculo  da  como  resultado  un  
mayor  riesgo  de  infección  que  ningún  antibiótico.
Machine Translated by Google
82  CAPÍTULO  13  ENFERMEDADES  INFECCIOSAS  QUIRÚRGICAS

PUNTOS  CLAVE:  ANTIBIÓTICO  PREOPERATORIO
PROFILAXIS
1.  El  momento  de  la  administración  es  el  factor  más  importante.

2.  Dosifique  30  minutos  antes  de  la  incisión  para  que  el  antibiótico  circule  antes  que  el  inóculo.

3.  Ninguna  evidencia  respalda  la  continuación  de  la  profilaxis  más  allá  de  las  24  horas.

19.  ¿Qué  factores  determinan  el  momento  de  la  administración  de  antibióticos  bajo  el
criterios  de  profilaxis?
El  elemento  más  importante  en  el  momento  de  la  profilaxis  es  que  el  fármaco  esté  circulando  antes  que  el  
inóculo.  ¿Cuándo  debería  parar?  Cuando  la  reducción  del  riesgo  de  infección  ya  no  es  comprobable  y  antes  de  
que  el  uso  continuado  anule  el  propósito  profiláctico  (como  se  explicó  anteriormente).
Para  resumir  con  una  regla  empírica  arbitraria:  no  hay  justificación  para  el  antibiótico  profiláctico  24  horas  después  
del  inóculo  de  un  procedimiento  invasivo.
¿Qué  implica  esta  regla?  ¿No  deberíamos  continuar  la  profilaxis  durante  semanas  para  cubrir  la  presencia  de  
una  prótesis  de  cadera?  Presumiblemente,  la  prótesis  de  cadera  estará  en  el  paciente  durante  muchos  años;  ¡pero  
seguramente  usted  no  discute  que  el  antibiótico  debe  continuar  diariamente  mientras  la  cadera  esté  en  su  lugar!  
Lo  que  se  ''profilaxia''  no  es  la  prótesis  de  cadera  sino  el  procedimiento  de  implantación.  Y  no  es  sólo  la  implantación  
lo  que  supone  un  riesgo  para  el  paciente  con  prótesis,  también  lo  es  la  hemorroidectomía  practicada  años  después,  
cuya  profilaxis  se  hace  obligatoria  por  la  presencia  de  la  prótesis  de  cadera.
La  válvula  cardíaca  protésica  o  reumática  es  un  riesgo,  pero  la  indicación  para  el  uso  de  profilácticos
los  antibióticos  son  un  procedimiento  invasivo;  un  conducto  radicular  es  un  ejemplo  en  el  que  un  inóculo  
es  inevitable.  Las  operaciones  están  cubiertas  por  antibióticos  profilácticos;  las  condiciones  que  son  factores  
de  riesgo  durante  la  operación  no  lo  son.

20.  Para  estar  seguro,  ¿por  qué  no  administrar  antibióticos  profilácticos  a  todos  los  pacientes?
someterse  a  algún  tipo  de  operación?
¿Me  puede  dar  la  indicación  de  un  antibiótico  profiláctico  en  un  paciente  sometido  a  un  procedimiento  
quirúrgico  limpio  electivo  que  no  implanta  prótesis,  como  la  reparación  de  una  hernia?
"Claro",  respondió  una  vez  uno  de  mis  estudiantes  más  brillantes,  "el  paciente  que  tiene  un
alteración  de  la  respuesta  del  huésped,  como  la  leucemia  granulocítica  aguda  en  crisis  blástica.''
Respondí:  "¿Por  qué  diablos  estás  arreglando  su  hernia?  Ese  es  un  error  limpio  (esperemos  que  no  sea  una  
muerte  limpia)  en  el  juicio  quirúrgico  que  no  tiene  nada  que  ver  con  los  antibióticos  en  absoluto.  Un  paciente  con  
ese  grado  de  deterioro  del  huésped  no  se  somete  a  un  procedimiento  quirúrgico  electivo”.
Regla  general:  si  puede  proporcionar  la  indicación  de  un  antibiótico  profiláctico  para  cubrir  un
operación  no  protésica  electiva  para  un  paciente,  ha  proporcionado  la  contraindicación  para  la  operación.

MANEJO  DE  INFECCIONES  DEL  SITIO  QUIRÚRGICO

21.  ¿Cuál  es  el  fármaco  de  elección  para  el  tratamiento  de  un  absceso?
Un  cuchillo.  Drenar  quirúrgicamente  el  absceso.  Los  abscesos  no  tienen  circulación  de  sangre  dentro  de  ellos  
para  administrar  un  antibiótico.  El  antibiótico,  incluso  si  se  inyecta  directamente  en  el  absceso,  sería  inútil  porque  
el  absceso  contiene  una  sopa  de  microorganismos  muertos  y  glóbulos  blancos  (GB).
Incluso  si  los  organismos  apenas  estuvieran  vivos,  no  se  reproducirían  ni  incorporarían  el  antibiótico.  Lo  más  
probable  es  que  el  fármaco  no  funcione  en  absoluto  en  las  condiciones  de  equilibrio  ácido­base  (pH)  y  pKa  del  
entorno  del  absceso.
Machine Translated by Google
CAPÍTULO  13  ENFERMEDAD  INFECCIOSA  QUIRÚRGICA  83

Si  hay  indicación  de  antibiótico  sería  en  la  circulación  alrededor  del  borde  inflamatorio  comprimido  del  absceso  
y  la  celulitis  (en  la  ''piel  de  naranja''  vascularizada)  y  planos  tisulares  no  contaminados  por  donde  se  debe  hacer  
el  drenaje  necesario.  llevado  a  cabo.
Una  infección  focal  se  maneja  con  un  tratamiento  local,  que  es  necesario  en  todos  los  abscesos  y  tratamiento  
suficiente  en  muchos.  En  ocasiones  se  indican  antibióticos  sistémicos  adyuvantes  para  la  protección  de  los  
tejidos  a  través  de  los  cuales  se  lleva  a  cabo  el  drenaje.  Si  ayuda  a  aclarar  este  principio  quirúrgico  
fundamental,  aquí  está  la  regla  general  para  el  manejo  de  los  abscesos:  donde  hay  pus,  que  haya  acero.  Quizás  
uno  de  los  procedimientos  más  gratificantes  de  toda  la  medicina  es  el  drenaje  de  pus  con  alivio  inmediato  de  
los  síntomas  locales  y  sistémicos  (p.  ej.,  un  absceso  perirrectal).

22.  ¿Qué  tratamiento  de  abscesos  es  el  más  importante  para  determinar  el  resultado  de  un  paciente  con  sepsis  
intraabdominal?
Lo  que  cuenta  es  el  drenaje  del  último  absceso.  Debe  haber  pocos  aplausos  para  el  drenaje  de  un  absceso  
pélvico  en  el  paciente  que  retiene  un  absceso  subfrénico.  El  paciente  responde  dramáticamente  cuando  se  
drena  el  último  pus.
Esta  ha  sido  un  área  de  avance  significativo  en  el  manejo  de  infecciones  quirúrgicas  porque
la  capacidad  de  exploración  no  invasiva  ha  facilitado  el  hallazgo  de  múltiples  bolsas  de  pus.
Además,  modalidades  como  la  tomografía  computarizada  (TC)  no  solo  encuentran  sino  que  también  dirigen  
percutáneamente  la  reparación  del  último  absceso.  Lo  que  podría  haber  sido  una  indicación  para  un  viaje  
exploratorio  de  regreso  al  quirófano  solo  una  década  antes  (es  decir,  un  paciente  que  falla  con  la  terapia  adecuada  
debería  desencadenar  la  primera  respuesta,  "¿Dónde  está  el  pus?")  ahora  es  una  buena  indicación  para  un  
Tomografía  computarizada  para  encontrar  y  drenar  la  infección  focal.

23.  ¿Qué  se  prefiere  para  drenar  un  absceso  intraabdominal,  una  aguja  o  un  bisturí?
¿Qué  se  puede  hacer  más  rápidamente?  El  paciente  con  sepsis  intraabdominal  está  bastante  enfermo,  y  el  
drenaje  seguro  más  temprano  es  el  procedimiento  de  elección.  Puede  haber  ventajas  en  la  tomografía  
computarizada  menos  invasiva,  que  puede  repetirse  y  tiene  menos  morbilidad  si  los  resultados  son  negativos.
La  cirugía,  por  otro  lado,  puede  corregir  las  condiciones  asociadas  que  pueden  haber  causado  el  absceso,  
como  el  asa  de  intestino  desvitalizado  o  la  fuga  en  la  anastomosis  que  puede  exteriorizarse.
Es  probable  que  cada  método  encuentre  múltiples  colecciones,  y  cada  uno  puede  dejar  drenajes  externos  para  
el  lavado  y  el  drenaje  continuo.  Ya  sea  con  aguja  o  bisturí,  la  urgencia  y  adecuación  del  tratamiento  local  de  la  
infección  focal  determina  qué  métodos  tienen  prioridad.

24.  ¿Cuál  es  el  papel  de  la  gammagrafía  con  galio  en  la  detección  temprana  de  abscesos  en  el  abdomen?

No  hay  ninguno.  Ordenar  una  gammagrafía  con  galio  es  un  medio  temporal  de  autoengaño  de  que  se  está  
logrando  algún  progreso  para  descubrir  qué  le  pasa  al  paciente.  De  hecho,  simplemente  pospone  las  decisiones  
sobre  la  intervención  en  enfermedades  críticas  durante  varios  días,  a  menudo  hasta  un  punto  más  allá  del  rescate.
La  exploración  con  galio  implica  la  preparación  del  intestino,  una  respuesta  vigorosa  de  glóbulos  blancos  
de  una  médula  ósea  activa  y  resultados  de  prueba  falsos  positivos  en  los  sitios  de  los  tubos  y  las  incisiones.  Es  
una  prueba  lenta  y  poco  confiable  que  es  el  anverso  de  los  principios  del  manejo  temprano  y  definitivo.  No  ordene  
una  gammagrafía  con  galio  para  convencer  a  un  consultor  de  que  "algo  se  está  haciendo  por  este  paciente".

PREGUNTAS  DE  CRÉDITO  EXTRA

25.  ¿Todos  los  pacientes  que  se  someten  a  una  laparotomía  electiva  deben  recibir  protección  antibiótica  
profiláctica?
No.  Hacerlo  contribuiría  a  aumentar  el  costo  de  los  antibióticos  y  su  tasa  de  complicaciones  y  devaluar  los  
medicamentos  que  antes  eran  buenos  al  hacerlos  inútiles  contra  la  flora  común  contra  la  que  alguna  vez  
fueron  muy  potentes.  Las  enfermeras  de  quirófano  siempre  han  clasificado  las
Machine Translated by Google
84  CAPÍTULO  13  ENFERMEDADES  INFECCIOSAS  QUIRÚRGICAS

tipo  de  operación  por  su  estado  con  respecto  a  la  exposición  microbiana:  limpia,  contaminada  o  séptica.  
Estas  categorías  son  una  aproximación  al  riesgo  microbiano  de  exposición,  y  si  además  se  superponen  categorías  
de  resistencia  de  los  pacientes  (mayor  riesgo  asociado  al  envejecimiento,  obesidad  u  otra  desnutrición,  
medicamentos  concomitantes,  compromiso  inmunológico  a  enfermedades  virales,  micobacterianas  o  neoplásicas),  
estos  mismos  estratos  se  denominan  clase  I,  II  y  III.

26.  ¿Qué  absceso  es  el  más  importante  a  drenar?
Es  el  último  absceso  el  que  cuenta  en  el  drenaje  porque  la  respuesta  dramática  del  paciente  a  menudo  solo  se  
logra  cuando  se  drena  el  último  pus.  Drenar  un  absceso  pélvico,  por  ejemplo,  pero  dejar  atrás  un  absceso  
subfrénico,  no  daría  como  resultado  la  extinción  de  los  mediadores  inflamatorios  del  síndrome  de  sepsis.

27.  ¿Es  la  fiebre  postoperatoria  el  signo  más  temprano  y  más  frecuente  de  una  lesión  incisional?
¿infección?
Las  fiebres  posoperatorias  son  mucho  más  frecuentes  que  las  infecciones  de  heridas,  y  la  típica  infección  de  
heridas  se  presenta  mucho  más  tarde.  Las  principales  fuentes  de  fiebre  postoperatoria  son:  Viento  (atelectasia  o  
neumonía)
Agua  (infección  del  tracto  urinario)
Camine  (haga  que  su  paciente  se  levante  y  camine;  tromboflebitis)
Herida

28.  ¿Debería  comenzar  con  anfotericina  en  el  primer  aislamiento  de  especies  de  Candida  extraído  de  cualquier  línea  
de  catéter  intravenoso?
No.  Nuevamente,  recuerde  la  distinción  entre  colonización  e  infección,  y  la  fuente  de  donde  se  toma  la  muestra.  
Las  vías  intravenosas  a  través  de  las  cuales  se  infunden  las  soluciones  de  hiperalimentación  hacen  posible  la  
colonización.  La  presencia  de  un  hongo  como  la  especie  Candida  es  frecuente  en  pacientes  que  no  tienen  una  
infección  fúngica  invasiva  o  una  verdadera  candidemia.  Esta  última  podría  distinguirse  de  la  colonización  por  
catéter  mediante  un  hemocultivo  extraído  de  otra  fuente,  como  una  venopunción.  Si  también  hay  evidencia  de  
cualquier  infección  micótica  invasiva  (p.  ej.,  como  biopsia  endoscópica  de  mucositis  inflamatoria),  ahora  se  indica  
una  opción  de  terapias  antimicóticas.
Las  soluciones  fúngicas  tópicas  (p.  ej.,  enjuagues  bucales  o  lavados  de  micostatina)  pueden  
controlar  la  infección  fúngica  local  y,  a  veces,  se  pueden  instituir  como  profilaxis  en  pacientes  de  alto  riesgo  
(p.  ej.,  pacientes  con  terapia  antirrechazo  para  trasplante  de  médula  ósea  o  de  órganos  sólidos).
Los  agentes  antimicóticos  sistémicos  incluyen  fluconazol,  caspifungina  y  anfotericina.

29.  ¿Las  combinaciones  de  fármacos  antibióticos  son  siempre  superiores  a  un  único  agente  antibiótico?
La  monoterapia  es  superior  a  los  regímenes  de  tratamiento  con  antibióticos  combinados,  pero  esto  
probablemente  solo  se  puede  demostrar  en  los  pacientes  de  mayor  riesgo.  Con  los  agentes  antibióticos  de  la  
clase  carbapenem,  un  gran  ensayo  clínico  multicéntrico  demostró  que  la  terapia  con  imipenem  es  superior  al  
aminoglucósido  y  un  antibiótico  macrólido,  con  una  supervivencia  demostrablemente  superior  solo  en  los  
pacientes  con  los  puntajes  más  altos  de  fisiología  aguda  y  evaluación  de  salud  crónica  (APACHE).  La  monoterapia  
con  ertapenem  fue  equivalente  a  ceftriaxona  y  metronidazol  en  un  ensayo  más  pequeño  y  más  reciente.
Más  no  siempre  es  mejor,  y  las  R  y  S  en  los  informes  de  cultivo  no  se  traducen  directamente  en  M  y  M  
(morbilidad  y  mortalidad)  en  los  informes  de  la  Conferencia  de  Muerte  y  Complicaciones.  No  solo  es  importante  
que  el  régimen  antibiótico  eficaz  elimine  las  bacterias;  también  son  importantes  cómo  se  lleva  a  cabo  este  efecto  
microbicida  y  qué  efecto  puede  tener  sobre  el  paciente  para  apagar  o  prolongar  la  respuesta  inflamatoria  sistémica.

30.  ¿Es  el  tratamiento  con  anticuerpos  de  la  endotoxina  circulante  una  herramienta  clínicamente  importante?
Aún  no.  La  neutralización  de  la  endotoxina  circulante  podría  brindar  un  beneficio  teórico  a  los  pacientes  con  
sepsis,  y  los  estudios  en  animales  parecían  prometedores.  Pero  los  complejos  antígeno/anticuerpo  inician  la  
cascada  del  complemento  y  la  liberación  de  productos  leucocitarios  activos  como  los  leucotrienos  que
Machine Translated by Google
CAPÍTULO  13  ENFERMEDAD  INFECCIOSA  QUIRÚRGICA  85

puede  aumentar  aún  más  el  proceso  inflamatorio.  Los  complejos  también  se  filtran  en  el  riñón,  donde  pueden  
deteriorar  aún  más  la  función  renal.  Hasta  la  fecha,  no  se  ha  demostrado  ningún  beneficio  terapéutico  clínico  
para  dicha  terapia  con  anticuerpos  monoclonales.

31.  ¿Cuál  es  el  papel  de  la  proteína  C  activada  recombinante  humana  en  pacientes  con
¿septicemia?
De  los  múltiples  ensayos  clínicos  de  neutralización  de  mediadores  o  bloqueo  de  receptores,  la  evidencia  hasta  
la  fecha  parece  marginalmente  favorable  solo  para  unos  pocos,  y  la  principal  respuesta  al  tratamiento  proviene  
del  control  temprano  y  completo  del  foco  de  sepsis  (no  de  las  secuelas  de  citoquinas).

SITIOS  WEB

www.acssurgery.com/abstracts/acs/acs0102.htm

www.medscape.com
Buscar:  antibióticos  preoperatorios

BIBLIOGRAFÍA

1.  Bartlett  JG:  sepsis  intraabdominal.  Med  Clin  North  Am  79:599­617,  1995.
2.  Bernard  GR,  Vincent  JL,  Laterre  PF  et  al.:  Eficacia  y  seguridad  de  la  proteína  C  activada  humana  recombinante  para
sepsis  severa.  N  Engl  J  Med  344:699,  2001.
3.  Bilik  R,  Burnweit  C,  Shandling  B:  ¿El  cultivo  de  la  cavidad  abdominal  tiene  algún  valor  en  la  apendicitis?  Am  J  Surg
175:267­270,  1998.

4.  Castaldo  ET,  Yang  EY:  Sepsis  grave  atribuible  a  Staphylococcus  resistente  a  la  meticilina  asociado  a  la  comunidad
aureus:  un  problema  fatal  emergente.  Am  Surg  73:684­687;  discusión  687­688,  2007.
5.  Christou  NV,  Turgeon  P,  Wassef  R  et  al.:  Manejo  de  infecciones  intraabdominales.  El  caso  de  los  cultivos  intraoperatorios  y  
la  cobertura  integral  de  antibióticos  de  amplio  espectro.  El  Grupo  Canadiense  de  Estudio  de  Infecciones  Intraabdominales.  
Arch  Surg  131:1193­1201,  1996.
6.  Ciftci  AO,  Tanyei  FC,  Buyukpamukcu  N  et  al.:  Ensayo  comparativo  de  cuatro  combinaciones  de  antibióticos  para
apendicitis  en  niños.  Eur  J  Surg  163:591­596,  1997.
7.  Falagas  ME,  Barefoot  L,  Griffith  J  et  al.:  Factores  de  riesgo  que  conducen  al  fracaso  clínico  en  el  tratamiento  de  la
o  infecciones  de  piel/tejidos  blandos.  Eur  J  Clin  Microbiol  Infect  Dis  15:913­921,  1996.

8.  Geelhoed  GW:  Preparación  preoperatoria  de  la  piel:  evaluación  de  eficacia,  tiempo,  conveniencia  y  costo.  Infectar  cirugía
85:648­669,  1985.
Machine Translated by Google

RIESGOS  DE  LAS  ENFERMEDADES  TRANSMITIDAS  POR  LA  SANGRE
CAPÍTULO  
14

Natasha  D.  Bir,  MD,  MHS

1.  ¿Qué  enfermedades  infecciosas  se  transmiten  por  transfusión  de  sangre?

Se  han  encontrado  virus,  parásitos  y  bacterias  y  las  enfermedades  que  transmiten  en  la  sangre  donada.  La  
detección  y  las  pruebas  de  los  donantes  han  reducido  drásticamente  el  riesgo  de  infecciones  relacionadas  con  las  
transfusiones  en  el  mundo  desarrollado;  sin  embargo,  la  infección  sigue  siendo  un  riesgo  significativo  en  los  países  
menos  desarrollados,  donde  más  de  10  millones  de  unidades  de  sangre  no  se  analizan  para  detectar  el  VIH  o  las  
hepatitis.  Las  enfermedades  de  transmisión  más  comunes  en  los  países  desarrollados  incluyen  el  virus  de  la  hepatitis  B  
(VHB)  y  el  virus  de  la  hepatitis  C  (VHC);  La  transmisión  del  virus  de  la  inmunodeficiencia  humana  (VIH)  y  el  citomegalovirus  
(CMV)  es  mucho  menos  común.  Los  parásitos  como  la  malaria  (Plasmodium),  la  enfermedad  de  Chagas  (Trypanosoma  
cruzii),  la  toxoplasmosis  (Toxoplasma  gondii)  y  la  babesiosis  (Babseium),  son  solo  un  problema  donde  estas  enfermedades  
son  endémicas.
Los  linfomas  y  las  leucemias  pueden  ser  causados  por  el  virus  linfotrópico  de  células  T  humanas  (HTLV­1)  y  la  
mononucleosis  infecciosa  por  el  virus  de  Epstein­Barr  (EBV).  La  contaminación  bacteriana  de  los  productos  sanguíneos  
ocurre  con  mayor  frecuencia  en  las  plaquetas,  que  se  almacenan  a  temperatura  ambiente.  La  contaminación  bacteriana  
puede  provocar  sepsis  o  un  síndrome  similar  al  shock  tóxico.

2.  ¿Cuáles  son  los  riesgos  estimados  de  transmisión  de  VHB,  VHC  y  VIH  por  transfusión  de  sangre  en  los  Estados  
Unidos?

Las  tasas  de  transmisión  de  enfermedades  virales  son  más  bajas  que  nunca,  particularmente  después  de  que  en  1999  
comenzaron  las  pruebas  de  ácido  nucleico  para  VIH,  VHB  y  VHC.  En  la  actualidad,  se  emplean  modelos  matemáticos  
para  estimar  los  riesgos  de  transmisión  viral.

Frecuencia  por  millón Riesgo  de  enfermedad  por  real
Enfermedad Unidades  de  Sangre Unidad  transfundida

VHB 17 1/60.000  a  1/200.000  

VHC 1 1/800.000  a  1/1.600.000  

VIH 1 1/1.400.000  a  1/2.400.000

transmisión  bacteriana;   2 1/500,000
glóbulos  rojos  empaquetados

transmisión  bacteriana;   500 1/2,000


plaquetas

3.  ¿Qué  patógenos  transmitidos  por  la  sangre  representan  un  riesgo  para  los  cirujanos?
Más  de  8  millones  de  trabajadores  de  la  salud  están  expuestos  a  sangre  u  otros  fluidos  corporales  anualmente.
El  ochenta  y  dos  por  ciento  están  expuestos  a  través  de  lesiones  percutáneas,  como  pinchazos  con  agujas,  y  otro  14%  
a  través  del  contacto  con  las  membranas  mucosas  de  los  ojos,  la  boca  o  la  nariz.  El  VIH,  el  VHB  y  el  VHC  son  
enfermedades  que  preocupan  a  los  cirujanos  debido  a  la  morbilidad  y  mortalidad  asociadas  con  estas  enfermedades.  A  
partir  de  2003  (los  datos  más  recientes  disponibles  en  2008)  solo  57  confirmaron

86
Machine Translated by Google
CAPÍTULO  14  RIESGOS  DE  LAS  ENFERMEDADES  TRANSMITIDAS  POR  LA  SANGRE  87

Se  han  informado  casos  de  infección  por  VIH  en  trabajadores  de  la  salud  por  parte  de  pacientes,  incluidos  
seis  médicos  (todos  no  cirujanos).  En  todos  los  casos,  la  lesión  incitante  involucró  cortes  significativos  o  
penetración  con  agujas  huecas  de  gran  calibre.  Los  pinchazos  con  agujas  sólidas  nunca  han  resultado  en  la  
transmisión  del  VIH.  La  transmisión  del  VIH  entre  el  paciente  y  el  cirujano  en  el  quirófano  nunca  se  ha  documentado  en  
los  Estados  Unidos.

La  infección  por  VHB  de  los  cirujanos  ha  disminuido  con  el  uso  generalizado  de  la  vacuna  contra  el  VHB  (ver  
más  abajo).  Un  pinchazo  con  una  aguja  hueca  puede  provocar  la  transmisión  del  VHB  hasta  en  un  30  %  de  los  casos.
El  riesgo  de  VHC  en  el  quirófano  sigue  siendo  importante  porque  no  hay  vacuna  disponible  y  el  número  de  
pacientes  con  infección  crónica  supera  los  4  millones.  Después  de  la  exposición,  la  tasa  de  seroconversión  al  VHC  
es  de  aproximadamente  10%.  Entre  el  cincuenta  y  el  ochenta  por  ciento  de  los  seroconvertidores  desarrollan  una  
infección  crónica  persistente  por  el  VHC,  y  el  20%  de  estos  avanzan  a  la  cirrosis  hepática.  Dada  la  falta  de  cura  y  las  
consecuencias  potencialmente  devastadoras  de  esta  enfermedad,  la  infección  por  VHC  es  la  mayor  amenaza  para  los  
cirujanos.

4.  ¿Cuál  es  el  riesgo  de  exposición  al  virus  de  la  hepatitis  B  para  los  trabajadores  de  la  salud?
En  los  Estados  Unidos,  hay  2  millones  de  personas  con  HBV,  aproximadamente  100  000  casos  nuevos  cada  año.  La  
prevalencia  más  alta  se  da  en  individuos  de  20  a  49  años.  El  30%  de  los  casos  agudos  de  VHB  están  clínicamente  
ocultos  y  el  10%  siguen  siendo  portadores  crónicos  de  por  vida.  Alrededor  del  25  por  ciento  de  las  personas  con  VHB  
crónico  eventualmente  mueren  de  enfermedad  hepática.  Entre  los  trabajadores  de  la  salud,  la  vacunación  y  la  adopción  
de  precauciones  universales  condujeron  a  una  fuerte  disminución  de  las  nuevas  infecciones  por  VHB,  de  12  000  casos  
en  1985  a  aproximadamente  1000  en  1994.  Sin  embargo,  doscientos  cincuenta  trabajadores  de  la  salud  mueren  a  
causa  del  VHB  crónico  cada  año.

5.  ¿Cuál  es  el  riesgo  de  exposición  al  virus  de  la  hepatitis  C  (VHC)  para  los  trabajadores  de  la  salud?
El  VHC  se  transmite  a  través  de  la  sangre,  y  los  pacientes  con  mayor  riesgo  incluyen  usuarios  de  drogas  inyectables,  
pacientes  que  recibieron  una  transfusión  de  sangre  antes  de  1999,  hemofílicos,  pacientes  en  hemodiálisis  y  trabajadores  
de  la  salud.  La  infección  aguda  por  VHC  es  asintomática  en  el  70%  de  los  casos.  El  sesenta  por  ciento  de  las  
infecciones  agudas  por  VHC  resultan  en  una  infección  crónica  y  persistente.  Aunque  estos  datos  siguen  siendo  
controvertidos,  el  50%  de  los  pacientes  infectados  por  el  VHC  desarrollarán  cirrosis  y  la  mitad  de  ellos  desarrollarán  un  
hepatoma.  Aproximadamente  el  10%  de  los  pinchazos  con  agujas  resultan  en  una  infección  aguda  por  VHC.

6.  ¿Cuál  es  el  riesgo  de  exposición  al  VIH  para  los  trabajadores  de  la  salud?
Entre  los  trabajadores  de  la  salud,  solo  se  han  producido  57  casos  confirmados  y  139  casos  "posibles" (no  confirmados  
como  resultado  de  una  documentación  deficiente)  de  transmisión  del  VIH  desde  1983.  La  mayoría  de  los  casos  
confirmados  fueron  enfermeras  (n¼24),  mientras  que  seis  fueron  médicos.  Ninguno  era  cirujano.
Ochenta  y  cuatro  por  ciento  de  los  casos  sufrieron  vías  de  transmisión  percutáneas,  es  decir,  cortes  o  pinchazos.  El  
riesgo  de  seroconversión  al  VIH  tras  una  exposición  percutánea  es  del  0,3%.  No  ha  habido  casos  documentados  de  
transmisión  de  un  paciente  VIH  positivo  a  un  cirujano.

7.  ¿Qué  tan  bien  protege  la  vacunación  contra  la  hepatitis  B  contra  la  enfermedad?
El  noventa  por  ciento  de  las  personas  que  completan  la  serie  de  inmunización  de  tres  dosis  contra  el  
VHB  desarrollan  títulos  de  anticuerpos  de  superficie  anti­VHB  (anti­HBs)  de  10  mIU/ml.  Un  8%  adicional  
muestra  niveles  de  título  apropiados  después  de  dosis  adicionales.  Un  nivel  de  anti­HBs  de  10  o  más  mIU/ml  
confiere  una  eficacia  protectora  de  casi  el  100%.  Si  bien  aproximadamente  la  mitad  de  los  adultos  vacunados  con  
éxito  muestran  un  nivel  de  título  disminuido  o  no  detectable  dentro  de  los  10  años,  persiste  una  "memoria  
inmunitaria"  de  por  vida  al  antígeno  viral  y  las  personas  no  requieren  dosis  de  refuerzo.

En  2001,  la  Administración  de  Drogas  y  Alimentos  de  los  Estados  Unidos  (FDA,  por  sus  siglas  en  inglés)  aprobó  
una  vacuna  bivalente  que  inmuniza  contra  la  hepatitis  A  y  B  para  personas  mayores  de  18  años,  y  tiene  el  mismo  éxito  
que  la  vacuna  monovalente  para  conferir  protección  contra  la  infección  por  VHB  con  la  beneficio  adicional  de  proteger  
contra  la  infección  viral  de  la  hepatitis  A.  También  hay  dos  marcas  de  vacunas  de  ADN  recombinante  monovalente  
disponibles.
Machine Translated by Google
88  CAPÍTULO  14  RIESGOS  DE  LAS  ENFERMEDADES  TRANSMITIDAS  POR  LA  SANGRE

8.  ¿Están  los  pacientes  en  riesgo  de  infección  por  parte  de  cirujanos  que  están  infectados  con  el  VHB?
Se  han  documentado  algunos  casos  de  transmisión  del  VHB  de  cirujano  a  paciente.  Aquellos  con  la  mayor  
posibilidad  de  transmitir  la  enfermedad  a  los  pacientes  son  positivos  para  el  antígeno  e  de  la  hepatitis  B,  un  
producto  de  degradación  de  la  nucleocápsida  viral  que  representa  la  replicación  activa  en  el  hígado.  Las  personas  
con  antígeno  E  positivo  generalmente  exhiben  una  alta  carga  viral.
Sin  embargo,  se  ha  documentado  la  transmisión  de  la  enfermedad  incluso  cuando  un  cirujano  dio  negativo  para  el  
antígeno  e.

9.  ¿Cuál  es  la  respuesta  adecuada  tras  la  exposición  percutánea  a  un  paciente  con
¿Hepatitis  B  conocida?
Para  los  médicos  que  han  sido  inmunizados  y  alguna  vez  demostraron  títulos  positivos,  no  se  necesita  una  respuesta  
adicional.  Los  niveles  de  títulos  de  algunas  personas  pueden  disminuir  con  el  tiempo,  por  lo  que  no  se  indican  los  niveles  
de  títulos  en  el  momento  de  la  exposición.  Las  personas  que  fueron  inmunizadas  o  que  tuvieron  una  respuesta  débil  o  
incompleta  a  la  vacuna  deben  recibir  una  dosis  de  inmunoglobulina  contra  la  hepatitis  B  y  luego  comenzar  de  nuevo  la  
serie  de  vacunación.

10.  ¿Cuáles  son  las  recomendaciones  para  la  vacunación  contra  la  hepatitis  B?
Se  requiere  la  vacunación  contra  la  hepatitis  B  para  los  cirujanos  en  formación  y  se  recomienda  encarecidamente  a  todos  
los  trabajadores  de  la  salud.  Las  recomendaciones  del  Servicio  de  Salud  Pública  de  los  Estados  Unidos  son  que  todos  los  

trabajadores  de  la  salud  que  realicen  tareas  que  puedan  implicar  exposición  a  sangre  o  fluidos  corporales  deben  recibir  
una  serie  de  3  dosis  de  la  vacuna  contra  la  hepatitis  B  a  intervalos  de  0,  1  y  6  meses.
Luego  se  les  debe  hacer  la  prueba  de  anticuerpos  de  superficie  contra  la  hepatitis  B  (anti­BH)  1  o  2  después  de  la  
vacunación,  para  documentar  la  inmunidad.  Si  el  nivel  de  anti­BH  es  de  al  menos  10  mIU/ml,  el  paciente  es  inmune.
Si  es  inferior  a  10  mUI/ml,  el  paciente  queda  desprotegido;  se  debe  administrar  otra  serie  de  vacunas  de  3  dosis  y  se  
deben  volver  a  verificar  los  títulos  1  o  2  meses  después.  Si  los  títulos  son  adecuados,  el  paciente  es  inmune.  Si  sigue  
siendo  inferior  a  10  mIU/ml,  se  considera  que  el  paciente  no  responde.
Deben  ser  considerados  susceptibles  al  VHB  y  deben  mantener  precauciones  estrictas  y  obtener  inmunoglobulina  
contra  la  hepatitis  B  para  cualquier  exposición  conocida  o  probable.

11.  ¿Cuáles  son  las  recomendaciones  para  la  vacunación  contra  la  hepatitis  C?
No  hay  ninguno.

La  única  protección  eficaz  contra  el  VHC  es  el  uso  riguroso  de  precauciones  universales  para  evitar  la  exposición  
a  fluidos  corporales  infectados.  No  existe  una  vacuna  eficaz  y  la  inmunoglobulina  no  confiere  protección.

12.  ¿La  cirugía  laparoscópica  minimiza  el  riesgo  de  contaminación  por  VIH?
La  técnica  laparoscópica  reduce  la  exposición  a  sangre  e  instrumentos  cortantes.  Sin  embargo,  la  descarga  de  
neumoperitoneo  puede  liberar  sangre  en  aerosol  y  líquido  peritoneal  al  quirófano  si  no  se  evacua  a  un  sistema  cerrado.

13.  ¿Es  el  doble  guante  un  método  efectivo  de  protección?
Sí,  las  tasas  de  contacto  entre  la  sangre  y  la  piel  disminuyen  en  un  70  %  con  la  adición  de  un  segundo  par  de  
guantes.  El  dedo  índice  no  dominante  es  el  objetivo  más  común.

14.  ¿Son  las  exposiciones  no  percutáneas  (salpicaduras  en  los  ojos)  una  amenaza  importante  para  los  cirujanos?
Según  los  Centros  para  el  Control  y  la  Prevención  de  Enfermedades  (CDC),  el  riesgo  de  seroconversión  después  de  la  
exposición  mucocutánea  (ojos,  nariz  o  boca)  es  del  0,1  %,  o  aproximadamente  1  en  1000.
El  contacto  mucocutáneo  es  responsable  del  13%  de  las  transmisiones  de  VIH  documentadas.  Entre  los  cirujanos,  
las  lesiones  por  salpicaduras  en  los  ojos  a  menudo  se  pasan  por  alto  como  un  riesgo  importante  de  transmisión  de  
enfermedades.  Un  estudio  de  procedimientos  quirúrgicos  examinó  los  protectores  oculares  de  160  cirujanos  y  asistentes.  
Todas  las  operaciones  duraron  30  minutos  o  más.  Aunque  los  cirujanos  estaban  conscientes  del  rociado  en  solo  el  8%  de  
los  casos,  las  salpicaduras  de  sangre  eran  macroscópicamente  visibles  en  el  16%  de  los  casos  y  microscópicamente  
positivas  en  el  44%  de  los  casos.  La  protección  de  los  ojos  es  importante.
Machine Translated by Google
CAPÍTULO  14  RIESGOS  DE  LAS  ENFERMEDADES  TRANSMITIDAS  POR  LA  SANGRE  89

15.  ¿Cuál  es  la  tasa  de  exposición  de  los  cirujanos  a  sangre  y  fluidos  corporales?
La  exposición  no  se  informa  ampliamente,  pero  la  exposición  percutánea  ocurre  en  un  1%  a  6%  estimado  de  los  
procedimientos  quirúrgicos  y  la  exposición  mucocutánea  en  hasta  el  50%  de  los  casos  quirúrgicos.  Ningún  trabajador  de  la  
salud  ha  sido  infectado  por  exposición  a  través  de  la  piel  intacta.

16.  Nuevamente,  ¿cuáles  son  las  tasas  de  seroconversión  para  la  exposición  al  VIH,  VHB  y  VHC?
Las  tasas  de  seroconversión  de  un  pinchazo  con  una  aguja  hueca  son  del  0,3  %  para  el  VIH,  del  10  %  para  el  VHC  y  
oscilan  entre  el  6  %  y  el  30  %  para  el  VHB.

17.  ¿Existen  métodos  efectivos  para  reducir  el  riesgo  de  transmisión  de  sangre
enfermedades  a  los  cirujanos?
Obviamente,  la  forma  más  efectiva  de  reducir  la  transmisión  de  enfermedades  es  limitar  la  exposición  a  sangre  o  fluidos  
corporales  infectados  (precauciones  de  barrera  universales).  Para  el  VHB,  la  administración  de  inmunoglobulina  posterior  a  
la  exposición  reduce  la  infección.  Existe  una  vacuna  altamente  efectiva  y  es  necesaria  para  la  mayoría  de  los  trabajadores  
de  la  salud.  Sorprendentemente,  la  mayoría  de  los  cirujanos  mayores  de  50  años  no  han  sido  vacunados.

18.  ¿Cuál  es  el  riesgo  para  los  cirujanos  en  formación?
Una  encuesta  multicéntrica  reciente  de  cirujanos  en  formación  señaló  que  los  pinchazos  con  agujas  son  frecuentes  
y,  a  menudo,  no  se  informan.  El  número  de  pinchazos  con  agujas  aumentó  con  cada  año  de  capacitación  (año  de  
posgrado  (PGY)  ­2,  3,7;  PGY­3,  4,1;  PGY­4,  5,3;  y  PGY­5,  7,7),  y  por  su  último  año  de  capacitación,  99  %  de  los  residentes  
tuvo  al  menos  un  pinchazo  de  aguja.  Aproximadamente  la  mitad  de  los  residentes  habían  estado  expuestos  a  sangre  de  
pacientes  de  alto  riesgo  (pacientes  con  antecedentes  de  VIH,  VHB,  VHC  o  uso  de  drogas  inyectables).
La  razón  más  común  para  no  reportar  un  pinchazo  de  aguja  fue  la  "falta  de  tiempo".

BIBLIOGRAFÍA

1.  Barrie  PS,  Patchen  Dellinger  E,  Dougherty  SH  et  al.:  Evaluación  del  estado  de  vacunación  contra  el  virus  de  la  hepatitis  B
entre  los  cirujanos  norteamericanos.  Arch  Surg  129:27­32,  1994.

2.  Bell  DM:  Riesgo  ocupacional  de  infección  por  el  virus  de  la  inmunodeficiencia  humana  en  trabajadores  de  la  salud:  una  descripción  general.
Am  J  Med  102  (suplemento  5B):  81S­85S,  1997.

3.  Dodd  RY,  Notari  EP,  Stramer  SL:  prevalencia  e  incidencia  actuales  de  marcadores  de  enfermedades  infecciosas  y  riesgo  estimado  del  período  
ventana  en  la  población  de  donantes  de  sangre  de  la  Cruz  Roja  Americana.  Transfusión  42:975­979,  2002.

4.  Eubanks  S,  Newman  L,  Lucas  G:  Reducción  de  la  transmisión  del  VIH  durante  los  procedimientos  laparoscópicos.  cirugía  laparoscópica
Endosc  3:2­5,  1993.

5.  Fry  DE:  Enfermedades  ocupacionales  de  transmisión  sanguínea  en  cirugía.  Am  J  Surg  190(2):249­254,  2005.

6.  Gershon  RR,  Sherman  M,  Mitchell  C  et  al.:  Prevalencia  y  factores  de  riesgo  de  exposición  e  infección  transmitidas  por  la  sangre
en  los  trabajadores  de  la  salud  correccional.  Infect  Control  Hosp  Epidemiol  28(1):24­30,  2007.

7.  Goodnough,  LT:  Riesgos  de  la  transfusión  de  sangre,  Anes  Clin  N  Am  23:241­252,  2005.

8.  Jaffray  CE,  Flint  LM:  Enfermedades  virales  transmitidas  por  la  sangre  y  el  cirujano.  Curr  Probl  Surg  40(4):195­251,  2003.

9.  Klein  HG,  Spahn  DR,  Carson  JL:  Transfusión  de  glóbulos  rojos  en  la  práctica  clínica.  Lanceta  370  (9585):  415­426,
2007.

10.  Koff  RS:  Hepatitis  A,  hepatitis  B  y  vacunas  combinadas  contra  la  hepatitis  para  inmunoprofilaxis:  una  actualización.
Digest  Dis  Sci  47:1183­1194,  2002.

11.  Makary  MA,  Al­Attar  A,  Holzmueller  CG  et  al .:  Lesiones  por  pinchazo  de  aguja  entre  cirujanos  en  formación.  N  Inglés  J  Med
356(26):2693­2699,  2007.

12.  Marasco  S,  Woods  S:  El  riesgo  de  lesiones  por  salpicaduras  oculares  en  la  cirugía.  Aust  NZJ  Surg  68:785­787,  1998.

13.  Mast  EE,  Weinbaum  CM,  Fiore  AE  et  al.:  Una  estrategia  de  inmunización  integral  para  eliminar  la  transmisión  de  la  infección  por  el  virus  de  
la  hepatitis  B  en  los  Estados  Unidos:  recomendaciones  del  Comité  Asesor  sobre  Prácticas  de  Inmunización  (ACIP)  Parte  II:  inmunización  
de  adultos.  MMWR  recomienda  Rep  55  (RR­16):  1­33,  2006.

14.  Weiss  ES,  Makary  MA,  Wang  T  et  al .:  Prevalencia  de  patógenos  transmitidos  por  la  sangre  en  una  práctica  quirúrgica  general  urbana  y  
universitaria.  Ann  Surg  241(5):803­807;  discusión  807­809,  2005.
Machine Translated by Google

II.  TRAUMA

EVALUACIÓN  INICIAL
CAPÍTULO  
15

Jeffry  L.  Kashuk,  MD,  FACS

1.  ¿Qué  es  la  ''hora  dorada''?
Esta  primera  hora  de  la  lesión  brinda  una  oportunidad  única  para  iniciar  intervenciones  que  salvan  vidas.
Más  de  la  mitad  de  las  muertes  por  trauma  ocurren  durante  este  período  de  tiempo  como  resultado  de  una  lesión  
cerebral  o  hemorragia  exanguinante,  por  lo  tanto,  el  transporte  rápido,  la  clasificación  adecuada  y  los  sistemas  
organizados  de  evaluación  (apoyo  vital  avanzado  en  trauma  [ATLS])  son  procedimientos  estandarizados  
importantes  que  pueden  salvar  vidas.

2.  Nombre  los  componentes  principales  de  la  evaluación  inicial  del  paciente  traumatizado.
Examen  primario,  reanimación,  examen  secundario,  reevaluación  y  atención  definitiva.

3.  ¿Cuál  es  el  propósito  de  la  encuesta  primaria?
Identificar  las  lesiones  que  amenazan  la  vida  a  través  de  un  sistema  de  prioridades  reproducible  y  un  marco  de  tiempo.

4.  Defina  el  mnemotécnico  ABCDE  de  la  encuesta  primaria  que  refuerza  el  hecho
que  las  lesiones  que  amenazan  la  vida  matan  en  un  orden  predecible.
Control  de  vía  aérea  con  protección  de  columna  cervical  
Respiración  con  oxigenación  y  ventilación  Circulación  
con  control  de  hemorragia  Incapacidad  o  estado  
neurológico  Exposición  del  paciente  con  control  de  
temperatura

5.  ¿Cuáles  son  los  adjuntos  a  la  encuesta  primaria?
Todos  los  pacientes  traumatizados  deben  recibir  inicialmente  oxígeno  suplementario  de  alto  flujo  mediante  cánula  
nasal  o  mascarilla  facial.  El  monitoreo  continuo  debe  incluir  oximetría  de  pulso,  monitor  de  electrocardiograma  (ECG)  
cardíaco  y  un  manguito  de  presión  arterial  (PA)  cíclico.  Se  colocan  dos  vías  intravenosas  (IV)  de  gran  calibre  a  medida  
que  se  extrae  sangre  para  las  pruebas  de  detección,  incluido  el  tipo  de  sangre  y  la  compatibilidad  cruzada.
Se  colocan  sondas  nasogástricas  (NG)  u  orogástricas  para  la  descompresión  gástrica  y  para  evitar  la  aspiración.  
Se  inserta  un  catéter  de  Foley  para  evaluar  el  flujo  de  orina  y  el  carácter  de  la  orina.  Las  radiografías  deben  incluir  los  
"tres  grandes"  para  el  "mecanismo"  de  trauma  mayor:  columna  cervical,  radiografía  de  tórax  y  radiografía  pélvica.  La  
radiografía  de  la  columna  cervical  se  puede  diferir  si  el  paciente  va  a  someterse  a  una  tomografía  computarizada  (TC).

6.  Identificar  el  único  concepto  que  puede  prevenir  el  deterioro  agudo  inesperado  del  paciente  traumatizado  
durante  la  evaluación  inicial.
Reevaluación:  si  se  produce  un  deterioro,  vuelva  al  ABC  en  orden  secuencial.

7.  Nombre  las  dos  principales  causas  de  muerte  durante  las  primeras  24  horas  después  de  la  lesión.
Exanguinación  secundaria  a  hemorragia  por  heridas  traumáticas  y  lesión  del  sistema  nervioso  central  (SNC).

91
Machine Translated by Google
92  CAPÍTULO  15  EVALUACIÓN  INICIAL

8.  ¿Cómo  se  evalúa  la  vía  aérea?
Hágale  una  pregunta  al  paciente.  Una  respuesta  con  voz  normal  sugiere  que  las  vías  respiratorias  no  están  en  
peligro  inmediato.  Una  respuesta  inestable,  débil  o  estridorosa  puede  implicar  un  compromiso  de  las  vías  respiratorias.
Una  respuesta  agitada  o  combativa  indica  hipoxia,  hasta  que  se  demuestre  lo  contrario.  Ninguna  respuesta  indica  la  
necesidad  de  una  "vía  aérea  definitiva" (idealmente,  un  tubo  con  manguito  en  la  tráquea).

9.  ¿Cuáles  son  las  causas  de  la  obstrucción  de  las  vías  respiratorias  superiores  en  el  paciente  traumatizado?
La  lengua,  seguida  de  sangre,  dientes  flojos  o  dentaduras  postizas,  vómito  y  edema  de  tejidos  blandos.

10.  ¿Cuáles  son  las  maniobras  iniciales  que  se  utilizan  para  restaurar  una  vía  aérea  abierta?
La  elevación  del  mentón  y  la  tracción  de  la  mandíbula  desplazan  físicamente  la  mandíbula  y  la  lengua  hacia  
delante  para  abrir  las  vías  respiratorias,  lo  que  facilitará  la  limpieza  manual  de  los  desechos  y  la  succión  de  la  
orofaringe  para  optimizar  la  permeabilidad.  Las  vías  respiratorias  orofaríngeas  y  nasofaríngeas  (trompetas)  son  
complementos  útiles  para  mantener  abiertas  las  vías  respiratorias  en  un  paciente  obnubilado.  Siempre  se  debe  asumir  
la  presencia  de  una  lesión  en  la  columna  cervical  hasta  que  se  demuestre  lo  contrario,  realizando  una  estabilización  
en  línea  mientras  se  evalúa  la  vía  aérea.

11.  ¿Cuáles  son  las  indicaciones  para  una  vía  aérea  definitiva?
Apnea,  incapacidad  para  mantener  o  proteger  las  vías  respiratorias  (conciencia  comprometida),  incapacidad  para  
mantener  la  oxigenación,  inestabilidad  hemodinámica,  necesidad  de  relajación  muscular  o  sedación  y  necesidad  de  
hiperventilación.

12.  Enumere  los  tipos  de  vía  aérea  definitiva  que  están  disponibles  en  su  orden  de  prioridad.
&  Intubación  orotraqueal.
&  Intubación  nasotraqueal.

  vía  aérea  quirúrgica  (cricotiroidotomía  o  traqueotomía).

13.  ¿Cuáles  son  las  indicaciones  de  una  vía  aérea  quirúrgica?
Traumatismos  y  maxilofaciales  extensos,  traumatismos  cervicales  anteriores  de  alto  riesgo  o  cualquier  situación  en  la  que  la  
intubación  de  las  vías  respiratorias  no  se  pueda  realizar  de  forma  segura.
Las  contraindicaciones  incluyen:  traumatismo  laríngeo  directo,  sospecha  de  rotura  traqueal  y  niños,  que  
tienen  un  riesgo  mucho  mayor  de  estenosis  en  esta  región  después  del  procedimiento.
Las  opciones  preferidas  en  este  grupo  son  la  traqueotomía  y  la  ventilación  transtraqueal.

14.  ¿Cómo  se  "limpia  la  columna  C"?
Se  debe  descartar  una  lesión  antes  de  mover  la  cabeza  o  el  cuello  de  un  paciente  traumatizado.  Los  pacientes  alerta  
sin  otras  "lesiones  que  distraigan"  significativas  pueden  ser  dados  de  alta  si  están  asintomáticos  y  no  tienen  
sensibilidad  en  el  examen  por  palpación  directa.  Otros  pacientes  requieren  evaluación  radiológica.  La  mayoría  de  las  
lesiones  óseas  se  pueden  encontrar  con  una  tomografía  computarizada  definitiva  del  cuello,  reservando  la  evaluación  
por  imágenes  de  resonancia  magnética  (IRM)  para  la  sospecha  de  lesión  ligamentosa  del  tejido  blando  que  puede  causar  
inestabilidad.  En  ausencia  de  capacidad  de  tomografía  computarizada,  se  requiere  una  serie  de  tres  vistas  de  la  columna  
cervical  (anteroposterior  [AP],  lateral  y  odontoides),  con  visualización  hasta  el  nivel  de  C7­T1.
Este  nivel  es  frecuentemente  difícil  de  ver,  requiriendo  una  "vista  de  nadador"  para  acentuar  la  visualización  
de  esta  región  anatómica.

15.  ¿Cuáles  son  las  cinco  condiciones  no  relacionadas  con  las  vías  respiratorias  que  representan  una  amenaza  
inmediata  para  la  respiración  del  paciente  traumatizado?
Neumotórax  a  tensión:  aire  en  el  espacio  pleural  a  presión  que  obstruye  el  flujo  venoso  por  torcedura  de  la  vena  
cava,  se  trata  con  descompresión  urgente  con  toracostomía  con  aguja  o  tubo.

Neumotórax  abierto:  herida  abierta  de  la  pared  torácica  que  deja  libre  la  comunicación  del  espacio  pleural  con  el  
aire  e  interfiere  con  el  mecanismo  del  fuelle  torácico  tratado  con  toracostomía  con  sonda.
Machine Translated by Google
CAPÍTULO  15  EVALUACIÓN  INICIAL  93

Tórax  inestable:  múltiples  fracturas  costales  con  un  segmento  flotante  y  posible  contusión  pulmonar  
subyacente,  tratadas  con  toracostomía  con  sonda  y  con  frecuencia  intubación  endotraqueal.

Hemotórax  masivo:  una  gran  acumulación  de  sangre  en  el  espacio  pleural  que  limita  el  pulmón
ventilación  y  oxigenación  tratada  con  tubo  de  toracostomía  y  posible  toracotomía.
Taponamiento  pericárdico:  la  inhibición  del  llenado  diastólico  y  el  gasto  cardíaco  (CO)  asociado,  la  principal  causa  
de  shock  cardiogénico  en  el  traumatismo,  requiere  la  evacuación  del  taponamiento  inicialmente  mediante  
aspiración  si  es  posible,  y  una  toracotomía  de  emergencia  subsiguiente  con  corrección  de  la  lesión  subyacente.

16.  ¿Cuáles  son  los  sitios  preferidos  para  el  acceso  intravenoso  de  emergencia?
Acceso  venoso  periférico  en  las  extremidades  superiores  con  catéter  de  gran  calibre  (calibre  14  a  16).
Otras  opciones  incluyen  la  vena  safena  del  tobillo  o  la  ingle.  El  acceso  venoso  central  (ruta  subclavia  o  yugular)  
está  indicado  para  la  medición  de  la  presión  venosa  central  (PVC)  después  de  los  bolos  de  líquido  iniciales  para  
evaluar  la  inestabilidad  hemodinámica.  En  niños  menores  de  6  años,  la  vía  interósea  en  el  fémur  distal  o  la  tibia  
proximal  es  una  alternativa  eficaz.

17.  ¿Cuáles  son  las  medidas  comunes  y  sencillas  para  evaluar  la  estabilidad  hemodinámica  en
un  paciente  traumatizado?
Estado  mental  (alerta,  verbal,  con  dolor  o  sin  respuesta).
Perfusión  de  la  piel  (rosada/cálida  versus  pálida/fría).
Parámetros  hemodinámicos  (PA,  frecuencia  cardíaca  [FC]  y  frecuencia  respiratoria  [FR]).
Estimaciones  brutas  de  la  presión  arterial  sistólica:  pulso  radial:  >80  mm  Hg;  femoral:  >70  mm  Hg;  carótida:  >60  mm  
Hg.  El  flujo  de  orina  de  >½  mililitros  por  kilogramo  por  hora  sugiere  una  buena  perfusión  de  órganos  finales.

18.  ¿Qué  es  la  escala  de  coma  de  Glasgow  y  qué  mide?
La  escala  de  coma  de  Glasgow  (GCS)  es  una  evaluación  del  estado  mental,  el  estado  papilar  y  la  mejor  
actividad  motora.
Mejor  respuesta  reveladora,  puntuada  de  1  a  4  Mejor  
respuesta  verbal,  puntuada  de  1  a  5  Mejor  respuesta  
motora,  puntuada  de  1  a  6
Los  puntos  se  suman.  Una  puntuación  general  de  13  a  15  indica  una  lesión  leve  en  la  cabeza,  de  9  a  12  indica  una  
lesión  moderada  y  <8  indica  una  lesión  grave  y  obliga  a  la  intubación  endotraqueal.

19.  ¿Qué  líquidos  se  deben  usar  para  la  reanimación  inicial?
Ringer  lactato  o  solución  salina  normal  son  el  pilar  de  la  reanimación  con  líquidos  a  través  de  una  infusión  rápida.
Debe  administrarse  sangre  y  plasma  temprano  para  optimizar  la  capacidad  de  transporte  de  oxígeno  y  prevenir  
la  coagulopatía  progresiva  en  pacientes  que  presentan  signos  de  acidosis  (pH  <7,25),  hipotermia  (temperatura  <34  
C),  coagulopatía  (índice  internacional  normalizado  [INR]  >1,5)  en  ante  un  shock  severo  (PA  sistólica  <70  mm  Hg).

Las  infusiones  de  coloides  son  más  costosas  y  no  tienen  ninguna  ventaja  comprobada  en  el  entorno  de  trauma.

20.  ¿Qué  significa  FAST  y  cómo  ayuda  en  la  evaluación  del  trauma?
FAST  significa  ecografía  abdominal  enfocada  en  trauma.  Las  cuatro  áreas  examinadas  por  ultrasonido  incluyen  
el  área  pericárdica,  el  cuadrante  superior  derecho,  el  cuadrante  superior  izquierdo  y  la  pelvis.
La  prueba  se  informa  como  positiva  (sangre)  o  negativa.

21.  ¿Qué  es  DPL  y  tiene  un  papel  en  la  evaluación  del  trauma?
DPL  significa  lavado  peritoneal  de  diagnóstico,  en  el  que  se  inserta  un  pequeño  catéter  en  el  paciente  para  evaluar  el  
sangrado  intraperitoneal.  Su  principal  utilidad  en  la  era  del  FAST  y  la  tomografía  computarizada  es  en  el  paciente  
hemodinámicamente  inestable  cuyo  FAST  inicial  es  negativo  para  descartar  mejor  el  abdomen  como  fuente  de  
hemorragia.
Machine Translated by Google
94  CAPÍTULO  15  EVALUACIÓN  INICIAL

22.  ¿Cómo  puedo  aprender  competencia  en  la  evaluación  inicial?
Tome  el  curso  ATLS  del  American  College  of  Surgeons,  que  enfatiza  las  habilidades  necesarias  para  tratar  
inicialmente  al  paciente  con  trauma.

PUNTOS  CLAVE  DE  LA  EVALUACIÓN  INICIAL
1.  Siga  los  ABCDE  del  sistema  ATLS  cuando  evalúe  a  un  paciente  traumatizado  y  regrese  al
mismo  orden  secuencial  al  reevaluar  al  paciente.

2.  Asumir  que  todo  paciente  traumatizado  tiene  una  lesión  en  la  columna  cervical  hasta  que  se  demuestre  lo  contrario,  y
evaluar  cuidadosamente  los  métodos  para  evaluar  o  limpiar  la  columna  cervical.

3.  Establecer  una  vía  aérea  segura  según  el  patrón  de  lesión  presente  o  el  estado  neurológico  de
el  paciente  (GCS).

4.  Evalúe  la  presencia  de  shock  e  inicie  la  reanimación  con  líquidos,  sangre  o  plasma  según  el  nivel  de  shock  y  los  
signos  asociados  de  coagulopatía,  hipotermia  y  acidosis  a  través  de  infusiones  intravenosas  periféricas  de  gran  
calibre.

5.  Establecer  un  cateterismo  venoso  central  para  ayudar  a  evaluar  la  estabilidad  homodinámica.

6.  Use  la  exploración  FAST,  DPL  y  CT  para  evaluar  el  alcance  de  las  lesiones  y  clasificar  al  paciente
adecuadamente.

BIBLIOGRAFÍA

1.  Comité  de  Trauma  del  Colegio  Estadounidense  de  Cirujanos:  Curso  avanzado  de  soporte  vital  en  trauma  7.ª  ed.,  Chicago,
2004,  Colegio  Americano  de  Cirujanos.

2.  Cha  J,  Kashuk  JL,  Moore  EE:  El  lavado  peritoneal  de  diagnóstico  sigue  siendo  un  complemento  valioso  para  las  técnicas  de  imagen  
modernas.  J  Trauma,  en  prensa.

3.  Cothren  CC,  Moore  EE:  toracotomía  del  departamento  de  emergencias  En:  Trauma,  6.ª  ed.,  Nueva  York,  2008,  McGraw  Hill.

4.  Fisher  A,  Young  WF:  ¿Está  obsoleta  la  radiografía  lateral  de  la  columna  cervical  durante  la  evaluación  inicial  de  pacientes  con
¿trauma?  Surg  Neurol  70(1):25­28,  2008.

5.  Kashuk  JL,  Moore  EE,  Johnson  JL  et  al .:  Coagulopatía  postoperatoria  potencialmente  mortal:  ¿es  1:  1  FFP:  RBC  la  respuesta?  J  
Trauma  66:xx,  2008.

6.  Kaufmann  CR:  Evaluación  inicial  y  gestión.  En  Trauma,  6ª  ed.,  Nueva  York,  2008,  McGraw­Hill.

7.  Rabb  CH,  Johnson  JL,  VanSickle  D  et  al .:  ¿Son  radiografías  cervicales  laterales  en  posición  vertical  en  el  paciente  con  trauma  obnubilado?
¿útil?  Un  estudio  retrospectivo,  World  J  Emerg  Surg  2:4,  2007.

8.  Sanchez  B,  Waxman  K,  Jones  T  et  al .:  Aclaramiento  de  la  columna  cervical  en  trauma  cerrado:  evaluación  de  un
Protocolo  basado  en  tomografía.  J  Trauma  59:179,  2005.
Machine Translated by Google

SHOCK  HEMORRÁGICO  POSTRAUMÁTICO
CAPÍTULO  
16

Ryan  P.  Merkow,  MD  y  Ernest  E.  Moore,  MD

1.  ¿Son  lo  mismo  shock  hemorrágico  y  shock  hipovolémico?
Sí.

2.  ¿Qué  es  el  shock  hemorrágico?
El  shock  existe  cuando  el  sistema  cardiovascular  ya  no  puede  satisfacer  las  necesidades  metabólicas  y  de  
oxígeno  del  cuerpo,  lo  que  provoca  una  lesión  celular.  En  otras  palabras,  los  tejidos  no  se  perfunden  
adecuadamente  para  satisfacer  sus  necesidades  de  oxígeno  y  nutrientes.

3.  ¿Cuál  es  el  manejo  inicial  del  shock  hemorrágico  o  hipovolémico?
Reanimación  rápida  y  agresiva  con  líquidos  en  un  intento  de  restaurar  el  volumen  de  sangre  circulante.  La  hemorragia  es  la  
causa  más  común  de  shock  en  el  paciente  lesionado.  El  agotamiento  del  volumen  sanguíneo  da  como  resultado  una  
disminución  de  la  presión  de  conducción  que  devuelve  la  sangre  al  corazón,  una  disminución  del  volumen  ventricular  al  final  
de  la  diástole  y  una  disminución  del  volumen  sistólico;  resultando  en  una  disminución  del  gasto  cardíaco  (GC).

4.  Describir  las  manifestaciones  celulares  del  shock  hemorrágico.
La  perfusión  tisular  inadecuada  da  como  resultado  una  disminución  de  la  tensión  de  oxígeno  celular  y  la  
interrupción  de  la  fosforilación  oxidativa  normal  con  una  disminución  en  la  generación  de  trifosfato  de  adenosina  
(ATP).  La  Naþ  Kþ  ATPasa  se  ralentiza  y  la  célula  ya  no  puede  mantener  la  integridad  de  la  polarización  de  la  
membrana,  lo  que  perjudica  una  serie  de  procesos  celulares  importantes.  El  metabolismo  anaeróbico  se  
produce  dando  como  resultado  la  producción  de  ácido  láctico,  creando  una  acidosis  metabólica  "brecha".  La  
primera  evidencia  de  esta  disfunción  es  la  hinchazón  del  retículo  endoplásmico,  seguida  de  daño  mitocondrial,  
ruptura  de  la  lisozima  y  entrada  de  agua  intersticial  en  la  célula  a  medida  que  se  acumula  sodio  intracelular  
(Na+).  Esta  pérdida  de  agua  extracelular  exacerba  el  déficit  de  volumen  intravascular.

5.  Enumerar  las  manifestaciones  clínicas  del  shock  hemorrágico.
  Frecuencia  cardíaca  (FC)  >100  latidos  por  minuto  y  presión  arterial  (PA)  <90  mm  Hg.
  Estado  mental  alterado  con  letargo  y  confusión.
  Disminución  de  la  producción  de  orina  <0,5  mililitros  por  kilogramo  por  hora  y  bajo  flujo  venoso  central.
presión  (PVC).

6.  ¿Cómo  se  puede  estimar  el  volumen  de  sangre  en  adultos  y  niños?
En  adultos  y  niños,  el  volumen  sanguíneo  medio  representa  el  7%  y  el  9%  del  peso  corporal  ideal,  
respectivamente.  Por  tanto,  en  adultos  multiplicar  el  peso  ideal  en  kg  por  un  7%  (70  ml/kg).  En  niños,  multiplicar  
el  peso  ideal  en  kg  por  9%  (90  ml/kg).

7.  Indique  la  primera  respuesta  fisiológica  a  la  hipovolemia.
El  paciente  intenta  compensar  la  disminución  del  volumen  sistólico  aumentando  la  FC  (taquicardia).

8.  ¿Cuáles  son  las  manifestaciones  cutáneas?
La  piel  se  vuelve  fría,  húmeda  y  pálida.  Las  venas  subcutáneas  colapsan  (lo  que  dificulta  el  inicio  de  una  línea  
intravenosa  [IV]).  El  llenado  capilar  se  retrasa  de  2  a  3  segundos.  Esto  resulta  del  intento  del  cuerpo  de  
redistribuir  el  suministro  de  oxígeno  a  los  órganos  vitales.

95
Machine Translated by Google
96  CAPÍTULO  16  SHOCK  HEMORRÁGICO  POSTRAUMÁTICO

9.  ¿Pueden  decirte  algo  las  venas  del  cuello?
La  ausencia  de  pulsaciones  o  el  colapso  de  las  venas  yugulares  externas  indican  una  presión  de  llenado  del  lado  
derecho  del  corazón  baja  (es  decir,  hipovolemia);  por  el  contrario,  las  venas  distendidas  indican  insuficiencia  cardíaca  o  
shock  cardiogénico,  incluido  el  taponamiento  pericárdico.

10.  ¿Es  el  hematocrito  una  guía  confiable  para  estimar  la  pérdida  de  sangre  aguda?
No.  Se  produce  una  disminución  del  hematocrito  con  el  relleno  del  espacio  intravascular  desde  el  espacio  
intersticial  o  durante  la  administración  de  líquido  de  reanimación  con  cristaloides  exógenos.
Sin  embargo,  este  proceso  no  es  inmediato  y  los  hematocritos  en  serie  son  más  útiles  para  evaluar  la  pérdida  de  
sangre.

11.  ¿Cuál  es  la  elección  adecuada  para  la  solución  intravenosa  durante
¿resucitación?

Ringer  lactato  o  solución  salina  normal.  Los  requerimientos  de  líquidos  cristaloides  isotónicos  en  el  shock  hemorrágico  
se  estiman  en  3  veces  la  pérdida  de  sangre  (regla  3:1).  El  reemplazo  de  volumen  inicial  debe  estar  dirigido  por  la  
respuesta  a  la  terapia  en  lugar  de  depender  de  la  pérdida  de  sangre  estimada.
No  agregue  dextrosa  a  los  líquidos  iniciales;  esto  exacerba  la  hiperglucemia  fisiológica  y  provoca  una  diuresis  osmótica.  
Se  agrega  dextrosa  al  5%  a  las  soluciones  IV  después  de  la  reanimación  inicial  por  su  efecto  ahorrador  de  proteínas  en  el  
paciente  traumatizado  en  ayunas.

12.  ¿Qué  es  el  déficit  de  base  y  cómo  es  útil  durante  la  reanimación?
El  déficit  de  bases  refleja  el  grado  de  acidosis  metabólica  en  sangre  y  se  utiliza  en  el  shock  hemorrágico  como  marcador  
indirecto  de  hipoxia  tisular.  Cuanto  peor  es  el  déficit  de  base  (cuanto  más  positivo  es),  peor  (menos  adecuada)  es  la  
perfusión  periférica  del  paciente.  El  déficit  de  bases  depende  del  hematocrito,  del  equilibrio  ácido­base  (pH)  y  de  la  presión  
parcial  de  dióxido  de  carbono  (pCO2);  si  corrige  la  pCO2  de  nuevo  a  40  mm  Hg,  el  pH  debería  ser  7,40.  Si  su  paciente  
todavía  está  acidótico,  tiene  un  déficit  de  bases.

13.  ¿Cuáles  son  las  clasificaciones  clínicas  del  shock  y  los  síntomas  clínicos  asociados?
manifestaciones?

Consulte  la  Tabla  16­1.  Estas  son  estimaciones  y  no  son  tan  precisas  ni  valiosas  como  determinar  la  respuesta  de  su  
paciente  a  la  terapia  o  la  reanimación.

TABLA  16­1.  C  LIN  ICALC  LA  SIFICACI  ÓN  SOF  SHO  CK
Clase Descripción Manifestaciones  clínicas

Clase  1 Pérdida  de  volumen  de  sangre  ¼  15% Taquicardia  leve,  dolor  de  cabeza  y  mareos  posturales


Se  puede  comparar  esto  con  un  
donante  de  sangre.

Clase  2 Pérdida  de  volumen  de  sangre  ¼  30% Taquicardia  moderada,  taquipnea  y  disminución  


de  la  presión  del  pulso

Clase  3 Pérdida  de  volumen  de  sangre  ¼  40% Taquicardia  marcada,  taquipnea,  disminución  del  


estado  mental,  hipotensión  y  disminución  de  la  diuresis

Clase  4 Pérdida  de  volumen  de  sangre  >  40% Taquicardia  marcada,  taquipnea  marcada,  


disminución  de  la  presión  arterial  sistólica,  
obnubilación  hasta  un  estado  mental  inconsciente  y  
ausencia  de  diuresis
Machine Translated by Google
CAPÍTULO  16  SHOCK  HEMORRÁGICO  POSTRAUMÁTICO  97

14.  ¿Cuáles  son  los  otros  tipos  de  shock  y  en  qué  se  diferencian  del  hemorrágico?
¿choque?
Además  del  shock  hemorrágico  o  hipovolémico,  existen  tipos  de  shock  neurogénico,  cardiogénico  y  séptico.  El  
shock  neurogénico  (esto  es  poco  común)  es  causado  por  la  pérdida  repentina  del  tono  vascular  autónomo,  lo  
que  resulta  en  vasodilatación.  La  PA  sistólica  es  baja,  la  presión  del  pulso  es  baja,  pero  la  piel  permanece  caliente.  El  
shock  cardiogénico  se  produce  por  falla  de  la  bomba  secundaria  a  daño  intrínseco  del  músculo  cardíaco  (infarto  de  
miocardio  [MI])  o  compresión  mecánica  (taponamiento  cardíaco).  En  este  contexto,  el  CO  es  bajo,  sin  embargo,  el  
volumen  intravascular  es  adecuado  y  se  refleja  en  un  aumento  de  la  CVP.  El  shock  séptico  (más  común  en  pacientes  
de  la  unidad  de  cuidados  intensivos  quirúrgicos  [UCI])  se  caracteriza  por  hipotensión  y  baja  resistencia  vascular  
sistémica  (RVS).  Es  importante  recordar  que  estas  categorías  de  shock  no  siempre  existen  de  forma  aislada.  Por  
ejemplo,  un  paciente  traumatizado  puede  tener  un  taponamiento  cardíaco  y  también  una  hemorragia  en  la  pelvis.

15.  ¿Cuándo  se  debe  iniciar  la  reanimación  con  líquidos  en  el  paciente  con  múltiples
traumas?

¡Inmediatamente!  Comience  la  terapia  (líquido  a  través  de  líneas  intravenosas  de  gran  calibre)  mientras  
realiza  el  examen  primario  dirigido  a  las  lesiones  que  amenazan  la  vida.  Es  inapropiado  esperar  hasta  que  el  paciente  
traumatizado  encaje  en  una  clasificación  fisiológica  precisa  de  shock  antes  de  iniciar  una  restauración  agresiva  del  volumen.

16.  ¿Cuáles  son  las  posibles  fuentes  de  pérdida  de  sangre  oculta  cuando  se  trata  de  determinar  el  estado  hemodinámico  
de  un  paciente?
Los  espacios  pleurales,  la  cavidad  abdominal,  el  espacio  retroperitoneal  o  pélvico  (fracturas  pélvicas),  las  
fracturas  mayores  de  huesos  largos  y  en  la  escena  externamente  ("en  la  acera").  Las  fracturas  de  fémur  pueden  
ocultar  más  de  1  L  de  sangre,  mientras  que  cada  fractura  de  costilla  puede  representar  150  ml.

17.  ¿Cómo  se  llama  el  paciente  que  se  vuelve  inestable  después  de  la  reanimación  inicial,
y  ¿por  qué  es  importante  reconocer  este  fenómeno?
Este  paciente  es  un  "respondedor  transitorio".  ¡Esto  indica  una  pérdida  de  sangre  continua!  La  mala  gestión  en  este  
entorno  puede  ser  fatal.

18.  ¿Cuándo  está  indicada  la  transfusión  de  sangre  durante  la  reanimación  inicial?
Si  llega  un  paciente  que  no  responde  a  la  infusión  de  cristaloides  agresiva  ("abierta  de  par  en  par"),  el  paciente  
debe  recibir  concentrados  de  glóbulos  rojos  O­negativos  sin  cruzar.  No  espere  sangre  específica  del  tipo  si  se  
requiere  una  infusión  inmediata;  el  banco  de  sangre  generalmente  no  está  usando  el  mismo  reloj  (no  están  tan  
asustados  porque  no  pueden  ver  al  paciente).

19.  ¿Cómo  conduce  el  shock  hemorrágico  a  una  falla  multiorgánica?
La  insuficiencia  orgánica  múltiple  (MOF,  por  sus  siglas  en  inglés)  es  un  síndrome  que  representa  una  vía  
fisiopatológica  complicada  y  dinámica  que  conduce  al  trastorno  funcional  de  los  órganos  y,  finalmente,  a  la  muerte.  El  
shock  hemorrágico  severo  inicia  una  cascada  inflamatoria  que  no  se  puede  revertir  en  algunos  pacientes  a  pesar  de  
la  reanimación  adecuada.  Se  cree  que  esta  vía  comienza  a  las  pocas  horas  de  la  lesión.
Durante  la  guerra  de  Vietnam,  los  pacientes  en  shock  hemorrágico  fueron  tratados  rápidamente,  pero  luego  
fallecieron  como  resultado  de  una  insuficiencia  pulmonar  o  síndrome  de  dificultad  respiratoria  del  adulto  (SDRA).  
Los  pacientes  con  ARDS  pueden  recibir  ventilación  mecánica,  pero  luego  mueren  por  una  combinación  de  
insuficiencia  hepática,  cardíaca  y  de  la  médula  ósea  o  MOF.  MOF  es  la  principal  causa  de  mortalidad  postlesión  tardía  
en  el  85%  de  estas  muertes.  Además  del  trastorno  celular  en  la  síntesis  de  ATP;  El  shock  provoca  la  liberación  de  
metabolitos  del  factor  activador  de  plaquetas,  interleucina­8  y  ácido  araquidónico  que  preparan  a  los  neutrófilos  para  
que  se  adhieran  a  las  células  endoteliales  y  liberen  mediadores  citotóxicos,  lo  que  produce  defectos  en  la  
endovasculatura,  inundando  el  espacio  intersticial  y  provocando  daños  en  los  órganos.
La  circulación  mesentérica  es  un  semillero  de  síntesis  de  mediadores  proinflamatorios  (el  intestino  es  el  "motor  de  
MOF")  y  parece  liberar  agentes  (probablemente  araquidonato  y  otros  lípidos  tóxicos)  en  la  linfa  mesentérica  que  
provoca  la  preparación  de  neutrófilos  sistémicos  y,  en  última  instancia,  lesión  pulmonar  aguda. .
Machine Translated by Google
98  CAPÍTULO  16  SHOCK  HEMORRÁGICO  POSTRAUMÁTICO

PUNTOS  CLAVE:  CLASIFICACIONES  DEL  CHOQUE
1.  Hemorrágico:  causa  más  frecuente  de  shock  postraumático;  bajas  presiones  de  llenado  y  CO,
saturación  venosa  mixta  de  oxígeno  (SVO2)  baja,  RVS  alta.

2.  Neurogénico:  poco  común;  RVS  baja  con  bradicardia;  la  piel  permanece  caliente.

3.  Cardiogénico:  falla  de  la  bomba  secundaria  a  daño  miocárdico  intrínseco  (MI)  o  mecánico
compresión  (taponamiento);  altas  presiones  de  llenado,  bajo  CO,  bajo  SVO2.

4.  Séptica:  más  frecuente  en  la  unidad  de  cuidados  intensivos  quirúrgicos  que  en  la  sala  de  traumatología;  CO  inicialmente  
alto,  SVR  bajo,  SVO2  alto.

BIBLIOGRAFÍA

1.  Ciesla  DJ,  Moore  EE,  Johnson  JL  et  al .:  Un  estudio  prospectivo  de  12  años  de  insuficiencia  orgánica  múltiple  posterior  a  una  lesión:  ha
algo  cambió?  Arch  Surg  140(5):432­438;  discusión  438­440,  2005.

2.  Durham  RM,  Moran  JJ,  Mazuski  JE  et  al.:  Falla  orgánica  múltiple  en  pacientes  con  trauma.  J  Trauma  55(4):608­616,
2003.

3.  Feliciano  DV,  Mattox  KL,  Moore  EE:  Trauma,  6.ª  ed.,  Nueva  York,  2008,  McGraw­Hill.

4.  Gonzalez  EA,  Moore  FA,  Holcomb  JB  et  al.:  El  plasma  fresco  congelado  debe  administrarse  antes  a  los  pacientes  que  requieren  una  
transfusión  masiva.  J  Trauma  62(1):112­119,  2007.

5.  Gutierrez  G,  Reines  HD,  Wulf­Gutierrez  ME:  Revisión  clínica:  shock  hemorrágico.  Crit  Care  8(5):373­381,  2004.

6.  Moore  FA,  Moore  EE,  American  College  of  Surgeons:  Cirugía  ACS:  principios  y  práctica  2006.  Sección  7  Inicial
Manejo  del  Trauma  que  Amenaza  la  Vida.  Nueva  York:  2006,  WebMD  Professional  Pub.

7.  Moore  FA,  McKinley  BA,  Moore  EE:  La  próxima  generación  en  reanimación  por  choque.  Lanceta  363  (9425):  1988­1996,
2004.
Machine Translated by Google

LESIÓN  CEREBRAL  TRAUMÁTICA
CAPÍTULO  
17

Brian  P.  Callahan,  MD  y  Craig  H.  Rabb,  MD

1.  ¿Es  la  lesión  cerebral  traumática  (TBI,  por  sus  siglas  en  inglés)  un  problema  común?
Sí.  En  los  Estados  Unidos,  1  de  cada  12  muertes  se  debe  a  lesiones.  Alrededor  del  40%  de  las  muertes  traumáticas  están  
asociadas  con  TBI.  De  las  muertes  resultantes  de  accidentes  automovilísticos,  el  60%  son  el  resultado  de  una  lesión  cerebral.  
Aún  más  común  es  la  LCT  menor,  que  representa  el  75%  de  las  admisiones  por  traumatismo  craneoencefálico.  Hay  más  de  
200.000  pacientes  hospitalizados  con  TBI  en  los  Estados  Unidos  por  año  y  más  de  1,7  millones  de  TBI  leves  que  requieren  la  
atención  de  un  médico.  Se  estima  que  de  2  a  6  millones  de  personas  en  los  Estados  Unidos  viven  con  discapacidades  
asociadas  a  TBI.

2.  ¿Qué  es  una  conmoción  cerebral?

La  definición  de  conmoción  cerebral  o  TBI  leve  según  los  Centros  para  el  Control  y  la  Prevención  de  Enfermedades  (CDC,  
por  sus  siglas  en  inglés)  es  un  proceso  fisiopatológico  complejo  secundario  a  un  trauma  que  resulta  en  una  constelación  de  
síntomas  físicos,  cognitivos,  emocionales  o  relacionados  con  el  sueño  que  pueden  o  no  implicar  pérdida  de  conciencia  (LOC).  
Los  síntomas  incluyen  dolor  de  cabeza,  mareos,  amnesia  y  vómitos.  Hay  alrededor  de  128/100,000  conmociones  cerebrales  
en  la  población  de  los  Estados  Unidos  por  año.
En  pacientes  pediátricos,  el  deporte  es  la  causa  más  común,  mientras  que  las  caídas  y  los  accidentes  automovilísticos  
son  la  causa  más  común  en  adultos.  La  puntuación  de  la  escala  de  coma  de  Glasgow  (GCS)  se  utiliza  para  categorizar  las  
lesiones  cerebrales  de  la  siguiente  manera:  leve,  13  a  14;  moderado,  9  a  12;  y  grave,  8.

3.  ¿Cómo  se  obtiene  la  puntuación  GCS?

La  GCS  es  un  medio  para  identificar  cambios  en  el  estado  neurológico.  Sus  principales  puntos  fuertes  son  la  facilidad  
de  uso  y  la  reproducibilidad  entre  los  observadores.  Es  una  escala  de  15  puntos;  15  es  la  mejor  puntuación  y  3  es  la  peor.  
La  puntuación  se  deriva  de  la  suma  de  los  tres  componentes  individuales:  mejor  respuesta  reveladora  (1  a  4  puntos),  mejor  
respuesta  verbal  (1  a  5  puntos)  y  mejor  respuesta  motora  (1  a  6  puntos).  La  GCS  es  insensible  a  la  respuesta  pupilar  y  la  
focalidad.

4.  ¿Cuándo  se  debe  consultar  a  un  neurocirujano?
Para  pacientes  con  LOC  y  anomalía  neurológica  subsiguiente  o  anomalía  en  la  tomografía  computarizada  (TC).  Estos  
pacientes  a  menudo  tienen  una  puntuación  GCS  igual  a  13.

5.  ¿Cómo  evalúa  inicialmente  al  paciente  con  una  lesión  cerebral?
Como  cualquier  paciente  traumatizado.  Los  primeros  pasos  son  la  evaluación  del  ABC  (vías  respiratorias,  respiración  y  
circulación)  y  la  reanimación  fisiológica  rápida.  El  examen  neurológico  es  crucial.
El  examen  inicial  incluye  (1)  evaluación  GCS;  (2)  evaluación  de  los  reflejos  de  los  nervios  craneales,  incluidos  el  tamaño  y  la  
reactividad  de  la  pupila,  el  reflejo  oculocefálico  (ojos  de  muñeca),  el  reflejo  corneal  y  el  reflejo  nauseoso;  (3)  y  examen  motor.  
La  repetición  del  examen  neurológico  también  es  crucial.
Finalmente,  evalúe  las  lesiones  de  la  columna  cervical  concurrentes.

6.  Lo  que  tiene  prioridad  en  un  paciente  hipotenso  también  con  un  traumatismo  craneoencefálico
¿lesión?
La  hipotensión  en  pacientes  con  traumatismo  craneoencefálico  suele  acompañar  a  otras  lesiones.  No  asuma  que  la  
hipotensión  es  el  resultado  de  la  lesión  cerebral  únicamente.  Un  solo  episodio  de  hipotensión  duplica  la  tasa  de  mortalidad.  
Además,  la  hipoxemia,  definida  como  una  PaO2  inferior  a  60  o  una  saturación  de  O2  inferior  al  90%,  aumenta  significativamente  
la  mortalidad  en  el  TCE.

99
Machine Translated by Google
100  CAPÍTULO  17  LESIÓN  CEREBRAL  TRAUMÁTICA

7.  ¿Cuál  es  el  significado  de  la  anisocoria  en  un  paciente  con  un  nivel  de  conciencia  disminuido?

La  anisocoria  (pupilas  desiguales)  es  una  verdadera  urgencia  neurológica.  Por  lo  general,  una  lesión  masiva  
(p.  ej.,  hematoma  subdural  o  epidural,  contusión  o  tumefacción  difusa  de  un  hemisferio)  conduce  a  una  hernia  uncal  y  
estiramiento  del  tercer  nervio  ipsilateral.  El  tiempo  es  crucial.  Administre  manitol,  obtenga  una  tomografía  computarizada  
y  proceda  con  la  descompresión  quirúrgica  (si  es  posible).

8.  ¿Qué  pasa  si  la  pupila  más  grande  es  reactiva?
Si  la  pupila  más  grande  es  reactiva,  el  tercer  par  craneal  está  funcionando.  Piense  en  el  síndrome  de  Horner  
(miosis,  ptosis  y  anhidrosis)  del  otro  lado.  Este  síndrome  puede  ser  el  resultado  de  una  lesión  en  los  nervios  simpáticos  
que  viajan  con  la  arteria  carótida  en  el  cuello.  Considere  la  evaluación  (angiografía)  para  una  disección  carotídea.

9.  ¿Es  inexacto  el  término  "semicomatoso"?

Sí.  Los  pacientes  están  alerta,  letárgicos  (la  excitación  se  mantiene  mediante  la  interacción  verbal),  obnubilados  
(estimulación  mecánica  constante  para  mantener  la  excitación)  o  comatosos  (ni  la  estimulación  verbal  ni  la  mecánica  
provocan  la  excitación).  El  cambio  en  el  nivel  de  conciencia  suele  ser  el  primer  signo  de  aumento  de  la  presión  intracraneal  
(PIC);  también  es  la  parte  peor  documentada  del  examen  neurológico.  ¡Documente  todos  los  hallazgos!

10.  ¿Cómo  se  prueba  la  respuesta  motora?
Determinar  la  capacidad  de  seguir  órdenes  pidiéndole  al  paciente  que  levante  los  dedos  y  mueva  los  brazos  y  las  piernas.  
Si  el  paciente  no  sigue  las  órdenes,  pruebe  la  respuesta  al  estímulo  central  doloroso.  La  localización  del  estímulo  doloroso  
se  confirma  cuando  la  mano  del  paciente  se  estira  hacia  un  roce  esternal.  El  paciente  puede  estar  en  mayor  peligro  si  en  
respuesta  al  dolor  exhibe  una  postura  flexora  (decorticación),  una  postura  extensora  (descerebración)  o  no  responde.  La  
postura  flexora  indica  una  lesión  del  tronco  encefálico  alto,  y  la  postura  extensora  se  asocia  con  una  disfunción  del  tronco  
encefálico  bajo.

11.  ¿Cuál  es  el  significado  de  la  equimosis  periorbitaria  (ojos  de  mapache)  y  la  equimosis  sobre  la  mastoides  
(signo  de  Battle)?
En  ausencia  de  traumatismo  directo  en  los  ojos  o  las  regiones  mastoideas,  la  equimosis  periorbitaria  y  la  equimosis  
sobre  la  mastoides  son  signos  confiables  de  fracturas  de  la  base  del  cráneo.  De  los  pacientes  con  fracturas  de  la  base  
del  cráneo,  el  10%  tiene  fugas  de  líquido  cefalorraquídeo  (LCR),  incluidas  rinorrea  u  otorrea.
Las  fugas  persistentes  de  LCR  se  relacionan  con  meningitis;  sin  embargo,  los  antibióticos  profilácticos  no  disminuyen  
el  riesgo  de  meningitis.

12.  ¿Deberían  explorarse  las  laceraciones  del  cuero  cabelludo  en  el  servicio  de  urgencias?
Usualmente  no.  Primero  se  debe  realizar  una  tomografía  computarizada  para  buscar  patología  intracraneal  o  fractura  
de  cráneo.  Si  se  observa  patología  quirúrgica  en  la  TC,  la  laceración  se  cerrará  en  el  quirófano  (OR).  De  lo  contrario,  la  
laceración  se  puede  lavar  y  cerrar  en  el  departamento  de  emergencias  (ED).  Si  no  se  puede  controlar  el  sangrado  antes  
de  la  TC,  se  debe  cerrar  la  laceración  con  grapas  para  detener  la  pérdida  de  sangre.

13.  ¿Qué  pacientes  necesitan  tomografías  computarizadas  de  la  cabeza?
La  tomografía  computarizada  se  usa  en  parte  como  una  herramienta  de  clasificación  con  lesiones  cerebrales  menores  
y  puede  ser  rentable  en  comparación  con  la  admisión  a  la  unidad  de  cuidados  intensivos  (UCI)  para  observación.
Por  el  contrario,  los  pacientes  con  focalidad  en  el  examen  no  proceden  al  quirófano  sin  una  tomografía  
computarizada.  Los  pacientes  que  definitivamente  necesitan  una  tomografía  computarizada  después  de  una  TBI  
leve  son  aquellos  menores  de  16  años  o  mayores  de  65  años,  que  están  intoxicados,  no  son  confiables,  toman  
anticoagulantes,  tienen  amnesia  persistente  u  otros  síntomas  neurológicos,  signos  de  una  fractura  basilar  del  
cráneo  o  examen  neurológico  anormal. .
Machine Translated by Google
CAPÍTULO  17  LESIÓN  CEREBRAL  TRAUMÁTICA  101

14.  ¿Cuáles  son  las  lesiones  quirúrgicas  traumáticas  comunes?
Si  los  ventrículos  son  grandes  (ventriculomegalia),  una  ventriculostomía  puede  drenar  el  exceso  de  LCR  si  la  PIC  está  
elevada.  Los  hematomas  epidurales  (por  hemorragia  arterial),  los  hematomas  subdurales  (por  hemorragia  venosa)  y  los  
hematomas  intraparenquimatosos  con  efecto  de  masa  significativo  deben  evacuarse  quirúrgicamente.  Una  fractura  de  
cráneo  deprimida  o  un  cuerpo  extraño  (p.  ej.,  una  bala)  puede  requerir  un  viaje  al  quirófano  en  ciertas  situaciones  clínicas.

15.  ¿Cuándo  está  indicada  la  monitorización  de  la  presión  intracraneal?
La  PIC  debe  monitorearse  en  todos  los  pacientes  rescatables  con  un  TBI  grave  (GCS  de  3  a  8  después  de  la  
reanimación)  y  una  TC  anormal  (definida  como  hematomas,  contusiones,  hinchazón,  hernia  o  cisternas  comprimidas).  La  
monitorización  de  la  PIC  también  está  indicada  en  pacientes  con  TBI  grave  y  TC  normal  si  se  observan  2  o  más  de  los  
siguientes:  edad  >  40  años,  postura  o  presión  arterial  sistólica  (PAS)  <  90  mm  Hg.

16.  Describa  el  tratamiento  inicial  de  los  pacientes  con  sospecha  de  aumento  de  la  PIC.
El  cerebro,  al  igual  que  cualquier  otro  órgano,  debe  tener  un  flujo  sanguíneo  y  un  suministro  de  oxígeno  adecuados.
El  ABC  es  lo  primero.  Se  debe  establecer  la  vía  aérea  y,  si  es  necesario,  se  debe  intubar  al  paciente.  Mantenga  la  
presión  arterial  sistólica  >  100  mm  Hg  y  evite  la  hipoxia.  La  cabecera  de  la  cama  debe  estar  elevada  y  se  deben  seguir  
las  precauciones  de  la  columna  cervical.  Se  debe  administrar  manitol  si  el  paciente  tiene  signos  de  hernia  inminente,  
como  anisocoria,  o  signos  neurológicos  focales  en  el  examen,  como  posturas.

17.  ¿Se  debe  hiperventilar  a  todos  los  pacientes  con  PIC  elevada?
La  disminución  de  la  presión  parcial  de  dióxido  de  carbono  (pCO2)  es  el  tratamiento  eficaz  más  rápido  para  
la  PIC  elevada.  El  objetivo  suele  ser  una  pCO2  de  30  a  35  mm  Hg.  Cualquier  paciente  con  un  nivel  de  conciencia  
deprimido  e  incapacidad  para  proteger  las  vías  respiratorias  debe  ser  intubado.  Antes  de  obtener  una  tomografía  
computarizada,  los  pacientes  que  se  cree  que  tienen  una  lesión  de  masa  según  el  examen  neurológico  deben  
hiperventilarse  hasta  que  se  logre  el  tratamiento  definitivo.  Evite  la  hiperventilación  crónica,  que  puede  causar  lesión  
cerebral  isquémica  como  resultado  de  la  vasoconstricción  de  los  vasos  sanguíneos  cerebrales  y  la  disminución  del  flujo  
sanguíneo  cerebral  (FSC).  Debido  a  este  efecto  sobre  el  CBF,  la  hiperventilación  solo  debe  usarse  como  una  medida  
temporal.

18.  En  pacientes  hemodinámicamente  estables,  ¿cómo  se  disminuye  la  PIC?
Manitol,  1  g/kg,  como  bolo  intravenoso  (IV).  La  evidencia  más  reciente  también  sugiere  que  la  solución  salina  hipertónica  
puede  disminuir  la  PIC  y  mantener  la  estabilidad  hemodinámica.  La  solución  salina  hipertónica  se  puede  administrar  como  
una  infusión  continua  con  un  objetivo  de  sodio  sérico  o  como  inyecciones  en  bolo  para  protuberancias  en  la  PIC.
La  dosis  en  bolo  varía  de  100  ml  de  solución  salina  normal  al  3%  a  10  ml  de  solución  salina  normal  al  23,4%.  Asegúrese  

también  de  que  el  collarín  cervical  no  obstruya  el  flujo  venoso  a  través  del  sistema  yugular.

19.  ¿Cuál  es  el  punto  final  del  tratamiento  con  diuréticos?
El  sodio  sérico  de  150  mEq/L  y  la  osmolalidad  sérica  de  320  mOsm  suelen  ser  los  límites  superiores  de  la  diuresis.  
Anticipe  la  hipovolemia  intravascular  y  trátela  en  consecuencia.  El  reciente  ensayo  SAFE  (Saline  vs  Albumin  Fluid  
Evaluation)  que  compara  la  albúmina  con  la  solución  salina  en  la  reanimación  de  pacientes  con  TBI  ha  demostrado  que  
la  reanimación  con  albúmina  aumenta  la  mortalidad  y  es  más  costosa  que  los  cristaloides.  Por  tanto,  los  cristaloides  
deben  utilizarse  para  reponer  el  volumen  perdido  secundario  a  la  diuresis  en  el  TCE.

20.  ¿Cuál  es  el  significado  de  la  presión  de  perfusión  cerebral  (PPC)?
La  PPC  es  la  diferencia  entre  la  presión  arterial  media  (PAM)  y  la  PIC:

PPC  ¼  PMA  PIC

El  PPC  es  importante.  El  resultado  neurológico  es  mejor  en  pacientes  con  CPP  en  los  años  60  y  debe  evitarse  CPP  
<50.  Algunos  pacientes  requieren  tratamiento  con  vasopresores  y  fluidos  para  mantener  la  CPP,  sin  embargo,  se  debe  
evitar  mantener  agresivamente  la  CPP  >70  debido  al  mayor  riesgo  de  síndrome  de  dificultad  respiratoria  del  adulto  (SDRA).
Machine Translated by Google
102  CAPÍTULO  17  LESIÓN  CEREBRAL  TRAUMÁTICA

21.  ¿Por  qué  se  debe  desvestir  y  examinar  minuciosamente  a  todos  los  niños  con  TBI?
La  mitad  de  los  niños  que  sufren  traumatismos  no  accidentales  (abuso  infantil)  tienen  TBI.  Un  examen  completo  
puede  revelar  lesiones  adicionales.

22.  ¿Deben  tratarse  profilácticamente  las  convulsiones  postraumáticas?
Los  pacientes  con  anomalías  del  parénquima  cerebral  en  la  tomografía  computarizada  después  de  una  lesión  en  la  cabeza  
pueden  beneficiarse  de  una  semana  de  profilaxis  anticonvulsiva.  Las  convulsiones  tempranas  pueden  aumentar  la  demanda  
metabólica  del  cerebro  lesionado  y  afectar  negativamente  a  la  PIC.  De  los  pacientes  que  tienen  convulsiones  dentro  de  los  
primeros  7  días  de  la  lesión,  el  10%  también  tiene  convulsiones  tardías.  Sin  embargo,  la  prevención  de  las  convulsiones  
tempranas  no  reduce  la  incidencia  de  las  convulsiones  tardías.  Los  pacientes  con  mayor  riesgo  de  convulsiones  son  aquellos  
con  GCS  <10,  contusiones,  fracturas  de  cráneo  deprimidas,  hematomas  cerebrales  o  lesiones  penetrantes.

23.  ¿Qué  coagulopatía  se  asocia  con  una  lesión  cerebral  grave?
Coagulación  intravascular  diseminada.  El  presunto  mecanismo  es  la  liberación  masiva  de  tromboplastina  
desde  el  cerebro  lesionado  hacia  la  circulación.  Los  niveles  séricos  de  productos  de  degradación  de  fibrina  se  
correlacionan  aproximadamente  con  la  extensión  de  la  lesión  del  parénquima  cerebral.  Todos  los  pacientes  con  
lesión  cerebral  grave  deben  evaluarse  con  tiempo  de  protrombina,  tiempo  de  tromboplastina  parcial,  recuento  de  
plaquetas  y  niveles  de  fibrinógeno.

24.  ¿Qué  otras  complicaciones  médicas  pueden  resultar  de  una  lesión  grave  en  la  cabeza?
La  diabetes  insípida  (DI)  secundaria  a  la  secreción  inadecuada  de  hormona  antidiurética  es  causada  por  una  lesión  
en  las  vías  hipofisaria  o  hipotalámica.  El  riñón  es  incapaz  de  disminuir  la  pérdida  de  agua  libre.  Por  lo  general,  la  
diuresis  es  >200  ml/h  y  la  gravedad  específica  de  la  orina  es  <1,003.  El  sodio  sérico  puede  aumentar  
precipitadamente  si  la  DI  no  se  trata  con  prontitud.  El  tratamiento  de  elección  en  traumatismos  es  la  infusión  IV  de  
vasopresina  sintética  (Pitressin),  que  tiene  una  semivida  de  20  minutos  y  puede  titularse  para  producir  la  diuresis  
adecuada.  Debido  a  que  la  mayoría  de  las  DI  inducidas  por  traumatismos  son  autolimitadas,  la  1­desamino­8­D­
arginina  vasopresina  (DDAVP)  a  largo  plazo,  que  tiene  una  vida  media  de  12  horas,  por  lo  general  no  es  necesaria.

25.  Si  un  paciente  está  despierto  con  síntomas  neurológicos  significativos  pero  sin  anomalías  en
Tomografía  computarizada,  ¿cuáles  son  las  posibles  explicaciones?
Una  lesión  de  la  médula  espinal  o  una  disección  de  la  arteria  carótida  o  vertebral.

26.  ¿Las  heridas  de  bala  que  cruzan  la  línea  media  del  cerebro  son  uniformemente  mortales?
No.  El  trayecto  que  toma  la  bala  es  importante,  pero  también  lo  es  la  energía  que  imparte  al  cerebro.

27.  ¿Cuál  es  el  significado  de  la  conmoción  cerebral?
En  la  mayoría  de  los  estudios  de  TBI  menor,  >50%  de  los  pacientes  se  quejan  de  dolor  de  cabeza,  fatiga,  mareos,  
irritabilidad  y  alteraciones  de  la  cognición  y  la  memoria  a  corto  plazo.  Esta  constelación  de  síntomas  se  ha  
denominado  síndrome  posconmocional.  Es  importante  alertar  al  paciente  sobre  la  probabilidad  de  desarrollar  estos  
síntomas.  Los  problemas  neuroconductuales  afectan  significativamente  la  vida  de  los  pacientes.  Los  síntomas  
generalmente  se  resuelven  dentro  de  los  3  a  6  meses  posteriores  a  la  lesión.

28.  ¿Se  puede  dar  de  alta  del  servicio  de  urgencias  a  los  pacientes  con  TCE  leves?
Los  pacientes  cuyo  examen  (incluida  la  memoria  a  corto  plazo)  vuelve  a  la  normalidad  y  que  tienen  una  tomografía  
computarizada  de  la  cabeza  normal  pueden  ser  dados  de  alta  a  su  hogar  si  están  acompañados  por  una  persona  
responsable  y  se  les  dan  instrucciones  escritas  para  regresar  al  hospital  si  el  dolor  de  cabeza  continúa  empeorando,  
aumentando  aparecen  vómitos,  debilidad,  somnolencia  o  pérdida  de  LCR.

29.  ¿La  lesión  cerebral  es  permanente?  ¿El  resultado  es  siempre  pobre?
No  y  no.  La  lesión  cerebral  ocurre  en  dos  fases.  La  lesión  primaria  se  produce  en  el  momento  del  impacto.  La  
lesión  secundaria  es  prevenible  y  tratable.  Los  ejemplos  incluyen  hipoxia,  hipotensión,  PIC  elevada  y  disminución  
de  la  perfusión  al  cerebro  secundaria  a  isquemia,  inflamación  cerebral,
Machine Translated by Google
CAPÍTULO  17  LESIÓN  CEREBRAL  TRAUMÁTICA  103

y  lesiones  masivas  en  expansión.  El  manejo  quirúrgico  rápido  y  la  prevención  de  lesiones  secundarias  
mejoran  el  resultado.  Aunque  anteriormente  se  creía  que  el  cerebro  no  era  capaz  de  repararse,  ahora  está  claro  
que  la  reparación  y  la  reorganización  neuronal  ocurren  después  de  una  lesión.

30.  ¿Cuál  es  el  umbral  para  tratar  el  aumento  de  la  PIC?
La  mayoría  de  los  estudios  coinciden  en  que  el  umbral  para  el  tratamiento  de  la  PIC  debe  ser  de  20  a  25  mm  Hg.

31.  ¿Se  deben  administrar  dosis  altas  de  esteroides  a  los  pacientes  con  TBI  para  tratar  el  aumento  de  la  PIC?
No.  Existe  evidencia  (estudio  CRASH  [Corticosteroid  Randomization  After  Significant  Head  Injury])  que  muestra  
que  las  dosis  altas  de  esteroides  en  TBI  están  asociadas  con  una  mayor  morbilidad  y  mortalidad.

32.  ¿Los  pacientes  con  lesión  cerebral  traumática  tienen  riesgo  de  trombosis  venosa  profunda?
y  embolia  pulmonar?
Sí.  El  riesgo  de  trombosis  venosa  profunda  (TVP)  y  embolia  pulmonar  (EP)  en  pacientes  con  TBI  puede  
llegar  al  30%.  Se  ha  demostrado  que  los  SCD  reducen  la  tasa  de  DVT/PE  y  deben  usarse  en  todos  los  pacientes  
con  TBI  a  menos  que  una  lesión  en  las  extremidades  inferiores  impida  su  uso.
También  se  ha  demostrado  que  la  heparina  de  bajo  peso  molecular  (HBPM)  disminuye  el  riesgo,  pero  también  
puede  aumentar  el  riesgo  de  hemorragia  cerebral.

PUNTOS  CLAVE

1.  Se  debe  evitar  la  hipotensión  y  la  hipoxia  en  la  TBI.

2.  Piense  en  la  disección  de  la  arteria  carótida  o  vertebral  en  pacientes  traumatizados  con  síntomas  neurológicos
pero  una  tomografía  computarizada  normal.

3.  CPP  ¼  PAM  PIC.  Trate  de  mantener  la  PPC  entre  50  y  60  mm  Hg  en  TCE  grave,  especialmente  en  pacientes  
con  problemas  continuos  de  PIC.  No  sobretratar  (CPP  >70),  ya  que  esto  aumenta  el  riesgo  de  ARDS.

4.  No  use  esteroides  en  dosis  altas  en  TBI.

SITIOS  WEB

www.acssurgery.com/abstracts/acs/acs0501.htm

www.surgery.ucsf.edu/eastbaytrauma/Protocols/ER%20protocol%20pages/closedheadinjury.htm
www.emedicine.com/pmr/topic212.htm

www.cdc.gov/ncipc/tbi/mtbi/report.htm

BIBLIOGRAFÍA

1.  Fundación  de  Trauma  Cerebral;  Asociación  Estadounidense  de  Cirujanos  Neurológicos;  Congreso  de  Cirujanos  Neurológicos;  
Sección  Conjunta  de  Neurotrauma  y  Cuidados  Críticos,  AANS/CNS:  Directrices  para  el  manejo  del  traumatismo  
craneoencefálico  grave.  J  Neurotrauma  24  Suplemento  1:  S1­S106,  2007.
2.  Brain  Trauma  Foundation:  Manejo  y  pronóstico  de  lesiones  cerebrales  traumáticas  graves,  Nueva  York,  2000,  Brain  
Trauma  Foundation.  Disponible  en  www.braintrauma.org.
Machine Translated by Google
104  CAPÍTULO  17  LESIÓN  CEREBRAL  TRAUMÁTICA

3.  Carson  J,  Tator  C,  Johnston  K  et  al .:  Nuevas  pautas  para  el  manejo  de  conmociones  cerebrales.  Médico  Can  Fam
52:756­757,  2006.

4.  Marion  DW:  Pautas  basadas  en  evidencia  para  lesiones  cerebrales  traumáticas.  Prog  Neurol  Surg  19:171­196,  2006.

5.  Mazzola  CA,  Adelman  PD:  Manejo  de  cuidados  críticos  del  traumatismo  craneoencefálico  en  niños.  Crit  Care  Med  30:S393­S401,
2002.

6.  Narayan  RK,  Michel  ME,  Ansell  B  et  al .:  Ensayos  clínicos  en  lesiones  en  la  cabeza.  J  Neurotrauma  19:503­557,  2002.

7.  Ogden  AT,  Mayer  SA,  Connolly  ES  Jr:  Agentes  hiperosmolares  en  la  práctica  neuroquirúrgica:  el  papel  en  evolución  de
solución  salina  hipertónica.  Neurocirugía  57(2):207­215,  2005.

8.  Ropper  AH,  Gorson  KC:  Práctica  clínica.  Concusión.  N  Engl  J  Med  356(2):166­172,  2007.

9.  Investigadores  del  estudio  SAFE;  Grupo  de  Ensayos  Clínicos  de  la  Sociedad  de  Cuidados  Intensivos  de  Australia  y  Nueva  Zelanda;
Servicio  de  Sangre  de  la  Cruz  Roja  Australiana  et  al .:  solución  salina  o  albúmina  para  la  reanimación  con  líquidos  en  pacientes  con  lesión  
cerebral  traumática.  N  Engl  J  Med  357(9):874­884,  2007.

10.  Shaw  NA:  La  neurofisiología  de  la  conmoción  cerebral.  Prog  Neurobiol  67:281­344,  2002.
Machine Translated by Google

LESIONES  DE  LA  MÉDULA  ESPINAL
CAPÍTULO  
18

Brian  P.  Callahan,  MD  y  Craig  H.  Rabb,  MD

1.  ¿Cuál  es  la  diferencia  entre  una  lesión  en  la  columna  y  una  lesión  en  la  médula  espinal?
Las  lesiones  de  la  columna  incluyen  daño  al  hueso,  disco  o  ligamentos.  Estas  lesiones  a  veces  resultan  en  
inestabilidad  de  la  columna.  También  pueden  estar  asociados  con  una  lesión  de  la  médula  espinal  (LME),  que  es  un  daño  
al  tejido  neural,  a  menudo  con  déficit  clínico.  Es  crucial  determinar  si  hay  (1)  una  lesión  en  la  columna,  (2)  una  LME  o  (3)  
inestabilidad  en  la  columna.

2.  Describa  la  evaluación  de  un  paciente  con  sospecha  de  lesión  en  la  columna.
Primero,  asegúrese  de  que  el  paciente  esté  adecuadamente  inmovilizado  y  que  todos  sepan  mantener  las  precauciones  
espinales.  En  segundo  lugar,  inspeccione  y  palpe  la  columna  en  busca  de  traumatismos  externos  y  escalones.  Finalmente,  
realice  un  examen  neurológico  completo  que  incluya  las  cuatro  extremidades.  Evalúe  la  fuerza,  la  sensación  (tacto  ligero/
propiocepción  y  dolor/temperatura),  el  tono  muscular,  los  reflejos  y  el  tono  rectal.
Documente  cuidadosamente  sus  resultados.

3.  ¿Cómo  minimiza  el  riesgo  de  lesiones  adicionales  en  la  columna  en  el  hospital?
Los  pacientes  traumatizados  deben  estar  protegidos  con  un  collarín  cervical  rígido.  La  columna  torácica  y  lumbar  se  
protegen  mediante  la  inmovilización  inicial  con  tabla  espinal.  Se  debe  girar  al  paciente  durante  la  evaluación  inicial,  luego  
retirarlo  de  la  tabla  y  trasladarlo  a  una  cama  de  hospital  adecuada  para  prevenir  las  úlceras  por  decúbito.  Se  deben  
mantener  las  precauciones  de  la  columna  hasta  que  la  columna  se  "limpie",  lo  que  significa  que  no  hay  inestabilidad  de  la  
columna  o  se  ha  identificado  la  inestabilidad.

4.  ¿Cómo  se  define  el  nivel  de  la  lesión  medular?
El  nivel  no  se  refiere  al  nivel  de  la  lesión  de  la  columna  vertebral  (vértebras,  discos  y  ligamentos)  sino  al  nivel  más  
caudal  de  la  médula  con  función  intacta.  Si  un  paciente  tiene  una  función  normal  de  los  deltoides  (C5)  y  poca  o  ninguna  
función  de  los  bíceps  (C6)  o  inferior,  el  paciente  tiene  una  lesión  a  nivel  motor  C5.  Los  lados  derecho  e  izquierdo  deben  
documentarse  por  separado.

5.  ¿Qué  tipo  de  lesión  se  asocia  comúnmente  con  la  lesión  de  la  columna  cervical?
Lesión  craneal.  Las  fuerzas  asociadas  con  una  lesión  importante  en  la  cabeza  y  el  cerebro  pueden  transmitirse  a  través  de  
la  columna  cervical.  De  los  pacientes  con  lesiones  de  la  médula  espinal,  el  50%  tiene  lesiones  en  la  cabeza  asociadas.
Aproximadamente  el  15%  de  los  pacientes  con  una  lesión  en  la  columna  tienen  una  segunda  lesión  en  otra  parte  de  la  columna.

6.  ¿Cómo  se  puede  evaluar  la  médula  espinal  en  pacientes  con  traumatismo  craneoencefálico  asociado?
Todos  los  pacientes  deben  tener  un  examen  rectal  para  evaluar  el  tono.  Un  ano  patuloso  es  una  buena  indicación  
de  lesión  de  la  médula  espinal  o  de  la  cauda  equina.  El  tono  motor  flácido  y  los  reflejos  ausentes  deben  despertar  la  
sospecha  de  SCI.  Estos  hallazgos  son  extremadamente  inusuales  con  una  lesión  cerebral  aislada.  El  priapismo  es  común  
con  SCI  pero  no  es  causado  por  una  lesión  en  la  cabeza.  Las  imágenes  radiográficas  deben  usarse  generosamente  
cuando  se  sospecha  una  LME.

7.  ¿Qué  otra  lesión  significativa  puede  simular,  en  la  presentación,  una  médula  torácica  alta?
¿lesión?
La  disección  de  la  aorta  torácica  puede  presentarse  como  una  lesión  de  la  médula  en  la  región  T4.  T4  es  una  zona  divisoria  
de  aguas  en  la  médula  entre  la  distribución  arterial  vertebral  y  las  arterias  radiculares  aórticas.

105
Machine Translated by Google
106  CAPÍTULO  18  LESIONES  DE  LA  MÉDULA  ESPINAL

8.  ¿Qué  es  el  shock  espinal?
La  ausencia  total  de  función  de  la  médula  espinal  por  debajo  del  nivel  de  la  lesión  da  como  resultado  un  tono  
motor  flácido  y  arreflexia.  El  shock  neurogénico,  por  el  contrario,  se  refiere  a  la  hipotensión  que  puede  resultar  de  lesiones  
completas  de  la  médula  espinal  cervical  o  torácica  superior.  La  hipotensión  es  el  resultado  de  la  falta  de  inervación  
vasomotora  simpática  por  debajo  de  la  lesión  y  se  caracteriza  por  bradicardia  por  aferencias  vagales  desequilibradas  al  
corazón.  Se  debe  usar  reanimación  con  líquidos  y  vasopresores  para  mantener  la  presión  arterial  sistólica  (PAS)  >  90  mm  
Hg.  La  atropina  puede  ser  necesaria  para  tratar  la  bradicardia.

9.  Describir  una  evaluación  radiológica  adecuada.
Los  pacientes  despiertos,  alertas  y  confiables  sin  dolor  o  sensibilidad  en  el  cuello  no  requieren  imaginación.
Se  recomienda  una  serie  de  tres  vistas  de  la  columna  cervical  (vistas  anteroposterior  [AP],  lateral  y  odontoidea)  
para  la  evaluación  radiográfica  de  la  columna  cervical  en  pacientes  sintomáticos  después  de  una  lesión  traumática.  Debe  
visualizarse  la  relación  entre  C7  y  la  parte  superior  de  T1.  Esto  debe  complementarse  con  una  tomografía  computarizada  
(TC)  para  definir  mejor  las  áreas  sospechosas  o  que  no  se  visualizan  bien  en  las  radiografías  simples  de  cuello  uterino.  
La  inmovilización  de  la  columna  cervical  en  pacientes  despiertos  con  dolor  o  sensibilidad  en  el  cuello  y  radiografías  
normales  de  la  columna  cervical  (incluida  la  TC  complementaria,  según  sea  necesario)  puede  interrumpirse  después  de  
(a)  una  flexión  dinámica  normal  y  adecuada  (con  una  excursión  de  al  menos  30  grados  en  cada  dirección). /radiografías  
de  extensión,  o  (b)  se  obtiene  un  estudio  normal  de  imágenes  por  resonancia  magnética  (IRM)  dentro  de  las  48  horas  
posteriores  a  la  lesión  o  (c)  a  discreción  del  médico  tratante  de  la  columna  vertebral.  Los  pacientes  obnubilados  con  
radiografías  normales  de  la  columna  cervical  deben  someterse  a  una  TC  de  alta  calidad  con  imágenes  reconstruidas  
coronales  y  sagitales.  Esto  puede  complementarse  con  resonancia  magnética.

Para  la  columna  torácica  y  lumbosacra,  se  obtienen  vistas  anteroposterior  y  lateral.
Los  pacientes  con  evidencia  de  posibles  fracturas  en  radiografías  simples  deben  someterse  a  tomografías  computarizadas  para  
definir  la  lesión  con  mayor  detalle.  La  resonancia  magnética  también  es  útil  para  buscar  discos  herniados  y  lesiones  de  ligamentos.
Actualmente,  estas  áreas  se  pueden  visualizar  satisfactoriamente  mediante  imágenes  reconstruidas  a  partir  de  imágenes  
corporales  realizadas  durante  el  estudio  del  trauma  (TC  de  tórax,  abdomen  y  pelvis).

10.  Describa  la  forma  correcta  de  leer  una  radiografía  lateral  de  la  columna  cervical.
Acostúmbrate  a  hacer  una  revisión  sistemática  minuciosa  de  la  misma  manera  con  cada  película.  Primero  observe  el  
espacio  de  tejido  blando  prevertebral,  que  puede  ser  la  única  anomalía  radiográfica  en  el  40  %  de  las  fracturas  C1  y  
C2.  El  espacio  anterior  a  C3  no  debe  exceder  un  tercio  del  cuerpo  de  C3.  A  nivel  de  C6,  todo  el  cuerpo  de  C6  generalmente  
encaja  en  el  espacio  de  tejido  blando  prevertebral.  Compruebe  la  alineación  de  los  bordes  anterior  y  posterior  de  los  
cuerpos  vertebrales.  Asegúrese  de  que  los  espacios  de  los  discos  intervertebrales  tengan  una  altura  relativamente  igual.  
Evaluar  cada  articulación  facetaria.  Compruebe  las  apófisis  espinosas  en  busca  de  alineación  y  separación  anormal.  
Finalmente,  evalúe  cada  vértebra  en  busca  de  fracturas.

11.  ¿Qué  pasa  con  la  película  anteroposterior?
Inspeccione  cuidadosamente  la  alineación  de  las  apófisis  espinosas  de  la  línea  media.  Las  angulaciones  bruscas  
sugieren  una  luxación  facetaria  unilateral.  Los  cambios  más  sutiles  pueden  indicar  inestabilidad  o  fractura  de  la  faceta.
Las  fracturas  del  cuerpo  pueden  ser  más  evidentes  en  la  vista  AP.

12.  ¿Puede  un  paciente  tener  una  lesión  de  la  médula  espinal  y  radiografías  simples  normales?
La  lesión  de  la  médula  espinal  sin  anormalidad  radiográfica  (SCIWORA)  se  define  como  signos  y  síntomas  neurológicos  
consistentes  con  mielopatía  traumática  a  pesar  de  que  las  radiografías  son  normales.  SCIWORA  es  raro  y  es  más  común  
en  niños,  donde  la  mayoría  de  las  series  citan  alrededor  del  15%  de  SCI.  Esta  tasa  puede  llegar  al  40%  en  niños  <9  años.  
SCIWORA  es  menos  común  en  adultos  (alrededor  del  5%  de  las  LME).

13.  ¿Es  útil  la  resonancia  magnética  en  la  evaluación  del  trauma  agudo  de  columna?
Sí.  Si  las  radiografías  simples  y  las  tomografías  computarizadas  no  explican  adecuadamente  la  extensión  de  la  lesión  observada  en  
los  exámenes  neurológicos,  se  debe  usar  la  resonancia  magnética  para  evaluar  la  columna  en  busca  de  hernias  de  disco,  lesiones  
de  ligamentos  y  evidencia  de  SCI.  Además,  la  resonancia  magnética  se  usa  de  forma  rutinaria  para  aclarar  aún  más  las  lesiones  
identificadas  en  la  tomografía  computarizada.
Machine Translated by Google
CAPÍTULO  18  LESIONES  DE  LA  MÉDULA  ESPINAL  107

14.  ¿Con  qué  vista  se  visualizan  mejor  las  fracturas  de  C1  y  C2?
En  las  radiografías  simples,  la  vista  odontoidea  es  la  mejor,  de  lo  contrario,  las  imágenes  de  TC  
reconstruidas  coronal  y  sagital.  Busque  el  saliente  de  la  masa  lateral  de  C1  de  los  bordes  laterales  de  C2.
Esto  ocurre  en  las  fracturas  de  Jefferson  (fracturas  en  estallido  del  anillo  C1,  que  se  ven  mejor  en  imágenes  
axiales  de  TC).  La  proyección  total  de  ambas  masas  laterales  de  C1  sobre  C2  de  >7  mm  puede  estar  asociada  con  
rotura  del  ligamento  transverso  e  inestabilidad.  Si  ve  una  fractura  C1,  busque  cuidadosamente  una  fractura  C2.  Los  tres  
tipos  de  fracturas  de  odontoides  son:  &  El  tipo  I  ocurre  en  las  guaridas.

&  El  tipo  II  ocurre  en  la  base  de  las  guaridas  donde  se  une  al  cuerpo  de  C2.
  El  tipo  III  se  extiende  al  cuerpo  de  C2.

15.  ¿Qué  es  la  fractura  del  ahorcado?
Fracturas  bilaterales  a  través  de  los  pedículos  o  pars  interarticularis  (o  «istmo»)  de  C2  causadas  por  una  lesión  
grave  por  hiperextensión,  por  lo  general  secundaria  a  accidentes  automovilísticos  a  alta  velocidad.  En  los  
ahorcamientos  judiciales,  la  lesión  mortal  es  el  estiramiento  medular  provocado  por  la  caída  en  combinación  con  la  
fractura  de  C2.  La  mayoría  de  los  pacientes  con  fractura  del  ahorcado  se  presentan  neurológicamente  intactos  y  la  
mayoría  de  los  casos  de  fractura  del  ahorcado  pueden  tratarse  con  inmovilización  externa.

16.  Definir  déficits  en  mielopatía  transversa  completa,  síndrome  medular  anterior,  síndrome  medular  central  y  síndrome  
de  Brown­Sequard.
La  mielopatía  transversa  completa  puede  deberse  a  la  sección,  estiramiento  o  contusión  de  la  médula.  
Todas  las  funciones  por  debajo  del  nivel  de  la  lesión  (motoras,  sensoriales  y  reflejas)  se  pierden.  El  síndrome  
de  la  médula  anterior  resulta  de  una  lesión  de  los  dos  tercios  anteriores  de  la  médula  espinal  (la  distribución  de  
la  arteria  espinal  anterior),  que  transporta  vías  motoras,  dolorosas  y  térmicas.  El  tacto  ligero  y  la  propiocepción  
están  intactos  porque  se  conservan  las  columnas  posteriores.

El  síndrome  de  la  médula  central  resulta  de  una  lesión  en  el  área  central  de  la  médula  espinal.  A  menudo  es
Se  encuentra  en  pacientes  con  estenosis  cervical  preexistente  como  resultado  de  cambios  espondilóticos.
De  manera  característica,  los  déficits  son  más  graves  en  las  extremidades  superiores  que  en  las  
extremidades  inferiores,  y  se  cree  que  son  el  resultado  de  la  torcedura  del  ligamento  amarillo  posterior  
engrosado  en  la  médula  espinal,  con  extensión.  Patológicamente,  hay  hemorragia  en  el  centro  de  la  
médula  espinal.  La  función  motora  suele  verse  más  afectada  que  la  función  sensorial.

El  síndrome  de  Brown­Sequard  se  observa  característicamente  en  lesiones  penetrantes,  pero  también  puede  ser
visto  en  lesiones  contusas,  especialmente  con  discos  herniados  traumáticamente  unilaterales.  El  síndrome  resulta  
de  una  lesión  en  la  mitad  de  la  médula  espinal.  Clínicamente,  los  sentidos  motor,  de  posición  y  de  vibración  se  ven  
afectados  en  el  lado  ipsolateral  a  la  lesión;  estos  tractos  se  cruzan  en  el  tronco  encefálico.  La  sensación  de  dolor  y  
temperatura  desaparece  contralateral  a  la  lesión;  estos  tractos  se  cruzan  en  el  cordón  en  o  cerca  del  nivel  de  inervación.

17.  ¿Cuál  es  el  papel  de  la  metilprednisolona  en  el  tratamiento  de  la  médula  aguda?
¿lesión?
Los  resultados  del  Segundo  Estudio  Nacional  de  Lesión  Aguda  de  la  Médula  Espinal  (NASCIS  II)  sugieren  que  la  
metilprednisolona  en  dosis  altas  resultó  en  una  mejora  estadísticamente  significativa  en  el  resultado.  La  dosis  es  una  
carga  de  30  mg/kg,  seguida  de  5,4  miligramos  por  kilogramo  por  hora  durante  23  horas.  El  ensayo  NASCIS  III  informó  
que  los  pacientes  que  recibieron  dosis  de  3  a  8  horas  después  de  la  lesión  tuvieron  mejores  resultados  cuando  
recibieron  tratamiento  durante  48  horas  con  metilprednisolona  en  lugar  de  24  horas.  En  pacientes  que  recibieron  dosis  
dentro  de  las  3  horas  posteriores  a  la  lesión,  no  se  documentaron  ganancias  adicionales  al  tratar  más  allá  de  las  24  horas.
La  reevaluación  reciente  de  los  datos  disponibles  ha  puesto  en  duda  el  valor  de  estos  esteroides  y  los  riesgos  pueden  
superar  los  beneficios.  La  metilprednisolona  no  está  indicada  en  el  tratamiento  de  la  SCI  penetrante.
Machine Translated by Google
108  CAPÍTULO  18  LESIONES  DE  LA  MÉDULA  ESPINAL

18.  ¿Los  pacientes  con  lesiones  de  la  médula  espinal  alguna  vez  se  someten  a  cirugía  aguda?
Actualmente  no  existen  estándares  sobre  el  momento  de  la  cirugía,  en  particular  con  el  objetivo  en  mente  de  revertir  los  déficits  
neurológicos.  Idealmente,  las  lesiones  de  columna  inestables  (en  pacientes  por  lo  demás  sistémicamente  estables)  deben  
estabilizarse  quirúrgicamente  dentro  de  las  72  horas,  para  facilitar  su  cuidado  global  y  la  movilización  del  paciente.  Con  respecto  
a  la  cirugía  urgente  para  tratar  los  déficits  neurológicos,  la  cirugía  generalmente  se  realiza  en  pacientes  con  lesiones  incompletas  
o  en  un  paciente  con  SCI  y  deterioro  neurológico.  El  deterioro  puede  ser  el  resultado  de  material  discal  herniado,  hemorragia  
epidural  o  hinchazón  del  cordón  umbilical  en  un  canal  estrecho,  lo  que  causa  compresión  del  cordón  umbilical  y  empeoramiento  
de  los  síntomas.  Los  pacientes  que  no  deben  someterse  a  cirugía  de  emergencia  son  aquellos  con  lesiones  completas  de  más  
de  24  horas  de  evolución  y  pacientes  médicamente  inestables.

19.  ¿Cómo  se  trata  la  lesión  ósea?
1.  Prevención  de  lesiones  adicionales  utilizando  precauciones  espinales.
2.  Obtener  la  alineación  normal  utilizando  la  posición  del  cuerpo,  la  tracción  y  el  refuerzo.
3.  Reducción  abierta,  descompresión  y  fusión  según  sea  necesario.

20.  ¿Cuál  es  el  resultado  en  pacientes  con  lesión  de  la  médula  espinal?
Con  lesiones  completas  (sin  función  motora  o  sensorial  debajo  de  la  lesión),  las  posibilidades  de  recuperación  son  
escasas;  El  2%  de  los  pacientes  recuperan  la  deambulación.  El  pronóstico  es  notablemente  mejor  para  pacientes  con  
lesiones  incompletas:  el  75%  experimenta  una  recuperación  significativa.  El  tratamiento  adecuado  de  las  lesiones  óseas  ayuda  a  
prevenir  el  dolor  y  el  deterioro  neurológico  tardío.

21.  ¿Las  lesiones  de  la  columna  cervical  están  asociadas  con  lesiones  de  la  carótida  o  la  vértebra?
arterias?

Sí.  Se  ha  demostrado  que  la  incidencia  de  lesión  de  la  arteria  vertebral  contusa  es  del  0,53  %  al  1,03  %  de  todas  las  admisiones  
por  traumatismos  contusos.  Se  ha  observado  una  tasa  general  de  accidentes  cerebrovasculares  del  24  %  en  todos  los  pacientes  
diagnosticados  con  lesión  de  la  arteria  vertebral  contusa,  independientemente  del  grado  de  la  lesión,  y  la  tasa  de  mortalidad  por  
lesión  vertebral  contusa  fue  del  8  %.  Se  observó  que  los  factores  de  riesgo  correlacionados  con  la  lesión  contusa  de  la  arteria  
vertebral  eran  las  fracturas  cervicales  que  afectaban  al  foramen  transverso,  cualquier  fractura  que  afectaba  a  C1  a  C3  y  cualquier  
lesión  que  implicaba  subluxación.  La  detección  de  pacientes  con  este  tipo  de  lesiones  mediante  angiografía  tomográfica  
computarizada  (CTA)  multidetector  de  16  cortes  puede  identificar  tales  lesiones  vasculares,  lo  que  permite  medidas  preventivas  
como  la  anticoagulación,  antes  de  que  ocurra  un  accidente  cerebrovascular.

22.  ¿Deben  colocarse  filtros  en  la  vena  cava  inferior  a  todos  los  pacientes  con  lesiones  medulares  para  prevenir  la  embolia  pulmonar?

No,  pero  aquellos  pacientes  que  no  responden  a  la  anticoagulación  o  tienen  contraindicaciones  para  la  anticoagulación  
deben  colocarse  un  filtro  en  la  vena  cava  inferior  (VCI).

PUNTOS  CLAVE

1.  Debe  sospecharse  LME  en  todos  los  pacientes  traumatizados  que  están  inconscientes,  intoxicados,  tienen
lesiones  que  distraen,  o  se  quejan  de  dolor  o  sensibilidad  en  el  cuello.  Se  deben  aplicar  estrictas  precauciones  espinales  
hasta  la  limpieza  radiográfica  y  clínica  de  la  columna  vertebral.

2.  En  pacientes  con  LME  aguda,  la  presión  arterial  media  (PAM)  debe  mantenerse  >  90  mm  Hg
para  aumentar  la  perfusión  de  la  médula  espinal.

3.  Los  pacientes  con  lesiones  de  la  columna  cervical  deben  ser  evaluados  para  la  arteria  carótida  y  vertebral.
lesiones

4.  Los  pacientes  con  SCI  que  se  presenten  dentro  de  las  8  horas  pueden  ser  tratados  con  dosis  altas  IV
Protocolo  de  metilprednisolona.
Machine Translated by Google
CAPÍTULO  18  LESIONES  DE  LA  MÉDULA  ESPINAL  109

SITIO  WEB

www.asia­spinalinjury.org/

BIBLIOGRAFÍA

1.  Biffl  WL,  Egglin  T,  Benedetto  B  et  al.:  La  angiografía  por  tomografía  computarizada  de  dieciséis  cortes  es  una  prueba  de  detección  no  
invasiva  confiable  para  lesiones  cerebrovasculares  contusas  clínicamente  significativas.  J  Trauma  60(4):745­751,  2006.

2.  Bracken  MB,  Shepard  MJ,  Holford  TR  et  al.:  Administración  de  metilprednisolona  durante  24  o  48  horas  o  mesilato  de  tirilazad  durante  48  
horas  en  el  tratamiento  de  la  lesión  aguda  de  la  médula  espinal:  resultados  del  tercer  ensayo  controlado  aleatorio  nacional  sobre  lesión  
aguda  de  la  médula  espinal.  JAMA  277:1597­1604,  1997.

3.  Cothren  CC,  Moore  EE,  Biffl  WL  et  al .:  Patrones  de  fractura  de  la  columna  cervical  predictivos  de  lesión  de  la  arteria  vertebral  contundente.
J  Trauma  55(5):811­813,  2003.

4.  Cortez  R,  Levi  AD:  lesión  aguda  de  la  médula  espinal.  Curr  Treat  Options  Neurol  9  de  marzo  (2):  115­125,  2007.

5.  Fehlings  MG,  Perrin  RG:  El  momento  de  la  intervención  quirúrgica  en  el  tratamiento  de  la  lesión  de  la  médula  espinal:  una  revisión  sistemática  
de  la  evidencia  clínica  reciente.  Lomo  31  (11  Suplemento),  S28­S35,  2006.

6.  Hadley  MN,  Walters  BC,  Grabb  PA  et  al.:  Evaluación  radiográfica  de  la  columna  cervical  en  pacientes  con  trauma  sintomático.  Neurocirugía  
50(3  suplementos):S36­S43,  2002.

7.  Harris  MB,  Sethi  RK:  La  evaluación  inicial  y  el  manejo  del  paciente  politraumatizado  con  una  lesión  espinal  asociada.  Lomo  31  (11  
suplementos):  S9­S15,  2006.

8.  Holmes  JF,  Akkinepalli  R:  tomografía  computarizada  versus  radiografía  simple  para  detectar  lesiones  en  la  columna  cervical:  un  metanálisis.  
J  Trauma  58(5):902­905,  2005.

9.  Rabb  CH,  Johnson  JL,  VanSickle  D  et  al .:  ¿Son  radiografías  cervicales  laterales  verticales  en  el  paciente  con  trauma  obnubilado?
¿útil?  Un  estudio  retrospectivo.  World  J  Emerg  Surg  2­4,  2004.

10.  Sliker  CW,  Mirvis  SE,  Shanmuganathan  K:  Evaluación  de  la  estabilidad  de  la  columna  cervical  en  pacientes  obnubilados  con  trauma  cerrado:
revisión  de  la  literatura  médica.  Radiología.  234(3):733­739,  2005.
Machine Translated by Google

TRAUMA  PENETRANTE  DEL  CUELLO
CAPÍTULO  
19

C.  Clay  Cothren,  MD  y  Ernest  E.  Moore,  MD

1.  ¿Por  qué  las  heridas  penetrantes  del  cuello  son  únicas?
Aunque  comprende  solo  un  pequeño  porcentaje  de  la  superficie  corporal,  el  cuello  contiene  una  gran  concentración  
de  estructuras  vitales.

2.  ¿Qué  constituye  una  herida  penetrante  en  el  cuello?
La  violación  del  músculo  platisma  define  una  herida  penetrante  en  el  cuello.  Esta  capa  fascial  de  inversión  del  
cuello  es  superficial  a  las  estructuras  vitales.  Si  no  se  penetra  el  platisma,  la  herida  se  trata  como  una  laceración  
simple  y  el  paciente  es  dado  de  alta  del  servicio  de  urgencias  (SU).

3.  ¿Qué  lado  del  cuello  es  más  probable  que  se  lesione?
El  lado  izquierdo  porque  la  mayoría  de  los  asaltantes  son  diestros.

4.  ¿Las  heridas  por  arma  de  fuego  y  las  heridas  por  arma  blanca  causan  las  mismas  lesiones  relativas?
Las  heridas  por  arma  de  fuego  por  lo  general  tienden  a  infligir  más  daño  tisular  (v .  tabla  19­1).

TABLA  19­1.  GUN  SHO  TV  ER  SU  SST  AB  WOUND  S
Estructura Heridas  de  bala  (%) Heridas  de  arma  blanca  (%)

Artería 20 5

Vena 15 10

Vías  respiratorias
10 5

Digestivo 20 <5

5.  ¿Cuáles  son  las  prioridades  en  el  manejo  del  traumatismo  penetrante  del  cuello?
El  ABC  (vías  respiratorias,  respiración  y  circulación)  es  la  primera  prioridad  en  todo  paciente  traumatizado.  Si  está  
indicado,  los  pacientes  deben  ser  intubados  por  vía  oral,  aunque  puede  ser  necesaria  la  cricotiroidotomía  con  un  
hematoma  extenso  en  el  cuello  o  sangrado  continuo  en  la  orofaringe.  Aunque  el  paciente  puede  presentar  una  vía  
aérea  permeable,  se  debe  realizar  un  control  electivo  temprano  de  la  vía  aérea  en  pacientes  con  hematomas  en  
expansión.  Con  base  en  la  trayectoria  de  la  lesión,  debe  sospecharse  neumotórax,  hemotórax  o  lesión  de  grandes  
vasos.  La  hemorragia  externa  debe  controlarse  con  presión  digital  directa  y  el  acceso  intravenoso  (IV)  debe  asegurarse  
con  dos  catéteres  periféricos  de  gran  calibre.

110
Machine Translated by Google
CAPÍTULO  19  TRAUMA  PENETRANTE  DEL  CUELLO  111

6.  ¿Cómo  se  debe  controlar  el  sangrado  en  la  escena  del  accidente  y  en  la  emergencia?
¿departamento?
La  presión  digital  directa  casi  siempre  tiene  éxito,  incluso  en  el  caso  de  lesiones  arteriales  importantes.  Colocar  
abrazaderas  a  ciegas  dentro  de  una  herida  corre  el  riesgo  de  lesionar  otras  estructuras  vitales  no  lesionadas,  en  particular
nervios

7.  ¿Debe  explorar  la  herida  del  cuello  en  la  sala  de  traumatología?
Aunque  se  justifica  una  inspección  visual  cuidadosa,  el  sondaje  de  la  herida  (digitalmente,  con  un  Q­
tip  o  con  un  instrumento  quirúrgico)  puede  desprender  un  coágulo  de  sangre  y  causar  una  
hemorragia  notable.

8.  ¿Qué  hallazgos  del  examen  físico  deben  obtenerse?
La  hemorragia  continua  de  la  herida,  los  hematomas  expansivos  o  pulsátiles,  la  hemoptisis,  la  hematemesis,  
los  déficits  neurológicos,  la  disfagia,  la  disfonía,  la  ronquera  y  el  estridor  son  indicativos  de  una  lesión  importante.

9.  ¿Con  qué  frecuencia  los  pacientes  con  crepitación  cervical  tienen  una  lesión  significativa?
Un  tercio  de  los  pacientes  con  crepitación  tienen  una  lesión  en  la  faringe,  el  esófago,  la  laringe  o  la  tráquea.
Sin  embargo,  en  dos  tercios  de  estos  pacientes,  el  aire  se  ha  introducido  a  través  del  sitio  de  entrada  de  la  
herida  y  no  hay  una  lesión  subyacente  significativa.

10.  ¿Cuáles  son  las  tres  zonas  del  cuello?
La  zona  I  se  extiende  desde  la  muesca  esternal  hasta  el  cartílago  cricoides.
La  zona  II  se  extiende  desde  el  cartílago  cricoides  hasta  el  ángulo  de  la  mandíbula.
La  zona  III  comprende  el  área  cefálica  al  ángulo  de  la  mandíbula  (v .  fig.  19­1).

tercero

Yo

Figura  19­1.  Las  tres  
zonas  del  cuello.
Machine Translated by Google
112  CAPÍTULO  19  TRAUMA  PENETRANTE  DEL  CUELLO

11.  ¿Por  qué  las  lesiones  penetrantes  se  dividen  en  zonas?
Cada  zona  tiene  implicaciones  de  gestión.  Debido  a  las  dificultades  técnicas  de  la  exposición  de  las  lesiones  y  los  
diversos  enfoques  quirúrgicos,  es  deseable  un  diagnóstico  preoperatorio  preciso  para  las  lesiones  sintomáticas  de  
las  zonas  I  y  III.  Las  lesiones  de  la  zona  II  se  evalúan  más  fácilmente  con  la  exploración  física  (v .  fig.  19­2).

Inestable  hemodinámicamente
Hemorragia  no  controlada
TAC  
+
Zona  I de   broncoscopia  
cuello/tórax de  esofagograma  CTA
PENETRANTE Hemodinámicamente  estable
Zona  II
HERIDA  EN  EL  CUELLO Sintomático**
Zona  III angiografía
Operatorio
Exploración
+

IR  embo  

TAC  
+
Zona  I de   broncoscopia  
cuello/tórax de  esofagograma  CTA
asintomático

GSW  transcervical
Zona  II
**síntomas  =  hematoma  en  expansión   Todos  los  otros
Observar
compromiso  de  las  vías   Zona  III
respiratorias  disfagia  
enfisema  subcutáneo  ronquera

Figura  19­2.  Manejo  del  trauma  penetrante  de  cuello.

12.  ¿Cuáles  son  las  indicaciones  para  la  exploración  quirúrgica  inmediata?
Inestabilidad  hemodinámica  o  hemorragia  arterial  externa.

13.  ¿Qué  es  el  manejo  selectivo  del  traumatismo  penetrante  del  cuello?
Históricamente,  todas  las  lesiones  de  la  zona  II  que  violaban  el  platisma  se  exploraban  quirúrgicamente.  Sin  
embargo,  como  resultado  de  un  número  prohibitivo  de  exploraciones  negativas,  este  enfoque  perdió  apoyo.
Los  pacientes  asintomáticos  con  lesiones  en  la  zona  II  pueden  observarse  durante  12  a  24  horas.  La  excepción  
son  los  pacientes  con  heridas  de  bala  transcervicales,  que  deben  someterse  a  una  angiografía  tomográfica  
computarizada  (CTA)  para  determinar  el  trayecto  del  misil  y  la  necesidad  de  más  diagnóstico  por  imágenes.

14.  ¿Se  debe  realizar  una  arteriografía  a  todos  los  pacientes?
La  angiografía  se  realiza  en  pacientes  sintomáticos,  hemodinámicamente  estables  con  lesiones  en  la  zona  I  y  III.  
En  pacientes  con  trauma  de  zona  I,  la  angiografía  identifica  lesiones  de  grandes  vasos  en  la  salida  torácica  que  
pueden  requerir  un  abordaje  quirúrgico  torácico.  El  diagnóstico  angiográfico  de  las  lesiones  de  la  zona  III  puede  ir  
seguido  de  angioembolización  o  intervención  endovascular.

15.  ¿Cuál  es  el  valor  de  otros  estudios  de  diagnóstico,  como  esofagografía,  esofagoscopia,  laringoscopia  y  
broncoscopia?
Se  han  recomendado  la  esofagografía,  la  broncoscopia  y  la  laringoscopia  en  la  zona  I  y  en  pacientes  
seleccionados  de  la  zona  II  tratados  sin  cirugía,  pero  la  tomografía  computarizada  (TC)  multicorte  ha  limitado  su  
función.  La  esofagoscopia  se  combina  con  la  esofagografía  si  se  sospecha  lesión  esofágica;  si  el  material  de  
contraste  hidrosoluble  no  muestra  fugas,  se  utiliza  bario.
Machine Translated by Google
CAPÍTULO  19  TRAUMA  PENETRANTE  DEL  CUELLO  113

Las  lesiones  esofágicas  no  detectadas  pueden  ser  mortales,  con  una  tasa  de  mortalidad  del  20%  si  el  diagnóstico  se  retrasa  
solo  12  horas.  La  endoscopia  intraoperatoria  con  insuflación  puede  usarse  de  forma  provocativa  para  mostrar  una  fuga  de  aire  y  
la  lesión  esofágica  asociada.  La  angiografía  sigue  siendo  el  estándar  de  oro  para  el  diagnóstico  de  lesión  arterial  y  esta  modalidad  
puede  ser  terapéutica  para  las  lesiones  de  la  zona  III;  la  zona  III  es  difícil  de  exponer  quirúrgicamente.

16.  ¿Se  debe  enviar  a  casa  desde  el  departamento  de  emergencias  a  un  paciente  asintomático  con  una  herida  penetrante  en  el  cuello?

No.  Inicialmente,  las  heridas  penetrantes  en  el  cuello  que  ponen  en  peligro  la  vida  pueden  ser  difíciles  de  resolver;  la  política  
más  segura  es  observar  a  todos  los  pacientes  en  el  hospital  durante  al  menos  24  horas.

PUNTOS  CLAVE

1.  La  lesión  penetrante  implica  la  violación  del  platisma;  el  manejo  se  basa  en  los  síntomas  del  paciente  y  las  zonas  anatómicas  
de  la  lesión.

2.  La  intervención  quirúrgica  inmediata  está  indicada  en  pacientes  con  inestabilidad  hemodinámica  o  sangrado  externo  
significativo  en  curso.

3.  Las  lesiones  de  la  zona  I  a  menudo  tienen  lesiones  asociadas  de  grandes  vasos  que  pueden  requerir  una  exploración  torácica.
acercarse.

4.  No  es  necesaria  la  exploración  obligatoria  de  todas  las  lesiones  de  la  zona  II;  alerta  y  asintomático
los  pacientes  pueden  ser  observados  expectantes  durante  24  horas.

BIBLIOGRAFÍA

1.  Albuquerque  FC,  Javedan  SP,  McDougall  CG:  Manejo  endovascular  de  lesiones  penetrantes  de  la  arteria  vertebral.
J  Trauma  53:574­580,  2002.

2.  Atteberry  LR,  Dennis  JW,  Menawat  SS  et  al.:  El  examen  físico  por  sí  solo  es  seguro  y  preciso  para  la  evaluación  de  lesiones  
vasculares  en  traumatismos  penetrantes  del  cuello  en  la  zona  II.  J  Am  Coll  Surg  179:657­662,  1994.

3.  Biffl  WL,  Moore  EE,  Rehse  DH  et  al.:  Manejo  selectivo  del  traumatismo  penetrante  del  cuello  basado  en  el  nivel  cervical
de  herida  Am  J  Surg  174:678­682,  1997.

4.  Demetriades  D,  Velmahos  G,  Asensio  JA:  Lesiones  cervicales  faringoesofágicas  y  laringotraqueales.  Mundo  J
Surg  25:1044­1048,  2001.

5.  Ferguson  E,  Dennis  JW,  Vu  JH  et  al .:  Redefinición  del  papel  de  las  imágenes  arteriales  en  el  tratamiento  de  las  lesiones  
penetrantes  del  cuello  en  la  zona  3.  Vascular  13:158­163,  2005.

6.  Gracias  VH,  Reilly  PM,  Philpott  J  et  al.:  Tomografía  computarizada  en  la  evaluación  del  traumatismo  penetrante  del  cuello.
Arch  Surg  136:1231­1235,  2001.

7.  Hirshberg  A,  Wall  MJ,  Johnston  RH  et  al.:  Lesiones  transcervicales  por  arma  de  fuego.  Am  J  Surg  167:309,  1993.

8.  Inaba  K,  Munera  F,  McKenney  M  et  al.:  Evaluación  prospectiva  del  cribado  helicoidal  multicorte  computarizado
angiografía  tomográfica  en  la  evaluación  inicial  de  las  lesiones  penetrantes  del  cuello.  J  Trauma  61:144­149,  2006.

9.  Mazolewski  PJ,  Curry  JD,  Browder  T  et  al.:  La  tomografía  computarizada  se  puede  utilizar  para  la  decisión  quirúrgica
realizando  en  la  zona  II  lesiones  penetrantes  del  cuello.  J  Trauma  51:315­319,  2001.

10.  Woo  K,  Magner  DP,  Wilson  MT  et  al.:  La  angiografía  por  TC  en  el  traumatismo  penetrante  del  cuello  reduce  la  necesidad  de  cirugía
exploración  del  cuello.  Am  Surg  71:754,  2005.
Machine Translated by Google

TRAUMA  TORÁCICO  CONTUNDENTE
CAPÍTULO  
20

Jeffrey  L.  Johnson,  MD  y  Ernest  E.  Moore,  MD

1.  ¿Con  qué  frecuencia  los  pacientes  con  traumatismo  torácico  cerrado  aislado  necesitan  una  emergencia?
¿operación?
Casi  nunca.  En  los  pacientes  que  llegan  vivos  al  hospital,  las  lesiones  quirúrgicas  de  estructuras  pulmonares,  
vasculares  y  mediastínicas  son  sorprendentemente  raras;  sólo  el  5%  de  los  pacientes  con  lesión  cerrada  aislada  
del  tórax  requieren  toracotomía.

2.  En  un  paciente  con  hemotórax  tras  traumatismo  cerrado  de  tórax,  ¿cuál  es  la  guía  más  importante  para  la  
decisión  de  operar?
El  estado  hemodinámico  del  paciente.  El  hemotórax  después  de  una  lesión  contusa  suele  ser  el  resultado  de  
lesiones  no  quirúrgicas  del  pulmón  y  la  pared  torácica.  En  un  paciente  estable,  por  lo  tanto,  el  enfoque  inicial  debe  
ser  la  evacuación  del  hemotórax,  la  reexpansión  del  pulmón  y  la  corrección  de  la  coagulopatía,  la  hipotermia  y  la  
acidosis.  Debe  anotarse  la  salida  del  tubo  torácico,  pero  no  es  la  consideración  principal.

3.  ¿Qué  es  un  neumotórax  a  tensión?
Aire  en  el  espacio  pleural  bajo  presión  como  resultado  de  un  mecanismo  de  válvula  unidireccional.  Esta  es  una  
condición  que  pone  en  peligro  la  vida  porque  las  elevaciones  marcadas  en  la  presión  intrapleural  producen  
colapso  circulatorio  por  llenado  ventricular  derecho  deteriorado.

4.  ¿Cuáles  son  los  signos  clínicos  del  neumotórax  a  tensión?
Hipotensión,  ausencia  de  ruidos  respiratorios  en  el  lado  afectado  y  distensión  de  las  venas  del  cuello.  El  
neumotórax  a  tensión  debe  tratarse  ante  la  sospecha  clínica  y  sin  demora  para  la  confirmación  radiográfica.

5.  ¿Cómo  se  trata  el  neumotórax  a  tensión?
Haz  un  agujero  en  el  pecho.  Para  atención  prehospitalaria,  descompresión  con  aguja  a  través  del  quinto  espacio  
intercostal  en  la  línea  medioaxilar  o  línea  medioclavicular.  Sin  embargo,  en  el  entorno  hospitalario,  un  médico  
experimentado  puede  descomprimir  completamente  el  espacio  pleural  con  la  misma  rapidez  con  una  
toracostomía  con  tubo.

6.  ¿Importa  cuántas  costillas  se  rompen?
Sí.  Seis  o  más  fracturas  indican  un  mayor  riesgo  de  neumonía,  síndrome  de  dificultad  respiratoria  del  adulto  
(SDRA)  y  muerte,  especialmente  en  pacientes  de  edad  avanzada.

7.  ¿Qué  es  un  cofre  de  mayales?
Un  tórax  inestable  ocurre  cuando  una  parte  de  la  caja  torácica  pierde  la  continuidad  ósea  con  el  resto  del  tórax.  
Cuando  se  fracturan  múltiples  costillas  en  dos  o  más  lugares,  la  pared  torácica  se  mueve  de  forma  paradójica  
("flails")  con  la  respiración.

8.  ¿Cómo  afecta  el  tórax  inestable  a  la  ventilación?
En  pacientes  con  respiración  espontánea,  la  parte  de  la  caja  torácica  que  ha  perdido  la  continuidad  ósea  
se  retrae  hacia  adentro  durante  la  inspiración.  Este  movimiento  paradójico  puede  resultar  en  una  mala  
ventilación.

114
Machine Translated by Google
CAPÍTULO  20  TRAUMA  TORÁCICO  CONTUNDENTE  115

9.  ¿Todos  los  pacientes  con  un  segmento  inestable  necesitan  un  ventilador?
No.  El  impacto  de  un  segmento  de  mayal  en  la  ventilación  no  suele  ser  profundo  y,  con  una  buena  analgesia,  muchos  
pacientes  pueden  mantener  su  propio  trabajo  respiratorio.  Deben  utilizarse  las  indicaciones  estándar  para  la  intubación.

10.  ¿El  tórax  inestable  afecta  la  oxigenación?
El  tórax  inestable  per  se  tiene  poco  impacto  directo  en  la  oxigenación.  Sin  embargo,  prácticamente  todos  los  pacientes  
con  tórax  inestable  tienen  un  hematoma  subyacente  en  el  pulmón:  contusión  pulmonar.  La  gravedad  de  la  contusión  
pulmonar  es  un  factor  determinante  más  importante  del  resultado  y  la  necesidad  de  intubación  que  el  deterioro  de  la  
mecánica  de  la  pared  torácica.  La  fisiopatología  de  la  lesión  contusa  del  tórax  con  lesión  ósea  grave  debe  considerarse  
como  un  proceso  único  (es  decir,  tórax  inestable  o  contusión  pulmonar).

11.  ¿Cuál  es  la  evolución  natural  de  la  contusión  pulmonar?
Es  como  un  hematoma  en  el  pulmón.  Inicialmente,  el  pulmón  sufre  cizallamiento  del  parénquima  y  ruptura  de  pequeños  
vasos  sanguíneos;  esta  lesión  tisular  va  seguida  de  edema  e  inflamación.  Así,  al  igual  que  otros  hematomas,  los  pacientes  
con  contusión  pulmonar  suelen  desarrollar  deterioro  clínico  en  las  primeras  48  horas.  La  radiografía  de  tórax  inicial  puede  
parecer  engañosamente  benigna.

12.  ¿Cuál  es  la  presentación  inicial  más  común  de  una  lesión  contusa  de  la  aorta  torácica?
Muerte.  El  ochenta  y  cinco  por  ciento  de  los  pacientes  con  una  aorta  torácica  desgarrada  mueren  de  desangramiento  antes  
de  llegar  al  hospital.  La  rotura  del  corazón  y  de  los  grandes  vasos  ocupa  el  segundo  lugar  después  de  la  lesión  en  la  cabeza  
como  causa  de  muerte  como  resultado  de  un  traumatismo  cerrado.

13.  De  los  pacientes  que  sobreviven  para  llegar  al  hospital,  ¿dónde  está  la  lesión  más  común  de  la  aorta  torácica?

Un  desgarro  a  través  de  la  íntima  y  la  media  justo  distal  al  origen  de  la  arteria  subclavia  izquierda.  Debido  a  que  la  
adventicia  está  intacta,  el  paciente  no  se  desangra  inmediatamente  y,  si  la  lesión  se  detecta  y  trata  de  inmediato,  la  tasa  
de  supervivencia  es  del  85%.

14.  ¿Cuáles  son  los  signos  clínicos  del  desgarro  de  la  aorta  torácica?
No  hay  signos  definitivos.  La  sospecha  debe  basarse  en  el  mecanismo  de  la  lesión  (desaceleración  rápida).  
Los  signos  físicos  asociados  con  la  interrupción  aórtica  no  se  observan  con  frecuencia;  incluyen  hipertensión  de  las  
extremidades  superiores,  presiones  desiguales  de  las  extremidades  superiores,  pérdida  de  pulsos  de  las  extremidades  
inferiores  y  hematoma  en  expansión  en  la  raíz  del  cuello.

15.  ¿Qué  hallazgos  en  la  radiografía  de  tórax  se  asocian  con  la  rotura  de  la  aorta  torácica  descendente?

Al  igual  que  los  signos  físicos,  ningún  signo  radiográfico  inicial  es  definitivo.  Los  signos  que  se  han  asociado  con  
el  desgarro  de  la  aorta  torácica  incluyen  perilla  aórtica  indistinta,  mediastino  ensanchado  (>  8  cm  al  nivel  de  la  
perilla  aórtica),  capuchón  apical,  derrame  pleural  izquierdo,  depresión  del  bronquio  principal  izquierdo,  desplazamiento  
del  esófago  hacia  la  derecha.  buscar  la  sonda  nasogástrica),  fracturas  de  primeras  costillas,  desplazamiento  de  la  tráquea  
y  pérdida  de  la  ventana  aortopulmonar.
Aproximadamente  el  15%  de  los  pacientes  con  aorta  desgarrada  tienen  un  mediastino  normal  y  el  7%  tienen  una  
radiografía  de  tórax  completamente  normal.

16.  En  el  paciente  estable  con  un  mecanismo  importante  de  lesión  o  radiografías  de  tórax  compatibles  con  lesión  aórtica,  
¿cómo  se  realiza  el  diagnóstico?
La  tomografía  computarizada  (TC)  helicoidal  dinámica  del  tórax  se  acerca  al  100%  de  sensibilidad  para  detectar  lesión  
aórtica;  está  ampliamente  disponible  y  es  aplicable  a  todos  los  pacientes  estables.  La  aortografía  puede  identificar  con  
mayor  precisión  el  sitio  y  la  extensión  de  la  lesión,  pero  en  gran  medida  ha  sido  reemplazada  por  la  angiografía  por  TC.

17.  Junior  O'Flaherty  fue  golpeado  en  el  pecho  con  un  bate  de  béisbol.  ¿Cómo  puedo  saber  si  tiene
un  hematoma  en  el  corazón  (contusión  miocárdica)?
No  puedes  a  menos  que  estés  haciendo  su  autopsia.
Machine Translated by Google
116  CAPÍTULO  20  TRAUMA  TORÁCICO  CONTUNDENTE

18.  Bien,  entonces,  ¿cómo  sé  si  algo  malo  le  va  a  pasar  al  corazón  de  Junior?
Desde  un  punto  de  vista  práctico,  solo  le  suceden  dos  cosas  al  corazón  magullado:  arritmia  y  falla  de  la  bomba.  Con  mucho,  
la  manifestación  más  común  de  lesión  cardíaca  cerrada  es  la  arritmia.  Los  pacientes  con  un  electrocardiograma  (ECG)  inicial  
normal  tienen  una  posibilidad  extremadamente  pequeña  de  desarrollar  arritmias  clínicamente  significativas  durante  su  curso  
hospitalario.  Cualquier  anomalía  en  el  ECG  es  indicación  de  ingreso  y  monitorización  cardiaca  de  24  horas.  El  compromiso  
hemodinámico  por  lesión  cardiaca  cerrada  es  inusual  y  no  sutil;  la  ecocardiografía  debe  emplearse  en  pacientes  con  evidencia  
de  alteración  de  la  contractilidad.  Las  enzimas  cardíacas  son  malos  predictores  de  arritmia  o  falla  de  la  bomba  y  no  se  
recomiendan.

19.  ¿Dónde  suelen  ocurrir  las  lesiones  contusas  del  bronquio?  ¿Cómo  se  presentan?
A  pocos  centímetros  de  la  carina.  Los  bronquios  principales  están  separados  con  compresión  anteroposterior  severa  
del  tórax.  A  medida  que  los  pulmones  se  desplazan  lateralmente,  los  bronquios  principales  pueden  desgarrarse  cerca  del  
sitio  donde  están  fijados  en  la  carina.  La  presentación  típica  es  aire  subcutáneo,  fuga  masiva  de  aire  o  falta  de  reexpansión  
del  pulmón  ("pulmón  caído")  después  de  una  toracostomía  con  sonda.

20.  ¿Cuáles  son  las  indicaciones  para  la  toracotomía  en  el  servicio  de  urgencias  después  de  una  cirugía  roma?
lesión  en  el  pecho?
Colapso  cardiovascular  después  de  la  llegada  al  servicio  de  urgencias  (SU).  El  resultado,  sin  embargo,  suele  ser  deprimente;  
menos  del  1%  de  los  pacientes  sobrevive  neurológicamente  intacto.

21.  ¿Qué  es  la  asfixia  traumática?
La  asfixia  traumática  es  el  resultado  de  una  lesión  por  aplastamiento  prolongada  en  la  parte  superior  del  torso  o  el  epigastrio.  
En  tal  lesión,  la  hipertensión  venosa  se  transmite  a  las  venas  sin  válvulas  de  la  parte  superior  del  cuerpo.  Los  pacientes  
presentan  alteración  del  sensorio,  hemorragias  petequiales,  cianosis  y  edema  de  la  parte  superior  del  cuerpo.
Aunque  su  presentación  inicial  puede  ser  dramática,  con  atención  de  apoyo  el  resultado  suele  ser  bueno.

PUNTOS  CLAVE

1.  La  mayoría  de  las  lesiones  torácicas  cerradas,  incluso  las  más  importantes,  pueden  tratarse  sin  operación.  Un  tubo  
torácico  y  control  del  dolor  suele  ser  lo  más  que  se  necesita.

2.  El  neumotórax  a  tensión  es  un  evento  preterminal  y  debe  tratarse  inmediatamente  con  un  agujero
en  el  pecho.

3.  No  arruine  el  hospital  en  busca  de  un  hematoma  en  el  corazón  de  un  paciente.  Revise  su  ECG  y  asegúrese  de  que  esté  
bombeando.

4.  Los  pacientes  que  pierden  mucho  aire  de  los  pulmones  pueden  tener  un  desgarro  en  un  bronquio  principal.

5.  La  desaceleración  rápida  puede  provocar  un  desgarro  en  la  aorta  torácica  descendente.  A  pesar  de  una  radiografía  de  
tórax  normal,  los  pacientes  con  este  mecanismo  merecen  una  tomografía  computarizada.

BIBLIOGRAFÍA

1.  Allen  GS,  Coates  NE:  Contusión  pulmonar:  una  revisión  colectiva.  Am  Surg  62:895,  1996.

2.  Branney  SW,  Moore  EE,  Feldhaus  KM  et  al.:  Análisis  crítico  de  dos  décadas  de  experiencia  con  postlesión
toracotomía  del  departamento  de  emergencias  en  un  centro  regional  de  trauma.  J  Trauma  45:87,  1998.

3.  Bulger  EM,  Arneson  MA,  Mock  CN  et  al.:  Fracturas  de  costillas  en  ancianos.  J  Trauma  48:1040,  2000.

4.  Demetriades  D,  Velmahos  GC,  Scalea  TM  et  al .:  Reparación  quirúrgica  o  injerto  de  stent  endovascular  en  trauma  cerrado
lesiones  de  la  aorta  torácica:  el  estudio  multicéntrico  AAST.  J  Trauma  64:561,  2008.
Machine Translated by Google
CAPÍTULO  20  TRAUMA  TORÁCICO  CONTUNDENTE  117

5.  Dyer  DS,  Moore  EE,  Ilke  DN  et  al .:  Lesión  de  la  aorta  torácica:  qué  tan  predictivo  es  el  mecanismo  y  si  la  tomografía  computarizada  de  tórax  es  confiable
¿herramienta  de  visualización?  Un  estudio  prospectivo  de  1500  pacientes.  J  Trauma  48:673,  2000.

6.  Flagel  BT,  Luchette  FA,  Reed  L  et  al .:  Media  docena  de  costillas:  el  punto  de  corte  para  la  mortalidad.  Cirugía  138:717,  2005.

7.  Gomez­Caro  A,  Ausin  P,  Moradiliellos  FJ  et  al.:  Rol  del  manejo  conservador  de  las  lesiones  traqueobronquiales.
J  Trauma  61:1426,  2006.

8.  Karmy­Jones  R,  Jrukovich  GJ,  Nathens  AB  et  al .:  Momento  de  la  toracotomía  urgente  por  hemorragia  después  del  trauma:  un
estudio  multicéntrico.  Arch  Surg  136:513,  2001.

9.  Kiser  AC,  O'Brien  SM,  Detterbeck  FC:  Lesiones  traqueobronquiales  cerradas:  tratamiento  y  resultado.  Ann  Thorac  Cirugía
71:2059,  2001.

10.  Yeong  EK,  Chen  MT,  Chu  SH:  asfixia  traumática.  Plast  Reconstr  Surg  93:739,  1994.
Machine Translated by Google

TRAUMA  TORÁCICO  PENETRANTE
CAPÍTULO  
21

Jeffrey  L.  Johnson,  MD  y  Ernest  E.  Moore,  MD

1.  ¿Con  qué  frecuencia  necesitan  una  operación  los  pacientes  con  heridas  penetrantes  en  el  pecho?
La  mayoría  de  las  lesiones  penetrantes  observadas  en  la  práctica  civil  son  causadas  por  cuchillos  y  pistolas  de  baja  energía.
En  consecuencia,  aunque  las  lesiones  de  la  pared  torácica  y  los  pulmones  son  comunes,  la  gran  mayoría  se  puede  tratar  
con  toracostomía  con  sonda  sola.  Se  requiere  una  toracotomía  formal  o  una  esternotomía  media  en  menos  del  15%  de  las  
lesiones  torácicas  penetrantes  aisladas.

2.  ¿Cuáles  son  las  indicaciones  para  la  toracotomía  en  el  servicio  de  urgencias  después  de  heridas  torácicas  
penetrantes?
Los  pacientes  que  llegan  dentro  de  los  15  minutos  posteriores  al  colapso  circulatorio  (o  paro  después  de  la  llegada)  pueden  
beneficiarse  de  una  toracotomía  en  el  departamento  de  emergencias  (EDT).  A  diferencia  de  la  lesión  contusa,  una  causa  tratable  
se  encuentra  más  comúnmente  después  de  una  lesión  penetrante  (p.  ej.,  taponamiento  pericárdico).  EDT  da  como  resultado  una  
supervivencia  de  alrededor  del  20%  en  este  entorno.

3.  ¿Cuál  es  la  "regla  de  las  6  horas"  para  las  lesiones  torácicas?
Una  radiografía  de  tórax  en  bipedestación  sin  evidencia  de  neumotórax  después  de  6  horas  hace  poco  probable  un  
neumotórax  tardío  o  una  lesión  oculta  en  un  órgano  intratorácico.  La  regla  de  las  6  horas  identifica  a  los  pacientes  que  
pueden  ser  dados  de  alta  de  manera  segura.

4.  ¿Cuánta  sangre  en  el  espacio  pleural  se  puede  detectar  de  manera  confiable  mediante  el  tórax?
¿radiografía?  250  
ml  o  más.

5.  Si  a  un  paciente  estable  con  una  herida  penetrante  en  el  pecho  le  sale  mucha  sangre
de  un  tubo  torácico,  ¿cuándo  debo  operar?
Una  buena  regla  general  es  que  el  retorno  inmediato  de  más  de  1500  ml  de  sangre  o  un  sangrado  continuo  superior  a  250  
ml/h  durante  3  horas  consecutivas  debe  impulsar  la  operación.  Todos  los  pacientes  inestables  merecen  una  operación.

6.  ¿Qué  es  una  toracotomía  en  "concha  de  almeja"?
Toracotomías  anterolaterales  bilaterales  con  extensión  a  través  del  esternón.  Este  procedimiento  permite  un  acceso  rápido  
a  los  espacios  pleurales,  el  hilio  pulmonar  y  el  mediastino.

7.  ¿Qué  es  un  neumotórax  abierto?
Un  defecto  en  la  pared  torácica  que  está  abierto  al  espacio  pleural.  En  las  lesiones  torácicas  penetrantes,  la  mayoría  de  las  
veces  es  el  resultado  de  un  disparo  de  escopeta  a  corta  distancia.

8.  ¿Cómo  se  trata  un  neumotórax  abierto?
El  defecto  en  la  pared  torácica  debe  cubrirse  con  un  vendaje  que  se  fija  solo  en  tres  lados.  Esta  solución  temporal  
evita  la  entrada  de  más  aire  en  el  espacio  pleural  al  tiempo  que  permite  la  salida  de  aire  bajo  presión.  Luego  se  inserta  un  
tubo  torácico.  La  reparación  formal  de  la  pared  torácica  puede  esperar  hasta  que  se  descarten  otras  lesiones  importantes.

118
Machine Translated by Google
CAPÍTULO  21  TRAUMA  TORÁCICO  PENETRANTE  119

9.  ¿Dónde  está  ''la  caja''  y  por  qué  es  importante?
El  cuadro  describe  un  área  en  la  parte  anterior  del  tórax  donde  una  herida  debe  generar  preocupación  sobre  una  lesión  
cardíaca  subyacente.  Sus  límites  son  las  líneas  medioclaviculares  desde  la  clavícula  hasta  el  margen  costal.  Aunque  
el  paciente  típico  con  una  lesión  cardíaca  penetrante  tiene  una  herida  en  la  caja,  también  se  puede  llegar  al  corazón  
desde  la  raíz  del  cuello,  la  axila  y  el  epigastrio.

10.  ¿Qué  es  la  tríada  de  Beck?  ¿Es  útil  en  lesiones  penetrantes  del  tórax?
La  tríada  de  Beck  consiste  en  hipotensión,  distensión  de  las  venas  del  cuello  y  tonos  cardíacos  apagados.  Estos  signos  
son  difíciles  de  apreciar  en  el  paciente  traumatizado  (en  particular,  los  ruidos  cardíacos  apagados  en  una  sala  de  
reanimación  concurrida)  y  están  presentes  en  una  minoría  de  pacientes  con  taponamiento  por  lesiones  penetrantes  
(menos  del  40%).  Se  podría  esperar  la  ausencia  de  distensión  de  las  venas  del  cuello  porque  la  mayoría  de  los  
pacientes  tienen  hipovolemia  concomitante.

11.  En  un  paciente  estable  con  sospecha  de  lesión  cardíaca  penetrante,  ¿cuál  es  el  estudio  inicial  más  importante?

Después  de  completar  el  examen  primario  (vías  respiratorias,  respiración,  circulación),  se  debe  
realizar  una  ecografía  al  lado  de  la  cama.  Este  método  rápido  y  sensible  para  detectar  líquido  pericárdico  identificará  
los  resultados  de  una  lesión  cardíaca.  Aunque  el  estudio  inicial  puede  ser  negativo  con  un  pequeño  derrame,  los  
exámenes  seriados  detectan  prácticamente  todos  los  casos.

12.  Junior  O'Flaherty  acaba  de  ser  apuñalado  en  el  corazón.  ¿De  qué  es  probable  que  muera?
Taponamiento  cardíaco.  Las  heridas  de  cuchillo  suelen  hacer  una  abertura  en  forma  de  hendidura  en  el  pericardio,  
que  se  sella  con  un  coágulo  después  de  que  el  corazón  sangra  en  el  saco  pericárdico.  La  exanguinación  es  poco  
común;  El  taponamiento  es  la  amenaza  más  común  para  la  vida.

13.  ¿Cuál  es  la  maniobra  terapéutica  inicial  en  el  paciente  con  herida  cardiaca  penetrante  que  aún  no  está  hipotenso?

Drenaje  pericárdico  percutáneo.  Uno  de  los  primeros  efectos  del  taponamiento  es  la  isquemia  subendocárdica,  
que  pone  al  paciente  en  riesgo  de  arritmias  refractarias.  La  descompresión  inmediata  del  pericardio  garantiza  un  
transporte  más  seguro  al  quirófano  (OR)  para  la  reparación  definitiva.
La  ventana  pericárdica  suxifoidea  también  es  una  opción  (y  popular  en  la  televisión),  pero  la  descompresión  
guiada  por  ultrasonido  es  la  mejor  opción.

14.  En  una  herida  torácica  penetrante,  ¿cómo  puedo  saber  si  el  diafragma  también  está  lesionado?
Al  final  de  la  expiración,  la  cúpula  del  diafragma  alcanza  el  nivel  de  los  pezones.  Entonces,  en  principio,  cualquier  
paciente  con  lesión  penetrante  por  debajo  del  nivel  de  los  pezones  puede  tener  una  lesión  en  el  diafragma.  La  
tomografía  computarizada  (TC)  no  es  confiable  a  menos  que  muestre  una  herniación  obvia  de  las  vísceras  abdominales  
hacia  el  tórax.  El  lavado  peritoneal  de  diagnóstico  (DPL)  es  el  procedimiento  inicial  preferido.  Si  el  líquido  DPL  sale  por  
un  tubo  torácico,  hay  un  orificio  en  el  diafragma.  En  ausencia  de  este  hallazgo,  el  recuento  de  glóbulos  rojos  también  
se  puede  utilizar  como  guía.  Los  recuentos  de  glóbulos  rojos  inferiores  a  1000/mm3  son  negativos.  Los  recuentos  >10  
000  son  positivos;  para  recuentos  de  1  000  a  10  000,  a  menudo  se  utiliza  la  toracoscopia  o  laparoscopia  para  visualizar  
el  hemidiafragma  en  riesgo.

15.  ¿Por  qué  es  importante  detectar  una  pequeña  laceración  diafragmática?
Las  vísceras  abdominales  se  hernian  desde  la  cavidad  abdominal  de  presión  positiva  hacia  el  espacio  pleural  
de  presión  negativa.  La  morbilidad  de  una  hernia  diafragmática  estrangulada  no  es  trivial,  a  menudo  debido  a  la  demora  
en  el  diagnóstico.  ¡Lo  mejor  es  identificar  el  agujero  en  el  momento  de  la  lesión  inicial!

16.  Junior  O'Flaherty  recibió  un  disparo  en  todo  el  mediastino.  Parece  estable:  ¿necesita  una  operación?

Probablemente  no.  Sorprendentemente,  la  mayoría  de  las  heridas  que  parecen  atravesar  completamente  el  tórax  no  
lesionan  una  estructura  crítica.  De  hecho,  solo  alrededor  del  35%  de  los  pacientes  estables  requieren  exploración.  
Junior  debe  ser  evaluado  con  antecedentes  (¿odinofagia?  ¿ronquera?),  examen  físico  (¿enfisema  cervical  profundo?  
¿hematoma  en  expansión?  ¿extremidad  sin  pulso?)  y  tomografía  computarizada  para  evaluar  la  trayectoria  y  evaluar  si  
hay  lesiones.  Si  el  trayecto  de  la  bala  indica  que  hay  estructuras  críticas  en  riesgo,  puede  ser  necesario  realizar  una  
angiografía,  una  broncoscopia  y  una  esofagoscopia  de  seguimiento.
Machine Translated by Google
120  CAPÍTULO  21  TRAUMA  TORÁCICO  PENETRANTE

17.  ¿Están  justificados  los  antibióticos  profilácticos  para  prevenir  el  empiema  después  de  una  toracostomía  con  
sonda?
El  metanálisis  de  los  estudios  aleatorios  publicados  actualmente  sobre  antibióticos  profilácticos  para  la  toracostomía  
con  sonda  sugiere  un  beneficio.  El  número  de  dosis  requeridas  no  está  claro;  además,  el  uso  en  pacientes  con  lesiones  
multisistémicas  cerradas  puede  cuestionarse  debido  al  riesgo  de  aparición  de  resistencia.

18.  ¿Cuál  es  el  factor  de  riesgo  más  importante  para  el  empiema  postraumático?
hemotórax  persistente.  La  sangre  es  un  medio  de  incubación  excelente  para  las  bacterias;  por  lo  tanto,  la  
evacuación  oportuna  de  sangre  del  espacio  pleural  mediante  toracostomía  con  tubo  o  cirugía  toracoscópica  asistida  por  
video  es  central  en  el  manejo  del  hemotórax  traumático  (¡matar  a  los  insectos!).

19.  ¿Qué  es  una  embolia  aérea  broncovenosa?

Una  embolia  gaseosa  ocurre  cuando  el  gas  a  presión  se  escapa  de  un  bronquio  lacerado  hacia  una  vena  pulmonar  
lacerada  adyacente.  Luego,  el  aire  viaja  hacia  el  lado  izquierdo  del  corazón  y  hacia  las  arterias  coronarias.  La  
presentación  clásica  es  un  paciente  con  una  lesión  torácica  penetrante  que  se  detiene  después  de  la  intubación  y  la  
aplicación  de  ventilación  con  presión  positiva.

20.  ¿Cómo  se  diagnostica  y  trata  la  embolia  aérea  broncovenosa?
El  diagnóstico  se  basa  únicamente  en  la  historia  típica  (consulte  la  pregunta  18).  La  terapia  está  dirigida  a  la  extracción  
de  aire  del  ventrículo  izquierdo  y  las  arterias  coronarias:  posición  de  Trendelenberg  (cabeza  hacia  abajo)  con  el  lado  derecho  
hacia  abajo,  toracotomía  inmediata  y  aspiración  del  vértice  del  ventrículo  izquierdo,  la  raíz  aórtica  y,  en  ocasiones,  las  arterias  
coronarias.

21.  En  una  herida  esofágica  penetrante,  ¿dónde  puede  ser  evidente  el  aire  en  el  examen  físico?

Los  tejidos  subcutáneos  profundos  del  cuello.  En  posición  erguida,  el  aire  del  mediastino  se  diseca  en  un  plano  continuo  
con  la  fascia  cervical  profunda.

22.  ¿Cómo  se  presentan  las  lesiones  traqueobronquiales  penetrantes?
La  laceración  de  la  tráquea  y  los  bronquios  principales  se  presenta  con  enfisema  subcutáneo,  hemoptisis  y  
disnea.  Las  radiografías  de  tórax  revelan  un  neumotórax  o  neumomediastino.
Después  de  una  toracostomía  con  sonda,  la  fuga  continua  de  aire  y  la  falta  de  expansión  del  pulmón  ("pulmón  caído")  
deben  hacer  sospechar  una  lesión  bronquial  importante.

23.  ¿Qué  indica  una  bala  borrosa  en  una  radiografía  de  tórax?
Una  bala  alojada  en  el  miocardio.  El  movimiento  del  corazón  hace  que  la  imagen  sea  borrosa  en  la  radiografía.  Cuidado  con  
la  bala  borrosa.

PUNTOS  CLAVE

1.  La  mayoría  de  los  pacientes  con  un  agujero  en  el  pecho  no  necesitan  una  operación.  Un  tubo  torácico  suele  ser  el
sólo  es  necesario  el  tratamiento  definitivo.

2.  Si  no  hay  neumotórax  después  de  6  horas,  es  poco  probable  que  el  paciente  tenga  un  neumotórax  significativo.
herida  en  el  pecho

3.  Si  un  arma  penetra  en  la  parte  anterior  del  tórax  en  la  caja,  use  un  ultrasonido  para  buscar  sangre  pericárdica.

4.  El  taponamiento  cardíaco  es  lo  que  probablemente  lo  mate  después  de  una  puñalada  en  el  corazón.

5.  Las  lesiones  del  diafragma  son  importantes  de  encontrar,  pero  difíciles  de  detectar  con  precisión.

6.  Si  un  paciente  con  una  herida  de  arma  blanca  en  el  pecho  se  detiene  después  de  la  intubación,  un  aire  broncovenoso
debe  sospecharse  embolia.
Machine Translated by Google
CAPÍTULO  21  TRAUMA  TORÁCICO  PENETRANTE  121

BIBLIOGRAFÍA

1.  Branney  SW,  Moore  EE,  Feldhaus  KM  et  al.:  Análisis  crítico  de  dos  décadas  de  experiencia  con  la  toracotomía  del  departamento  
de  emergencias  posterior  a  una  lesión  en  un  centro  de  trauma  regional.  J  Trauma  45:87­95,  1998.

2.  Cothren  CC,  Moore  EE,  Biffl  WL  et  al.:  Las  técnicas  de  preservación  pulmonar  están  asociadas  con  mejores  resultados
en  comparación  con  la  resección  anatómica  para  lesiones  pulmonares  graves.  J  Trauma  53:483,  2002.

3.  Ibirogba  S,  Nicol  AN,  Navsaria  PH:  Detección  de  tomografía  computarizada  helicoidal  en  hemodinámicamente
pacientes  estables  con  heridas  transmediastínicas  por  arma  de  fuego.  Lesión  38:48,  2007.

4.  Karmy­Jones  R,  Nathens  A,  Jurkovich  GJ  et  al .:  Toracotomía  urgente  y  emergente  para  trauma  torácico  penetrante.  J  Trauma  
56:664,  2004.

5.  Mandal  AK,  Sanusi  M:  Heridas  torácicas  penetrantes:  24  años  de  experiencia.  World  J  Surg  25:1145­1149,  2001.

6.  Nagy  KK,  Lohmann  C,  Kim  DO  et  al .:  Papel  de  la  ecocardiografía  en  el  diagnóstico  de  lesión  cardíaca  penetrante  oculta.  J  Trauma  
38:859­862,  1995.

7.  Rhee  PM,  Foy  H,  Kaufmann  C  et  al .:  Lesiones  cardíacas  penetrantes:  un  estudio  basado  en  la  población.  J  Trauma  45:
366­370,  1998.

8.  Stassen  AA,  Lukan  JK,  España  DA  et  al.:  Reevaluación  de  procedimientos  diagnósticos  para  disparo  transmediastinal
heridas  J  Trauma  53:635­638,  2002.
Machine Translated by Google

TRAUMA  ABDOMINAL  CONTUNDENTE
CAPÍTULO  
22

David  J.  Ciesla,  MD,  MS  y  Ernest  E.  Moore,  MD

1.  ¿Qué  elementos  de  la  historia  son  importantes  en  la  evaluación  de  un  paciente  con
sospecha  de  traumatismo  abdominal  cerrado  (BAT)?
En  primer  lugar,  es  importante  el  mecanismo  de  la  lesión  (p.  ej.,  colisión  de  vehículos  motorizados,  accidente  
automovilístico­peatón,  caída).  En  los  accidentes  automovilísticos,  tenga  en  cuenta  la  posición  de  la  víctima  en  el  
automóvil,  la  velocidad  del  impacto  (alta,  moderada  o  baja),  el  tipo  de  accidente  (impacto  frontal,  lateral  o  trasero,  golpe  
lateral  o  vuelco)  y  el  tipo  de  sujeción.  utilizado  (restricción  de  hombro,  bolsa  de  aire  o  cinturón  de  regazo).  La  información  
sobre  daños  en  el  vehículo,  como  un  parabrisas  roto  o  un  volante  torcido,  puede  generar  sospechas  de  lesiones  
cervicales  y  torácicas.  En  una  caída,  es  importante  tener  en  cuenta  la  distancia  caída  y  el  sitio  del  impacto  anatómico.  
El  aterrizaje  vertical  sobre  los  pies  o  sentado  provoca  un  patrón  diferente  de  lesiones  que  el  aterrizaje  lateral  sobre  un  
costado.  Los  signos  vitales  seriados  y  el  estado  mental  siempre  son  importantes.

2.  ¿Es  precisa  la  exploración  física  en  el  diagnóstico  de  lesión  intraabdominal?
No.  Los  resultados  del  examen  pueden  ser  normales  hasta  en  el  50  %  de  los  pacientes  con  hemorragia  intraabdominal  
aguda.  Los  signos  de  lesión  intraabdominal  incluyen  abrasiones  y  contusiones  en  la  parte  inferior  del  tórax  y  el  
abdomen;  enfisema  subcutáneo  o  fractura  costal  palpable;  fractura  pélvica  clínicamente  evidente;  dolor  abdominal,  
sensibilidad,  protección  o  rigidez;  sangre  en  la  orina  o  meato  uretral;  próstata  elevada  o  sangre  en  el  examen  rectal;  y  
hematuria  microscópica.

3.  ¿Qué  órganos  se  lesionan  con  más  frecuencia  en  BAT?

Hígado,  50% Colón,  5%

Bazo,  40% Duodeno,  5%

Mesenterio,  10% Vasculares,  4%

Urológico,  10% Estómago,  2%

Páncreas,  10% Vesícula  biliar,  2%

Intestino  delgado,  10%

4.  ¿Qué  estudios  de  diagnóstico  son  útiles  en  BAT?
1.  Ultrasonido:  identifica  de  forma  fiable  el  líquido  peritoneal  (sangre)  y  el  líquido  pericárdico,  pero  puede  pasar  por  alto  hasta  
el  25  %  de  las  lesiones  aisladas  de  órganos  sólidos.
2.  Tomografía  computarizada  (TC):  identifica  la  presencia  y  la  gravedad  de  un  órgano  sólido
(hígado  y  bazo),  detecta  aire  y  fluidos  intraabdominales  (sangre,  mucosidad,  orina)  y  ayuda  en  la  evaluación  
de  fracturas  pélvicas.  La  tomografía  computarizada  también  puede  identificar  lesiones  intestinales,  pancreáticas,  
renales  y  vesicales.  La  sensibilidad  y  la  especificidad  de  las  tomografías  computarizadas  para  diagnosticar  
lesiones  en  BAT  continúan  mejorando  con  los  escáneres  de  nueva  generación.

122
Machine Translated by Google
CAPÍTULO  22  TRAUMATISMO  ABDOMINAL  CERDO  123

3.  Lavado  peritoneal  de  diagnóstico  (DPL):  DPL  muy  positivo  (>10  ml  de  sangre  devuelta  por  aspiración  
del  catéter)  indica  hemoperitoneo  significativo.  Un  recuento  celular  positivo  después  de  la  infusión  de  1  
L  de  líquido  cristaloide  (>100  glóbulos  rojos/mm3 ,  presencia  de  bilis  o  fibras)  indica  
intraabdominal,  
hemorragia   lesión  
de  víscera  hueca  o  lesión  del  sistema  hepatobiliar.  El  líquido  de  lavado  que  sale  a  través  de  un  tubo  
torácico  o  un  catéter  urinario  indica  una  lesión  diafragmática  o  vesical.

5.  ¿Cómo  ha  cambiado  la  evaluación  inicial  de  BAT  la  disponibilidad  de  ultrasonido?
La  ecografía  abdominal  enfocada  en  el  examen  de  trauma  (FAST)  ha  suplantado  en  gran  medida  al  DPL.  El  
examen  FAST  se  puede  realizar  en  un  paciente  hemodinámicamente  inestable  durante  el  examen  secundario  
temprano  con  transferencia  inmediata  a  la  sala  de  operaciones  (OR)  cuando  se  identifica  hemoperitoneo.  La  
tomografía  computarizada  es  segura  en  el  paciente  hemodinámicamente  estable.  La  DPL  sigue  siendo  útil  
cuando  la  ecografía  (US)  es  equívoca  o  no  está  disponible.  DPL  también  es  útil  para  evaluar  la  presencia  de  
lesión  de  víscera  hueca  en  casos  en  los  que  hay  líquido  libre  presente  en  la  tomografía  computarizada  en  
ausencia  de  lesión  de  órgano  sólido.

6.  ¿Cómo  se  diagnostica  la  lesión  de  órganos  huecos?
La  lesión  de  órganos  huecos  suele  detectarse  mediante  exploración  física  (peritonitis  en  pacientes  
despiertos)  o  en  el  momento  de  la  laparotomía  para  controlar  la  hemorragia.  Los  hallazgos  de  la  TC  incluyen  
líquido  peritoneal  sin  lesión  de  órganos  sólidos,  extravasación  de  contraste  oral  a  la  cavidad  peritoneal  y  aire  libre  
intraabdominal.  Los  signos  sugestivos  incluyen  hebras  mesentéricas  y  hematomas.  Los  resultados  del  lavado  
peritoneal  que  sugieren  lesión  de  órganos  huecos  incluyen  niveles  elevados  de  amilasa,  fosfatasa  alcalina  o  
bilirribo  y  la  presencia  de  partículas.

7.  ¿Cuáles  son  las  indicaciones  de  intervención  urgente  en  un  paciente  con  TAM?
Cualquier  paciente  hemodinámicamente  inestable  que  muestre  hemoperitoneo  significativo  (por  US  o  DPL)  
requiere  laparotomía  de  emergencia.  Otras  indicaciones  para  la  laparotomía  urgente  incluyen  aire  libre  
intraabdominal  y  evidencia  de  lesión  de  víscera  hueca.

8.  ¿Cómo  afecta  el  tiempo  en  el  departamento  de  emergencias  (ED)  la  mortalidad  de  los  pacientes  que  
requieren  una  operación  de  emergencia  para  BAT?
La  probabilidad  de  muerte  por  trauma  está  relacionada  tanto  con  la  extensión  de  la  hipotensión  como  con  
el  intervalo  desde  el  momento  de  la  lesión  hasta  la  cirugía  definitiva.  Se  incurre  en  un  aumento  estimado  de  la  
mortalidad  del  1%  por  cada  3  minutos  que  pasan  en  el  departamento  de  emergencias  (ED)  hasta  90  minutos.

9.  ¿Cuál  es  el  papel  de  la  embolización  angiográfica?
La  embolización  angiográfica  puede  ser  eficaz  para  el  control  de  la  hemorragia  en  pacientes  
hemodinámicamente  estables.  Los  sitios  de  embolización  favorables  incluyen  lesiones  en  el  hígado,  el  bazo  
y  los  riñones;  arterias  lumbares  con  hemorragia  retroperitoneal;  y  vasos  sanguíneos  pélvicos  asociados  con  fractura  pélvica.

10.  ¿Qué  es  el  ''ciclo  de  las  vísceras  sangrientas''?
El  ciclo  de  las  vísceras  sangrientas  es  un  síndrome  de  hipotermia,  acidosis  y  coagulopatía  que  ocurre  con  shock  
hemorrágico  profundo  y  transfusión  masiva.  Representa  una  cascada  circular  de  eventos  en  los  que  el  shock  
hemorrágico  grave  acompañado  de  insuficiencia  metabólica  provoca  una  coagulopatía  que  exacerba  el  sangrado  
adicional.

11.  ¿Qué  es  una  laparotomía  por  etapas  o  abreviada  (cirugía  de  control  de  daños)?
La  laparotomía  por  etapas  se  termina  antes  de  que  se  completen  todos  los  procedimientos  definitivos  con  la  
intención  de  regresar  al  quirófano  para  completar  la  operación  en  un  momento  posterior  (y  más  seguro).  El  
propósito  de  este  enfoque  es  retrasar  el  estrés  quirúrgico  adicional  hasta  que  el  paciente  se  encuentre  en  un  
estado  fisiológico  más  favorable.  Los  objetivos  de  la  operación  inicial  son  (1)  detener  el  sangrado  y  corregir  la  
coagulopatía;  (2)  limitar  la  contaminación  peritoneal  y  la  respuesta  inflamatoria  secundaria  (para  controlar  el  
derrame  gastrointestinal  [GI]);  y  (3)  encerrar  el  contenido  abdominal  para  proteger  las  vísceras  y  limitar  la  pérdida  
de  calor,  líquidos  y  proteínas  de  un  abdomen  abierto.
Machine Translated by Google
124  CAPÍTULO  22  TRAUMA  ABDOMINAL  CONTUNDENTE

12.  ¿Cuándo  se  utiliza  la  laparotomía  por  etapas  en  pacientes  traumatizados?
  Incapacidad  para  lograr  la  hemostasia  debido  a  una  coagulopatía  recalcitrante  (comprimir  la  hemorragia).
  Lesión  venosa  mayor  inaccesible  (lesión  de  la  cava  retrohepática).
  Demanda  de  control  de  una  lesión  extraabdominal  (p.  ej.,  craneal  o  torácica)  potencialmente  mortal.
  Imposibilidad  de  cerrar  la  incisión  abdominal  por  edema  visceral  extenso.
  Necesidad  de  reevaluar  el  contenido  abdominal  debido  a  viabilidad  cuestionable  en  el  momento  de  la
operación  inicial.

PUNTOS  CLAVE:  MODALIDADES  DE  DIAGNÓSTICO  ÚTILES

1.  Las  encuestas  primarias  y  secundarias  son  cruciales,  pero  se  requieren  más  pruebas  de  diagnóstico  en
la  mayoría  de  los  pacientes.

2.  RÁPIDO:  identifica  de  manera  confiable  el  líquido  intraabdominal  e  intrapericárdico,  pero  es  deficiente  en  la  víscera  hueca
evaluación.

3.  DPL:  eficaz  para  la  evaluación  del  hemoperitoneo  y  un  complemento  útil  junto  con  FAST
examen.

4.  TC:  excelente  modalidad  con  99,97%  de  valor  predictivo  negativo  para  BAT.

SITIO  WEB

www.east.org/tpg/bluntabd.pdf

BIBLIOGRAFÍA

1.  Clarke  JR,  Trooskin  SZ,  Doshi  PJ  et  al.:  El  tiempo  hasta  la  laparotomía  por  hemorragia  intraabdominal  a  causa  de  un  traumatismo  
afecta  la  supervivencia  para  retrasos  de  hasta  90  minutos.  J  Trauma  52:420,  2002.

2.  Davis  KA,  Fabian  TC,  Croce  MA  et  al.:  Éxito  mejorado  en  el  manejo  de  lesiones  esplénicas  contusas:  embolización
de  pseudoaneurismas  de  la  arteria  esplénica.  J  Trauma  44:1008,  1998.

3.  Peitzman  AB,  Harbrecht  BG,  Rivera  L  et  al.:  Fracaso  de  la  observación  de  lesión  esplénica  contundente  en  adultos:  variabilidad  en
práctica  y  consecuencias  adversas.  J  Am  Coll  Surg  201:179,  2005.

4.  Rodríguez  C,  Barone  JE,  Willbanks  TO  et  al .:  Líquido  libre  aislado  en  tomografía  computarizada  en  trauma  abdominal  cerrado:  
una  revisión  sistemática  de  incidencia  y  manejo.  J  Trauma  53:79,  2002.

5.  Stengal  D,  Bauwens  K,  Sehouli  J  et  al.:  Algoritmos  basados  en  ultrasonido  de  emergencia  para  diagnosticar  dolor  abdominal  cerrado
trauma.  Base  de  datos  Cochrane  Syst  Rev  18:CD004446,  2005.

6.  Tien  H,  Spencer  F,  Tremblay  L  et  al .:  Muertes  evitables  por  hemorragia  en  un  centro  de  trauma  canadiense  de  nivel  1.
J  Trauma  62:142,  2007.
Machine Translated by Google

TRAUMA  ABDOMINAL  PENETRANTE
CAPÍTULO  
23

C.  Clay  Cothren,  MD  y  Ernest  E.  Moore,  MD

1.  ¿Por  qué  la  evaluación  es  diferente  para  pacientes  con  heridas  de  arma  blanca  versus  bala?
heridas?
Aunque  un  tercio  de  las  heridas  de  arma  blanca  (SW)  en  el  abdomen  anterior  no  penetran  el  
peritoneo,  el  80%  de  las  heridas  de  bala  (GSW)  violan  el  peritoneo.  Además,  de  las  heridas  que  penetran  
en  el  peritoneo,  el  95%  de  los  GSW  tienen  lesiones  viscerales  o  vasculares  asociadas,  mientras  que  
solo  un  tercio  de  los  SW  las  tienen  (v .  fig.  23­1).

Hemodinámicamente  inestable  o  peritonitis

Anterior
Abdomen
PENETRANTE
ABDOMINAL
GSW RUQ
TRAUMA

+
Tangencial Connecticut

Espalda/Flanco Escanear
hemodinámicamente Operando
Estable Habitación

Espalda/Flanco

SUDOESTE LPD
AASW  con contra

Connecticut
+
+  LWE Escanear
AASW  =  herida  de  arma  blanca  en  el  abdomen  anterior
contra
LWE  =  exploración  local  de  heridas
De  serie
DPL  =  lavado  peritoneal  diagnóstico
Exámenes
TC  =  tomografía  computarizada

Figura  23­1.  Algoritmo  de  manejo  para  pacientes  con  trauma  abdominal  penetrante.

2.  ¿Cuáles  son  las  indicaciones  de  la  laparotomía  de  emergencia  en  pacientes  con  heridas  de  arma  blanca?
La  hipotensión,  la  peritonitis  y  los  signos  obvios  de  lesión  visceral  abdominal  (hematemesis,  
proctorragia,  palpación  del  defecto  diafragmático  en  la  inserción  del  tubo  torácico,  evidencia  radiológica  de  
lesión  del  tracto  gastrointestinal  [GI])  obligan  a  la  exploración  inmediata.  La  mayoría  de  las  autoridades  
también  abogó  por  la  pronta  exploración  en  busca  de  evisceración  omental  o  intestinal.

3.  ¿Cuáles  son  las  indicaciones  de  laparotomía  inmediata  en  pacientes  con  arma  de  fuego?
heridas?
Debido  a  la  alta  incidencia  de  lesiones  viscerales,  está  indicada  la  exploración  temprana  de  todas  las  GSW  
que  violan  el  peritoneo.  La  excepción  es  el  traumatismo  penetrante  aislado  en  el  cuadrante  superior  derecho;  
en  pacientes  hemodinámicamente  estables  con  trayectoria  de  bala  limitada  al  hígado  por  tomografía  
computarizada  (TC),  se  puede  considerar  la  observación  no  quirúrgica.  Del  mismo  modo,  en  obesos

125
Machine Translated by Google
CAPÍTULO  23  TRAUMA  ABDOMINAL  PENETRANTE  127

(ver  Capítulo  27),  y  aquellos  con  proximidad  a  las  vías  urinarias  deben  evaluarse  con  tomografía  computarizada  (ver  
Capítulo  30).

7.  ¿Cuál  es  la  diferencia  entre  una  herida  penetrante  en  el  abdomen  anterior
contra  el  flanco  o  la  espalda?

Debido  a  que  la  incidencia  de  lesiones  es  mayor  para  las  heridas  anteriores  y  las  lesiones  están  dentro  de  la  cavidad  
peritoneal,  la  evaluación  diagnóstica  difiere.

8.  ¿Cómo  se  evalúa  una  herida  de  arma  blanca  en  el  abdomen  anterior  en  pacientes  asintomáticos?
pacientes?
El  primer  paso  es  la  exploración  local  de  la  herida  para  determinar  la  penetración  peritoneal.  Si  el  tracto  claramente  
termina  superficialmente,  por  encima  de  la  fascia,  no  se  requiere  evaluación  adicional  y  el  paciente  es  dado  de  alta  del  
departamento  de  emergencias  (ED).  Si  se  penetra  la  fascia  o  se  viola  el  peritoneo,  se  justifica  una  evaluación  adicional.  El  
enfoque  de  diagnóstico  óptimo  sigue  siendo  objeto  de  debate  entre  el  examen  en  serie,  el  lavado  peritoneal  de  diagnóstico  
(DPL)  y  la  tomografía  computarizada.

9.  ¿Qué  constituye  un  resultado  positivo  de  lavado  peritoneal  de  diagnóstico  después  de  un  traumatismo  
penetrante?
Una  punción  macroscópicamente  positiva  (aspiración  de  >10  ml;  de  sangre  o  aspiración  de  contenido  gastrointestinal  o  
biliar)  obliga  a  una  exploración  inmediata.  Un  resultado  de  aspiración  inicial  negativo  es  seguido  por  la  instilación  de  1000  ml  
de  solución  salina  (15  ml/kg  en  niños)  en  el  abdomen  a  través  de  un  catéter  de  diálisis,  seguido  del  drenaje  por  gravedad  del  
líquido  de  regreso  a  la  bolsa  de  solución  salina.  El  hallazgo  de  >  100  000  glóbulos  rojos  (RBC)  por  ml,  >  500  ml  de  glóbulos  
blancos  (WBC),  amilasa  >  20  UI/L,  fosfatasa  alcalina  >  3  UI/L  o  nivel  elevado  de  bilirrubina  también  son  indicaciones  para  la  
exploración.

10.  ¿Cómo  se  evalúan  las  heridas  de  arma  blanca  en  el  flanco  y  la  espalda  en  pacientes  asintomáticos?
pacientes?
La  incidencia  de  lesiones  significativas  es  del  10%  para  SW  en  la  espalda  y  del  25%  para  SW  en  el  flanco.  Los  SW  
del  flanco  y  la  espalda  deben  someterse  a  una  TC  de  triple  contraste  para  detectar  lesiones  retroperitoneales  ocultas  
en  el  colon,  el  duodeno  y  las  vías  urinarias.  El  aspecto  más  valioso  de  la  tomografía  computarizada  es  determinar  la  
trayectoria  de  la  herida.

11.  ¿Cómo  se  evalúa  una  herida  de  arma  blanca  en  la  parte  inferior  del  tórax?

Las  heridas  de  arma  blanca  en  la  parte  inferior  del  tórax  se  asocian  con  lesión  visceral  abdominal  en  el  15  %  de  los  
casos,  mientras  que  las  heridas  de  bala  en  la  parte  inferior  del  tórax  se  asocian  con  lesión  visceral  abdominal  en  casi  el  
50  %  de  los  casos.  La  parte  inferior  del  tórax  se  define  como  el  área  entre  la  línea  del  pezón  (cuarto  espacio  intercostal)  
en  la  parte  anterior,  la  punta  de  la  escápula  (séptimo  espacio  intercostal)  en  la  parte  posterior  y  los  márgenes  costales  en  
la  parte  inferior.  Debido  a  que  el  diafragma  alcanza  el  cuarto  espacio  intercostal  durante  la  espiración,  los  órganos  
abdominales  corren  el  riesgo  de  lesionarse  incluso  después  de  lo  que  parece  ser  una  herida  «torácica  aislada».  Por  lo  
tanto,  las  heridas  en  la  parte  inferior  del  tórax  también  deben  tratarse  como  heridas  abdominales  para  descartar  una  lesión  
intraabdominal.  Debe  descartarse  una  lesión  oculta  del  diafragma  en  pacientes  con  SW  en  la  parte  inferior  del  tórax.  Los  
pacientes  que  se  someten  a  una  evaluación  de  DPL  tienen  diferentes  valores  de  corte  de  laboratorio  que  las  heridas  de  
arma  blanca  abdominales  anteriores  estándar.  Un  recuento  de  glóbulos  rojos  de  más  de  10  000/mL  se  considera  positivo  y  
una  indicación  para  laparotomía,  mientras  que  los  pacientes  con  un  recuento  de  glóbulos  rojos  DPL  entre  1000/mL  y  10  000/
mL  deben  someterse  a  laparoscopia  o  toracoscopia.

12.  ¿Qué  pacientes  con  heridas  abdominales  por  arma  de  fuego  son  candidatos  para  tratamiento  conservador?

Pacientes  hemodinámicamente  estables  con  tractos  subcutáneos  en  misil  tangenciales  o  aquellos  con  trauma  hepático  
aislado.  El  manejo  selectivo  de  los  GSW  en  la  espalda  y  el  flanco  se  basa  en  los  resultados  de  la  tomografía  computarizada  
de  triple  contraste.
Machine Translated by Google
128  CAPÍTULO  23  TRAUMA  ABDOMINAL  PENETRANTE

13.  Si  está  indicada  la  exploración  quirúrgica  abdominal,  ¿cuál  es  el  abordaje  general?
Una  incisión  abdominal  en  la  línea  media  proporciona  una  entrada  rápida  y  una  amplia  exposición;  puede  extenderse  como  
una  esternotomía  mediana  para  acceder  al  tórax.  La  aorta  debe  palparse  justo  debajo  del  diafragma  para  evaluar  la  presión  
arterial  (PA)  del  paciente.  La  sangre  líquida  y  coagulada  se  evacua  con  múltiples  almohadillas  de  laparotomía  y  succión  para  
identificar  la(s)  fuente(s)  principal(es)  de  sangrado  activo.
Después  de  localizar  el  origen  de  la  hemorragia,  se  utiliza  la  oclusión  digital  directa  (lesión  vascular)  o  el  relleno  de  
almohadillas  de  laparotomía  (lesión  de  órganos  sólidos)  para  controlar  la  hemorragia.  Las  lesiones  viscerales  huecas  se  
aíslan  temporalmente  con  pinzas  que  no  aplastan  o  se  suturan  rápidamente.  Todo  el  abdomen  se  explora  sistemáticamente  
antes  de  realizar  reparaciones  extensas  para  que  las  lesiones  puedan  priorizarse  para  el  tratamiento  definitivo.

PUNTOS  CLAVE

1.  Las  GSW  en  el  abdomen  generalmente  requieren  exploración  quirúrgica;  una  excepción  podría  ser  correcta
Heridas  en  el  cuadrante  superior  con  lesión  hepática  aislada.

2.  Después  de  una  SW,  los  pacientes  con  hipotensión,  peritonitis  o  evisceración  deben  someterse  a
exploración  operativa.

3.  Los  SW  abdominales  anteriores  en  pacientes  estables  se  evalúan  inicialmente  con  herida  local
exploración;  la  penetración  del  peritoneo  requiere  una  evaluación  adicional  (examen  en  serie,  DPL  o  tomografía  
computarizada).

4.  Los  SW  de  flanco  y  espalda  en  pacientes  estables  se  evalúan  con  tomografía  computarizada  de  triple  contraste.

CONTROVERSIA

14.  ¿Cuál  es  el  papel  de  la  laparoscopia  y  la  toracoscopia  tras  un  traumatismo  abdominal  penetrante?

Aunque  es  una  modalidad  diagnóstica  intrigante  con  capacidades  terapéuticas  adicionales,  hasta  ahora  la  laparoscopia  parece  
tener  una  aplicación  limitada  después  de  un  traumatismo.  Con  la  excepción  de  la  sospecha  de  lesión  diafragmática,  una  lesión  
aislada  de  un  órgano  sólido  o  la  evaluación  de  penetración  peritoneal,  la  laparoscopia  aún  tiene  que  demostrar  ventajas  sobre  el  
algoritmo  descrito  anteriormente.  La  posibilidad  de  que  se  pasen  por  alto  las  lesiones,  la  evaluación  deficiente  del  retroperitoneo  
y  el  costo  son  los  principales  inconvenientes.

BIBLIOGRAFÍA

1.  Biffl  WL,  Cothren  CC,  Brasel  KJ  et  al.:  Un  estudio  prospectivo  observacional  multicéntrico  del  manejo  óptimo  de  pacientes  con  heridas  de  
arma  blanca  en  el  abdomen  anterior.  J  Trauma  64:250,  2008.

2.  Boyle  EM  Jr,  Maier  RV,  Salazar  JD  et  al .:  Diagnóstico  de  lesiones  después  de  heridas  de  arma  blanca  en  la  espalda  y  el  costado.  J  Trauma
42:260,  1997.

3.  Chiu  WC,  Shanmuganathan  K,  Mirvis  SE  et  al .:  Determinación  de  la  necesidad  de  laparotomía  en  el  traumatismo  penetrante  del  
torso:  un  estudio  prospectivo  que  utiliza  tomografía  computarizada  abdominopélvica  mejorada  con  triple  contraste.
J  Trauma  51:860­868,  2001.

4.  Demetriades  D,  Hadjizacharia  P,  Constantinou  C  et  al.:  Manejo  no  quirúrgico  selectivo  de  penetrantes
Lesiones  de  órganos  sólidos  abdominales.  Ann  Surg  244:620,  2006.

5.  Freeman  RK,  Al­Dossari  G,  Hutcheson  KA  et  al .:  Indicaciones  para  el  uso  de  cirugía  toracoscópica  asistida  por  video  para  diagnosticar  
lesiones  diafragmáticas  después  de  un  traumatismo  torácico  penetrante.  Ann  Thorac  Surg  72:342­347,  2001.
Machine Translated by Google
126  CAPÍTULO  23  TRAUMA  ABDOMINAL  PENETRANTE

Si  se  cree  que  el  GSW  es  tangencial  a  través  de  los  tejidos  subcutáneos,  la  tomografía  computarizada  
puede  delinear  el  tracto  y  excluir  la  violación  peritoneal.  La  laparoscopia  es  otra  opción  para  evaluar  la  
penetración  peritoneal  y  puede  ir  seguida  de  una  laparotomía  para  reparar  las  lesiones.  En  caso  de  duda,  
siempre  es  más  seguro  explorar  el  abdomen  que  equivocarse.

4.  ¿Cuándo  está  indicada  la  toracotomía  en  el  servicio  de  urgencias  por  una  herida  abdominal  
penetrante?
La  toracotomía  de  reanimación  se  debe  considerar  cuando  un  paciente  presenta  un  paro  cardíaco  
(reanimación  cardiopulmonar  [RCP]  <15  minutos)  o  con  hipotensión  profunda  (presión  arterial  sistólica  
[PAS]  <70  mm  Hg)  que  es  refractaria  a  la  reanimación  inicial.  Después  de  la  toracotomía  anterolateral,  
se  pinza  la  aorta  descendente  para  disminuir  la  hemorragia  subdiafragmática  y  mejorar  la  perfusión  
coronaria  y  cerebral.  Si  es  necesario,  se  realiza  un  masaje  cardíaco  abierto.

5.  ¿Cuáles  son  los  elementos  clave  de  la  encuesta  secundaria?
Examinar  al  paciente  sistemáticamente;  es  fácil  pasar  por  alto  las  lesiones  sincrónicas.  El  examen  incluye  
buscar  sitios  adicionales  de  entrada  o  salida;  asegúrese  de  examinar  minuciosamente  la  axila  y  el  perineo,  
ya  que  las  heridas  pueden  quedar  ocultas  en  los  pliegues  de  la  piel.  Se  debe  evaluar  si  hay  sangre  en  los  
sistemas  GI,  genitourinario  (GU)  y  ginecológico,  y  se  deben  considerar  los  mecanismos  cerrados  de  lesión  
asociados;  algunos  pacientes  desafortunados  son  agredidos  tanto  con  cuchillos  como  con  puños  (v .  fig.  
23­2).

Figura  23­2.  Un  ejemplo  de  cómo  la  trayectoria  de  una  bala  a  través  de  un  cuerpo  contorsionado  puede  
producir  confusión  cuando  se  examina  al  paciente  en  el  servicio  de  urgencias.  Se  encontrará  una  herida  de  
entrada  en  la  parte  superior  del  brazo  izquierdo  y  una  herida  de  salida  en  la  cara  medial  de  la  rodilla  derecha.
La  bala  podría  haber  dañado  cualquier  estructura  entre  estas  dos  heridas  cuando  el  cuerpo  del  paciente  
estaba  contorsionado.

6.  ¿Cuáles  son  los  estudios  iniciales  adecuados?
En  pacientes  estables,  una  radiografía  de  tórax  excluye  el  hemotórax  o  el  neumotórax  y  determina  la  
posición  de  los  catéteres  intravenosos  (IV)  (p.  ej.,  tubos  endotraqueales,  nasogástricos  [NG]  y  pleurales).  
Las  radiografías  abdominales  biplanares  son  útiles  para  localizar  cuerpos  extraños  retenidos  (es  decir,  
balas)  y  pueden  revelar  neumoperitoneo.  La  ubicación  de  las  heridas  de  entrada  y  salida,  identificadas  
con  un  marcador  radiopaco  aplicado  a  la  piel,  puede  ser  útil  para  determinar  la  trayectoria  de  los  misiles.  
Lesiones  en  la  proximidad  del  recto  sigmoidoscopia  obligada
Machine Translated by Google
CAPÍTULO  23  TRAUMA  ABDOMINAL  PENETRANTE  129

6.  Henneman  PL,  Marx  JA,  Moore  EE  et  al .:  Lavado  peritoneal  diagnóstico:  precisión  en  la  predicción  necesaria
laparotomía  después  de  un  trauma  cerrado  y  penetrante.  J  Trauma  30:1345­1355,  1990.

7.  McAnena  OJ,  Marx  JA,  Moore  EE:  determinaciones  de  enzimas  de  lavado  peritoneal  después  de  un  traumatismo  abdominal  
cerrado  y  penetrante.  J  Trauma  31:1161­1164,  1991.

8.  Moore  EE,  Marx  JA:  Heridas  abdominales  penetrantes:  una  justificación  para  la  laparotomía  exploradora.  JAMA  
253:2705­2708,  1985.

9.  Reber  PU,  Schmied  B,  Seiler  CA  et  al.:  Lesiones  diafragmáticas  perdidas  y  sus  secuelas  a  largo  plazo.  J  Trauma
44:183­188,  1998.

10.  Simon  RJ,  Rabin  J,  Kuhls  D:  Impacto  del  mayor  uso  de  la  laparoscopia  en  las  tasas  de  laparotomía  negativa  después  de  
un  traumatismo  penetrante.  J  Trauma  53:297­302,  2002.
Machine Translated by Google

TRAUMA  HEPÁTICO  Y  BILIAR
CAPÍTULO  
24

Jarrod  N.  Keith,  MD  y  Ernest  E.  Moore,  MD

1.  ¿Con  qué  frecuencia  se  lesiona  el  hígado  en  un  traumatismo?
El  hígado  es  grande  y  central,  por  lo  que  es  vulnerable  a  los  traumatismos  cerrados  y  un  blanco  fácil  para  las  
heridas  penetrantes.

2.  ¿El  hígado  y  el  bazo  responden  de  manera  similar  a  las  lesiones?
No.  El  hígado  tiene  una  capacidad  única  para  establecer  hemostasia  espontánea  incluso  con  
lesiones  extensas.  Por  esta  razón,  la  mayoría  de  las  lesiones  hepáticas  en  pacientes  hemodinámicamente  
estables  pueden  tratarse  sin  cirugía.  Por  el  contrario,  muchas  fracturas  esplénicas  siguen  sangrando;  por  lo  
tanto,  un  mayor  porcentaje  de  pacientes  con  lesiones  esplénicas  requieren  intervención  quirúrgica.

3.  ¿Cuáles  son  los  determinantes  de  la  mortalidad  después  de  una  lesión  hepática  aguda?
El  mecanismo  de  lesión,  el  grado  de  lesión  y  los  órganos  abdominales  asociados  lesionados  determinan  la  
mortalidad.  La  mortalidad  por  heridas  de  arma  blanca  (SW)  en  el  hígado  es  del  2%;  por  heridas  de  bala  (GSW),  8%;  
y  por  traumatismos  contusos,  el  15%.  La  tasa  de  mortalidad  de  las  lesiones  hepáticas  aisladas  de  grado  III  es  del  2%;  
para  grado  IV,  20%;  y  para  el  grado  V,  65%.  Las  lesiones  de  la  vena  cava  retrohepática  conllevan  tasas  de  mortalidad  
del  80%  para  traumatismos  penetrantes  y  del  95%  para  traumatismos  cerrados.

4.  ¿Qué  antecedentes  y  signos  físicos  sugieren  una  lesión  hepática  aguda?
Se  debe  suponer  que  cualquier  paciente  que  sufra  un  traumatismo  abdominal  contuso  (BAT)  con  
hipotensión  tiene  una  lesión  hepática  hasta  que  se  demuestre  lo  contrario.  Los  signos  específicos  que  aumentan  la  
probabilidad  de  lesión  hepática  son  contusión  en  la  parte  inferior  derecha  del  tórax,  fractura  de  las  costillas  inferiores  
derechas  (especialmente  fracturas  posteriores  de  las  costillas  9  a  12)  y  lesiones  penetrantes  en  la  parte  inferior  
derecha  del  tórax  (por  debajo  del  cuarto  espacio  intercostal,  flanco ,  y  parte  superior  del  abdomen).  Los  signos  físicos  
de  hemoperitoneo  pueden  estar  ausentes  hasta  en  un  tercio  de  los  pacientes  con  lesión  hepática  importante.

5.  ¿Qué  pruebas  diagnósticas  son  útiles  para  confirmar  una  lesión  hepática  aguda?
El  lavado  peritoneal  de  diagnóstico  (DPL)  es  sensible  para  el  hemoperitoneo  (99%),  pero  no  específico  
para  la  lesión  hepática.  La  ecografía  (US)  es  muy  sensible  para  identificar  >200  ml  de  líquido  
intraperitoneal.  No  es  invasivo  y  puede  repetirse  a  intervalos  frecuentes,  pero  es  relativamente  pobre  para  
estadificar  las  lesiones  hepáticas.  Actualmente,  la  tomografía  computarizada  (TC)  abdominal  se  utiliza  en  
pacientes  hemodinámicamente  estables  que  son  candidatos  para  tratamiento  no  quirúrgico.  La  clasificación  
de  las  lesiones  hepáticas  por  tomografía  computarizada  es  útil  para  determinar  la  tasa  de  éxito  del  tratamiento  no  
quirúrgico  porque  es  más  probable  que  las  lesiones  de  mayor  grado  requieran  intervención.

6.  ¿Cuál  es  el  papel  de  la  angiografía  hepática?
La  embolización  selectiva  de  la  arteria  hepática  es  una  terapia  eficaz  para  el  sangrado  arterial  hepático,  tanto  
para  evitar  la  cirugía  como  para  el  sangrado  postoperatorio  recurrente.  Se  debe  considerar  la  embolización  para  
pacientes  con  extravasación  activa  de  contraste  en  el  peritoneo  observada  en  la  tomografía  computarizada  porque  es  
menos  probable  que  se  taponen.

130
Machine Translated by Google
CAPÍTULO  24  TRAUMA  HEPÁTICO  Y  BILIAR  131

ANATOMÍA  QUIRÚRGICA  DEL  HÍGADO

7.  ¿Cuántos  lóbulos  anatómicos  hay  en  el  hígado?  ¿Cuál  es  su  topografía?
¿Perímetro?
El  hígado  se  divide  en  dos  lóbulos  anatómicos,  el  derecho  y  el  izquierdo.  Su  límite  se  encuentra  en  un  
plano  oblicuo  que  se  extiende  desde  la  fosa  de  la  vesícula  biliar  por  delante  hasta  la  vena  cava  inferior  
(VCI)  por  detrás.  Las  tres  venas  hepáticas  definen  la  división  entre  los  segmentos  lobulares  y  los  planos  de  
resección  quirúrgica.  Los  segmentos  lobares  se  numeran  del  I  al  VIII,  según  la  nomenclatura  de  Couinaud  
(v .  fig.  24­1).

Figura  24­1.  La  división  funcional  del  hígado  y  los  segmentos  
según  la  nomenclatura  de  Couinaud.  (Tomado  de  Bismuth  H:  
Surgical  Anatomy  and  anatomical  Surgery  of  the  liver,  World  J  
Surg  6:6,  1982,  con  autorización.)

8.  ¿Cuál  es  el  suministro  de  sangre  al  hígado  y  la  contribución  relativa  de  cada  estructura  a  la  oxigenación  
hepática?
La  arteria  hepática  suministra  aproximadamente  el  30%  del  flujo  sanguíneo  al  hígado  y  el  50%  de  su  
suministro  de  oxígeno.  La  vena  porta  proporciona  el  70%  del  flujo  sanguíneo  del  hígado  y  el  50%  de  su  oxígeno.
La  importancia  relativa  del  flujo  arterial  en  pacientes  cirróticos  es  mayor;  por  lo  tanto,  no  se  recomienda  la  
ligadura  de  la  arteria  hepática  en  pacientes  con  cirrosis.

9.  ¿Cuáles  son  las  variaciones  más  comunes  en  el  suministro  arterial  hepático  a  la  derecha?
y  los  lóbulos  izquierdos  del  hígado?
En  la  mayoría  de  las  personas,  la  arteria  hepática  común  se  origina  en  el  eje  celíaco  y  se  divide  en  ramas  
arteriales  hepáticas  derecha  e  izquierda  dentro  de  la  porta  hepatis.  Aproximadamente  el  15  %  de  las  
personas  tienen  una  arteria  hepática  derecha  reemplazada  (suministro  arterial  único  al  lóbulo  derecho)  que  
se  origina  en  la  arteria  mesentérica  superior  (SMA).  Una  arteria  hepática  derecha  reemplazada  siempre  irriga  
una  arteria  cística;  por  lo  tanto,  la  ligadura  debe  ir  seguida  de  colecistectomía.  Una  arteria  hepática  izquierda  
reemplazada  (aproximadamente  el  15%  de  las  personas)  surge  de  la  arteria  gástrica  izquierda;  puede  ser  el  
único  suministro  de  sangre  al  lóbulo  izquierdo  o  puede  contribuir  al  suministro  de  sangre  junto  con  una  arteria  
hepática  izquierda  normal.  En  el  5%  de  las  personas,  el  riego  arterial  hepático  no  surge  del  eje  celíaco.
Machine Translated by Google
132  CAPÍTULO  24  TRAUMA  HEPÁTICO  Y  BILIAR

En  estas  personas,  las  arterias  hepáticas  derecha  e  izquierda  se  reemplazan  o  un  solo  tronco  hepático  principal  
se  deriva  de  la  SMA.

10.  ¿Qué  es  el  drenaje  venoso  del  hígado?
Las  venas  hepáticas  derecha,  media  e  izquierda  son  las  principales  venas  tributarias  y  entran  en  la  VCI  por  debajo  
del  hemidiafragma  derecho.

MANEJO  OPERATIVO  DE  LA  LESIÓN  HEPÁTICA

11.  ¿Cómo  se  clasifican  las  lesiones  hepáticas  agudas?
Las  heridas  hepáticas  generalmente  se  clasifican  en  una  escala  de  I  a  VI  según  la  profundidad  de  la  laceración  
parenquimatosa  y  la  afectación  de  las  venas  hepáticas  o  la  porción  retrohepática  de  la  VCI.  Los  métodos  óptimos  
para  lograr  la  hemostasia  varían  según  la  gravedad  de  la  lesión  ( cuadro  24­1).

TABLA  24­1.  HÍGADO  EN  ESCALA  DEL  JURADO

Descripción  de  la  lesión

Lesión subcapsular parenquimatoso Vascular

Calificación Hematoma laceración Hematoma

I <10%   — —
1  cm  de  profundidad
de  superficie

10  %–50  %   10  cm  de   —
Yo
1–3  cm  de  profundidad
de  superficie longitud

>50  %   >10  cm  de   —
tercero
>3  cm  de  profundidad
de  superficie longitud
IV— 25%–75%  del  lóbulo  o  1– — —

3  Couinaud
segmentos  
EN  ­ >75%  del  lóbulo  o — Vena  cava  inferior  
>3  segmentos   retrohepática  o  venas  
Couinaud hepáticas  mayores

Datos  de  Moore  EE,  Cogbill  TH,  Jurkovich  GJ  et  al.:  Scaling  Organ  Injury:  Spleen  and  Liver  (Revisión  
de  1994),  J  Trauma  38:323­324,  1995.

12.  ¿Todos  los  pacientes  con  lesión  traumática  del  hígado  requieren  cirugía?
No.  El  tratamiento  conservador  es  el  estándar  para  las  víctimas  de  un  traumatismo  cerrado  que  
permanecen  hemodinámicamente  estables  (aproximadamente  el  85  %  de  los  pacientes).  Un  tercio  de  estos  
pacientes  requieren  transfusiones  de  sangre,  pero  si  el  volumen  supera  las  6  unidades  en  las  primeras  24  horas,  
se  debe  realizar  una  angiografía.  La  tomografía  computarizada  debe  repetirse  en  5  a  7  días  para  lesiones  de  grado  
IV  y  V.  Se  han  informado  complicaciones,  que  incluyen  infección  perihepática,  biloma,  bilhemia  y  hemobilia,  en  10%  
de  los  pacientes  no  operados.

13.  ¿Qué  pacientes  tienen  más  probabilidades  de  fracasar  en  el  tratamiento  conservador?
Aquellos  pacientes  que  requieren  reanimación  continua  con  líquidos  para  mantener  la  estabilidad  hemodinámica,  acumulación  
o  rubor  del  contraste  visto  en  la  tomografía  computarizada  y  aquellos  con  lesiones  en  múltiples  órganos  sólidos.
Machine Translated by Google
CAPÍTULO  24  TRAUMA  HEPÁTICO  Y  BILIAR  133

14.  ¿Cuáles  son  las  opciones  para  el  control  temporal  de  hemorragia  significativa  en  víctimas  de  trauma  hepático?

La  hemorragia  continua  conduce  al  círculo  vicioso  de  acidosis,  hipotermia  y  coagulopatía.
La  compresión  manual,  el  taponamiento  perihepático,  la  angioembolización  y  la  maniobra  de  Pringle  son  las  
estrategias  temporales  más  efectivas.

15.  ¿Qué  es  la  maniobra  de  Pringle?
La  maniobra  de  Pringle  es  una  oclusión  con  abrazadera  manual  o  vascular  del  ligamento  hepatoduodenal  
para  interrumpir  el  flujo  sanguíneo  hacia  el  hígado.  Incluidos  en  el  ligamento  hepatoduodenal  están  la  arteria  
hepática,  la  vena  porta  y  el  conducto  biliar  común.  El  fracaso  de  la  maniobra  de  Pringle  para  controlar  la  hemorragia  
hepática  sugiere  (1)  lesión  de  la  vena  cava  retrohepática  o  de  la  vena  hepática  o  (2)  irrigación  arterial  de  una  arteria  
hepática  derecha  o  izquierda  anómala  (ver  pregunta  9).

16.  ¿Qué  es  la  técnica  de  fractura  de  dedo?
La  hepatotomía  o  tractotomía  por  fractura  de  dedo  es  el  método  para  exponer  puntos  sangrantes  profundos  
dentro  de  las  laceraciones  hepáticas  mediante  disección  roma.  La  separación  del  parénquima  hepático  permite  
identificar  y  ligar  los  puntos.  Este  método  es  más  comúnmente  requerido  para  heridas  penetrantes.

17.  ¿Cuál  es  el  papel  de  la  ligadura  selectiva  de  la  arteria  hepática  para  asegurar  la  hemostasia  en  pacientes  con  una  
lesión  hepática  importante?
Las  laceraciones  profundas  del  lóbulo  hepático  derecho  o  izquierdo  pueden  provocar  una  hemorragia  que  no  se  puede  
controlar  por  completo  mediante  la  ligadura  con  suturas  de  puntos  específicos  de  hemorragia  dentro  del  parénquima  
hepático.  En  esta  situación,  se  puede  ligar  la  arteria  derecha  o  izquierda  para  controlar  el  sangrado  con  poco  riesgo  de  
necrosis  hepática  isquémica.

18.  ¿Por  qué  es  letal  la  laceración  de  la  vena  cava  retrohepática?
La  exposición  requiere  hepatotomía  extensa,  movilización  extensa  del  lóbulo  derecho,  lobectomía  derecha  o  
sección  transversal  de  la  vena  cava.  El  gran  calibre  y  el  alto  flujo  de  la  IVC  dan  como  resultado  una  hemorragia  
masiva  durante  la  exposición  quirúrgica,  mientras  que  el  pinzamiento  de  la  IVC  a  menudo  produce  hipotensión  
atribuible  a  una  disminución  abrupta  del  retorno  venoso  al  corazón.

19.  ¿Cuál  es  la  justificación  fisiológica  para  el  uso  de  una  derivación  en  el  intento  de  reparación?
de  las  lesiones  de  la  vena  cava  retrohepática?
El  control  de  la  hemorragia  requiere  el  mantenimiento  del  retorno  venoso  al  corazón  mientras  se  detiene  
la  hemorragia  anterógrada  y  retrógrada  a  través  de  la  laceración.  Estos  requisitos  se  cumplen  mediante  la  derivación  
de  sangre  a  través  de  un  tubo  que  atraviesa  la  laceración  entre  la  aurícula  derecha  y  la  VCI  inferior.

20.  ¿Qué  es  el  dispositivo  de  taponamiento  con  balón  intrahepático?
Para  lesiones  penetrantes  transhepáticas,  se  sutura  un  drenaje  Penrose  de  1  pulgada  alrededor  de  un  catéter  de  
goma  roja.  Esto  forma  un  globo  largo  que  se  pasa  a  través  de  la  lesión  hepática  sangrante  y  se  infla  con  medio  de  
contraste  a  través  de  una  llave  de  paso  en  el  catéter  de  goma  roja.  El  globo  tapona  la  hemorragia  hepática.  El  catéter  
se  saca  a  través  de  la  pared  abdominal,  se  desinfla  y  se  retira  24  a  48  horas  después.

21.  ¿Cuáles  son  las  indicaciones  del  taponamiento  perihepático?
El  taponamiento  hepático  con  reoperación  planificada  para  el  tratamiento  definitivo  de  lesiones  en  pacientes  con  
hipotermia,  acidosis  y  coagulopatías  es  una  maniobra  que  salva  vidas  (laparotomía  de  control  de  daños).  Se  
colocan  compresas  de  laparotomía  (>20)  alrededor  del  hígado  para  comprimir  y  controlar  la  hemorragia.  Luego  se  
coloca  un  vendaje  temporal  sobre  el  abdomen  abierto  (laparotomía  de  control  de  daños),  y  las  anomalías  
metabólicas  y  la  coagulopatía  del  paciente  se  corrigen  con  una  reoperación  planificada  dentro  de  las  24  horas.
Machine Translated by Google
134  CAPÍTULO  24  TRAUMA  HEPÁTICO  Y  BILIAR

22.  ¿Qué  es  el  síndrome  compartimental  abdominal?
El  síndrome  compartimental  abdominal  es  una  complicación  potencialmente  letal  del  taponamiento  perihepático  o  de  la  
reanimación  de  gran  volumen.  Puede  ocurrir  cuando  la  presión  intraabdominal  supera  los  20  cm  H2O.
La  presión  intraabdominal  aumenta  debido  al  edema  intestinal  y  hepático  secundario  a  isquemia  y  lesión  por  
reperfusión  o  hemorragia  continua  en  la  cavidad  abdominal.  A  medida  que  la  presión  aumenta  más  allá  de  20  cm  H2O,  
disminuyen  el  retorno  venoso,  el  gasto  cardíaco  (CO)  y  la  diuresis,  pero  aumentan  las  presiones  ventilatorias.  Los  
pacientes  deben  regresar  de  inmediato  al  quirófano  (OR)  para  la  descompresión  del  abdomen.  Un  manómetro  conectado  
al  catéter  de  Foley  es  útil  para  controlar  la  presión  intraabdominal.

23.  ¿Cuáles  son  las  complicaciones  comunes  relacionadas  con  la  lesión  hepática?
Después  de  una  lesión  contusa  del  hígado,  en  general,  el  13%  desarrolla  complicaciones  hepáticas.  Las  
complicaciones  incluyen  sangrado,  fugas  o  fístulas  biliares,  síndrome  compartimental  abdominal  e  infección.  Las  
complicaciones  ocurren  con  mayor  frecuencia  en  los  grados  más  altos  de  lesión:  5%  en  el  grado  III,  22%  en  el  grado  IV  y  52%  en  el  gr

LESIÓN  DEL  TRACTO  BILIAR

24.  ¿Por  qué  se  asocian  complicaciones  con  las  fugas  de  los  conductos  biliares?
Los  bilomas  (es  decir,  acumulaciones  de  bilis)  se  infectan  con  frecuencia  y  pueden  provocar  una  peritonitis  letal.
La  fístula  biliopleural,  una  comunicación  entre  el  sistema  biliar  y  la  cavidad  pleural,  persiste  debido  a  la  presión  
negativa  relativa  en  el  tórax  y  puede  provocar  un  empiema  biliar.
La  bilhemia  resulta  de  una  fístula  intrahepática  entre  los  conductos  biliares  y  las  venas  hepáticas,  lo  que  resulta  en  una  
bilirrubinemia  severamente  elevada.  La  hemobilia  ocurre  por  la  ruptura  de  un  pseudoaneurisma  arterial  en  el  sistema  
biliar,  lo  que  resulta  en  una  hemorragia  digestiva  alta.

25.  ¿Cuál  es  la  incidencia  de  fuga  del  conducto  biliar?

Para  los  pacientes  manejados  sin  cirugía,  la  tasa  de  fuga  es  del  3%  y  rara  vez  se  los  ve  por  lesiones  de  grado  I,  II  o  III.  
Las  tasas  de  fuga  son  más  altas  para  aquellos  que  se  someten  a  operaciones  o  angioembolización.
Las  colecciones  de  líquido  perihepático  identificadas  por  US  sugieren  una  fuga;  sin  embargo,  se  identifican  con  mayor  
precisión  mediante  gammagrafía  con  ácido  iminodiacético  hepatobiliar  (HIDA)  o  colangiopancreatografía  retrógrada  
endoscópica  (CPRE).

26.  ¿Cuál  es  el  manejo  inicial  de  una  fuga  biliar?
La  CPRE  suele  ser  bastante  útil  para  diagnosticar  y  tratar  fugas.  La  colocación  de  endoprótesis  biliar  con  o  sin  
esfinterotomía  y  drenaje  percutáneo  de  bilomas  frecuentemente  permite  la  resolución  espontánea  de  las  lesiones  de  las  
vías  biliares.  Las  lesiones  extensas  requieren  hepaticoyeyunostomía  para  su  reconstrucción.

PUNTOS  CLAVE

1.  Las  lesiones  hepáticas  son  comunes  después  del  BAT  y  deben  considerarse  en  todos  los  pacientes  que  están
hipotenso

2.  El  ochenta  y  cinco  por  ciento  de  los  pacientes  con  lesiones  hepáticas  pueden  tratarse  sin  cirugía.
La  angioembolización  es  un  complemento  importante.

3.  Es  más  probable  que  las  lesiones  de  mayor  grado  fracasen  en  el  tratamiento  conservador  y  tengan  un
mayor  tasa  de  complicaciones.

4.  Debe  considerarse  la  laparotomía  de  control  de  daños  para  lesiones  hepáticas  graves.

5.  La  arteria  hepática  entrega  el  30%  del  flujo  sanguíneo  y  la  vena  porta  entrega  el  70%  del  flujo  sanguíneo.

6.  Las  lesiones  biliares  pueden  ocurrir  con  una  lesión  hepática  grave,  pero  la  mayoría  se  tratan  con  técnicas  
mínimamente  invasivas.
Machine Translated by Google
CAPÍTULO  24  TRAUMA  HEPÁTICO  Y  BILIAR  135

SITIO  WEB

www.acssurgery.com

BIBLIOGRAFÍA

1.  Croce  MA,  Fabian  TC,  Menke  PG  et  al.:  El  tratamiento  no  quirúrgico  del  traumatismo  hepático  cerrado  es  el  tratamiento  de
elección  para  pacientes  hemodinámicamente  estables.  Ann  Surg  221:744­753,  1995.
2.  Franklin  GA,  Casos  SR:  Avances  actuales  en  el  abordaje  quirúrgico  del  trauma  abdominal.  Lesión  37:1143­1156,
2006.

3.  Hiatt  JR,  Gabbay  J,  Busutill  RW:  Anatomía  quirúrgica  de  las  arterias  hepáticas  en  1000:  casos.  Ann  Surg  220:50­52,
1994.

4.  Hurtuk  M,  Reed  RL,  Esposito  TJ  et  al.:  Los  cirujanos  de  trauma  practican  lo  que  predican:  la  historia  de  NTDB  en  sólidos
Manejo  de  lesiones  de  órganos.  J  Trauma  61:243­255,  2006.

5.  Kozar  RA,  Moore  FA,  Cothren  CC  et  al .:  Factores  de  riesgo  de  morbilidad  hepática  después  del  tratamiento  no  quirúrgico.
Arch  Surg  141:451­459,  2006.
6.  Lui  F,  Sangosanya  A,  Kaplan  LJ:  Síndrome  compartimental  abdominal:  aspectos  clínicos  y  seguimiento.  cuidado  crítico
Clin  23:415­433,  2007.

7.  Meredith  JW,  Young  JR,  Bowling  J  et  al .:  Manejo  no  quirúrgico  del  trauma  hepático  cerrado  en  adultos:  ¿la  excepción  
o  la  regla?  J  Trauma  36:529­534,  1994.
8.  Moore  EE:  laparotomía  por  etapas  para  el  síndrome  de  hipotermia,  acidosis  y  coagulopatía.  Am  J  Surg
172:405­410,  1996.

9.  Moore  EE,  Cogbill  TH,  Malangoni  MA  et  al .:  Lesión  de  órganos  escalados.  Surg  Clin  North  Am  75:293–303.
10.  Poggetti  RS,  Moore  EE,  Moore  FA  et  al .:  taponamiento  con  globo  para  heridas  de  bala  hepáticas  transfijadoras  bilobulares.
J  Trauma  33:694­697,  1992.

11.  Tai  NR,  Boffard  KD,  Goosen  J  et  al .:  Una  experiencia  de  10  años  de  trauma  hepático  complejo.  Br  J  Surg  89:1532­1537,
2002.

12.  Velmahos  GC,  Toutouzas  K,  Radin  F  et  al .:  Gran  éxito  con  el  manejo  no  quirúrgico  del  trauma  hepático  cerrado.  El  
hígado  es  un  órgano  robusto.  Arch  Surg  138:475­481,  2003.
13.  Verous  M,  Cillo  U,  Brolese  A  et  al .:  Lesión  hepática  contusa:  desde  el  manejo  no  quirúrgico  hasta  el  trasplante  de  hígado.
Lesión  34:181­186,  2003.
14.  Wahl  WL,  Brandt  MM,  Hemmila  MR  et  al .:  Diagnóstico  y  manejo  de  fugas  de  bilis  después  de  una  lesión  hepática  contusa.
Cirugía  138:742­748,  2005.
Machine Translated by Google

TRAUMA  ESPLÉNICO
CAPÍTULO  
25

Sarah  Judkins,  MD,  y  Ernest  E.  Moore,  MD

1.  ¿Cuál  es  el  papel  fisiológico  del  bazo?
En  el  desarrollo  fetal,  el  bazo  sirve  como  un  sitio  importante  para  la  hematopoyesis.  En  la  primera  infancia,  el  bazo  produce  
inmunoglobulina  M  (IgM)  y  tuftsina.  El  bazo  también  funciona  como  un  filtro,  lo  que  permite  que  los  macrófagos  residentes  
eliminen  los  glóbulos  rojos  (RBC)  anormales,  los  desechos  celulares  y  las  bacterias  encapsuladas  y  poco  opsonizadas.

2.  ¿Qué  patrones  de  lesión  están  asociados  con  el  traumatismo  esplénico?
Fuerza  contundente  directa,  desaceleración  y  compresión  al  torso  izquierdo.  Piense  en  el  bazo  después  de  un  accidente  
automovilístico  o  una  caída:  las  fracturas  de  costillas  inferiores,  las  fracturas  de  costillas  solo  del  lado  izquierdo  y  la  transferencia  
de  alta  energía  (golpes  grandes)  aumentan  la  probabilidad  de  lesión  esplénica.

3.  ¿Cuáles  son  los  signos  y  síntomas  de  la  lesión  esplénica?
El  signo  principal  es  dolor  en  el  cuadrante  superior  izquierdo.  Esto  se  produce  al  estirar  la  cápsula  esplénica.  La  irritación  
peritoneal  (dolor  de  rebote)  es  causada  por  sangre  extravasada.  Los  signos  vitales  varían  según  la  pérdida  de  sangre  asociada  y  
no  son  específicos  de  las  lesiones  del  bazo.
Desafortunadamente,  una  gran  cantidad  de  pacientes  con  una  lesión  esplénica  significativa  no  presentan  ningún  signo  o  
síntoma.

4.  ¿Qué  estudios  pueden  ayudar  en  el  diagnóstico  de  traumatismo  esplénico?
La  ecografía  (US)  se  realiza  de  forma  rutinaria  en  el  departamento  de  emergencias  (ED)  y  puede  identificar  rápidamente  tan  
solo  200  ml  de  líquido  o  sangre.  Cuando  la  ecografía  es  equívoca,  el  lavado  peritoneal  de  diagnóstico  (DPL)  es  una  medida  
precisa  y  sensible  del  sangrado  intraabdominal.  La  tomografía  computarizada  (TC)  es  la  prueba  diagnóstica  estándar  de  oro  
porque  es  casi  100%  precisa  para  identificar  lesiones  esplénicas,  pero  también  define  la  magnitud  de  la  lesión  y  cuantifica  la  
cantidad  de  sangre  intraperitoneal.

5.  ¿Cómo  se  clasifican  las  lesiones  esplénicas  y  por  qué  es  importante?
El  manejo  se  rige  por  el  estado  hemodinámico  del  paciente,  pero  la  terapia  también  se  ve  influida  por  el  grado  de  lesión  
esplénica  en  la  TC.  El  manejo  conservador  es  más  exitoso  en  los  grados  I  a  III,  mientras  que  la  intervención  quirúrgica  a  
menudo  se  requiere  para  las  lesiones  de  grado  IV.  Las  lesiones  de  grado  V  exigen  una  intervención  quirúrgica  inmediata  ( cuadro  
25­1).

TABLA  25­1.  LESIONES  DE  GR  AD  ESOFSPL  EN  IC

Calificación Descripción

I Hematoma:  subcapsular  no  expansivo  <10%  del  área  de  superficie
Laceración:  capsular  no  sangrante  <1  cm  de  profundidad  parenquimatosa

Yo
Hematoma:  no  expansivo,  subcapsular  <50%  de  superficie
Intraparenquimatosa  no  expansiva  <5  cm  de  diámetro
Laceración:  sangrante,  capsular  <3  cm  de  profundidad  parenquimatosa

136
Machine Translated by Google
CAPÍTULO  25  TRAUMA  ESPLÉNICO  137

TABLA  25­1.  LESIONES  DE  GR  AD  ESOFSPL  EN  IC—  CON  T  'D

Calificación Descripción

tercero
Hematoma:  subcapsular  >50%  del  área  superficial,  expandiéndose,  roto  con  sangrado  
activo  Intraparenquimatoso  >5  cm  de  diámetro  o  expandiéndose
Laceración:  capsular  >3  cm  de  profundidad  parenquimatosa,  involucrando  vaso  trabecular

IV Hematoma:  roto,  intraparenquimatoso,  con  sangrado  activo
Laceración:  involucra  vasos  segmentarios  o  hiliares  con  >25%  de  desvascularización  
esplénica

EN Laceración:  bazo  destrozado
Vascular:  avulsión  hiliar  o  desvascularización  esplénica  completa

6.  ¿Las  lesiones  esplénicas  requieren  laparotomía?
No.  El  tratamiento  conservador  tiene  éxito  en  aproximadamente  el  95  %  de  los  pacientes  con  grados  I  a  III.
Los  pacientes  hemodinámicamente  estables  con  evidencia  de  sangrado  continuo  (que  requieren  transfusión)  pueden  
ser  tratados  mediante  embolización  arterial  selectiva  si  se  identifica  un  sitio  de  sangrado  en  la  angiografía.

7.  ¿Cuáles  son  las  contraindicaciones  para  el  tratamiento  conservador  de  las  lesiones  esplénicas?
&  Inestabilidad  hemodinámica.
  Coagulopatía  persistente.
  Lesión  intraabdominal  adicional  que  requiere  intervención  quirúrgica.

8.  ¿Cuál  es  la  tasa  de  fracaso  del  tratamiento  conservador  de  la  lesión  esplénica?
Cualquier  paciente  con  signos  de  inestabilidad  hemodinámica,  hemorragia  persistente,  empeoramiento  del  dolor  o  
la  sensibilidad,  o  lesión  progresiva  en  la  tomografía  computarizada  ha  fracasado  en  el  tratamiento  conservador.
Aproximadamente  el  60  %  de  todas  las  lesiones  esplénicas  pueden  tratarse  sin  cirugía  con  una  tasa  de  fracaso  del  
12  %.  Los  factores  que  predicen  el  fracaso  no  quirúrgico  incluyen  lesiones  múltiples,  lesiones  del  bazo  de  grado  >  III  
y  necesidad  de  transfusión  de  sangre.

9.  ¿Qué  es  la  rotura  retardada  del  bazo?
Esta  es  una  complicación  rara  que  ocurre  en  <1%  de  los  pacientes  con  una  lesión  esplénica.  La  rotura  tardía  del  bazo  
debe  distinguirse  del  retraso  en  el  diagnóstico  de  lesión  esplénica  y  de  la  rotura  de  una  lesión  esplénica  conocida.  La  
ruptura  esplénica  retardada  verdadera  ocurre  >48  horas  en  un  paciente  con  antecedentes  de  trauma  abdominal  y  sin  
evidencia  clínica  manifiesta  de  lesión  intraabdominal  en  la  presentación  inicial.

10.  ¿Cuáles  son  los  principios  generales  del  tratamiento  quirúrgico  del  bazo  lesionado?
La  primera  prioridad  es  controlar  el  sangrado.  Por  lo  general,  esto  se  puede  lograr  mediante  empaquetamiento  y  
compresión  manual  del  bazo.  Si  tiene  éxito,  se  explora  minuciosamente  el  abdomen  en  busca  de  otras  lesiones.  Se  
requiere  la  movilización  completa  del  bazo  por  división  de  los  ligamentos  esplenocólico,  esplenorrenal,  frenosplénico  y  
gastroesplénico  para  una  evaluación  completa  del  bazo.
Los  vasos  gástricos  cortos  pueden  ligarse  con  división  del  ligamento  gastroesplénico.  La  reparación  del  bazo  se  
puede  lograr  mediante  la  aplicación  de  agentes  hemostáticos,  la  reparación  directa  con  sutura  del  parénquima  esplénico,  
la  esplenectomía  parcial  y  la  construcción  de  una  "envoltura  esplénica"  con  una  malla  absorbible.  Si  se  requiere  
esplenectomía,  la  arteria  y  la  vena  esplénica  deben  ligarse  individualmente  antes  de  extirpar  el  bazo.

11.  ¿Qué  complicaciones  tempranas  surgen  después  de  la  esplenectomía?
Sangrado  recurrente,  dilatación  gástrica  aguda,  perforación  gástrica,  pancreatitis  (la  arteria  esplénica  discurre  a  lo  largo  
de  la  parte  superior  del  páncreas)  y  absceso  subfrénico.
Machine Translated by Google
138  CAPÍTULO  25  TRAUMA  ESPLÉNICO

12.  ¿Qué  es  el  autotrasplante  de  bazo?
El  autotrasplante  se  logra  mediante  la  implantación  de  cortes  de  parénquima  de  tejido  esplénico  en  bolsas  
creadas  en  el  epiplón  gastrocólico.

13.  ¿El  autotrasplante  esplénico  preserva  la  función  esplénica?
El  autotrasplante  después  de  la  esplenectomía  es  controvertido.  Se  necesita  al  menos  el  30%  de  la  masa  esplénica  
original  para  proporcionar  una  función  normal.  Después  del  autotrasplante,  los  niveles  de  inmunoglobulina  G  (IgG)  
e  IgM  aumentan  en  respuesta  a  la  vacuna  antineumocócica  en  comparación  con  los  pacientes  después  de  la  
esplenectomía  sola.

14.  ¿La  leucocitosis  posesplenectomía  predice  infección?
Las  elevaciones  en  el  recuento  de  glóbulos  blancos  (WBC)  y  el  recuento  de  plaquetas  (PC)  después  de  la  
esplenectomía  son  un  evento  fisiológico  común.  Sin  embargo,  después  del  cuarto  día  posoperatorio,  un  WBC  >  15  
103  y  un  PC/WBC  <  20  están  muy  asociados  con  sepsis  y  no  deben  confundirse  con  la  respuesta  fisiológica  a  la  
esplenectomía.

15.  ¿Debería  realizarse  una  tomografía  computarizada  de  seguimiento  después  de
tratamiento  no  quirúrgico  de  las  lesiones  esplénicas  antes  del  alta  del  paciente?
No.  La  mayoría  de  los  pacientes  que  fallan  en  el  tratamiento  conservador  lo  hacen  dentro  de  los  5  
días  y  exhibirán  evidencia  hemodinámica  de  hemorragia  en  curso.  Sin  embargo,  se  debe  realizar  una  TC  de  
seguimiento  para  las  lesiones  de  grado  III  y  IV  a  las  4  a  6  semanas  antes  de  volver  a  la  actividad  física  vigorosa.

16.  ¿Qué  es  la  sepsis  abrumadora  posesplenectomía  y  cómo  se  previene?
La  sepsis  postesplenectomía  abrumadora  (OPSS)  es  una  bacteriemia  devastadora  (típicamente  
bacterias  encapsuladas)  que  ocurre  en  el  2%  de  los  pacientes  después  de  la  esplenectomía.  El  riesgo  de  OPSS  
es  mayor  cuando  la  esplenectomía  se  realiza  durante  la  infancia.  Los  organismos  más  comunes  son  neumococo  
(50%),  meningococo,  Escherichia  coli,  Haemophilus  influenzae,  estafilococo  y  estreptococo.  Aunque  es  raro,  el  
OPSS  conlleva  una  tasa  de  mortalidad  del  75%  y  ha  estimulado  el  interés  en  la  preservación  del  bazo.  El  OPSS  se  
previene  principalmente  con  la  vacunación  posoperatoria.  Las  vacunas  contra  la  gripe  neumocócica,  meningocócica  
y  Haemophilus  deben  administrarse  2  semanas  después  de  la  esplenectomía  y  se  recomiendan  cada  5  años.  La  
sepsis  puede  ocurrir  a  pesar  de  la  vacunación;  en  consecuencia,  se  recomienda  la  profilaxis  a  largo  plazo  con  
penicilina  oral  para  los  niños.

PUNTOS  CLAVE:  MANEJO  EXPECTANTE  DEL  BAZO
LESIONES
1.  El  tratamiento  conservador  tiene  éxito  en  el  95  %  de  las  lesiones  de  grado  I  a  III.

2.  El  ochenta  por  ciento  de  todas  las  lesiones  esplénicas  se  manejan  sin  cirugía,  con  un  12%  de  fracaso  o
tasa  de  conversión.

3.  Los  factores  que  predicen  el  fracaso  o  la  conversión  al  tratamiento  quirúrgico  incluyen  lesión  >  grado  III,
y  necesidad  de  transfusión  de  sangre.

4.  Los  pacientes  con  evidencia  de  sangrado  en  curso  (p.  ej.,  "rubor"  de  contraste  en  la  TC  o  requisitos  de  
transfusión  en  curso)  pueden  tratarse  con  embolización  arterial  selectiva.
Machine Translated by Google
CAPÍTULO  25  TRAUMA  ESPLÉNICO  139

SITIOS  WEB

www.east.org/tpg/bluntabd.pdf

www.acssurgery.com/abstracts/acs/acs0506.htm

BIBLIOGRAFÍA

1.  Bala  M,  Edden  Y,  Mintz  Y  et  al .:  Trauma  esplénico  contundente:  predictores  para  un  manejo  no  quirúrgico  exitoso.  Isr
Med  Assoc  J  9:857­861,  2007.

2.  Cadeddu  M,  Garnett  A,  Al­Anezi  K  et  al .:  Manejo  de  las  lesiones  del  bazo  en  la  población  adulta  con  traumatismos:  una  
experiencia  de  diez  años.  Can  J  Surg  49:386­390,  2006.

3.  Leemans  R,  Harms  G,  Rijkers  GT  et  al .:  El  autotrasplante  de  bazo  proporciona  la  restauración  de  los  compartimentos  linfoides  
esplénicos  funcionales  y  mejora  la  respuesta  inmune  humoral  a  la  vacuna  neumocócica  de  polisacáridos.  Clin  Exp  Immunol  
17:596­604,  1999.

4.  Moore  EE,  Cogbill  TH,  Jurkovich  GJ  et  al.:  Scaling  Organ  Injury:  Spleen  and  Liver  (revisión  de  1994).  J  Trauma
38:323­324,  1995.

5.  Shatz  DV:  Prácticas  de  vacunación  entre  los  cirujanos  de  trauma  de  América  del  Norte  en  la  esplenectomía  por  trauma.  J  Trauma
53:950­956,  2002.

6.  Taylor  M,  Genuit  T,  Napolitano  L:  Abrumadora  sepsis  y  trauma  posesplenectomía:  ¿es  hora  de  considerar  la  revacunación?  
J  Trauma  59:1482­1485,  2005.

7.  Uecker  J,  Pickett  C,  Dunn  E:  El  papel  de  los  estudios  radiográficos  de  seguimiento  en  el  tratamiento  no  quirúrgico  del  trauma  del  
bazo.  Am  Surg  67:22­25,  2001.
Machine Translated by Google

LESIÓN  PÁNCREAS  Y  DUODENAL
CAPÍTULO  
26

Jeffry  L.  Kashuk,  MD,  FACS

1.  ¿Qué  tan  comunes  son  las  lesiones  pancreáticas  y  duodenales?
Debido  a  que  el  duodeno  y  el  páncreas  están  íntimamente  asociados  con  estructuras  vitales  en  un  área  
profunda  y  angosta  del  retroperitoneo,  lo  que  brinda  un  grado  significativo  de  protección,  la  frecuencia  de  
lesiones  reportadas  está  entre  el  7%  y  el  10%  para  todas  las  celiotomías  por  trauma.  La  lesión  duodenal  
cerrada  ocurre  en  menos  del  1%  de  los  traumatismos  cerrados.

2.  ¿Qué  otras  lesiones  se  asocian  típicamente  con  la  penetración  del  páncreas?
¿trauma?
La  lesión  hepática  es  la  lesión  concomitante  más  frecuente,  con  una  incidencia  reportada  del  50%.  Otras  
lesiones  comúnmente  asociadas  incluyen  el  estómago  (40%),  grandes  vasos  abdominales  como  la  aorta  y  la  
vena  cava  (40%),  bazo  (25%),  riñón  (2%)  y  duodeno  (20%).

3.  ¿Cómo  se  diagnostican  preoperatoriamente  las  lesiones  pancreáticas?
El  traumatismo  penetrante  del  páncreas  suele  descubrirse  durante  la  exploración  en  busca  de  lesiones  
asociadas.  Dichos  pacientes  pueden  presentar  inestabilidad  hemodinámica  por  sangrado,  ecografía  
abdominal  enfocada  positiva  en  el  examen  de  trauma  (FAST)  o  peritonitis.  Los  pacientes  con  lesión  contusa  que  
están  hemodinámicamente  estables  deben  someterse  a  una  tomografía  computarizada  (TC)  abdominal  y  una  
posible  colangiopancreatografía  retrógrada  endoscópica  (CPRE).  Las  concentraciones  elevadas  de  amilasa  
sérica  no  son  específicas  para  la  lesión  pancreática  y  pueden  ser  normales  en  una  alta  proporción  de  pacientes.

4.  ¿Cuáles  son  algunas  de  las  opciones  quirúrgicas  comúnmente  utilizadas  para  el  tratamiento  de  lesiones  
pancreáticas?
La  mayoría  de  las  lesiones  penetrantes  y  contusas  de  bajo  grado  se  tratan  adecuadamente  con  
drenajes  de  succión  cerrados  colocados  en  la  cirugía.  En  lesiones  más  graves,  se  debe  evaluar  la  integridad  
del  conducto  pancreático  principal,  ya  sea  por  inspección  directa  o  por  pancreatografía  intraoperatoria.  Las  
lesiones  del  conducto  distal  se  tratan  con  pancreatectomía  distal,  con  o  sin  esplenectomía,  y  drenaje  cerrado  
del  muñón  pancreático.  Es  preferible  la  preservación  del  bazo.  La  lesión  del  conducto  pancreático  en  la  
cabeza  o  el  cuello  puede  requerir  la  resección  de  porciones  significativas  del  páncreas  distal  y  generalmente  
se  realiza  de  manera  tardía  después  de  los  procedimientos  de  control  de  daños.

5.  Describir  las  complicaciones  comunes  de  las  lesiones  pancreáticas.
La  exanguinación  es  la  causa  más  común  de  muerte  prematura,  lo  que  lleva  al  uso  de  control  de  daños.
Para  los  pacientes  que  sobreviven  a  su  operación  inicial,  las  dos  complicaciones  más  comunes  son  las  
fístulas  pancreáticas  y  los  abscesos  intraabdominales.  Otros  problemas  tardíos  son  la  pancreatitis,  el  
seudoquiste  pancreático  y  la  hemorragia  pancreática.  La  mayoría  de  los  pacientes  que  mueren  después  de  
sufrir  lesiones  en  el  páncreas  lo  hacen  como  resultado  de  complicaciones  tardías  y  no  por  la  lesión  pancreática  
en  sí.

140
Machine Translated by Google
CAPÍTULO  26  LESIÓN  PÁNCREAS  Y  DUODENAL  141

PUNTOS  CLAVE:  OPCIONES  QUIRÚRGICAS  PARA  EL  PANCREÁTICO
LESIONES

1.  Las  lesiones  de  bajo  grado  se  tratan  con  drenaje  por  succión  cerrado  simple  en  el  momento  de  la  celiotomía.

2.  Las  lesiones  asociadas  son  comunes  y  deben  buscarse  y  tratarse.

3.  Los  pacientes  inestables  deben  someterse  a  desbridamiento  del  tejido  desvascularizado,  
hemostasia  y  drenaje  con  reconstrucción  diferida  hasta  que  el  paciente  esté  estable.

4.  Si  se  sospecha  lesión  ductal  en  un  paciente  estable,  visualice  con  CPRE  o  colangiografía.

5.  Si  hay  lesión  ductal  en  la  cabeza  o  el  cuello  del  páncreas,  ligar  proximalmente  e  intentar
para  preservar  el  tejido  pancreático  con  pancreaticoyeyunostomía  en  Y  de  Roux.

6.  Considere  establecer  un  acceso  nutricional  enteral  mediante  la  colocación  de  una  sonda  de  alimentación  yeyunal  en  pacientes
con  heridas  más  que  leves.

DUODENO

6.  ¿Cuál  es  el  papel  de  la  tomografía  computarizada  en  el  diagnóstico  de  lesiones  duodenales  cerradas?

Aunque  la  TC  es  una  herramienta  excelente  para  visualizar  lesiones  sólidas,  la  TC  es  menos  útil  con  lesiones  
de  órganos  huecos  como  el  duodeno.  Incluso  la  adición  de  un  agente  de  contraste  oral  al  estudio  tiene  una  
alta  especificidad  pero  poca  sensibilidad.  Los  signos  sutiles  de  lesión  duodenal  en  las  tomografías  
computarizadas  incluyen  edema  paraduodenal,  líquido,  aire  retroduodenal  y  acumulación  de  grasa  con  pérdida  
de  planos  tisulares  afilados,  lo  que  generalmente  puede  indicar  una  ruptura  duodenal  y  derrame  de  pequeñas  
cantidades  de  contenido  intraluminal  hacia  el  retroperitoneo.  Tales  hallazgos  sutiles  en  un  paciente  con  un  
mecanismo  de  lesión  de  alto  riesgo  pueden  justificar  la  exploración  quirúrgica.

7.  ¿Cuál  es  la  importancia  de  la  maniobra  de  Kocher?
En  1903,  Kocher  describió  lo  que  ahora  se  ha  convertido  en  una  maniobra  de  rutina  durante  la  celiotomía  
exploradora  para  visualizar  y  reparar  lesiones  en  el  duodeno,  el  colédoco  distal  y  la  cabeza  pancreática.  Las  
inserciones  peritoneales  laterales  avasculares  al  duodeno  se  cortan  con  precisión;  luego  se  eleva  el  barrido  
duodenal  y  se  refleja  medialmente,  lo  que  permite  la  inspección  y  palpación  de  su  superficie  posterior,  así  como  
de  la  cabeza  del  páncreas.

8.  ¿Cuáles  son  las  cuatro  porciones  del  duodeno  y  sus  relaciones  quirúrgicas?
La  primera  porción  del  duodeno  comienza  en  el  píloro  (por  vía  intraperitoneal)  y  pasa  hacia  atrás  (por  vía  
retroperitoneal)  hacia  la  vesícula  biliar  (el  resto  del  duodeno  es  retroperitoneal).
La  segunda  porción  desciende  de  7  a  8  cm  y  es  anterior  a  la  vena  cava.  El  borde  izquierdo  del  duodeno  se  une  
a  la  cabeza  del  páncreas,  en  el  sitio  donde  entran  los  conductos  biliar  común  y  pancreático;  comparte  un  riego  
sanguíneo  común  con  la  cabeza  del  páncreas  a  través  de  las  arcadas  pancreatoduodenales.  La  tercera  porción  
del  duodeno  gira  horizontalmente  hacia  la  izquierda,  con  su  superficie  craneal  en  contacto  con  el  proceso  
uncinado  del  páncreas,  y  pasa  posterior  a  la  arteria  y  vena  mesentéricas  superiores.  La  cuarta  porción  continúa  
hacia  la  izquierda,  asciende  ligeramente  y  cruza  la  columna  anterior  a  la  aorta,  donde  se  fija  al  ligamento  
suspensorio  de  Treitz  en  el  ángulo  duodenoyeyunal.

9.  ¿Cómo  se  clasifican  las  lesiones  duodenales?
Se  ha  adoptado  una  escala  de  lesión  de  órganos  que  permite  descripciones  estandarizadas  de  lesiones  
duodenales,  que  se  extienden  desde  el  grado  1  (menos  grave)  al  grado  V  (más  grave).  La  clasificación  de  las  
lesiones  duodenales  ayuda  a  los  cirujanos  a  seleccionar  el  procedimiento  quirúrgico  apropiado  para  la  reparación  
o  reconstrucción  de  estas  lesiones  frecuentemente  complejas  ( cuadro  26­1).
Machine Translated by Google
142  CAPÍTULO  26  LESIÓN  PÁNCREAS  Y  DUODENAL

TABLA  26­1.  GR  AD  ESOFP  AN  CR  EATIC  LESIONES

Lesión  de  grado Descripción

I Hematoma  que  afecta  una  sola  porción  del  duodeno

Laceración De  espesor  parcial;  sin  perforación

Yo
Hematoma  que  involucra  más  de  una  porción

Laceración Alteración  <50%  de  la  circunferencia

tercero Laceración Interrupción  del  50  %  al  75  %  de  la  circunferencia  de  D2  o  interrupción  del  50  


%  al  100  %  de  D1,  D3  o  D4*

IV Laceración Interrupción  >75%  de  D2  o  compromiso  de  ampolla  o  colédoco  distal

EN Laceración Interrupción  masiva  del  complejo  duodenopancreático

Vascular Desvascularización  del  duodeno

*  D1,  D2,  D3  y  D4  se  refieren  a  las  porciones  del  duodeno  (es  decir,  de  la  primera  a  la  cuarta).

10.  ¿Cuáles  son  las  principales  opciones  quirúrgicas  para  las  lesiones  penetrantes  del  duodeno?
La  mayoría  de  las  laceraciones  simples  (grado  1  a  2)  pueden  repararse  principalmente.  Las  laceraciones  complejas  (grado  3)  
con  márgenes  desvitalizados  o  las  laceraciones  que  afectan  >50%  de  la  circunferencia  duodenal  requieren  desbridamiento  de  
los  márgenes  y  reanastomosis  de  los  extremos  divididos.  Si  se  anticipa  tensión  en  la  línea  de  sutura  debido  a  la  pérdida  
extensa  de  tejido  (grado  3  a  4),  las  técnicas  complementarias  como  la  duodenoyeyunostomía  en  Y  de  Roux  o  la  exclusión  
pilórica  son  más  apropiadas.  Las  lesiones  duodenales  graves  que  afectan  el  conducto  biliar  distal  y  la  cabeza  del  páncreas  
(grado  5)  pueden  justificar  una  duodenopancreatectomía  (es  decir,  procedimiento  de  Whipple)  después  de  los  procedimientos  
de  control  de  daños.  En  pacientes  con  todas  las  reparaciones  excepto  las  simples,  se  debe  considerar  el  establecimiento  de  
un  acceso  enteral  a  través  de  una  yeyunostomía.

PUNTOS  CLAVE:  OPCIONES  QUIRÚRGICAS  PARA  DUODENAL
LESIONES

1.  Las  lesiones  asociadas  son  comunes,  particularmente  en  el  páncreas,  y  deben  buscarse
para  y  dirigida.

2.  Aunque  las  lesiones  penetrantes  generalmente  se  descubren  en  la  laparotomía  por  sangrado  o
peritonitis,  las  lesiones  contusas  son  difíciles  de  diagnosticar  incluso  con  tomografía  computarizada,  y  la  decisión  de  
operar  debe  incluir  la  consideración  de  signos  clínicos  sutiles.

3.  La  exploración  minuciosa  requiere  una  maniobra  de  Kocher  y  una  evaluación  completa  de  todas  las  regiones  
anatómicas  del  duodeno.

4.  La  reparación  quirúrgica  está  determinada  por  la  clasificación  de  la  gravedad  de  la  lesión.  La  mayoría  de  las  lesiones  se  
tratan  con  reparación  primaria  simple  y  las  resecciones  extensas  deben  retrasarse  en  la  mayoría  de  los  pacientes  
mediante  procedimientos  de  control  de  daños.

5.  Debe  considerarse  el  acceso  enteral  en  todas  las  lesiones  excepto  en  las  más  simples.
Machine Translated by Google
CAPÍTULO  26  LESIÓN  PÁNCREAS  Y  DUODENAL  143

BIBLIOGRAFÍA

1.  Huerta  S,  Bui  T,  Porral  D  et  al.:  Predictores  de  morbimortalidad  en  pacientes  con  lesiones  duodenales  traumáticas.  Am  
Surg  71:763,  2005.

2.  Kap  LS,  Bulger  EM,  Parks  DL  et  al .:  Predictores  de  morbilidad  después  de  una  lesión  pancreática  traumática.  J  Trauma  55:898,
2003.

3.  Lopez  PP,  Benjamin  R,  Cockbum  M  et  al.:  Tendencias  recientes  en  el  manejo  de  las  lesiones  pancreatoduodenales  combinadas.  
Am  Surg  71:847,  2005.

4.  Moore  EE,  Cogbill  T,  Malangoni  M  et  al .:  Escalamiento  de  lesión  de  órganos  II:  páncreas,  duodeno,  intestino  delgado,  colon  y  
recto.  J  Trauma  30:1427;

5.  Patel  SV,  Spencer  JA,  el­Hansani  S  et  al .:  Imágenes  del  trauma  pancreático.  Br  J  Radiol  71:985,  1998.

6.  Supramann  A,  Dente  CJ,  Feliciano  DY:  El  manejo  del  trauma  pancreático  en  la  era  moderna.  Cirugía  Clin  Norte
Am  87:1515,  2007.

7.  Takishima  T,  Sugimoto  K,  Hirata  M  et  al .:  Nivel  de  amilasa  sérica  al  ingreso  en  el  diagnóstico  de  lesión  contusa  del  páncreas:  su  
importancia  y  limitaciones.  Ann  Surg  226:70,  1997.

8.  Timaran  CH,  Daley  BJ,  Enderson  BL:  papel  de  la  duodenografía  en  el  diagnóstico  de  lesiones  duodenales  cerradas.
J  Trauma  51:648,  2001.

9.  Vassiliu  P,  Toutouzas  KG,  Velahos  GC:  Estudio  prospectivo  de  fístulas  biliares  y  pancreáticas  postraumáticas.
El  papel  de  la  conducta  expectante.  Lesión  35:223,  2004.

10.  Velmahos  GC,  Constantimon  C,  Kassotakis  G:  Seguridad  de  la  reparación  de  lesiones  duodenales  graves.  Mundo  J  Surg  32:7,
2008.

11.  Wales  PW,  Shuckett  B,  Kim  PC:  Resultado  a  largo  plazo  después  del  tratamiento  no  quirúrgico  de  la  lesión  traumática  completa
Transacción  pancreática  en  niños.  J  Pediatr  Surg  36:823,  2001.
Machine Translated by Google

CAPÍTULO  
27

TRAUMA  EN  EL  COLON  Y  RECTO
Dr.  Walter  L.  Biffl,  FACS

TRAUMA  DEL  COLON

1.  ¿Cómo  ocurren  la  mayoría  de  las  lesiones  de  colon?
Casi  todas  las  lesiones  de  colon  (>95%)  son  causadas  por  traumatismo  penetrante  por  arma  de  fuego,  arma  blanca,  
iatrogénica  o  lesión  sexual.  El  traumatismo  cerrado  del  colon  es  poco  común  y  por  lo  general  resulta  de  los  cinturones  de  
seguridad  durante  accidentes  automovilísticos.

2.  ¿Cómo  se  diagnostican  las  lesiones  de  colon?
Por  lo  general,  se  diagnostican  durante  la  laparotomía  por  traumatismo  penetrante.  Para  los  pacientes  en  los  que  no  se  
ha  establecido  la  necesidad  de  laparotomía,  las  radiografías  de  tórax  y  de  abdomen  en  bipedestación  pueden  revelar  
aire  libre  y  detectar  la  ubicación  de  objetos  penetrantes.  La  tomografía  computarizada  (TC)  de  triple  contraste  (es  decir,  
oral,  intravenosa  [IV]  y  rectal)  o  las  radiografías  con  contraste  soluble  pueden  diagnosticar  lesiones  retroperitoneales  del  
colon.  Los  recuentos  elevados  de  glóbulos  blancos  o  los  niveles  de  enzimas  (amilasa,  fosfatasa  alcalina)  o  materia  fecal  
en  el  lavado  peritoneal  de  diagnóstico  (DPL)  son  altamente  sugestivos  de  una  lesión  intestinal.

3.  ¿Cómo  se  clasifican  las  lesiones  del  colon?
Grado  I:  hematoma  por  contusión  sin  desvascularización  o  laceración  de  espesor  parcial.
Grado  II:  laceración  <50%  de  circunferencia.
Grado  III:  laceración  >50%  de  circunferencia.
Grado  IV:  transección  del  colon.

Grado  V:  transección  con  pérdida  de  tejido  segmentario.

4.  ¿Cuáles  son  las  tres  opciones  quirúrgicas  principales  para  tratar  una  lesión  de  colon?
1.  Reparación  primaria:  sutura  de  perforaciones  simples  o  resección  y  anastomosis  primaria  para  lesiones  más  
complejas.
2.  Colostomía:  se  reseca  el  colon  lesionado  y  se  saca  el  colon  proximal  como  una  colostomía  o  se  repara  la  lesión  
pero  se  saca  una  ileostomía  o  colostomía  más  proximal  para  desviar  el  flujo  fecal.

5.  ¿Cuáles  son  las  ventajas  y  desventajas  de  cada  una  de  estas  opciones?
1.  La  reparación  primaria  es  deseable  porque  el  tratamiento  definitivo  se  lleva  a  cabo  en  la  operación  inicial  y  el  
paciente  se  ahorra  la  morbilidad  de  una  colostomía  y  su  reversión.  La  desventaja  es  que  las  líneas  de  sutura  se  
crean  en  condiciones  subóptimas,  por  lo  que  pueden  producirse  fugas.
2.  La  colostomía  proximal  evita  una  línea  de  sutura  sin  protección  en  el  abdomen  pero  requiere  una  segunda  
operación  para  cerrar  la  colostomía.  Pueden  ocurrir  complicaciones  estomales,  que  incluyen  necrosis,  
estenosis,  obstrucción  y  prolapso.

6.  ¿Cómo  se  maneja  quirúrgicamente  a  la  mayoría  de  los  pacientes  con  lesiones  de  colon?
La  reparación  primaria  es  segura  y  eficaz  en  prácticamente  todos  los  pacientes  con  traumatismo  de  colon.  Las  
anastomosis  cosidas  a  mano  y  con  grapas  tienen  las  mismas  tasas  de  complicaciones.  Los  antibióticos  profilácticos  se  
administran  durante  no  más  de  24  horas  después  de  la  operación.

144
Machine Translated by Google
CAPÍTULO  27  TRAUMA  EN  EL  COLON  Y  EL  RECTO  145

7.  ¿Cómo  se  debe  manejar  la  incisión  quirúrgica  y  la  herida  penetrante?
Las  heridas  deben  dejarse  abiertas  (para  el  cierre  primario  tardío)  para  disminuir  la  incidencia  de  infección  de  la  herida  y  
dehiscencia  fascial.

8.  ¿Qué  complicaciones  se  asocian  con  la  lesión  colónica  y  su  tratamiento?
  Infección  de  la  herida  (¼  65%  si  la  incisión  en  la  piel  se  cierra  primero;  no  se  sienta  tentado  a  cerrar
una  incisión  sucia).
&  Absceso  intraabdominal  (20%).
&  Dehiscencia  fascial  (10%).
&  Complicaciones  estomales  (5%).
&  Fuga  anastomótica  (5%).
&  Mortalidad  (<1%).

TRAUMA  RECTAL

9.  ¿Cómo  se  producen  las  lesiones  rectales?
De  manera  similar  a  las  lesiones  del  colon,  la  mayoría  de  las  lesiones  rectales  son  el  resultado  de  un  traumatismo  
penetrante  como  resultado  de  un  disparo,  una  puñalada,  una  lesión  iatrogénica  o  sexual.  Las  fracturas  pélvicas  cerradas  
deben  evaluarse  con  una  fuerte  sospecha  de  lesión  rectal  (y  uretral).

10.  ¿Cómo  se  diagnostican  las  lesiones  rectales?
Un  examen  completo  es  crucial,  y  el  diagnóstico  se  sugiere  por  el  curso  de  los  proyectiles  y  la  presencia  de  sangre  en  el  
tacto  rectal.  Si  se  sospecha  un  traumatismo  rectal,  el  paciente  debe  someterse  a  una  proctoscopia  para  buscar  hematoma,  
contusión,  laceración  o  sangre  macroscópica.
Si  el  diagnóstico  está  en  duda,  se  deben  realizar  radiografías  con  enemas  de  contraste  soluble.

11.  ¿En  qué  se  diferencia  el  tratamiento  de  los  pacientes  con  lesiones  rectales  intraperitoneales  de  aquellos  con  lesiones  
extraperitoneales?
La  porción  del  recto  proximal  al  reflejo  peritoneal  se  denomina  segmento  intraperitoneal.  Las  lesiones  de  esta  porción  
se  tratan  de  manera  similar  a  las  lesiones  colónicas.

12.  ¿Cuáles  son  los  cuatro  principios  básicos  para  el  manejo  del  recto  extraperitoneal  simple?
lesiones?
1.  Derivación:  es  adecuada  una  colostomía  o  ileostomía  sigmoidea  en  asa  o  terminal.
2.  Drenaje:  se  debe  utilizar  una  incisión  retroanal  para  colocar  drenajes  Penrose  o  de  succión  cerrada.
cerca  del  sitio  de  la  perforación.
3.  Reparación:  adecuada  cuando  sea  posible,  pero  no  obligatoria  4.  
Lavado:  irrigación  del  recto  distal  con  solución  isotónica  hasta  que  el  efluente  sea  transparente.
El  papel  del  lavado  sigue  siendo  controvertido,  pero  puede  beneficiar  a  los  pacientes  cuyo  recto  está  lleno  de  
heces.

Estos  principios  han  sido  cuestionados  recientemente,  pero  todavía  están  respaldados  por  la  literatura  y  las  
recomendaciones  de  la  mayoría  de  los  expertos.

13.  ¿Cómo  se  manejan  las  lesiones  rectales  extraperitoneales  complejas?
En  pacientes  con  trauma  pélvico  masivo  y  una  lesión  rectal  asociada,  puede  ser  necesaria  una  resección  abdominoperineal  
para  un  desbridamiento  y  hemostasia  adecuados.  También  se  requiere  una  resección  abdominoperineal  en  casos  raros  
en  los  que  se  han  destruido  los  esfínteres  anales.

14.  ¿Qué  complicaciones  se  asocian  con  el  traumatismo  rectal  y  su  tratamiento?
Son  similares  a  los  de  las  lesiones  colónicas.  Además,  puede  ocurrir  osteomielitis  pélvica.
En  este  caso,  puede  ser  necesario  el  desbridamiento  y  se  deben  administrar  antibióticos  intravenosos  específicos  
para  el  cultivo  durante  2  a  3  meses.
Machine Translated by Google
146  CAPÍTULO  27  TRAUMA  EN  EL  COLON  Y  EL  RECTO

15.  ¿Cuál  es  el  papel  de  los  antibióticos  en  el  trauma  colorrectal?

Los  antibióticos  son  importantes.  Deben  iniciarse  en  el  preoperatorio  (se  necesita  un  buen  nivel  en  sangre  en  el  momento  de  
realizar  la  incisión)  y  finalizar  rápidamente  (12  a  24  horas  después  de  la  operación).
La  terapia  de  combinación  de  amplio  espectro  es  superior  a  la  terapia  de  agente  único.

PUNTOS  CLAVE:  TRAUMA  COLORRECTAL

1.  La  reparación  primaria  de  las  lesiones  del  colon  es  segura.

2.  Las  anastomosis  cosidas  a  mano  y  con  grapas  tienen  las  mismas  tasas  de  complicaciones.

3.  Se  recomienda  una  dosis  preoperatoria  de  terapia  antibiótica,  que  debe  continuarse  durante  24  horas  o  menos.
ventajoso.

4.  El  tratamiento  de  las  lesiones  rectales  extraperitoneales  está  evolucionando.  La  derivación  y  el  drenaje  es  la  estrategia  
más  conservadora.

BIBLIOGRAFÍA

1.  Curran  TJ,  Borzotta  AP:  Complicaciones  de  la  reparación  primaria  de  lesiones  de  colon:  revisión  de  la  literatura  de  2964  casos.
Am  J  Surg  177:42­47,  1999.

2.  Demetriades  D,  Murray  J,  Chan  LS  et  al .:  anastomosis  con  sutura  manual  versus  anastomosis  con  grapas  en  lesiones  penetrantes  de  
colon  que  requieren  resección:  un  estudio  multicéntrico.  J  Trauma  52:117­121,  2002.

3.  Demetriades  D,  Murray  J,  Chan  L  et  al.:  Lesiones  penetrantes  del  colon  que  requieren  resección:  derivación  o  primaria
¿anastomosis?  Un  estudio  prospectivo  multicéntrico  de  la  AAST.  J  Trauma  50:765­775,  2001.

4.  Gonzalez  RP,  Phelan  H  3rd,  Hassan  M  et  al.:  ¿Es  necesaria  la  desviación  fecal  para  la  penetración  no  destructiva?
lesiones  rectales  extraperitoneales?  J  Trauma  61:815­819,  2006.

5.  Miller  PR,  Fabian  TC,  Croce  MA  et  al.:  Mejora  de  los  resultados  después  de  heridas  penetrantes  en  el  colon.  ann  surg
235:775­781,  2002.
Machine Translated by Google

FRACTURAS  PÉLVICAS
CAPÍTULO  
28

Steven  J.  Morgan,  MD,  FACS  y  Wade  R.  Smith,  MD

1.  ¿Cuáles  son  los  primeros  pasos  en  la  evaluación  y  tratamiento  de  una  paciente  con  trauma  pélvico?

El  ABC  (evaluación  de  las  vías  respiratorias,  la  respiración  y  la  circulación).  La  respuesta  a  esta  primera  
pregunta  sobre  el  trauma  es  siempre  la  misma.  Los  pacientes  traumatizados  con  fracturas  pélvicas  desplazadas  
tienen  una  alta  incidencia  de  lesiones  asociadas  en  la  cabeza,  el  tórax  y  el  abdomen.

2.  ¿Cuáles  son  las  fuentes  y  el  volumen  potencial  de  sangrado  en  la  fractura  pélvica  desplazada?

Las  fracturas  pélvicas  sangran  de  las  superficies  óseas  esponjosas  expuestas,  las  venas  pélvicas  y  las  arterias  pélvicas.
Los  estudios  de  inyección  cadavérica  han  demostrado  que  el  90  %  de  los  pacientes  con  muertes  por  trauma  con  
fracturas  pélvicas  mueren  desangrados  debido  al  hueso  expuesto  y  las  venas  lesionadas.  Sólo  el  10%  sangra  por  las  
arterias.  El  volumen  total  que  puede  contener  la  pelvis  es  de  4  a  6  L  antes  de  que  un  efecto  de  taponamiento  disminuya  
el  sangrado  venoso  y  óseo.

3.  ¿Se  debe  colocar  un  catéter  de  Foley  en  pacientes  traumatizados  con  desplazamiento  pélvico?
fracturas?
Sí.  Las  contraindicaciones  incluyen  lesiones  uretrales,  que  deben  sospecharse  ante  la  observación  de  sangre  
en  el  meato  peneano  o  en  el  introito  vaginal.  Un  examen  rectal  manual  en  hombres  y  un  examen  bimanual  en  
mujeres  son  obligatorios  para  excluir  una  fractura  abierta  en  la  vagina  o  el  recto  o  una  próstata  elevada.  Si  hay  una  
lesión  uretral,  se  puede  insertar  fácilmente  un  catéter  suprapúbico  por  vía  percutánea  y  se  realizan  tanto  un  
uretrograma  como  un  cistograma.

4.  ¿Cuál  es  la  incidencia  de  lesión  urológica  asociada  con  fracturas  pélvicas?
La  incidencia  global  es  del  16%.

5.  ¿Cuáles  son  los  esquemas  de  clasificación  radiográfica  de  uso  común  para
fracturas  pélvicas?
La  clasificación  mecanicista  describe  las  fracturas  pélvicas  como  compresión  anteroposterior  (APC),  compresión  
lateral  (LC),  cizallamiento  vertical  (VS)  o  mecanismo  combinado  (CM).  La  clasificación  de  Tile  clasifica  las  fracturas  
en  tres  grupos,  A,  B  o  C,  con  subgrupos  numerados  según  la  gravedad  creciente  de  la  rotura  de  ligamentos  y  huesos.

6.  ¿Qué  es  una  fractura  pélvica  abierta?
Una  fractura  abierta  se  ha  contaminado  a  través  de  una  laceración  en  la  piel,  la  vagina  o  el  recto.
Cuando  se  sospecha  una  fractura  pélvica  abierta,  las  pacientes  deben  recibir  un  examen  rectal  con  un  anoscopio  
y  un  examen  vaginal  realizado  de  forma  bimanual  y  con  un  espéculo.
Con  las  fracturas  abiertas,  las  tasas  de  morbilidad  y  mortalidad  aumentan  tanto  en  el  período  agudo  (por  
hemorragia)  como  en  el  período  tardío  (por  infección).  Las  lesiones  abiertas  en  la  región  rectal  o  perirrectal  a  
menudo  requieren  una  colostomía  de  derivación  para  prevenir  una  infección  pélvica  profunda.

147
Machine Translated by Google
148  CAPÍTULO  28  FRACTURAS  PÉLVICAS

7.  ¿Cuándo  está  indicada  la  estabilización  mecánica  aguda  de  una  fractura  pélvica?
Las  fracturas  de  libro  abierto  y  de  cizallamiento  vertical  con  desplazamiento  pueden  beneficiarse  de  una  estabilización  
mecánica  aguda.  Cuando  la  inestabilidad  hemodinámica  persiste  frente  a  la  reanimación  agresiva  en  curso,  la  
estabilización  pélvica  con  una  bolsa  de  frijoles,  una  envoltura  externa  o  un  dispositivo  de  fijación  externo  puede  ayudar  
a  disminuir  el  sangrado  pélvico  al  disminuir  el  volumen  pélvico  (efecto  de  taponamiento),  estabilizar  las  superficies  de  
fractura  y  promover  la  formación  de  coágulos.

8.  ¿Qué  es  el  taponamiento  pélvico  y  cuándo  se  usa?
El  taponamiento  pélvico  es  la  técnica  de  apertura  del  retroperitoneo,  ya  sea  directamente  a  través  de  un  
abordaje  tipo  Pfannenstiel  o  indirectamente  a  través  del  reflejo  peritoneal,  durante  la  laparotomía  emergente  y  la  
colocación  de  esponjas  para  absorber  y  taponar  el  sangrado.  Históricamente,  el  empaquetamiento  se  usaba  como  
último  esfuerzo  desesperado  al  final  de  las  reanimaciones  fallidas.  Informes  recientes  han  demostrado  que  el  
taponamiento  es  seguro  y  potencialmente  beneficioso  para  reducir  la  mortalidad  en  pacientes  con  trauma  pélvico  
inestable,  siempre  que  el  procedimiento  se  realice  en  las  primeras  etapas  de  la  reanimación  y  como  parte  de  un  
protocolo  multidisciplinario  con  indicaciones  específicas  y  acceso  a  angiografía  de  emergencia.

9.  ¿Cuál  es  el  papel  de  la  angiografía  en  una  fractura  pélvica  aguda?
La  angiografía  puede  identificar  y  embolizar  el  sangrado  arterial  causado  por  fracturas  pélvicas.  Pero  solo  un  bajo  
porcentaje  del  sangrado  pélvico  se  debe  a  una  lesión  arterial.  La  sospecha  debe  aumentar  cuando  los  pacientes  con  
hipotensión  no  responden  a  la  estabilización  del  anillo  pélvico  y  la  reanimación  agresiva  con  líquidos.  Si  se  emplea  
taponamiento  pélvico  retroperitoneal  durante  la  reanimación,  el  requisito  de  transfusión  continua  indica  una  alta  
probabilidad  de  hemorragia  arterial  y  la  subsiguiente  necesidad  de  angiografía.

10.  ¿Por  qué  mueren  los  pacientes  por  fracturas  pélvicas?
La  mortalidad  generalmente  es  causada  por  lesiones  asociadas  más  que  por  la  fractura  pélvica.  Solo  el  2%  de  los  
pacientes  con  una  fractura  pélvica  experimentan  un  traumatismo  aislado  en  la  pelvis.  Por  ejemplo,  los  pacientes  
con  fracturas  pélvicas  de  LC  tienen  más  probabilidades  de  morir  por  lesiones  en  la  cabeza  asociadas  que  por  
hemorragia  pélvica.  Sin  embargo,  en  las  fracturas  pélvicas  con  hemorragia  importante,  la  muerte  suele  ser  el  resultado  
de  un  shock  y  una  insuficiencia  orgánica  multisistémica  (MOF).  Limitar  la  pérdida  de  sangre  a  través  de  la  reanimación  
agresiva  con  transfusión  de  sangre/plasma  fresco  congelado  1:1,  la  estabilización  mecánica,  el  tratamiento  quirúrgico  
de  las  lesiones  asociadas,  el  taponamiento  pélvico  y  la  angiografía  para  el  shock  refractario  son  las  claves  para  
reducir  la  mortalidad  por  fractura  pélvica.
La  mortalidad  general  es  alta  para  los  pacientes  con  shock,  pero  puede  reducirse  mediante  un  protocolo  
multidisciplinario.

11.  ¿Qué  es  la  fijación  externa?
La  fijación  externa  mediante  el  uso  de  clavijas  colocadas  en  las  alas  ilíacas  y  conectadas  a  un  marco  o  mediante  
clavijas  colocadas  en  el  hueso  justo  por  encima  del  acetábulo  y  conectadas  a  una  abrazadera  en  C  puede  usarse  
como  un  método  temporal  de  reducción  y  estabilización  de  la  fractura.  La  fijación  externa  no  evita  el  desplazamiento  
vertical  y  posterior  de  la  pelvis  en  caso  de  rotura  posterior  completa.  El  dispositivo  de  fijación  debe  colocarse  de  
manera  que  permita  el  acceso  abdominal  para  la  laparotomía,  el  diagnóstico  por  imágenes  y  el  abordaje  quirúrgico  
definitivo  para  la  reducción  abierta  y  la  fijación  interna.

12.  ¿Tienen  algún  papel  las  prendas  neumáticas  antishock  en  el  tratamiento  de  las  fracturas  pélvicas?

Las  prendas  antichoque  neumáticas  (PASG)  están  perdiendo  popularidad  en  el  tratamiento  de  las  fracturas  
pélvicas.  Su  papel  potencial  se  limita  al  transporte  de  emergencia  y  la  estabilización  inicial  de  pacientes  con  una  
fractura  pélvica  compleja.  Los  PASG  pueden  reducir  el  desplazamiento  de  las  fracturas  APC  pero  pueden  aumentar  
el  desplazamiento  de  una  fractura  LC.  La  prenda  también  restringe  el  acceso  al  paciente,  compromete  la  reserva  
pulmonar  y  se  asocia  con  un  mayor  riesgo  de  síndrome  compartimental.
Machine Translated by Google
CAPÍTULO  28  FRACTURAS  PÉLVICAS  149

13.  ¿Cuándo  pueden  deambular  los  pacientes  con  una  fractura  pélvica?
Los  pacientes  con  fracturas  que  afectan  solo  al  anillo  pélvico  anterior,  como  las  fracturas  unilaterales  o  
bilaterales  de  las  ramas  púbicas,  pueden  cargar  peso  inmediatamente.  Si  el  patrón  de  fractura  involucra  las  
estructuras  posteriores,  como  la  articulación  sacroilíaca  o  el  ala  ilíaca,  los  pacientes  no  deben  soportar  peso  
durante  10  semanas.

14.  ¿Cuál  es  la  fuente  más  común  de  hemorragia  arterial  asociada  con  una  fractura  pélvica?

La  arteria  glútea  superior.

15.  ¿Qué  género  y  qué  porción  de  la  uretra  se  lesiona  con  mayor  frecuencia  en  pacientes  con  una  fractura  
pélvica  desplazada?
La  uretra  masculina  se  lesiona  con  más  frecuencia.  La  uretra  pasa  a  través  del  diafragma  urogenital  o  
piso  pélvico,  pasando  de  manera  abrupta  de  la  uretra  membranosa  a  la  bulbosa.  La  uretra  en  este  punto  
está  atenuada  y  relativamente  fija  arriba,  lo  que  explica  el  gran  número  de  lesiones  en  la  unión  bulbo­
membranosa.  La  uretra  femenina  es  mucho  más  corta  y  el  suelo  pélvico  está  menos  desarrollado,  lo  que  permite  
una  mayor  movilidad  de  la  uretra  femenina  (o  quizás  se  deba  a  que  las  niñas  son  más  inteligentes,  más  
cautelosas  y  no  se  lesionan  con  tanta  frecuencia).  El  sitio  más  común  de  lesión  uretral  en  niñas  y  mujeres  es  el  
cuello  de  la  vejiga.

16.  Describa  el  mecanismo  que  provoca  la  ruptura  de  la  vejiga.
La  vejiga  es  una  estructura  tanto  intraperitoneal  como  extraperitoneal.  La  compresión  de  una  vejiga  
distendida  da  como  resultado  una  ruptura  intraperitoneal  a  lo  largo  de  la  cúpula  de  la  vejiga.  La  ruptura  
extraperitoneal,  una  lesión  más  común,  se  debe  a  la  laceración  de  la  vejiga  por  fragmentos  de  fractura  de  ramas  
púbicas  desplazadas.

17.  ¿Cuáles  son  las  tres  vistas  radiográficas  requeridas  para  evaluar  pacientes  con
fracturas?
1.  Vista  anteroposterior  (AP)  de  la  pelvis  2.  
Vista  de  entrada  3.  Vista  de  salida

18.  ¿Cuál  es  la  ubicación  de  inserción  adecuada  para  un  lavado  peritoneal  de  diagnóstico?
catéter  en  presencia  de  una  fractura  pélvica?
Una  ubicación  supraumbilical  evita  la  descompresión  inadvertida  del  hematoma  pélvico  y  un  resultado  falso  
positivo.

19.  ¿Qué  porcentaje  de  pacientes  con  una  fractura  pélvica  inestable  sufrirá  una  lesión  neurológica  asociada?

Según  los  informes,  las  lesiones  asociadas  del  plexo  lumbosacro,  los  agujeros  sacros  y  el  canal  sacro  alcanzan  
el  50%.

20.  ¿Cuál  es  una  trampa  potencial  de  la  transfusión  de  sangre  agresiva  de  pacientes  con
¿Fractura  pélvica  hemodinámicamente  inestable?
Coagulopatía.  El  cuarenta  por  ciento  de  los  pacientes  con  fracturas  pélvicas  inestables  pueden  requerir  10  
unidades  de  sangre.  El  plasma  fresco  congelado  y  las  plaquetas  deben  transfundirse  al  principio  de  la  reanimación.

21.  ¿Cuál  es  el  significado  de  una  fractura  de  apófisis  transversa  L5  en  un  paciente
con  una  fractura  de  pelvis?
Una  fractura  de  apófisis  transversa  (TP)  a  nivel  de  L5  puede  indicar  inestabilidad  vertical  de  la  fractura  pélvica.  
Los  ligamentos  iliolumbares  se  unen  a.
Machine Translated by Google
150  CAPÍTULO  28  FRACTURAS  PÉLVICAS

PUNTOS  CLAVE:  PÉRDIDA  DE  SANGRE  POR  FRACTURAS  PÉLVICAS

1.  El  noventa  por  ciento  de  las  muertes  relacionadas  con  el  sangrado  pélvico  son  el  resultado  de  sangrado  venoso  y  óseo.

2.  El  10%  restante  son  el  resultado  de  una  hemorragia  arterial,  más  comúnmente  de  la  parte  superior

arteria  glútea.

3.  Normalmente,  la  pelvis  puede  contener  de  4  a  6  litros  de  sangre  antes  de  que  se  produzca  un  efecto  de  taponamiento.

4.  Las  envolturas  pélvicas  o  la  fijación  pueden  limitar  el  sangrado,  reducir  el  cizallamiento  óseo  y  promover  la  formación  de  coágulos.

5.  La  angiografía  es  terapéutica  y  diagnóstica,  pero  solo  el  10%  de  las  lesiones  son  predominantemente
arterial.

6.  La  mortalidad  general  es  alta  para  los  pacientes  con  shock,  pero  puede  reducirse  mediante  un  enfoque  multidisciplinario.
protocolo.

SITIOS  WEB

www.east.org/tpg/pelvis.pdf

BIBLIOGRAFÍA

1.  Biffl  WL,  Smith  WR,  Moore  EE  et  al.:  Evolución  de  una  vía  clínica  clave  multidisciplinaria  para  el  manejo
de  fracturas  de  pelvis  inestables.  Ann  Surg  233:843­850,  2001.

2.  Burgess  AR,  Eastridge  BJ,  Young  JW  et  al .:  Disrupciones  del  anillo  pélvico:  sistema  de  clasificación  y  tratamiento  efectivos
protocolos  J  Trauma  30:848­856,  1990.

3.  Cook  RE,  Keating  JF,  Gillespie  I:  El  papel  de  la  angiografía  en  el  manejo  de  la  hemorragia  por
fracturas  de  la  pelvis.  J  Bone  Joint  Surg  84B:178­182,  2002.

4.  Ghaemmaghami  V,  Sperry  J,  Gunst  M  et  al .:  Efectos  del  uso  temprano  de  compresión  pélvica  externa  sobre  los  requisitos  de  transfusión  
y  la  mortalidad  en  fracturas  pélvicas.  Am  J  Surg  194(6):720­723;  discusión  723,  2007.

5.  Hauschild  O,  Strohm  PC,  Culemann  U  et  al.:  Mortalidad  en  pacientes  con  fracturas  pélvicas:  resultados  de  la
Registro  alemán  de  lesiones  pélvicas.  J  Trauma  64(2):449­455,  2008.

6.  Pérez  JV,  Hughes  TM,  Bowers  K:  Embolización  angiográfica  en  fractura  pélvica.  Lesión  29:187­191,  1998.

7.  Smith  W,  Williams  A,  Agudelo  J  et  al .:  Predictores  tempranos  de  mortalidad  en  fracturas  de  pelvis  hemodinámicamente  
inestables.  J  Orthop  Trauma  21(1):31­37,  2007.

8.  Smith  WR,  Moore  EE,  Osborn  P  et  al.:  Empaquetamiento  retroperitoneal  como  técnica  de  reanimación  para
Pacientes  hemodinámicamente  inestables  con  fracturas  pélvicas:  reporte  de  dos  casos  representativos  y  descripción  de  la  técnica.  J  
Trauma  59(6):1510­1514,  2005.

9.  Starr  AJ,  Griffin  DR,  Reinert  CM  et  al .:  rupturas  del  anillo  pélvico:  predicción  de  lesiones  asociadas,  necesidad  de  transfusión,  
arteriografía  pélvica,  complicaciones  y  mortalidad.  J  Orthop  Trauma  16:553­561,  2002.

10.  Stover  MD,  Summers  HD,  Ghanayem  AJ  et  al.:  Análisis  tridimensional  del  volumen  pélvico  en  una  fractura  pélvica  inestable.  J  
Trauma  61(4):905­908,  2006.

11.  To¨tterman  A,  Madsen  JE,  Skaga  NO  et  al.:  Empaquetamiento  pélvico  extraperitoneal:  un  procedimiento  de  rescate  para  controlar
Hemorragia  pélvica  traumática  masiva.  J  Trauma  62(4):843­852,  2007.

12.  Velmahos  GC,  Toutouzas  KG,  Vassiliu  P  et  al.:  Un  estudio  prospectivo  sobre  la  seguridad  y  eficacia  de  angiografía
embolización  de  lesiones  pélvicas  y  viscerales.  J  Trauma  53:303­308,  2002.
Machine Translated by Google

LESIONES  DEL  TRACTO  URINARIO  SUPERIOR
CAPÍTULO  
29

Fernando  J.  Kim,  MD,  FACS;

1.  ¿Cuál  es  el  tipo  más  común  de  traumatismo  renal  en  los  Estados  Unidos,  cerrado  o  penetrante?

Contundente,  de  lejos.

2.  ¿La  mayoría  de  las  lesiones  renales  requieren  cirugía?
No.  Menos  del  2  %  de  las  lesiones  contusas  requieren  cirugía,  y  muchas  lesiones  penetrantes  también  pueden  
tratarse  sin  cirugía.

3.  ¿Son  los  riñones  pediátricos  más  susceptibles  a  lesiones  mayores?
Sí.  Debido  a  los  músculos  abdominales  más  débiles  de  los  niños,  la  caja  torácica  menos  osificada,  la  grasa  
perirrenal  disminuida  y  el  tamaño  renal  aumentado  en  relación  con  el  resto  del  cuerpo,  el  riesgo  de  lesión  renal  
es  mayor  en  la  población  pediátrica.

4.  ¿Cuándo  debe  investigarse  un  posible  traumatismo  renal?
Todos  los  pacientes  con  trauma  cerrado  con  hematuria  macroscópica  o  con  hematuria  microscópica  y  
shock  (presión  arterial  sistólica  <90  mm  Hg)  deben  ser  examinados  de  cerca.  Se  deben  obtener  imágenes  de  
lesiones  penetrantes  con  cualquier  grado  de  hematuria.  Para  pacientes  pediátricos,  se  recomienda  el  uso  liberal  
de  estudios.  Cuando  los  niños  derraman  menos  de  50  glóbulos  rojos  por  campo  de  alta  potencia  (hpf)  en  el  
análisis  microscópico,  la  lesión  renal  significativa  es  rara.

5.  ¿Cuándo  se  sospecha  un  traumatismo  renal?
El  mecanismo  de  la  lesión,  el  examen  físico  (es  decir,  equimosis  en  el  flanco,  ubicación  de  heridas  penetrantes)  
y  las  lesiones  asociadas  (p.  ej.,  fracturas  costales)  deben  hacer  sospechar  un  traumatismo  renal.  Aunque  el  
grado  de  hematuria  no  se  correlaciona  con  el  grado  de  lesión  renal,  cuando  la  hematuria  no  guarda  proporción  
con  los  antecedentes  de  traumatismo,  sugiere  una  anomalía  renal  preexistente  (p.  ej.,  hidronefrosis,  riñón  
ectópico,  tumor,  enfermedad  quística,  malformación  vascular).  Por  el  contrario,  las  lesiones  del  pedículo  renal  
(grado  4)  pueden  causar  poca  o  ninguna  hematuria  debido  a  la  interrupción  arterial.

6.  ¿Qué  estudio  de  imagen  es  el  mejor  para  evaluar  el  traumatismo  renal?
Se  debe  realizar  una  tomografía  computarizada  (TC)  del  abdomen  y  la  pelvis  con  y  sin  contraste  intravenoso  (IV),  
pero  es  fundamental  que  las  fases  de  perfusión  y  excreción  (10  minutos  después  de  administrar  el  contraste  IV)  
se  obtengan  durante  el  estudio.

7.  ¿Qué  es  un  "IVP  de  disparo  único"  y  cuándo  se  realiza?
La  IVP  de  disparo  único  es  un  estudio  de  imágenes  en  el  que  solo  se  toma  una  película  10  minutos  después  de  la  
inyección  intravenosa  (IVP)  de  2  ml/kg  de  material  de  contraste.  Se  realiza  en  situaciones  en  las  que  no  se  puede  
realizar  la  estadificación  preoperatoria  por  TC  renal  (es  decir,  el  paciente  se  somete  a  exploración  inmediata  y  
está  hemodinámicamente  inestable).

151
Machine Translated by Google
152  CAPÍTULO  29  LESIONES  DEL  TRACTO  URINARIO  SUPERIOR

8.  ¿Cómo  se  clasifica  el  traumatismo  renal?
Grado  1:  Contusión.  Hay  hematoma  subcapsular,  no  expansivo  sin  parénquima
laceración

Grado  2:  laceración  superficial  (<1  cm  de  profundidad  del  parénquima  de  la  corteza  renal  sin  orina)
extravasación)
Grado  3:  laceración  profunda  (>1  cm  de  profundidad  del  parénquima  de  la  corteza  renal  sin  rotura  del  sistema  
colector  ni  extravasación  urinaria)
Grado  4:  laceración  del  parénquima  que  se  extiende  a  través  de  la  corteza  renal,  la  médula  y  el  sistema  colector  
o  lesión  de  la  arteria  o  vena  renal  con  hemorragia  contenida
Grado  5:  Riñón  destrozado  o  avulsión  del  hilio  renal

9.  ¿Cómo  es  el  manejo  según  el  grado  de  traumatismo?
Un  paciente  hemodinámicamente  estable  con  una  lesión  bien  estadificada  por  TC  por  lo  general  puede  
manejarse  sin  exploración  renal;  de  hecho,  el  98%  de  las  lesiones  renales  cerradas  pueden  tratarse  sin  cirugía.  
Las  lesiones  de  grado  IV  y  V  requieren  con  mayor  frecuencia  exploración  quirúrgica,  pero  incluso  estas  lesiones  de  alto  
grado  pueden  tratarse  sin  operación  renal  si  se  las  selecciona  y  clasifica  cuidadosamente.

10.  ¿Cuáles  son  los  diferentes  tipos  de  traumatismo  del  pedículo  renal?
El  pedículo  renal  puede  interrumpirse  por  trombosis  o  avulsión  completa;  ambos  eventos  se  caracterizan  por  no  
visualización  urográfica  y  hematuria  mínima.  El  sitio  más  común  de  interrupción  arterial  es  la  unión  de  los  tercios  
proximal  y  medio  de  la  arteria  renal  principal.
Aunque  la  hematuria  suele  estar  ausente,  se  puede  observar  hematuria  o  microhematuria  macroscópica  transitoria,  lo  
que  enfatiza  la  necesidad  de  un  análisis  de  orina  en  todas  las  circunstancias.

11.  ¿Cuánto  tiempo  puede  tolerar  un  riñón  no  perfundido  la  isquemia  caliente?
Se  puede  observar  daño  renal  irreversible  en  los  riñones  después  de  30  minutos  de  isquemia  caliente  y  después  de  8  
horas  de  isquemia,  la  recuperación  renal  es  mínima.  Recientemente,  informes  únicos  de  traumatismo  renovascular  con  
desgarro  de  la  íntima  tratados  con  stents  endovasculares  han  sido  alentadores.

12.  ¿Cuál  es  el  significado  de  la  hematuria  macroscópica  retardada?
Esto  ocurre  de  3  a  4  semanas  después  del  trauma  y  puede  indicar  una  fístula  arteriovenosa.  La  embolización  
selectiva  es  el  siguiente  paso  si  falla  la  terapia  conservadora  (reposo  en  cama).  En  raras  ocasiones,  es  necesaria  
una  intervención  quirúrgica,  por  lo  general  para  una  nefrectomía  parcial.

13.  ¿Cómo  maneja  un  sangrado  retroperitoneal  inesperado  durante  una  cirugía?
¿exploración?
Un  hematoma  pulsátil  sugiere  una  lesión  vascular  importante,  y  la  exploración  debe  ir  precedida  de  control  vascular  
(tanto  proximal  como  distal)  y  preparación  para  una  rápida  reposición  de  sangre.
Los  hematomas  estables  (por  encima  del  borde  pélvico)  pueden  dejarse  intactos  a  menos  que  los  
estudios  (preoperatorios  o  intraoperatorios)  revelen  daño  renal  grave.  Cuando  existe  duda,  se  justifica  la  exploración,  
con  probabilidad  de  pérdida  de  un  riñón.

14.  ¿Cómo  se  manejan  los  pacientes  con  extravasación  de  orina  postraumática?
Cuando  la  extravasación  de  orina  es  causada  por  una  laceración  importante  en  el  sistema  colector  y  coexiste  
con  un  sangrado  persistente  significativo,  se  recomienda  la  corrección  quirúrgica.  De  lo  contrario,  la  extravasación  
de  orina  comúnmente  se  resuelve  rápidamente.  La  nueva  imagen  a  las  48  a  72  horas  define  los  casos  que  
requieren  drenaje,  colocación  de  stent  o  reparación  quirúrgica.

15.  ¿Qué  incluye  el  manejo  conservador  del  trauma  renal?
El  tratamiento  conservador  incluye  reposo  en  cama  hasta  que  haya  remitido  la  hematuria  macroscópica.  Se  evitan  las  
actividades  extenuantes  hasta  que  la  microhematuria  haya  desaparecido  (generalmente  dentro  de  las  3  semanas).  El  
paciente  seguido  por  un  traumatismo  renal  de  grado  5  debe  someterse  a  una  ecografía,  una  tomografía  computarizada  
del  abdomen  y  la  pelvis  o  una  urografía  a  las  6  semanas.  No  se  requiere  hospitalización  durante  estos  períodos.
Machine Translated by Google
CAPÍTULO  29  LESIONES  DEL  TRACTO  URINARIO  SUPERIOR  153

16.  ¿Cuál  es  la  probabilidad  de  hipertensión  posterior?
La  hipertensión  postraumática  documentada  ocurre  en  <2%  de  los  pacientes  y  está  mediada  por  la  renina.  El  
inicio  generalmente  ocurre  dentro  de  los  primeros  meses  de  la  lesión.  Los  mecanismos  de  la  hipertensión  
postraumática  pueden  ser  el  resultado  de  estenosis  u  oclusión  de  la  arteria  renal,  compresión  del  parénquima  
renal  (extravasación  de  sangre  u  orina)  y  fístula  arteriovenosa  postraumática.

PUNTOS  CLAVE:  PRINCIPIOS  DE  LA  REPARACIÓN  URETERAL

1.  Cualquier  nivel  de  anastomosis  ureteral  primaria  debe  realizarse  manteniendo  un  suministro  vascular  
ureteral  adecuado  durante  la  disección  de  los  extremos  ureterales,  sitio  de  anastomosis  ureteral  libre  
de  tensión,  sobre  stent  con  sutura  absorbible.

2.  Para  una  lesión  distal  en  el  tercio  inferior  del  uréter  por  debajo  de  los  vasos  ilíacos,  se  recomienda  la  
ureteroneocistostomía.  Si  se  necesita  más  longitud  ureteral  para  mantener  la  anastomosis  libre  de  tensión,  
se  puede  realizar  un  colgajo  de  Bohari  (colgajo  de  vejiga)  y  un  enganche  de  psoas.

3.  Para  las  lesiones  del  tercio  medio  y  proximal,  se  recomienda  la  anastomosis  terminoterminal,  pero  la  
tasa  de  estenosis  de  la  anastomosis  es  más  alta  que  la  reparación  ureteral  proximal  o  distal.

4.  Para  una  lesión  proximal  (lesión  de  la  unión  ureteropélvica),  más  común  en  niños,  se  necesita  una  laparotomía  
inmediata  y  reparación  quirúrgica.

17.  ¿Cómo  se  dañan  la  mayoría  de  los  uréteres?
El  uréter  es  el  órgano  genitourinario  que  se  lesiona  con  menos  frecuencia  y  representa  menos  del  1%  de  todos  los  
traumatismos  urológicos  secundarios  a  violencia  externa.  La  lesión  ureteral  ocurre  con  mayor  frecuencia  durante  la  
operación  (80%)  que  por  un  traumatismo  violento  externo  (20%).  Los  traumatismos  cerrados  y  las  heridas  de  arma  blanca  
(SW,  por  sus  siglas  en  inglés)  rara  vez  provocan  lesiones  en  el  uréter.  Los  traumatismos  cerrados  representan  el  4,1%  y  los  
SW  el  5,2%  de  todos  los  traumatismos  ureterales.  Las  lesiones  ureterales  más  frecuentes  por  violencia  externa  son  las  
heridas  por  arma  de  fuego  (TSG;  90,7%).

18.  ¿Cómo  evalúa  e  identifica  una  lesión  ureteral?
El  sitio  y  el  mecanismo  del  traumatismo  deben  llevar  al  cirujano  a  sospechar  una  lesión  ureteral.
Las  manifestaciones  clínicas  son  característicamente  sutiles  y,  a  menudo,  oscurecidas  por  lesiones  y  
molestias  coexistentes.  La  mayoría  de  los  SW  y  GSW  también  dañarían  el  intestino,  el  colon,  el  hígado,  el  
bazo,  los  vasos  sanguíneos  o  el  páncreas.  La  hematuria  suele  ser  microscópica,  pero  puede  estar  ausente.
La  extravasación  de  contraste  se  puede  detectar  con  estudios  de  imagen  no  invasivos  (IVP  y  CT)  e  invasivos  
(ureteropielograma  anterógrado  y  retrógrado).  Si  se  sospecha  una  lesión  ureteral  durante  la  laparotomía,  se  debe  
administrar  índigo  carmín  (1  vial  en  bolo  IV)  para  identificar  el  sitio  de  la  fuga  (coloración  azul).

19.  ¿Cuáles  son  las  posibles  consecuencias  de  una  lesión  ureteral  no  detectada?
Fiebre,  leucocitosis,  azotemia,  dolor  en  el  costado,  íleo,  urinoma  o  fístula  urinaria.  La  presentación  a  menudo  se  
retrasa  varias  semanas  después  de  la  lesión.

20.  Uréter  distal  lesionado  y  reimplante  ureteral  con  enganche  de  psoas
(fijar  la  vejiga  al  músculo  psoas).  Después  de  la  operación,  el  paciente  se  queja  de  entumecimiento  del  
muslo  anterior.  ¿Qué  hiciste  mal?

El  nervio  genitofemoral  se  encuentra  en  la  cara  anterior  del  músculo  ileopsoas.  Atrapó  este  nervio  cuando  lo  
sincronizó  con  el  tendón  del  músculo  psoas.
Machine Translated by Google
154  CAPÍTULO  29  LESIONES  DEL  TRACTO  URINARIO  SUPERIOR

SITIOS  WEB

www.east.org/tpg/GUmgmt.pdf

BIBLIOGRAFÍA

1.  Alsikafi  NF,  McAninch  JW,  Elliott  SP  et  al .:  Resultados  del  manejo  no  quirúrgico  de  la  extravasación  urinaria  
aislada  después  de  laceraciones  renales  debido  a  un  trauma  externo.  J  Urol  176(6  Pt  1):2494,  2006.
2.  Alsikafi  NF,  Rosenstein  DI:  Estadificación,  evaluación  y  tratamiento  no  quirúrgico  de  las  lesiones  renales.  Urol  Clín  Norte
Am  33:13,  2006.

3.  Broghammer  JA,  Fisher  MB,  Santucci  RA:  Manejo  conservador  del  trauma  renal:  una  revisión.  Urología  70:623,
2007.

4.  Elliott  SP,  McAninch  JW:  Lesiones  ureterales:  externas  e  iatrogénicas.  Urol  Clin  N  Am  33:55,  2006.
5.  Elliott  SP,  McAninch  JW:  Lesiones  ureterales  por  violencia  externa:  la  experiencia  de  25  años  en  San  Francisco
Hospital  General.  JUrol  170:1213,  2003.

6.  Santucci  RA,  Fisher  MB:  La  literatura  respalda  cada  vez  más  el  manejo  expectante  (conservador)  de  la  insuficiencia  renal.
trauma:  una  revisión  sistemática.  J  Trauma  59(2):493,  2005.
Machine Translated by Google

LESIÓN  DEL  TRACTO  URINARIO  INFERIOR
Y  TRAUMA  PÉLVICO
CAPÍTULO  
30

Mario  F.  Chammas,  Jr.,  MD;

1.  ¿Cuáles  son  las  causas  de  la  lesión  de  la  vejiga?
Manipulación  iatrogénica  y  traumatismo  penetrante  o  cerrado.  Debido  a  la  abundante  irrigación  sanguínea  del  
detrusor,  la  lesión  de  la  vejiga  suele  ir  acompañada  de  hematuria.  Otros  signos  pueden  incluir  dolor  suprapúbico,  
incapacidad  para  orinar  o  recuperación  incompleta  de  la  irrigación  del  catéter.

2.  ¿Qué  tipos  de  lesiones  vesicales  pueden  ocurrir  con  un  traumatismo  cerrado?
La  laceración  o  perforación  puede  ser  intraperitoneal  o  extraperitoneal.  La  hematuria  con  un  cistograma  normal  
define  la  contusión  de  la  vejiga  en  ausencia  de  lesión  del  tracto  superior.  Las  lesiones  extraperitoneales  constituyen  la  
mayoría  de  los  traumatismos  vesicales  y  tienden  a  concentrarse  en  la  base  de  la  vejiga  o  en  el  área  parasinfisaria.  Por  
lo  general,  estos  pueden  manejarse  de  manera  conservadora  con  un  catéter  urinario  durante  10  días.  Las  rupturas  
intraperitoneales  ocurren  típicamente  cuando  la  vejiga  se  distiende  en  el  momento  del  traumatismo,  lo  que  provoca  un  
estallido  de  la  cúpula  de  la  vejiga.  La  ruptura  vesical  intraperitoneal  debe  repararse  quirúrgicamente  mediante  un  cierre  
de  dos  capas  con  suturas  absorbibles  y  la  colocación  de  catéteres  suprapúbicos  y  uretrales.

3.  ¿Cuál  es  la  probabilidad  de  lesión  vesical  en  pacientes  con  fractura  de  pelvis?
La  lesión  vesical  extraperitoneal  ocurre  en  el  10%  de  todas  las  fracturas  pélvicas.  Por  el  contrario,  aproximadamente  
el  85%  de  las  lesiones  vesicales  cerradas  se  asocian  con  una  fractura  pélvica.  Las  lesiones  vesicales  ocurren  más  a  
menudo  con  las  fracturas  del  arco  púbico  parasinfisario  y  más  a  menudo  con  las  fracturas  bilaterales  que  con  las  
unilaterales.  Las  fracturas  aisladas  de  rama  producen  laceración  de  la  vejiga  en  el  10%  de  los  casos.

4.  ¿Cómo  se  evalúa  la  lesión  vesical?
Tanto  la  cistografía  por  tomografía  computarizada  (TC)  como  la  cistouretrografía  retrógrada  brindan  una  gran  
precisión  diagnóstica  para  la  ruptura  vesical.  La  vejiga  debe  llenarse  por  gravedad  con  un  total  de  300  a  400  ml  de  una  
dilución  al  50  %  de  un  agente  de  contraste  de  radio  estándar  utilizando  el  catéter  de  Foley.  Las  radiografías  deben  
incluir  vistas  anteroposterior  (AP),  lateral  y  oblicua.  Finalmente,  se  debe  obtener  una  película  posmiccional.  Cuando  se  
sospecha  lesión  renal  o  ureteral  distal,  las  imágenes  del  tracto  superior  (pielograma  intravenoso  [PIV]  o  tomografía  
computarizada)  deben  preceder  al  cistograma.

5.  ¿Cuáles  son  los  patrones  cistouretrográficos  retrógrados  de  la  lesión  vesical?
La  lesión  extraperitoneal  permite  que  el  agente  de  contraste  escape  junto  a  la  sínfisis,  pero  queda  
confinado  a  la  base  de  la  vejiga  por  el  peritoneo  intacto.  La  extravasación  intraperitoneal  produce  una  
apariencia  de  "rayo  de  sol"  desde  la  cúpula  de  la  vejiga,  que  puede  acumularse  en  los  canalones  paracólicos,  perfilar  
las  asas  intestinales  o  acumularse  debajo  del  hígado  o  el  bazo.  Es  fundamental  obtener  películas  posmiccionales.

6.  ¿Cómo  se  maneja  la  ruptura  de  la  vejiga?
Las  laceraciones  extraperitoneales  se  pueden  tratar  con  un  catéter  permanente  durante  7  a  10  días,  momento  en  
el  cual  el  cistograma  suele  confirmar  la  resolución  de  la  extravasación.  Las  laceraciones  intraperitoneales  requieren  
reparación  quirúrgica.  En  pacientes  seleccionados  un  abordaje  laparoscópico  podría  ser  una  opción  para  estos  
pacientes.  La  contusión  de  la  vejiga  requiere  drenaje  con  catéter  hasta  que  la  hematuria  macroscópica  haya  remitido.

155
Machine Translated by Google
156  CAPÍTULO  30  LESIÓN  DEL  TRACTO  URINARIO  INFERIOR  Y  TRAUMA  PÉLVICO

7.  ¿Cuándo  debe  investigarse  la  lesión  uretral?
El  mecanismo  de  la  lesión  (p.  ej.,  aplastamiento  o  desaceleración/impacto,  lesiones  a  horcajadas)  
y  el  trauma  asociado  (p.  ej.,  fractura  pélvica),  sangre  en  el  meato,  inflamación  y  equimosis  del  pene  o  el  
escroto,  desplazamiento  prostático  hacia  arriba  en  el  tacto  rectal  e  incapacidad  para  orinar  o  para  pasar  un  
catéter  uretral  debe  ser  investigado.

8.  Cuando  un  paciente  presenta  una  fractura  pélvica,  es  concomitante  uretral
lesión  una  preocupación  importante?
Sí.  El  traumatismo  uretral  ocurre  en  el  10%  de  las  fracturas  pélvicas;  es  más  común  con  la  ruptura  
anterior  del  anillo  pélvico,  incluido  el  20%  de  las  fracturas  parasinfisarias  unilaterales  y  el  50%  de  las  
bilaterales.  La  avulsión  posterior  (prostatomembranosa)  se  asocia  con  posibles  secuelas  incapacitantes  y  
requisitos  para  correcciones  quirúrgicas  complejas  y  desafiantes.  Por  el  contrario,  las  lesiones  de  la  uretra  
más  distal  evitan  problemas  de  impotencia  e  incontinencia  y  son  más  accesibles  quirúrgicamente.

9.  ¿Cómo  se  evalúa  mejor  la  lesión  uretral?
Siempre  se  debe  realizar  una  uretrografía  retrógrada  antes  de  insertar  un  catéter  de  
Foley.  La  transacción  uretral  incompleta  produce  extravasación  local  del  medio  de  contraste  y  opacificación  
de  la  vejiga.  La  avulsión  total  produce  una  extravasación  local  extensa  y  no  llega  medio  de  contraste  a  la  
vejiga.  La  sección  incompleta  es  más  común  con  lesiones  uretrales  anteriores  (50%)  que  posteriores  (10%).

10.  ¿Cómo  se  maneja  la  lesión  uretral?
Para  la  transacción  incompleta  sin  importar  el  sitio,  ya  sea  la  colocación  de  un  stent  a  través  del  
defecto  (realizado  por  el  urólogo)  o  la  derivación  por  cistostomía  suprapúbica  permite  la  resolución.
Con  la  sección  uretral  completa,  la  vejiga  debe  descomprimirse  inicialmente  a  través  de  una  cistostomía  
suprapúbica.  Si  es  posible,  la  restauración  temprana  de  la  continuidad  mediante  la  colocación  de  un  catéter  
uretral  puente  debe  realizarse  por  vía  endoscópica.

11.  ¿Cuáles  son  las  complicaciones  de  la  lesión  uretral?
Estenosis,  incontinencia  e  impotencia  (asociadas  con  desplazamiento  prostático  traumático).
Las  complicaciones  iatrogénicas  se  asocian  con  la  disección  retropúbica  durante  el  manejo  
quirúrgico  de  la  lesión.

12.  ¿Cuál  es  el  diagnóstico  diferencial  en  el  trauma  cerrado  de  escroto?
Rotura  testicular,  hematocele,  hematoma  escrotal,  hematoma  intratesticular  y  torsión  testicular.  La  
ecografía  ayuda  a  resolver  esto.

13.  ¿Cuál  es  el  signo  ecográfico  de  rotura  testicular?
El  signo  de  pérdida  de  la  ecotextura  homogénea  normal  del  testículo,  con  áreas  de  hiperecogenicidad  o  
hipoecogenicidad  irregular.

14.  ¿Cómo  se  manejan  los  pacientes  con  ruptura  testicular  aguda?
El  tratamiento  incluye  la  exploración  quirúrgica  y  el  desbridamiento  de  los  túbulos  extruidos  no  viables  y  
la  evacuación  del  hematoma.  Una  vez  lograda  la  hemostasia  adecuada,  la  túnica  albugínea  debe  cerrarse  
con  suturas  absorbibles  continuas.

15.  ¿Cuál  es  la  causa  más  común  de  las  fracturas  de  pene?
La  fractura  de  pene  es  una  ruptura  del  cuerpo  cavernoso,  más  comúnmente  asociada  con  las  
relaciones  sexuales,  la  masturbación  agresiva  o  una  flexión  anormalmente  forzada  del  pene  erecto.  
De  manera  característica,  el  paciente  oye  un  chasquido,  seguido  de  dolor  y  detumescencia.
Machine Translated by Google
CAPÍTULO  30  LESIÓN  DEL  TRACTO  URINARIO  INFERIOR  Y  TRAUMA  PÉLVICO  157

16.  ¿Cuáles  son  los  hallazgos  del  examen  físico  con  una  fractura  de  pene?
La  lesión  de  la  túnica  albugínea  provoca  la  formación  de  un  hematoma  y  la  desviación  de  la  diáfisis  hacia  el  lado  
opuesto  de  la  lesión.  Si  la  fascia  de  Buck  está  intacta,  el  hematoma  se  limitará  al  pene  ("deformidad  en  berenjena");  
la  interrupción  de  la  fascia  de  Buck  permite  la  propagación  del  hematoma  debajo  de  la  fascia  de  Colle  y  Scarpa  
hacia  el  perineo  y  la  pared  abdominal.

17.  ¿Cómo  se  manejan  las  fracturas  de  pene?
Quirúrgicamente.  El  cierre  del  defecto  (o  defectos)  a  lo  largo  de  la  túnica  albugínea  y  la  evacuación  del  
hematoma  se  realizan  después  de  desenguantar  el  pene.  Se  debe  realizar  una  uretrografía  retrógrada  
cuando  se  sospeche  una  lesión  uretral  porque  puede  ocurrir  en  aproximadamente  el  20%  de  los  casos.

18.  En  las  lesiones  por  amputación  del  pene,  ¿cómo  se  debe  realizar  la  parte  amputada  del  pene?
ser  conservado  para  el  transporte?
La  porción  amputada  del  pene  debe  envolverse  en  una  gasa  empapada  en  solución  salina  y  colocarse  en  una  bolsa  
esterilizada  sellada,  luego  la  bolsa  que  contiene  el  pene  protegido  se  coloca  en  un  baño  de  aguanieve  (procedimiento  
de  doble  bolsa),  el  hielo  no  debe  en  contacto  directo  con  la  piel  del  pene.
El  pene  debe  reimplantarse  dentro  de  las  primeras  24  horas  después  de  la  lesión.

19.  ¿Cómo  se  maneja  la  pérdida  importante  de  piel  escrotal?
Si  la  reparación  primaria  no  es  posible,  se  pueden  usar  injertos  de  piel  de  espesor  parcial  en  malla  para  cubrir  
el  testículo.  Cuando  es  necesaria  una  reparación  diferida,  se  deben  crear  bolsas  en  los  muslos  hasta  que  sea  factible  
una  reconstrucción  permanente.

20.  Una  mujer  de  50  años  se  queja  de  pérdidas  de  orina  por  la  vagina  después  de  una
histerectomía.  ¿Cuál  es  el  diagnóstico  más  probable?
La  lesión  vesical  no  detectada  durante  la  histerectomía  con  la  subsiguiente  extravasación  de  orina  en  el  
campo  quirúrgico  y  el  drenaje  a  través  de  la  línea  de  sutura  vaginal  conduce  a  la  formación  de  una  fístula  vesicovaginal.

21.  ¿Cuál  es  el  mejor  momento  para  reparar  una  fístula  vesicovaginal  secundaria  a  una  histerectomía  no  
complicada?
Aunque  en  el  pasado  se  ha  recomendado  de  3  a  6  meses  después  de  la  lesión,  la  reparación  temprana  puede  
tener  éxito  si  hay  una  inflamación  mínima  y  no  hay  factores  de  complicación.

SITIO  WEB

www.east.org/tpg/GUmgmt.pdf

BIBLIOGRAFÍA

1.  Chapple  C,  Turner­Warwick  R:  fístula  vesico­vaginal.  BJU  Int  95:193,  2005.
2.  Corriere  JN  Jr,  Sandler  CM:  Diagnóstico  y  tratamiento  de  las  lesiones  vesicales.  Urol  Clin  Norte  Am  33:67,  2006.
3.  Kim  FJ,  Chammas  MF  Jr,  Gewehr  EV  et  al .:  Manejo  laparoscópico  de  la  ruptura  de  la  vejiga  intraperitoneal  
secundaria  a  un  traumatismo  abdominal  cerrado  mediante  la  técnica  de  sutura  intracorpórea  de  una  sola  capa.  J  
Trauma  65(1):234­236,  2008.
4.  Kommu  SS,  Illahi  I,  Mumtaz  F:  Patrones  de  lesión  uretral  y  manejo  inmediato.  Curr  Opinión  Urol  17:383,
2007.

5.  Morey  AF,  Metro  MJ,  Carney  KJ  et  al .:  Consenso  sobre  trauma  genitourinario:  genitales  externos.  BJU  Int  94:507,
2004.

6.  Wessells  H,  Long  L:  Lesiones  genitales  y  del  pene.  Urol  Clin  Norte  Am  33:117,  2006.
Machine Translated by Google

LESIONES  VASCULARES  DE  LAS  EXTREMIDADES
CAPÍTULO  
31

Ashok  N.  Babu,  MD  y  Ernest  E.  Moore,  MD

1.  ¿Cuáles  son  los  «signos  duros»  de  lesión  arterial?
  Déficit  circulatorio  distal:  isquemia  o  pulsos  disminuidos  o  ausentes.
y  ruido.
  Hematoma  expansivo  o  pulsátil.
  Sangrado  arterial  (pulsátil).

2.  ¿Cuáles  son  las  cuatro  formas  en  que  se  puede  presentar  una  lesión  arterial?
1.  Hemorragia.
2.  Trombosis.
3.  Fístula  arteriovenosa.
4.  Pseudoaneurisma.

3.  ¿Cuáles  son  los  signos  «blandos»  de  lesión  arterial?
Hematoma  estable  de  tamaño  pequeño  a  moderado.
Lesión  del  nervio  adyacente.
Shock  no  explicado  por  otras  lesiones.
Proximidad  de  la  herida  penetrante  a  una  estructura  vascular  importante.

4.  ¿Cuáles  son  los  síntomas  de  la  oclusión  arterial  aguda?
Las  seis  P:  dolor,  palidez,  déficit  de  pulso,  parestesia,  parálisis  y  poiquilotermia  (frío).

5.  ¿Qué  prueba  de  detección  inicial  se  utiliza  para  evaluar  una  extremidad  en  busca  de  lesiones  vasculares  ocultas?
¿lesión?
Cálculo  de  índices  de  presión  arterial  (APIs).

6.  ¿Cómo  se  llaman  las  API  para  la  extremidad  superior  y  la  extremidad  inferior?
Un  API  para  la  extremidad  superior  es  el  índice  braquial  de  la  muñeca  (WBI).
Un  API  para  la  extremidad  inferior  es  el  índice  tobillo  braquial  (ABI).

7.  ¿Cómo  se  miden  el  WBI  y  el  ABI  y  qué  se  considera  un  valor  normal?
Se  utiliza  un  Doppler  de  mano  y  un  manguito  de  presión  arterial  para  medir  la  presión  arterial  sistólica  (PAS)  en  
las  arterias  braquial,  radial,  cubital,  dorsal  del  pie  (DP)  y  tibial  posterior  (PT)  bilateralmente.
El  ABI  para  cada  pierna  es  el  DP  o  PT  más  alto  dividido  por  la  presión  braquial  más  alta.  El  WBI  para  cada  brazo  
es  la  presión  arterial  radial  o  cubital  más  alta  dividida  por  la  presión  braquial  más  alta.  Un  valor  de  1,0  es  normal.

8.  ¿Qué  valor  de  API  genera  preocupación  por  una  lesión  arterial  y  cuál  es  la  sensibilidad  y
especificidad?
  Un  valor  API  <0,9  tiene  una  sensibilidad  del  95  %  y  una  especificidad  del  97  %  para  lesiones  arteriales  importantes.
&  Un  API  >0,9  tiene  un  valor  predictivo  negativo  del  99  %.

158
Machine Translated by Google
CAPÍTULO  31  LESIONES  VASCULARES  DE  LAS  EXTREMIDADES  159

9.  Cuando  el  valor  API  es  <0,9  en  una  extremidad  lesionada,  ¿cuál  debe  ser  la  siguiente  prueba  diagnóstica?

Angiografía  por  TC  con  arteriografía  de  seguimiento  para  establecer  el  diagnóstico  y  planificar  la  intervención  
quirúrgica.

10.  ¿Qué  anormalidades  en  la  arteriografía  determinan  un  resultado  positivo  de  la  prueba?
&  Obstrucción  del  flujo
&  Extravasación  de  contraste
&  Relleno  venoso  temprano  o  fístula  arteriovenosa
&  Irregularidad  de  la  pared  o  defecto  de  relleno
&  Falso  aneurisma  (pseudoaneurisma)

11.  ¿Qué  estudio  se  debe  realizar  en  pacientes  con  lesiones  de  proximidad  o  signos  blandos?
(API  >0.9)?
Ecografía  dúplex  para  descartar  lesión  vascular  oculta.

12.  ¿Qué  lesiones  vasculares  ocultas  pueden  detectarse  mediante  ecografía  dúplex?
&  Colgajo  intimal.
y  pseudoaneurisma.
&  Fístula  arteriovenosa.
  Estrechamiento  de  vasos  focales.
&  La  observación  no  quirúrgica  de  estas  lesiones  es  segura  y  efectiva:  el  89%  de  ellas  no
requieren  cirugía.

13.  ¿Qué  es  un  pseudoaneurisma?
Es  una  ruptura  de  la  pared  arterial  que  conduce  a  un  hematoma  pulsátil  contenido  por  tejido  conjuntivo  
fibroso  (pero  no  por  las  tres  capas  de  la  pared  arterial).  (Véanse  las  figuras  31­1  y  31­2.)

Figura  31­1.  Angiografía  por  
sustracción  que  demuestra  un  
colgajo  de  íntima  con  estenosis  
de  la  arteria  femoral  superficial.
Machine Translated by Google
160  CAPÍTULO  31  LESIONES  VASCULARES  DE  LAS  EXTREMIDADES

Figura  31­2.  Ultrasonido  dúplex  de  la  arteria  femoral  común  que  muestra  el  saco  
del  pseudoaneurisma  y  el  cuello  asociado  entre  el  saco  del  pseudoaneurisma  y  la  arteria  
femoral  después  del  acceso  percutáneo  para  la  angiografía.

14.  ¿Qué  es  un  verdadero  aneurisma?
Dilatación  de  las  tres  capas  de  la  pared  del  vaso  (es  decir,  íntima,  media  y  adventicia).

15.  ¿Cuál  es  la  forma  más  efectiva  de  controlar  el  sangrado  arterial  en  un  lesionado?
¿extremidad?
Presión  digital  directa.

16.  ¿Qué  medios  de  controlar  la  lesión  vascular  deben  evitarse?  ¿Por  qué?
Debe  evitarse  un  torniquete  porque  se  ocluye  la  circulación  colateral  y  aumenta  la  isquemia  tisular.

También  se  debe  evitar  la  sujeción  ciega  porque  puede  causar  más  daño  al  vaso,  haciendo
la  reconstrucción  es  más  difícil  y  puede  lesionar  los  nervios  adyacentes.

17.  ¿Cómo  se  debe  preparar  y  envolver  en  el  quirófano  a  un  paciente  con  una  lesión  vascular  en  una  extremidad?

Toda  la  extremidad  afectada  debe  estar  en  el  campo  estéril.  El  tronco  arterial  principal  proximal  al  sitio  de  la  
lesión  (para  el  control  proximal)  y  una  porción  de  la  extremidad  inferior  que  permita  el  acceso  a  la  vena  safena  
deben  incluirse  en  el  campo  estéril.

18.  ¿Qué  más  se  debe  preparar  y  cubrir  para  las  lesiones  de  las  extremidades  proximales?
El  tórax  debe  estar  preparado  para  lesiones  proximales  de  la  extremidad  superior.  El  abdomen  debe  estar  
preparado  para  lesiones  proximales  de  la  extremidad  inferior.  (Puede  ser  necesario  acceder  al  tórax  o  al  
abdomen  para  obtener  un  control  vascular  proximal  seguro).

19.  ¿Cuáles  son  los  principios  operativos  relativos  a  la  reparación  de  lesiones  vasculares?
  Realizar  incisiones  longitudinales  sobre  los  vasos  a  explorar.
  La  disección  inicial  debe  realizarse  lejos  del  lugar  donde  se  sospecha  la  lesión  y  el  hematoma  adyacente.
Machine Translated by Google
CAPÍTULO  31  LESIONES  VASCULARES  DE  LAS  EXTREMIDADES  161

  Obtenga  control  proximal  y  distal  del  vaso  lesionado.
&  Desbridar  el  vaso  lesionado.
&  Realice  la  reparación  primaria  si  no  hay  tensión  (extienda  completamente  la  extremidad  para  asegurarse  de  que  no  haya  tensión).
reparar).

  Reparación  con  injerto  de  vena  de  interposición  autógena  si  la  longitud  (tensión)  es  inadecuada.

20.  ¿Cuál  es  el  mejor  conducto  para  lesiones  vasculares  de  las  extremidades  si  no  es  posible  la  reparación  primaria?  ¿Por  qué?

Vena  safena  o  cefálica  de  la  extremidad  no  lesionada  porque  las  tasas  de  permeabilidad  a  largo  plazo  son  mejores  y  
hay  menos  riesgo  de  infección.

21.  ¿Deben  repararse  las  lesiones  de  las  venas  principales  de  las  extremidades?
Sí.  La  reparación  de  una  vena  principal  mejora  el  éxito  de  una  reparación  arterial  concomitante  al  mejorar  el  flujo  de  
salida.  Esto  es  más  aplicable  a  las  lesiones  venosas  poplíteas.  La  trombosis  tardía  a  menudo  ocurre  después  de  la  
reparación  venosa,  pero  la  permeabilidad  inicial  ayuda  al  permitir  que  se  desarrolle  la  circulación  colateral.  Esto  también  
puede  reducir  la  incidencia  de  insuficiencia  venosa  posoperatoria.

22.  ¿Cuándo  deben  ligarse  las  venas  principales  lesionadas?
Las  venas  principales  deben  ligarse  en  lugar  de  repararse  cuando  el  paciente  está  hemodinámicamente  inestable  o  la  
reparación  es  demasiado  compleja.

23.  ¿Qué  complicaciones  pueden  desarrollarse  después  de  la  ligadura  de  las  principales  venas  de  las  extremidades?
Las  posibles  complicaciones  incluyen  un  aumento  rápido  de  la  presión  del  compartimento  muscular,  lo  que  lleva  a  
un  compromiso  del  flujo  venoso  o  arterial  y  al  síndrome  compartimental.  También  puede  ocurrir  estasis  venosa  
posoperatoria,  que  puede  atenuarse  con  compresión  neumática  intermitente  de  la  pantorrilla  y  elevación  de  la  pierna.

24.  ¿Qué  es  un  síndrome  compartimental?
Desarrollo  de  presiones  tisulares  patológicamente  elevadas  (que  impiden  la  perfusión)  dentro  de  envolturas  
no  expansivas  (dentro  de  los  compartimentos  fasciales)  del  brazo  o  la  pierna.

25.  ¿Cuál  es  la  causa  más  común  de  un  síndrome  compartimental?
Lesión  por  isquemia­reperfusión  cuando  la  isquemia  agota  las  reservas  de  energía  intracelular  y  luego  la  
reperfusión  conduce  a  radicales  de  oxígeno  tóxicos,  que  causan  inflamación  celular  y  acumulación  de  líquido  
intersticial.

26.  ¿Cuál  es  el  signo  más  temprano  de  síndrome  compartimental  después  de  la  reparación  vascular  de  un
¿extremidad?
Déficit  neurológico  en  la  distribución  del  nervio  peroneo  con  dorsiflexión  débil  y  entumecimiento  en  el  primer  espacio  
web  dorsal.

27.  ¿Hay  otros  signos  de  un  síndrome  compartimental  en  desarrollo  de  un
¿extremidad?
  Aumento  del  dolor  con  el  movimiento  pasivo  del  tobillo.
  Dolor  fuera  de  proporción  con  los  hallazgos  clínicos  (dolor  de  isquemia).
  Compartimentos  musculares  tensos  que  son  sensibles  a  la  palpación.
  Los  pulsos  distales  pueden  permanecer  intactos.
Machine Translated by Google
162  CAPÍTULO  31  LESIONES  VASCULARES  DE  LAS  EXTREMIDADES

28.  ¿Cómo  se  realiza  el  diagnóstico  objetivo  de  un  síndrome  compartimental?
Midiendo  las  presiones  del  compartimiento  con  una  aguja  percutánea  y  un  transductor  de  presión.
Los  criterios  para  el  síndrome  compartimental  son  los  
siguientes:  &  Cuando  la  presión  diastólica­presión  compartimental  es  ¼  20  mm  Hg  
o  &  Cuando  la  presión  arterial  media­presión  compartimental  es  ¼  30  mm  Hg.

29.  ¿Cuál  es  el  tratamiento  del  síndrome  compartimental  de  una  extremidad?
Fasciotomía  emergente  con  descompresión  de  los  cuatro  compartimentos  de  la  parte  inferior  de  la  
pierna  (anterior,  lateral,  posterior  superficial  y  posterior  profundo)  o  descompresión  de  los  compartimentos  
del  antebrazo.

30.  ¿Cuál  es  el  resultado  del  síndrome  compartimental  no  tratado?
La  pérdida  de  perfusión  promueve  la  eventual  mioneuronecrosis.

31.  ¿Cuáles  son  las  arterias  más  comúnmente  lesionadas  en  la  extremidad  superior?
Arteria  braquial 30%  (causado  con  mayor  
frecuencia  por  cateterismo  para  arteriografía)
Arteria  radial  o  cubital  20%
Arteria  axilar  10%
Arteria  subclavia  5%

32.  ¿Cuáles  son  las  arterias  lesionadas  con  mayor  frecuencia  en  la  extremidad  inferior?
Arteria  femoral  superficial  20  %  Arteria  poplítea  10  %  Arteria  femoral  
común  <5  %  Arterias  tibial  anterior,  posterior  y  peronea  <5  %  Arteria  
femoral  profunda  2  %

33.  ¿Puede  un  paciente  con  una  lesión  arterial  en  una  extremidad  tener  pulsos  distales  palpables?
Sí.  En  el  20%  de  las  lesiones  arteriales  comprobadas,  se  palpa  un  pulso  distal  (a  menudo  debido  a  la  
circulación  colateral).

34.  ¿Qué  lesiones  ortopédicas  suelen  tener  lesiones  vasculares  asociadas?
  Las  fracturas  supracondíleas  del  húmero  se  asocian  con  lesiones  de  la  arteria  braquial.
  Las  luxaciones  de  rodilla  se  asocian  con  lesiones  de  la  arteria  poplítea.
  Las  fracturas  de  fémur  pueden  asociarse  con  lesión  de  la  arteria  femoral  superficial.

35.  Para  una  extremidad  lesionada  con  fractura  concomitante  y  lesión  vascular,  ¿qué  reparación  debe  realizarse  
primero?
La  reparación  vascular  debe  realizarse  primero  para  restaurar  el  flujo  y  revertir  la  isquemia  tisular.

36.  Después  de  reducir  o  reparar  una  fractura  en  una  extremidad,  ¿qué  debe  hacer  siempre?
Evalúe  los  pulsos  distales  para  garantizar  un  flujo  de  entrada  vascular  adecuado  (especialmente  si  la  fijación  o  
cualquier  manipulación  sigue  a  una  reparación  vascular).
Machine Translated by Google
CAPÍTULO  31  LESIONES  VASCULARES  DE  LAS  EXTREMIDADES  163

37.  ¿Cuál  es  el  diagnóstico  probable  en  un  paciente  con  traumatismo  palmar  repetitivo  e  isquemia  o  necrosis  de  los  dedos?

Síndrome  del  martillo  hipotenar  (HHS).  Se  cree  que  el  mecanismo  es  un  trauma  palmar  repetitivo  en  pacientes  
con  fibrodisplasia  de  la  arteria  palmar  preexistente.  (El  arteriograma  muestra  oclusiones  arteriales  digitales  
con  oclusión  segmentaria  de  la  arteria  cubital  o  elongación  en  "sacacorchos").
(Véase  la  figura  31­3.)

Figura  31­3.  Angiografía  que  
muestra  un  colgajo  de  íntima  
en  la  arteria  femoral  superficial  
asociado  con  una  fractura  de  
fémur.

38.  ¿Qué  complicaciones  pueden  ocurrir  después  de  la  angiografía  cuando  se  utiliza  un  dispositivo  de  cierre  percutáneo  en  
la  arteria  femoral?
  Trombosis,  isquemia  o  ambas  cuando  la  sutura  de  cierre  afecta  a  la  pared  posterior
pared)  de  la  arteria.
  Pseudoaneurisma  infectado.
  Embolización  distal  cuando  se  utiliza  un  dispositivo  de  cierre  de  tapón  hemostático.

39.  ¿Cuáles  son  algunas  de  las  características  que  hacen  de  la  angiografía  por  tomografía  computarizada  una  
alternativa  eficaz  a  la  angiografía  convencional  en  la  evaluación  de  la  lesión  vascular  de  las  extremidades?

Mayor  sensibilidad  (95%  a  100%),  menor  costo,  menor  tiempo  de  diagnóstico  en  comparación  con  la  angiografía,  
efectivo  incluso  en  vasos  distales  de  brazos  y  piernas,  puramente  diagnóstico  (sin  posibilidad  de  intervención  como  
con  la  angiografía  convencional).
Machine Translated by Google
164  CAPÍTULO  31  LESIONES  VASCULARES  DE  LAS  EXTREMIDADES

PUNTOS  CLAVE:  SÍNDROME  COMPARTIMENTAL
1.  Las  presiones  tisulares  patológicamente  elevadas  en  los  compartimentos  fasciales  no  expansivos  evitan
perfusión  tisular.

2.  La  causa  más  común  es  la  lesión  por  isquemia­reperfusión  después  de  lesiones  traumáticas  en  las  extremidades.

3.  El  signo  clínico  más  temprano  es  entumecimiento  en  el  primer  espacio  web  dorsal  asociado  con
Compromiso  del  nervio  peroneo  profundo.  Otros  signos:  dolor  con  el  movimiento  articular  pasivo,  dolor  
desproporcionado  con  respecto  a  la  lesión  y  compartimentos  musculares  tensos  y  sensibles.

4.  Los  pulsos  distales  son  evidentes  hasta  una  etapa  avanzada  del  diagnóstico  y  no  deben  utilizarse  para  
descartar  un  síndrome  compartimental.

5.  El  manómetro  de  mano  se  utiliza  para  medir  los  compartimentos  musculares.

Presión  normal  10  mm  Hg;  presión  patológica  35  mm  Hg.

6.  El  tratamiento  es  fasciotomía  emergente.

SITIOS  WEB

www.east.org/tpg/lepene.pdf

www.surgery.ucsf.edu/eastbaytrauma/Protocols/ER%20protocol%20pages/extremity.htm

BIBLIOGRAFÍA

1.  Ferris  BL,  Taylor  LM  Jr,  Oyama  K  et  al.:  Síndrome  del  martillo  hipotenar:  etiología  propuesta.  J  Vasc  Cirugía
31:104­113,  2000.
2.  LeBus  GF,  Collinge  C:  anomalías  vasculares  evaluadas  con  angiografía  por  TC  en  fracturas  de  pilón  tibial  de  
alta  energía.  J  Orthop  Trauma  22(1):16­22,  2008.
3.  McCroskey  BL,  Moore  EE,  Pearce  WH  et  al.:  Lesiones  traumáticas  de  la  arteria  braquial.  Am  J  Surg  
156(6):553­555,  1988.
4.  Peng  PD,  España  DA,  Tataria  M  et  al.:  La  angiografía  por  TC  evalúa  eficazmente  el  traumatismo  vascular  de  las  extremidades.  Soy  quirúrgico
74(2):103­107,  2008.
5.  Rutherford  RB,  editor:  Cirugía  vascular,  5ª  ed.,  Filadelfia,  2000,  WB  Saunders.
6.  Schwartz  SI,  editor:  Principios  de  cirugía,  7ª  ed.,  Nueva  York,  1999,  McGraw­Hill.
Machine Translated by Google

LACERACIONES  FACIALES
CAPÍTULO  
32

Raffi  Gurunluoglu,  MD,  PhD

1.  ¿Qué  distingue  a  las  laceraciones  faciales  de  otras  laceraciones?
La  apariencia  es  claramente  de  importancia  primordial.  La  calidad  del  resultado  final  depende  del  cumplimiento  
estricto  de  los  principios  básicos  del  manejo  de  heridas  y  la  técnica  minuciosa.  La  irrigación  copiosa,  el  
desbridamiento  juicioso,  el  manejo  suave  del  tejido,  la  hemostasia  meticulosa  y  la  minimización  de  las  suturas  
combinadas  con  la  remoción  temprana  de  los  puntos  son  fundamentales  para  un  resultado  óptimo.
Se  deben  utilizar  suturas  finas  e  instrumentos  afilados;  La  eversión  del  margen  de  la  herida  con  cierre  en  capas,  la  
obliteración  del  espacio  muerto  y  la  falta  de  tensión  son  obligatorios.

2.  ¿Qué  factores  influyen  en  el  tratamiento  de  la  herida?

El  mecanismo,  la  ubicación  y  la  profundidad  de  la  lesión,  la  evaluación  clínica  de  la  contaminación  y  el  tiempo  
transcurrido  desde  la  lesión  dictan  el  tratamiento.  Las  lesiones  de  tejidos  blandos  son  de  mayor  preocupación  cuando  
se  localizan  en  la  cara  posterior  e  inferior  de  la  mejilla.  Dentro  de  esta  área  vital  se  encuentran  las  ramas  del  nervio  
facial,  la  glándula  y  el  conducto  parótidos  y  el  músculo  masetero.  Las  laceraciones  limpias,  las  heridas  muy  
contaminadas,  las  lesiones  por  aplastamiento  y  las  mordeduras  se  tratan  de  manera  muy  diferente.

3.  ¿Cómo  se  reparan  las  laceraciones  limpias?
Las  laceraciones  limpias  se  tratan  con  la  colocación  mínima  de  suturas  de  monofilamento  fino,  sin  tensión,  y  el  retiro  
temprano  de  las  suturas  (3  a  5  días).  El  uso  de  suturas  absorbibles  catgut  simple  en  niños  parece  ser  una  alternativa  
aceptable  a  la  sutura  no  absorbible  porque  el  resultado  cosmético  a  largo  plazo  parece  ser  al  menos  igual  de  bueno.

4.  ¿Cómo  se  reparan  las  laceraciones  sucias?
Las  heridas  muy  contaminadas  deben  permanecer  abiertas  después  de  la  irrigación  y  el  desbridamiento  para  que  se  
cierren  más  tarde.  Sin  embargo,  debido  a  consideraciones  cosméticas,  este  enfoque  es  inaceptable  en  la  cara.  Por  esta  
razón,  es  esencial  el  desbridamiento  meticuloso  del  tejido  desvitalizado  y  la  eliminación  de  todo  el  material  extraño.  La  
herida  debe  ser  cultivada  antes  de  la  irrigación  copiosa  y  debe  instituirse  un  antibiótico  de  amplio  espectro  de  manera  
profiláctica.  El  paciente  debe  ser  informado  de  la  posibilidad  de  una  infección  posterior  a  la  reparación.

5.  ¿Qué  factores  influyen  en  la  selección  de  la  sutura?

Cualquier  método  de  sutura  provoca  daño  tisular,  altera  las  defensas  del  huésped,  aumenta  la  proliferación  
de  cicatrices  e  invita  a  la  infección.  La  presencia  de  una  sola  sutura  de  seda  en  una  herida  reduce  el  umbral  
infeccioso  por  un  factor  de  10.000.  Por  lo  tanto,  se  debe  utilizar  una  sutura  de  monofilamento  fina,  lo  suficientemente  
fuerte  como  para  superar  la  tensión  de  la  herida  en  reposo.  Use  la  menor  cantidad  de  suturas  posible.
Las  heridas  con  poca  o  ninguna  retracción  pueden  cerrarse  solo  con  cinta  adhesiva.

6.  ¿Qué  heridas  son  adecuadas  para  el  cierre  con  adhesivos  tisulares  y  tiras  estériles?
El  N­butil­2­cianoacrilato  puede  ser  suficiente  para  el  cierre  cutáneo  de  laceraciones  de  baja  tensión  en  niños  (método  
preferido)  y  adultos.  Este  adhesivo  cierra  eficazmente  las  laceraciones  de  baja  tensión.  Este  método  es  rápido  y  
relativamente  indoloro.  Tiene  una  baja  tasa  de  complicaciones  y  produce  excelentes  resultados  cosméticos.  En  muchos  
casos,  si  la  orientación  inicial  de  la  herida  está  en  contra  de  las  líneas  de  Langer,  de  hecho  puede  ofrecer  una  ventaja  
sobre  la  sutura  manual  convencional.  Las  tiras  Steri  también  pueden  ser  suficientes

165
Machine Translated by Google
166  CAPÍTULO  32  LACERACIONES  FACIALES

para  el  cierre  de  laceraciones  simples  de  baja  tensión  tanto  en  adultos  como  en  niños  y  proporciona  resultados  
estéticos  similares.

7.  ¿Se  deben  afeitar  las  cejas  cuando  se  reparan  laceraciones  faciales?
No.  Proporcionan  un  punto  de  referencia  para  la  realineación  de  los  bordes  del  tejido  dañado  y  no  siempre  vuelven  
a  crecer.

8.  ¿Cómo  deben  repararse  las  lesiones  por  avulsión  por  aplastamiento  con  pérdida  de  piel  asociada?
Los  elementos  no  viables  deben  extirparse  quirúrgicamente  porque  predisponen  a  la  infección  y  provocan  una  
cicatrización  excesiva.  Si  hay  dudas  sobre  la  viabilidad,  la  herida  debe  irrigarse  completamente  y  dejarse  abierta  con  
apósitos  húmedos.  Se  puede  lograr  un  cierre  diferido  cuando  las  áreas  cuestionables  se  hayan  declarado.  A  menudo  es  
prudente  cerrar  el  tejido  facial  tal  como  está;  esta  técnica  a  menudo  produce  una  cicatriz  menos  prominente  que  el  
desbridamiento  y  cierre  en  línea  recta.

9.  ¿Cómo  se  deben  tratar  las  mordeduras?

Tanto  las  heridas  por  mordeduras  de  animales  como  las  de  humanos  están  muy  contaminadas  y  son  propensas  a  
infecciones.  La  herida  debe  dejarse  abierta  y  cerrarse  con  retraso.  Sin  embargo,  a  pesar  del  riesgo  de  infección,  la  
herida  se  puede  irrigar  y  se  puede  realizar  una  reparación  primaria  en  un  intento  de  lograr  el  mejor  resultado  
cosmético.  Está  indicada  la  profilaxis  antibiótica.  Si  la  herida  se  infecta,  se  deben  quitar  las  suturas  y  dejar  que  la  herida  
drene  y  cicatrice.  Se  debe  informar  al  paciente  que  puede  ser  necesaria  una  revisión  de  la  cicatriz.

10.  ¿Se  deben  utilizar  injertos  o  colgajos  de  piel  para  el  cierre  primario  de  una  herida?
Las  técnicas  complicadas  de  transferencia  de  tejido  no  tienen  cabida  en  el  tratamiento  agudo  de  las  heridas  
faciales.  El  cierre  debe  lograrse  de  la  manera  más  sencilla  posible  y  los  esfuerzos  reconstructivos  complejos  
deben  diferirse  hasta  que  la  cicatriz  haya  madurado  (meses).  Cuando  la  pérdida  de  tejido  impide  el  cierre,  puede  
ser  necesario  usar  un  injerto  de  piel  de  espesor  completo  (preferiblemente  como  un  injerto  de  hoja)  o  un  colgajo  
de  piel  para  cubrir.  Las  opciones  reconstructivas  deben  considerarse  de  forma  temprana  en  heridas  con  pérdida  de  
sustancia  (es  decir,  defectos  de  labio,  nariz  u  oreja).

11.  ¿Cuándo  están  indicados  los  antibióticos  en  el  tratamiento  de  laceraciones  faciales?

La  irrigación  copiosa,  el  desbridamiento  y  el  manejo  cuidadoso  de  los  tejidos  son  más  pertinentes  para  la  prevención  de  
infecciones  que  el  uso  de  antibióticos  en  heridas  limpias  y  limpias­contaminadas.

La  cobertura  antibiótica  está  indicada,  sin  embargo,  en  lesiones  por  avulsión  por  aplastamiento,  mordeduras  y  
lesiones  muy  contaminadas.

12.  ¿Qué  determina  la  calidad  de  la  cicatriz?
La  ubicación  de  la  herida,  la  edad  del  paciente  y  el  tipo  y  calidad  de  la  piel  la  determinan.  Los  determinantes  menores  
son  el  tipo  y  la  cantidad  de  material  de  sutura  y  el  cuidado  de  la  herida.  El  aspecto  final  depende  poco  del  método  de  
sutura.  La  contusión,  la  infección,  el  cuerpo  extraño  retenido,  la  orientación  incorrecta  de  la  laceración,  el  patrón  de  la  
laceración  (p.  ej.,  en  forma  de  "U"),  la  tensión  y  el  biselado  de  los  bordes  predicen  un  mal  resultado.  Las  diferencias  entre  
los  materiales  de  sutura  son  insignificantes;  sin  embargo,  los  factores  técnicos  de  la  colocación  de  la  sutura  para  producir  
la  eversión  de  la  herida  y  el  tiempo  de  remoción  afectan  el  resultado  final.

13.  ¿Cuándo  se  deben  revisar  las  cicatrices?

Una  cicatriz  generalmente  tiene  su  peor  apariencia  entre  2  semanas  y  2  meses  después  de  la  sutura.  La  revisión  de  la  
cicatriz  debe  esperar  la  maduración  completa,  que  puede  tardar  de  4  a  24  meses.  Una  buena  regla  general  es  no  realizar  
revisiones  durante  al  menos  6  a  12  meses  después  de  la  reparación  inicial.  La  maduración  de  la  herida  puede  evaluarse  
por  su  grado  de  molestia,  eritema  e  induración.
Machine Translated by Google
CAPÍTULO  32  LACERACIONES  FACIALES  167

14.  ¿Qué  técnicas  existen  para  la  revisión  de  cicatrices?
Las  cicatrices  circulares  o  en  forma  de  U  o  aquellas  que  cruzan  los  pliegues  naturales  de  la  piel  por  lo  general  se  
benefician  de  algún  tipo  de  revisión  que  involucre  un  cambio  de  dirección,  alargamiento  o  escalonamiento.  Las  técnicas  
quirúrgicas  incluyen  plastia  en  Z  y  plastia  en  W.

CONTROVERSIAS

15.  ¿Qué  controversias  existen  respecto  al  cuidado  y  reparación  de  las  laceraciones  faciales?
Hay  poca  controversia  sobre  el  cuidado  y  la  reparación  de  las  laceraciones  faciales.  La  atención  a  los  principios  
básicos  del  cuidado  de  heridas  suele  producir  una  cicatriz  satisfactoria.  Debido  a  las  consideraciones  cosméticas  en  
los  traumatismos  faciales,  en  algunos  casos  la  reparación  primaria  se  lleva  a  cabo  por  el  bien  de  la  apariencia  a  pesar  del  
riesgo  de  infección  que  se  consideraría  inaceptable  en  otras  áreas  del  cuerpo.  Un  buen  ejemplo  sería  una  herida  por  
mordedura  humana  o  animal  que  puede  repararse  principalmente  después  de  una  abundante  irrigación  y  desbridamiento  de  
los  tejidos  no  viables.

PUNTOS  CLAVE:  LACERACIONES  FACIALES

1.  La  apariencia  y  la  función  son  de  suma  importancia.

2.  Las  laceraciones  limpias  se  tratan  con  la  colocación  mínima  de  suturas  de  monofilamento  fino,  sin  tensión,  y  la  retirada  
temprana  de  las  suturas  (3  a  5  días).

3.  Las  tiras  de  N­butil­2­cianoacrilato  (Dermabond)  o  steri  se  utilizan  preferentemente  para  reparar  lesiones  pediátricas.
laceraciones  faciales,  particularmente  con  baja  tensión.

4.  Las  heridas  muy  contaminadas  se  irrigan,  desbridan  y  reparan  con  la  administración  de
antibióticos

BIBLIOGRAFÍA

1.  Hochberg  J,  Ardenghy  M,  Toledo  S  et  al .:  Lesiones  de  tejidos  blandos  en  la  cara  y  el  cuello:  evaluación  y  reparación  tempranas.
World  J  Surg  25:1023­1027,  2001.

2.  Holger  JS,  Wandersee  SC,  Hale  DB:  Resultados  estéticos  de  laceraciones  faciales  reparadas  con  suturas  adhesivas,  absorbibles  
y  no  absorbibles.  Am  J  Emerg  Med  22:254­257,  2004.

3.  Karounis  H,  Gouin  S,  Eisman  H  et  al.:  Un  ensayo  aleatorizado  y  controlado  que  compara  los  resultados  estéticos  a  largo  plazo  de  las  
laceraciones  pediátricas  traumáticas  reparadas  con  suturas  absorbibles  de  nailon  versus  no  absorbibles.
Acad  Emerg  Med  11:730­735,  2004.

4.  Leach  J:  manejo  adecuado  de  los  tejidos  blandos  en  la  fase  aguda.  Cirugía  plástica  facial  17:227­238,  2001.

5.  Mattick  A,  Clegg  G,  Beattie  T  et  al.:  Un  ensayo  aleatorizado  y  controlado  que  compara  un  adhesivo  tisular
(2­octilcianoacrilato)  con  tiras  adhesivas  (steristrips)  para  reparación  pediátrica.  Emerg  Med  J  19:405­407,  2002.

6.  Mcheik  JN,  Vergnes  P,  Bondonny  JM:  Tratamiento  de  lesiones  faciales  por  mordedura  de  perro  en  niños:  un  estudio  retrospectivo.
J  Pediatr  Surg  35:580­583,  2000.

7.  Mitchell  RB,  Nanez  G,  Wagner  JD  et  al .:  Mordeduras  de  perro  en  el  cuero  cabelludo,  la  cara  y  el  cuello  en  niños.  Laringoscopio
113:492­495,  2003.

8.  Moscati  RM,  Mayrose  J,  Reardon  RF  et  al.:  Una  comparación  multicéntrica  de  agua  del  grifo  versus  solución  salina  estéril  para
irrigación  de  heridas.  Acad  Emerg  Med  14:404­409,  2007.

9.  Schalamon  J,  Ainoedhofer  H,  Singer  G  et  al.:  Análisis  de  mordeduras  de  perro  en  niños  menores  de  17  años.
Pediatría  117:374­379,  2006.

10.  Singer  AJ,  Quinn  JV,  Clark  RE  et  al.:  Cierre  de  laceraciones  e  incisiones  con  octilcianoacrilato:  estudio  multicéntrico
ensayo  controlado  aleatorizado.  Cirugía  131:270­276,  2002.
Machine Translated by Google
168  CAPÍTULO  32  LACERACIONES  FACIALES

11.  Singer  AJ,  Quinn  JV,  Thode  HC  et  al .:  Determinantes  de  un  resultado  deficiente  después  de  la  laceración  y  la  reparación  de  la  incisión  quirúrgica.
Plast  Reconstr  Surg  110:429­435,  2002.

12.  Stefanopoulos  PF,  Tarantzopoulou  AD:  Heridas  faciales  por  mordedura:  actualización  de  la  gestión.  Int  J  Oral  Maxillofac  Surg
34:464­472,  2005.

13.  Valente  JH,  Forti  RJ,  Freundlich  LF  et  al.:  Irrigación  de  heridas  en  niños:  ¿agua  salina  o  del  grifo?  Ann  Emerg  Med
41:609­616,  2003.

14.  Zempsky  WT,  Parrotti  D,  Grem  C  et  al.:  Comparación  aleatoria  controlada  de  los  resultados  cosméticos  de  laceraciones  faciales  simples  
cerradas  con  cierres  de  piel  steri  strip  o  adhesivo  tisular  dermabond.  Pediatr  Emerg  Care  20:519­524,  2004.
Machine Translated by Google

CUIDADOS  BÁSICOS  DE  LESIONES  EN  LAS  MANOS
CAPÍTULO  
33

Kagan  Ozer,  MD,  y  Laura  DiMatteo,  MD

1.  ¿Cómo  se  entablillan  las  fracturas  y  lesiones  de  la  mano?
La  inmovilización  específica  de  la  lesión  debe  incluir  el  entablillado  de  la  articulación  por  encima  y  por  debajo  de  la  lesión.
Por  ejemplo,  una  fractura  del  hueso  metacarpiano  debe  inmovilizarse  en  la  muñeca  y  la  articulación  metacarpiano­
falángica.
Férula  palmar  para  muñeca:  férula  de  descanso  ideal  para  la  mano  después  de  quemaduras  y  lesiones  de  tejidos  blandos.

Férula  en  espiga  para  pulgar:  Ideal  para  lesiones  localizadas  en  el  lado  radial  de  la  mano  incluyendo  tendinitis
del  primer  compartimento  dorsal  (de  Quervain).
Férula  de  canalón  cubital:  comúnmente  utilizada  para  las  fracturas  del  cuarto  y  quinto  metacarpianos
(fractura  de  boxeador).
Férula  de  pila:  se  utiliza  para  lesiones  en  el  dedo  en  martillo  y  traumatismo  en  el  lecho  ungueal.  Permite  la  
flexión  de  la  articulación  interfalángica  proximal  (PIP)  evitando  el  desarrollo  de  cualquier  contractura.

2.  ¿Cuáles  son  los  signos  de  la  tenosinovitis  de  los  flexores?
Signos  de  Kanaval:  postura  flexionada  del  dedo,  tumefacción  circunferencial,  sensibilidad  a  lo  largo  de  la  vaina  del  
tendón  flexor  y  dolor  con  la  extensión  pasiva  del  dedo.

3.  ¿Cómo  se  trata  la  tenosinotivitis  flexora?
Descompresión  urgente  de  la  vaina  tendinosa  en  quirófano  (OR),  cultivo  de  antibióticos  específicos  y  cuidado  de  
heridas.

4.  ¿Cómo  y  dónde  se  deben  explorar  las  lesiones  de  la  mano?
Las  heridas  de  la  mano  deben  explorarse  bajo  control  de  torniquete  con  analgesia  adecuada  utilizando  instrumentos  
delicados  en  una  sala  de  cirugía  bien  iluminada.  La  ampliación  visual  suele  ser  obligatoria.

5.  ¿Cómo  se  logra  la  hemostasia  de  emergencia  de  las  manos  lesionadas?
En  el  entorno  agudo  (fuera  del  quirófano),  no  se  debe  aplicar  torniquete;  y  no  debe  haber  abrazaderas  ciegas  ni  atado  con  
suturas  de  ninguna  estructura.  La  hemostasia  puede  lograrse  elevando  la  extremidad  y  con  compresión  directa  de  la  herida  
durante  10  minutos.  Este  enfoque  evita  lesiones  en  estructuras  subyacentes  delicadas  que  son  difíciles  de  ver.

6.  ¿Cómo  se  tratan  las  lesiones  en  las  yemas  de  los  dedos?
Si  se  rompe  <1  cm  de  pulpa,  la  herida  sanará  espontáneamente  con  limpieza  diaria  y  vendaje  con  gasa  húmeda  no  
adherente.  Los  defectos  más  grandes  pueden  requerir  un  injerto  de  piel,  que  a  menudo  se  puede  proporcionar  eliminando  
la  grasa  de  la  pieza  amputada.  La  exposición  ósea  requiere  cobertura  con  colgajo  si  se  quiere  mantener  la  longitud  digital.  
Los  nervios  digitales  no  pueden  repararse  distalmente  a  la  articulación  interfalángica  distal  (DIP).

7.  ¿Cuál  es  el  sistema  de  clasificación  de  las  amputaciones  de  las  yemas  de  los  dedos?
La  clasificación  de  las  amputaciones  de  las  yemas  de  los  dedos  se  basa  en  la  cantidad  de  piel  sensible  volar  restante.
Aunque  la  amputación  con  un  ángulo  favorable  comúnmente  elimina  parte  de  la  uña  y  el  hueso,  la  piel  volar  está  disponible  
para  una  fácil  cobertura.  Este  tipo  de  amputación  es  ''favorable''  para  el  tratamiento  por

169
Machine Translated by Google
170  CAPÍTULO  33  CUIDADOS  BÁSICOS  DE  LAS  LESIONES  EN  LAS  MANOS

solo  apósitos,  permitiendo  la  reparación  de  heridas  por  contracción  y  epitelización.  El  ángulo  de  amputación  
con  angulación  volar  es  «desfavorable»  para  el  tratamiento  conservador  y  suele  requerir  un  procedimiento  
reconstructivo  (v .  fig.  33­1).

Figura  33­1.  Amputaciones  de  la  punta  de  los  dedos.  Imagen  de  Ditmars  DM  Jr:  Lesiones  en  la  punta  de  los  dedos  y  en  el  
lecho  ungueal.  En  Kasdan  ML,  editor:  Lesiones  y  enfermedades  laborales  de  las  manos  y  las  extremidades  superiores.  
Filadelfia,  1991,  Hanley  &  Belfus,  1991,  con  autorización.)

8.  ¿Cómo  se  reparan  las  lesiones  del  lecho  ungueal?
La  reparación  de  la  ruptura  de  la  matriz  germinal  debe  aproximarse  meticulosamente  con  aumento;  y  
el  lecho  ungueal  entablillado,  preferiblemente  con  la  parte  avulsionada.  Los  hematomas  subungueales  deben  
evacuarse  con  un  clip  de  punta  caliente  o  un  cauterio  eléctrico  a  batería.  La  reparación  de  la  rotura  del  
pliegue  eponiquial  estéril  debe  mantenerse  durante  3  semanas  con  gasa  Xeroform  o  con  el  clavo  original.  A  
menudo,  la  interrupción  del  lecho  ungueal  no  se  puede  diagnosticar  sin  la  extracción  de  la  uña.

9.  ¿Cuál  es  el  manejo  inicial  de  la  laceración  del  tendón  flexor?
La  laceración  del  tendón  flexor  no  es  una  emergencia  y  la  reparación  no  debe  realizarse  en  el  
departamento  de  emergencias  (ED).  Si  no  se  dispone  de  un  cirujano  de  la  mano,  se  debe  irrigar  y  suturar  
abundantemente  la  herida  e  instituir  antibióticos  profilácticos.  Esta  lesión  se  puede  reparar  principalmente  
en  3  semanas.

10.  ¿Cuál  es  el  manejo  adecuado  de  una  fractura  abierta?
Las  fracturas  abiertas  deben  limpiarse  y  vendarse  en  el  servicio  de  urgencias,  pero  no  sondarse  ni  
cultivarse.  Se  debe  administrar  una  cefalosporina  de  primera  generación,  como  Ancef;  se  debe  actualizar  
la  inmunización  contra  el  tétanos;  un  vendaje  empapado  en  solución  salina  aplicado  sobre  la  herida;  y  la  
mano  debe  ser  entablillada  en  la  posición  de  función  con  un  vendaje  voluminoso.  La  herida  y  la  fractura  
deben  llevarse  con  urgencia  al  quirófano  (OR)  para  irrigación  formal,  desbridamiento  y  cierre  de  la  herida  si  
es  posible.  Las  heridas  que  cumplan  los  criterios  de  una  fractura  abierta  de  Gustilo  y  Anderson  también  
deben  recibir  penicilina  y  un  aminoglucósido,  como  la  gentamicina.

11.  ¿Cuál  es  el  tratamiento  adecuado  para  la  infección  de  manos?
La  extremidad  debe  inmovilizarse  y  elevarse,  y  deben  administrarse  antibióticos  por  vía  parenteral.
El  paciente  debe  ser  derivado  inmediatamente  para  posible  drenaje  quirúrgico.

12.  ¿Cuál  es  el  manejo  adecuado  de  las  mordeduras  humanas  y  animales?
Después  de  la  limpieza  de  la  herida,  se  debe  tomar  una  radiografía.  La  herida  debe  dejarse  abierta;  nunca  
cerrado  Deben  iniciarse  antibióticos  y  la  herida  debe  volver  a  revisarse  a  las  24  y  48  horas.  Si  hay  
evidencia  de  infección,  se  deben  instituir  antibióticos  parenterales.
Machine Translated by Google
CAPÍTULO  33  ATENCIÓN  BÁSICA  DE  LAS  LESIONES  DE  LA  MANO  171

y  referido  para  posible  drenaje  quirúrgico.  En  las  mordeduras  humanas,  el  microorganismo  más  común  es  
Eikenella  Corrodens  y  se  trata  mejor  con  penicilina  resistente  a  la  penicilinasa,  como  amoxicilina/clavulanato.  
En  las  mordeduras  de  animales,  Pasteurella  Multocida  es  el  organismo  más  común  y  debe  tratarse  con  
amoxicilina/clavulanato.  El  llamado  mordisco  de  lucha  ocurre  sobre  la  articulación  metacarpofalángica  (MCF)  o  
la  articulación  interfalángica  proximal  cuando  un  puño  cerrado  se  empala  en  los  dientes  frontales  de  un  
adversario.  Esto  a  menudo  inocula  la  articulación  MCP  con  estreptococos  anaerobios.  Cuando  se  diagnostica  
una  infección,  se  debe  realizar  una  artrotomía  y  un  lavado  inmediatos.

13.  ¿Cómo  se  tratan  las  lesiones  por  inyección?
A  pesar  de  su  apariencia  inocua,  las  lesiones  por  inyección  pueden  causar  una  destrucción  profunda  de  las  
estructuras  de  la  mano.  Cualquier  lesión  de  este  tipo  requiere  hospitalización  inmediata  con  descompresión,  
drenaje  y  desbridamiento  rápidos  y  extensos.

14.  ¿Cuáles  son  las  causas  más  prevenibles  de  deformidad  en  las  lesiones  de  la  mano?
El  edema  y  la  infección  provocan  un  aumento  de  la  cicatrización  y  una  función  restringida.  La  
inmovilización  prolongada  en  una  mala  posición  también  altera  la  función,  al  igual  que  el  cierre  tardío  de  la  
piel.  Si  no  se  obtiene  una  radiografía,  se  pierde  el  diagnóstico  y  se  retrasa  el  reconocimiento  de  una  lesión.

15.  ¿Cuál  es  el  tratamiento  adecuado  en  el  departamento  de  emergencias  del  paciente  con
parte  amputada?
Primero,  el  paciente  debe  ser  tratado  de  acuerdo  con  el  protocolo  de  soporte  vital  avanzado  para  traumatismos  
(ATLS).  Una  vez  que  se  estabiliza  la  condición  del  paciente,  se  debe  administrar  la  inmunización  antitetánica  y  
antibióticos  profilácticos.  Deben  administrarse  antibióticos  de  amplio  espectro  si  la  herida  está  muy  contaminada  
o  si  el  paciente  es  diabético  o  inmunocomprometido.  Para  una  evaluación  más  detallada  del  paciente  con  una  
parte  potencialmente  reimplantable,  se  deben  ordenar  las  siguientes  pruebas:  radiografías  de  la  extremidad  
lesionada  y  de  la  parte  amputada,  contenido  de  hemoglobina/hematocrito,  tipo  de  sangre  y  detección,  y  otras  
pruebas  según  se  indique  (glucosa  en  sangre  para  diabéticos,  etc.).  Mientras  estas  pruebas  están  pendientes,  
se  debe  contactar  un  centro  de  reimplantación  para  la  transferencia.
Debido  a  que  la  isquemia  prolongada  de  la  parte  amputada  resulta  en  la  pérdida  completa  de  la  parte  y  la  
función,  una  vez  acordada,  la  transferencia  debe  instituirse  lo  antes  posible.  Para  un  dedo,  la  reimplantación  
suele  ser  posible  hasta  12  horas.  Sin  embargo,  las  partes  que  contienen  músculo  (antebrazo,  brazo)  no  se  
pueden  reimplantar  después  de  6  horas.

16.  ¿Cómo  se  debe  transportar  la  parte  amputada  al  centro  de  reimplantación?
Primero,  la  parte  debe  colocarse  sobre  una  gasa  húmeda  con  solución  salina  normal.  Luego  se  debe  colocar  
en  una  bolsa  de  plástico  sellada  que  luego  se  debe  colocar  en  un  recipiente  con  solución  salina  helada  
manteniendo  la  temperatura  de  4  C.  Se  debe  evitar  el  contacto  directo  con  el  hielo  para  evitar  la  congelación.  
No  se  deben  utilizar  soluciones  hipotónicas  e  hipertónicas  para  evitar  el  daño  osmótico  de  la  pieza.

17.  ¿Qué  es  el  síndrome  del  túnel  carpiano  agudo?
Compresión  aguda  del  nervio  mediano  en  el  túnel  carpiano.  Asociado  con  traumatismo  en  la  mano,  la  
muñeca  o  el  antebrazo;  por  ejemplo,  una  fractura  de  radio  distal.  Se  diagnostica  clínicamente  y  se  presenta  
con  empeoramiento  del  dolor  en  la  muñeca  y  cambios  sensoriales  en  la  distribución  del  nervio  mediano.  Los  
cambios  sensoriales  típicamente  incluyen  parestesias  en  el  pulgar  volar,  índice,  medio  y  la  mitad  del  dedo  
anular.  La  mejor  prueba  es  la  discriminación  de  2  puntos  con  menos  de  5  mm  en  las  puntas  de  los  dedos  en  
manos  normales.

18.  ¿Cómo  se  trata  el  túnel  carpiano  agudo?
Descompresión  quirúrgica  emergente  del  nervio  mediano  mediante  una  incisión  en  el  ligamento  carpiano  
transverso,  que  es  el  techo  fascial  del  túnel  carpiano.
Machine Translated by Google
172  CAPÍTULO  33  CUIDADOS  BÁSICOS  DE  LAS  LESIONES  DE  LA  MANO

PUNTOS  CLAVE:  AGUDO  VERSUS  CRÓNICO  CARPIANO
SÍNDROME  DEL  TÚNEL
1.  Ambas  son  neuropatías  por  compresión.

2.  La  forma  crónica  tiene  predilección  por  las  mujeres  después  de  los  40,  se  puede  observar  el  síndrome  del  túnel  carpiano  agudo

a  cualquier  edad.

3.  La  forma  aguda  está  relacionada  con  un  trauma,  mientras  que  la  forma  crónica  se  ve  más  comúnmente  en  casos  repetitivos.
Trabajadores  manuales.

4.  El  dolor  es  el  síntoma  predominante  en  el  síndrome  del  túnel  carpiano  agudo  seguido  de  entumecimiento  y  hormigueo.  En  forma  crónica,  los  

síntomas  dominantes  son  entumecimiento,  hormigueo  y  despertares  nocturnos.

SITIO  WEB

www.ninds.nih.gov  

Buscar:  túnel  carpiano

BIBLIOGRAFÍA

1.  Dunn  R,  Watson  S:  Sutura  versus  manejo  conservador  de  laceraciones  en  las  manos.  Las  laceraciones  de  la  mano  deben  explorarse  
antes  del  tratamiento  conservador.  Br  Med  J  325:299,  2002.

2.  Goldner  RD,  Urbaniak  JR:  Replantación.  En  Green  DP,  editor:  Cirugía  de  la  mano  operativa  de  Green,  5.ª  ed.,
Filadelfia,  2005,  Elsevier.

3.  Gustilo  RB,  Anderson  JT:  Prevención  de  la  infección  en  el  tratamiento  de  mil  veinticinco  abiertos
fracturas  de  huesos  largos:  análisis  retrospectivo  y  prospectivo.  J  Bone  Joint  Surg  Am  58(4):453­458,  1976.

4.  Hansen  TB,  Carstensen  O:  Lesiones  en  las  manos  en  accidentes  agrícolas.  J  Hand  Surg  [Br]  24B:190­192,  1999.

5.  Irvine  AJ:  sutura  versus  manejo  conservador  de  laceraciones  en  las  manos.  Las  incisiones  no  son  laceraciones.  Br  Med  J
325(7359):299,  2002.

6.  Jebson  PJL,  Louis  DS,  editores:  Infecciones  de  manos.  Mano  Clin  14(4):511­711,  1998.

7.  Lee  SJ,  Montgomery  K:  lesiones  atléticas  en  la  mano.  Orthop  Clin  North  Am  33:547­554,  2002.

8.  Mack  GR,  McPherson  SA,  Lutz  RB:  neuropatía  mediana  aguda  después  de  un  traumatismo  en  la  muñeca.  El  papel  del  carpo  emergente
liberación  del  túnel.  Clin  Orthop  Relat  Res  300:141­146,  1994.

9.  Martin  C,  Gonzalez  Del  Pino  J:  Controversias  en  el  tratamiento  de  las  amputaciones  de  punta  de  los  dedos:  conservador  versus
reconstrucción  quirúrgica.  Clin  Orthop  Relat  Res  353,  63­73,  1998.

10.  Riaz  M,  Hill  C,  Khan  K  et  al.:  Resultado  a  largo  plazo  de  la  movilización  activa  temprana  después  de  la  reparación  del  tendón  flexor  
en  la  zona  2.  J  Hand  Surg  24B:157­160,  1999.

11.  Taras  JS,  Lamb  MJ:  Tratamiento  de  las  lesiones  del  tendón  flexor:  perspectiva  de  los  cirujanos.  J  Hand  Ther  12:141­148,  1999.

12.  VanderMolen  AB,  Matloub  HS,  Dzwierzynski  W  et  al.:  El  sistema  de  puntuación  de  la  gravedad  de  las  lesiones  en  las  manos  y  el
Casos  de  compensación  en  Wisconsin,  EE.UU.  J  Hand  Surg  24B:184­186,  1999.
Machine Translated by Google

QUEMADURAS
CAPÍTULO  
34

Janeen  R.  Jordan,  MD,  y  Walter  L.  Biffl,  MD,  FACS

1.  ¿Dónde  ocurren  las  lesiones  por  quemaduras?
El  ochenta  por  ciento  de  las  lesiones  relacionadas  con  quemaduras  ocurren  en  el  hogar,  principalmente  en  viviendas  multifamiliares  de  

bajos  ingresos.

2.  ¿Quién  tiene  riesgo  de  sufrir  quemaduras?
La  incidencia  de  lesiones  por  quemaduras  y  muertes  en  los  Estados  Unidos  es  sustancialmente  mayor  que  la  del  resto  
del  mundo  industrializado.  La  proporción  de  hombres  a  mujeres  para  las  lesiones  por  quemaduras  es  de  aproximadamente  2:1.
Las  lesiones  por  quemaduras  relacionadas  con  el  trabajo  representan  la  mayor  parte  de  la  disparidad  entre  hombres  y  
mujeres,  y  los  accidentes  en  las  industrias  petroquímica  y  de  transporte  son  responsables  de  una  proporción  significativa.  El  
abuso  de  alcohol  y  la  actividad  de  drogas  ilícitas  también  aumentan  el  riesgo  de  lesiones  por  quemaduras  y  muerte.  La  tasa  
de  mortalidad  de  niños  menores  de  5  años  y  adultos  de  65  años  o  más  es  de  5  a  6  veces  mayor  que  la  del  resto  de  la  población.

3.  ¿Qué  factores  influyen  más  profundamente  en  los  resultados  de  las  quemaduras?
El  riesgo  global  de  mortalidad  por  quemaduras  es  del  7,6%.  Los  centros  de  quemados  pediátricos  registran  mortalidades  
entre  el  2%  y  el  3%,  mientras  que  la  mortalidad  en  mayores  de  50  años  es  más  de  3  veces  superior  a  la  media  nacional,  y  por  
encima  de  los  70  años  la  mortalidad  supera  el  33%.  Se  han  identificado  tres  factores  de  riesgo  principales,  con  un  peso  
esencialmente  igual  en  la  predicción  de  la  mortalidad:  tamaño  de  la  quemadura  de  >40%  del  área  de  superficie  corporal  total  
(TBSA),  edad  del  paciente  >60  años  y  presencia  de  lesión  por  inhalación  en  los  pulmones.  La  probabilidad  acumulada  de  
muerte  cuando  uno  o  más  de  estos  factores  se  presenta  en  el  cuadro  34­1.

TABLA  34­1.  TASAS  DE  MOR  TA  LIDAD  ASOCIADAS  CON  EL  JURADO  DE  LESIONES  POR  QUEMADURAS

Número  de  factores  de  riesgo  presentes Mortalidad

0 0,3%

1 3%

2 33%

3 90%

4.  ¿Alguna  otra  variable  influye  en  la  supervivencia?
El  abuso  de  etanol  y  el  abuso  de  drogas  ilícitas  se  pueden  agregar  a  los  tres  factores  enumerados  anteriormente,  lo  
que  aumenta  el  riesgo  de  muerte  en  un  factor  de  2  a  4  veces.

5.  Como  modo  único  de  lesión,  ¿por  qué  las  quemaduras  representan  un  desafío  y  una  amenaza  tan  devastadores  para  las  víctimas?

&  Daño  extenso  a  la  piel  (considerada  el  órgano  individual  más  grande  del  cuerpo  y
consume  casi  el  20%  del  gasto  cardíaco  [CO])  prepara  el  escenario  para  la  invasión  bacteriana.

173
Machine Translated by Google
174  CAPÍTULO  34  QUEMADURAS

  Dado  que  los  seres  humanos  son  casi  un  70  %  de  agua,  encerrados  por  un  complejo  sistema  tegumentario,  
se  producen  trastornos  graves  en  la  homeostasis  de  los  fluidos  cuando  se  destruye  la  envoltura  de  la  piel.
  La  desnaturalización  inducida  por  el  calor  de  las  proteínas  tegumentarias  entra  en  la  circulación .  sistémico
la  infección  o  la  sepsis  sigue  siendo  el  desencadenante  dominante  de  la  insuficiencia  orgánica  y  la  muerte;  esto  apunta  a  
una  disfunción  o  falla  inmunitaria  relacionada  con  la  lesión  por  quemadura.

6.  ¿Qué  sucede  localmente  en  las  lesiones  por  quemadura?
El  sitio  de  la  lesión  se  puede  dividir  en  tres  zonas  mediante  microscopía  de  luz  estándar:  una  zona  interna  de  necrosis,  
una  zona  intermedia  de  estasis  y  una  zona  externa  de  hiperemia.  En  la  zona  de  necrosis,  todas  las  proteínas  se  desnaturalizan;  
se  destruyen  todas  las  estructuras  y  funciones  microvasculares  y  macrovasculares.
Rodeando  esta  zona  central  hay  una  zona  de  estasis.  Aquí,  la  morfología  celular  está  intacta  pero  las  células  están  hinchadas  
con  cambios  microestructurales  con  extravasación  de  leucocitos  y  glóbulos  rojos  en  el  espacio  intersticial,  aumento  del  líquido  
intersticial  y  estasis  capilar.  Luego,  una  tercera  zona  de  hiperemia  pasa  suavemente  a  los  tejidos  normales  adyacentes  donde  no  
se  observan  anomalías.

7.  ¿Qué  cambios  ocurren  sistémicamente?
Los  eventos  sistémicos  se  vuelven  clínicamente  significativos  más  allá  de  un  tamaño  de  lesión  del  10%  TBSA.  Ocurren  dos  
anomalías  importantes:  (1)  una  tendencia  a  la  retención  de  líquidos  con  edema  generalizado,  causada  por  un  aumento  de  la  
permeabilidad  microvascular  sistémica  de  inicio  rápido  (minutos  a  horas)  y  (2)  una  disminución  definitiva  y  reproducible  del  GC  
que  se  resuelve  gradualmente  en  12  a  36  horas  para  evolucionar  a  un  hiperdinamismo  cardiovascular  subsiguiente  a  las  36  horas  
después  de  la  lesión.  En  resumen,  la  bomba  está  fallando  y  la  microvasculatura  tiene  fugas.

8.  ¿Cómo  se  puede  manejar  de  manera  racional  a  las  víctimas  de  quemaduras  desde  el  momento  de  la  lesión?
Se  pueden  identificar  cinco  fases  de  atención:  1.  Primeros  
auxilios  para  quemaduras.

2.  Atención  prehospitalaria.
3.  Departamento  de  emergencias  (ED).
4.  Transporte  a  la  unidad  de  quemado.

5.  Estabilización  en  la  unidad  de  quemados  o  en  la  habitación  del  paciente.

9.  ¿Qué  pueden  hacer  los  socorristas  cuando  presencian  una  lesión  por  quemadura?
Primero,  no  hagas  daño.  No  se  debe  aplicar  hielo,  mantequilla,  hielo  seco  o  cualquier  otra  sustancia  a  la  herida  después  de  
extinguir  el  fuego.  Si  la  quemadura  es  menor  (<10%  TBSA),  es  beneficioso  dejar  correr  agua  tibia  del  grifo  sobre  la  quemadura  con  
una  ducha  de  mano  durante  20  minutos.  La  aplicación  de  toallas  húmedas  no  parece  brindar  ningún  beneficio  y  puede  provocar  
hipotermia.  Si  se  encuentra  varado  en  un  área  remota,  fomente  la  ingesta  de  líquidos  por  vía  oral  y  cubra  la  herida  con  toallas  
limpias.  La  aspirina  o  el  ibuprofeno  pueden  beneficiar  al  paciente  ya  la  herida.  Eleve  las  extremidades  quemadas  y  estimule  el  
rango  completo  de  movimiento  de  todas  las  articulaciones.

10.  ¿Qué  acciones  se  necesitan  por  parte  de  los  proveedores  prehospitalarios  (es  decir,  después  de  la
llega  la  tripulación,  ¿cuáles  son  sus  prioridades)?
El  Comité  de  Trauma  del  Colegio  Estadounidense  de  Cirujanos  (ACS­COT,  por  sus  siglas  en  inglés)  recomienda  que  todos  los  
equipos  de  ambulancias  sigan  las  pautas  del  procedimiento  de  "recoger  y  correr"  para  todas  las  víctimas  de  quemaduras  dentro  
de  los  60  minutos  de  un  hospital  apropiado  (centro  de  trauma  de  nivel  I  o  II  o  centro  para  quemados). ).  Intente  colocar  una  línea  
intravenosa  (IV)  en  el  camino,  pero  esto  no  es  esencial  si  el  tiempo  de  viaje  es  <60  minutos.  Las  líneas  se  pueden  colocar  a  través  
de  la  piel  quemada,  preferiblemente  en  las  venas  antecubitales.

11.  ¿Cómo  contribuye  el  servicio  de  urgencias  hospitalario  a  la  atención  de  los  pacientes  con  grandes  quemaduras?

La  urgencia  en  el  cuidado  de  la  víctima,  no  la  herida,  es  fundamental  para  la  supervivencia  final  de  la  víctima:
A.  Vía  aérea.  Busque  hollín  en  la  faringe  y  quemaduras  faciales  extensas.
B.  Respiración.  Identificar  ronquera  o  estridor.  Escuche  los  sonidos  de  la  respiración  en  ambos  lados.
Machine Translated by Google
CAPÍTULO  34  QUEMADURAS  175

C.  Circulación.  Coloque  dos  líneas  intravenosas  periféricas,  comience  con  líquidos  como  solución  de  Ringer  lactato;
calcule  la  fórmula  de  Parkland  ¼  4  ml  kg  peso  corporal  %  cuerpo  quemado  [la  mitad  del  volumen  en  las  primeras  8  
horas;  la  otra  mitad  durante  16  horas].
D.  Déficit  neurológico.  Examinar  el  sistema  nervioso  central  (SNC)  y  los  nervios  craneales;  evaluar
el  estado  neurológico  de  las  extremidades  quemadas.
E.  Exponga  y  examine  la  piel,  gire  el  registro  y  determine  meticulosamente  el  tamaño  de  la  quemadura  en  la
cuerpo  posterior,  y  luego  cubrir  y  conservar  el  calor  corporal.  El  entorno  del  paciente  debe  calentarse  a  90  F.

F.  Se  debe  evaluar  el  efecto  de  la  terapia  de  fluidos  según  lo  demostrado  por  1  ml  de  producción  de  orina  por  kilogramo  
de  peso  corporal  cada  hora.
El  manejo  del  dolor  y  el  apoyo  psicoemocional  también  son  de  vital  importancia.  Evite  la  sobredosis  de  narcóticos.

12.  ¿Cuál  es  el  tamaño  de  las  quemaduras?

Esta  determinación  se  realiza  clínicamente,  con  la  ayuda  de  tres  importantes  herramientas  clínicas:
1.  La  superficie  volar  de  la  mano  abierta  de  la  víctima  (incluidos  los  dedos)  ¼  0,8%  a  1,0%  de  TBSA;  más  útil  para  
determinar  el  tamaño  de  áreas  de  heridas  pequeñas  y  dispersas  2.  Regla  de  los  nueves:  más  comúnmente  utilizada;  
fácil  de  memorizar;  no  es  correcto;  generalmente
sobrestima
Cabeza  adulta  ¼  9%

Total  extremidad  superior  ¼  2  9%
Extremidad  inferior  anterior  total  ¼  2  9%
Extremidad  inferior  posterior  total  ¼  2  9%
Torso  anterior  ¼  2  9%

Torso  posterior  incluyendo  glúteos  ¼  2  9%
Genitales  ¼  1%

Tenga  en  cuenta  que  los  adultos  y  los  niños  difieren  significativamente  por  la  diferencia  en  el  tamaño  relativo  de  la  cabeza  
(9%  en  adultos,  15%  en  bebés).  Por  el  contrario,  el  muslo  de  un  bebé  es  mucho  más  pequeño  que  el  de  un  adulto  (6%  frente  
a  10%).
3.  Gráfico  de  Lund  y  Browder:  más  preciso;  pérdida  de  tiempo;  requiere  práctica;  no  es  fácil  de
memorizar

13.  Además  de  la  lesión  cutánea  real,  ¿qué  otras  lesiones  asociadas  pueden  ocurrir?
La  lesión  por  inhalación  se  diagnostica  en  el  10%  de  todas  las  víctimas  de  quemaduras  hospitalizadas.  Otros  traumas  físicos  
se  asocian  frecuentemente  con  explosiones  o  simplemente  con  los  intentos  de  escapar  del  fuego.  La  conciencia  del  trauma  
asociado  justifica  la  importancia  de  una  evaluación  cuidadosa  del  trauma  de  Soporte  Vital  Avanzado  para  Trauma  (ATLS).

14.  ¿Cómo  se  define  la  lesión  por  inhalación?
En  contraste  con  la  lesión  por  quemadura  externa  visible  y  algo  cuantificable  en  la  piel,  la  inhalación  de  calor,  monóxido  
de  carbono,  cianuro  y  otros  vapores  tóxicos  o  nocivos  es  menos  visible  y  menos  cuantificable,  pero  bastante  peligrosa.  Cuatro  
mecanismos  separados  de  lesión  de  las  vías  respiratorias  a  veces  se  agrupan  incorrectamente  como  lesión  por  inhalación:

1.  Intoxicación  por  monóxido  de  carbono.

2.  Daño  por  calor  en  las  vías  respiratorias  superiores.

3.  Inhalación  de  componentes  tóxicos  del  humo  que  se  producen  por  la  combustión  de  materiales  sintéticos  modernos  
utilizados  en  la  decoración  interior  de  casas,  edificios  y  automóviles.
4.  Envenenamiento  por  cianuro:  la  combustión  de  muchos  materiales  sintéticos  también  produce  gas  cianuro,  que  se  une  
al  sistema  enzimático  del  citocromo  e  inhibe  la  función  mitocondrial  y  la  respiración  celular.  Los  niveles  de  cianuro  en  
sangre  deben  evaluarse  en  todos  los  pacientes  con  niveles  de  monóxido  de  carbono  >10%.
Machine Translated by Google
176  CAPÍTULO  34  QUEMADURAS

15.  ¿Cómo  se  diagnostica  la  lesión  por  inhalación?
1.  Ronquera  o  estridor  evidente.
2.  Quemaduras  sustanciales  en  la  cabeza  y  el  cuello  o  en  la  cara.

3.  Atrapamiento  en  un  espacio  cerrado  o  proximidad  directa  a  una  explosión.
4.  Quemaduras  extensas  en  todo  el  cuerpo  (>50%  en  adultos  jóvenes;  menos  en  ancianos).
5.  Historial  de  eventos  de  vapor  sobrecalentado.

16.  ¿Qué  tratamiento  ha  influido  más  en  el  resultado  de  las  víctimas  de  quemaduras  durante  el
últimos  100  años?
Reanimación  adecuada  y  oportuna  con  líquidos.

17.  ¿Por  qué  se  debe  reanimar  con  líquidos  y  por  qué  vía?
Todos  los  pacientes  con  quemaduras  >10%  TBSA  deben  recibir  líquidos.  La  reanimación  con  líquidos  a  través  del  
tracto  gastrointestinal  (GI)  con  una  sonda  orogástrica  se  usa  con  bastante  frecuencia  en  los  centros  de  quemados  pediátricos.
En  adultos,  la  vía  IV  es  obligatoria.

18.  ¿Cómo  se  maneja  la  fluidoterapia?
El  plan  de  manejo  de  fluidos  tiene  dos  componentes.  En  primer  lugar,  determine  el  tamaño  de  la  herida  por  quemadura  
y  el  peso  del  paciente  en  kilogramos,  calcule  la  tasa  de  líquidos  por  hora  mediante  la  fórmula  de  Parkland  y  administre  
solución  de  Ringer  lactato  a  la  tasa  por  hora  calculada.
El  segundo  componente  del  plan  es  igual  de  importante.  Supervise  la  eficacia  de  su  fluido
plan  de  terapia  y  ajustarlo  de  inmediato  cuando  se  indique.  Nuestro  objetivo  es  un  paciente  
hemodinámicamente  normalizado,  con  una  diuresis  de  0,5  a  1,0  ml  de  orina  por  kilogramo  de  peso  corporal  por  hora.

19.  ¿Qué  se  debe  hacer  si  este  algoritmo  de  tratamiento  no  logra  lograr  resultados  clínicos?
mejora  y  estabilización  del  paciente?
La  falta  de  respuesta  a  la  fórmula  de  Parkland  es  indicativa  de  un  mal  pronóstico.  Sin  embargo,  algunas  medidas  
adicionales  pueden  ser  beneficiosas,  pero  actualmente  no  se  consideran  parte  del  estándar  de  atención.  Estos  incluyen  el  
uso  de  soluciones  salinas  hipertónicas  en  quemaduras  masivas,  el  uso  temprano  de  coloides  en  quemaduras  masivas  y  el  
uso  temprano  de  inotrópicos  (se  prefiere  la  dopamina  en  la  mayoría  de  los  textos  sobre  quemaduras).

20.  ¿Cómo  se  calculan  los  requerimientos  de  líquidos  cuando  ha  habido  un  retraso  en  el  inicio  de  la  terapia?

A  veces  la  demora  es  inevitable.  En  un  intento  de  abordar  una  acumulación  percibida  de  reanimación  con  líquidos,  
la  enseñanza  actual  es  aumentar  proporcionalmente  el  volumen  de  líquido  en  un  intento  de  obtener  el  volumen  total  deseado  
durante  8  horas  en  el  paciente  antes  de  que  transcurra  el  período  de  8  horas.  Este  remedio  requiere  algo  de  sentido  común;  
no  se  debe  aplicar  esta  guía  si  el  paciente  llega  al  sitio  de  reanimación  más  tarde  de  3  horas  después  de  la  lesión.  En  su  
lugar,  administre  líquidos  en  función  de  la  presión  arterial  (PA),  el  pulso  y  la  producción  de  orina.

21.  ¿Cuál  es  la  mejor  manera  de  cuidar  inicialmente  las  heridas  por  quemaduras?
Al  principio,  estas  heridas  necesitan  una  cobertura  simple  con  una  sábana  quirúrgicamente  limpia  o,  si  está  disponible,  estéril  
o  un  paño  quirúrgico.  Las  pautas  de  trauma  y  quemaduras  de  ACS­COT  establecen  que  el  cuidado  definitivo  de  la  herida  no  
necesita  ocurrir  hasta  24  horas  después  de  la  lesión.  Inicialmente  no  se  requiere  pomada  ni  tratamiento  antibacteriano  
específico.  El  paciente  debe  mantenerse  caliente  porque  la  exposición  precipita  la  hipotermia  sistémica.

Para  el  cuidado  definitivo  de  la  herida,  se  lava  o  se  ducha  a  todo  el  paciente  y  se  eliminan  los  desechos  residuales  
y  la  epidermis  dañada.  Luego  se  mapea  la  extensión  de  la  lesión,  por  lo  general  en  un
Machine Translated by Google
CAPÍTULO  34  QUEMADURAS  177

Gráfica  de  Lund  y  Browder,  junto  con  intentos  preliminares  para  determinar  la  profundidad  de  la  lesión.  Una  herida  por  

quemadura  suele  ser  un  mosaico  de  diferentes  áreas  lesionadas  en  diversos  grados  (profundidades).
Todas  las  heridas  por  quemadura  se  profundizan  hasta  cierto  punto  durante  las  primeras  48  a  96  horas,  por  lo  que  con  el  tiempo  se  

podrá  predecir  mejor  qué  áreas  requieren  injerto.

22.  ¿Por  qué  y  cómo  se  clasifica  la  profundidad  de  una  lesión  por  quemadura?
Esto  depende  de  la  presencia  de  apéndices  cutáneos  (folículo  piloso  y  glándulas  sudoríparas)  que  llevan  la  capa  germinal  a  lo  más  
profundo  de  la  dermis,  a  partir  de  la  cual  se  puede  producir  la  reepitelización.  El  día  de  la  lesión,  la  capacidad  visual  para  diferenciar  la  
herida  por  quemadura  que  sanará  de  la  que  no  lo  hará  es  pobre  (50%  de  precisión).  Con  el  tiempo  (próximos  3  a  7  días),  la  precisión  

de  la  predicción  clínica  mejorará  algo  (90%).  El  cuadro  34­2  ayuda  a  dilucidar  estos  aspectos.

Las  quemaduras  de  cuarto  grado  implican  daño  a  estructuras  más  profundas  que  la  dermis  (p.  ej.,  grasa,  músculo,
hueso,  tendón,  nervio,  cápsula  articular).  Las  quemaduras  se  designan  como  de  quinto  grado  cuando  el  tejido  se  pierde,  se  
desprende  o  se  vaporiza  por  la  quemadura  o  la  explosión.

TABLA  34­2.  PROFUNDIDAD  DE  LA  LESIÓN  CON  SIGNOS  CLÍNICOS  Y  SALIDA  PROBABLE
Signos  Clínicos  y

Profundidad  de  la  lesión Síntomas Resultado

Primer  grado  (lesión  superficial   Eritema  de  la  piel  con  molestias   Las  heridas  cicatrizan  espontáneamente  en  


limitada  a  la  epidermis) leves  a  moderadas. 5  a  10  días;  el  epitelio  dañado  se  despega,  
sin  dejar  efectos  residuales.

Segundo  grado Las  heridas  son  ampollas  o   Las  heridas  cicatrizan  espontáneamente  

Superficial  (involucra  la   supurantes,  eritematosas  y  dolorosas. en  2  a  3  semanas  sin  dejar  cicatrices  

totalidad  de  la  epidermis  y  la   residuales  y  con  una  piel  de  buena  calidad;  
porción  superficial  de  la  dermis)   La  piel  está  desecada,  con  ampollas,  a   la  pigmentación  puede  verse  alterada.  Las  

Profunda  (involucra  la  dermis  más   menudo  se  ven  escaras  blancas. heridas  cicatrizan  espontáneamente  

profunda,  pero  quedan  partes   En  ocasiones,  las  heridas  están   después  de  3  a  4  semanas;  a  menudo  se  


viables  de  los  apéndices   húmedas  y  son  difíciles  de  distinguir   produce  cicatrización  hipotrófica  y,  en  
epidérmicos) de  las  quemaduras  de  tercer  grado. ocasiones,  epitelio  inestable.  Para  obtener  
los  mejores  resultados,  elimine  la  escara  

mediante  escisión  tangencial  y  cubra  con  
un  injerto  de  piel  de  espesor  parcial.

Tercer  grado  (todos  los   Escara  insensible  avascular,   A  menos  que  sean  de  tamaño  pequeño  (<2  


apéndices  epidérmicos   cerosa,  blanca,  coriácea  marrón   cm  de  diámetro),  las  heridas  requieren  la  
destruidos) o  negra. eliminación  de  la  escara  y  la  cobertura  con  
injerto  de  piel  para  su  cicatrización.

23.  ¿Cuándo  debe  comenzar  la  escisión  quirúrgica  de  la  herida  por  quemadura?
Debe  comenzar  lo  antes  posible,  pero  debe  combinarse  con  sentido  común  y  pragmatismo,  lo  que  implica  un  paciente  
hemodinámicamente  "normalizado"  sin  signos  de  sepsis  u  otra  contraindicación  para  una  cirugía  mayor.  Esto  puede  ser  tan  pronto  
como  24  horas  después  de  la  lesión  en  quemaduras  de  tamaño  pequeño  a  moderado  (30%  dependiendo  de  la  edad),  pero  puede  

demorar  de  4  a  10  días  en  pacientes  inestables,  sépticos  o  frágiles.
Machine Translated by Google
178  CAPÍTULO  34  QUEMADURAS

24.  ¿Cómo  se  maneja  el  área  extirpada?
Un  avance  significativo  en  el  tratamiento  de  quemaduras  se  produjo  a  principios  de  la  década  de  1970,  cuando  
Janzekovic  demostró  que  las  heridas  extirpadas  debían  injertarse  inmediatamente  con  piel.  Este  sigue  siendo  el  objetivo.  
Si  los  sitios  donantes  son  insuficientes,  se  puede  usar  piel  de  cadáver,  piel  de  cerdo  o  productos  biosintéticos  (p.  ej.,  
Integra,  Biobrane  o  Transcyte)  para  cubrir  la  herida.  Estas  áreas  requieren  autoinjerto  posteriormente.  Los  queratinocitos  
autólogos  cultivados  son  una  alternativa  teórica  atractiva,  pero  todavía  carecen  de  un  alto  porcentaje  de  injerto  consistente  
cuando  se  usan  en  áreas  grandes.

25.  ¿Cuál  es  el  impacto  de  una  quemadura  grave  en  el  cuerpo?
Una  gran  quemadura  encabeza  la  lista  de  enfermedades  o  lesiones  que  es  mejor  evitar.  La  respuesta  metabólica  
alcanza  un  máximo  de  2,5  veces  la  tasa  metabólica  basal  (TMB)  en  todas  las  quemaduras  >50  %  de  TBSA.  Esta  
aceleración  máxima  del  metabolismo  del  cuerpo  por  lesión  por  quemadura  conduce  a  un  catabolismo  rápido  y  grave,  
agravado  aún  más  por  períodos  de  septicemia  y  pérdida  de  calor  a  través  de  una  mayor  evaporación.

26.  ¿Cuál  es  la  mejor  manera  de  suministrar  combustible  al  horno  metabólico  del  cuerpo?
El  apoyo  nutricional  de  la  víctima  de  la  quemadura  es  primordial.  La  nutrición  enteral  total  puede  tener  el  beneficio  adicional  
de  mantener  la  función  de  barrera  intestinal,  que  supuestamente  reduce  los  eventos  sépticos  al  prevenir  la  translocación  
bacteriana.

27.  ¿Cuál  es  el  papel  de  los  antibióticos  en  el  cuidado  de  las  quemaduras?

Los  antibióticos  nunca  se  administran  profilácticamente  para  las  lesiones  por  quemaduras.  Sin  embargo,  la  terapia  
antibiótica  temprana  y  apropiada  es  una  herramienta  críticamente  importante  y  que  salva  vidas  en  el  manejo  de  infecciones  
establecidas  en  pacientes  quemados.

28.  ¿Cómo  se  abordan  las  lesiones  por  quemaduras  químicas?
Cepille  todos  los  productos  químicos  que  quedan  en  forma  de  polvo  en  la  víctima.  Posteriormente,  se  debe  realizar  una  
irrigación  inmediata  y  prolongada  (30  minutos)  de  la  piel  contaminada  con  agua  corriente  del  grifo;  en  el  caso  de  
quemaduras  por  álcalis,  irrigar  durante  60  minutos.  Algunas  sustancias  químicas  pueden  absorberse;  por  lo  tanto,  está  
indicado  el  contacto  inmediato  con  un  centro  de  toxicología.

29.  ¿Cómo  se  manejan  los  pacientes  con  quemaduras  eléctricas?
Una  lesión  causada  por  la  electricidad  puede  ser  una  quemadura  eléctrica  o  una  lesión  por  contacto  o  conducción.  En  la  
lesión  por  relámpago  eléctrico,  la  descarga  eléctrica  ioniza  el  aire  o  la  atmósfera,  sin  conducción  de  corriente  a  través  del  
cuerpo.  Así,  la  lesión  es  sólo  cutánea.  Una  verdadera  quemadura  por  relámpago  eléctrico  cura  con  mayor  frecuencia  sin  
mucho  injerto.  El  compromiso  de  la  vía  aérea  es  raro.  Sin  embargo,  en  la  lesión  por  conducción  eléctrica,  el  tejido  se  daña  
a  través  de  la  transferencia  real  de  energía  eléctrica  a  través  del  paciente  desde  el  punto  de  entrada  hasta  el  de  salida.  La  
energía  térmica  se  genera  dentro  de  los  tejidos  debido  a  la  relativa  resistencia  a  la  conducción  de  la  corriente,  con  la  
desnaturalización  proteica  resultante  y  la  muerte  celular.  Diferentes  estructuras  (p.  ej.,  hueso,  piel,  músculo,  nervio,  tendón  
y  pulmón)  exhiben  una  conductividad  eléctrica  diferente,  lo  que  da  como  resultado  vías  de  conducción  impredecibles.

Por  lo  tanto,  la  piel  a  menudo  solo  está  mínimamente  afectada  en  los  sitios  de  entrada  y  salida,  con  necrosis  extensa  de  
músculos,  nervios,  tendones  e  incluso  huesos  en  patrones  erráticos.  Las  lesiones  neurológicas,  el  síndrome  compartimental  
y  la  mioglobinuria  son  complicaciones  frecuentes.  La  descompresión  rápida  del  tejido  (es  decir,  la  fasciotomía)  es  esencial  
con  una  reexploración  temprana  y  repetida  para  eliminar  el  tejido  necrótico.
El  objetivo  de  la  terapia  de  fluidos  debe  ser  lograr  altos  volúmenes  de  orina  (>1,0  a  1,5  mililitros  por  kilogramo  por  hora).  
La  alcalinización  de  la  orina  también  es  beneficiosa.

30.  Una  vez  curadas  las  lesiones  por  quemaduras,  ¿qué  cuestiones  importantes  quedan  por  abordar  en
el  período  de  rehabilitación?
La  rehabilitación  de  una  víctima  de  quemaduras  debe  comenzar  el  día  de  la  admisión  y  es  un  trabajo  de  equipo  total  que  
involucra  a  fisiatras,  cirujanos  plásticos,  terapeutas  ocupacionales,  fisioterapeutas,  nutricionistas,  psicólogos,  trabajadores  
sociales,  neumólogos,  microbiólogos,  farmacéuticos,  logopedas  y  enfermeras. .  La  rehabilitación  de  la  mente  y  el  cuerpo  
debe  ocurrir  en  concierto.
Machine Translated by Google
CAPÍTULO  34  QUEMADURAS  179

PUNTOS  CLAVE:  FACTORES  FUERTEMENTE  ASOCIADOS  CON
MORTALIDAD  DESPUÉS  DE  UNA  LESIÓN  POR  QUEMADURA

1.  Tamaño  de  la  quemadura  >40%  TBSA.

2.  Edad  del  paciente  >  60  años.

3.  Presencia  de  lesión  por  inhalación.

4.  Un  factor  de  riesgo:  tasa  de  mortalidad  del  3%;  los  tres  factores  de  riesgo:  tasa  de  mortalidad  del  90%.

SITIO  WEB

www.ameriburn.org/

BIBLIOGRAFÍA

1.  Blumetti  J,  Hunt  JL,  Arnoldo  BD  et  al.:  La  fórmula  de  Parkland  bajo  fuego:  ¿se  justifica  la  crítica?  Cuidado  de  quemaduras  J
Resolución  29:180,  2008.

2.  Demling  RL:  Atención  de  quemados  en  el  período  de  reanimación  inmediata.  En  Colegio  Americano  de  Cirujanos:  Cirugía:
Principios  y  Práctica,  Chicago,  2002,  Colegio  Americano  de  Cirujanos.

3.  Gibbons  J:  Prevención.  En  Gibbons  J,  editor:  ¡Fuego!  38  Consejos  para  salvar  vidas  para  usted  y  su  familia,  Seattle,  1995,
Editorial  Ballard.

4.  Heyland  DK,  Novak  F,  Drover  JW  et  al.:  ¿Debería  la  inmunonutrición  convertirse  en  una  rutina  en  pacientes  en  estado  crítico?  JAMA
29:944,  2001.

5.  Krzywiecki  A,  Ziora  D,  Niepsuj  G  et  al.:  Consecuencias  tardías  de  las  quemaduras  del  sistema  respiratorio.  J  Physiol  Pharmacol  58
(Suplemento  5):  319,  2007.

6.  McDonald­Smith  GP,  Saffle  JR,  Edelman  L  et  al.:  Informe  de  2002  del  National  Burn  Repository,  Chicago,  2002,
Asociación  Americana  de  Quemaduras.

7.  McGill  V,  Kahn  S,  Gamelli  RL  et  al .:  El  impacto  del  uso  de  sustancias  en  la  mortalidad  y  morbilidad  por  lesión  térmica.
J  Trauma  38:931,  1995.

8.  Mustonen  KM,  Vuola  J:  Insuficiencia  renal  aguda  en  pacientes  quemados  en  cuidados  intensivos.  J  Burn  Care  Res  29(1):227­237,  2008.

9.  Pruitt  BA,  Goodwin  CW,  Mason  AD  Jr:  Características  epidemiológicas,  demográficas  y  de  resultados  de  las  lesiones  por  quemaduras.  En  
Herndon  DN,  editor:  Total  burn  care,  2.ª  ed.,  Filadelfia,  2002,  WB  Saunders.

10.  Ryan  CM,  Sheridan  RL,  Tompkins  RG  et  al .:  Estimaciones  objetivas  de  la  probabilidad  de  muerte  por  lesiones  por  quemaduras.
N  Engl  J  Med  338:362­368,  1998.
Machine Translated by Google

TRAUMA  PEDIÁTRICO
CAPÍTULO  
35

Jonathan  P.  Roach,  MD  y  David  A.  Partrick,  MD

1.  ¿Cuál  es  la  principal  causa  de  muerte  en  niños  en  los  Estados  Unidos?
Las  lesiones  causan  más  muertes  y  discapacidades  en  niños  de  1  a  18  años  que  todas  las  demás  causas  
combinadas.  Las  muertes  por  lesiones  no  intencionales  representan  el  65%  de  todas  las  muertes  por  lesiones  en  
niños  menores  de  19  años.  Cada  año,  aproximadamente  20.000  niños  y  adolescentes  mueren  como  resultado  de  
lesiones  y  50.000  niños  sufren  discapacidades  permanentes.  Cada  año,  casi  1  de  cada  4  niños  recibe  tratamiento  
médico  por  una  lesión.  El  costo  anual  estimado  es  de  $  15  mil  millones.

2.  ¿Qué  grupos  de  edad  tienen  un  riesgo  particular  de  muerte  traumática?
Los  bebés  menores  de  2  años  tienen  una  tasa  de  mortalidad  consistentemente  más  alta  para  el  mismo  nivel  
de  lesión.  Sin  embargo,  durante  la  adolescencia,  las  lesiones  cobran  el  mayor  número  de  víctimas  y  representan  casi  el  
80%  de  las  muertes.

3.  ¿Qué  mecanismos  primarios  explican  las  lesiones  traumáticas  pediátricas?
Lesiones  contusas  (90%),  penetrantes  (9%)  y  por  aplastamiento  (<1%).  Los  accidentes  automovilísticos  son  la  causa  
más  común  de  lesiones  (50%)  y  muerte  en  la  niñez.

4.  ¿Cuál  es  la  incidencia  de  lesiones  por  región  del  cuerpo?
Múltiples  (50%),  extremidades  (20%),  cabeza  y  cuello  (15%),  abdomen  (3%),  cara  (2%)  y  tórax  (1%).

5.  ¿Cuál  es  la  mortalidad  general  por  lesiones  en  los  niños?
El  dos  por  ciento  de  todos  los  niños  heridos  y  el  3%  de  los  niños  heridos  hospitalizados.

6.  ¿Cuál  es  la  tasa  de  mortalidad  de  las  lesiones  por  mecanismo?
Mecanismo  Mortalidad  (%)

Golpeo  13  Herida  de  bala  (GSW)  8  
Accidente  de  vehículo  de  motor  5  Peatón  
5  
Motocicleta  3  Bicicleta  2  Deporte  1  Caída  
1  
Otro  3

7.  ¿Son  los  niños  y  las  niñas  igualmente  susceptibles  a  las  lesiones?
No.  Los  niños  se  lesionan  el  doble  que  las  niñas.  Los  niños  y  los  hombres  tienen  un  riesgo  4  veces  mayor  de  
suicidio  "con  éxito" (aunque  los  niños  lo  intentan  con  menos  frecuencia),  un  riesgo  3  veces  mayor  de  ahogamiento,  
un  riesgo  2,5  veces  mayor  de  homicidio  y  un  riesgo  2  veces  mayor  de  accidentes  relacionados  con  vehículos  
motorizados.  trauma.  El  segundo  cromosoma  X  es  claramente  protector.

180
Machine Translated by Google
CAPÍTULO  35  TRAUMA  PEDIÁTRICO  181

8.  ¿En  qué  se  diferencian  las  vías  respiratorias  de  un  niño  de  las  de  un  adulto?
Los  niños  corren  un  mayor  riesgo  de  obstrucción  de  las  vías  respiratorias  debido  a  su  lengua  grande;  epiglotis  
flácida;  aumento  del  tejido  linfoide;  y  tráquea  corta  y  de  pequeño  diámetro.  Los  tubos  endotraqueales  sin  manguito  
son  apropiados  en  niños  menores  de  8  años  para  minimizar  el  trauma  de  las  cuerdas  vocales,  el  edema  subglótico  y  
la  ulceración.  La  parte  más  estrecha  de  las  vías  respiratorias  de  un  niño  es  el  anillo  cricoides,  que  funciona  como  un  
sello  para  el  tubo  endotraqueal  sin  manguito.

9.  ¿Cuál  es  el  tamaño  apropiado  de  tubo  endotraqueal  para  colocar  en  un  niño?
El  tubo  endotraqueal  debe  ser  del  mismo  tamaño  que  el  dedo  meñique  del  niño.  Para  recién  nacidos,  use  un  tubo  de  
3  mm;  niños  en  primer  año  de  vida,  tubo  de  4  mm;  niños  mayores  de  1  año,  diámetro  interno  del  tubo  endotraqueal  ¼  
18  þ  edad  del  paciente  en  4  años  (pero,  en  una  situación  urgente,  no  recurra  a  cálculos  extensos;  simplemente  mire  
el  dedo  meñique  del  niño).

10.  ¿Qué  sucede  si  no  se  puede  realizar  la  intubación  endotraqueal  oral?
Una  cricotirotomía  con  aguja  es  preferible  a  la  cricotirotomía  quirúrgica  y  se  puede  realizar  con  un  catéter  de  calibre  
14.  Conceptualmente,  esto  es  lo  mismo  que  la  insuflación  a  chorro  en  adultos.  La  cricotiroidotomía  quirúrgica  es  
mucho  más  difícil  en  niños  pequeños  y  tiene  una  alta  asociación  con  estenosis  subglótica  secundaria.

11.  ¿Cuál  es  el  volumen  total  de  sangre  de  un  niño?

Es  de  80  ml/kg  (8%  del  peso  corporal).

12.  ¿Cuál  es  el  primer  signo  de  pérdida  significativa  de  sangre  en  los  niños?
Taquicardia.  Los  niños  pequeños  son  increíblemente  duros  y  tienen  una  notable  tolerancia  a  la  pérdida  de  sangre.
Una  hemorragia  del  30  %  del  volumen  sanguíneo  puede  no  provocar  cambios  en  la  presión  arterial  (PA),  pero  dicha  pérdida  
de  sangre  provoca  un  aumento  rápido  de  la  frecuencia  cardíaca.  El  gasto  cardíaco  (CO)  de  un  niño  depende  en  gran  medida  
de  la  frecuencia  cardíaca  (FC);  a  diferencia  de  los  adultos,  los  niños  tienen  una  capacidad  limitada  para  aumentar  el  volumen  
sistólico.

13.  ¿Cuáles  son  los  signos  de  shock  hipovolémico  en  niños?
Taquicardia  (que  progresa  a  bradicardia),  estado  mental  alterado,  compromiso  respiratorio,  relleno  capilar  
retrasado  (>2  segundos)  y  pulsos  periféricos  disminuidos  o  ausentes.

14.  ¿Es  la  hipotensión  un  indicador  fiable  de  la  pérdida  de  sangre  en  los  niños?
No.  Menos  de  la  mitad  de  los  niños  lesionados  con  hipotensión  documentada  tienen  un  insulto  identificable  que  resulta  
en  una  pérdida  significativa  de  volumen.  La  hipotensión  a  menudo  se  asocia  con  una  lesión  cerrada  aislada  en  la  
cabeza,  especialmente  en  niños  menores  de  6  años.

15.  ¿Por  qué  los  niños  tienen  un  mayor  riesgo  de  hipotermia  durante  la  reanimación?
El  área  de  superficie  del  niño  es  grande  en  relación  con  la  masa  corporal  interna;  un  niño  desnudo  puede  perder  calor  
rápidamente.  Los  líquidos  intravenosos  (IV)  fríos  y  los  gases  inhalados  pueden  exacerbar  la  hipotermia  y  provocar  
hipoxemia,  que  provoca  hipertensión  pulmonar  y  acidosis  metabólica  progresiva.
Particularmente  vulnerables  son  los  lactantes  <6  meses  de  edad,  que  carecen  de  grasa  subcutánea  significativa  y  de  
un  mecanismo  eficaz  de  escalofríos.

16.  ¿Qué  sitios  se  prefieren  para  el  acceso  venoso  en  niños?
Se  deben  insertar  percutáneamente  dos  catéteres  intravenosos  de  gran  calibre  en  las  extremidades  superiores.  La  
segunda  opción  es  el  acceso  percutáneo  a  la  vena  safena  distal  (o  una  incisión).

17.  ¿Qué  sucede  si  no  puede  establecer  una  vía  intravenosa?
La  ruta  intraósea  es  segura  y  en  realidad  requiere  menos  tiempo  que  una  incisión  venosa.  La  superficie  
anteromedial  de  la  tibia  proximal  se  usa  con  mayor  frecuencia,  con  la  aguja  colocada  3  cm  distal  a  la  tuberosidad  tibial.  
El  fémur  proximal,  el  fémur  distal  y  la  tibia  distal  son  otros
Machine Translated by Google
182  CAPÍTULO  35  TRAUMA  PEDIÁTRICO

sitios  potenciales.  Se  han  administrado  con  éxito  solución  salina,  glucosa,  sangre,  bicarbonato,  
atropina,  dopamina,  epinefrina,  diazepam,  antibióticos,  fenitoína  y  succinilcolina  por  vía  intraósea.  Las  
complicaciones  son  raras  y  resultan  principalmente  de  infección  o  extravasación.  La  reanimación  con  
volumen  intraóseo  facilita  la  posterior  canulación  de  la  circulación  venosa.

18.  ¿Cuáles  son  los  volúmenes  apropiados  de  reanimación  con  cristaloides  y  sangre  en
¿niños?
Administrar  20  ml/kg  de  solución  de  lactato  de  Ringer  o  solución  salina  normal  en  bolo.  Una  respuesta  es  
una  disminución  de  la  FC  y  un  aumento  de  la  diuresis.  El  bolo  de  20  ml/kg  debe  repetirse  si  la  evaluación  
revela  una  perfusión  tisular  inadecuada.  Si  la  evidencia  de  shock  persiste  después  de  dos  infusiones  en  bolo  
de  solución  cristaloide,  se  deben  administrar  10  ml/kg  de  glóbulos  rojos  concentrados  (tipo  específico  si  está  
disponible  u  O­negativo).  Desafortunadamente,  una  respuesta  favorable  a  la  reanimación  no  excluye  una  
gran  lesión  abdominal  o  torácica.

19.  ¿Por  qué  las  lesiones  en  la  cabeza  son  más  comunes  en  niños  que  en  adultos?
Hasta  los  10  años,  las  cabezas  de  los  niños  son  más  grandes  en  relación  con  el  cuerpo  que  las  cabezas  de  los  adultos.
La  lesión  del  sistema  nervioso  central  (SNC)  es  la  principal  causa  de  muerte  entre  los  niños  lesionados  y,  por  
lo  tanto,  es  el  principal  determinante  del  resultado.

20.  ¿Qué  tipos  de  lesiones  en  la  cabeza  son  más  comunes  en  los  niños?
La  hemorragia  epidural  es  la  más  común;  la  hemorragia  subdural  es  relativamente  rara.  Sin  embargo,  la  
mortalidad  por  hemorragia  subdural  es  del  40%  frente  al  4%  de  una  hemorragia  epidural.  Los  pacientes  
pediátricos  también  tienden  a  sufrir  lesiones  que  producen  edema  difuso  en  lugar  de  lesiones  focales  que  
ocupan  espacio.

21.  ¿Pueden  los  niños  tener  un  trauma  torácico  significativo  sin  fracturas  de  costillas?
Absolutamente.  La  pared  torácica  es  mucho  más  complaciente  en  niños  que  en  adultos;  por  lo  tanto,  la  energía  
cinética  se  transmite  más  fácilmente  a  las  estructuras  dentro  del  tórax.  Un  niño  con  un  traumatismo  torácico  cerrado  
significativo  tiene  un  mayor  riesgo  de  sufrir  una  contusión  potencialmente  mortal  en  los  pulmones  o  el  corazón,  incluso  si  
no  tiene  fracturas  costales  o  las  tiene  relativamente  pocas.  Además,  el  neumotórax  puede  resultar  fatal  rápidamente  en  
niños  debido  a  un  mediastino  más  móvil.  Cuando  están  presentes,  las  fracturas  de  costillas  en  los  niños  reflejan  un  
traumatismo  no  accidental.  La  lesión  torácica  es  la  segunda  causa  principal  de  muerte  (después  del  traumatismo  
craneoencefálico)  en  niños.

22.  ¿Qué  tipos  de  lesiones  torácicas  son  comunes  o  poco  comunes  en  los  niños?
Son  frecuentes  la  contusión  pulmonar,  la  asfixia  traumática  y  las  lesiones  traqueobronquiales.  La  ruptura  
aórtica  traumática,  el  tórax  inestable,  la  ruptura  diafragmática  y  el  neumotórax  abierto  son  inusuales.

23.  ¿Cuál  es  la  frecuencia  de  lesión  de  órganos  abdominales  en  trauma  cerrado?
En  orden  decreciente  de  frecuencia,  son  el  bazo,  el  hígado,  los  riñones,  el  intestino,  el  páncreas,  la  vejiga  
urinaria  y  los  principales  vasos  sanguíneos.  Aproximadamente  un  tercio  de  los  niños  con  trauma  mayor  
tienen  lesiones  intraperitoneales  significativas  que  deben  reconocerse  y  tratarse  con  rapidez.

24.  ¿Qué  precisión  tiene  el  examen  físico  en  la  evaluación  del  traumatismo  abdominal  cerrado  pediátrico?

Pobre.  El  examen  físico  es  engañoso  en  el  50%  de  los  niños  lesionados.

25.  ¿Cuáles  son  las  ventajas  y  desventajas  del  lavado  peritoneal  diagnóstico  en
¿niños?
El  lavado  peritoneal  de  diagnóstico  (DPL)  tiene  una  precisión  del  96  %  para  detectar  lesiones  intraabdominales.  Sin  
embargo,  puede  dar  lugar  a  tasas  de  laparotomía  no  terapéutica  del  15%.
Machine Translated by Google
CAPÍTULO  35  TRAUMA  PEDIÁTRICO  183

26.  ¿Cuáles  son  las  ventajas  y  desventajas  de  la  tomografía  computarizada  en  niños?
La  tomografía  computarizada  (TC)  abdominal  es  segura,  no  invasiva  y  puede  evaluar  estructuras  retroperitoneales  e  
identificar  lesiones  de  órganos  específicos.  La  TC  es  crítica  en  la  decisión  de  manejar  a  los  niños  sin  cirugía.  Las  
desventajas  incluyen  la  insensibilidad  para  la  lesión  visceral  hueca  y  la  necesidad  de  agentes  de  contraste  intravenosos  y  
enterales.  Además,  la  TC  requiere  mucho  tiempo  (la  TC  en  espiral  puede  resultar  mejor)  y  requiere  el  transporte  y  la  
sedación  del  paciente.  Un  viaje  al  escáner  deja  a  los  pacientes  vulnerables  y  sin  control.  Por  lo  tanto,  la  TC  es  riesgosa  en  
pacientes  inestables.

27.  ¿Es  efectiva  la  ultrasonografía  en  la  evaluación  de  niños  con  trauma  abdominal?
Sí.  Es  simple,  rápido,  fácilmente  disponible  y  se  puede  realizar  al  lado  de  la  cama.  Además,  no  es  invasivo  y  es  fácilmente  
repetible.  La  sensibilidad  y  especificidad  de  una  ecografía  abdominal  enfocada  en  el  examen  de  trauma  (FAST)  supera  el  
95%.  La  ecografía  abdominal  se  utiliza  mejor  como  una  herramienta  de  clasificación  para  detectar  líquido  intraperitoneal  
significativo,  identificando  así  a  los  pacientes  hemodinámicamente  inestables  que  podrían  beneficiarse  de  una  laparotomía.

28.  ¿Existe  un  método  fiable  para  diagnosticar  lesiones  de  vísceras  huecas  en  niños?
No.  Los  exámenes  físicos  en  serie  siguen  siendo  el  estándar  de  oro.  Es  obligatorio  repetir  el  examen  físico  por  parte  del  
equipo  quirúrgico  de  trauma.

29.  ¿Cuáles  son  los  «signos  leves»  de  lesión  intraabdominal  pediátrica?
  La  equimosis  en  cinturón  abdominal  corresponde  a  una  alta  incidencia  de  lesiones  de  órganos  sólidos,  vísceras  huecas
lesión  y  lesión  de  la  columna  lumbar.
  La  hematuria  macroscópica  tiene  un  riesgo  del  30%  de  lesión  intraabdominal  significativa  que  ni  siquiera  afecta  al  
sistema  genitourinario.
  La  elevación  de  las  enzimas  hepáticas  aspartato  aminotransferasa  (>250  U/L)  o  alanina  aminotransferasa  
(>450  U/L)  corresponde  a  un  50%  de  riesgo  de  daño  hepático.
&  Los  niños  con  fractura  pélvica  documentada  tienen  al  menos  un  20  %  de  riesgo  de
lesión  intraabdominal.
  Los  niños  con  deterioro  neurológico  grave  (puntuación  en  la  escala  de  coma  de  Glasgow  <8)  suelen  sufrir  lesiones  
intraabdominales  concurrentes.

30.  ¿Qué  se  debe  sospechar  en  niños  con  lesiones  en  el  cinturón  de  seguridad  o  en  el  manillar?
El  complejo  del  cinturón  de  seguridad  consiste  en  equimosis  de  la  pared  abdominal,  lesión  por  flexión­distracción  de  
la  columna  lumbar  (fractura  de  Chance)  y  lesión  intestinal.  Aproximadamente  el  30%  de  los  niños  con  el  signo  del  
cinturón  de  seguridad  tienen  una  lesión  intestinal  asociada.  Una  lesión  en  el  manillar  clásicamente  provoca  la  rotura  
del  páncreas  en  la  unión  del  cuerpo  y  la  cola,  donde  el  páncreas  cruza  la  columna  vertebral  y  es  vulnerable  a  la  
compresión  contundente  anterior.

31.  ¿La  presencia  de  hemoperitoneo  en  niños  requiere  laparotomía?
No.  A  diferencia  de  los  adultos,  <15%  de  los  niños  con  hemoperitoneo  requieren  laparotomía  para  controlar  el  sangrado  o  
reparar  una  lesión.

32.  ¿Todos  los  niños  con  lesiones  de  órganos  sólidos  requieren  reparación  quirúrgica?
No.  Al  igual  que  con  los  pacientes  adultos,  el  tratamiento  no  quirúrgico  selectivo  de  las  lesiones  de  órganos  sólidos  
ha  revolucionado  el  tratamiento  de  los  traumatismos  pediátricos.  De  hecho,  la  mayoría  de  los  pacientes  que  sufren  una  
lesión  hepática  o  esplénica  de  Grado  I  a  IV  no  requerirán  una  operación.

33.  ¿Cuándo  es  adecuado  el  tratamiento  no  quirúrgico  de  una  lesión  de  órganos  sólidos  en  niños?

Cuando  los  signos  vitales  permanecen  estables,  se  reemplaza  el  50%  o  menos  del  volumen  sanguíneo  y  no  hay  otras  
lesiones  intraabdominales  significativas.  La  decisión  de  manejo  conservador  versus  laparotomía  debe  basarse  en  la  
condición  fisiológica  del  niño  y  no  en  la  extensión  de  la  lesión  documentada  radiográficamente.
Machine Translated by Google
184  CAPÍTULO  35  TRAUMA  PEDIÁTRICO

34.  ¿Cuáles  son  las  consecuencias  a  largo  plazo  del  tratamiento  no  quirúrgico  de  una  lesión  esplénica  en  niños?

El  pseudoaneurisma  vascular  y  el  quiste  esplénico  son  secuelas  conocidas  del  traumatismo  esplénico.  Sin  embargo,  en  
pacientes  pediátricos  estas  complicaciones  son  extremadamente  raras  (<1%).  Hasta  el  15  %  de  los  niños  (dependiendo  
del  grado  de  la  lesión)  pueden  experimentar  dolor  prolongado  >4  semanas  después  de  la  lesión.  Sin  embargo,  en  casi  
todos  los  casos,  este  dolor  está  relacionado  con  el  proceso  de  curación  o  no  tiene  ninguna  relación  con  el  bazo.

35.  ¿Cuáles  son  las  indicaciones  para  la  intervención  quirúrgica  en  lesiones  de  órganos  sólidos?
Sangrado  masivo  en  la  presentación  y  transfusión  de  >50%  del  volumen  sanguíneo  (40  ml/kg)  dentro  de  las  24  horas  
posteriores  a  la  lesión.

36.  ¿Qué  es  SCIWORA?
SCIWORA  significa  lesión  de  la  médula  espinal  sin  anomalías  radiológicas.  Es  un  problema  exclusivo  de  los  niños.  La  
columna  de  un  niño  tiene  mayor  elasticidad,  articulaciones  facetarias  poco  profundas  y  orientadas  horizontalmente,  
acuñamiento  anterior  de  los  cuerpos  vertebrales  y  procesos  uncinados  poco  desarrollados.  La  médula  espinal  puede  
romperse  por  completo  en  los  niños  pequeños  sin  una  ruptura  aparente  de  los  elementos  vertebrales.  Sin  embargo,  la  
mayoría  de  los  pacientes  tienen  evidencia  de  lesión  de  la  médula  espinal  (LME)  en  la  resonancia  magnética  nuclear  
(RMN).  Dos  tercios  de  los  casos  de  SCIWORA  se  ven  en  niños  de  8  años  de  edad.

37.  ¿Cuál  es  el  sello  distintivo  de  SCIWORA?
Un  déficit  neurológico  documentado  que  puede  haber  cambiado  o  resuelto  cuando  el  niño  llega  al  departamento  de  
emergencias  (ED).  El  peligro  es  que  una  nueva  lesión  inmediata  de  la  misma  área  puede  producir  una  discapacidad  
permanente.  Muchos  niños  con  SCIWORA  tienden  a  desarrollar  déficits  neurológicos  horas  o  días  después  de  la  lesión  
informada.  Por  lo  tanto,  la  inmovilización  espinal  debe  continuar,  y  la  evaluación  neuroquirúrgica  completa  es  esencial  
en  cualquier  niño  con  evidencia  confiable  de  un  déficit  neurológico  incluso  transitorio.

38.  ¿Qué  porcentaje  de  muertes  pediátricas  atribuidas  a  lesiones  son  intencionales?

Veinticinco  porciento.  Más  del  80%  de  las  muertes  por  traumatismo  craneoencefálico  en  niños  menores  de  2  años  
son  causadas  por  abuso  intencional.

39.  ¿Qué  signos  son  sospechosos  de  trauma  no  accidental  (NAT)?
&  Historial  de  falta  de  crecimiento.
&  Retraso  en  la  obtención  de  atención  médica.
&  Múltiples  lesiones  previas.
&  Cuidador  ausente  o  desinteresado.
&  Historias  fluctuantes  o  contradictorias.
&  Antecedentes  incompatibles  con  la  lesión  o  el  nivel  de  desarrollo  de  la  víctima.
Los  hallazgos  físicos  sospechosos  incluyen  mordeduras,  pellizcos,  bofetadas  o  marcas  de  cordón  o  hematomas  en  
varias  etapas  de  curación;  fracturas  de  cráneo  múltiples  o  bilaterales;  una  fractura  de  cráneo  en  una  caída  <4  pies;  
hemorragias  retinianas  (por  sacudidas),  fracturas  de  costillas  y  quemaduras  perineales  o  bordes  de  quemaduras  lineales  
(por  "sumergir"  al  niño  en  líquido  hirviendo).

40.  Enumera  las  características  del  síndrome  del  bebé  sacudido.
&  Hemorragia  retinal.
  hemorragia  subdural  o  subaracnoidea.
  Escasa  evidencia  de  traumatismo  externo.

&  Edad  <2  años.
Machine Translated by Google
CAPÍTULO  35  TRAUMA  PEDIÁTRICO  185

41.  ¿Qué  patrones  de  fractura  son  sospechosos  de  NAT?
  Múltiples  fracturas  costales  de  diferentes  edades.
  Fracturas  de  las  extremidades,  como  fracturas  metafisarias  en  «astilla»  o  «en  asa  de  cubo».
  Fractura  espiral  diafisaria  en  niños  <9  meses  de  edad.
  Fractura  transversa  de  eje  medio  de  huesos  largos.
  Fractura  de  fémur  en  lactantes  <2  años  de  edad.
  Fractura  del  proceso  acromion  de  la  escápula.
  Fractura  de  húmero  proximal.

42.  ¿Qué  porcentaje  de  casos  de  NAT  involucra  lesiones  por  quemaduras?  Cuáles  son  sus
¿características?

El  veinte  por  ciento  de  los  casos  de  abuso  involucran  quemaduras.  Las  escaldaduras  por  agua  caliente  son  las  más  comunes.
Los  patrones  específicos  de  lesión  pueden  generar  sospechas  de  abuso,  incluidas  las  quemaduras  que  afectan  las  nalgas  
y  el  perineo  (distribución  en  tronco  de  baño),  la  espalda,  el  dorso  de  la  mano  y  la  distribución  en  calcetín­guante.  Las  
quemaduras  de  cigarrillos  se  ven  como  úlceras  perforadas  circulares  de  tamaño  similar.

43.  ¿Cuáles  son  los  pasos  necesarios  en  la  evaluación  de  niños  con  sospecha  de  NAT?
Cualquier  niño  con  sospecha  de  NAT  debe  someterse  a  un  examen  físico  detallado  con  documentación  completa  
(los  dibujos  y  las  fotografías  pueden  ser  muy  útiles)  de  todas  las  lesiones,  una  tomografía  computarizada  de  la  cabeza,  
un  estudio  esquelético  (babygram)  y  un  examen  de  fondo  de  retina.  Se  debe  contactar  de  inmediato  a  los  servicios  de  
protección  infantil  apropiados.

44.  ¿Qué  tan  común  es  la  falla  multiorgánica  posterior  a  una  lesión  en  los  niños?
Es  extraño.  Con  una  gravedad  de  lesión  equivalente,  la  insuficiencia  multiorgánica  (FMO)  en  niños  es  mucho  menor  que  
en  adultos  y  conlleva  una  mortalidad  mucho  menor.

45.  ¿Importa  el  nivel  de  glucosa  en  sangre  en  pacientes  pediátricos  con  trauma?
Sí.  Si  bien  este  tema  de  la  unidad  de  cuidados  intensivos  (UCI)  ha  recibido  gran  atención  en  la  población  traumatizada  de  
adultos,  solo  recientemente  se  han  examinado  los  efectos  beneficiosos  del  control  estricto  de  la  glucemia  en  la  población  
traumatizada  pediátrica.  La  euglucemia  (glucosa  de  90  a  130  mg/dl)  se  asocia  con  una  menor  tasa  de  infección,  una  
menor  duración  de  la  estancia  hospitalaria  y  una  mayor  supervivencia.

PUNTOS  CLAVE:  HEMODINÁMICA  PEDIÁTRICA

1.  Volumen  de  sangre:  80  ml/kg.

2.  El  primer  signo  de  hipovolemia  es  la  taquicardia,  que  progresa  a  bradicardia.

3.  La  hipotensión  no  es  un  indicador  fiable  de  pérdida  de  sangre;  los  niños  pueden  perder  el  30%  del  volumen  de  sangre
sin  cambios  detectables  en  la  PA.

4.  Vías  de  acceso  IV  preferidas  en  orden:  (1)  dos  IV  de  extremidad  superior  de  gran  calibre;  (2)  vena  safena  distal  
o  corte;  (3)  acceso  intraóseo.

5.  El  líquido  de  reanimación  es  lactato  de  Ringer,  20  ml/kg  2;  luego  concentrado  de  glóbulos  rojos  (10  ml/kg)
si  la  inestabilidad  continúa.

BIBLIOGRAFÍA

1.  Calkins  CM,  Bensard  DD,  Moore  EE  et  al.:  El  niño  lesionado  es  resistente  a  la  falla  multiorgánica:  ¿una  respuesta  inflamatoria  
diferente?  J  Trauma  53:1058,  2002.

2.  Dare  AO,  Dias  MS,  Li  V:  correlación  de  imágenes  por  resonancia  magnética  en  lesiones  pediátricas  de  la  médula  espinal  sin
anomalía  radiográfica.  J  Neurosurg  97(1  suplemento):33,  2002.
Machine Translated by Google
186  CAPÍTULO  35  TRAUMA  PEDIÁTRICO

3.  Kristoffersen  KW,  Mooney  DP:  Resultado  a  largo  plazo  del  manejo  no  quirúrgico  de  lesiones  esplénicas  pediátricas.
J  Pediatr  Surg  42:1038,  2007.

4.  Holmes  JF,  Gladman  A,  Chang  CH:  Rendimiento  de  la  ecografía  abdominal  en  trauma  cerrado  pediátrico
pacientes:  un  metanálisis.  J  Pediatr  Surg  42:1588,  2007.

5.  Mazzola  CA,  Adelson  PD:  Manejo  de  cuidados  críticos  del  traumatismo  craneoencefálico  en  niños.  Crit  Care  Med  30  (11  suplementos):
S393,  2002.

6.  Mehall  JR,  Ennis  JS,  Saltzman  DA  et  al .:  Resultados  prospectivos  de  un  algoritmo  estandarizado  basado  en  el  estado  hemodinámico  
para  el  manejo  de  lesiones  pediátricas  de  órganos  sólidos.  J  Am  Coll  Surg  193:347,  2001.

7.  Partrick  DA,  Bensard,  DD,  Janik  JS  et  al.:  ¿Es  la  hipotensión  un  indicador  fiable  de  la  pérdida  de  sangre  por  lesión  traumática?
¿en  ninos?  Am  J  Surg  184:555,  2002.

8.  Roaten  JB,  Partrick  DA,  Nydam  TL  et  al.:  El  trauma  no  accidental  es  una  causa  importante  de  morbilidad  y  mortalidad
entre  los  pacientes  de  un  centro  regional  de  trauma  pediátrico  de  nivel  I.  J  Pediatr  Surg  41:2013,  2006.

9.  St.  Peter  SD,  Keckler  SJ,  Spilde  TL  et  al.:  Justificación  de  un  protocolo  abreviado  en  el  manejo  de  embotamiento
lesión  del  bazo  y  el  hígado  en  los  niños.  J  Pediatr  Surg  43:191,  2008.

10.  Stafford  PW,  Blinman  TA,  Nance  ML:  puntos  prácticos  en  la  evaluación  y  reanimación  del  niño  lesionado.  Cirugía
Clin  North  Am  82:273,  2002.

11.  Tuggle  DW,  Kuhn  MA,  Jones  SK  et  al.:  Hiperglucemia  e  infecciones  en  pacientes  con  trauma  pediátrico.  Soy  quirúrgico
74:195,  2008.
Machine Translated by Google

tercero  CIRUGÍA  ABDOMINAL

APENDICITIS CAPÍTULO  
36

Laurel  R.  Imhoff,  MD,  MPH,  y  Alden  H.  Harken,  MD

1.  ¿Cuál  es  la  presentación  clásica  de  la  apendicitis  aguda?
Dolor  periumbilical  que  migra  al  cuadrante  inferior  derecho  (RLQ)  en  una  paciente  anoréxica.
Los  síntomas  asociados  incluyen:  náuseas,  vómitos  y  cambios  intestinales.

2.  ¿Cuál  es  la  fisiopatología  de  la  apendicitis?
El  apéndice  es  susceptible  a  la  obstrucción  luminal,  por  hiperplasia  linfoide,  fecalito  retenido,  tumor,  cuerpo  
extraño  o  torcedura.  Cualquiera  de  estos  procesos  puede  resultar  en  obstrucción  linfática  y  venosa  que  aumenta  
la  presión  intraluminal  y  causa  distensión  de  la  luz  apendicular.
En  consecuencia,  se  desarrolla  una  respuesta  inflamatoria  aguda  que  conduce  a  isquemia,  sobrecrecimiento  
bacteriano  y  eventualmente  necrosis.  A  menos  que  se  extirpe  quirúrgicamente,  el  apéndice  gangrenoso  se  
perforará,  liberando  el  contenido  apendicular  en  la  cavidad  peritoneal.  Posteriormente  se  desarrolla  un  flemón,  
un  absceso  intraperitoneal  o  una  peritonitis  local.

3.  ¿Cuál  es  el  mecanismo  del  dolor  periumbilical?
Los  intestinos  son  insensibles  al  tacto  oa  la  inflamación,  a  menos  que  esté  afectado  el  peritoneo  que  los  
envuelve.  El  dolor  epigástrico  resulta  de  una  sección  distendida  del  intestino.  Este  dolor  se  refiere  a  lo  largo  de  la  
línea  media.

4.  ¿Dónde  está  el  punto  de  McBurney?
Un  tercio  de  la  distancia  entre  la  espina  ilíaca  anterosuperior  y  el  ombligo.

5.  ¿Cuál  es  el  punto  de  McBurney?
El  punto  de  máxima  sensibilidad  en  la  apendicitis  aguda.  Resulta  de  la  inflamación  local  del  peritoneo  parietal.

6.  ¿Era  McBurney  un  policía  de  Boston?
Probablemente.  Otro  McBurney  fue  un  cirujano  de  Nueva  York  que,  en  colaboración  con  un  cirujano  llamado  Fitz,  
acuñó  el  término  apendicitis  en  artículos  clásicos  publicados  en  1886  y  1889.

7.  ¿Cuáles  son  los  hallazgos  de  laboratorio  típicos  de  un  paciente  con  apendicitis?
&  Recuento  de  glóbulos  blancos  (WBC):  12,000  a  14,000  &  
Resultados  de  análisis  de  orina  negativos  (sin  WBC)
&  Resultado  negativo  de  la  prueba  de  embarazo

8.  ¿Qué  capas  encuentra  el  cirujano  al  exponer  el  apéndice  a  través  de  una  incisión  de  Rockey­Davis?

Piel,  grasa  subcutánea,  aponeurosis  del  músculo  oblicuo  externo,  músculo  oblicuo  interno,  músculo  
transversalis  abdominus,  fascia  tranvsersalis  y  peritoneo.

9.  Otros  posibles  signos  de  apendicitis  incluyen:
Signo  de  Rovsings:  dolor  en  el  RLQ  a  la  palpación  del  cuadrante  inferior  izquierdo  (LLQ).
Signo  de  Dunphy:  aumento  del  dolor  al  toser  (la  tos  empuja  el  peritoneo  inflamado).

187
Machine Translated by Google
188  CAPÍTULO  36  ANEXOS

Signo  del  psoas:  dolor  a  la  extensión  pasiva  del  muslo  derecho.  Está  presente  cuando  el  inflamado
el  apéndice  es  retrocecal  y  recubre  el  músculo  psoas  derecho.
Signo  del  obturador:  dolor  en  la  rotación  interna  pasiva  de  la  cadera  cuando  se  flexiona  la  rodilla  derecha.  Está  
presente  cuando  el  apéndice  inflamado  está  en  contacto  con  el  músculo  obturador  interno.

10.  ¿Quién  fue  Rockey­Davis?
Rockey­Davis  fue  un  par  de  cirujanos,  AE  Rockey  y  GG  Davis,  que  desarrollaron  incisiones  transversales  RLQ  que  
dividen  los  músculos  y  se  extienden  hasta  la  vaina  del  recto.

11.  ¿Cuál  es  el  suministro  de  sangre  al  apéndice  y  al  colon  derecho?
Las  arterias  ileocólica  y  cólica  derecha,  que  se  desprenden  de  la  arteria  mesentérica  superior.

12.  ¿La  cirugía  de  la  apendicitis  implica  riesgo  de  mortalidad?
Ningún  procedimiento  quirúrgico  está  exento  de  riesgos.

Tasa  de  
mortalidad  Apéndice  no   <0,1%  
perforado  Apéndice  perforado 5,0%

13.  ¿Qué  grupos  de  pacientes  tienen  mayor  riesgo  de  muerte  por  apendicitis  perforada?
1.  Pacientes  muy  jóvenes  (menores  de  2  años).
2.  Pacientes  de  edad  avanzada  (mayores  de  70  años)  que  presentan  inervación  abdominal  disminuida  y
presente  tarde.
3.  Pacientes  diabéticos,  que  se  presentan  tarde  por  neuropatía  visceral  diabética.
4.  Pacientes  que  toman  esteroides;  los  esteroides  enmascaran  todo.

14.  ¿Qué  es  un  ''gusano  blanco''?

Un  apéndice  normal.

15.  ¿Cuál  es  el  diagnóstico  diferencial  del  dolor  en  el  cuadrante  inferior  derecho?
Divertículo  de  Meckel  Absceso  tubo­ovárico  (TOA)
Diverticulitis  Enfermedad  inflamatoria  pélvica  (EPI)

Embarazo  ectópico  Tumor  carcinoide  Enfermedad  de  
Crohn  Colecistitis  Torsión  ovárica  Quiste  ovárico  roto

16.  ¿Cuál  es  una  tasa  de  apendicectomía  negativa  aceptable?
Este  es  un  tema  controvertido  que  se  debate  actualmente  en  la  literatura  quirúrgica.  Tradicionalmente  se  
consideraba  aceptable  una  tasa  de  apendicectomía  negativa  de  hasta  el  20%.  Ahora,  con  el  complemento  de  las  
imágenes  (ultrasonido  [US]  y  tomografía  computarizada  [CT]),  se  esperan  tasas  negativas  más  bajas.

17.  ¿Cuál  es  el  papel  de  las  imágenes  en  el  diagnóstico  de  apendicitis  aguda?
La  ecografía  y  la  TC  pueden  ser  útiles  tanto  negativa  como  positivamente.  Pueden  eliminar  diagnósticos  
alternativos  como  el  embarazo  ectópico  o  TOA  cuando  se  observa  una  trompa  de  Falopio  y  un  ovario  derechos  
perfectamente  normales.  Pueden  establecer  el  diagnóstico  cuando  se  visualiza  un  apéndice  edematoso  e  inflamado.  La  
TC  es  particularmente  útil  para  visualizar  el  tejido  periapendicular  y  puede  revelar  que  el  apéndice  ya  se  ha  perforado  al  
mostrar  un  flemón  o  un  absceso.

18.  ¿Cuáles  son  los  hallazgos  ecográficos  y  de  TC  que  sugieren  apendicitis?
1.  Un  apéndice  de  7  mm  o  más  de  diámetro  anteroposterior.
2.  La  presencia  de  un  apendicolito.
3.  Líquido  o  masa  periapendicular.
Machine Translated by Google
CAPÍTULO  36  APÉNDICES  189

19.  ¿La  apendicectomía  laparoscópica  reemplaza  el  abordaje  tradicional?
En  manos  de  un  cirujano  laparoscópico  experto,  tanto  el  apéndice  normal  como  el  apéndice  inflamado  y  perforado  
pueden  extraerse  con  seguridad  con  el  laparoscopio.  Además,  cuando  el  diagnóstico  está  en  duda,  el  laparoscopio  se  
puede  utilizar  para  ayudar  a  identificar  la  patología  correcta.  Ocasionalmente  está  indicada  la  conversión  de  
apendicectomía  laparoscópica  a  abierta.

PUNTOS  CLAVE:  CARCINOIDE  APÉNDICE
1.  El  sesenta  por  ciento  de  los  tumores  carcinoides  ocurren  en  el  apéndice;  El  0,03%  de  las  apendicectomías  revelan  
un  carcinoide  incidental.

2.  Este  tumor  maligno  pero  lento  se  disemina  a  los  ganglios  linfáticos,  el  hígado  y  el  corazón  derecho.

3.  Si  el  tamaño  del  tumor  es  <2  cm  y  no  afecta  la  base  del  apéndice,  la  apendicectomía  sola  puede  ser  suficiente;  sin  
embargo,  se  debe  evaluar  el  intestino  debido  al  30%  de  probabilidad  de  lesión  sincrónica.

4.  Si  el  tamaño  del  tumor  es  >  2  cm  o  afecta  la  base  del  apéndice,  se  recomienda  una  hemicolectomía  derecha.
necesario.

20.  ¿Qué  es  un  divertículo  de  Meckel?

El  divertículo  de  Meckel  es  un  remanente  congénito  de  mucosa  onfalomesentérica  que  puede  contener  
mucosa  gástrica  ectópica.  Se  encuentra  en  el  2%  de  la  población,  2  pies  por  encima  de  la  válvula  ileocecal.  Se  
inflama  en  el  2%  de  los  pacientes  (es  decir,  la  regla  de  los  2).

21.  ¿Puede  la  diverticulitis  crónica  hacerse  pasar  por  apendicitis?
Sí.  El  cincuenta  por  ciento  de  los  pacientes  de  50  años  o  más  tienen  divertículos  colónicos.  El  apéndice  es  solo  un  gran  
divertículo  cecal.  Por  lo  tanto,  tiene  sentido  que  la  apendicitis  y  la  diverticulitis  se  vean,  actúen  y  huelan  de  la  misma  
manera.

22.  ¿Una  mujer  con  test  de  embarazo  negativo  puede  presentar  un  embarazo  ectópico?
Sí.  La  trompa  de  Falopio  debe  inspeccionarse  en  busca  de  un  bulto  del  tamaño  de  una  nuez.  El  tratamiento  quirúrgico  
apropiado  es  una  incisión  longitudinal  para  "desenvainar"  al  feto  con  la  subsiguiente  reparación  de  la  trompa.  Este  
enfoque  (a  diferencia  de  la  salpingectomía)  está  diseñado  para  preservar  la  fertilidad.  El  metotrexato  también  puede  
precipitar  la  evacuación  espontánea.

23.  ¿La  enfermedad  de  Crohn  puede  presentarse  inicialmente  como  apendicitis?
Sí;  esta  presentación  es  típica.  La  enfermedad  de  Crohn  es  una  inflamación  granulomatosa,  edematosa  y  pantanosa  del  
íleon  distal.  El  dictamen  quirúrgico  tradicional  sugiere  que  es  apropiado  extirpar  el  apéndice  en  pacientes  con  enfermedad  
de  Crohn  a  menos  que  esté  afectado  el  ciego  en  la  base  apendicular.

24.  ¿Se  puede  confundir  apendicitis  con  TOA?
Por  supuesto.  Un  absceso  ovárico  enterrado  profundamente  en  un  anexo  derecho  inflamado,  edematoso  y  
apelmazado  puede  tratarse  con  éxito  con  antibióticos  intravenosos  (IV)  solos.  No  drene  pus  en  la  cavidad  peritoneal  
libre;  esto  solo  hará  que  el  paciente  se  enferme  más.

25.  ¿Puede  la  enfermedad  inflamatoria  pélvica  parecerse  a  la  apendicitis?
La  EPI  puede  parecerse  exactamente  a  la  apendicitis  excepto  por  un  "signo  de  araña"  positivo.  En  el  examen  
pélvico,  el  tirón  manual  del  cuello  uterino  mueve  los  anexos  inflamados  y  dolorosos  y  la  paciente  golpea  la  araña.  Los  
pacientes  con  EPI  deben  recibir  tratamiento  con  antibióticos  (ya  sea  por  vía  oral  o  intravenosa,  según  el  grado  de  
enfermedad  del  paciente).
Machine Translated by Google
190  CAPÍTULO  36  APÉNDICES

26.  ¿Cómo  se  trata  un  tumor  carcinoide  apendicular?
Los  tumores  carcinoides  pueden  presentarse  en  cualquier  parte  del  tracto  gastrointestinal  (GI);  El  60%,  
sin  embargo,  se  encuentran  en  el  apéndice.  Un  tumor  carcinoide  obstructivo,  al  igual  que  un  fecalito,  
puede  provocar  apendicitis;  y  en  el  0,3%  de  las  apendicectomías,  los  tumores  carcinoides  son  los  
culpables.  La  mayoría  de  los  tumores  carcinoides  son  pequeños  (<1,5  cm)  y  benignos;  El  70%  se  ubican  
en  el  apéndice  distal.  Se  tratan  eficazmente  con  apendicectomía  sola.  Un  tumor  carcinoide  grande  (>2,0  
cm)  en  la  base  apendicular,  especialmente  con  invasión  del  mesoapéndice,  debe  considerarse  maligno  y  
obliga  a  una  hemicolectomía  derecha.

27.  ¿Se  puede  confundir  la  apendicitis  con  la  colecistitis  aguda?
De  vez  en  cuando,  sí.  Ambas  entidades  reflejan  una  inflamación  intraperitoneal  aguda,  localizada.  Los  estudios  de  laboratorio  
pueden  ser  idénticos:  recuento  de  glóbulos  blancos  de  12  000  a  14  000,  resultado  negativo  del  análisis  de  orina  y  resultado  
negativo  de  la  prueba  de  embarazo.  Por  lo  tanto,  si  uno  está  pensando  en  "apendicitis",  la  principal  diferencia  puede  ser  sólo  
el  dolor  en  el  cuadrante  superior  derecho  frente  al  dolor  en  el  cuadrante  inferior  izquierdo.  La  colecistectomía  laparoscópica  
es  posible  para  la  colecistitis  aguda,  pero  la  conversión  a  un  procedimiento  abierto  debe  ser  más  frecuente.

SITIOS  WEB

www.websurg.com  (conferencias,  videos,  excelentes  fotografías  de  cirugía  laparoscópica)  

www.pmppals.org/appendiceal_carcinoid.htm

BIBLIOGRAFÍA

1.  Cope  Z,  Rev.  by  Silen  W:  Cope's  Early  Diagnostic  of  the  Acute  Abdomen,  19th  ed.,  New  York,  1996,  Oxford
Prensa  Universitaria.

2.  Fitz  RH:  Inflamación  perforante  del  apéndice  vermiforme  con  especial  referencia  a  su  diagnóstico  y  tratamiento  precoz.  Trans  Assoc  Am  
Physicians  1:107,  1886.

3.  Flum  DR,  McClure  TD,  Morris  A  et  al .:  Diagnóstico  erróneo  de  apendicitis  y  el  uso  de  imágenes  de  diagnóstico.  J  Am  Coll  Surg  
201(6):933­939,  2005.  Epub  13  de  octubre  de  2005.

4.  Guss  DA,  Behling  CA,  Munassi  D:  Impacto  de  la  tomografía  computarizada  helicoidal  abdominal  en  la  tasa  de  resultados  negativos
Apendicitis  J  Emerg  Med  34:7­11,  2008.  Epub  26  de  diciembre  de  2007.

5.  Huynh  V,  Lalezarzadeh  F,  Lawandy  S  et  al.:  Tomografía  computarizada  abdominal  en  la  evaluación  de
apendicitis  perforada  en  el  ámbito  comunitario.  Am  Surg  73(10):1002­1005,  2007.

6.  Meakins  JL:  Apendicectomía  y  apendicitis.  Can  J  Surg  42:90,  1999.

7.  Pokala  N,  Sadhasivam  S,  Kiran  RP  et  al.:  Apendicitis  complicada:  ¿es  apropiado  el  abordaje  laparoscópico?
Un  estudio  comparativo  con  el  enfoque  abierto:  resultado  en  un  entorno  hospitalario  comunitario.  Am  Surg  73(8):  737­741;  
discusión  741­742,  2007.

8.  Rockey  AE:  Incisiones  transversales  en  operaciones  abdominales.  MedRec  68:779,  1905.

9.  Samuel  M:  puntuación  de  apendicitis  pediátrica.  J  Pediatric  Surg  37:877­881,  2002.

10.  Urbach  DR,  Cohen  MM:  ¿La  perforación  del  apéndice  es  un  factor  de  riesgo  para  la  infertilidad  tubárica  y  el  embarazo  ectópico?
Una  valoración  de  la  evidencia.  Can  J  Surg  42:101­108,  1999.
Machine Translated by Google

ENFERMEDAD  DE  LA  VESÍCULA
CAPÍTULO  
37

Dr.  Walter  L.  Biffl,  FACS

1.  ¿Cuál  es  la  prevalencia  de  cálculos  biliares  en  la  sociedad  occidental  para  mujeres  y  hombres  de  60  
años  de  edad?
Mujeres,  50%;  hombres,  15%,  aunque  existe  una  formidable  predilección  étnica  por  los  cálculos  biliares  
endémicos  en  los  indios  americanos.

2.  ¿Cuál  es  la  diferencia  entre  colelitiasis,  colecistitis,  coledocolitiasis  y  colangitis?

La  colelitiasis  se  refiere  a  la  presencia  de  cálculos  en  la  vesícula  biliar.  La  colelitiasis  sintomática  es  la  
indicación  más  común  para  la  colecistectomía.  La  colecistitis  es  una  afección  inflamatoria  de  la  vesícula  biliar,  
generalmente  iniciada  por  la  impactación  de  cálculos  biliares  en  el  cuello  de  la  vesícula  biliar  con  obstrucción  del  
conducto  cístico.  La  coledocolitiasis  es  la  presencia  de  cálculos  en  el  conducto  biliar  común  (CBD).
La  colangitis  es  una  infección  del  árbol  biliar,  generalmente  como  resultado  de  una  obstrucción,  
generalmente  secundaria  a  coledocolitiasis.

3.  ¿Qué  porcentaje  de  cálculos  biliares  asintomáticos  se  vuelve  sintomático?
Diez  por  ciento  a  los  5  años,  15%  a  los  10  años  y  18%  a  los  15  años.

4.  ¿Los  pacientes  con  cálculos  biliares  asintomáticos  deben  someterse  a  una  colecistectomía?
No.  El  riesgo  de  observación  de  pacientes  con  cálculos  biliares  asintomáticos  es  menor  o  igual  al  riesgo  de  
operación.

5.  ¿En  qué  grupos  de  pacientes  con  cálculos  biliares  asintomáticos  es  beneficiosa  la  colecistectomía  
profiláctica?
  Pacientes  con  anemia  hemolítica  congénita  que  tienen  cálculos  biliares  en  el  momento  de  la  esplenectomía.
  Pacientes  obesos  sometidos  a  cirugía  bariátrica  que  ya  han  desarrollado  cálculos  biliares.

6.  ¿Cuál  es  el  momento  óptimo  para  la  colecistectomía  laparoscópica  en  la  colecistitis  aguda?

"Enfriar  la  vesícula  biliar"  y  retrasar  la  cirugía  durante  6  semanas  se  asocia  con  colecistitis  recurrente  en  el  20%  
de  los  pacientes.  Los  estudios  aleatorios  prospectivos  han  encontrado  consistentemente  que  la  colecistectomía  
temprana  permite  estadías  hospitalarias  más  cortas,  sin  diferencias  en  la  morbilidad  o  mortalidad,  en  comparación  
con  la  colecistectomía  tardía.  Los  procedimientos  realizados  dentro  de  las  primeras  24  horas  generalmente  son  
más  fáciles  porque  el  área  de  disección  aún  no  está  inflamada  al  máximo  y  aún  no  se  ha  producido  fibrosis  ni  
aumento  de  la  proliferación  de  vasos  sanguíneos.

7.  ¿Cuál  es  la  tasa  de  conversión  de  la  laparoscopia  al  abordaje  abierto  en  la  colecistitis  aguda  y  en  la  colelitiasis  
sintomática?
Diez  a  15%  para  colecistitis  aguda  y  <5%  para  colelitiasis  sintomática.

191
Machine Translated by Google
192  CAPÍTULO  37  ENFERMEDAD  DE  LA  VESÍCULA  BILIAR

8.  ¿Cuál  es  la  incidencia  de  la  colecistitis  acalculosa?
Diez  por  ciento  de  todos  los  casos  de  colecistitis.

9.  ¿Qué  organismos  requieren  cobertura  antibiótica  en  infecciones  biliares?
Escherichia  coli,  especies  de  Klebsiella,  Streptococcus  faecalis,  Clostridium  welchii,  especies  de  Proteus,  especies  de  
Enterobacter  y  especies  de  Streptococcus  anaerobios.

10.  ¿Cuál  es  la  incidencia  de  lesión  del  colédoco  en  la  colecistectomía  abierta  y  laparoscópica?
Es  del  0,2  %  al  0,3  %  para  la  colecistectomía  abierta  y  del  0,4  %  al  0,6  %  para  la  colecistectomía  laparoscópica.

11.  ¿Cómo  se  compara  la  ecografía  intraoperatoria  laparoscópica  con  la  colangiografía  intraoperatoria  laparoscópica?

La  sensibilidad  de  la  ecografía  intraoperatoria  laparoscópica  (LUS)  es  alta  (90  %)  y  comparable  a  la  de  la  colangiografía  
intraoperatoria  laparoscópica  (LIOC).  Las  ventajas  potenciales  de  LUS  incluyen  menos  tiempo  y  menos  disección  que  
LIOC.

12.  ¿La  LUS  o  la  LIOC  previenen  las  lesiones  del  colédoco  durante  la  colecistectomía?
En  estudios  basados  en  la  población,  hay  menos  lesiones  de  CBD  entre  los  pacientes  que  se  someten  a  LIOC;  sin  
embargo,  no  hay  evidencia  de  nivel  I  de  que  el  LIOC  fuera  preventivo.  Sin  embargo,  LIOC  identifica  las  lesiones  en  
el  momento  en  que  ocurren.  Los  defensores  de  LUS  argumentan  que  identificar  la  anatomía  antes  de  seccionar  un  
conducto  (e  insertar  un  catéter  LIOC)  puede  prevenir  lesiones  en  el  CBD.

13.  ¿Qué  porcentaje  de  pacientes  sometidos  a  colecistectomía  tienen  coledocolitiasis  no  sospechada?

Sobre  2%.  Sin  embargo,  si  el  paciente  no  tiene  antecedentes  de  ictericia  y  un  colédoco  inferior  a  6  mm  de  diámetro,  el  
examen  de  laboratorio  de  rutina  no  sirve.  La  gran  mayoría  de  estas  piedras  pasarán  espontáneamente.

14.  ¿Cuándo,  si  alguna  vez,  se  debe  realizar  una  colecistectomía  laparoscópica  durante
¿el  embarazo?
La  mayoría  de  los  ataques  de  cólico  biliar  agudo  durante  el  embarazo  se  resuelven  espontáneamente.  Para  
evitar  un  aborto  inducido  quirúrgicamente,  se  debe  realizar  una  colecistectomía  después  del  parto.  Sin  embargo,  si  es  
necesaria  la  cirugía,  se  prefiere  el  segundo  trimestre  para  cualquier  intervención  quirúrgica.

15.  ¿Cuál  es  la  prevalencia  del  carcinoma  de  vesícula  biliar  encontrado  incidentalmente  durante  la  colecistectomía?

Abierto,  1%;  laparoscópica,  0,1%.

16.  ¿Por  qué  está  aumentando  la  colecistectomía  en  la  población  pediátrica?
La  identificación  de  cálculos  biliares  ha  aumentado  debido  al  uso  más  liberal  de  la  ecografía  en  pacientes  con  dolor  
abdominal.

PUNTOS  CLAVE:  ENFERMEDAD  DE  LA  VESÍCULA  BILIAR

1.  Incidencia  general  en  Estados  Unidos:  mujeres  >60  años,  50%;  hombres  >60  años,  15%.

2.  Del  quince  al  20%  de  los  pacientes  con  cálculos  biliares  se  vuelven  sintomáticos.

3.  Los  pacientes  con  colecistitis  aguda  deben  someterse  a  cirugía  lo  antes  posible  después  del  inicio  de  la
síntomas.

4.  La  LUS  de  rutina  puede  identificar  cálculos  de  CBD  y  puede  ayudar  a  evitar  lesiones  de  CBD.
Machine Translated by Google
CAPÍTULO  37  ENFERMEDAD  DE  LA  VESÍCULA  BILIAR  193

BIBLIOGRAFÍA

1.  Adamsen  S,  Hansen  OH,  Funch­Jensen  P  et  al .:  Lesión  del  conducto  biliar  durante  la  colecistectomía  laparoscópica:
Una  serie  prospectiva  a  nivel  nacional.  J  Am  Coll  Surg  184:571­578,  1997.
2.  Biffl  WL,  Moore  EE,  Offner  PJ  et  al.:  La  ultrasonografía  laparoscópica  intraoperatoria  de  rutina  con  colangiografía  
selectiva  reduce  las  complicaciones  del  conducto  biliar  durante  la  colecistectomía  laparoscópica.  J  Am  Coll  Surg  
193:272­280,  2001.

3.  Fletcher  DR,  Hobbs  MST,  Tan  P  et  al.:  Complicaciones  de  la  colecistectomía:  riesgos  del  abordaje  laparoscópico  
y  efectos  protectores  de  la  colangiografía  quirúrgica.  Un  estudio  de  base  poblacional.  Ann  Surg  229:449­457,  
1999.

4.  Ghumman  E,  Barry  M,  Grace  PA  et  al.:  Manejo  de  los  cálculos  biliares  en  el  embarazo.  Br  J  Surg  84:1646­1650,  1997.
5.  Machi  J,  Tateishi  T,  Oishi  A  et  al .:  Ultrasonografía  laparoscópica  versus  colangiografía  quirúrgica  durante  la  colecistectomía  
laparoscópica:  revisión  de  la  literatura  y  una  comparación  con  la  ultrasonografía  intraoperatoria  abierta.  J  Am  Coll  Surg  
188:361­367,  1999.
6.  Robinson  TN,  Biffl  WL,  Moore  EE  et  al.:  Los  análisis  de  laboratorio  preoperatorios  de  rutina  son  innecesarios  antes
Colecistectomía  laparoscópica  electiva.  Surg  Endosc  17:438­441,  2003.
7.  Siddiqui  T,  MacDonald  A,  Chong  PS  et  al.:  Colecistectomía  laparoscópica  temprana  versus  tardía  para
colecistitis:  un  metanálisis  de  ensayos  clínicos  aleatorios.  Am  J  Surg  195:40­47,  2008.
Machine Translated by Google

CÁNCER  DE  PÁNCREAS
CAPÍTULO  
38

Martín  D.  McCarter,  MD

1.  ¿Cuál  es  la  magnitud  del  problema?
Para  el  año  más  reciente  en  el  que  hay  estadísticas  disponibles,  hubo  aproximadamente  37  000  casos  nuevos  de  cáncer  
de  páncreas  en  los  Estados  Unidos  y  más  de  33  000  muertes,  lo  que  lo  convierte  en  uno  de  los  tumores  más  letales.  Es  la  
cuarta  causa  más  común  de  muerte  por  cáncer  en  los  Estados  Unidos  tanto  para  hombres  como  para  mujeres.

2.  ¿Cuáles  son  los  tipos  histológicos  del  cáncer  de  páncreas?
El  adenocarcinoma  es,  de  lejos,  el  tipo  más  común  (y  letal).  Los  tumores  neuroendocrinos  constituyen  aproximadamente  
el  1%  de  los  casos,  generalmente  tienen  un  curso  más  indolente.  También  pueden  ocurrir  otros  tipos  de  tumores  raros,  
como  sarcoma,  linfoma,  tumor  peusdopapilar  y  mucinoso  papilar  intraductal  (IPMN).

3.  ¿Cuáles  son  los  signos  de  presentación  del  cáncer  de  páncreas?
  Ictericia  indolora:  40%  de  los  pacientes.
&  Dolor  (epigástrico,  cuadrante  superior  derecho,  espalda)  con  ictericia:  40%.
  Enfermedad  metastásica  (p.  ej.,  hepatomegalia,  ascitis,  nódulos  pulmonares)  con  o  sin  ictericia:  20%.
La  mayoría  de  los  pacientes  también  tienen  otros  síntomas  gastrointestinales  (GI)  no  específicos,  como  distensión  
abdominal,  intolerancia  alimentaria  o  insuficiencia  pancreática  y  pérdida  de  peso.

4.  ¿Cuál  es  la  supervivencia  estimada  de  los  pacientes  con  cáncer  de  páncreas?
La  supervivencia  general  a  los  5  años  para  los  que  se  someten  a  una  resección  completa  oscila  entre  el  5  %  y  el  20  %.

Escenario En  el  Diagnóstico Supervivencia  media  estimada

resecable 10%–20% 18  meses

localmente  avanzado 40%–45% 9–12  meses

metastásico 40%–45% 6–9  meses

5.  ¿Por  qué  hay  una  tasa  tan  alta  de  enfermedad  avanzada  en  el  momento  del  diagnóstico?
El  páncreas  es  retroperitoneal,  relativamente  insensible,  y  los  síntomas  de  la  enfermedad  no  se  manifiestan  hasta  que  se  
forma  una  obstrucción  local  del  conducto  duodenal,  pancreático  o  biliar.  Alrededor  del  80%  surgen  en  la  cabeza  de  la  
glándula,  el  10%  surgen  en  el  cuerpo  y  el  10%  en  la  cola.

6.  Un  paciente  de  73  años  previamente  sano  presenta  prurito,  orina  oscura  y  esclerótica  ictérica  después  de  un  viaje  reciente  al  
extranjero.  ¿Qué  es  un  diagnóstico  diferencial  razonable?
1.  Cálculos  biliares.

2.  Cáncer  de  las  vías  biliares  extrahepáticas.
3.  Cáncer  de  páncreas.
4.  hepatitis.

194
Machine Translated by Google
CAPÍTULO  38  CÁNCER  DE  PÁNCREAS  195

7.  ¿Cuál  es  el  primer  paso  para  evaluar  al  paciente?
El  primer  paso  son  las  pruebas  de  función  hepática  (LFT)  para  determinar  el  grado  de  ictericia  y  disfunción  
hepática.  Luego  se  realiza  una  ecografía  (US)  para  determinar  si  la  causa  es  intrahepática  (vías  biliares  
normales)  o  extrahepática  (vías  biliares  dilatadas).  La  ecografía  puede  detectar  cálculos  en  la  vesícula  biliar  o  en  el  
conducto  común  con  un  95  %  de  precisión.  Por  lo  tanto,  si  un  paciente  con  ictericia  tiene  conductos  biliares  
normales  en  la  ecografía,  el  problema  es  una  colestasis  intrahepática,  probablemente  por  hepatitis.

8.  ¿Qué  pasa  si  una  ecografía  muestra  conductos  biliares  extrahepáticos  dilatados?
Proceder  a  colangiopancreatografía  retrógrada  endoscópica  (CPRE)  o  colangiograma  transhepático  
para  determinar  si  la  obstrucción  es  alta  o  baja  en  el  conducto  biliar  común  (CBD)  y  para  determinar  su  causa  
probable  (estenosis,  cálculo,  tumor).  El  tracto  biliar  se  puede  descomprimir  con  un  stent  interno  en  este  momento,  
lo  que  permite  que  la  función  hepática  mejore  antes  de  una  cirugía  mayor.  Si  hay  cálculos,  se  debe  realizar  una  
esfinterotomía  endoscópica,  lo  que  permite  el  paso  de  los  cálculos  y  simplifica  la  cirugía  futura.

9.  ¿Qué  otras  imágenes,  si  las  hay,  están  indicadas?
En  general,  el  estudio  más  útil  es  una  tomografía  computarizada  (TC)  multifásica  de  corte  fino  del  abdomen.  Esto  
proporciona  información  sobre  la  posible  enfermedad  metastásica  y  ayuda  a  evaluar  la  afectación  vascular  local  
(vena  porta,  vena  mesentérica  superior,  eje  celíaco,  arteria  hepática,  arteria  mesentérica  superior).  La  ecografía  
endoscópica  (EUS)  puede  ser  útil  si  la  masa  no  está  clara  y  se  necesita  un  diagnóstico  de  tejido.  La  tomografía  por  
emisión  de  positrones  (PET)  o  las  tomografías  computarizadas  a  menudo  se  obtienen  para  ayudar  a  descartar  la  
enfermedad  metastásica,  pero  su  beneficio  adicional  (algunos  estudios  sugieren  nuevos  hallazgos  en  el  20%  de  
los  pacientes)  aún  se  debate.  Asimismo,  a  medida  que  ha  mejorado  la  imaginología,  ha  disminuido  el  papel  de  la  
laparoscopia  de  rutina  para  evaluar  la  resecabilidad.

10.  ¿Cuál  es  el  significado  de  un  letrero  de  ''doble  conducto''?
Esto  se  refiere  a  la  presencia  de  un  sistema  pancreático  y  de  conductos  biliares  dilatados  identificados  en  una  
tomografía  computarizada  o  una  CPRE.  En  ausencia  de  un  cálculo  biliar,  esto  casi  siempre  implica  la  presencia  de  
un  cáncer  subyacente  como  etiología.

11.  ¿Qué  demonios  es  CA19­9?
CA  19­9  significa  antígeno  carbohidrato  19­9.  Es  un  marcador  tumoral  asociado  a  tumores  pancreáticos  y  biliares,  
que  se  mide  en  el  suero  del  paciente.  No  es  específico  (puede  estar  elevado  en  la  inflamación  y  otras  afecciones  
benignas),  pero  puede  ser  útil  para  monitorear  el  progreso/respuesta  de  un  paciente  a  la  terapia.

12.  En  este  caso,  la  ecografía,  la  CPRE  y  la  tomografía  computarizada  muestran  conductos  biliares  extrahepáticos  
dilatados,  una  masa  en  la  cabeza  del  páncreas  y  ninguna  otra  causa  evidente  aparte  del  cáncer.  El  tumor  
parece  separado  de  la  vena  porta  y  no  hay  metástasis  hepáticas.  ¿Qué  se  debe  hacer  a  continuación?

Realizar  una  evaluación  del  riesgo  operativo.  Si  el  paciente  tiene  un  riesgo  quirúrgico  bajo,  se  debe  considerar  la  
aspiración  con  aguja  fina  (PAAF)  percutánea  o  endoscópica  guiada  por  US  para  documentar  el  cáncer,  si  es  
posible,  y  la  colocación  de  un  stent  endoscópico  en  el  conducto  biliar;  la  cirugía  probablemente  no  sea  una  buena  
opción.  Si  el  paciente  tiene  un  buen  riesgo  operatorio,  el  siguiente  paso  es  la  cirugía.  El  cuadro  clínico  es  preciso  
en  al  menos  el  90%  de  los  casos,  y  la  FNA  no  agrega  información  útil  en  este  momento.  Si  no  se  obtiene  tejido  
maligno,  la  cirugía  aún  está  indicada  porque  la  aguja  puede  haber  pasado  por  alto  la  lesión,  muestreando  solo  la  
pancreatitis  que  rodea  a  todos  esos  tumores.

13.  Estamos  en  la  sala  de  operaciones,  el  abdomen  está  abierto  y  la  discusión  gira  en  torno  a  la  extracción  del  
tumor.  ¿Qué  es  un  procedimiento  de  Whipple?
La  pancreaticoduodenectomía  implica  la  extirpación  de  la  vesícula  biliar,  el  conducto  común  distal,  el  
duodeno,  el  antro  gástrico  y  la  porción  del  páncreas  a  la  derecha  de  la  vena  porta;  en  esencia,  una  
pancreatectomía  proximal.
Machine Translated by Google
196  CAPÍTULO  38  CÁNCER  DE  PÁNCREAS

14.  ¿Qué  es  la  pancreatectomía  distal?  ¿Una  pancreatectomía  total?
La  pancreatectomía  distal  extirpa  la  porción  de  glándula  a  la  izquierda  de  la  vena  porta,  junto  con  el  bazo.  La  
pancreatectomía  total  combina  ambos  procedimientos,  nuevamente,  con  antrectomía  en  algunos  centros.

15.  ¿Por  qué  extirpar  la  vesícula  biliar,  el  duodeno  y  el  estómago  si  el  problema  está  en  el
¿páncreas?
Después  de  que  se  extrae  la  ampolla  de  Vater,  la  vesícula  biliar  no  funciona  bien  y  se  forman  cálculos  biliares.  
Las  porciones  segunda  y  tercera  del  duodeno  comparten  un  suministro  de  sangre  con  la  cabeza  del  páncreas  y  
generalmente  se  desvascularizan  cuando  se  extrae  la  cabeza.  Históricamente,  se  extirpaba  el  antro  gástrico  para  
mejorar  los  márgenes  de  resección.  Se  agregó  vagotomía  para  reducir  la  incidencia  de  ulceración  marginal  en  la  
anastomosis  entre  el  remanente  gástrico  y  el  yeyuno.
Sin  embargo,  la  extirpación  del  antro  agrega  poco  al  alcance  de  la  operación,  y  la  ulceración  marginal  se  
puede  prevenir  colocando  la  gastroyeyunostomía  aguas  abajo  de  donde  las  secreciones  biliares  y  pancreáticas  
ingresan  al  intestino.  Por  lo  tanto,  muchos  cirujanos  ahora  realizan  un  procedimiento  de  Whipple  para  preservar  el  
píloro  siempre  que  sea  posible,  preservando  también  el  nervio  vago.  Un  «Whipple»  con  preservación  del  píloro  tiene  
la  misma  supervivencia  general,  se  asocia  con  una  reducción  del  tiempo  operatorio  y  pérdida  de  sangre,  pero  ha  sido  
difícil  probar  algún  beneficio  fisiológico.

16.  ¿Cómo  se  determina  si  realizar  un  procedimiento  de  Whipple,  una  pancreatectomía  distal  o  una  pancreatectomía  
total?  ¿Cuál  es  la  tasa  de  curación?
Los  procedimientos  de  Whipple  se  usan  para  tumores  móviles  en  la  cabeza  sin  signos  de  metástasis  en  los  
ganglios  linfáticos  en  el  eje  celíaco  o  la  raíz  del  mesenterio.  La  pancreatectomía  distal  se  usa  para  lesiones  del  
cuerpo  y  la  cola  que  no  se  acompañan  de  signos  de  diseminación.  La  pancreatectomía  total  generalmente  se  
reserva  para  unas  pocas  situaciones  raras  en  las  que  un  cáncer  difuso  afecta  a  la  mayor  parte  de  la  glándula  pero  
no  a  ninguna  otra  parte;  este  es  un  evento  raro.  La  mediana  de  supervivencia  con  cada  procedimiento  es  de  unos  20  
meses  y  la  supervivencia  a  los  5  años  es  de  alrededor  del  15  %.  Este  procedimiento  tiene  alrededor  de  1%  a  3%  de  
mortalidad  operatoria  y  25%  a  40%  de  morbilidad  en  centros  con  amplia  experiencia;  en  otros  entornos,  el  riesgo  
operatorio  y  la  tasa  de  complicaciones  pueden  ser  mucho  mayores.

17.  Qué  se  debe  hacer  si  hay  metástasis  ganglionares  en  el  eje  celíaco
o  la  raíz  del  mesenterio?
El  paciente  no  se  puede  curar  con  cirugía,  por  lo  que  el  objetivo  es  la  paliación.  Si  hay  ictericia  obstructiva,  se  debe  
realizar  un  bypass  bilioentérico.  Si  un  tumor  obstruye  el  duodeno,  también  se  debe  realizar  una  gastroenterostomía.  
Algunos  cirujanos  creen  que  la  gastroenterostomía  debe  realizarse  de  forma  rutinaria  para  los  cánceres  de  la  cabeza  
del  páncreas,  independientemente  de  si  hay  compromiso  duodenal,  porque  hasta  el  20  %  de  los  pacientes  sin  este  
problema  en  el  momento  de  la  cirugía  pueden  necesitar  una  intervención  por  vaciado  gástrico  deficiente  en  el  futuro.

18.  ¿Existen  otros  signos  de  inoperabilidad?
Las  contraindicaciones  generales  para  la  resección  incluyen  enfermedad  metastásica,  invasión  de  la  vena  cava  inferior  
(VCI)  o  arterias  locales  principales  (eje  celíaco,  arteria  hepática,  arteria  mesentérica  superior).
Las  contraindicaciones  relativas  a  la  resección  incluyen  la  invasión  de  la  vena  porta  o  mesentérica  superior.  La  
resección  de  la  vena  porta  y  la  reconstrucción  se  pueden  realizar  con  menos  morbilidad  que  en  el  pasado,  pero  este  
ejercicio  técnico  no  ha  mejorado  la  supervivencia.

19.  Se  encuentra  que  un  paciente  tiene  una  diseminación  insospechada  al  eje  celíaco.  Realizas  un  bypass  biliar  y  gástrico.  
¿Hay  algo  más  que  pueda  ofrecerle  al  paciente,  ya  sea  quirúrgicamente  o  no  quirúrgicamente?

Algunos  de  estos  pacientes,  si  sufren  de  dolor  de  espalda  preoperatorio,  pueden  aliviarse  mediante  el  bloqueo  
intraoperatorio  del  ganglio  celíaco  con  alcohol.  Alternativamente,  tal  tratamiento  puede  ser  realizado  
postoperatoriamente  por  un  radiólogo  intervencionista.  La  quimioterapia  paliativa  y  la  radioterapia  también  pueden  
aliviar  el  dolor,  pero  por  lo  general  no  mejoran  la  supervivencia.
Machine Translated by Google
CAPÍTULO  38  CÁNCER  DE  PÁNCREAS  197

20.  ¿Existen  otros  tratamientos  (quimioterapia,  radioterapia,  terapia  con  mascotas)
que  mejoran  los  resultados  en  el  cáncer  de  páncreas?
Aún  no.  Esto  no  es  como  resultado  de  una  falta  de  interés  o  esfuerzo.  Hay  alguna  evidencia  de  que  la  quimioterapia  
(gemcitabina)  o  una  combinación  de  quimioterapia,  radioterapia  e  inmunoterapia  pueden  agregar  algunos  meses  a  la  
supervivencia  general  después  de  la  resección  (con  algún  costo  para  la  calidad  de  vida);  sin  embargo,  la  gran  mayoría  de  los  
ensayos  no  lograron  establecer  una  diferencia  significativa  en  la  supervivencia  global.  ¡Esta  es  una  oportunidad  genuina  para  
que  estudiantes  brillantes  como  el  presente  lector  marquen  la  diferencia!

21.  Con  una  alta  morbilidad  y  bajas  tasas  de  curación,  ¿por  qué  los  cirujanos  están  tan  ansiosos  por  realizar  procedimientos  
de  Whipple?
Desafortunadamente,  esto  representa  la  única  posibilidad  de  curación.  Además,  la  resección  pancreática,  cuando  se  lleva  a  cabo  
de  manera  segura,  probablemente  ofrezca  el  mejor  paliador  a  largo  plazo  en  aquellos  destinados  a  morir  por  su  enfermedad.
Finalmente,  los  avances  futuros  en  la  terapia  adyuvante  pueden  brindar  esperanza  de  mejora  en  la  supervivencia  general.

PUNTOS  CLAVE:  EVALUACIÓN  DIAGNÓSTICA  DE  UN  PACIENTE
CON  ICTERICIA

1.  LFT:  determinar  el  grado  de  ictericia  (obstructiva  versus  no  obstructiva)  y  hepática
disfunción.

2.  Ecografía  del  cuadrante  superior  derecho:  descarta  cálculos  biliares,  evalúa  intrahepáticos  versus  extrahepáticos
dilatación  ductal.

3.  Si  los  conductos  hepáticos  están  dilatados:  CPRE  o  colangiografía  transhepática  percutánea  para  delinear  el  
sitio  de  la  obstrucción  mecánica.

4.  TC:  evalúa  el  tamaño  del  tumor,  si  está  presente,  el  grado  de  diseminación  regional  o  las  metástasis  hepáticas.

SITIOS  WEB

www.nccn.org/

www.cancer.org

BIBLIOGRAFÍA

1.  Diener  MK,  Knaebel  HP,  Heukaufer  C  et  al.:  Una  revisión  sistemática  y  metanálisis  de  la  preservación  del  píloro
versus  duodenopancreatectomía  clásica  para  el  tratamiento  quirúrgico  del  carcinoma  pancreático  y  periampular.
Ann  Surg  245:187­200,  2007.

2.  Garofalo  M,  Flannery  T,  Regine  W:  El  caso  de  la  quimiorradiación  adyuvante  para  el  cáncer  de  páncreas.  Res.  de  mejores  prácticas
Clin  Gastroenterol  20:403­416,  2006.

3.  Goldin  SB,  Bradner  MW,  Zervos  EE  et  al.:  Evaluación  de  las  neoplasias  pancreáticas:  revisión  de  las  técnicas  de  biopsia.
J  Gastrointest  Surg  11:783­790,  2007.

4.  Jemal  A,  Siegel  R,  Ward  E  et  al.:  Estadísticas  de  cáncer,  2007.  CA  Cancer  J  Clin  57(1):43­66,  2007.

5.  Kalra  MK,  Maher  MM,  Mueller  PR  et  al.:  Imágenes  de  vanguardia  de  las  neoplasias  pancreáticas.  Br  J  Radiol  76
(912):857­865,  2003.
Machine Translated by Google
198  CAPÍTULO  38  CÁNCER  DE  PÁNCREAS

6.  Ryan  DP,  Willett  CG:  Manejo  del  adenocarcinoma  de  páncreas  localmente  avanzado.  Hematol  Oncol  Clínica
América  del  Norte  16:95­103,  2002.

7.  Siriwardana  HP,  Siriwardena  AK:  Revisión  sistemática  del  resultado  de  la  resección  sincrónica  de  la  vena  mesentérica  portal  superior  durante  
la  pancreatectomía  por  cáncer.  Br  J  Surg  93:662­673,  2006.

8.  Sohn  TA,  Lillemoe  KD:  paliación  quirúrgica  del  cáncer  de  páncreas.  Adv  Surg  34:249­271,  2000.

9.  Stojadinovic  A,  Hoos  A,  Brennan  MF  et  al.:  Ensayos  clínicos  aleatorizados  en  cáncer  de  páncreas.  Surg  Oncol  Clin  N  Am
11:207­229,  2002.

10.  Ujiki  MB,  Talamonti  MS.:  Directrices  para  el  tratamiento  quirúrgico  del  adenocarcinoma  de  páncreas.  Semin  Oncol,
34:311­320,  2007.

11.  Varadhachary  GR,  Tamm  EP,  Abbruzzese  JL  et  al .:  Cáncer  de  páncreas  resecable  límite:  definiciones,  manejo  y  papel  de  la  terapia  
preoperatoria.  Ann  Surg  Oncol  13:1035­1046,  2006.
Machine Translated by Google

PANCREATITIS  AGUDA
CAPÍTULO  
39

Adam  H.  Lackey,  MD  y  C.  Clay  Cothren,  MD

1.  ¿Cuáles  son  las  causas  comunes  y  la  incidencia  de  la  pancreatitis  aguda?
Cálculos  biliares  (45%),  alcohol  (35%)  y  otros  (20%).  Se  estima  que  la  pancreatitis  aguda  es  responsable  de  
210  000  ingresos  hospitalarios  en  los  Estados  Unidos  por  año.

2.  ¿Cuáles  son  las  causas  poco  frecuentes?

Hiperlipidemia,  hipercalcemia  (hiperparatiroidismo,  mieloma  múltiple),  factores  iatrogénicos  (hasta  el  15%  de  los  
pacientes  sometidos  a  colangiopancreatografía  retrógrada  endoscópica  [CPRE]  pueden  experimentar  pancreatitis),  
fármacos  (didanosina,  diuréticos  tiazídicos,  bloqueadores  H2,  tetraciclina,  azatioprina,  salicilatos,  ácido  valproico) ,  
infecciones  (paperas,  coxsackievirus),  páncreas  divisum  y  picaduras  de  escorpión,  y  pancreatitis  autoinmune.  
Aproximadamente  el  10%  de  los  casos  se  consideran  verdaderamente  idiopáticos.

3.  ¿Cuáles  son  los  síntomas  característicos?
Inicio  agudo  de  dolor  epigástrico  severo  que  es  de  naturaleza  aburrida  y  que  a  menudo  se  irradia  a  la  espalda.  El  dolor  
se  acompaña  frecuentemente  de  náuseas  y  vómitos.

4.  ¿Qué  se  puede  encontrar  en  el  examen  físico?
Sensibilidad  abdominal  difusa,  distensión  abdominal,  defensa  abdominal  "en  forma  de  tabla"  y  ruidos  intestinales  
hipoactivos.  Los  pacientes  pueden  estar  febriles,  taquicárdicos  y  deshidratados.  La  evidencia  de  ictericia  o  la  
identificación  de  cálculos  biliares  en  la  ecografía  (US)  del  cuadrante  superior  derecho  indica  una  causa  biliar  de  
pancreatitis.  La  pancreatitis  grave  puede  provocar  hemorragia  retroperitoneal,  lo  que  lleva  a  una  decoloración  
periumbilical  o  del  flanco  (denominada  signo  de  Cullen  y  signo  de  Gray  Turner,  respectivamente).

5.  ¿Cuál  es  el  tratamiento  adecuado  para  la  pancreatitis  leve  a  moderada?
El  componente  crítico  de  la  terapia  de  apoyo  es  la  reanimación  adecuada  con  líquidos  para  mantener  la  
diuresis  (colocar  una  sonda  de  Foley);  los  pacientes  pueden  requerir  hasta  10  L  de  cristaloides  como  resultado  de  la  
gran  cantidad  de  tercer  espacio  en  el  abdomen.  El  tratamiento  apropiado  también  incluye  analgésicos,  profilaxis  de  
abstinencia  de  alcohol,  descompresión  nasogástrica  (NG)  para  emesis  persistente  y  evitar  la  ingesta  oral.  Los  pacientes  
comienzan  con  una  dieta  regular  una  vez  que  sus  marcadores  de  laboratorio  están  disminuyendo  o  son  normales,  su  
dolor  abdominal  se  está  resolviendo  y  tienen  ruidos  intestinales.

6.  ¿Cuál  es  mejor  prueba  de  laboratorio,  la  amilasa  o  la  lipasa?
Los  niveles  de  amilasa  sérica  tienden  a  alcanzar  su  punto  máximo  antes  que  los  niveles  de  lipasa,  que  pueden  
permanecer  elevados  durante  4  a  5  días.  Hasta  el  30%  de  los  pacientes  tienen  niveles  normales  de  amilasa,  sobre  
todo  los  alcohólicos  con  pancreatitis  crónica  "quemada".  La  lipasa  sérica  tiene  una  mayor  sensibilidad  y  especificidad  
que  los  niveles  de  amilasa,  siendo  3  veces  el  límite  normal  diagnóstico  de  pancreatitis.

7.  ¿Qué  otros  estados  de  enfermedad  causan  hiperamilasemia?
Las  úlceras  pépticas  perforadas,  la  obstrucción  del  intestino  delgado,  la  inflamación  o  el  tumor  de  la  glándula  parótida,  
la  insuficiencia  renal  y  los  tumores  de  ovario  se  asocian  con  niveles  elevados  de  amilasa.

8.  ¿Cuál  es  la  importancia  de  la  hipoxemia  en  las  primeras  etapas  de  la  evolución  de  la  pancreatitis?
Los  pacientes  con  pancreatitis  necrosante  pueden  desarrollar  insuficiencia  respiratoria  que  requiera  ventilación  
mecánica,  que  puede  progresar  a  insuficiencia  orgánica  múltiple  (MOF).  La  hipoxemia  es  un  signo  ominoso,  al  igual  
que  los  infiltrados  en  la  radiografía  de  tórax  de  admisión.

199
Machine Translated by Google
200  CAPÍTULO  39  PANCREATITIS  AGUDA

9.  ¿Qué  es  la  puntuación  de  Ranson?
Los  índices  de  Ranson  son  11  medidas  que  son  útiles  para  indicar  la  gravedad  y  predecir  el  resultado  de  la  
pancreatitis.  Una  puntuación  de  3  o  más  se  considera  enfermedad  grave.
Al  ingreso Después  de  las  48  horas  
Edad  >55  años iniciales  Aumento  del  nitrógeno  ureico  en  sangre  (BUN)  
Recuento  de  glóbulos  blancos  >16.000  ml >5  mg/dl  Disminución  del  nivel  de  hematocrito  >10%  
Glucosa  >350  mg/dl Calcio  <8  mg/dl  Presión  parcial  de  oxígeno  (PaO2)  <60  
Lactato  deshidrogenasa  >350  U/L mm  Hg  Déficit  de  bases  >4  mmol/L
Aspartato  aminotransferasa  (AST)  >  250  
m/L
Secuestro  de  fluidos  >6  L

10.  ¿Cómo  se  relacionan  los  índices  de  Ranson  con  la  mortalidad?
Número  de  criterios  Tasa  de  mortalidad  (%)  0–2  5  3–4  
15  5–6  50  7–8  100

11.  ¿Cuál  es  la  limitación  de  usar  la  puntuación  de  Ranson  para  predecir  la  gravedad?
de  pancreatitis?
No  se  puede  calcular  una  puntuación  de  Ranson  verdadera  hasta  después  de  48  horas  de  tratamiento  hospitalario.  
Otros  índices  predictivos  incluyen  el  sistema  de  puntuación  de  fisiología  aguda  y  evaluación  crónica  de  la  salud  II  
(APACHE  II)  y  el  sistema  de  puntuación  de  Balthazar.  El  sistema  de  puntuación  APACHE  II  logra  valores  predictivos  
positivos  y  negativos  comparables  a  los  criterios  de  Ranson  para  predecir  la  gravedad  y  la  mortalidad  esperada  de  la  
pancreatitis  aguda.  La  puntuación  de  Balthazar,  basada  en  los  hallazgos  de  la  tomografía  computarizada  (TC)  (cantidad  
de  inflamación,  presencia  y  extensión  de  la  necrosis  y  presencia  de  colecciones  líquidas),  también  es  aceptable  para  
la  evaluación  de  la  morbilidad  y  mortalidad  esperadas.

12.  ¿Qué  es  la  pancreatitis  necrosante?
La  inflamación  y  el  edema  de  la  pancreatitis  aguda  pueden  progresar  con  la  subsiguiente  desvitalización  del  tejido  
pancreático  y  peripancreático.  La  necrosis  pancreática  ocurre  en  aproximadamente  el  20%  de  los  episodios  agudos.

13.  ¿Por  qué  es  importante  diferenciar  la  pancreatitis  aguda  de  la  pancreatitis  necrosante?

La  presencia  y  extensión  de  la  necrosis  son  determinantes  clave  del  curso  clínico.  Aproximadamente  del  10%  al  20%  
de  los  pacientes  con  necrosis  pancreática  desarrollan  necrosis  pancreática  infectada.
La  infección  representa  el  80%  de  todas  las  muertes  por  pancreatitis,  y  el  desbridamiento  quirúrgico  es  el  estándar  
actual  de  atención.  Aunque  existe  interés  en  el  tratamiento  conservador  de  la  necrosis  pancreática  infectada  
comprobada  por  biopsia,  ningún  estudio  ha  demostrado  que  sea  una  estrategia  clínica  segura.

14.  ¿Cuál  es  el  método  óptimo  para  el  diagnóstico  de  necrosis  pancreática  con  o  sin  infección  asociada?

Las  tomografías  computarizadas  dinámicas  con  contraste  intravenoso  permiten  la  visualización  y  la  diferenciación  
del  parénquima  sano  perfundido  del  tejido  necrótico  en  parches  mal  perfundido.  La  sensibilidad  de  las  imágenes  por  
TC  aumenta  en  los  días  posteriores  a  la  presentación,  y  la  identificación  óptima  de  la  necrosis  ocurre  >4  días.  Por  lo  
tanto,  se  debe  obtener  una  tomografía  computarizada  en  pacientes  que  no  mejoran  clínicamente  en  respuesta  a  la  
reanimación  con  líquidos  y  al  tratamiento  de  apoyo.  Se  puede  realizar  una  aspiración  guiada  por  TC  del  tejido  necrótico  
para  determinar  la  presencia  de  infección.

15.  ¿Cuándo  está  indicada  la  cirugía  en  pacientes  con  pancreatitis  aguda?
La  necrosis  pancreática  infectada  es  la  única  indicación  absoluta  para  la  cirugía.  El  drenaje  abierto  se  logra  mejor  a  
través  de  una  incisión  subcostal  bilateral,  la  colocación  del  epiplón  mayor  sobre  el
Machine Translated by Google
CAPÍTULO  39  PANCREATITIS  AGUDA  201

colon  transverso  para  prevenir  fístulas  entéricas  y  extracción  de  material  necrótico  del  saco  menor.
El  paciente  puede  requerir  múltiples  viajes  al  quirófano  (OR)  para  el  desbridamiento  repetido;  por  lo  general,  el  abdomen  
no  se  cierra  formalmente  hasta  que  solo  queda  tejido  viable.  La  intervención  quirúrgica  para  la  necrosis  pancreática  estéril  
es  controvertida.  Las  únicas  indicaciones  absolutas  para  la  cirugía  en  la  necrosis  pancreática  estéril  son  1)  síndrome  
compartimental  abdominal,  2)  sospecha  de  perforación  entérica  o  3)  hemorragia  (los  pseudoaneurismas  de  la  arteria  
esplénica  pueden  complicar  la  enfermedad).  Aunque  existe  una  alta  incidencia  de  infección  en  pacientes  con  >50%  de  
necrosis,  el  desbridamiento  «preventivo»  se  asocia  con  una  alta  morbilidad  y  mortalidad.

16.  ¿Cuándo  se  debe  agregar  la  terapia  con  antibióticos?
Los  antibióticos  no  alteran  el  curso  de  la  pancreatitis  ni  disminuyen  las  complicaciones  sépticas  de  la  enfermedad;  por  
lo  tanto,  los  pacientes  con  casos  leves  de  pancreatitis  deben  tratarse  con  medidas  de  soporte.  La  literatura  es  confusa  con  
respecto  al  uso  de  antibióticos  en  pacientes  con  pancreatitis  necrotizante.  A  pesar  de  la  escasez  de  evidencia  de  clase  I  
que  apoye  el  uso  de  antibióticos  profilácticos,  muchas  instituciones  continúan  administrándolos.  En  pacientes  con  signos  de  
sepsis,  el  tratamiento  con  antibióticos  empíricos  es  razonable  mientras  se  busca  una  fuente  de  infección  (es  decir,  aspiración  
de  necrosis  pancreática  guiada  por  TC).

17.  ¿Cuál  es  la  complicación  más  frecuente  de  la  pancreatitis  aguda?
Pseudoquistes  pancreáticos.  Los  pacientes  con  seudoquistes  suelen  presentar  dolor  abdominal  persistente,  
náuseas  y  vómitos  o  una  masa  abdominal.  La  tomografía  computarizada  es  diagnóstica.
La  intervención  está  indicada  en  pacientes  sintomáticos  o  en  aquellos  pacientes  asintomáticos  con  aumento  
documentado  del  tamaño  del  seudoquiste  mediante  tomografías  computarizadas  seriadas.  El  drenaje  transmural  quirúrgico  
(quiste­gastrostomía  o  quiste­yeyunostomía)  o  endoscópico  se  puede  realizar  de  6  a  12  semanas  después  de  la  presentación  
aguda,  una  vez  que  el  seudoquiste  es  "maduro".

18.  ¿Cuál  es  la  evolución  natural  de  la  colelitiasis  después  de  pancreatitis  por  cálculos  biliares?
La  colecistectomía  es  curativa  y  debe  realizarse  antes  del  alta  del  paciente  en  el  80%  de  los  pacientes  con  pancreatitis  
aguda  edematosa  no  complicada.  La  colecistectomía  se  realiza  cuando  el  dolor  del  paciente  se  resuelve,  los  signos  vitales  
se  han  normalizado  y  los  marcadores  de  laboratorio  están  disminuyendo.  La  colecistectomía  debe  acompañarse  de  una  
colangiografía  intraoperatoria  o  una  ecografía  laparoscópica;  si  se  observa  un  cálculo  retenido  en  el  conducto  biliar  común  
(CBD),  se  debe  realizar  una  exploración  laparoscópica  del  conducto  común  o  una  CPRE  antes  del  alta.

19.  ¿Cuál  es  la  evolución  natural  de  la  pancreatitis  alcohólica?
Los  ataques  se  repiten.  Debe  alentarse  la  abstinencia  del  alcohol  porque  muchos  pacientes  desarrollan  pancreatitis  crónica.

PUNTOS  CLAVE

1.  Causas:  cálculos  biliares  (45%),  alcohol  (35%),  otras  (10%),  idiopáticas  (10%).

2.  Síntomas:  aparición  aguda  de  dolor  epigástrico  que  se  irradia  a  la  espalda  con  náuseas  o
emesis

3.  Pruebas  de  laboratorio:  amilasa  y/o  lipasa  elevada  (más  sensible).

4.  Imágenes:  la  tomografía  computarizada  diagnostica  necrosis  pancreática,  colecciones  de  líquido  peripancreático  y
pseudoquistes.

5.  Tratamiento:  reanimación  con  cristaloides,  reposo  intestinal  y  descompresión  NG  (si  persiste  la  emesis);  El  10%  
de  los  casos  progresan  a  pancreatitis  necrosante,  que  requiere  desbridamiento  quirúrgico  si  se  infecta.
Machine Translated by Google
202  CAPÍTULO  39  PANCREATITIS  AGUDA

BIBLIOGRAFÍA

1.  Banks  PA,  Freeman  ML:  Pautas  de  práctica  en  pancreatitis  aguda.  Am  J  Gastroenterología  101:2379­2400,  2006.

2.  Dellinger  EP,  Tellado  JM,  Soto  NE  et  al.:  Tratamiento  antibiótico  temprano  para  la  pancreatitis  necrotizante  aguda  grave.
Un  estudio  aleatorizado,  doble  ciego,  controlado  con  placebo.  Ann  Surg  245:674­683,  2007.

3.  Gotzinger  P,  Sautner  T,  Kriwanek  S  et  al .:  Tratamiento  quirúrgico  para  la  pancreatitis  aguda  grave:  la  extensión  y  el  control  quirúrgico  de  
la  necrosis  determinan  el  resultado.  World  J  Surg  26:474­478,  2002.

4.  LeMee  J,  Paye  F,  Sauvanet  A  et  al .:  Incidencia  y  reversibilidad  de  la  falla  orgánica  en  el  curso  de  estéril  o  infectado
pancreatitis  necrotizante.  Arch  Surg  136:1386­1390,  2001.

5.  Morton  JM,  Brown  A,  Galanko  JA  et  al .:  Una  comparación  nacional  de  drenaje  quirúrgico  versus  percutáneo  de  pseudoquistes  
pancreáticos:  1997–2001.  J  Gastrointest  Surg  9:15,  2005.

6.  Nealon  WH,  Bawduniak  J,  Walser  EM:  Momento  adecuado  de  la  colecistectomía  en  pacientes  que  presentan  pancreatitis  aguda  
moderada  a  grave  asociada  con  cálculos  biliares  con  colecciones  de  líquido  peripancreático.  Ann  Surg  239:741,  2004.

7.  Rosing  DK,  de  Virgilio  C,  Yaghoubian  A  et  al.:  colecistectomía  temprana  para  la  pancreatitis  por  cálculos  biliares  de  leve  a  moderada
acorta  la  estancia  hospitalaria.  J  Am  Coll  Surg  205:762­766,  2007.

8.  Uhl  W,  Warshaw  A,  Imrie  C  et  al .:  Pautas  de  IAP  para  el  manejo  quirúrgico  de  la  pancreatitis  aguda.
Pancreatología  2:565,  2002.

9.  Vriens  PW,  van  de  Linde  P,  Slotema  E  et  al .:  El  índice  de  gravedad  de  la  tomografía  computarizada  es  una  herramienta  de  pronóstico  
temprano  para  la  pancreatitis  aguda.  J  Am  Coll  Surg  201:497­502,  2005.

10.  Whitcomb  DC:  Práctica  clínica.  Pancreatitis  aguda.  N  Engl  J  Med  354(20):2142­2150,  2006.
Machine Translated by Google

DIAGNÓSTICO  Y  TERAPIA  DE  LAS  CRÓNICAS
PANCREATITIS
CAPÍTULO  
40

Adam  H.  Lackey,  MD  y  C.  Clay  Cothren,  MD

1.  ¿Qué  es  la  pancreatitis  crónica?
El  síndrome  clásico  consiste  en  dolor  abdominal  latente  y  evidencia  de  insuficiencia  
pancreática.  Histológicamente,  la  inflamación  crónica  da  como  resultado  la  destrucción  de  las  células  
pancreáticas  endocrinas  y  exocrinas  en  funcionamiento.

2.  ¿Cuál  es  la  causa  más  común?
El  abuso  de  alcohol  representa  más  del  70%  de  los  casos.  Otras  causas  conocidas  incluyen  estenosis  
postraumáticas,  páncreas  divisum,  mutaciones  genéticas,  trastornos  autoinmunes  y  trastornos  metabólicos  
(hipertrigliceridemia  e  hipercalcemia).  Se  estima  que  la  incidencia  general  es  de  3  a  10  por  cada  100.000  
personas.

3.  ¿La  pancreatitis  crónica  es  el  resultado  de  una  pancreatitis  aguda?
Es  posible  que  los  pacientes  no  hayan  tenido  pancreatitis  aguda,  aunque  el  alcoholismo  es  común  en  ambos.
Una  hipótesis  es  que  la  inflamación  de  episodios  recurrentes  de  pancreatitis  aguda  causa  fibrosis  acinar  
intersticial  con  dilatación  secundaria  del  conducto  pancreático  principal.  Paradójicamente,  la  edad  promedio  para  
la  pancreatitis  crónica  es  13  años  menor  que  para  la  enfermedad  aguda.

4.  ¿Cuáles  son  los  signos  de  insuficiencia  pancreática?
Diabetes  mellitus  insulinodependiente  (encontrada  hasta  en  el  30%  de  los  pacientes)  y  esteatorrea  (en  el  
25%).  La  forma  de  diabetes  asociada  con  pancreatitis  crónica  se  denomina  IIIc;  puede  ser  particularmente  
difícil  de  manejar  debido  a  la  destrucción  de  las  células  productoras  de  insulina  y  glucagón.

5.  ¿Qué  parte  del  páncreas  debe  destruirse  antes  de  que  se  desarrolle  la  diabetes?
Aproximadamente  el  90%.

6.  ¿Qué  es  la  esteatorrea?  ¿Cómo  se  confirma  el  diagnóstico?
La  esteatorrea  son  heces  blandas,  grasosas  y  malolientes.  Se  puede  realizar  un  análisis  de  grasa  fecal  de  72  
horas  para  confirmar  el  diagnóstico.  La  prueba  de  D­xilosa  muestra  resultados  normales  y  la  prueba  de  Schilling  
no  es  sensible  para  la  insuficiencia  pancreática.  Los  pacientes  con  esteatorrea  se  tratan  con  una  combinación  
variable  de  dietas  bajas  en  grasas,  enzimas  pancreáticas,  antiácidos  y  cimetidina.  La  presencia  de  esteatorrea  
indica  que  la  producción  de  lipasa  está  >10%  por  debajo  de  los  niveles  normales.

7.  ¿La  amilasa  sérica  está  elevada  en  pacientes  con  pancreatitis  crónica?
No.  El  nivel  de  amilasa  sérica  suele  ser  normal  en  los  casos  de  pancreatitis  «quemada».

8.  ¿Cuáles  son  las  complicaciones  de  la  pancreatitis  crónica?
Puede  ocurrir  seudoquiste  pancreático,  absceso  o  fístula.  La  obstrucción  del  árbol  biliar  con  la  ictericia  
resultante  puede  deberse  a  áreas  de  fibrosis.  Es  más  probable  que  coexistan  la  desnutrición  y  la  adicción  a  los  
narcóticos  que  las  complicaciones  reales  de  la  insuficiencia  pancreática.

203
Machine Translated by Google
204  CAPÍTULO  40  DIAGNÓSTICO  Y  TRATAMIENTO  DE  LA  PANCREATITIS  CRÓNICA

9.  ¿Cuál  es  una  posible  fuente  de  hemorragia  digestiva  alta  en  un  paciente  con  pancreatitis  crónica?

Aunque  la  gastritis  y  la  úlcera  péptica  son  causas  más  frecuentes  de  hemorragia  digestiva  alta  (HDA),  también  se  
debe  considerar  la  trombosis  de  la  vena  esplénica  con  varices  gástricas  asociadas  e  hiperesplenismo.  (¡A  su  
asistente  le  encantará  esta  respuesta!)

10.  ¿Qué  es  la  ''cadena  de  lagos''?
Durante  la  colangiopancreatografía  retrógrada  endoscópica  (CPRE),  se  inyecta  medio  de  contraste  en  el  conducto  
pancreático;  las  áreas  secuenciales  de  estrechamiento  seguidas  de  dilatación  del  conducto  provocan  la  apariencia  
de  una  "collar  de  cuentas"  o  "cadena  de  lagos".

11.  ¿Cuáles  son  las  opciones  de  tratamiento  para  la  pancreatitis  crónica?
Inicialmente,  la  terapia  médica  incluye  medicamentos  para  el  dolor,  una  dieta  baja  en  grasas,  abstinencia  de  
alcohol  y  reemplazo  de  enzimas  pancreáticas  o  terapia  con  insulina,  según  se  indique.  Los  pacientes  con  evidencia  
de  insuficiencia  pancreática  y  dolor  abdominal  persistente  que  requieren  hospitalizaciones  repetidas  deben  
considerar  opciones  terapéuticas  más  invasivas.  Para  los  pacientes  con  estenosis  del  conducto  pancreático  proximal  
y  dilatación  ductal  aguas  arriba,  el  tratamiento  endoscópico  (esfinterotomía,  dilatación  de  la  estenosis,  extracción  de  
cálculos,  colocación  de  stent)  puede  tener  éxito.  El  resto  de  pacientes  con  síntomas  refractarios  pueden  ser  
intervenidos  quirúrgicamente.

12.  ¿Cuáles  son  las  indicaciones  de  la  cirugía?
No  hay  reglas  fijas.  Las  indicaciones  relativas  incluyen  dolor  persistente  refractario  al  tratamiento  médico,  conducto  
pancreático  principal  dilatado,  obstrucción  de  la  salida  biliar  o  gástrica,  páncreas  divisum,  seudoquiste  sintomático  
o  agrandado  y  sospecha  de  malignidad.

13.  ¿Qué  procedimientos  quirúrgicos  se  realizan  comúnmente?
Una  pancreaticoyeyunostomía  lateral  en  Y  de  Roux  (es  decir,  un  procedimiento  de  Peustow)  proporciona  
alivio  del  dolor  a  través  del  drenaje  ductal  mientras  se  preserva  el  parénquima  pancreático.  Se  abre  todo  el  
conducto  pancreático  de  la  «cadena  de  lagos»  desde  la  cabeza  hasta  la  cola,  y  se  sutura  la  rama  yeyunal  de  Roux  
a  la  cápsula  pancreática  alrededor  del  conducto  «fileteado»  para  proporcionar  una  ruta  de  drenaje.  Se  puede  realizar  
una  duodenopancreatectomía  (es  decir,  un  procedimiento  de  Whipple)  en  pacientes  con  una  masa  inflamatoria  en  
la  cabeza  del  páncreas.  El  procedimiento  de  Frey  implica  "sacar"  la  cabeza  pancreática  combinado  con  el  drenaje  
tanto  de  los  conductos  de  la  cabeza  pancreática  como  de  la  longitud  del  conducto  pancreático.  La  pancreatectomía  
distal  se  puede  utilizar  para  la  enfermedad  distal  aislada  o  el  drenaje  retrógrado  en  una  pancreaticoyeyunostomía.

14.  ¿Cuál  es  el  resultado  de  la  intervención  quirúrgica?
El  alivio  del  dolor  ocurre  en  aproximadamente  el  70%  de  los  pacientes  al  final  de  1  año  y  en  el  50%  de  los  
pacientes  al  final  de  5  años.  Aunque  la  morbilidad  asociada  oscila  entre  el  6  %  y  el  50  %  según  el  tipo  de  
operación,  la  mortalidad  general  de  estos  procedimientos  es  del  1  %  al  3  %.

PUNTOS  CLAVE

1.  Causas:  alcohol  (70%),  otras/idiopáticas  (30%).

2.  Síntomas:  dolor  abdominal  latente  e  insuficiencia  pancreática  (diabetes,  esteatorrea).

3.  Pruebas  de  laboratorio:  ninguna.

4.  Imágenes:  la  tomografía  computarizada  (TC)  diagnostica  masas  pancreáticas,
dilatación,  calcificaciones  y  seudoquistes;  La  CPRE  evalúa  el  conducto  pancreático,  incluidas  las  estenosis.

5.  Tratamiento:  analgésicos,  dieta  baja  en  grasas,  abstinencia  de  alcohol,  enzima  pancreática
terapia  de  reemplazo  e  insulina;  el  dolor  constante,  refractario  al  tratamiento  médico,  puede  tratarse  con  
intervención  endoscópica  o  quirúrgica.
Machine Translated by Google
CAPÍTULO  40  DIAGNÓSTICO  Y  TRATAMIENTO  DE  LA  PANCREATITIS  CRÓNICA  205

BIBLIOGRAFÍA

1.  Asociación  Americana  de  Gastroenterología:  Revisión  técnica  de  la  AGA:  tratamiento  del  dolor  en  la  pancreatitis  crónica.
Gastroenterología  115:765­776,  1998.

2.  Cahen  DL,  Gouma  DJ,  Nio  Y  et  al.:  Drenaje  endoscópico  versus  quirúrgico  del  conducto  pancreático  en  la  pancreatitis  crónica.  
N  Engl  J  Med  356(7):676–684.

3.  Fernández­del  Castillo  C,  Rattner  DW,  Warshaw  AL:  Estándares  para  la  resección  pancreática  en  la  década  de  1990.  Cirugía  de  arco
130:295­300,  1995.

4.  Heiko  W,  Apte  MV,  Volker  K  et  al:  Revisiones  en  gastroenterología  básica  y  clínica,  pancreatitis  crónica:
desafíos  y  avances  en  patogénesis,  genética,  diagnóstico  y  terapia.  Gastroenterología  132:1557­1573,  2007.

5.  Jordan  Jr  PH,  Pikoulis  M:  tratamiento  quirúrgico  para  el  dolor  de  pancreatitis  crónica.  J  Am  Coll  Surg  192:498­509,  2001.

6.  Schnelldorfer  T,  Lewin  DN,  Adams  DB:  Manejo  operativo  de  la  pancreatitis  crónica:  resultados  a  largo  plazo  en  372  pacientes,  J  Am  
Coll  Surg  204:1039­1047,  2007.

7.  Steer  ML,  Waxman  I,  Freedman  S:  Pancreatitis  crónica,  N  Engl  J  Med  332:1482­1490,  1995.
Machine Translated by Google

HIPERTENSION  PORTAL  Y  ESOFAGO
VÁRICES
CAPÍTULO  
41

Ramin  Jamshidi,  MD,  y  Gregory  V.  Stiegmann,  MD

1.  Describa  el  suministro  de  sangre  al  hígado.
El  flujo  sanguíneo  hepático  total  es  de  aproximadamente  1500  ml/min  o  25%  del  gasto  cardíaco  (CO).  La  arteria  
hepática  normalmente  suministra  alrededor  del  30%  del  flujo  sanguíneo  y  la  vena  porta  aporta  el  70%.  Sin  
embargo,  la  arteria  hepática  y  la  vena  porta  suministran  cada  una  el  50%  del  oxígeno  del  hígado.  Con  la  
hipertensión  portal,  el  flujo  portal  disminuye  y  necesariamente  aumenta  la  contribución  relativa  de  la  arteria  hepática.

2.  ¿Cómo  se  define  la  hipertensión  portal?
La  presión  venosa  porta  normalmente  es  de  5  a  10  mm  Hg;  >20  mm  Hg  se  define  como  hipertensión  
portal.  La  medición  directa  es  riesgosa,  por  lo  que  en  su  lugar  se  utiliza  el  gradiente  de  presión  venosa  hepática  
(HVPG).  Este  es  el  cambio  en  la  presión  de  la  vena  hepática  cuando  se  ocluye  el  flujo  encajando  un  catéter  con  
globo  en  ella  (análogo  a  la  estimación  de  la  presión  auricular  izquierda  encajando  una  arteria  pulmonar).
Un  GPVH  normal  es  de  2  a  6  mm  Hg;  >12  mm  Hg  se  considera  hipertensión  portal.

3.  ¿Qué  es  el  flujo  hepatopétalo?
El  flujo  sanguíneo  portal  fisiológico  hacia  el  hígado  se  denomina  flujo  hepatopétalo.  La  inversión  del  flujo  en  la  
vena  porta  puede  ocurrir  con  un  gran  aumento  de  la  resistencia  vascular  hepática  y  se  denomina  flujo  hepatofugo.  
En  este  caso,  la  arteria  hepática  debe  proporcionar  la  perfusión  dominante  del  hígado.

4.  ¿Cuáles  son  las  causas  más  frecuentes  de  hipertensión  portal?
&  En  el  mundo:  esquistosomiasis.
  En  los  Estados  Unidos:  infección  crónica  por  el  virus  de  la  hepatitis  C  (VHC)  o  cirrosis  alcohólica  
(enfermedad  de  Laennec).
  en  niños:  oclusión  venosa  portal  extrahepática  (como  en  la  trombosis  de  la  vena  porta)  o  atresia  biliar.

5.  ¿Qué  son  la  esquistosomiasis  y  la  fiebre  de  Katayama?
Infección  por  un  trematodo  sanguíneo  de  agua  dulce  que  causa  una  dermatitis  inicial  ("prurito  del  nadador")  y  
exantema  seguido  después  de  1  a  2  meses  por  fiebre,  mialgias,  dolor  abdominal  y  diarrea  sanguinolenta  (fiebre  
de  Katayama).  A  medida  que  estos  parásitos  se  aparean  y  ponen  huevos  en  el  sistema  venoso,  la  inflamación  
resultante  causa  fibrosis  obstructiva  crónica  de  órganos  y  vasos,  que  se  manifiesta  por  hipertensión  portal.  La  
fiebre  de  Katayama  dura  unas  pocas  semanas  y  solo  es  superada  por  la  malaria  como  causa  de  enfermedad  
tropical  crónica.  Tratar  con  praziquantel.

6.  ¿Cómo  se  pueden  clasificar  anatómicamente  las  causas  de  hipertensión  portal?
Presinusoidal:  &  
Extrahepática:  trombosis  de  la  vena  porta  o  esplénica,  atresia  biliar  congénita,  compresión  extrínseca  
(p.  ej.,  tumor).
  Intrahepática:  cirrosis  biliar  primaria,  esquistosomiasis,  metástasis  hepáticas,  enfermedad  poliquística,  
sarcoidosis.
Sinusoidal:  cirrosis  hepática  (p.  ej.,  infección  viral,  alcohol,  hemocromatosis).
Postsinusoidal:  síndrome  de  Budd­Chiari,  obstrucción  de  la  vena  cava  inferior  (IVC),  del  lado  derecho
insuficiencia  cardiaca.

206
Machine Translated by Google
CAPÍTULO  41  HIPERTENSIÓN  PORTAL  Y  VARICES  ESOFÁGICAS  207

7.  Enumere  las  cuatro  conexiones  anatómicas  principales  entre  el  portal  y  los  sistemas  venosos  sistémicos.

1.  Vena  gástrica  (coronaria)  izquierda  a  la  vena  esofágica  (várices  esofágicas  potenciales).
2.  Vena  mesentérica  inferior  a  través  de  las  venas  hemorroidales  superiores  a  la  hipogástrica
vena  (posibles  várices  rectales).
3.  Vena  porta  a  vena  umbilical  a  venas  superficiales  de  la  pared  abdominal  (caput  potencial
Medusa).
4.  Venas  mesentéricas  a  las  venas  perlumbares  de  Retzius  en  la  VCI  (potencial  retroperitoneal
hemorragia).
Tenga  en  cuenta  que  la  razón  por  la  que  estas  anastomosis  pueden  desviar  la  sangre  (alrededor  del  hígado)  es  que  las  venas  esplácnicas  

carecen  de  válvulas  unidireccionales.

8.  Definir  hipertensión  portal  sinistral.
Derivado  de  siniestro  (en  latín,  "izquierda"),  se  trata  de  hipertensión  portal  del  "lado  izquierdo"  causada  
específicamente  por  trombosis  u  obstrucción  de  la  vena  esplénica.  Esto  provoca  la  derivación  de  las  ramas  gástricas  
cortas  de  la  vena  esplénica  a  la  vena  gástrica  izquierda,  lo  que  da  como  resultado  várices  gástricas.  La  esplenectomía  
es  el  tratamiento  definitivo.

9.  ¿Cuáles  son  las  complicaciones  comunes  de  la  hipertensión  venosa  portal?
  Ascitis  y  peritonitis  bacteriana  espontánea.
  Hemorragia  por  várices  esofágicas  (la  principal  causa  de  mortalidad).
e  hiperesplenismo.
&  várices  rectales  (hemorroides).
&  Encefalopatía  portosistémica.
&  Gastropatía  y  colopatía  hipertensiva  portal.

10.  ¿Qué  impacto  puede  tener  la  hipertensión  portal  en  otros  sistemas  de  órganos?
  Circulación  hiperdinámica  (disminución  de  la  resistencia  vascular  sistémica  con  aumento  del  GC  y  presión  
arterial  baja).
&  Síndrome  hepatorrenal.
  Síndrome  hepatopulmonar  o  hipertensión  portopulmonar.

11.  ¿Según  qué  sistema  se  clasifica  la  función  hepática?
El  sistema  Child­Turcott­Pugh  modificado  define  tres  clases  de  enfermedad  hepática  en  función  de  la  
mortalidad;  los  puntos  se  deben  sumar  de  la  Tabla  41­1.
Clase  A  (5  a  6  puntos):  100  %  de  supervivencia  a  1  año  
Clase  B  (7  a  9  puntos):  80  %  de  supervivencia  a  1  año  
Clase  C  (10  puntos):  45  %  de  supervivencia  a  1  año

TABLA  41­1.  CHI  LD  ­  T  UR  CO  TT  ­  PUGH  SISTEMA  DE  PUNTUACIÓN  DE  ENFERMEDADES  HEPÁTICAS

Parámetro 1  punto 2  puntos 3  puntos

Albúmina  (g/dl) >3.5 2,8–3,5 <2.8

Bilirrubina  (mg/dl) <2 2–3 >3

Razón  internacional  normalizada  (INR) <1.7 1.7–2.2 >2.2

ascitis Ninguno Moderado Severo

Encefalopatía Ninguno Moderado Severo


Machine Translated by Google
208  CAPÍTULO  41  HIPERTENSIÓN  PORTAL  Y  VARICES  ESOFÁGICAS

12.  ¿Qué  es  MELD?
La  puntuación  de  enfermedad  hepática  en  etapa  terminal  de  Mayo  es  una  medida  completamente  objetiva  
de  la  enfermedad  calculada  con  el  índice  internacional  normalizado  (INR),  la  bilirrubina  y  la  creatinina.  En  2002,  
la  United  Network  for  Organ  Sharing  (UNOS)  adoptó  MELD  para  determinar  la  prioridad  del  trasplante  de  hígado.

13.  ¿Cómo  se  calcula  MELD?

MELD  ¼  10  ½0:957  ln  creatinina  
dmg  Þþ  =dl  0:378  ð  ln  bilirrubina  

mg  Þ  =dl  þ  1:120  ln  INR  ð  Þ

La  creatinina  máxima  ingresada  es  4.0.  El  resultado  se  redondea  al  entero  más  próximo.

14.  ¿Qué  tan  comunes  son  las  várices  esofágicas?
En  el  momento  del  diagnóstico  de  cirrosis,  aproximadamente  el  30%  de  los  pacientes  tienen  várices  esofágicas  
y  la  incidencia  de  formación  de  nuevas  várices  en  pacientes  con  cirrosis  conocida  es  de  aproximadamente  el  6%  
por  año.  Hay  una  prevalencia  puntual  del  50%  de  várices  en  pacientes  cirróticos.  Sin  embargo,  el  sangrado  ocurre  
en  solo  alrededor  de  un  tercio  de  los  pacientes  con  várices.

15.  ¿La  hemorragia  digestiva  alta  en  pacientes  cirróticos  con  várices  documentadas  siempre  es  várice?

Las  habilidades  perfeccionadas  para  tomar  exámenes  le  dicen  que  la  respuesta  debe  ser  no.  El  20%  de  estos  
pacientes  sangra  por  otra  fuente  (p.  ej.,  ulceraciones  gástricas  alcohólicas,  úlcera  péptica).  Esto  incluye  pacientes  
con  ascitis,  araña  angiomata  y  asterixis.

16.  ¿Son  las  várices  gástricas  una  fuente  frecuente  de  sangrado  en  pacientes  con  hipertensión  portal?

No.  Solo  alrededor  del  5%  de  las  hemorragias  por  várices  en  pacientes  cirróticos  son  causadas  por  várices  gástricas.  
La  hipertensión  portal  con  várices  gástricas  y  sin  várices  esofágicas  suele  asociarse  con  trombosis  de  la  vena  
esplénica.  Las  várices  gástricas  sangran  con  mucha  menos  frecuencia,  pero  con  mayor  severidad,  que  sus  
contrapartes  esofágicas.

17.  ¿Qué  factores  son  predictivos  de  sangrado  por  várices?
  Tamaño  de  las  várices  (el  factor  más  importante),  que  aumenta  la  tensión  de  la  pared  del  vaso.
  Marcas  rojas  en  las  várices  («marcas  de  látigo»  longitudinales)  por  disminución  de  la  pared
espesor.
  Gravedad  de  la  enfermedad  hepática.
&  Abuso  activo  de  alcohol.
En  total,  la  hemorragia  por  várices  ocurre  en  el  30%  de  los  pacientes  dentro  de  los  2  años  posteriores  a  la  documentación  de  várices.

18.  ¿El  grado  de  hipertensión  portal  predice  sangrado?
Sorprendentemente,  no.  El  riesgo  de  sangrado  se  correlaciona  mal  con  la  magnitud  de  la  presión  portal.  Sin  
embargo,  rara  vez  ocurre  sangrado  con  HVPG  <12  mm  Hg;  este  umbral  de  presión  se  considera  necesario  pero  no  
suficiente  para  la  hemorragia.

19.  ¿Con  qué  mortalidad  y  riesgo  de  resangrado  se  asocia  una  hemorragia  varicosa  inicial?
El  cuarenta  por  ciento  de  estos  pacientes  vuelven  a  sangrar  dentro  de  las  6  semanas,  y  el  40%  ocurre  en  los  primeros  5  días.  Si  no  
se  trata,  hasta  el  75  %  de  los  pacientes  vuelve  a  sangrar  durante  el  primer  año  y  aproximadamente  el  50  %  muere.

20.  ¿Se  deben  usar  bloqueadores  beta  selectivos  o  no  selectivos  en  el  tratamiento  de  las  várices  esofágicas?

El  bloqueo  beta  no  selectivo  minimiza  mejor  el  sangrado  al  disminuir  la  presión  arterial  y  reducir  el  flujo  
esplácnico.  El  antagonismo  adrenérgico  β1  causa  vasoconstricción  esplácnica  por  activación  refleja  de  los  
receptores  α  y  disminuye  la  contractilidad  miocárdica.  El  bloqueo  b2  previene  la  vasodilatación  esplácnica  y  
periférica.  El  nadolol  es  el  fármaco  de  elección.
Machine Translated by Google
CAPÍTULO  41  HIPERTENSIÓN  PORTAL  Y  VÁRICES  ESOFÁGICAS  209

21.  ¿Cuáles  son  los  principales  componentes  del  tratamiento  de  la  hemorragia  aguda  por  várices?
  Reanimación  con  líquidos  o  hemoderivados  (tenga  cuidado  de  no  empeorar  la  ascitis).
  Agentes  farmacológicos  para  disminuir  la  presión  y  el  flujo  portal.
&  Endoscopia  para  confirmar  el  diagnóstico  y  tratar  con  bandas  o  escleroterapia.
&  Profilaxis  antibiótica.
  Catarsis  de  lactulosa  (el  sangrado  gastrointestinal  [GI]  aumenta  la  carga  proteica;
rico  en  proteínas  y  puede  empeorar  la  encefalopatía).
  Taponamiento,  cirugía  o  derivación  portosistémica  intrahepática  transyugular  (TIPS,  por  sus  siglas  en  inglés)  si  es  
refractaria  o  presenta  una  hemorragia  recurrente  temprana.

22.  ¿Qué  tratamientos  farmacológicos  se  utilizan  en  el  sangrado  variceal  agudo?
La  vasopresina  (comience  con  0,2  U/min  por  vía  intravenosa  [IV]  y  aumente  el  nivel  mientras  observa  el  
electrocardiograma  [ECG])  disminuye  la  perfusión  esplácnica  y,  por  lo  tanto,  la  presión  portal.  Ten  cuidado;  la  
vasoconstricción  sistémica  puede  causar  isquemia  e  infarto  miocárdico  o  mesentérico.

La  terlipresina  (2  mg  IV  cada  4  horas)  es  un  análogo  sintético  de  la  vasopresina  con  menos  efectos  secundarios.
efectos  y  dosificación  más  simple.  Se  ha  mostrado  claramente  prometedor  en  ensayos  controlados  aleatorios,  pero  
aún  no  está  disponible  en  los  Estados  Unidos  fuera  de  los  ensayos  clínicos.
Octreotide  (bolo  de  50  mg  IV,  luego  25  mg/h  IV)  es  un  análogo  sintético  de  la  somatostatina  que
disminuye  el  flujo  sanguíneo  portal  por  vasoconstricción  esplácnica  selectiva,  por  lo  que  los  efectos  secundarios  
son  limitados.  Octreotide  actúa  a  través  de  péptidos  vasoactivos  sustancia  P  y  glucagón.

23.  ¿Qué  tratamientos  endoscópicos  se  utilizan  en  el  sangrado  variceal  agudo?
  Ligadura  endoscópica  con  bandas  (LBE):  estrangulación  directa  de  várices  con  bandas  elásticas,  similar
a  la  banda  de  hemorroides.
  Escleroterapia:  inyección  intravariceal  de  una  sustancia  química  esclerosante.
Cualquiera  de  las  dos  técnicas  suele  controlar  la  hemorragia  aguda  en  90%  de  las  hemorragias  por  várices,  
pero  aunque  la  escleroterapia  puede  ser  más  fácil  ante  una  hemorragia  importante,  la  EBL  es  más  segura  (menor  
posibilidad  de  perforación)  y  tiende  a  requerir  menos  repeticiones  (v .  fig.  41­1). .

Figura  41­1.  Ligadura  
endoscópica  con  banda.  
A,  El  endoscopio  se  
coloca  sobre  una  várice  y  
se  aplica  succión  para  
atraerlo  hacia  el  ligador.  
Luego  se  expulsa  una  
banda  elástica  sobre  la  
base  de  la  lesión.  B,  La  
banda  estrangula  la  várice,  
que  se  desprende  y  atraviesa  
el  cuerpo  en  unos  5  a  7  días.

24.  ¿Por  qué  se  deben  administrar  antibióticos  a  los  pacientes  cirróticos  ingresados  por  hemorragia  digestiva?
Estos  pacientes  tienen  casi  el  doble  de  riesgo  de  desarrollar  infecciones  bacterianas  mientras  están  hospitalizados  que  
los  pacientes  cirróticos  ingresados  por  otras  razones  (las  tasas  de  infección  nosocomial  se  aproximan  al  50%).  La  
peritonitis  bacteriana  espontánea,  la  bacteriemia  y  la  neumonía  son  las  más  frecuentes.
Machine Translated by Google
210  CAPÍTULO  41  HIPERTENSIÓN  PORTAL  Y  VARICES  ESOFÁGICAS

infecciones  La  profilaxis  antibiótica  a  corto  plazo  reduce  la  incidencia  de  infecciones  y  las  
hemorragias  tempranas  con  el  consiguiente  aumento  de  la  supervivencia.  La  norfloxacina  oral  (400  mg  dos  veces  al  
día  durante  7  días)  es  un  régimen  comprobado;  la  ceftriaxona  intravenosa  (1  g  una  vez  al  día  durante  7  días)  es  
superior  para  pacientes  con  shock  hipovolémico.

25.  ¿Qué  es  un  tubo  de  Sengstaken­Blakemore?
Una  sonda  nasogástrica  (NG)  grande  con  dos  globos  inflables  que  se  puede  usar  para  taponar  tanto  el  esófago  
como  el  cardias  gástrico.  El  globo  gástrico  se  infla  en  el  estómago  (inserte  150  ml  de  solución  salina  más  25  ml  de  
Gastrografin  para  que  pueda  confirmar  la  posición  adecuada  mediante  una  radiografía)  y  tire  suavemente  de  este  globo  
inflado  contra  la  unión  gastroesofágica.  La  mayoría  de  las  hemorragias  ocurren  en  los  5  cm  distales  del  esófago,  por  lo  
que  si  la  hemorragia  continúa,  también  se  debe  inflar  el  globo  esofágico.  Para  evitar  la  isquemia  o  la  ruptura  esofágica  
inducida  por  el  globo,  no  infle  este  globo  a  más  de  30  mm  Hg  (excede  la  presión  venosa  portal)  y  limite  el  uso  a  24  
horas.  La  mitad  de  los  pacientes  vuelve  a  sangrar  después  de  desinflar  el  globo  y  10  a  25%  sufre  neumonía  por  
aspiración  (v .  fig.  41­2).

Figura  41­2.  Tubo  
Sengstaken  Blakemore,  
con  dos  balones  y  puertos  de  succión.

26.  ¿Cuáles  son  las  opciones  para  prevenir  las  hemorragias  varicosas  recurrentes?
Sin  tratamiento,  el  75%  de  los  pacientes  vuelven  a  sangrar  dentro  de  1  año.  los  bloqueadores  beta  reducen  esto  
al  40%;  cuando  se  combina  con  escleroterapia,  la  tasa  es  del  35  %,  y  cuando  se  combina  con  EBL,  la  tasa  se  reduce  
al  25  %.  Por  lo  tanto,  las  tasas  más  bajas  de  hemorragia  recurrente  se  logran  con  EBL  y  nadolol  crónico.  Curiosamente,  
la  EBL  con  bloqueadores  beta  no  ha  demostrado  diferencias  en  la  supervivencia  a  los  2  años  en  comparación  con  el  
tratamiento  con  bloqueadores  beta  y  nitratos  solos.  La  cirugía  de  derivación  y  los  TIPS  son  ligeramente  mejores  que  
todas  estas  opciones  con  un  15  %  de  nuevas  hemorragias  por  año,  pero  estas  intervenciones  invasivas  también  son  
más  morbosas.
Machine Translated by Google
CAPÍTULO  41  HIPERTENSIÓN  PORTAL  Y  VARICES  ESOFÁGICAS  211

27.  ¿Cómo  se  debe  tratar  a  un  paciente  con  hemorragias  varicosas  recurrentes?
El  tratamiento  primario  debe  ser  EBL  combinado  con  bloqueo  beta.  Si  este  tratamiento  falla,  la  opción  de  segunda  
línea  es  descomprimir  el  sistema  venoso  portal  mediante  la  derivación  de  sangre  con  una  anastomosis  
portosistémica.  La  decisión  de  derivación  abierta  versus  radiológica  se  basa  en  la  urgencia  y  la  aptitud  del  
paciente  para  la  cirugía.

28.  ¿Qué  es  la  derivación  portosistémica  intrahepática  transyugular?
TIPS  es  una  técnica  radiológica  percutánea  para  desviar  sangre  venosa  portal  directamente  a  la  VCI.  Bajo  
fluoroscopia,  se  coloca  un  stent  a  través  del  parénquima  hepático  para  unir  las  venas  hepática  y  porta.  Aunque  la  
TIPS  alivia  la  ascitis  y  es  superior  a  la  EBL  para  reducir  el  riesgo  de  hemorragia  por  várices,  también  exacerba  la  
encefalopatía  sin  disminuir  la  mortalidad.  La  encefalopatía  nueva  o  empeorada  ocurre  en  al  menos  el  25%  de  los  
pacientes  después  de  TIPS.  La  estenosis  y  la  disfunción  del  stent  ocurren  en  30%  al  año  y  en  50%  a  los  2  años  
(fig.  41­3).

Figura  41­3.  Colocación  de  TIPS.  A,  Desde  una  vena  hepática,  una  aguja  perfora  el  hígado  para  llegar  a  
una  vena  porta.  B,  El  túnel  se  ensancha  con  un  catéter  con  balón.  C,  Se  coloca  un  stent  permanente.
(De  McNally  PR,  editor:  GI/Liver  Secrets,  2nd  ed.,  Philadelphia,  2001,  Hanley  &  Belfus.)

29.  Describa  las  opciones  básicas  para  la  derivación  quirúrgica.
Derivación  no  selectiva  (central):  las  derivaciones  portovacal  y  mesocava  descomprimen  de  manera  no  
selectiva  el  sistema  venoso  portal,  lo  que  pone  en  riesgo  el  flujo  hepatofugo  y  empeora  la  insuficiencia  hepática.
Grandes  cantidades  de  sangre  portal  (no  desintoxicada  en  el  hígado)  en  la  circulación  sistémica  
empeoran  la  encefalopatía.  La  creación  de  un  conducto  de  menor  diámetro  (derivación  parcial)  ayuda  a  
preservar  algo  del  flujo  portal  anterógrado  y  limita  este  efecto.
Derivación  esplenorrenal  (Warren)  selectiva:  anastomosis  de  la  vena  esplénica  distal  a  la  vena  renal  izquierda  
con  ligadura  de  la  gástrica  izquierda.  Esto  no  descomprime  tan  a  fondo  y,  por  lo  tanto,  esta  técnica  tiene  un  
menor  riesgo  de  encefalopatía.
Como  regla  general,  cuanto  más  central  sea  el  sitio  de  la  derivación,  más  extensa  será  la  descompresión  portal,  
pero  la  contrapartida  es  el  mayor  riesgo  de  encefalopatía  (como  lo  demuestra  el  TIPS).

30.  ¿Cómo  se  puede  estimar  la  mortalidad  operatoria  de  la  derivación  portosistémica  electiva?
La  mortalidad  perioperatoria  se  correlaciona  bien  con  la  clase  de  Child­Pugh  (este  fue  el  propósito  original  de  la  
clasificación).  Las  clases  A,  B  y  C  muestran  hasta  un  5  %,  10  %  y  40  %  de  mortalidad,  respectivamente,  a  los  30  
días.

31.  ¿Existe  un  tratamiento  definitivo  para  el  sangrado  recurrente  de  varices?
El  trasplante  de  hígado  proporciona  descompresión  portal  y  restaura  la  función  hepática.  Los  criterios  de  
inclusión  son  estrictos  y  la  evaluación  psicológica  del  "alcohólico  reformado"  es  particularmente  ardua.  TIPS  
anteriores  o  operaciones  de  derivación  no  son  contraindicaciones  para  el  trasplante.
Machine Translated by Google
212  CAPÍTULO  41  HIPERTENSIÓN  PORTAL  Y  VARICES  ESOFÁGICAS

CONTROVERSIA

32.  ¿Cómo  se  debe  tratar  a  un  paciente  con  várices  esofágicas  conocidas  para  prevenir  una  hemorragia  inicial  por  
várices?
La  combinación  de  bloqueadores  beta  y  nitrato  se  usa  para  la  profilaxis  primaria,  pero  la  EBL  es  al  menos  
equivalente  a  la  farmacoterapia  sin  los  efectos  secundarios  (un  tercio  de  los  pacientes  no  puede  tolerar  los  
bloqueadores  beta  debido  a  la  fatiga  o  el  broncoespasmo,  y  el  20%  no  puede  tolerar  los  nitratos  secundarios  
a  fuertes  dolores  de  cabeza).  Estos  tratamientos  reducen  la  incidencia  de  una  hemorragia  inicial  de  30%  a  
<10%  y  la  mortalidad  de  una  hemorragia  de  30%  a  20%.  La  EBL  se  sugirió  anteriormente  para  la  profilaxis  
solo  en  la  enfermedad  de  clase  C,  pero  la  evidencia  creciente  sugiere  que  la  EBL  es  al  menos  tan  efectiva  
como  la  farmacoterapia  en  todos  los  pacientes.

PUNTOS  CLAVE:  HIPERTENSIÓN  PORTAL

1.  Presión  venosa  portal  >20  mm  Hg  (normal  ¼  5  a  10  mm  Hg).

2.  Las  causas  más  comunes  en  los  Estados  Unidos  son  la  hepatitis  C  y  el  alcoholismo.

3.  Causas  anatómicas  caracterizadas  como  presinusoidales,  sinusoidales  o  postsinusoidales.

4.  Las  complicaciones  incluyen  ascitis,  várices  esofágicas,  encefalopatía,  hiperesplenismo  y
hemorroides.

5.  El  manejo  inicial  es  médico  y  endoscópico;  la  cirugía  se  reserva  para  los  casos  refractarios.

BIBLIOGRAFÍA

1.  Abraldes  JG,  Bosch  J:  El  tratamiento  del  sangrado  varicoso  agudo.  J  Clin  Gastroenterol  41:S312­S317,  2007.
2.  Albillos  A:  Prevención  de  la  primera  hemorragia  varicosa  en  la  cirrosis.  J  Clin  Gastroenterol  41:S305­S311,  2007.
3.  deFranchis  R,  Dell'Era  A,  Iannuzzi  F:  Diagnóstico  y  tratamiento  de  la  hipertensión  portal.  Dig  Hígado  Dis  36:787­798,
2004.

4.  Hayes  PC:  profilaxis  primaria  de  la  hemorragia  varicosa:  un  ensayo  controlado  aleatorio  que  compara  la  ligadura  con  banda,
propranolol  y  mononitrato  de  isosorbida.  Gastroenterología  123:735­744,  2002.
5.  Jensen  DM:  Detección  endoscópica  de  várices  en  cirrosis:  hallazgos,  implicaciones  y  resultados.
Gastroenterología  122:1620­1630,  2002.
6.  Lo  GH,  Chen  WC,  Chen  MH  et  al .:  Ligadura  con  bandas  versus  nadolol  y  mononitrato  de  isosorbida  para  la  prevención  del  
resangrado  de  várices  esofágicas.  Gastroenterología  123:728­734,  2002.
7.  Lui  HF,  Stanley  AJ,  Forrest  EH  et  al .:  Hemorragia  por  várices  gastroesofágicas.  N  Engl  J  Med  345:669­681,  2001.
Machine Translated by Google

ENFERMEDAD  POR  REFLUJO  GASTROESOFÁGICO
CAPITULO  
42

Michael  E.  Fenoglio,  MD  y  Lawrence  W.  Norton,  MD

1.  ¿Qué  síntomas  sugieren  enfermedad  por  reflujo  gastroesofágico  (ERGE)?
Un  síntoma  es  el  ardor  subesternal  después  de  las  comidas  o  por  la  noche,  asociado  ocasionalmente  con  
regurgitación  de  jugos  gástricos.  El  malestar  se  alivia  al  pararse  o  sentarse.  La  disfagia,  una  complicación  tardía  de  
la  ERGE,  es  causada  por  edema  de  la  mucosa  o  estenosis  del  esófago  distal.  Sin  embargo,  ningún  síntoma  es  
específico  de  la  ERGE  y  las  decisiones  terapéuticas  no  deben  tomarse  únicamente  en  función  de  los  síntomas.

2.  ¿Cuál  es  la  diferencia  entre  la  acidez  estomacal  y  la  ERGE?
La  acidez  estomacal  es  un  término  común  para  el  reflujo  leve  e  intermitente  de  contenido  gástrico  hacia  el  esófago  
sin  lesión  tisular.  Es  relativamente  común  entre  los  adultos.  GERD  implica  esofagitis  con  diversos  grados  de  eritema,  
edema  y  friabilidad  de  la  mucosa  esofágica  distal.  Ocurre  en  el  10%  de  la  población.

3.  ¿Qué  causa  la  ERGE?
La  anomalía  subyacente  de  la  ERGE  es  la  incompetencia  funcional  del  esfínter  esofágico  inferior  (EEI),  que  
permite  que  el  ácido  gástrico,  la  bilis  y  las  enzimas  digestivas  dañen  la  mucosa  esofágica  desprotegida.  La  
acalasia,  la  esclerodermia  y  otros  trastornos  de  la  motilidad  esofágica  a  veces  se  asocian  con  la  ERGE.

4.  ¿La  hernia  hiatal  es  un  defecto  esencial  en  pacientes  con  ERGE?
No.  No  todos  los  pacientes  con  ERGE  tienen  una  hernia  hiatal  y  no  todos  los  pacientes  con  hernia  hiatal  tienen  
ERGE.  Un  total  del  50%  de  los  pacientes  con  ERGE  tienen  una  hernia  hiatal  asociada.

5.  ¿Qué  estudios  son  útiles  para  diagnosticar  la  ERGE?
La  endoscopia  con  biopsia  es  esencial  en  el  diagnóstico  de  ERGE.  El  trago  de  bario  con  o  sin  fluoroscopia  
puede  diagnosticar  el  reflujo  pero  no  puede  identificar  la  esofagitis.  La  prueba  del  equilibrio  acidobásico  (pH)  
esofágico  de  veinticuatro  horas  asocia  el  reflujo  con  los  síntomas  y  es  útil  en  algunos  pacientes.  Las  pruebas  de  
secreción  gástrica  o  de  vaciamiento  gástrico  son  útiles  en  ocasiones.  La  manometría  del  esófago  y  el  EEI  se  requiere  
siempre  que  se  sospeche  un  trastorno  de  la  motilidad  esofágica  y  antes  de  cualquier  intervención  quirúrgica.

6.  ¿Cuál  es  el  manejo  inicial  de  un  paciente  con  sospecha  de  ERGE?
  Cambie  la  dieta  para  evitar  los  alimentos  que  se  sabe  que  inducen  el  reflujo  (p.  ej.,  chocolate,  alcohol  y  café).
&  Evite  las  comidas  copiosas  antes  de  acostarse.
&  Deje  de  fumar.
&  No  use  ropa  ceñida  y  ceñida.
&  Eleve  la  cabecera  de  la  cama  de  4  a  5  pulgadas.
&  Tome  antiácidos  cuando  tenga  síntomas.
&  La  pérdida  de  peso  puede  ser  muy  eficaz  para  reducir  los  síntomas  de  la  ERGE.

7.  Si  falla  el  tratamiento  inicial,  ¿qué  se  debe  recomendar?
Alrededor  del  50  %  de  los  pacientes  muestran  una  curación  significativa  con  bloqueadores  H2,  pero  solo  el  10  %  de  estos  
pacientes  permanecen  curados  1  año  después.  La  metoclopramida  promueve  el  vaciado  gástrico,  pero  rara  vez  alivia  los  
síntomas  de  manera  consistente  en  ausencia  de  reducción  de  ácido.

213
Machine Translated by Google
214  CAPÍTULO  42  ENFERMEDAD  POR  REFLUJO  GASTROESOFÁGICO

PUNTOS  CLAVE:  TRABAJO  DE  DIAGNÓSTICO  DE
ENFERMEDAD  POR  REFLUJO  GASTROESOFÁGICO

1.  La  anormalidad  anatómica  subyacente  puede  causar  incompetencia  funcional  del  EEI.

2.  La  endoscopia  y  la  biopsia  son  primordiales  en  el  diagnóstico.

3.  Los  estudios  de  deglución  delimitan  las  posibles  causas  anatómicas.

4.  La  monitorización  del  pH  durante  veinticuatro  horas  puede  relacionar  el  reflujo  con  los  síntomas  del  paciente.

5.  Se  requiere  manometría  del  EEI  si  se  sospecha  un  trastorno  de  la  motilidad  esofágica.

8.  ¿Cuál  es  el  papel  del  inhibidor  de  la  bomba  de  protones  (IBP)  en  la  ERGE?
Los  PPI  (omeprazol  y  otros)  inhiben  de  manera  irreversible  la  bomba  de  iones  de  hidrógeno  de  las  células  
parietales  y  tienen  >80%  de  éxito  en  la  curación  de  la  esofagitis  erosiva  grave.  Dos  tercios  de  los  pacientes  que  continúan  
con  la  medicación  permanecen  curados.  Una  preocupación  en  el  tratamiento  prolongado  con  PPI  es  la  hipergastrinemia  
secundaria  a  la  alcalinización  del  antro.  La  gastrina  es  trófica  para  la  mucosa  gastrointestinal  (GI),  pero  los  estudios  de  
seguimiento  no  han  confirmado  el  temor  inicial  de  una  neoplasia  inducida.

9.  ¿Cuándo  se  debe  recomendar  la  operación  por  ERGE?
El  fracaso  de  la  terapia  no  quirúrgica  (médica)  es  la  principal  indicación  para  la  cirugía.  El  incumplimiento  del  tratamiento  
prescrito  es  una  causa  frecuente  de  fracaso  e  incluso  de  estenosis  que  no  responde  a  la  dilatación.  Con  los  PPI,  los  
síntomas  de  la  mayoría  de  los  pacientes  pueden  controlarse  durante  largos  períodos  de  tiempo.
Las  recomendaciones  actuales  para  la  intervención  quirúrgica  incluyen:  (1)  tratamiento  médico  fallido  (p.  ej.,  enfermedad  
intratable,  intolerancia  o  alergia  a  los  medicamentos,  incumplimiento  y  recurrencia  de  los  síntomas  durante  el  tratamiento  
médico),  (2)  complicaciones  (p.  ej.,  estenosis,  síntomas  respiratorios,  problemas  medicosociales).  y  cambios  mucosos  
premalignos),  (3)  preferencia  del  paciente  (p.  ej.,  costo  —las  prescripciones  médicas  a  largo  plazo  pueden  ser  costosas
—  o  problemas  de  estilo  de  vida).

10.  ¿Cuál  es  el  objetivo  del  tratamiento  quirúrgico?
Las  operaciones  para  GERD  intentan  prevenir  el  reflujo  aumentando  mecánicamente  la  presión  del  EEI  y,  en  la  mayoría  
de  los  procedimientos,  restaurar  una  longitud  suficiente  de  esófago  distal  a  la  zona  de  alta  presión  del  abdomen.  La  
hernia  de  hiato,  cuando  está  presente,  se  reduce  simultáneamente.

11.  ¿Qué  procedimientos  pueden  lograr  este  objetivo  y  cómo  lo  hacen?
  En  la  fundoplicatura  de  Nissen,  que  se  usa  en  >95%  de  los  pacientes,  se  moviliza  el  fondo  del  estómago,  
se  envuelve  alrededor  del  esófago  distal  en  la  parte  posterior  y  se  asegura  a  sí  mismo  en  la  parte  anterior  
(es  decir,  envoltura  de  360  grados).  El  procedimiento  altera  el  ángulo  de  la  unión  gastroesofágica  y  mantiene  
el  esófago  distal  dentro  del  abdomen  para  evitar  el  reflujo.  La  operación  se  realiza  por  vía  transabdominal  
mediante  laparotomía  o  laparoscopia  (v .  fig.  42­1).

  La  operación  Belsey  Mark  IV  logra  los  mismos  cambios  anatómicos,  pero  se  realiza  mediante  una  toracotomía  (v .  
fig.  42­2).
  La  gastropexia  de  Hill  restablece  el  esófago  a  la  cavidad  abdominal  asegurando  el  cardias  gástrico  a  la  fascia  
preaórtica  (v .  fig.  42­3).
  La  fundoplicatura  de  Toupet  (parcial)  se  utiliza  en  pacientes  que  tienen  trastornos  de  la  motilidad  asociados.  
Debido  a  que  la  envoltura  no  es  circunferencial,  la  incidencia  de  disfagia  posoperatoria  se  reduce  
significativamente  con  esta  envoltura  parcial  en  comparación  con  una  envoltura  completa  de  360  grados  
(funduplicatura  de  Nissen).  Sin  embargo,  la  durabilidad  a  largo  plazo  puede  no  ser  tan  buena  como  con  una  
fundoplicatura  de  Nissen.  Esta  operación  puede  realizarse  por  vía  transabdominal  mediante  laparotomía  o  
laparoscopia  (v .  fig.  42­4).
Machine Translated by Google
CAPÍTULO  42  ENFERMEDAD  POR  REFLUJO  GASTROESOFÁGICO  215

Figura  42­1.  En  la  
fundoplicatura  de  Nissen,  que  
se  usa  en  >95%  de  los  
pacientes,  se  moviliza  el  
fondo  del  estómago,  se  
envuelve  alrededor  del  
esófago  distal  en  la  parte  
posterior  y  se  asegura  a  sí  
mismo  en  la  parte  anterior  (es  
decir,  envoltura  de  360  grados).  
El  procedimiento  altera  el  
ángulo  de  la  unión  
gastroesofágica  y  mantiene  el  
esófago  distal  dentro  del  abdomen  para  evitar  el  refluj
La  operación  se  realiza  
transabdominalmente  por  
laparotomía  o  laparoscopia.

Figura  42­2.  La  operación  
Belsey  Mark  IV  logra  los  
mismos  cambios  
anatómicos  que  la  
fundoplicatura  de  Nissen,  pero  
se  realiza  a  través  de  una  toracotomía.

Figura  42­3.  La  gastropexia  de  Hill  restaura  el  esófago  a  la  cavidad  abdominal  asegurando  el  
cardias  gástrico  a  la  fascia  preaórtica.
Machine Translated by Google
216  CAPÍTULO  42  ENFERMEDAD  POR  REFLUJO  GASTROESOFÁGICO

Figura  42­4.  La  fundoplicatura  
de  Toupet  (parcial)  se  usa  en  
pacientes  que  tienen  trastornos  
de  la  motilidad  asociados.  
Debido  a  que  la  envoltura  no  es  
circunferencial,  la  incidencia  de  
disfagia  posoperatoria  se  reduce  
significativamente  con  esta  
envoltura  parcial  en  comparación  
con  una  envoltura  completa  de  
360  grados  (funduplicatura  de  
Nissen).
Sin  embargo,  la  
durabilidad  a  largo  plazo  
puede  no  ser  tan  buena  como  
con  una  fundoplicatura  de  
Nissen.  Esta  operación  se  
puede  realizar  por  vía  
transabdominal  mediante  laparotomía  o  laparoscopia.

12.  ¿Cuáles  son  las  tasas  de  éxito  de  dichos  procedimientos?
Todos  los  procedimientos  descritos  en  la  pregunta  11  eliminan  la  ERGE  en  casi  el  90  %  de  los  pacientes  que  
son  seguidos  durante  10  años.  Pero  la  funduplicatura  de  Nissen  gana  en  los  estudios  de  comparación.
Los  síntomas  recurrentes  deben  analizarse  a  fondo  porque  con  frecuencia  se  asocian  con  otros  trastornos  y  no  
con  ERGE  recurrente.

13.  ¿Cuáles  son  las  complicaciones  a  largo  plazo  de  dichos  procedimientos?
La  reparación  puede  fallar,  con  recurrencia  del  reflujo,  después  de  cualquiera  de  estas  operaciones.  La  colocación  incorrecta  o  
el  deslizamiento  de  la  envoltura  del  estómago  pueden  complicar  la  fundoplicatura  de  Nissen  y  el  procedimiento  Belsey  Mark  IV.  
La  disfagia  y  la  incapacidad  para  eructar  (es  decir,  el  síndrome  de  hinchazón  por  gases)  son  el  resultado  de  una  venda  
demasiado  apretada.

14.  ¿Cómo  se  puede  manejar  la  estenosis  por  ERGE?
Las  estenosis  flexibles  (no  fijadas)  pueden  dilatarse.  Las  estenosis  fijas  requieren  reparación  quirúrgica.  Un  
parche  de  Thal  expande  la  estenosis  interponiendo  un  trozo  de  estómago.

CONTROVERSIAS

15.  ¿La  ERGE  se  trata  mejor  a  largo  plazo  con  terapia  con  IBP  o  fundoplicatura  de  Nissen?
Los  PPI  realmente  funcionan  para  resolver  la  esofagitis  y  eliminar  los  síntomas  de  la  ERGE,  pero  los  efectos  
secundarios  a  largo  plazo  no  se  conocen  por  completo.  La  fundoplicatura  potencialmente  libera  al  paciente  de  la  
medicina  diaria  (esto  ha  sido  cuestionado  recientemente)  y  puede  causar  morbilidad  en  el  10%  de  los  pacientes.
Machine Translated by Google
CAPÍTULO  42  ENFERMEDAD  POR  REFLUJO  GASTROESOFÁGICO  217

16.  ¿Se  debe  realizar  una  fundoplicatura  de  Nissen  por  laparoscopia  o  laparotomía?
El  mismo  procedimiento  puede  llevarse  a  cabo  mediante  cualquiera  de  los  dos  enfoques.  La  morbilidad  y  la  
mortalidad  postoperatorias  son  comparables.  Las  claras  ventajas  de  la  laparoscopia  son  menos  dolor  posoperatorio,  
hospitalización  más  breve  y  regreso  más  temprano  al  trabajo.

17.  ¿Se  puede  tratar  esta  enfermedad  por  otros  medios  mínimamente  invasivos?
Sí.  Otros  métodos  endoscópicos  incluyen:  &  
Sutura  endoluminal.
&  Tratamiento  por  radiofrecuencia  del  EEI.
  Inyección  de  agentes  formadores  de  masa  alrededor  del  EEI.

SITIO  WEB

www.emedicine.com/med/topic857.htm

BIBLIOGRAFÍA

1.  Anand  O,  Wani  S,  Sharma  P:  Enfermedad  por  reflujo  gastroesofágico  y  esófago  de  Barrett.  Endoscopy  40:126­130,  2008.  Epub  4  de  
diciembre  de  2007.

2.  Bremner  RM,  DeMeester  TR,  Crookes  F  et  al .:  El  efecto  de  los  síntomas  y  las  anomalías  de  la  motilidad  no  específicas  en  los  resultados  
de  la  terapia  quirúrgica  para  el  reflujo  gastroesofágico.  J  Thorac  Cardiovasc  Surg  107:1244­1250,  1994.

3.  DeMeester  TR,  Peters  JH,  Bremner  CG  et  al.:  Biología  de  la  enfermedad  por  reflujo  gastroesofágico:  fisiopatología
relacionados  con  el  tratamiento  médico  y  quirúrgico.  Annu  Rev  Med  50:469­506,  1999.

4.  Hinder  RA,  Filipi  CJ,  Wetscher  G  et  al.:  La  fundoplicatura  laparoscópica  de  Nissen  es  un  tratamiento  eficaz  para
enfermedad  por  reflujo  gastroesofágico.  Ann  Surg  220:472­481,  1994.

5.  Lagergren  J,  Bergstrom  R,  Lindgren  A  et  al.:  reflujo  gastroesofágico  sintomático  como  factor  de  riesgo  de
adenocarcinoma  de  esófago.  N  Engl  J  Med  340:825­831,  1999.

6.  Lord  RV,  Kaminski  A,  Oberg  S  et  al.:  Ausencia  de  enfermedad  por  reflujo  gastroesofágico  en  la  mayoría  de  los  pacientes  que  toman
Medicamentos  para  la  supresión  de  ácido  después  de  la  fundoplicatura  de  Nissen.  J  Gastrointest  Surg  6:3­9,  2002.

7.  Peters  JH,  DeMeester  TR,  editores:  Cirugía  mínimamente  invasiva  del  intestino  anterior.  St.  Louis,  1994,  Calidad  Médica
Publicación.

8.  Roy­Shapira  A,  Stein  HJ,  Scwartz  D  et  al .:  Métodos  endoluminales  para  tratar  la  enfermedad  por  reflujo  gastroesofágico.  Dis
Esófago  15:132­136,  2002.

9.  Spechler  SJ:  Comparación  de  la  terapia  médica  y  quirúrgica  para  la  enfermedad  por  reflujo  gastroesofágico  complicada  en
veteranos  N  Engl  J  Med  326:786­792,  1992.

10.  Spechler  SJ,  Lee  E,  Ahnen  D  et  al.:  Resultado  a  largo  plazo  de  las  terapias  médicas  y  quirúrgicas  para  gastroesofágico
enfermedad  por  reflujo:  seguimiento  de  un  ensayo  controlado  aleatorio.  JAMA  285:2331­2338,  2001.

11.  Spivak  H,  Lulcuk  S,  Hunter  JG:  Cirugía  laparoscópica  de  la  unión  gastroesofágica.  Cirugía  mundial  J
23:356­367,  1999.

12.  Strate  U,  Emmermann  A,  Fibbe  C  et  al .:  Funduplicatura  laparoscópica:  resultado  de  dos  años  de  Nissen  versus  Toupet  de  un  estudio  
prospectivo  aleatorizado  de  200  pacientes  con  respecto  a  la  motilidad  esofágica  preoperatoria.  Surg  Endosc  22:21­30,  2008.  Epub  20  de  
noviembre  de  2007.

13.  Triadafilopoulos  G,  DiBaise  JK,  Nostrant  TT  et  al.:  El  procedimiento  de  Stretta  para  el  tratamiento  de  la  ERGE:  6  y  12
mes  de  seguimiento  del  ensayo  de  etiqueta  abierta  de  EE.  UU.  Gastrointest  Endosc  55:149­156,  2002.

14.  Trus  TL,  Laycock  WS,  Waring  JP  et  al.:  Mejora  en  las  medidas  de  calidad  de  vida  después  del  antirreflujo  laparoscópico
cirugía.  Ann  Surg  229:331­336,  1999.

15.  Watson  DI,  Jamieson  JG,  Pike  GK  et  al.:  Ensayo  doble  ciego  aleatorizado  prospectivo  entre  Nissen  laparoscópico
fundoplicatura  y  fundoplicatura  parcial  anterior.  Br  J  Surg  86:120­130,  1999.
Machine Translated by Google

CÁNCER  DE  ESÓFAGO
CAPÍTULO  
43

Ricardo  J.  Gonzalez,  MD

1.  ¿Cuáles  son  los  factores  de  riesgo  para  desarrollar  cáncer  de  esófago?
Tanto  el  alcohol  como  el  tabaco  aumentan  el  riesgo  de  carcinoma  de  esófago  por  un  factor  de  10.
Los  factores  de  riesgo  adicionales  incluyen  esófago  de  Barrett  con  displasia,  exposición  a  carcinógenos  (p.  ej.,  
nitrosaminas  en  el  mundo  oriental),  deficiencias  de  vitaminas  y  oligoelementos  y  síndrome  de  Plummer­Vinson.

2.  ¿Cuál  es  la  epidemiología  del  carcinoma  de  esófago?
El  cáncer  de  esófago  representa  el  1%  de  todos  los  cánceres  y  el  2%  de  las  muertes  relacionadas  con  el  cáncer.  
Generalmente,  es  3  veces  más  común  en  hombres  y  ocurre  con  mayor  frecuencia  en  la  séptima  década  de  la  vida.
En  todo  el  mundo,  el  95  %  de  todos  los  cánceres  de  esófago  se  originan  en  células  escamosas;  sin  embargo,  en  el  
mundo  occidental,  la  incidencia  relativa  de  adenocarcinoma  ha  aumentado  de  forma  espectacular  en  los  últimos  20  
años  debido  al  aumento  comparable  de  la  incidencia  del  esófago  de  Barrett.

3.  ¿Qué  es  el  esófago  de  Barrett  y  cómo  se  relaciona  con  el  cáncer  de  esófago?
El  reflujo  crónico  de  contenido  gástrico  hacia  el  esófago  puede  conducir  al  esófago  de  Barrett,  que  se  
caracteriza  por  el  reemplazo  de  la  mucosa  esofágica  escamosa  normal  con  una  mucosa  columnar  glandular  
que  se  asemeja  al  estómago.  Esto  también  se  llama  metaplasia  intestinal.  Si  el  esófago  de  Barrett  progresa  a  
displasia  de  alto  grado,  los  pacientes  tienen  un  riesgo  cuarenta  veces  mayor  de  adenocarcinoma  esofágico.  Los  
pacientes  con  displasia  de  alto  grado  se  tratan  tradicionalmente  mediante  esofagectomía;  sin  embargo,  la  terapia  
fotodinámica  (TFD)  puede  eliminar  la  mucosa  de  Barrett  displásica,  obviando  la  resección  quirúrgica.  La  PDT  sigue  
sin  aprobarse  y  es  experimental.

4.  ¿Cuáles  son  los  síntomas  de  presentación  más  comunes  del  cáncer  de  esófago?
La  disfagia  ocurre  en  el  85%  de  los  pacientes.  Otros  síntomas  incluyen  pérdida  de  peso  (60%),  dolor  torácico  o  
epigástrico  (25%),  regurgitación  de  alimentos  no  digeridos  (25%),  ronquera  causada  por  afectación  del  nervio  
laríngeo  recurrente  (5%),  tos  o  disnea  (3%)  y  hematemesis.  (2%).

5.  ¿Cuál  es  el  estudio  de  diagnóstico  para  los  pacientes  que  presentan  estos  síntomas?
1.  Historia  y  examen  físico.
2.  Serie  gastrointestinal  superior  (GI)  (estudio  de  contraste  del  tracto  GI  superior).
3.  Endoscopia  digestiva  alta  con  biopsias  de  todas  las  estructuras  luminales  concernientes.
4.  Tomografía  computarizada  (TC)  de  tórax  y  abdomen  para  definir  enfermedad  metastásica  potencial  y  
ganglionar.
5.  Ecografía  endoscópica  (EUS)  para  definir  el  estadio  T  (es  decir,  el  tamaño)  de  la  masa  primaria  y  la  
afectación  de  los  ganglios  linfáticos  regionales  con  una  posible  biopsia  por  aspiración  con  aguja  fina  (FNA).
6.  Tomografía  por  emisión  de  positrones  (PET)  para  definir  la  diseminación  metastásica  a  distancia.

6.  ¿Cuál  es  la  distribución  anatómica  del  cáncer  de  esófago?
El  esófago  se  divide  en  tres  segmentos  anatómicos:  tercio  superior,  medio  e  inferior.  El  quince  por  ciento  de  los  
cánceres  de  esófago  surgen  en  el  tercio  superior,  el  50%  en  el  tercio  medio  y  el  35%  en  el  tercio  inferior.

218
Machine Translated by Google
CAPÍTULO  43  CÁNCER  DE  ESÓFAGO  219

7.  ¿Qué  es  la  quimioterapia  neoadyuvante?  ¿Cuáles  son  sus  ventajas  y
desventajas?
Esto  es  quimioterapia,  radioterapia  o  ambas  a  la  lesión  primaria  antes  de  la  resección  quirúrgica.
Las  ventajas  incluyen:  &  
Potencial  reducción  del  estadio  (para  reducir  el  tamaño  del  tumor  o  tratar  la  afectación  de  los  ganglios  linfáticos  locorregionales).
&  Tratamiento  precoz  de  la  enfermedad  micrometastásica.
  El  tratamiento  se  tolera  mejor  antes  del  estrés  quirúrgico.
&  Calibra  la  capacidad  del  paciente  para  tolerar  una  cirugía  mayor.
&  Verificación  de  la  sensibilidad  del  tumor  primario  a  la  quimioterapia  o  radioterapia  para
plan  para  una  terapia  adyuvante  eficaz.
Las  desventajas  incluyen:     
Retraso  en  el  tratamiento  de  la  lesión  primaria,  particularmente  cuando  el  tumor  primario  progresa  a  pesar  de  la  
terapia  neoadyuvante.
&  Selección  de  líneas  celulares  quimiorresistentes.

8.  ¿Cuáles  son  las  opciones  quirúrgicas  para  el  tratamiento  del  carcinoma  de  esófago?
La  cirugía  sola  o  combinada  con  quimiorradioterapia  ofrece  la  única  esperanza  de  curación.  Los  abordajes  quirúrgicos  
incluyen:  (1)  resección  transabdominal  de  lesiones  localizadas  en  la  unión  gastroesofágica;  (2)  resección  con  anastomosis  
intratorácica  por  toracoabdominal  izquierda  (procedimiento  de  Sweet)  o  laparotomía  de  línea  media  combinada  y  
toracotomía  derecha  (procedimiento  de  Ivor­Lewis);  y  (3)  esofagectomía  transhiatal  con  anastomosis  cervical.  La  terapia  
con  láser,  los  procedimientos  de  colocación  de  stents  esofágicos  y  la  dilatación  se  reservan  para  la  paliación.

9.  ¿Cuáles  son  los  riesgos  de  la  cirugía?
Fuga  anastomótica.
Estenosis  anastomótica.
Muerte.

Disfagia.
Hemorragia.
Empiema  y  sepsis.
Recurrencia  local  del  cáncer.

10.  ¿Cuál  es  la  historia  natural  del  cáncer  de  esófago?
En  una  serie  recopilada  de  casi  1000  pacientes  no  tratados,  las  tasas  de  supervivencia  a  1  y  2  años  fueron  del  6,0  %  
y  el  0,3  %,  respectivamente.  Los  pacientes  que  no  reciben  tratamiento  suelen  sucumbir  a  una  desnutrición  progresiva  
complicada  con  neumonía  por  aspiración,  sepsis  y  muerte.  La  formación  de  una  fístula  entre  la  aorta  o  la  arteria  pulmonar  
y  el  esófago  o  el  árbol  pulmonar  es  un  modo  de  salida  algo  más  dramático  (o  tal  vez  misericordioso).  Con  o  sin  
tratamiento,  el  cáncer  de  esófago  es  una  enfermedad  grave.

11.  Describir  los  estadios  del  cáncer  de  esófago  y  la  respectiva  supervivencia  a  5  años
tasa  después  de  la  esofagectomía.
El  estadio  I  es  el  cáncer  confinado  a  la  capa  interna  (muscularis  mucosae  o  submucosa),  y  la  supervivencia  a  los  5  años  
es  de  hasta  el  80  %.  El  estadio  II  describe  tumores  que  están  confinados  a  las  capas  fuera  de  la  submucosa  con  
afectación  de  los  ganglios  linfáticos  locales,  y  la  supervivencia  a  los  5  años  puede  llegar  al  35  %.
Los  tumores  en  estadio  III  han  invadido  las  estructuras  circundantes  (pulmón,  aorta  o  tráquea)  
independientemente  de  la  afectación  de  los  ganglios  linfáticos  regionales  o  atraviesan  la  pared  del  esófago  con  
afectación  de  los  ganglios.  La  supervivencia  a  5  años  suele  ser  <10%.  El  cáncer  de  esófago  en  estadio  IV  se  diseminó  
a  los  ganglios  linfáticos  no  regionales  (ganglios  supraclaviculares  o  celíacos)  o  a  órganos  distantes  (pulmón,  hígado  o  
hueso).  Esencialmente,  todos  los  pacientes  con  enfermedad  en  etapa  IV  mueren  dentro  de  los  2  años  posteriores  al  diagnóstico.

12.  ¿Qué  es  una  resección  ''R0'' (o  ''R  cero'')  y  cómo  afecta  la  supervivencia?
Se  elimina  toda  la  enfermedad  macroscópica  y,  microscópicamente,  los  márgenes  de  resección  son  negativos  para  
el  tumor.  Lograr  una  resección  R0  es  el  objetivo  del  cirujano  y  es  el  predictor  más  sólido  de  una
Machine Translated by Google
220  CAPÍTULO  43  CÁNCER  DE  ESÓFAGO

resultado  favorable  después  de  la  cirugía  para  el  cáncer  de  esófago.  Una  resección  R1  representa  la  extirpación  de  
toda  la  enfermedad  macroscópica,  pero  los  márgenes  de  resección  son  microscópicamente  positivos  para  el  tumor.  La  
supervivencia  general  a  5  años  (cualquier  etapa)  para  pacientes  con  márgenes  microscópicamente  positivos  disminuye  
en  un  orden  de  magnitud  (p.  ej.,  30  %  hasta  3  %).

PUNTOS  CLAVE:  CARCINOMA  DE  ESÓFAGO
1.  Más  frecuente  en  pacientes  mayores  con  disfagia  (85  %  de  los  casos)  y  pérdida  de  peso  (60  %  de  los  casos).
casos).

2.  Los  principales  factores  causales  son  el  alcohol  y  el  tabaco  (el  riesgo  se  multiplica  por  diez).

3.  El  diagnóstico  se  realiza  mediante  endoscopia  digestiva  alta  y  biopsia.

4.  La  variante  más  común  es  el  adenocarcinoma;  El  segundo  más  común  es  el  cáncer  de  células  escamosas.

5.  El  estudio  radiográfico  es  necesario  para  estadificar  la  enfermedad.

SITIOS  WEB

www.emedicine.com/med/topic741.htm

www.acssurgery.com/abstracts/acs/acs0309.htm

BIBLIOGRAFÍA

1.  Alexiou  C:  Supervivencia  después  de  la  resección  esofágica  por  carcinoma:  la  importancia  del  tipo  de  célula  histológica.  Ann  Thorac  
Surg  82:1073­1077,  2006.

2.  Cordero  JA:  Stents  esofágicos  metálicos  autoexpandibles  en  el  tratamiento  de  la  obstrucción  esofágica.  Soy  Cirugía:
66:958­959,  2000.

3.  Hofstetter  W:  Resultados  del  tratamiento  del  cáncer  de  esófago  resecado.  Ann  Surg  236:376­384,  2002.

4.  Kato  H:  Comparación  de  PET  y  tomografía  computarizada  en  el  uso  de  la  evaluación  de  esófago
carcinoma.  Cáncer  15:921­928,  2002.

5.  Kelsen  DP:  Resultados  a  largo  plazo  del  ensayo  RTOG  8911  (Intergrupo  113  de  EE.  UU.):  una  comparación  de  ensayo  de  asignación  
aleatoria  de  quimioterapia  seguida  de  cirugía  en  comparación  con  cirugía  sola  para  el  cáncer  de  esófago.  J  Clin  Oncol  25:  3719­3725,  
2007.

6.  Malaisrie  SC:  La  adición  de  quimioterapia  de  inducción  a  la  quimiorradioterapia  concurrente  preoperatoria  mejora
Respuesta  tumoral  en  pacientes  con  adenocarcinoma  de  esófago.  Cáncer  107:967­974,  2006.

7.  Grupo  de  Cáncer  de  Esófago:  Resección  quirúrgica  con  o  sin  quimioterapia  preoperatoria  en  esófago
cáncer.  Lancet  359:1727­1733,  2002.

8.  Overholt  BF:  Terapia  fotodinámica  en  el  manejo  del  esófago  de  Barrett  con  displasia.  J  Gastrointestinal
Surg  4:129­130,  2002.

9.  Reed  C:  Técnicas  de  cirugía  esofágica.  Chest  Surg  Clin  North  Am  5:379­574,  1995.

10.  Shumaker  DA:  Impacto  potencial  de  la  EUS  preoperatoria  en  el  manejo  y  costo  del  cáncer  de  esófago.  Gastrointestinal
Endosc  56:391­396,  2002.

11.  Swaroop  VS:  Re:  Pautas  de  práctica  para  el  cáncer  de  esófago.  Am  J  Gastroenterol  94:2319­2320,  1999.

12.  Urba  S:  Terapia  de  modalidad  combinada  para  el  cáncer  de  esófago:  ¿estándar  de  atención?  Cirugía  Oncológica  Clin  North  Am
11:377­386,  2002.

13.  Walsh  T:  Una  comparación  de  terapia  multimodal  y  cirugía  para  el  adenocarcinoma  esofágico.  N  Inglés  J  Med
335:462­467,  1996.
Machine Translated by Google

ENFERMEDAD  ÚLCERA  ÁCIDO­PÉPTICA
CAPÍTULO  
44

Erik  D.  Peltz,  DO  y  Thomas  N.  Robinson,  MD

ENFERMEDAD  DE  ÚLCERA  DUODENAL

1.  ¿Cuál  es  el  riesgo  de  enfermedad  de  úlcera  duodenal?

El  riesgo  de  por  vida  de  úlcera  duodenal  es  de  aproximadamente  1  en  14.  Por  lo  general,  ocurre  entre  los  20  y  los  60  
años,  con  una  incidencia  máxima  en  la  cuarta  década  de  la  vida.  Es  más  común  en  los  hombres.
La  hemorragia  es  la  causa  más  frecuente  de  ingreso  hospitalario.  El  número  anual  de  muertes  en  los  Estados  Unidos  es  
de  unas  10.000  muertes  causadas  por  úlceras  duodenales.

2.  ¿Cuál  es  el  papel  de  Helicobacter  pylori  en  la  úlcera  duodenal?
H.  pylori,  un  bacilo  gramnegativo,  está  fuertemente  asociado  con  la  enfermedad  de  úlcera  péptica.  Se  aísla  de  la  mucosa  
antral  en  el  80%  de  los  pacientes  con  úlcera  péptica.  Pueden  ocurrir  úlceras  en  ausencia  de  H.  pylori.  Estas  úlceras  ocurren  
en  el  contexto  de  secreción  hiperácida,  secreción  ácida  normal  o  después  de  operaciones  de  reducción  de  ácido  como  la  
vagotomía.  Las  ulceraciones  recurrentes  o  múltiples  pueden  indicar  una  enfermedad  endocrina  subyacente.  La  ruptura  de  la  
barrera  de  la  mucosa  duodenal  probablemente  también  contribuya  a  la  ulcerogénesis.

3.  ¿Es  necesaria  la  hipersecreción  ácida  para  la  úlcera  péptica?
No.  La  hipersecreción  gástrica  de  ácido  y  pepsina  juega  un  papel  importante  en  la  formación  de  úlceras;  sin  embargo,  
solo  el  40%  de  los  que  padecen  úlceras  manifiestan  hipersecreción  ácida.

4.  ¿Cuáles  son  las  complicaciones  clínicamente  importantes  de  la  infección  por  H.  pylori?
Enfermedad  de  úlcera  péptica:  como  se  señaló,  H.  pylori  está  presente  en  el  80%  de  las  úlceras  pépticas.  En  cambio,
El  50%  de  la  población  general  alberga  este  organismo.  En  las  personas  infectadas  con  H.  pylori,  el  riesgo  de  por  vida  
de  desarrollar  una  úlcera  péptica  es  del  10%  al  20%.  H.  pylori  puede  ser  parte  de  la  flora  gástrica  humana  autóctona;  
Se  detectaron  antígenos  en  momias  precolombinas  centroamericanas  cuya  última  comida  fue  hace  1700  años.

Carcinoma  gástrico:  H.  pylori  está  fuertemente  relacionado  con  el  cáncer  gástrico  y  ahora  se  clasifica  como  carcinógeno  
del  grupo  I.  También  puede  causar  linfoma  de  tejido  linfoide  asociado  a  mucosas  (MALT).

El  esófago  de  Barrett  es  una  posible  enfermedad  asociada  con  H.  pylori,  aunque  es  más  común
asociado  con  reflujo  gastroesofágico  crónico.
Es  probable  que  H.  pylori  tenga  sinergia  con  el  uso  de  fármacos  antiinflamatorios  no  esteroideos  (AINE).

5.  ¿Cuál  es  la  prueba  más  utilizada  para  H.  pylori?
La  prueba  CLO  detecta  la  presencia  de  H.  pylori.  H.  pylori  libera  ureasa,  que  descompone  la  urea  en  amoníaco  y  
bicarbonato,  aumentando  así  el  equilibrio  ácido­base  (pH).  La  prueba  CLO  se  puede  realizar  en  el  momento  de  la  endoscopia  
obteniendo  raspados  de  la  mucosa  antral.
Si  la  endoscopia  no  está  disponible,  se  puede  usar  el  ensayo  inmunoabsorbente  ligado  a  enzimas  (ELISA).
para  detectar  anti­H.  títulos  de  anticuerpos  de  inmunoglobulina  A  (IgA)  e  inmunoglobulina  G  (IgG)  de  pylori.
El  cultivo  directo  del  organismo  debe  reservarse  para  los  casos  en  los  que  la  resistencia  a  los  antibióticos
se  convierte  en  el  problema.

221
Machine Translated by Google
222  CAPÍTULO  44  ENFERMEDAD  ÚLCERA  ÁCIDO­PÉPTICA

6.  ¿Qué  otros  factores  de  riesgo  están  asociados  con  la  enfermedad  de  úlcera  duodenal?
&  Fumar  cigarrillos  es  un  factor  de  riesgo  importante;  su  cese  es  un  componente  clave  de  la  terapia  de  la  úlcera.
  El  grupo  sanguíneo  O  se  asocia  con  una  mayor  incidencia  de  úlcera  duodenal,  al  igual  que  los  leucocitos.
antígenos  HLA­B5,  B12  y  BW35.
  Los  AINE  promueven  la  formación  de  úlceras  al  suprimir  la  producción  sistémica  de  prostaglandinas.
  La  pancreatitis  crónica,  la  cirrosis,  el  enfisema  y  la  deficiencia  de  antitripsina  a1  también  son
asociado  con  la  condición.

7.  ¿Qué  trastorno  endocrino  se  asocia  con  enfermedad  ulcerosa  grave?
Los  pacientes  con  neoplasia  endocrina  múltiple  (NEM)  tipo  I  tienen  una  incidencia  del  75  %  de  gastrinoma  con  
diátesis  ulcerosa  grave.

8.  ¿Qué  otros  trastornos  endocrinos  deben  examinarse?
Se  debe  sospechar  un  tumor  hipofisario  e  hiperparatiroidismo  cuando  se  considera  MEN  tipo  I.

9.  ¿Cuáles  son  las  presentaciones  clínicas  de  la  úlcera  péptica?
  El  dolor  suele  tener  un  origen  epigástrico,  aunque  la  radiación  a  la  espalda  puede  indicar  afectación  
pancreática.  A  menudo  se  alivia  con  la  ingestión  de  alimentos  o  antiácidos.  Pueden  ocurrir  náuseas  y  vómitos.

  Hemorragia  digestiva  alta  (GI).  (Las  úlceras  pépticas  son  responsables  del  28  %  al  59  %  de  todas  las  
hemorragias  digestivas  altas)
  La  obstrucción  de  la  salida  gástrica  (GOO)  puede  deberse  a  espasmo  pilórico,  constricción  de  una  
masa  inflamatoria,  cicatrización  duodenal  o  fibrosis.
  La  perforación  es  una  urgencia  quirúrgica  con  una  tasa  de  mortalidad  de  hasta  el  10%.  La  perforación  
puede  ocurrir  sin  antecedentes  de  úlcera  péptica,  especialmente  si  la  úlcera  está  situada  en  la  superficie  
anterior  del  duodeno.

10.  ¿Cómo  afecta  la  localización  de  la  úlcera  a  su  presentación  clínica?
Las  úlceras  de  la  pared  anterior  (generalmente  la  primera  porción  del  duodeno)  pueden  perforar  y  causar  
peritonitis  con  aire  libre  en  el  abdomen.  Las  úlceras  posteriores  pueden  erosionar  la  arteria  gastroduodenal  o  el  
páncreas.

11.  ¿Cuáles  son  los  diagnósticos  diferenciales  del  dolor  epigástrico?
Además  de  la  enfermedad  de  úlcera  péptica,  se  debe  considerar  la  enfermedad  por  reflujo  gastroesofágico  
(ERGE),  gastritis,  carcinoma  gástrico,  enfermedad  de  las  vías  biliares,  pancreatitis  o  carcinoma  pancreático,  
aneurisma  aórtico,  angina  intestinal  (isquemia)  e  isquemia  miocárdica.

12.  ¿Qué  prueba  inicial  se  debe  realizar  al  evaluar  el  dolor  epigástrico  de  presunto  origen  gastrointestinal?

Se  prefiere  la  esofagogastroduodenoscopia  flexible  (EGD),  aunque  el  estudio  de  contraste  GI  superior  con  bario  
puede  ser  aceptable.  La  prueba  CLO  se  puede  realizar  en  el  momento  de  la  EGD  si  está  indicado.  Se  debe  realizar  
una  ecografía  (US)  si  se  sospechan  enfermedades  vasculares  o  de  la  vesícula  biliar.  Se  debe  obtener  un  angiograma  
de  vista  lateral  para  la  angina  intestinal,  una  tomografía  computarizada  (TC)  para  un  aneurisma  y  un  
electrocardiograma  (ECG)  de  referencia  porque  la  cardiopatía  isquémica  siempre  es  posible.

13.  ¿Cómo  se  tratan  los  pacientes  con  úlcera  duodenal?
  Dieta:  deben  suspenderse  la  aspirina  y  los  AINE.  El  alcohol  y  la  nicotina  deben  ser
evitado
  Antiácidos:  la  neutralización  del  pH  gástrico  puede  aliviar  los  síntomas,  pero  su  impacto  en  la  cicatrización  
de  la  úlcera  no  está  bien  definido.
  Antagonistas  de  los  receptores  H2:  el  uso  de  cimetidina  o  ranitidina  previene  la  acidez  gástrica.
secreciones  al  bloquear  el  receptor  de  histamina  H2.
Machine Translated by Google
CAPÍTULO  44  ENFERMEDAD  ÚLCERA  ÁCIDO­PÉPTICA  223

  Sucralfato:  un  medicamento  de  barrera  protectora  que  se  adhiere  a  la  base  de  la  úlcera  y  proporciona  una  
capa  protectora.  Los  medicamentos  que  disminuyen  la  secreción  de  ácido  no  deben  usarse  al  mismo  
tiempo  porque  el  sucralfato  requiere  un  ambiente  ácido  para  activarse.
Inhibidores  de  la  bomba  de  protones  (IBP):  el  omeprazol  bloquea  la  bomba  de  adenosina  trifosfatasa  de  
hidrógeno  y  potasio  en  las  células  parietales  gástricas  e  inhibe  la  liberación  de  iones  de  hidrógeno.
Por  lo  general,  se  reserva  para  los  fracasos  de  la  terapia  de  primera  línea  (es  decir,  antagonistas  de  los  receptores  H2).

&  Erradicación  de  H.  pylori:  Si  se  diagnostica  infección  por  H.  pylori,  la  combinación  de  triple
la  terapia  (bismuto,  tetraciclina  y  metronidazol)  con  un  régimen  antagonista  del  receptor  H2  parece  proporcionar  
una  tasa  de  curación  de  90%.  Se  puede  agregar  eritromicina,  amoxicilina­omeprazol  o  eritromicina­omeprazol  
para  las  fallas  iniciales.

14.  ¿Cuáles  son  las  tasas  de  recurrencia  después  de  la  terapia  médica?
Aproximadamente  el  80%  de  las  úlceras  duodenales  curan  en  6  semanas.  La  tasa  de  recurrencia  dentro  de  1  año  de  
tratamiento  es  del  70%;  por  lo  tanto,  puede  ser  necesario  repetir  el  tratamiento.

15.  ¿Qué  complicaciones  se  asocian  con  el  tratamiento  médico?
Los  antagonistas  de  los  receptores  H2  pueden  inducir  cambios  en  el  estado  mental  y  ginecomastia.  La  cimetidina,  en  
particular,  puede  afectar  el  metabolismo  hepático  de  la  warfarina,  la  fenitoína,  la  teofilina,  el  propranolol  y  la  digoxina,  
lo  que  lleva  a  niveles  séricos  anormales.  El  omeprazol  puede  causar  hipergastrinemia  al  bloquear  la  secreción  de  
ácido  gástrico.  Puede  desarrollarse  resistencia  de  H.  pylori  a  los  antibióticos,  especialmente  al  metronidazol;  por  lo  
tanto,  se  recomienda  una  combinación  triple  de  al  menos  dos  antimicrobianos  con  un  fármaco  inhibidor  de  ácido  como  
terapia  inicial.

16.  ¿Cómo  se  deben  evaluar  las  úlceras  recurrentes  o  múltiples?
Además  de  los  estudios  antes  mencionados,  se  deben  obtener  los  niveles  de  gastrina  sérica  para  evaluar  posibles  
trastornos  endocrinos.  Los  pacientes  no  deben  tomar  omeprazol  cuando  se  miden  los  niveles  de  gastrina.  En  el  
síndrome  de  Zollinger­Ellison,  la  hipersecreción  de  gastrina  del  tumor  de  los  islotes  pancreáticos  produce  úlceras  
múltiples  o  intratables  (gastrina  sérica  normal,  <200  pg/ml;  síndrome  de  Zollinger­Ellison,  por  lo  general  >500  pg/ml).

17.  ¿Cómo  evalúa  un  valor  límite  de  gastrina  sérica  (200  a  500  pg/ml)?
La  prueba  de  estimulación  con  secretina  puede  usarse  para  diagnosticar  el  síndrome  de  Zollinger­Ellison.  
Un  bolo  intravenoso  (IV)  de  secretina  (2  U/kg)  debe  resultar  en  un  aumento  de  gastrina  de  150  pg/ml  en  15  
minutos  si  el  paciente  tiene  este  síndrome.

18.  ¿Cuáles  son  las  indicaciones  para  el  tratamiento  quirúrgico  de  las  úlceras  duodenales?
Las  indicaciones  habituales  son  el  fracaso  del  tratamiento  médico  para  controlar  el  dolor,  el  sangrado  (<6  
unidades  de  transfusiones  de  glóbulos  rojos  concentrados  en  24  horas  o,  mejor  aún,  dos  tercios  de  la  pérdida  de  
volumen  de  sangre  calculada  del  paciente  en  24  horas)  y  la  obstrucción.  La  perforación  de  la  úlcera  generalmente  se  
trata  quirúrgicamente  a  menos  que  el  paciente  se  presente  24  horas  después  del  evento  sin  peritonitis  y  la  serie  GI  
superior  de  Gastrografin  confirme  que  la  perforación  ha  sido  bien  sellada  (generalmente  con  epiplón).

19.  ¿Qué  operaciones  se  utilizan  para  tratar  las  úlceras  duodenales?
  Vagotomía  troncal  y  piloroplastia  (V  y  P)  o  gastroyeyunostomía.
  Vagotomía  troncal  y  antrectomía  con  anastomosis  Billroth  I  o  II.
  Gastrectomía  subtotal  con  anastomosis  Billroth  I  o  II.
  Vagotomía  selectiva  (sólo  las  ramas  vagales  a  las  células  parietales  en  el  estómago).
&  Gastrectomía  total.

20.  ¿Qué  son  las  anastomosis  Billroth  I  y  Billroth  II?

La  operación  de  Billroth  I  es  una  anastomosis  entre  el  duodeno  y  el  remanente  gástrico  (gastroduodenostomía).  La  
operación  Billroth  II  se  construye  cosiendo  un  asa  de  yeyuno  al  remanente  gástrico  (gastroyeyunostomía).  Cualquier  
método  es  aceptable.
Machine Translated by Google
224  CAPÍTULO  44  ENFERMEDAD  ÚLCERA  ÁCIDO­PÉPTICA

21.  ¿Qué  procedimiento  se  prefiere,  Billroth  I  o  Billroth  II?
Billroth  I  tiene  la  ventaja  de  eliminar  el  muñón  duodenal  y  requerir  solo  una  línea  de  sutura  en  lugar  de  dos  
(como  en  Billroth  II).  El  estallido  del  muñón  duodenal  es  una  emergencia  quirúrgica  crítica  que  requiere  una  
laparotomía  inmediata.  El  síndrome  del  asa  aferente  (es  decir,  acumulación  de  material  en  el  asa  que  no  está  en  la  
corriente  entérica)  también  es  una  complicación  de  Billroth  II.  La  gastritis  por  reflujo  biliar  puede  ocurrir  en  ambos  
procedimientos.  Billroth  I  es  más  fisiológico;  por  lo  tanto,  resulta  en  una  mejor  digestión  de  proteínas  y  grasas.  Billroth  
I  es  más  susceptible  a  la  obstrucción  de  la  salida  gástrica  con  úlcera  o  recurrencia  tumoral;  por  lo  tanto,  no  se  
recomienda  una  conexión  Billroth  I  para  pacientes  con  carcinoma  gástrico.

22.  ¿Qué  es  el  síndrome  del  asa  aferente?
El  dolor  abdominal  posprandial  a  menudo  se  alivia  con  vómitos  biliosos.  Un  estrechamiento  en  la  unión  del  estómago  
y  el  lado  duodenal  de  una  anastomosis  Billroth  II  conduce  a  la  acumulación  de  líquido  biliar  y  pancreático  dentro  de  
la  rama  aferente  del  intestino.  El  dolor  se  alivia  cuando  el  contenido  de  líquido  se  vacía  en  el  estómago,  lo  que  puede  
provocar  vómitos  biliosos  y  gastritis  por  reflujo  grave.

23.  ¿Cómo  se  previene  el  síndrome  del  asa  aferente?
La  prevención  requiere  evitar  una  rama  aferente  larga  o  torcida  con  una  anastomosis  demasiado  estrecha  durante  la  
construcción  de  Billroth  II.  Un  procedimiento  Billroth  I  elimina  este  posible  problema.

24.  ¿Quién  fue  Billroth?
Christian  Albert  Theodor  Billroth  (1829–1894)  fue  un  cirujano  austriaco  al  que  se  le  atribuye  la  realización  
de  la  primera  resección  gástrica  exitosa  en  1881  y  la  introducción  de  innovaciones  en  la  cirugía  de  derivación  
intestinal.  El  padre  de  la  cirugía  estadounidense  moderna,  William  Halsted,  fue  una  vez  aprendiz  de  Billroth  en  
Viena.

25.  ¿Cómo  se  produce  la  gastritis  alcalina  o  por  reflujo  biliar?
El  reflujo  de  la  bilis  y  las  secreciones  pancreáticas  hacia  el  estómago  después  de  una  anastomosis  Billroth  II  (a  veces  
Billroth  I)  puede  causar  irritación  gástrica  marcada,  lo  que  lleva  a  dolor  posprandial  crónico.
El  dolor  persistente  se  debe  evaluar  con  endoscopia  y  se  debe  considerar  la  reconstrucción  quirúrgica,  por  lo  
general  con  una  gastroyeyunostomía  en  Y  de  Roux  de  una  rama  yeyunal  eferente  de  40  cm.

26.  ¿Qué  es  la  vagotomía  selectiva?
En  esta  vagotomía  proximal  limitada,  las  células  parietales  gástricas  se  denervan  selectivamente.  Las  fibras  del  antro,  
el  píloro,  el  hígado,  el  tracto  biliar  y  el  resto  del  tracto  intestinal  se  dejan  intactas,  lo  que  evita  la  necesidad  de  un  
procedimiento  de  vaciado  gástrico.  La  recurrencia  de  la  enfermedad  ulcerosa  puede  ser  del  10  %  o  mayor,  pero  sus  
efectos  secundarios,  a  saber,  el  vaciado  (causado  por  la  resección  del  píloro)  o  la  diarrea  (causada  por  la  vagotomía),  
se  reducen  al  2  %.

27.  ¿Qué  es  el  síndrome  de  dumping?
La  resección  del  píloro  puede  conducir  a  un  vaciado  rápido  e  incontrolado  del  contenido  gástrico  hiperosmolar  
hacia  el  intestino  delgado  proximal.  La  carga  osmótica  y  de  glucosa  en  el  intestino  absorbe  el  volumen  intravascular  
hacia  el  intestino,  lo  que  hace  que  el  paciente  tenga  hipovolemia  transitoria.  La  respuesta  adrenérgica  
fisiológicamente  apropiada  a  este  cambio  de  volumen  produce  taquicardia,  sudoración,  sofocos,  debilidad,  
náuseas,  calambres  abdominales  e  incluso  síncope.  Ingerir  una  comida  pequeña,  seca  y  baja  en  carbohidratos  (para  
limitar  los  osmoles  disponibles)  puede  prevenir  este  síndrome.
Los  medicamentos  anticolinérgicos  también  pueden  ayudar.  Hasta  el  20%  de  los  pacientes  experimentan  el  
síndrome  de  dumping  en  el  período  posoperatorio  temprano,  pero  solo  el  2%  desarrolla  problemas  crónicos.

28.  ¿Qué  debe  acompañar  a  la  vagotomía  troncal?
La  vagotomía  troncal  desnerva  el  estómago,  lo  que  provoca  hipomotilidad  gástrica.  Se  debe  realizar  algún  
procedimiento  de  vaciado  gástrico  como  una  piloroplastia  o  una  gastroduodenostomía  laterolateral.
Machine Translated by Google
CAPÍTULO  44  ENFERMEDAD  ÚLCERA  ÁCIDO­PÉPTICA  225

29.  ¿Qué  es  una  piloroplastia  de  Heinecke­Mikulicz?
Una  incisión  piloduodenal  a  lo  largo  del  eje  longitudinal  seguida  de  un  cierre  transversal  abre  el  píloro  y  promueve  el  
vaciamiento  gástrico.

30.  ¿Qué  es  una  piloroplastia  de  Finney?
Una  anastomosis  gastroduodenal  de  lado  a  lado  que  corta  y  desfuncionaliza  el  píloro  y  promueve  el  vaciamiento  gástrico.

31.  ¿Qué  es  una  piloroplastia  de  Jaboulay?
Este  procedimiento  de  vaciado  gástrico  comprende  una  anastomosis  gastroduodenal  de  lado  a  lado  que  no  corta  el  
píloro.  Es  ideal  si  existe  una  cicatrización  pilórica  severa.

32.  ¿Cuáles  son  las  tasas  de  recurrencia  de  la  úlcera  después  del  tratamiento  quirúrgico?
Vagotomía  y  piloroplastia:  10%.
Vagotomía  y  antrectomía:  2%.
Vagotomía  altamente  selectiva:  10%.
Gastrectomía  subtotal:  1%.
Gastrectomía  total:  <1%.

33.  ¿Cuál  es  la  tasa  de  mortalidad  de  estas  operaciones?
Vagotomía  y  piloroplastia:  1%.
Vagotomía  y  antrectomía:  2%.
Vagotomía  altamente  selectiva:  0,1%.
Gastrectomía  subtotal:  2%.
Gastrectomía  total:  5%.

34.  ¿Cómo  se  tratan  los  pacientes  con  úlceras  duodenales  perforadas?
Primero  se  debe  reanimar  al  paciente,  siguiendo  el  ABC  de  las  vías  respiratorias,  la  respiración  y  la  circulación.
El  contenido  del  estómago  se  vacía  a  través  de  una  sonda  nasogástrica  (NG).  El  cierre  quirúrgico  con  parche  omental  
(cierre  de  Graham)  se  practica  ampliamente.  Para  pacientes  hemodinámicamente  estables,  es  apropiada  la  sutura  de  la  
úlcera  seguida  de  una  vagotomía  selectiva.  La  antrectomía  con  vagotomía  para  extirpar  la  úlcera  es  apropiada  si  el  
paciente  tiene  una  úlcera  péptica  intratable.

35.  ¿Qué  (pregunta  específica  sobre  la  úlcera)  debe  hacer  siempre  antes  de  pasar  al  quirófano?

Antecedentes  de  enfermedad  ulcerosa.  La  elección  de  la  operación  dependerá  de  la  enfermedad  ulcerosa  aguda  o  crónica.

36.  ¿Cuál  es  el  resultado  a  largo  plazo  después  del  cierre  de  Graham  de  una  úlcera  perforada?
Un  tercio  de  los  pacientes  permanece  asintomático,  un  tercio  tiene  síntomas  controlados  con  tratamiento  médico  y  un  
tercio  requiere  una  operación  adicional  de  úlcera.

37.  ¿Cuáles  son  las  complicaciones  de  la  cirugía  de  las  úlceras  duodenales?
La  fuga  del  muñón  duodenal  puede  ocurrir  dentro  de  la  primera  semana  después  de  la  resección  antral  y  la  anastomosis  
de  Billroth.  El  tratamiento  consiste  en  una  reintervención  inmediata  para  drenar  y  controlar  la  fuga.
La  nutrición  parenteral  total  puede  ser  necesaria  como  complemento  del  «descanso  intestinal».
La  retención  gástrica  puede  ocurrir  por  edema  en  la  anastomosis  o  atonía  del  estómago  después  de  la  vagotomía.  Por  
lo  general,  se  resuelve  espontáneamente  en  3  a  4  semanas.
El  sangrado  puede  ocurrir  por  una  línea  de  sutura,  una  úlcera  pasada  por  alto  u  otras  lesiones  de  la  mucosa  gástrica.
La  mayor  parte  del  sangrado  posterior  a  la  gastrectomía  cesa  espontáneamente,  pero  en  algunos  casos  puede  ser  
necesaria  la  endoscopia.

38.  ¿Dónde  reaparecen  las  úlceras  después  de  la  operación?
Las  úlceras  suelen  recidivar  junto  a  la  anastomosis  gástrica  en  el  lado  intestinal  (es  decir,  yeyuno,  duodeno).
Machine Translated by Google
226  CAPÍTULO  44  ENFERMEDAD  ÚLCERA  ÁCIDO­PÉPTICA

39.  ¿Por  qué  se  repiten?
Los  factores  responsables  son  resección  gástrica  inadecuada,  vagotomía  incompleta,  drenaje  inadecuado  del  
remanente  gástrico  (estasis  del  contenido  gástrico  proximal  a  la  anastomosis)  o  antro  gástrico  retenido  (células  
productoras  de  gastrina)  después  de  un  procedimiento  Billroth  II.

40.  ¿Cómo  se  trata  la  estenosis  pilórica?
Se  debe  iniciar  la  reanimación  con  líquidos  y  la  descompresión  de  la  sonda  NG.  La  alcalosis  metabólica  puede  
resultar  de  vómitos  prolongados  (pérdida  de  iones  de  hidrógeno)  y  debe  corregirse  con  una  infusión  de  solución  salina  
normal.  Es  aceptable  la  vagotomía  con  gastroyeyunostomía  o  la  resección  de  la  estenosis  con  un  bypass  Billroth  II.  
La  gastrectomía  parcial  se  requiere  con  menos  frecuencia.

ENFERMEDAD  ÚLCERA  GÁSTRICA

41.  ¿Cuál  es  el  factor  más  importante  en  el  manejo  de  las  úlceras  gástricas?
Todas  las  úlceras  gástricas  deben  ser  evaluadas  para  malignidad.  La  incidencia  de  malignidad  es  de  alrededor  del  10%.

42.  ¿Cómo  se  evalúa  la  úlcera  gástrica?
La  biopsia  es  obligatoria.  La  EGD  con  múltiples  biopsias  (típicamente,  seis)  del  cráter  de  la  úlcera  es  el  mejor  método.  
La  serie  GI  superior  puede  ser  útil,  pero  la  biopsia  no  es  posible.  La  prueba  CLO  se  puede  realizar  en  el  momento  de  
la  EGD  para  detectar  H.  pylori.  Las  úlceras  benignas  generalmente  sanan  a  las  12  semanas.
La  intratabilidad  debe  despertar  la  sospecha  de  malignidad.

43.  En  pacientes  con  úlceras  pépticas  sangrantes,  ¿cuáles  son  los  hallazgos  endoscópicos  sugestivos  de  resangrado?

Hallazgo  endoscópico   Riesgo  de  resangrado  (%)   Clasificación  de  Forrest

Sangrado  activo   90%  55% I  A

Exudación  activa  Vaso   BI

visible  que  no  sangra  Coágulo   50%   IIA


adherente 33% IIB

Mancha  roja  o  azul  en  la  superficie 7%   CII


fondo  limpio 3% tercero

44.  ¿Cómo  se  clasifican  las  úlceras  gástricas?
Tipo  I   En  la  incisura  o  porción  más  inferior  de  la  curvatura  menor  Úlcera  gástrica  þ  
Tipo  II   úlcera  duodenal  Prepíloro  Unión  gastroesofágica  o  cardias  proximal  Cualquier  

Tipo  III   úlcera  por  uso  de  AINE  o  aspirina.
Tipo  IV  
Tipo  V

45.  ¿Cuál  es  el  tipo  de  úlcera  gástrica  más  común?
Tipo  i.

46.  ¿En  qué  se  diferencian  las  úlceras  gástricas  benignas  de  las  úlceras  duodenales?
Las  úlceras  gástricas  benignas  son  difíciles  de  tratar  y  tienen  una  mayor  tasa  de  recurrencia  y  
complicaciones.  La  úlcera  gástrica  y  el  carcinoma  gástrico  tienen  un  probable  factor  etiológico  común,  que  es  la  gastritis  
atrófica  inducida  por  H.  pylori.  Por  el  contrario,  los  factores  asociados  con  la  úlcera  duodenal  pueden  proteger  contra  el  
cáncer  gástrico.

47.  ¿Cómo  se  relaciona  H.  pylori  con  la  enfermedad  de  úlcera  gástrica?
La  colonización  por  H.  pylori  induce  gastritis  crónica  activa,  que  se  asocia  con  la  formación  de  úlceras,  aunque  
no  se  ha  establecido  claramente  una  relación  directa  de  causa  y  efecto.  Otro
Machine Translated by Google
CAPÍTULO  44  ENFERMEDAD  ÚLCERA  ÁCIDO­PÉPTICA  227

factores  como  el  defecto  focal  en  la  neutralización  del  ácido  que  permite  la  difusión  del  ácido  en  la  mucosa  del  estómago  

o  la  hipersecreción  de  ácido  (en  casos  de  úlceras  de  tipo  II  y  III)  pueden  desempeñar  un  papel  importante.

48.  ¿Qué  es  una  "prueba  de  curación"?
Una  combinación  de  antagonistas  de  los  receptores  H2  o  inhibidores  de  la  bomba  de  hidrógeno  con  anti­H.  los  
medicamentos  para  pylori,  si  están  indicados,  pueden  probarse  durante  6  a  12  semanas.  Se  debe  realizar  una  segunda  
EGD  para  evaluar  la  úlcera.  Se  acepta  una  prueba  adicional  de  12  semanas  siempre  que  los  resultados  de  la  biopsia  
para  malignidad  sean  negativos.

49.  ¿Cuál  es  el  objetivo  de  la  erradicación  de  H.  pylori  en  el  contexto  de  la  úlcera  gástrica?
La  terapia  dirigida  a  la  erradicación  de  H.  pylori  se  asocia  con  una  mayor  cicatrización  de  la  úlcera  y  una  menor  recidiva  
de  la  úlcera.  Varias  series  han  mostrado  disminuciones  en  las  recurrencias  de  50%  a  <10%  con  la  erradicación  de  H.  
pylori.  H.  pylori  está  fuertemente  relacionado  con  el  cáncer  gástrico  y  ahora  se  clasifica  como  carcinógeno  del  grupo  I.  
También  puede  causar  linfoma  MALT.

50.  ¿Cómo  se  tratan  los  pacientes  con  infección  por  H.  pylori?
Deben  recibir  una  terapia  triple  de  bismuto,  metronidazol  y  tetraciclina,  generalmente  complementada  con  
medicamentos  reductores  de  ácido.

51.  ¿La  cicatrización  de  la  úlcera  gástrica  garantiza  una  úlcera  benigna?
No.  Las  úlceras  gástricas  con  focos  de  malignidad  pueden  curarse  completamente  con  tratamiento  médico.

52.  ¿Cuáles  son  las  indicaciones  para  el  tratamiento  quirúrgico  de  las  úlceras  gástricas  benignas?
Hemorragia,  perforación,  obstrucción  e  intratabilidad  (igual  que  las  úlceras  duodenales).

53.  ¿Cuál  es  el  procedimiento  definitivo  para  las  úlceras  gástricas  benignas?
La  hemigastrectomía  o  antrectomía  (incluida  la  úlcera)  sin  vagotomía  para  las  úlceras  de  tipo  I  y  IV  es  el  
procedimiento  estándar.  Las  úlceras  tipo  I  y  IV  tienen  niveles  de  ácido  bajos  o  normales.  Para  los  tipos  II  y  III,  se  debe  
agregar  la  vagotomía.

54.  ¿Cuáles  son  las  opciones  bajo  emergencia  (es  decir,  hemorragia  o  perforación)?
¿condiciones?

Hemodinámicamente  estable:  vagotomía  troncal  y  gastrectomía  distal.  Los  pacientes  hemodinámicamente  estables  
con  hemorragia  digestiva  alta  por  úlceras  pépticas  también  pueden  tratarse  con  manejo  endoscópico.  La  EGD  con  
el  uso  de  cauterización,  endoclips  o  inyección  de  vasoconstrictores  o  agentes  esclerosantes  es  eficaz  hasta  en  el  
90%  de  las  úlceras  pépticas  sangrantes.  El  riesgo  de  resangrado  después  de  la  terapia  endoscópica  es  del  15%  
al  20%  y  la  toma  de  decisiones  de  manejo  requiere  evaluación  quirúrgica.  El  resangrado  después  de  un  segundo  
intento  endoscópico  de  hemostasia  es  una  indicación  absoluta  para  la  intervención  quirúrgica.

Inestable:  vagotomía  troncal  y  procedimiento  de  drenaje  con  biopsia  seguida  de  escisión  y  sutura  de  la  úlcera.

55.  ¿Cuál  es  la  tasa  de  resangrado  si  la  úlcera  se  deja  in  situ?
es  el  33%.

56.  ¿Qué  es  la  úlcera  gástrica  gigante?
Úlcera  >3  cm  de  diámetro,  generalmente  localizada  a  lo  largo  de  la  curvatura  menor.  El  riesgo  de  malignidad  es  de  
alrededor  del  30%  y  aumenta  con  el  diámetro.  La  resección  quirúrgica  temprana  está  indicada  debido  al  riesgo  de  
malignidad.  Se  puede  agregar  vagotomía.
Machine Translated by Google
228  CAPÍTULO  44  ENFERMEDAD  ÚLCERA  ÁCIDO­PÉPTICA

57.  ¿Qué  es  la  úlcera  de  Cushing?
Una  úlcera  por  estrés  que  se  encuentra  en  pacientes  gravemente  enfermos  con  lesión  del  sistema  nervioso  central  (SNC).
Típicamente,  único,  profundo  y  con  tendencia  a  perforar.

58.  ¿Qué  es  la  úlcera  de  Curling?
Una  úlcera  por  estrés  que  se  encuentra  en  pacientes  que  están  gravemente  enfermos  con  lesiones  por  quemaduras.

59.  ¿Qué  es  la  úlcera  de  Dieulafoy?
Erosión  de  la  mucosa  gástrica  que  recubre  una  malformación  vascular,  que  a  menudo  conduce  a  hemorragia.  La  
inflamación  crónica  no  está  asociada  con  esta  lesión.

60.  ¿Qué  es  una  úlcera  marginal?
Una  úlcera  que  se  encuentra  cerca  del  margen  de  la  anastomosis  gastroentérica,  generalmente  en  el  lado  del  intestino  delgado.

61.  ¿Cuándo  ocurre  la  gastritis  por  estrés?  ¿Por  qué?
El  sesenta  por  ciento  de  ellos  ocurren  dentro  de  las  24  a  48  horas  posteriores  al  trauma,  shock  o  sepsis.  Por  lo  general,  las  
erosiones  de  la  mucosa  comienzan  proximalmente  en  el  estómago  y  viajan  distalmente.  Estos  finalmente  se  ven  en  casi  todos  
los  pacientes  que  están  gravemente  enfermos.  La  integridad  de  la  barrera  celular  en  la  lámina  propia  está  comprometida,  
probablemente  por  la  disminución  del  suministro  de  sangre,  lo  que  lleva  a  la  retrodifusión  de  ácido;  erosión  de  la  submucosa;  y,  
finalmente,  sangrado.

62.  ¿Cómo  se  tratan  los  pacientes  con  gastritis  por  estrés  sangrante?
Los  coágulos  de  sangre  deben  eliminarse  de  la  luz  del  estómago  mediante  succión  y  lavado  con  sonda  NG.
Las  fibrinolisinas  de  los  coágulos  aumentan  el  sangrado.  El  pH  del  estómago  debe  mantenerse  por  encima  de  4,0  
con  medicamentos  reductores  de  ácido.

PUNTOS  CLAVE:  HELICOBACTER  PYLORI

1.  H.  pylori  es  un  bacilo  gramnegativo  productor  de  ureasa.

2.  Tiene  una  fuerte  asociación  con  la  enfermedad  ulcerosa  péptica  (80%  de  los  pacientes  con  úlcera).

3.  Está  relacionado  con  el  desarrollo  de  linfoma  MALT.

4.  Se  asocia  con  el  desarrollo  de  carcinoma  gástrico.

5.  La  infección  por  H.  pylori  se  diagnostica  mediante  biopsia  EGD  y  prueba  CLO.

6.  El  tratamiento  incluye  terapia  antibiótica  triple  complementada  con  medicamentos  para  reducir  el  ácido
(90%  tasa  de  curación).

SITIO  WEB

www.emedicine.com/med/topic1776.htm

BIBLIOGRAFÍA

1.  Aabakken  L:  Tratamiento  endoscópico  y  farmacológico  actual  de  la  hemorragia  por  úlcera  péptica.  Best  Pract  Res  Clin
Gastroenterol  22(2):243­259,  2008.
2.  Calam  J,  Baron  JH:  ABC  del  tracto  gastrointestinal  superior:  fisiopatología  de  la  úlcera  duodenal  y  gástrica  y
cáncer  gástrico.  Br  Med  J  323:980,  2001.
Machine Translated by Google
CAPÍTULO  44  ENFERMEDAD  ÚLCERA  ÁCIDO­PÉPTICA  229

3.  Correa  P,  Willis  D,  Allison  MJ  et  al.:  Helicobacter  pylori  en  momias  precolombinas.  Gastroenterología
114  (suplemento  4):  A956,  1998.

4.  Hansson  LE,  Nyren  O,  Hsing  AW  et  al.:  El  riesgo  de  cáncer  de  estómago  en  pacientes  con  úlcera  gástrica  o  duodenal
enfermedad.  N  Engl  J  Med  335:242,  1996.

5.  Kokoska  ER,  Kauffman  GL:  Helicobacter  pylori  y  la  mucosa  gastroduodenal.  Cirugía  130:13,  2001.

6.  Leung  WK,  Graham  DY:  Úlcera  y  gastritis.  Endoscopia  33:8,  2001.

7.  Palmer  K:  Hemorragia  gastrointestinal  superior  aguda.  British  Med  Bull  83:307,  2007.

8.  Rollhauser  C,  Fleischer  DE:  Hemorragia  digestiva  alta  no  varicosa.  Endoscopia  34:111,  2002.

9.  Rosin  D,  Rosenthal  RJ,  Bonner  G  et  al .:  Linfoma  MALT  gástrico  en  un  paciente  negativo  para  Helicobacter  pylori:  un
Reporte  de  un  caso  y  reseña  de  literatura.  J  Am  Coll  Surg  192:652,  2001.

10.  Schwesinger  WH,  Page  CP,  Sirinek  KR  et  al .:  Operaciones  para  la  enfermedad  de  úlcera  péptica:  pérdida  de  paradigma.  J  Gastrointestinal
Surg  5:438,  2001.

11.  Talamini  G,  Tommasi  M,  Vantini  I  et  al .:  Factores  de  riesgo  de  úlcera  péptica  en  4943  pacientes  hospitalizados.  J  Clin  Gastroenterol
42(4):373­380,  2008.

12.  van  Lanschot  JJ,  van  Leerdam  M,  van  Delden  OM  et  al.:  Manejo  de  las  úlceras  gastroduodenales  sangrantes.
Dig  Surg  19:99,  2002.

13.  van  Leerdam  M:  Epidemiología  de  la  hemorragia  digestiva  alta  aguda.  Mejor  práctica  Res  Clin  Gastroenterol
22(2):209­224,  2008.
Machine Translated by Google

OBSTRUCCIÓN  DEL  INTESTINO  DELGADO
CAPÍTULO  
45

Elizabeth  L.  Cureton,  MD  y  Joyce  A.  Majure,  MD

1.  Mencione  tres  mecanismos  de  obstrucción  intestinal  y  dé  ejemplos  y
incidencia  de  cada  tipo.
1.  Compresión  extrínseca:  adherencias  (60%),  malignidad  (20%),  hernias  (10%),  vólvulo  y
otros  (5%).
2.  Obstrucción  interna  de  la  luz  por  materiales  anormales  (obturación):  bezoares,  cálculos  biliares,  gusanos  o  
cuerpo  extraño  (generalmente  obstruye  la  válvula  ileocecal).
3.  Enfermedad  mural  que  invade  la  luz  (enfermedad  inflamatoria  intestinal  [5%]),  estenosis  fibrosa  secundaria  
a  traumatismo,  isquemia  o  radiación,  intususcepción).

2.  ¿Cuáles  son  los  síntomas  más  comunes  de  la  obstrucción  del  intestino  delgado  (SBO)?
1.  Dolor  abdominal:  inicialmente  inespecífico,  a  menudo  cólico,  coincidiendo  con  ondas  de  peristalsis  que  
intentan  pasar  el  punto  de  obstrucción.
2.  Meteorismo:  cuanto  más  distal  es  la  obstrucción,  más  grave  es  la  distensión  abdominal  provocada  por  
la  dilatación  del  intestino  proximal.
3.  Vómitos:  biliosos,  frecuentes  y  profusos  con  obstrucción  proximal,  menos  frecuentes  pero  de  mayor  
volumen  ya  menudo  feculentos  con  obstrucción  distal.
4.  Obstipación:  falta  de  evacuación  de  gases  o  heces;  en  ocasiones,  el  paciente  tiene  algunas  heces  
blandas  al  principio,  a  medida  que  se  vacía  el  intestino  distal  a  la  obstrucción.

3.  ¿Cuáles  son  las  preguntas  pertinentes  en  la  historia  del  paciente?
&  ¿Alguna  cirugía  abdominal  o  pélvica  previa?
&  Cualquier  SBO  anterior?
&  ¿Algún  historial  de  cáncer?  ¿Qué  tipo  y  cómo  se  trata?  ¿Alguna  radiación?
  ¿Alguna  infección  o  inflamación  abdominal  previa  (incluye  enfermedad  pélvica  inflamatoria  [EPI],  apendicitis,  
diverticulitis,  enfermedad  inflamatoria  intestinal,  perforación  y  traumatismo)?
&  ¿Alguna  historia  de  cálculos  biliares?
&  Medicamentos  actuales,  particularmente  anticoagulantes,  anticolinérgicos,  quimioterapia  o
diuréticos?

4.  ¿Cuáles  son  los  hallazgos  en  el  examen  físico?
El  paciente  a  menudo  está  deshidratado  y  puede  tener  febrícula,  hipotensión  postural  y  distensión  abdominal.  
Los  ruidos  intestinales  pueden  ser  hiperactivos  con  "tintineos  y  ráfagas"  o  pueden  ser  totalmente  silenciosos  si  el  
paciente  se  ha  demorado  en  buscar  tratamiento.  La  percusión  suele  revelar  tímpanos  difusos,  y  los  pacientes  
ancianos  delgados  pueden  incluso  tener  asas  visibles  de  intestino  delgado  distendido.
La  palpación  puede  aumentar  el  dolor  abdominal,  pero  la  sensibilidad  localizada  o  los  signos  peritoneales  indican  
un  posible  estrangulamiento  u  otro  diagnóstico.

5.  ¿Es  necesario  un  tacto  rectal?
Absolutamente.  El  examen  rectal  puede  revelar  signos  de  cáncer,  como  una  plataforma  rectal  rígida  por  
carcinomatosis,  y  la  sangre  en  el  examen  hemoculto  puede  anunciar  isquemia  o  estrangulación  o  puede  
indicar  enfermedad  inflamatoria  intestinal.  Una  hernia  obturatriz  puede  palparse  mejor  por  vía  transrectal  o  
transvaginal.

230
Machine Translated by Google
CAPÍTULO  45  OBSTRUCCIÓN  DEL  INTESTINO  DELGADO  231

6.  ¿Dónde  debe  buscar  el  examinador  las  hernias  obstructivas?
Examine  las  ingles  cerca  del  tubérculo  púbico  ya  lo  largo  del  piso  inguinal,  revise  los  triángulos  femorales  en  busca  de  
protuberancias  o  sensibilidad,  realice  un  examen  rectal  para  buscar  una  hernia  obturatriz  (consulte  la  pregunta  5)  y  palpe  
todas  las  incisiones  existentes.  Compruebe  todos  los  sitios  de  trocar  de  cirugías  laparoscópicas  anteriores.

7.  ¿Cuál  es  la  forma  más  económica  de  confirmar  el  diagnóstico?
La  «serie  abdominal  de  cuatro  vías» (radiografías  abdominales  planas  y  verticales,  más  radiografías  posterolaterales  [PA]  
y  laterales  de  tórax)  es  diagnóstica  aproximadamente  el  75%  de  las  veces.  Buscar:
  Niveles  hidroaéreos  en  el  intestino  delgado  dilatado  (también  conocido  como  signo  de  «escalón  de  escalera»  o  «collar  de  perlas»).
  Aire  ausente  o  mínimo  en  el  colon  distal  y  el  recto.
  Aspecto  de  «vidrio  esmerilado»  y  oscurecimiento  de  las  sombras  del  psoas  por  líquido  extraperitoneal.
  A  veces,  un  asa  de  intestino  delgado  distendida  con  un  «pico»  en  cada  extremo,  lo  que  indica  una  obstrucción  de  asa  
cerrada  en  un  abdomen  sin  gas  o  un  asa  única  fija  que  permanece  en  la  misma  ubicación  en  las  radiografías  tanto  
en  decúbito  supino  como  en  bipedestación.
  Las  radiografías  de  tórax  pueden  mostrar  un  infiltrado,  con  íleo  acompañante,  en  lugar  de  SBO.  La  radiografía  de  
tórax  lateral  es  la  más  sensible  para  identificar  aire  libre  en  el  abdomen;  esto  requiere  una  laparotomía  urgente  por  
víscera  perforada.

8.  ¿Qué  otros  estudios  por  imágenes  se  pueden  utilizar?
Los  estudios  de  contraste  oral  con  contraste  hidrosoluble  (Gastrografin)  ayudan  a  distinguir  la  obstrucción  parcial  de  la  
completa,  el  tumor  o  cuerpo  extraño  intraluminal  y  la  enfermedad  inflamatoria  intestinal;  también  pueden  definir  el  
punto  de  obstrucción.  Gastrografin  en  realidad  puede  ayudar  a  resolver  las  obstrucciones  parciales  por  su  efecto  
osmótico.
La  tomografía  computarizada  (TC)  y  la  resonancia  magnética  nuclear  (RMN)  pueden  ayudar  a  delinear  las  obstrucciones  
intestinales.  La  resonancia  magnética  tiene  la  ventaja  de  la  velocidad  (de  6  a  10  minutos  usando  la  técnica  HASTE  
[Hast  Fourier  single  shot  turbo  spin  echo]),  no  necesita  agente  de  contraste  y  una  mayor  tasa  de  precisión.  Una  serie  
reciente  de  Mayo  Clinic  también  afirma  una  precisión  superior  (95%  versus  71%).
La  ecografía  (US)  no  ha  resultado  útil.

9.  ¿Qué  estudios  de  laboratorio  están  indicados?
1.  Recuento  completo  de  células  sanguíneas  (CSC,  por  sus  siglas  en  inglés)  para  verificar  si  hay  leucocitosis  o  
anemia  inesperada  2.  Análisis  de  orina  para  buscar  una  infección  del  tracto  urinario  (que  también  puede  causar  un  íleo  y  
presentarse  con  un  cuadro  similar  al  SBO)  y  para  evaluar  la  hidratación  (gravedad  específica  de  la  orina). )
3.  Panel  de  química  para  verificar  anomalías  electrolíticas  como  hipopotasemia  o
alcalosis  metabólica  hipoclorémica  (asociada  con  vómitos  de  contenido  gástrico  ácido),  hiponatremia  y  azotemia  
prerrenal  (niveles  elevados  de  nitrógeno  ureico  en  sangre  [BUN]  y  creatinina).

4.  Amilasa  y  lipasa  para  descartar  pancreatitis;  la  amilasa  también  puede  estar  elevada,  aunque  no  tanto,  con  SBO  o  
intestino  isquémico.

10.  ¿Cuáles  son  los  pasos  iniciales  del  tratamiento?
Deben  instituirse  succión  nasogástrica  (NG)  y  líquidos  intravenosos  (IV)  para  restablecer  el  equilibrio  de  electrolitos  y  líquidos,  
y  debe  colocarse  una  sonda  de  Foley  para  controlar  la  producción  de  orina.  Tan  pronto  como  se  completa  la  reanimación,  es  
obligatoria  la  intervención  quirúrgica  inmediata  para  las  obstrucciones  completas  y  para  cualquier  persona  con  signos  y  
síntomas  de  estrangulamiento.

11.  ¿Cómo  puedo  distinguir  entre  una  obstrucción  completa  y  una  parcial?
  clínicamente:  si  es  parcial,  el  paciente  puede  continuar  eliminando  pequeñas  cantidades  de  gases  o  heces.  El  dolor  y  la  
distensión  disminuyen  rápidamente  con  la  succión  nasogástrica.
  radiográficamente:  las  radiografías  muestran  gas  moviéndose  hacia  el  colon  (obstrucción  parcial).
  Con  estudios  de  contraste  oral:  el  agente  de  contraste  hidrosoluble  o  de  bario  administrado  a  través  de  la  sonda  NG  
pasa  al  colon  en  obstrucciones  parciales.
Machine Translated by Google
232  CAPÍTULO  45  OBSTRUCCIÓN  DEL  INTESTINO  DELGADO

12.  ¿Qué  condiciones  deben  incluirse  en  el  diagnóstico  diferencial?
Debe  incluirse  el  íleo  por  otras  causas  (p.  ej.,  como  infección  del  tracto  urinario  [ITU],  neumonía,  hipopotasemia),  
gastroenteritis  viral,  apendicitis  (generalmente  con  perforación),  cálculos  ureterales,  diverticulitis,  trombosis  
mesentérica  y  cáncer  de  colon  obstructivo.

PUNTOS  CLAVE:  OBSTRUCCIÓN  DEL  INTESTINO  DELGADO

1.  La  causa  más  frecuente  es  la  enfermedad  adhesiva,  seguida  de  las  hernias.

2.  La  malignidad  debe  ser  considerada  como  una  posible  causa.

3.  El  tratamiento  consiste  en  descompresión  NG,  reposición  de  líquidos  y  electrolitos  y  conducta  expectante.
gestión.

4.  Se  requiere  intervención  quirúrgica  si  se  sospecha  estrangulación  u  obstrucción  de  asa  cerrada.

13.  ¿Cuáles  son  los  tres  tipos  de  SBO,  según  la  viabilidad  intestinal?
1.  Obstrucción  simple:  nada  pasa  por  el  punto  de  obstrucción,  pero  el  suministro  vascular  no  se  ve  
comprometido.  Puede  ser  parcial  y  resolverse  con  manejo  conservador.

2.  Obstrucción  estrangulada:  el  mesenterio  está  torcido  o  hay  tanta  dilatación  del  intestino  que  se  corta  el  flujo  
arterial  o  venoso  y  el  intestino  se  vuelve  isquémico.  La  cirugía  urgente  es  obligatoria.

3.  Obstrucción  de  asa  cerrada:  el  intestino  se  obstruye  proximal  y  distalmente,  por  lo  general  en  un  segmento  
corto,  y  ese  segmento  se  dilata  masivamente  y  es  susceptible  de  estrangulación  y  perforación.  La  cirugía  
urgente  es  obligatoria.

14.  ¿Cuáles  son  los  ''cinco  signos  clásicos''  de  estrangulamiento?  ¿Qué  tan  precisos  son?
1.  Dolor  continuo  (no  cólico).
2.  Fiebre.
3.  Taquicardia.
4.  Signos  peritoneales  (protección  o  hipersensibilidad  localizada,  hipersensibilidad  de  rebote).
5.  Leucocitosis.
Estos  signos  suelen  indicar  isquemia  irreversible.  El  dolor  persistente,  la  fiebre  progresiva  y  la  leucocitosis  son  
indicaciones  para  la  cirugía.

15.  ¿Cuál  es  la  tasa  de  mortalidad  de  SBO?
  Obstrucción  simple:  Mortalidad  5%  si  se  opera  dentro  de  las  24  horas.
  Obstrucción  estrangulada:  Tasa  de  mortalidad  del  25%.  La  mortalidad  depende  de  la
resiliencia  del  paciente  (enfermedad  comórbida);  pero  la  estrangulación  aumenta  cinco  veces  la  
mortalidad.

16.  ¿Qué  intervenciones  quirúrgicas  pueden  ser  necesarias  para  el  tratamiento  de  SBO?
  Lisis  abierta  o  laparoscópica  de  adherencias  en  el  punto  de  obstrucción.
&  Reducción  y  reparación  de  hernia.
  Resección  de  lesiones  obstructivas  con  anastomosis  primaria.
  Resección  de  segmento  estrangulado  con  anastomosis  primaria.
  Bypass  de  lesiones  obstructivas  (usado  principalmente  para  carcinomatosis).
  Colocación  de  un  tubo  largo  a  través  del  duodeno  hasta  el  intestino  delgado
(un  tubo  de  Baker  es  el  más  utilizado).
Machine Translated by Google
CAPÍTULO  45  OBSTRUCCIÓN  DEL  INTESTINO  DELGADO  233

17.  Describa  los  criterios  para  distinguir  el  intestino  viable  del  muerto  en  el  momento  de  la  operación.

El  color  rosado,  el  peristaltismo  y  las  pulsaciones  arteriales  son  la  forma  más  obvia  de  identificar  un  intestino  viable.
En  casos  dudosos,  la  ecografía  Doppler  puede  detectar  pulsaciones  arteriales,  pero  la  más  confiable  es  la  inyección  
intravenosa  de  colorante  de  fluoresceína  con  el  uso  de  una  lámpara  de  Wood.  El  intestino  viable  emite  fluorescencia  violeta.

18.  ¿Cuál  es  el  riesgo  de  desarrollo  de  SBO  después  de  la  laparotomía  inicial?  ¿Después  de  una  laparotomía  previa  por  SBO?  
¿Qué  operaciones  están  asociadas  con  altas  tasas  de  SBO?
Aproximadamente  el  15%  de  todos  los  pacientes  sometidos  a  laparotomía  finalmente  desarrollan  una  SBO.  Alrededor  
del  12%  de  los  pacientes  con  una  SBO  previa  desarrollan  otra.  Cuantas  más  recurrencias,  mayor  es  la  tasa  de  
recurrencia.  La  colectomía  total  o  subtotal  tiene  una  tasa  de  11%  a  1  año  y  de  30%  a  10  años.  La  histerectomía  
también  conlleva  una  alta  tasa  de  SBO:  alrededor  del  5  %  para  los  procedimientos  de  rutina  y  hasta  el  15  %  después  
de  la  histerectomía  radical.

19.  ¿Qué  pueden  hacer  los  cirujanos  para  disminuir  el  riesgo  de  SBO?
&  Utilice  guantes  sin  talco  o  lave  el  talco  de  los  guantes.
  Evite  suturar  a  través  del  peritoneo  durante  el  cierre.
  Use  una  película  de  barrera  entre  la  incisión  y  el  intestino  delgado.

20.  ¿Cuál  es  el  papel  de  la  laparoscopia  en  SBO?
La  lisis  laparoscópica  de  adherencias  generalmente  se  reserva  para  pacientes  que  no  han  tenido  múltiples  
laparotomías  previas.  Aproximadamente  un  tercio  de  ellos  puede  tratarse  con  éxito  solo  con  laparoscopia,  un  tercio  
requiere  una  laparotomía  mínima  ("asistida  por  el  regazo")  y  alrededor  de  un  tercio  requiere  una  laparotomía  abierta  
completa.  Las  series  recientes  reclaman  más  del  80%  de  éxito.

21.  ¿Qué  debo  considerar  si  el  paciente  ha  tenido  un  bypass  gástrico  en  Y  de  Roux  (BGYR)?
Las  causas  más  comunes  de  SBO  después  de  RYGB  son  hernias  internas  (42  %),  enfermedad  adhesiva  (22  %),  
estenosis  de  yeyunoyeyunostomía  (15  %)  y  hernia  incisional  (9  %).  Las  hernias  internas  pueden  ocurrir  a  través  del  
mesocolon  transverso,  el  mesentario  del  intestino  delgado  en  la  yeyunoyejonostomía  y  entre  el  mesentario  yeyunal  y  el  
mesocolon  (hernia  de  Peterson).  La  evaluación  de  SBO  debe  incluir  estudios  gastrointestinales  superiores  o  tomografía  
computarizada,  manteniendo  un  umbral  bajo  para  la  exploración  quirúrgica.

22.  ¿Qué  se  puede  hacer  en  pacientes  con  obstrucciones  intestinales  recurrentes  múltiples  para
adherencias?

Se  puede  colocar  un  tubo  largo,  ya  sea  a  través  de  NG,  gastrostomía  o  yeyunostomía  con  el  tubo  avanzado  a  
través  de  la  válvula  ileocecal.  El  tubo  largo  se  deja  en  posición  durante  aproximadamente  7  días  y,  según  se  informa,  
permite  que  el  intestino  vuelva  a  formar  adherencias  en  curvas  más  suaves.  Se  han  probado  y  abandonado  muchas  
otras  técnicas,  incluida  la  plicatura  Noble  (es  decir,  sutura  del  intestino  en  asas  ordenadas)  y  la  adición  de  varios  
irrigantes  (p.  ej.,  heparina,  dextrano,  solución  salina)  a  la  cavidad  peritoneal  antes  del  cierre.

23.  Nombre  cinco  complicaciones  asociadas  con  la  cirugía  para  SBO.
1.  Enterotomía.
2.  Íleo  prolongado.
3.  Infección  de  heridas.
4.  Absceso.
5.  Obstrucción  recurrente.

24.  Nombre  los  productos  que  pretenden  disminuir  la  formación  de  adherencias.
&  Celulosa  oxidada  (Interceed).
&  Hialuronato  de  sodio  y  carboximetilcelulosa  (Seprafilm).
&  icodextrina  (Adept;  en  investigación).
&  Gel  de  hialuronato  férrico  al  0,5  %  (Intergel;  en  fase  de  investigación).
Machine Translated by Google
234  CAPÍTULO  45  OBSTRUCCIÓN  DEL  INTESTINO  DELGADO

BIBLIOGRAFÍA

1.  Beall  DP,  Fortman  BJ,  Lawler  BC  et  al.:  Imágenes  de  obstrucción  intestinal:  una  comparación  entre  imágenes  magnéticas  rápidas
resonancia  magnética  y  tomografía  computarizada  helicoidal.  Clin  Radiol  57:719­724,  2002.

2.  Beck  DE,  Opelka  FG,  Bailey  HR  et  al .:  Incidencia  de  obstrucción  del  intestino  delgado  y  adhesiolisis  después  de  cirugía  abierta
Cirugía  colorrectal  y  general.  Dis  Colon  Rectum  42:241­248,  1999.

3.  Choi  HK,  Chu  KW,  Law  WL:  valor  terapéutico  de  Gastrografin  en  la  obstrucción  del  intestino  delgado  adhesivo  después  de  un  
tratamiento  conservador  fallido:  un  ensayo  prospectivo  aleatorizado.  Ann  Surg  236:1­6,  2002.

4.  DeCherney  AH,  diZerega  GS:  Problema  clínico  de  la  formación  de  adherencias  posquirúrgicas  intraperitoneales  después  de  la  cirugía  
general  y  el  uso  de  barreras  de  prevención  de  adherencias.  Surg  Clin  North  Am  77:671­688,  1997.

5.  Hayanga  AJ,  Bass­Wilins  K,  Bulkley  GB:  Manejo  actual  de  la  obstrucción  del  intestino  delgado.  Adv  Surg  29:1­33,
2005.

6.  Helton  WS,  Fisichella  PM:  Obstrucción  intestinal.  En  Cirugía  ACS:  Principios  y  práctica,  Nueva  York,  2008,
WebMD  Inc.

7.  Koppman  JS,  Li  C,  Gandsas  A:  Obstrucción  del  intestino  delgado  después  de  un  bypass  gástrico  laparoscópico  en  Y  de  Roux:  una  revisión
de  9.527  pacientes,  J  Am  Coll  Surg  206(3):571­584,  2008.

8.  Zerey  M,  Sechrist  CW,  Kercher  KW  et  al .:  El  manejo  laparoscópico  de  la  obstrucción  del  intestino  delgado.
Cirugía  Am  J  194(6):882­888,  2007.
Machine Translated by Google

ISQUEMIA  INTESTINAL CAPITULO  
46

Dr.  Thomas  F.  Rehring,  FACS

1.  ¿Cuál  es  el  riego  arterial  del  intestino?
El  intestino  anterior  (estómago  y  duodeno)  recibe  su  riego  sanguíneo  de  la  arteria  celíaca,  el  intestino  
medio  (yeyuno  al  colon  descendente  proximal)  de  la  arteria  mesentérica  superior  (AMS)  y  el  intestino  
posterior  (el  resto  del  intestino  intraperitoneal)  de  la  mesentérica  inferior.  arteria  (AMI).

2.  Nombre  las  posibles  vías  colaterales  entre  el  eje  celíaco  y  la  AME.
¿SMA  e  IMA?  ¿Iliaca  e  IMA?
Las  arterias  pancreatoduodenales  forman  las  principales  colaterales  entre  la  arteria  celíaca  y  la  SMA.  La  
arteria  gastroduodenal  da  origen  a  la  arteria  pancreaticoduodenal  superior  que  rodea  la  cabeza  del  páncreas  
y  se  anastomosa  con  la  arteria  pancreaticoduodenal  inferior,  la  primera  rama  de  la  SMA.

El  SMA  y  el  IMA  tienen  dos  conexiones  principales.  La  arteria  marginal  de  Drummond  se  encuentra  dentro
el  mesenterio  del  colon  y  está  formado  por  ramas  de  las  arterias  ileocólica,  cólica  derecha,  media  e  
izquierda.  El  arco  de  Riolano  (arteria  mesentérica  serpenteante)  es  más  central  y  conecta  la  rama  cólica  
media  de  la  SMA  y  la  rama  cólica  izquierda  de  la  AMI.
La  arteria  ilíaca  interna  da  lugar  a  la  arteria  rectal  media,  que  puede  proporcionar  flujo  a  la  rectal  superior  
y,  por  tanto,  a  la  AMI.

3.  Para  crédito  adicional,  ¿para  quién  se  nombra  la  arteria  marginal  de  Drummond?  Qué
sobre  el  arco  de  Riolan?
Hamilton  Drummond,  un  cirujano  británico,  probó  la  conexión  anastomótica  que  lleva  su  nombre  ligando  los  
orígenes  de  las  arterias  cólicas  derecha,  media  e  izquierda  y  demostrando  el  flujo  hacia  las  arterias  
sigmoideas  en  1913  y  1914.
Jean  Riolan  (1577­1657)  fue  un  conocido  anatomista  francés  que  (irónicamente)  se  opuso  a  la  teoría  
de  la  circulación  de  Harvey,  pero  se  reconoce  que  fue  la  primera  persona  en  señalar  la  comunicación  
entre  la  SMA  y  la  IMA.

4.  Nombre  las  causas  comunes  de  isquemia  intestinal  aguda.
Embolia  aguda  de  SMA  (50  %  de  todos  los  casos),  trombosis  aguda  de  SMA,  isquemia  mesentérica  no  
oclusiva  (NOMI),  trombosis  venosa  mesentérica,  vasculitis  y  causas  iatrogénicas  (p.  ej.,  agentes  
inotrópicos,  cirugía  aórtica).

5.  ¿Cuál  es  la  tasa  de  mortalidad  de  los  pacientes  con  isquemia  mesentérica  aguda?
Aunque  el  pronóstico  de  la  oclusión  embólica  es  algo  mejor  debido  a  la  presentación  espectacular,  el  
diagnóstico  de  isquemia  mesentérica  aguda  a  menudo  se  hace  después  de  un  infarto.  El  resultado  es  
una  alta  tasa  de  mortalidad  (60%  a  80%),  independientemente  de  la  causa.  A  pesar  de  los  avances  en  
diagnóstico,  intervención  y  cuidados  intensivos,  esta  cifra  no  ha  cambiado  durante  más  de  50  años.

6.  ¿Qué  es  un  «émbolo  paradójico»?
Un  émbolo  paradójico  ocurre  en  el  contexto  de  un  trombo  venoso  que  emboliza  a  la  circulación  arterial  a  través  de  
un  defecto  cardíaco  (típicamente  un  defecto  del  tabique  interauricular  que  permite  la  derivación  de  derecha  a  izquierda).

235
Machine Translated by Google
236  CAPÍTULO  46  ISQUEMIA  INTESTINAL

7.  ¿Cuál  es  la  tríada  diagnóstica  de  la  isquemia  intestinal  embólica  aguda?
Inicio  repentino  de  (1)  dolor  abdominal  intenso,  (2)  evacuación  intestinal  (vómitos  o  diarrea)  y  (3)  antecedentes  
de  enfermedad  cardíaca  (embolia  arterial).  Un  sello  distintivo  adicional  es  el  dolor  fuera  de  proporción  con  los  
hallazgos  físicos.

8.  ¿Cómo  difiere  la  presentación  de  los  pacientes  con  oclusión  trombótica  aguda?
La  oclusión  trombótica  se  presenta  típicamente  en  pacientes  de  edad  avanzada  con  enfermedad  oclusiva  aterosclerótica  
difusa  o  en  pacientes  con  antecedentes  compatibles  con  isquemia  mesentérica  crónica  (consulte  la  pregunta  25).  
Particularmente  en  el  primer  grupo  de  pacientes,  la  oclusión  embólica  aguda  puede  ser  indistinguible  de  la  oclusión  
trombótica.

9.  ¿Qué  valor  de  laboratorio  es  diagnóstico  de  isquemia  intestinal  aguda?  ¿La  acidosis?
Ningún  valor  de  laboratorio  es  diagnóstico  de  isquemia  intestinal  aguda.  La  acidosis  metabólica  es  un  hallazgo  
tardío  e  implica  isquemia  avanzada  o  infarto.  De  manera  similar,  los  niveles  elevados  de  lactato  y  fosfato  son  hallazgos  
inespecíficos  y  frecuentemente  tardíos.  Aunque  la  leucocitosis  se  encuentra  en  la  mayoría  de  los  pacientes,  ningún  
estudio  de  laboratorio  es  específico.  El  diagnóstico  se  persigue  únicamente  con  la  sospecha  clínica.

PUNTOS  CLAVE:  TRÍADA  DIAGNÓSTICA  DE  EMBOLIA  AGUDA
ISQUEMIA  INTESTINAL
1.  Inicio  súbito  de  dolor  abdominal  intenso  fuera  de  proporción  con  el  examen  físico.

2.  Evacuación  intestinal  repentina  (vómitos  o  diarrea).

3.  Antecedentes  de  enfermedad  cardíaca  (p.  ej.,  fibrilación  auricular  que  explica  la  fuente  embólica).

4.  Ningún  hallazgo  de  laboratorio  (p.  ej.,  nivel  de  lactato)  es  diagnóstico;  la  acidosis  metabólica  es  un  hallazgo  tardío.

5.  Está  indicada  una  arteriografía  urgente.

10.  Ante  la  sospecha  de  isquemia  intestinal  aguda,  ¿qué  estudio  es  diagnóstico?
La  arteriografía  emergente  es  diagnóstica.  Es  importante  incluir  vistas  laterales  de  la  aorta  para  visualizar  los  
vasos  viscerales.  En  muchas  instituciones,  la  angiografía  por  tomografía  computarizada  (CTA)  multidetector  está  
reemplazando  rápidamente  a  la  arteriografía.  Para  esta  indicación,  tiene  las  claras  ventajas  de  la  velocidad,  la  
accesibilidad  y  la  evaluación  de  otras  fuentes  en  el  diagnóstico  diferencial  del  dolor  abdominal  agudo.

11.  ¿Cómo  difieren  los  hallazgos  operatorios  en  pacientes  con  oclusión  aterosclerótica  y  pacientes  con  embolia  AME?

Un  émbolo  de  SMA  generalmente  se  aloja  de  3  a  4  cm  distal  a  su  origen  y,  por  lo  tanto,  más  allá  de  las  arterias  
cólica  media  y  yeyunal  proximal.  Por  lo  tanto,  las  6  a  10  pulgadas  proximales  del  yeyuno  generalmente  se  
conservan.  La  oclusión  trombótica  se  produce  directamente  en  los  ostium,  donde  el  estrechamiento  aterosclerótico  
es  más  grave  y  causa  isquemia  de  todo  el  intestino  medio.

12.  ¿Cuál  es  el  manejo  adecuado  de  un  émbolo  de  AME?  ¿Hay  un  papel  para  la  trombólisis?

Heparinización  inmediata,  exploración  urgente,  embolectomía,  evaluación  de  viabilidad  intestinal  y  resección  de  
cualquier  intestino  infartado.  La  anticoagulación  postoperatoria  es  esencial  para  evitar  una  mayor  embolización.

La  trombólisis  actualmente  tiene  poco  o  ningún  papel  en  el  tratamiento  de  la  isquemia  mesentérica  
aguda.  La  revascularización  del  intestino  debe  llevarse  a  cabo  rápidamente  y  la  viabilidad  del  intestino  debe  
determinarse  directamente.
Machine Translated by Google
CAPÍTULO  46  ISQUEMIA  INTESTINAL  237

13.  ¿Cómo  se  maneja  la  isquemia  visceral  de  origen  trombótico?
El  manejo  general  sigue  al  de  una  embolia;  sin  embargo,  la  isquemia  mesentérica  por  oclusión  trombótica  es  la  
etapa  final  de  la  oclusión  aterosclerótica  progresiva.  Por  lo  tanto,  la  trombectomía  por  sí  sola  no  es  suficiente;  es  
necesaria  la  derivación  o  la  endarterectomía  del  vaso  o  vasos  proximales  enfermos.  Nuevamente,  la  viabilidad  
intestinal  se  evalúa  después  de  la  reperfusión.

14.  ¿Qué  pruebas  intraoperatorias  ayudan  a  los  cirujanos  a  determinar  la  viabilidad  intestinal?
Tanto  la  infusión  intravenosa  (IV)  sistémica  de  fluoresceína,  que  se  evalúa  con  una  lámpara  de  Wood,  como  el  
examen  Doppler  intraoperatorio  del  intestino  son  útiles,  pero  en  última  instancia,  la  decisión  se  basa  en  el  juicio  
clínico.

15.  Cuando  se  cuestiona  el  grado  de  viabilidad  intestinal,  ¿qué  se  debe  hacer?
Todo  el  intestino  no  viable  y  necrótico  debe  ser  resecado.  El  cirujano  debe  programar  una  segunda  
operación  de  revisión  12  a  24  horas  más  tarde  para  evaluar  la  viabilidad  marginal  del  intestino.
Algunos  segmentos  que  inicialmente  eran  cuestionables  pueden  volverse  claramente  viables  o  necróticos  durante  
este  período.

16.  ¿Cuánto  intestino  delgado  se  requiere  para  mantener  una  nutrición  adecuada?
Se  requieren  alrededor  de  100  cm  de  intestino  delgado  para  mantener  una  nutrición  adecuada.  El  íleon  distal  y  la  
válvula  ileocecal  son  los  segmentos  más  importantes  a  retener  para  la  función  y  absorción  vital  del  intestino.

17.  ¿Se  debe  cancelar  una  operación  de  revisión  porque  un  paciente  mejora?
Nunca.  La  decisión  se  toma  en  el  quirófano  (OR)  según  los  hallazgos  en  el  momento  de  la  cirugía.  Ningún  
parámetro  clínico  dentro  de  las  siguientes  12  a  24  horas  indica  con  precisión  el  estado  del  intestino  en  cuestión.

18.  ¿Qué  es  NOMI?
La  isquemia  mesentérica  no  oclusiva  (NOMI)  representa  aproximadamente  el  20%  de  los  casos  de  isquemia  aguda.  
Esto  ocurre  típicamente  en  pacientes  que  están  gravemente  enfermos  con  hipoperfusión  sistémica.  En  tales  estados  
de  bajo  flujo,  el  flujo  sanguíneo  esplácnico  se  reduce  en  un  intento  de  preservar  la  perfusión  de  los  lechos  cardíaco  
y  cerebral.  Los  agentes  farmacológicos  como  los  alcaloides  del  cornezuelo  del  centeno,  los  digitálicos,  la  cocaína  y  
los  vasoconstrictores  también  pueden  predisponer  a  NOMI.

19.  ¿Cómo  se  diagnostica  y  maneja  la  NOMI?
La  angiografía  documenta  vasoespasmo  en  ausencia  de  una  oclusión  anatómica.  El  colon  derecho  es  el  más  
comúnmente  afectado  debido  a  su  flujo  sanguíneo  colateral  menos  constante.  Está  asociado  con  (y  puede  ser  
exacerbado  por)  el  uso  concomitante  de  digitálicos  en  pacientes  con  hipoperfusión  sistémica.  En  los  casos  graves  
asociados  con  insuficiencia  orgánica  multisistémica  (MOF),  la  tasa  de  mortalidad  se  acerca  al  75%.  El  tratamiento  
consiste  en  optimización  hemodinámica,  destete  de  inotrópicos  e  infusión  arterial  selectiva  de  vasodilatadores  
(papaverina)  a  través  del  catéter  de  angiografía.
La  intervención  quirúrgica  se  reserva  para  el  infarto  o  la  perforación  intestinal.

20.  Si  se  sospecha  trombosis  de  la  vena  mesentérica  (TVM),  ¿cuál  es  la  mejor  prueba?
Los  signos  y  síntomas  de  la  MVT  son  similares  a  los  de  la  isquemia  intestinal  aguda,  pero  a  menudo  son  más  
sutiles.  Se  cree  que  la  demora  en  el  diagnóstico  contribuye  a  la  alta  tasa  de  mortalidad  informada  del  50%.  La  
tomografía  computarizada  (TC)  con  contraste  sigue  siendo  el  estándar  de  oro  para  el  diagnóstico.

21.  ¿Cuáles  son  los  factores  de  riesgo  de  TVM?  Como  es  tratado?
Aproximadamente  la  mitad  de  los  pacientes  con  TVM  tienen  un  estado  de  hipercoagulabilidad  subyacente.  Otras  
causas  incluyen  esplenectomía,  hipertensión  portal,  infecciones  viscerales,  pancreatitis,  malignidad  y  traumatismo  
abdominal  cerrado.
Machine Translated by Google
238  CAPÍTULO  46  ISQUEMIA  INTESTINAL

El  tratamiento  de  la  MVT  incluye  anticoagulación,  antibióticos  de  amplio  espectro,  tratamiento  de  la  
causa  subyacente  y  medidas  de  apoyo.  La  cirugía  se  reserva  para  la  resección  del  intestino  no  viable.  La  
trombectomía  venosa  no  ha  demostrado  ser  beneficiosa  a  largo  plazo.  Se  ha  explorado  el  uso  de  agentes  
trombolíticos,  pero  solo  en  informes  anecdóticos.  Además,  el  acceso  a  la  circulación  venosa  esplácnica  para  
la  lisis  dirigida  es  difícil.

22.  ¿Cuál  es  la  causa  principal  de  la  isquemia  mesentérica  crónica?
Aterosclerosis.  Como  la  circulación  colateral  al  intestino  es  robusta,  los  síntomas  generalmente  no  
ocurren  a  menos  que  dos  de  las  tres  arterias  principales  (celíaca,  mesentérica  superior  y  mesentérica  
inferior)  estén  estrechadas  u  ocluidas.

23.  ¿Cuál  es  el  único  factor  de  riesgo  de  isquemia  mesentérica  crónica  que  difiere
de  otros  fenómenos  ateroscleróticos?
Ocurre  con  mayor  frecuencia  en  mujeres.

24.  ¿Cuáles  son  las  características  clínicas  de  los  pacientes  con  isquemia  mesentérica  crónica?
La  pérdida  de  peso  es  el  signo  más  constante  de  isquemia  mesentérica  crónica.  Los  pacientes  
gradualmente  y,  a  veces,  sin  saberlo,  tienen  miedo  de  comer  (miedo  a  la  comida)  debido  al  dolor  
posprandial  (angina  intestinal).  En  ausencia  de  pérdida  de  peso,  el  diagnóstico  de  isquemia  intestinal  
crónica  es  poco  probable.  Por  el  contrario,  en  pacientes  con  aterosclerosis  severa  y  pérdida  de  peso  de  
causa  desconocida,  se  debe  considerar  seriamente  la  isquemia  mesentérica.  Un  soplo  epigástrico  es  un  
signo  importante  que  sugiere  una  enfermedad  oclusiva  mesentérica.

25.  ¿Cómo  se  debe  evaluar  a  los  pacientes  con  isquemia  mesentérica  crónica?
La  ecografía  dúplex  no  invasiva  (US)  puede  proporcionar  información  fisiológica  importante  sobre  el  eje  
celíaco  y  la  AME.  Sin  embargo,  este  procedimiento  depende  del  técnico  y  no  está  ampliamente  disponible.  
Si  no  está  disponible  o  es  equívoco,  se  debe  realizar  una  angiografía  mesentérica.
Si  se  considera  la  intervención  quirúrgica,  la  arteriografía  es  esencial.  Aunque  controvertida,  la  CTA  
multidetector  está  reemplazando  rápidamente  a  la  angiografía  tradicional,  ya  que  es  rápida,  no  invasiva  y  
ampliamente  accesible.

26.  ¿Cuáles  son  los  objetivos  de  la  derivación  arterial  en  la  isquemia  mesentérica  crónica?
Resolución  de  síntomas,  mejora  de  la  nutrición  y  prevención  del  infarto  visceral.

27.  Si  se  considera  la  revascularización  mesentérica,  ¿qué  cinco  decisiones  esenciales  deben  considerarse?

&  Abordaje  quirúrgico  (transabdominal,  retroperitoneal,  toracoabdominal).
&  Cuáles  y  cuántos  vasos  revascularizar.
&  Endarterectomía,  reimplantación  o  derivación.
&  Si  derivación,  anterógrada  o  retrógrada.
&  Si  es  bypass,  qué  tipo  de  conducto  (venoso  versus  protésico).
Ver  también  las  preguntas  31  y  32.

28.  ¿Qué  es  la  colitis  isquémica?
La  colitis  isquémica  es  una  insuficiencia  circulatoria  del  colon  que  puede  deberse  a  causas  
oclusivas,  no  oclusivas  y  farmacológicas  (p.  ej.,  cocaína,  fármacos  antiinflamatorios  no  esteroideos  [AINE]).  
El  siete  por  ciento  de  todos  los  pacientes  que  se  someten  a  una  cirugía  de  aneurisma  aórtico  abdominal  no  
emergente  y  hasta  el  60%  de  los  pacientes  que  sobreviven  a  un  aneurisma  aórtico  abdominal  roto  sufren  de  
colitis  isquémica.  La  mayoría  de  los  casos  son  leves,  por  lo  general  involucran  solo  la  mucosa  y  provocan  
dolor  abdominal  y  diarrea  sanguinolenta.  La  enfermedad  grave  (15%  de  los  casos)  se  caracteriza  por  infarto  
gangrenoso  transmural  que  se  presenta  con  signos  claros  de  peritonitis  y  diarrea  sanguinolenta.
Machine Translated by Google
CAPÍTULO  46  ISQUEMIA  INTESTINAL  239

29.  ¿Cómo  se  diagnostica  y  trata  la  colitis  isquémica?  ¿Cuáles  son  sus  implicaciones  pronósticas?

El  diagnóstico  se  realiza  por  endoscopia.  En  casos  idiopáticos,  la  angiografía  demuestra  vasos  grandes  
permeables  porque  se  cree  que  los  émbolos  o  lesiones  responsables  afectan  vasos  arteriales  periféricos.  La  
enfermedad  leve  generalmente  se  trata  de  manera  conservadora  con  reposo  intestinal,  hidratación  vigorosa  y  
antibióticos  de  amplio  espectro.  La  enfermedad  grave  requiere  resección  quirúrgica.
Las  tasas  generales  de  mortalidad  son  de  alrededor  del  50  %,  pero  en  pacientes  que  requieren  resección  de  colon,  la  tasa  
de  mortalidad  puede  superar  el  85  %.  La  alta  mortalidad  en  este  último  grupo  se  atribuye  al  shock  endotoxémico  y  al  MOF.

CONTROVERSIAS

30.  ¿Qué  es  el  síndrome  de  compresión  celíaca  (síndrome  de  Dunbar)?
La  compresión  celíaca  es  un  trastorno  raro  y  controvertido  que  se  describe  con  mayor  frecuencia  en  mujeres  (la  
proporción  de  mujeres  a  hombres  es  de  4:  1)  entre  las  edades  de  20  y  50  años.  También  conocido  como  síndrome  
del  ligamento  arcuato  mediano,  estos  pacientes  parecen  sufrir  isquemia  mesentérica  crónica  sin  evidencia  angiográfica  
de  enfermedad  aterosclerótica.  Se  cree  que  la  compresión  mecánica  es  causada  por  el  pilar  izquierdo  del  diafragma  
(es  decir,  el  ligamento  arcuato  medio)  y,  en  ocasiones,  el  diagnóstico  se  confirma  demostrando  una  compresión  celíaca  
transitoria  durante  la  espiración.  El  dolor  asociado  es  el  resultado  de  una  redirección  del  flujo  complicada  y  aún  muy  
debatida  (robo  del  intestino  anterior)  lejos  de  la  SMA.  El  tratamiento  efectivo  ha  requerido  no  solo  la  liberación  de  la  
compresión  sino  también  la  derivación  para  mejorar  la  probabilidad  de  resolución  del  dolor.

31.  ¿Cuál  es  el  tratamiento  de  elección  para  la  isquemia  mesentérica  crónica,  el  bypass  arterial  visceral  anterógrado  o  
retrógrado?  ¿Es  necesario  reconstruir  más  de  un  vaso  mesentérico?

Como  se  aplican  a  la  derivación  intestinal,  los  términos  anterógrado  y  retrógrado  se  refieren  al  origen  del  injerto  de  
la  aorta  como  proximal  al  eje  celíaco  o  distal  a  la  SMA,  respectivamente.
Las  ventajas  declaradas  de  la  derivación  anterógrada  son  una  menor  torsión  del  injerto  y  posiblemente  mejores  
características  de  flujo  sanguíneo.  Las  desventajas  son  que  la  exposición  supracelíaca  es  técnicamente  más  difícil  
y  el  pinzamiento  puede  provocar  isquemia  renal  o  de  la  médula  espinal.  Los  injertos  de  derivación  retrógrados  son  
más  difíciles  de  colocar  para  evitar  torceduras.
Series  recientes  sugieren  que  los  resultados  para  la  reconstrucción  de  uno  o  varios  vasos  en
la  forma  anterógrada  o  retrógrada  son  excelentes,  con  tasas  de  supervivencia  sin  síntomas  >90%  a  los  cinco  años.

32.  ¿Cuál  es  el  papel  de  la  angioplastia  transluminal  percutánea  en  la  mesentérica  crónica?
¿isquemia?

El  tratamiento  endovascular  de  la  isquemia  mesentérica  crónica  es  una  técnica  relativamente  nueva.  La  evitación  obvia  
de  la  cirugía  es  una  ventaja  importante,  pero  las  raras  complicaciones  de  disección  y  embolia  pueden  ser  devastadoras  
en  lechos  arteriales  sin  garantías  adecuadas.
Las  tasas  de  éxito  se  aproximan  al  70%;  la  reestenosis  y  los  síntomas  recurrentes  se  informan  en  el  50%  de  los  
pacientes.  Ningún  ensayo  prospectivo  ha  comparado  la  ATP  con  el  bypass  arterial;  sin  embargo,  las  revisiones  
retrospectivas  sugieren  que  los  resultados  iniciales  con  cualquiera  de  las  técnicas  son  similares  con  respecto  a  la  
morbilidad,  la  muerte  y  la  estenosis  recurrente.  Sin  embargo,  las  tasas  de  recurrencia  de  los  síntomas  son  más  altas  con  la  PTA.

SITIO  WEB

www.emedicine.com/emerg/topic311.htm
Machine Translated by Google
240  CAPÍTULO  46  ISQUEMIA  INTESTINAL

BIBLIOGRAFÍA

1.  Brown  DJ,  Schermerhorn  ML,  Powell  RJ  et  al.:  Colocación  de  stents  mesentéricos  para  la  isquemia  mesentérica  crónica.  J  
Vasc  Surg  2005;42(2):268­274.

2.  Fisher  DF  Jr,  Fry  WJ:  Circulación  mesentérica  colateral.  Surg  Gynecol  Obstet  164(5):487­492,  1987.

3.  Gewertz  BL,  Schwartz  LB:  isquemia  mesentérica.  Surg  Clin  North  Am  77(2):275­502,  1977.
4.  Kasirajan  K,  O'Hara  PJ,  Gray  BH  et  al.:  Isquemia  mesentérica  crónica:  cirugía  abierta  versus  percutánea
angioplastia  y  colocación  de  stent.  J  Vasc  Surg  33(1):63­71,  2001.

5.  Kazmers  A:  manejo  operativo  de  la  isquemia  mesentérica  aguda.  Parte  1,  Ann  Vasc  Surg  12(2):187­197,  1998.

6.  Kazmers  A:  manejo  operativo  de  la  isquemia  mesentérica  crónica.  Ann  Vasc  Surg  12(3):299­308,  1998.
7.  Oldenburg  WA,  Lau  LL,  Rodenberg  TJ  et  al .:  Isquemia  mesentérica  aguda:  una  revisión  clínica.  Arch  Intern  Med  164
(10):1054­1062,  2004.
8.  Seeger  JM:  El  manejo  de  las  lesiones  y  trastornos  vasculares  esplácnicos.  En:  Rutherford  RB,  editor,  Cirugía  vascular,  6.ª  ed.,  
vol.  2,  Filadelfia,  2005  Elsevier.

9.  Shih  MC,  Hagspiel  KD:  CTA  y  MRA  en  la  isquemia  mesentérica:  parte  1,  Papel  en  el  diagnóstico  y  diferencial
diagnóstico.  AJR  188(2):452­461,  2007.
10.  Sivamurthy  N,  Rhodes  JM,  Lee  D  et  al.:  Revascularización  mesentérica  endovascular  versus  abierta:  los  beneficios  
inmediatos  no  equivalen  a  resultados  funcionales  a  corto  plazo.  J  Am  Coll  Surg  202(6):859­867,  2006.
Machine Translated by Google

ENFERMEDAD  DIVERTICULAR  DEL  COLON CAPITULO  
47

Alexander  Q.  Ereso,  MD,  Elizabeth  L.  Cureton,  MD;

1.  ¿Qué  es  un  divertículo  colónico?
Protrusión  de  mucosa  y  submucosa  a  través  de  las  capas  musculares  de  la  pared  intestinal.  No  tiene  cubierta  
muscular.  Como  los  divertículos  no  afectan  a  todas  las  capas  de  la  pared  intestinal,  en  realidad  son  divertículos  
«falsos».  La  formación  de  divertículos  puede  estar  relacionada  con  la  debilidad  de  la  pared  intestinal  en  los  
sitios  de  perforación  de  los  vasos  o  con  el  aumento  de  la  presión  intraluminal  causado  por  la  escasez  de  fibra  
dietética  y  el  estreñimiento.

2.  ¿Cuál  es  la  diferencia  entre  diverticulosis  y  diverticulitis?
La  diverticulosis  es  un  divertículo  colónico  sin  inflamación  asociada.  La  diverticulitis  es  inflamación  
e  infección.  Solo  el  15%  de  los  pacientes  con  diverticulosis  desarrollan  diverticulitis.

3.  ¿Cómo  causa  dolor  un  divertículo?
Aparentemente,  el  dolor  se  debe  a  la  perforación  del  divertículo.  La  fuga  resultante  puede  ser  escasa  
y  estar  contenida  en  la  grasa  pericólica  o  extensa  y  afectar  el  mesenterio,  otros  órganos  o  la  cavidad  
peritoneal.  La  diverticulitis  sigmoidea  generalmente  causa  dolor  en  el  cuadrante  inferior  izquierdo.

4.  ¿En  qué  lugar  del  colon  se  localizan  habitualmente  los  divertículos?
En  los  Estados  Unidos,  el  95%  de  todos  los  divertículos  ocurren  en  el  colon  izquierdo,  principalmente  en  el  colon  sigmoide.
Sin  embargo,  los  divertículos  pueden  ocurrir  en  cualquier  parte  del  colon.  En  Asia,  los  divertículos  del  colon  derecho  se
más  común.

5.  ¿A  qué  edad  es  más  común  la  diverticulitis?
La  sexta  o  séptima  década  de  la  vida.  Los  pacientes  menores  de  50  años  con  diverticulitis  tienden  a  tener  más  
complicaciones.  Los  pacientes  más  jóvenes  son  más  propensos  que  los  pacientes  mayores  a  tener  diverticulitis  
colónica  derecha.

6.  ¿Qué  estrategia  puede  disminuir  la  diverticulitis  en  pacientes  con  divertículos?
Una  dieta  alta  en  fibra.  Gran  volumen  en  el  colon  disminuye  la  segmentación  y  la  presión  intraluminal.

7.  ¿Cuál  es  la  mejor  prueba  de  imagen  para  el  diagnóstico  de  diverticulitis  aguda?
Tomografía  computarizada  (TC),  que  también  puede  diagnosticar  complicaciones  locales  de  la  
diverticulitis.

8.  ¿Qué  complicaciones  pueden  resultar  de  la  perforación  de  un  divertículo  colónico?
  Flemón  inflamatorio  o  absceso  en  el  mesenterio  intestinal.
y  peritonitis.
  Absceso  intraabdominal.
&  fístula  interna.
&  Obstrucción  intestinal.

241
Machine Translated by Google
242  CAPÍTULO  47  ENFERMEDAD  DIVERTICULAR  DEL  COLON

9.  ¿La  enfermedad  diverticular  puede  causar  sangrado?
Sí.  La  diverticulosis  (no  ­itis)  es  una  causa  común  de  hemorragia  gastrointestinal  inferior  (GI).
El  sangrado  por  diverticulitis  es  poco  común.

10.  ¿Cómo  se  puede  localizar  el  sitio  del  sangrado  diverticular?
Se  localiza  con  angiografía  realizada  a  través  de  la  arteria  mesentérica  inferior  (AMI)  y,  si  es  necesario,  la  
arteria  mesentérica  superior  (AMS).  Los  estudios  de  glóbulos  rojos  marcados  son  menos  útiles.
La  colonoscopia  rara  vez  es  útil.

PUNTOS  CLAVE:  LOCALIZACIÓN  DE  INFERIOR
HEMORRAGIA  GASTROINTESTINAL
1.  Causas  comunes:  diverticulosis,  cáncer,  angiodisplasia.

2.  La  proctosigmoidoscopia  sin  preparación  es  útil  para  descartar  el  origen  del  sangrado  rectal  (más
el  sangrado  proximal  limita  el  uso  de  la  endoscopia).

3.  Las  exploraciones  nucleares  de  glóbulos  rojos  marcados  son  útiles  para  el  sangrado  gastrointestinal  más  lento  (detecta  el  
sangrado  de  0,2  a  0,5  ml/min).

4.  La  arteriografía  es  la  modalidad  de  imagen  preferida  porque  puede  ser  terapéutica  y  detecta  sangrado  a  0,5  a  2  
ml/min.

5.  Iniciar  la  arteriografía  con  la  AMI,  luego  la  SMA,  luego  el  eje  celíaco  si  es  necesario;  administrar  vasopresina  o  
embolizar  (tasa  de  éxito  del  85%).

11.  ¿Cuándo  se  debe  operar  por  un  divertículo  colónico  sangrante?
El  reemplazo  de  5  a  6  unidades  de  sangre  (dos  tercios  del  volumen  de  sangre  de  un  paciente)  dentro  
de  las  24  horas  y  el  resangrado  durante  la  hospitalización  son  indicaciones  estándar  para  la  resección  del  
segmento  de  colon  que  contiene  un  divertículo  sangrante.

12.  Si  el  sangrado  es  potencialmente  mortal  pero  no  se  puede  localizar  dentro  del  colon,  ¿qué  tratamiento  se  requiere?

Se  requiere  colectomía  subtotal  con  ileostomía  y  cierre  del  colon  sigmoide  distal  en  el  reflejo  peritoneal  (operación  
de  Hartmann)  o  colectomía  abdominal  total  con  anastomosis  ileorrectal.

13.  ¿Qué  tres  procedimientos  se  pueden  utilizar  cuando  la  perforación  del  divertículo  da  lugar  a  un  absceso?  ¿Cuál  
tiene  la  tasa  de  mortalidad  operatoria  más  baja?
1.  Colostomía  de  derivación  y  drenaje  de  abscesos  (primera  de  tres  etapas).
2.  Resección  del  colon  afectado  con  colostomía  proximal  y  fístula  mucosa  distal  o  cierre
por  la  operación  de  Hartmann  (primera  de  dos  etapas).
3.  Resección  con  anastomosis  primaria  (una  etapa).
La  mortalidad  operatoria  es  más  baja  después  de  la  resección  y  la  colostomía  proximal  para  la  desviación  fecal.  A  
pesar  de  los  informes  de  éxito  con  el  procedimiento  de  una  etapa,  la  mayoría  de  los  cirujanos  favorecen  un  método  
más  seguro  de  dos  etapas  para  la  diverticulitis  perforada  (esta  estrategia  requiere  una  segunda  operación  después  
de  3  meses  para  retirar  la  colostomía  y  reanastomosis  colónica).

14.  ¿Cuál  es  la  evidencia  clínica  de  una  fístula  vesicocólica  o  ureterocólica  después  de
perforación  diverticular?
Neumaturia,  fecaluria  e  infecciones  crónicas  del  tracto  urinario  (polimicrobianas).
Machine Translated by Google
CAPÍTULO  47  ENFERMEDAD  DIVERTICULAR  DEL  COLON  243

15.  ¿Qué  procedimiento  se  requiere  para  reparar  una  fístula  vesicocólica?
Un  procedimiento  por  etapas  era  el  estándar  hasta  hace  poco  tiempo.  Ahora,  la  mayoría  de  los  pacientes  
pueden  ser  tratados  con  un  solo  procedimiento  que  incluye  resección  sigmoidea,  anastomosis  colónica  y  
reparación  primaria  del  defecto  vesical  con  sutura  absorbible.  Por  lo  general,  se  deja  colocado  un  catéter  de  
Foley  durante  10  días  después  de  la  cirugía.  Se  debe  colocar  algo  de  tejido  viable  entre  las  reparaciones  del  colon  
y  la  vejiga  para  prevenir  una  fístula  recurrente.

BIBLIOGRAFÍA

1.  Bouillot  JL,  Berthou  JC,  Champault  G  et  al.:  Resección  colónica  laparoscópica  electiva  para  la  enfermedad  diverticular:
resultados  de  un  estudio  multicéntrico  en  179  pacientes.  Surg  Endosc  16:1320­1323,  2002.

2.  Constantinides  VA,  Heriot  A,  Remzi  F  et  al .:  Estrategias  operativas  para  la  peritonitis  diverticular:  un  análisis  de  decisión  entre  la  resección  
primaria  y  la  anastomosis  versus  los  procedimientos  de  Hartmann.  Ann  Surg  245:94­103,  2007.

3.  Frattini  J,  Longo  WE:  Diagnóstico  y  tratamiento  de  la  diverticulitis  crónica  y  recurrente.  J  Clin  Gastroenterol
7  (Suplemento  3):  S145­149,  2006.

4.  Guller  U,  Jain  N,  Hervey  S  et  al.:  Colectomía  laparoscópica  versus  colectomía  abierta:  comparación  de  resultados  basada  en  grandes
bases  de  datos  a  nivel  nacional.  Arch  Surg  138:1179­1186,  2003.

5.  Richter  S,  Lindemann  W,  Kollmar  O  et  al .:  resección  de  colon  sigmoide  en  una  etapa  para  la  diverticulitis  sigmoidea  perforada  
(etapas  III  y  IV  de  Hinchey).  World  J  Surg  30:1027­1032,  2006.

6.  Salem  L,  Veenstra  DL,  Sullivan  SF  et  al .:  El  momento  de  la  colectomía  electiva  en  diverticulitis:  un  análisis  de  decisión.
J  Am  Coll  Surg  199:904­912,  2004.

7.  Simpson  J,  Scholefield  JH,  Spiller  RC:  Patogénesis  de  los  divertículos  colónicos.  Br  J  Surg  89:546­554,  2002.

8.  Wolff  BG,  Devine  RM:  Manejo  quirúrgico  de  la  diverticulitis.  Am  Surg  66:153­156,  2000.
Machine Translated by Google

OBSTRUCCIÓN  AGUDA  DEL  INTESTINO  GRUESO
CAPÍTULO  
48

Erik  D.  Peltz,  DO  y  Elizabeth  C.  Brew,  MD

1.  ¿Cuáles  son  las  causas  mecánicas  de  la  obstrucción  del  intestino  grueso?
Las  causas  mecánicas  más  comunes  son  carcinoma  (50%),  vólvulo  (15%),  adherencias  (15%)  y  enfermedad  diverticular  
(10%).  La  compresión  extrínseca  del  carcinoma  metastásico  es  otra  causa  de  obstrucción.  Las  causas  menos  frecuentes  
incluyen  hernia,  intususcepción,  tumor  benigno  e  impactación  fecal.

2.  ¿Cómo  se  realiza  el  diagnóstico?
1.  El  paciente  se  queja  de  dolor  abdominal  tipo  cólico  y  distensión  abdominal.  Las  náuseas  y  los  vómitos  ocurren  
más  tarde  en  LBO  y  pueden  ser  feculentos.  Un  inicio  agudo  de  los  síntomas  es  más  consistente  con  un  vólvulo  
en  comparación  con  el  desarrollo  gradual  de  molestias  obstructivas  en  pacientes  con  carcinoma  de  colon.

2.  El  examen  físico  revela  distensión  abdominal  y  ruidos  intestinales  agudos.  El  examen  rectal  puede  revelar  un  
cáncer  rectal  obstructivo  o  evidencia  de  retención  fecal.
La  ausencia  de  ruidos  intestinales  y  la  sensibilidad  localizada  pueden  ser  signos  de  peritonitis.
La  progresión  de  los  síntomas  acompañada  de  fiebre  alta  o  taquicardia  requiere  atención  quirúrgica  inmediata.

3.  Las  radiografías  abdominales  planas  y  verticales  revelan  un  colon  dilatado  proximal  a  la  obstrucción.
Una  radiografía  de  tórax  en  bipedestación  puede  mostrar  aire  libre  debajo  del  diafragma  si  se  ha  producido  
una  perforación.

3.  ¿Cómo  se  confirma  el  diagnóstico?
Es  necesario  un  enema  de  contraste  (contraste  de  bario  o  soluble  en  agua)  para  delinear  el  nivel  y  la  naturaleza  de  
una  obstrucción.  Un  vólvulo  se  puede  identificar  por  un  estrechamiento  en  “pico  de  pájaro”  en  el  cuello  del  vólvulo.  La  
sigmoidoscopia  o  colonoscopia  es  una  parte  esencial  de  la  evaluación;  permite  la  visualización  del  colon  y  puede  ser  
terapéutico  en  el  caso  de  un  vólvulo  sigmoideo.

4.  ¿Cuál  es  el  papel  de  la  tomografía  computarizada  (TC)  en  el  diagnóstico  de  grandes
¿obstrucción  intestinal?

La  tomografía  computarizada  multidetector  helicoidal  (MDCT)  de  adquisición  rápida  moderna  está  comenzando  
a  reemplazar  el  enema  de  contraste  para  confirmar  el  diagnóstico  de  LBO  en  muchos  pacientes.  MDCT  tiene  una  
sensibilidad  y  especificidad  reportadas  de  96%  y  94%,  respectivamente,  en  el  diagnóstico  de  LBO.  Las  tomografías  
computarizadas  (TC)  pueden  distinguir  entre  obstrucción  mecánica  o  pseudoobstrucción  y  pueden  ayudar  con  el  
diagnóstico  de  diverticulitis  o  carcinoma  de  colon.  La  TC  evita  el  riesgo  de  perforación  con  instrumentación  durante  el  
enema  o  la  endoscopia  y  puede  ser  beneficiosa  en  pacientes  ancianos  o  frágiles  que  no  pueden  cooperar  o  tolerar  otros  
procedimientos  de  diagnóstico.

5.  ¿Por  qué  es  importante  la  sensibilidad  en  el  cuadrante  inferior  derecho  (RLQ)?
El  ciego  es  el  área  que  es  más  probable  que  se  perfore.  Cuando  el  ciego  alcanza  los  15  cm  en  su  diámetro  más  
ancho,  la  tensión  en  la  pared  es  tan  grande  que  la  descompresión  es  fundamental  para  evitar  la  perforación.  El  mayor  
diámetro  del  ciego  provoca  más  tensión  de  la  pared  cecal  a  la  misma  presión  intraluminal  (ley  de  Laplace).  La  otra  área  
en  riesgo  de  perforación  es  el  sitio  de  un  cáncer  de  colon  primario.

244
Machine Translated by Google
CAPÍTULO  48  OBSTRUCCIÓN  AGUDA  DEL  INTESTINO  GRUESO  245

6.  ¿Dónde  suele  localizarse  el  cáncer  obstructivo?
La  mayoría  de  los  carcinomas  colorrectales  obstructivos  ocurren  en  el  ángulo  esplénico,  el  colon  descendente  
o  el  ángulo  hepático.  Por  el  contrario,  las  lesiones  del  colon  derecho  suelen  presentarse  con  sangrado  oculto.  Los  
cánceres  de  recto  y  cecal  son  causas  poco  frecuentes  de  obstrucción.

7.  ¿Qué  es  un  vólvulo?  ¿Donde  está  localizado?
Un  vólvulo  es  una  rotación  anormal  del  colon  sobre  un  eje  formado  por  su  mesenterio  y  ocurre  en  el  colon  
sigmoide  (75%)  o  en  el  ciego  (25%).  El  vólvulo  sigmoide  ocurre  en  una  población  de  edad  avanzada  cuando  el  
estreñimiento  crónico  hace  que  el  colon  sigmoide  se  alargue  y  se  vuelva  redundante.  El  vólvulo  cecal  requiere  
un  ciego  hipermóvil  como  resultado  de  la  fijación  embriológica  incompleta  del  colon  ascendente.

8.  ¿Cuándo  está  indicada  la  cirugía?
La  cirugía  se  realiza  de  forma  temprana  en  la  obstrucción  del  colon.  La  laparotomía  urgente  es  necesaria  en  
pacientes  con  sospecha  de  perforación  o  isquemia.  Los  signos  de  peligro  son  abdomen  tranquilo,  sensibilidad  en  el  
RLQ  y  aumento  del  dolor.  El  estado  cardiopulmonar  del  paciente  debe  evaluarse  y  optimizarse  antes  de  la  
operación.  Es  fundamental  corregir  la  deshidratación  y  administrar  antibióticos  perioperatorios.
El  marcado  de  posibles  sitios  de  estoma  y  la  profilaxis  de  trombosis  venosa  profunda  (TVP)  son  otras  
consideraciones  preoperatorias  importantes.

9.  ¿Qué  operación  se  debe  realizar  para  una  obstrucción  del  intestino  grueso?
El  procedimiento  tradicional  para  una  LBO  ha  sido  una  colostomía  de  descompresión.  Sin  embargo,  la  
evaluación  cuidadosa  del  estado  del  paciente,  la  viabilidad  del  intestino,  la  ubicación  de  la  obstrucción  y  la  ausencia  
de  contaminación  intraabdominal  a  menudo  permiten  la  resección  con  o  sin  anastomosis  primaria.  De  hecho,  no  se  
ha  demostrado  que  una  colostomía  de  derivación  inicial  tenga  ninguna  ventaja  de  supervivencia  e  incurre  en  el  
riesgo  de  más  cirugías.
Un  carcinoma  obstructivo  se  puede  resecar  satisfactoriamente  en  condiciones  de  emergencia  en  el  90%
de  pacientes  Los  carcinomas  del  colon  derecho  y  transverso  (proximal  al  ángulo  esplénico)  se  tratan  de  forma  
rutinaria  con  resección  y  anastomosis  primaria.  Recientemente,  los  cánceres  obstructivos  del  colon  descendente  
se  han  tratado  con  resección  y  colostomía  o  lavado  intraoperatorio  seguido  de  resección  y  anastomosis  primaria.  
Se  están  investigando  técnicas  para  la  descompresión  no  quirúrgica  del  colon,  como  la  dilatación  con  globo,  la  
terapia  con  láser  y  la  colocación  de  stents.  En  teoría,  estas  técnicas  permitirán  la  paliación,  la  preparación  intestinal  
y  la  resección  electiva  del  colon.

Un  vólvulo  debe  reducirse  y  resecarse.  Se  puede  lograr  la  reducción  de  un  vólvulo  sigmoideo
no  quirúrgicamente  por  sigmoidoscopia  o  descompresión  hidrostática  con  un  enema  de  contraste.  La  tasa  de  
recurrencia  del  vólvulo  después  de  la  reducción  no  quirúrgica  simple  es  del  75%.  El  tratamiento  quirúrgico  
incluye  detorsión  con  colopexia  o  colectomía  sigmoidea.  El  vólvulo  cecal  se  puede  tratar  de  manera  similar  con  
descompresión  no  quirúrgica,  cecopexia  o  resección  quirúrgica.
El  tratamiento  óptimo  de  la  enfermedad  diverticular  es  el  reposo  intestinal  inicial;  antibióticos  
intravenosos  (IV);  y  drenaje  percutáneo  de  abscesos,  si  es  necesario.  La  resección  de  colon  y  la  anastomosis  
primaria  se  pueden  realizar  después  de  una  adecuada  preparación  intestinal.

10.  ¿Cuál  es  el  papel  de  la  colocación  de  stent  endoluminal  para  la  obstrucción  aguda  del  intestino  grueso?
La  colocación  de  un  stent  colorrectal  puede  ser  útil  para  la  descompresión  colónica,  la  limpieza  intestinal  y  la  
optimización  médica  antes  de  la  resección  quirúrgica  definitiva.  En  este  entorno  preoperatorio,  la  colocación  de  
stents  colorrectales  puede  permitir  una  resección  quirúrgica  en  una  sola  etapa.  Los  stents  endoluminales  también  
pueden  ser  útiles  como  una  alternativa  a  la  colostomía  para  la  descompresión  paliativa  en  pacientes  con  obstrucción  
maligna  irresecable.  Las  complicaciones  de  la  colocación  del  stent  incluyen  la  oclusión  y  migración  del  stent,  
descompresión  inadecuada  y  perforación  intestinal.  En  este  momento,  su  aplicación  limitada  a  un  grupo  selecto  de  
pacientes  requiere  una  evaluación  cuidadosa.
Machine Translated by Google
246  CAPÍTULO  48  OBSTRUCCIÓN  AGUDA  DEL  INTESTINO  GRUESO

11.  ¿Cuáles  son  las  causas  no  mecánicas  de  la  obstrucción  del  intestino  grueso?
Íleo  paralítico  (es  decir,  seudoobstrucción  colónica)  o  megacolon  tóxico.

12.  ¿Qué  es  el  síndrome  de  Ogilvie?
El  síndrome  de  Ogilvie  es  un  íleo  paralítico  (adinámico)  agudo  o  una  seudoobstrucción  (es  decir,  una  dilatación  
enorme  del  colon  sin  una  lesión  obstructiva  distal  mecánica).  Los  pacientes  presentan  un  abdomen  masivamente  
dilatado  y  una  pequeña  cantidad  de  dolor.  El  tratamiento  no  quirúrgico,  que  incluye  reposo  intestinal,  líquidos  
intravenosos  y  enemas  suaves,  es  el  tratamiento  de  elección.  El  enema  de  gastrografín  o  colonoscopia  es  
diagnóstico  y  terapéutico.  La  neostigmina  es  otra  modalidad  de  tratamiento  en  pacientes  con  colon  >10  cm  de  
diámetro.

13.  ¿Qué  es  el  megacolon  tóxico?
El  megacolon  tóxico  es  la  dilatación  de  todo  el  colon  secundaria  a  una  enfermedad  inflamatoria  intestinal  aguda.
La  enfermedad  se  manifiesta  por  un  inicio  agudo  de  dolor  abdominal,  distensión  y  sepsis.  La  terapia  inicial  
incluye  reanimación  con  líquidos  IV,  descompresión  nasogástrica  (NG)  y  antibióticos  de  amplio  espectro.  Si  
los  síntomas  no  se  resuelven  en  unas  pocas  horas,  el  paciente  requiere  una  operación  para  evitar  la  
perforación.  El  tratamiento  quirúrgico  suele  consistir  en  una  colectomía  abdominal  de  urgencia  con  formación  
de  una  ileostomía.

PUNTOS  CLAVE:  CAUSAS  DE  LA  OBSTRUCCIÓN  DEL  INTESTINO  GRUESO

1.  Carcinoma:  causa  más  común:  50%.

2.  Vólvulo:  15%.

3.  Adherencias  15%.

4.  Enfermedad  diverticular:  10%.

5.  Otras  causas:  hernia,  intususcepción,  impactación  fecal.

SITIO  WEB

www.emedicine.com/emerg/topic65.htm

BIBLIOGRAFÍA

1.  Adler  DG,  Baron  TH:  paliación  endoscópica  del  cáncer  colorrectal.  Hematol  Oncol  Clin  North  Am  16:1015,  2002.
2.  Beattie  GC,  Peters  RT,  Guy  S  et  al.:  Tomografía  computarizada  en  la  evaluación  de  la  sospecha  de  intestino  grueso
obstrucción.  ANZ  J  Surg  77:160­165,  2007.
3.  Dauphine  CE,  Tan  P,  Beart  RW  Jr  et  al .:  Colocación  de  stents  metálicos  autoexpandibles  para  la  obstrucción  
maligna  aguda  del  intestino  grueso:  una  revisión  colectiva.  Ann  Surg  Oncol  9:574,  2002.
4.  Frager  D:  Obstrucción  intestinal:  papel  de  la  TC.  Gastroenterol  Clin  North  Am  31:777­799,  2002.
5.  Jost  RS  et  al .:  Stent  colorrectal:  una  terapia  eficaz  para  el  tratamiento  preoperatorio  y  paliativo.  Cardiovasc  Intervent  
Radiol  30:433­440,  2007.

6.  Lopez­Kostner  F,  Hool  GR,  Lavery  IC:  Manejo  y  causas  de  la  obstrucción  aguda  del  intestino  grueso.  Surg  Clin  North  
Am  77:1265­1290,  1997.
Machine Translated by Google
CAPÍTULO  48  OBSTRUCCIÓN  AGUDA  DEL  INTESTINO  GRUESO  247

7.  Markogiannakis  H,  Messaris  E,  Dardamanis  D  et  al.:  Obstrucción  intestinal  mecánica  aguda:  presentación  clínica,  
etiología,  manejo  y  resultado,  World  J  Gastroenterol  13(3):432­437,  2007.
8.  Murray  JJ,  Schoetz  DJ,  Coller  JA  et  al.:  Lavado  de  colon  intraoperatorio  y  anastomosis  primaria  en  resección  de  
colon  no  electiva,  Dis  Colon  Rectum  34:527,  1991.
9.  Paran  H,  Silverberg  D,  Mayo  A:  Tratamiento  de  la  seudoobstrucción  colónica  aguda  con  neostigmina,  J  Am  Coll
Surg  190(3):315­318,  2000.
10.  Tan  SG,  Nambiar  R,  Rauff  A  et  al.:  Resección  primaria  y  anastomosis  en  colon  descendente  obstruido  debido  a  
cáncer,  Arch  Surg  126:748,  1991.
Machine Translated by Google

ENFERMEDAD  INFLAMATORIA  INTESTINAL
CAPITULO  
49

David  E.  Stein,  MD  y  Jeffry  L.  Kashuk,  MD,  FACS

1.  ¿Qué  dos  entidades  clínicas  engloban  el  diagnóstico  de  colon  inflamatorio?
¿enfermedad?
Enfermedad  de  Crohn  y  colitis  ulcerosa.

2.  Aunque  las  dos  enfermedades  a  menudo  se  superponen,  por  lo  general  se  pueden  distinguir  por
criterios  clinicos.  ¿Cuáles  son  las  principales  diferencias  clínicas?
El  sangrado  rectal  es  inusual  en  la  enfermedad  de  Crohn,  pero  común  en  la  colitis  ulcerosa  (CU).  Una  masa  
abdominal,  antecedentes  de  enfermedades  anorrectales  (fisuras,  fístulas,  abscesos)  se  encuentran  comúnmente  en  la  
enfermedad  de  Crohn.

3.  ¿Cuáles  son  las  principales  diferencias  radiológicas  entre  las  dos  enfermedades?
Las  tomografías  computarizadas  (TC)  pueden  mostrar  engrosamiento  del  íleon  terminal,  áreas  salteadas,  estenosis  y  
fístulas  internas  con  flemones  asociados  en  la  enfermedad  de  Crohn.  En  la  CU  se  puede  observar  engrosamiento  de  la  
pared  del  colon.

4.  ¿Cuáles  son  las  principales  diferencias  histológicas?
Los  granulomas  en  la  pared  intestinal  y  los  ganglios  linfáticos  adyacentes  pueden  ocurrir  en  el  60%  de  los  pacientes  
con  enfermedad  de  Crohn.  En  la  CU,  la  inflamación  se  limita  a  la  mucosa,  mientras  que  en  la  enfermedad  de  Crohn  
puede  ser  una  inflamación  de  espesor  total.  La  distorsión  de  la  cripta  y  la  metaplasia  de  las  células  del  paneth  son  
hallazgos  histopatológicos  comunes  en  la  CU.  La  CU  grave  puede  tener  ulceraciones  y  erosiones  desde  la  mucosa  hacia  
la  submucosa  y  puede  confundirse  con  la  enfermedad  de  Crohn.

5.  Aunque  la  enfermedad  de  Crohn  puede  afectar  el  tracto  gastrointestinal  (GI)  desde  la  faringe  hasta  el  ano,  ¿cuáles  
son  los  patrones  clínicos  más  comunes  de  afectación  GI?

Solo  intestino  delgado:  28%;  tanto  íleon  como  colon  (ileocolitis):  41%;  y  colon  solo:  27%.  La  afectación  del  colon  por  
Crohn  también  se  denomina  colitis  de  Crohn  o  colitis  granulomatosa.

6.  La  colitis  de  Crohn  y  la  colitis  ulcerosa  suelen  ser  difíciles  de  distinguir  clínicamente.
¿Cuáles  son  las  principales  diferencias  observadas  en  la  colonoscopia?
La  colitis  de  Crohn  es  focal  y  predominantemente  del  lado  derecho.  Las  ulceraciones  aftosas  son  las  primeras  
lesiones  que  se  observan  en  la  enfermedad  de  Crohn.  A  medida  que  avanza  la  enfermedad,  pueden  formar  úlceras  
lineales  o  serpiginosas  (ulceraciones  en  garra  de  oso)  en  las  áreas  afectadas.  Cuando  estas  grandes  ulceraciones  migran  
transversalmente,  aparecen  islas  de  mucosa  normal,  lo  que  se  conoce  como  empedrado.  En  la  CU,  la  enfermedad  
comienza  en  el  recto  y  progresa  proximalmente.  Las  áreas  salteadas  son  más  comunes  en  la  enfermedad  de  Crohn,  
pero  tenga  en  cuenta  que  los  pacientes  con  CU  tratados  con  supositorios  o  enemas  rectales  pueden  tener  un  recto  de  
apariencia  normal.  Los  hallazgos  endoscópicos  en  la  CU  pueden  variar  desde  un  simple  edema  de  la  mucosa  hasta  
tejido  indurado  y  friable  que  sangra  al  contacto.

7.  ¿Cuáles  son  las  principales  indicaciones  de  cirugía  en  la  enfermedad  de  Crohn?
Depende  del  sitio  de  afectación  y  la  importancia  de  los  síntomas.  El  principio  rector  es  operar  solo  cuando  la  enfermedad  
está  afectando  la  fisiología  del  paciente.  Un  paciente  con  leve

248
Machine Translated by Google
CAPÍTULO  49  ENFERMEDAD  INFLAMATORIA  INTESTINAL  249

la  diarrea  y  una  fístula  enteroentérica  no  necesitan  cirugía  y  deben  tratarse  médicamente.
Un  paciente  con  una  fístula  enterorrectal  con  diarrea  profunda  y  pérdida  de  peso,  o  un  paciente  con  una  
obstrucción  intestinal  como  resultado  de  una  estenosis,  necesita  una  intervención  quirúrgica.  Enfermedad  perianal  
con  absceso  necesita  drenaje  urgente.

8.  ¿Cuáles  son  las  principales  indicaciones  de  cirugía  en  la  colitis  ulcerosa?
Intratabilidad  médica  (incluyendo  retraso  en  el  crecimiento  en  niños,  diarrea,  pérdida  de  peso  y  brotes  repetidos),  
incapacidad  para  tolerar  medicamentos  (psicosis  por  esteroides,  pancreatitis)  colitis  tóxica  aguda  con  o  sin  perforación,  
displasia  (células  premalignas  en  la  colonoscopia  de  vigilancia)  o  cáncer  colorrectal  son  las  indicaciones  de  cirugía  
en  la  CU.

9.  ¿Cuál  es  el  tratamiento  quirúrgico  de  la  colitis  ulcerosa?
Una  proctocolectomía  total  proporciona  una  cura  quirúrgica  para  la  CU.  La  mayoría  de  los  pacientes  no  desean  una  
ileostomía  permanente,  por  lo  que  se  realiza  una  reconstrucción  para  crear  un  neo­recto.  Se  puede  formar  una  bolsa  
hecha  con  el  intestino  delgado  del  paciente  y  anastomosarla  a  los  esfínteres  anales  para  proporcionar  continencia  
fecal.

10.  ¿Cuál  es  el  procedimiento  quirúrgico  para  una  estenosis  ileal?  Cuál  es  el  procedimiento
para  múltiples  estenosis?
Debido  a  que  la  cirugía  para  la  enfermedad  de  Crohn  no  es  curativa,  la  preservación  del  intestino  es  importante  
ya  que  los  procedimientos  repetidos  pueden  provocar  el  síndrome  del  intestino  corto.  Para  una  estenosis  aislada,  la  
resección  y  anastomosis  es  el  procedimiento  de  elección.  Se  debe  anastomosar  el  íleon  macroscópicamente  normal  con  el  
colon  macroscópicamente  normal.  Para  aquellos  pacientes  con  estenosis  múltiples,  se  realiza  un  procedimiento  llamado  
estricturoplastia.  Se  hace  una  incisión  longitudinal  a  lo  largo  de  la  estenosis  y  se  extiende  hacia  la  pared  intestinal  normal.
Luego,  la  abertura  se  cierra  transversalmente,  abriendo  la  luz  para  que  el  contenido  entérico  pueda  pasar.

11.  ¿Cómo  evalúa  la  colocación  de  un  estoma  (ostomía)?
La  ubicación  de  un  estoma  es  un  factor  importante  en  la  morbilidad  del  paciente  y  en  el  mantenimiento  de  una  
calidad  de  vida  aceptable.  La  colocación  es  óptima  cuando  el  intestino  pasa  a  través  de  la  vaina  del  recto  por  encima  
del  ombligo.  Esto  proporciona  la  fascia  más  fuerte  para  anclar  la  ostomía.  Una  forma  de  identificar  esta  área  es  trazar  
una  línea  desde  la  espina  ilíaca  anterosuperior  hasta  el  ombligo.  Es  importante  marcar  el  sitio  del  estoma  antes  de  la  
operación  con  el  paciente  acostado,  sentado  y  de  pie.  Trate  de  identificar  dónde  usa  el  paciente  los  cinturones  y  evite  
esta  área  y  la  piel  se  pliega  y  se  arruga.

12.  ¿Cómo  se  monitorea  a  un  paciente  con  CU  para  displasia?
Después  de  una  colonoscopia  índice  cuando  se  diagnostica  al  paciente,  la  colonoscopia  de  vigilancia  debe  
comenzar  después  de  8  a  10  años  de  enfermedad.  Las  biopsias  de  cuatro  cuadrantes  cada  10  cm  son  estándar  para  
lograr  al  menos  35  muestras.  Esto  proporciona  un  tamaño  de  muestra  adecuado  para  detectar  displasia.

13.  ¿La  EII  tiene  una  base  genética?
Se  han  identificado  cinco  enlaces  genéticos  confirmados  (lo  que  significa  que  se  encontró  un  enlace  significativo  en  
un  estudio  y  la  replicación  del  enlace  con  un  valor  de  P  nominal  <0,01  en  un  estudio  independiente)  para  la  
enfermedad  inflamatoria  intestinal  (EII):  (1)  enlace  entre  la  enfermedad  de  Crohn  y  un  locus  en  el  cromosoma  16  (el  
locus  IBD1);  (2)  vínculo  entre  IBD  (especialmente  UC)  y  un  locus  en  el  cromosoma  12q  (el  locus  IBD2);  (3)  vínculo  
entre  la  EII  (especialmente  la  enfermedad  de  Crohn)  y  un  locus  en  el  cromosoma  6p  (el  locus  IBD3);  (4)  vínculo  
entre  la  enfermedad  de  Crohn  y  un  locus  en  el  cromosoma  14q  (el  locus  IBD4);  y  (5)  vínculo  entre  la  EII  (especialmente  
la  enfermedad  de  Crohn)  y  un  locus  en  el  cromosoma  3p21.  Hay  otros  cuatro  vínculos  significativos  con  la  EII  que  
aún  no  se  han  confirmado  en  poblaciones  de  estudios  independientes.

14.  ¿Cuáles  son  algunas  de  las  terapias  médicas  para  la  enfermedad  inflamatoria  intestinal?
Muchos  tratamientos  de  primera  línea  implican  el  uso  de  medicamentos  antiinflamatorios  que  pueden  administrarse  
por  vía  oral  o  rectal.  Estos  compuestos  de  5­ASA  tienen  toxicidades  mínimas.  Otras  modalidades  incluyen
Machine Translated by Google
250  CAPÍTULO  49  ENFERMEDAD  INFLAMATORIA  INTESTINAL

medicamentos  inmunomoduladores.  Los  esteroides  son  el  pilar  del  tratamiento  para  los  brotes  o  la  exacerbación  
de  los  síntomas  de  la  enfermedad.  Los  medicamentos  inmunosupresores,  como  la  azatioprina,  su  metabolito  6­
mercaptopurina  e  incluso  la  ciclosporina,  se  han  utilizado  en  la  EII.  La  clase  más  nueva  de  medicamentos  utilizados,  los  
biológicos,  son  anticuerpos  contra  objetivos  específicos  (infliximab  y  adalimumab)  como  el  factor  de  necrosis  tumoral­a  
(TNF­a).  Los  antibióticos  también  se  pueden  usar  en  la  terapia  a  largo  plazo  de  pacientes  con  enfermedad  de  Crohn  y  
fístulas.

15.  ¿Qué  es  una  ileostomía  de  Brooke?
La  ileostomía  de  Brooke  es  una  ileostomía  de  espesor  total  doblada  sobre  sí  misma  aproximadamente  1  cm  por  
encima  de  la  piel.  Esto  permite  un  sellado  adecuado  entre  el  dispositivo  de  estoma  y  la  abertura  del  intestino.  Esto  evita  el  
derrame  de  contenido  intestinal  sobre  la  piel,  lo  que  puede  causar  una  irritación  e  inflamación  significativas  de  la  piel.

16.  ¿Qué  es  la  reservoritis  y  qué  pacientes  es  probable  que  la  padezcan?
La  reservoritis  se  define  como  una  inflamación  de  la  bolsa  del  intestino  delgado.  De  todos  los  pacientes  que  se  someten  
a  una  proctocolectomía  total  con  reservorio  ileoanal,  el  27%  tendrá  al  menos  un  episodio  de  reservoritis  en  su  vida.  
Aunque  no  se  ha  encontrado  la  causa,  la  investigación  se  ha  centrado  en  las  posibilidades  autoinmunes,  el  crecimiento  
excesivo  de  bacterias  en  el  intestino  delgado  y  la  falta  de  las  bacterias  apropiadas  que  generalmente  se  encuentran  en  el  
colon.  La  mayoría  de  los  episodios  se  tratan  con  antibióticos  como  fluoroquinolonas  y  metronidazol.  Los  casos  refractarios  
son  raros  y  pueden  requerir  medicamentos  antiinflamatorios  o  incluso  medicamentos  inmunosupresores.

La  reservoritis  rara  vez  requiere  la  escisión  de  la  bolsa.  Es  interesante  señalar  que  no  se  observa  en  pacientes  que  
tienen  una  bolsa  pélvica  por  otras  enfermedades,  como  la  poliposis  adenomatosa  familiar,  lo  que  da  crédito  a  la  
teoría  de  que  la  bolsa  puede  ser  el  resultado  del  mismo  mecanismo  que  la  EII.

CONTROVERSIAS

17.  ¿Todos  los  pacientes  con  fístulas  enteroenterales  secundarias  a  enfermedad  de  Crohn
operarse  cuando  se  descubre  la  fístula?
Para:  A  estos  pacientes  finalmente  les  va  mal,  desarrollan  más  complicaciones  sépticas  intraperitoneales
y  eventualmente  requieren  cirugía.
En  contra:  muchos  de  estos  pacientes  evolucionan  bien  sin  tratamiento  quirúrgico  con
gestión.  La  cirugía  no  es  una  cura,  así  que  ¿por  qué  someter  al  paciente  a  un  riesgo  innecesario?
Si  los  síntomas  empeoran  podemos  ofrecer  cirugía.

18.  En  caso  de  que  todos  los  pacientes  con  colitis  ulcerosa  documentada  durante  10  años,
independientemente  de  si  la  enfermedad  está  activa  o  no,  someterse  a  una  proctocolectomía  para  evitar  el  riesgo  de  
carcinoma  de  colon  y  recto?

A  favor:  El  riesgo  de  cáncer  de  colon  en  la  colitis  ulcerosa  aumenta  aproximadamente  un  1%  por  año  después  de  8  a  
10  años  de  enfermedad,  por  lo  que  esto  puede  eliminar  su  riesgo  de  cáncer.
En  contra:  mediante  la  colonoscopia  de  vigilancia  y  la  biopsia,  podemos  determinar  los  pacientes  que  están  en
alto  riesgo  de  cáncer,  entonces,  ¿por  qué  someter  a  cirugía  a  muchos  pacientes  normales?

19.  ¿La  anastomosis  ileorrectal  es  una  operación  aceptable  después  de  una  colectomía  por  colitis  ulcerosa?

Para:  estos  pacientes  tendrán  hábitos  intestinales  más  normales  y  evitarán  la  mayor  tasa  de  complicaciones
de  cirugía  pélvica.
En  contra:  Al  menos  el  50%  de  los  pacientes  eventualmente  requieren  reoperación  por  recurrencia  de
síntomas  de  la  enfermedad.  El  recto  restante  también  puede  ser  un  sitio  para  el  desarrollo  de
cáncer.
Machine Translated by Google
CAPÍTULO  49  ENFERMEDAD  INFLAMATORIA  INTESTINAL  251

20.  ¿Deberíamos  ofrecer  proctocolectomía  total  y  reservorio  ileal  a  pacientes  con  colitis  de  Crohn?

Para:  Con  el  uso  de  medicamentos  más  nuevos,  estos  pacientes  pueden  estar  libres  de  estoma  durante  muchos  años.
En  contra:  la  tasa  de  formación  de  fístulas  y  fallas  de  la  bolsa  hacen  que  esta  cirugía  sea  demasiado  riesgosa  en
Enfermedad  de  Crohn.

BIBLIOGRAFÍA

1.  Duerr  RH:  La  genética  de  la  enfermedad  inflamatoria  intestinal.  Gastroenterol  Clin  North  Am  31:63­76,  2002.

2.  Farouk  R:  resultados  funcionales  después  de  la  anastomosis  del  reservorio  ileal­anal  para  la  colitis  ulcerosa  crónica.  ann  surg
231:919­926,  2000.

3.  Fazio  V:  Estado  actual  de  la  cirugía  de  la  enfermedad  inflamatoria  intestinal.  Digestión  59:470­480,  1998.

4.  Heuschen  UA,  Hinz  U,  Allemeyer  EH  et  al .:  Procedimientos  de  una  o  dos  etapas  para  la  proctocolectomía  restauradora:
justificación  de  una  estrategia  en  la  colitis  ulcerosa.  Ann  Surg  234:788­794,  2001.

5.  Hurst  RD,  Michelassi  F:  Estricturoplastia  para  la  enfermedad  de  Crohn:  técnicas  y  resultados  a  largo  plazo.  Cirugía  mundial  J
22:359­363,  1998.

6.  Konda  A,  Duffy  MC:  Vigilancia  de  pacientes  con  mayor  riesgo  de  cáncer  de  colon:  enfermedad  inflamatoria  intestinal  y  otras  
afecciones.  Gastroenterol  Clin  37:191­213,  2008.

7.  Mathew  CG,  Lewis  CM:  Genética  de  la  enfermedad  inflamatoria  intestinal:  progreso  y  perspectivas.  Hum  Mol  Genet
13  (Número  de  revisión  1):  R161­R168,  2004.

8.  Presente  DH,  Rutgeerts  P,  Targan  S  et  al.:  Infliximab  para  el  tratamiento  de  fístulas  en  pacientes  con  enfermedad  de  Crohn.
New  Engl  J  Med  340:1398­1405,  1999.

9.  Solomon  MJ,  Schmirz  M:  Cáncer  y  enfermedad  inflamatoria  intestinal:  sesgo,  epidemiología,  vigilancia  y  tratamiento.  World  
J  Surg  22:352­358,  1998.

10.  Stocchi  L,  Pemberton  JH:  Bolsa  y  reservoritis.  Gastroenterol  Clin  North  Am  30:223­241,  2001.

11.  Sugarman  HJ:  anastomosis  reservorio  ileal­anal  sin  derivación  ileal.  Ann  Surg  232:530­541,  2000.

12.  Wolff  BG:  Factores  que  determinan  la  recurrencia  después  de  la  cirugía  para  la  enfermedad  de  Crohn.  World  J  Surg  22:  364­369,
1998.
Machine Translated by Google

SANGRADO  GASTROINTESTINAL  SUPERIOR
CAPÍTULO  
50

G.  Edward  Kimm,  Jr.,  MD,  y  Allen  T.  Belshaw,  MD

1.  ¿Qué  es  la  hemorragia  digestiva  alta?
Sangrado  proximal  al  ligamento  de  Treitz  (el  punto  de  transición  entre  el  duodeno  y  el  yeyuno).

2.  ¿Cuáles  son  las  causas  más  frecuentes  de  hemorragia  digestiva  alta?
En  orden  decreciente  de  frecuencia,  son  la  enfermedad  ulcerosa  péptica,  las  várices  esofagogástricas,  las  
malformaciones  vasculares  y  los  desgarros  de  Mallory­Weiss.  Todas  las  demás  causas  representan  <5%  de  los  casos.

3.  ¿Cuál  es  la  tasa  de  mortalidad  general  de  la  hemorragia  digestiva  alta?
Aproximadamente  el  10%  para  todos  los  pacientes.  La  mortalidad  es  mucho  más  alta  con  factores  comórbidos  como  
enfermedad  cardiaca,  pulmonar,  hepática  y  renal,  así  como  con  la  edad  avanzada  (>60  años)  y  grandes  necesidades  
de  transfusión  (>5  unidades  de  sangre).

4.  ¿Cuál  es  la  presentación  más  común  de  la  hemorragia  digestiva  alta?
El  ochenta  por  ciento  de  los  pacientes  presenta  melena  (la  sangre  es  un  catártico  y  los  pacientes  expulsan  heces  
negras,  alquitranadas  o  de  color  granate)  o  hematoquecia  (sangre  de  color  rojo  brillante  en  el  recto).
La  hematemesis  (emesis  de  color  rojo  brillante  o  café  molido)  es  diagnóstica  de  una  fuente  superior  de  
hemorragia  gastrointestinal  (GI).  El  sangrado  oculto  puede  presentarse  solo  con  heces  positivas  para  guayaco.

5.  ¿Cuánta  pérdida  de  sangre  gastrointestinal  es  necesaria  para  causar  melena?
Tan  poco  como  50  a  100  ml.  El  sangrado  oculto  (positivo  para  guayaco  o  Hematest)  se  puede  detectar  con  tan  solo  10  
ml  de  pérdida  de  sangre.

6.  Un  hombre  de  45  años  acude  al  servicio  de  urgencias  con  hematemesis  masiva,  taquicardia  e  hipotensión.  
¿Cuál  debe  ser  el  enfoque  inicial?

La  hemorragia  GI  aguda  requiere  un  abordaje  rápido  y  sistemático.  Como  en  todos  los  pacientes  críticos,  evalúe  
inicialmente  el  ABC  (vías  aéreas,  respiración  y  circulación).  Inicie  dos  líneas  intravenosas  (IV)  de  gran  calibre  y  
administre  1  L  de  lactato  de  Ringer  mientras  monitorea  al  paciente.
Coloque  una  sonda  nasogástrica  (NG)  y  un  catéter  de  Foley  e  irrigue  la  sonda  NG  con  solución  salina.  Enviar  sangre  
para  tipo  y  cross­match  y  pruebas  de  coagulación  y  función  hepática  (LFT).

7.  Este  paciente  se  estabiliza  después  de  sus  intervenciones.  ¿Tiene  algún  valor  la  historia  clínica  para  determinar  la  
causa  del  sangrado?
Sí.  Los  siguientes  son  pertinentes:     
Síntomas  previos  de  enfermedad  ulcerosa  péptica  o  uso  de  antiinflamatorios  no  esteroideos  (AINE):  hemorragia  
duodenal  o  úlcera  gástrica.
  Antecedentes  de  enfermedad  por  reflujo  gastroesofágico  (ERGE):  esofagitis.
  Consumo  excesivo  de  alcohol:  gastritis  o  varices  sangrantes.
  arcadas  o  vómitos  recientes:  desgarro  de  Mallory­Weiss.
  Pérdida  de  peso:  malignidad  del  tubo  digestivo  alto.

252
Machine Translated by Google
CAPÍTULO  50  SANGRADO  GASTROINTESTINAL  SUPERIOR  253

8.  ¿Qué  hallazgo  físico  puede  ser  útil  para  establecer  el  origen  del  sangrado?
El  examen  físico  generalmente  no  es  útil.  Los  estigmas  de  la  enfermedad  hepática  (ictericia,  cabeza  de  medusa,  
ascitis,  atrofia  muscular)  hacen  sospechar  hemorragia  varicosa  o  erosiones  gástricas  superficiales  múltiples.

9.  ¿Qué  porcentaje  de  pacientes  con  varices  esofágicas  conocidas  están  sangrando  por  las  varices  en  el  momento  
de  la  presentación?
Solo  el  50%.

10.  ¿El  aspirado  de  NGT  bilioso  o  claro  descarta  una  fuente  de  hemorragia  GI  superior?
No.  Aunque  la  aspiración  por  sonda  NG  puede  ser  útil  para  dirigir  la  búsqueda  de  un  sitio  de  sangrado,  se  
debe  tener  en  cuenta  que  la  tasa  de  falsos  negativos  puede  llegar  al  20%.

11.  ¿Qué  estudios  se  pueden  utilizar  para  determinar  el  origen  del  sangrado?
La  esofagogastroduodenoscopia  (EGD)  es  la  primera  y  mejor  prueba.  Los  estudios  con  bario  pueden  pasar  por  
alto  una  fuente  importante  de  hemorragia  digestiva  alta,  como  la  gastritis  erosiva,  e  interferir  con  otras  pruebas  más  
definitivas,  especialmente  la  arteriografía.  Las  exploraciones  nucleares  tienen  un  valor  limitado  en  la  hemorragia  
digestiva  alta  aguda.

12.  ¿Cuál  es  la  sensibilidad  de  EGD?
EGD  identifica  la  fuente  de  sangrado  en  hasta  el  95%  de  los  casos.  La  EGD  tiene  la  ventaja  de  visualizar  
directamente  el  origen  de  la  pérdida  de  sangre  y  brinda  la  oportunidad  de  biopsiar  una  lesión  y  realizar  maniobras  
terapéuticas.

13.  ¿Cómo  se  puede  usar  la  EGD  para  controlar  el  sangrado  no  varicoso?
Se  pueden  utilizar  intervenciones  de  inyección,  térmicas  y  mecánicas  (p.  ej.,  clips).  Un  metanálisis  reciente  mostró  
que  la  terapia  endoscópica  dual  fue  significativamente  superior  a  la  inyección  de  epinefrina  sola.

14.  ¿Qué  cantidad  de  sangrado  se  requiere  para  ver  un  "rubor"  en  la  arteriografía?
Menos  de  5  ml/min.  Aunque  la  angiografía  es  la  más  invasiva  de  estas  pruebas,  el  catéter  puede  dejarse  colocado  y  
utilizarse  para  administrar  vasopresina  terapéutica  o  embolización.

15.  ¿Qué  opciones  de  tratamiento  existen  para  controlar  el  sangrado  por  várices?
La  endoscopia  superior  con  ligadura  con  banda  es  más  eficaz  que  la  escleroterapia.  En  manos  experimentadas,  la  
colocación  de  un  tubo  de  Sengstaken­Blakemore  (un  tubo  de  doble  globo  que  permite  el  taponamiento  directo  de  las  
várices  gástricas  y  esofágicas)  controla  temporalmente  el  sangrado  en  el  90%  de  los  casos.  La  infusión  intravenosa  
(IV)  de  vasopresina  u  octreótido  debería  disminuir  el  flujo  sanguíneo  a  las  várices,  pero  tiene  menos  éxito  en  pacientes  
con  enfermedad  hepática  más  grave.

16.  ¿Cuáles  son  las  indicaciones  de  cirugía  en  pacientes  con  enfermedad  digestiva  alta?
¿hemorragia?
Alrededor  del  10%  de  los  pacientes  eventualmente  requieren  cirugía.  Las  indicaciones  
incluyen:     Hipotensión  persistente  o  shock  (fracaso  de  la  terapia  de  reanimación).
  Sangrado  recurrente  durante  el  tratamiento  médico  máximo.
  Pacientes  de  alto  riesgo  con  enfermedad  comórbida  significativa.
  Grandes  necesidades  de  transfusión  (transfusión  de  más  de  dos  tercios  del  volumen  sanguíneo  del  paciente  
en  24  horas).

17.  ¿Cuál  es  el  abordaje  quirúrgico  de  un  paciente  inestable  con  hemorragia  digestiva  alta  no  localizada  que  no  responde  a  
la  reanimación  inicial?
En  la  laporotomía  comience  con  una  generosa  gastroduodenotomía  centrada  sobre  el  píloro.  Si  esto  no  revela  una  
fuente  de  sangrado,  proceda  con  una  gastrotomía  proximal.
Machine Translated by Google
254  CAPÍTULO  50  SANGRADO  GASTROINTESTINAL  SUPERIOR

18.  Un  paciente  presenta  hematemesis  y  tiene  antecedentes  remotos  de  reparación  de  aneurisma  aórtico  abdominal.  
¿Qué  causa  poco  común  de  hemorragia  digestiva  alta  debe  considerarse?

Fístula  aortoduodenal.  Cualquier  paciente  con  antecedentes  de  cirugía  aórtica  y  evidencia  de  hemorragia  
gastrointestinal  debe  ser  investigado  agresivamente  por  fístula  aortoentérica.  El  estudio  de  elección  es  la  endoscopia.

19.  ¿Qué  es  una  úlcera  de  Dieulafoy?
Una  malformación  vascular  gástrica  con  una  arteria  submucosa  expuesta,  generalmente  dentro  de  los  2  a  5  cm  
de  la  unión  gastroesofágica.  Se  presenta  con  hematemesis  indolora,  a  menudo  masiva  (afortunadamente,  esto  
es  poco  común).

20.  Se  trata  a  un  paciente  ingresado  recientemente  con  una  laceración  hepática  traumática
no  quirúrgicamente  y  más  tarde  desarrolla  hematemesis  indolora.  ¿Qué  sospechas?
¿Cómo  debe  tratar  a  este  paciente?
La  hemobilia,  otra  causa  rara  de  hemorragia  digestiva  alta,  por  lo  general  ocurre  después  de  un  traumatismo  hepático  o  
una  resección  hepática.  El  tratamiento  consiste  en  embolización  angiográfica.

21.  ¿Cuáles  son  otras  causas  raras  de  hemorragia  digestiva  alta?
Estómago  en  sandía,  gastropatía  hipertensiva  portal,  neoplasia  GI  superior,  divertículo  duodenal  y  
pancreatitis  (que  provoca  erosión  en  la  arteria  esplénica  o  trombosis  de  la  vena  esplénica  con  hipertensión  
portal).

PUNTOS  CLAVE:  SANGRADO  GASTROINTESTINAL  SUPERIOR

1.  La  hemorragia  digestiva  alta  se  define  como  hemorragia  proximal  al  ligamento  de  Treitz.

2.  Las  causas  más  frecuentes  son  gastritis,  úlcera  duodenal,  várices  esofágicas,  úlcera  gástrica  benigna,  
esofagitis  y  desgarro  de  Mallory­Weiss.

3.  El  ochenta  por  ciento  de  los  pacientes  presentan  melena  o  hematoquecia.

4.  EGD  identifica  la  fuente  de  sangrado  en  el  95%  de  los  casos.

BIBLIOGRAFÍA

1.  Conrad  SA:  Hemorragia  digestiva  alta  aguda  en  pacientes  críticos:  causas  y  modalidades  de  tratamiento.  crítico
Care  Med  30:365­368,  2002.

2.  Fallah  MA,  Prakash  C,  Edmundowitz  S:  Hemorragia  gastrointestinal  aguda.  Med  Clin  North  Am  84:1183­1208,  2000.
3.  Jamieson  GG:  Estado  actual  de  las  indicaciones  de  cirugía  en  la  enfermedad  de  úlcera  péptica.  World  J  Surg  24:256,  2000.
4.  Kwan  V,  Norton  ID:  Manejo  endoscópico  de  hemorragia  gastrointestinal  superior  no  varicosa.  Cirugía  ANZ  J
77:222­230,  2007.

5.  Marmo  R,  Rotondano  G,  Piscopo  R  et  al.:  Terapia  dual  versus  monoterapia  de  úlceras  sangrantes  de  alto  riesgo:
un  metanálisis  de  ensayos  controlados.  Am  J  Gastroenterol  102:279­289,  2007.
6.  Savides  TJ,  Jensen  DM:  Endoscopia  terapéutica  para  hemorragia  gastrointestinal  no  varicosa.  Clínica  de  Gastroenterol
North  Am  29:465­487,  2000.
Machine Translated by Google

SANGRADO  GASTROINTESTINAL  INFERIOR CAPÍTULO  
51

Dra.  Kathleen  R.  Liscum

1.  Describa  el  tratamiento  inicial  de  un  paciente  que  presenta  hemorragia  gastrointestinal  (GI)  inferior  masiva.

El  tratamiento  comienza  con  la  reanimación.  Coloque  dos  catéteres  intravenosos  (IV)  de  gran  calibre  en  las  
extremidades  superiores.  Obtenga  los  niveles  de  hemoglobina  y  hematocrito,  tipo  de  sangre  y  pruebas  cruzadas.  
Se  debe  colocar  un  catéter  de  Foley  para  ayudar  a  controlar  el  estado  del  volumen.

2.  ¿Cuál  es  el  siguiente  paso  en  la  evaluación  del  paciente?
Se  debe  colocar  una  sonda  nasogástrica  (NG)  para  descartar  una  fuente  GI  superior.  Si  el  aspirado  es  bilioso,  
el  examinador  puede  estar  bastante  seguro  de  que  la  fuente  es  distal  al  ligamento  de  Treitz.
Sin  embargo,  si  el  aspirado  no  revela  bilis,  el  paciente  aún  puede  estar  sangrando  en  el  duodeno  con  un  píloro  
competente.

3.  ¿Cuáles  son  las  dos  causas  más  frecuentes  de  hemorragia  digestiva  baja  masiva?
Hemorragia  diverticular  (diverticulosis)  y  ectasias  vasculares  sangrantes.  Anteriormente  se  pensaba  que  la  
enfermedad  diverticular  era  la  causa  más  común  de  hemorragia  digestiva  baja,  pero  ahora  las  ectasias  vasculares  
son  bastante  frecuentes.

4.  ¿Cuáles  son  otras  posibles  causas  de  sangre  del  recto?
Cáncer  de  colon  
Enfermedad  inflamatoria  intestinal  
Pólipos  Trastornos  anorrectales  
(p.  ej.,  hemorroides,  fisuras)
Colitis  isquémica  
Divertículo  de  Meckel  
Colitis  infecciosa

5.  Después  de  una  anamnesis  y  un  examen  físico  exhaustivos,  ¿cuál  es  el  primer  paso  hacia
identificar  el  sitio  específico  del  sangrado?
Anoscopia  y  proctosigmoidoscopia  rígida  para  descartar  fuentes  anorrectales.  Antes  de  proceder  con  un  estudio  
exhaustivo  del  colon  intraperitoneal,  el  médico  debe  estar  seguro  de  que  la  etiología  no  se  encuentra  en  las  porciones  
más  distales  del  tracto  gastrointestinal.

6.  Mencione  cuatro  opciones  para  localizar  el  sangrado  gastrointestinal  inferior.
1.  Exploración  de  glóbulos  rojos  marcados.
2.  Exploración  con  coloides  de  azufre.

3.  Angiografía.
4.  Colonoscopia.

7.  Discuta  las  diferencias  entre  la  exploración  con  coloides  de  azufre  y  la  sangre  roja  etiquetada
escaneo  celular

La  exploración  con  coloides  de  azufre  se  puede  realizar  rápidamente  y  detecta  un  sangrado  mínimo  de  0,1  
ml/min.  El  hígado  y  el  bazo  eliminan  rápidamente  el  coloide  de  azufre  radiomarcado,  lo  que  puede

255
Machine Translated by Google
256  CAPÍTULO  51  HEMORRAGIA  GASTROINTESTINAL  INFERIOR

ocultar  el  sitio  de  sangrado  si  se  encuentra  en  el  ángulo  hepático  o  esplénico.  La  prueba  se  completa  dentro  de  los  20  
minutos  posteriores  a  la  administración  del  radionúclido.

La  exploración  de  glóbulos  rojos  etiquetados  requiere  un  retraso  de  60  minutos  mientras  los  glóbulos  rojos  autólogos  
se  etiquetan  con  isótopos.  La  prueba  detecta  un  sangrado  tan  lento  como  0,5  ml/min.  Debido  a  que  las  células  marcadas  
permanecen  en  el  sistema  del  paciente,  también  es  útil  para  identificar  la  fuente  cuando  el  paciente  sangra  de  forma  
intermitente.  El  estudio  se  puede  repetir  si  el  sangrado  reaparece  dentro  de  las  24  horas.  El  estudio  tarda  al  menos  2  
horas  en  realizarse.

8.  ¿Cuál  es  el  papel  de  la  angiografía?
Este  estudio  es  el  próximo  paso  óptimo  para  los  pacientes  que  tienen  sangrado  persistente  significativo  y  están  lo  
suficientemente  estables  hemodinámicamente  como  para  no  requerir  una  laparotomía  de  emergencia.
La  angiografía  detecta  tasas  de  sangrado  de  0,5  a  1,0  ml/min,  pero  solo  si  el  paciente  está  sangrando  activamente.  
Cuando  se  identifica  un  sitio  de  sangrado,  la  apariencia  angiográfica  puede  proporcionar  más  información  sobre  la  causa  
del  sangrado.  Mientras  que  el  sangrado  diverticular  a  menudo  se  observa  como  una  extravasación  de  contraste,  las  
ectasias  vasculares  pueden  identificarse  por  un  penacho  vascular  o  una  vena  de  llenado  temprano.

9.  ¿Qué  opciones  terapéuticas  están  disponibles  con  la  angiografía?
(1)  Infusión  de  vasopresina  (Pitressin)  en  un  vaso  seleccionado  y  (2)  embolización  del  vaso  sangrante.

10.  ¿Qué  papel  debe  jugar  la  colonoscopia  en  la  evaluación  del  paciente  con  menor
¿hemorragia  gastrointestinal?
Para  los  pacientes  con  sangrado  intermitente  leve,  la  colonoscopia  es  un  buen  estudio  para  localizar  el  origen.  Hay  un  
poco  más  de  controversia  sobre  el  papel  de  la  colonoscopia  para  pacientes  con  pérdida  de  sangre  de  mayor  volumen  que  
continúan  sangrando.  Unos  pocos  estudios  muestran  una  estadía  hospitalaria  más  corta  asociada  con  la  endoscopia  
temprana  en  este  grupo  de  pacientes,  mientras  que  otro  estudio  mostró  que  la  colonoscopia  tenía  más  probabilidades  de  
identificar  el  origen  del  sangrado,  pero  esta  identificación  no  dio  como  resultado  ninguna  reducción  en  la  mortalidad  o  los  
requisitos  de  transfusión.

PUNTOS  CLAVE:  SANGRADO  GASTROINTESTINAL  INFERIOR
1.  Las  causas  más  frecuentes  de  hemorragia  digestiva  baja  masiva  son  la  hemorragia  diverticular  y  las  ectasias  
vasculares  hemorrágicas.

2.  La  causa  más  frecuente  de  hemorragia  digestiva  baja  en  niños  es  el  divertículo  de  Meckel.

3.  Después  de  una  historia  clínica  y  un  examen  físico  exhaustivos,  los  primeros  pasos  para  identificar  el
el  sitio  de  sangrado  son  la  anoscopia  y  la  proctosigmoidoscopia  rígida.

4.  La  gammagrafía  con  glóbulos  rojos  marcados,  la  gammagrafía  con  coloides  de  azufre,  la  colonoscopia  y  la  angiografía  son  cuatro  

opciones  para  localizar  el  sangrado  gastrointestinal  inferior.

5.  Las  indicaciones  para  la  cirugía  incluyen  pacientes  que  han  recibido  6  U  de  sangre  sin  resolución  del  sangrado  y  
pacientes  que  continúan  sangrando  después  de  la  vasopresina  o  la  embolización  o  que  están  demasiado  inestables  
para  someterse  a  cualquier  procedimiento  de  localización.

11.  ¿A  qué  pacientes  se  les  debe  realizar  una  embolización  angiográfica  del  sitio  de  sangrado?
La  mayoría  de  los  cirujanos  cree  que  la  embolización  debe  reservarse  para  pacientes  que  tienen  riesgos  operativos  bajos  
en  los  que  se  asocia  una  tasa  de  complicaciones  del  15%  con  el  procedimiento.  Los  pacientes  pueden  perforarse  o  
desarrollar  una  estenosis  como  resultado  de  la  isquemia  de  la  pared  intestinal.
Machine Translated by Google
CAPÍTULO  51  HEMORRAGIA  GASTROINTESTINAL  INFERIOR  257

12.  ¿Cuál  es  el  papel  de  la  infusión  de  vasopresina?
La  vasopresina  es  sólo  una  medida  temporal.  El  control  del  sangrado  con  vasopresina  da  tiempo  para  la  resucitación  
y  esencialmente  convierte  un  caso  emergente  en  uno  urgente.  En  ocasiones,  la  vasopresina  se  puede  utilizar  como  
único  tratamiento  para  la  hemorragia  diverticular.  Si  el  paciente  tiene  un  episodio  repetido  de  sangrado  después  del  
destete  de  la  vasopresina,  el  cirujano  debe  decidir  entre  la  embolización  y  la  cirugía.

13.  ¿Alguna  vez  se  resuelven  espontáneamente  las  hemorragias  del  tubo  digestivo  bajo?
La  resolución  espontánea  ocurre  en  el  75%  de  los  pacientes  con  ectasias  vasculares  y  en  el  90%  de  los  pacientes  
con  hemorragia  diverticular.

14.  ¿Cuáles  son  las  indicaciones  para  la  intervención  quirúrgica?
Cuando  el  paciente  ha  recibido  6  unidades  de  sangre  (dos  tercios  del  volumen  sanguíneo  del  paciente  en  24  
horas)  sin  resolución  del  sangrado.  Cualquier  paciente  que  continúe  sangrando  o  tenga  sangrado  recurrente  después  
de  la  vasopresina  o  la  embolización  debe  someterse  a  una  resección.

15.  ¿Cuál  es  el  papel  de  la  colectomía  subtotal  ciega  en  el  manejo  de  pacientes  con
¿Sangrado  gastrointestinal  bajo  masivo?
La  colectomía  subtotal  a  ciegas  se  limita  al  pequeño  grupo  de  pacientes  en  los  que  no  se  puede  identificar  una  
fuente  de  sangrado  específica.  El  procedimiento  se  asocia  con  una  tasa  de  mortalidad  del  16%.  Los  pacientes  más  
jóvenes  tienden  a  tolerar  mejor  el  procedimiento  que  los  pacientes  de  edad  avanzada.  Los  pacientes  mayores  a  
menudo  sufren  de  diarrea  severa,  urgencia  e  incontinencia.  Sin  embargo,  la  colectomía  segmentaria  a  ciegas  se  
asocia  con  una  tasa  de  mortalidad  aún  mayor  (40  %)  y  una  tasa  de  resangrado  del  50  %.

16.  ¿Cuál  es  la  causa  más  común  de  hemorragia  digestiva  baja  en  el  pediatra?
¿población?
Divertículo  de  Meckel.

BIBLIOGRAFÍA

1.  Biondo  S,  Kreisler  E,  Millan  M  et  al.:  Diferencias  en  los  resultados  postoperatorios  y  a  largo  plazo  del  paciente  entre
Cáncer  colónico  obstructivo  y  perforado.  Am  J  Surg  195:427­432,  2008.

2.  Cynamon  J,  Atar  E,  Steiner  A  et  al .:  Vasoespasmo  inducido  por  catéter  en  el  tratamiento  del  sangrado  gastrointestinal  inferior  agudo.  J  
Vasc  Interv  Radiol  14:211­216,  2003.

3.  Green,  BT,  Rockey,  DC,  Portwood  G  et  al .:  Colonoscopia  urgente  para  la  evaluación  y  el  tratamiento  de  la  hemorragia  gastrointestinal  
inferior  aguda:  un  ensayo  controlado  aleatorio.  Am  J  Gastroenterol  100:2395,  2005.

4.  Lee  YS,  Lee  IK,  Kang  WK  et  al.:  Resultados  quirúrgicos  y  patológicos  de  la  cirugía  laparoscópica  para  el  cáncer  de  colon  transverso,  
Int  J  Colorrectal  Dis  23:669­673,  2008.  Epub  1  de  abril  de  2008.

5.  Mallant­Hent  RC,  Van  Bodegraven  AA,  Meuwissen  SG  et  al .:  enfoque  alternativo  al  sangrado  gastrointestinal  masivo  en  la  colitis  
ulcerosa:  embolización  transcatéter  altamente  selectiva.  Eur  J  Gastroenterol  Hepatol  15:189­193,  2003.

6.  Schmulewitz  N,  Fisher  DA,  Rockey  DC:  La  colonoscopia  temprana  para  el  sangrado  GI  inferior  agudo  predice  una  estancia  hospitalaria  
más  corta:  un  estudio  retrospectivo  de  la  experiencia  en  un  solo  centro.  Gastrointest  Endosc  58:841,  2003.

7.  Setya  V,  Singer  JA,  Minken  SL:  Colectomía  subtotal  como  último  recurso  para  pacientes  inferiores  implacables,  no  localizados
hemorragia  digestiva:  experiencia  con  12  casos.  Am  Surg  58:295­299,  1992.

8.  Estrato  LL:  Hemorragia  digestiva  baja:  epidemiología  y  diagnóstico.  Gastroenterol  Clin  North  Am  34:643­664,  2005.

9.  Strate  LL,  Syngal  S:  Momento  de  la  colonoscopia:  impacto  en  la  duración  de  la  estancia  hospitalaria  en  pacientes  con  hemorragia  
intestinal  aguda  inferior.  Am  J  Gastroenterol  98:317,  2003.

10.  Zuccaro  G:  Manejo  del  paciente  adulto  con  hemorragia  digestiva  baja  aguda.  Am  J  Gastroenterol
93:1202,  1998.
Machine Translated by Google

PÓLIPOS  COLORRECALES
CAPÍTULO  
52

Christina  L.  Roland,  MD  y  Carlton  C.  Barnett,  Jr.,  MD

1.  ¿Qué  son  los  pólipos?
Un  pólipo  es  una  elevación  de  la  superficie  de  la  mucosa  que  puede  ocurrir  en  cualquier  parte  del  tracto  gastrointestinal  (GI).  
Dos  tercios  de  los  pólipos  ocurren  en  el  colon  rectosigmoideo  y  descendente.

2.  ¿Cuáles  son  los  principales  tipos  de  pólipos?
Los  pólipos  se  clasifican  por  sus  características  morfológicas;  ya  sea  pedunculada  o  sésil.
Los  pólipos  pedunculados  tienen  una  cabeza  unida  por  un  tallo  a  la  mucosa  del  colon  o  del  recto.
El  tallo  suele  estar  cubierto  de  mucosa  normal  y  <1,5  cm.
Los  pólipos  sésiles  descansan  sobre  una  base  ancha.

En  cualquier  tipo,  la  muscular  de  la  mucosa  es  un  hito  importante  para  diferenciar  las  lesiones  invasivas  de  
las  no  invasivas.  Los  linfáticos  y  las  venas  no  se  extienden  a  través  de  la  muscularis  mucosa.  Las  lesiones  
submucosas,  como  los  carcinoides  y  los  lipomas,  pueden  parecerse  a  los  pólipos  colorrectales.

3.  ¿A  qué  edad  se  desarrollan  los  pólipos?
Los  pólipos  colorrectales  adenomatosos  ocurren  con  poca  frecuencia  antes  de  los  30  años.  La  incidencia  aumenta  con  la  
edad.  Sin  embargo,  las  series  de  autopsias  informan  una  frecuencia  microscópica  de  hasta  el  75%  en  pacientes  mayores  
de  45  años.  La  incidencia  clínica  es  de  hasta  un  40%  en  personas  mayores  de  60  años.

4.  ¿Qué  pólipos  no  tienen  potencial  maligno?
Los  pólipos  hiperplásicos  (metaplásicos)  son  pequeños  (1  a  5  mm)  y  son  10  veces  más  comunes  que  los  adenomas.  A  
diferencia  de  los  pólipos  adenomatosos,  los  pólipos  hiperplásicos  son  causados  por  una  falla  de  las  células  de  la  mucosa  
para  diseminarse  sobre  la  luz  de  la  mucosa.  Estas  células  se  acumulan  en  la  superficie  luminal,  formando  un  pólipo  de  
mucosa  engrosada.
Los  hamartomas  son  tumores  benignos  en  un  órgano  (colon)  compuestos  por  elementos  de  tejido  que  normalmente  se  
encuentran  en  ese  sitio  pero  que  crecen  en  una  masa  desorganizada.
Los  pólipos  inflamatorios  son  comunes  en  enfermedades  como  la  colitis  ulcerosa  (CU),  la  enfermedad  de  Crohn  y  
la  esquistosomiasis.  Representan  islas  de  epitelio  mucoso  cicatrizado  o  en  proceso  de  curación  que  no  son  permanentes.  
La  aparición  de  pólipos  inflamatorios  en  realidad  refleja  la  gravedad  de  la  enfermedad  subyacente.

5.  ¿Qué  pólipos  tienen  potencial  maligno?
Los  pólipos  adenomatosos  pueden  ser  precursores  del  cáncer.  Hay  tres  tipos  histológicos  de  pólipos  adenomatosos.  
Los  pólipos  que  contienen  >75%  de  elementos  glandulares  se  denominan  tubulares.  Los  que  contienen  >75  %  de  
elementos  vellosos  se  denominan  vellosos,  y  los  que  contienen  >25  %  de  elementos  tanto  glandulares  como  vellosos  se  
denominan  tubulovellosos.
En  general,  del  65%  al  80%  de  todos  los  pólipos  adenomatosos  extirpados  son  tubulares;  10%  a  25%  son  tubulovillosos;  
y  solo  del  5%  al  10%  son  adenomas  vellosos  puros.  Los  adenomas  tubulares  suelen  ser  pedunculados  y  los  adenomas  
vellosos  suelen  ser  sésiles,  pero  todos  los  tipos  histológicos  pueden  verse  en  cada  tipo  de  pólipo.

258
Machine Translated by Google
CAPÍTULO  52  PÓLIPOS  COLORRECALES  259

6.  ¿Algunos  tipos  de  pólipos  se  asocian  más  frecuentemente  con  el  adenocarcinoma?
Sí.  Los  pólipos  vellosos  son  "malos  actores".  Coutsoftide  y  sus  colegas  informaron  una  incidencia  del  5,6%,  
16%  y  41%  de  adenocarcinoma  en  adenomas  tubulares,  vellotubulares  y  vellosos,  respectivamente.

7.  ¿Cuál  es  la  relación  entre  el  tamaño  de  los  pólipos  y  el  riesgo  de  adenocarcinoma?
Los  pólipos  <2  cm  tienen  un  riesgo  del  2  %  de  contener  cáncer,  los  pólipos  de  2  cm  tienen  un  riesgo  del  10  %  y  los  
pólipos  >2  cm  tienen  un  riesgo  de  cáncer  del  40  %.  El  60%  de  los  pólipos  vellosos  miden  >2  cm  y  el  77%  de  los  
pólipos  tubulares  miden  <1  cm  en  el  momento  del  descubrimiento.

8.  ¿Qué  son  los  pólipos  juveniles?
Los  pólipos  juveniles  son  dilataciones  quísticas  de  estructuras  glandulares  dentro  de  la  lámina  propia  y  ocurren  en  el  
colon  y  el  recto  de  bebés,  niños  y  adolescentes.  La  causa  de  estos  pólipos  no  está  clara.  Pueden  representar  una  
respuesta  a  la  inflamación.  Los  pólipos  juveniles  son  la  causa  más  común  de  hemorragia  gastrointestinal  en  niños  o  
pueden  servir  como  puntos  de  partida  para  la  invaginación  intestinal.
Deben  dejarse  solos  a  menos  que  causen  problemas,  momento  en  el  que  la  polipectomía  endoscópica  suele  ser  un  
tratamiento  suficiente.

9.  ¿Cómo  se  diagnostican  los  pólipos  colorrectales?
La  prueba  de  sangre  oculta  en  heces  (FOBT,  por  sus  siglas  en  inglés)  es  la  prueba  más  común  en  los  Estados  
Unidos  que  conduce  al  descubrimiento  de  pólipos.  La  sigmoidoscopia  y  la  colonoscopia  confirman  el  diagnóstico.  La  
colonoscopia  tiene  la  ventaja  de  ser  diagnóstica  y  potencialmente  terapéutica.

10.  ¿Cuáles  son  los  riesgos  de  la  colonoscopia?
Sangrado  y  perforación.  Para  la  colonoscopia  diagnóstica,  estos  riesgos  son  extremadamente  bajos,  1,0  %  y  0,2  
%,  respectivamente.  Ambos  riesgos  siguen  siendo  <1%  para  la  colonoscopia  terapéutica.  El  sangrado  generalmente  
se  detiene  por  sí  solo  y  rara  vez  requiere  una  laparotomía.

11.  ¿Cómo  se  puede  determinar  si  la  polipectomía  endoscópica  es  adecuada?
¿tratamiento?
En  general,  si  se  puede  obtener  un  margen  >1  mm,  no  hay  invasión  de  la  muscularis  mucosa  y  el  grado  histológico  
de  la  lesión  es  I  o  II  (bien  a  moderadamente  bien  diferenciado),  se  debe  ofrecer  al  paciente  una  polipectomía  
endoscópica.  Los  pólipos  con  márgenes  <1  mm,  invasión  de  vasos  o  linfáticos  y  lesiones  histológicas  de  grado  III  
(poco  diferenciadas)  deben  someterse  a  resección  de  colon,  a  menos  que  las  condiciones  médicas  comórbidas  
contraindiquen  la  cirugía.

12.  ¿Cuáles  son  las  recomendaciones  de  cribado  para  detectar  pólipos?
La  Sociedad  Estadounidense  del  Cáncer  y  la  Asociación  Estadounidense  de  Gastoenterología  recomiendan  que  los  
hombres  y  las  mujeres  con  un  riesgo  promedio  de  cáncer  de  colon  sigan  uno  de  los  siguientes  programas:  FOBT  
anual  con  sigmoidoscopia  de  detección  cada  5  años,  enema  de  bario  de  doble  contraste  cada  5  años  o  colonoscopia  
cada  10  años .  Todas  las  pruebas  positivas  deben  ser  seguidas  con  una  colonoscopia.  Los  pacientes  con  mayor  
riesgo  (antecedentes  personales  de  cáncer  de  colon  o  pólipos,  antecedentes  familiares  importantes,  antecedentes  de  
enfermedad  inflamatoria  intestinal  [EII]  o  antecedentes  familiares  de  síndromes  de  cáncer  colorrectal  hereditario)  
deben  comenzar  a  hacerse  pruebas  de  detección  a  los  40  años  o  10  años  antes  que  el  cáncer  más  joven.  diagnóstico  
en  la  familia,  lo  que  ocurra  primero.

La  sensibilidad  y  especificidad  para  FOBT  es  40%  y  96%,  respectivamente,  para  pólipos  colorrectales.
La  sigmoidoscopia  y  la  colonoscopia  tienen  una  sensibilidad  del  90  %  y  una  especificidad  del  96  %  para  los  pólipos.
Sin  embargo,  la  sigmoidoscopia  no  permite  la  evaluación  del  colon  proximal,  lo  que  reduce  la  eficacia  general  de  esta  
técnica  de  detección.  Aunque  la  colonoscopia  es  muy  eficaz,  debe  ser  realizada  por  personas  capacitadas  y  conlleva  
el  riesgo  de  la  anestesia,  lo  que  compromete  su  valor  como  herramienta  de  detección.
Machine Translated by Google
260  CAPÍTULO  52  PÓLIPOS  COLORRECALES

13.  ¿Cuáles  son  las  recomendaciones  de  detección  para  pacientes  con  pólipos  conocidos?
Los  pacientes  con  bajo  riesgo  (1  o  2  adenomas  tubulares  <1  cm  con  displasia  de  bajo  grado)  deben  repetir  la  
colonoscopia  a  los  5  a  10  años.  Si  se  descubre  que  los  pacientes  tienen  múltiples  pólipos  o  lesiones  de  alto  
grado,  deben  someterse  a  una  colonoscopia  a  intervalos  más  frecuentes  (2  a  3  años).
Si  se  descubre  que  los  pacientes  no  tienen  lesiones  nuevas  después  de  un  ciclo  de  detección,  la  detección  
puede  extenderse  a  cada  5  años.

14.  ¿Qué  síndromes  clínicos  se  asocian  con  pólipos  colorrectales?
La  poliposis  adenomatosa  familiar  (FAP)  o  poliposis  coli  adenomatosa  (APC)  se  hereda  como  un  rasgo  autosómico  
dominante  caracterizado  por  múltiples  pólipos  adenomatosos  en  todo  el  tracto  GI.  El  diagnóstico  se  realiza  
clínicamente  al  observar  al  menos  100  pólipos  adenomatosos  en  el  colon;  más  de  1000  se  encuentran  en  muchos  
casos.  La  FAP  es  causada  por  la  pérdida  del  gen  supresor  de  tumores  APC  en  el  brazo  largo  del  cromosoma  5.  Con  
frecuencia,  a  varios  miembros  de  la  familia  se  les  diagnostica  cáncer  colorrectal,  generalmente  a  una  edad  temprana.  
El  sangrado,  la  diarrea  y  el  dolor  abdominal  son  síntomas  comunes  de  presentación.  La  FAP  está  asociada  con  un  
riesgo  de  cáncer  de  casi  el  100  %.  La  FAP  también  se  asocia  con  el  intestino  delgado,  especialmente  con  pólipos  
periampulares  o  cáncer  y  osteoma  mandibular.
El  síndrome  de  Gardner  también  se  asocia  con  la  pérdida  del  gen  APC.  Los  pacientes  tienen  poliposis,
al  igual  que  los  pacientes  con  FAP,  pero  también  tienen  osteomas  del  cráneo,  quistes  epidermoides,  anomalías  
en  la  pigmentación  de  la  retina  y  múltiples  tumores  de  tejidos  blandos  (desmoides).
El  síndrome  de  Turcot  también  se  asocia  con  mutaciones  de  APC  y  se  caracteriza  clínicamente  por
tumores  del  sistema  nervioso  central  (SNC)  y  pólipos  adenomatosos  múltiples.
El  síndrome  de  Peutz­Jeghers  consiste  en  múltiples  pólipos  hamartomatosos  en  todo  el  tracto  alimentario.  
Estos  pólipos  se  asocian  con  manchas  melanóticas  cutáneas  en  los  labios,  dentro  de  la  orofaringe  y  el  dorso  de  los  
dedos  de  manos  y  pies.  El  potencial  maligno  es  bastante  bajo.

15.  ¿Cuál  es  la  evolución  natural  de  la  poliposis  coli  adenomatosa?
Una  revisión  de  más  de  1000  casos  de  APC  mostró  que  la  edad  promedio  en  el  momento  del  diagnóstico  de  pólipos  
fue  de  34  años  y  la  edad  promedio  en  el  momento  del  diagnóstico  de  cáncer  colorrectal  fue  de  40  años.  La  edad  
media  de  muerte  fue  de  43  años.  Ahora  se  recomienda  que  los  pacientes  con  APC  se  sometan  al  menos  a  una  
colectomía  casi  total  a  los  25  años.  Los  pacientes  también  están  en  riesgo  de  desarrollo  tardío  de  adenocarcinoma  
del  intestino  anterior,  lo  que  requiere  la  necesidad  de  una  endoscopia  superior  cada  3  a  5  años.  A  pesar  de  la  
colectomía  profiláctica,  este  grupo  de  pacientes  no  tendrá  una  expectativa  de  vida  normal.

16.  ¿Cuáles  son  las  opciones  de  tratamiento  quirúrgico  de  la  poliposis  coli  adenomatosa?
Las  opciones  de  tratamiento  incluyen  proctocolectomía  total  con  ileostomía  permanente,  colectomía  
abdominal  con  preservación  rectal,  colectomía  abdominal  con  anastomosis  ileorrectal  y  anastomosis  ileoanal  con  
bolsa.  En  pacientes  en  los  que  se  conserva  el  recto,  es  necesaria  una  vigilancia  endoscópica  anual.

17.  ¿Qué  papel  juegan  los  defectos  genéticos  en  la  progresión  de  los  pólipos  colorrectales  a
adenocarcinoma?
Se  cree  que  la  progresión  de  pólipos  adenomatosos  a  cáncer  colorrectal  implica  una  acumulación  de  
defectos  genéticos  a  través  de  la  activación  de  protooncogenes  y/o  la  activación  de  genes  supresores  de  tumores.  
Los  pólipos  de  colon  han  proporcionado  el  mejor  modelo  disponible  de  mutaciones  genéticas  en  la  progresión  del  
tejido  normal  al  cáncer.  Vogelstein  y  otros  han  proporcionado  una  descripción  elegante  de  eventos  genéticos  en  los  
que  los  pólipos  acumulan  alteraciones  de  un  gen  supresor  de  tumores  en  el  cromosoma  18,  y  los  carcinomas  se  
relacionan  con  la  inactivación  del  gen  supresor  de  tumores  TP53  con  pérdida  de  la  función  de  la  proteína  p53.

18.  ¿Qué  papel  juegan  los  oncogenes  en  el  desarrollo  del  adenocarcinoma  de
pólipos  adenomatosos?
Los  oncogenes  son  copias  de  genes  celulares  normales  que  han  sido  activados  por  mutación.  Las  mutaciones  
activadoras  de  un  alelo  de  un  oncogén  pueden  interrumpir  el  crecimiento  y  la  diferenciación  celular  normal  y
Machine Translated by Google
CAPÍTULO  52  PÓLIPOS  COLORRECALES  261

aumentar  la  probabilidad  de  transformación  neoplásica.  El  gen  Ki­ras  es  el  oncogén  mutado  con  mayor  
frecuencia  en  la  neoplasia  colónica  esporádica.  Se  han  observado  mutaciones  puntuales  en  el  gen  K­ras  en  
aproximadamente  el  40%  de  los  adenomas  y  carcinomas  colorrectales  esporádicos.  Se  ha  propuesto  el  análisis  
de  mutaciones  en  el  ADN  de  las  células  que  se  desprenden  de  las  heces  como  una  forma  potencialmente  útil  de  
detectar  el  cáncer  colorrectal.  Además,  la  activación  de  la  tirosina  quinasa  del  producto  del  gen  c­src  pp60c­src  es  
frecuente  en  pólipos  de  alto  potencial  maligno;  la  actividad  de  la  tirosina  quinasa  está  significativamente  elevada  
por  encima  del  nivel  de  los  tumores  primarios  en  las  metástasis  hepáticas.

PUNTOS  CLAVE:  PÓLIPOS  COLORRECALES

1.  Un  pólipo  es  una  elevación  de  la  superficie  de  la  mucosa  que  puede  ocurrir  en  cualquier  parte  del  tracto  GI.

2.  Los  pólipos  hiperplásicos  son  pequeños  y  constituyen  >90%  de  los  pólipos  en  el  colon  y  el  recto.

3.  Los  pólipos  que  contienen  >75%  de  elementos  glandulares  se  denominan  tubulares;  aquellos  con  >75%  de  
elementos  vellosos  se  denominan  vellosos;  y  los  que  contienen  >25%  de  elementos  tanto  glandulares  como  
vellosos  se  denominan  tubulovellosos.

4.  FOBT  es  la  prueba  más  común  en  los  Estados  Unidos  que  conduce  al  descubrimiento  de  pólipos.

BIBLIOGRAFÍA

1.  Ahnen  DJ,  Feigl  P,  Quan  G:  La  mutación  y  la  sobreexpresión  de  Ki­ras  predicen  el  comportamiento  clínico  del  cáncer  colorrectal:  
un  estudio  del  Southwest  Oncology  Group.  Cáncer  Res  58:1149­1158,  1998.

2.  Brosens  LA,  van  Hattem  WA,  Jansen  M  et  al:  Síndromes  de  poliposis  gastrointestinal.  Curr  Mol  Med
7:29­46,  2007.

3.  Levine  JS,  Ahnen  DJ:  Pólipos  adenomatosos  del  colon.  N  Engl  J  Med  355:2551­2557,  2006.

4.  Duffy  MJ,  van  Dalen  A,  Haglund  C  et  al.:  Marcadores  tumorales  en  cáncer  colorrectal:  Directrices  del  Grupo  Europeo  sobre  
Marcadores  Tumorales  (EGTM)  para  uso  clínico.  Eur  J  Cancer  43:1348­1360,  2007.

5.  Fukami  N,  Lee  JH:  Tratamiento  endoscópico  de  grandes  lesiones  colorrectales  sésiles  y  planas.  Curr  Opin  Gastroenterol
22:54­59,  2006.

6.  Gallagher  MC,  Phillips  RK,  Bulow  S:  Vigilancia  y  manejo  de  enfermedades  gastrointestinales  superiores  en  Familial
Poliposis  adenomatosa.  Fam  Cancer  5:263­273,  2006.

7.  Hahn  WC,  Weinberg  RA:  Reglas  para  hacer  células  tumorales  humanas.  N  Engl  J  Med  347:1593­1602,  2002.

8.  Hassan  C,  Zullo  A,  Winn  S  et  al .:  El  pólipo  maligno  colorrectal:  abordando  un  dilema.  Excavar  enfermedad  del  hígado
39:92­100,  2007.

9.  Nivatvongs  S,  Rojanasakul  A,  Reimann  HM  et  al .:  El  riesgo  de  metástasis  en  los  ganglios  linfáticos  en  pólipos  colorrectales  con
adenocarcinoma  invasivo.  Dis  Colon  Rectum  34:323­328,  1991.

10.  Ransahoff  DF,  Sandler  RS:  Detección  de  cáncer  colorrectal.  N  Engl  J  Med  346:4044,  2002.

11.  Shih  IM,  Wang  TL,  Traverso  G:  Morfogénesis  de  arriba  hacia  abajo  de  los  tumores  colorrectales.  Proc  Natl  Acad  Sci  EE.  UU.
27:2640­2645,  2001.

12.  Winawer  S,  Fletcher  R,  Rex  D  et  al.:  Detección  y  vigilancia  del  cáncer  colorrectal:  guías  clínicas  y
justificación:  actualización  basada  en  nueva  evidencia.  Gastroenterología  124:544­560,  2003.

13.  Winawer  SJ,  Zauber  AG,  Fletcher  RH  et  al.:  Pautas  para  la  vigilancia  de  la  colonoscopia  después  de  la  polipectomía:
una  actualización  consensuada  del  Grupo  de  Trabajo  de  Sociedades  Múltiples  de  EE.  UU.  sobre  el  Cáncer  Colorrectal  y  la  Sociedad  Estadounidense  del  Cáncer.
CA  Cancer  J  Clin  56:143­159,  2006.

14.  Coutsoftides  T,  Lavery  I,  Benjamin  SP,  Sivak  MV  Jr.:  pólipos  malignos  del  colon  y  el  recto:  un  estudio  clinicopatológico.  
Dis  Colon  Rectum  22(2):82­86,  1979.
Machine Translated by Google

CARCINOMA  COLORRECTAL
CAPÍTULO  
53

Dra.  Kathleen  R.  Liscum

1.  ¿Cuáles  son  las  tres  principales  causas  de  muerte  por  cáncer  en  los  Estados  Unidos?
Cáncer  de  pulmón,  mama  o  próstata  y  colon.

2.  Enumere  algunos  de  los  síntomas  de  presentación  de  los  pacientes  con  cáncer  colorrectal.
Sangrado  rectal  intermitente,  dolor  abdominal  vago,  fatiga  secundaria  a  anemia,  cambio  en  los  hábitos  intestinales,  
estreñimiento,  tenesmo  y  dolor  perineal.

3.  ¿Qué  opciones  están  disponibles  para  evaluar  a  un  paciente  que  tiene  guayaco  positivo?
taburetes?
Para  evaluar  todo  el  colon  y  el  recto,  se  puede  realizar  un  enema  de  bario  y  una  proctoscopia  o  una  
colonoscopia.  La  colonoscopia  es  10  veces  más  costosa  pero  es  más  sensible  para  lesiones  <1  cm.

4.  Enumere  al  menos  cinco  factores  de  riesgo  para  el  cáncer  colorrectal.
Pólipos  adenomatosos  previos,  antecedentes  familiares  de  cáncer  colorrectal,  edad  mayor  de  40  años,  colitis  
ulcerosa  crónica  (CU),  colitis  de  Crohn,  antecedentes  de  cáncer  de  colon,  exposición  a  radiación  pélvica  por  
cáncer  de  próstata  o  de  cuello  uterino  y  poliposis  familiar.  Los  pólipos  hamartomatosos  (síndrome  de  Peutz­
Jeghers),  los  pólipos  inflamatorios  y  los  pólipos  hiperplásicos  no  se  consideran  premalignos.

5.  ¿Cuáles  son  las  recomendaciones  de  detección  actuales  de  la  Sociedad  Estadounidense  del  Cáncer  para  el  
cáncer  colorrectal?
Un  examen  rectal  digital  anual  con  prueba  de  sangre  oculta  para  pacientes  de  40  años  o  más.  Además,  para  
pacientes  mayores  de  50  años,  se  recomienda  una  sigmoidoscopia  flexible  cada  3  a  5  años.

6.  ¿En  qué  parte  del  colon  o  del  recto  se  encuentran  la  mayoría  de  los  cánceres?
Históricamente,  ha  habido  una  mayor  incidencia  de  cánceres  en  el  recto  y  el  colon  izquierdo.
Sin  embargo,  durante  los  últimos  50  años,  ha  habido  un  cambio  gradual  hacia  una  mayor  incidencia  de  
cánceres  de  colon  derecho.  Este  cambio  en  el  patrón  puede  reflejar  una  mejora  en  la  detección  temprana.

7.  Las  opciones  quirúrgicas  para  el  cáncer  colorrectal  dependen  de  la  ubicación  del  tumor.
¿Qué  operación  se  debe  realizar  a  un  paciente  con  una  lesión  a  25  cm  del  borde  anal?

Una  colectomía  sigmoidea.

8.  ¿Qué  pasa  con  una  lesión  a  9  cm  del  margen  anal?
Una  resección  anterior  baja  (LAR).

9.  ¿Qué  pasa  con  una  lesión  a  4  cm  del  borde  anal?
Una  resección  abdominoperineal  (APR).  Esto  requiere  una  colostomía  permanente.

262
Machine Translated by Google
CAPÍTULO  53  CARCINOMA  COLORRECTAL  263

10.  ¿Cuál  es  la  importancia  de  encontrar  pólipos  adenomatosos  en  el  colon  de  un  paciente?
Este  paciente  tiene  6  veces  más  probabilidades  de  desarrollar  cáncer  colorrectal  que  un  paciente  sin  pólipos.
La  evidencia  sugiere  que  la  mayoría  de  los  cánceres  de  colon  surgen  de  pólipos  adenomatosos.  La  ''secuencia  de  
carcinoma  de  adenoma''  describe  este  proceso  de  transformación.  Los  pacientes  con  poliposis  adenomatosa  familiar  
(PAF)  típicamente  albergan  más  de  100  pólipos  que  cubren  la  mucosa  colónica.  Si  estos  pacientes  no  reciben  tratamiento,  
sin  excepción,  desarrollarán  adenocarcinoma  de  colon  a  la  edad  de  40  años.

PUNTOS  CLAVE:  CARCINOMA  COLORRECTAL

1.  Los  síntomas  de  presentación  pueden  incluir  sangrado  rectal  intermitente,  dolor  abdominal  vago,  fatiga  
secundaria  a  anemia,  cambios  en  los  hábitos  intestinales,  estreñimiento,  tenesmo  y  dolor  perineal.

2.  Las  recomendaciones  actuales  de  la  Sociedad  Americana  Contra  El  Cáncer  para  las  pruebas  de  detección  son
examen  rectal  digital  con  prueba  de  sangre  oculta  a  la  edad  de  40  años  y  para  pacientes  mayores  de  50  años  una  
sigmoidoscopia  flexible  cada  3  a  5  años.

3.  Los  pacientes  con  afectación  de  los  ganglios  linfáticos  deben  recibir  quimioterapia  después  de  la  operación  para  tratar
micrometástasis.

11.  ¿Cómo  prepara  el  cirujano  el  colon  del  paciente  para  una  operación?
La  preparación  intestinal  incluye  tanto  una  limpieza  mecánica  como  una  profilaxis  antimicrobiana  apropiada.  
Esta  combinación  ha  resultado  en  una  disminución  significativa  de  la  morbilidad  y  mortalidad  de  la  cirugía  de  colon.  La  
limpieza  mecánica  se  puede  lograr  mediante  lavado  con  polietilenglicol  (Go­Lytely)  o  una  combinación  de  catárticos  y  
enemas  (Fleet's  Prep).
La  profilaxis  antimicrobiana  debe  cubrir  la  flora  aeróbica  y  anaeróbica  esperada  del  intestino.  Existe  
controversia  significativa  sobre  si  los  antibióticos  deben  administrarse  por  vía  enteral  (p.  ej.,  neomicina,  1  g  y  
metronidazol  [Flagyl],  1  g,  3  veces  por  vía  oral  a  intervalos  de  4  horas  la  noche  anterior  a  la  cirugía)  o  por  vía  parenteral  
(p.  ej.,  cefotetán,  2  g  por  vía  intravenosa  [IV]  dentro  de  1  hora  antes  de  la  cirugía).  Muchos  médicos  dan  ambos  para  
obtener  protección  tanto  intraluminal  como  sistémica.

12.  ¿Cuál  es  el  sistema  de  etapas  de  Dukes?
En  1932,  el  Dr.  Cuthbert  Dukes  describió  un  sistema  de  estadificación  para  el  cáncer  de  recto.  Originalmente  
describió  lo  siguiente:  Un  tumor  de  Dukes  confinado  a  la  pared  intestinal.

duques  B Tumor  que  invade  a  través  de  la  pared  intestinal.
C  de  los  duques Células  tumorales  encontradas  en  los  ganglios  linfáticos  regionales.

Desde  que  se  publicó  su  artículo  original,  esta  clasificación  ha  sido  modificada  varias  veces.
Una  de  las  modificaciones  más  utilizadas  es  la  inclusión  del  estadio  D  de  Dukes,  que  indica  metástasis  a  distancia.  
Si  bien  este  sistema  suele  ser  más  fácil  de  entender  para  los  pacientes,  la  investigación  y  las  publicaciones  se  basan  en  
el  sistema  de  estadificación  TNM.

13.  ¿Qué  pacientes  con  cáncer  colorrectal  requieren  adyuvante  (postoperatorio)
¿terapia?
Los  pacientes  con  afectación  de  los  ganglios  linfáticos  (C  de  Dukes)  en  estadio  III  deben  recibir  quimioterapia  
después  de  la  operación  para  tratar  las  micrometástasis.  Dos  grandes  estudios  han  documentado  una  ventaja  de  
supervivencia  para  estos  pacientes.  Sin  embargo,  ningún  estudio  ha  documentado  una  ventaja  de  supervivencia  para  
los  pacientes  con  enfermedad  de  Dukes  B  en  estadio  II  tratados  con  quimioterapia.
Machine Translated by Google
264  CAPÍTULO  53  CARCINOMA  COLORRECTAL

Los  pacientes  con  cáncer  de  recto  con  una  probabilidad  significativa  de  recurrencia  local  (B  y  C  de  
Dukes)  deben  recibir  tratamiento  con  radioterapia.  Esto  se  puede  administrar  en  el  preoperatorio,  el  
postoperatorio  o  con  una  técnica  combinada  de  "sándwich".

SITIO  WEB

www.nejm.org

BIBLIOGRAFÍA

1.  Alvarez  JA,  Baldonedo  RF,  Bear  IG  et  al.:  Cirugía  de  emergencia  para  carcinoma  colorrectal  complicado:  una  comparación  
de  pacientes  mayores  y  jóvenes.  Int  Surg  92:320­326,  2007.

2.  Grupo  Colaborativo  de  Cáncer  Colorrectal:  Radioterapia  adyuvante  para  el  cáncer  de  recto:  una  descripción  sistemática  de
22  ensayos  aleatorios  con  8507  pacientes.  Lancet  358:1291­1304,  2001.

3.  Jass  JR:  Patogénesis  del  cáncer  colorrectal.  Surg  Clin  North  Am  82:891­904,  2002.

4.  Levin  B,  Brooks  D,  Smith  RA,  Stone  A:  Tecnologías  emergentes  en  la  detección  del  cáncer  colorrectal:
Colonografía  por  tomografía  computarizada,  pruebas  inmunoquímicas  de  sangre  oculta  en  heces  y  detección  de  heces  con  marcadores  moleculares.
CA  Cáncer  J  Clin  53:44­55,  2003.

5.  Lynch  HT,  de  la  Chapelle  A:  Cáncer  colorrectal  hereditario.  N  Engl  J  Med  348:919­932,  2003.

6.  Nakamura  T,  Mitomi  H,  Ihara  A  et  al .:  Factores  de  riesgo  para  la  infección  de  la  herida  después  de  la  cirugía  por  cáncer  coloretal.  Mundo  J
Surg  32:1138­1141,  2008.  Epub  11  de  abril  de  2008.

7.  Ransohoff  DF:  Cribado  de  colonoscopia  en  cuestiones  de  equilibrio  de  aplicación.  Gastroenterol  Clin  North  Am
31:1031­1044,  2002.

8.  Salz  LB,  Minsky  B:  Terapia  adyuvante  de  cánceres  de  colon  y  recto.  Surg  Clin  North  Am  82:1035­1058,
2002.

9.  Scarpa  M,  Erroi  F,  Ruffolo  C  et  al.:  Cirugía  mínimamente  invasiva  para  el  cáncer  colorrectal:  calidad  de  vida,  imagen  corporal,  estética  y  
resultados  funcionales.  Surg  Endosc  [Epub  antes  de  imprimir]  4  de  abril  de  2008.

10.  Grupo  de  trabajo  multisociedad  de  EE.  UU.  sobre  el  cáncer  colorrectal:  Detección  y  vigilancia  del  cáncer  colorrectal:  Directrices  
clínicas  y  actualización  de  la  justificación  basada  en  nueva  evidencia.  Gastroenterología  124:544­560,  2003.
Machine Translated by Google

ENFERMEDAD  ANORRECTAL
CAPÍTULO  
54

Jeffry  L.  Kashuk,  MD,  FACS

PREGUNTAS  GENERALES

1.  ¿Qué  aspecto  del  encuentro  inicial  con  el  paciente  es  más  importante  en  el  diagnóstico  de
enfermedad  anorrectal?
Historial  clínico,  incluida  la  duración  de  las  quejas,  los  problemas  que  exacerban  o  alivian,  los  eventos  precipitantes,  
los  hábitos  alimentarios  y  intestinales,  los  tratamientos  actuales  o  anteriores  y  las  opciones  de  estilo  de  vida.

2.  ¿Cuál  es  la  causa  más  común  de  sangre  roja  brillante  e  indolora  por  el  recto?
Hemorroides  internas.

3.  ¿Cuáles  son  los  puntos  de  referencia  anatómicos  proximales  y  distales  del  canal  anal?  Qué  es
su  longitud  promedio?
El  canal  anal  comienza  en  la  unión  anorrectal,  que  es  el  borde  superior  del  músculo  del  esfínter  interno  
o  músculo  puborrectal,  y  termina  en  el  borde  anal.  La  longitud  promedio  es  de  solo  3  a  4  cm.  El  punto  medio  
del  canal  anal  se  llama  línea  dentada.

4.  ¿Cuál  es  el  significado  anatómico  y  quirúrgico  de  la  línea  dentada?
La  línea  dentada  es  la  ubicación  de  las  criptas  anales  que  drenan  las  glándulas  anales  
intramusculares  e  interesfinterianas,  que  son  el  sitio  de  abscesos  anorrectales  y  fístulas  en  el  ano.  Por  encima  
de  la  línea  dentada,  el  canal  anal  recibe  inervación  visceral  (control  involuntario),  está  cubierto  por  epitelio  
cilíndrico  y  es  el  origen  de  las  hemorroides  internas.  Debajo  de  la  línea  dentada,  el  canal  anal  recibe  inervación  
somática  (control  voluntario),  está  revestido  con  epitelio  escamoso  y  es  la  ubicación  de  las  hemorroides  
externas.

5.  ¿Cuál  es  la  causa  más  común  de  absceso  anorrectal?
El  noventa  por  ciento  resulta  de  una  infección  criptoglandular.

6.  ¿Cuáles  son  los  cuatro  espacios  anorrectales  potenciales  utilizados  para  clasificar
abscesos?
1.  Perianal  (área  del  borde  anal).
2.  Isquiorrectal  (área  lateral  a  los  músculos  del  esfínter  externo,  que  se  extiende  desde  los  músculos  
elevadores  del  ano  hasta  el  perineo).
3.  Interesfinteriana  (área  entre  los  músculos  del  esfínter  interno  y  externo,  continua
inferiormente  con  el  espacio  perianal  y  superiormente  con  la  pared  rectal).
4.  Supraelevador  (área  superior  a  los  músculos  elevadores  del  ano,  inferior  al  peritoneo  y  lateral  a  la  
pared  rectal).

7.  Definir  fístula  en  el  ano.
Una  fístula  es  una  comunicación  anormal  entre  dos  superficies  revestidas  de  epitelio.  La  abertura  interna  de  la  
fístula  en  el  ano  involucra  el  anodermo,  más  comúnmente  en  la  región  de  la  línea  dentada,  mientras  que  el  
orificio  externo  se  localiza  más  comúnmente  en  el  margen  anal.

265
Machine Translated by Google
266  CAPÍTULO  54  ENFERMEDAD  ANORRECTAL

8.  ¿Cuál  es  la  incidencia  de  fístula  en  el  ano  después  de  la  incisión  quirúrgica  apropiada  y
drenaje  de  abscesos  anorrectales  agudos?
es  el  50%.

9.  ¿Cuál  es  el  factor  más  importante  que  conduce  a  la  erradicación  quirúrgica  exitosa  de  abscesos  y  fístulas  anorrectales?

Conocimiento  de  la  anatomía  anorrectal,  incluidos  los  posibles  espacios  comprometidos  (memorizar  las  
respuestas  a  las  preguntas  anteriores),  y  establecer  un  drenaje  adecuado,  identificando  el  curso  completo  de  la  
fístula.

10.  ¿Qué  es  la  regla  de  Goodsall?
La  ubicación  de  la  abertura  interna  de  una  fístula  anorrectal  se  basa  en  la  posición  de  la  abertura  externa.  
Una  abertura  posterior  a  una  línea  trazada  transversalmente  a  través  del  perineo  se  origina  en  una  abertura  
interna  en  la  línea  media  posterior.  Una  abertura  externa  anterior  a  esta  línea  se  origina  en  la  cripta  anal  más  
cercana  en  dirección  radial.

11.  ¿Qué  es  un  setón?
Un  sedal  es  un  cuerpo  extraño  (como  una  sutura  gruesa)  que  se  coloca  a  través  del  trayecto  fistuloso  y  luego  se  
aprieta  en  serie,  lo  que  permite  una  operación  lenta  y  controlada  del  esfínter  y  la  extrusión  del  cuerpo  extraño.  La  
reacción  fibrosa  asociada  mantiene  la  integridad  del  esfínter.  Aunque  el  dolor  asociado  es  un  factor  limitante  en  su  
uso,  la  técnica  puede  evitar  eficazmente  la  lesión  del  esfínter  con  riesgos  mínimos  de  incontinencia  asociada.

FISURA  ANAL

12.  ¿Cuál  es  la  localización  más  frecuente  de  la  fisura  anal  idiopática?
El  noventa  por  ciento  son  posteriores  y  el  10%  son  anteriores.

13.  ¿Cuáles  son  los  síntomas  más  comunes  de  la  fisura  anal?
Dolor  anal  desgarrante  y  sangrado  al  defecar.

14.  ¿Cuál  es  la  fisiopatología  subyacente  de  la  fisura  anal?
Trauma  local  cíclico  del  canal  anal,  disfunción  del  esfínter  anal  interno  e  isquemia.

15.  ¿Cuál  es  el  diagnóstico  diferencial  de  la  fisura  anal,  especialmente  si  es  atípica  en
¿ubicación?
Absceso  anorrectal,  hemorroides  trombosadas,  enfermedad  inflamatoria  intestinal  (EII)  o,  en  raras  ocasiones,  
malignidad.

16.  ¿Cuál  es  la  mejor  forma  de  diagnosticar  la  fisura  anal?
Por  historia  clínica  e  inspección  visual;  no  por  examen  rectal  digital  o  anoscopia  (que  solo  sirve  para  convertir  a  
un  paciente  amistoso  en  uno  iracundo).

17.  ¿Cuáles  son  las  opciones  de  tratamiento  no  quirúrgico?
Dieta  alta  en  fibra;  agentes  de  carga  de  heces;  aumento  de  la  hidratación,  baños  de  asiento  tibios  frecuentes  y  
agentes  tópicos  que  contienen  agentes  antiinflamatorios,  anestésicos  locales  y  vasodilatadores  (nitroglicerina).
También  se  ha  informado  que  la  inyección  de  botox  (toxina  botulínica)  es  efectiva  al  relajar  los  músculos  del  
esfínter.

18.  ¿Cuál  es  la  operación  más  común  que  se  realiza  para  tratar  una  fisura  intratable?
en  el  ano?
Esfinterotomía  lateral  interna  con  o  sin  fisurotomía.
Machine Translated by Google
CAPÍTULO  54  ENFERMEDAD  ANORRECTAL  267

HEMORROIDES

19.  ¿Qué  son  los  tejidos  hemorroidales  y  cuáles  son  sus  funciones  normales?
Las  hemorroides  son  cojines  de  tejido  vascular  que  contribuyen  a  la  continencia  anal  y  protegen  el  mecanismo  del  
esfínter  durante  la  defecación.  Las  hemorroides  no  son  venas,  sino  sinusoides.  El  sangrado  se  origina  en  las  arteriolas  
presinusoidales,  lo  que  explica  el  sangrado  de  color  rojo  brillante.

20.  ¿Cuál  es  la  causa  más  común  de  hemorroides  patológicas?
Estreñimiento,  esfuerzo  prolongado,  embarazo,  disfunción  del  esfínter  interno  y  enfermedad  varicosa  asociada  
con  hipertensión  portal  hepática.

21.  ¿Cuál  es  la  diferencia  más  importante  entre  las  hemorroides  internas  y  externas?

Las  hemorroides  internas  están  por  encima  de  la  línea  dentada  con  inervación  visceral,  mientras  que  las  
hemorroides  externas  están  por  debajo  de  esta  región  con  inervación  somática.  La  ablación  de  las  hemorroides  
internas  provoca  una  sensación  de  presión  con  ganas  de  defecar,  pero  un  enfoque  similar  para  las  hemorroides  
externas  provoca  un  dolor  insoportable.

22.  ¿Cuáles  son  las  quejas  más  comunes  asociadas  con  las  hemorroides  internas  patológicas?

Sangrado,  secreción  de  moco  y  prolapso.

23.  ¿Cuáles  son  las  molestias  más  comunes  asociadas  con  las  hemorroides  externas?
Dolor,  inflamación,  trombosis  y  dificultades  de  higiene  anal.

24.  ¿Existen  opciones  de  tratamiento  para  las  hemorroides  internas  sintomáticas  basadas  en  características  físicas  
identificables?
Sí:  El  tratamiento  se  basa  en  el  grado  de  prolapso:  Grado  1:  Dieta,  
agentes  para  aumentar  el  volumen  de  las  heces.
Grado  2:  reducción  espontánea;  además  de  las  medidas  de  grado  1,  ligadura  con  banda  elástica,
escleroterapia  por  inyección,  crioterapia,  ablación  infrarroja,  dilatación  anal  y  electrocauterio.
Grado  3:  Reducción  manual  e  igual  que  las  medidas  de  grado  2  Grado  4:  
Irreductible,  igual  que  las  medidas  de  grado  3  o  hemorroidectomía  quirúrgica,  incluidos  los  procedimientos  de  
grapado  para  la  enfermedad  extensa.

ENFERMEDAD  PILNIDAL

25.  ¿Cuál  es  la  presentación  clínica  más  común  de  un  seno  pilonidal?
Dolor  e  hinchazón  en  la  región  sacrococcígea,  que  generalmente  se  asocia  con  uno  o  más  trayectos  sinusales  de  
drenaje  crónico.

26.  ¿Cómo  se  trata  el  absceso  pilonidal  agudo?
Incisión  y  drenaje  del  absceso  de  forma  ambulatoria,  con  posterior  atención  al  trayecto  fistuloso.

27.  ¿Cuál  es  el  tratamiento  definitivo  de  la  enfermedad  pilonidal?
Escisión  de  toda  la  cavidad  pilonidal  y  los  trayectos  sinusales  asociados  hasta  la  fascia.

28.  ¿Cuál  es  la  mejor  manera  de  tratar  la  herida?
Las  heridas  pequeñas  se  pueden  cerrar  principalmente,  mientras  que  los  defectos  grandes  pueden  requerir  aberturas  amplias  o  
cierres  con  colgajos.

29.  ¿Por  qué  la  enfermedad  pilonidal  es  rara  después  de  los  40  años?
Los  cambios  en  el  hábito  corporal  es  una  teoría.
Machine Translated by Google
268  CAPÍTULO  54  ENFERMEDAD  ANORRECTAL

PUNTOS  CLAVE:  ENFERMEDAD  ANORRECTAL

1.  Realizar  una  anamnesis  y  un  examen  físico  cuidadosos  con  especial  atención  a  los  hábitos  intestinales  antes
examinando  a  un  paciente  con  molestias  anorrectales.  Si  se  sospecha  una  fisura,  realice  una  inspección  visual  
limitada.

2.  Estar  particularmente  familiarizado  con  la  anatomía  anorrectal  cuando  se  trate  de  condiciones  de  este
región.

3.  Siempre  tenga  en  cuenta  la  ubicación  del  esfínter  anal  al  abordar  quirúrgicamente  la  enfermedad  anorrectal;  utilice  
un  seton  en  caso  de  duda.

4.  Sea  conservador  al  tratar  la  enfermedad  hemorroidal,  reservando  la  terapia  quirúrgica  en  la  mayoría  de  los  casos.
casos  como  último  recurso.

5.  La  enfermedad  pilonidal  debe  erradicarse  enérgicamente  hasta  la  fascia  en  la  operación  inicial
intervención.

BIBLIOGRAFÍA

1.  Beck  OE,  Wexner  SC,  editores:  Fundamentos  de  la  cirugía  anorrectal,  Filadelfia,  1999,  WB  Saunders.
2.  Brisinda  G,  Cadeddu  F,  Brandara  F  et  al.:  Ensayo  clínico  aleatorizado  que  compara  las  inyecciones  de  toxina  botulínica  con
Pomada  de  nitroglicerina  al  0,2  %  para  la  fisura  anal  crónica.  Br  J  Surg  94:162,  2007.
3.  Cintron  JR,  Park  JJ,  Orsay  CP  et  al .:  Reparación  de  fístulas  en  el  ano  con  adhesivo  de  fibrina:  beca  a  largo  plazo.
Dis  Colon  Rectum  43:944,  2000.

4.  Felt­Bersma  RJ:  Ultrasonido  endoanal  en  fístulas  y  abscesos  perianales.  Dig  Liver  Dis  38:537,  2006.
5.  Ky  AJ,  Sylla  P,  Steinhagen  R  et  al .:  Tapón  de  fístula  de  colágeno  para  el  tratamiento  de  fístulas  anales.  Este  Colon  Recto
51:838­843,  2008.

6.  Mattana  C,  Coco  C,  Manno  A  et  al .:  Hemorroidectomía  con  grapas  y  hemorroidectomía  de  Milligan  Morgan  en  la  cura  de  las  
hemorroides  de  cuarto  grado.  Dis  Colon  Rectum  50:1770,  2007.
7.  McCallum  I,  King  PM,  Bruce  J:  Curación  por  intención  primaria  versus  secundaria  después  del  tratamiento  quirúrgico  para
seno  pilonidal.  Cochrane  Database  Reviews  Número  4  Art:  D006213,  2007.

8.  Thornton  MJ,  Kennedy  ML,  King  DOV:  efecto  manométrico  del  trinitrato  de  glicerilo  tópico  y  su  impacto  en  la  cicatrización  de  
fisuras  anales  crónicas.  Dis  Colon  Rectum  48:1207,  2005.
Machine Translated by Google

HERNIA  INGUINAL CAPITULO  
55

Elizabeth  L.  Cureton,  MD,  Alexander  Q.  Ereso,  
MD;

1.  ¿A  qué  tres  hernias  se  refiere  la  hernia  de  la  "ingle"?
Hernias  inguinales  directas  e  indirectas  y  hernias  femorales.

2.  Francois  Poupart,  cirujano  y  anatomista  francés  (1616–1708),  describió  un
ligamento  que  lleva  su  nombre.  ¿Cuál  es  el  nombre  anatómico  del  ligamento  de  Poupart?
Ligamento  inguinal,  que  es  un  elemento  clave  en  la  mayoría  de  las  reparaciones  de  hernias  inguinales.

3.  Franz  K.  Hesselbach,  cirujano  y  anatomista  alemán  (1759–1816),  describió  un  triángulo  que  es  el  sitio  común  
de  las  hernias  directas.  ¿Cuáles  son  los  márgenes  anatómicos  del  triángulo  de  Hesselbach?

El  triángulo  está  definido  inferiormente  por  el  ligamento  inguinal,  superiormente  por  los  vasos  
epigástricos  inferiores  y  medialmente  por  la  fascia  del  recto.  La  fascia  transversalis  forma  el  suelo  del  
triángulo.  La  descripción  original  usaba  el  ligamento  de  Cooper  como  límite  inferior,  pero  debido  al  uso  
común  del  abordaje  anterior  de  las  hernias,  el  ligamento  inguinal  más  aparente  se  sustituyó  como  límite  
inferior  del  triángulo.  Con  el  uso  cada  vez  mayor  de  abordajes  preperitoneales  para  la  reparación  de  hernias,  
el  ligamento  de  Cooper  vuelve  a  ser  mucho  más  evidente  y  útil  como  piedra  de  toque  anatómica.

4.  Sir  Astley  Paston  Cooper,  cirujano  y  anatomista  inglés  (1768–1841),  describió  un  ligamento  que  lleva  
su  nombre.  ¿Cuál  es  el  nombre  anatómico  del  ligamento  y  el  nombre  propio  de  la  reparación  del  
ligamento  de  Cooper?
El  nombre  anatómico  del  ligamento  de  Cooper  es  ligamento  iliopectíneo.  Chester  McVay  (1911­1987)  
popularizó  la  reparación  del  ligamento  de  Cooper  o  reparación  de  McVay.  Con  Barry  Aston,  profesor  de  
anatomía  en  la  Universidad  Northwestern,  McVay  proporcionó  la  descripción  moderna  de  la  anatomía  de  la  ingle.

5.  Antonio  de  Gimbernat,  a  Spanish  surgeon  and  anatomist  (1734–1816),  had  
su  interesante  nombre  se  une  al  ligamento  lacunar,  que  marca  el  margen  medial  de  una  abertura  en  el  
área  de  la  ingle.  ¿Qué  es  la  apertura?  ¿Qué  hernia  sobresale  en  esta  abertura?

La  abertura  es  el  canal  femoral,  que  está  definido  medialmente  por  el  ligamento  lacunar,  anteriormente  por  
el  ligamento  inguinal,  posteriormente  por  la  fascia  pectínea  y  lateralmente  por  la  vena  femoral.
Una  hernia  femoral  sobresale  en  el  canal  femoral.

6.  La  hernia  inguinal  indirecta  (particularmente  en  niños)  y  el  hidrocele  son
¿A  qué  anomalía  congénita  se  asocia?
La  persistencia  de  un  proceso  vaginal  abierto,  en  el  caso  de  una  hernia,  permite  el  descenso  del  intestino  al  
canal  inguinal.  Con  la  acumulación  de  líquido,  la  obstrucción  parcial  se  presenta  como  un  hidrocele  del  
cordón  espermático.

7.  ¿Cuáles  son  los  criterios  de  diagnóstico  de  hernia  en  un  bebé  o  niño?
&  Bulto  inguinal,  escrotal  o  labial  que  puede  o  no  ser  reducible.
  Antecedentes  de  un  bulto  visto  por  un  proveedor  de  atención  médica.

269
Machine Translated by Google
270  CAPÍTULO  55  HERNIA  INGUINAL

&  Historia  de  un  bulto  visto  por  la  madre.
&  El  ''signo  de  seda'' (la  sensación  de  frotar  dos  superficies  de  tela  de  seda  cuando  se
frotando  juntas  las  dos  superficies  de  un  saco  herniario).
  A  veces  se  siente  un  encarcelamiento  en  el  examen  rectal.

8.  ¿Qué  se  puede  hacer  para  reducir  una  hernia  encarcelada  en  un  bebé  o  un  niño?

El  programa  de  cuatro  puntos  es  más  fácil  decirlo  que  hacerlo,  pero  vale  la  pena  el  esfuerzo:  1.  
Sedar  al  paciente.
2.  Coloque  al  paciente  en  posición  de  Trendelenburg.
3.  Aplique  una  compresa  fría  (sobre  una  gasa  de  petróleo  para  evitar  lesiones  en  la  piel)  en  el  área  inguinal.
4.  En  ausencia  de  reducción  espontánea,  y  si  el  paciente  está  tranquilo,  utilice  una  manipulación  suave.

9.  ¿Con  qué  frecuencia  se  puede  reducir  con  éxito  el  encarcelamiento?  Que  deberia  ser
hecho  a  continuación?

Alrededor  del  80%  de  las  hernias  encarceladas  se  pueden  reducir  en  los  niños;  en  adultos,  el  porcentaje  es  menor.
A  pesar  de  que  del  80%  al  90%  de  las  hernias  inguinales  ocurren  en  niños,  la  mayoría  de  las  encarcelaciones  
ocurren  en  niñas.  La  hernia  debe  repararse  de  forma  electiva  unos  días  después  de  la  incarceración.
El  20%  de  las  hernias  que  aún  están  encarceladas  se  operan  de  inmediato.

10.  ¿Qué  es  una  reparación  Bassini?
La  reparación  de  Bassini  sutura  el  tendón  conjunto  y  el  borde  inclinado  del  ligamento  inguinal  hasta  el  anillo  
interno  (fig.  55­1).  Este  procedimiento  clásico,  introducido  en  1887  en  la  Sociedad  Italiana  de  Cirugía  en  Génova,  revolucionó  
la  reparación  de  hernias.  Hasta  hace  poco,  ha  sido  el  estándar  de  reparación.  Después  de  graduarse  de  la  escuela  de  
medicina  y  mientras  luchaba  por  la  independencia  de  Italia,  Eduardo  Bassini  (1844­1924)  recibió  un  bayonetazo  en  la  ingle  
y,  como  prisionero,  fue  hospitalizado  durante  meses  con  una  fístula  fecal.

Figura  55­1.  La  reparación  estándar  de  hernia  inguinal  derecha  usando  el  tendón  conjunto  y  
el  ligamento  inguinal.
Machine Translated by Google
CAPÍTULO  55  HERNIA  INGUINAL  271

11.  ¿Cuál  es  la  tasa  de  recurrencia  con  hernias  indirectas  y  directas  que  han  sido  reparadas  con  la  técnica  de  

reparación  clásica  de  Bassini?
Durante  un  período  de  seguimiento  de  50  años,  la  tasa  de  recurrencia  de  las  hernias  indirectas  en  adultos  es  del  5%  
al  10%;  de  hernias  directas,  15%  a  30%.

12.  Describa  una  reparación  de  hernia  de  McVay.
La  línea  de  suturas  interrumpidas  comienza  en  el  tubérculo  púbico  y  une  el  arco  tendinoso  del  músculo  
transverso  del  abdomen  con  el  ligamento  de  Cooper  hasta  el  canal  femoral.  En  este  punto,  se  colocan  dos  o  tres  
suturas  de  transición  desde  el  ligamento  de  Cooper  hasta  la  fascia  femoral  anterior,  cerrando  efectivamente  el  extremo  
medial  del  canal  femoral.  El  juego  final  de  suturas  une  el  arco  del  transverso  del  abdomen  y  la  fascia  femoral  anterior.  
Los  puntos  generalmente  incorporan  el  ligamento  inguinal  en  el  límite  superior  de  la  reparación,  el  sitio  del  nuevo  anillo  
inguinal  interno  y  las  estructuras  del  cordón.  Hace  unos  15  años,  McVay  describió  acostarse  en  un  parche  de  malla  y  
coserlo,  en  su  periferia,  a  las  mismas  estructuras  anatómicas.  Esta  aplicación  de  malla  se  parece  mucho  a  la  reparación  
de  Lichtenstein  (ver  pregunta  17),  excepto  que  usa  el  ligamento  de  Cooper.

13.  ¿Para  qué  tipo  de  hernias  es  más  útil  la  reparación  del  ligamento  de  McVay  Cooper?
Hernias  femorales  y  directas.

14.  ¿Qué  es  la  reparación  de  Shouldice?
La  reparación  de  Shouldice,  popularizada  en  la  Clínica  Shouldice  cerca  de  Toronto,  imbrica  o  superpone  la  fascia  
transversalis  y  el  tendón  unido  con  cuatro  líneas  continuas,  utilizando  dos  suturas  de  alambre  fino.  El  tracto  de  sutura  
se  extiende  desde  el  tubérculo  púbico  hasta  un  nuevo  anillo  interno.  Se  tiene  cuidado  con  los  vasos  epigástricos  
inferiores.  El  resultado  es  una  aproximación  en  capas  del  tendón  conjunto  al  trayecto  del  ligamento  inguinal.

15.  ¿Cuál  es  la  tasa  de  recurrencia  informada  para  la  reparación  de  Shouldice?
La  tasa  de  recurrencia  es  del  1%,  la  tasa  más  baja  informada  para  reparaciones  sin  malla  de  hernias  inguinales  en  
adultos.

16.  ¿Para  qué  tipo  de  hernia  inguinal  no  es  adecuada  la  reparación  de  Shouldice?
Hernia  femoral.

17.  Describa  la  reparación  de  Lichtenstein.
La  reparación  de  Lichtenstein  consiste  en  un  parche  suturado  de  malla  de  polipropileno  (Marlex,  CR
Bard,  Inc.,  Covington,  GA)  que  cubre  el  triángulo  de  Hesselbach  y  el  área  de  la  hernia  indirecta.
Se  considera  una  reparación  sin  tensión  porque  la  malla  se  sutura  en  su  lugar  sin  tirar  de  ligamentos  o  tejidos  como  
en  todas  las  demás  reparaciones.  La  malla  se  divide  en  su  extremo  superior  para  envolver  estrechamente  el  cordón  
espermático  y  sus  estructuras  asociadas  en  la  posición  normal  del  canal  inguinal  interno.  El  procedimiento  de  
Lichtenstein  se  está  convirtiendo  rápidamente  en  la  reparación  más  utilizada  de  la  hernia  inguinal  en  adultos.  La  
tasa  de  recurrencia  informada  es  <1%.

18.  ¿Cuáles  son  las  ventajas  de  utilizar  la  malla  Marlex?
Para  la  aceptación  y  el  éxito  de  la  reparación  de  la  hernia  de  Lichtenstein  ha  sido  fundamental  el  
desarrollo  y  la  experiencia  con  la  malla  de  Marlex.  La  malla  de  monofilamento  es  fuerte,  inerte  y  resistente  a  la  
infección.  Los  intersticios  se  infiltran  rápida  y  completamente  con  fibroblastos  y  la  malla  no  sufre  deterioro,  rechazo  
ni  fragmentación  (v .  fig.  55­2).
Machine Translated by Google
272  CAPÍTULO  55  HERNIA  INGUINAL

Figura  55­2.  Reparación  con  malla  de  Marlex  de  una  hernia  inguinal  derecha.  Tenga  en  cuenta  
que  se  utilizan  las  mismas  estructuras  pero  no  se  unen;  de  ahí  el  nombre  de  la  reparación  "sin  tensión".

19.  ¿Para  qué  área  de  la  ingle  no  es  apropiada  la  reparación  de  Lichtenstein?
Hernia  femoral.

20.  ¿Qué  tipo  de  reparación  es  aceptable  para  la  hernia  femoral?
Se  pueden  utilizar  varias  reparaciones  diferentes.  La  malla  en  forma  de  tapón  puede  insertarse  en  el  
canal  femoral  y  fijarse  en  su  lugar.  Se  puede  realizar  una  reparación  del  ligamento  de  McVay  Cooper.  Se  
puede  utilizar  un  abordaje  preperitoneal  de  la  hernia  para  suturar  o  tapar  el  defecto.  También  se  puede  usar  
una  reparación  con  sutura  o  un  colgajo  facial  de  sartorio  aplicado  desde  debajo  del  ligamento  inguinal  en  un  
abordaje  femoral.  El  abordaje  preperitoneal  se  usa  cada  vez  más  para  las  hernias  inguinales  y  femorales  
complicadas.

21.  ¿Qué  es  el  procedimiento  preperitoneal  o  Stoppa?
El  procedimiento  preperitoneal  o  Stoppa  es  una  reparación  de  hernia  inguinal  en  el  lado  interno  de  
la  pared  abdominal  entre  el  peritoneo  y  las  superficies  fasciales  que  no  se  abren  hacia  la  cavidad  
peritoneal.  Los  puntos  de  referencia  anatómicos  son  bastante  diferentes  e  inicialmente  bastante  desafiantes  
para  los  cirujanos  acostumbrados  al  abordaje  de  la  pared  abdominal  externa.  La  técnica  es  adecuada  para  
las  hernias  recurrentes  en  las  que  la  cicatrización  y  la  anatomía  obliterada  aumentan  el  riesgo  de  lesión  
medular  y  recurrencia.  Otros  problemas  como  hernias  grandes  y  hernias  femorales  se  corrigen  con  este  
enfoque.  Conceptualmente,  la  reparación  laparoscópica  de  hernia  utiliza  el  mismo  método  (v .  fig.  55­3).
Machine Translated by Google
CAPÍTULO  55  HERNIA  INGUINAL  273

Figura  55­3.  Las  diferentes  apariencias  y  puntos  de  referencia  se  ven  en  la  vista  anterior  (arriba)  y  la  vista  posterior  
(abajo)  del  área  inguinal­femoral.  En  la  vista  posterior  es  evidente  la  importancia  de  los  vasos  epigástricos  inferiores,  
la  vejiga  y  el  ligamento  de  Cooper  como  puntos  de  referencia  anatómicos.
Machine Translated by Google
274  CAPÍTULO  55  HERNIA  INGUINAL

22.  ¿Dónde  están  los  espacios  de  Retzius  y  Bogros?  ¿Por  qué  son  cada  vez  más
¿importante?
El  espacio  de  Retzius  se  encuentra  entre  el  pubis  y  la  vejiga  urinaria.  El  espacio  de  Bogros  se  encuentra  
entre  el  peritoneo  y  la  fascia  y  los  planos  musculares  en  la  cara  posterior  de  la  pared  abdominal  por  debajo  
del  ombligo  y  hasta  el  ligamento  de  Cooper.  Lateralmente,  el  espacio  va  a  las  espinas  ilíacas.
Tanto  en  el  procedimiento  abierto  de  Stoppa  como  en  la  reparación  preperitoneal  laparoscópica,  los  espacios  
de  Retzius  y  Bogros  se  desarrollan  para  la  colocación  de  la  malla  y  la  exposición  quirúrgica.

23.  ¿Qué  tan  ajustado  alrededor  del  cordón  espermático  debe  estar  un  anillo  inguinal  interno  hecho  quirúrgicamente?

Unos  5  mm,  que  es  menos  que  la  yema  de  un  dedo  y  más  que  la  punta  de  una  pinza.

24.  ¿Cuál  es  el  defecto  fascial  común  de  los  inguinales  indirectos  más  grandes  y  todos  los  directos?
hernias?
Debilidad  o  atenuación  de  la  fascia  transversalis.

25.  En  el  examen,  la  hernia  femoral  puede  confundirse  con  otras
¿hernia  inguinal?
La  hernia  femoral  puede  confundirse  con  una  hernia  inguinal  directa  debido  a  la  tendencia  de  la  hernia  femoral  a  
presentarse  en  el  borde  lateral  del  ligamento  inguinal.

PUNTOS  CLAVE:  TIPOS  DE  REPARACIÓN  DE  HERNIA  INGUINAL

1.  La  reparación  de  Bassini  sutura  el  tendón  conjunto  y  el  borde  inclinado  del  ligamento  inguinal  hasta  el  
anillo  interno.

2.  La  reparación  de  McVay  es  más  útil  para  las  hernias  femorales  y  directas.

3.  La  reparación  de  Shouldice  imbrica  la  fascia  transversalis  y  el  tendón  conjunto  con  cuatro  líneas  continuas,  
usando  dos  suturas  de  alambre  fino  (no  apropiado  para  hernias  femorales).

4.  La  reparación  de  Lichtenstein  consiste  en  un  parche  suturado  de  malla  de  polipropileno  que  cubre
El  triángulo  de  Hesselbach  y  el  saco  herniario  indirecto.

26.  ¿Cuál  es  la  diferencia  entre  una  hernia  incarcerada  y  una  estrangulada?
Encarcelado:  las  estructuras  en  el  saco  de  la  hernia  aún  tienen  un  buen  suministro  de  sangre  pero  están  atrapadas  en  el
saco  debido  a  adherencias  o  a  un  cuello  estrecho  del  saco  herniario.
Estrangulado:  las  estructuras  herniadas,  como  el  intestino  o  el  epiplón,  han  perdido  su  sangre
suministro  debido  a  la  constricción  anatómica  en  el  cuello  de  la  hernia.  El  tejido  isquémico  herniado  se  
encuentra,  por  lo  tanto,  en  diversas  etapas  de  cambios  gangrenosos.  Las  hernias  estranguladas  son  
emergencias  quirúrgicas.

27.  ¿Qué  operación  se  realiza  para  una  hernia  infantil  indirecta  no  complicada?
Ligadura  alta  del  saco  herniario.

28.  ¿Qué  operación  se  realiza  para  una  hernia  indirecta  no  complicada  en  adultos  jóvenes?
La  operación  adecuada  consiste  en  una  ligadura  alta  y  posiblemente  uno  o  dos  puntos  en  la  fascia  
transversalis  para  apretar  el  anillo  interno.  Esta  es  la  técnica  Marcy  básica,  desarrollada  por  Henry  Orlando  
Marcy  (1837–1924);  es  más  pequeña  y  más  enfocada  anatómicamente  que  la  reparación  de  Bassini.
Machine Translated by Google
CAPÍTULO  55  HERNIA  INGUINAL  275

29.  ¿Qué  operación  se  realiza  para  una  hernia  directa  no  complicada  pero  considerable  en  ancianos?
adultos?

Tradicionalmente  se  optaba  por  la  reparación  de  Bassini  o  McVay.  Más  recientemente,  debido  a  la  baja  tasa  de  
recurrencia,  se  prefiere  la  reparación  de  Shouldice  o  Lichtenstein.

30.  ¿Qué  sistemas  de  órganos  deben  revisarse  con  especial  cuidado  en  el
evaluación  de  pacientes  con  hernia  (especialmente  pacientes  de  edad  avanzada  con  aparición  reciente  de  hernia)?

Los  sistemas  gastrointestinal  (GI),  urinario  y  pulmonar  deben  revisarse  con  especial  cuidado.  Uno  está  
buscando  causas  de  tensión  crónica  o  fuerzas  repentinas  que  pueden  haber  inducido  la  hernia.  El  esfuerzo  durante  la  
defecación  o  la  micción,  la  tos  inusual  o  la  dificultad  para  respirar,  si  se  corrigen,  pueden  ser  de  gran  valor  para  el  
paciente  y  reducir  la  posibilidad  de  hernia  recurrente.

31.  ¿Qué  es  una  hernia  deslizante?
Una  hernia  por  deslizamiento  se  forma  cuando  un  órgano  retroperitoneal  sobresale  (se  hernia)  fuera  de  la  cavidad  
abdominal  de  tal  manera  que  el  propio  órgano  y  la  superficie  peritoneal  suprayacente  constituyen  un  lado  del  saco  
herniario.

32.  ¿Qué  órganos  se  pueden  encontrar  en  las  hernias  por  deslizamiento?
Colon  
Vejiga  
Ciego  

Trompas  de  Falopio  
Apéndice  Útero  
(raro)
Ovario

33.  ¿Cuáles  son  las  complicaciones  operatorias  y  posoperatorias  comunes  de  las  reparaciones  de  hernias?
Complicaciones  intraoperatorias:  &  
Lesión  del  cordón  espermático,  especialmente  en  niños.
  lesión  de  los  vasos  espermáticos,  que  produce  atrofia  o  necrosis  aguda  de  los  testículos.
  Lesión  del  nervio  ilioinguinal,  del  nervio  genitofemoral  y  del  nervio  cutáneo  femoral  lateral  (el  nervio  cutáneo  femoral  
lateral  es  especialmente  vulnerable  en  los  procedimientos  laparoscópicos  y  ownitoneales).

  Lesión  de  los  vasos  femorales.
Complicaciones  postoperatorias:
  Infección:  alto  riesgo  en  niños  con  dermatitis  del  pañal  y  pacientes  con  lesión  intestinal  o  necrosis.
  Hematoma:  debe  resolverse  con  el  tiempo.

  Lesión  nerviosa:  el  nervio  no  siempre  se  divide  y,  con  el  tiempo,  puede  mejorar.  Si  el  dolor  persiste,  pruebe  el  bloqueo  
con  lidocaína  tanto  para  el  diagnóstico  como  para  el  tratamiento.  Si  un  bloqueo  nervioso  no  tiene  éxito,  se  puede  
considerar  la  reexploración  para  liberar  el  nervio  de  la  cicatriz  o  extirpar  un  neuroma  posquirúrgico.

34.  ¿Cuáles  son  los  sitios  comunes  de  recurrencia  de  la  hernia?

Las  hernias  directas  recurren  a  menudo  en  el  tubérculo  púbico.  Las  hernias  indirectas  reaparecen  en  el  anillo  interno.
La  causa  suele  estar  relacionada  con  puntos  mal  colocados  o  insuficientes.  Otras  posibles  causas  incluyen  infección,  
tejido  deficiente,  mala  formación  de  colágeno  o  demasiada  tensión  en  la  línea  de  sutura  quirúrgica.  Una  sola  línea  de  
reparación  bajo  tensión  moderada  falla  en  un  número  significativo  de  pacientes,  independientemente  de  la  idoneidad  
de  la  reparación  o  del  proceso  de  curación.  La  tensión  es  casi  siempre  mala  en  la  cirugía.
Machine Translated by Google
276  CAPÍTULO  55  HERNIA  INGUINAL

35.  ¿Cuánto  tiempo  debe  evitar  el  paciente  levantar  objetos  pesados  después  de  una  reparación  de  hernia?
El  consejo  estándar  durante  décadas  ha  sido  de  6  semanas.  El  consejo  actual  varía  desde  ninguna  
limitación  con  las  reparaciones  de  Lichtenstein  o  preperitoneal  hasta  6  semanas  para  una  reparación  de  Bassini.
La  autolimitación  del  dolor  es  una  excelente  guía.

CONTROVERSIAS

36.  ¿Cuáles  son  algunos  de  los  problemas  anatómicos  relacionados  con  las  hernias  inguinales?
En  cuestión  está  el  tracto  iliopúbico,  que  es  central  en  la  descripción  anatómica  de  Anson/McVay  del  área  inguinal  
y  aparece  en  la  reparación  del  ligamento  de  McVay  Cooper.  Aunque  la  reparación  de  McVay  se  usa  en  Inglaterra,  
el  tracto  iliopúbico  no  se  menciona  ni  se  describe  en  los  textos  anatómicos  ingleses.

El  término  tendón  conjunto,  aunque  se  usa  comúnmente,  es  considerado  por  muchos  como  anatómicamente
inexacta  y  engañosa.  Los  músculos  oblicuo  interno  y  transverso  del  abdomen  que  forman  el  tendón  conjunto  son  
obvios  y  pueden  usarse  quirúrgicamente  solos  o  juntos.  El  borde  tendinoso  del  músculo  transverso  del  abdomen  
y  el  borde  tendinoso  del  músculo  oblicuo  interno  comienzan  en  su  inserción  en  el  tubérculo  púbico  y  discurren  lateral  
y  superiormente  hacia  el  borde  medial  del  anillo  interno.  En  este  punto,  los  elementos  tendinosos  disminuyen,  
dejando  solo  tejidos  musculares,  y  continúan  lateral  y  superiormente  a  sus  orígenes.

Si  el  ligamento  lacunar  o  el  tracto  iliopúbico  definen  el  borde  medial  del  fémur.
canal  es  controvertido.  La  posición  de  compromiso  es  que  el  tracto  iliopúbico  es  el  borde,  mientras  que  en  
el  estado  normal  sin  estiramiento,  el  ligamento  lacunar  (ligamento  de  Gimbernat)  es  el  borde  en  presencia  de  
hernia  (estado  estirado).  En  la  cirugía  es  suficiente  decir  que  un  ligamento  curvo  palpable  y  visible  está  presente  y  
se  usa  en  algunas  reparaciones  femorales.

37.  ¿Todas  las  hernias  requieren  reparación  urgente?
Tal  vez  no.  La  incarceración  aguda  o  la  estrangulación  del  contenido  abdominal  en  una  hernia  es  una  urgencia  
quirúrgica.  Los  pacientes  con  molestias  crónicas  se  benefician  de  la  reparación  electiva.  Sin  embargo,  datos  recientes  
sobre  hombres  con  hernias  asintomáticas  o  mínimamente  sintomáticas  indican  que  puede  ser  seguro  esperar  para  
operar  hasta  que  los  síntomas  empeoren.  Al  igual  que  con  cualquier  procedimiento  quirúrgico,  se  deben  considerar  
los  factores  de  riesgo  y  la  función  inicial  de  un  individuo  junto  con  el  riesgo  de  un  enfoque  quirúrgico  o  no  quirúrgico.  
En  el  caso  de  las  hernias,  un  pequeño  porcentaje  de  pacientes  asintomáticos  en  los  que  se  difiere  la  reparación  
electiva  desarrollarán  encarcelamiento  herniario  agudo  en  algún  momento  en  el  futuro.

38.  ¿Cuáles  son  algunos  problemas  quirúrgicos  en  la  reparación  de  hernias  inguinales?
Existe  controversia  sobre  el  uso  del  laparoscopio  para  la  reparación  de  hernias.  Los  datos  sugieren  que  la  reparación  laparoscópica  
da  como  resultado  menos  dolor  y  un  retorno  más  rápido  a  las  actividades  normales,  aunque  los  tiempos  quirúrgicos  son  más  
prolongados  y  el  riesgo  de  complicaciones  viscerales  y  vasculares  es  mayor.  La  reparación  laparoscópica  requiere  un  período  de  
aprendizaje  mucho  más  prolongado  para  lograr  tasas  de  recurrencia  comparables  a  las  de  la  reparación  abierta  sin  tensión  con  malla.

BIBLIOGRAFÍA

1.  Avisse  C,  Delattre  JF,  Flament  JB:  Los  anillos  inguinales.  Surg  Clin  North  Am  80:49­69,  2000.

2.  Avisse  C,  Delattre  JF,  Flament  JB:  El  área  inguinofemoral  desde  un  punto  de  vista  laparoscópico.  historia,  anatomia,
y  aplicaciones  quirúrgicas.  Surg  Clin  North  Am  80:35­48,  2000.

3.  Bendavid  R,  Howarth  D:  fascia  transversalis  redescubierta.  Surg  Clin  North  Am  80:25­33,  2000.

4.  Colaboración  de  investigadores  de  hernia  de  la  UE:  reparación  de  hernia  inguinal  con  malla  sintética:  metanálisis  de  ensayos  
controlados  aleatorios.  Ann  Surg  235:322­332,  2002.

5.  Fitzgibbons  RJ  Jr,  Giobbie­Hurder  A,  Gibbs  JO  et  al .:  espera  vigilante  frente  a  reparación  de  hernia  inguinal  en  hombres  
mínimamente  sintomáticos:  un  ensayo  clínico  aleatorizado.  JAMA  295:285­292,  2006.

6.  Neumayer  L,  Giobbie­Hurder  A,  Jonasson  O  et  al .:  Reparación  de  hernia  inguinal  con  malla  abierta  versus  malla  laparoscópica.
N  Engl  J  Med  350:1819,  2004.
Machine Translated by Google

CIRUGÍA  BARIÁTRICA
CAPITULO  
56

Dr.  Jeffrey  L.  Johnson

1.  Mi  paciente  pesa  250  libras  (114  kg).  ¿Es  él  o  ella  con  obesidad  mórbida?
Tal  vez.  La  definición  más  utilizada  de  obesidad  mórbida  utiliza  el  concepto  de  índice  de  masa  corporal  (IMC),  que  
es  el  peso  (kg)  dividido  por  la  altura  al  cuadrado  (m).  Esta  es  simplemente  una  descripción  de  qué  tan  pesado  es  un  
paciente  para  su  estatura.  Un  IMC  de  40  se  considera  obesidad  mórbida.
Un  paciente  que  pesa  250  libras  y  mide  5'6''  de  altura  tiene  obesidad  mórbida  (IMC  =  40),  pero  un  paciente  que  pesa  
250  libras  y  mide  6'6''  de  altura  simplemente  tiene  sobrepeso  (IMC  =  29).

2.  ¿La  obesidad  mórbida  por  sí  sola  es  realmente  tan  mórbida?
Sí.  Incluso  sin  comorbilidades  manifiestas  (como  diabetes  e  hipertensión),  las  personas  con  obesidad  mórbida  tienen  
un  riesgo  considerable.  Muchos  sistemas  de  órganos  críticos  se  ven  afectados.  Por  ejemplo,  en  el  sistema  
cardiopulmonar,  la  apnea  obstructiva  del  sueño,  la  hipoventilación  crónica  y  la  hipertensión  pulmonar  son  hallazgos  
comunes.  Esto  se  traduce  en  una  mayor  probabilidad  de  malos  resultados  después  del  tratamiento  médico  o  quirúrgico  
de  una  amplia  variedad  de  condiciones.  Los  pacientes  con  obesidad  mórbida  tienen  una  esperanza  de  vida  
considerablemente  más  corta.  En  muchos  sentidos,  es  una  condición  potencialmente  letal.

3.  ¿Qué  es  el  ''síndrome  metabólico''?
El  síndrome  metabólico  describe  un  conjunto  de  cambios  en  la  fisiología  que  se  asocian  con  un  alto  riesgo  
cardiovascular.  La  obesidad  es  una  característica  central,  junto  con  la  resistencia  a  la  insulina,  los  triglicéridos  
elevados,  el  colesterol  de  lipoproteínas  de  baja  densidad  (LDL)  elevado  y  la  hipertensión.

4.  Mi  paciente  tiene  un  IMC  de  40.  Debido  a  que  parece  estar  tan  bien  alimentado,  ¿es  seguro  asumir  que  su  estado  
nutricional  y  cicatrización  de  heridas  son  normales?
No.  Aunque  su  ingesta  calórica  total  es  alta,  no  es  raro  que  los  pacientes  con  obesidad  mórbida  tengan  una  
ingesta  pobre  de  proteínas,  reservas  deficientes  de  proteínas  y  deficiencias  vitamínicas.
Además,  la  diabetes  concomitante  puede  contribuir  al  deterioro  de  la  cicatrización  de  heridas.

5.  Entonces,  si  los  pacientes  con  obesidad  mórbida  están  enfermos  y  no  se  curan  bien,  ¿por  qué  un  cirujano  racional  elegiría  
realizar  operaciones  de  pérdida  de  peso?
Porque  funciona  muy  bien.  Existen  pocos  enfoques  conductuales,  farmacológicos  o  combinados  para  el  tratamiento  
de  la  obesidad  mórbida  que  hayan  demostrado  promover  la  pérdida  de  peso  incluso  a  corto  plazo.
Más  pastillas,  programas  y  prensa  no  han  resultado  en  una  población  más  delgada.  Además,  estos  enfoques  no  
quirúrgicos  ni  siquiera  se  aproximan  a  la  cantidad  o  duración  de  la  pérdida  de  peso  observada  en  pacientes  que  
se  someten  a  cirugía  bariátrica.  La  pérdida  de  peso  después  de  la  cirugía  bariátrica  es  sustancial  y  parece  
mantenerse  durante  al  menos  15  años.

6.  ¿Los  pacientes  que  se  someten  a  una  cirugía  bariátrica  en  realidad  se  vuelven  más  saludables  a  medida  que  crecen?
¿más  delgada?

Sí.  La  mayoría  de  los  pacientes  con  diabetes,  hipertensión,  incontinencia  urinaria  y  apnea  obstructiva  del  sueño  se  
curan  esencialmente  de  estas  enfermedades  a  medida  que  pierden  peso.  Pregúntale  a  un  internista  cuándo  fue  la  última  
vez  que  curó  (no  palió…)  alguna  de  estas  condiciones.

277
Machine Translated by Google
278  CAPÍTULO  56  CIRUGÍA  BARIÁTRICA

7.  Si  los  pacientes  con  obesidad  mórbida  tienen  una  menor  esperanza  de  vida,  ¿los  pacientes  que  se  
someten  a  una  cirugía  bariátrica  en  realidad  viven  más  tiempo?
Parece  que  sí.  Dos  grandes  estudios  han  demostrado  una  mejor  supervivencia  en  pacientes  sometidos  a  cirugía  para  
promover  la  pérdida  de  peso.

8.  Algunas  operaciones  bariátricas  (como  el  bypass  yeyunoileal)  se  abandonaron  debido  a  complicaciones  metabólicas.  
¿Hay  algunas  operaciones  que  realmente  funcionan  y  se  consideran  seguras?

Sí.  El  bypass  gástrico  en  Y  de  Roux  (RYGB)  tiene  los  mejores  datos  de  seguridad  y  eficacia  a  largo  plazo.
Otras  opciones  incluyen  la  gastroplastia  con  banda  vertical,  la  gastrectomía  en  manga,  la  banda  gástrica  ajustable  
laparoscópica  (banda  gástrica)  y  el  cruce  duodenal.

9.  Un  bypass  gástrico  en  Y  de  Roux  suena  complicado.  ¿Qué  implica?
No  es  complicado.  El  estómago  proximal  se  divide  por  completo  para  producir  una  bolsa  proximal  de  unos  50  ml  
de  tamaño.  El  resto  del  estómago  simplemente  se  deja  en  su  lugar.  El  intestino  delgado  proximal  (la  rama  roux  o  rama  
alimentaria)  se  divide  y  se  une  a  la  bolsa.
Luego,  el  intestino  delgado  se  vuelve  a  conectar  corriente  abajo.

10.  ¿Por  qué  los  pacientes  pierden  peso  después  de  un  bypass  gástrico?
Hay  tres  razones  básicas.  Primero,  los  pacientes  no  pueden  comer  mucho  al  mismo  tiempo.  Detener.  Reflexiona  por  
un  segundo. . .  50  ml  son  10  cucharaditas.  Esta  es  la  cantidad  que  los  pacientes  pueden  comer  (o  beber)  inicialmente  a  
la  vez.  En  realidad,  se  convierte  en  trabajo  obtener  suficientes  proteínas,  calorías  y  líquidos.  En  segundo  lugar,  los  
pacientes  no  pueden  (al  principio)  tolerar  los  dulces  concentrados.  La  rama  alimentaria  está  hecha  de  intestino  delgado  
que  reaccionará  a  altas  cargas  osmolares  con  síndrome  de  dumping. . .  una  combinación  desagradable  de  dolor  
abdominal,  náuseas,  sudoración  y  diarrea.  Por  lo  tanto,  existe  un  desincentivo  significativo  para  "hacer  trampa"  después  
del  bypass  gástrico.  En  tercer  lugar,  debido  a  que  la  comida  en  la  rama  alimentaria  no  se  mezcla  con  la  bilis  y  el  jugo  
pancreático  hasta  que  se  encuentra  con  la  otra  rama  (acertadamente  llamada  biliopancreática),  75  cm  o  más  aguas  
abajo,  no  se  absorbe  de  manera  eficiente.

11.  ¿Cuánto  suelen  perder  los  pacientes  después  de  un  bypass  gástrico?
Inicialmente,  alrededor  del  70%  de  su  exceso  de  peso.  Esto  ocurre  durante  los  primeros  12  a  24  meses.
Los  pacientes  (y  los  médicos)  deben  comprender  que  es  bastante  inusual  llegar  al  peso  ideal.  Estas  operaciones  no  
están  destinadas  a  producir  modelos  de  aptitud,  sino  pacientes  más  saludables  con  una  mejor  calidad  de  vida  y  una  
mayor  longevidad.

12.  ¿Quiénes  son  los  mejores  candidatos  para  la  cirugía  bariátrica?
La  mayoría  de  los  cirujanos  bariátricos  utilizan  las  pautas  de  los  Centros  para  el  Control  y  la  Prevención  de  
Enfermedades  (CDC),  que  incluyen  el  IMC  (>40  o  >35  con  comorbilidades  relacionadas  con  el  peso)  y  la  capacidad  
de  comprender  y  cumplir  con  la  rutina  perioperatoria.  Esto  último  es  sumamente  importante  porque  el  paciente  debe  
volver  a  aprender  a  comer  con  su  nueva  anatomía.  Esta  operación  no  está  exenta  de  riesgos  y  tiene  importantes  
consecuencias  sanitarias  y  sociales:  imagínese  salir  a  cenar  cuando  puede  comer  solo  10  cucharaditas.

13.  ¿Cuáles  son  las  complicaciones  más  graves  del  bypass  gástrico?
La  fuga  de  la  anastomosis  gastroyeyunal  es  la  complicación  más  temida,  aunque  en  la  mayoría  de  las  series  le  
sigue  el  embolismo  pulmonar  como  causa  de  muerte.  La  tasa  de  mortalidad  es  <1%,  pero  no  0%.  Las  complicaciones  de  
la  herida  (hernia,  infección)  se  observan  en  alrededor  del  10  %  de  los  pacientes  que  se  someten  a  cirugía  abierta  y  solo  
alrededor  del  1  %  en  los  pacientes  que  se  someten  a  cirugía  laparoscópica.

14.  ¿Cuál  es  el  signo  más  fiable  de  fuga  gastroyeyunal?
Taquicardia.  Una  frecuencia  cardíaca  >110  debe  generar  preocupación  por  una  fuga.  Algunos  cirujanos  solicitan  estudios  de  
contraste  de  rutina  en  todos  los  pacientes.
Machine Translated by Google
CAPÍTULO  56  CIRUGÍA  BARIÁTRICA  279

PUNTOS  CLAVE

1.  La  obesidad  mórbida  es  una  afección  médica  grave  que  acorta  la  vida.

2.  La  pérdida  de  peso  quirúrgica  promueve  una  mejor  salud  y  probablemente  alarga  la  vida.

3.  Aunque  hay  varias  opciones  quirúrgicas  posibles,  el  bypass  gástrico  es  la  más  probada.

4.  La  cirugía  bariátrica  requiere  un  paciente  informado  y  cumplidor  que  comprenda  que  hay
riesgos  significativos.

BIBLIOGRAFÍA

1.  Adams  TD,  Gress  RE,  Smith  SC  et  al .:  Mortalidad  a  largo  plazo  después  de  la  cirugía  de  bypass  gástrico.  N  
Engl  J  Med  357:753,  2007.

2.  DeMaria  EJ,  Sugerman  HJ,  Kellum  JM  et  al.:  Results  of  281  consecutive  total  laparoscopic  Roux­en­Y  gastric  
bypass  para  tratar  la  obesidad  mórbida.  Ann  Surg  235:640,  2002.

3.  Hutter  MM,  Randall  S,  Khuri  SF  et  al.:  Bypass  gástrico  laparoscópico  versus  abierto  para  la  obesidad  mórbida:
un  análisis  multicéntrico,  prospectivo  y  ajustado  al  riesgo  del  Programa  Nacional  de  Mejoramiento  de  la  Calidad  Quirúrgica.
Ann  Surg  243:657,  2006.

4.  Madan  AK,  Orth  W,  Ternovits  CA  et  al.:  Síndrome  metabólico:  otra  comorbilidad  ayuda  a  la  derivación  gástrica
curar.  Surg  Obes  Relat  Dis  2:48,  2006.
Machine Translated by Google

IV.  CIRUGÍA  ENDOCRINA

HIPERPARATIROIDISMO CAPITULO  
57

Christopher  D.  Raeburn,  MD  y  Robert  C.  McIntyre,  Jr.,  MD

1.  ¿Cuál  es  la  prevalencia  del  hiperparatiroidismo  (HPT)?
Hay  aproximadamente  100.000  nuevos  casos  de  HPT  anualmente  en  los  Estados  Unidos.  Las  mujeres  superan  en  
número  a  los  hombres  en  una  proporción  de  2  a  1  y  el  riesgo  aumenta  con  la  edad.  El  HPT  primario  ocurre  en  1  de  cada  
500  mujeres  y  en  1  de  cada  2000  hombres  mayores  de  40  años.  Aproximadamente  el  10%  de  los  pacientes  con  HPT  
primario  son  derivados  para  cirugía.

2.  ¿Cuáles  son  los  síntomas  del  hiperparatiroidismo?
''Huesos  dolorosos,  cálculos  renales,  gemidos  abdominales,  gemidos  psíquicos  y  sobretonos  de  fatiga''.  Los  
síntomas  y  signos  clásicos  son:  Huesos:  artralgia,  osteoporosis  y  fracturas  patológicas.

Piedras:  cálculos  renales,  insuficiencia  renal,  poliuria  y  polidipsia.
Gemidos  abdominales:  pancreatitis,  úlcera  péptica  y  estreñimiento.
Lamentos  psíquicos:  fatiga,  debilidad  y  depresión.

3.  ¿Cuál  es  la  causa  más  frecuente  de  hipercalcemia  en  un  paciente  ambulatorio  frente  a
a  un  paciente  hospitalizado?

El  HPT  es  la  causa  más  común  de  hipercalcemia  entre  los  pacientes  ambulatorios  y  la  segunda  causa  más  
común  en  el  ámbito  hospitalario.  La  causa  más  común  de  hipercalcemia  en  pacientes  hospitalizados  es  la  
malignidad.  El  HPT  primario  y  la  malignidad  representan  el  90%  de  los  casos  de  hipercalcemia.

4.  ¿Cuál  es  el  diagnóstico  diferencial  de  la  hipercalcemia?
Endocrino:  
HPT.
Hipertiroidismo.
La  enfermedad  de  Addison.

Neoplasia  
maligna:  metástasis  ósea.

Síndromes  paraneoplásicos.
Tumores  sólidos  (carcinoma  de  pulmón  escamoso  o  de  células  pequeñas).
Neoplasia  maligna  hematológica  (mieloma,  leucemia,  linfoma).
Aumento  de  la  ingesta:  

síndrome  de  leche  alcalina.
Intoxicación  por  vitamina  D.
Enfermedad  granulomatosa:  
Sarcoidosis.
Tuberculosis.
Varios:  
Hipercalcemia  hipocalciúrica  familiar  (FHH).
Tiazida.
Litio.

281
Machine Translated by Google
282  CAPÍTULO  57  HIPERPARATIROIDISMO

5.  ¿Cuál  es  la  evaluación  de  laboratorio  esencial  para  HPT?
El  calcio  sérico  elevado  (>10,3  mg/dl)  debe  evaluarse  al  menos  dos  veces.  La  hipercalcemia  debe  estar  asociada  con  la  
elevación  de  la  hormona  paratiroidea  intacta.  Los  niveles  de  fosfato  sérico  son  bajos  en  casi  el  80%  de  los  pacientes  y  el  
cloruro  sérico  aumenta  en  el  40%  de  los  pacientes.  Una  relación  cloruro/fosfato  superior  a  33  asociada  con  hipercalcemia  
sugiere  HPT  primario.
Los  niveles  elevados  de  fosfatasa  alcalina  ocurren  solo  en  el  contexto  de  una  enfermedad  ósea  avanzada.
Los  pacientes  con  fosfatasa  alcalina  elevada  asociada  con  un  aumento  de  nitrógeno  ureico  en  sangre  (BUN)  y  creatinina  
tienen  un  mayor  riesgo  de  síndrome  del  hueso  hambriento  después  de  la  paratiroidectomía.
Una  recolección  de  orina  de  24  horas  para  la  excreción  de  calcio  y  creatinina  excluye  FHH  benigna.  En  pacientes  con  
HPT  primario,  el  calcio  en  orina  de  24  horas  es  >150  mg/día  versus  <100  mg/día  en  aquellos  con  FHH.

6.  Describir  la  embriología  y  anatomía  de  las  glándulas  paratiroides.
Las  glándulas  paratiroides  superiores  surgen  de  la  parte  dorsal  de  la  cuarta  bolsa  braquial  junto  con  los  lóbulos  laterales  
de  la  tiroides.  Las  glándulas  paratiroides  inferiores  surgen  de  la  parte  dorsal  de  la  tercera  bolsa  braquial  junto  con  el  timo.

El  peso  promedio  de  una  glándula  paratiroides  normal  es  de  35  a  50  mg.  La  glándula  paratiroides  superior
Se  encuentra  en  la  porción  posterior  de  la  mitad  superior  de  la  tiroides,  cefálica  a  la  arteria  tiroidea  inferior  y  
posterior  al  nervio  laríngeo  recurrente  (NLR).  La  glándula  paratiroides  inferior  normal  se  encuentra  en  la  superficie  
lateral  o  posterior  del  polo  inferior  de  la  glándula  tiroides,  caudal  y  anterior  al  punto  en  que  la  arteria  tiroidea  inferior  cruza  el  
NLR.
Cuatro  glándulas  están  presentes  en  el  89%  de  los  pacientes,  cinco  en  el  8%,  seis  en  el  3%  y  menos  de  cuatro  en  el  0%.
Se  ha  informado  la  presencia  de  hasta  ocho  glándulas.
La  ubicación  de  las  glándulas  paratiroides  superiores  es  más  constante.  Los  sitios  ectópicos  más  comunes  de  las  
glándulas  superiores  son  posteriores  al  esófago  o  en  el  surco  traqueoesofágico  hacia  el  mediastino  posterosuperior.  Las  
glándulas  paratiroides  inferiores  son  más  comúnmente  ectópicas  y  pueden  estar  en  el  ligamento  tirotímico,  el  timo,  el  
mediastino  (pero  fuera  del  timo),  la  vaina  carotídea  o  dentro  de  la  tiroides  (3%).

7.  ¿Cuáles  son  las  indicaciones  de  la  paratiroidectomía?
El  taller  de  NIH  de  2002  sobre  hiperparatiroidismo  asintomático  enumeró  las  indicaciones  como  calcio  sérico  >1,0  mg/dl  
por  encima  de  los  límites  superiores  de  lo  normal,  calcio  en  orina  de  24  horas  >400  mg,  eliminación  de  creatinina  reducida  >30  
%,  osteoporosis  (puntuación  t  de  densidad  mineral  ósea  <–  2.5  en  cualquier  sitio),  edad  <50  años  y  pacientes  en  los  que  la  
vigilancia  no  es  posible  o  deseable.
La  anamnesis  cuidadosa  indica  que  la  mayoría  de  los  pacientes  (>90%)  tienen  síntomas.  el  tratamiento  de
pacientes  asintomáticos  con  elevación  mínima  (10,3  a  11,0  mg/dl)  de  calcio  sérico  es  controvertido.  Sin  embargo,  al  
menos  cuatro  factores  favorecen  la  operación:
&  Los  pacientes  con  HPT  primario  no  tratado  tienen  una  mayor  tasa  de  mortalidad  causada  por  problemas  cardiovasculares.
enfermedad.

  Los  pacientes  con  HPT  tienen  puntajes  de  calidad  de  vida  anormales,  y  estos  puntajes  mejoran  a
normal  después  del  éxito  operatorio.
  El  coste  de  la  paratiroidectomía  es  equivalente  al  seguimiento  médico  a  los  5  años.
&  Los  cirujanos  endocrinos  experimentados  tienen  una  alta  tasa  de  éxito  (=95  %)  con  baja  morbilidad  y
tasas  de  mortalidad.

8.  ¿Puede  un  paciente  tener  hiperparatiroidismo  y  una  hormona  paratiroidea  normal?
¿nivel?

Sí.  Una  respuesta  paratiroidea  normal  al  calcio  sérico  elevado  es  disminuir  la  secreción  de  hormona  paratiroidea.  Cuando  
el  calcio  sérico  es  alto,  el  nivel  de  hormona  paratiroidea  debe  estar  en  el  extremo  inferior  de  lo  normal.  Un  nivel  de  hormona  
paratiroidea  en  el  extremo  superior  de  lo  normal  en  el  contexto  de  hipercalcemia  está  inapropiadamente  elevado  y  esto  es  
compatible  con  HPT.  Se  debe  descartar  FHH  haciendo  un  análisis  de  calcio  y  creatinina  en  orina  de  24  horas.
Machine Translated by Google
CAPÍTULO  57  HIPERPARATIROIDISMO  283

9.  ¿Qué  es  el  hiperparatiroidismo  normocalcémico?
Ocasionalmente,  los  pacientes  serán  identificados  (a  menudo  en  el  estudio  de  la  osteoporosis)  con  
un  nivel  elevado  de  hormona  paratiroidea  pero  con  calcio  sérico  normal.  El  calcio  está  típicamente  en  los  
límites  superiores  del  rango  normal.  La  mayoría  de  estos  pacientes  tendrán  calcio  ionizado  elevado.  Si  el  
nivel  de  calcio  es  bajo,  el  diagnóstico  es  HPT  secundario,  que  puede  ser  el  resultado  de  una  deficiencia  de  
vitamina  D.

10.  Resuma  la  estrategia  quirúrgica  tradicional  de  una  exploración  inicial  para  HPT  primario.
La  operación  tradicional  es  una  exploración  bilateral  del  cuello  para  identificar  las  cuatro  glándulas.  Se  debe  
mantener  un  campo  quirúrgico  meticulosamente  seco  y  libre  de  sangre.  El  tejido  en  la  región  del  nervio  laríngeo  
recurrente  (NLR)  no  debe  pinzarse  ni  dividirse  hasta  que  se  identifique  definitivamente  el  nervio.
Si  se  encuentra  un  adenoma  solitario  y  tres  glándulas  normales,  se  extirpa  el  adenoma  y  se  realiza  
una  biopsia  de  una  de  las  glándulas  normales.  El  examen  de  sección  congelada  confirma  que  el  tejido  
es  paratiroideo,  pero  no  es  confiable  para  diferenciar  el  adenoma  de  la  hiperplasia.  El  agrandamiento  de  
cuatro  glándulas  (hiperplasia)  indica  una  paratiroidectomía  subtotal  (que  deja  aproximadamente  50  mg  de  
tejido  paratiroideo  bien  vascularizado  en  el  cuello)  o  una  paratiroidectomía  total  con  autotrasplante  de  50  mg  
de  tejido  paratiroideo.  Si  queda  un  remanente  en  el  cuello,  debe  marcarse  con  una  sutura  no  absorbible  o  una  
grapa.  En  el  marco  de  la  hiperplasia,  una  timectomía  elimina  la  posibilidad  de  glándulas  supernumerarias  
tímicas.  Si  se  encuentra  más  de  una  glándula  agrandada  en  asociación  con  glándulas  de  apariencia  normal  
(adenoma  doble),  se  deben  extirpar  todas  las  glándulas  anormales.

11.  ¿Cuál  es  la  patología  del  HPT  primario?
El  HPT  primario  está  causado  por  un  solo  adenoma  en  el  87%  de  los  casos;  hiperplasia  en  9%;  
adenoma  doble  en  3%;  y  carcinoma  en  <1%.  En  el  HPT  familiar,  los  síndromes  de  neoplasia  endocrina  
múltiple  (MEN)  (I  y  II)  y  el  HPT  como  resultado  de  una  enfermedad  renal  en  etapa  terminal,  la  regla  es  la  hiperplasia.

12.  ¿Qué  alternativas  mínimamente  invasivas  existen  al  abordaje  quirúrgico  estándar?
La  paratiroidectomía  enfocada,  la  paratiroidectomía  mínimamente  invasiva  guiada  por  radio  (MIRP)  y  la  
exploración  endoscópica  del  cuello  son  técnicas  nuevas.  Una  paratiroidectomía  enfocada  utiliza  la  localización  
preoperatoria  para  guiar  una  paratiroidectomía,  evitando  una  exploración  bilateral.  Este  abordaje  se  combina  
con  una  prueba  de  hormona  paratiroidea  "rápida"  intraoperatoria.  La  hormona  paratiroidea  se  mide  mediante  
un  ensayo  modificado  antes  de  la  operación  y  10  minutos  después  de  la  resección  adecuada.  Un  nivel  
posterior  a  la  resección  <50%  del  nivel  preoperatorio  y  dentro  del  rango  normal  predice  el  éxito.  El  MIRP  utiliza  
una  gammagrafía  con  sestamibi  la  mañana  de  la  cirugía  y  una  sonda  gamma  intraoperatoria  para  guiar  la  
paratiroidectomía.  Se  mide  la  relación  entre  la  radiactividad  ex  vivo  y  la  de  fondo  para  determinar  el  éxito  y,  
por  lo  tanto,  el  final  de  la  operación.

13.  ¿Qué  estudios  de  localización  preoperatoria  existen?
El  mejor  estudio  de  localización  individual  es  la  gammagrafía  con  sestamibi.  Otros  estudios  de  localización  no  
invasivos  incluyen  la  ecografía  (US),  la  tomografía  computarizada  (TC)  y  la  resonancia  magnética  nuclear  
(RMN).  Los  procedimientos  de  localización  invasivos  incluyen  arteriografía  y  muestreo  venoso.  Las  pruebas  
son  más  precisas  con  una  sola  glándula  paratiroides  anormal.  Los  procedimientos  de  localización  en  casos  
de  hiperplasia  pueden  ser  engañosos.
La  gammagrafía  preoperatoria  con  sestamibi  o  la  ecografía  se  utilizan  para  permitir  una  cirugía  mínimamente  invasiva.

paratiroidectomía.
Los  estudios  de  localización  son  obligatorios  antes  de  todas  las  paratiroidectomías  reoperatorias  por  HPT  
persistente  o  recurrente  y  en  pacientes  con  cirugía  tiroidea  previa.

14.  ¿Cómo  se  realiza  una  gammagrafía  con  sestamibi  y  qué  precisión  tiene?
Sestamibi  es  un  radionúclido  que  es  absorbido  por  el  corazón,  la  tiroides,  las  glándulas  salivales  y  el  tejido  
paratiroideo  anormal.  Una  exploración  estándar  consiste  en  administrar  el  sestamibi  y  realizar  imágenes  
planares  del  cuello  y  la  parte  superior  del  tórax  a  los  30  minutos  y  de  2  a  3  horas  más  tarde.  El
Machine Translated by Google
284  CAPÍTULO  57  HIPERPARATIROIDISMO

el  radionúclido  se  ve  típicamente  en  todos  los  tejidos  antes  mencionados  en  la  exploración  inicial,  pero  por  lo  
general  “se  elimina”  rápidamente  del  corazón  y  la  tiroides.  Por  razones  poco  claras,  el  radionúclido  se  retiene  en  las  
paratiroides  por  más  tiempo,  por  lo  que  cualquier  captación  remanente  observada  en  la  gammagrafía  diferida  es  específica  
de  tejido  paratiroideo  anormal.  La  sensibilidad  y  especificidad  de  la  gammagrafía  con  sestamibi  es  de  alrededor  del  85%.
Un  enfoque  alternativo  es  administrar  sestamibi  y  yodo  o  pertecnatato  que  solo  es  absorbido  por  la  tiroides.  La  tiroides  
se  puede  sustraer  de  la  gammagrafía  con  Sestamibi  y  revelar  un  adenoma.  Una  alternativa  a  las  imágenes  planares  es  la  
tomografía  computarizada  por  emisión  de  fotón  único  (SPECT),  que  proporciona  información  tridimensional  presentada  
como  cortes  transversales  a  través  del  paciente  y  puede  reformatearse  según  sea  necesario.

15.  ¿Qué  se  debe  hacer  si  no  se  encuentra  un  adenoma  en  las  localizaciones  habituales?

Cada  glándula  normal  debe  ser  biopsiada  para  confirmación  y  marcada.  Las  glándulas  paratiroides  normales  no  deben  
extirparse.  Si  se  identifican  tres  glándulas  normales,  el  cirujano  debe  evaluar  si  la  glándula  faltante  es  superior  o  inferior.  Una  
glándula  superior  que  falta  a  menudo  se  encuentra  en  el  surco  traqueoesofágico,  posterior  al  esófago  o  en  el  mediastino  
posterosuperior.
El  error  común  es  que  la  tiroides  superior  no  se  moviliza  satisfactoriamente  y  la  disección  no  se  realiza  lo  suficientemente  
posterior.  Esta  exploración  puede  requerir  la  ligadura  de  la  arteria  y  la  vena  tiroideas  superiores.  La  ubicación  de  una  
glándula  inferior  faltante  es  más  variable.  Primero,  se  debe  inspeccionar  el  ligamento  tirotímico.  Luego,  el  timo  se  puede  
resecar  a  través  de  la  incisión  del  cuello.  A  continuación,  se  debe  abrir  la  vaina  carotídea.  Finalmente,  el  lóbulo  tiroideo  del  
lado  de  la  paratiroides  faltante  debe  palparse  o  examinarse  mediante  ecografía  intraoperatoria  en  busca  de  nódulos.  Si  se  
encuentra  un  nódulo,  se  realiza  una  lobectomía  y  se  examina  el  tejido  por  sección  congelada;  puede  ser  una  glándula  
paratiroides  intratiroidea.  Una  lobectomía  tiroidea  a  ciegas  rara  vez  es  útil.

Nunca  se  debe  realizar  una  esternotomía  como  parte  de  una  exploración  inicial.  Si  las  maniobras  antes  mencionadas  
no  logran  revelar  un  adenoma  paratiroideo,  el  cirujano  debe  detenerse.
Se  debe  hacer  un  diagrama  de  la  ubicación  de  las  glándulas  identificadas  para  futuras  referencias.  La  hipercalcemia  
persistente  indica  la  necesidad  de  procedimientos  de  localización.
La  paratiroides  no  descendida  es  rara  (<1%)  y  se  localiza  en  la  bifurcación  carotídea  o  por  encima  de  ella.  Esta  
área  no  se  puede  explorar  a  través  de  la  incisión  estándar  y  requiere  estudios  de  localización  preoperatorios  precisos.

16.  ¿Cuál  es  el  resultado  de  la  cirugía  por  hipocalcemia  primaria?
La  tasa  de  curación  esperada  debe  ser  del  95  %  para  los  pacientes  que  se  someten  a  una  exploración  inicial  por  HPT  
primario.  La  mejoría  sintomática  supera  el  95%.  Las  puntuaciones  de  calidad  de  vida  vuelven  a  la  normalidad  a  los  6  meses.  
Después  de  la  paratiroidectomía,  el  80%  de  los  pacientes  sintomáticos  mejoran  la  densidad  ósea  y  la  función  renal.  Incluso  
en  pacientes  asintomáticos,  los  niveles  urinarios  de  calcio  y  desoxipiridinolina  disminuyen.  Los  pacientes  tienen  menos  
episodios  de  nefrolitiasis,  gota  y  úlcera  péptica.
La  paratiroidectomía  también  parece  mejorar  la  longevidad  en  pacientes  con  HPT  primario.
Una  serie  reciente  sugiere  que  la  tasa  de  curación  es  menor  en  un  paciente  que  tiene  una
gammagrafías  preoperatorias  con  sestamibi  (99,3  %  frente  a  92,7  %).

17.  ¿Cuáles  son  las  complicaciones  de  la  paratiroidectomía?
La  lesión  permanente  del  NLR  ocurre  en  <1%  de  los  pacientes;  sin  embargo,  una  paresia  nerviosa  temporal  ocurre  
en  el  3%.  La  hipocalcemia  temporal  ocurre  en  el  10%  de  los  pacientes,  pero  el  HPT  permanente  ocurre  solo  en  el  2%  de  los  
casos.  Un  nivel  elevado  de  fosfatasa  alcalina  preoperatoria  y  una  función  renal  anormal  pueden  predecir  qué  pacientes  
tienen  probabilidades  de  tener  el  síndrome  de  «huesos  hambrientos».

18.  ¿Cuáles  son  los  signos  y  síntomas  de  hipocalcemia  después  de  la  cirugía?
El  signo  de  Chvostek  es  un  espasmo  de  los  músculos  faciales  causado  por  un  golpeteo  en  el  tronco  del  nervio  facial.
El  signo  de  Trousseau  es  un  espasmo  del  carpo  provocado  por  la  oclusión  de  la  arteria  braquial  durante  3  minutos  con  un  
manguito  de  presión  arterial.
El  síntoma  más  temprano  de  la  hipocalcemia  es  el  entumecimiento  perioral  o  parestesias  típicamente  en  las  manos  
o  los  pies.  La  hipocalcemia  grave  no  tratada  puede  provocar  espasmo  del  carpelo  o  tetania.
Machine Translated by Google
CAPÍTULO  57  HIPERPARATIROIDISMO  285

19.  ¿Cómo  se  debe  tratar  a  los  pacientes  con  hipocalcemia?
Los  pacientes  con  tetania  causada  por  hipoparatiroidismo  requieren  tratamiento  de  emergencia  con  
calcio  intravenoso  (IV)  para  prevenir  el  estridor  laríngeo  y  las  convulsiones.  Se  debe  administrar  una  ampolla  
de  gluconato  de  calcio  al  10  %  (90  mg  de  calcio  elemental  por  10  ml)  en  100  ml  de  solución  salina  durante  15  
minutos,  seguida  de  una  infusión  de  calcio  (5  ampollas  de  gluconato  de  calcio  en  500  ml  de  solución  salina)  a  50  ml/
h. .  Mantener  niveles  de  calcio  de  7,5  a  9,0  mg/dl  es  adecuado.  El  calcio  oral  debe  iniciarse  lo  antes  posible  en  
forma  de  carbonato  de  calcio  (Tums  u  Oscal)  a  razón  de  2  a  3  g/día  en  dosis  divididas  (3  a  4  veces/día).  Se  prefiere  
el  citrato  de  calcio  para  pacientes  con  litiasis  renal  porque  el  citrato  puede  ser  profiláctico  contra  la  litiasis  renal.  En  
la  mayoría  de  los  pacientes,  las  preparaciones  de  vitamina  D  aumentan  la  absorción  intestinal  y  se  pueden  
administrar  como  calcitriol  (Rocaltrol),  de  0,25  a  0,75  mg  por  día.

20.  Defina  HPT  persistente  y  recurrente.
El  éxito  operatorio  se  define  por  la  normocalcemia  a  largo  plazo.  El  HPT  persistente  se  define  como  
hipercalcemia  dentro  de  los  6  meses  posteriores  a  la  cirugía;  El  HPT  recurrente  se  define  como  hipercalcemia  
después  de  6  meses.

21.  ¿Cuál  es  la  estrategia  para  el  manejo  de  pacientes  con  HPT  persistente  o  recurrente?
Primero,  el  paciente  debe  ser  reevaluado  para  asegurar  que  la  hipercalcemia  sea  causada  por  HPT  primario  y  no  
por  alguna  otra  causa.  Los  pacientes  deben  ser  evaluados  por  hipercalcemia  hipocalciúrica  familiar,  que  no  justifica  
la  reoperación.  Se  evalúa  la  gravedad  de  la  enfermedad  para  garantizar  que  se  justifique  la  repetición  de  la  
operación.  Se  deben  revisar  las  notas  operativas  anteriores  y  los  informes  de  patología  para  ayudar  a  planificar  la  
terapia  repetida.  Los  estudios  de  localización  deben  usarse  ampliamente.  Antes  de  la  reexploración,  se  debe  evaluar  
la  función  de  las  cuerdas  vocales  en  todos  los  pacientes.
Se  repite  la  exploración  cervical  a  través  de  la  incisión  anterior.  Porque  los  músculos  de  la  correa
suelen  adherirse  a  la  tiroides,  se  puede  utilizar  un  abordaje  lateral  a  través  del  plano  entre  el  
esternocleidomastoideo  y  los  músculos  infrarrojos  en  lugar  del  abordaje  medial  habitual.  Con  estudios  de  localización  
positivos  o  determinación  retrospectiva  del  lado  del  adenoma  que  falta,  la  disección  puede  limitarse  si  se  encuentra  
un  adenoma.
Una  alternativa  a  la  exploración  repetida  en  pacientes  que  se  adaptan  bien  es  la  ablación  angiográfica  de
tejido  paratiroideo,  que  es  especialmente  útil  para  los  adenomas  mediastínicos  porque  evita  una  esternotomía  
mediana.  Se  realiza  administrando  contraste  iónico  a  través  de  un  catéter  arterial  encajado  en  el  vaso  de  
alimentación.
Cuando  la  localización  es  exitosa,  la  reoperación  por  HPT  persistente  o  recurrente  es  curativa  en  85%
al  90%.  La  reexploración  cuando  la  localización  no  tiene  éxito  tiene  una  tasa  de  fracaso  del  50%.

22.  Definir  hiperparatiroidismo  secundario  y  terciario.
La  sobreproducción  de  hormona  paratiroidea  (PTH)  provocada  en  respuesta  a  un  nivel  bajo  de  calcio  en  la  sangre  
causado  por  otra  afección  se  denomina  hiperparatiroidismo  secundario.  Las  causas  más  comunes  incluyen  
deficiencia  de  vitamina  D,  insuficiencia  renal  crónica,  deficiencia  de  calcio  y  trastornos  del  metabolismo  del  fosfato.
El  HPT  terciario  generalmente  ocurre  después  de  un  HP  secundario  prolongado  y  se  caracteriza  por
secreción  autónoma  de  PTH  que  causa  hipercalcemia.  La  secreción  de  PTH  no  responde  a  la  administración  
de  calcio  o  vitamina  D.

23.  ¿Cuáles  son  las  indicaciones  de  la  paratiroidectomía  en  la  enfermedad  renal  terminal?
Las  principales  indicaciones  para  la  paratiroidectomía  son:
  Hipercalcemia  grave.
  Enfermedad  ósea  hiperparatiroidea  progresiva  y  debilitante.
  Prurito  que  no  responde  a  la  diálisis.
  Calcificación  extraesquelética  progresiva  o  calcifilaxis  que  suele  asociarse  a  hiperfosfatemia.

  Miopatía  sintomática  sin  otra  explicación.
&  Receptores  de  trasplante  renal  con  hiperparatiroidismo  persistente  asociado  con  hipercalcemia  y
insuficiencia.
Machine Translated by Google
286  CAPÍTULO  57  HIPERPARATIROIDISMO

24.  ¿Cuáles  son  las  opciones  de  tratamiento  quirúrgico  del  HPT  secundario  y  terciario?
Tanto  la  paratiroidectomía  subtotal  como  la  paratiroidectomía  total  con  autotrasplante  corrigen  eficazmente  
el  HPT  secundario  y  terciario.
La  paratiroidectomía  subtotal  se  realiza  mediante  la  escisión  de  todo  el  tejido  paratiroideo  identificable  excepto
40  a  60  mg  de  una  glándula.  Las  desventajas  de  la  paratiroidectomía  subtotal  incluyen  el  riesgo  de  
enfermedad  recurrente,  que  se  complica  por  una  mayor  morbilidad  si  se  requiere  repetir  la  exploración  del  
cuello.  La  paratiroidectomía  total  con  autotrasplante  de  pequeñas  cantidades  de  tejido  paratiroideo  resecado  
en  el  músculo  braquiorradial  del  antebrazo  tiene  la  principal  ventaja  de  la  facilidad  de  extirpar  las  glándulas  
hiperplásicas  recurrentes  sin  la  morbilidad  adicional  de  la  reexploración  del  cuello.

25.  Enumerar  las  endocrinopatías  en  MEN  I  y  II.
MEN  I  (3  P's):  
Hiperparatiroidismo.
Adenoma  pituitario.
Tumor  endocrino  pancreático.

MEN  II  (3  C's):  
Hiperparatiroidismo  (cálcico).
Cáncer  medular  de  tiroides  (calcitonina).
Feocromocitoma  (catecolaminas).

26.  ¿Cuál  es  el  abordaje  quirúrgico  preferido  para  HPT  en  pacientes  con  MEN?
El  HPT  se  desarrolla  en  más  del  90  %  de  los  pacientes  con  MEN  I.  Los  pacientes  suelen  tener  múltiples  tumores,  
pero  pueden  tener  un  tamaño  asimétrico.  La  operación  preferida  es  la  paratiriodectomía  subtotal  con  timectomía  
transcervical  (la  ubicación  más  probable  de  las  glándulas  supernumerarias).  El  tejido  paratiroideo  se  puede  
criopreservar  para  autoinjertos  posteriores  en  caso  de  hipoparatiroidismo.
En  MEN  II,  20%  a  30%  de  los  pacientes  desarrollan  HPT  y  los  tumores  múltiples  son  la  norma.  El  HPT  tiende
ser  más  leve  que  en  MEN  I.  Es  necesario  inspeccionar  las  cuatro  glándulas  con  resección  solo  de  glándulas  
agrandadas  o  paratiroidectomía  subtotal.  Los  niveles  de  PTH  intraoperatorios  pueden  ayudar  a  guiar  la  resección.

27.  ¿Quién  realizó  la  primera  paratiroidectomía?
En  1925,  Felix  Mendl  realizó  con  éxito  la  primera  paratiroidectomía  en  la  Clínica  Hochenegg  de  Viena.  Su  
paciente  era  Albert,  un  conductor  de  tranvía  de  34  años  que  no  podía  trabajar  debido  a  una  osteítis  fibrosa  quística  
severa.

28.  ¿Quién  fue  el  Capitán  Martell?
Oficial  de  la  Marina  Mercante  de  los  EE.  UU.,  el  Capitán  Martell  fue  el  primer  paciente  en  los  Estados  Unidos  en  
someterse  a  una  cirugía  por  HPT  primario.  El  Capitán  Martell  tenía  HPT  progresivo  que  redujo  su  altura  de  6  pies  a  5  
pies  y  6  pulgadas  cifóticos.  Después  de  siete  operaciones,  finalmente  se  extirpó  el  adenoma  del  mediastino;  sin  
embargo,  el  capitán  murió  de  insuficiencia  renal  crónica.

BIBLIOGRAFÍA

1.  Ambrogini  E,  Cetani  F,  Cianferotti  L  et  al.:  Cirugía  o  vigilancia  de  la  enfermedad  primaria  leve  asintomática
hiperparatiroidismo:  un  ensayo  clínico  prospectivo  y  aleatorizado.  J  Clin  Endocrinol  Metab  92:3114­3121,  2007.

2.  Eigelberger  MS,  Cheah  WK,  Ituarte  PH  et  al.:  Los  criterios  NIH  para  paratiroidectomía  en  pacientes  primarios  asintomáticos
hiperparatiroidismo:  ¿son  demasiado  limitados?  Ann  Surg  239(4):528­535,  2004.
3.  Gil­Cardenas  A,  Gamino  R,  Reza  A  et  al.:  ¿Es  obligatorio  el  ensayo  intraoperatorio  de  hormona  paratiroidea  para  el
éxito  de  la  paratiroidectomía  dirigida?  J  Am  Coll  Surg  204(2):286­290,  2007.
4.  Kebebew  E,  Hwang  J,  Reiff  E  et  al .:  Predictores  de  enfermedad  paratiroidea  de  una  sola  glándula  frente  a  multiglándula  en  
el  hiperparatiroidismo  primario:  un  modelo  de  puntuación  simple  y  preciso.  Arco  Surg  2006;  141:777­782.

5.  Lambert  LA,  Shapiro  SE,  Lee  JE  et  al.:  Tratamiento  quirúrgico  del  hiperparatiroidismo  en  pacientes  con  múltiples
neoplasia  endocrina  tipo  1.  Arch  Surg  140:374­382,  2005.
Machine Translated by Google
CAPÍTULO  57  HIPERPARATIROIDISMO  287

6.  Lo  CY,  Lang  BH,  Chan  WF  et  al .:  una  evaluación  prospectiva  de  la  localización  preoperatoria  por  tecnecio­99m
Gammagrafía  y  ultrasonografía  con  sestamibi  en  el  hiperparatiroidismo  primario.  Am  J  Surg  193:155­159,  2007.

7.  Nilsson  IL,  Aberg  J,  Rastad  J  et  al.:  Normalización  mantenida  de  la  disfunción  cardiovascular  5  años  después  de  la  
paratiroidectomía  en  el  hiperparatiroidismo  primario.  Cirugía  2005;  137(6):632­638,  2005.

8.  Pappu  S,  Donovan  P,  Cheng  D  et  al.:  No  todas  las  exploraciones  con  Sestamibi  se  crean  por  igual.  Arch  Surg  140:383­386,  2005.

9.  Phitayakorn  R,  McHenry  CR.  Incidencia  y  ubicación  de  glándulas  paratiroides  anormales  ectópicas.  Am  J  Surg  191:
418­423,  2006.

10.  Richards  ML,  Wormuth  J,  Bingener  J  et  al.:  Paratiroidectomía  en  el  hiperparatiroidismo  secundario:  ¿existe  un
manejo  operativo  óptimo?  Cirugía  139:174­180,  2006.
Machine Translated by Google

CAPITULO  
58

HIPERTIROIDISMO
Robert  C.  McIntyre,  Jr.,  MD  y  Christopher  D.  Raeburn,  MD

1.  ¿Cuáles  son  los  signos  y  síntomas  del  hipertiroidismo?
General:  intolerancia  al  calor,  transpiración,  sofocos,  temblores,  trastornos  del  sueño  o  pérdida  de  cabello.
Psicológicos:  Nerviosismo,  labilidad  emocional,  ansiedad,  agresividad  o  delirios.
Cardiovascular:  palpitaciones,  taquicardia  o  arritmias  supraventriculares.
Respiratorio:  Falta  de  aire  o  ronquera.
Gastrointestinales:  aumento  del  apetito,  pérdida  de  peso  o  aumento  de  la  frecuencia  de  las  deposiciones
movimientos
Reproductivo:  Ginecomastia  o  menstruaciones  irregulares.
Hueso:  Osteoporosis.
Otros:  Oftalmopatía  o  dermopatía.

2.  ¿Cuáles  son  las  tres  causas  más  comunes  de  hipertiroidismo?
La  enfermedad  de  Graves.

Bocio  multinodular  tóxico  (TMNG).
Nódulo  tóxico  (enfermedad  de  Plummer).

3.  ¿Cómo  debe  investigarse  el  hipertiroidismo?
Un  nivel  de  hormona  estimulante  de  la  tiroides  (TSH)  es  la  mejor  prueba  inicial.  Una  TSH  baja  con  un  nivel  sérico  
alto  de  tiroxina  libre  (T4)  o  triyodotironina  (T3)  es  diagnóstica.  Una  TSH  alta  con  un  aumento  de  la  T4  libre  indica  
el  raro  paciente  con  un  tumor  hipofisario  productor  de  tirotropina.  El  hipertiroidismo  subclínico  es  una  TSH  
suprimida  con  una  T4  o  T3  normal  alta.
Después  de  realizar  el  diagnóstico  de  hipertiroidismo,  la  captación  de  yodo  radiactivo  (RAIU)  y  la  exploración  
pueden  diferenciar  las  muchas  causas.  Una  captación  elevada  confirma  el  hipertiroidismo  resultante  de  la  
sobreproducción  de  hormona  tiroidea.  La  captación  generalmente  se  mide  a  las  4  a  6  horas  y  luego  nuevamente  
a  las  24  horas.  La  captación  uniforme  en  la  exploración  sugiere  enfermedad  de  Grave,  la  captación  en  parches  
sugiere  un  TMNG  y  un  área  unifocal  con  supresión  del  resto  de  la  tiroides  es  diagnóstico  de  un  adenoma  solitario  
tóxico.  La  captación  baja  y  difusa  sugiere  tiroiditis,  que  puede  causar  un  curso  autolimitado  de  hipertiroidismo  
secundario  a  la  liberación  de  hormona  tiroidea  preformada.

4.  ¿Cómo  se  establece  el  diagnóstico  de  la  enfermedad  de  Graves?
La  enfermedad  de  Grave  casi  siempre  se  puede  diagnosticar  sobre  la  base  de  los  hallazgos  clínicos.  La  
TSH  será  baja  en  asociación  con  un  aumento  de  T4  libre.  Si  la  T4  libre  es  normal,  se  obtiene  un  nivel  de  T3  libre  
para  descartar  toxicosis  por  T3 .  Graves  es  una  enfermedad  autoinmune  que  resulta  en  la  producción  de  
autoanticuerpos  que  estimulan  el  receptor  de  TSH  (inmunoglobulinas  estimulantes  de  la  tiroides,  TSI).  La  
oftalmopatía  es  exclusiva  de  la  enfermedad  de  Graves  y  es  el  resultado  de  autoanticuerpos  tiroideos  que  
reaccionan  de  forma  cruzada  con  los  músculos  extraoculares.  Una  RAIU  muestra  una  mayor  captación  uniforme.

5.  ¿Cuáles  son  las  tres  opciones  de  tratamiento?
Fármacos  antitiroideos  (ATD),  yodo  radioactivo  y  cirugía.

288
Machine Translated by Google
CAPÍTULO  58  HIPERTIROIDISMO  289

6.  ¿Qué  fármacos  son  útiles  para  el  tratamiento  del  hipertiroidismo?  ¿Cuáles  son  sus  mecanismos  de  acción?

El  metimazol  (MMI)  y  el  propiltiouracilo  (PTU)  son  los  pilares  del  tratamiento.  El  objetivo  del  tratamiento  es  la  remisión  
de  la  enfermedad  de  Graves  durante  la  terapia  o  el  eutiroidismo  antes  del  tratamiento  con  yodo  radioactivo  o  la  cirugía.  
Ambos  fármacos  inhiben  la  organificación  del  yodo  y  el  acoplamiento  de  las  yodotironinas.  PTU  también  inhibe  la  
monodesyodación  periférica  de  T4  a  la  T3  más  fisiológicamente  activa .  La  terapia  generalmente  se  mantiene  durante  
2  años.  Los  pacientes  deben  ser  monitoreados  por  efectos  secundarios,  que  incluyen  erupción  cutánea,  prurito,  hepatitis,  
ictericia  colestásica,  síndrome  similar  al  lupus  y  la  complicación  rara  pero  potencialmente  mortal  de  agranulocitosis.

Los  antagonistas  b­adrenérgicos  mejoran  los  signos  y  síntomas  de  la  enfermedad.  no  deberían  ser
se  usa  solo,  excepto  por  períodos  breves  antes  del  yodo  radiactivo  o  la  terapia  quirúrgica.  El  yodo  administrado  como  
solución  de  Lugol  (yodo  al  5%  y  yoduro  de  potasio  al  10%  en  agua,  0,3  ml/día)  o  yoduro  de  potasio  (60  mg  3  veces  
al  día)  inhibe  la  liberación  de  hormona  tiroidea.  Es  útil  para  la  terapia  a  corto  plazo  en  preparación  para  la  cirugía,  después  
de  la  terapia  con  yodo  radiactivo  para  acelerar  la  caída  de  los  niveles  hormonales  y  para  el  tratamiento  de  la  tormenta  
tiroidea.  Provoca  una  disminución  en  la  perfusión  de  la  tiroides,  lo  que  puede  reducir  el  sangrado  durante  la  tiroidectomía.

7.  ¿Cuáles  son  las  indicaciones  y  los  resultados  del  tratamiento  farmacológico?
La  ATD  se  reserva  para  el  hipertiroidismo  leve  y  una  glándula  pequeña.  Casi  todos  los  pacientes  se  volverán  eutiroideos  
dentro  de  las  6  semanas  posteriores  al  inicio  de  la  terapia;  sin  embargo,  el  hipertiroidismo  recurrente  ocurre  en  50%  de  
los  pacientes  cuando  se  suspende  el  medicamento.  La  remisión  a  largo  plazo  solo  ocurre  en  el  30%  de  los  pacientes.  
La  recaída  es  más  común  en  los  primeros  6  meses  después  de  la  interrupción  del  tratamiento.  Los  efectos  secundarios  de  
los  ATD  incluyen  agranulocitosis,  hepatotoxicidad  y  erupciones.

8.  ¿Cuál  es  el  régimen  de  tratamiento  con  yodo  radiactivo?
El  yodo  radiactivo  es  la  terapia  más  común.  La  dosis  habitual  de  yodo  radiactivo  es  de  10  a  15  mCi  para  la  
enfermedad  de  Graves.  TMNG  se  trata  con  dosis  ligeramente  más  altas  de  25  a  30  mCi.  Los  pacientes  mayores  y  
los  pacientes  con  comorbilidad  significativa  deben  recibir  ATD  antes  que  el  yodo  radiactivo  para  prevenir  la  tormenta  
tiroidea  inducida  por  I131 .  Los  pacientes  con  enfermedad  ocular  significativa  y  los  fumadores  deben  recibir  
corticosteroides  antes  que  el  yodo  radiactivo  para  prevenir  la  progresión  de  la  oftalmopatía.
El  embarazo  es  una  contraindicación  absoluta.  Las  mujeres  en  edad  fértil  deben  ser  evaluadas
con  una  prueba  de  embarazo  antes  del  tratamiento  y  debe  evitar  el  embarazo  durante  6  meses  después  del  
tratamiento.

9.  ¿Cuál  es  el  resultado  del  tratamiento  con  yodo  radiactivo?

El  eutiroidismo  no  se  logra  durante  meses  después  del  tratamiento.  Una  vez  que  se  logra  el  eutiroidismo,  la  
recurrencia  del  hipertiroidismo  es  rara.  El  hipotiroidismo,  el  único  efecto  secundario  grave,  depende  de  la  dosis.  Ocurre  
a  una  tasa  del  3%  por  año,  afecta  al  50%  de  los  pacientes  a  los  10  años  y  casi  al  100%  a  los  25  años.

10.  ¿Cuáles  son  las  indicaciones  de  la  tiroidectomía  por  hipertiroidismo?
1.  Las  pacientes  que  están  embarazadas  son  difíciles  de  tratar  médicamente.
2.  Pacientes  con  bocio  grande  y  baja  captación  de  yodo  radiactivo.
3  niños.

4.  Pacientes  no  cumplidores.
5.  Pacientes  con  nódulos  sospechosos  de  cáncer  (resfriado).
6.  Pacientes  con  compresión  de  la  tráquea  o  el  esófago.
7.  Pacientes  con  problemas  estéticos.

8.  Pacientes  con  oftalmopatía.
9.  Alergia  o  efectos  secundarios  significativos  a  los  ATD.
10.  Pacientes  con  comorbilidad  importante  que  necesiten  una  rápida  consecución  del  estado  eutiroideo.
Machine Translated by Google
290  CAPÍTULO  58  HIPERTIROIDISMO

11.  ¿Cómo  se  debe  preparar  a  los  pacientes  para  la  cirugía?
Cualquier  paciente  con  hipertiroidismo  debe  volverse  eutiroideo  antes  de  la  cirugía.  Los  pacientes  
pueden  ser  tratados  con  medicación  antitiroidea  más  yodo  potásico.  Los  antagonistas  b­adrenérgicos  también  
se  pueden  usar  solos  o  en  combinación  con  el  régimen  anterior.

12.  ¿Cuál  es  la  extensión  de  la  tiroidectomía?
Las  dos  opciones  quirúrgicas  para  la  enfermedad  de  Graves  son  la  tiroidectomía  subtotal  o  la  
tiroidectomía  casi  total.  Se  prefiere  la  tiroidectomía  casi  total  para  neoplasias  malignas  coexistentes  y  
pacientes  con  oftalmopatía  grave.  El  objetivo  de  la  tiroidectomía  subtotal  es  conservar  de  4  a  8  g  de  tejido  
tiroideo  bien  vascularizado  para  evitar  el  hipotiroidismo.  Sin  embargo,  debido  al  pequeño  riesgo  de  
recurrencia  (10%),  algunos  cirujanos  prefieren  la  tiroidectomía  casi  total.  En  la  enfermedad  de  Plummer,  la  
lobectomía  o  la  tiroidectomía  parcial  para  lesiones  unilaterales  y  la  tiroidectomía  subtotal  contralateral  para  
lesiones  múltiples  hacen  que  el  paciente  sea  eutiroideo.

13.  ¿Cuál  es  la  incidencia  de  hipotiroidismo  después  de  la  cirugía?
Todos  los  pacientes  que  se  someten  a  una  tiroidectomía  casi  total  se  vuelven  hipotiroideos  y  necesitan  reemplazo  de  T4 .
El  hipotiroidismo  ocurre  en  el  30%  de  los  pacientes  después  de  la  tiroidectomía  subtotal.

14.  ¿Cuál  es  el  tratamiento  adecuado  para  el  bocio  nodular  tóxico?
El  hipertiroidismo  por  bocio  nodular  tóxico  es  permanente  y  sin  remisión  espontánea;  Los  ATD  no  son  
una  terapia  adecuada  a  largo  plazo.  El  yodo  radiactivo  es  la  forma  más  común  de  terapia.  Dosis  mayores  (25  
a  30  mCi)  minimizan  el  riesgo  de  hipertiroidismo  persistente  en  estos  pacientes,  que  tienden  a  ser  mayores  y  
tener  síntomas  cardiovasculares  prominentes  de  hipertiroidismo.  La  cirugía  también  es  bastante  efectiva,  da  
como  resultado  el  logro  más  rápido  del  eutiroidismo  y  tiene  una  baja  tasa  de  recurrencia.

15.  ¿Cuál  es  el  tratamiento  adecuado  para  el  hipertiroidismo  resultante  de
tiroiditis?
Debe  sospecharse  tiroiditis  subaguda  si  el  paciente  tiene  dolor  y  sensibilidad  en  la  región  tiroidea.  
Una  RAIU  y  una  exploración  muestran  una  captación  disminuida.  El  hipertiroidismo  suele  ser  leve  y  de  corta  
duración  (semanas).  La  mayoría  de  los  pacientes  no  necesitan  tratamiento,  pero  se  utilizan  antagonistas  b­
adrenérgicos  y  salicilato  o  glucocorticoides  para  controlar  los  síntomas.  Puede  ocurrir  hipotiroidismo,  pero  por  
lo  general  no  es  permanente.

16.  ¿Cuál  es  el  tratamiento  adecuado  para  la  tormenta  tiroidea?
La  crisis  tirotóxica  debe  tratarse  en  la  unidad  de  cuidados  intensivos  (UCI).  Las  medidas  generales  incluyen  
hidratación,  antipiresis  (paracetamol)  y  nutrición.  Las  medidas  específicas  incluyen  la  inhibición  de  la  síntesis  
de  T4  y  la  conversión  a  T3  con  PTU  a  una  dosis  de  100  mg  por  vía  oral,  por  sonda  nasogástrica  (NG)  o  por  
vía  rectal  cada  6  horas.  Los  yoduros  inhiben  la  liberación  de  T4  (solución  saturada  de  yoduro  de  potasio,  
SSKI  5  gotas  por  vía  oral  o  sonda  NG  cada  6  horas).  Los  esteroides  (dexametasona,  2  mg  cada  6  horas)  
también  inhiben  la  liberación  de  T4  y  la  conversión  a  T3.  Los  antagonistas  b­adrenérgicos  (propranolol  o  
esmolol)  pueden  controlar  las  manifestaciones  cardiovasculares.  Otros  agentes  que  reducen  la  hormona  
tiroidea  son  el  ácido  iopanoico,  el  litio  y  el  perclorato  de  potasio.  La  última  opción  de  manejo  es  la  eliminación  
de  T4  por  plasmaféresis,  hemoperfusión  o  diálisis.

17.  ¿Qué  cirujano  ganó  el  premio  Nobel  por  su  trabajo  con  la  enfermedad  de  la  tiroides?
Theodor  Kocher  ganó  el  Premio  Nobel  de  Medicina  en  1909  por  su  trabajo  sobre  la  fisiología,  patología  
y  cirugía  de  la  glándula  tiroides.  Logró  reducir  la  alta  tasa  de  mortalidad  de  la  tiroidectomía  a  fines  del  siglo  
XIX.  En  1850,  la  tasa  de  mortalidad  era  del  50  %,  pero  en  1898  la  tasa  de  mortalidad  en  la  clínica  de  Kocher  
era  del  0,18  %.  Su  logro  más  significativo  fue  describir  el  hipotiroidismo  posoperatorio  como  caquexia  
strumipriva.
Machine Translated by Google
CAPÍTULO  58  HIPERTIROIDISMO  291

SITIOS  WEB

www.endocrinesurgery.org/

www.tiroides.org/
www.aace.com/

BIBLIOGRAFÍA

1.  Boger  MS,  Perrier  ND:  Ventajas  y  desventajas  de  la  terapia  quirúrgica  y  extensión  óptima  de  la  tiroidectomía  para  el  tratamiento  del  
hipertiroidismo.  Surg  Clin  North  Am  84(3):849­874,  2004.

2.  Cooper  DS:  Fármacos  antitiroideos.  N  Engl  J  Med  352(9):905­917,  2005.

3.  Erbil  Y,  Giris  M,  Salmaslioglu  A  et  al .:  El  efecto  de  la  duración  del  tratamiento  con  fármacos  antitiroideos  en  la  densidad  
de  microvasos  de  la  glándula  tiroides  y  la  pérdida  de  sangre  intraoperatoria  en  pacientes  con  enfermedad  de  Graves.  
Cirugía  143(2):216­225,  2008.

4.  Kang  AS,  Grant  CS,  Thompson  GB  et  al.:  Tratamiento  actual  del  bocio  nodular  con  hipertiroidismo  (Plummer's
enfermedad):  cirugía  versus  yodo  radiactivo.  Cirugía  132(6):916­923;  discusión  923,  2002.

5.  Lal  G,  Ituarte  P,  Kebebew  E  et  al.:  Debería  la  tiroidectomía  total  convertirse  en  el  procedimiento  preferido  para  la  cirugía
manejo  de  la  enfermedad  de  Graves?  Tiroides  15(6):569­574,  2005.

6.  Palit  TK,  Miller  CC  3rd,  Miltenburg  DM:  La  eficacia  de  la  tiroidectomía  para  la  enfermedad  de  Graves:  un  metanálisis.
J  Surg  Res  90(2):161­165,  2000.

7.  Schussler­Fiorenza  CM,  Bruns  CM,  Chen  H:  El  manejo  quirúrgico  de  la  enfermedad  de  Graves.  J  Surg  Res  133
(2):207­214,  2006.

8.  Vidal­Trecan  GM,  Stahl  JE,  Eckman  MH:  Yodo  radiactivo  o  cirugía  para  el  adenoma  tiroideo  tóxico:  disección  de  un
decision  importante.  Un  análisis  de  rentabilidad.  Tiroides  14(11):933­945,  2004.

9.  Weetman  AP:  Enfermedad  de  Graves,  N  Engl  J  Med  343(17):1236­1248,  2000.

10.  Witte  J,  Goretzki  PE,  Dotzenrath  C  et  al .:  Cirugía  para  la  enfermedad  de  Graves:  tiroidectomía  total  versus  subtotal:  resultados  de  un  
ensayo  prospectivo  aleatorizado.  World  J  Surg  24(11):1303­1311,  2000.
Machine Translated by Google

NÓDULOS  TIROIDEOS  Y  CÁNCER
CAPÍTULO  
59

Trevor  L.  Nydam,  MD  y  Robert  C.  McIntyre,  Jr.,  MD

1.  ¿Cuál  es  la  prevalencia  de  nódulos  tiroideos  y  cáncer?
La  prevalencia  de  nódulos  tiroideos  aumenta  a  lo  largo  de  la  vida.  Los  nódulos  son  4  veces  más  comunes  
en  mujeres  que  en  hombres.  La  prevalencia  depende  del  método  de  detección:  por  palpación  5%,  por  ultrasonido  
(US)  35%  y  por  autopsia  50%.  Después  de  la  exposición  a  la  radiación,  los  nódulos  se  desarrollan  en  
aproximadamente  un  2%  anual,  alcanzando  un  pico  a  los  25  años.  Cada  año  en  los  Estados  Unidos,  hay  
aproximadamente  24.000  nuevos  casos  de  cáncer  de  tiroides.  Hasta  el  35%  de  las  glándulas  tiroideas  examinadas  
en  la  autopsia  contienen  cáncer  papilar  oculto  (<1,0  cm).

2.  ¿Cuál  es  la  importancia  de  la  distinción  entre  tiroides  único  y  múltiple?
nódulos?
Tradicionalmente,  los  nódulos  tiroideos  múltiples  se  consideraban  benignos  y  los  nódulos  tiroideos  solitarios  
eran  malignos.  Sin  embargo,  múltiples  series  sugieren  que  un  nódulo  dominante  en  una  glándula  multinodular  
conlleva  el  mismo  riesgo  de  cáncer  que  un  nódulo  solitario  (5%).  La  ecografía  a  menudo  revelará  una  enfermedad  
multinodular  en  una  glándula  con  un  nódulo  solitario  conocido.

3.  ¿Qué  características  de  la  historia  y  el  examen  físico  indican  un  mayor  riesgo  de
¿cáncer?
Los  nódulos  que  aparecen  en  los  extremos  de  la  edad  tienen  más  probabilidades  de  ser  cancerosos,  especialmente  en  los  hombres.
El  rápido  crecimiento  y  la  invasión  local  aumentan  la  posibilidad  de  malignidad,  pero  los  síntomas  asociados  (p.  
ej.,  ronquera,  disfagia)  son  poco  frecuentes.  Una  historia  de  exposición  a  la  radiación  aumenta  la  frecuencia  de  
nódulos  tanto  benignos  como  malignos.  Una  historia  familiar  de  cáncer  de  tiroides  medular  o  papilar  o  síndrome  
de  Gardner  (es  decir,  poliposis  familiar)  aumenta  el  riesgo  de  cáncer.
El  cáncer  se  encuentra  con  mayor  frecuencia  en  pacientes  con  nódulos  solitarios  y  firmes.  La  fijación  a  
estructuras  adyacentes,  la  parálisis  de  las  cuerdas  vocales  y  los  ganglios  linfáticos  agrandados  también  se  
asocian  con  un  mayor  riesgo  de  malignidad.

4.  ¿Cuál  es  la  evaluación  de  laboratorio  adecuada  de  un  paciente  con  un  nódulo  tiroideo?
La  única  prueba  bioquímica  que  se  necesita  rutinariamente  es  una  concentración  sérica  de  hormona  estimulante  
de  la  tiroides  (TSH)  para  identificar  a  los  pacientes  con  hipertiroidismo  no  sospechado.  En  pacientes  con  sospecha  
de  carcinoma  medular  de  tiroides  (MTC),  se  debe  medir  la  calcitonina  sérica.  Los  pacientes  con  MTC  deben  
someterse  a  un  análisis  de  ADN  derivado  de  linfocitos  para  el  análisis  de  mutación  del  protooncogén  ret.  En  
pacientes  con  neoplasia  endocrina  múltiple  conocida  (MEN)  2,  se  deben  realizar  niveles  de  calcio  sérico  y  
recolección  de  orina  de  24  horas  para  evaluar  las  catecolaminas  y  sus  productos  metabólicos  para  evaluar  
hiperparatiroidismo  (HPT)  y  feocromocitoma  antes  de  la  tiroidectomía.

5.  ¿Qué  estudios  de  imagen  se  deben  realizar  en  la  evaluación  de  un  nódulo  tiroideo?
Los  pacientes  con  TSH  suprimida  (<0.5  mIU/mL)  deben  someterse  a  captación  de  yodo  radiactivo  (RAIU)  y  
exploración  para  evaluar  hipertiroidismo  resultante  de  la  enfermedad  de  Graves,  bocio  multinodular  tóxico  (TMNG)  
o  un  nódulo  autónomo.  Los  pacientes  con  TSH  normal  o  elevada  no  requieren  exploración  porque  esto  no  puede  
distinguir  de  manera  confiable  los  nódulos  benignos  de  los  malignos.
La  ecografía  de  tiroides  debe  realizarse  en  todos  los  pacientes  con  nódulos.  Determina  si  hay  un  nódulo
que  corresponde  a  una  anormalidad  palpable,  evalúa  otros  nódulos,  determina  si  un

292
Machine Translated by Google
CAPÍTULO  59  NÓDULOS  TIROIDEOS  Y  CÁNCER  293

nódulo  es  quístico,  sólido  o  mixto,  y  es  la  mejor  medida  del  tamaño  de  un  nódulo.  Las  características  
ecográficas  son  superiores  al  tamaño  para  determinar  el  riesgo  de  malignidad  e  incluyen  microcalcificación,  
hipoecogenicidad  de  un  nódulo  sólido  e  hipervascularización  intranodular.  La  ecografía  mejora  la  precisión  de  
la  biopsia  por  aspiración  con  aguja  fina  (FNA),  que  debe  realizarse  en  el  nódulo  que  más  preocupa.

6.  ¿Cuál  es  el  diagnóstico  diferencial  de  los  nódulos  tiroideos?
Adenoma:  
Macrofolicular  (coloide).
microfolicular.
embrionario.
Celda  de  Hürthle.

Carcinoma:  
papilar.
folicular.
Medular.
Anaplásico.
Linfoma.
metastásico.

Quiste  Bocio  nodular  con  un  nódulo  
dominante  Otras:  Enfermedades  inflamatorias  
(p.  ej.,  tiroiditis  de  Hashimoto).
Anomalías  del  desarrollo.

7.  ¿Qué  prueba  única  predice  mejor  la  necesidad  de  una  intervención  quirúrgica?
La  mejor  prueba  individual  para  predecir  la  necesidad  de  cirugía  es  la  aspiración  con  aguja  fina  (FNA).  Si  se  
obtiene  una  muestra  adecuada,  los  tres  resultados  posibles  son  benigno  (70%),  sospechoso  (15%)  y  maligno  
(5%).  La  FNA  es  más  confiable  para  el  diagnóstico  de  carcinoma  papilar  y  en  pacientes  con  cáncer  medular  y  
anaplásico.  Es  menos  fiable  para  distinguir  las  neoplasias  foliculares  y  de  células  de  Hurthle  benignas  de  las  
malignas.  La  precisión  general  supera  el  95%  en  manos  experimentadas.  Cuando  FNA  revela  cáncer,  es  97%  
correcto  (3%  de  tasa  de  falsos  positivos);  cuando  indica  un  nódulo  benigno,  el  cáncer  está  presente  en  el  4%  
de  los  casos  (tasa  de  falsos  negativos  del  4%).  Cuando  la  PAAF  es  sospechosa,  el  25%  de  los  nódulos  son  
malignos.

8.  ¿Es  útil  el  tratamiento  con  levotiroxina  en  el  manejo  de  los  nódulos  tiroideos?
En  regiones  del  mundo  con  baja  captación  de  yodo,  la  supresión  de  la  TSH  sérica  a  niveles  inferiores  a  los  normales  
mediante  el  uso  de  levotiroxina  puede  disminuir  el  tamaño  de  los  nódulos  benignos.  Sin  embargo,  en  áreas  con  suficiente  
yodo  no  se  recomienda  el  tratamiento  de  supresión.

9.  ¿Cuáles  son  los  tipos  y  la  distribución  del  cáncer  de  tiroides?
Papilar  80%.
Folicular  15%.
Medular  5%.
Anaplásico  y  linfoma  <1%.

10.  ¿Cuáles  son  los  axiomas  de  la  cirugía  de  tiroides?
&  Debe  mantenerse  un  campo  operatorio  meticulosamente  seco.
  El  tejido  de  la  región  del  nervio  laríngeo  recurrente  (NLR)  no  debe  cortarse  ni  pinzarse  hasta  que  se  
identifique  definitivamente  el  nervio.
  Cada  glándula  paratiroides  debe  tratarse  como  si  fuera  la  última  glándula  en  funcionamiento.
&  Si  se  sospecha  malignidad,  toda  la  operación  debe  realizarse  como  si  la  lesión  fuera
cáncer.
Machine Translated by Google
294  CAPÍTULO  59  NÓDULOS  TIROIDEOS  Y  CÁNCER

11.  Definir  los  distintos  tipos  de  procedimientos  tiroideos.
La  tiroidectomía  subtotal  está  destinada  a  tratar  el  hipertiroidismo  y  deja  de  4  a  8  g  de  tejido  tiroideo  para  lograr  
un  estado  eutiroideo.  Se  utiliza  una  tiroidectomía  casi  total  para  tratar  el  carcinoma  y  dejar  menos  de  1  g  de  tejido  
tiroideo  en  el  ligamento  de  Berry  para  evitar  la  lesión  del  NLR.  Esta  operación  tiene  resultados  oncológicos  
equivalentes  a  la  tiroidectomía  total  que  elimina  todo  el  tejido  tiroideo  visible.  Sin  embargo,  la  tiroidectomía  casi  
total  puede  tener  una  tasa  de  complicaciones  más  baja  que  la  tiroidectomía  total.

12.  ¿Cuál  es  la  extensión  mínima  de  la  tiroidectomía  para  un  nódulo  tiroideo  solitario?
A  excepción  de  pequeñas  lesiones  en  el  istmo  tiroideo,  el  procedimiento  mínimo  ante  la  sospecha  de  malignidad  
debe  ser  la  lobectomía,  incluido  el  istmo  (como  biopsia  diagnóstica).  Debe  evitarse  la  enucleación  o  la  
nodulectomía.  Los  nódulos  autónomos  solitarios  deben  tratarse  con  lobectomía.

13.  ¿Cuál  es  el  tratamiento  quirúrgico  del  carcinoma  de  tiroides?
El  carcinoma  de  tiroides  debe  tratarse  con  tiroidectomía  casi  total  o  total,  excepto  en  pacientes  jóvenes  con  
tumores  pequeños  bien  diferenciados  (¼  de  cm),  sin  invasión  extratiroidea  o  metástasis  a  ganglios  linfáticos  
o  sitios  distantes.  En  tales  casos,  la  lobectomía  con  resección  del  istmo  es  la  terapia  adecuada.  La  
tiroidectomía  casi  total  elimina  el  cáncer  multifocal  en  la  tiroides,  permite  el  uso  de  yodo  radiactivo  
posoperatorio  para  el  diagnóstico  y  tratamiento  de  la  enfermedad  metastásica,  disminuye  el  riesgo  de  recurrencia  
locorregional  y  mejora  la  precisión  de  la  tiroglobulina  sérica  como  marcador  de  enfermedad  persistente  o  
recurrente.  Para  el  carcinoma  papilar  de  1  cm,  la  tiroidectomía  total  o  casi  total  mejora  la  supervivencia  global.  
Los  ganglios  linfáticos  cervicales  agrandados  deben  extirparse  y  examinarse  por  sección  congelada.  Si  se  
identifica  cáncer  metastásico,  se  realiza  una  disección  de  cuello.  La  «recogida  de  bayas»  da  como  resultado  
una  mayor  tasa  de  recurrencia  regional  y  debe  evitarse  en  favor  de  las  disecciones  anatómicas.

Debido  a  que  el  cáncer  medular  de  tiroides  no  responde  al  yodo  radioactivo  ni  a  la  levotiroxina,  un  total
se  debe  realizar  tiroidectomía.  Una  disección  central  del  cuello  es  obligatoria  para  evaluar  la  enfermedad  
metastásica.  Si  los  ganglios  centrales  son  positivos  para  cáncer  en  la  sección  congelada,  se  realiza  una  disección  
de  cuello  modificada  ipsilateral.  Puede  observarse  el  cuello  contralateral.
La  cirugía  para  el  carcinoma  anaplásico  es  paliativa  y  por  lo  general  se  limita  a  la  citorreducción  y  la  
traqueotomía  para  aliviar  los  síntomas  compresivos.

14.  ¿Cuál  es  la  incidencia  de  enfermedad  metastásica  en  los  ganglios  linfáticos?
En  el  momento  del  diagnóstico  de  cáncer  de  tiroides  bien  diferenciado  (WDTC;  papilar  y  folicular),  del  20%  al  
50%  de  los  pacientes  tendrán  metástasis  microscópicas  si  se  realiza  una  disección  profiláctica  del  cuello.  Sin  
embargo,  múltiples  estudios  no  logran  mostrar  un  beneficio  significativo  de  la  disección  profiláctica  del  cuello.
Múltiples  autores  abogan  por  una  disección  central  del  cuello  en  el  momento  de  la  tiroidectomía  en  pacientes  con  
WDTC  porque  la  tiroidectomía  implica  operar  dentro  del  cuello  central.  Sin  embargo,  el  riesgo  de  hipoparatiroidismo  
y  lesión  nerviosa  puede  ser  mayor  que  el  de  la  tiroidectomía  total  sola.
En  el  cáncer  medular  de  tiroides,  el  80%  de  los  pacientes  tendrán  ganglios  positivos  en  el  cuello  central  
e  ipsilateral.  Aproximadamente  el  50%  de  los  pacientes  tendrán  enfermedad  ganglionar  en  el  cuello  contralateral.
Debido  a  que  el  MTC  es  insensible  al  yodo  radioactivo  y  a  la  TSH,  la  cirugía  es  la  única  terapia  y  es  
necesaria  la  disección  del  cuello.

15.  ¿Se  puede  hacer  algo  para  identificar  a  los  pacientes  con  enfermedad  ganglionar  antes  de  la
¿operación?
La  ecografía  del  cuello  antes  de  la  tiroidectomía  identifica  enfermedad  ganglionar  no  palpable  en  20%  a  
30%  de  los  pacientes  y  altera  la  operación  realizada  según  el  examen  físico.  La  ecografía  no  es  sensible  en  la  
parte  central  del  cuello  cuando  no  se  ha  extirpado  la  tiroides.  Es  mucho  más  preciso  en  el  cuello  lateral.

16.  Describa  el  riego  arterial  y  el  drenaje  venoso  de  la  tiroides.
El  suministro  de  sangre  a  la  glándula  tiroides  proviene  de  las  arterias  tiroideas  superior  e  inferior.
Ocasionalmente,  una  arteria  imma  tiroidea  en  la  línea  media  surge  del  arco  aórtico.  La  arteria  tiroidea  superior  
es  la  primera  rama  de  la  arteria  carótida  externa.  La  arteria  tiroidea  inferior  surge  del  tronco  tirocervical.
Machine Translated by Google
CAPÍTULO  59  NÓDULOS  TIROIDEOS  Y  CÁNCER  295

Las  tres  venas  principales  son  las  venas  tiroideas  superior,  media  e  inferior.  Las  venas  tiroideas  superior  
y  media  drenan  en  la  vena  yugular  interna  y  la  vena  inferior  drena  en  la  vena  innominada.

17.  Describa  la  anatomía  de  los  nervios  laríngeos  recurrentes.
El  RLN  derecho  surge  del  nervio  vago  y  gira  alrededor  de  la  arteria  subclavia  derecha.  El  nervio  vago  izquierdo  
emite  el  RLN  izquierdo  y  gira  alrededor  de  la  aorta  en  el  ligamento  arterioso.  Los  RLN  discurren  oblicuamente  a  
través  del  cuello,  generalmente  en  el  surco  traqueoesofágico.  En  la  parte  baja  del  cuello,  los  nervios  son  más  
laterales  y  discurren  medialmente  a  medida  que  ascienden.  El  nervio  derecho  corre  más  oblicuo  que  el  izquierdo.  
Ocasionalmente,  el  NLR  puede  ramificarse  antes  de  ingresar  a  la  laringe,  generalmente  en  el  lado  izquierdo.  Las  
fibras  motoras  suelen  estar  en  la  rama  más  medial.  En  el  1%  de  los  casos,  el  NLR  derecho  no  es  recurrente  y  
entra  al  cuello  desde  una  dirección  lateral  y  superior  y  se  asocia  con  una  arteria  subclavia  derecha  aberrante.

18.  ¿Qué  defecto  resulta  de  la  lesión  del  NLR?
La  lesión  de  un  solo  RLN  da  como  resultado  una  cuerda  vocal  paralizada,  lo  que  provoca  una  voz  débil  y  ronca.
Los  pacientes  también  pueden  tener  deglución  anormal  y  problemas  con  la  aspiración.  La  lesión  de  ambos  nervios  
provoca  parálisis  de  ambas  cuerdas  y  obstrucción  del  flujo  de  aire.  Esta  situación  requiere  una  traqueotomía.  La  
lesión  del  NLR  ocurre  en  el  1%  de  las  tiroidectomías.

19.  Describir  la  anatomía  del  nervio  laríngeo  superior  y  el  defecto  que
ocurre  con  su  lesión.
El  nervio  laríngeo  superior  emite  la  rama  externa  del  nervio  laríngeo  superior  (EBSLN),  que  discurre  medial  
a  los  vasos  del  polo  superior  para  entrar  en  el  músculo  cricotiroideo.  Este  nervio  motor  (es  decir,  el  nervio  Amelita  
Galli­Curci)  aumenta  la  tensión  de  las  cuerdas  vocales,  lo  que  permite  notas  altas.  El  nervio  laríngeo  interno  
proporciona  la  inervación  sensorial  a  la  faringe  posterior.  Se  encuentra  superior  al  cartílago  tiroides.  La  lesión  del  
nervio  conduce  a  una  voz  baja  y  débil  que  carece  de  resonancia.  Los  pacientes  también  pueden  tener  problemas  
con  la  aspiración.

20.  ¿Los  pacientes  tienen  cambios  en  la  voz  independientemente  de  la  lesión  de  los  nervios?
En  ausencia  de  lesión  nerviosa,  entre  el  80  %  y  el  85  %  de  los  pacientes  tienen  un  cambio  en  al  menos  un  parámetro  
de  la  voz;  sin  embargo,  solo  entre  el  40  %  y  el  50  %  de  los  pacientes  reportan  una  disfunción  leve  de  la  voz.  Estos  
primeros  síntomas  vocales  se  resuelven  en  la  mayoría  pero  pueden  persistir  en  aproximadamente  el  15%  de  los  pacientes.  
Es  importante  discutir  estos  datos  con  el  paciente  antes  de  la  operación.

21.  ¿Cuál  es  la  otra  complicación  importante  de  la  tiroidectomía?
La  hipocalcemia  temporal  ocurre  en  10%  a  15%  de  los  pacientes,  mientras  que  el  hipoparatiroidismo  
permanente  ocurre  en  3%  de  los  pacientes  que  se  han  sometido  a  tiroidectomías.

22.  ¿Cuáles  son  las  terapias  postoperatorias  para  la  tiroides  bien  diferenciada?
¿carcinoma?
Los  pacientes  seleccionados  deben  ser  tratados  con  yodo  radiactivo  posoperatorio  (I131)  para  la  ablación  del  
tejido  remanente  y  la  enfermedad  distal.  Todos  los  pacientes  con  WDTC  deben  recibir  tratamiento  con  levotiroxina  
(Synthroid)  para  suprimir  los  niveles  séricos  de  TSH.  Esta  terapia  de  tres  componentes  (es  decir,  cirugía,  I131  y  
levotiroxina)  da  como  resultado  la  tasa  de  recurrencia  más  baja.

23.  ¿Cuáles  son  las  indicaciones  de  la  ablación  posoperatoria  con  yodo  radiactivo  (I131) ?
Recomendado  para  todas  las  enfermedades  en  estadios  2  a  4  de  tumores,  ganglios  y  metástasis  (TNM)  y  pacientes  
seleccionados  en  estadio  1  con  factores  de  riesgo.  Los  factores  de  riesgo  incluyen  la  edad  avanzada  (>45  años),  el  género  
masculino,  el  tamaño  del  tumor,  la  invasión  local  directa,  la  diseminación  ganglionar  y  la  enfermedad  a  distancia.  La  ablación  
posoperatoria  y  el  seguimiento  posterior  de  la  enfermedad  persistente  o  recurrente  requieren  estimulación  con  TSH.
La  estimulación  con  TSH  endógena  se  realiza  con  retirada  de  levotiroxina.  La  estimulación  con  TSH  exógena  se  
realiza  con  TSH  humana  recombinante  (rhTSH).
Machine Translated by Google
296  CAPÍTULO  59  NÓDULOS  TIROIDEOS  Y  CÁNCER

24.  ¿Cuál  es  el  grado  adecuado  de  supresión  de  la  hormona  tiroidea  de  TSH?
Los  estudios  retrospectivos  sugieren  que  los  pacientes  de  alto  riesgo  deben  tener  la  TSH  suprimida  a  
<0,1  mIU/ml.  Los  pacientes  de  bajo  riesgo  deben  mantenerse  entre  0,1  y  0,5  mUI/ml  (normal  =  0,5  a  5,0  mUI/
ml).

25.  ¿Cuál  es  el  método  apropiado  para  seguir  a  los  pacientes  después  de  su  curso  inicial?
de  tratamiento?
Los  pacientes  con  WDTC  tienen  un  30%  de  riesgo  de  recurrencia  durante  30  años  de  seguimiento.  Inicialmente,  
los  pacientes  son  seguidos  a  intervalos  de  6  a  12  meses  dependiendo  de  su  estratificación  de  riesgo.  Se  realiza  
un  examen  físico.  Los  niveles  de  TSH,  tiroglobulina  (Tg)  y  anticuerpos  contra  la  tiroglobulina  deben  controlarse  
con  el  paciente  que  recibe  levotiroxina.  Un  nivel  elevado  de  Tg  con  TSH  suprimida  es  preocupante  para  la  
recurrencia.  Si  el  nivel  de  Tg  es  indetectable  durante  la  supresión  de  TSH,  se  realiza  un  nivel  de  Tg  con  
estimulación  de  TSH  (endógena  o  exógena).  Un  aumento  en  el  nivel  de  Tg  con  estimulación  con  TSH  también  
es  motivo  de  preocupación  para  la  recurrencia.  Entre  el  quince  y  el  20  %  de  la  población  tiene  anticuerpos  Tg  
que  interfieren  con  el  ensayo  de  Tg  y  pueden  causar  una  sobremedición  o  una  submedición.  La  ecografía  del  
cuello  ha  reemplazado  en  gran  medida  a  la  ecografía  con  yodo  para  detectar  la  recurrencia.

26.  ¿Cuál  es  el  manejo  adecuado  de  los  pacientes  con  enfermedad  metastásica?
La  enfermedad  local  o  regional  en  el  cuello  se  trata  mediante  reoperación.  La  disección  del  cuello  orientada  
hacia  los  compartimentos  se  recomienda  para  las  metástasis  regionales.  La  enfermedad  a  distancia  debe  
tratarse  con  yodo  radiactivo  si  las  metástasis  captan  yodo.  Las  micrometástasis  pulmonares  deben  tratarse  en  
intervalos  de  6  a  12  meses  mientras  la  enfermedad  continúe  respondiendo.  La  enfermedad  ávida  sin  yodo  
radiactivo  no  se  beneficia  con  el  tratamiento  de  rutina.  En  estos  pacientes,  está  indicado  el  seguimiento  con  
terapia  supresora  de  TSH.  Para  pacientes  seleccionados,  la  metastectomía  o  la  radioterapia  de  haz  externo  
(EBRT)  pueden  proporcionar  un  beneficio  paliativo.  Se  debe  considerar  la  resección  quirúrgica  completa  de  
metástasis  óseas  aisladas.  Las  alternativas,  cuando  las  metástasis  no  están  aisladas,  son  el  tratamiento  con  
yodo  radiactivo  o  la  EBRT.  Los  pacientes  con  enfermedad  progresiva  avanzada  que  no  son  ávidos  de  yodo  
radiactivo  pueden  participar  en  ensayos  clínicos  de  quimioterapia.  Si  no  se  dispone  de  ensayos  clínicos,  la  
quimioterapia  citotóxica  consiste  en  un  régimen  basado  en  doxorrubicina.

SITIOS  WEB

www.acssurgery.com

www.endocrinesurgery.org/
www.tiroides.org/

www.aace.com/

BIBLIOGRAFÍA

1.  Bilimoria  KY,  Bentrem  DJ,  Ko  CY  et  al.:  La  extensión  de  la  cirugía  afecta  la  supervivencia  del  cáncer  papilar  de  tiroides.  ann  surg
246:375­381,  2007.

2.  Brandi  ML,  Gagel  RF,  Angeli  A  et  al.:  Directrices  para  el  diagnóstico  y  tratamiento  de  MEN  tipo  1  y  tipo  2.  J  Clin
Endocrinol  Metab  86:5658­5671,  2001.

3.  Cooper  DS,  Doherty  GM,  Haugen  BR  et  al.:  Pautas  de  manejo  para  pacientes  con  nódulos  tiroideos  y
cáncer  diferenciado  de  tiroides.  Tiroides  16:109­142,  2006.

4.  Haugen  BR,  Ridgway  EC,  McLaughlin  BA  et  al.:  Comparación  clínica  de  gammagrafía  con  yodo  radiactivo  de  cuerpo  entero  y  
tiroglobulina  sérica  después  de  la  estimulación  con  tirotropina  humana  recombinante.  Tiroides  12:37­43,  2002.
Machine Translated by Google
CAPÍTULO  59  NÓDULOS  TIROIDEOS  Y  CÁNCER  297

5.  Ito  Y,  Miyauchi  A:  Disección  de  ganglios  linfáticos  laterales  y  mediastínicos  en  carcinoma  diferenciado  de  tiroides:  
indicaciones,  beneficios  y  riesgos.  World  J  Surg  31:905­915,  2007.

6.  Jonklaas  J,  Sarlis  NJ,  Litofsky  D  et  al.:  Resultados  de  pacientes  con  carcinoma  de  tiroides  diferenciado  después
terapia  inicial.  Tiroides  16:1229­1242,  2006.
7.  Leboulleux  S,  Girard  E,  Rose  M  et  al.:  Criterios  ecográficos  de  malignidad  para  los  ganglios  linfáticos  cervicales  en  pacientes
seguimiento  por  cáncer  diferenciado  de  tiroides.  J  Clin  Endocrinol  Metab  92:3590­3594,  2007.
8.  Lim  CY,  Yun  JS,  Lee  J  et  al .:  Terapia  de  inyección  percutánea  de  etanol  para  la  tiroides  papilar  localmente  recurrente
carcinoma.  Tiroides  17:347­350,  2007.
9.  Mazzaferri  EL,  Robbins  RJ,  Spencer  CA  et  al.:  Un  informe  de  consenso  sobre  el  papel  de  la  tiroglobulina  sérica  como  
método  de  seguimiento  para  pacientes  de  bajo  riesgo  con  carcinoma  papilar  de  tiroides.  J  Clin  Endocrinol  Metab  
88:1433­1441,  2003.

10.  Mittendorf  EA,  Wang  X,  Perrier  ND  et  al.:  Seguimiento  de  pacientes  con  cáncer  papilar  de  tiroides:  en  busca  del
algoritmo  óptimo.  J  Am  Coll  Surg  205:239­247,  2007.

11.  Pacini  F,  Molinaro  E,  Castagna  MG  et  al .:  Tiroglobulina  sérica  estimulada  con  tirotropina  humana  recombinante
combinado  con  la  ecografía  del  cuello  tiene  la  mayor  sensibilidad  en  el  seguimiento  del  carcinoma  diferenciado  de  tiroides.
J  Clin  Endocrinol  Metab  88:3668­3673,  2003.

12.  Sawka  AM,  Thephamongkhol  K,  Brouwers  M  et  al.:  Clinical  review  170:  Una  revisión  sistemática  y  metanálisis  de  la  efectividad  
de  la  ablación  con  restos  de  yodo  radiactivo  para  el  cáncer  de  tiroides  bien  diferenciado.  J  Clin  Endocrinol  Metab  
89:3668­3676,  2004.

13.  Stojadinovic  A,  Shaha  AR,  Orlikoff  RF  et  al .:  Evaluación  prospectiva  de  la  voz  funcional  en  pacientes  sometidos  a  cirugía  de  
tiroides.  Ann  Surg  236(6):823­832,  2002.
14.  Stulak  JM,  Grant  CS,  Farley  DR  et  al.:  Valor  de  la  ultrasonografía  preoperatoria  en  el  manejo  quirúrgico  de
Cáncer  papilar  de  tiroides  inicial  y  reoperatorio.  Arch  Surg  141:489­494,  2006.
Machine Translated by Google

HIPERTENSIÓN  QUIRÚRGICA
CAPITULO  
60

Thomas  A.  Whitehill,  MD  y  Joel  Baumgartner,  MD

1.  ¿Cuáles  son  las  causas  de  hipertensión  corregibles  quirúrgicamente?
Hipertensión  renovascular,  feocromocitoma,  síndrome  de  Cushing,  hiperaldosteronismo  
primario  (síndrome  de  Conn),  coartación  de  la  aorta  y  enfermedad  del  parénquima  renal  
unilateral.  La  hipertensión  quirúrgica  representa  el  5%  de  todos  los  pacientes  hipertensos.

2.  ¿Qué  forma  de  hipertensión  quirúrgica  es  más  común?
La  hipertensión  renovascular  es  la  más  frecuente.  Aunque  la  frecuencia  general  de  hipertensión  
renovascular  entre  pacientes  con  presión  arterial  diastólica  (PAD)  elevada  es  de  alrededor  del  3%,  la  
hipertensión  diastólica  moderada  o  grave  puede  ser  causada  por  enfermedad  oclusiva  de  la  arteria  renal  
hasta  en  el  25%  de  los  casos.  El  feocromocitoma,  el  hiperaldosteronismo,  la  enfermedad  de  Cushing  y  la  
coartación  de  la  aorta  se  encuentran  cada  uno  en  sólo  el  0,1%  de  todos  los  pacientes  hipertensos.

3.  ¿Cuáles  son  las  causas  más  frecuentes  de  hipertensión  renovascular?
La  aterosclerosis  causa  el  70%  de  los  casos;  Afecta  a  los  hombres  con  el  doble  de  frecuencia  que  a  las  
mujeres.  La  segunda  causa  más  común  es  la  displasia  fibromuscular  (25%).  De  los  muchos  subtipos  
patológicos,  el  más  común  es  la  fibrodisplasia  medial  (85%);  invariablemente  afecta  a  las  mujeres.  La  
última  es  la  estenosis  de  la  arteria  renal  del  desarrollo  (10%),  que  a  menudo  se  asocia  con  neurofibromatosis  
y  coartación  aórtica  abdominal.

4.  ¿Qué  criterios  clínicos  respaldan  la  realización  de  estudios  de  investigación  para  casos  sospechosos?
hipertensión  renovascular?
Aunque  ninguna  característica  clínica  es  patognomónica  de  la  hipertensión  renovascular,  los  
siguientes  hallazgos  sugieren  fuertemente  la  presencia  de  una  lesión  estenótica  subyacente  de  la  arteria  
renal:     Hipertensión  en  personas  muy  jóvenes  o  en  mujeres  menores  de  50  años.
  Inicio  rápido  de  hipertensión  grave  después  de  los  50  años.
  Hipertensión  refractaria  a  regímenes  de  tres  fármacos.
&  Presentación  inicial  con  PAD  >  115  mm  Hg  o  empeoramiento  repentino  de  la  supuesta  preexistencia
hipertensión.
  Hipertensión  acelerada  o  maligna.
  Deterioro  de  la  función  renal  después  del  inicio  de  agentes  antihipertensivos,  especialmente
Inhibidores  de  la  enzima  convertidora  de  angiotensina  (ECA).
  Soplos  sistólicos  o  diastólicos  en  el  abdomen  superior  o  en  el  flanco.

5.  ¿Qué  es  el  sistema  renina­angiotensina­aldosterona?
La  renina  se  libera  del  aparato  yuxtaglomerular  del  riñón  en  respuesta  a  cambios  en  la  presión  de  perfusión  
arteriolar  aferente  de  la  corteza  renal.  La  renina  actúa  localmente  y  en  la  circulación  sistémica  sobre  el  
sustrato  de  la  renina  (angiotensinógeno),  una  globulina  α2  no  vasoactiva  que  se  produce  en  el  hígado  para  
formar  angiotensina  I.  La  angiotensina  I  sufre  escisión  enzimática  por  la  ECA  en  la  circulación  pulmonar  
para  producir  angiotensina  II,  un  potente  vasopresor  responsable  de  el  elemento  vasoconstrictor  de  la  
hipertensión  renovascular.  La  angiotensina  II  aumenta  la  producción  de  aldosterona  en  las  glándulas  
suprarrenales  con  la  subsiguiente  retención  de  sodio  y  agua;  este  proceso  establece  el  elemento  de  
volumen  de  la  hipertensión  renovascular.

298
Machine Translated by Google
CAPÍTULO  60  HIPERTENSIÓN  QUIRÚRGICA  299

6.  ¿Cómo  funcionan  los  inhibidores  de  la  ECA?
La  inhibición  directa  de  la  ACE  disminuye  las  concentraciones  de  angiotensina  II,  lo  que  conduce  a  una  disminución  de  la  
actividad  vasopresora  y  una  disminución  de  la  secreción  de  aldosterona.  La  eliminación  de  la  retroalimentación  negativa  de  la  
angiotensina  II  sobre  la  secreción  de  renina  conduce  a  un  aumento  de  la  actividad  de  la  renina  plasmática.

7.  ¿Los  pacientes  con  hipertensión  renovascular  deben  recibir  tratamiento  médico  o
¿quirúrgicamente?
Aunque  no  se  han  publicado  grandes  estudios  prospectivos  aleatorizados  que  comparen  el  tratamiento  
farmacológico  y  el  intervencionista,  la  mayoría  de  los  médicos  prefieren  el  tratamiento  quirúrgico  y  la  angioplastia  
renal  transluminal  percutánea  (PTRA)  al  tratamiento  farmacológico.  La  clave  es  el  reconocimiento  temprano  del  
problema.

8.  ¿Cuándo  se  debe  tratar  a  los  pacientes  con  hipertensión  renovascular  con
PTRA?
Las  indicaciones  claras  para  PTRA  incluyen  lesiones  ateroscleróticas  no  orificios  y  lesiones  fibrodisplásicas  mediales  
limitadas  a  la  arteria  renal  principal.

9.  ¿Qué  hallazgos  de  la  anamnesis  y  la  exploración  física  deben  hacer  sospechar  un  feocromocitoma?

Los  feocromocitomas  son  tumores  principalmente  de  la  médula  suprarrenal  y  de  las  células  paraganglionares  
extraadrenales.  Aproximadamente  el  90%  de  ellos  se  encuentran  dentro  de  la  glándula  suprarrenal  y  el  10%  restante  
se  encuentran  dispersos  a  lo  largo  de  la  cadena  simpática  paravertebral  abdominal  o  en  ganglios  ubicados  de  
manera  remota  (p.  ej.,  vejiga  urinaria,  nervios  pélvicos).  Los  tumores  se  clasifican  como  funcionantes  cuando  
producen  catecolaminas,  siempre  de  forma  autónoma  y  habitualmente  en  gran  exceso.  Los  efectos  clínicos  
predecibles  del  aumento  de  la  secreción  de  catecolaminas  endógenas  son  hipertensión  sostenida  con  episodios  de  
aumento  de  la  presión  arterial,  taquicardia,  dolor  de  cabeza,  palpitaciones  o  sofocos.
En  raras  ocasiones,  los  pacientes  mantienen  periodos  de  normotensión  con  episodios  paroxísticos  de  hipertensión  
poco  frecuentes  e  impredecibles.

10.  ¿Cómo  se  diagnostica  el  feocromocitoma?
El  diagnóstico  se  confirma  mejor  mediante  la  recolección  de  orina  de  24  horas  para  detectar  
catecolaminas,  metanefrinas  y  ácido  vanililmandélico  excretados.  Todavía  se  debate  cuál  es  la  mejor  
prueba  individual  para  confirmar  el  diagnóstico  de  feocromocitoma;  algunos  creen  que  el  nivel  de  metanefrina  es  
el  más  preciso  (85%).  Las  catecolaminas  plasmáticas  también  son  una  prueba  específica,  pero  dada  la  variabilidad  
de  los  resultados  en  pacientes  individuales  y  en  muchos  ensayos,  el  enfoque  actual  debe  continuar  enfatizando  
el  uso  de  catecolaminas  urinarias.  El  ochenta  por  ciento  de  los  pacientes  con  feocromocitoma  tienen  al  menos  un  
metabolito  urinario  mayor  que  el  doble  del  valor  normal.  El  diagnóstico  de  feocromocitoma  debe  ir  seguido  de  
estudios  para  localizar  el  tumor.

11.  ¿Cuál  es  la  mejor  prueba  para  localizar  un  feocromocitoma?
La  exploración  por  tomografía  computarizada  (TC),  la  resonancia  magnética  nuclear  (RMN)  y  la  exploración  
con  I131­  metaiodobencilguanidina  (MIBG)  son  tres  modalidades  de  imagenología  disponibles.  Debido  a  que  97%  
de  los  feocromocitomas  son  intraabdominales  y  casi  siempre  >2  cm,  una  tomografía  computarizada  abdominal  
(cortes  delgados  a  través  del  lecho  suprarrenal  desde  el  diafragma  hasta  la  bifurcación  aórtica)  rara  vez  pasa  por  
alto  una  lesión  y  brinda  buenos  detalles  anatómicos.  La  resonancia  magnética  se  ha  utilizado  cada  vez  más  
porque  el  90%  de  los  feocromocitomas  son  característicamente  brillantes  en  las  imágenes  ponderadas  en  T2.  
MIBG  se  utiliza  mejor  en  pacientes  con  sospecha  de  feocromocitoma  extraadrenal,  multifocal  o  recurrente.  Es  
menos  sensible  que  la  TC  y  la  RM.  Es  mejor  reservar  MIBG  para  pacientes  con  mayor  riesgo  de  tumores  múltiples  
o  extraadrenales  y  feocromocitoma  maligno.
Machine Translated by Google
300  CAPÍTULO  60  HIPERTENSIÓN  QUIRÚRGICA

12.  Describir  el  tratamiento  antihipertensivo  inmediato  en  pacientes  con
feocromocitoma.
La  hipertensión  por  feocromocitoma  es  causada  por  la  activación  de  los  receptores  a1  del  músculo  liso  
vascular,  lo  que  da  como  resultado  vasoconstricción.  Por  lo  tanto,  el  mejor  tratamiento  agudo  es  la  administración  
intravenosa  (IV)  de  un  antagonista  α1  o  un  bloqueador  α;  las  opciones  incluyen  fenoxibenzamina,  prazosina  o  
terazosina.  Los  agentes  de  segunda  línea  incluyen  bloqueadores  de  los  canales  de  calcio  e  inhibidores  de  la  ECA.
El  bloqueo  antiarrítmico  β  debe  evitarse  inicialmente  porque  estos  agentes  provocan  tanto  la  estimulación  
de  los  receptores  α1  periféricos  sin  oposición  como  la  disminución  del  gasto  cardíaco  (CO;  secundario  a  la  alta  
resistencia  vascular).  La  insuficiencia  cardíaca  congestiva  puede  precipitarse  bloqueando  el  corazón  antes  de  
disminuir  la  presión  arterial.

13.  ¿Cómo  se  diagnostica  el  hiperaldosteronismo  primario  (síndrome  de  Conn)?
El  síndrome  de  Conn,  que  resulta  de  la  hipersecreción  autónoma  de  mineralocorticoides,  se  caracteriza  
por  hipertensión,  hipopotasemia,  hipernatremia,  alcalosis  metabólica  y  debilidad  muscular  y  parálisis  periódicas,  
a  menudo  causadas  por  un  adenoma  secretor  de  aldosterona.  El  síndrome  ahora  se  identifica  por  los  hallazgos  
combinados  de  hipopotasemia,  actividad  de  renina  plasmática  suprimida  a  pesar  de  la  restricción  de  sodio  y  
niveles  altos  de  aldosterona  urinaria  y  plasmática  después  de  la  reposición  de  sodio  en  pacientes  hipertensos.

14.  ¿Por  qué  el  síndrome  de  Cushing  o  la  enfermedad  de  Cushing  causan  hipertensión?
Ambos  causan  hipercortisolismo  o  cantidades  excesivas  de  glucocorticoides.  En  el  sistema  cardiovascular,  los  
glucocorticoides  producen  un  aumento  de  los  efectos  cronotrópicos  e  inotrópicos  cardíacos,  junto  con  un  
aumento  de  la  resistencia  vascular  periférica.  Los  receptores  en  los  túbulos  renales  distales  responden  a  los  
glucocorticoides  aumentando  la  reabsorción  tubular  de  sodio.  Estos  receptores  pertenecen  a  una  clase  diferente  
de  los  receptores  que  median  las  acciones  más  potentes  de  la  aldosterona.

15.  ¿Qué  hallazgos  sugieren  coartación  aórtica?
Presión  arterial  más  baja  en  las  piernas  que  en  los  brazos  y  pulsos  femorales  disminuidos  o  ausentes.
Las  muescas  en  las  costillas  pueden  ser  evidentes  en  la  radiografía  de  tórax  en  pacientes  con  
coartación  hemodinámicamente  significativa  de  larga  evolución.  Se  pueden  escuchar  soplos  sobre  el  tórax  o  la  pared  abdominal.
Los  adultos  pueden  incluso  desarrollar  insuficiencia  cardíaca  congestiva  e  insuficiencia  renal.

16.  ¿Cómo  provoca  hipertensión  la  coartación  aórtica?
No  se  ha  identificado  una  causa  única.  La  obstrucción  mecánica  de  la  eyección  ventricular  es  un  
componente  que  conduce  a  la  hipertensión  de  las  extremidades  superiores.  La  hipoperfusión  de  los  riñones  
con  la  activación  resultante  del  RAAS  probablemente  contribuya.  También  se  han  implicado  la  distensibilidad  
aórtica  anormal,  la  capacidad  variable  de  los  vasos  colaterales  y  la  configuración  anormal  de  los  barorreceptores.

PUNTOS  CLAVE:  HIPERTENSIÓN  QUIRÚRGICA

1.  Las  causas  de  hipertensión  corregible  quirúrgicamente  incluyen  hipertensión  renovascular,
feocromocitoma,  síndrome  de  Cushing,  síndrome  de  Conn,  coartación  de  la  aorta  y  enfermedad  del  
parénquima  renal  unilateral.

2.  La  causa  más  frecuente  de  hipertensión  renovascular  es  la  aterosclerosis.

3.  El  diagnóstico  de  feocromocitoma  se  confirma  mediante  la  recolección  de  orina  de  24  horas  para  detectar  
catecolaminas,  metanefrinas  y  ácido  vanililmandélico  excretados.

4.  El  síndrome  de  Conn  se  caracteriza  por  hipertensión,  hipopotasemia,  hipernatremia,  alteraciones  metabólicas
alcalosis  y  debilidad  muscular  periódica  y  parálisis.
Machine Translated by Google
CAPÍTULO  60  HIPERTENSIÓN  QUIRÚRGICA  301

BIBLIOGRAFÍA

1.  Bloch  MJ,  Basile  J:  Diagnóstico  y  tratamiento  de  la  enfermedad  renovascular  y  la  hipertensión  renovascular.  J  clin
Hipertensión  (Greenwich)  9:381,  2007.
2.  Coen  G,  Calabria  S,  Lai  S  et  al.:  Enfermedad  renal  isquémica  aterosclerótica:  diagnóstico  y  prevalencia  en  un
población  anciana  hipertensa  y/o  urémica.  BMC  Nephrol  4:2,  2003.

3.  Hansen  KJ,  Deitch  JS,  Oskin  TC  et  al.:  Reparación  de  la  arteria  renal:  consecuencias  de  los  fracasos  operativos.  ann  surg
277:678­690,  1998.

4.  Kebebew  E,  Duh  QY:  feocromocitoma  benigno  y  maligno:  diagnóstico,  tratamiento  y  seguimiento.  Surg  Oncol  Clin  North  Am  
7:765,  1998.

5.  Mittendorf  EA,  Evans  DB,  Lee  JE  et  al.:  Feocromocitoma:  avances  en  genética,  diagnóstico,  localización  y
tratamiento.  Hematol  Oncol  Clin  North  Am  21:509,  2007.

6.  Lairmore  TC,  Ball  DW,  Baylin  SB  et  al.:  Manejo  de  feocromocitomas  en  pacientes  con  síndromes  de  neoplasia  endocrina  múltiple  
tipo  2.  Ann  Surg  217:595,  1993.

7.  Rossi  GP,  Pessina  AC,  Heagerty  AM:  Aldosteronismo  primario:  una  actualización  sobre  detección,  diagnóstico  y  tratamiento.
J  Hypertens  26:613,  2008.

8.  Nicholson  T:  Angiografía  por  resonancia  magnética  para  el  diagnóstico  de  estenosis  de  la  arteria  renal.  Clin  Radiol  58:257,
2003.

9.  Oskin  TC,  Hansen  KJ,  Deitch  JS  et  al.:  Oclusión  crónica  de  la  arteria  renal:  nefrectomía  versus  revascularización.
J  Vasc  Surg  29:140,  1999.

10.  Palmaz  JC:  Estado  actual  del  intervencionismo  vascular  en  la  nefropatía  isquémica.  J  Vasc  Interv  Radiol  9:439,
1998.

11.  Stanley  JC:  Tratamiento  quirúrgico  de  la  hipertensión  renovascular.  Am  J  Surg  174:102,  1997.

12.  Wong  JM,  Hansen  KJ,  Oskin  TC  et  al .:  Cirugía  después  de  una  angioplastia  percutánea  de  la  arteria  renal  fallida.  J  Vasc  Cirugía
30:468,  1999.
Machine Translated by Google

LAPAROSCÓPICA  SUPRARRENAL
ADRENALECTOMÍA  INCIDENTALOMA
CAPITULO  
61

Janeen  R.  Jordan,  MD  y  Robert  C.  McIntyre,  Jr.,  MD

1.  ¿Cuáles  son  la  anatomía  y  los  productos  secretores  de  la  glándula  suprarrenal?
Hay  dos  glándulas  suprarrenales,  cada  una  ubicada  en  el  retroperitoneo  superior  a  los  riñones.
El  suministro  de  sangre  consta  de  tres  arterias:  la  arteria  suprarrenal  superior  que  es  una  rama  de  la  arteria  
frénica  inferior;  la  arteria  suprarrenal  media  que  es  una  rama  de  la  aorta  abdominal;  y  la  arteria  suprarrenal  
inferior  que  es  una  rama  de  la  arteria  renal.  La  vena  central  principal  de  la  derecha  suele  salir  del  tercio  superior  
de  la  glándula  y  drena  directamente  a  la  vena  cava.  La  vena  central  izquierda  es  más  larga  y  drena  a  la  vena  
renal  izquierda.  Además  de  las  venas  centrales,  una  serie  de  pequeñas  venas  son  paralelas  a  las  arterias.

La  corteza  tiene  tres  zonas  distintas:  la  zona  glomerulosa  adyacente  a  la  cápsula  externa  donde  se  
produce  la  aldosterona;  la  zona  fasciculada  que  produce  glucocorticoides  (cortisol)  y  algunos  esteroides  sexuales;  
y  la  zona  reticularis  que  está  adyacente  a  la  médula  y  produce  cortisol,  andrógenos  y  estrógenos.

y  sal  de  Glomerulsa  (aldosterona)
&  Fasiculata  Azúcar  (Cortisol)
&  Reticularis  Sex  (andrógenos  y  estrógenos)
La  médula  se  deriva  de  las  células  de  la  cresta  neural,  actúa  como  un  ganglio  simpático  y  secreta  
catecolaminas,  específicamente,  noradrenalina,  adrenalina  y  dopamina.

2.  ¿Qué  preguntas  deben  considerarse  cuando  se  identifica  un  tumor  suprarrenal?
¿El  tumor  es  funcional?
¿El  tumor  es  benigno  o  maligno?
¿El  tumor  es  primario  o  metastásico?
¿El  tumor  es  cortical  o  medular?

3.  ¿Cuál  es  la  incidencia  de  los  tumores  suprarrenales  incidentales?
Un  tumor  suprarrenal  incidental  (incidentaloma)  es  un  tumor  suprarrenal  clínicamente  inaparente  que  se  descubre  
mediante  imágenes  realizadas  para  otra  indicación.  Estos  tumores  se  encuentran  en  el  1,4%  al  9%  de  los  
pacientes  en  series  de  autopsias  y  en  el  0,4%  al  4%  de  los  pacientes  sometidos  a  una  tomografía  computarizada  
(TC)  del  abdomen.  La  incidencia  aumenta  con  la  edad,  la  obesidad  y  la  hipertensión.

4.  ¿Cuál  es  el  diagnóstico  diferencial  de  un  tumor  suprarrenal  incidental?
El  diagnóstico  diferencial  de  un  incidentaloma  suprarrenal  con  la  distribución  informada  de  múltiples  series  
es  el  siguiente:  corteza  suprarrenal  y  adenoma  no  funcionante  36%  a  94%.

  Adenoma  productor  de  cortisol  (CPA)  2%  a  15%.
  Adenoma  productor  de  aldosterona  (APA)  0%  a  2%.
  Cáncer  de  corteza  suprarrenal  0%  a  25%.
Médula  suprarrenal  
y  feocromocitoma  0%  a  11%.

302
Machine Translated by Google
CAPÍTULO  61  ADRENALECTOMÍA  LAPAROSCÓPICA  SUPRARRENAL  INCIDENTALOMA  303

Metástasis  
y  sin  antecedentes  de  cáncer  0%  a  21%.
  Antecedentes  de  cáncer  del  32  %  al  73  %.
Otros  
y  Quistes  4%  a  22%.
&  Mielipomas  7%  a  15%.
&  Hematoma  0%  a  4%.
&  Ganglioneuroma  0%  a  6%.
&  Granulomas  0%  a  5%.

5.  ¿Cuál  es  la  función  de  la  aldosterona?
La  aldosterona  es  el  principal  mineralocorticoide.  Aumenta  la  reabsorción  de  sodio  (Na)  y  agua  por  el  riñón  a  
cambio  de  potasio  o  hidrógeno.  La  liberación  de  aldosterona  es  estimulada  por  la  angiotensina  II,  que  se  deriva  
de  la  conversión  mediada  por  renina  del  angiotensinógeno  en  angiotensina  I  que  se  convierte  en  angiotensina  II  
por  la  enzima  convertidora  de  angiotensina  (ECA).  La  liberación  de  renina  del  riñón  es  estimulada  por  la  hipovolemia,  
la  estimulación  β­adrenérgica  y  las  prostaglandinas.  La  hiponatremia  y  la  hiperpotasemia  estimulan  directamente  la  
secreción  de  aldosterona.
La  secreción  de  aldosterona  también  es  estimulada  por  la  hormona  adrenocorticotrópica  (ACTH).  Debido  a  que  la  
ACTH  tiene  una  variación  diurna,  los  niveles  de  aldosterona  y  cortisol  varían  según  la  hora  del  día.

6.  ¿Qué  es  el  hiperaldosteronismo  primario?
El  hiperaldosteronismo  primario  (hiperaldo),  también  conocido  como  enfermedad  de  Conn,  es  un  síndrome  clínico  
caracterizado  por  hipertensión  e  hipopotasemia  (aunque  el  20%  de  los  pacientes  pueden  tener  niveles  normales  de  
potasio  sérico).  Cualquier  paciente  con  hipertensión  de  aparición  temprana  que  sea  difícil  de  controlar  o  que  esté  
asociada  con  hipopotasemia  debe  someterse  a  exámenes  de  detección.  Entre  el  0,05%  y  el  2,0%  de  la  hipertensión  
es  causada  por  hiperaldosteronismo.  El  hiperaldo  primario  es  causado  por  APA  (65%  de  los  casos)  o  hiperplasia  
suprarrenal  primaria.  Otras  entidades  clínicas  menos  comunes  que  comprenden  hiperaldo  primario  incluyen  
carcinoma  suprarrenal  idiopático  y  aldosteronismo  remediable  con  glucocorticoides.

7.  ¿Cómo  se  detecta  el  hiperaldosteronismo?
El  diagnóstico  se  realiza  en  presencia  de  un  nivel  de  aldosterona  plasmática  (PA)  inapropiadamente  elevado  con  
una  actividad  de  renina  plasmática  (PRA)  suprimida.  Los  pacientes  deben  estar  sin  diuréticos,  bloqueadores  beta  e  
inhibidores  de  la  ECA  durante  2  semanas.  Deben  consumir  al  menos  150  mEq  de  Na  por  día  y  se  les  dan  
suplementos  de  potasio.  Se  recolectan  muestras  de  sangre  en  posición  vertical  y  se  sugiere  una  relación  PA/PRA  
>20.  Un  nivel  de  aldosterona  en  orina  de  24  horas  de  20  mg/día  también  es  sugestivo.
Para  confirmar  el  diagnóstico,  a  menudo  se  realiza  una  prueba  de  carga  de  solución  salina.  Un  paciente  
recibe  2  L  de  solución  salina  durante  4  horas.  Se  obtienen  PA  basal  y  de  4  horas,  PRA  y  18­hidroxicorticosterona.  
La  solución  salina  debería  suprimir  los  niveles  de  aldosterona,  pero  los  pacientes  con  hiperaldo  primario  tendrán  
PRA  indetectable  con  niveles  de  PA  >15  ng/dl  antes  y  después  de  la  carga.

8.  ¿Cómo  imagina  al  paciente  con  hiperaldosteronismo  primario?
La  tomografía  computarizada  abdominal  identificará  lesiones  >  5  mm.  La  resonancia  magnética  nuclear  (RMN)  es  
igual  de  precisa  pero  no  brinda  información  adicional  y  es  más  costosa.  Las  imágenes  anatómicas  no  pueden  
diferenciar  la  APA  de  la  hiperplasia.  Por  esa  razón,  los  pacientes  deben  someterse  a  un  muestreo  de  la  vena  
suprarrenal  (AVS)  que  puede  influir  en  el  manejo  hasta  en  el  25%  de  los  pacientes.  La  proporción  de  aldosterona  en  
el  lado  afectado  a  la  glándula  no  afectada  es  superior  a  10:1.  También  se  mide  el  cortisol  para  confirmar  que  las  
muestras  son,  de  hecho,  de  las  venas  suprarrenales.  La  prueba  a  menudo  se  realiza  con  y  sin  estimulación  con  
ACTH.

9.  ¿Cuál  es  el  tratamiento  del  hiperaldosteronismo  primario?
Los  pacientes  con  APA  deben  someterse  a  adrenalectomía  unilateral.  Si  AVS  revela  hiperplasia,  la  
espironolactona  (50  a  200  mg  dos  veces  al  día)  es  el  agente  de  elección.  La  amilorida  es  una  alternativa  y  se  informa  
cierto  éxito  con  los  bloqueadores  de  los  canales  de  calcio  y  los  inhibidores  de  la  ACE.
Machine Translated by Google
304  CAPÍTULO  61  ADRENALECTOMÍA  LAPAROSCÓPICA  SUPRARRENAL  INCIDENTALOMA

10.  ¿Cuál  es  el  resultado  de  la  suprarrenalectomía  unilateral  por  AAF?
Entre  el  70  y  el  85  %  de  los  pacientes  se  curarán  de  la  hipertensión  o  tendrán  una  mejoría  significativa  y  necesitarán  
menos  tratamiento  farmacológico.  La  hipopotasemia  se  cura  en  >90%  de  los  casos.  Los  factores  que  sugieren  un  
resultado  favorable  incluyen  edad  joven,  menor  duración  de  la  hipertensión,  hipertensión  menos  severa,  mayor  
depuración  de  creatinina.

11.  ¿Cuáles  son  las  características  clínicas  comunes  del  feocromocitoma?
Los  feocromocitomas  son  tumores  de  la  médula  suprarrenal  que  producen  catecolaminas,  incluidas  la  norepinefrina,  
la  epinefrina  y  la  dopamina.  Las  cinco  P  del  paroxismo  son:  &  Presión  (presión  arterial  alta).

&  Dolor  (dolores  de  cabeza).
&  Transpiración  (profusa).
y  palpitaciones.
y  palidez.

12.  ¿Qué  es  la  «regla  de  los  10»  en  relación  con  el  feocromocitoma?
Los  siete  10  del  feocromocitoma  son:
&  10%  son  malignos.  &  
10%  son  extra  suprarrenales.  
&  10%  son  múltiples  o  bilaterales.  &  
10%  surgen  en  la  infancia.  &  10%  son  
familiares.  &  10%  descubierto  
incidentalmente.  &  10%  recurre  
después  de  la  resección.

13.  ¿Qué  trastornos  neuroendocrinos  se  asocian  con  los  feocromocitomas?
  Neoplasia  endocrina  múltiple  (NEM)  IIA:  asociada  con  mutación  del  protooncogén  RET.  Asociado  con  
hiperparatiroidismo  (HPT)  y  cáncer  medular  de  tiroides.
(3  C's:  catecolaminas,  calcio,  calcitonina).
  MEN  IIB:  también  asociado  con  mutación  del  protooncogén  RET.  Asociado  con
neurogangliomas  y  cáncer  medular  de  tiroides.
&  Enfermedad  de  Von  Hippel  Lindau  (VHL):  causada  por  mutaciones  del  gen  supresor  de  tumores  VHL  en  el  
cromosoma  3.  Los  feocromocitomas  se  acompañan  de  angiomatosis,  hemangioblastomas,  carcinoma  o  
tumores  de  células  renales  o  quistes  de  páncreas.

14.  ¿Cuál  es  el  estudio  de  un  feocromocitoma?
La  detección  se  realiza  en  pacientes  con  "episodios",  hipertensión  resistente,  trastornos  familiares  (MEN  
II,  VHL,  feocromocitoma  familiar)  e  incidentalomas.  Los  estudios  de  cribado  más  sensibles  son  las  metanefrinas  y  
las  catecolaminas  en  orina  de  24  horas.  Las  metanefrinas  totales  suelen  ser  >1  000  mg,  la  normetanefrina  >400  mg  
y  las  catecolaminas  son  >2  veces  lo  normal.
La  sensibilidad  es  del  98%  en  casos  esporádicos  y  del  90%  incluyendo  pacientes  sindrómicos.  La  especificidad  es  
del  98%.  El  entusiasmo  reciente  por  las  metanefrinas  libres  de  plasma  fraccionadas  es  el  resultado  de  la  alta  
sensibilidad  del  97  %  al  100  %,  pero  esta  prueba  tiene  una  especificidad  más  baja  (85  %  a  89  %)  y  solo  debe  
realizarse  en  pacientes  de  alto  riesgo  (estudios  de  orina  no  concluyentes). ,  masa  suprarrenal  vascular,  
antecedentes  familiares,  MEN  II,  BVS).
Las  imágenes  no  deben  realizarse  hasta  que  la  prueba  bioquímica  confirme  el  diagnóstico.  El  90%  se  encuentran  
en  las  suprarrenales,  el  98%  en  el  abdomen  y,  por  lo  general,  no  son  difíciles  de  localizar.  Se  prefiere  la  RM  a  la  
TC.  Si  se  encuentra  un  feocromocitoma  suprarrenal  unilateral,  entonces  no  es  necesaria  la  123I­
methaiodobenzylguanidine  (MIBG).  Sin  embargo,  si  se  encuentra  un  paraganglioma,  se  realiza  MIBG  para  evaluar  
tumores  adicionales  y  enfermedades  malignas.  En  pacientes  con  imágenes  abdominales  negativas,  considere  la  TC  
de  cabeza,  cuello  y  tórax.
Machine Translated by Google
CAPÍTULO  61  ADRENALECTOMÍA  LAPAROSCÓPICA  SUPRARRENAL  INCIDENTALOMA  305

15.  ¿Cómo  se  debe  preparar  para  la  cirugía  a  un  paciente  con  feocromocitoma?  El  bloqueo  a­adrenérgico  debe  
iniciarse  al  menos  7  a  10  días  antes  de  la  operación  para  controlar  la  presión  arterial  y  expandir  el  volumen  
sanguíneo  contraído.  La  fenoxibenzamina  es  el  agente  preferido  comenzando  con  10  mg  todos  los  días,  dos  
veces  al  día  y  aumentando  de  10  a  20  mg  en  dosis  divididas  cada  2  a  3  días  hasta  el  control  de  la  presión  arterial  
y  los  episodios.  Se  debe  advertir  a  los  pacientes  acerca  de  la  ortostasis.  El  bloqueo  β  nunca  debe  iniciarse  antes  
del  bloqueo  α  y  está  indicado  para  la  taquicardia  asociada  con  el  bloqueo  α.  La  hipotensión  intraoperatoria  puede  
controlarse  con  nitroprusiato,  fentolamina  o  nicardipina.

16.  ¿Qué  es  el  Síndrome  de  Cushing?
El  Síndrome  de  Cushing  es  un  conjunto  de  manifestaciones  clínicas  causadas  por  un  exceso  de  cortisol.
Las  características  incluyen:

  Obesidad,  especialmente  troncal,  facies  de  luna  llena  y  joroba  de  búfalo.
  Piel  fina,  estrías,  hirsutismo.
&  Debilidad  muscular  y  desgaste.
e  hipertensión.
&  Irregularidad  menstrual.
y  osteoporosis.
y  pancreatitis.
&  Mayor  riesgo  de  infección.
y  depresión.

17.  ¿En  qué  se  diferencia  el  síndrome  de  Cushing  de  la  enfermedad  de  Cushing?
La  enfermedad  de  Cushing  es  el  resultado  de  un  tumor  hipofisario  que  libera  ACTH  que  estimula  la  liberación  
de  cortisol  de  la  glándula  suprarrenal.  El  síndrome  de  Cushing  es  la  manifestación  clínica  del  exceso  de  cortisol  
a  pesar  de  la  etiología.

18.  ¿Cuáles  son  las  causas  del  síndrome  de  Cushing?
Administración  de  esteroides  exógenos.
Síndrome  de  Cushing  hipofisario.
Producción  ectópica  de  ACTH.
CPA  suprarrenal.
Carcinomas  suprarrenales.

Hiperplasia  suprarrenal  micronodular  y  macronodular,  hiperplasia  suprarrenal.

19.  ¿Cómo  se  pueden  clasificar  las  causas  del  síndrome  de  Cushing  y  cuál  es  la  más
¿común?
En  general,  el  síndrome  de  Cushing  exógeno  es  el  más  común.  De  lo  contrario,  el  síndrome  de  Cushing  se  puede  
clasificar  como  dependiente  de  ACTH  (80  %)  e  independiente  de  ACTH  (20  %):  Independiente  de  ACTH  (20  %)
Dependiente  de  ACTH  (80%)
Adenoma  pituitario  (85%) Adenoma  suprarrenal  (>50%)
ACTH  ectópica  (15%) Carcinoma  suprarrenal  (<50%)
Hiperplasia  suprarrenal
Exógeno  (común)

20.  ¿Cuál  es  la  modalidad  de  diagnóstico  y  tratamiento  del  síndrome  de  Cushing?
La  mejor  prueba  de  detección  individual  es  un  cortisol  libre  de  orina  de  24  horas.  Otra  prueba  ampliamente  utilizada  
es  la  prueba  de  supresión  con  dexametasona.  Se  realiza  un  nivel  basal  de  cortisol  y  ACTH.  El  diagnóstico  se  
sugiere  con  un  cortisol  elevado.  El  paciente  recibe  1  mg  de  dexametasona  a  las  11  de  la  noche  y  el  cortisol  se  mide  
a  las  8  de  la  mañana.  Los  pacientes  con  síndrome  de  Cushing  tendrán  un  cortisol  >5  mg/dl  y  no  se  suprimirá.  La  
prueba  de  supresión  con  dexametasona  ayudará  a  determinar  la  localización  del  tumor.  Si  se  administra  una  dosis  
baja  de  dexametasona,  los  niveles  de  cortisol  no  se  suprimirán  si  el  tumor  está  en  la  glándula  suprarrenal.  Si  no  
hay  supresión,  se  repite  el  estudio  con
Machine Translated by Google
306  CAPÍTULO  61  ADRENALECTOMÍA  LAPAROSCÓPICA  SUPRARRENAL  INCIDENTALOMA

dosis  altas  de  dexametasona  (8  mg)  que  suprimen  la  secreción  de  ACTH  de  los  tumores  hipofisarios.
A  continuación,  se  mide  la  ACTH  en  el  plasma.  Si  la  ACTH  es  alta,  es  probable  que  el  tumor  sea  extra  
suprarrenal.  Si  la  ACTH  plasmática  es  baja,  esto  sugiere  una  causa  suprarrenal.  Por  lo  tanto,  supresión  de  
ACTH  plasmática  baja  con  dosis  altas  de  dexametasona  =  origen  suprarrenal.

21.  ¿Cuál  es  el  resultado  de  la  resección  de  los  adenomas  suprarrenales  productores  de  cortisol?
La  suprarrenalectomía  produce  una  excelente  mejoría  en  los  síntomas  del  síndrome  de  Cushing  y  mejora  la  
calidad  de  vida  del  paciente.  La  hipertensión  y  la  diabetes  se  resolverán  en  el  65%  al  80%  de  los  pacientes.  Los  
cambios  físicos  del  síndrome  de  Cushing  se  resolverán  en  un  85%.  Estas  mejoras  tardan  entre  6  y  12  meses  en  
producirse.  Los  esteroides  exógenos  serán  necesarios  durante  3  a  36  meses  después  de  la  suprarrenalectomía  
unilateral.  Los  estudios  de  estimulación  con  cosintropina  se  realizan  cada  3  a  6  meses  para  determinar  cuándo  se  
pueden  suspender  los  esteroides.

22.  ¿Cuál  es  la  evaluación  funcional  de  un  incidentaloma?
Una  historia  clínica  y  un  examen  físico  cuidadosos  pueden  sugerir  un  tipo  de  tumor  en  particular  y  estudios  
bioquímicos  a  realizar.  Las  pruebas  iniciales  deben  incluir  una  prueba  de  supresión  con  dosis  bajas  de  
dexametasona,  metanefrinas  y  catecolaminas  en  orina  de  24  horas  (especialmente  en  pacientes  hipertensos)  y  
nivel  de  potasio  sérico  y  aldosterona  plasmática.

23.  ¿Cuáles  son  las  reglas  de  resección  de  los  incidentalomas?
Los  incidentalomas  con  las  siguientes  características  deben  ser  resecados:
  Todos  los  tumores  funcionantes.
&  Tumores  >5  cm.
  Tumor  de  cualquier  tamaño  con  características  imagenológicas  preocupantes  para  malignidad.

&  Tumores  que  crecen  con  imágenes  seriadas.

24.  ¿Qué  características  de  imagen  de  los  incidentalomas  sugieren  malignidad?
La  mayoría  de  los  carcinomas  suprarrenales  son  grandes  (>4  cm).  Los  carcinomas  pueden  tener  una  forma  
irregular  con  márgenes  poco  claros  y  ser  heterogéneos.  Es  común  la  necrosis,  hemorragia  o  calcificación.
Por  lo  general,  son  tumores  unilaterales  solitarios  con  una  atenuación  de  >10  unidades  Hounsfield  (HU)  en  la  tomografía  
computarizada  sin  contraste.  Tienden  a  ser  vasculares  y  tienen  un  lavado  de  contraste  <50%  a  los  10  minutos  en  la  tomografía  
computarizada  mejorada.  Por  resonancia  magnética,  son  hiperintensos  en  comparación  con  el  hígado  en  las  imágenes  T2 .

25.  ¿Qué  tumores  o  trastornos  suelen  aparecer  como  masas  suprarrenales  bilaterales?
Tumores  metastásicos,  hiperplasia  (hiperplasia  suprarrenal  congénita)  y  adenomas.

26.  ¿Cómo  debe  tratarse  el  carcinoma  cortical  suprarrenal  y  cuál  es  el  principal
determinante  del  resultado?
En  ausencia  de  enfermedad  a  distancia,  la  resección  abierta  con  el  objetivo  de  una  resección  R0  (R  cero)  
es  el  tratamiento  de  elección.  En  pacientes  que  no  son  susceptibles  de  cirugía,  el  mitotano  (solo  o  en  combinación  
con  fármacos  citotóxicos)  sigue  siendo  el  tratamiento  de  elección.  Los  agentes  citotóxicos  de  uso  común  incluyen  
etopósido,  doxorrubicina,  cisplatino  y  estreptozotocina.  La  supervivencia  postoperatoria  libre  de  enfermedad  a  los  
5  años  es  del  30%.  La  recurrencia  local,  peritoneal  oa  distancia  ocurre  en  el  70%.

27.  ¿Qué  es  la  enfermedad  de  Addison?
La  enfermedad  de  Addison  es  una  insuficiencia  suprarrenal  en  la  que  la  glándula  suprarrenal  produce  una  
cantidad  insuficiente  de  glucocorticoides  y,  a  veces,  mineralocorticoides.  Clínicamente,  los  pacientes  presentan  
hiperpigmentación,  presión  arterial  baja  con  hiperpotasemia,  hipercalcemia,  hipoglucemia  e  hiponatremia.  La  
enfermedad  de  Addison  puede  ser  el  resultado  de  insuficiencia  suprarrenal  primaria,  síndrome  de  deficiencia  
poliendocrina,  tuberculosis  y  amiloidosis.  La  insuficiencia  suprarrenal  secundaria  es  el  resultado  de  la  liberación  
reducida  de  ACTH  hipofisaria.  La  insuficiencia  suprarrenal  secundaria  suele  ser  el  resultado  de  panhipopituitarismo,  
más  comúnmente  de  lesiones  que  ocupan  espacio  en  la  silla  turca.  Se  trata  mediante  reposición  de  glucocorticoides  
y  mineralocorticoides  (hidrocortisona  y  fludrocortisona).
Machine Translated by Google
CAPÍTULO  61  ADRENALECTOMÍA  LAPAROSCÓPICA  SUPRARRENAL  INCIDENTALOMA  307

28.  ¿Qué  es  la  crisis  de  Addison  y  cómo  se  trata?
La  crisis  de  Addison  es  una  insuficiencia  suprarrenal  grave  que  resulta  de  la  enfermedad  de  Addison  no  
tratada  o  no  diagnosticada.  También  puede  ocurrir  como  resultado  de  un  traumatismo  (hemorragia  
suprarrenal  bilateral),  infección  o  con  el  cese  repentino  de  glucocorticoides  exógenos.  Se  presenta  con  
deshidratación,  hipotensión,  estado  mental  alterado,  hipoglucemia  y  posiblemente  convulsiones.  Es  una  
emergencia  médica  que  puede  ser  fatal.  El  diagnóstico  se  realiza  mediante  pruebas  de  estimulación  con  
cosintropina.  En  esta  prueba,  el  cortisol  en  la  sangre  se  mide  antes  y  después  de  administrar  una  forma  sintética  
de  ACTH  por  inyección.  La  medición  de  cortisol  en  sangre  se  repite  de  30  a  60  minutos  después  de  una  
inyección  intravenosa  (IV)  de  ACTH.  Para  el  tratamiento  se  requiere  reemplazo  intravenoso  de  cortisol,  glucosa  
y  solución  salina.

29.  ¿Cuáles  son  las  potencias  relativas  de  los  esteroides?

Glucocorticoide   Mineralocorticoide   Duración


hidrocortisona 1,0  0,7   1,0  0,7   Corto
Cortisona 4,0  5,0   0,7  0,5   Corto
prednisona 30,0  10,0 0,0  400,0 Corto
Metilprednisolona Corto
Dexametasona Largo
fludrocortisona Largo

30.  ¿Qué  presidente  de  los  EE.  UU.  tenía  la  enfermedad  de  Addison?
A  John  F.  Kennedy  se  le  diagnosticó  oficialmente  la  enfermedad  de  Addison  en  1947.  Sin  embargo,  es  posible  
que  se  le  haya  diagnosticado  mucho  antes  de  lo  que  comúnmente  se  pensaba.  En  ese  momento,  la  
enfermedad  de  Addison  se  describía  clásicamente  como  causada  por  una  tuberculosis  que  Kennedy  nunca  
tuvo.  Cuando  la  campaña  de  Lyndon  B.  Johnson  hizo  referencia  a  la  enfermedad  de  Addison  de  Kennedy  
usándola  como  argumento  en  contra  de  su  nominación  a  la  presidencia,  la  campaña  de  Kennedy  respondió  
que  nunca  tuvo  la  enfermedad.

BIBLIOGRAFÍA

1.  Allolio  B,  Fassnacht  M:  Carcinoma  adrenocortical:  actualización  clínica.  J  Clin  Endocrinol  Metab  91:2027­2037;  2006.
2.  Annane  D,  Maxime  V,  Ibrahim  F  et  al .:  Diagnóstico  de  insuficiencia  suprarrenal  en  sepsis  grave  y  shock  séptico.
Am  J  Respir  Crit  Care  Med  174(12):1319­1326,  2006.
3.  Barnett  CC  Jr,  Varma  DG,  El­Naggar  AK  et  al.:  Limitaciones  de  tamaño  como  criterio  en  la  evaluación  de  la  suprarrenal
tumores  Cirugía  128(6):973­982;  discusión  982­983,  2000.
4.  Brunt  LM,  Moley  JF:  Incidentaloma  suprarrenal.  World  J  Surg  25(7):905­913,  2001.
5.  Goldstein  RE,  O'Neill  JA  Jr,  Holcomb  GW  3rd  et  al.:  Experiencia  clínica  durante  48  años  con  feocromocitoma.
Ann  Surg  229(6):755­764;  discusión  764­766,  1999.
6.  Gonzalez  RJ,  Tamm  EP,  Ng  C  et  al.:  La  respuesta  al  mitotano  predice  el  resultado  en  pacientes  con  carcinoma  de  la  
corteza  suprarrenal  recurrente.  Cirugía  142(6):867­875;  discusión  867­875,  2007.
7.  Grumbach  MM,  Biller  BM,  Braunstein  GD  et  al.:  Manejo  de  la  masa  suprarrenal  clínicamente  inaparente
("incidentaloma").  Ann  Intern  Med  138(5):424­429,  2003.
8.  Kudva  YC,  Sawka  AM,  Young  WF  Jr:  Revisión  clínica  164:  el  diagnóstico  de  laboratorio  de  suprarrenal
feocromocitoma:  la  experiencia  de  Mayo  Clinic.  J  Clin  Endocrinol  Metab  88(10):4533­4539,  2003.
9.  Porterfield  JR,  Thompson  GB,  Young  WF  Jr  et  al.:  Cirugía  para  el  síndrome  de  Cushing:  una  revisión  histórica  y
Experiencia  reciente  de  diez  años.  World  J  Surg  32:659­677,  2008.
10.  Quayle  FJ,  Spitler  JA,  Pierce  RA  et  al.:  La  biopsia  con  aguja  de  masas  suprarrenales  descubiertas  incidentalmente  
rara  vez  es  informativa  y  potencialmente  peligrosa.  Cirugía  142(4):497­502;  discusión  502­504,  2007.
11.  Sawka  AM,  Young  WF,  Thompson  GB  et  al .:  Aldosteronismo  primario:  factores  asociados  con  la  normalización  de  la  
presión  arterial  después  de  la  cirugía.  Ann  Intern  Med  135(4):258­261,  2001.
Machine Translated by Google
308  CAPÍTULO  61  ADRENALECTOMÍA  LAPAROSCÓPICA  SUPRARRENAL  INCIDENTALOMA

12.  Sprung  CL,  Annane  D,  Keh  D  et  al .:  Terapia  con  hidrocortisona  para  pacientes  con  shock  séptico.  N  Engl  J  Med  
358(2):111­124,  2008.
13.  Tan  YY,  Ogilvie  JB,  Triponez  F  et  al .:  Uso  selectivo  de  muestreo  venoso  suprarrenal  en  la  lateralización  de  
adenomas  productores  de  aldosterona.  World  J  Surg  30(5):879­885;  discusión  886­887,  2006.
14.  Terzolo  M,  Angeli  A,  Fassnacht  M  et  al .:  tratamiento  adyuvante  con  mitotano  para  el  carcinoma  adrenocortical.  N  
Engl  J  Med  356(23):2372–2380;
15.  Young  WF  Jr:  Práctica  clínica.  La  masa  suprarrenal  descubierta  incidentalmente.  N  Engl  J  Med  356(6):601­610,  2007.
Machine Translated by Google

V.  CIRUGÍA  DE  MAMA

MASA  DE  MAMA CAPITULO  
62

Ann  Marie  Kulungowski,  MD;

1.  ¿Cuáles  son  las  tres  partes  del  examen  de  mama  que  ayudan  en  el  diagnóstico  temprano  de
¿cáncer  de  mama?

El  autoexamen  de  mamas  (BSE)  debe  comenzar  a  los  20  años  y  realizarse  mensualmente.  El  seno  suele  ser  más  fácil  
de  examinar  en  los  días  inmediatamente  posteriores  al  ciclo  menstrual.  La  BSE  puede  ser  frustrante  para  los  pacientes,  
particularmente  cuando  tienen  un  cambio  fibroquístico  porque  no  están  seguros  de  lo  que  sienten  o  se  supone  que  
deben  sentir.  La  técnica  de  BSE  debe  enseñarse  temprano  y  reforzarse  regularmente.  Si  se  desarrolla  un  tumor  
palpable,  las  mujeres  que  realizan  BSE  regularmente  presentan  tumores  de  1  cm  o  menos  con  mayor  frecuencia  que  
las  mujeres  que  no  realizan  BSE.  Sin  embargo,  no  se  ha  demostrado  una  mejoría  en  la  supervivencia  del  cáncer  de  
mama.
Algunas  mujeres  no  deberían  practicar  BSE  debido  al  trauma  psicológico  que  sufren  por  los  resultados  falsos  
positivos  repetitivos.  Esas  mujeres  necesitan  confiar  en  que  su  médico  les  haga  un  examen  de  los  senos  una  o  dos  
veces  al  año.
El  examen  médico  o  clínico  de  las  mamas  (CBE,  por  sus  siglas  en  inglés)  también  debe  comenzar  a  los  
20  años  y  realizarse  anualmente  para  mujeres  con  un  riesgo  promedio  de  cáncer  de  mama.  Aunque  los  tumores  
entre  0,5  cm  y  1,0  cm  ocasionalmente  pueden  ser  detectados  por  un  médico  experimentado,  los  tumores  entre  1,0  y  
1,5  cm  pueden  detectarse  el  60  %  de  las  veces.  A  medida  que  el  tumor  crece,  el  96  %  de  los  tumores  mayores  de  2,0  
cm  pueden  identificarse  en  el  examen  físico  del  médico.  El  examen  clínico  de  las  mamas  debe  formar  parte  del  
programa  de  detección  y  mantenimiento  de  la  salud  del  médico  de  atención  primaria.
La  mamografía  de  detección  ha  tenido  el  impacto  más  sustancial  en  el  diagnóstico  temprano  y  la  subsiguiente  
disminución  de  la  mortalidad  por  cáncer  de  mama.

2.  ¿Cuándo  debe  comenzar  la  mamografía  de  rutina?
Todas  las  principales  organizaciones  médicas  de  los  EE.  UU.  actualmente  recomiendan  que  las  mujeres  que  tienen  un  
riesgo  promedio  de  desarrollar  cáncer  de  mama  deben  comenzar  a  hacerse  mamografías  a  los  40  años.
La  mamografía  debe  realizarse  anualmente.  Este  régimen  de  detección  dará  como  resultado  una  disminución  del  30  
%  o  más  en  las  muertes  por  cáncer  de  mama.  Las  mujeres  con  antecedentes  familiares  de  cáncer  de  mama  temprano  
deben  comenzar  con  la  mamografía  de  detección  10  años  antes  de  la  edad  en  que  se  le  diagnosticó  cáncer  de  mama  
al  familiar  de  primer  grado  más  joven.

3.  ¿Una  mamografía  normal  o  negativa  garantiza  que  no  hay  cáncer  presente?
No.  La  mamografía  tiene  una  tasa  de  falsos  negativos  de  al  menos  el  15%.  Para  que  un  cáncer  de  mama  se  
detecte  en  una  mamografía,  debe  tener  características  de  tejido  que  sean  diferentes  del  tejido  circundante.  
Algunos  tumores,  particularmente  el  carcinoma  lobulillar,  invaden  el  tejido  mamario  circundante  de  una  manera  que  no  
altera  las  características  del  tejido  mamario.  Dichos  tumores  a  menudo  no  son  visibles  en  la  mamografía.

4.  ¿Cuál  es  el  papel  de  la  resonancia  magnética  como  complemento  de  la  mamografía?

Las  pautas  de  la  Sociedad  Estadounidense  del  Cáncer  publicadas  en  2007  recomiendan  la  detección  de  imágenes  por  
resonancia  magnética  (MRI)  para  mujeres  con  un  riesgo  de  desarrollar  cáncer  de  mama  mayor  al  20%  durante  su  vida.  
Esto  incluye  a  mujeres  que  tienen  un  fuerte  historial  familiar  de  cáncer  de  mama  o  de  ovario  y  mujeres  que  han  recibido  
radiación  en  el  pecho  para  la  enfermedad  de  Hodgkin  cuando  eran  adolescentes  o  adultas  jóvenes.  Anual

309
Machine Translated by Google
310  CAPÍTULO  62  MASAS  MAMARAS

Se  debe  agregar  una  resonancia  magnética  de  detección  para  las  mujeres  que  tienen  una  mutación  BRCA,  las  mujeres  
que  no  se  han  sometido  a  la  prueba  de  mutaciones  BRCA  pero  tienen  un  familiar  de  primer  grado  con  una  mutación  BRCA,  
o  las  mujeres  que  tienen  un  riesgo  de  por  vida  de  más  del  20  %  determinado  por  BRCAPRO  o  otros  modelos  estadísticos  
que  predicen  el  riesgo.  También  se  recomienda  el  examen  de  resonancia  magnética  anual  para  mujeres  con  síndromes  de  
Li­Fraumeni,  Cowden  o  Bannayan­Riley­Ruvalcaba  y  sus  familiares  de  primer  grado.
Actualmente  no  hay  evidencia  suficiente  para  recomendar  a  favor  o  en  contra  de  la  detección  en  resonancia  magnética  en
mujeres  con  un  riesgo  de  por  vida  del  15  %  al  20  %,  carcinoma  lobulillar  in  situ  o  hiperplasia  lobulillar  o  ductal  
atípica  con  antecedentes  familiares  asociados,  mamas  heterogéneamente  densas  por  mamografía,  o  mujeres  con  
antecedentes  de  cáncer  de  mama,  incluido  el  carcinoma  ductal  in  situ  (DCIS) .  No  se  recomienda  el  examen  de  
resonancia  magnética  para  ninguna  mujer  con  menos  del  15%  de  riesgo  de  cáncer  de  mama  durante  su  vida.

La  resonancia  magnética  es  bastante  sensible  para  identificar  el  cáncer  de  mama,  pero  es  criticada  por  tener  
una  menor  especificidad  que  conduce  a  imágenes  y  biopsias  adicionales  para  caracterizar  mejor  las  lesiones  que  
resultan  ser  falsos  positivos.  Al  igual  que  con  todos  los  estudios  de  diagnóstico,  la  necesidad  de  una  evaluación  
adicional  debe  sopesarse  frente  al  beneficio  de  encontrar  un  cáncer  de  mama  oculto.  La  capacidad  de  detectar  el  
cáncer  por  resonancia  magnética  está  fuertemente  influenciada  por  la  calidad  de  la  imagen  y  la  familiaridad  del  radiólogo  
con  la  lectura  de  la  resonancia  magnética  de  mama.  La  resonancia  magnética  de  mama  solo  debe  realizarse  en  
instalaciones  que  tengan  la  capacidad  de  evaluar  cualquier  anomalía  que  se  encuentre,  incluida  la  capacidad  de  realizar  
una  biopsia  guiada  por  resonancia  magnética.  La  resonancia  magnética  debe  usarse  junto  con  la  mamografía  y  no  es  una  
modalidad  de  detección  independiente  para  el  cáncer  de  mama.

5.  ¿Cuál  es  el  papel  de  la  ecografía  en  el  diagnóstico  del  cáncer  de  mama?
Hasta  la  fecha,  la  ecografía  (EE.  UU.)  se  ha  utilizado  como  complemento  para  evaluar  más  a  fondo  las  anomalías  
encontradas  en  la  mamografía  y  el  examen  físico,  pero  no  se  utiliza  como  herramienta  de  detección.  La  ecografía  
puede  distinguir  quistes  simples  benignos  de  masas  sólidas  o  quistes  complejos.  Es  particularmente  valioso  en  
mujeres  jóvenes  en  las  que  la  densidad  del  tejido  mamario  limita  el  valor  de  la  mamografía.  Si  un  paciente  con  una  
queja  de  una  masa  palpable  que  tiene  características  benignas  en  el  examen  físico  del  médico  tiene  una  ecografía  dirigida  
negativa,  el  valor  predictivo  negativo  para  el  cáncer  es  del  99,8%.

6.  ¿Cuál  es  la  diferencia  entre  una  mamografía  de  detección  y  una  de  diagnóstico?
La  mamografía  de  detección  se  realiza  en  mujeres  asintomáticas  para  buscar  cáncer  de  mama  clínicamente  oculto.  Se  
obtienen  dos  vistas  de  cada  mama.  Cuando  una  mujer  tiene  una  queja  en  los  senos,  como  una  masa  o  una  mamografía  
de  detección  anormal,  se  realiza  una  mamografía  de  diagnóstico.
Una  mamografía  de  diagnóstico  presta  especial  atención  al  área  de  interés  clínico.  Vistas  adicionales  tomadas  en  
múltiples  ángulos  o  vistas  de  compresión  tomadas  con  mayor  aumento  de  la  anormalidad  ayudan  a  distinguir  entre  cambios  
benignos  y  malignos.

7.  ¿Cómo  se  caracterizan  las  anomalías  mamográficas?
El  American  College  of  Radiography  ha  desarrollado  una  puntuación  de  interpretación  estándar  para  disminuir  la  
ambigüedad  en  los  informes  mamográficos:  Bi­Rads  0  Requiere  una  evaluación  adicional.

1  Negativo  (examen  normal  sin  hallazgos).
2  Benigno  (examen  normal  con  un  hallazgo  definitivamente  benigno).
3  Probablemente  benigno  (<3%  de  probabilidad  de  malignidad).
4  Sospechoso  (30%  de  probabilidad  de  malignidad).
5  Altamente  sospechoso  o  maligno.
  La  categoría  0  es  una  designación  temporal  que  requiere  imágenes  diagnósticas  adicionales  mediante  ecografía  
o  vistas  de  compresión  (ampliación)  de  la  anomalía.  Después  de  una  evaluación  adicional,  dichas  mamografías  
se  reclasifican  en  una  de  las  otras  categorías.
Machine Translated by Google
CAPÍTULO  62  MASA  EN  LAS  MAMA  311

&  Las  categorías  1  y  2  no  requieren  más  evaluación;  el  calendario  habitual  de  mamografías  es
no  alterado.
&  Para  la  categoría  3,  una  mamografía  de  diagnóstico  de  intervalo  corto  (6  meses)  del  afectado
Se  recomienda  el  pecho.  Alternativamente,  se  puede  realizar  una  biopsia.
&  Las  categorías  4  y  5  requieren  una  biopsia.

8.  ¿Qué  técnicas  de  biopsia  ayudan  en  el  diagnóstico  de  anomalías  mamográficas?
Varias  técnicas  de  biopsia  guiada  por  imágenes  maximizan  el  rendimiento  del  diagnóstico  y  minimizan  la  incomodidad  
del  paciente  y  la  pérdida  de  tejido  normal:

La  biopsia  de  núcleo  Tru­cut  se  realiza  con  una  aguja  de  extracción  de  núcleo  de  calibre  14  a  18.  varios  tejidos
se  obtienen  muestras  (al  menos  siete).
La  biopsia  de  Mammotome  se  realiza  con  una  aguja  de  biopsia  asistida  por  vacío  de  calibre  11.
Mammotome  puede  eliminar  una  lesión  completa  o  un  área  de  calcificación.  Se  puede  dejar  un  clip  de  marcado  en  el  
seno  en  el  sitio  de  la  anomalía.  La  biopsia  central  y  el  mammotomo  se  pueden  realizar  solo  con  anestesia  local.

El  instrumento  avanzado  de  biopsia  mamaria  (ABBI)  extrae  hasta  2  cm  de  tejido  mamario.  Él
por  lo  general  requiere  anestesia  local  y  sedación  leve  y  generalmente  se  realiza  en  la  sala  de  operaciones  (OR).

La  biopsia  mamaria  de  localización  con  aguja  es  un  procedimiento  quirúrgico  que  requiere  que  el  radiólogo  coloque  
un  alambre  delgado  en  el  seno  en  el  sitio  de  la  anomalía.  En  el  quirófano,  se  extraen  el  alambre  y  el  tejido  mamario  que  
lo  rodea.  Este  procedimiento  se  puede  realizar  con  anestesia  local  con  o  sin  sedación.

Aunque  la  aspiración  con  aguja  fina  (FNA)  es  excelente  para  la  evaluación  de  anomalías  palpables,
su  sensibilidad  y  especificidad  para  la  biopsia  guiada  por  imagen  no  son  aceptables.
Con  la  excepción  de  FNA,  cada  una  de  estas  técnicas  tiene  una  tasa  de  éxito  equivalente  en  la  identificación  
de  la  patología  asociada  con  la  anomalía  mamográfica.  Se  asocia  una  tasa  de  falsos  negativos  del  5%  con  cada  una  de  
estas  técnicas.

9.  ¿Cuáles  son  las  características  de  una  masa  mamaria  dominante?

La  identificación  de  una  masa  dominante,  especialmente  en  mujeres  premenopáusicas,  puede  ser  un  desafío.
Por  lo  general,  una  masa  dominante  se  puede  palpar  en  tres  dimensiones  y  su  densidad  es  distinta  del  tejido  
mamario  circundante.  Los  síntomas  de  igual  importancia  son  nódulo,  bulto,  engrosamiento  y  asimetría.  El  cáncer  de  
mama  no  se  puede  excluir  mediante  un  examen  físico  solo.  "El  no  dejarse  impresionar  por  los  hallazgos  del  examen  
físico"  es  la  razón  citada  con  más  frecuencia  para  el  retraso  en  el  diagnóstico  del  cáncer  de  mama.

10.  ¿Cuáles  son  las  cuatro  masas  mamarias  palpables  que  se  encuentran  con  mayor  frecuencia?
La  mayoría  de  las  masas  dominantes  son  benignas.  Los  ejemplos  incluyen  quistes,  fibroadenomas  y  masas  
fibroquísticas.  El  carcinoma,  aunque  no  es  la  forma  más  común  de  masa  mamaria,  es  la  razón  por  la  que  todas  las  
masas  dominantes  persistentes  requieren  un  diagnóstico.  Otras  causas  menos  comunes  de  masas  mamarias  
palpables  son  lipomas,  granulomas,  necrosis  grasa,  quistes  de  inclusión  epidérmica  y  adenomas  de  lactancia.

11.  ¿Cuáles  son  las  características  diferenciales  de  las  masas  palpables  más  frecuentes?
Un  quiste  es  una  masa  móvil  regular  que  puede  ser  sensible.  Puede  ser  bastante  firme  o  fluctuante.
Un  fibroadenoma  suele  ser  liso,  firme,  alargado  (más  largo  que  ancho)  y  móvil  con  bordes  discretos.  Los  cambios  
fibroquísticos  a  menudo  se  describen  como  tejido  mamario  "abultado".  Puede  haber  un  área  focal  discreta  de  fibrosis  
que  es  más  dominante  que  el  tejido  irregular  de  fondo.
El  carcinoma  es  una  masa  irregular,  dura  e  indolora.  En  etapas  avanzadas  puede  quedar  fijado  a
la  pared  torácica  o  asociarse  con  cambios  en  la  piel  suprayacente.  Aunque  esta  es  la  presentación  clásica,  el  
carcinoma  puede  presentarse  de  forma  similar  a  las  lesiones  benignas.  El  carcinoma  lobulillar  a  menudo  aparece  
como  una  masa  blanda  o  un  área  de  engrosamiento.  Debido  a  que  el  examen  físico  por  sí  solo  no  es  confiable  para  
excluir  definitivamente  el  cáncer  de  mama,  se  debe  obtener  una  biopsia  para  todas  las  masas  sólidas  persistentes  y  
dominantes.
Machine Translated by Google
312  CAPÍTULO  62  MASAS  MAMARAS

12.  Una  mujer  de  32  años  se  presenta  con  la  queja  de  un  bulto  en  el  seno.  ¿Qué  preguntas  sobre  la  historia  del  paciente  
son  importantes  en  la  evaluación  de  la  masa?

El  tamaño  de  la  masa,  si  ha  cambiado  de  tamaño,  cuánto  tiempo  ha  estado  presente,  si  es  dolorosa,  cambios  en  la  piel,  
secreción  del  pezón  o  cambios  en  relación  con  el  ciclo  menstrual  pueden  ser  útiles.  La  evaluación  de  cualquier  afección  
mamaria  incluye  una  evaluación  de  los  factores  de  riesgo  de  cáncer  de  mama,  incluidos  los  antecedentes  personales  o  
familiares  de  cáncer  de  mama,  de  ovario  u  otros;  edad  de  la  menarquia;  edad  del  primer  embarazo  a  término;  edad  de  la  
menopausia,  si  corresponde;  uso  de  métodos  anticonceptivos  o  reemplazo  hormonal;  y  antecedentes  de  biopsia  mamaria  
previa.

13.  La  masa  identificada  en  la  pregunta  10  es  discreta,  no  dolorosa,  fácilmente  palpable  y  ha  aumentado  gradualmente  de  
tamaño.  ¿Cuál  es  el  próximo  paso  más  apropiado?
Las  imágenes  mamarias  pueden  ser  útiles  para  definir  mejor  las  características  de  una  masa  mamaria.  La  ecografía  
de  una  masa  discreta  puede  determinar  si  es  quística  o  sólida.  Existen  criterios  ecográficos  específicos  para  definir  un  
quiste  simple.  Un  quiste  simple  se  puede  aspirar  u  observar.  Un  quiste  complejo  debe  evaluarse  más  a  fondo  mediante  
aspiración  (para  ver  si  se  resuelve  por  completo)  o  mediante  biopsia  por  escisión.  Con  un  quiste  complejo,  la  FNA  o  la  
biopsia  central  tienen  un  mayor  riesgo  de  error  de  muestreo  del  componente  sólido.  Una  masa  sólida  requiere  un  
diagnóstico  de  tejido.

14.  ¿Cómo  se  realiza  la  aspiración  de  un  quiste?
Se  inserta  una  aguja  de  calibre  22  en  el  quiste  y  se  extrae  líquido.  En  general,  una  jeringa  de  10  ml  es  adecuada,  aunque  
en  ocasiones  los  quistes  contienen  mayores  cantidades  de  líquido.  Si  el  quiste  es  bastante  profundo  y  difícil  de  arreglar  
entre  los  dedos  del  médico,  la  aspiración  se  puede  realizar  bajo  guía  ecográfica.  La  aspiración  de  un  quiste  es  tanto  
diagnóstica  como  terapéutica.  Después  de  la  aspiración,  la  masa  debe  resolverse  por  completo.  Si  una  masa  persiste  o  
reaparece  después  de  dos  aspiraciones,  debe  extirparse.  El  líquido  del  quiste  puede  ser  transparente  o  turbio  de  color  
amarillo,  verde,  gris  o  marrón.  Debe  enviarse  un  aspirado  puramente  de  sangre  o  un  aspirado  de  lo  que  parece  ser  
sangre  vieja  para  citología,  y  se  debe  realizar  la  escisión  de  la  lesión.

15.  ¿Qué  técnicas  existen  para  el  diagnóstico  de  una  mama  sólida  palpable?
¿masa?

La  FNA,  la  biopsia  central,  la  biopsia  por  incisión  y  la  biopsia  por  escisión  tienen  un  papel  en  el  diagnóstico  de  masas  
mamarias  palpables.  La  técnica  utilizada  depende  de  la  naturaleza  de  las  lesiones  y  del  soporte  técnico  disponible.

FNA  recupera  células  de  la  masa  y  requiere  un  citopatólogo  dedicado  para  una  interpretación  precisa.  Varias  
lesiones  benignas  y  malignas  pueden  caracterizarse  con  precisión  mediante  FNA,  pero  FNA  no  puede  discriminar  
entre  carcinoma  invasor  e  in  situ.  Para  ser  utilizado  de  manera  efectiva,  debe  correlacionarse  con  el  examen  físico  y  
las  imágenes  mamarias.
La  biopsia  central  también  es  una  técnica  de  muestreo  que  extrae  piezas  de  tejido  de  calibre  14  a  18  para
evaluación  histológica  por  el  patólogo.  Debido  a  que  es  una  muestra,  existe  el  riesgo  de  pasar  por  alto  la  lesión  y  
obtener  un  resultado  falso  negativo.  Una  vez  más,  la  correlación  con  el  examen  físico  y  las  imágenes  es  importante  para  
evitar  que  no  se  diagnostique  un  cáncer  de  mama.
La  biopsia  por  incisión  rara  vez  se  usa  hoy  en  día.  Tiene  un  papel  cuando  una  lesión  altamente  sospechosa  que  es  un
candidato  para  tratamiento  neoadyuvante  no  se  diagnostica  definitivamente  en  la  biopsia  central.
La  biopsia  por  escisión  elimina  por  completo  la  lesión  diana.  Proporciona  la  mayor  cantidad  de  tejido  para
evaluación  patológica  y,  en  la  enfermedad  benigna,  es  tanto  diagnóstica  como  terapéutica.

16.  ¿Cuál  es  el  papel  de  las  imágenes  mamarias  en  la  evaluación  de  una  mama  palpable?
¿masa?

Las  imágenes  de  mama  ayudan  a  definir  la  lesión  y  examinan  el  resto  de  la  mama  en  busca  de  lesiones  secundarias.  En  
general,  las  imágenes  del  seno  se  realizan  antes  de  la  biopsia  porque  el  artefacto  de  la  biopsia  puede  interferir  con  la  
interpretación  del  estudio.
Machine Translated by Google
CAPÍTULO  62  MASAS  MAMARAS  313

En  mujeres  menores  de  30  años,  en  quienes  el  riesgo  de  malignidad  es  bajo,  la  mamografía  debe
reservarse  para  las  lesiones  más  sospechosas.  Para  las  mujeres  mayores  de  30  años,  la  evaluación  de  
una  masa  sospechosa  de  malignidad  incluye  una  mamografía  para  caracterizar  la  masa  y  evaluar  el  resto  de  la  
mama.  La  ecografía  puede  diferenciar  de  forma  fiable  entre  masas  quísticas  y  sólidas.  Es  bastante  inusual  (menos  
del  2%)  que  la  ecografía  no  identifique  una  masa  mamaria  clínicamente  significativa.  En  consecuencia,  se  puede  
realizar  una  aspiración  o  biopsia  del  quiste.

17.  ¿Qué  es  la  «prueba  triple  negativa»  o  «tríada  diagnóstica»?
Hay  tres  componentes  para  diagnosticar  una  anomalía  mamaria  palpable:  examen  físico,  imágenes  
mamarias  y  biopsia.  Las  lesiones  benignas  no  tienen  que  ser  extirpadas,  pero  la  dificultad  está  en  diferenciar  
entre  una  lesión  benigna  y  una  maligna.  Cuando  las  características  de  una  masa  al  examen  físico  indican  baja  
sospecha  de  malignidad,  la  mamografía  es  benigna  y  la  PAAF  recupera  células  benignas,  la  probabilidad  de  que  
la  lesión  sea  benigna  es  del  98%.
Las  opciones  de  tratamiento  incluyen  la  escisión  para  el  diagnóstico  definitivo  o  la  observación.  Si  se  opta  por  
la  observación,  la  anomalía  debe  volver  a  examinarse  en  un  plazo  de  3  meses  para  confirmar  que  es  estable.  Si  
algún  componente  de  la  tríada  diagnóstica  es  preocupante,  es  necesario  un  diagnóstico  definitivo,  por  lo  general  
con  biopsia  por  escisión.

BIBLIOGRAFÍA

1.  Elmore  JG,  Armstrong  K,  Lehman  CD  et  al.:  Detección  de  cáncer  de  mama.  JAMA  293:1245­1256,  2005.
2.  Geller  BM,  Barlow  WE,  Ballard­Barbash  R  et  al.:  uso  del  American  College  of  Radiology  BI­RADS  para  informar  sobre  la  
evaluación  mamográfica  de  mujeres  con  signos  y  síntomas  de  enfermedad  mamaria.  Radiología  222:536­542,  2002.

3.  Hendrick  RE:  Reducción  de  la  mortalidad  por  mamografía  de  detección.  Mama  Dis  13:303­307,  2003.
4.  Mendelson,  EB:  Ultrasonido  de  resolución  de  problemas.  Radiol  Clin  N  Am  42:909­918,  2004.
5.  Pruthi  S,  Brandt  KR,  Degnim  AC  et  al.:  Un  enfoque  multidisciplinario  para  el  manejo  del  cáncer  de  mama,  parte  1:  
prevención  y  diagnóstico.  Mayo  Clin  Proc  82:999­1012,  2007.
6.  Saslow  D,  Boetes  C,  Burke  W  et  al.:  Directrices  de  la  Sociedad  Estadounidense  del  Cáncer  para  la  detección  de  mamas  con  resonancia  magnética  como
complemento  de  la  mamografía.  CA  Cancer  J  Clin  57:75­89,  2007.
7.  Yang  W,  Dempsey  P:  Diagnóstico  por  ultrasonido  de  mama:  estado  actual  y  direcciones  futuras.  Radiol  Clin  N  Am
45:845­861,  2007.
Machine Translated by Google

TERAPIA  PRIMARIA  PARA  LA  MAMA
CÁNCER
CAPITULO  
63

Kristine  E.  Calhoun,  MD  y  Benjamin  O.  Anderson,  MD

1.  ¿Cómo  se  diagnostica  el  cáncer  de  mama?
Un  diagnóstico  de  cáncer  de  mama  requiere  confirmación  de  tejido  mediante  muestreo  con  aguja  o  biopsia  quirúrgica.
Históricamente,  la  biopsia  por  escisión  era  el  estándar  de  oro,  pero  el  muestreo  con  aguja  se  ha  convertido  en  el  método  de  
diagnóstico  inicial  preferido  mediante  la  biopsia  con  aguja  gruesa  o,  si  un  citólogo  de  mama  experto  está  disponible  para  
interpretar  la  muestra,  la  aspiración  con  aguja  fina  (FNA).  El  muestreo  con  aguja  (1)  permite  una  planificación  quirúrgica  
completa,  incluidas  las  decisiones  sobre  la  lumpectomía  o  el  uso  del  mapeo  del  ganglio  centinela  y  (2)  no  distorsiona  la  forma  o  
la  arquitectura  de  la  mama  para  futuros  exámenes  clínicos  de  mama  (CBE)  e  imágenes  mamarias.

2.  ¿Cuáles  son  las  limitaciones  del  muestreo  con  aguja?
Tanto  la  FNA  como  la  biopsia  con  aguja  gruesa  pueden  tener  resultados  falsos  negativos  causados  por  un  error  de  muestreo.
Si  el  diagnóstico  de  la  muestra  con  aguja  es  negativo  para  el  cáncer  y  estos  hallazgos  se  correlacionan  con  la  presentación  
clínica  y  los  hallazgos  de  las  imágenes  mamarias  (mamografía  y  ecografía  [US]),  todos  los  cuales  sugieren  un  proceso  
mamario  benigno  (concordancia),  la  paciente  puede  tener  un  seguimiento  clínico.  exploración  sin  más  intervención.  Sin  embargo,  
si  los  resultados  de  la  muestra  con  aguja  no  coinciden  con  los  hallazgos  del  examen  clínico  o  las  imágenes  del  seno  (discordancia),  
se  debe  realizar  una  muestra  de  tejido  adicional,  como  una  biopsia  por  escisión.

3.  ¿En  qué  se  diferencian  la  aspiración  con  aguja  fina  y  la  biopsia  con  aguja  gruesa?
La  citología  FNA  es  técnicamente  simple  de  realizar,  se  puede  leer  de  inmediato  y  cuesta  solo  unos  centavos.  Sin  
embargo,  la  citología  FNA  requiere  un  citólogo  experto  para  una  interpretación  correcta  y  no  se  puede  usar  para  distinguir  el  
cáncer  no  invasivo  (in  situ)  del  invasivo.  Por  el  contrario,  la  biopsia  con  aguja  gruesa  (utilizando  muestreo  asistido  por  vacío  
de  calibre  14  estándar  o  calibre  8  de  gran  calibre)  obtiene  muestras  histológicas  reales  que  funcionalmente  se  asemejan  a  
biopsias  quirúrgicas  en  miniatura  pero  no  distorsionan  el  tejido  mamario  ni  dejan  cicatrices  grandes  después  de  la  cicatrización.  
La  biopsia  con  aguja  gruesa  puede  distinguir  el  cáncer  invasivo  del  no  invasivo,  la  histología  ductal  de  la  lobulillar  y  la  enfermedad  
de  alto  grado  de  la  de  bajo  grado.  Se  pueden  preparar  secciones  especiales  de  muestras  de  biopsia  con  aguja  gruesa  para  la  
tinción  inmunohistoquímica  (IHC)  para  determinar  el  receptor  de  estrógeno  (ER),  el  estado  del  receptor  de  progesterona  (PR)  y  la  
sobreexpresión  de  Her­2/neuoncogén.  Un  patólogo  experto  en  la  lectura  de  portaobjetos  de  mama  quirúrgicos  estándar  también  
debe  sentirse  cómodo  leyendo  portaobjetos  de  aguja  central  de  mama,  pero  es  posible  que  no  se  sienta  cómodo  interpretando  
una  PAAF  de  mama.  Debido  a  su  versatilidad  y  la  relativa  escasez  de  experiencia  en  citología  mamaria,  la  biopsia  con  aguja  
gruesa  se  ha  convertido  en  gran  medida  en  el  estándar  en  los  Estados  Unidos.

4.  ¿Por  qué  se  debe  obtener  una  imagen  del  seno  antes  de  realizar  una  biopsia  mamaria  quirúrgica?
El  cáncer  de  mama  generalmente  comienza  como  una  enfermedad  clínicamente  oculta  que  gradualmente  se  vuelve  palpable  a  
medida  que  el  cáncer  evoluciona  y  crece  en  la  mama.  Incluso  los  cirujanos  experimentados  pueden  sorprenderse  al  descubrir  
que  los  cánceres  palpables  aparentemente  pequeños  pueden  ser  mucho  más  extensos  en  el  seno  de  lo  anticipado  basándose  
únicamente  en  la  CBE.  Las  imágenes  preoperatorias  ayudan  a  los  cirujanos  a  optimizar  los  resultados  quirúrgicos  al  evitar  estas  
sorpresas.

314
Machine Translated by Google
CAPÍTULO  63  TERAPIA  PRIMARIA  PARA  EL  CÁNCER  DE  MAMA  315

La  mamografía  es  el  mapa  de  ruta  del  cirujano,  que  ilustra  la  distribución  de  grasa  y  denso
tejidos  dentro  del  seno  y  al  mismo  tiempo  identificar  lesiones  adicionales  en  el  mismo  seno  o  en  el  opuesto.  
La  ecografía  mamaria  es  buena  para  visualizar  una  lesión  específica  o  una  masa  palpable  dentro  de  la  
mama  y  puede  usarse  para  guiar  el  muestreo  con  aguja.  Muchos  centros  oncológicos  utilizan  actualmente  la  
resonancia  magnética  (RM)  de  mama  para  evaluar  la  extensión  de  la  enfermedad  más  allá  de  lo  que  se  ve  en  
las  imágenes  estándar  una  vez  que  el  cáncer  ya  se  ha  diagnosticado.

5.  ¿La  demora  entre  la  biopsia  y  el  tratamiento  definitivo  afecta  negativamente  a  la  curación?
No,  si  el  retraso  es  solo  de  días  o  semanas.  En  general,  los  cánceres  de  mama  evolucionan  
lentamente,  el  tratamiento  debe  iniciarse  dentro  de  las  3  a  4  semanas  posteriores  al  diagnóstico  inicial.  Deben  
evitarse  retrasos  de  más  de  3  a  6  meses.  Hay  más  urgencia  con  el  cáncer  de  mama  asociado  con  el  embarazo,  
en  el  que  el  crecimiento  del  tumor  puede  ser  mucho  más  rápido.  No  es  apropiado  retrasar  el  tratamiento  de  un  
cáncer  de  mama  hasta  el  final  del  embarazo,  particularmente  cuando  algunos  agentes  quimioterapéuticos  (p.  ej.,  
doxorrubicina  [Adriamicina])  se  pueden  administrar  de  manera  segura  durante  el  embarazo.

6.  ¿Cómo  se  clasifica  por  etapas  el  cáncer  de  mama?
Consulte  la  Tabla  63­1.

TABLA  63­1.  ST  AG  IN  GOF  BR  EASTC  AN  C  ER

Nodal Distante
TNM Histología Tamaño  del  tumor metástasis metástasis

0 no  invasivo — —

I Invasor Cualquiera  2   No  (N0) No

IIA Invasor cm  (T1)  2   Sí,  1–3  (N1) No

cm  (T1)  2–5   No  (N0) No

IIB Invasor cm  (T2)  2–5   Sí,  1–3  (N1) No

cm  (T2)  >5   No  (N0) No

IIIA Invasor cm  (T3)  <2   Sí,  4–9  (N2) No

cm  (T1)  2–5  cm  (T2) Sí,  4–9  (N2) No

>5cm  (T3) Sí,  1–3  (N1) No

>5cm  (T3) Sí,  4–9  (N2) No

IIIB Invasor Músculo  o  piel   No/Sí  (N0,  N1, No


involucrados  (T4) N2)
IIIC Invasor Cualquier  tamaño  (Cualquier  T) Sí,  10þ  (N3) No

IV Invasor Cualquier  tamaño sí  o  no Sí

7.  ¿Por  qué  es  importante  la  estadificación  del  cáncer  de  mama?
La  etapa  del  cáncer  de  mama  se  correlaciona  con  la  probabilidad  de  recaída  y  fatalidad.  La  estadificación  de  
tumor,  ganglio  y  metástasis  (TMN)  resume  los  datos  sobre  el  tamaño  del  tumor,  las  metástasis  en  los  ganglios  
axilares  y  las  metástasis  a  distancia.  Los  cánceres  en  etapa  0  son  cánceres  no  invasivos  (p.  ej.,  carcinoma  ductal  in  
situ  [DCIS]);  los  cánceres  de  mama  en  etapa  I  son  cánceres  invasivos  con  ganglios  negativos  pequeños;  los  cánceres  
en  etapa  II  son  cánceres  de  tamaño  intermedio  con  o  sin  metástasis  en  los  ganglios  axilares;  los  cánceres  en  etapa  III  
son  cánceres  localmente  avanzados,  por  lo  general  con  metástasis  en  los  ganglios  axilares;  y  los  cánceres  en  etapa  IV  
son  aquellos  que  ya  han  hecho  metástasis  en  sitios  distantes.
Machine Translated by Google
316  CAPÍTULO  63  TERAPIA  PRIMARIA  PARA  EL  CÁNCER  DE  MAMA

8.  ¿Cuál  es  la  tasa  de  supervivencia  global  tras  el  tratamiento  multimodal  definitivo?
Etapa  0  (DCIS):  Casi  el  100  %  de  la  tasa  de  supervivencia  general  específica  de  la  enfermedad  a  10  años.
Etapa  I:  90  %  de  tasa  de  supervivencia  específica  de  la  enfermedad  general  a  10  años.

Etapa  II:  75  %  de  tasa  de  supervivencia  específica  de  la  enfermedad  general  a  10  años.

Etapa  III:  40  %  de  tasa  de  supervivencia  específica  de  la  enfermedad  general  a  10  años.

Una  mejora  incremental  gradual  en  la  supervivencia  del  cáncer  de  mama  en  los  últimos  años  se  ha  atribuido  a  la  detección  
más  temprana  y  al  tratamiento  sistémico  mejorado.  Quimioterapia  citotóxica  (p.  ej.,  CMF,  adriamicina,  paclitaxel  [Taxol])  para  

cánceres  con  receptores  de  hormonas  negativos,  terapia  hormonal  (p.  ej.,  tamoxifeno,  inhibidores  de  la  aromatasa)  para  cánceres  
con  receptores  de  hormonas  positivos  y  terapia  biológica  (p.  ej.,  herceptin  para  Her­2/  cánceres  que  sobreexpresan  neuoncogén)  
han  mejorado  la  supervivencia  libre  de  enfermedad  y  la  supervivencia  general  en  pacientes  con  cáncer  de  mama,  incluso  en  aquellas  
con  enfermedad  avanzada.

9.  ¿Cuál  es  la  diferencia  entre  los  cánceres  de  mama  no  invasivos  (in  situ)  e  invasivos?
Los  cánceres  no  invasivos  (in  situ)  son  lesiones  en  las  que  las  células  malignas  permanecen  confinadas  en  el  conducto  o  lóbulo  
en  el  que  se  originaron.  Los  cánceres  in  situ  tienen  una  probabilidad  mínima  de  propagarse  a  los  ganglios  o  sitios  distantes.  Los  
cánceres  invasivos  han  atravesado  la  membrana  basal  de  su  conducto  o  lóbulo  de  origen  y,  al  mismo  tiempo,  pueden  haber  

desarrollado  potencial  metastásico.  Los  cánceres  in  situ  tienen  células  que  en  gran  medida  son  biológicamente  incompetentes  y  no  
pueden  establecer  crecimiento  en  tejidos  distantes,  por  lo  que  incluso  si  las  células  de  estos  cánceres  tempranos  "escapan"  del  
conducto  o  son  empujadas  hacia  los  tejidos  circundantes  durante  el  muestreo  con  aguja,  siguen  siendo  incapaces  de  crear  
enfermedad  metastásica.  Por  lo  tanto,  la  razón  principal  para  tratar  el  cáncer  in  situ  es  evitar  que  se  transforme  en  un  cáncer  invasivo  
que  tenga  el  potencial  de  diseminarse  a  sitios  distantes.  La  disección  completa  de  los  ganglios  linfáticos  no  está  justificada  para  las  
lesiones  in  situ.  Sin  embargo,  el  mapeo  del  ganglio  centinela  se  puede  usar  junto  con  el  tratamiento  quirúrgico  del  CDIS,  cuando  la  
paciente  se  va  a  someter  a  una  mastectomía  o  si  también  se  sospecha  de  cáncer  invasivo.

10.  ¿Adónde  se  propaga  el  cáncer  de  mama  invasivo  (aparte  de  los  ganglios  linfáticos)?  ¿Qué  pruebas  diagnósticas  son  útiles  para  identificar  
dichas  metástasis?
El  cáncer  de  mama  se  puede  propagar  a  los  huesos,  los  pulmones,  el  hígado,  las  superficies  peritoneales  y  el  cerebro.  Las  
gammagrafías  óseas  son  bastante  sensibles  pero  menos  específicas  para  las  metástasis  óseas.  Las  radiografías  estándar  ayudan  a  
distinguir  las  metástasis  de  las  condiciones  inflamatorias  benignas.  Las  metástasis  pulmonares  se  identifican  mediante  radiografías  
de  tórax  o  tomografía  computarizada  (TC).  Las  metástasis  hepáticas  se  pueden  identificar  mediante  pruebas  de  función  hepática  
(LFT),  pero  estas  pruebas  no  son  específicas  ni  sensibles,  y  el  25  %  de  las  pacientes  con  cáncer  de  mama  con  metástasis  hepáticas  
documentadas  tienen  resultados  normales  de  LFT.  Las  pruebas  de  diagnóstico  por  imágenes  del  hígado  (tomografía  computarizada  

abdominal,  ecografía  o  resonancia  magnética)  son  más  costosas  pero  más  confiables.  Las  metástasis  cerebrales  se  visualizan  
mediante  tomografía  computarizada  o  resonancia  magnética  de  la  cabeza,  pero  solo  en  el  paciente  sintomático.

11.  ¿Qué  pruebas  se  deben  obtener  antes  de  la  cirugía  para  detectar  metástasis?
Todos  los  pacientes  con  síntomas  que  sugieran  enfermedad  metastásica  (dolor  óseo,  síntomas  pulmonares,  ictericia,  convulsiones  
o  síntomas  neurológicos  focales)  deben  ser  evaluados  completamente  después  de  que  se  haya  diagnosticado  cáncer  de  mama  
invasivo.
Un  estudio  preoperatorio  mínimo  estándar  para  la  enfermedad  invasiva  consiste  en  una  radiografía  de  tórax  y  LFT.  En  realidad,  la  
utilidad  de  estas  pruebas  entre  los  cánceres  en  etapa  temprana  es  baja.  La  radiografía  de  tórax  de  rutina  identifica  metástasis  
pulmonares  no  sospechadas  en  <1%  de  los  pacientes.  La  radiografía  de  tórax  a  menudo  se  justifica  para  la  planificación  preoperatoria  
y  es  útil  como  prueba  de  referencia  para  futuras  comparaciones.

La  medición  de  marcadores  tumorales  circulantes  (CEA,  CA­125,  etc.)  tiene  poco  o  ningún  valor  en  la  mayoría  de  las  circunstancias  
y  debe  desaconsejarse.

12.  ¿Cuáles  son  las  alternativas  para  el  tratamiento  quirúrgico  primario  de  la  mama  invasiva?
¿cáncer?

1.  Mastectomía  A.  
Mastectomía  radical  modificada  (MRM):  Mastectomía  radical  modificada  (extirpación  del
mama  y  los  ganglios  linfáticos  axilares  de  nivel  1  y  nivel  2),  ha  reemplazado  a  la  mastectomía  radical
Machine Translated by Google
CAPÍTULO  63  TERAPIA  PRIMARIA  PARA  EL  CÁNCER  DE  MAMA  317

(extirpación  de  la  mama,  los  ganglios  linfáticos  y  el  músculo  pectoral)  como  estándar  de  atención  para  
pacientes  con  enfermedad  de  ganglios  positivos  que  se  someten  a  una  mastectomía.  El  músculo  pectoral  
menor  se  puede  extirpar  con  una  morbilidad  mínima  en  una  mastectomía  radical  modificada  para  facilitar  
la  disección  de  los  ganglios  linfáticos  más  altos  (nivel  III)  (si  están  involucrados),  aunque  la  mayoría  de  los  
cirujanos  no  están  capacitados  en  esta  técnica  en  la  actualidad.
B.  Mastectomía  total  o  simple:  esta  variación  de  la  mastectomía  implica  la  extirpación  de  toda  la  mama,  pero  
elimina  la  disección  rutinaria  de  los  ganglios  axilares.  A  menudo  se  combina  con  la  biopsia  del  ganglio  
centinela  en  aquellas  con  cánceres  de  seno  con  ganglios  negativos  desde  el  punto  de  vista  clínico,  porque  
una  vez  que  se  extirpa  el  seno,  la  biopsia  del  ganglio  centinela  ya  no  es  posible.
2.  Mastectomía  parcial  (lumpectomía  o  cuadrantectomía):  la  terapia  de  conservación  mamaria  requiere  la  
extirpación  del  tumor  con  un  margen  de  tejido  mamario  normal  (márgenes  negativos)  y  va  seguida  de  
irradiación  mamaria  posoperatoria.  Los  ensayos  con  un  seguimiento  de  20  años  han  demostrado  que  la  
supervivencia  es  equivalente  para  las  pacientes  tratadas  con  lumpectomía  y  radiación,  mastectomía  total  y  
mastectomía  radical.  Se  prefiere  la  mastectomía  cuando  no  se  pueden  lograr  márgenes  negativos.  La  
lumpectomía  se  puede  combinar  con  una  biopsia  del  ganglio  centinela  si  el  paciente  tiene  ganglios  clínicamente  
negativos  o  con  una  disección  del  ganglio  axilar  si  el  paciente  tiene  enfermedad  ganglionar  documentada.

13.  ¿Qué  es  el  Programa  Nacional  de  Cirugía  Adyuvante  de  Mama  e  Intestino?
El  Programa  Nacional  Quirúrgico  Adyuvante  de  Mama  e  Intestino  (NSABP,  por  sus  siglas  en  inglés)  es  un  grupo  de  
investigadores  con  sede  en  los  EE.  UU.  que  realizó  muchos  de  los  ensayos  aleatorios  clásicos  que  han  dado  forma  
a  nuestro  enfoque  moderno  para  la  terapia  del  cáncer  de  mama.  El  NSABP  ayudó  a  demostrar  que  el  cáncer  de  
mama  puede  ser  un  problema  sistémico  en  el  momento  del  diagnóstico  y  que  las  operaciones  más  pequeñas  
pueden  ser  equivalentes  a  las  más  grandes  para  el  potencial  curativo.  Más  recientemente,  la  NSABP  informó  que  el  
tamoxifeno  puede  disminuir  las  posibilidades  de  que  una  mujer  con  un  alto  riesgo  de  desarrollar  cáncer  de  mama  lo  
haga.

14.  ¿Cuál  es  la  importancia  del  ensayo  NSABP  B­06?
NSABP  B­06  fue  un  estudio  multicéntrico  que  asignó  al  azar  a  casi  2000  mujeres  con  tumores  en  estadio  I  y  II  (<4  cm)  
a  uno  de  los  tres  brazos  de  tratamiento:  mastectomía  segmentaria  (SM;  también  conocida  como  lumpectomía)  sola,  
SM  con  radiación  y  mastectomía  total  ( TM).  Todas  las  pacientes  se  sometieron  a  disección  axilar  y  las  pacientes  con  
ganglios  positivos  recibieron  quimioterapia  adyuvante.  No  hubo  diferencia  en  las  tasas  de  supervivencia  general  entre  
los  grupos,  pero  la  radioterapia  disminuyó  la  recurrencia  local  en  pacientes  tratados  con  lumpectomía.  No  hubo  
diferencias  en  la  supervivencia  libre  de  enfermedad  ni  en  las  tasas  de  supervivencia  general  entre  ninguno  de  los  tres  
grupos  de  tratamiento,  lo  que  indica  que  la  terapia  de  conservación  de  la  mama  es  eficaz  para  lograr  el  control  de  la  
enfermedad  tanto  a  nivel  local  como  a  distancia.

15.  ¿Cuál  es  la  diferencia  entre  cuadrantectomía,  lumpectomía  y  mastectomía  parcial?

Las  diferencias  son  mínimas  porque  todas  se  refieren  a  la  extirpación  de  parte  del  seno,  solo  en  cantidades  variables,  
con  el  fin  de  tratar  el  cáncer  de  seno.  La  cuadrantectomía  original  promovida  por  los  italianos  en  la  década  de  1980  
incluía  la  escisión  de  todo  el  cuadrante  mamario  afectado,  junto  con  la  piel  que  lo  recubría.  Las  lumpectomías  estándar  
extraen  menos  tejido  y  pueden  o  no  implicar  la  extirpación  de  piel,  pero  aun  así  exigen  márgenes  quirúrgicos  negativos  
tanto  para  el  cáncer  invasivo  como  para  el  CDIS.  En  estas  operaciones  de  cáncer,  el  cirujano  tiene  la  intención  de  
lograr  márgenes  "negativos"  o  "libres",  lo  que  significa  que  el  cáncer  no  se  encuentra  rastreando  microscópicamente  
hasta  el  borde  de  los  tejidos  extirpados.  Esto  contrasta  con  la  "biopsia  quirúrgica",  en  la  que  el  cirujano  trata  de  extirpar  
la  menor  cantidad  de  tejido  posible  para  hacer  un  diagnóstico  histológico  de  una  lesión  cuestionable,  y  no  tiene  la  
intención  de  extirpar  el  exceso  de  tejido  o  lograr  un  estado  de  margen  negativo.

16.  ¿Son  algunas  pacientes  malas  candidatas  para  la  terapia  de  conservación  mamaria?
Las  contraindicaciones  (relativas  o  absolutas)  para  la  conservación  del  seno  incluyen  (1)  cánceres  que  no  se  pueden  
extirpar  con  márgenes  negativos  sin  mastectomía,  (2)  cánceres  que  son  demasiado  grandes
Machine Translated by Google
318  CAPÍTULO  63  TERAPIA  PRIMARIA  PARA  EL  CÁNCER  DE  MAMA

en  relación  con  la  mama  para  obtener  resultados  estéticos  aceptables,  (3)  cánceres  multicéntricos  y  (4)  
pacientes  que  no  desean  o  que  tienen  una  contraindicación  específica  para  la  radioterapia  adyuvante  (p.  ej.,  
esclerodermia).

17.  ¿Qué  es  la  cirugía  oncoplástica?
Esta  es  una  colección  de  procedimientos  que  utilizan  principios  oncológicos  y  reconstructivos  combinados  para  
realizar  una  mastectomía  parcial.  Se  extirpan  grandes  segmentos  de  mama  de  espesor  completo,  por  lo  general  
junto  con  la  piel  suprayacente.  Usando  técnicas  de  mastopexia,  la  glándula  se  remodela  en  la  pared  torácica  para  
preservar  la  forma  y  apariencia  natural  del  seno  sin  crear  una  muesca  de  tejido  antiestético  debajo  de  la  piel.

18.  Después  de  la  mastectomía,  ¿qué  pacientes  pueden  someterse  a  una  reconstrucción  mamaria  
inmediata  (es  decir,  durante  la  misma  operación)?
La  selección  de  pacientes  para  reconstrucción  inmediata  puede  ser  controvertida.  La  mayoría  está  de  acuerdo  en  que  
a  las  pacientes  con  cánceres  de  mama  no  invasivos  (in  situ)  o  invasivos  tempranos  (etapa  I  y  etapa  II  seleccionada)  
se  les  puede  ofrecer  una  reconstrucción  inmediata  mediante  un  colgajo  miocutáneo,  un  expansor  tisular  temporal  que  
se  reemplaza  por  un  implante  o  una  combinación  de  ambos. .  Es  desventajoso  realizar  una  reconstrucción  inmediata  
en  pacientes  con  cánceres  de  mama  localmente  avanzados  (etapa  III)  porque  estos  pacientes  en  última  instancia  
pueden  requerir  irradiación  de  la  pared  torácica  posterior  a  la  mastectomía.  La  radiación  afecta  negativamente  el  
resultado  cosmético  en  los  colgajos  de  tejido  reconstruidos,  puede  provocar  la  pérdida  del  colgajo  de  tejido  y  puede  
promover  la  contractura  capsular  alrededor  de  los  implantes.

19.  ¿Cuándo  está  indicada  la  radioterapia  de  la  pared  torácica  después  de  una  mastectomía?
La  mayoría  de  las  pacientes  de  mastectomía  no  requieren  radioterapia.  Las  excepciones  son  aquellos  con  
cánceres  invasivos  primarios  grandes,  T3  (>5  cm),  márgenes  de  mastectomía  positivos  o  cerrados,  o  cuatro  o  más  
ganglios  axilares  positivos,  todos  los  cuales  están  asociados  con  un  mayor  riesgo  de  recurrencia  locorregional.  
Actualmente  se  está  estudiando  el  posible  beneficio  de  la  radiación  con  uno  a  tres  ganglios  axilares  positivos.

20.  ¿Qué  es  el  mapeo  de  ganglios  linfáticos  centinela  para  el  cáncer  de  mama?
El  estándar  de  oro  histórico  para  la  estadificación  de  la  axila  en  pacientes  con  cáncer  de  mama  invasivo  fue  una  
disección  de  ganglios  linfáticos  axilares  de  nivel  1  +  nivel  2,  que  proporciona  información  importante  sobre  la  
estadificación,  pero  a  menudo  se  asoció  con  morbilidad,  la  más  notable  de  las  cuales  es  el  linfedema  del  brazo.  
Mapeo  de  ganglios  linfáticos  centinela,  en  el  que  se  inyecta  en  la  mama  un  trazador  radiactivo  (coloide  de  azufre  
marcado  con  tecnecio),  un  colorante  azul  (linfazurina)  o  una  combinación  de  ambos  para  identificar  los  primeros  
ganglios  axilares  aguas  arriba  a  los  que  drena  un  cáncer  de  mama  primario.  introducido  en  la  década  de  1990  y  
actualmente  es  el  tratamiento  estándar  para  la  estadificación  axilar.  Si  los  ganglios  centinela  son  negativos  para  
cáncer,  no  es  necesario  realizar  una  disección  de  ganglios  para  realizar  una  compleción.

21.  ¿Existen  riesgos  de  la  estadificación  axilar  mediante  el  mapeo  del  ganglio  centinela?
El  mapeo  del  ganglio  centinela  parece  más  apropiado  para  los  cánceres  de  mama  con  axilas  clínicamente  
negativas  para  los  ganglios.  La  técnica  puede  ser  menos  confiable  con  cánceres  grandes  (T3)  y  ganglios  
reemplazados  extensamente  por  cáncer.  Por  lo  tanto,  el  principal  riesgo  del  mapeo  del  ganglio  centinela  es  que  puede  
subestadificar  a  un  paciente  al  sugerir  que  el  cáncer  es  ganglio  negativo  cuando,  de  hecho,  hay  metástasis  
ganglionares  en  otros  ganglios  linfáticos  "no  centinela" (es  decir,  un  resultado  falso  negativo). ).  Como  resultado,  el  
paciente  puede  ser  tratado  con  una  quimioterapia  menos  agresiva  que  la  apropiada  para  minimizar  la  mortalidad  por  
cáncer.  El  riesgo  de  una  biopsia  de  ganglio  centinela  falso  negativo  es  inferior  al  5  %  para  los  cirujanos  capacitados  
en  el  procedimiento.

22.  ¿Qué  pruebas  se  deben  obtener  después  de  la  cirugía  para  detectar  metástasis  o  como
estudios  de  base  para  futuras  comparaciones?
La  utilidad  de  las  pruebas  de  cribado  metastásico  se  correlaciona  con  la  estadificación  del  tumor  locorregional  y  
ganglionar  (TN)  determinada  en  la  cirugía.  Los  pacientes  con  cánceres  más  avanzados  tienen  un  mayor  riesgo  de
Machine Translated by Google
CAPÍTULO  63  TERAPIA  PRIMARIA  PARA  EL  CÁNCER  DE  MAMA  319

desarrollar  recurrencia  del  cáncer  con  metástasis,  lo  que  hace  que  los  estudios  de  diagnóstico  adicionales  sean  valiosos.
La  gammagrafía  ósea,  la  tomografía  computarizada  (tórax,  abdomen,  pelvis)  y  la  PET  se  utilizan  entre  los  pacientes  de  
mayor  riesgo  y,  en  ocasiones,  revelan  una  enfermedad  metastásica  no  apreciada  previamente.  Algunos  médicos  también  
usan  marcadores  tumorales  circulantes  como  CEA  CA­27,  29  para  seguir  los  resultados  del  tratamiento  y  monitorear  la  
evidencia  de  recurrencia  del  cáncer,  aunque  el  valor  de  estos  estudios  es  discutible.
Por  el  contrario,  es  mejor  evitar  los  estudios  de  referencia  en  pacientes  asintomáticos  con  cánceres  tempranos  porque  
la  posibilidad  de  una  prueba  falsa  positiva  es  mayor  que  las  posibilidades  de  encontrar  metástasis  a  distancia  clínicamente  
ocultas.  Por  ejemplo,  con  el  cáncer  de  mama  en  etapa  I,  la  probabilidad  de  un  resultado  falso  positivo  en  las  gammagrafías  
óseas  supera  con  creces  la  probabilidad  de  un  resultado  positivo  verdadero.  De  manera  similar,  las  imágenes  cerebrales  (CT  
o  MRI)  deben  reservarse  para  aquellos  pacientes  con  síntomas  neurológicos  debido  al  bajo  rendimiento  en  pacientes  
asintomáticos.

23.  ¿Qué  es  la  terapia  "neoadyuvante"  para  el  cáncer  de  mama?
Los  cánceres  localmente  avanzados  pero  operables  (etapa  IIIA,  B,  C  y  algunos  en  etapa  II)  tienen  una  mayor  probabilidad  
de  recurrencia  después  de  la  cirugía.  La  terapia  neoadyuvante  (antes  de  la  cirugía),  también  conocida  como  quimioterapia  
primaria,  se  usa  para  disminuir  la  carga  tumoral  local  y  comenzar  el  tratamiento  de  la  supuesta  enfermedad  micrometastásica  
lo  antes  posible.  No  parece  que  el  momento  de  la  quimioterapia  en  relación  con  la  cirugía  influya  en  el  tiempo  de  supervivencia  
desde  el  diagnóstico,  aunque  esto  se  está  estudiando.  La  quimioterapia  neoadyuvante  puede  reducir  el  estadio  de  algunos  
cánceres  que  de  otro  modo  serían  marginales  para  la  terapia  de  conservación  de  la  mama  en  candidatos  exitosos  para  la  
lumpectomía.

24.  ¿Qué  es  el  cáncer  de  mama  «inoperable»?
El  cáncer  de  mama  inoperable  ha  avanzado  más  allá  de  los  límites  de  la  resección  quirúrgica.
La  diseminación  puede  ser  regional  (involucrando  grandes  cantidades  de  piel  de  la  pared  torácica)  o  distante  (metástasis  a  
distancia,  estadio  IV).  Las  metástasis  en  los  ganglios  linfáticos  supraclaviculares  homolaterales  son  un  indicador  de  mal  
pronóstico,  pero  actualmente  se  clasifican  como  enfermedad  en  estadio  III,  no  en  estadio  IV.  La  terapia  primaria  para  estos  
cánceres  avanzados  es  el  tratamiento  sistémico  (quimioterapia  o  terapia  hormonal)  en  lugar  de  cirugía.  La  cirugía  combinada  
con  radioterapia  se  convierte  en  una  terapia  adyuvante  para  el  control  local  de  la  enfermedad  después  de  una  buena  
respuesta  al  tratamiento  sistémico,  y  cada  vez  más  informes  abogan  por  la  resección  de  la  lesión  primaria  incluso  en  la  
enfermedad  en  estadio  IV.

25.  ¿Cómo  se  trata  el  CDIS?

Como  la  forma  más  temprana  de  cáncer  de  mama  que  requiere  tratamiento,  DCIS  tiene  la  gama  más  amplia  de  
opciones  de  tratamiento.  También  llamado  carcinoma  intraductal,  el  CDIS  se  puede  tratar  de  manera  segura  con  terapia  
de  conservación  de  la  mama  (tumorectomía  más  radiación  adyuvante),  siempre  que  la  enfermedad  se  extirpe  con  márgenes  
negativos.  Si  no  se  pueden  lograr  márgenes  negativos,  se  recomienda  la  mastectomía  para  el  control  de  la  enfermedad.  
Aunque  no  está  indicada  la  disección  axilar  para  la  estadificación,  se  puede  ofrecer  la  biopsia  del  ganglio  linfático  centinela  
si  la  mastectomía  es  la  operación  de  elección.  Debido  a  que  carece  de  potencial  metastásico,  DCIS  no  requiere  tratamiento  
farmacológico  sistémico.  El  tamoxifeno  puede  desempeñar  un  papel  en  la  prevención  del  cáncer  de  mama  y  redujo  la  
recurrencia  local  después  de  la  lumpectomía  y  la  radiación  en  el  ensayo  NSABP  B­24.

26.  ¿Se  pueden  tratar  algunos  casos  de  CDIS  mediante  lumpectomía  sin  radioterapia?
Utilizando  datos  retrospectivos  recopilados  cuidadosamente,  Silverstein  y  sus  colegas  desarrollaron  un  índice  de  pronóstico  
(sistema  de  puntuación)  para  DCIS  basado  en  el  grado  histológico,  el  tamaño  del  tumor  y  el  ancho  del  margen.  Sus  datos  
sugieren  que  las  lesiones  pequeñas  (<1  cm)  de  CDIS  que  no  son  de  alto  grado  extirpadas  con  amplios  márgenes  quirúrgicos  
no  requieren  radioterapia  además  de  la  lumpectomía.  Sin  embargo,  eliminando
Machine Translated by Google
320  CAPÍTULO  63  TERAPIA  PRIMARIA  PARA  EL  CÁNCER  DE  MAMA

el  tratamiento  de  radiación  después  de  la  lumpectomía  por  CDIS  sigue  siendo  controvertido.  Varios  estudios  
aleatorizados  de  fase  III  han  demostrado  los  beneficios  del  control  local  con  radioterapia.  Además,  un  grupo  reciente  
demostró  una  tasa  de  recurrencia  local  del  12  %  a  los  5  años  después  de  la  escisión  local  solamente,  lo  que  llevó  a  los  
autores  a  concluir  que  la  eliminación  de  la  radiación  resultó  en  tasas  de  recurrencia  inaceptablemente  altas.

27.  ¿En  qué  se  diferencia  el  manejo  del  CDIS  del  del  carcinoma  lobulillar  in  situ?
(LCIS)?
El  CDIS  se  considera  una  neoplasia  maligna  preinvasiva  o  no  invasiva.  Se  trata  quirúrgicamente  con  lumpectomía  
o  mastectomía,  con  o  sin  radioterapia,  de  manera  similar  a  cómo  se  maneja  el  cáncer  de  mama  invasivo.  El  objetivo  
general  es  la  resección  del  margen  negativo.  Por  el  contrario,  el  carcinoma  lobulillar  in  situ  (LCIS)  se  considera  una  
lesión  de  "factor  de  riesgo"  para  el  desarrollo  de  un  cáncer  de  mama  subsiguiente  y,  en  general,  no  se  considera  que  
sea  un  "cáncer"  per  se.  Si  se  diagnostica  en  una  biopsia  con  aguja  gruesa,  se  recomienda  una  biopsia  por  escisión  
posterior  para  descartar  la  existencia  simultánea  de  DCIS  o  un  cáncer  invasivo.  Si  se  identifica  LCIS  en  una  muestra  
quirúrgica,  no  se  requiere  resección  de  margen  negativo.

28.  ¿Por  qué  los  pacientes  con  LCIS  no  son  tratados  quirúrgicamente?
CLIS  no  degenera  invariablemente  en  cáncer  invasivo,  aunque  las  mujeres  con  CLIS  comprobado  por  biopsia  tienen  
hasta  un  25%  de  posibilidades  de  desarrollar  cáncer  de  mama  durante  su  vida.  El  cáncer  puede  ser  ductal  o  lobulillar  y  
puede  desarrollarse  en  cualquiera  de  los  senos.  Por  lo  tanto,  CLIS  se  considera  un  marcador  de  alto  riesgo  de  cáncer  
de  mama,  lo  que  justifica  una  vigilancia  cuidadosa  con  mamografías  seriadas,  examen  físico  y  resonancia  magnética.  
Debido  a  que  el  riesgo  futuro  de  cáncer  de  mama  es  el  mismo  para  ambas  mamas,  la  mastectomía  bilateral  sería  el  
único  procedimiento  quirúrgico  lógico  para  esta  afección.  Esta  terapia  agresiva  no  está  justificada  en  la  mayoría  de  los  
pacientes,  aunque  se  puede  considerar  en  individuos  de  alto  riesgo  caso  por  caso.

29.  ¿Se  pueden  usar  medicamentos  para  prevenir  el  cáncer  de  mama  entre  mujeres  con  alto  riesgo?
En  el  ensayo  de  prevención  de  tamoxifeno  NSABP  P­01,  las  mujeres  con  mayor  riesgo  de  desarrollar  cáncer  de  mama  
(>1,66  %  de  riesgo  a  5  años)  desarrollaron  menos  cáncer  de  mama  cuando  recibieron  tamoxifeno  en  comparación  con  
placebo.  Para  las  mujeres  con  LCIS,  la  incidencia  de  cáncer  de  mama  a  5  años  fue  del  6,8  %  en  el  grupo  de  placebo  y  
del  2,5  %  en  el  grupo  de  tamoxifeno,  lo  que  representa  una  reducción  absoluta  del  56  %  en  los  cánceres  de  mama.  Sin  
embargo,  la  cantidad  de  cánceres  de  mama  que  se  previnieron  rivalizó  con  la  cantidad  de  complicaciones  asociadas  
con  el  tamoxifeno,  incluidos  los  cánceres  de  endometrio  y  los  eventos  trombóticos.
Aún  no  se  ha  observado  ningún  beneficio  de  supervivencia  con  la  profilaxis  con  tamoxifeno.  En  este  momento,  
a  las  mujeres  con  LCIS  y  sin  contraindicaciones  médicas  importantes  se  les  puede  ofrecer  tamoxifeno  como  una  
opción  para  la  prevención  del  cáncer,  aunque  pueden  rechazarlo  razonablemente  cuando  se  les  presenten  los  
datos  completos.

PUNTOS  CLAVE:  TERAPIA  PRIMARIA  PARA  EL  CÁNCER  DE  MAMA
1.  Históricamente,  la  biopsia  por  escisión  fue  el  estándar  de  oro  para  el  diagnóstico  de  cáncer  de  mama.

2.  Ahora,  el  método  de  diagnóstico  inicial  preferido  es  la  biopsia  con  aguja  gruesa  o  FNA.

3.  Las  alternativas  quirúrgicas  para  el  tratamiento  del  cáncer  de  mama  invasivo  primario  son  la  mastectomía  
radical  modificada,  la  mastectomía  total  o  la  mastectomía  parcial.

4.  El  ensayo  NSABP  B­06  no  encontró  diferencias  en  la  supervivencia  general  en  mujeres  con  cáncer  de  mama  en  
estadio  I  y  II  que  se  sometieron  a  SM,  SM  con  radiación  y  TM,  pero  la  radiación  disminuyó  la  recurrencia  local  en  
la  mama  tumorectomizada.
Machine Translated by Google
CAPÍTULO  63  TERAPIA  PRIMARIA  PARA  EL  CÁNCER  DE  MAMA  321

SITIO  WEB

www.acssurgery.com

BIBLIOGRAFÍA

1.  Anderson  BO,  Calhoun  KE,  Rosen  EL:  Conceptos  en  evolución  en  el  manejo  de  la  neoplasia  lobular.  J  Natl  Compr  Canc  Netw  4(5):511­522,  2006.

2.  Buzdar  AU:  tratamiento  de  quimioterapia  preoperatoria  del  cáncer  de  mama:  una  revisión.  Cáncer  110:2394­2407,  2007.

3.  Calhoun  KE,  Anderson  BO:  Mastectomía  profiláctica  y  el  manejo  clínico  de  pacientes  con  cáncer  de  mama  de  alto  riesgo.  Comunidad  Oncol  
3(6):379­382,  2006.

4.  Chen  CY,  Calhoun  KE,  Masetti  R  et  al.:  Cirugía  conservadora  de  mama  oncoplástica:  un  renacimiento  de  la
Tratamiento  quirúrgico  basado.  Minerva  Chir  61:421­434,  2006.

5.  Fisher  B,  Anderson  S,  Bryant  J  et  al.:  Veinte  años  de  seguimiento  de  un  ensayo  aleatorizado  que  comparó  la  mastectomía  total,  la  lumpectomía  y  
la  lumpectomía  más  irradiación  para  el  tratamiento  del  cáncer  de  mama  invasivo.  N  Engl  J  Med  347:1233­1241,  2002.

6.  Fisher  B,  Costantino  JP,  Wickerham  DL  et  al.:  Tamoxifeno  para  la  prevención  del  cáncer  de  mama:  estado  actual  del  Estudio  P­1  del  Proyecto  
Nacional  Quirúrgico  Adyuvante  de  Mama  e  Intestino.  J  Natl  Cancer  Inst  97:1652­1662,  2005.

7.  Lehman  CD,  Gatsonis  C,  Kuhl  CK  et  al.:  Evaluación  por  resonancia  magnética  de  la  mama  contralateral  en  mujeres  con
cáncer  de  mama  diagnosticado.  N  Engl  J  Med  356:1295­1303,  2007.

8.  Lyman  GH,  Giuliano  AE,  Somerfield  MR  et  al.:  Recomendaciones  de  la  guía  de  la  Sociedad  Estadounidense  de  Oncología  
Clínica  para  la  biopsia  del  ganglio  linfático  centinela  en  el  cáncer  de  mama  en  estadio  temprano.  J  Clin  Oncol  23:7703­7720,  
2005.

9.  Morrow  M,  Strom  EA,  Bassett  LW  et  al.:  Estándar  para  la  terapia  de  conservación  mamaria  en  el  manejo  de
Carcinoma  de  mama  invasivo.  Ca  Cancer  J  Clin  52:277­300,  2002.

10.  Saslow  D,  Boetes  C,  Burke  W  et  al.:  Directrices  de  la  Sociedad  Americana  del  Cáncer  para  el  cribado  de  mamas  con  resonancia  magnética  como
complemento  de  la  mamografía.  CA  Cancer  J  Clin  57:75­89,  2007.

11.  Silverstein  MJ:  Índice  de  pronóstico  de  la  Universidad  del  Sur  de  California/Van  Nuys  para  el  carcinoma  ductal  in  situ  de  la  mama.  Am  J  Surg  
186(4):337­343,  2003.

12.  Truong  PT,  Olivotto  IA,  Kader  HA  et  al.:  Selección  de  pacientes  con  cáncer  de  mama  con  tumores  T1­2  y  de  uno  a  tres  ganglios  axilares  
positivos  con  alto  riesgo  de  recurrencia  locorregional  posmastectomía  para  radioterapia  adyuvante.
Int  J  Radiat  Oncol  Biol  Phys  61:1337­1347,  2005.

13.  Wong  JS,  Kaelin  CM,  Troyan  SL  et  al.:  Estudio  prospectivo  de  escisión  amplia  sola  para  carcinoma  ductal  in  situ  de  mama.  J  Clin  Oncol  
24:1031­1036,  2006.

14.  Veronesi  U,  Cascinelli  N,  Mariani  L  et  al.:  Veinte  años  de  seguimiento  de  un  estudio  aleatorizado  que  comparó  la  cirugía  conservadora  de  la  
mama  con  la  mastectomía  radical  para  el  cáncer  de  mama  temprano.  N  Engl  J  Med  34:1232,  2002.

15.  Yen  TW,  Kuerer  HM,  Ottesen  RA  et  al.:  Impacto  de  los  resultados  de  ensayos  clínicos  aleatorizados  en  el
red  integral  de  cáncer  sobre  el  uso  de  tamoxifeno  después  de  la  cirugía  de  mama  por  carcinoma  ductal  in  situ.  J  Clin  Oncol  25:3251­3258,  2007.
Machine Translated by Google

VI.  OTROS  CÁNCERES

¿QUÉ  ES  EL  CÁNCER?
CAPITULO  
64

Karyn  Stitzenberg,  MD,  MPH,  y  John  A.  Ridge,  MD,  PhD

1.  ¿Qué  es  una  neoplasia?
Una  neoplasia  (tumor)  es  un  nuevo  crecimiento  en  el  que  las  células  crecen  progresivamente  en  condiciones  que  
no  estimulan  el  crecimiento  de  las  células  normales.  Una  neoplasia  maligna  (cáncer)  está  compuesta  por  células  que  
invaden  otros  tejidos  y  se  diseminan.

2.  ¿Qué  tipos  de  cánceres  hay?

Los  tumores  malignos  de  las  células  epiteliales  (tejido  superficial)  son  carcinomas.  Los  tumores  malignos  de  las  
células  mesenquimales  (tejido  conectivo)  son  sarcomas.  Los  carcinomas  y  los  sarcomas  son  tumores  sólidos.  Las  
neoplasias  malignas  hematológicas,  como  la  leucemia,  son  tumores  líquidos  de  origen  mesenquimatoso.

3.  ¿Qué  pasa  con  los  cánceres  de  piel?

El  melanoma  se  comporta  de  manera  similar  a  otros  tumores  sólidos.  Por  el  contrario,  la  mayoría  de  los  cánceres  
de  piel  de  células  basales  y  escamosos  son  potencialmente  mortales  solo  si  se  descuidan.  Ocurren  en  gran  número  y  rara  
vez  son  fatales  con  el  tratamiento  adecuado.  Aunque  los  principios  generales  del  tratamiento  del  cáncer  se  aplican  a  los  
cánceres  de  piel  distintos  del  melanoma,  por  lo  general  no  se  consideran  en  la  misma  clase  que  otros  tumores  sólidos.

4.  ¿Por  qué  el  cáncer  es  malo  para  usted?
No  hay  una  respuesta  sencilla.  El  reemplazo  de  tejido  normal  por  tumor  eventualmente  causa  disfunción  orgánica.  
Si  un  tumor  supera  su  suministro  de  sangre  y  se  vuelve  necrótico,  se  produce  una  inflamación  local.  A  menudo,  la  
obstrucción  (con  compromiso  de  la  luz)  del  tracto  gastrointestinal  (GI),  los  conductos  biliares  o  las  vías  respiratorias  se  
desarrolla  a  medida  que  crece  el  tumor.  Ocasionalmente,  el  cáncer  sangra  (pero  el  sangrado  que  amenaza  la  vida  es  
raro).  La  invasión  o  inflamación  de  los  nervios  generalmente  causa  dolor,  que  puede  ser  insoportable.  Los  cánceres  
también  pueden  elaborar  factores  humorales  (p.  ej.,  gastrina)  que  causan  síntomas.

5.  ¿Todos  los  cánceres  ponen  en  peligro  la  vida?
El  cáncer  es  una  enfermedad  mortal.  Es  poco  común  que  un  paciente  con  un  cáncer  no  tratado  muera  por  otra  causa.  
Aún  así,  actualmente  más  del  50%  de  los  pacientes  con  cáncer  en  los  Estados  Unidos  se  curan.

6.  ¿Cómo  comienzan  los  cánceres?

Nadie  lo  sabe,  pero  las  células  comienzan  a  crecer  en  circunstancias  en  las  que  no  deberían  hacerlo.  Dejan  de  responder  
a  las  señales  contra  el  crecimiento,  promueven  su  propio  suministro  de  sangre,  aparentemente  pueden  replicarse  sin  fin  
y  no  sufren  muerte  celular  programada  (apoptosis).

7.  ¿Este  proceso  es  el  mismo  para  todos  los  cánceres?
No,  el  orden  en  que  ocurren  estos  cambios  parece  variar  entre  tipos  de  cáncer  e  incluso  entre  tumores  individuales  
con  el  mismo  tipo  histológico.  Ocasionalmente,  una  sola  mutación  por  sí  sola  causa  cáncer,  pero  generalmente  están  
involucradas  muchas  alteraciones  genéticas.

8.  ¿Cómo  se  diagnostica  el  cáncer?
El  cáncer  se  diagnostica  histopatológicamente.  Los  patólogos  tienen  una  variedad  de  criterios  para  diagnosticar  el  
cáncer.  Buscan  crecimiento  desordenado,  cambios  en  el  núcleo  celular,  cambios  en  la  estructura  de  la  célula  e  invasión  
de  células  anormales  en  tejidos  cercanos  o  distantes,  vasos  sanguíneos  y  linfáticos  adyacentes.

323
Machine Translated by Google
324  CAPÍTULO  64  ¿QUÉ  ES  EL  CÁNCER?

9.  ¿Qué  es  la  inmunohistoquímica?
Los  patólogos  usan  anticuerpos  marcados  dirigidos  contra  ciertas  proteínas  celulares  para  ayudarlos  a  identificar  el  tipo  de  
célula.  Por  ejemplo,  si  un  anticuerpo  dirigido  contra  el  receptor  de  estrógeno  tiñe  las  células  cancerosas  en  un  ganglio  linfático,  
es  probable  que  el  cáncer  se  haya  diseminado  desde  el  seno.

10.  ¿Qué  es  una  metástasis?

Una  metástasis  es  un  grupo  de  células  cancerosas  que  se  han  propagado  desde  la  ubicación  original  del  tumor  a  otro  órgano  
no  adyacente.  Las  células  de  la  metástasis  son  similares  a  las  del  tumor  original.
Las  metástasis  se  desarrollan  a  partir  de  células  cancerosas  que  viajan  por  el  torrente  sanguíneo  o  los  vasos  linfáticos.  Sin  
embargo,  la  mayoría  de  las  células  cancerosas  en  el  torrente  sanguíneo  y  los  vasos  linfáticos  no  son  capaces  de  crear  metástasis.
Solo  en  raras  ocasiones  las  células  del  torrente  sanguíneo  realmente  se  convierten  en  implantes  tumorales  distantes.  Este  
proceso  se  conoce  como  la  cascada  de  invasión­metástasis.  Hay  muchos  pasos  en  la  cascada.  Las  células  cancerosas  deben  
descansar  en  tejidos  propicios  para  su  crecimiento  y  adquirir  rasgos  que  les  permitan  colonizar  y  desarrollar  un  suministro  de  
sangre  en  el  nuevo  entorno.

11.  ¿Todos  los  cánceres  se  propagan?
Alrededor  del  25%  de  los  pacientes  con  tumores  sólidos  tienen  metástasis  detectables  en  el  momento  del  diagnóstico.
Menos  del  50%  del  resto  desarrollan  metástasis  durante  el  curso  del  tratamiento.  En  el  momento  del  diagnóstico,  el  cáncer  
suele  tener  al  menos  1  cm  de  diámetro  (ya  menudo  es  mucho  más  grande)  y  contiene  millones  de  células.  Es  sorprendente  
que  no  se  hayan  producido  metástasis  en  todos  los  pacientes  en  el  momento  del  diagnóstico.

12.  ¿Este  proceso  tiene  algún  efecto  sobre  cómo  los  cirujanos  tratan  a  los  pacientes  con  cáncer?
Las  operaciones  para  tratar  afecciones  benignas  están  diseñadas  para  extirpar  la  menor  cantidad  de  tejido  posible  mientras  
se  crea  un  estado  fisiológico  o  anatómico  nuevo  y  deseable.  Las  operaciones  de  cáncer,  por  otro  lado,  están  diseñadas  para  
eliminar  la  mayor  cantidad  de  tejido  posible  y  dejar  al  paciente  con  una  función  aceptable.  Las  operaciones  de  cáncer  suelen  
extirpar  el  tumor  primario  y  los  ganglios  linfáticos  que  drenan  el  sitio  primario.  La  resección  quirúrgica  es  el  tratamiento  más  
efectivo  para  los  tumores  sólidos.

13.  ¿Por  qué  se  extirpan  los  ganglios  linfáticos  durante  las  operaciones  de  cáncer?
Hace  más  de  100  años,  William  S.  Halsted  (si  no  sabe  la  respuesta  a  alguna  pregunta  histórica  planteada  sobre  las  rondas,  
siempre  debe  adivinar  ''Halsted'')  se  dio  cuenta  de  que  la  recurrencia  del  tumor  en  la  pared  torácica  después  de  la  mastectomía  
estaba  relacionada  con  el  tumor.  en  los  ganglios  linfáticos  restantes.
Halsted  creía  que  el  cáncer  de  mama  se  diseminaba  de  manera  ordenada  (o  tal  vez  incluso  de  forma  contigua)  
desde  el  tumor  primario  hasta  los  ganglios  linfáticos  regionales  y,  finalmente,  a  sitios  distantes.  Popularizó  la  disección  en  bloque  
de  la  mama  con  los  ganglios  linfáticos  axilares  para  el  tratamiento  del  cáncer  de  mama.  Conceptualmente,  este  enfoque  se  
adoptó  para  el  tratamiento  quirúrgico  de  la  mayoría  de  los  tumores  sólidos.

14.  ¿Qué  es  un  ganglio  linfático  centinela?
Los  ganglios  linfáticos  centinela  son  la  primera  parada  para  que  las  células  tumorales  hagan  metástasis  a  través  de  los  
vasos  linfáticos  del  tumor  primario.  A  menudo  hay  más  de  un  ganglio  centinela,  incluso  para  un  tumor  pequeño.
Si  no  hay  tumor  presente  en  uno  o  más  ganglios  linfáticos  centinela,  es  poco  probable  que  el  tumor  esté  presente  en  cualquiera  
de  los  otros  ganglios.  El  mapeo  de  ganglios  linfáticos  centinela  se  ha  utilizado  para  cánceres  de  muchos  órganos  (incluyendo  la  
piel,  el  seno,  el  colon,  la  tiroides  y  la  cabeza  y  el  cuello).  La  evaluación  cuidadosa  de  los  ganglios  linfáticos  centinela  ha  demostrado  
ser  fiable  en  la  estadificación  del  melanoma  y  el  cáncer  de  mama.  Con  un  ganglio  centinela  libre  de  tumores,  los  pacientes  con  
melanoma  y  cáncer  de  mama  ahora  pueden  evitar  muchas  más  linfadenectomías  morbosas  (disecciones  de  ganglios  linfáticos).

15.  ¿Los  tumores  sólidos  se  diseminan  de  manera  ordenada?
No  necesariamente.  Otra  visión  del  comportamiento  del  cáncer  de  mama  se  hizo  popular  en  la  década  de  1970.
Bernard  Fisher  postuló  que  el  cáncer  está  muy  extendido  en  sus  inicios.  Afirmó  que  ''el  cáncer  de  mama  es  una  enfermedad  
sistémica. . .  y  que  es  poco  probable  que  las  variaciones  en  el  tratamiento  regional  local  efectivo  afecten  sustancialmente  la  
supervivencia”.
Machine Translated by Google
CAPÍTULO  64  ¿QUÉ  ES  EL  CÁNCER?  325

16.  ¿Cómo  afectan  al  tratamiento  estos  diferentes  modelos  de  cáncer?

Los  cirujanos  que  creen  que  los  tumores  se  diseminan  de  manera  ordenada  tienden  a  realizar  disecciones  completas  de  
los  ganglios  linfáticos  junto  con  la  resección  del  tumor  primario.  Por  lo  general,  creen  que  la  linfadenectomía  curará  a  
algunos  pacientes  que  tienen  afectación  de  los  ganglios  linfáticos  sin  metástasis  a  distancia  y  que  la  recurrencia  local  o  
regional  es  una  causa  de  muerte  prevenible.
Los  cirujanos  que  creen  que  las  metástasis  de  los  ganglios  linfáticos  son  simplemente  marcadores  de  una  enfermedad  
sistémica  suelen  ser  mucho  menos  agresivos  al  realizar  disecciones  de  ganglios  linfáticos  porque,  en  su  opinión,  la  
extirpación  de  los  ganglios  linfáticos  que  contienen  un  tumor  no  curará  a  los  pacientes  que  probablemente  ya  tengan  
enfermedad  metastásica.

17.  ¿Sabemos  qué  modelo  es  el  correcto?

Ambos  son  probablemente  inadecuados.  Incluso  cuando  tienen  afectación  de  los  ganglios  linfáticos,  algunos  
tumores  sólidos  (p.  ej.,  cáncer  escamoso  de  cabeza  y  cuello,  cáncer  de  colon)  a  menudo  no  tienen  metástasis  a  
distancia.  Su  propagación  parece  ser  un  proceso  ordenado.  Otros  tumores  sólidos  (p.  ej.,  cáncer  de  pulmón  de  células  
pequeñas  y  cáncer  de  próstata)  a  menudo  metastatizan  ampliamente  incluso  cuando  son  pequeños.  Para  tales  tipos  de  
cáncer,  la  afectación  de  los  ganglios  linfáticos  es  un  signo  confiable  de  cáncer  diseminado.  Los  sarcomas  rara  vez  hacen  
metástasis  en  los  ganglios  linfáticos,  pero  los  pacientes  pueden  desarrollar  metástasis  a  distancia  limitadas  a  los  
pulmones  únicamente.  Sorprendentemente,  estos  pacientes  a  veces  se  curan  mediante  la  resección  de  las  lesiones  
pulmonares  distantes.

18.  ¿De  qué  otra  forma  se  pueden  tratar  los  tumores  sólidos  con  intención  curativa?

En  lugar  de  la  extirpación  quirúrgica  del  tumor  primario  y  los  ganglios  linfáticos  apropiados,  toda  el  área  puede  
tratarse  con  radiación  curativa.  Algunos  tipos  de  cáncer  responden  más  a  la  radiación  que  otros.  Los  efectos  
secundarios  del  tratamiento  con  radiación  curativa  son  formidables.  Similares  a  las  de  la  cirugía,  deben  ser  explicadas  al  

paciente.  Cuando  la  radiación  mata  el  cáncer,  daña  los  tejidos  normales  adyacentes.  El  daño  a  los  tejidos  normales  
continúa  a  lo  largo  de  la  vida  del  paciente.  Aunque  los  radioterapeutas  están  mejorando  en  la  dirección  de  sus  haces,  la  
tolerancia  de  los  tejidos  cercanos  a  la  radiación  sigue  siendo  el  factor  limitante  en  el  tratamiento  de  cánceres  con  
radiación  sola.  Por  la  misma  razón,  la  radiación  generalmente  solo  se  puede  usar  una  vez  para  cualquier  área  de  
tratamiento  determinada.

19.  ¿Qué  es  la  terapia  adyuvante?
Adyuvante  significa  ''asistir  o  ayudar'',  pero  usamos  este  término  para  referirnos  a  ayudar  después  del  control  quirúrgico  
o  radioterapéutico  del  tumor  primario.  La  quimioterapia  adyuvante  tiene  un  beneficio  documentado  en  el  tratamiento  del  
cáncer  de  mama,  cáncer  colorrectal,  cáncer  de  estómago,  cáncer  de  páncreas  y  tumores  de  ovario  y  testicular.  La  
radioterapia  adyuvante  es  eficaz  para  reducir  el  riesgo  de  recurrencia  del  tumor  alrededor  del  sitio  quirúrgico.  La  
radioterapia  adyuvante  se  usa  a  menudo  en  el  tratamiento  de  pacientes  con  cánceres  y  sarcomas  de  recto,  mama,  
cabeza  y  cuello  y  estómago.  Conceptualmente,  tanto  la  cirugía  como  la  radiación  son  terapias  locales  y  regionales.  
Aunque  la  quimioterapia  es  obviamente  un  tratamiento  sistémico,  puede  ayudar  a  sensibilizar  los  tumores  a  la  radiación.

20.  ¿Qué  es  la  terapia  neoadyuvante?
El  término  neoadyuvante  en  realidad  no  significa  nada,  pero  a  menudo  se  usa  para  describir  la  quimioterapia  
o  la  radioterapia  administrada  antes  del  tratamiento  que  realmente  pretende  ser  curativo.  Podría  describirse  con  
mayor  precisión  como  tratamiento  de  "inducción".

21.  ¿Qué  tratamientos  contra  el  cáncer  están  disponibles  además  de  la  cirugía,  la  radioterapia  y  la  quimioterapia  citotóxica?

La  manipulación  hormonal  se  ha  utilizado  durante  décadas  para  retardar  el  crecimiento  de  algunos  tumores.
La  estimulación  del  sistema  inmunitario  del  paciente  para  combatir  el  cáncer  es  potencialmente  prometedora.  Este  
enfoque  puede  implicar  vacunas,  formación  de  células  T  o  mejora  de  la  respuesta  inmunitaria.
Los  nuevos  tipos  de  agentes  anticancerígenos  incluyen  fármacos  que  interfieren  con  la  angiogénesis  tumoral,  anticuerpos  
y  otros  fármacos  que  interfieren  con  los  receptores  del  factor  de  crecimiento,  otros  tipos  de  fármacos  que  alteran  la  
señalización  intracelular  y  fármacos  que  restablecen  el  control  del  ciclo  celular.  La  limitación  de  todos  estos
Machine Translated by Google
326  CAPÍTULO  64  ¿QUÉ  ES  EL  CÁNCER?

enfoques  reside  en  nuestra  incapacidad  para  especificar  un  objetivo  único  para  las  células  cancerosas.  Por  lo  tanto,  
mientras  dañan  las  células  tumorales,  los  tratamientos  también  dañan  el  resto  del  cuerpo  con  una  toxicidad  potencialmente  fatal.

22.  ¿Qué  es  la  terapia  dirigida?
El  término  terapia  dirigida  se  refiere  a  medicamentos  u  otros  agentes  que  interactúan  con  moléculas  específicas  u  
"objetivos"  en  la  célula  cancerosa,  lo  que  interfiere  con  la  capacidad  del  tumor  para  crecer,  dividirse  y  comunicarse  
con  otras  células.  La  mayoría  de  los  agentes  dirigidos  consisten  en  un  anticuerpo  monoclonal  que  interactúa  con  un  
objetivo  molecular  en  la  célula  cancerosa.  Las  terapias  dirigidas  están  dirigidas  a  moléculas  exclusivas  de  las  células  
cancerosas,  lo  que  les  permite  atacar  los  tumores  sin  atacar  las  células  normales.  Los  ejemplos  de  dianas  moleculares  
incluyen  ciertos  tipos  de  tirosina  quinasas  y  receptores  de  factores  de  crecimiento.

23.  ¿El  cuerpo  combate  el  cáncer  por  sí  solo?
Ciertamente.  Algunos  científicos  creen  que  los  cánceres  tempranos  son  extirpados  regularmente  por  el  sistema  
inmunitario  (ya  que  "capturamos"  cáncer  todos  los  días)  y  que  los  cánceres  clínicos  reflejan  una  falla  en  la  
vigilancia  inmunológica.  Los  pacientes  inmunocomprometidos  con  trasplantes  o  síndrome  de  inmunodeficiencia  
adquirida  (SIDA)  tienen  una  vigilancia  inmunológica  menos  eficaz.  Estos  pacientes  desarrollan  cánceres  con  una  
frecuencia  aterradora.  De  hecho,  el  rechazo  y  la  sepsis  ya  no  son  las  causas  más  comunes  de  muerte  entre  los  
pacientes  trasplantados  de  riñón;  el  cáncer  es.  Ocurren  "remisiones  espontáneas"  de  melanoma  y  carcinoma  de  
células  renales  y  presumiblemente  son  mediadas  inmunológicamente.

24.  ¿Qué  es  un  linfocito  infiltrante  de  tumores?
Los  linfocitos  infiltrantes  de  tumores  (TIL,  por  sus  siglas  en  inglés)  son  células  linfoides  (células  T)  que  se  infiltran  en  
tumores  sólidos  y  parecen  reaccionar  naturalmente  a  los  antígenos  tumorales  autólogos.  En  comparación  con  los  
linfocitos  circulantes,  los  TIL  se  dirigen  de  manera  más  agresiva  al  cáncer.  Se  cree  que  estos  TIL  desempeñan  un  
papel  importante  en  la  respuesta  inmunitaria  del  cuerpo  al  cáncer.  En  general,  la  presencia  de  TIL  circulantes  es  un  
buen  signo  de  pronóstico.  El  campo  de  la  inmunoterapia  adoptiva  se  centra  en  aprovechar  el  poder  de  los  TIL  para  
tratar  el  cáncer.

25.  ¿Qué  son  los  tratamientos  paliativos?
Paliativo  significa  ''proporcionar  alivio  al  no  curar''.

26.  ¿Cuáles  son  algunos  ejemplos  de  procedimientos  paliativos?
La  resección  del  tumor  primario  ante  metástasis  a  distancia  se  puede  realizar  para  tratar  el  sangrado  o  la  
obstrucción.  Los  procedimientos  para  evitar  la  obstrucción  intestinal  o  biliar  en  pacientes  con  cáncer  irresecable  
son  comunes.  Las  traqueotomías  se  crean  para  pacientes  que  no  pueden  respirar  debido  a  una  obstrucción  de  las  
vías  respiratorias  superiores.  Las  sondas  de  alimentación  pueden  permitir  la  nutrición  enteral  en  pacientes  que  no  
pueden  comer.  La  eliminación  de  metástasis  cerebrales  aisladas  a  menudo  mejora  la  calidad  de  vida  del  paciente.  
Muchos  pacientes  con  tumores  endocrinos  funcionantes  se  benefician  de  la  reducción  de  la  masa  tumoral.

27.  ¿Qué  es  la  cirugía  citorreductora?
Los  procedimientos  citorreductores  ("debulking")  están  diseñados  para  disminuir  la  carga  tumoral.  La  simple  
reducción  del  volumen  del  tumor  rara  vez  es  suficiente  para  prolongar  la  supervivencia.  Para  que  la  cirugía  
citorreductora  sea  beneficiosa,  la  terapia  no  quirúrgica  (adyuvante)  debe  ser  muy  eficaz,  como  la  radiación  para  el  
glioblastoma  o  la  quimioterapia  para  el  cáncer  de  ovario.

CONTROVERSIA

28.  ¿El  tratamiento  de  los  ganglios  linfáticos  axilares  para  el  cáncer  de  mama  tiene  valor  terapéutico  o  simplemente  ayuda  
a  seleccionar  pacientes  que  deben  recibir  quimioterapia?
Quienes  creen  que  la  disección  de  los  ganglios  linfáticos  axilares  solo  proporciona  información  sobre  el  
comportamiento  del  tumor  en  lugar  de  un  beneficio  terapéutico,  suelen  citar  la  National  Surgical  Adjuvant  Breast  and
Machine Translated by Google
CAPÍTULO  64  ¿QUÉ  ES  EL  CÁNCER?  327

Ensayo  B­04  del  Programa  Intestinal  (NSABP).  No  hubo  diferencias  estadísticamente  significativas  
en  las  curvas  de  supervivencia  entre  los  pacientes  cuyas  axilas  fueron  tratadas  inicialmente  y  los  pacientes  
que  recibieron  un  tratamiento  tardío  en  la  axila.  Además  de  otros  problemas,  sin  embargo,  el  estudio  carecía  
del  poder  para  probar  el  punto.  Para  tener  un  90  %  de  posibilidades  de  detectar  una  diferencia  de  supervivencia  
del  7  %  entre  los  grupos  de  tratamiento,  el  NSABP  debería  haber  inscrito  2000  pacientes  (no  solo  550)  en  
cada  grupo.  Por  lo  tanto,  es  posible  que  no  se  haya  reconocido  una  ventaja  de  supervivencia  sustancial  
resultante  de  la  disección  axilar.  El  estudio  no  fue  diseñado  para  probar  que  los  dos  enfoques  fueran  
equivalentes  y  ha  sido  sobreinterpretado.  Se  necesita  un  ensayo  mucho  más  grande  para  probar  la  equivalencia  
que  para  mostrar  una  diferencia.  De  hecho,  los  ensayos  aleatorios  posteriores  en  el  tratamiento  del  cáncer  de  
mama  y  los  estudios  observacionales  demuestran  una  ventaja  de  supervivencia  independiente  conferida  por  
el  tratamiento  de  la  axila.  Esta  experiencia  con  el  cáncer  de  mama  refuerza  la  importancia  de  comprender  
realmente  los  ensayos  clínicos.

BIBLIOGRAFÍA

1.  Bland  KI,  Scott­Conner  CEH,  Menck  H  et  al.:  Disección  axilar  en  cirugía  conservadora  de  mama  para  estado  I  y
II  cáncer  de  mama:  un  estudio  de  la  base  de  datos  nacional  del  cáncer  sobre  los  patrones  de  omisión  y  las  implicaciones  para  la  supervivencia.  
J  Am  Coll  Surg  188:586­596,  1999.

2.  Cabanes  PA,  Salmon  RJ,  Vilcoq  JR  et  al.:  Valor  de  la  disección  axilar  además  de  lumpectomía  y  radioterapia  en  el  cáncer  de  
mama  temprano.  Lancet  339:1245­1248,  1992.

3.  Fisher  B,  Jeong  JH,  Anderson  S  et  al.:  Seguimiento  de  veinticinco  años  de  un  ensayo  clínico  aleatorizado  que  comparó  la  mastectomía  
radical,  la  mastectomía  total  y  la  mastectomía  total  seguida  de  irradiación.  N  Engl  J  Med  312:674­681,  1985.

4.  Hanahan  D,  Weinberg  RA:  Las  características  del  cáncer.  Celda  100:57­70,  2000.

5.  Harris  JR,  Osteen  RT:  Los  pacientes  con  cáncer  de  mama  temprano  se  benefician  de  un  tratamiento  axilar  efectivo.  Cáncer  de  Mama  Res.  
Tratar  5:17­21,  1985.

6.  Hellman  S:  Historia  natural  de  los  cánceres  de  mama  pequeños.  J  Clin  Oncol  12:2229­2234,  1994.

7.  Morton  DL,  Thompson  JF,  Cochran  AJ  et  al.:  Biopsia  de  ganglio  centinela  u  observación  de  ganglios  en  el  melanoma.  N  Inglés
J  Med  355:1307­1317,  2006.

8.  Rosenberg  SA:  Avances  en  inmunología  e  inmunoterapia  de  tumores  humanos.  Naturaleza  411:380­384,  2001.

9.  Scheel  C,  Onder  T,  Karnoub  A  et  al .:  Adaptación  versus  selección:  los  orígenes  del  comportamiento  metastásico.  Cáncer  Res  67:11476­11479,  
2007.

10.  Veronesi  U,  Paganelli  G,  Viale  G  et  al .:  Biopsia  de  ganglio  linfático  centinela  como  procedimiento  de  estadificación  en  el  cáncer  de  mama:
actualización  de  un  estudio  controlado  aleatorizado.  Lanceta  Oncol.  7:983­990,  2006.

11.  Whelan  TJ,  Julian  J,  Wright  J:  ¿La  radioterapia  locorregional  mejora  la  supervivencia  en  el  cáncer  de  mama?  un  meta
análisis.  J  Clin  Oncol  18:1220­1229,  2000.
Machine Translated by Google

MELANOMA
CAPITULO  
65

Martín  D.  McCarter,  MD

1.  ¿Qué  es  el  melanoma?

El  término  melanoma  implica  un  tumor  maligno;  el  melanoma  maligno  es  redundante.  El  melanoma,  el  más  maligno  
de  todos  los  cánceres  de  piel,  generalmente  se  forma  a  partir  de  un  nevus  o  lunar  preexistente,  pero  puede  desarrollarse  de  
novo.

2.  ¿Cuál  es  la  incidencia  del  melanoma?

Es  el  sexto  cáncer  más  común  en  los  Estados  Unidos  y  actualmente  es  el  cáncer  con  el  aumento  más  rápido  en  la  
incidencia  en  los  Estados  Unidos.  El  riesgo  de  por  vida  en  el  año  2000  era  de  1  en  75  frente  a  1  en  150  en  1985.  Cada  año  
se  informan  más  de  59  000  casos  nuevos  de  melanoma  con  más  de  8000  muertes  a  causa  de  la  enfermedad.

3.  ¿Cuáles  son  los  tipos  de  lunares?  ¿Cuáles  son  más  propensos  al  cambio  maligno?
Intradérmica:  la  forma  más  benigna.
De  unión:  el  componente  de  unión  puede  ser  el  sitio  de  formación  del  melanoma.
Compuesto:  intradérmico  y  de  unión  juntos;  actividad  intermedia.
Spitz:  una  vez  llamado  melanoma  juvenil,  en  realidad  es  un  nevus  epitelioide  de  células  fusiformes  que  es  bastante
benigno.
Displásico:  el  más  propenso  a  malignizarse  (especialmente  en  el  síndrome  de  nevus  displásico).

4.  ¿Cuáles  son  los  factores  de  riesgo  en  la  formación  de  melanoma?

  Gran  número  de  moles  (>50  moles  >2  mm  de  diámetro).
&  Cambio  de  nevus.
&  Historia  del  melanoma.
&  Antecedentes  familiares  de  melanoma.
&  Piel  clara,  mal  bronceada;  cabello  rubio  o  castaño  rojizo.
  Antecedentes  de  quemaduras  solares  episódicas,  agudas  y  graves.

  Síndrome  de  nevus  displásico  o  síndrome  familiar  de  melanoma  con  múltiples  lunares  atípicos
(FAMMM).

5.  ¿Qué  lesiones  cutáneas  suelen  simular  un  melanoma  primario?
  nevo  de  Spitz  (nevo  epitelioide  de  células  fusiformes).
&  Micosis  fungoide.
  Nevus  benigno  atípico.
&  Enfermedad  de  Paget  extramamaria.
&  Hola  nevus.
y  enfermedad  de  Bowen.

  Nevus  benigno  recurrente  después  de  una  escisión  inadecuada.
&  Queratosis  sebáceas  oscuras.
  Melanoma  metastásico  en  la  piel.

y  sarcoma  de  Kaposi.
  Carcinoma  basocelular  pigmentado.

328
Machine Translated by Google
CAPÍTULO  65  MELANOMA  329

6.  ¿Qué  es  el  síndrome  de  melanoma  familiar?
El  síndrome  FAMMM  hereditario  se  ha  definido  como  la  aparición  de  melanoma  en  uno  o  más  familiares  de  primer  o  
segundo  grado  y  la  presencia  de  >50  lunares  de  tamaño  variable,  algunos  de  los  cuales  son  histológicamente  atípicos.  
El  riesgo  de  melanoma  en  este  síndrome  llega  al  100%  durante  la  vida  de  la  persona.  Las  personas  con  FAMMM  con  
frecuencia  tienen  una  mutación  en  p16  asignada  al  cromosoma  9.

7.  ¿Cuáles  son  los  sitios  comunes  de  desarrollo  de  melanoma?
Los  sitios  más  comunes  son  el  tronco  posterior  en  hombres  y  las  extremidades  inferiores  en  mujeres.  Todas  las  áreas  
expuestas  al  sol  son  sitios  posibles.  Los  sitios  poco  comunes  para  la  formación  de  melanoma  son  las  plantas  de  los  pies,  
las  palmas  de  las  manos  y  los  genitales.  Los  sitios  no  cutáneos  inusuales  para  la  formación  de  melanoma  son  el  ojo,  el  
ano  y  el  tracto  gastrointestinal  (GI).

8.  ¿Dónde  es  la  incidencia  de  melanoma  más  alta  del  mundo?
El  melanoma  es  más  común  en  Australia,  especialmente  en  la  parte  norte  del  continente,  donde  los  descendientes  de  
piel  clara  de  los  colonos  originales  están  expuestos  al  sol  tropical.

9.  ¿Cuáles  son  las  señales  de  advertencia  del  melanoma?
Lesiones  cutáneas  que  
presenten:  A  ¼  Asimetría.
B  ¼  Borde  irregular.
C  ¼  Color:  variable;  manchado;  a  menudo  muy  negro  con  áreas  bronceadas  irregulares;  manchas  rojas  o  rosadas;  
ulcerado  cuando  está  avanzado  (sangra  fácilmente).
D  ¼  Diámetro  (>5  a  6  mm).
E  ¼  Ampliación  o  Elevación.

10.  ¿Cuáles  son  los  tipos  de  melanoma  y  su  incidencia?
Extensión  superficial:  75%  de  todos  los  casos;  más  común.
Nodular:  15%  de  los  casos;  más  maligno;  bien  circunscrito;  profundamente  invasivo.
Melanoma  léntigo  maligno:  5%  de  los  casos;  pronóstico  relativamente  bueno.
Lentiginoso  acral:  5%  de  los  casos;  tipo  más  común  en  personas  de  color;  aparece  en  las  suelas,
palmas,  sitios  subungueales.

11.  ¿Qué  lunares  se  deben  considerar  para  su  eliminación?

Deben  extirparse  los  nevos  que  crecen  y  se  oscurecen,  especialmente  en  pacientes  sensibles  al  sol.  La  picazón  es  
un  signo  de  cambio  maligno  temprano.  La  ulceración  es  un  signo  tardío.  Debido  a  que  el  origen  del  melanoma  
puede  ser  familiar,  los  hijos  de  pacientes  con  melanoma  deben  ser  examinados  cuidadosamente  para  detectar  
nevus  muy  oscuros.

12.  ¿Cómo  se  debe  realizar  la  biopsia  de  los  nevos  sospechosos?
Debe  realizarse  una  escisión  total  de  la  lesión  con  un  margen  estrecho  (1  mm)  de  piel  normal  más  reparación  primaria.  
La  biopsia  por  incisión  parcial  es  aceptable  si  la  lesión  es  grande  o  si  la  escisión  total  requeriría  cirugía  reconstructiva.  La  
biopsia  con  sacabocados,  la  biopsia  por  incisión  o  la  saucerización  son  apropiadas  siempre  que  se  obtenga  una  muestra  
de  espesor  total.  Un  estudio  anatomopatológico  completo  es  esencial.

13.  ¿Los  melanomas  retroceden  espontáneamente  o  incluso  desaparecen?
Sorprendentemente,  algunos  melanomas  pueden  retroceder  o  incluso  desaparecer.  Aproximadamente  el  10%  de  los  
pacientes  con  melanoma  con  metástasis  presentan  metástasis  de  un  sitio  primario  desconocido.

14.  ¿Cuáles  son  las  clasificaciones  de  invasión  de  melanoma  de  Breslow  y  Clark?
Clark  seleccionó  cinco  niveles  de  grosor  del  melanoma  en  la  piel:

  Nivel  I:  melanoma  intradérmico  que  no  hace  metástasis;  puede  denominarse  mejor  hiperplasia  melanótica  
atípica:  una  lesión  benigna.
Machine Translated by Google
330  CAPÍTULO  65  MELANOMA

  Nivel  II:  melanoma  que  penetra  la  membrana  basal  hacia  la  dermis  papilar.
  Nivel  III:  melanoma  que  llena  la  dermis  papilar  e  invade  la  dermis  reticular  de  manera  forzada.

  Nivel  IV:  melanoma  que  invade  la  dermis  reticular.
&  Nivel  V:  melanoma  que  se  abre  camino  hacia  la  grasa  subcutánea.
El  método  Breslow  requiere  un  micrómetro  óptico  ajustado  a  la  posición  ocular  de  un  microscopio  estándar.  Esta  técnica  
es  una  determinación  más  exacta  de  la  invasión  tumoral.  Las  lesiones  se  clasifican  de  la  siguiente  manera:  =  1,0  mm  
1,01  a  2,0  mm  2,01  a  4,0  mm  =  4,0  mm

Las  lesiones  <1  mm  incluyen  melanoma  in  situ  y  tumores  invasivos  delgados.  La  tasa  de  curación  en  este  último  es  
superior  al  95%  con  la  escisión.  Los  tumores  de  1,0  a  4,0  mm  se  denominan  intermedios  pero  implican  riesgo  de  

metástasis.  Las  lesiones  >4,0  mm  son  lesiones  de  alto  riesgo  con  una  baja  tasa  de  curación.
Todos  los  melanomas  deben  ser  revisados  por  ambos  métodos  porque  algunos  tumores  pueden  mostrar  una  
medida  de  Breslow  baja  con  un  nivel  de  Clark  más  profundo,  lo  que  indica  un  gran  riesgo  de  recurrencia  y  diseminación.
La  medición  del  grosor  es  importante  y  el  tumor  debe  medirse  desde  la  altura  total  de  la  lesión  verticalmente  en  el  punto  
de  máximo  grosor.  Además,  si  hay  ulceración,  la  medición  debe  realizarse  desde  el  fondo  del  cráter  de  la  úlcera  hasta  
el  borde  más  profundo  de  la  lesión  (v .  fig.  65­1).

1.0

0.9
Estadio  I  (n=9175)
0.8

0.7

0.6
Estadio  II  (n=5739)
sobreviviente
Proporción  

0.5

0.4
Estadio  III  (n=1528)
0.3

0.2

0.1 Estadio  IV  (n=1158)

0.0
0  1  2  3  4  5  6  7  8  9  10  11  12  13  14  15

Supervivencia  (años)

Figura  65­1.  Supervivencia  estimada  por  etapa  del  melanoma  (Balch  CM,  Buzaid  AC,  Soong  SJ  et  al.:  Versión  final  
del  sistema  de  estadificación  del  American  Joint  Committee  on  Cancer  para  el  melanoma  cutáneo,  J  Clin  Oncol  
19(16):  3635­3648,  2001.  Reimpreso  con  autorización  de  la  Sociedad  Americana  de  Oncología  Clínica.)
Machine Translated by Google
CAPÍTULO  65  MELANOMA  331

15.  ¿Cuál  es  el  sistema  de  estadificación  de  tumores,  ganglios  y  metástasis  para  el  melanoma?
El  sistema  de  estadificación  tumor,  ganglio,  metástasis  (TNM)  es  la  clasificación  más  completa  del  melanoma.  Mediante  
el  uso  de  factores  de  riesgo  establecidos  para  la  enfermedad  avanzada,  estratifica  a  los  pacientes  según  el  grosor  del  
melanoma,  la  ulceración,  las  micrometástasis  o  metástasis  ganglionares  y  la  enfermedad  metastásica  a  distancia.  Revisado  
por  última  vez  en  2001,  predice  con  mayor  precisión  el  pronóstico  general.  Consulte  la  Tabla  65­1.

TABLA  65­1.  TUM  O , ASIENTO  E , Y  METASTA  SI  SCLAS  SI  FI  CA  CIÓN  OFMEL  ANOM  A

Clasificación  T Espesor Estado  de  ulceración

T1 1  milímetro a  ¼  sin  ulceración  y  nivel  II/
III  b  ¼  con  ulceración  
o  nivel  IV/V

T2 1,01–2,0  mm a  ¼  sin  ulceración  b  ¼  con  
ulceración

T3 2,01–4,0  mm a  ¼  sin  ulceración  b  ¼  con  
ulceración

T4 >4,0  mm a  ¼  sin  ulceración  b  ¼  con  
ulceración

N  Clasificación Número  de  ganglios  metastásicos Masa  metastásica  ganglionar

N1 1  nodo a  ¼  micrometástasis  

(descubierta  
microscópicamente)  
b  ¼  macrometástasis  

(clínicamente  evidente)  

N2 2–3  nodos a  ¼  micrometástasis  b  ¼  
macrometástasis  c  ¼  
metástasis  en  tránsito/satélite  
sin  ganglios  metastásicos

N3 4  o  más  ganglios,  o  ganglios  
enmarañados,  o  en  metástasis  stansit/
satélite  con  ganglios  metastásicos

Clasificación  M Sitio suero  de  leche

deshidrogenasa

M1a Piel  distante,  subcutánea  o  ganglios Normal

M1b metástasis  pulmonar Normal

M1c Todas  las  demás  metástasis  viscerales Normal

Cualquier  metástasis  a  distancia Elevado

Usado  con  el  permiso  del  Comité  Conjunto  Estadounidense  sobre  el  Cáncer  (AJCC),  Chicago,  Illinois.  La  
fuente  original  de  este  material  es  el  Manual  de  estadificación  del  cáncer  del  AJCC,  sexta  edición  (2002)  
publicado  por  Springer  Science  and  Business  Media  LLC,  www.springerlink.com.
Machine Translated by Google
332  CAPÍTULO  65  MELANOMA

16.  ¿Cuáles  son  las  posibilidades  de  diseminación  ganglionar  y  sistémica  de  los  distintos  grados  de  invasión  del  melanoma?

Consulte  la  Tabla  65­2.

TABLA  65­2.  RIESGO  ESTIMADO  DE  METÁSTASIS  EN  LOS  GANGLIOS  LINFÁTICOS  EN  TI  NEL  BASADO  
EN  EL  ESPESOR  DEL  TUMOR

Grosor  del  tumor Riesgo  relativo  de  metástasis  en  el  ganglio  linfático  centinela

1,0  mm <5%

1,01–2,0  mm 10%–20%

2,01–4,0  mm 25%–35%

4mm 35%–55%

17.  ¿Cuáles  son  las  características  de  un  melanoma  subungueal?
Las  lesiones  subungueales  a  menudo  se  confunden  con  un  proceso  inflamatorio  crónico;  por  lo  tanto,  la  
mayoría  de  los  pacientes  se  presentan  bastante  tarde.  Suelen  ser  mayores  que  los  pacientes  con  otras  formas  
de  melanoma  cutáneo.  El  dedo  gordo  del  pie  es  el  sitio  de  origen  más  común.  Se  recomienda  la  amputación  en  
la  articulación  metatarsofalángica  o  proximal  a  ella  y  la  biopsia  del  ganglio  linfático  centinela  regional.  Las  
lesiones  primarias  suelen  ser  profundamente  invasivas  y  los  ganglios  linfáticos  son  positivos  para  cáncer  en  la  
mayoría  de  los  casos,  ya  sea  en  el  momento  del  diagnóstico  original  o  en  el  seguimiento  posterior.

18.  Describa  la  técnica  de  biopsia  del  ganglio  centinela.
La  biopsia  del  ganglio  linfático  centinela  (SLN,  por  sus  siglas  en  inglés)  se  basa  en  la  teoría  de  que  la  linfa  de  una  
neoplasia  sólida  inicialmente  drena  a  un  ganglio  centinela  central.  Estos  ganglios  centinela  son  los  primeros  ganglios  
en  riesgo  de  enfermedad  metastásica.  Los  ganglios  se  pueden  biopsiar  y  examinar  con  cortes  en  serie  y  tinción  
inmunohistoquímica.  La  técnica  de  identificación  de  SLN  requiere  la  cooperación  de  un  cirujano,  un  radiólogo  y  un  
patólogo.  Se  realiza  una  linfogammagrafía  con  inyección  de  coloide  de  azufre  de  tecnecio  radiactivo  (99mTeSC)  
alrededor  del  sitio  del  melanoma  primario.
Esto  identifica  las  cuencas  nodales  regionales  en  riesgo.  En  el  quirófano  para  inyección  intradérmica  de  colorante  de  
contraste  azul  (linfazurina  al  1%)  alrededor  del  sitio  primario.  Un  detector  gamma  portátil  identifica  el  punto  caliente  y  
se  realiza  una  pequeña  incisión  sobre  esta  área  para  extraer  el  SLN.
Una  combinación  de  colorante  de  contraste  azul  y  radiocoloide  proporciona  el  mayor  rendimiento  de  identificación  del  
ganglio  centinela.

PUNTOS  CLAVE:  MELANOMA
1.  El  término  melanoma  implica  un  tumor  maligno.

2.  El  melanoma  es  el  sexto  cáncer  más  común  en  los  Estados  Unidos  y  el  de  más  rápido  crecimiento
cáncer  en  los  hombres.

3.  Las  señales  de  advertencia  del  melanoma  son  lesiones  cutáneas  que  muestran  asimetría,  bordes  irregulares,  
cambios  de  color,  diámetro  >5  a  6  mm  y  agrandamiento  o  elevación  (ABCDE).

4.  El  papel  del  cirujano  es  proporcionar  un  control  local  con  márgenes  adecuados  (1  o  2  cm)  y  valorar  el  pronóstico  
(biopsia  del  ganglio  centinela).
Machine Translated by Google
CAPÍTULO  65  MELANOMA  333

19.  ¿Cómo  está  cambiando  la  biopsia  del  ganglio  linfático  centinela  el  tratamiento  del  melanoma?
La  presencia  de  metástasis  en  el  GC  es  un  predictor  independiente  de  supervivencia  global.  La  biopsia  del  SLN  también  
identifica  selectivamente,  con  una  morbilidad  mínima,  a  los  pacientes  que  tienen  un  alto  riesgo  de  recurrencia.
Aunque  no  se  ha  demostrado  un  beneficio  absoluto  para  la  disección  de  los  ganglios  linfáticos  regionales  o  la  terapia  
adyuvante,  a  menudo  se  ofrecen  para  proporcionar  control  local  de  la  enfermedad  e  información  pronóstica.

20.  ¿La  disección  electiva  de  los  ganglios  linfáticos  mejora  las  tasas  de  curación  en  pacientes  con  melanoma?
No.

21.  ¿Cuál  es  la  precisión  de  la  biopsia  del  ganglio  linfático  centinela  para  el  melanoma?
En  general,  el  procedimiento  tiene  una  precisión  del  95  %  en  la  predicción  de  la  presencia  de  metástasis  ganglionares  
adicionales  en  la  región  muestreada.  Hay  una  tasa  de  falsos  negativos  del  5%.  De  aquellos  con  una  disección  completa  
de  los  ganglios  linfáticos,  aproximadamente  el  20  %  tendrá  ganglios  positivos  adicionales.  Una  ventaja  de  la  técnica  SLN  
es  el  análisis  patológico  mejorado  con  múltiples  cortes  finos  de  solo  unos  pocos  ganglios  seleccionados.

22.  ¿Qué  características  del  melanoma  son  desfavorables  para  el  pronóstico  y  el  riesgo  de  metástasis?
El  grosor  tumoral  (Breslow),  la  invasión  anatómica  de  la  dermis  (Clark),  el  estado  ganglionar,  la  invasión  angiolinfática,  
la  regresión,  la  microsatelitosis,  el  neurotropismo,  el  índice  mitótico  (>6/mm2 ),  el  tronco  frente  a  las  extremidades,  la  
ulceración  y  el  sexo  masculino  son  desfavorables.

23.  Si  está  indicado,  ¿qué  tipos  de  disección  de  ganglios  deben  realizarse?
Si  no  hay  evidencia  de  afectación  macroscópica  de  los  ganglios  excepto  por  el  SLN  histológicamente  positivo,  se  prefiere  
una  disección  de  tipo  funcional  (es  decir,  que  preserva  la  función,  no  radical)  porque  preserva  nervios  y  vasos  vitales.

24.  ¿Qué  margen  es  el  adecuado  para  tratar  un  melanoma  primario?
¡La  ciencia  quirúrgica  está  viva  y  bien!  A  través  de  una  serie  de  ensayos  aleatorios  prospectivos,  se  han  
establecido  las  siguientes  pautas:  Grosor  del  tumor  Margen  clínico  recomendado  1,0  mm  1,0  cm  1,01  a  2,0  mm  
1  a  2  cm  2,01  a  4,0  mm  2,0  cm  4  mm  2,0  cm

25.  ¿Existen  otros  tratamientos  que  mejoren  la  supervivencia  en  pacientes  con  melanoma?
Aún  no.  Se  han  empleado  innumerables  tratamientos  adyuvantes,  vacunas,  patas  de  conejo  y  combinaciones  
(bioquimioterapia).  Se  ha  logrado  una  tasa  de  respuesta  de  hasta  el  40  %,  pero  no  se  traduce  en  una  mejor  supervivencia.  
La  dacarbazina  (DTIC)  es  la  quimioterapia  aprobada  por  la  Administración  de  Alimentos  y  Medicamentos  (FDA)  para  el  
melanoma  avanzado.

26.  ¿Qué  pasa  con  la  inmunoterapia?
La  interleucina­2  y  el  interferón­gamma  son  tratamientos  aprobados  para  el  melanoma,  pero  solo  son  activos  en  
el  5  %  al  10  %  de  los  pacientes.  También  se  han  probado  muchas  vacunas  sin  mejorar  la  supervivencia  general.

27.  ¿Puede  la  radioterapia  ser  útil  en  el  tratamiento  del  melanoma?
La  radioterapia  es  bastante  útil  como  tratamiento  paliativo  de  la  enfermedad  metastásica.

28.  ¿Cuándo  debe  utilizarse  la  amputación  en  el  tratamiento  de  la  enfermedad  localmente  avanzada?
¿melanoma?

Casi  nunca.  Con  el  desarrollo  de  la  perfusión  de  aislamiento,  las  indicaciones  para  amputaciones  mayores  de  miembros  
son  raras.  La  amputación  no  afecta  la  supervivencia  y,  como  tal,  debe  usarse  solo  para  el  control  local  de  enfermedades  
que  no  pueden  manejarse  de  manera  que  se  preserve  la  extremidad.  La  amputación  digital  parcial  es  la  terapia  
recomendada  para  el  melanoma  subungueal  para  lograr  el  control  local.
Machine Translated by Google
334  CAPÍTULO  65  MELANOMA

29.  ¿Qué  es  la  perfusión  de  extremidades  de  aislamiento?  ¿Cómo  se  usa  en  el  melanoma?
Aunque  los  estudios  no  han  demostrado  que  la  perfusión  de  aislamiento  transmita  una  ventaja  de  supervivencia  
en  el  melanoma  primario,  esta  técnica  se  usa  a  menudo  en  el  contexto  de  metástasis  en  tránsito  múltiples  o  
recurrentes  de  una  extremidad.  El  melfalán  (de  uso  común)  u  otras  preparaciones  de  quimioterapia  (p.  ej.,  interferón,  
factor  de  necrosis  tumoral  [TNF])  se  hacen  circular  a  través  de  una  extremidad  aislada  utilizando  un  oxigenador  de  
bomba  a  temperaturas  hipertérmicas  leves.  La  perfusión  de  aislamiento  exitosa  preserva  una  extremidad  funcional,  
tiene  una  tasa  de  respuesta  superior  e  incurre  en  menos  morbilidad  que  la  terapia  sistémica.

30.  ¿Cuál  es  el  tratamiento  de  un  paciente  con  ganglios  metastásicos  confinados  a  un  solo
área  cuando  el  sitio  primario  es  desconocido?
Si  un  estudio  cuidadoso  no  revela  otros  focos  de  melanoma,  se  debe  realizar  una  disección  de  los  ganglios  linfáticos  
regionales.

31.  Ahora  lo  has  hecho.  Ese  paciente  con  una  disección  de  ganglios  linfáticos  ha  desarrollado  linfedema.  ¿Cómo  lo  
manejas?
El  reconocimiento  y  el  tratamiento  tempranos  son  la  clave.  Ocurre  del  10%  al  20%  de  los  pacientes.  La  fisioterapia  y  
las  prendas  compresivas  hechas  a  medida  pueden  ayudar  a  reducir  la  gravedad  del  edema.

32.  ¿Qué  debe  hacer  en  la  atención  de  seguimiento  de  pacientes  sometidos  a  tratamientos  curativos?
operacion  de  melanoma?
Además  de  los  exámenes  físicos  frecuentes,  son  importantes  las  radiografías  de  tórax  y  las  pruebas  de  función  
hepática  (LFT).

33.  ¿Tiene  alguna  función  la  cirugía  en  pacientes  con  melanoma  en  estadio  IV  (metastásico)?
Absolutamente.  En  pacientes  seleccionados  (generalmente  un  intervalo  largo  sin  enfermedad,  un  solo  sitio  de  enfermedad),  
hasta  el  30  %  de  los  que  se  someten  a  una  resección  estarán  vivos  a  los  5  años  (en  comparación  con  menos  del  5  %  de  los  que  
no  se  someten  a  una  resección).

SITIOS  WEB

www.nccn.org/

www.cancer.org

BIBLIOGRAFÍA

1.  Balch  CM,  Buzaid  AC,  Soong  SJ  et  al.:  Versión  final  del  sistema  de  estadificación  del  American  Joint  Committee  on  Cancer
para  melanoma  cutáneo.  J  Clin  Oncol  19(16):  3635­3648,  2001.
2.  Balch  CM,  Houghton  AN,  Sober  AJ  et  al.:  Melanoma  cutáneo,  4.ª  ed.,  St.  Louis,  2003,  Quality  Medical
Publicación.
3.  Blazer  DG  3rd,  Sondak  VK,  Sabel  MS:  Terapia  quirúrgica  del  melanoma  cutáneo.  Semin  Oncol  34(3):270­280,
2007.

4.  Jakub  JW,  Reintgen  DS,  Shivers  S  et  al .:  Disección  de  ganglios  regionales  para  melanoma:  técnicas  e  indicaciones.
Surg  Oncol  Clin  N  Am  16(1):247­261,  2007.
5.  Morton  DL,  Thompson  JF,  Cochran  AJ  et  al.:  Biopsia  de  ganglio  centinela  u  observación  de  ganglios  en  melanoma.
N  Engl  J  Med  355(13):1307­1317,  2006.
6.  Noorda  EM,  Vrouenraets  BC,  Nieweg  OE  et  al .:  Perfusión  de  extremidades  aisladas  en  melanoma  regional.  Surg  Oncol  
Clin  N  Am  15(2):373­384,  2006.
Machine Translated by Google
CAPÍTULO  65  MELANOMA  335

7.  Rousseau  DL  Jr,  Ross  MI,  Johnson  MM  et  al.:  Los  criterios  de  estadificación  revisados  del  American  Joint  Committee  on  
Cancer  predicen  con  precisión  la  positividad  del  ganglio  linfático  centinela  en  pacientes  con  melanoma  con  ganglios  
clínicamente  negativos.  Ann  Surg  Oncol  10(5):569­574,  2003.

8.  Spanknebel  K,  Temple  L,  Hiotis  S  et  al.:  Ensayos  clínicos  aleatorizados  en  melanoma,  Surg  Oncol  Clin  N  Am
11(1):23­52,  2002.

9.  Thompson  JF,  Scolyer  RA,  Uren  RF:  Manejo  quirúrgico  del  melanoma  cutáneo  primario:  márgenes  de  escisión  y  el  papel  del  examen  
del  ganglio  linfático  centinela.  Surg  Oncol  Clin  N  Am  15(2):301­318,  2006.

10.  Young  SE,  Martinez  SR,  Essner  R:  El  papel  de  la  cirugía  en  el  tratamiento  del  melanoma  en  estadio  IV.  J  Surg  Oncol  94
(4):344­351,  2006.
Machine Translated by Google

TUMORES  DE  PARÓTIDA
CAPITULO  
66

Dr.  Michael  L.  Lepore,  FACS

1.  Describir  la  ubicación  y  características  de  la  glándula  parótida.
Las  glándulas  parótidas  pares  son  las  más  grandes  de  las  tres  glándulas  salivales  principales  que  
surgen  como  invaginación  del  ectodermo  oral  en  el  tejido  mesenquimatoso  circundante.  La  característica  
histológica  distintiva  de  la  glándula  parótida  es  que  sus  células  acinares  están  formadas  principalmente  por  
células  secretoras  serosas.  El  compartimento  de  la  glándula  parótida  tiene  una  forma  aproximadamente  
triangular,  delimitado  superiormente  por  el  arco  cigomático,  posteriormente  por  el  conducto  auditivo  externo  e  
inferiormente  por  la  apófisis  estiloides,  el  músculo  estiloides  y  los  vasos  yugulares  y  carotídeos  internos;  
anteriormente  está  delimitado  por  el  músculo  masetero.  La  cola  de  la  glándula  parótida  puede  extenderse  
inferior­posteriormente  al  nivel  del  músculo  esternocleidomastoideo  y  la  apófisis  mastoides.

2.  ¿Cuál  es  la  relación  del  nervio  facial  con  la  glándula  parótida?
El  nervio  atraviesa  la  glándula  parótida,  dividiendo  la  glándula  parótida  en  un  lóbulo  superficial  y  uno  profundo.  
El  nervio  se  encuentra  lateralmente  a  la  apófisis  estiloides  y  al  vientre  posterior  del  músculo  digástrico  y  medial  
a  la  punta  de  la  mastoides.  A  medida  que  el  nervio  sale  del  agujero  estilomastoideo,  emite  tres  ramas  motoras:  
una  al  músculo  estilohioideo,  otra  al  vientre  posterior  del  músculo  digástrico  y,  por  último,  a  los  tres  músculos  
postauriculares.  Luego,  el  nervio  girará  lateralmente  para  entrar  en  la  cara  posterior  de  la  glándula  parótida.  
Después  de  entrar  en  la  glándula,  se  divide  en  el  pes  anserinus  en  una  división  temporofacial  y  cervicofacial.  
La  división  temporofacial  se  divide  en  las  ramas  temporal,  cigomática  y  bucal.  La  división  cervicofacial  se  divide  
en  las  ramas  mandibular  marginal  y  cervical.  El  lóbulo  profundo  se  encuentra  entre  las  divisiones  temporofacial  
y  cervicofacial.

3.  ¿Qué  rama  del  nervio  facial  sigue  el  conducto  parotídeo  a  lo  largo  de  su  trayecto  y  cuál  es  su  significado?

La  rama  bucal  del  nervio  facial  discurre  superiormente  a  lo  largo  de  todo  el  trayecto  del  conducto  de  Stenson.  
En  parotidectomías  difíciles  cuando  el  nervio  facial  no  se  puede  ubicar  en  el  agujero  estilomastoideo,  la  
relación  del  nervio  facial  con  el  conducto  de  Stenson  es  un  método  para  identificar  el  nervio  periféricamente.  
Seguir  este  nervio  distalmente  conducirá  directamente  a  la  división  temporofacial.

4.  ¿Qué  es  la  unidad  de  glándulas  salivales?
La  unidad  de  la  glándula  salival  está  compuesta  por  células  acinares  serosas  o  mucosas.  Las  secreciones  
de  estas  células  drenan  en  un  conducto  intercalado  que  se  conecta  a  un  conducto  estriado  que  a  su  vez  drena  
en  un  conducto  excretor.  Alrededor  de  los  ácinos  y  los  conductos  intercalados  hay  células  mioepiteliales.
Estas  células  mioepiteliales  se  contraen  para  forzar  la  entrada  de  saliva  en  el  sistema  ductal.

5.  ¿Cuál  es  la  importancia  de  la  unidad  de  la  glándula  salival  en  el  desarrollo  del  tumor?
Actualmente  existen  dos  teorías  de  desarrollo  tumoral  basadas  en  la  unidad  de  glándulas  salivales.  a.  
Teoría  bicelular:  los  tumores  surgen  a  partir  de  células  madre.  La  célula  de  reserva  del  conducto  excretor  dará  
lugar  a  carcinomas  de  células  escamosas  y  mucoepidermoides.  La  célula  de  reserva  del  conducto  
intercalado  da  lugar  al  adenoma  pleomórfico,  oncocitomas,  carcinomas  adenoides  quísticos,  
adenocarcinomas  y  carcinomas  de  células  acinares.

336
Machine Translated by Google
CAPÍTULO  66  TUMORES  DE  PARÓTIDA  337

b.  Teoría  multicelular:  cada  tipo  de  tumor  se  asocia  con  una  célula  de  origen  específica  diferenciada  dentro  de  la  unidad  
de  glándulas  salivales.  Por  lo  tanto,  las  células  de  los  conductos  excretores  dan  lugar  a  carcinomas  de  células  
escamosas;  las  células  de  los  conductos  intercalados  dan  lugar  a  adenomas  pleomórficos;  los  conductos  estriados  
dan  lugar  a  oncocitomas;  y  las  células  acinares  dan  lugar  a  carcinomas  de  células  acinares.

6.  ¿Cuáles  son  los  cuatro  tumores  benignos  más  frecuentes  de  origen  en  las  glándulas  salivales?
y  sus  caracteristicas?

a.  El  adenoma  pleomorfo  (tumor  mixto)  representa  aproximadamente  el  80%  de  todos  los  tumores  parotídeos  
benignos.  Son  de  crecimiento  lento  y  no  están  bien  encapsulados.  La  tasa  de  recurrencia  es  del  1%  al  5%  con  
la  escisión  adecuada.  La  degeneración  maligna  puede  ocurrir  en  aproximadamente  2%  a  10%  de  los  casos.

b.  Tumor  de  Warthin  (cistoadenoma  linfomatoso  papilar  o  adenolinfoma).  Este  tumor  se  presenta  más  adelante  en  la  
vida.  Es  el  segundo  tumor  más  frecuente  representando  aproximadamente  el  5%  de  todos  los  tumores  benignos.  
Hay  un  predominio  masculino.  Aproximadamente  el  12%  de  los  tumores  de  Warthin  ocurren  bilateralmente.  C.  El  
oncitoma  se  presenta  en  la  sexta  década  de  la  vida  y  está  compuesto  por  grandes  células  oxifílicas.

Los  oncocitos  que  se  encuentran  en  estos  tumores  y  en  los  tumores  de  Warthin  son  responsables  de  la  
concentración  de  pertecnetato  de  tecnecio  99m.
d.  El  adenoma  monomórfico  incluye  lo  siguiente:  adenoma  de  células  basales,  adenoma  de  células  claras  y  
adenoma  rico  en  glucógeno.  El  más  común  de  los  tres  es  el  adenoma  de  células  basales.
Estos  tumores  están  bien  circunscritos  y  encapsulados.

7.  ¿Cuál  es  el  tratamiento  de  los  tumores  benignos  de  la  glándula  parótida?
El  tratamiento  es  una  parotidectomía  superficial  con  preservación  del  nervio  facial.  Antes  de  que  se  extirpe  la  glándula,  
debe  orientarse  y  etiquetarse  adecuadamente  para  el  patólogo.  Si  hay  un  margen  estrecho,  se  debe  observar  al  
paciente  en  busca  de  recurrencia,  particularmente  en  el  caso  de  adenomas  pleomórficos.

8.  ¿Cuál  es  el  papel  de  la  monitorización  intraoperatoria  del  nervio  facial  en  la  glándula  parótida?
¿cirugía?
El  monitoreo  del  nervio  facial  es  un  medio  útil  para  identificar  el  nervio  facial,  particularmente  durante  casos  
quirúrgicos  difíciles  de  la  glándula  parótida.  Normalmente  se  colocan  múltiples  sondas  periféricas  en  cuatro  
ubicaciones:  en  la  región  de  la  rama  temporal  que  inerva  el  músculo  frontal;  en  la  región  de  la  rama  cigomática  que  
inerva  el  músculo  orbicularis  oculi;  en  la  región  de  la  rama  bucal  que  inerva  el  músculo  orbicularis  oris;  y  en  la  región  del  
nervio  mandibular  marginal  que  inerva  el  músculo  depresor  del  labio  inferior.  Cuando  se  estimulan  distalmente  las  
principales  divisiones  del  nervio  facial,  el  movimiento  facial  será  evidente.

9.  ¿Cuál  es  el  significado  de  un  «tumor  en  mancuerna»?
Ocasionalmente,  el  tumor  parotídeo  del  lóbulo  profundo  puede  presentarse  en  el  examen  como  una  masa  en  la  
pared  faríngea  lateral.  Esto  es  principalmente  el  resultado  de  una  debilidad  en  la  membrana  estilomandibular.

10.  De  las  tres  glándulas  salivales  mayores  emparejadas,  ¿cuál  glándula  tiene  la  mayor
incidencia  de  las  neoplasias  de  las  glándulas  salivales?
La  glándula  parótida  tiene  la  mayor  incidencia  de  neoplasias  de  las  glándulas  salivales.  Aproximadamente  el  80%  de  
todas  las  neoplasias  localizadas  en  esta  glándula  son  benignas.  Una  buena  regla  general  para  recordar  con  respecto  a  
los  tumores  malignos  es  la  regla  25/50/75.  A  medida  que  la  glándula  salival  se  hace  más  pequeña,  aumenta  la  incidencia  
de  tumores  malignos.  Así,  en  la  glándula  parótida  la  incidencia  de  neoplasias  malignas  es  del  25%,  en  la  glándula  
submandibular  es  del  50%  y  en  la  glándula  sublingual  es  del  75%.

11.  ¿Cuál  es  el  estudio  de  una  masa  en  el  espacio  parotídeo?
El  estudio  se  basa  en  la  historia  clínica  y  los  hallazgos  físicos  en  el  examen  del  paciente.
Clásicamente,  los  pacientes  con  un  tumor  que  involucra  la  glándula  parótida  se  quejan  de  un  dolor  indoloro.
Machine Translated by Google
338  CAPÍTULO  66  TUMORES  DE  PARÓTIDA

masa  de  crecimiento  lento  en  la  región  preauricular  (80%),  o  en  el  ángulo  de  la  mandíbula  (cola  de  la  glándula  
parótida).  Si  la  masa  es  dolorosa  (30%)  o  las  ramas  del  nervio  facial  no  funcionan  (7%  a  20%),  entonces  se  debe  tener  
un  alto  índice  de  sospecha  de  malignidad  de  la  parótida.  Aproximadamente  el  80%  de  los  pacientes  con  parálisis  del  
nervio  facial  tienen  metástasis  en  los  ganglios  en  el  momento  del  diagnóstico.  Se  debe  realizar  un  examen  cuidadoso  
de  la  cavidad  oral,  el  cuero  cabelludo  y  una  endoscopia  flexible  para  descartar  otros  tumores  que  puedan  haber  hecho  
metástasis  a  la  glándula.
El  examen  del  cuello  debe  buscar  adenopatías  que  indiquen  enfermedad  metastásica  o  linfoma.
La  aspiración  con  aguja  fina  (FNA)  se  puede  realizar  fácilmente  en  el  entorno  clínico.  La  tomografía  computarizada  
(TC)  de  contraste  y  la  resonancia  magnética  nuclear  (RMN)  son  útiles  para  determinar  la  ubicación  y  extensión  de  la  
masa;  sin  embargo,  la  patología  benigna  puede  dar  hallazgos  similares:  bordes  mal  definidos  y  realce.

12.  ¿Cuál  es  la  importancia  de  la  debilidad  o  parálisis  del  nervio  facial  en  relación  con  el  agrandamiento  de  la  glándula  
parótida?
La  afectación  del  nervio  facial  en  presencia  de  una  masa  parotídea  suele  indicar  la  presencia  de  un  proceso  
maligno.  El  grado  de  parálisis  debe  anotarse  clínicamente  y  tomarse  fotografías  para  la  documentación.

13.  Describa  los  cinco  tumores  parotídeos  malignos  más  frecuentes  y  sus  características.

a.  El  carcinoma  mucoepidermoide  es  el  tumor  maligno  más  común  de  la  glándula  parótida  y  representa  el  30%  
de  todos  los  tumores  malignos  de  la  parótida.  Se  clasifica  en  neoplasias  malignas  de  bajo  o  alto  grado.  La  
forma  de  bajo  grado  tiene  una  mayor  proporción  de  células  mucosas  a  células  epidermoides  y  se  comporta  
como  tumores  benignos.  En  el  caso  de  tumores  de  alto  grado,  hay  una  mayor  porción  de  células  epidermoides  
y  pueden  asemejarse  a  carcinomas  de  células  escamosas.  Los  últimos  tienen  una  alta  propensión  a  la  
metástasis.  b.  El  adenocarcinoma  representa  aproximadamente  el  15%  de  los  tumores  de  la  glándula  parótida.  
Las  neoplasias  se  presentan  como  masas  firmes  o  duras  adheridas  al  tejido  circundante.  Los  adenocarcinomas  
carecen  de  queratina  y,  por  lo  tanto,  se  diferencian  fácilmente  de  los  carcinomas  mucoepidermoides.  C.  El  
carcinoma  adenoide  quístico  (cilindromas)  representa  el  6%  de  todas  las  neoplasias  de  las  glándulas  salivales.

Es  la  neoplasia  maligna  más  frecuente  de  las  glándulas  salivales  menores  y  submandibulares.
Los  carcinomas  adenoides  quísticos  son  impredecibles  y  pueden  permanecer  inactivos  durante  mucho  tiempo.
Estos  tumores  crecen  a  lo  largo  de  los  planos  perineurales  y  tienen  una  alta  incidencia  de  metástasis  
a  distancia,  particularmente  en  los  pulmones.  Hay  tres  tipos  histológicos:  cribiforme,  sólido,  cilindromatoso  
y  tubular.  La  forma  sólida  es  la  de  peor  pronóstico  y  la  cribosa  se  considera  la  más  benigna  del  grupo.

d.  Se  cree  que  los  tumores  mixtos  malignos  (carcinoma  adenoma  expleomorfo)  se  desarrollan  a  partir  de  un  
adenoma  pleomorfo  preexistente.  Parece  representar  del  2%  al  5%  de  los  tumores  malignos  de  la  parótida.

mi.  El  linfoma  de  la  glándula  parótida  ocurre  con  mayor  frecuencia  en  hombres  de  edad  avanzada.  Toda  la  glándula  
parótida  y  los  ganglios  linfáticos  regionales  están  agrandados.  La  FNA  con  citometría  de  flujo  puede  ayudar  a  
diagnosticar  esta  afección  porque  el  tratamiento  consiste  en  quimioterapia  seguida  de  radioterapia.

14.  ¿Cuál  es  la  clasificación  actual  de  los  tumores  malignos  de  la  glándula  parótida?
Los  tumores  de  parótida  se  clasifican  según  el  Comité  Conjunto  Estadounidense  sobre  el  Cáncer  (AJCC).
tumor  primario
TX  Extensión  del  tumor  desconocida  o  no  se  puede  evaluar

T0  Sin  evidencia  de  un  tumor  primario
T1  Tumor  <2  cm  de  diámetro  mayor
T2  Tumor  >2  cm  pero  <4  cm  en  diámetro  mayor
T3  Tumor  >4  cm  o  tumor  con  extensión  extraparenquimatosa
T4a  El  tumor  invade  la  piel,  la  mandíbula,  el  canal  auditivo  o  el  nervio  facial
Machine Translated by Google
CAPÍTULO  66  TUMORES  DE  PARÓTIDA  339

T4b  El  tumor  invade  la  base  del  cráneo  o  las  placas  pterigoideas  o  encierra  la  arteria  carótida
Todas  las  categorías  se  subdividen  en  a)  sin  extensión  local,  b)  extensión  local
Ganglios  linfáticos
N0  Sin  metástasis  en  ganglios  linfáticos  regionales
N1  Metástasis  a  un  solo  ganglio  homolateral  <3  cm
N2a  Metástasis  a  un  solo  ganglio  ipsilateral  >3  cm  pero  <6  cm
N2b  Metástasis  en  múltiples  ganglios  linfáticos  ipsilaterales,  ninguno  de  más  de  6  cm  en  su  dimensión  mayor
N2c  Metástasis  en  ganglios  linfáticos  bilaterales  o  contralaterales,  ninguno  de  más  de  6  cm  en  su  mayor
dimensión

N3  Metástasis  en  un  ganglio  linfático  >6  cm  en  su  mayor  dimensión
Metástasis
MO  Sin  metástasis  a  distancia
Metástasis  a  distancia  M1

15.  ¿Cómo  se  manejan  los  tumores  de  parótida?
Aunque  las  opciones  de  tratamiento  pueden  variar  según  el  caso  y  la  ubicación  (lóbulo  superficial  o  profundo),  los  
tumores  de  parótida  se  pueden  clasificar  en  los  siguientes  grupos:
Grupo  1:  neoplasias  malignas  T1  o  T2N0  de  bajo  grado  (carcinoma  de  células  acinares  y
carcinomas  mucoepidermoides).
Se  realiza  una  parotidectomía  superficial  con  preservación  del  nervio  facial.
Grupo  2:  tumores  malignos  de  alto  grado  T1  o  T2N0  (carcinomas  mucoepidermoides  de  alto  grado,  adenocarcinomas  
y  tumores  malignos  mixtos).
a.  La  parotidectomía  total  se  realiza  si  el  lóbulo  profundo  está  involucrado.  b.  Si  el  
nervio  facial  no  está  afectado,  debe  conservarse.  Si  está  involucrado,  debe  ser  resecado.
e  inmediatamente  injertar  con  un  injerto  de  nervio  sural.
C.  Se  realiza  disección  de  ganglios  linfáticos.  d.  El  
paciente  debe  recibir  radioterapia  posoperatoria.
Grupo  3:  cánceres  de  alto  grado  T3N0  o  N1  y  cánceres  recurrentes.
a.  El  tratamiento  de  elección  es  la  resección  quirúrgica  radical  agresiva  para  incluir  el  lóbulo  profundo  si
involucrado.

b.  Si  las  ramas  del  nervio  facial  están  involucradas,  deben  extraerse  e  injertarse  en  el  momento  de  la  cirugía.  Si  el  tumor  
involucra  el  agujero  estilomastoideo,  se  debe  realizar  una  mastoidectomía  y  seguir  el  nervio  en  el  canal  de  Falopio  
hasta  obtener  márgenes  negativos.

C.  Se  realiza  disección  de  cuello  modificada  en  todas  las  categorías  T3N0  y  cuello  radical
disección  en  categorías  T3Nþ.  El  paciente  debe  recibir  radioterapia  postoperatoria.
Grupo  4:  Categoría  T4.
a.  En  este  grupo  se  realiza  una  parotidectomía  radical  para  incluir  el  tejido  circundante
involucrados  (grasa  bucal,  piel,  canal  auditivo,  hueso  mastoideo).  
b.  El  nervio  facial  suele  estar  afectado  y  sacrificado.  C.  La  
reconstrucción  primaria  de  todas  las  áreas  involucradas  se  realiza  en  el  momento  de  la  cirugía.  El  paciente  necesitará  
radioterapia  postoperatoria.

16.  ¿Cuáles  son  los  tumores  malignos  de  parótida  más  frecuentes  en  niños?
Carcinoma  mucoepidermoide  bien  diferenciado.

17.  ¿Cuáles  son  las  posibles  complicaciones  de  la  cirugía  de  la  glándula  parótida?
Colgajo  de  
piel  Sangrado  (hematoma)
Infección  

Fístula  de  las  glándulas  
salivales  Parálisis  facial  temporal  en  el  10%  de  los  pacientes  (generalmente  como  resultado  de  un  nervio  estirado)  
y  paresia  facial  permanente  en  <2%  de  los  pacientes.
Machine Translated by Google
340  CAPÍTULO  66  TUMORES  DE  PARÓTIDA

Síndrome  de  Frey  (enrojecimiento  y  sudoración  de  la  piel  que  recubre  el  sitio  quirúrgico).  Este  es  el  resultado  de  ramas  
nerviosas  parasimpáticas  preganglionares  ininterrumpidas  posoperatorias  hacia  la  parótida  hacia  las  glándulas  sudoríparas  
más  superficiales  de  la  piel.

18.  ¿Cuál  es  el  papel  de  la  biopsia  por  aspiración  con  aguja  fina  en  el  diagnóstico  del  agrandamiento  de  la  glándula  parótida?

La  FNA  es  un  complemento  diagnóstico  útil  en  la  evaluación  de  masas  en  la  cabeza  y  el  cuello.  La  FNA  depende  en  gran  
medida  de  la  experiencia  del  patólogo.  Por  lo  tanto,  su  papel  en  la  evaluación  de  los  tumores  de  las  glándulas  salivales  es  
algo  controvertido.  Tiene  una  sensibilidad  >90%  y  una  especificidad  >95%.  Tiene  un  valor  predictivo  positivo  de  
aproximadamente  84%  y  un  valor  predictivo  negativo  de  aproximadamente  77%.  Es  un  excelente  método  para  diferenciar  
entre  un  proceso  benigno  (linfadenopatía)  y  uno  maligno.

19.  ¿Qué  malignidad  de  la  glándula  parótida  tiene  una  alta  incidencia  de  invasión  perineural?
El  carcinoma  quístico  adenoide  se  asocia  comúnmente  con  una  alta  incidencia  de  invasión  perineural.

20.  ¿Por  qué  debe  tener  cuidado  cuando  se  trata  de  lesiones  quísticas  de  la  parótida?
¿glándula?
Antes  se  pensaba  que  las  lesiones  quísticas  de  la  glándula  parótida  eran  lesiones  raras.  Sin  embargo,  en  los  últimos  20  años,  
la  incidencia  de  lesiones  quísticas  se  ha  incrementado  particularmente  en  la  población  afectada  por  el  virus  de  la  
inmunodeficiencia  humana  (VIH).  Cuando  se  nota  una  masa  en  la  glándula  parótida  en  un  paciente  VIH  positivo,  se  debe  
pensar  en  un  quiste  linfoepitelial  que  se  asocia  frecuentemente  con  pacientes  VIH  positivos.

21.  ¿Son  confiables  las  secciones  congeladas  intraoperatorias  para  diferenciar  entre  lesiones  benignas
y  tumores  parotídeos  malignos,  y  ¿resectaría  el  nervio  facial  sobre  la  base  de  una  sección  congelada?

A  veces,  es  difícil  para  un  patólogo  hacer  un  diagnóstico  concluyente  basado  en  cortes  congelados,  por  lo  que  tenderá  a  
diferir  el  diagnóstico  hasta  que  se  realice  un  estudio  adecuado.  Por  lo  tanto,  la  mayoría  de  los  cirujanos  dudarán  antes  de  
realizar  un  procedimiento  destructivo  mayor  hasta  que  se  reciba  por  escrito  el  diagnóstico  anatomopatológico  final.  El  cirujano  
normalmente  realizará  una  parotidectomía  del  lóbulo  superficial  y  en  una  segunda  operación  obtendrá  márgenes  quirúrgicos  
claros,  que  pueden  incluir  la  resección  y  el  injerto  del  nervio  facial.

22.  ¿Tiene  alguna  función  la  quimioterapia  en  el  tratamiento  de  las  neoplasias  malignas  de  la  glándula  parótida?
Los  tumores  de  la  glándula  parótida  normalmente  responden  mal  a  la  quimioterapia.  La  quimioterapia  adyuvante  
actualmente  está  indicada  solo  para  paliación.  Los  agentes  a  base  de  platino  se  usan  con  mayor  frecuencia  porque  inducen  
la  apoptosis  y  la  muerte  celular.  Por  otro  lado,  los  agentes  a  base  de  doxorrubicina  promueven  la  detención  celular.

23.  ¿Tiene  alguna  función  la  tinción  inmunohistoquímica  en  la  identificación  de  tumores  de  la  glándula  parótida?

Las  investigaciones  más  recientes  sobre  los  procesos  microcelulares  de  las  neoplasias  de  las  glándulas  salivales  se  
pueden  realizar  en  tejido  biopsiado  o  seccionado  para  ayudar  en  el  trabajo  de  diagnóstico.  La  tinción  para  la  región  del  
organizador  nucleolar  de  plata  puede  ayudar  a  diferenciar  las  lesiones  benignas  o  inflamatorias  de  las  malignas.  Las  
inmunotinciones  citoplasmáticas  para  pRb  o  p130  (familia  de  genes  supresores  de  tumores)  se  correlacionaron  directamente  
con  un  mayor  grado  tumoral  en  las  neoplasias  malignas  de  las  glándulas  salivales.  La  pérdida  de  inmunotinción  para  p63  en  
células  mioepiteliales  se  puede  utilizar  para  buscar  células  malignas  para  distinguir  adenomas  pleomórficos  de  adenomas  
expleomórficos  de  carcinoma.  La  inmunotinción  para  la  expresión  de  mucina  puede  ayudar  a  diferenciar  el  carcinoma  de  
células  acinares  de  los  carcinomas  mucoepidermoides.  Los  carcinomas  de  células  acinares  expresan  MUC3  pero  no  MUC5AC,  
por  otro  lado,  los  carcinomas  mucoepidermoides  expresan  únicamente  MUC5AC  pero  no  MUC3.
Machine Translated by Google
CAPÍTULO  66  TUMORES  DE  PARÓTIDA  341

24.  ¿Existe  un  papel  de  la  inmunohistoquímica  en  la  predicción  de  la  supervivencia  en  la  glándula  parótida?
malignidades?  a.  La  
tinción  inmunohistoquímica  para  la  expresión  de  mucina  en  carcinomas  mucoepidermoides  demostró  una  
mayor  expresión  de  MUC1  en  estos  tumores  correlacionada  con  una  mayor  progresión  tumoral  y  un  peor  
pronóstico;  sin  embargo,  un  aumento  en  la  expresión  de  MUC4  demostró  una  menor  progresión  y  una  
mejor  supervivencia.  b.  También  se  evaluaron  proteínas  como  heparinasa  y  endo­bD­glucuronidasa.  
Cuando  se  expresaron  estas  sustancias,  se  correlacionaron  negativamente  con  la  supervivencia,  particularmente  
en  carcinomas  mucoepidermoides,  adenocarcinoma,  carcinomas  de  células  escamosas  y  de  células  acinares.  
C.  Ki­67,  un  antígeno  nuclear  que  mide  la  capacidad  proliferativa,  se  estudió  en  tumores  de  parótida.

Cuando  se  encontraron  altos  niveles  de  Ki­67  en  los  tumores,  se  notó  una  pobre  supervivencia.

PUNTOS  CLAVE

1.  El  tumor  benigno  más  común  de  la  glándula  parótida  es  un  adenoma  pleomórfico.

2.  El  tumor  maligno  más  común  de  la  glándula  parótida  es  un  mucoepidermoide
carcinoma.

3.  El  carcinoma  quístico  adenoide  tiene  la  mayor  incidencia  de  invasión  perineural.

4.  La  biopsia  FNA  es  una  herramienta  diagnóstica  útil  que  puede  ayudar  al  cirujano  en  el  preoperatorio
evaluación.

5.  La  presencia  de  parálisis  del  nervio  facial  es  una  buena  indicación  de  un  tumor  maligno  subyacente.
proceso.

BIBLIOGRAFÍA

1.  Arabi  Mianroodi  AA,  Sigston  EA,  Vallance  NA:  Sección  congelada  para  cirugía  de  parótida:  ¿debería  convertirse  en  una  rutina?
ANZ  J  Surg  76:736­739,  2006.

2.  Balakrishnan  K,  Castling  B,  McMahon  J  et  al.:  Citología  por  aspiración  con  aguja  fina  en  el  tratamiento  de  una  parótida
masa:  un  estudio  retrospectivo  de  dos  centros.  Cirujano  3:67­72,  2005.

3.  Brennan  JA,  Moore  EJ,  Shuler  KJ:  Análisis  prospectivo  de  la  eficacia  de  la  monitorización  nerviosa  intraoperatoria  continua  durante  la  
tiroidectomía,  paratiroidectomía  y  parotidectomía.  Otolaryngol  Head  Neck  Surg  1:  537­554.

4.  Carlson  GW:  Las  glándulas  salivales:  embriología,  anatomía  y  aplicaciones  quirúrgicas.  Surg  Clin  North  Am  80:
261­273,  2000.

5.  English  GM,  editor:  Otolaryngology,  vol  5  Filadelfia,  2000,  JB  Lippincott.

6.  Huang  RD,  Pearlman  S,  Friedman  WH  et  al.:  Lesiones  quísticas  benignas  frente  a  lesiones  sólidas  de  la  glándula  parótida  en  pacientes  con  VIH.
Cabeza  Cuello  13:522­526,  1991.

7.  Koyuncu  M,  Sesen  T,  Akan  H  et  al .:  Comparación  de  tomografía  computarizada  y  resonancia  magnética  en  el  diagnóstico  de  tumores  de  
parótida.  Otolaryngol  Head  Neck  Surg  129:726­732,  2003.

8.  Lee  JH,  Lee  JH,  Kim  A  et  al .:  expresión  única  de  MUC3,  MUC5AC  y  citoqueratinas  en  la  glándula  salival
carcinomas.  Pathol  Int  55:386­390,  2005.

9.  Medina  JE:  Disección  de  cuello  en  el  tratamiento  del  cáncer  de  glándulas  salivales  mayores.  Otolaryngol  Clin  North  Am
31:815­822,  1998.

10.  Rabinov  JD:  Imágenes  de  la  patología  de  las  glándulas  salivales.  Radiol  Clin  North  Am  38:1047­1057,  2000.

11.  Spiro  RH:  Diagnóstico  y  trampas  en  el  tratamiento  de  tumores  de  parótida.  Semin  Surg  Oncol  7:20­24,  1991.

12.  Zbaren  P,  Schar  C,  Hotz  MA  et  al .:  Valor  de  la  citología  por  aspiración  con  aguja  fina  de  masas  de  glándulas  parótidas.
Laringoscopio  111:1989­1992,  2001.
Machine Translated by Google

MASAS  CUELLO
CAPITULO  
67

Nathan  W.  Pearlman,  MD

1.  Un  hombre  de  34  años  presenta  una  masa  ipsolateral  de  2  a  3  cm  justo  debajo  del  ángulo  de  la  mandíbula.  ¿Cuál  es  el  
diagnóstico  diferencial?
Agrandamiento  inespecífico  de  los  ganglios  linfáticos
Quiste  de  hendidura  branquial
Infección  intraoral

Tumores  de  las  glándulas  salivales

Proceso  infeccioso  viral  o  bacteriano
linfoma
Tumor  del  cuerpo  carotideo
Carcinoma  metastásico
Tuberculosis

Anatomía  normal  prominente

2.  ¿Anatomía  normal?
Sí.  En  algunos  pacientes,  una  «masa»  en  el  cuello  no  es  más  que  una  glándula  submaxilar  normal  o  un  músculo  
omohioideo  que  se  ha  vuelto  prominente  como  resultado  de  un  ligero  agrandamiento  o  pérdida  de  la  grasa  circundante.  
El  hallazgo  clave  es  una  masa  similar,  pero  quizás  algo  más  pequeña,  en  el  mismo  lugar  del  otro  lado.

3.  ¿Este  paciente  parece  muy  joven  para  el  cáncer  metastásico?
No  precisamente.  El  cáncer  de  tiroides  metastásico  se  presenta  con  frecuencia  como  un  ganglio  agrandado  
en  el  cuello  en  pacientes  de  entre  20  y  40  años.  Aunque  el  cáncer  de  nasofaringe  y  el  cáncer  de  lengua  son  poco  
comunes  en  este  grupo  de  edad,  no  son  raros.

4.  ¿Hay  alguna  forma  de  acotar  esta  lista  de  posibilidades?
  Linfadenopatía  inespecífica:  relativamente  asintomática  o  levemente  dolorosa,  aparición  reciente,  los  ganglios  
son  móviles  y  <3  cm,  la  piel  suprayacente  es  normal.
  Mononucleosis  infecciosa:  ganglios  linfáticos  grandes,  confluentes  y  dolorosos,  de  aparición  reciente  (con  o  sin  
febrícula),  bilaterales,  de  textura  blanda  y  piel  suprayacente  normal.
  Infección  bacteriana:  ganglios  linfáticos  agrandados,  indurados  y  dolorosos,  piel  suprayacente  y  unilateral
eritematoso
  Tumores  del  cuerpo  carotídeo:  sensibles  o  indoloros,  unilaterales  o  bilaterales,  de  larga  duración,  de  
consistencia  gomosa,  no  pueden  separarse  del  pulso  carotídeo.
  Quistes  de  la  hendidura  branquial:  unilaterales,  blandos,  no  dolorosos,  de  larga  duración  y  transiluminados.
  Linfoma:  ganglios  no  dolorosos,  >3  cm,  unilateral  o  bilateral,  relativamente  blandos,  de  aparición  reciente.
Los  signos  de  enfermedad  sistémica  (fiebres  recurrentes,  pérdida  de  peso,  etc.)  pueden  o  no  estar  presentes.
  Tumor  de  las  glándulas  salivales  (submaxilar,  parótida):  gomoso,  indoloro,  relativamente  inmóvil.
Recuerde,  al  examinar  al  paciente,  la  cola  de  la  parótida  se  extiende  hasta  el  ángulo  de  la  mandíbula.

  Carcinoma  metastásico:  ganglios  linfáticos  duros,  no  dolorosos,  a  menudo  >3  a  4  cm,  por  lo  general  presentes  durante  
al  menos  varias  semanas.
&  Tuberculosis:  imita  todas  estas  condiciones.

342
Machine Translated by Google
CAPÍTULO  67  MASAS  EN  EL  CUELLO  343

5.  ¿Por  qué  no  simplemente  extirpar  la  masa  o  el  ganglio  linfático  para  ver  qué  es?
La  biopsia  por  escisión  nunca  debe  ser  la  maniobra  diagnóstica  inicial  a  menos  que  sea  absolutamente  necesaria.
Si  hay  linfoma  o  una  infección  inusual,  pero  no  se  sospecha,  el  ganglio  puede  ser  maltratado  cuando  se  
envía  a  patología  o  microbiología.  Si  el  cáncer  metastásico  es  el  problema,  la  cicatriz  creada  por  la  biopsia  
hará  que  cualquier  disección  posterior  del  cuello  sea  más  difícil  de  lo  necesario.  Una  mejor  opción  para  el  
diagnóstico  inicial  es  la  aspiración  con  aguja  fina  (FNA),  que  tiene  una  precisión  del  95  %  y  evita  los  problemas  
de  la  biopsia  abierta.

6.  Un  examen  completo  de  cabeza  y  cuello  no  muestra  nada  anormal,  pero  la  aspiración  con  aguja  fina  de  la  masa  
revela  cáncer  escamoso  en  un  ganglio  linfático.
¿Qué  se  debe  hacer  a  continuación?
Examen  de  la  boca,  faringe,  laringe,  esófago  y  árbol  traqueobronquial  (triple  endoscopia)  bajo  anestesia.  Si  
no  se  ve  nada,  proceda  a  una  biopsia  ciega  de  la  nasofaringe,  la  base  de  la  lengua  y  los  senos  piriformes.

7.  ¿Es  esto  un  poco  demasiado?
No.  El  cáncer  escamoso  provino  de  algún  lugar  de  la  región  y  el  lugar  más  probable  se  encuentra  en  algún  lugar  
del  tracto  aerodigestivo  superior  (boca,  faringe,  etc.).  En  aproximadamente  el  15%  de  los  casos,  el  tumor  primario  
se  detecta  solo  mediante  examen  bajo  anestesia,  y  en  otro  10%  de  los  pacientes  se  encontrará  un  segundo  tumor  
primario  sincrónico  del  tracto  aerodigestivo.

8.  ¿Por  qué  realizar  un  examen  de  cabeza  y  cuello  en  la  oficina  si  voy  a
proceder  a  la  triple  endoscopia  de  todos  modos?
El  examen  del  paciente  mientras  está  despierto  proporciona  información  sobre  la  función  de  la  lengua  y  la  
laringe  que  no  se  puede  obtener  mientras  duerme  y  ayuda  en  la  planificación  del  tratamiento.  Además,  el  
examen  bajo  anestesia  puede  ser  una  búsqueda  a  ciegas  debido  al  colapso  de  la  lengua  y  la  faringe,  a  menos  
que  lo  indiquen  los  hallazgos  observados  mientras  está  despierto.

9.  ¿Debería  hacerme  una  tomografía  computarizada  o  una  resonancia  magnética?

Sí.  Ambas  modalidades  brindan  información  que  puede  ser  difícil  de  obtener  mediante  un  examen  físico,  como  la  
afectación  de  los  tejidos  en  la  base  del  cráneo.  Sin  embargo,  estas  pruebas  no  reemplazan  las  medidas  ya  descritas.

10.  Todo  eso  está  hecho,  y  todavía  no  puedo  encontrar  un  tumor  primario.  ¿Ahora  que?
Existen  dos  opciones.  Se  puede  proceder  a  una  disección  del  cuello  homolateral  funcional  o  modificada,  
conservando  la  mayor  función  posible;  luego  use  irradiación  posoperatoria  para  tratar  el  cuello  y  el  sitio  probable  del  
tumor  primario.  Como  alternativa,  se  puede  utilizar  la  irradiación  primaria  sola,  reservando  la  cirugía  para  la  
persistencia  o  recurrencia  de  la  enfermedad  en  el  cuello.

11.  ¿Qué  pasa  si  el  tumor  primario  nunca  aparece?  ¿Esto  influye  en  la  supervivencia?
No.  El  pronóstico  está  determinado  por  la  presencia  de  enfermedad  metastásica,  en  este  caso  en  los  ganglios  
del  cuello.

12.  Si  la  masa  o  el  ganglio  agrandado  está  en  el  triángulo  posterior  del  cuello,  ¿es  el
el  trabajo  sigue  igual?
Sí.  Aunque  la  mayoría  de  los  tumores  orales  o  faríngeos  se  propagan  primero  a  los  ganglios  del  triángulo  anterior,  
no  es  raro  que  los  tumores  nasofaríngeos  o  hipofaríngeos,  los  cánceres  de  tiroides  y  los  linfomas  se  presenten  
como  ganglios  del  triángulo  posterior  agrandados.

13.  ¿Qué  pasa  si  la  aspiración  con  aguja  fina  de  los  ganglios  solo  muestra  linfocitos  o  muestra
adenocarcinoma?
Lo  más  probable  es  que  la  presencia  de  solo  linfocitos  represente  inflamación  o  linfoma;  sin  embargo,  si  el  ''nódulo  
se  encuentra  justo  debajo  del  lóbulo  de  la  oreja,  puede  ser  un  tumor  de  Warthin  (cistoadenoma
Machine Translated by Google
344  CAPÍTULO  67  MASAS  EN  EL  CUELLO

linfomatoso)  de  la  glándula  parótida.  El  adenocarcinoma  encontrado  en  FNA  generalmente  indica  metástasis  
de  cáncer  de  tiroides  o  un  sitio  primario  debajo  de  las  clavículas;  sin  embargo,  también  podría  representar  la  
propagación  de  un  cáncer  de  glándula  salival.
Si  la  FNA  obtiene  solo  linfocitos,  ahora  puede  ser  razonable  extirpar  el  ganglio  para  una  mejor
estadificación  histológica;  pero,  como  se  señaló  anteriormente,  la  escisión  no  debe  ser  el  primer  paso.

14.  Los  bultos  en  el  cuello  son  comunes  y  relativamente  pocos  pacientes  tienen  cáncer.
¿Es  este  tipo  de  trabajo  excesivo  y  demasiado  caro?
No.  La  mayoría  de  los  pacientes  con  un  bulto  en  el  cuello  tienen  un  proceso  benigno  que  mejorará  por  sí  solo  sin  
cirugía.  Por  lo  tanto,  la  observación  inicial  durante  varias  semanas  es  realmente  el  enfoque  más  rentable.  Luego,  si  
usted  o  el  paciente  todavía  quieren  saber  "qué  es",  la  FNA  es  más  barata,  menos  morbosa  y,  por  lo  general,  tan  
informativa  como  la  biopsia  abierta.
Por  otro  lado,  si  los  bultos  en  el  cuello  se  extirpan  de  forma  rutinaria  sin  un  diagnóstico  diferencial,  la
el  médico  estará  constantemente  sorprendido  por  los  hallazgos.  Si  el  ganglio  es  benigno,  el  paciente  se  
sometió  a  una  cirugía  innecesaria.  Si  hay  cáncer  o  infección,  aún  será  necesario  realizar  el  estudio  detallado  
anteriormente,  y  luego  en  un  campo  contaminado  por  la  biopsia.  Esto  no  es  rentable;  sino,  más  bien  una  pérdida  de  
tiempo  y  recursos.

BIBLIOGRAFÍA

1.  Attie  JN,  Setzon  M,  Klein  I.  Cáncer  de  tiroides  que  se  presenta  como  ganglios  linfáticos  cervicales  agrandados.  Am  J  Surg
166:428­430,  1993.

2.  King  AD,  Ahuja  AT,  Yeung  DKW  et  al .:  Linfadenopatía  cervical  maligna:  precisión  diagnóstica  de  la  resonancia  
magnética  ponderada  por  difusión.  Radiología  245:806­813,  2007.

3.  Lee  NK,  Byers  RM,  Abbruzzese  JL  et  al.:  Adenocarcinoma  metastásico  en  el  cuello  de  un  tumor  primario  desconocido
fuente.  Am  J  Surg  162:306­309,  1991.

4.  Nasuti  JF,  Yu  G,  Boudousquie  A  et  al .:  Valor  diagnóstico  de  la  citología  por  aspiración  con  aguja  fina  de  ganglios  
linfáticos:  una  experiencia  institucional  de  387  casos  observados  durante  un  período  de  5  años.  Citopatología  11:18­31,  
2000.

5.  Rice  DH,  Sprio  RH:  Carcinoma  metastásico  del  cuello,  primario  desconocido.  En  Current  Concepts  in  Head  and  Neck  Cancer,  
Atlanta,  1989,  Sociedad  Americana  del  Cáncer.

6.  Tarantino  DR,  McHenry  CR,  Strickland  T  et  al.:  El  papel  de  la  biopsia  por  aspiración  con  aguja  fina  y  la  citometría  de  flujo  en  la  
evaluación  de  la  adenopatía  persistente  del  cuello.  Am  J  Surg  176:413­417,  1998.

7.  Troost  EG,  Vogel  WV,  Merkx  MA  et  al.:  18­FLT  PET  no  discrimina  entre  reactivo  y  metastásico
ganglios  linfáticos  en  pacientes  con  cáncer  primario  de  cabeza  y  cuello.  J  Nucl  Med  48:726­735,  2007.
Machine Translated by Google

VIII.  CIRUGÍA  VASCULAR

¿QUÉ  ES  LA  ATEROSCLEROSIS? CAPITULO  
68

Nancy  C.  Andersen,  MD  y  Craig  H.  Selzman,  MD

1.  ¿Hay  que  ser  mayor  para  tener  aterosclerosis?
No.  La  lesión  inicial  (o  tipo  I),  que  consiste  en  depósitos  de  lípidos  en  la  íntima,  ha  sido  bien  caracterizada  en  
lactantes  y  niños.

2.  ¿Qué  es  una  veta  grasa?
Las  estrías  de  grasa  o  las  lesiones  de  tipo  II  son  visibles  como  estrías,  parches  o  manchas  de  color  amarillo  en  la  
superficie  íntima  de  las  arterias.  Microscópicamente,  se  caracterizan  por  la  acumulación  intracelular  de  lípidos.

3.  ¿Qué  es  una  celda  de  espuma?
Una  célula  espumosa  es  cualquier  célula  que  ha  ingerido  lípidos,  dando  así  el  aspecto  histológico  de  una  vacuola  
jabonosa.  En  general,  una  célula  espumosa  se  refiere  a  un  macrófago  cargado  de  lípidos;  sin  embargo,  otras  
células  que  captan  lípidos,  en  particular  las  células  del  músculo  liso  vascular,  también  pueden  considerarse  células  
espumosas.

4.  Describa  la  progresión  de  la  aterosclerosis.
Aunque  la  secuencia  de  eventos  no  siempre  es  consistente,  las  estrías  grasas  progresan  a  lesiones  de  tipo  III  o  
intermedias.  Este  crecimiento  se  caracteriza  por  depósitos  extracelulares  de  lípidos,  que  por  lo  general  están  
clínicamente  ocultos.  Sin  embargo,  cuando  los  charcos  se  unen  para  crear  un  núcleo  de  lípido  extracelular  (lesión  de  
tipo  IV  o  ateroma),  la  arquitectura  de  los  vasos  sanguíneos  se  ha  alterado  lo  suficiente  como  para  volverse  clínicamente  
evidente.  Con  la  proliferación  de  células  de  músculo  liso  (SMC)  y  el  depósito  de  colágeno,  el  ateroma  se  convierte  en  
un  fibroateroma  (tipo  V).  El  fibroateroma  se  caracteriza  por  defectos  superficiales  trombogénicos  que  provocan  
hemorragia  intramural  o  trombo  intraluminal  (lesión  tipo  V),  lo  que  resulta  en  la  oclusión  del  vaso,  que,  en  el  caso  de  
una  arteria  coronaria,  resulta  en  un  infarto  de  miocardio  (IM).

5.  De  100  estudiantes  de  medicina  voluntarios,  ¿cuántos  tienen  aterosclerosis  significativa?

En  1953,  Enos  informó  sobre  los  resultados  de  la  autopsia  de  300  hombres  estadounidenses  muertos  en  
combate  en  Corea  (edad  promedio,  22  años).  Observó  que  el  77%  de  los  corazones  tenían  alguna  evidencia  
macroscópica  de  aterosclerosis  coronaria.  Alrededor  del  39  %  de  los  hombres  tenían  estrechamiento  luminal,  estimado  
entre  un  10  %  y  un  90  %,  y  el  3  %  tenía  placas  que  causaban  la  oclusión  completa  de  uno  o  más  vasos  coronarios.  Sin  
embargo,  un  estudio  posterior  que  evaluó  105  bajas  en  combate  en  Vietnam  demostró  que  solo  el  45  %  presentaba  
aterosclerosis  y  menos  del  5  %  se  consideraba  grave.  Finalmente,  un  estudio  reciente  que  analizó  a  105  víctimas  de  
trauma  corroboró  el  estudio  de  la  Guerra  de  Corea  al  demostrar  una  incidencia  del  78  %  de  aterosclerosis,  con  una  
afectación  significativa  de  dos  y  tres  vasos  principales  izquierdos  en  un  20  %.

6.  ¿Cuáles  son  los  factores  de  riesgo  clásicos  de  la  enfermedad  cardiovascular  aterosclerótica?
Los  factores  de  riesgo  clásicos  incluyen  el  tabaquismo,  la  hiperlipidemia,  la  hipertensión,  la  diabetes  mellitus  y  los  
antecedentes  familiares  de  enfermedad  cardiovascular.  La  evidencia  más  reciente  sugiere  la  importancia  de  la  obesidad,  
el  estrés  emocional  (más  débil)  y  la  inactividad  física  (ese  es  usted).

345
Machine Translated by Google
346  CAPÍTULO  68  ¿QUÉ  ES  LA  ATEROSCLEROSIS?

7.  ¿Cómo  factores  de  riesgo  tan  diversos  producen  enfermedades  similares?
Esa  es  la  pregunta  del  millón.  ¿Las  vías  paralelas  conducen  a  una  lesión  aterosclerótica  final,  o  los  factores  
de  riesgo  aparentemente  diferentes  activan  señales  que  convergen  en  unos  pocos  eventos  dominantes,  
promoviendo  el  desarrollo  de  la  aterosclerosis?  Ciertamente,  esta  pregunta  tiene  amplias  implicaciones  
terapéuticas.  Sería  mucho  más  fácil  inhibir  un  solo  punto  proximal  en  este  proceso  que  tratar  múltiples  eventos  
patológicos  celulares  más  distales  y  divergentes.

8.  ¿Cuál  es  la  respuesta  a  la  lesión?
La  premisa  de  que  la  aterogénesis  representa  una  respuesta  fibroproliferativa  inflamatoria  exagerada  a  la  
lesión  se  ha  convertido  en  una  atractiva  hipótesis  unificadora  de  la  enfermedad  vascular  y  la  reparación.  Las  
agresiones  mecánicas,  metabólicas  y  tóxicas  pueden  dañar  la  pared  del  vaso.  El  denominador  común  es  la  
lesión  endotelial.  La  alteración  del  endotelio  no  sólo  produce  disfunción  de  las  células  endoteliales,  sino  que  
también  permite  la  adhesión  y  transmigración  de  monocitos,  plaquetas  y  linfocitos  T  circulantes.  Dentro  de  la  
lesión  en  desarrollo,  las  células  activadas  liberan  potentes  moléculas  reguladoras  del  crecimiento  que  pueden  
actuar  tanto  de  forma  paracrina  como  autocrina.  Bajo  la  influencia  de  las  citocinas  y  los  factores  de  crecimiento,  
las  células  del  músculo  liso  vascular  (VSMC)  se  adaptan  a  un  fenotipo  sintético  y  comienzan  la  proliferación  y  la  
migración  a  través  de  la  lámina  elástica  interna  hacia  la  capa  íntima.  Las  VSMC  estimuladas  permiten  el  depósito  
de  matriz  extracelular,  convirtiendo  así  la  lesión  inicial  en  una  placa  fibrosa.

9.  ¿Qué  es  la  proteína  C  reactiva?  ¿Es  simplemente  otro  azar,  no  clínicamente  relevante?
marcador  de  inflamación?
La  proteína  C  reactiva  (PCR)  es  una  de  las  muchas  proteínas  de  fase  aguda  elaboradas  a  partir  de  hepatocitos  
bajo  estimulación  inflamatoria.  Aislado  originalmente  del  suero  de  pacientes  con  neumonía,  tiene  una  alta  
afinidad  de  unión  por  el  polisacárido  C  neumocócico.  Aunque  la  PCR  es  mejor  conocida  como  un  péptido  activo  
que  neutraliza  los  antígenos  extraños,  controla  el  daño  tisular  y  promueve  la  reparación  tisular,  se  considera  
cada  vez  más  como  un  marcador  sensible  de  la  inflamación.  A  diferencia  de  otros  marcadores  de  inflamación,  
los  niveles  de  CRP  son  estables  durante  largos  períodos  de  tiempo,  no  tienen  variación  diurna,  se  pueden  medir  
de  forma  económica  con  los  ensayos  de  alta  sensibilidad  disponibles  y  han  demostrado  especificidad  en  la  
predicción  del  riesgo  de  eventos  cardiovasculares.  De  hecho,  la  elevación  de  los  niveles  de  PCR  podría  ser  más  
predictiva  de  eventos  cardíacos  que  la  elevación  de  los  niveles  de  lipoproteínas  de  baja  densidad  (LDL).
Estas  observaciones  pueden  influir  en  la  terapia  porque  los  pacientes  no  hiperlipidémicos  con  niveles  elevados  
de  PCR  podrían  beneficiarse  de  una  terapia  agresiva  con  estatinas  (inhibidores  de  la  3­hidroxi­3­metilglutaril  
[HMG]­reductasa).

10.  ¿La  lesión  vascular  significa  solo  una  lesión  física  directa,  como  con  una  angioplastia?
¿catéter?
No.  Lesión  es  una  palabra  comodín  que  incluye  lesiones  físicas,  como  angioplastia,  hipertensión  y  fuerzas  de  
cizallamiento  (las  lesiones  ateroscleróticas  generalmente  ocurren  en  las  bifurcaciones)  y  otros  insultos  diversos,  
incluidos  virus,  bacterias,  nicotina,  homocisteína  y  LDL  oxidadas.

11.  ¿Son  importantes  los  lípidos?
La  hipótesis  de  los  lípidos  de  la  aterosclerosis  sugiere  que  los  cambios  celulares  en  la  aterosclerosis  son  eventos  
reactivos  en  respuesta  a  la  infiltración  de  lípidos.  De  hecho,  la  terapia  antilipídica  es  una  de  las  pocas  estrategias  
que  ha  inducido  la  regresión  de  la  aterosclerosis  en  ensayos  clínicos  prospectivos  aleatorizados.  La  evidencia  
fuerte  también  se  deriva  de  pacientes  con  hiperlipidemias  genéticas;  los  homocigotos  rara  vez  viven  más  allá  de  
los  26  años.

12.  ¿Qué  es  el  síndrome  metabólico?
A  menudo  denominado  síndrome  X,  el  síndrome  metabólico  es  un  fenómeno  en  personas  mayores  
sedentarias  que  tienen  hiperinsulinemia  asociada  con  niveles  elevados  de  azúcar  en  la  sangre,  presión  arterial  
alta  y  triglicéridos  elevados  con  niveles  reducidos  de  colesterol  de  lipoproteínas  de  alta  densidad  (HDL).
Se  estima  que  la  prevalencia  en  los  Estados  Unidos  es  de  casi  el  25%  de  la  población.  Clínicamente,
Machine Translated by Google
CAPÍTULO  68  ¿QUÉ  ES  LA  ATEROSCLEROSIS?  347

tales  pacientes  desarrollan  enfermedad  cardiovascular  prematura.  La  resistencia  a  la  insulina  con  niveles  elevados  
de  insulina,  con  o  sin  diabetes  manifiesta,  alimenta  aspectos  importantes  de  la  aterogénesis,  incluidas  las  
dislipidemias,  la  disfunción  endotelial,  la  hipertensión  y  la  proliferación  de  SMC.  Para  los  fanáticos  de  las  trivias  y  
para  que  no  se  confundan  con  sus  suegros,  se  ha  observado  una  condición  similar  en  caballos  con  sobrepeso  
conocida  como  síndrome  metabólico  equino.

13.  ¿Qué  es  la  leptina?  ¿Cuál  es  su  asociación  con  la  aterosclerosis?
La  leptina  es  una  hormona  secretada  por  los  adipocitos  que  mantiene  la  homeostasis  entre  las  reservas  de  energía  
y  el  gasto  de  energía  mediante  la  regulación  del  apetito  y  la  ingesta  de  alimentos.  Las  alteraciones  en  la  vía  de  
señalización  de  la  leptina,  ya  sea  a  través  de  la  resistencia  a  la  leptina  o  del  agotamiento  de  la  leptina,  pueden  
provocar  una  ingesta  excesiva  de  alimentos  y  obesidad.  La  leptina  también  es  proaterogénica.  Señala  la  
proliferación  de  monocitos,  promueve  el  estrés  oxidativo  en  la  célula  endotelial  y  provoca  la  hipertrofia  y  proliferación  
de  las  células  del  músculo  liso  vascular.

14.  ¿Por  qué  la  vitamina  E  sería  (incluso  teóricamente)  protectora  contra  enfermedades  cardiovasculares?
¿enfermedad?

La  terapia  antioxidante  con  vitaminas  C  y  E  y  betacaroteno  es  intuitivamente  sólida.  In  vitro,  estos  agentes  
proporcionan  resistencia  de  LDL  a  la  oxidación  y  reducen  la  elaboración  de  especies  reactivas  de  oxígeno  que  
dañan  los  vasos.  Los  metabolitos  reactivos  del  oxígeno  (hasta  el  5%  del  oxígeno),  como  el  superóxido  y  el  peróxido  
de  hidrógeno,  dañan  directamente  las  células  vasculares,  alteran  la  función  vasomotora  endotelial,  promueven  la  
agregación  plaquetaria  y  la  adhesión  de  leucocitos,  y  estimulan  la  proliferación  de  SMC  vasculares.  Aunque  los  
estudios  descriptivos,  de  casos  y  controles  y  de  cohortes  prospectivos  han  encontrado  asociaciones  inversas  entre  
la  frecuencia  de  la  enfermedad  de  las  arterias  coronarias  (CAD)  y  la  ingesta  dietética  de  vitaminas  antioxidantes,  
los  ensayos  terapéuticos  aleatorios  hasta  el  momento  no  han  demostrado  ningún  beneficio  al  hacerlo.

15.  ¿Qué  es  la  homocisteína?
Este  aminoácido  intermedio  en  el  metabolismo  de  la  metionina  es  un  aminoácido  esencial  en  la  síntesis  de  
proteínas  tanto  animales  como  vegetales.  El  exceso  de  homocisteína  en  la  pared  del  vaso  reacciona  con  las  
proteínas  de  baja  densidad  (LDP)  para  crear  especies  reactivas  de  oxígeno  dañinas.
La  evidencia  epidemiológica  correlaciona  los  niveles  elevados  de  homocisteína  y  los  niveles  reducidos  de  folato  
con  la  enfermedad  cardiovascular.

16.  ¿Cómo  se  clasifica  la  homocisteína  como  factor  de  riesgo  de  aterosclerosis?
Se  estima  que  el  10%  del  riesgo  de  CAD  en  la  población  general  es  atribuible  a  la  homocisteína.  Un  
aumento  de  5  mmol/L  en  la  concentración  plasmática  de  homocisteína  (normal,  5  a  15  mmol/L)  aumenta  el  
riesgo  de  enfermedad  coronaria  tanto  como  un  aumento  de  20  mg/dl  en  la  concentración  de  colesterol.

17.  ¿Todo  el  mundo  debería  tomar  suplementos  de  folato?
El  ácido  fólico,  las  vitaminas  B12  y  B6  y  la  piridoxina  son  cofactores  importantes  para  el  procesamiento  
enzimático  de  la  homocisteína.  De  hecho,  la  reducción  de  la  mortalidad  por  causas  cardiovasculares  desde  
1960  se  ha  correlacionado  con  el  aumento  de  la  suplementación  con  vitamina  B6  en  el  suministro  de  alimentos.  
Aunque  estos  suplementos  pueden  disminuir  los  niveles  de  homocisteína,  la  disminución  esperada  de  los  eventos  
cardiovasculares  aún  no  se  ha  documentado  en  ensayos  clínicos  prospectivos  aleatorizados.
De  hecho,  los  resultados  del  estudio  HOPE­2  concluyeron  que  en  pacientes  con  enfermedad  vascular  o  
diabetes  mayores  de  55  años,  la  suplementación  dietética  con  ácido  fólico,  B6  y  B12  durante  5  años  no  redujo  el  
riesgo  de  eventos  cardiovasculares  mayores.

18.  ¿Qué  microorganismos  se  han  implicado  en  la  aterosclerosis?
Las  bacterias  incluyen  Chlamydia  pneumoniae,  Helicobacter  pylori,  estreptococos  y  Bacillus  typhosus.  
Los  virus  incluyen  influenza,  virus  del  herpes,  adenovirus  y  citomegalovirus.
Machine Translated by Google
348  CAPÍTULO  68  ¿QUÉ  ES  LA  ATEROSCLEROSIS?

19.  ¿Las  personas  con  enfermedades  de  transmisión  sexual  tienen  mayor  riesgo  de
¿enfermedad  cardiovascular?
La  descripción  epidemiológica  inicial  que  vincula  a  las  especies  de  Chlamydia  con  la  aterosclerosis  fue  reportada  por  
venerólogos  en  América  del  Sur  en  la  década  de  1940.  C.  pneumoniae,  un  organismo  respiratorio  ubicuo,  es  la  
especie  predominante  identificada  posteriormente  en  las  lesiones  cardiovasculares.  Más  del  50%  de  la  población  
tiene  anticuerpos  anticlamidiales  (ACA)  a  la  edad  de  50  años;  sin  embargo,  este  50%  de  la  población  no  tiene  esta  
enfermedad  de  transmisión  sexual  (ETS).

20.  ¿Existe  un  equivalente  de  úlcera  péptica  por  H.  pylori  en  la  aterosclerosis?  ¿Deberíamos  todos
tomar  un  macrólido  al  día?
El  jurado  aún  está  deliberando.  Es  poco  probable  que  la  erradicación  de  especies  de  Chlamydia  tenga  el  
mismo  efecto  profundo  sobre  la  enfermedad  que  la  erradicación  de  H.  pylori.  Sin  embargo,  C.  pneumoniae  
puede  ser  otro  factor  que  exacerbe  la  respuesta  a  la  lesión.  La  evidencia  sugiere  que  la  terapia  con  antibióticos  
disminuye  el  número  de  eventos  cardiovasculares  en  pacientes  con  títulos  elevados  de  ACA.

21.  Si  tiene  múltiples  caries,  ¿debería  programar  electivamente  su  cirugía  coronaria?
cirugía  de  derivación  arterial?
En  varios  estudios  de  cohortes,  la  periodontis  crónica  se  asoció  con  un  riesgo  15%  mayor  de  desarrollar  
enfermedad  coronaria.  Una  evaluación  más  cercana  sugiere  que  este  vínculo  es  en  realidad  mucho  más  débil.  El  
principal  problema  de  los  estudios  existentes  es  la  alta  incidencia  de  abuso  de  tabaco  en  pacientes  con  enfermedades  
dentales.  Curiosamente,  cuando  se  maximiza  la  higiene  oral  mediante  la  extracción  dental  completa,  las  personas  
desdentadas  tenían  un  riesgo  similar  de  enfermedad  cardíaca  que  las  personas  con  periodontitis  crónica.

22.  ¿Cuál  es  el  papel  del  endotelio?
La  pared  de  un  vaso  sanguíneo  sano  está  revestida  por  una  monocapa  de  células  endoteliales  
extraordinariamente  metabólicamente  activas.  El  área  superficial  del  endotelio  es  de  aproximadamente  5000  m2  
pero  comprende  sólo  el  1%  del  peso  corporal  total.  Mientras  actúa  como  una  barrera  física  para  proteger  el  vaso  
subyacente  y  permite  que  los  elementos  sanguíneos  formados  fluyan  libremente,  evitando  así  la  trombosis,  esta  
capa  aparentemente  bucólica  es  un  centro  de  control  central  de  la  fisiología  vascular.  El  endotelio  es  un  punto  de  
acoplamiento  clave  para  monocitos,  neutrófilos  y  linfocitos  en  virtud  de  su  capacidad  para  expresar  moléculas  de  
adhesión  pegajosas  y  específicas  de  células.  El  endotelio  es  una  fuente  de  citocinas  y  factores  de  crecimiento  
peptídicos  que  actúan  tanto  de  forma  autocrina  como  paracrina  para  promover  la  aterogénesis.

23.  ¿Cuáles  son  algunos  de  los  productos  de  las  células  endoteliales  que  gobiernan  el  tono  vasomotor?
Los  factores  que  favorecen  la  relajación  vascular  incluyen  el  óxido  nítrico  y  la  prostaciclina.  Por  el  contrario,  
los  factores  que  favorecen  la  constricción  vascular  incluyen  tromboxano,  leucotrienos,  radicales  libres,  
endotelinas  y  citocinas  (p.  ej.,  factor  de  necrosis  tumoral  [TNF]  e  interleucina­1).

24.  ¿Cuál  es  la  importancia  de  la  trombosis  vascular?
La  trombosis  es  central  en  la  patogenia  de  la  insuficiencia  arterial  aguda  y  los  síndromes  coronarios  o  
cerebrovasculares  agudos,  incluida  la  angina  inestable,  el  infarto  de  miocardio  sin  onda  Q,  agudo  (elevación  del  segmento  ST)
IM  y  oclusión  de  vasos  después  de  una  intervención  vascular  (angioplastia).

25.  Describa  las  tres  fases  principales  de  la  afectación  plaquetaria  con  trombo.
formación.
Las  tres  fases  principales  de  la  participación  de  las  plaquetas  en  la  formación  de  trombos  son  la  adhesión,  la  
activación  y  la  agregación  de  las  plaquetas.  Con  la  exposición  del  espacio  subendotelial  después  de  una  lesión  
vascular,  las  plaquetas  se  adhieren  a  las  proteínas  de  la  membrana  basal  expuestas,  como  los  proteoglicanos,  
el  colágeno,  la  fibulina  y  la  laminina,  y  a  las  moléculas  secretadas  localmente,  como  el  factor  von  Willebrand  
(vWF),  a  través  de  sus  receptores  de  glicoproteínas  de  membrana.  La  activación  plaquetaria  se  produce  después  
de  la  adhesión,  lo  que  mejora  la  capacidad  de  las  plaquetas  cercanas  para  unirse  al  trombo  en  desarrollo.
Este  proceso  depende  de  la  energía  y  requiere  trifosfato  de  adenosina  (ATP).  el  predominante
Machine Translated by Google
CAPÍTULO  68  ¿QUÉ  ES  LA  ATEROSCLEROSIS?  349

los  estimuladores  de  la  activación  incluyen  colágeno,  vWF,  epinefrina  y  tromboxano  A2.  Por  último,  se  
produce  la  agregación  plaquetaria,  en  la  que  las  plaquetas  se  acumulan  de  manera  amplificada  y  
conducen  a  la  formación  final  del  trombo.  Este  paso  está  mediado  por  el  receptor  de  glicoproteína  IIb/IIIa  
y  su  interacción  con  vWF,  fibronectina  y  fibrinógeno.  Este  proceso  toma  solo  unos  minutos.
Nuestros  colegas  cardiólogos  utilizan  activamente  el  bloqueo  farmacológico  del  receptor  de  glicoproteína  IIb/
IIIa  para  el  tratamiento  de  los  síndromes  coronarios  agudos.

26.  ¿Cuál  es  el  mecanismo  de  ruptura  de  la  placa?
El  soporte  estructural  de  una  placa  aterosclerótica  es  la  cubierta  fibrosa,  una  capa  organizada  de  SMC  y  
tejido  conectivo.  Esta  tapa  sirve  como  barrera  subendotelial  entre  la  luz  del  vaso  y  el  núcleo  necrótico  
aterosclerótico,  lleno  de  gotitas  de  lípidos,  células  inflamatorias  y  sales  de  calcio.  Cuando  la  cubierta  fibrosa  
es  delgada,  puede  dañarse  por  las  citocinas  y  proteasas  inflamatorias  liberadas  por  los  macrófagos,  las  
células  T  y  los  mastocitos.  Una  vez  destruido,  el  contenido  del  núcleo  necrótico  queda  expuesto,  lo  que  
provoca  trombosis  y  oclusión  arterial  total  o  casi  total.  Este  proceso  ocurre  hasta  en  un  70%  de  las  trombosis  
de  arterias  coronarias.

27.  ¿Cuáles  son  algunas  de  las  complicaciones  clínicas  de  la  placa  aterosclerótica?
¿formación?
Dilatación  aneurismática,  estenosis  y  oclusión  arterial,  ruptura  de  la  pared  arterial  y  eventos  tromboembólicos  que  conducen  a  
infarto  de  miocardio  y  accidente  cerebrovascular.

28.  Si  la  aterosclerosis  es  una  enfermedad  inflamatoria,  ¿deberíamos  estar  todos  tomando  una  aspirina?
¿un  día?
Tal  vez.  Las  estrategias  destinadas  a  limitar  la  cascada  inflamatoria  son  prometedoras  
como  terapia  antiaterosclerosis.  Los  ejemplos  de  uso  diario  incluyen  aspirina,  fibrinolíticos,  inhibidores  de  la  
HMG­reductasa  y  estrógenos.  Otros  en  el  ámbito  preclínico  incluyen  la  terapia  génica,  la  terapia  con  
anticitoquinas  y  la  terapia  con  factores  de  anticrecimiento.  Ciertamente,  la  prevención  primaria  es  importante  
para  limitar  el  estímulo  lesional  inicial.  Sin  embargo,  la  inflamación  latente  relacionada  con  la  aterosclerosis  
se  puede  atacar  mejor  modificando  la  respuesta  de  las  células  vasculares  a  estas  agresiones.

BIBLIOGRAFÍA

1.  Ayada  K,  Yokota  K,  Kabayashi  K  et  al.:  Infecciones  crónicas  y  aterosclerosis.  Ann  NY  Acad  Sci  1108:  594–602;
2007.

2.  Davi  G,  Patrono  C:  Activación  plaquetaria  y  aterotrombosis.  N  Engl  J  Med  357:2482­2494,  2007.

3.  Enos  WF,  Holmes  RH,  Beyer  J:  enfermedad  coronaria  entre  los  soldados  estadounidenses  muertos  en  acción  en  Corea.  JAMA
152:1090­1093,  1953.

4.  Fruchart  JC,  Nierman  MC,  Stroes  ESG:  Nuevos  factores  de  riesgo  para  la  aterosclerosis  y  evaluación  del  riesgo  del  paciente.
Circulación  109:III­15­III­19,  2004.

5.  Hansson  GK:  Inflamación,  aterosclerosis  y  enfermedad  de  las  arterias  coronarias.  N  Engl  J  Med  352:1685­1695,  2005.

6.  Selzman  CH,  Miller  SA,  Harken  AH:  implicaciones  terapéuticas  de  la  inflamación  en  la  enfermedad  cardiovascular  aterosclerótica.  
Ann  Thorac  Surg  71:2066­2074,  2001.

7.  The  Heart  Outcomes  Prevention  Evaluation  (HOPE)  2  Investigadores:  Reducción  de  la  homocisteína  con  ácido  fólico  y
Vitaminas  B  en  enfermedades  vasculares.  N  Engl  J  Med  354:1567­1577,  2006.

8.  Zimmerman  MA,  Selzman  CH,  Raeburn  CD  et  al.:  Implicaciones  diagnósticas  de  la  proteína  C  reactiva  en  la  
aterosclerosis.  Arch  Surg  138:220­224,  2003.
Machine Translated by Google

CAPITULO  
69

INSUFICIENCIA  ARTERIAL
Carlos  A.  Rueda,  MD  y  Mark  R.  Nehler,  MD

1.  Describir  la  claudicación  y  su  fisiología.
La  claudicación  intermitente  consiste  en  dolor  muscular  reproducible  en  las  extremidades  inferiores  inducido  por  el  
ejercicio  y  que  se  alivia  con  breves  períodos  de  descanso.  Se  produce  por  la  obstrucción  arterial  de  los  lechos  
musculares  afectados,  lo  que  restringe  el  aumento  normal  del  flujo  sanguíneo  inducido  por  el  ejercicio,  produciendo  
una  isquemia  muscular  transitoria.  Los  estudios  han  demostrado  que  más  de  la  mitad  de  los  pacientes  con  claudicación  
intermitente  nunca  se  han  quejado  de  este  síntoma  a  sus  médicos,  asumiendo  que  la  dificultad  para  caminar  es  una  
consecuencia  normal  del  envejecimiento.  Finalmente,  solo  un  tercio  o  menos  de  los  pacientes  con  enfermedad  arterial  
periférica  (EAP)  tienen  claudicación  típica;  otros  tienen  dolor  atípico  en  las  piernas  o  son  asintomáticos  porque  las  
comorbilidades  médicas  limitan  la  deambulación.

2.  Enumerar  las  diferentes  terapias  no  quirúrgicas  para  la  claudicación  intermitente.
Modificación  de  factores  de  riesgo,  ejercicio  y  terapias  farmacológicas.  El  abandono  del  hábito  de  fumar  duplica  de  
forma  fiable  las  distancias  recorridas,  y  la  necesidad  de  una  eventual  amputación  en  pacientes  con  enfermedad  
oclusiva  arterial  de  las  extremidades  inferiores  disminuye  después  de  dejar  de  fumar.  El  ejercicio  (definido  como  
caminar  hasta  el  inicio  del  dolor  en  la  pierna,  descansar  y  luego  reanudar  la  caminata)  durante  30  a  60  minutos,  3  días  
a  la  semana  durante  6  meses  también  ha  demostrado  en  múltiples  ensayos  aleatorios  que  aumenta  la  distancia  
recorrida  en  más  del  100  %.  Actualmente,  los  únicos  medicamentos  aprobados  por  la  Administración  de  Drogas  y  
Alimentos  (FDA,  por  sus  siglas  en  inglés)  para  el  tratamiento  de  la  claudicación  son  la  pentoxifilina  (mínimamente  
efectiva)  y  el  cilostazol  (aparentemente  más  efectivo).  La  terapia  farmacológica  también  debe  dirigirse  a  la  dislipidemia,  
la  hipertensión  y  el  control  de  la  glucemia.  Además,  la  terapia  antiplaquetaria  de  por  vida  es  esencial.

3.  Definir  isquemia  crítica  de  extremidades.
La  isquemia  crítica  de  la  extremidad  amenaza  potencialmente  la  viabilidad  de  la  extremidad.  Los  síntomas  incluyen  
dolor  en  reposo  (etapa  III  de  Fontaine  [p.  ej.,  dolor  en  el  pie  en  reposo])  que  suele  ocurrir  por  la  noche  cuando  el  
paciente  está  en  decúbito  supino  y  ya  no  está  presente  la  contribución  de  la  gravedad  a  la  presión  arterial  del  pie.  Este  
dolor  se  alivia  con  la  dependencia  del  pie  o  períodos  cortos  de  deambulación.  La  mala  circulación  de  los  tejidos  no  
cura  las  lesiones  menores  de  la  piel  causadas  por  un  traumatismo  incidental.  Estas  úlceras  isquémicas  (etapa  IV  de  
Fontaine)  suelen  ser  dolorosas  y  pueden  progresar  a  gangrena.  La  isquemia  crítica  de  las  extremidades  implica  
cronicidad  y  debe  distinguirse  de  la  isquemia  aguda  de  las  extremidades,  que  se  debe  a  una  reducción  súbita  (definida  
como  2  semanas  o  menos)  en  la  perfusión  de  las  extremidades.

4.  ¿Qué  es  el  índice  tobillo­brazo  (ITB)?
ABI  es  la  presión  más  alta  del  tobillo  (arteria  tibial  anterior  o  tibial  posterior)  dividida  por  la  mayor  de  las  dos  presiones  
braquiales.  El  ITB  normal  es  ligeramente  >  1  (1,10).  Un  ABI  de  1,0  a  0,5  es  típico  de  pacientes  con  claudicación.  Los  
pacientes  con  dolor  en  reposo  tienen  un  ABI  <0,5  y  los  pacientes  con  necrosis  tisular  a  menudo  tienen  un  ABI  mucho  
más  bajo.

5.  Describir  la  historia  natural  de  la  claudicación.
Múltiples  estudios  de  historia  natural  han  documentado  la  naturaleza  benigna  de  la  claudicación.  La  tasa  de  
amputación  acumulada  de  10  años  es  del  10%.  Un  tercio  de  los  pacientes  experimenta  un  deterioro  de  los  
síntomas  y  la  mitad  de  estos  pacientes  requieren  algún  tipo  de  revascularización.  El  tabaquismo  continuo  y  la  
diabetes  son  factores  de  riesgo  importantes  para  la  progresión.

350
Machine Translated by Google
CAPÍTULO  69  INSUFICIENCIA  ARTERIAL  351

6.  Describa  la  historia  natural  de  la  isquemia  crítica  de  las  extremidades.
La  isquemia  crítica  de  las  extremidades  a  menudo  requiere  revascularización  o  amputación  primaria.  Las  intervenciones  
percutáneas  se  utilizan  cada  vez  más  como  terapia  primaria  con  procedimientos  quirúrgicos  o  amputación  si  fallan.  
Los  grupos  de  tratamiento  no  intervencionista  de  varios  ensayos  farmacológicos  para  la  isquemia  crítica  de  las  
extremidades  notaron  una  mejoría  de  los  síntomas  con  el  tiempo  en  el  40  %  de  los  pacientes.
Debido  a  que  los  pacientes  con  isquemia  crítica  de  las  extremidades  tienen  un  riesgo  tres  veces  mayor  de  mortalidad  
cardiovascular  que  aquellos  con  claudicación  intermitente,  el  50%  de  los  pacientes  con  isquemia  crítica  de  las  
extremidades  sucumben  a  la  enfermedad  cardíaca  dentro  de  los  5  años.  Finalmente,  un  subgrupo  de  pacientes  con  
isquemia  crítica  de  extremidades  no  puede  tratarse  de  manera  efectiva  con  revascularización  quirúrgica  o  endovascular.
El  manejo  meticuloso  de  la  herida  puede  ayudar  a  los  pacientes  con  úlceras  crónicas  no  complicadas  que  no  cicatrizan  
sin  revascularización.  En  ensayos  aleatorizados,  del  15%  al  20%  de  los  pacientes  que  recibieron  placebo  curan  a  los  6  
meses,  pero  las  tasas  de  recurrencia  son  altas.  Otras  terapias,  incluida  la  angiogénesis,  se  encuentran  en  etapa  de  ensayo  
clínico.

7.  ¿Qué  son  las  presiones  segmentarias  de  las  extremidades?  ¿Cómo  se  usan?
Así  como  el  ABI  se  registra  en  el  tobillo,  los  manguitos  en  la  parte  alta  del  muslo,  por  encima  de  la  rodilla  y  por  debajo  
del  nivel  de  la  rodilla  pueden  registrar  presiones.  Observar  la  ubicación  de  las  disminuciones  de  la  presión  arterial  puede  
determinar  el  nivel  de  la  obstrucción  vascular.  Por  lo  general,  una  reducción  de  la  presión  de  20  mm  Hg  o  más  entre  los  
segmentos  se  considera  significativa  y  ayudará  a  determinar  el  nivel  de  obstrucción.

8.  Describa  la  historia  natural  de  las  oclusiones  del  injerto.
Aunque  los  injertos  de  derivación  pueden  mejorar  drásticamente  la  circulación  de  las  extremidades  inferiores,  tienen  una  
esperanza  de  vida  limitada.  Cuando  fallan  estos  injertos,  la  extremidad  afectada  suele  tener  peores  problemas  circulatorios  
que  antes  de  la  derivación.  Esto  se  debe  a  la  división  de  las  principales  vías  colaterales  arteriales  durante  la  operación  y  
la  propagación  o  embolización  del  trombo  para  ocluir  las  arterias  distales  en  el  momento  de  la  oclusión  del  injerto.

9.  ¿Cuál  es  el  pronóstico  de  los  pacientes  jóvenes  con  enfermedad  vascular?
La  aterosclerosis  significativa  en  pacientes  jóvenes  (edad  <40  años)  es  infrecuente.  Estos  pacientes  son  casi  
exclusivamente  fumadores  empedernidos  con  una  alta  incidencia  de  estados  de  hipercoagulabilidad  (fibrinólisis  defectuosa,  
anticuerpos  anticardiolipina,  homocisteinemia  o  deficiencias  en  anticoagulantes  naturales).  Aquellos  con  condiciones  que  
amenazan  las  extremidades  con  frecuencia  progresan  a  la  pérdida  de  una  extremidad  a  pesar  de  los  intentos  de  
revascularización.  Los  procedimientos  reconstructivos  tienen  una  longevidad  limitada  y  requieren  revisiones  frecuentes  en  
esta  población.

10.  Describa  la  distribución  anatómica  de  la  enfermedad  vascular  en  la  diabetes.

Los  pacientes  con  diabetes  son  únicos.  Tienen  predilección  por  la  calcificación  de  la  pared  arterial,  lo  que  hace  que  los  
estudios  de  diagnóstico  (presión  del  tobillo,  ABI)  no  sean  confiables  debido  a  la  elevación  falsa.  Las  arterias  digitales  
suelen  estar  respetadas  y  la  presión  del  dedo  gordo  del  pie  puede  usarse  para  aproximarse  a  la  presión  del  tobillo.  Las  
arterias  de  entrada  (es  decir,  la  aorta,  las  ilíacas,  las  femorales  comunes)  suelen  estar  respetadas.
La  enfermedad  intermitente  suele  estar  presente  en  las  arterias  femoral  superficial  y  poplítea.  La  enfermedad  oclusiva  
significativa  afecta  con  mayor  frecuencia  a  las  arterias  femoral  profunda,  tibial  posterior  y  anterior  y  pedal,  con  relativa  
preservación  de  la  arteria  peronea.

11.  ¿Cuáles  son  las  implicaciones  de  la  insuficiencia  renal  en  los  resultados?
Los  pacientes  con  insuficiencia  renal  terminal  que  tienen  isquemia  crítica  de  las  extremidades  se  encuentran  al  final  de  la  
vida,  con  tasas  de  supervivencia  a  3  años  <30  %,  similares  a  los  pacientes  con  cáncer  metastásico.  Además,  el  potencial  
de  curación  de  las  amputaciones  parciales  del  pie  después  de  una  revascularización  exitosa  es  limitado.
Las  reconstrucciones  en  estos  pacientes  son  técnicamente  difíciles  debido  a  los  objetivos  distales  calcificados.  La  
combinación  de  estos  problemas  ha  hecho  que  muchos  cirujanos  vasculares  desaconsejen  las  reconstrucciones  
vasculares  en  estos  pacientes.
Machine Translated by Google
352  CAPÍTULO  69  INSUFICIENCIA  ARTERIAL

12.  Discuta  el  concepto  de  flujo  de  entrada  versus  flujo  de  salida.
La  extremidad  se  considera  como  una  red  de  circulación  separada  cuando  se  planifican  los  procedimientos  de  
revascularización.  La  circulación  adecuada  de  las  piernas  requiere  que  la  sangre  entre  en  la  pierna  desde  el  corazón  (entrada)  y  
llegue  al  pie  desde  el  muslo  (salida).  En  la  extremidad  normal,  el  flujo  de  entrada  a  la  pierna  se  realiza  a  través  de  la  aorta  y  las  
arterias  ilíaca  y  femoral  común  y  profunda.  La  salida  normal  al  pie  es  la  poplítea  y  tres  arterias  tibiales  (anterior,  posterior  y  
peronea).  Para  que  las  derivaciones  funcionen,  necesitan  un  flujo  de  entrada  (es  decir,  que  entre  sangre)  y  un  flujo  de  salida  (es  
decir,  un  lecho  vascular  que  abastecer)  adecuados.  El  tratamiento  del  flujo  de  entrada  y  salida  a  menudo  se  logra  a  través  de  
procedimientos  híbridos  que  requieren  el  uso  de  procedimientos  endovasculares  y  quirúrgicos  para  tratar  todas  las  lesiones  
críticas  en  un  paciente.  Por  ejemplo,  una  estenosis  ilíaca  puede  tratarse  con  angioplastia  transluminal  percutánea  y  una  oclusión  
poplítea  con  derivación  quirúrgica.

13.  ¿Cuáles  son  las  opciones  para  los  conductos  autógenos?
El  éxito  de  la  derivación  infrainguinal  depende  en  gran  medida  del  conducto  (de  qué  está  hecho  el  injerto).  Las  mejores  opciones  
de  conducto  en  orden  de  preferencia  serían  un  segmento  único  de  vena  safena  mayor,  piezas  empalmadas  de  vena  safena,  
venas  safenas  menores  empalmadas,  venas  del  brazo,  venas  del  brazo  empalmadas  y  material  protésico  con  un  manguito  de  
vena  distal.  Las  venas  de  cadáveres  criopreservadas  son  costosas  y  generalmente  tienen  una  durabilidad  limitada.  Los  injertos  
protésicos  se  utilizan  mejor  para  objetivos  poplíteos  por  encima  de  la  rodilla  porque  se  doblan  en  la  articulación  de  la  rodilla  y  la  
falta  de  coincidencia  de  tamaño  en  las  arterias  más  distales  reduce  notablemente  su  longevidad  en  estas  posiciones.  Sin  
embargo,  los  injertos  de  politetrafluoroetileno  (PTFE)  unidos  a  heparina  se  han  utilizado  con  tasas  de  permeabilidad  comparables  
a  los  injertos  de  vena  autóloga  para  el  tratamiento  de  la  enfermedad  vascular  oclusiva  en  las  extremidades  inferiores.

Este  material  se  muestra  prometedor  para  pacientes  con  opciones  limitadas  de  conductos  autógenos.

14.  ¿Cuáles  son  las  indicaciones  de  la  arteriografía?
La  arteriografía  solo  se  realiza  para  planificar  futuras  operaciones  o  intervenciones.  La  arteriografía  diagnóstica  sin  intervención  
rara  vez  se  usa  en  la  enfermedad  oclusiva  de  las  extremidades  inferiores.  La  arteriografía  es  costosa  y  conlleva  un  riesgo  finito  
de  hemorragia,  lesión  arterial  con  trombosis  e  insuficiencia  renal  por  toxicidad  del  agente  de  contraste  (combinado  3%).

15.  ¿Cuáles  son  las  tasas  de  permeabilidad  de  los  procedimientos  de  entrada?
La  durabilidad  de  las  reconstrucciones  vasculares  se  mide  por  la  permeabilidad.  La  permeabilidad  tiene  tres  tipos,  
todos  medidos  a  través  de  un  método  de  tabla  de  vida,  lo  que  explica  el  número  moderado  de  muertes  (principalmente  de  origen  
cardíaco)  que  ocurren  en  pacientes  vasculares  a  lo  largo  del  tiempo.  La  permeabilidad  puede  ser  primaria  (el  injerto  se  ha  
mantenido  funcional  sin  ninguna  intervención),  primaria  asistida  (el  injerto  nunca  se  ha  trombosado  pero  ha  requerido  alguna  
intervención  para  mantener  su  funcionamiento)  o  secundaria  (el  injerto  se  ha  trombosado,  pero  una  intervención  lo  ha  reabierto  y  
vuelve  a  funcionar).  Los  cuatro  procedimientos  más  comunes  para  mejorar  el  flujo  de  entrada  son  la  angioplastia  ilíaca,  el  bypass  
aortofemoral,  el  bypass  femorofemoral  y  el  bypass  axilofemoral.  El  más  duradero  es  el  bypass  aortofemoral,  que  tiene  una  
permeabilidad  primaria  a  10  años  del  80%.  Las  tasas  de  permeabilidad  primaria  a  cinco  años  para  la  angioplastia  ilíaca  y  la  
derivación  axilofemoral  y  femorofemoral  son  del  65  %,  70  %  y  70  %,  respectivamente.

16.  ¿Cuáles  son  las  tasas  de  permeabilidad  de  los  procedimientos  de  derivación  infrainguinal?
Las  derivaciones  infrainguinales  incluyen  injertos  en  las  arterias  poplítea  por  encima  de  la  rodilla,  poplítea  por  debajo  de  la  
rodilla,  tibiales  y  podales.  Las  tasas  de  permeabilidad  primaria  a  cinco  años  para  injertos  poplíteos  por  encima  de  la  rodilla  con  
vena  safena  y  prótesis  son  del  80  %  y  65  %,  respectivamente.  Las  tasas  de  permeabilidad  primaria  a  cinco  años  para  los  injertos  
poplíteos  de  vena  safena  por  debajo  de  la  rodilla  son  del  75  %.  Las  tasas  de  permeabilidad  primaria  a  cinco  años  para  las  
derivaciones  tibiales  son  del  65  %.  La  tasa  de  permeabilidad  primaria  a  cinco  años  para  la  derivación  con  pedal  es  del  50%.
Los  injertos  de  PTFE  adheridos  con  heparina  han  logrado  resultados  comparables  a  los  de  la  vena  safena  autóloga  en  pacientes  
de  bajo  riesgo  en  series  pequeñas  y  son  una  posible  alternativa  para  los  pacientes  que  no  tienen  una  vena  adecuada.
Machine Translated by Google
CAPÍTULO  69  INSUFICIENCIA  ARTERIAL  353

17.  Mencione  la  causa  principal  de  mortalidad  perioperatoria.
La  mayoría  (>90  %)  de  todos  los  pacientes  con  enfermedad  vascular  periférica  tienen  enfermedad  arterial  coronaria  
(EAC)  subyacente.  Debido  a  las  limitaciones  ambulatorias  de  su  enfermedad  vascular  periférica,  la  mayoría  de  
estos  pacientes  no  tienen  síntomas  coronarios  evidentes.  La  causa  más  frecuente  de  mortalidad  perioperatoria  en  
cirugía  vascular  es  el  infarto  de  miocardio  (IM).  La  decisión  de  evaluar  y  revascularizar  (quirúrgicamente  o  con  
angioplastia  y  colocación  de  stent)  la  EAC  en  estos  pacientes  antes  de  la  operación  vascular  es  un  área  de  
controversia  constante.

18.  Mencione  la  causa  primaria  de  morbilidad  perioperatoria.
Las  complicaciones  de  la  herida  ocurren  en  aproximadamente  el  25%  de  los  pacientes  que  se  someten  a  derivación  de  las  
extremidades  inferiores  por  isquemia  crítica  de  las  extremidades.  El  linfedema  posoperatorio,  la  neuropatía  isquémica  y  la  
cicatrización  de  heridas  prolongada  (a  menudo  medida  en  meses  en  lugar  de  semanas)  son  aspectos  importantes  para  estos  
pacientes.

19.  ¿Cuáles  son  las  causas  del  fracaso  del  injerto?
Alrededor  del  20%  al  50%  de  los  pacientes  que  se  someten  a  una  cirugía  de  derivación  infrainguinal  
experimentarán  una  falla  del  injerto.  El  fracaso  temprano  (dentro  de  los  30  días)  es  causado  por  problemas  técnicos  
con  la  operación  (doblez  o  torsión  del  injerto,  estrechamiento  de  la  anastomosis,  sangrado,  infección,  colgajos  de  
íntima  o  embolización).  El  fracaso  del  injerto  entre  los  meses  2  y  18  suele  deberse  a  hiperplasia  fibrointimal  en  las  
anastomosis  distales  o  en  los  sitios  de  las  válvulas  venosas  dentro  del  injerto.  El  fracaso  tardío  del  injerto  (>18  
meses)  es  causado  con  mayor  frecuencia  por  aterosclerosis  recurrente.  Los  estados  de  hipercoagulabilidad  son  
una  causa  inusual  de  fracaso  del  injerto.

20.  ¿Qué  opciones  terapéuticas  existen  para  el  fracaso  del  injerto?
Si  un  injerto  de  vena  falla  inmediatamente  después  de  la  operación,  el  enfoque  correcto  es  explorar  la  
anastomosis  distal  y  corregir  el  supuesto  problema  técnico.  Si  un  injerto  falla  semanas  o  meses  después  de  la  
implantación,  el  curso  correcto  es  algo  controvertido.  Explorar  el  injerto  para  eliminar  mecánicamente  el  trombo  y  
reparar  cualquier  estenosis  tiene  una  tasa  de  éxito  baja  y  no  se  recomienda.
El  uso  de  terapia  trombolítica  para  abrir  el  injerto  y  luego  reparar  cualquier  estenosis  subyacente  parece  atractivo,  
pero  la  longevidad  de  los  injertos  tratados  de  esta  manera  ha  sido  pobre,  con  menos  del  50  %  de  permeabilidad  
restante  al  cabo  de  1  año.  Reemplazar  el  injerto  de  vena  con  un  nuevo  bypass  proporciona  la  alternativa  más  
duradera  siempre  que  sea  técnicamente  posible  y  el  paciente  sea  un  candidato  quirúrgico.  Los  principales  desafíos  
al  reemplazar  el  injerto  de  vena  son  encontrar  los  vasos  de  entrada  y  salida  adecuados  y  el  conducto  de  injerto  de  
derivación  adecuado.  Los  injertos  de  entrada  que  ocluyen  generalmente  se  manejan  con  trombectomía  quirúrgica  
y  revisión  de  la  estenosis  anastomótica  distal.

21.  ¿Qué  método  de  vigilancia  del  injerto  se  debe  utilizar?
Debido  a  las  opciones  limitadas  para  los  injertos  de  derivación  de  vena  ocluidos,  se  utilizan  estudios  de  
ultrasonido  (US)  para  detectar  estenosis  dentro  del  injerto  antes  de  la  oclusión.  Se  han  defendido  varios  criterios  
para  detectar  con  precisión  >50%  de  estrechamiento  dentro  del  injerto  o  de  las  arterias  nativas  de  entrada  y  salida.
Los  exámenes  del  injerto  se  realizan  1,  3,  6,  9  y  12  meses  después  de  la  operación  y,  posteriormente,  
anualmente.  Los  datos  de  la  historia  natural  indican  que  los  injertos  con  >50%  de  estenosis  que  no  se  tratan  
tienen  altas  tasas  de  fracaso  a  mediano  plazo.  Los  síntomas  recurrentes  y  los  cambios  en  el  ABI  son  demasiado  
insensibles  para  detectar  estas  lesiones.

22.  ¿Qué  opciones  terapéuticas  existen  para  las  estenosis  del  injerto?
La  mayoría  de  las  estenosis  de  injertos  de  vena  son  causadas  por  hiperplasia  fibrointimal  de  porciones  escleróticas  
de  los  sitios  de  injerto  o  válvula.  Estas  lesiones  tienen  una  consistencia  de  goma  firme  y  son  menos  susceptibles  
de  éxito  a  largo  plazo  con  angioplastia  percutánea,  aunque  las  lesiones  focales  generalmente  se  tratan  inicialmente  
de  esta  manera.  Las  técnicas  abiertas  (resección  e  interposición  de  injertos  de  vena  o  angioplastia  con  parche  de  
vena)  son  más  duraderas  pero  también  causan  más  morbilidad  en  el  paciente.  En  resumen,  los  resultados  de  la  
intervención  sobre  injertos  fallidos  son  superiores  a  los  resultados  de  la  intervención  sobre  aquellos  que  ya  han  
fallado.
Machine Translated by Google
354  CAPÍTULO  69  INSUFICIENCIA  ARTERIAL

23.  ¿Cuál  es  el  papel  de  la  angioplastia  ilíaca  y  la  colocación  de  stent?
Las  lesiones  ateroscleróticas  de  la  arteria  ilíaca  que  responden  mejor  a  la  angioplastia  con  balón  son  de  corta  
longitud  (<3  cm)  y  están  confinadas  a  la  arteria  ilíaca  común.  A  los  pacientes  sin  diabetes  les  va  mejor  que  a  
los  pacientes  con  diabetes.  Los  informes  actuales  de  éxito  inicial  son  >  90  %,  lo  que  ha  mejorado  con  el  uso  de  
stents  para  tratar  disecciones  arteriales  iatrogénicas  (separación  de  la  pared  arterial  en  las  capas  íntima  o  
media).  Las  técnicas  endovasculares  más  nuevas  que  combinan  angioplastia  y  stents  tienen  tasas  de  
permeabilidad  a  largo  plazo  (6  a  8  años)  de  más  del  80  %  en  pacientes  bien  seleccionados.  La  mayoría  de  los  
pacientes  con  enfermedad  aortoilíaca  sin  oclusiones  completas  de  segmentos  largos  se  tratan  inicialmente  con  
técnicas  endovasculares.

24.  ¿Cómo  se  determina  la  viabilidad  en  casos  de  isquemia  aguda?
Las  cinco  P  de  la  isquemia  aguda  son  dolor,  palidez,  falta  de  pulso,  parestesia  y  parálisis.  Los  primeros  
hallazgos  con  isquemia  aguda  incluyen  ausencia  de  pulso,  dolor  y  palidez.  Parestesia  y  parálisis  son  hallazgos  
posteriores.  La  enseñanza  clásica  establece  isquemia  muscular  irreversible  después  de  6  horas.
Sin  embargo,  en  la  práctica  clínica,  hay  muchas  superposiciones.  Quizás  el  hallazgo  más  sensible  que  
indica  la  inviabilidad  de  una  extremidad  es  el  rigor  muscular  en  la  pantorrilla.  La  gran  mayoría  de  las  
extremidades  isquémicas  se  pueden  tratar  con  un  tratamiento  inicial  con  heparina  seguido  de  angiografía  y  
cirugía  o  trombólisis  al  día  siguiente.

25.  ¿Cómo  se  distingue  el  trombo  del  émbolo  en  la  isquemia  aguda?
El  diagnóstico  de  oclusión  arterial  aguda  trombótica  versus  embólica  de  las  extremidades  inferiores  es  
complicado.  Los  hallazgos  sugestivos  de  embolia  incluyen  antecedentes  de  enfermedad  vascular,  circulación  
normal  de  la  pierna  contralateral,  antecedentes  de  arritmia  cardíaca  o  infarto  de  miocardio  reciente  y  trombo  
cardíaco  no  conocido.  Los  pacientes  con  embolia  a  menudo  tienen  una  isquemia  bastante  profunda  de  la  pierna  
debido  a  la  naturaleza  proximal  de  la  oclusión  (bifurcación  aórtica  o  femoral)  y  la  ausencia  de  colaterales  
desarrollados.  Ocasionalmente,  se  requiere  una  arteriografía  para  diferenciar  entre  los  dos.

26.  ¿Cuándo  está  indicada  la  trombólisis?
La  terapia  trombolítica  requiere  un  paciente  sin  contraindicaciones  (riesgo  de  sangrado)  y  un  trombo  
que  se  pueda  atravesar  con  una  guía.  El  medicamento  lítico  (uroquinasa,  estreptoquinasa  o  activador  tisular  del  
plasminógeno)  debe  colocarse  directamente  dentro  del  trombo.  Las  oclusiones  arteriales  nativas  agudas  no  
deben  tener  evidencia  de  arterias  de  salida  permeables  (p.  ej.,  un  aneurisma  de  la  arteria  poplítea  trombosado).  
Embolia  arterial  en  una  extremidad  que  no  presenta  isquemia  grave  y  puede  tolerar  el  curso  temporal  de  una  
trombólisis  exitosa  (con  frecuencia  varias  horas  de  infusión  intraarterial  y  viajes  repetidos  a  la  sala  de  angiografía  
para  angiogramas  que  ayuden  a  determinar  el  reposicionamiento  óptimo  del  catéter  para  la  lisis  completa  del  
trombo).  Los  beneficios  de  la  trombectomía  percutánea  incluyen  completar  la  trombectomía  en  menos  tiempo,  
evitar  la  lesión  por  reperfusión  tardía  y  disminuir  el  riesgo  de  sangrado  por  el  uso  de  trombolíticos.  El  uso  de  la  
terapia  trombolítica  para  las  oclusiones  del  injerto  es  más  controvertido  debido  a  la  durabilidad  a  largo  plazo  
relativamente  baja  de  estos  injertos  después  de  que  se  restablece  el  flujo.

27.  ¿Qué  es  el  síndrome  compartimental?
La  reperfusión  después  de  una  isquemia  aguda  puede  provocar  una  inflamación  profunda  del  tejido  en  la  extremidad  afectada.
El  edema  del  músculo  afectado  puede  aumentar  la  presión  dentro  de  los  compartimentos  musculares  
delimitados  por  la  fascia  (es  decir,  anterior,  lateral,  posterior  profundo  y  posterior  superficial)  a  un  nivel  que  
excede  la  presión  de  perfusión  capilar  (>30  mm  Hg).  Entonces,  la  muerte  del  músculo  es  inevitable  a  menos  
que  se  alivie  la  presión  abriendo  los  compartimentos  quirúrgicamente,  un  procedimiento  conocido  como  
fasciotomía.  Los  pacientes  se  quejan  de  dolor  intenso  e  hinchazón,  con  parestesia  asociada.  Los  pulsos  de  los  
pedales  pueden  permanecer  palpables.

28.  ¿Cuál  es  el  papel  de  la  terapia  endovascular  en  la  enfermedad  oclusiva  infrainguinal?
La  terapia  endovascular  se  usa  cada  vez  más  para  tratar  la  enfermedad  oclusiva  infrainguinal  en  pacientes  
con  claudicación  e  isquemia  crítica  de  las  extremidades.  Esto  incluye  angioplastia  y  colocación  de  stent  para
Machine Translated by Google
CAPÍTULO  69  INSUFICIENCIA  ARTERIAL  355

estenosis  y  recanalización  para  oclusiones  de  segmentos  largos.  Además,  se  han  utilizado  dispositivos  de  aterectomía  para  tratar  
estenosis  en  las  arterias  femoral,  poplítea  y  tibial.  Los  primeros  resultados  son  favorables  y  actualmente  estas  técnicas  se  utilizan  
con  la  misma  o  más  frecuencia  que  el  bypass  quirúrgico.  La  durabilidad  a  largo  plazo  es  relativamente  desconocida  en  la  actualidad.

PUNTOS  CLAVE:  INSUFICIENCIA  ARTERIAL
1.  ABI  es  la  presión  más  alta  del  tobillo  dividida  por  la  más  alta  de  las  dos  presiones  braquiales.

2.  La  isquemia  crítica  de  la  extremidad  amenaza  potencialmente  la  viabilidad  de  la  extremidad.

3.  Los  pacientes  con  insuficiencia  renal  terminal  que  tienen  isquemia  crítica  de  las  extremidades  se  encuentran  al  final  de  la  vida,
con  tasas  de  supervivencia  a  3  años  <30%.

4.  Si  un  injerto  de  vena  falla  inmediatamente  después  de  la  operación,  el  enfoque  correcto  es  explorar  la  anastomosis  distal  y  
solucionar  el  supuesto  problema  técnico.

SITIOS  WEB

www.acssurgery.com

www.vascularweb.org

BIBLIOGRAFÍA

1.  Amonkar  SJ,  Cleanthis  M,  Nice  C  et  al .:  Resultados  de  la  trombólisis  intraarterial  para  la  isquemia  aguda  de  las  extremidades.
Angiología  58(6):734­742,  2007.

2.  Gerhard­Herman  M,  Gardin  JM,  Jaff  M  et  al.:  Pautas  para  pruebas  de  laboratorio  vasculares  no  invasivas:  un  informe  de  la  Sociedad  
Estadounidense  de  Ecocardiografía  y  la  Sociedad  de  Medicina  y  Biología  Vascular.  Vasc  Med  11(3):183­200,  2006.

3.  Hiatt  WR,  Krantz  MJ:  Serie  Masterclass  en  enfermedad  arterial  periférica.  Terapia  antiplaquetaria  para  arterias  periféricas
enfermedad  y  claudicación.  Vasc  Med  11(1):55­60,  2006.

4.  Klein  WM,  van  der  GY,  Seegers  J  et  al.:  Ensayo  de  stent  ilíaco  holandés:  resultados  a  largo  plazo  en  pacientes  aleatorizados  para  
colocación  primaria  o  selectiva  de  stent.  Radiología  238(2):734­744,  2006.

5.  Landis  GS,  FariesPL:  Nuevas  técnicas  y  desarrollos  para  tratar  oclusiones  arteriales  infrainguinales  largas:  uso  de  dispositivos  
de  reentrada,  angioplastia  subintimal  y  endoprótesis.  Perspect  Vasc  Surg  Endovasc  Ther  19(3):285­290,  2007.

6.  Lau  H,  Cheng  SW,  Hui  J:  experiencia  de  dieciocho  años  con  derivación  femorofemoral.  Aust  NZJ  Surg  70:  275­278,
2000.

7.  Nehler  MR,  Hiatt  WR:  Terapia  de  ejercicio  para  la  claudicación.  Ann  Vasc  Surg  13:109­114,  1999.

8.  Novo  S,  Coppola  G,  Milio  G:  Isquemia  crítica  de  las  extremidades:  definición  e  historia  natural.  El  fármaco  actual  se  dirige  a  Cardiovasc
Haematol  Disord  4(3):219­225,  2004.

9.  Taylor  SM,  Kalbaugh  CA,  Blackhurst  DW  et  al.:  Resultados  posoperatorios  según  el  estado  médico  y  funcional  preoperatorio  después  
de  la  revascularización  infrainguinal  para  la  isquemia  crítica  de  las  extremidades  en  pacientes  de  80  años  o  más.
Am  Surg  71(8):640­645,  2005.

10.  Wind  J,  Koelemay  MJ:  Terapia  de  ejercicio  y  el  efecto  adicional  de  la  supervisión  sobre  la  terapia  de  ejercicio  en
pacientes  con  claudicación  intermitente.  Revisión  sistemática  de  ensayos  controlados  aleatorios.  Eur  J  Vasc  Endovasc  Surg  34(1):1­9,  
2007.
Machine Translated by Google

ENFERMEDAD  CARÓTIDA
CAPITULO  
70

Jennifer  M.  Worth,  MD  y  Bernard  Timothy  Baxter,  MD

1.  ¿Qué  enfermedades  primarias  afectan  a  las  arterias  carótidas?
La  aterosclerosis  es,  con  mucho,  la  enfermedad  más  común  que  afecta  a  las  arterias  carótidas  y  representa  
el  90%  de  las  lesiones  en  el  mundo  occidental.  La  arteria  carótida  también  puede  verse  afectada  por  
torceduras  secundarias  a  elongación  arterial,  displasia  fibromuscular,  compresión  extrínseca  (p.  ej.,  
neoplasia),  cambios  inducidos  por  radiación,  trauma  (que  causa  sangrado,  oclusión  o  disección)  y  
arteriopatías  inflamatorias  (p.  ej.,  arteritis  temporal,  enfermedad  de  Takayasu).  arteritis).

2.  ¿Cuáles  son  las  características  histológicas  de  las  placas  ateroscleróticas?
Es  probable  que  las  placas  se  formen  de  acuerdo  con  la  hipótesis  de  la  respuesta  a  la  lesión.  Las  placas  
comienzan  en  la  íntima  y  la  media  a  través  de  una  serie  compleja  de  eventos.  Inicialmente,  se  reclutan  células  
de  músculo  liso  (SMC)  y  se  producen  proteínas  de  tejido  conectivo  en  exceso.  La  incorporación  de  colesterol  
de  lipoproteínas  de  baja  densidad  (LDL),  monocitos  y  plaquetas  conduce  a  la  formación  de  la  placa  madura  
que  consta  de  un  núcleo  lipídico  y  una  capa  fibrótica  que  cubre  el  núcleo.  La  hemorragia  en  la  placa  (hemorragia  
intraplaca)  puede  causar  un  crecimiento  repentino  que  aumente  de  forma  aguda  el  grado  de  estenosis  o  
provoque  una  oclusión.

3.  ¿Cuáles  son  las  secuelas  clínicas  de  la  enfermedad  aterosclerótica?
La  trombosis  y  la  embolización  son  las  complicaciones  más  comunes  de  las  placas  ateroscleróticas.  
La  trombosis  y  la  embolización  ocurren  típicamente  cuando  la  capa  fibrosa  externa  de  la  placa  es  degradada  
por  las  enzimas  de  las  células  inflamatorias  que  exponen  el  núcleo  lipídico.  Este  núcleo  es  altamente  
trombogénico  y  friable  predisponiendo  a  la  trombosis  y  embolización  de  lípidos  y  agregados  plaquetarios.  La  
aterosclerosis  también  puede  ser  un  factor  en  el  desarrollo  de  aneurismas  de  la  arteria  carótida,  aunque  esto  
es  controvertido  porque  pueden  ocurrir  sin  aterosclerosis.

4.  ¿Cuáles  son  los  síntomas  más  comunes  de  la  enfermedad  de  la  arteria  carótida?
&  Ataque  isquémico  transitorio  (AIT)
&  Accidente  cerebrovascular  (ACV)
&  tímido  de  amor

5.  Definir  TIA,  CVA  y  amaurosis  fugax.
Estos  términos  clínicos  describen  un  espectro  de  síndromes  isquémicos  cerebrales.  Un  TIA  es  un  déficit  
neurológico  que  dura  menos  de  24  horas,  pero  por  lo  general  dura  solo  unos  minutos.  Desde  el  punto  de  vista  
clínico,  un  CVA,  o  accidente  cerebrovascular  agudo,  se  define  por  un  déficit  neurológico  persistente  que  dura  
>24  horas.  La  amaurosis  fugax  es  un  episodio  de  ceguera  monocular  transitoria  (de  minutos  a  horas),  a  menudo  
comparada  con  una  persiana  que  se  coloca  sobre  el  ojo.  Se  produce  por  una  disminución  aguda  del  flujo  
sanguíneo  secundaria  a  la  embolización  a  través  de  la  arteria  oftálmica  hacia  la  retina.  Cuando  esto  ocurre,  el  
flujo  de  sangre  a  la  periferia  de  la  retina  se  pierde  primero  con  la  isquemia  (pérdida  de  visión)  avanzando  hacia  
el  centro  de  la  retina,  de  ahí  la  sensación  de  que  la  sombra  está  bajando.

6.  ¿Qué  son  las  placas  de  Hollenhorst?
Hollenhorst  fue  un  oftalmólogo  que  describió  por  primera  vez  las  placas  refringentes  de  colesterol  en  
los  vasos  de  la  retina.  Suelen  verse  en  los  puntos  de  bifurcación  del  vaso  y  son  el  resultado  de

356
Machine Translated by Google
CAPÍTULO  70  ENFERMEDAD  CARÓTIDA  357

embolismo  arterial  a  arterial.  La  fuente  más  común  es  una  placa  de  bifurcación  carotídea,  pero  pueden  originarse  por  
enfermedad  en  la  carótida  común  o  en  la  aorta  ascendente.

7.  ¿Qué  mecanismos  producen  déficits  neurológicos?
  Embolización  de  arterias  ateroscleróticas  proximales  (aorta  ascendente,  cayado  aórtico,  arterias  carótidas),  el  corazón  
o,  en  el  caso  de  un  cortocircuito  cardíaco  de  derecha  a  izquierda,  el  sistema  venoso  (émbolos  paradójicos)

  Disminución  del  flujo  sanguíneo  cerebral  por  shock     

Enfermedad  aterosclerótica  oclusiva  que  progresa  a  trombosis     Hemorragia  
intracraneal

8.  ¿Cuál  es  la  historia  natural  de  un  AIT?
La  historia  natural  de  un  AIT  está  definida  por  la  patología  de  la  arteria  carótida  ipsilateral.
En  pacientes  con  estenosis  grave  (>  70  %),  el  riesgo  de  accidente  cerebrovascular  ipsolateral  dentro  de  los  24  meses  es  del  26  %.
Para  aquellos  con  enfermedad  moderada  (50%  a  69%),  el  riesgo  es  del  22%  a  los  5  años.  Con  estenosis  mínima  (<30  
%),  el  riesgo  es  del  1  %  a  los  3  años  (consulte  Lecturas  obligatorias  en  el  Capítulo  1).

9.  ¿Cuál  es  el  efecto  de  la  medicación  sobre  los  AIT  y  los  accidentes  cerebrovasculares?

El  ácido  acetilsalicílico  (aspirina)  es  un  inhibidor  de  la  ciclooxigenasa  que  disminuye  la  pegajosidad  y  la  inflamación  de  
las  plaquetas.  Disminuye  la  incidencia  de  ictus  en  las  mujeres.  Aunque  no  se  ha  demostrado  que  reduzca  la  incidencia  de  
accidentes  cerebrovasculares  como  criterio  de  valoración  único  en  los  hombres,  se  ha  demostrado  que  reduce  la  incidencia  
de  infarto  de  miocardio  (IM)  y  accidente  cerebrovascular  cuando  se  combinan  estos  criterios  de  valoración.  Se  pueden  
agregar  otros  agentes  antiplaquetarios  (clopidogrel,  dipiridamol)  cuando  los  pacientes  presentan  síntomas  con  aspirina,  pero  
no  hay  datos  prospectivos  que  demuestren  que  alguna  combinación  sea  superior  a  la  aspirina  sola.

La  terapia  con  estatinas  está  aprobada  y  recomendada  para  la  prevención  de  accidentes  cerebrovasculares  en  pacientes  con  AIT,

accidente  cerebrovascular  o  estenosis  carotídea  significativa.

10.  ¿Qué  significa  un  soplo  carotídeo?
Un  soplo  carotídeo  es  un  marcador  general  de  aterosclerosis  que  carece  de  especificidad;  indica  un  mayor  riesgo  de  
eventos  vasculares  cardíacos  y  cerebrales.  La  ausencia  de  un  soplo  no  indica  ausencia  de  enfermedad.

11.  ¿El  sonido  de  un  soplo  se  correlaciona  con  el  grado  de  estenosis?
No.  A  medida  que  avanza  la  estenosis,  el  soplo  puede  disminuir  o  desaparecer  a  medida  que  disminuye  el  
flujo.

12.  ¿Qué  prueba  preliminar  debe  ordenarse  para  evaluar  un  soplo  cervical  o  una  estenosis  carotídea  con  base  en  hallazgos  
clínicos  como  TIA  o  CVA?
La  ecografía  dúplex  (US)  es  precisa  y  económica.  Para  confirmar  la  enfermedad  carotídea  o  buscar  otros  sitios  de  la  
enfermedad  cuando  el  dúplex  carotídeo  no  se  correlaciona  con  los  síntomas,  ahora  se  utilizan  la  angiografía  por  resonancia  
magnética  o  la  angiografía  por  tomografía  computarizada  (CTA)  para  evaluar  los  vasos  del  arco  y  la  circulación  intracraneal.  
Debido  a  que  la  angiografía  cerebral  es  invasiva  y  se  asocia  con  un  riesgo  adicional,  incluido  un  pequeño  riesgo  de  accidente  
cerebrovascular,  se  reserva  para  casos  en  los  que  se  requiere  información  anatómica  adicional  específica.

13.  ¿Cuándo  está  indicada  la  intervención  para  la  enfermedad  de  la  arteria  carótida  sintomática?
La  intervención  está  fuertemente  indicada  para  la  enfermedad  de  la  arteria  carótida  sintomática  asociada  con  estenosis  
>70%.  La  reducción  absoluta  del  riesgo  de  accidente  cerebrovascular  es  del  17%  a  los  2  años.  Hay  un  beneficio  menor  en  
pacientes  con  estenosis  sintomática  del  50%  al  69%  (reducción  del  riesgo  del  6,5%  a  los  5  años).
Los  pacientes  sintomáticos  con  estenosis  <50%  no  se  benefician  de  la  intervención.
Machine Translated by Google
358  CAPÍTULO  70  ENFERMEDAD  CARÓTIDA

PUNTOS  CLAVE:  ENFERMEDAD  CARÓTIDA

1.  Los  síntomas  de  la  enfermedad  carotídea  incluyen  TIA,  CVA  y  amaurosis  fugax.

2.  Un  soplo  carotídeo  es  un  marcador  general  de  aterosclerosis  que  incluye  tanto  la  arteria  coronaria  como
enfermedad  cerebrovascular.

3.  La  intervención  está  fuertemente  indicada  para  la  enfermedad  de  la  arteria  carótida  sintomática  asociada
con  >70%  de  estenosis.

14.  ¿Se  debe  operar  a  un  paciente  con  estenosis  asintomática?
La  reducción  absoluta  del  riesgo  de  accidente  cerebrovascular  es  del  6  %  durante  un  período  de  5  años  en  pacientes  
asintomáticos  con  estenosis  >60  %  que  se  someten  a  endarterectomía  carotídea  (CEA)  más  aspirina  versus  pacientes  
tratados  con  aspirina  sola  (5,1  %  versus  11  %).  Por  lo  tanto,  la  CEA  se  debe  realizar  para  la  enfermedad  carotídea  
asintomática  cuando  se  espera  que  el  paciente  viva  al  menos  3  años  y  cuando  la  CEA  se  puede  realizar  con  una  tasa  
combinada  de  accidente  cerebrovascular  y  mortalidad  <3%.

15.  ¿Cuáles  son  las  complicaciones  de  la  endarterectomía  carotídea?
AIT  o  accidente  cerebrovascular  (aproximadamente  2%)
Hematoma  

Lesión  de  pares  
craneales  Hipertensión  
Hipotensión

16.  ¿Qué  nervios  craneales  pueden  lesionarse  durante  la  CEA?  ¿Cuáles  son  los  signos  clínicos?
de  lesión?
La  rama  mandibular  marginal  del  nervio  facial  (nervio  craneal  [NC]  VII):  la  lesión  puede
causar  caída  de  la  comisura  ipsilateral  de  la  boca
Nervio  glosofaríngeo  (CN  IX):  dificultad  para  tragar  tanto  sólidos  como  líquidos
Rama  laríngea  recurrente  del  nervio  vago  (NC  X):  ronquera,  pérdida  de  tos  efectiva
Nervio  laríngeo  superior  (rama  del  vago,  CN  X):  fatiga  de  la  voz,  pérdida  del  tono  alto
fonación
Nervio  hipogloso  (NC  XII):  desviación  de  la  lengua  hacia  el  lado  ipsilateral,  dificultad  para  hablar  y  masticar

17.  ¿Cuál  es  el  peligro  de  hematoma  en  la  herida  después  de  la  cirugía?
El  principal  peligro  es  el  compromiso  de  las  vías  respiratorias,  lo  que  puede  requerir  una  descompresión  urgente  
mediante  la  apertura  de  la  herida.  No  está  claro  si  los  drenajes  colocados  en  el  lecho  de  la  herida  previenen  la  formación  
de  hematomas.

18.  ¿Cuándo  ocurren  los  eventos  neurológicos  durante  la  EAC?
  Disección:  desprendimiento  de  material  de  la  pared  arterial  con  embolización     Pinza:  infarto  
isquémico     Postoperatorio :  colgajo  de  íntima,  reperfusión

19.  ¿Qué  es  una  derivación?  ¿Cuándo  se  usa?

Una  derivación  es  un  pequeño  tubo  de  plástico  que  desvía  el  flujo  de  sangre  alrededor  de  la  arteria  carótida  abierta  
quirúrgicamente  mientras  se  realiza  la  endarterectomía.  Se  utiliza  una  derivación  para  garantizar  un  flujo  sanguíneo  
cerebral  adecuado  y  evitar  la  isquemia  cerebral  intraoperatoria.  Muchos  cirujanos  usan  derivaciones  de  forma  rutinaria,  
pero  otros  las  usan  de  manera  selectiva,  si  es  que  las  usan.  La  decisión  de  utilizar  una  derivación  se  basa  en  la  ausencia  
conocida  de  circulación  colateral  o  en  la  evaluación  intraoperatoria,  incluido  el  pinzamiento  temporal  de  la
Machine Translated by Google
CAPÍTULO  70  ENFERMEDAD  CARÓTIDA  359

carotídea  bajo  anestesia  local,  medición  de  la  presión  del  muñón,  electroencefalografía  
intraoperatoria  o  Doppler  transcraneal.  Ninguno  de  estos  métodos  es  100%  exacto.

20.  ¿Qué  es  la  presión  del  muñón?
La  presión  del  muñón  es  la  contrapresión  de  la  arteria  carótida  interna  después  del  pinzamiento.  Se  utiliza  
para  evaluar  la  adecuación  de  la  perfusión  cerebral.  La  presión  "segura"  varía  de  un  médico  a  otro,  pero  la  
presión  media  debe  ser  de  al  menos  40  mm  Hg.

21.  ¿Recurre  la  estenosis  después  de  la  endarterectomía  carotídea?
Sí.  La  incidencia  informada  ha  sido  bastante  variable  y  oscila  entre  <2%  y  hasta  un  36%.
Durante  los  primeros  24  meses  después  de  la  operación,  se  cree  que  la  reestenosis  es  secundaria  a  hiperplasia  
miointimal.  Más  allá  de  este  tiempo,  es  causado  por  la  progresión  de  la  enfermedad  (aterosclerosis).  La  
incidencia  es  menor  cuando  la  arteriotomía  se  cierra  con  una  angioplastia  con  parche  venoso.

22.  ¿Cuál  es  la  complicación  más  común  asociada  con  la  endarterectomía  de  reoperación?

Lesión  NC  (incidencia  reportada  2%  a  20%).  Sin  embargo,  la  mayoría  de  las  lesiones  son  transitorias.

23.  ¿En  qué  capa  de  la  arteria  se  realiza  la  endarterectomía  carotídea?
Las  capas  externas  de  la  túnica  media.

24.  ¿Qué  referencia  anatómica  es  útil  para  identificar  el  nivel  de  la  arteria  carótida?
¿bifurcación?
La  vena  facial  común.

25.  ¿Cuántas  ramas  de  la  arteria  carótida  interna  se  encuentran  en  el  cuello?
Ninguno.

26.  Cuando  se  ocluye  la  arteria  carótida  interna,  ¿qué  ramas  de  la  externa
¿La  arteria  carótida  forma  colaterales  y  restablece  la  circulación  en  el  polígono  de  Willis?
Las  ramas  periorbitarias  de  la  arteria  carótida  externa  forman  comunicaciones  con  la  arteria  oftálmica,  una  
rama  de  la  carótida  interna.

27.  ¿Cuáles  son  las  funciones  del  seno  carotídeo  y  del  cuerpo  carotídeo?
Ambos  se  localizan  en  la  bifurcación  carotídea  y  están  inervados  por  los  nervios  glosofaríngeo  y  vago,  
respectivamente.  La  función  del  seno  carotídeo  es  la  regulación  de  la  presión  arterial.
La  hipertensión  estimula  los  impulsos  eferentes  al  centro  vasomotor  en  el  bulbo  raquídeo,  lo  que  inhibe  el  
tono  simpático  y  aumenta  el  tono  vagal.  El  cuerpo  carotídeo  regula  el  impulso  respiratorio  y  el  estado  ácido­
base  a  través  de  quimiorreceptores.  También  induce  bradicardia  cuando  se  manipula  (este  es  su  objetivo  
durante  el  masaje  carotídeo  para  las  arritmias  cardíacas).

28.  ¿Cuándo  fue  el  primer  procedimiento  quirúrgico  exitoso  de  la  carótida  extracraneal?
arteria  realizada?  ¿A  quién  se  le  atribuye?
En  1954  por  Eastcott.

CONTROVERSIA

29.  ¿Cuál  es  el  papel  de  la  colocación  de  un  stent  en  la  arteria  carótida?
Aunque  la  CEA  sigue  siendo  el  estándar  de  atención  para  la  enfermedad  de  la  arteria  carótida,  la  angioplastia  percutánea  
con  colocación  de  stent  (CAS)  se  ha  investigado  como  una  alternativa.  El  fundamento  subyacente  es  disminuir  la  
morbilidad,  los  costos  hospitalarios  y  los  riesgos  anestésicos  además  de  mejorar  la  permeabilidad  a  largo  plazo.  
Actualmente,  los  pacientes  sintomáticos  de  «alto  riesgo»  son  considerados  para  la  colocación  de  stents  carotídeos.
Machine Translated by Google
360  CAPÍTULO  70  ENFERMEDAD  CARÓTIDA

Se  han  publicado  múltiples  informes  en  los  últimos  10  años  sobre  el  uso  de  stents  en  la  enfermedad  de  la  
arteria  carótida.  Estos  estudios  han  establecido  claramente  que  se  requiere  algún  tipo  de  dispositivo  de  
protección  cerebral  para  reducir  el  riesgo  de  embolización.  Los  dispositivos  más  utilizados  son  filtros  colocados  
distalmente  a  la  estenosis  para  capturar  los  desechos  liberados  durante  la  angioplastia  y  la  colocación  de  stents.
Ensayos  aleatorizados  recientes  sugieren  que  CAS  no  es  equivalente  a  CEA,  pero  los  estudios  se  han  visto  
limitados  por  poblaciones  de  pacientes  heterogéneas  (sintomáticas  y  asintomáticas).  Los  defensores  de  CAS  
citan  problemas  con  estos  estudios  que  pueden  haber  favorecido  la  cirugía  y  señalan  las  continuas  mejoras  
en  la  tecnología.  Hay  una  serie  de  ensayos  aleatorios  importantes  que  reclutan  pacientes  en  este  momento,  
incluido  el  ensayo  CREST  (endarterectomía  de  revascularización  carotídea  versus  stent)  patrocinado  por  los  
Institutos  Nacionales  de  Salud  (NIH)  que  asignará  al  azar  a  1400  pacientes  sintomáticos  y  1100  pacientes  
asintomáticos.

SITIO  WEB

www.acssurgery.com

BIBLIOGRAFÍA

1.  Barnett  HJM,  Taylor  DW,  Eliasziw  M  et  al.:  Beneficio  de  la  endarterectomía  carotídea  en  pacientes  con
estenosis  moderada  o  severa.  N  Engl  J  Med  339:1415­1425,  1998.

2.  Comité  Ejecutivo  del  Estudio  de  Aterosclerosis  Carótida  Asintomática:  endarterectomía  para  la  estenosis  de  la  arteria  carótida  
asintomática.  JAMA  273:1421­1429,  1995.

3.  Halliday  A,  Mansfield  A,  Marro  J  et  al .:  Prevención  de  accidentes  cerebrovasculares  incapacitantes  y  fatales  mediante  
endarterectomía  carotídea  exitosa  en  pacientes  sin  síntomas  neurológicos  recientes:  ensayo  controlado  aleatorio.  Grupo  
colaborativo  del  ensayo  de  cirugía  carotídea  asintomática  (ACST)  del  MRC.  Lancet  363:1491,  2004.

4.  Illig  KA,  Narins  CR:  Selección  de  pacientes  para  colocación  de  stent  carotídeo  versus  endarterectomía:  una  revisión  sistemática.  J  
Vasc  Surg  44:661­672,  2006.

5.  Luebke  T,  Aleksic  M,  Brunkwall  J:  Metanálisis  de  ensayos  aleatorios  que  comparan  la  endarterectomía  carotídea  y  el  tratamiento  
endovascular.  Eur  J  Vasc  Endovasc  Surg  34:470­479,  2007.

6.  Mas  JL,  Chantellier  G,  Beyssen  B  et  al.:  Endarterectomía  versus  colocación  de  stent  en  pacientes  con  estenosis  carotídea  severa  
sintomática.  N  Engl  J  Med  355:1660­1671,  2006.

7.  Moore  WS,  Kempszinski  RF,  Nelson  JJ  et  al .:  Estenosis  carotídea  recurrente:  resultados  del  estudio  de  aterosclerosis  carotídea  
asintomática.  Carrera  29:2018­2025,  1998.

8.  Colaboradores  del  ensayo  de  endarterectomía  carotídea  sintomática  de  América  del  Norte:  efecto  beneficioso  de  la  
endarterectomía  carotídea  en  pacientes  sintomáticos  con  estenosis  de  alto  grado.  N  Engl  J  Med  325:445­453,  1991.

9.  Paraskevas  KI,  Hamilton  G,  Mikhailidis  DP:  Estatinas:  un  componente  esencial  en  el  manejo  de  la  arteria  carótida
enfermedad.  J  Vasc  Surg  46:373­386,  2007.

10.  Grupo  colaborativo  SPACE:  resultados  a  30  días  del  ensayo  SPACE  de  angioplastia  protegida  con  stent  versus  endarterectomía  
carotídea  en  pacientes  sintomáticos:  un  ensayo  aleatorizado  de  no  inferioridad.  Lancet  368:1239­1247,  2006.
Machine Translated by Google

ANEURISMA  AÓRTICO  ABDOMINAL
CAPITULO  
71

Carlos  A.  Rueda,  MD  y  Mark  R.  Nehler,  MD

1.  ¿Qué  es  un  aneurisma  aórtico  abdominal  (AAA)?
Aorta  Aumento  del  50%  del  diámetro  aórtico  normal.  El  diámetro  normal  de  la  aorta  infrarrenal  es  de  2,0  cm  para  los  

hombres.  Es  apropiada  una  definición  de  AAA  como  una  aorta  =  3,0  cm  de  diámetro.

2.  ¿Cuál  es  la  incidencia  de  AAA?

&  Tres  por  ciento  en  pacientes  adultos  no  seleccionados  examinados  con  ultrasonido  (EE.  UU.)
&  Cinco  por  ciento  en  pacientes  con  enfermedad  arterial  coronaria  conocida  (CAD)
&  Diez  por  ciento  en  pacientes  con  enfermedad  vascular  periférica  conocida

3.  ¿Cuál  es  la  etiología  del  AAA?
La  elastina  es  el  principal  elemento  de  carga  de  la  aorta.  En  la  aorta  humana  normal,  hay  una  reducción  gradual  en  la  
cantidad  de  elastina  presente  en  la  aorta  distal  en  comparación  con  la  proximal.  Histológicamente  se  observa  
fragmentación  y  degeneración  de  elastina  en  las  paredes  del  AAA.  Estas  observaciones  ayudan  a  explicar  la  predilección  
de  los  AAA  en  la  aorta  infrarrenal.  La  ausencia  de  vasa  vasorum  en  la  aorta  infrarrenal  ha  llevado  a  sugerir  una  
deficiencia  nutritiva.  La  degradación  de  la  media  aórtica  en  la  enfermedad  aneurismática  implica  una  alteración  del  
equilibrio  entre  las  enzimas  proteolíticas  y  sus  inhibidores.

4.  ¿Los  AAA  tienen  un  componente  genético?
Múltiples  informes  describen  un  subgrupo  familiar  de  AAA.  Por  lo  tanto,  la  evaluación  de  los  familiares  de  primer  
grado  de  los  pacientes  con  AAA  que  tienen  50  años  o  más  tiene  sentido.  Dos  estudios  prospectivos  demostraron  que  
aproximadamente  el  30%  de  estos  familiares  también  albergan  un  AAA.  El  defecto  genético  propuesto  se  ha  
relacionado  con  el  colágeno  tipo  III  anormal.

5.  ¿Los  pacientes  con  AAA  son  propensos  a  aneurismas  en  otros  lechos  vasculares?
Sí.  El  cuarenta  por  ciento  de  los  pacientes  con  un  aneurisma  de  la  arteria  poplítea  albergan  un  AAA.  El  setenta  y  
cinco  por  ciento  de  los  pacientes  con  un  aneurisma  de  la  arteria  femoral  también  tienen  un  AAA.  Los  pacientes  con  
aneurismas  torácicos  tienen  un  20%  de  posibilidades  de  tener  un  AAA  simultáneo.  El  cinco  por  ciento  de  los  pacientes  
desarrollan  aneurismas  aórticos  proximales  a  su  injerto  5  años  después  de  la  reparación  del  AAA  infrarrenal.

6.  ¿Pueden  detectarse  los  AAA  de  forma  fiable  en  el  examen  físico?
No.  La  bifurcación  aórtica  está  a  la  altura  del  ombligo.  Por  tanto,  la  masa  pulsátil  de  un  AAA  se  localiza  en  el  epigastrio.  
Por  lo  tanto,  solo  pueden  detectarse  AAA  relativamente  grandes  en  pacientes  delgados.

7.  ¿Se  pueden  detectar  los  AAA  por  radiografía?
Las  radiografías  simples  de  abdomen  o  columna  lumbar  pueden  detectar  AAA  oculto  en  alrededor  del  20%  de  los  casos.
Un  borde  delgado  de  calcificación  identifica  la  pared  aórtica  aneurismática.  La  mayoría  de  los  AAA  contienen  calcio  
insuficiente  para  ser  visualizados  por  radiografía.

8.  ¿Qué  método  de  imagen  es  el  mejor  para  detectar  AAA  en  los  pacientes?
La  ecografía  abdominal  permite  una  precisión  de  medición  de  0,3  cm  y  datos  en  dimensiones  transversales  y  
longitudinales.

361
Machine Translated by Google
362  CAPÍTULO  71  ANEURISMA  AÓRTICO  ABDOMINAL

9.  ¿Cuál  es  la  mejor  modalidad  de  imagen  única  para  planificar  la  reparación  del  AAA?
La  tomografía  computarizada  (TC)  con  contraste  es  la  mejor.  Las  medidas  de  diámetro  tienen  una  
precisión  de  0,2  cm.  Las  anomalías  venosas  (es  decir,  vena  renal  izquierda  retroaórtica  o  circumaórtica,  duplicación  
de  la  vena  cava  inferior  [IVC]  y  IVC  del  lado  izquierdo)  que  alteran  drásticamente  el  abordaje  quirúrgico  se  
visualizan  bien  en  la  TC.  Aunque  la  TC  es  excelente  para  detectar  ruptura  o  fuga  aneurismática  (92  %  de  precisión  
y  100  %  de  especificidad),  es  menos  útil  para  predecir  el  compromiso  de  un  aneurisma  suprarrenal  (sensibilidad,  83  
%;  especificidad,  90  %;  valor  predictivo  positivo,  48  %).

10.  ¿Cuál  es  la  manifestación  de  un  AAA  sintomático?
El  dolor  lumbar  agudo  es  el  síntoma  de  presentación  más  común  (82%),  pero  solo  un  tercio  de  los  AAA  se  
diagnostican  antes  de  la  ruptura.  Un  anciano  hipotenso  con  inicio  agudo  de  dolor  lumbar  tiene  una  fuga  de  AAA  
hasta  que  se  demuestre  lo  contrario.

11.  ¿Cuál  es  el  manejo  adecuado  de  un  paciente  con  sospecha  de  rotura  de  AAA?
Justo  antes  de  la  exploración  quirúrgica  urgente,  los  pacientes  hemodinámicamente  inestables  con  una  masa  
abdominal  pulsátil  deben  someterse  a  un  electrocardiograma  (ECG)  para  descartar  un  infarto  de  miocardio  (IM).

12.  ¿Todos  los  pacientes  que  presentan  rotura  de  AAA  deben  someterse  a  reparación?
Los  pacientes  en  shock  profundo  o  paro  cardíaco  en  el  momento  de  la  presentación  tienen  pocas  posibilidades  
de  supervivencia.  La  edad  extrema,  la  demencia,  el  cáncer  metastásico  y  otros  problemas  médicos  graves  en  
etapa  terminal  deberían  obligarlo  a  reevaluar  esta  asignación  de  recursos  médicos.

13.  ¿Todos  los  pacientes  con  AAA  rotos  llegan  a  la  cirugía?
Aproximadamente  la  mitad  de  los  pacientes  con  un  AAA  roto  mueren  antes  de  llegar  al  hospital.  Una  cuarta  parte  
de  los  que  llegan  al  hospital  mueren  antes  de  que  puedan  ser  llevados  a  la  sala  de  operaciones  (OR).  Por  lo  tanto,  
solo  el  25%  de  los  pacientes  llegan  a  la  cirugía.

14.  ¿Cómo  se  trata  quirúrgicamente  un  AAA  roto?
No  se  debe  anestesiar  al  paciente  hasta  que  esté  completamente  preparado  y  cubierto  y  listo  para  la  
incisión  inmediata  porque  la  presión  arterial  puede  disminuir  drásticamente  durante  la  inducción  de  la  anestesia.  El  
control  rápido  de  la  aorta  proximal  es  la  clave  para  el  éxito  de  las  operaciones  de  rotura  de  AAA.  Puede  ser  en  el  
diafragma  (en  un  paciente  inestable,  con  sangrado  intraperitoneal  libre  o  un  hematoma  retroperitoneal  que  se  
extiende  proximal  a  la  vena  renal  izquierda)  o  en  el  segmento  aórtico  infrarrenal  (en  un  paciente  estable  con  un  
hematoma  retroperitoneal  inferior).
La  oclusión  con  balón  intraluminal  de  la  aorta  es  una  opción  con  rotura  intraperitoneal  libre.  Tan  pronto  como  se  
obtiene  el  control,  se  resucita  al  paciente  y  se  mueven  las  pinzas  a  la  ubicación  infrarrenal  más  estándar.  El  control  
distal  también  se  puede  obtener  con  balones  o  compresas  para  prevenir  lesiones  en  las  venas  ilíacas.

15.  ¿Cómo  deben  manejarse  los  pacientes  con  AAA  sintomáticos  no  rotos?
Los  AAA  sintomáticos  se  expanden  rápidamente  y  tienen  un  alto  riesgo  de  ruptura.  Por  lo  tanto,  la  mayoría  
de  los  cirujanos  vasculares  están  de  acuerdo  en  que  los  AAA  sintomáticos  pero  intactos  deben  repararse  
rápidamente  (tan  pronto  como  sea  posible).

16.  ¿Existen  alternativas  a  la  reparación  quirúrgica  abierta  del  AAA  roto?
Los  injertos  protésicos  endovasculares  se  han  colocado  con  éxito  en  pacientes  de  alto  riesgo  con  AAA  
sintomáticos  o  roturas  contenidas  tanto  en  la  posición  aórtica  como  aortoilíaca.

17.  ¿Cuáles  son  las  tasas  de  ruptura  de  los  AAA?
Un  AAA  de  5  cm  de  diámetro  tiene  un  riesgo  de  rotura  anual  <1%.  El  riesgo  de  ruptura  del  AAA  aumenta  con  el  
tamaño.  El  riesgo  de  rotura  anual  es  del  10  %  para  un  AAA  de  6  cm  y  del  30  %  para  AAA  >  7  cm.
Machine Translated by Google
CAPÍTULO  71  ANEURISMA  AÓRTICO  ABDOMINAL  363

18.  ¿A  qué  velocidad  se  agrandan  los  AAA?
La  tasa  de  expansión  promedio  de  todos  los  AAA  es  de  0,4  cm/año.  Sin  embargo,  el  20%  de  todos  los  AAA  no  muestran  
cambios  de  tamaño  con  el  tiempo.  Por  el  contrario,  el  20%  se  expande  a  un  ritmo  >0,5  cm/año.  La  expansión  rápida  
(0,5  cm/6  meses)  se  considera  predictiva  de  rotura  e  indicación  de  reparación.

19.  ¿Cuándo  son  útiles  los  angiogramas  en  el  diagnóstico  de  AAA?
Tradicionalmente,  la  angiografía  se  ha  indicado  en  pacientes  cuando  existe  preocupación  sobre  la  extensión  del  cuello  
proximal,  enfermedad  oclusiva  visceral  concomitante,  anomalías  de  la  arteria  renal,  una  colectomía  previa  con  
necesidad  de  visualizar  la  circulación  visceral  o  enfermedad  oclusiva  o  aneurismática  de  las  extremidades  inferiores.  
Más  recientemente,  los  angiogramas  por  TC  de  corte  fino  (3  mm  o  menos)  se  utilizan  para  planificar  la  reparación  
endovascular  de  un  AAA  (EVAR).  La  angiografía  estándar  rara  vez  se  usa  antes  de  la  operación,  ya  que  ahora  es  parte  
de  la  terapia  endovascular.

20.  ¿Cuál  es  la  diferencia  entre  el  abordaje  extraperitoneal  y  el  transabdominal?
La  colocación  electiva  del  injerto  aórtico  se  puede  realizar  igualmente  bien  a  través  de  un  abordaje  
transperitoneal  o  extraperitoneal.  El  primero  proporciona  una  mejor  exposición  pélvica.  El  abordaje  extraperitoneal  
brinda  una  exposición  superior  de  la  aorta  suprarrenal  y  facilita  el  manejo  pulmonar  posoperatorio.

21.  ¿Qué  son  las  endoprótesis?  ¿Son  duraderos?
Se  estima  que  del  50  al  60%  de  los  AAA  se  reparan  con  un  injerto  endovascular.  Los  injertos  endovasculares  son  stents  
cubiertos  de  injerto  que  se  colocan  a  través  de  la  arteria  femoral  mediante  métodos  intervencionistas  (es  decir,  
radiográficos)  para  excluir  el  aneurisma  sin  necesidad  de  una  incisión  abdominal  o  pinzamiento  cruzado  de  la  aorta.  Se  
han  informado  múltiples  series  diferentes  de  reparación  endovascular  exitosa  de  AAA.  Se  ha  informado  de  la  colocación  
exitosa  de  endoprótesis  en  una  amplia  variedad  de  candidatos  quirúrgicos  de  alto  riesgo.  Muchos  cirujanos  vasculares  
e  intervencionistas  están  haciendo  de  la  colocación  de  una  endoprótesis  aórtica  su  tratamiento  preferido  para  los  
pacientes  con  AAA.  Los  principales  inconvenientes  son  las  fugas  tardías  o  la  ruptura  del  injerto,  el  costo  del  procedimiento  
y  la  necesidad  de  un  seguimiento  del  paciente  a  largo  plazo.

22.  ¿Cuáles  son  las  ventajas  y  desventajas  de  una  reparación  endovascular  de  un
AAA  (EVAR)?
Estudios  recientes  sugieren  que  EVAR  tiene  una  mortalidad  operatoria  reducida  a  corto  plazo  sobre  la  reparación  
abierta  de  AAA  en  pacientes  anatómicamente  adecuados  y  relativamente  sanos.  Sin  embargo,  esta  reducción  
de  la  mortalidad  operatoria  desapareció  en  los  pacientes  considerados  no  aptos  para  cirugía  mayor.  Además,  la  EVAR  
requiere  vigilancia  continua  y  un  seguimiento  más  prolongado  que  la  reparación  abierta  de  AAA.  Algunas  evidencias  
sugieren  que  el  EVAR  es  más  costoso  y  conduce  a  un  mayor  número  de  complicaciones  y  reintervenciones.  La  
reparación  abierta  de  AAA  puede  ser  preferible  para  pacientes  más  jóvenes  y  saludables  y  para  quienes  la  durabilidad  
a  largo  plazo  y  el  seguimiento  son  motivo  de  preocupación.  Por  el  contrario,  los  pacientes  mayores  y  más  enfermos  con  
mayores  riesgos  quirúrgicos  y  una  esperanza  de  vida  más  corta  pueden  ser  mejor  tratados  con  EVAR.

23.  ¿Cuáles  son  las  complicaciones  de  la  EVAR?  ¿Cómo  son  tratados?
Puede  haber  perfusión  arterial  continua  del  saco  del  aneurisma  después  de  la  REVA.  Esto  se  denomina  endofuga  
y  ocurre  en  15%  a  20%  de  EVAR.  La  endofuga  de  tipo  I  es  el  resultado  del  flujo  anterógrado  en  el  sitio  de  inserción  de  
la  endoprótesis  cubierta.  Estos  se  tratan  con  injertos  de  extensión  proximal  o  distal  en  el  momento  del  descubrimiento.  
La  endofuga  de  tipo  II  resulta  del  flujo  retrógrado  en  una  rama  lateral  colateral  del  aneurisma,  como  las  arterias  lumbares  
o  la  arteria  mesentérica  inferior.  Debido  a  que  la  mayoría  de  estas  endofugas  son  autolimitadas,  la  observación  es  
adecuada  cuando  el  tamaño  del  AAA  no  aumenta.  Sin  embargo,  las  endofugas  de  tipo  II  se  pueden  tratar  con  
embolización  percutánea  con  espiral  o  reparación  abierta  si  el  AAA  aumenta  de  tamaño.  La  endofuga  de  tipo  III  resulta  
del  flujo  anterógrado  en  el  punto  de  unión  entre  los  componentes  del  injerto.  Estos  se  reparan  con  una  endoprótesis  
secundaria.
La  endofuga  de  tipo  IV  se  debe  a  la  porosidad  de  la  pared  del  injerto  y  se  puede  tratar  con  una  endoprótesis  secundaria  o  con  
observación.  Las  tomografías  computarizadas  de  la  aorta  se  realizan  a  intervalos  regulares  después  de  la  REVA  para  identificar  
endofugas  tardías  de  tipo  II  que  pueden  llevar  a  una  ruptura  tardía  del  AAA.
Machine Translated by Google
364  CAPÍTULO  71  ANEURISMA  AÓRTICO  ABDOMINAL

24.  Describa  la  evaluación  necesaria  para  un  paciente  que  recibe  EVAR.
Una  historia  y  un  examen  físico  con  un  enfoque  cuidadoso  en  el  sistema  cardiopulmonar  son  esenciales  
junto  con  los  datos  de  laboratorio  apropiados.  Una  ATC  con  cortes  finos  y  reconstrucciones  tridimensionales  puede  
ayudar  a  medir  el  diámetro  y  evaluar  la  longitud  del  cuello,  la  angulación  y  el  trombo.  También  proporcionará  datos  
valiosos  sobre  el  diámetro  de  los  vasos  (arterias  ilíaca  y  femoral),  calcificación  y  tortuosidad  para  determinar  la  
idoneidad  del  paciente  y  el  tipo  de  dispositivo  que  sellará  mejor  el  aneurisma.  En  general,  el  diámetro  del  cuello  
aórtico  proximal  por  debajo  de  la  arteria  renal  más  baja  debe  estar  entre  20  y  28  mm  y  al  menos  15  mm  o  más  de  longitud.

25.  ¿Cuáles  son  los  aspectos  técnicos  de  EVAR?
Las  incisiones  longitudinales  en  la  ingle  exponen  las  arterias  femorales,  aunque  muchos  centros  lo  están  haciendo  de  
forma  percutánea.  El  control  posterior  proximal  y  distal  de  las  arterias  femoral  común,  superficial  y  profunda  permitirá  la  
introducción  de  vainas,  alambres  y  el  dispositivo  endovascular  para  reparar  el  AAA.  El  paciente  recibe  heparinización  
sistémica  antes  de  la  punción  arterial.  Un  aortograma  confirma  los  puntos  de  referencia  anatómicos  necesarios  para  el  
despliegue  del  injerto  y  la  distancia  desde  las  arterias  renales  hasta  la  bifurcación  de  la  arteria  ilíaca  común  para  
seleccionar  con  precisión  la  longitud  del  injerto.  Se  debe  anotar  la  ubicación  de  las  arterias  renales  y  no  mover  el  arco  en  
C  hasta  que  se  complete  el  despliegue  del  injerto.  En  algunos  casos,  después  del  despliegue  del  injerto  aórtico,  el  sellado  
se  logra  con  una  angioplastia  con  balón  suave.  Se  despliega  una  rama  contralateral  dentro  de  la  arteria  ilíaca  y  se  
superpone  al  injerto  principal  para  lograr  un  buen  sellado  y  evitar  una  endofuga  de  tipo  III.  Un  angiograma  repetido  confirma  
la  colocación  e  identifica  las  endofugas  tipo  I  y  tipo  III.  Después  de  la  extracción  de  los  alambres  y  vainas,  las  arterotomías  
femorales  se  cierran  con  sutura  no  reabsorbible  y  las  incisiones  se  cierran  en  dos  capas  o  se  utilizan  dispositivos  de  cierre  
percutáneo.

26.  ¿A  qué  tamaño  se  deben  reparar  los  AAA  asintomáticos  de  forma  electiva?
Deben  repararse  de  forma  electiva  cuando  el  AAA  alcanza  los  5,5  cm  de  diámetro.  El  único  beneficio  de  la  reparación  de  
un  AAA  asintomático  es  prevenir  la  ruptura  y  la  muerte  posteriores.  Por  lo  tanto,  todos  los  candidatos  para  la  reparación  
electiva  deben  esperar  vivir  al  menos  5  años.

27.  ¿Cuáles  son  los  aspectos  técnicos  de  la  cirugía  del  AAA?
Las  dos  decisiones  importantes  son  la  ubicación  de  las  pinzas  arteriales  y  el  tipo  de  injerto  a  colocar.  La  mayoría  de  los  
casos  se  pueden  manejar  colocando  la  pinza  arterial  debajo  de  las  arterias  renales.
Esto  evita  la  isquemia  prolongada  de  los  riñones.  El  aneurisma  se  abre  después  de  pinzar  proximal  y  distalmente.  
Los  orificios  de  la  arteria  lumbar  se  suturan  para  evitar  el  sangrado  de  las  arterias  colaterales.  La  arteria  mesentérica  
inferior  a  menudo  se  ocluye,  pero  cuando  es  permeable  y  no  sangra  intensamente,  puede  requerir  reimplantación.

28.  ¿Cuáles  son  las  principales  complicaciones  no  cardíacas  de  la  reparación  del  AAA?
Insuficiencia  renal  (elevación  de  la  creatinina)  e  isquemia  intestinal  (diarrea  sanguinolenta).

PUNTOS  CLAVE

1.  Un  AAA  se  define  como  un  aumento  del  50%  del  diámetro  aórtico  normal.

2.  El  cuarenta  por  ciento  de  los  pacientes  con  un  aneurisma  de  la  arteria  poplítea  albergan  un  AAA.

3.  La  TC  es  la  mejor  modalidad  de  imagen  individual  para  planificar  una  reparación  de  AAA.

4.  El  AAA  debe  repararse  de  forma  electiva  cuando  el  tamaño  alcanza  los  5,5  cm  de  diámetro.

5.  La  longitud  del  cuello,  la  angulación  y  el  trombo  del  AAA  determinan  si  la  EVAR  es  factible.
Machine Translated by Google
CAPÍTULO  71  ANEURISMA  AÓRTICO  ABDOMINAL  365

SITIOS  WEB

www.acssurgery.com

www.vascularweb.org

BIBLIOGRAFÍA

1.  Harkin  DW,  Dillon  M,  Blair  PH  et  al .:  Reparación  endovascular  de  aneurisma  aórtico  abdominal  roto  (EVRAR):  una  revisión  
sistemática.  Eur  J  Vasc  Endovasc  Surg  34(6):673­681,  2007.

2.  Tambyraja  A,  Murie  J,  Chalmers  R:  Predictores  del  resultado  después  de  la  ruptura  del  aneurisma  aórtico  abdominal:  Edimburgo
Puntuación  de  aneurisma  roto.  World  J  Surg  31(11):2243­2247,  2007.

3.  Reparación  endovascular  de  aneurisma  versus  reparación  abierta  en  pacientes  con  aneurisma  aórtico  abdominal  (ensayo  EVAR  1):
ensayo  controlado  aleatorio.  Lancet  365  (9478):  2179­2186,  2005.

4.  Greenhalgh  RM,  Brown  LC,  Kwong  GP  et  al.:  Comparación  de  la  reparación  endovascular  del  aneurisma  con  la  reparación  abierta  
en  pacientes  con  aneurisma  aórtico  abdominal  (ensayo  EVAR  1),  resultados  de  mortalidad  quirúrgica  a  los  30  días:  ensayo  
controlado  aleatorizado.  Lancet  364  (9437):  843­848,  2004.

5.  Reparación  endovascular  de  aneurisma  y  resultado  en  pacientes  no  aptos  para  la  reparación  abierta  de  aneurisma  aórtico  
abdominal  (ensayo  EVAR  2):  ensayo  controlado  aleatorizado.  Lancet  365  (9478):  2187­2192,  2005.

6.  Faries  PL,  Cadot  H,  Agarwal  G  et  al.:  Manejo  de  la  endofuga  después  de  la  reparación  endovascular  de  un  aneurisma:  manguitos,  
espirales  y  conversión.  J  Vasc  Surg  37(6):1155­1161,  2003.

7.  Lecroy  C,  Passman  MA,  Taylor  S  et  al.:  ¿Debería  utilizarse  la  reparación  endovascular  para  los  aneurismas  aórticos  
abdominales  pequeños?  Vasc  Endovascular  Surg  42:113­119;  discusión,  120­121,  2008.

8.  Powell  JT,  Brown  LC,  Forbes  JF  et  al.:  Seguimiento  final  de  12  años  de  cirugía  versus  vigilancia  en  el  ensayo  de  aneurismas  
pequeños  del  Reino  Unido.  Br  J  Surg  94(6):702­708,  2007.

9.  Alonso­Perez  M,  Segura  RJ,  Sanchez  J  et  al.:  Factores  que  aumentan  la  tasa  de  mortalidad  de  los  pacientes  con  rotura
aneurismas  de  aorta  abdominal.  Ann  Vasc  Surg  15(6):601­607,  2001.

10.  Lederle  FA,  Johnson  GR,  Wilson  SE  et  al.:  Tasa  de  ruptura  de  aneurismas  aórticos  abdominales  grandes  en  pacientes  que  se  niegan
o  no  apto  para  reparación  electiva.  JAMA  287:2968­2972,  2002.

11.  Lederle  FA,  Wilson  SE,  Johnson  GR  et  al.:  Reparación  inmediata  comparada  con  vigilancia  de  dolor  abdominal  pequeño
aneurismas  aórticos.  N  Engl  J  Med  346:1437­1444,  2002.
Machine Translated by Google

ENFERMEDAD  VENOSA
CAPITULO  
72

Franklin  L.  Wright,  MD  y  Thomas  A.  Whitehill,  MD

1.  ¿Dónde  se  origina  la  trombosis  venosa  profunda  (TVP)?
Más  del  95  %  de  las  TVP  se  desarrollan  en  las  venas  profundas  de  las  extremidades  inferiores;  la  mayoría  se  originan  en  los  
senos  de  las  válvulas  de  las  venas  de  la  pantorrilla.

2.  ¿Cuál  es  el  origen  habitual  de  una  embolia  pulmonar?
La  trombosis  de  la  vena  de  la  pantorrilla  puede  propagarse  proximalmente  hacia  el  sistema  venoso  profundo  para  
afectar  las  venas  poplítea,  femoral  o  ilíaca  (o  una  combinación  de  venas).  Estas  TVP  proximales  son  las  culpables  
de  más  del  90  %  de  las  embolias  pulmonares  (EP).

3.  ¿Qué  es  la  tríada  de  Virchow?
(1)  hipercoagulabilidad,  (2)  alteración  de  un  revestimiento  de  la  íntima  venosa  intacto  y  (3)  estasis  del  flujo  sanguíneo  
venoso.  En  la  mayoría  de  los  pacientes  con  TVP,  al  menos  dos  de  estos  tres  componentes  están  operativos.

4.  ¿Cuáles  son  los  principales  síndromes  de  hipercoagulabilidad  (trombofilia)?
La  mutación  del  factor  V  Leiden,  la  deficiencia  de  antitrombina  III,  la  deficiencia  de  proteína  C,  la  deficiencia  de  
proteína  S,  la  disfibrinogenemia,  el  anticoagulante  lúpico,  el  síndrome  antifosfolípido,  el  factor  VIII  elevado,  la  mutación  
de  la  protrombina  20210A  y  las  anomalías  de  la  fibrinólisis  son  los  principales  ejemplos.  La  más  común  es  la  mutación  
del  factor  V  Leiden  (es  decir,  resistencia  a  la  proteína  C  activada).

5.  ¿Qué  causa  la  lesión  de  la  íntima  venosa?
Los  cambios  en  la  íntima  venosa  pueden  ser  secundarios  a  un  traumatismo  en  la  pared  de  la  vena,  infección,  
inflamación,  catéteres  permanentes  o  cirugía.  La  venodilatación  durante  la  anestesia  y  la  cirugía  puede  producir  
desgarros  microscópicos  de  la  íntima  y  estasis.  La  íntima  venosa  lesionada  inicia  la  liberación  de  sustancias  
tromboplásticas  que  pueden  activar  la  cascada  de  la  coagulación.

6.  ¿Qué  causa  la  estasis  del  flujo  sanguíneo  venoso?
La  venostasis  es  común  en  pacientes  quirúrgicos;  ocurre  durante  la  anestesia,  después  de  ciertos  tipos  de  trauma  
y  con  la  inmovilidad  perioperatoria.

7.  ¿Cuáles  son  los  factores  de  riesgo  clínicos  habituales  para  la  TVP?

Los  factores  de  riesgo  incluyen  malignidad  (especialmente  cáncer  de  páncreas,  genitourinario,  estómago,  pulmón,  
colon,  cerebro,  ovario,  riñón  y  mama),  edad  mayor  de  40  años,  obesidad,  antecedentes  de  trombosis  venosa  o  
EP,  antecedentes  familiares,  procedimientos  quirúrgicos  mayores,  inflamación  enfermedad  intestinal  (EII),  embarazo,  
terapia  hormonal,  movilidad  limitada,  parálisis,  estado  de  hipercoagulabilidad  y  trauma.

8.  ¿Qué  signos  y  síntomas  sugieren  TVP?  ¿Cómo  se  puede  determinar  con  precisión  la  TVP?
diagnosticado?
Los  signos  y  síntomas  son  dolor  en  la  pantorrilla  o  el  muslo,  hipersensibilidad,  aumento  de  la  temperatura  de  
la  piel,  hinchazón  o  dilatación  venosa  superficial.  Ninguno  de  estos  signos  es  específico  de  la  TVP.  Incluso  el  
conocido  signo  de  Homan  (es  decir,  dolor  en  la  pantorrilla  con  dorsiflexión  del  pie)  no  es  fiable;  su  precisión  es  sólo  
del  50%.  El  examen  de  ultrasonido  Doppler  (US)  (escaneo  dúplex)  detecta  TVP  proximal  a  las  venas  de  la  pantorrilla  
con  una  precisión  >95%;  desafortunadamente,  no  es  tan  sensible  para  detectar  la  TVP  de  la  vena  de  la  pantorrilla.  La  
venografía  ascendente  sigue  siendo  el  estándar  de  referencia.

366
Machine Translated by Google
CAPÍTULO  72  ENFERMEDAD  VENOSA  367

9.  ¿Tiene  algún  valor  la  prueba  del  dímero  D?
La  medición  de  los  productos  de  degradación  de  fibrina  reticulada  (FDP)  con  dímero  D,  formada  por  la  
acción  de  la  plasmina  sobre  la  fibrina  reticulada,  se  ha  propuesto  como  una  alternativa  a  las  pruebas  no  
invasivas  iniciales.  Se  informó  una  sensibilidad  del  96,8  %  y  una  especificidad  del  35,2  %  para  la  prueba  de  
ensayo  inmunoabsorbente  ligado  a  enzimas  (ELISA),  lo  que  teóricamente  hace  posible  limitar  las  pruebas  no  
invasivas  a  aquellos  con  pruebas  de  dímero  D  positivas.  Desafortunadamente,  la  prueba  ELISA  consume  mucho  
tiempo  y  es  poco  práctica  como  prueba  de  detección.  Ahora  están  disponibles  ensayos  ELISA  más  rápidos  (1  
hora).  La  evaluación  prospectiva  de  la  seguridad  de  suspender  el  tratamiento  anticoagulante  en  pacientes  con  
dímero  D  negativo  ha  sido  limitada.  Los  resultados  falsos  positivos  son  un  problema  en  pacientes  con  malignidad,  
infección,  embarazo,  traumatismo,  hemorragia  o  cirugía  reciente.

10.  ¿Qué  métodos  de  profilaxis  de  la  TVP  perioperatoria  se  deben  utilizar?  En  el  cual
pacientes  quirúrgicos?
Se  recomienda  encarecidamente  la  profilaxis  perioperatoria  de  la  TVP  en  todos  los  pacientes  de  alto  riesgo  
mayores  de  40  años  que  se  someten  a  procedimientos  ortopédicos  o  generales  importantes.  En  pacientes  
quirúrgicos  generales,  las  medidas  profilácticas  bien  aplicadas  disminuyen  el  riesgo  relativo  de  TVP  en  un  67%.  
La  mejor  profilaxis  para  la  TVP  incluye  caminar  antes  y  después  de  la  operación.  Se  recomiendan  medias  de  
compresión  neumática  intermitente  y  alguna  forma  de  terapia  profiláctica  anticoagulante  (dosis  bajas  de  heparina  
no  fraccionada  [LDUH]  o  heparina  de  bajo  peso  molecular  [HBPM])  a  medida  que  aumenta  el  perfil  de  riesgo  del  
paciente;  La  profilaxis  anticoagulante  es  frecuentemente  infraaplicada  dada  la  alta  prevalencia  de  factores  de  
riesgo  entre  los  pacientes  hospitalizados.

11.  ¿Cómo  actúa  la  heparina?
La  heparina  se  une  a  la  antitrombina  III  (ATIII),  haciéndola  más  activa.  La  heparina  en  dosis  bajas  (5  000  U  
administradas  por  vía  subcutánea  cada  8  a  12  h  hasta  que  el  paciente  deambula  por  completo)  activa  la  ATIII,  
inhibe  la  agregación  plaquetaria  y  disminuye  la  disponibilidad  de  trombina.

12.  ¿Qué  es  la  HBPM?
LMWH  es  un  fragmento  de  heparina  producido  por  descomposición  química.  Ejerce  su  efecto  anticoagulante  
uniéndose  a  la  ATIII  e  inhibiendo  varias  enzimas  de  la  coagulación,  principalmente  el  factor  Xa.  Tiene  una  vida  
media  más  larga  que  la  heparina  de  preparación  estándar  y  se  puede  administrar  una  vez  al  día.  La  HBPM  da  una  
respuesta  anticoagulante  más  predecible  a  dosis  altas  y,  por  lo  tanto,  puede  administrarse  sin  vigilancia  (no  es  
necesario  seguir  el  tiempo  de  tromboplastina  parcial).

13.  ¿Debería  considerarse  alguna  vez  la  colocación  de  un  filtro  en  la  vena  cava  inferior  (VCI)?
En  pacientes  con  EP  recurrente  documentada  mientras  reciben  una  terapia  anticoagulante  adecuada  o  con  
una  contraindicación  absoluta  para  la  anticoagulación,  se  puede  colocar  un  filtro  en  la  vena  cava  inferior  (VCI)  
para  evitar  la  embolización  o  la  propagación  del  coágulo  a  los  pulmones.  Los  filtros  IVC  están  asociados  con  
un  aumento  significativo  posterior  en  la  tasa  de  TVP.  A  pesar  del  renovado  interés  en  los  filtros  IVC  como  resultado  
de  los  avances  en  dispositivos  recuperables,  existe  evidencia  insuficiente  para  ampliar  estas  indicaciones.

14.  ¿Cuánto  tiempo  debe  continuarse  la  anticoagulación  después  de  la  TVP?
Los  pacientes  con  una  causa  transitoria/reversible  (p.  ej.,  traumatismo,  cirugía)  de  una  TVP  inicial  pueden  
recibir  anticoagulación  durante  3  meses  (HBPM  o  antagonista  de  la  vitamina  K  [es  decir,  warfarina]).  El  
tratamiento  debe  extenderse  de  6  a  12  meses  para  una  TVP  inicial  de  origen  desconocido.  Los  factores  de  
riesgo  para  la  TVP  recurrente  incluyen  antecedentes  personales  o  familiares  fuertes  de  TVP,  estado  
trombofílico  (p.  ej.,  cáncer,  síndrome  de  hipercoagulabilidad),  dímero  D  persistentemente  elevado  después  
de  la  terapia  o  coágulo  residual  en  la  ecografía  Doppler  dúplex  de  seguimiento.  En  presencia  de  uno  de  estos  
factores  de  riesgo,  el  médico  debe  sopesar  los  riesgos  y  beneficios  de  la  anticoagulación  de  por  vida  en  consulta  
con  el  paciente.
Machine Translated by Google
368  CAPÍTULO  72  ENFERMEDAD  VENOSA

15.  ¿Cuáles  son  las  características  de  la  insuficiencia  venosa  crónica  y  del  síndrome  posflebítico  o  postrombótico?

La  característica  primaria  es  insuficiencia  valvular  venosa  u  obstrucción  venosa  persistente  con  
hipertensión  venosa  ambulatoria  distal.  Después  de  la  TVP,  los  segmentos  venosos  afectados  eventualmente  se  
recanalizan  hasta  cierto  punto.  Sin  embargo,  sus  delicadas  válvulas  permanecen  cicatrizadas  o  atrapadas  por  trombos  
organizados  residuales.  La  pérdida  de  la  función  valvular  inhabilita  la  bomba  venomotora.  Las  paredes  de  las  venas  se  
vuelven  más  gruesas  y  menos  distensibles,  aumentando  la  resistencia  al  flujo  sanguíneo  proximal.  Estos  factores  
resultan  en  hipertensión  venosa  distal.  Los  líquidos  ricos  en  proteínas,  la  fibrina  y  los  glóbulos  rojos  se  extravasan  y  
depositan  a  través  de  poros  grandes  en  la  microcirculación  distendida  durante  los  períodos  de  hipertensión  venosa.  
Este  proceso  conduce  a  inflamación,  cicatrización,  fibrosis  de  los  tejidos  subcutáneos  y  decoloración  por  depósito  de  
hemosiderina  (edema  "musculoso").  La  reacción  inflamatoria  resultante,  la  cicatrización  y  el  edema  intersticial  crean  
una  barrera  adicional  para  el  flujo  capilar  y  la  difusión  de  oxígeno;  se  inhibe  la  nutrición  adecuada  de  la  piel.  Estos  
cambios  pueden  conducir  a  la  atrofia  y  ulceración  del  tejido  (es  decir,  úlcera  por  estasis  venosa).

16.  ¿Todos  los  pacientes  con  TVP  desarrollan  síndrome  posflebítico  o  postrombótico?
No.  Aproximadamente  entre  un  tercio  y  la  mitad  de  los  pacientes  con  TVP  desarrollarán  un  síndrome  
postflebítico  clínicamente  relevante,  generalmente  en  los  primeros  2  años  posteriores  a  la  TVP.  Estudios  epidemiológicos  
recientes  sugieren  que  la  incidencia  de  ulceración  venosa  es  de  alrededor  del  5%.  De  interés,  el  50%  de  los  pacientes  
con  úlceras  venosas  no  tienen  antecedentes  de  TVP  (probablemente  debido  a  una  TVP  previa  asintomática  de  la  vena  
de  la  pantorrilla).

17.  ¿Cómo  se  tratan  los  pacientes  con  síndrome  posflebítico?
Con  la  educación  adecuada  del  paciente  y  el  cumplimiento,  las  secuelas  de  estasis  posflebíticas  pueden  controlarse  
por  medios  no  quirúrgicos  en  más  del  90%  de  los  pacientes,  particularmente  si  ninguna  obstrucción  del  flujo  venoso  
residual  complica  la  incompetencia  valvular.  El  tratamiento  no  quirúrgico  consiste  en  medias  de  compresión  elásticas  
graduadas  (o  botas  de  Unna)  para  retardar  la  hinchazón  y  la  elevación  periódica  de  las  piernas  durante  el  día.  Se  debe  
enseñar  a  los  pacientes  a  elevar  las  piernas  por  encima  del  nivel  del  corazón  ("los  dedos  de  los  pies  por  encima  de  la  
nariz")  a  intervalos  regulares  (p.  ej.,  de  10  a  15  minutos  cada  2  horas).  El  cumplimiento  es  fundamental.

18.  Distinguir  entre  flegmasia  alba  dolens  y  flegmasia  cerulea  dolens.
La  trombosis  venosa  iliofemoral  se  caracteriza  por  dolor  unilateral  y  edema  de  toda  una  extremidad  inferior,  decoloración  
y  sensibilidad  en  la  ingle.  Un  total  de  75%  de  los  casos  de  trombosis  venosa  iliofemoral  ocurren  en  el  lado  izquierdo,  
presumiblemente  debido  a  la  compresión  de  la  vena  ilíaca  común  izquierda  por  la  arteria  ilíaca  común  derecha  
suprayacente  (síndrome  de  May­Thurner).  En  la  flegmasia  alba  dolens  (literalmente,  hinchazón  blanca  y  dolorosa),  la  
pierna  se  vuelve  pálida  y  blanca.  Los  pulsos  arteriales  permanecen  normales.  La  trombosis  progresiva  puede  ocurrir  
con  propagación  proximal  o  distal  y  hacia  los  afluentes  vecinos.  Toda  la  pierna  se  vuelve  edematosa  y  moteada  o  
cianótica.  Esta  etapa  se  llama  flegmasia  cerulea  dolens  (literalmente,  hinchazón  morada  dolorosa).

Cuando  el  flujo  de  salida  venoso  se  ve  gravemente  impedido,  el  flujo  de  entrada  arterial  puede  reducirse  
secundariamente  hasta  en  un  30%.  La  pérdida  de  extremidades  es  una  preocupación  seria;  es  necesario  un  manejo  
agresivo  (es  decir,  trombectomía  venosa,  terapia  lítica  dirigida  por  catéter,  o  ambas).

19.  ¿Qué  es  la  claudicación  venosa?
Cuando  no  se  produce  la  recanalización  venosa  después  de  una  trombosis  venosa  iliofemoral,  se  desarrollan  
venas  colaterales  para  sortear  la  obstrucción  del  flujo  venoso.  Estas  garantías  suelen  ser  suficientes  mientras  el  
paciente  está  en  reposo.  Sin  embargo,  el  ejercicio  de  piernas  induce  un  aumento  del  flujo  arterial,  que  puede  exceder  
la  capacidad  del  lecho  venoso  colateral  y  dar  como  resultado  una  hipertensión  venosa  progresiva.
La  acumulación  de  presión  en  el  sistema  venoso  da  como  resultado  un  dolor  en  la  pantorrilla  que  comúnmente  se  
describe  como  apretado,  pesado  o  estallante  (claudicación  venosa).  El  alivio  se  obtiene  con  reposo  y  elevación,  pero  
no  es  tan  rápido  como  con  la  claudicación  arterial.
Machine Translated by Google
CAPÍTULO  72  ENFERMEDAD  VENOSA  369

20.  ¿Cómo  se  pueden  distinguir  las  venas  varicosas  primarias  de  las  venas  varicosas  secundarias?
Las  venas  varicosas  primarias  son  el  resultado  de  una  válvula  venosa  safenofemoral  sin  complicaciones.
incompetencia  y  mayor  distribución  safena,  prueba  de  torniquete  positiva,  sin  secuelas  de  estasis  (dermatitis  o  
ulceración),  y  sin  edema  matutino  de  tobillo  (linfedema).
Las  venas  varicosas  secundarias  son  más  comúnmente  una  consecuencia  de  venas  profundas  y  perforantes.
incompetencia  secundaria  al  síndrome  posflebítico.

21.  ¿Por  qué  las  personas  desarrollan  venas  varicosas  primarias?
La  causa  más  común  es  la  ausencia  congénita  de  válvulas  venosas  proximales  a  la  unión  safenofemoral.  Normalmente  
no  hay  válvulas  en  la  vena  cava  o  en  las  venas  ilíacas  comunes  y  solo  una  válvula  ocasional  en  las  venas  ilíacas  
externas.  Por  lo  tanto,  la  válvula  centinela  en  la  vena  femoral  común  justo  por  encima  de  la  unión  safenofemoral  es  de  
importancia  crítica.  Sin  embargo,  los  estudios  anatómicos  revelan  que  esta  válvula  está  ausente  en  uno  u  otro  lado  en  
el  30%  de  los  pacientes.

22.  ¿Cómo,  cuándo  y  en  quién  se  deben  tratar  las  varices?
Las  venas  varicosas  que  causan  molestias  o  vergüenza  estética  grave  requieren  tratamiento.
Se  obtienen  mejores  resultados  con  el  tratamiento  temprano  antes  de  que  la  presión  retrógrada  continua  y  el  flujo  por  
el  sistema  superficial  y  hacia  las  venas  perforantes  comunicantes  (siempre  que  el  paciente  esté  de  pie)  causen  una  
incompetencia  perforante  secundaria  e  irreversible.  La  ligadura  de  la  vena  safena  alta  en  una  etapa  temprana  puede  
detener  la  progresión  de  este  proceso  gravitacional.  Las  várices  distales  pueden  tratarse  mediante  extracción  
quirúrgica  selectiva,  escleroterapia  o  ambas.

PUNTOS  CLAVE:  ENFERMEDAD  VENOSA

1.  Más  del  95  %  de  las  TVP  se  desarrollan  en  las  venas  profundas  de  las  extremidades  inferiores;  la  mayoría
se  originan  en  los  senos  de  las  válvulas  de  las  venas  de  la  pantorrilla.

2.  La  tríada  de  Virchow  consiste  en  hipercoagulabilidad,  alteración  de  un  revestimiento  de  la  íntima  venosa  intacto,
y  estasis  del  flujo  sanguíneo  venoso.

3.  La  mejor  profilaxis  para  la  TVP  incluye  caminar  antes  y  después  de  la  operación;  sin  embargo,  se  debe  
considerar  la  anticoagulación  profiláctica  para  muchos  pacientes  quirúrgicos  que  tienen  factores  de  riesgo  
significativos  para  la  TVP.

SITIO  WEB

www.acssurgery.com

BIBLIOGRAFÍA

1.  Clarke­Pearson  DL,  Dodge  RK,  Synan  I  et  al.:  Profilaxis  del  tromboembolismo  venoso:  pacientes  con  alto  riesgo  de  fracasar
compresión  neumática  intermitente.  Obstet  Gynecol  101:157,  2003.

2.  Franks  PJ,  Sharp  EJ,  Moffatt  CJ:  Factores  de  riesgo  para  la  recurrencia  de  úlceras  en  las  piernas:  un  ensayo  aleatorio  de  dos  tipos  
de  medias  de  compresión.  Edad  Envejecimiento  24:490,  1995.

3.  Gallix  BP,  Achard­Lichere  C,  Dauzat  M  et  al .:  Venografía  por  resonancia  magnética  independiente  del  flujo  de  la  pantorrilla.
J  Mag  Reson  Imaging  17:421,  2003.
Machine Translated by Google
370  CAPÍTULO  72  ENFERMEDAD  VENOSA

4.  Geerts  WH,  Pineo  GF,  Heit  JA  et  al.:  Prevención  del  tromboembolismo  venoso.  Cofre  126:338S–400S,  2004.

5.  Ginsberg  JS:  Manejo  del  tromboembolismo  venoso.  N  Engl  J  Med  335:1816,  1996.

6.  Janssen  MC,  Wollersheim  H,  Verbruggen  B  et  al .:  Ensayos  rápidos  de  dímero  D  para  excluir  la  trombosis  venosa  profunda  y  la  embolia  
pulmonar:  estado  actual  y  nuevos  desarrollos.  Semin  Thromb  Hemost  24:393,  1998.

7.  Kahn  SR:  Frecuencia  y  determinantes  del  síndrome  postrombótico  tras  tromboembolismo  venoso.
Curr  Opin  Pulm  Med  12:299,  2006.

8.  Meissner  MH,  Wakefield  TW,  Ascher  E  et  al.:  Enfermedad  venosa  aguda:  trombosis  venosa  y  traumatismo  venoso.
J  Vasc  Surg  46S:  25S,  2008.

9.  Philbrick  JT,  Heim  S:  La  prueba  del  dímero  D  para  la  trombosis  venosa  profunda:  estándares  de  oro  y  sesgo  en  negativo
valor  predictivo.  Clin  Chem  49:570,  2003.

10.  Sorensen  HT,  Mellemkjaer  L,  Steffensen  FH  et  al.:  El  riesgo  de  un  diagnóstico  de  cáncer  después  de  un  tratamiento  venoso  profundo  primario.
trombosis  o  embolia  pulmonar.  N  Engl  J  Med  338:1169,  1998.

11.  Young  T,  Tang  H,  Aukes  J  et  al .:  Filtros  de  vena  cava  para  la  prevención  de  la  embolia  pulmonar.  Cochrane
Sistema  de  base  de  datos  Rev  4:  CD006212,  2007.
Machine Translated by Google

DIAGNÓSTICO  VASCULAR  NO  INVASIVO
LABORATORIO
CAPITULO  
73

Darrell  N.  Jones,  PhD,  Jason  Q.  Alexander,  MD  y  Tony  T.  Nguyen,  DO

1.  ¿Cuál  es  el  papel  del  laboratorio  de  diagnóstico  vascular  (VDL)  en  la  evaluación
y  el  tratamiento  de  pacientes  con  sospecha  de  enfermedad  vascular?
Aunque  la  evaluación  tradicional  realizada  por  un  médico  experimentado  sigue  siendo  la  base  del  diagnóstico  
vascular,  la  evaluación  clínica  tiene  sus  limitaciones.  Por  ejemplo,  solo  un  tercio  de  los  soplos  cervicales  se  relacionan  
con  enfermedad  importante  de  la  arteria  carótida;  por  el  contrario,  hasta  dos  tercios  de  los  pacientes  con  enfermedad  
carotídea  grave  se  presentan  sin  soplo  cervical.  La  mitad  de  los  pacientes  con  trombosis  venosa  profunda  (TVP)  extensa  
de  las  extremidades  inferiores  carecen  de  signos  y  síntomas  atribuibles  a  las  extremidades  inferiores,  y  más  de  la  mitad  
de  los  pacientes  que  presentan  signos  clínicos  de  TVP  son  venográficamente  normales.  Hasta  el  40%  de  los  pacientes  
con  diabetes  no  tienen  enfermedad  oclusiva  arterial  periférica  de  grandes  vasos.  El  laboratorio  de  diagnóstico  vascular  
(VDL)  proporciona  datos  de  estado  objetivos,  cuantitativos  y  funcionales  para  delinear  la  gravedad  de  la  enfermedad  
cerebrovascular  extracraneal,  la  enfermedad  oclusiva  arterial  periférica  y  la  enfermedad  venosa  aguda  y  crónica.

2.  ¿Qué  diferencia  al  VDL  de  la  radiología  diagnóstica  y  la  ecografía?
El  VDL  proporciona  información  funcional  en  lugar  o  además  de  los  datos  morfológicos  proporcionados  por  las  
pruebas  de  radiología  y  las  imágenes  generales  de  ultrasonido  (US).  Esta  información  es  particularmente  importante  
para  la  enfermedad  oclusiva  arterial  periférica,  la  insuficiencia  venosa  crónica  y  la  vigilancia  posoperatoria  de  las  
intervenciones  vasculares.  La  información  anatómica  sobre  el  sitio  de  la  estenosis  o  la  oclusión  tiene  un  valor  limitado  
sin  el  conocimiento  de  la  importancia  funcional  al  evaluar  la  enfermedad  arterial  periférica.  Las  opciones  de  tratamiento  
para  pacientes  con  insuficiencia  venosa  crónica  dependen  de  la  función  de  los  sistemas  venosos  superficiales  y  
profundos  más  que  de  la  evidencia  ultrasonográfica  de  obstrucciones.  La  identificación  temprana  de  estrechamientos  
en  el  período  postoperatorio  puede  permitir  una  intervención  temprana  para  prolongar  la  durabilidad  de  la  reparación.

ENFERMEDAD  CEREBROVASCULAR

3.  ¿Qué  pruebas  no  invasivas  deben  usarse  para  diagnosticar  la  arteria  carótida  extracraneal?
¿enfermedad?

La  ecografía  dúplex  (US)  tiene  una  sensibilidad  del  97  %  para  detectar  la  enfermedad  de  la  arteria  carótida  y  una  
precisión  del  95  %  para  clasificar  correctamente  las  estenosis  carotídeas  como  una  reducción  del  diámetro  superior  al  50  %.
Aunque  la  angiografía  con  catéter  es  el  estándar  de  oro  para  la  evaluación  de  la  enfermedad  oclusiva  de  la  arteria  
carótida,  este  procedimiento  se  asocia  con  una  tasa  de  accidente  cerebrovascular  del  1,2%.  La  angiografía  por  
tomografía  computarizada  (CTA)  y  la  angiografía  por  resonancia  magnética  (MRA)  también  demuestran  una  alta  
sensibilidad  y  especificidad  para  la  enfermedad  oclusiva  de  la  carótida,  pero  para  obtener  estos  resultados  se  requiere  la  
administración  de  agentes  de  contraste  (generalmente  renagraffin  o  gadolinio)  que  conllevan  un  riesgo  de  nefropatía  o  
sistémico.  fibrosis.  Estos  riesgos  no  están  asociados  con  la  ecografía  dúplex.

371
Machine Translated by Google
372  CAPÍTULO  73  LABORATORIO  DE  DIAGNÓSTICO  VASCULAR  NO  INVASIVO

4.  ¿Qué  es  la  ecografía  dúplex?
La  ecografía  dúplex  utiliza  datos  de  imagen  y  velocidad  (de  ahí  el  nombre  dúplex)  en  una  presentación  casi  
simultánea  de  imágenes  ecográficas  ecográficas  (a  menudo  denominadas  "escala  de  grises"  o  ecografía  en  modo  B  
y  formas  de  onda  de  velocidad  de  la  sangre  obtenidas  mediante  ecografía  Doppler.  las  señales  se  obtienen  de  una  
sola  región  pequeña  del  vaso  sanguíneo.  Las  velocidades  promedio  se  pueden  estimar  para  múltiples  regiones  de  
este  tipo  en  un  área  grande  del  vaso.  Al  asignar  colores  a  las  velocidades,  el  flujo  sanguíneo  se  puede  representar  
visualmente.  Tal  presentación,  llamada  flujo  de  color  ecografía  dúplex,  ayuda  al  examen  dúplex  pero  no  puede  
reemplazar  la  información  obtenida  de  la  forma  de  onda  de  velocidad  Doppler.

5.  ¿Por  qué  es  importante  la  velocidad  de  la  sangre  para  evaluar  el  grado  de  la  arteria  carótida?
¿estenosis?
A  menudo  es  difícil  medir  con  precisión  la  luz  arterial  en  una  imagen  de  ecografía  en  modo  B  porque  las  
propiedades  acústicas  (y,  por  lo  tanto,  la  imagen)  de  la  placa  no  calcificada,  el  trombo  e  incluso  la  sangre  pueden  ser  
similares.  El  estrechamiento  arterial  fuerza  la  sangre  a  través  de  un  canal  más  estrecho,  lo  que  aumenta  la  velocidad  
de  la  sangre.  Esta  velocidad  puede  caracterizar  el  grado  de  estrechamiento  arterial.  La  práctica  actual  clasifica  el  
grado  de  estenosis  de  la  carótida  interna  basándose  exclusivamente  en  los  datos  de  velocidad  del  Doppler.

6.  ¿Cuáles  son  los  criterios  de  velocidad  y  los  rangos  categóricos  de  la  estenosis  de  la  arteria  carótida?
Los  criterios  desarrollados  en  la  Universidad  de  Washington  (cuadro  73­1)  son  los  más  ampliamente  
aceptados.  Tenga  en  cuenta  que  la  estenosis  progresiva  de  la  carótida  aumenta  la  señal  de  velocidad  del  flujo  
a  medida  que  el  volumen  de  sangre  pasa  a  través  de  un  orificio  cada  vez  más  pequeño.  Desafortunadamente,  no  
existe  un  criterio  universalmente  exceptuado  para  la  estimación  de  la  estenosis  carotídea  basada  en  mediciones  de  velocidad.
La  ecografía  no  solo  depende  de  cada  ecografista  individual,  sino  también  del  tipo  de  máquina.  En  última  instancia,  
la  ecografía  solo  puede  estimar  los  rangos  de  estenosis  resultantes  de  las  variables  mencionadas  anteriormente  y  la  
variación  en  diferentes  pacientes  (p.  ej.,  los  pacientes  con  una  fracción  de  eyección  [FE]  mejorada  tendrán  velocidades  
elevadas  en  comparación  con  los  pacientes  que  tienen  una  FE  menor  pero  estrechamientos  similares).  Por  lo  tanto,  
cualquier  VDL  no  invasivo  debe  evaluar  continuamente  los  criterios  que  emplea  y  compararlos  con  las  medidas  
estándar  de  oro  (angiografía  con  catéter)  para  verificar  sus  criterios.

TABLA  73­1.  UN  IV  IS  SITYOFWA  SH  IN  G  TON  CR  IT  IS  IA

Estenosis Criterios

0% Velocidad  sistólica  máxima  <125  cm/seg  y  sin  alteración  de  la  
velocidad

1%–15% Velocidad  sistólica  máxima  <125  cm/s  con  turbulencia  durante  la  
desaceleración  sistólica

16%–49% Velocidad  sistólica  máxima  <125  cm/seg  con  turbulencia  en  todo  el  ciclo  
cardíaco

50%–79% Velocidad  sistólica  máxima  >125  cm/seg  y  velocidad  diastólica  <140  
cm/seg

80%–99% Velocidad  diastólica  >140  cm/seg

100% Señal  de  velocidad  de  flujo  ausente

7.  ¿La  ultrasonografía  dúplex  es  capaz  de  determinar  si  la  carótida  interna  está
ocluido?
A  medida  que  la  estenosis  de  un  vaso  se  hace  más  estrecha,  aumenta  la  velocidad  del  flujo  sanguíneo.  Sin  
embargo,  en  un  punto  crítico,  el  diámetro  de  la  estenosis  se  vuelve  tan  estrecho  que  la  sangre  ya  no  puede
Machine Translated by Google
CAPÍTULO  73  LABORATORIO  DE  DIAGNÓSTICO  VASCULAR  NO  INVASIVO  373

atravesará  a  alta  velocidad  y  de  hecho  disminuirá  la  velocidad.  Cuando  la  estenosis  se  acerca  a  la  
oclusión,  la  velocidad  del  flujo  sanguíneo  se  ralentizará  hasta  el  punto  de  que  ya  no  estará  dentro  del  alcance  de  la  
máquina  de  ecografía.  El  estudio  dúplex  puede  entonces  interpretar  una  arteria  carótida  interna  como  ocluida  
cuando  en  realidad  hay  un  "flujo  de  goteo"  o  un  "signo  de  cuerda".  Esta  es  una  diferenciación  importante,  pero  la  
oclusión  de  la  arteria  carótida  interna  rara  vez  requiere  intervención,  sino  una  estenosis  crítica.  asociado  con  flujo  
de  goteo  o  una  señal  de  cuerda  conlleva  un  riesgo  significativo  de  accidente  cerebrovascular.

8.  ¿Qué  tan  precisa  es  la  ecografía  dúplex  de  la  carótida  interna  si  la  contralateral
carótida  interna  está  ocluida?
Cuando  se  ocluye  la  arteria  carótida  interna  contralateral,  el  cuerpo  a  menudo  se  ajustará  para  mantener  la  
misma  perfusión  cerebral  anterior  aumentando  el  flujo  en  la  carótida  interna  de  interés.  Este  flujo  aumentado  
conduce  a  velocidades  elevadas  determinadas  por  Doppler.  Debido  a  que  estas  velocidades  se  utilizan  para  estimar  
la  estenosis,  el  Doppler  puede  predecir  artificialmente  una  estenosis  más  estrecha  de  lo  que  en  realidad  está  presente.

ENFERMEDAD  VENOSA

9.  ¿Qué  prueba  no  invasiva  se  utiliza  para  diagnosticar  la  TVP  aguda?
La  ecografía  dúplex  ha  reemplazado  a  la  pletismografía  de  oclusión  venosa  como  el  estándar  aceptado.  Colorflow  
dúplex  es  útil  porque  ayuda  a  identificar  las  venas  pequeñas  de  las  capas  musculares  y  fasciales.
La  evaluación  ecográfica  implica  los  siguientes  pasos:  1.  
Examinar  la  vena  en  busca  de  trombos  ecogénicos.
2.  Comprimir  la  vena,  ejerciendo  presión  sobre  la  sonda  US,  buscando  el  colapso  completo.
La  incapacidad  para  comprimir  la  vena  sugiere  trombosis.  La  compresión  parcial  sugiere  trombosis  parcial.

3.  Una  señal  Doppler  de  la  vena  que  es  fásica  con  la  respiración  sugiere  que  no  hay
trombo  oclusivo.  Una  señal  que  está  espontáneamente  presente  pero  no  fásica  sugiere  flujo  alrededor  de  una  
oclusión  a  través  de  pequeñas  venas  colaterales.  La  ausencia  de  una  señal  Doppler  en  la  vena  sugiere  
ausencia  de  flujo.

10.  ¿Se  puede  utilizar  la  ecografía  dúplex  para  la  vigilancia  de  pacientes  con  alto  riesgo  de  TVP?
El  diagnóstico  de  TVP  en  pacientes  asintomáticos  presenta  un  dilema.  La  sensibilidad  de  la  ecografía  dúplex  se  
reduce  del  95%  informado  a  <80%  para  la  detección  de  TVP  por  encima  de  la  rodilla  en  pacientes  asintomáticos.  La  
detección  de  la  TVP  en  la  pantorrilla  es  mucho  peor,  con  sensibilidades  tan  bajas  como  el  20  %  en  muchas  series  
informadas.  Sin  embargo,  la  venografía  de  contraste  seriada,  aunque  más  específica,  no  es  una  estrategia  práctica  
de  vigilancia.

11.  ¿Qué  venas  son  anatómicamente  venas  profundas  y  qué  venas  son  venas  superficiales?
Es  importante  diferenciar  entre  venas  superficiales  y  profundas,  ya  que  la  trombosis  venosa  superficial  casi  no  
conlleva  riesgo  de  embolia  pulmonar  (EP),  a  menos  que  primero  se  propague  al  sistema  profundo.  El  sistema  
profundo  se  identifica  debajo  de  las  capas  fasciales  musculares  en  el  cuerpo.  Para  mayor  simplicidad,  a  menudo  es  
más  fácil  recordar  que  si  una  vena  corre  con  una  arteria  nombrada,  se  considera  una  vena  profunda.  Por  lo  tanto,  la  
vena  que  corre  con  la  arteria  femoral  superficial  (anteriormente  llamada  vena  femoral  superficial),  la  vena  femoral,  es  
una  vena  profunda.  Hay  algunas  excepciones  a  esta  regla,  pero  se  limitan  en  gran  medida  a  las  venas  musculares  
de  la  pantorrilla,  el  gastrocnemio  y  las  venas  del  sóleo.

12.  ¿Qué  pruebas  no  invasivas  son  útiles  para  evaluar  la  insuficiencia  venosa?
La  ecografía  Doppler  puede  detectar  el  reflujo  venoso  en  las  venas  profundas  de  las  piernas  y  en  las  venas  safenas  
mayores  y  menores.  Con  experiencia,  la  prueba  se  puede  realizar  usando  un  Doppler  simple  (onda  continua  versus  
Doppler  pulsado),  pero  la  ecografía  dúplex  a  menudo  se  usa  para  facilitar  la  identificación  de  los  segmentos  de  la  
vena  y  las  válvulas  y  para  posicionar  una  muestra  de  Doppler  pulsado  de  manera  confiable.  Cuando  una  válvula  
venosa  es  incompetente,  el  flujo  en  la  vena  se  perpetúa  en  la  periferia  si  se  aplica  presión  en  la  porción  más  central  
de  la  extremidad.  La  diferenciación  entre  válvulas  venosas  competentes  e  incompetentes  en  el  sistema  superficial  y  
profundo  afecta  las  opciones  de  tratamiento.
Machine Translated by Google
374  CAPÍTULO  73  LABORATORIO  DE  DIAGNÓSTICO  VASCULAR  NO  INVASIVO

ENFERMEDAD  OCLUSIVA  ARTERIAL  PERIFÉRICA

13.  ¿Cuál  es  la  prueba  principal  para  el  diagnóstico  de  isquemia  de  las  extremidades  inferiores?
El  índice  tobillo­brazo  (ITB)  o  índice  de  presión  sistólica  normalmente  es  mayor  o  igual  a  1,0.  Por  lo  general,  se  usa  la  
ecografía  Doppler  (en  lugar  de  un  estetoscopio)  como  sensor  de  flujo  distal  al  manguito  de  presión,  pero  también  se  
pueden  usar  instrumentos  pletismográficos.  Las  señales  Doppler  generalmente  se  monitorean  en  la  arteria  tibial  
posterior  o  en  la  arteria  dorsal  del  pie.  El  ABI  es  la  proporción  de  la  presión  arterial  sistólica  (PAS)  más  alta  medida  en  
la  arteria  dorsal  del  pie  o  en  la  arteria  tibial  posterior  en  comparación  con  la  medida  de  PAS  más  alta  en  el  braquio  de  
cualquiera  de  los  brazos.
Desafortunadamente,  los  vasos  calcificados  pueden  evitar  la  compresión  con  manguitos  de  presión,  lo  que  sugiere  
una  PAS  artificialmente  alta.  Esto  es  más  notable  en  pacientes  diabéticos  que  pueden  demostrar  un  ABI  superior  a  1,0  a  
pesar  de  la  enfermedad  estenótica  de  las  extremidades  inferiores  hemodinámicamente  significativa.

14.  ¿Qué  se  gana  midiendo  las  presiones  a  nivel  de  las  extremidades  distintas  del  tobillo?
Las  mediciones  de  la  presión  segmentaria  de  las  extremidades  (SLP),  realizadas  en  la  parte  superior  e  inferior  del  muslo,  
la  pantorrilla  y  el  tobillo,  localizan  los  segmentos  arteriales  implicados  en  la  enfermedad  oclusiva  arterial  periférica.

15.  ¿Qué  pruebas  se  utilizan  para  evaluar  la  enfermedad  arterial  periférica  en  pacientes  que
Son  diabéticos  que  pueden  tener  arterias  incompresibles  causadas  por  calcificación  medial?
El  registro  del  volumen  del  pulso  (PVR,  por  sus  siglas  en  inglés)  es  una  técnica  neumopletismográfica  que  realiza  un  
seguimiento  de  los  cambios  en  el  volumen  de  las  extremidades  a  lo  largo  del  ciclo  cardíaco.  Mide  los  cambios  de  presión  
segmentaria  con  manguitos  neumáticos  en  función  de  los  cambios  de  volumen  de  la  extremidad.  Las  amplitudes  relativas  
de  la  RVP  identifican  la  presencia  de  enfermedad  arterial  periférica  y  localizan  el  segmento  arterial  afectado.  La  PVR  no  se  
ve  afectada  por  la  calcificación  medial.  La  presión  del  dedo  gordo  del  pie  también  se  puede  usar  para  diagnosticar  y  evaluar  
la  gravedad  de  la  enfermedad  en  pacientes  diabéticos  porque  la  calcificación  medial  rara  vez  afecta  las  arterias  digitales.

16.  ¿Cómo  se  debe  evaluar  al  paciente  con  sospecha  de  claudicación  intermitente?
Primero  se  debe  evaluar  al  paciente  obteniendo  ABI  o  SLP  en  reposo.  El  paciente  con  isquemia  en  reposo  normalmente  
no  necesita  más  evaluación.  El  paciente  con  insuficiencia  arterial  leve  en  reposo  o  incluso  con  presiones  normales  en  
reposo  debe  realizar  una  prueba  de  esfuerzo  (caminar  en  cinta  rodante  utilizando  protocolos  de  carga  fija  o  variable)  
seguido  de  ABI.  La  distancia  que  el  paciente  puede  caminar  permite  evaluar  la  discapacidad  funcional,  y  la  reducción  de  
la  presión  en  el  tobillo  después  del  ejercicio,  o  la  falta  de  ella,  permite  evaluar  si  la  discapacidad  es  causada  por  
insuficiencia  arterial  en  lugar  de  dolor  musculoesquelético  o  neurológico.

VIGILANCIA  POR  ULTRASONIDO  DUPLEX  DE  LA  TERAPIA  VASCULAR

17.  ¿Cuál  es  la  importancia  de  la  vigilancia  ecográfica  dúplex  de  autógenos  inferiores
injertos  de  derivación  de  extremidades?
La  vigilancia  dúplex  de  los  injertos  de  derivación  autógenos  infrainguinales  es  fundamental  para  el  cuidado  
posoperatorio  en  el  paciente  vascular.  Los  injertos  autógenos  ocluidos  por  lo  general  no  responden  a  los  intentos  de  
recuperar  la  permeabilidad  y,  a  menudo,  requieren  derivaciones  completamente  nuevas.  Múltiples  estudios  han  
demostrado  que  el  examen  físico  (p.  ej.,  pérdida  o  disminución  del  pulso  distal)  o  el  retorno  de  los  síntomas  isquémicos  
no  identifican  nuevas  estenosis  antes  de  la  oclusión.  La  identificación  temprana  de  estenosis  dentro  o  en  la  periferia  de  
los  injertos  puede  tratarse  mediante  técnicas  abiertas  o  endovasculares  mínimamente  invasivas  antes  de  que  ocurra  la  
oclusión.  La  ecografía  dúplex  puede  identificar  fácilmente  las  estenosis  tanto  dentro  del  injerto  como  en  las  anastomosis  
al  demostrar  cambios  de  velocidad  significativos  en  estos  puntos.

18.  ¿La  ecografía  dúplex  tiene  algún  papel  en  la  vigilancia  de  la  revascularización  infrainguinal?

La  revascularización  infrainguinal  puede  clasificarse  simplemente  como  derivación  de  injerto  o  vena  autógena  y  
derivación  por  encima  o  por  debajo  de  la  rodilla.  Numerosos  estudios  han  comprobado  que  la  tasa  de  permeabilidad  difiere
Machine Translated by Google
CAPÍTULO  73  LABORATORIO  DE  DIAGNÓSTICO  VASCULAR  NO  INVASIVO  375

según  el  tipo  de  conducto  que  se  utilice  y  la  ubicación  de  la  anastomosis  distal:  el  injerto  de  vena  por  
encima  de  la  rodilla  es  el  mejor  y  el  injerto  protésico  por  debajo  de  la  rodilla  es  el  peor  en  términos  de  fracaso  del  
injerto.  Se  ha  documentado  que  la  detección  temprana  del  fracaso  del  injerto  resultante  de  una  estenosis  
significativa  y  la  intervención  subsiguiente  tienen  un  mejor  resultado  en  comparación  con  la  recuperación  de  un  
injerto  ocluido.  Los  injertos  de  vena  tienen  más  probabilidades  de  desarrollar  una  estenosis  progresiva  que  conduce  
a  la  oclusión  en  comparación  con  el  injerto  protésico,  que  generalmente  no  se  estenosa  antes  de  la  oclusión.  La  
ecografía  dúplex,  por  lo  tanto,  tiene  un  papel  en  la  vigilancia  del  injerto  de  vena  para  mejorar  la  permeabilidad.  
Aunque  los  injertos  protésicos  no  muestran  de  manera  rutinaria  estenosis  intrainjerto,  pueden  exhibir  progresión  de  
la  enfermedad  aterosclerótica  de  estrechamiento  hemodinámicamente  significativo  en  las  anastomosis.  Si  estos  
estrechamientos  se  identifican  y  tratan,  es  posible  que  también  se  pueda  extender  la  permeabilidad  del  injerto  
protésico.  Además,  la  bibliografía  actual  tiende  a  respaldar  la  vigilancia  ecográfica  después  de  la  colocación  de  
stents  endovasculares  y  la  angioplastia  en  un  intento  de  identificar  la  reestenosis  temprana  para  acelerar  la  
reintervención.

19.  ¿Cuál  es  el  papel  de  la  ecografía  dúplex  de  vigilancia  después  de  la  carótida?
endarterectomía  (CEA)?
La  justificación  de  la  ecografía  dúplex  de  vigilancia  después  de  la  endarterectomía  carotídea  (CEA)  es  doble.  La  
mayoría  de  los  cirujanos  vasculares  evaluarán  la  reparación  de  la  carótida  ipsilateral  cada  6  meses  o  una  vez  al  
año.  La  incidencia  de  estenosis  recurrente  es  rara,  pero  la  identificación  temprana  puede  permitir  una  intervención  
más  inmediata  o  mínimamente  invasiva  antes  de  que  el  paciente  presente  síntomas.  De  importancia  potencialmente  
mayor  durante  el  examen  ecográfico  dúplex  de  vigilancia  es  la  evaluación  de  la  arteria  carótida  contralateral.  Hasta  
una  cuarta  parte  de  los  pacientes  que  se  someten  a  CEA  demostrarán  progresión  de  enfermedad  no  
hemodinámicamente  significativa  a  enfermedad  aterosclerótica  significativa  en  la  arteria  carótida  interna  contralateral  
en  los  próximos  10  años.

CONTROVERSIAS

20.  ¿Se  puede  realizar  una  endarterectomía  carotídea  basándose  únicamente  en  el  estudio  dúplex?
El  argumento  a  favor  de  la  eliminación  de  la  arteriografía  en  casos  seleccionados  es  persuasivo  porque  la  
arteriografía  carotídea  sola  tiene  una  tasa  de  morbilidad  >1%.  Esta  tasa  puede  representar  el  25%  de  la  morbilidad  
total  habitual  asociada  con  CDEA.  Sin  embargo,  para  darse  cuenta  del  beneficio  de  la  cirugía  basada  en  ecografía  
dúplex,  el  estudio  dúplex  debe  tener  un  valor  predictivo  positivo  (VPP)  alto.  Por  fortuna,  el  VPP  es  alto  para  lesiones  
graves  que  cumplen  criterios  adecuadamente  estrictos  (p.  ej.,  velocidades  sistólicas  máximas  >290  cm/seg  y  
velocidades  telediastólicas  >80  cm/seg).

21.  ¿La  ecografía  dúplex  tiene  algún  papel  en  la  evaluación  preoperatoria  de  la  enfermedad  vascular  periférica?

La  arteriografía  de  contraste  (CA)  sigue  siendo  la  modalidad  de  imagen  estándar  de  oro  en  el  estudio  de  
la  isquemia  de  las  extremidades.  Sin  embargo,  la  ecografía  dúplex  está  ganando  popularidad  como  la  modalidad  
preferida  para  obtener  imágenes  de  la  vasculatura  arterial  para  evaluar  la  posible  revascularización.  Duplex  tiene  
varias  ventajas  sobre  la  arteriografía  de  contraste  que  la  hace  atractiva.  Puede  identificar  áreas  de  vasos  engrosados  
o  calcificados  que  aún  son  permeables  y,  por  lo  tanto,  se  identificarían  falsamente  como  un  objetivo  distal  potencial  
en  imágenes  de  contraste  o  ARM.  Las  mediciones  de  flujo  de  volumen  por  dúplex  permiten  una  evaluación  más  
objetiva  de  la  lesión  hemodinámicamente  significativa  en  comparación  con  una  evaluación  subjetiva  por  CA.  Otros  
beneficios  del  dúplex  son  la  capacidad  de  evaluar  la  enfermedad  subyacente  del  vaso  para  determinar  si  una  lesión  
es  una  oclusión  crónica  o  un  émbolo  agudo  con  poca  enfermedad  subyacente,  aneurismas  con  trombo  parcial  y  sin  
dilatación  luminal,  lo  que  parecería  normal  en  CA,  y  la  presencia  de  placas  ulceradas  o  irregulares,  que  serían  
fuente  de  embolización.  La  viabilidad  y  portabilidad  del  dúplex  como  estudio  de  cabecera  permite  un  estudio  más  
rentable  y  eficiente  en  el  tiempo  y,  por  lo  tanto,  reduce  los  costos  y  las  estadías  en  el  hospital.  La  capacidad  de  
evaluar  simultáneamente  el  sistema  venoso  en  busca  de  conductos  potenciales  también  es  atractiva.
Machine Translated by Google
376  CAPÍTULO  73  LABORATORIO  DE  DIAGNÓSTICO  VASCULAR  NO  INVASIVO

22.  ¿Cuáles  son  los  posibles  efectos  adversos  de  la  angiografía  por  resonancia  magnética  en  pacientes  con  insuficiencia  
renal?
La  fibrosis  sistémica  nefrogénica  (NSF),  reconocida  por  primera  vez  en  1997,  es  una  enfermedad  que  afecta  a  
pacientes  con  insuficiencia  renal  o  diálisis.  Se  caracteriza  por  una  dermatosis  similar  a  la  esclerodermia  con  
engrosamiento  de  la  piel  que  da  como  resultado  lesiones  de  placas  induradas.  Comúnmente  involucra  extremidades  
que  causan  contracturas,  pero  también  puede  tener  compromiso  visceral,  lo  que  aumenta  la  morbilidad  y  la  mortalidad.
Aunque  actualmente  se  desconoce  la  patogenia  real,  existen  muchos  factores  de  riesgo  asociados,  incluidos  los  
medios  de  contraste  que  contienen  gadolinio.  Debido  a  que  la  MRA  conlleva  el  riesgo  de  NSF  y  la  CTA  conlleva  el  
riesgo  de  nefropatía  por  medio  de  contraste,  la  ecografía  dúplex  puede  ser  la  modalidad  diagnóstica  de  elección  en  
pacientes  con  insuficiencia  renal.

23.  ¿Cuáles  son  las  desventajas  de  la  ecografía  dúplex?
Duplex  US  tiene  limitaciones.  Los  factores  que  limitan  su  visualización  son  los  vasos  severamente  calcificados,  
el  linfedema  severo,  la  dermatitis,  las  heridas  ulcerosas  o  la  hiperqueratosis  y  el  dolor  en  reposo  o  los  pacientes  
no  adherentes.  Los  estados  de  flujo  extremadamente  bajo  con  velocidad  sistólica  máxima  (PSV)  <20  cm/s  pueden  
hacer  que  las  interpretaciones  dúplex  no  sean  confiables,  como  es  el  caso  de  la  enfermedad  de  la  arteria  carótida  
interna  (ICA)  críticamente  estenótica  que  puede  interpretarse  incorrectamente  como  ocluida.

24.  ¿Deberían  ser  necesarios  análisis  de  sangre  con  dímero  D  antes  de  que  los  pacientes  sean  evaluados  por  
ultrasonido  para  TVP?
El  dímero  D  es  un  producto  de  degradación  de  la  fibrina  reticulada.  Los  niveles  de  plasma  sanguíneo  de  
dímero  D  a  menudo  están  elevados  en  pacientes  con  TVP.  Sin  embargo,  la  TVP  no  es  la  única  causa  de  dímeros  D  
elevados  y  no  se  puede  usar  en  lugar  de  la  ecografía  para  diagnosticar  la  presencia  de  TVP.  Por  el  contrario,  en  
subgrupos  de  pacientes  seleccionados,  un  nivel  bajo  de  dímero  D  tiene  un  alto  valor  predictivo  negativo  y  puede  evitar  
pruebas  ecográficas  innecesarias.  La  prueba  debe  restringirse  a  pacientes  no  quirúrgicos,  pacientes  que  no  están  
anticoagulados,  pacientes  ambulatorios  y  pacientes  en  los  que  existe  una  baja  sospecha  clínica  de  TVP,  como  dolor  
en  una  extremidad  sin  inflamación  o  inflamación  bilateral  del  tobillo.

25.  ¿Existe  un  componente  terapéutico  en  el  diagnóstico  vascular  no  invasivo?
¿laboratorio?
La  ecografía  dúplex  ahora  se  utiliza  para  localizar  y  guiar  el  tratamiento  de  muchos  pseudoaneurismas  
iatrogénicos.  Cuando  un  paciente  desarrolla  un  pseudoaneurisma  después  de  la  canulación  arterial  con  un  
cuello  largo  y  estrecho,  se  puede  instituir  el  tratamiento  empleando  compresión  en  el  cuello  usando  la  sonda  de  
ultrasonido.  Si  el  flujo  puede  obstruirse  con  la  sonda  (lo  que  generalmente  requiere  de  20  a  30  minutos  de  presión),  
el  pseudoaneurisma  a  menudo  se  trombosará.  Además,  la  ecografía  se  puede  utilizar  para  guiar  la  inyección  de  
trombina  en  los  propios  pseudoaneurismas.  La  inyección  se  realiza  solo  en  pseudoaneurismas  con  cuellos  largos  y  
estrechos.  La  canulación  del  aneurisma  con  aguja  se  confirma  con  ecografía.  Colorflow  se  aplica  al  aneurisma  durante  
la  inyección  de  trombina  para  confirmar  la  trombosis.  Este  método,  aunque  conlleva  un  pequeño  riesgo  de  embolización  
distal,  elimina  la  incomodidad  para  el  paciente  de  la  compresión  ultrasónica  extendida.

26.  ¿Cuál  es  el  papel  de  la  ecografía  dúplex  en  el  tratamiento  y  manejo  de
aneurismas  de  aorta  abdominal  (AAA)?
La  ecografía  convencional  es  bastante  buena  para  identificar  el  aneurisma  de  la  aorta  abdominal  (AAA),  pero  
falla  al  intentar  determinar  el  tamaño  del  aneurisma.  Esto  es  más  significativo  en  pacientes  grandes  o  aneurismas  
que  contienen  una  gran  cantidad  de  trombos.  La  tomografía  computarizada  (TC)  sigue  siendo  la  prueba  de  elección  
para  la  delimitación  anatómica  del  AAA,  especialmente  cuando  se  evalúa  la  idoneidad  de  la  reparación  endovascular  
del  AAA  (EVAR).  La  ecografía  dúplex  puede  ser  valiosa  para  determinar  el  éxito  de  la  reparación  durante  la  vigilancia  
después  de  la  REVA.  Aunque  esta  aplicación  sigue  siendo  controvertida,  la  ecografía  dúplex  posterior  a  la  REVA  
puede  identificar  fugas  y  evaluar  el  crecimiento  del  saco  del  aneurisma  mientras  evita  la  carga  de  contraste  propia  de  
la  vigilancia  por  tomografía  computarizada  posterior  a  la  REVA.
Machine Translated by Google
CAPÍTULO  73  LABORATORIO  DE  DIAGNÓSTICO  VASCULAR  NO  INVASIVO  377

PUNTOS  CLAVE:  DIAGNÓSTICO  VASCULAR  NO  INVASIVO
LABORATORIO
1.  La  ecografía  dúplex  tiene  una  sensibilidad  del  97  %  para  detectar  la  enfermedad  de  la  arteria  carótida  y  una  precisión  del  95  
%  para  clasificar  correctamente  las  estenosis  carotídeas  como  una  reducción  del  diámetro  superior  al  50  %.

2.  La  prueba  principal  para  el  diagnóstico  de  isquemia  de  las  extremidades  inferiores  es  el  ABI.

3.  La  prueba  no  invasiva  utilizada  para  diagnosticar  la  TVP  aguda  es  la  ecografía  dúplex.

4.  La  vigilancia  ecográfica  dúplex  de  la  revascularización  de  las  extremidades  inferiores  y  la  CEA  se  ha  convertido  en  un
componente  del  VDL  no  invasivo.

5.  El  papel  de  la  ecografía  dúplex  se  está  expandiendo  rápidamente  junto  con  la  expansión  de  las  terapias  vasculares  
mínimamente  invasivas.  Este  rol  incluye  no  solo  el  diagnóstico,  sino  también  la  vigilancia  y  la  terapia.

SITIO  WEB
www.acssurgery.com

BIBLIOGRAFÍA

1.  Ascher  E,  Marks  NA,  Hingorani  AP  et  al.:  Tratamiento  endovascular  guiado  por  dúplex  para  lesiones  oclusivas  y  estenóticas  del  
segmento  arterial  femoropoplíteo:  un  estudio  comparativo  en  los  primeros  253  casos.  J  Vasc  Surg  44:1230­1237,  2006.

2.  Baker  WF:  Diagnóstico  de  trombosis  venosa  profunda  y  embolismo  pulmonar.  Med  Clin  North  Am  82:459­476,  1998.

3.  Ballotta  E,  Da  Giau,  Meneghetti  G  et  al.:  Progresión  de  la  aterosclerosis  en  arterias  carótidas  asintomáticas  después
endarterectomía  contralateral:  un  estudio  prospectivo  de  10  años.  J  Vasc  Surg  45:516­522,  2007.

4.  Carter  A,  Murphy  MO,  Halka  AT  et  al .:  La  historia  natural  de  las  estenosis  en  los  injertos  de  derivación  arterial  de  las  extremidades  
inferiores  mediante  un  programa  de  vigilancia  del  injerto.  Ann  Vasc  Surg  21:695­703,  2007.

5.  Clorius  S,  Technau  K,  Watter  T  et  al.:  Fibrosis  sistémica  nefrogénica  tras  la  exposición  al  gadolinio
que  contiene  agente  de  contraste.  Clin  Nephrol  68:249­252,  2007.

6.  Gerlock  AJ,  Giyanani  VL,  Krebs  C:  Aplicaciones  de  técnicas  vasculares  no  invasivas,  Filadelfia,  1988,  WB
Saunders.

7.  Hanson  JM,  Atri  M,  Power  N:  Seudoaneurisma  inguinal  por  inyección  de  trombina  guiada  por  ecografía:  características  del  Doppler  y  
consejos  técnicos.  Br  J  Radiología  81:154­163,  2008.

8.  Hingorani  AP,  Ascher  E,  Marks  N:  arteriografía  dúplex  para  la  revascularización  de  las  extremidades  inferiores.  Perspectiva  Vasca
Surg  Endovasc  Ther  19:6­20,  2007.

9.  Luedde  M,  Krumsdorf  U,  Zehelein  J  et  al.:  Tratamiento  del  pseudoaneurisma  femoral  iatrogénico  por  ultrasonido
terapia  de  compresión  guiada  e  inyección  de  trombina.  Angiología  58:435­439,  2007.

10.  Moneta  GL,  Edwards  JM,  Papanicolaou  G  et  al .:  Detección  de  estenosis  de  la  arteria  carótida  interna  asintomática:
criterios  dúplex  para  discriminar  entre  el  60%  y  el  99%  de  estenosis.  J  Vasc  Surg  21:989­997,  1995.

11.  Mueller­Hulsbeck  S,  Orden  BM,  Jahnke  T:  Intervenciones  en  injertos  de  derivación  infrainguinal.  Cardiovasc  Intervent  Radiol  29:17­28,  
2006.

12.  Olsen  DM,  Rodriguez  JA,  Vranic  M  et  al .:  Un  estudio  prospectivo  de  la  inyección  de  trombina  guiada  por  ecografía  del  pseudoaneurisma  
femoral:  una  tendencia  hacia  la  medicación  mínima.  J  Vasc  Surg  36:779­782,  2002.

13.  Roth  SM,  Bandyk  DF:  imágenes  dúplex  de  derivaciones,  angioplastias  y  stents  de  las  extremidades  inferiores.  Cirugía  Semin  Vasc
12:275­284,  1999.
Machine Translated by Google
378  CAPÍTULO  73  LABORATORIO  DE  DIAGNÓSTICO  VASCULAR  NO  INVASIVO

14.  Sato  DT,  Goff  CD,  Gregory  RT  et  al .:  Endofuga  después  de  la  reparación  del  injerto  de  stent  aórtico:  diagnóstico  
mediante  ecografía  dúplex  a  color  versus  tomografía  computarizada.  J  Vasc  Surg  28:657­663,  1998.

15.  Shirit  D,  Levi  H,  Huerta  M  et  al .:  Indicaciones  apropiadas  para  la  exploración  dúplex  venosa  basada  en  el  ensayo  de  dímero  D.
Ann  Vasc  Surg  16:304­308,  2002.

16.  Wolf  YG,  Johnson  BL,  Hill  BB  et  al.:  Ecografía  dúplex  versus  tomografía  computarizada
angiografía  para  la  evaluación  postoperatoria  de  la  reparación  endovascular  de  aneurisma  aórtico  abdominal.  J  Vasc  
Surg  32:1142­1148,  2000.

17.  Zierler  RE,  Sumner  DS:  evaluación  fisiológica  de  la  enfermedad  oclusiva  arterial  periférica.  En  Rutherford  RB,  editor:  
Cirugía  vascular,  5ª  ed.,  Filadelfia,  2000,  WB  Saunders.
Machine Translated by Google

VIII.  CIRUGÍA  CARDIOTORÁCICA

ARTERIOPATÍA  CORONARIA
CAPITULO  
74

Joseph  C.  Cleveland,  Jr.,  MD

1.  ¿Qué  es  la  angina  y  qué  la  causa?
La  angina  de  pecho  refleja  isquemia  miocárdica.  Los  pacientes  a  menudo  describen  la  sensación  como  presión,  asfixia  
o  tirantez.  La  angina  se  produce  típicamente  por  un  desequilibrio  entre  el  suministro  de  oxígeno  al  miocardio  y  la  
demanda  de  oxígeno  al  miocardio.  La  presentación  clásica  es  la  de  un  hombre  (proporción  hombre/mujer  de  4:1)  que  sale  
a  palear  nieve  en  una  noche  fría  después  de  una  gran  comida  después  de  haber  tenido  una  pelea  con  su  esposa.

2.  ¿Cómo  se  trata  la  angina?
Las  opciones  de  tratamiento  para  la  angina  incluyen  terapia  médica  o  revascularización  miocárdica.
El  tratamiento  médico  está  dirigido  a  disminuir  la  demanda  de  oxígeno  del  miocardio.  Las  estrategias  incluyen  nitratos  
(nitroglicerina,  isosorbida),  que  dilatan  mínimamente  las  arterias  coronarias  pero  también  disminuyen  la  presión  arterial  
(poscarga)  y,  por  lo  tanto,  la  demanda  miocárdica  de  oxígeno;  antagonistas  de  los  receptores  b,  que  disminuyen  la  
frecuencia  cardíaca,  la  contractilidad  y  la  poscarga;  y  antagonistas  de  los  canales  de  calcio,  que  disminuyen  la  poscarga  y  
pueden  prevenir  la  vasoconstricción  coronaria.  La  aspirina  (terapia  antiplaquetaria)  también  es  importante.  Los  agentes  
antiplaquetarios  más  nuevos  como  el  clopidogrel  (Plavix)  y  la  eptifibatida  (Integrilin)  se  promueven  en  el  tratamiento  de  los  
síndromes  coronarios  agudos.  Plavix,  sin  embargo,  es  un  agente  potente  y  eficaz,  y  la  operación  (es  decir,  injerto  de  
derivación  de  la  arteria  coronaria  [CABG])  dentro  de  la  primera  semana  de  exposición  a  Plavix  aumenta  el  riesgo  de  
sangrado  posoperatorio  tres  veces.
Si  la  terapia  médica  no  logra  aliviar  la  angina,  puede  ser  apropiada  la  revascularización  miocárdica  con  intervención  
coronaria  percutánea  (PCI),  con  o  sin  colocación  de  un  stent,  o  CABG.

3.  ¿Cuáles  son  las  indicaciones  para  el  injerto  de  derivación  de  arteria  coronaria?
1.  Estenosis  de  la  arteria  coronaria  principal  izquierda:  estenosis  >  50  %  que  afecta  a  la  arteria  coronaria  principal  izquierda
arterial  es  un  predictor  sólido  de  mal  resultado  a  largo  plazo  en  pacientes  que  reciben  tratamiento  médico.
Una  porción  sustancial  del  miocardio  es  irrigada  por  esta  arteria.  Incluso  si  el  paciente  está  asintomático,  la  
supervivencia  mejora  notablemente  con  CABG.  La  enfermedad  de  la  coronaria  principal  izquierda  es  una  
indicación  de  clase  I  para  CABG  de  acuerdo  con  las  pautas  para  cirugía  de  CABG  de  la  American  Heart  
Association/American  College  of  Cardiology  (AHA/ACC).
2.  Enfermedad  de  la  arteria  coronaria  de  tres  vasos  (estenosis  del  70  %)  con  depresión  del  ventrículo  izquierdo
(VI)  (es  decir,  <0,50)  o  enfermedad  de  la  arteria  coronaria  (CAD)  de  dos  vasos  con  afectación  de  la  descendente  
anterior  izquierda  proximal  (LAD):  En  ensayos  aleatorizados,  los  pacientes  con  enfermedad  de  tres  vasos  y  función  
del  VI  deprimida  mostraron  un  beneficio  de  supervivencia  con  CABG  en  comparación  con  terapia  médica.

3.  La  CABG  también  confiere  un  beneficio  de  supervivencia  en  pacientes  con  CAD  de  dos  vasos  con  una  estenosis  
estrecha  de  la  LAD  proximal  y  una  fracción  de  eyección  (EF)  <0,50  o  isquemia  demostrable  en  pruebas  no  
invasivas.  Sin  embargo,  una  advertencia  importante  en  el  manejo  de  pacientes  con  función  ventricular  izquierda  
deprimida  es  que  la  mortalidad  operatoria  aumenta  cuando  la  FE  cae  por  debajo  del  30%.
4.  Angina  a  pesar  de  un  tratamiento  médico  agresivo:  pacientes  con  limitaciones  en  el  estilo  de  vida
debido  a  CAD  son  candidatos  apropiados  para  CABG,  siempre  que  la  cirugía  se  pueda  realizar  con  un  riesgo  
aceptable.  Los  datos  del  Estudio  de  Cirugía  de  la  Arteria  Coronaria  (CASS)  sugieren  que  los  pacientes  tratados  con  
cirugía  tienen  menos  angina,  menos  limitaciones  de  actividad  y  un  aumento  objetivo  en  la  tolerancia  al  ejercicio  en  
comparación  con  los  pacientes  tratados  médicamente.

379
Machine Translated by Google
380  CAPÍTULO  74  ENFERMEDAD  DE  LAS  ARTERIAS  CORONARIAS

4.  ¿Qué  se  hace  durante  un  procedimiento  CABG  "tradicional"?
CABG  es  un  procedimiento  de  bypass  arterial  que  se  puede  realizar  tanto  con  bypass  como  sin  bypass.  La  arteria  
mamaria  interna  izquierda  (LIMA)  se  extrae  como  un  injerto  pediculado,  con  otros  conductos  que  incluyen  la  vena  
safena  mayor  o  la  arteria  radial  también.  El  bypass  cardiopulmonar  (CPB)  se  establece  canulando  la  aorta  ascendente  
y  la  aurícula  derecha,  y  el  corazón  se  detiene  con  cardioplejía  de  sangre  fría.  Luego  se  invierten  los  segmentos  de  la  
vena  safena  mayor  y  se  cosen  con  la  porción  proximal  (entrada)  del  injerto  de  derivación  que  se  origina  en  la  aorta  
ascendente  y  la  porción  distal  (salida)  del  injerto  de  derivación  anastomosada  a  la  arteria  coronaria  distal  a  la  lesión  
obstructiva.  El  LIMA  normalmente  se  cose  al  LAD.

Cuando  se  terminan  las  anastomosis,  se  retira  al  paciente  de  la  CEC  y  se  cierra  el  tórax.  Por  lo  general,  
se  construyen  de  uno  a  seis  injertos  de  derivación  (de  ahí  los  términos  derivación  triple  o  cuádruple).

5.  ¿Qué  es  una  CABG  sin  bomba  (OPCAB)?
La  CABG  se  puede  realizar  sin  CPB  y  parada  del  corazón.  Cuando  se  realiza  con  el  corazón  latiendo  a  través  de  una  
esternotomía  mediana,  la  CABG  se  denomina  derivación  de  la  arteria  coronaria  sin  circulación  extracorpórea  (OPCAB,  
por  sus  siglas  en  inglés).  El  corazón  se  coloca  con  dispositivos  de  estabilización  comercialmente  disponibles,  y  el  
vaso  que  se  va  a  desviar  se  inmoviliza  y  se  sujeta  para  proporcionar  una  oclusión  temporal.  A  continuación,  se  cose  
el  conducto  venoso  o  arterial  a  la  arteria  coronaria  inmovilizada  y  se  libera  la  oclusión  del  vaso.

6.  ¿Por  qué  uno  elegiría  un  OPCAB  en  lugar  de  un  CABG  tradicional?
CABG  con  CEC  sigue  siendo  el  estándar  de  oro  con  el  85%  de  los  procedimientos  de  CABG  informados  a  la  
base  de  datos  cardíaca  de  adultos  nacional  de  la  Sociedad  de  Cirujanos  Torácicos  que  aún  se  realizan  con  CEC.
Sin  embargo,  la  CPD  se  asocia  con  varias  consecuencias  clínicas  adversas,  como  disfunción  pulmonar  aguda,  
accidente  cerebrovascular,  insuficiencia  renal,  insuficiencia  hepática,  hemorragia  y  la  promoción  de  un  estado  
proinflamatorio.  Se  cree,  aunque  todavía  no  está  bien  delineado,  que  realizar  CABG  sin  CEC  puede  reducir  estas  
complicaciones.  Los  pacientes  con  comorbilidades  de  enfermedad  pulmonar,  enfermedad  cerebrovascular,  enfermedad  
renal  o  enfermedad  vascular  periférica  grave  pueden  tener  mejores  resultados  cuando  la  CABG  se  realiza  sin  el  uso  
de  CEC.  Desafortunadamente,  la  compensación  por  evitar  la  CEC  puede  incluir  una  permeabilidad  del  injerto  
comprometida,  ya  que  la  mayoría  de  los  informes  que  promueven  OPCAB  no  incluyen  datos  de  permeabilidad  del  
injerto,  y  los  primeros  informes  de  OPCAB  describieron  más  oclusiones  tempranas  del  injerto  con  esta  técnica.

7.  ¿La  CABG  mejora  la  función  miocárdica?
Sí.  El  miocardio  en  hibernación  mejora  con  CABG.  La  hibernación  miocárdica  se  refiere  a  la  función  contráctil  
miocárdica  reversible  asociada  con  una  disminución  del  flujo  coronario  en  el  contexto  de  viabilidad  miocárdica  
preservada.  Algunos  pacientes  con  disfunción  sistólica  global  exhiben  una  mejoría  dramática  en  la  contractilidad  
miocárdica  después  de  CABG.

8.  ¿Es  útil  la  CABG  en  pacientes  con  insuficiencia  cardíaca  congestiva?
Posiblemente.  La  CABG  mejora  los  síntomas  de  CHF  que  están  relacionados  con  la  disfunción  
miocárdica  isquémica.  Por  el  contrario,  si  la  insuficiencia  cardiaca  es  secundaria  a  un  músculo  infartado  
irreversiblemente  desde  hace  mucho  tiempo  (es  decir,  una  cicatriz),  la  CABG  no  resulta  beneficiosa.  La  evaluación  
preoperatoria  crítica  debe  evaluar  la  viabilidad  del  miocardio  disfuncional.  Una  gammagrafía  con  talio  de  
redistribución  en  reposo  puede  determinar  los  segmentos  de  miocardio  que  aún  son  viables;  sin  embargo,  la  
resonancia  magnética  (RM)  cardíaca  está  reemplazando  a  las  imágenes  con  radionúclidos  como  una  mejor  prueba  
para  detectar  el  miocardio  en  hibernación.

9.  ¿La  CABG  es  valiosa  para  prevenir  las  arritmias  ventriculares?
No.  La  mayoría  de  las  arritmias  ventriculares  en  pacientes  con  CAD  se  originan  en  el  borde  del  miocardio  irritable  
que  rodea  el  músculo  infartado.  La  implantación  de  un  desfibrilador  cardíaco  automático  implantable  (AICD)  está  
indicada  para  pacientes  con  taquiarritmias  ventriculares  potencialmente  mortales.
Machine Translated by Google
CAPÍTULO  74  ENFERMEDAD  DE  LAS  ARTERIAS  CORONARIAS  381

10.  ¿Cuál  es  la  diferencia  entre  PCI  y  CABG?

Varios  ensayos  clínicos  aleatorizados  y  controlados  han  comparado  la  PCI  con  la  CABG.  Aunque  colectivamente  
analizaron  datos  de  varios  miles  de  pacientes,  la  gran  mayoría  (>80  %)  de  los  pacientes  que  originalmente  cumplieron  con  
los  criterios  de  inclusión  fueron  excluidos  de  la  participación  por  una  variedad  de  razones.  Es  importante  entender  que  
aunque  los  ensayos  controlados  aleatorios  son  el  estándar  de  oro  para  la  comparación  entre  dos  terapias,  una  crítica  
significativa  de  estos  ensayos  de  CABG  versus  PCI  incluye  el  riesgo  relativamente  bajo  de  las  poblaciones  estudiadas,  que  
puede  no  ser  un  reflejo  de  los  pacientes  del  mundo  real  que  se  someten  a  ya  sea  CABG  o  PCI  (ver  Controversias).

Varias  características  importantes  surgieron  de  estos  ensayos.  Mortalidad  global  y  eventos  cardíacos  adversos
(infarto  de  miocardio  [MI])  las  tasas  no  fueron  diferentes  para  CABG  e  PCI.  Un  estudio,  el  estudio  de  Investigación  de  
Revascularización  de  Angioplastia  de  Bypass  (BARI)  mostró  una  mayor  supervivencia  clínicamente  relevante  en  diabéticos  
tipo  2  sometidos  a  CAB  que  aquellos  diabéticos  que  tenían  PCI.  Esta  supervivencia  diferencial  persistió  durante  10  años  de  
seguimiento.
La  principal  diferencia  entre  las  dos  estrategias  de  tratamiento  fue  la  ausencia  de  angina  y
reintervención  En  general,  el  40  %  de  los  pacientes  tratados  con  angioplastia  coronaria  transluminal  percutánea  
(PTCA)  requirieron  una  nueva  PTCA  o  CABG,  mientras  que  aproximadamente  el  5  %  de  los  pacientes  tratados  con  CABG  
requirieron  una  nueva  intervención.  Los  pacientes  que  se  sometieron  a  CABG  también  experimentaron  menos  episodios  de  
angina  en  comparación  con  los  pacientes  tratados  con  PTCA.
Los  ensayos  más  recientes  que  comparan  los  stents  liberadores  de  fármacos  (DES)  versus  CABG  establecen  una  tasa  
mucho  más  baja  de  reestenosis  con  DES,  aproximadamente  del  8%  al  10%.  Sin  embargo,  los  DES  se  han  asociado  con  
trombosis  catastrófica,  que  ocurre  repentinamente  y  sin  signos  clínicos  previos,  meses  o  incluso  años  después  de  su  
colocación.  Es  mucho  más  probable  que  la  trombosis  de  estos  stents  cause  muerte  súbita  o  un  infarto  de  miocardio  
considerable  que  la  reestenosis  que  se  observa  con  los  stents  metálicos  desnudos  (BMS).
Debido  a  esta  propensión  a  la  trombosis  con  DES,  los  pacientes  están  comprometidos  con  la  terapia  antiplaquetaria  
doble  (tanto  ácido  acetildisílico  [AAS]  como  clopidogrel)  durante  un  mínimo  de  1  año,  y  muchos  pacientes  con  DES  ahora  
continúan  con  la  terapia  antiplaquetaria  doble  indefinidamente.

La  conclusión  ineludible  es  que  la  recomendación  de  PCI  o  CABG  debe  ser  individualizada  para  cada  paciente.  

Las  dos  terapias  no  deben  verse  como  excluyentes  o  competitivas;  algunos  pacientes  pueden  beneficiarse  de  una  
combinación  de  PCI  y  CABG.  CABG  da  como  resultado  una  revascularización  más  duradera,  aunque  con  el  riesgo  inherente  
de  complicaciones  perioperatorias.

PUNTOS  CLAVE:  ENFERMEDAD  DE  LAS  ARTERIAS  CORONARIAS

1.  La  CABG  mejora  el  miocardio  en  hibernación.

2.  La  CABG  no  es  útil  para  prevenir  las  arritmias  ventriculares.

3.  La  regla  general  para  la  permeabilidad  de  los  vasos  es  una  permeabilidad  del  90  %  a  los  10  años  para  el  injerto  mamario  
interno,  una  permeabilidad  del  50  %  a  los  10  años  para  los  injertos  de  vena  safena  y  una  permeabilidad  del  80  %  a  1  
año  para  la  ICP  de  un  vaso  estenótico  con  BMS.

11.  ¿Cuál  es  la  regla  general  para  la  permeabilidad  de  los  vasos?
Injerto  mamario  interno:  90  %  de  permeabilidad  a  los  10  años  Injerto  de  vena  
safena:  50  %  de  permeabilidad  a  los  10  años  PCI  c%  
on  BMS  
de   de  vaso  estenótico:  
permeabilidad  a  1  año 80  

12.  ¿Qué  problemas  operativos  y  técnicos  están  asociados  con  CABG?
Las  complicaciones  quirúrgicas  incluyen,  en  términos  generales,  problemas  técnicos  con  la  anastomosis  del  injerto  de  
derivación,  complicaciones  del  esternón  y  complicaciones  de  la  incisión  asociadas  con  la  incisión  de  extracción  de  la  vena  
safena.  Los  problemas  técnicos  con  la  anastomosis  de  la  arteria  coronaria  generalmente  conducen  a  un  IM.
Machine Translated by Google
382  CAPÍTULO  74  ENFERMEDAD  DE  LAS  ARTERIAS  CORONARIAS

Las  complicaciones  esternales  resultan  previsiblemente  en  sepsis  y  fallo  multiorgánico  (MOF).  Las  incisiones  
para  la  extracción  de  la  vena  safena  también  pueden  provocar  problemas  de  edema,  infección  y  dolor  posoperatorio.

13.  ¿Cuáles  son  los  riesgos  de  la  CABG?  ¿Qué  factores  comórbidos  aumentan  el  riesgo  operatorio  de  CABG?  ¿Por  qué  
las  grandes  bases  de  datos  son  útiles  para  el  reporte  de  datos?
La  estimación  del  riesgo  operatorio  es  un  componente  crítico  del  asesoramiento  a  los  pacientes  antes  
de  la  revascularización  quirúrgica.  La  Sociedad  de  Cirujanos  Torácicos  (STS)  y  la  Administración  de  Veteranos  
han  desarrollado  y  promovido  dos  grandes  bases  de  datos  nacionales.  La  base  de  datos  STS  ahora  incluye  los  
resultados  de  más  de  2  millones  de  pacientes  y  representa  el  mayor  programa  de  mejora  de  calidad  y  resultados  
cardiotorácicos  del  mundo.  Los  factores  que  aumentan  el  riesgo  de  CABG  incluyen  fracción  de  eyección  del  
ventrículo  izquierdo  (FEVI)  deprimida,  cirugía  cardíaca  previa,  prioridad  de  operación  (emergencia  versus  electiva),  
clasificación  de  la  Asociación  del  Corazón  de  Nueva  York,  edad,  enfermedad  vascular  periférica,  enfermedad  
pulmonar  obstructiva  crónica  (EPOC),  e  insuficiencia  cardíaca  descompensada  en  el  momento  de  la  cirugía.  Estas  
comorbilidades  ocupan  un  lugar  destacado  en  el  resultado.  Sencillamente,  los  datos  brutos  de  mortalidad  por  
CABG  pueden  ser  engañosos.  Diferentes  cirujanos  pueden  realizar  operaciones  idénticas  pero  tener  tasas  de  
mortalidad  brutas  diferentes  si  un  cirujano  opera  a  triatletas  jóvenes  con  CAD  y  el  otro  cirujano  opera  a  viejos  
sedentarios  que  fuman  dos  paquetes  de  cigarrillos  por  día.  A  través  de  la  evaluación  de  estos  factores  comórbidos,  
se  puede  determinar  una  representación  más  justa  de  los  resultados  previstos  y  observados.  De  esta  manera,  el  
uso  de  los  resultados  observados  con  los  esperados  con  modelos  ajustados  al  riesgo  representa  una  comparación  
más  honesta  de  las  tasas  de  mortalidad  de  la  CABG.  Dado  que  tanto  el  público  como  los  pagadores  de  la  atención  
médica  exigen  transparencia  en  los  resultados,  la  base  de  datos  STS  sirve  como  modelo  para  que  todas  las  
demás  especialidades  recopilen  y  ajusten  los  datos  según  el  riesgo  para  mejorar  la  calidad.

14.  ¿Qué  pasos  se  toman  si  un  paciente  no  puede  retirarse  de  la  CEC?
De  hecho,  el  cirujano  está  tratando  el  shock.  Como  en  el  shock  hipovolémico  (p.  ej.,  una  bala  que  corta  la  aorta),  
los  principios  básicos  incluyen  lo  siguiente:     Reanimación  con  volumen  hasta  que  se  optimicen  las  presiones  
de  llenado  del  lado  izquierdo  y  del  lado  derecho.
  Cuando  las  presiones  de  llenado  son  adecuadas,  inicio  de  soporte  inotrópico.
  Dar  soporte  inotrópico  a  la  toxicidad  (generalmente  taquiarritmias  ventriculares)  e  insertar  un  balón  de  
contrapulsación  intraaórtico  (IABP).  La  última  extensión  de  la  CEC  incluye  la  colocación  de  un  dispositivo  
de  asistencia  del  VI  o  del  ventrículo  derecho  (o  ambos).  Estos  dispositivos  pueden  apoyar  la  circulación  
mientras  permiten  la  recuperación  del  miocardio.

CONTROVERSIAS

15.  ¿Tiene  alguna  ventaja  la  revascularización  quirúrgica  con  todos  los  conductos  arteriales?
Probablemente,  sin  embargo,  los  datos  son  mucho  menos  sólidos  que  los  datos  que  respaldan  la  clara  ventaja  de  
un  bypass  de  LIMA  a  LAD  sobre  un  injerto  de  vena  a  un  bypass  de  LAD.  La  extensión  lógica  de  la  observación  de  
que  una  arteria  mamaria  interna  tiene  una  permeabilidad  superior  a  la  de  una  vena  safena  ha  despertado  el  interés  
por  la  revascularización  arterial  total.  En  lugar  de  utilizar  las  venas  safenas  como  conductos  de  derivación,  algunos  
cirujanos  también  utilizan  la  arteria  mamaria  interna  derecha,  la  arteria  gastroepiploica  y  la  arteria  radial  como  
conductos  de  derivación  en  lugar  de  venas.  Datos  convincentes  sugieren  un  beneficio  de  supervivencia  y  ausencia  
de  angina  cuando  se  utiliza  la  arteria  LIM  como  conducto.  Los  datos  que  respaldan  la  revascularización  arterial  
total  son  mucho  menos  claros.

16.  ¿Cuáles  son  las  opciones  para  un  paciente  con  angina  continua  que  no  se  considera  apto  para  CABG?

Para  los  pacientes  con  tratamiento  médico  optimizado  que  no  son  candidatos  quirúrgicos  (debido  a  
comorbilidades  prohibitivas  o  objetivos  de  arteria  coronaria  de  mala  calidad  para  el  bypass),  una  alternativa  es  un  
procedimiento  llamado  revascularización  transmiocárdica  (TMR).  TMR  utiliza  un  láser  para  quemar  pequeños
Machine Translated by Google
CAPÍTULO  74  ENFERMEDAD  DE  LAS  ARTERIAS  CORONARIAS  383

orificios  desde  el  endocardio  hasta  el  epicardio.  Aunque  originalmente  se  creía  que  el  láser  llevaba  
sangre  desde  la  red  de  capilares  endocárdicos  al  miocardio,  se  ha  observado  repetidamente  que  los  
canales  creados  con  láser  se  llenan  de  trombos  en  24  horas  y  posteriormente  se  ocluyen.  Por  lo  
tanto,  se  postula  que  la  energía  del  láser  provoca  una  respuesta  inflamatoria  con  un  aumento  
resultante  de  los  factores  angiogénicos  (factor  de  crecimiento  endotelial  vascular,  factor  de  crecimiento  
tumoral  by  factor  de  crecimiento  de  fibroblastos).  Aunque  los  datos  experimentales  prometedores  y  
los  ensayos  clínicos  respaldan  la  TMR  como  terapéutica,  uno  se  pregunta  si  un  efecto  placebo  no  es  
operativo  para  promover  el  alivio  de  la  angina.

17.  ¿Qué  terapia  debo  ofrecer  a  un  varón  de  65  años  con  diabetes  mellitus,
estilo  de  vida  estable  que  limita  la  angina,  la  enfermedad  de  la  arteria  coronaria  de  múltiples  vasos  (sin  
afectación  de  la  descendente  anterior  izquierda  proximal)  y  la  función  ventricular  normal  (fracción  de  
eyección  =  65%)?
Este  tipo  de  paciente  explora  el  debate  y  la  interfaz  entre  tres  opciones:  (1)  terapia  médica  continua,  
(2)  PCI  multivaso  y  (3)  CABG.  Se  puede  hacer  un  argumento  persuasivo  para  cada  terapia,  aunque  
como  cirujanos,  le  ofreceremos  a  este  paciente  una  CABG.  La  clave  para  la  toma  de  decisiones  adecuada  
para  este  paciente  incluye  un  equipo  multidisciplinario  de  médicos,  incluidos  cardiólogos  y  cirujanos  
cardíacos,  para  informar  completamente  al  paciente  sobre  sus  opciones  y  los  beneficios  esperados,  los  
resultados  y  los  problemas  a  largo  plazo  con  cada  línea  de  terapia.  La  decisión  de  someterse  a  una  ICP  
multivaso  mientras  este  paciente  está  en  la  mesa  de  cateterismo  porque  "el  problema  se  puede  solucionar  
en  este  momento  sin  una  gran  operación"  deja  a  este  paciente  sin  una  oportunidad  justa  de  ser  
adecuadamente  asesorado  e  informado  sobre  sus  opciones.  En  la  discusión  con  este  paciente,  resalte  
que  aunque  la  CABG  representa  la  terapia  más  invasiva  para  su  CAD,  le  ofrece  el  tratamiento  a  largo  
plazo  más  duradero  de  su  enfermedad  con  un  pequeño  riesgo  inicial  de  mortalidad  o  morbilidad  y  un  
tiempo  relativamente  corto  (semanas  a  meses)  período  de  recuperación.

SITIO  WEB

www.acssurgery.com

BIBLIOGRAFÍA

1.  Cleveland  JC  Jr,  Shroyer  ALW,  Chen  AY  et  al.:  El  injerto  de  derivación  de  arteria  coronaria  sin  bomba  reduce  el  riesgo
mortalidad  y  morbilidad  ajustadas.  Ann  Thorac  Surg  72:1282­1288,  2001.

2.  Eagle  KA,  Guyton  RA,  Davidoff  R  et  al.:  Actualización  de  la  guía  ACC/AHA  2004  para  el  injerto  de  bypass  de  arteria  coronaria
cirugía:  un  informe  del  Grupo  de  Trabajo  sobre  Directrices  de  Participación  del  Colegio  Americano  de  Cardiología/Asociación  
Americana  del  Corazón  (Comité  para  actualizar  las  Directrices  de  1999  para  la  Cirugía  de  Derivación  de  la  Arteria  Coronaria).  Sitio  
Web  de  la  Asociación  Americana  del  Corazón.  Disponible  en:  www.americanheart.org/presenter.jhtml?identifier=9181.

3.  Gundry  SR,  Romano  MA,  Shattuck  OH  et  al.:  Seguimiento  de  siete  años  de  derivaciones  de  arteria  coronaria  realizadas  con
y  sin  circulación  extracorpórea.  J  Thorac  Cardiovasc  Surg  115:1273­1277,  1998.

4.  Hannan  EL,  Racz  MJ,  Walford  G  et  al .:  Resultados  a  largo  plazo  del  injerto  de  derivación  de  la  arteria  coronaria  versus  la  
implantación  de  stent.  N  Engl  J  Med  352:2174­2183,  2005.

5.  Horvath  KA,  Aranki  SF,  Cohn  LH  et  al.:  Alivio  sostenido  de  la  angina  5  años  después  de  la  revascularización  con  
láser  transmiocárdico  con  láser  de  CO2.  Circulación  104  (suplemento  I):  I81­I84,  2001.

6.  Taggart  DP:  el  injerto  de  derivación  de  la  arteria  coronaria  sigue  siendo  el  mejor  tratamiento  para  la  enfermedad  de  múltiples  vasos  y  del  tronco  principal  izquierdo,  pero
los  pacientes  necesitan  saber.  Ann  Thorac  Surg  82:1966­1975,  2006.

7.  El  seguimiento  final  de  10  años:  resultados  del  ensayo  aleatorio  BARI.  J  Am  Coll  Cardiología  49:1600­1606,  2007.
Machine Translated by Google

ESTENOSIS  MITRAL
CAPITULO  
75

T.  Brett  Reece,  MD  y  David  A.  Fullerton,  MD

1.  ¿Qué  causa  la  estenosis  mitral?
Fiebre  reumática.

2.  ¿A  qué  sexo  le  da  estenosis  mitral  con  más  frecuencia?
Mujeres,  en  una  proporción  de  2:1.

3.  ¿Cuáles  son  los  hallazgos  físicos  de  la  estenosis  mitral?
En  la  auscultación,  un  chasquido  de  apertura  y  un  soplo  diastólico  se  escuchan  mejor  en  el  vértice.

4.  ¿Cómo  se  confirma  el  diagnóstico?
Por  ecocardiografía,  preferentemente  ecocardiografía  transesofágica  (ETE).

5.  ¿Qué  es  la  fórmula  de  Gorlin?
Fórmula  utilizada  para  calcular  el  área  de  una  válvula  cardíaca.  En  términos  simplificados:

Área  de  la  válvula  mitral  =  gasto  cardíaco  (CO)  √gradiente  de  presión  medio  a  través  de  la  válvula

6.  ¿Cuál  es  el  tamaño  normal  de  la  válvula  mitral?
El  área  transversal  normal  es  de  4  a  6  cm2 .
La  estenosis  mitral  leve  es  <2  cm2 .
La  estenosis  mitral  severa  es  <1  cm2 .

7.  ¿Cómo  se  determina  el  área  de  la  válvula  mitral  por  ecocardiograma?
Midiendo  la  velocidad  del  flujo  sanguíneo  a  través  de  la  válvula  (la  velocidad  aumenta  con  la  estenosis)  y  midiendo  
el  tiempo  requerido  para  que  disminuya  la  velocidad  del  flujo  (denominado  semitiempo  de  presión).

Área  de  la  válvula  mitral  =  220  medio  tiempo  de  presión

8.  ¿Cuál  es  la  fisiopatología  de  la  estenosis  mitral?
Es  necesario  aumentar  la  presión  de  la  aurícula  izquierda  para  impulsar  la  sangre  a  través  de  una  válvula  mitral  estenótica  
desde  la  aurícula  izquierda  hacia  el  ventrículo  izquierdo.  El  aumento  de  la  presión  en  la  aurícula  izquierda  se  transmite  de  
forma  retrógrada  a  las  venas  pulmonares  y  los  capilares  pulmonares  y,  en  última  instancia,  a  las  arterias  pulmonares.
Da  al  paciente  una  sensación  de  disnea.  Una  presión  auricular  izquierda  de  aproximadamente  25  mm  Hg  aumenta  
la  presión  capilar  pulmonar  lo  suficiente  como  para  producir  edema  pulmonar.
Ejemplo:  para  mantener  un  llenado  adecuado  del  ventrículo  izquierdo  a  través  de  una  válvula  de  1,5  cm2 ,  se  
requiere  un  gradiente  de  presión  de  20  mm  Hg.  Con  una  presión  diastólica  final  del  ventrículo  izquierdo  normal  de  
5  mm  Hg,  un  gradiente  de  20  mm  Hg  produce  una  presión  en  la  aurícula  izquierda  de  25  mm  Hg.  La  presión  de  la  aurícula  
izquierda  aumenta  aún  más  a  medida  que  aumenta  el  flujo  a  través  de  la  válvula  (aumento  del  GC).  Esta  presión  alta  en  
la  aurícula  izquierda  retrocede  e  inunda  los  pulmones  (edema  pulmonar).

9.  ¿Cuál  es  el  principal  síntoma  de  la  estenosis  mitral?
Disnea  de  esfuerzo  (DOE).

384
Machine Translated by Google
CAPÍTULO  75  ESTENOSIS  MITRAL  385

10.  ¿Qué  condiciones  hemodinámicas  precipitan  los  síntomas  en  pacientes  con  mitral
¿estenosis?

Taquicardia:  debido  a  que  la  sangre  fluye  a  través  de  la  válvula  mitral  durante  la  diástole,  una  diástole  más  corta  
(taquicardia)  significa  menos  tiempo  para  que  la  sangre  se  mueva  a  través  de  la  válvula  mitral  estenótica,  lo  que  
disminuye  el  volumen  sistólico.

Pérdida  del  impulso  auricular:  a  medida  que  aumenta  la  presión  en  la  aurícula  izquierda,  la  aurícula  izquierda  se  estira  más  y

el  impulso  auricular  normalmente  organizado  se  vuelve  caótico  (es  decir,  fibrilación  auricular).  Se  requiere  una  
mayor  presión  para  mover  la  sangre  a  través  de  la  válvula  estenótica.  La  pérdida  de  la  contracción  auricular  
presistólica  puede  disminuir  el  llenado  del  ventrículo  izquierdo  hasta  en  un  30%.

11.  ¿Qué  complicaciones  pueden  resultar  de  la  estenosis  mitral?
1.  Hemoptisis  por  congestión  venosa  pulmonar  grave.
2.  Tromboembolismo  en  pacientes  en  fibrilación  auricular.
3.  Endocarditis.
4.  Hipertensión  pulmonar  e  insuficiencia  cardíaca  derecha.

12.  ¿Por  qué  la  estenosis  mitral  provoca  hipertensión  pulmonar?
  Transmisión  retrógrada  del  aumento  de  la  presión  auricular  izquierda.
  vasoconstricción  pulmonar  refleja  iniciada  por  distensión  de  la  aurícula  izquierda.
  Hipertrofia  de  las  arterias  pulmonares,  que  conduce  a  la  remodelación  de  las  arterias  pulmonares.
vasculatura

13.  ¿Cuál  es  el  tratamiento  médico  de  la  estenosis  mitral?
Los  bloqueadores  beta  disminuyen  la  frecuencia  ventricular  a  alrededor  de  60  latidos  por  minuto .
  La  digoxina  reduce  la  frecuencia  ventricular  (al  disminuir  la  conducción  del  nódulo  auriculoventricular)  en  
pacientes  con  fibrilación  auricular.

y  furosemida  (Lasix)  para  reducir  el  edema  pulmonar.
  Se  usa  warfarina  (Coumadin)  si  el  paciente  tiene  fibrilación  auricular.

14.  ¿Cuál  es  la  historia  natural  de  la  estenosis  mitral?
La  supervivencia  con  estenosis  mitral  moderada  es  de  aproximadamente  el  50%  a  los  10  años.
La  supervivencia  en  pacientes  asintomáticos  es  del  80%  a  los  10  años,  pero  cae  en  picado  al  0%  al  15%  una  
vez  que  se  desarrollan  los  síntomas.  Además,  la  supervivencia  media  en  estos  pacientes  con  hipertensión  pulmonar  
es  <3  años.

15.  ¿Cuáles  son  las  indicaciones  de  la  intervención  mecánica  en  la  estenosis  mitral?

  Pacientes  sintomáticos  con  estenosis  mitral  de  moderada  a  grave  (área  valvular  <1,5  cm2  o  gradiente  
transvalvular  de  5  a  10  mm  Hg).
&  Pacientes  asintomáticos  con  un  área  de  la  válvula  mitral  <1  cm2 , gradiente  transvalvular  de
>10  mmHg,  o  presión  sistólica  de  la  arteria  pulmonar  >50  mmHg.

16.  ¿Cuál  es  el  procedimiento  de  elección  en  la  estenosis  mitral?
Si  el  paciente  tiene  valvas  móviles,  ausencia  de  calcio  en  las  valvas  y  regurgitación  mitral  concurrente  mínima,  entonces  
la  valvuloplastia  con  balón  con  catéter  puede  ser  una  opción.

17.  ¿Qué  pacientes  pueden  ser  apropiados  para  una  valvuloplastia  con  balón?
Una  vez  más,  esta  puede  ser  una  decisión  difícil,  pero  los  pacientes  pueden  ser  candidatos  si  no  tienen  calcificación  
del  anillo  o  de  las  valvas  mitrales,  tienen  poca  o  ninguna  regurgitación  mitral  y  tienen  poca  o  ninguna  fusión  de  las  
cuerdas  tendinosas  mitrales.  El  trombo  auricular  izquierdo  también  es  una  contraindicación  para  el  procedimiento.

18.  ¿Cuáles  son  los  resultados  de  la  valvuloplastia  con  balón?
  Tasa  de  mortalidad  <1%.
  El  éxito  inicial  puede  llegar  al  95%  en  pacientes  debidamente  seleccionados.
Machine Translated by Google
386  CAPÍTULO  75  ESTENOSIS  MITRAL

&  El  área  de  la  válvula  puede  aumentar  a  2  
cm2 .  &  <90%  de  supervivencia  libre  de  eventos  (usted  y  su  paciente  no  quieren  "eventos")  a  los  7  años,  de  nuevo  en  
pacientes  adecuadamente  seleccionados.

19.  ¿Qué  operaciones  se  pueden  hacer  para  la  estenosis  mitral?
  Comisurotomía  mitral:  la  tasa  de  mortalidad  es  <2%  y  la  recurrencia  de  la  estenosis  mitral  es
2%  por  año.  Esta  operación  rara  vez  se  realiza  hoy  en  día.
  Reemplazo  de  válvula  mitral:  la  tasa  de  mortalidad  es  del  6%.

PREGUNTA  EXTRA

20.  ¿Qué  es  el  síndrome  de  Lutembacher?
Estenosis  mitral  asociada  a  comunicación  interauricular.  Esto  da  como  resultado  una  derivación  de  izquierda  a  derecha  y  
sobrecarga  el  ventrículo  derecho.

PUNTOS  CLAVE:  ESTENOSIS  MITRAL

1.  La  estenosis  mitral  es  causada  por  fiebre  reumática.

2.  Los  hallazgos  físicos  incluyen  la  auscultación  de  un  chasquido  de  apertura  y  un  soplo  diastólico,  escuchado
mejor  en  el  vértice.

3.  Los  síntomas  suelen  manifestarse  con  taquicardia  por  ejercicio  o  fibrilación  auricular.

4.  La  valvuloplastia  mitral  con  balón  es  el  procedimiento  de  elección  para  la  estenosis  mitral.  Mitral
la  comisurotomía  y  el  reemplazo  de  la  válvula  mitral  son  las  dos  operaciones  que  se  pueden  realizar  para  la  estenosis  
de  la  válvula  mitral.

SITIO  WEB

www.acssurgery.com

BIBLIOGRAFÍA

1.  Bonow  RO,  Carabello  B,  Chatterjee  K  et  al.:  Directrices  ACC/AHA  2006  para  el  manejo  de  pacientes  con
enfermedad  cardíaca  valvular.  Circulación  114:e84­e231,  2006.

2.  Fawzy  ME,  Shoukri  M,  Al  Buraiki  et  al.:  Diecisiete  años  de  seguimiento  clínico  y  ecocardiográfico  de  mitral
valvuloplastia  con  globo  en  520  pacientes  y  predictores  de  resultados  a  largo  plazo.  J  Heart  Valve  Dis  16:454­460,  2007.

3.  Hammermeister  K,  Sethi  GK,  Henderson  WG  et  al.:  Resultados  15  años  después  del  reemplazo  valvular  con  válvula  mecánica  
versus  bioprotésica:  Informe  final  del  ensayo  aleatorizado  VA.  J  Am  Coll  Cardiol  36:1152­1158,  2000.

4.  Lung  B,  Garbarz  E,  Michand  P  et  al.:  Resultados  tardíos  de  la  comisurotomía  mitral  percutánea  en  una  serie  de  1024  pacientes.  
Análisis  del  deterioro  clínico  tardío:  frecuencia,  hallazgo  anatómico  y  factores  predictivos.  Circulación  99:3270­3278,  1999.

5.  Palacios  IF,  Sanchez  PL,  Harrell  KC  et  al.:  ¿Qué  pacientes  se  benefician  del  balón  mitral  percutáneo?
valvuloplastia?  Variables  prevalvuloplastia  y  posvalvuloplastia  que  predicen  el  resultado  a  largo  plazo.  Circulación  
106:1183­1188,  2002.
Machine Translated by Google

REGURGITACIÓN  MITRAL
CAPITULO  
76

T.  Brett  Reece,  MD  y  David  A.  Fullerton,  MD

1.  Enumerar  las  causas  de  la  insuficiencia  mitral.
Fiebre  reumática
Calcificación  anular  mitral  senil
Endocarditis
Disfunción  del  músculo  papilar  por  isquemia
tendones  rotos
Dilatación  anular  por  dilatación  del  ventrículo  izquierdo

2.  ¿Cuál  es  la  fisiopatología  de  la  insuficiencia  mitral?
El  ventrículo  izquierdo  expulsa  sangre  por  dos  vías:  (1)  anterógrada,  a  través  de  la  válvula  aórtica  o  
(2)  retrógrada,  a  través  de  la  válvula  mitral.  La  cantidad  de  cada  volumen  sistólico  expulsado  retrógradamente  
hacia  la  aurícula  izquierda  es  la  fracción  regurgitante.  Para  compensar  la  fracción  regurgitante,  el  ventrículo  
izquierdo  debe  aumentar  su  volumen  sistólico  total.  Esto  finalmente  produce  una  sobrecarga  de  volumen  
del  ventrículo  izquierdo  y  conduce  a  una  disfunción  ventricular.

3.  ¿Cuáles  son  los  síntomas  de  la  insuficiencia  mitral?
La  disnea  de  esfuerzo  y  la  pérdida  de  tolerancia  al  ejercicio  son  los  síntomas  de  la  insuficiencia  cardíaca.

4.  ¿Qué  determina  la  presión  de  la  aurícula  izquierda  en  la  insuficiencia  mitral?
La  distensibilidad  de  la  aurícula  izquierda.

5.  ¿Por  qué  la  regurgitación  mitral  aguda  provoca  síntomas  graves?
Con  insuficiencia  mitral  aguda,  la  aurícula  izquierda  normal  no  es  compatible.  Por  lo  tanto,  la  presión  de  la  
aurícula  izquierda  aumenta  rápidamente,  inundando  los  pulmones  (es  decir,  insuficiencia  cardíaca  congestiva  [CHF])  y  
provocando  síntomas  graves.  Por  el  contrario,  la  insuficiencia  mitral  crónica  se  asocia  con  dilatación  progresiva  de  la  
aurícula  izquierda.  Con  una  mayor  distensibilidad  de  la  aurícula  izquierda,  es  posible  que  la  presión  de  la  aurícula  
izquierda  no  aumente.

6.  ¿Qué  condiciones  hemodinámicas  exacerban  la  insuficiencia  mitral?
Aumento  de  la  poscarga  del  ventrículo  izquierdo:  el  aumento  de  la  presión  arterial  sistémica  aumenta  
la  impedancia  contra  la  cual  el  ventrículo  izquierdo  debe  bombear  para  expulsar  sangre  anterógrada.  
Por  lo  tanto,  la  fracción  regurgitante  aumenta  (más  sangre  retrocede  a  través  de  la  válvula  mitral).

7.  ¿Qué  es  el  soplo  de  insuficiencia  mitral?
Un  soplo  holosistólico  se  escucha  mejor  en  el  vértice  con  radiación  a  la  axila  izquierda.

8.  ¿Cómo  se  confirma  el  diagnóstico?
Por  ecocardiografía  Doppler  color,  especialmente  ecocardiografía  transesofágica  (ETE;  la  aurícula  izquierda  
se  encuentra  justo  sobre  el  esófago).  El  chorro  regurgitante  puede  visualizarse  y  cuantificarse  con  precisión.  
La  ecocardiografía  también  permite  determinar  la  anomalía  anatómica  del  aparato  de  la  válvula  mitral  
responsable  de  la  regurgitación.  Además,  la  anatomía  definida  define  la  probabilidad  de  reparación  de  la  
válvula.

387
Machine Translated by Google
388  CAPÍTULO  76  REGURGITACIÓN  MITRAL

9.  ¿Cuál  es  el  tratamiento  médico  de  la  insuficiencia  mitral?
  Reducción  de  la  poscarga  con  inhibidores  de  la  enzima  convertidora  de  angiotensina  (ECA).
  Diuréticos  (furosemida)  para  la  precarga  del  ventrículo  izquierdo  inferior.
&  La  digoxina  proporciona  control  de  la  frecuencia  ventricular  para  pacientes  con  fibrilación  auricular.
  La  warfarina  (Coumadin)  se  usa  para  pacientes  con  fibrilación  auricular.

10.  ¿Cuáles  son  las  indicaciones  de  cirugía  en  pacientes  con  insuficiencia  mitral?
  Insuficiencia  mitral  grave,  especialmente  con  rotura  de  cuerdas  tendinosas.
  Síntomas  a  pesar  del  tratamiento  médico.
  Insuficiencia  mitral  progresiva  por  ecocardiografía.
  Deterioro  de  la  función  sistólica  del  ventrículo  izquierdo.  Debido  a  que  la  regurgitación  mitral  reduce  la
impedancia  total  de  la  eyección  del  ventrículo  izquierdo  (gran  parte  de  cada  volumen  sistólico  escapa  a  través  de  la  
"puerta  trasera"  de  la  válvula  mitral  de  baja  resistencia),  la  fracción  de  eyección  del  ventrículo  izquierdo  (FEVI)  debe  ser  
mayor  de  lo  normal  en  presencia  de  insuficiencia  mitral.  Una  FEVI  <55%  en  presencia  de  insuficiencia  mitral  sugiere  
disfunción  ventricular  izquierda.
  La  presión  arterial  pulmonar  aumenta  con  el  ejercicio.

11.  ¿Cómo  se  corrige  la  insuficiencia  mitral?
Reparación  de  válvula  mitral.  La  reparación  de  la  válvula  mitral  es  el  procedimiento  quirúrgico  preferido.  Esto  preserva  el  
aparato  mitral,  manteniendo  la  continuidad  entre  el  músculo  ventricular  izquierdo  y  el  anillo  mitral  a  través  de  las  
cuerdas  tendinosas.  La  pérdida  de  esta  continuidad  por  la  resección  del  aparato  coloca  al  ventrículo  izquierdo  en  una  
desventaja  mecánica  que  con  el  tiempo  conduce  a  la  dilatación  y  disfunción  del  ventrículo  izquierdo.

Reemplazo  de  válvula  mitral.  La  incapacidad  para  reparar  la  válvula  regurgitante  obliga  a  reemplazarla.
Si  es  necesario  el  reemplazo,  se  deben  hacer  esfuerzos  para  preservar  la  valva  posterior  de  la  válvula  mitral.  En  la  
mayoría  de  las  series,  se  requiere  el  reemplazo  de  la  válvula  mitral  en  <30%  de  los  casos.

12.  ¿Por  qué  es  preferible  reparar  en  lugar  de  reemplazar  la  válvula  mitral?
&  Menor  mortalidad  operatoria.
&  Menos  riesgo  de  tromboembolismo.
&  Menos  riesgo  de  endocarditis.

&  Menos  necesidad  (si  la  hay)  de  anticoagulación  crónica.
&  Mejor  función  ventricular  izquierda  a  largo  plazo.
&  Evita  complicaciones  relacionadas  con  la  válvula.

13.  ¿Cómo  se  repara  la  válvula  mitral?
Se  resecan  las  porciones  redundantes  de  la(s)  valva(s)  de  la  válvula,  se  reaproxima  la  valva  y  se  pliega  el  anillo  mitral  y  
se  refuerza  con  un  anillo  de  anuloplastia  protésico.  El  anillo  de  anuloplastia  se  cose  alrededor  del  perímetro  del  anillo  en  el  lado  
auricular  izquierdo  de  la  válvula.  Al  hacerlo,  las  valvas  mitrales  reciben  el  apoyo  de  cuerdas  tendinosas  competentes  y  se  reduce  
la  circunferencia  del  anillo  mitral.  La  competencia  de  la  válvula  reparada  se  evalúa  intraoperatoriamente  mediante  TEE.

14.  ¿Cuál  es  la  mortalidad  operatoria  de  la  reparación  de  la  válvula  mitral  frente  al  reemplazo  de  la  válvula  mitral?

Reparación:  2%;  reposición:  6%.

15.  ¿Qué  tan  duraderas  son  las  reparaciones  de  la  válvula  mitral?
El  riesgo  de  requerir  otra  operación  de  válvula  mitral  es  de  aproximadamente  2%  por  año.

16.  ¿Cuál  es  el  papel  de  la  cirugía  mínimamente  invasiva  en  pacientes  con  insuficiencia  mitral?

Los  enfoques  mínimamente  invasivos  se  pueden  aplicar  a  muchos  pacientes  que  se  someten  a  una  cirugía  de  válvula  mitral.
"Mínimamente  invasivo"  puede  referirse  a  muchas  cosas  que  van  desde  una  incisión  limitada  hasta  un
Machine Translated by Google
CAPÍTULO  76  REGURGITACIÓN  MITRAL  389

esternotomía,  a  una  toracotomía,  o  incluso  con  el  uso  de  robótica.  Todos  estos  enfoques  se  pueden  utilizar  con  reparación  
o  reemplazo.  Las  ventajas  teóricas  de  los  abordajes  mínimamente  invasivos  incluyen  una  posible  disminución  del  sangrado  
posoperatorio  y  del  dolor  posoperatorio.  Finalmente,  los  enfoques  mínimamente  invasivos  se  han  realizado  de  manera  segura  en  
pacientes  con  esternotomía  previa  que  se  sometieron  a  procedimientos  cardíacos  posteriores.  El  abordaje  robótico  puede  ser  
particularmente  útil  para  facilitar  la  reparación  de  la  válvula  mitral  por  degeneración  mixomatosa.

PREGUNTA  EXTRA

17.  ¿Qué  es  el  movimiento  anterior  sistólico  (SAM)  de  la  válvula  mitral?
SAM  es  una  complicación  de  la  reparación  de  la  válvula  mitral.  Después  de  la  reparación  de  la  válvula  mitral,  la  valva  anterior  
de  la  válvula  mitral  puede  hincharse  hacia  el  tracto  de  salida  del  ventrículo  izquierdo  durante  la  sístole,  creando  dos  problemas:  
(1)  obstrucción  dinámica  del  tracto  de  salida  del  ventrículo  izquierdo  y  (2)  regurgitación  mitral  (desplazamiento  anterior  de  las  
causas  de  la  valva  anterior).  que  sea  en  escorzo).  Se  debe  sospechar  SAM  si  el  gasto  cardíaco  (CO)  es  bajo  después  de  la  
reparación  de  la  válvula  mitral  y  se  puede  diagnosticar  mediante  ecocardiografía.  Se  exacerba  por  un  aumento  del  estado  contráctil  
del  miocardio,  por  lo  que  deben  evitarse  los  agentes  inotrópicos.  Los  pacientes  con  SAM  son  tratados  con  agentes  de  carga  de  
volumen  y  bloqueadores  beta.  Si  estas  medidas  fallan,  la  válvula  debe  ser  reemplazada.

PUNTOS  CLAVE:  REGURGITACIÓN  MITRAL
1.  Los  síntomas  son  disnea  de  esfuerzo  y  pérdida  de  tolerancia  al  ejercicio.

2.  El  soplo  de  regurgitación  mitral  es  un  soplo  holosistólico  que  se  escucha  mejor  en  el  vértice  con  radiación  a  la  axila  izquierda.

3.  La  regurgitación  de  la  válvula  mitral  se  corrige  con  reparación  de  la  válvula  mitral  o  reemplazo  de  la  válvula  mitral.

4.  La  reparación  de  la  válvula  mitral  es  preferible  al  reemplazo  debido  a  las  menores  tasas  de  mortalidad  operatoria,  menor  riesgo  
de  tromboembolismo,  menor  riesgo  de  endocarditis,  mejor  función  ventricular  izquierda  a  largo  plazo  y  menor  necesidad  (si  
la  hay)  de  anticoagulación  crónica.

5.  La  reparación  también  evita  complicaciones  relacionadas  con  la  válvula  protésica.

SITIO  WEB
www.acssurgery.com

BIBLIOGRAFÍA

1.  Bonow  RO,  Carabello  B,  Chatterjee  K  et  al.:  Directrices  ACC/AHA  2006  para  el  manejo  de  pacientes  con
enfermedad  cardíaca  valvular.  Circulación  114:e84­e231,  2006.

2.  Carabello  BA:  Las  diez  preguntas  más  frecuentes  sobre  la  insuficiencia  mitral.  CardiolRev  10:321­322,  2002.

3.  Enruquez­Sarano  M,  Nkomo  V,  Mohty  D  et  al.:  Regurgitación  mitral:  historia  natural  en  pacientes  operados  y  no  operados
pacientes  Adv  Cardiol  39:122­129,  2002.
Machine Translated by Google
390  CAPÍTULO  76  REGURGITACIÓN  MITRAL

4.  Galloway  AC,  Grossi  EA,  Bizekis  CS  et  al.:  Técnicas  en  evolución  para  la  reconstrucción  de  la  válvula  mitral.  Ann  
Thorac  Surg  236:288­293,  2002.
5.  Irvine  T,  Li  XK,  Shan  DJ  et  al.:  Evaluación  de  la  regurgitación  mitral.  Corazón  88  (suplemento  4):  iv11­iv19,  2002.
6.  Soltesz  EG,  Cohn  LH:  Cirugía  valvular  mínimamente  invasiva.  Revista  Cardiol  15:109­115,  2007.
Machine Translated by Google

ENFERMEDAD  VALVULAR  AÓRTICA
CAPITULO  
77

Dr.  Cyrus  J.  Parsa

1.  ¿Cuáles  son  las  causas  más  frecuentes  de  estenosis  aórtica?
La  estenosis  aórtica  calcificada  degenerativa  relacionada  con  la  edad  es  la  causa  más  común  en  adultos.  La  
estenosis  aórtica  es  más  común  en  hombres  que  en  mujeres.

2.  ¿Cuál  es  la  anomalía  anatómica  más  frecuente  en  la  estenosis  aórtica?
La  válvula  aórtica  bicúspide  (la  válvula  normal  es  la  tricúspide)  ocurre  en  el  2%  de  la  población  general.  Más  del  
50  %  de  los  pacientes  con  estenosis  aórtica  mayores  de  15  años  tienen  una  válvula  aórtica  bicúspide.

3.  ¿Cuáles  son  los  síntomas  más  comunes  de  la  estenosis  aórtica  en  adultos?  ¿Infantes?
La  mayoría  de  los  pacientes  con  estenosis  aórtica  son  asintomáticos.  En  adultos,  el  desarrollo  de  angina,  
síncope  o  disnea  de  esfuerzo  (insuficiencia  cardíaca  congestiva  [CHF])  presagia  un  mal  pronóstico  a  menos  que  
se  realice  un  reemplazo  valvular.  La  CHF  es  la  presentación  más  común  de  la  estenosis  aórtica  en  los  bebés.

4.  ¿Cuál  es  la  supervivencia  esperada  de  los  pacientes  con  estenosis  aórtica?
Los  pacientes  asintomáticos  con  estenosis  aórtica  tienen  una  esperanza  de  vida  casi  normal.  Una  vez  
sintomático,  la  supervivencia  estimada  si  se  sigue  un  tratamiento  expectante  es  de  2  años  (CHF),  3  años  (síncope)  
y  4  años  (angina).  Después  de  que  aparecen  los  síntomas,  la  mortalidad  compuesta  a  los  3  años  de  los  pacientes  
que  no  se  someten  a  una  cirugía  valvular  es  del  75  %.  Por  lo  tanto,  es  importante  detectar  a  los  pacientes  antes  de  
que  presenten  síntomas.

5.  ¿Cuál  es  la  complicación  más  temida  de  la  estenosis  aórtica?
La  muerte  súbita.

6.  ¿Qué  hallazgos  físicos  sugieren  estenosis  aórtica?
Soplo  sistólico  crescendo­decrescendo  (en  forma  de  diamante),  pulsos  periféricos  disminuidos  o  pulso  
ascendente  retrasado  (llámelo  pulsus  parvus  et  tardus  si  quiere  brillar  en  las  rondas  de  medicina).

7.  ¿Cuáles  son  los  hallazgos  típicos  de  la  estenosis  aórtica  en  las  radiografías  de  tórax  y
electrocardiograma  (ECG)?
Tanto  las  radiografías  de  tórax  como  el  ECG  pueden  mostrar  resultados  normales  incluso  con  estenosis  aórtica  
severa;  por  lo  tanto,  estas  no  son  buenas  pruebas  de  detección.  En  la  radiografía  de  tórax,  se  puede  ver  
calcificación  de  la  válvula  aórtica  y  una  silueta  cardíaca  agrandada.  El  ECG  es  bastante  sensible  para  detectar  la  
hipertrofia  del  ventrículo  izquierdo  (HVI)  y  también  puede  revelar  defectos  de  conducción  como  la  prolongación  del  
intervalo  PR  (esto  ocurre  como  consecuencia  de  la  extensión  de  la  calcificación  valvular  al  tejido  de  conducción  
adyacente).

8.  ¿Cómo  se  confirma  el  diagnóstico  de  estenosis  aórtica?
La  ecocardiografía  con  ultrasonido  Doppler  (EE.  UU.)  tiene  una  precisión  de  casi  el  100  %  en  el  diagnóstico  
de  estenosis  aórtica  hemodinámicamente  significativa.  La  variable  medida  es  en  realidad  la  velocidad  (v)  a  
través  de  la  válvula.  La  velocidad  se  convierte  en  gradiente  usando  la  ecuación  de  Bernoulli  donde  gradiente  
=  4v2 .  Estos  datos  luego  se  usan  para  derivar  o  estimar  el  área  de  la  válvula  aórtica  (AVA).

391
Machine Translated by Google
392  CAPÍTULO  77  ENFERMEDAD  VALVULAR  AÓRTICA

9.  ¿Cuándo  está  indicado  el  cateterismo  cardíaco  en  pacientes  con  estenosis  aórtica  confirmada  por  
ecocardiografía?
Las  recomendaciones  de  clase  I  de  las  directrices  más  recientes  del  grupo  de  trabajo  del  Colegio  Americano  
de  Cardiología/Asociación  Americana  del  Corazón  (ACC/AHA)  indican:  (1)  angiografía  coronaria  para  pacientes  
sometidos  a  reemplazo  de  válvula  aórtica  (AVR)  con  riesgo  de  enfermedad  arterial  coronaria  (CAD)  porque  
aproximadamente  El  50%  de  los  pacientes  con  estenosis  aórtica  tendrán  algún  grado  de  EAC  asociada.  Los  
pacientes  que  requieren  AVR  que  tienen  CAD  tratable  quirúrgicamente  deben  someterse  a  un  injerto  de  derivación  
de  la  arteria  coronaria  (CABG)  en  el  momento  de  la  cirugía  de  la  válvula.  (2)  Cateterismo  cardíaco  para  mediciones  
hemodinámicas  cuando  la  gravedad  de  la  estenosis  aórtica  (EA)  sintomática  no  es  concluyente  en  las  pruebas  no  
invasivas  o  los  hallazgos  clínicos  y  las  pruebas  no  invasivas  no  se  correlacionan,  y  (3)  Se  recomienda  la  angiografía  
coronaria  en  pacientes  con  EA  en  quienes  el  autoinjerto  pulmonar  (procedimiento  de  Ross)  y  los  orígenes  de  ambas  
arterias  coronarias  no  se  delinearon  claramente  mediante  estudios  no  invasivos.

10.  ¿Cuándo  está  indicada  una  operación  de  estenosis  aórtica?
Los  pacientes  asintomáticos  con  estenosis  aórtica  rara  vez  requieren  cirugía;  sin  embargo,  prácticamente  todos  
los  pacientes  con  estenosis  aórtica  sintomática  deben  someterse  a  un  reemplazo  de  la  válvula  aórtica.  Las  pautas  
de  clase  I  de  ACC/AHA  respaldan  la  AVR  para  pacientes  sintomáticos  con  EA  grave,  pacientes  con  EA  grave  que  
se  someten  a  cirugía  cardíaca  por  otras  razones  (p.  ej.,  CAD,  otros  problemas  de  válvulas  o  cirugía  aórtica)  y  
pacientes  con  EA  grave  con  fracción  de  eyección  ( FE)  <50%.
Las  recomendaciones  de  clase  IIa  son  que  se  puede  realizar  un  AVR  para  AS  moderado  en  pacientes  sometidos  
a  cirugía  cardíaca  por  otras  razones  porque  la  historia  natural  de  AS  es  la  de  una  enfermedad  progresiva  que  
probablemente  requerirá  reparación  quirúrgica  dentro  de  los  próximos  años  de  todos  modos.
El  aumento  promedio  del  gradiente  es  de  7  a  10  mm  Hg  por  año;  sin  embargo,  existe  una  variación  significativa  
entre  los  pacientes.

11.  ¿Qué  se  debe  dar  a  todos  los  pacientes  con  EA  sometidos  a  limpieza  electiva?
¿cirugía  contaminada  o  contaminada?
La  profilaxis  antibiótica  está  indicada  en  todos  los  pacientes  con  EA  para  la  prevención  de  la  endocarditis  infecciosa  
y  en  pacientes  con  EA  de  etiología  reumática  para  prevenir  la  fiebre  reumática  recurrente.

12.  ¿Qué  trastorno  hematológico  ocurre  en  pacientes  con  estenosis  aórtica  severa?
La  función  plaquetaria  deteriorada  y  los  niveles  reducidos  del  factor  de  von  Willebrand  (vWf)  ocurren  en  
pacientes  con  EA  grave.  Esta  deficiencia  adquirida  de  vWf  se  asocia  con  epistaxis  o  equimosis  en  el  20%  
de  los  pacientes.

13.  ¿Se  puede  utilizar  la  valvulotomía  aórtica  para  tratar  la  estenosis  aórtica?
Aunque  la  valvulotomía  es  un  paliativo  efectivo  para  los  pacientes  con  estenosis  aórtica  congénita,  rara  vez  es  
curativa.  La  mayoría  de  los  niños  con  la  afección  requerirán  AVR  más  adelante  en  la  vida.  AVR,  no  la  valvulotomía,  
es  el  procedimiento  de  elección  en  adultos.

14.  ¿Qué  es  el  procedimiento  de  Ross?
La  propia  válvula  pulmonar  del  paciente  y  la  arteria  pulmonar  proximal  se  extraen  (autoinjerto)  y  se  utilizan  para  
reemplazar  la  válvula  aórtica  nativa  enferma.  Luego  se  usa  un  aloinjerto  u  homoinjerto  pulmonar  (obtenido  y  
congelado  de  un  cadáver  humano)  para  reconstruir  el  tracto  de  salida  del  ventrículo  derecho.

15.  ¿Qué  tipo  de  prótesis  valvular  se  debe  utilizar  en  niños  que  requieren  reemplazo  de  válvula  aórtica?

En  niños  menores  de  15  años  (y  adultos  jóvenes  entre  15  y  40  años),  se  produce  una  rápida  calcificación  de  las  
válvulas  porcinas  colocadas  en  posición  aórtica.  Por  lo  tanto,  se  deben  usar  válvulas  mecánicas  (o  el  procedimiento  
de  Ross;  consulte  la  pregunta  14).
Machine Translated by Google
CAPÍTULO  77  ENFERMEDAD  VALVULAR  AÓRTICA  393

16.  ¿Qué  tipo  de  prótesis  valvular  se  debe  utilizar  en  adultos  que  requieren  válvula  aórtica?
¿reemplazo?
El  uso  de  una  válvula  mecánica  o  bioprotésica  depende  de  la  edad  del  paciente,  el  riesgo  de  anticoagulación  de  
por  vida  y,  en  última  instancia,  la  preferencia  del  paciente.  Aunque  de  ninguna  manera  unánime,  la  mayoría  de  los  
cirujanos  cardíacos  recomendarían  una  prótesis  mecánica  para  pacientes  menores  de  60  años.  Las  válvulas  aórticas  
mecánicas  brindan  un  excelente  alivio  a  largo  plazo  de  la  estenosis  aórtica  hemodinámicamente  significativa,  pero  
requieren  anticoagulación  de  por  vida.  Las  válvulas  bioprotésicas  en  posición  aórtica  no  requieren  anticoagulación;  
sin  embargo,  el  30%  de  estas  válvulas  presentan  deterioro  estructural  a  los  10  o  15  años.  La  ausencia  de  
reintervención  en  pacientes  con  válvulas  aórticas  bioprotésicas  es  >90  %  a  los  10  años,  pero  <70  %  a  los  15  años.  
Se  recomienda  una  prótesis  de  válvula  mecánica  a  los  pacientes  que  se  someten  a  una  nueva  cirugía  de  válvula  
porque  los  riesgos  de  reoperación  son  sustanciales  para  una  tercera  operación  de  válvula.

17.  ¿Cuáles  son  las  causas  más  frecuentes  de  insuficiencia  aórtica?
Endocarditis  valvular  infecciosa,  disección  aórtica,  enfermedad  del  tejido  conjuntivo  (p.  ej.,  síndromes  de  Marfan  
y  Ehlers­Danlos),  aortitis  (siflítica  o  de  células  gigantes),  iatrogénica  (después  de  una  valvulotomía  aórtica  con  
globo),  prolapso  de  la  cúspide  aórtica  asociado  con  defectos  del  tabique  ventricular  y  prótesis  (mecánica) )  
disfunción  valvular.

18.  ¿Qué  hallazgos  físicos  sugieren  insuficiencia  aórtica?
Un  rápido  ascenso  y  descenso  del  pulso  arterial  (referirse  a  esto  como  un  pulso  de  golpe  de  ariete  o,  mejor  
aún,  un  pulso  de  Corrigan  para  deslumbrar  a  su  jefe  de  medicina  residente).  La  presión  diastólica  puede  ser  baja  en  
casos  severos  con  una  presión  de  pulso  ampliada.  Estos  pacientes  exhibirán  un  soplo  diastólico  a  diferencia  de  los  
pacientes  con  EA  que  presentan  un  soplo  de  eyección  sistólico.

19.  ¿Qué  es  el  pulso  de  Quincke?
Pulsaciones  capilares  secundarias  a  insuficiencia  aórtica  que  pueden  detectarse  transmitiendo  una  luz  a  través  de  la  
yema  del  dedo  del  paciente  o  presionando  un  portaobjetos  de  vidrio  en  su  labio.

20.  ¿Cómo  se  confirma  el  diagnóstico  de  insuficiencia  aórtica?
Al  igual  que  con  la  estenosis  aórtica,  la  ecocardiografía  o  la  ecografía  Doppler  son  las  pruebas  de  elección.  
La  resonancia  magnética  nuclear  (RMN)  cardíaca  es  un  avance  reciente  que  puede  ser  un  complemento  diagnóstico  
útil  en  estos  pacientes.

PUNTOS  CLAVE:  ENFERMEDAD  VALVULAR  AÓRTICA

1.  Las  causas  más  comunes  de  estenosis  aórtica  son  anomalías  congénitas  y  enfermedad  calcificada  
(degenerativa).

2.  La  complicación  más  temida  de  la  estenosis  aórtica  es  la  muerte  súbita.

3.  La  estenosis  aórtica  se  asocia  con  un  soplo  de  eyección  sistólico,  mientras  que  la  insuficiencia  aórtica  es
asociado  con  un  soplo  diastólico.

4.  La  intervención  quirúrgica  está  indicada  para  todos  los  pacientes  sintomáticos  con  estenosis  aórtica  e  insuficiencia  
aórtica.  Las  guías  ACC/AHA  proporcionan  recomendaciones  de  clase  I  para  indicaciones  quirúrgicas  en  ambos  
grupos.

5.  La  cirugía  valvular  mínimamente  invasiva  es  factible  y  bien  utilizada.

6.  Los  pacientes  con  estenosis  aórtica  deben  recibir  profilaxis  antibiótica  antes  de
cirugías

7.  AVR  percutáneo  está  en  su  infancia;  solo  los  pacientes  de  edad  avanzada  con  comorbilidades  significativas  
pueden  inscribirse  en  estudios  en  centros  que  implementan  estas  válvulas.
Machine Translated by Google
394  CAPÍTULO  77  ENFERMEDAD  VALVULAR  AÓRTICA

21.  ¿Cuándo  está  indicada  una  operación  por  insuficiencia  aórtica?
Esto  depende  de  la  causa  de  la  insuficiencia  aórtica  y  de  si  es  aguda  o  crónica.  La  insuficiencia  
aórtica  causada  por  una  disección  de  la  aorta  ascendente  es  una  urgencia  quirúrgica.
La  insuficiencia  aórtica  secundaria  a  endocarditis  infecciosa  puede  requerir  o  no  el  reemplazo  de  la  
válvula  aórtica  (consulte  la  pregunta  22).  Los  pacientes  con  insuficiencia  aórtica  crónica  (leve  a  
moderada)  que  no  progresa  disfrutan  de  una  esperanza  de  vida  casi  normal.  Los  pacientes  con  insuficiencia  
aórtica  severa  requieren  cirugía  valvular  antes  de  que  desarrollen  una  disfunción  ventricular  izquierda  
irreversible.  Las  recomendaciones  de  clase  I  de  ACC/AHA  indican  que  AVR  está  indicado:  para  pacientes  
sintomáticos  graves  independientemente  de  la  función  del  ventrículo  izquierdo  (VI);  para  pacientes  asintomáticos  
con  insuficiencia  aórtica  severa  crónica  con  evidencia  de  disminución  de  la  función  del  VI  (EF  ¼  50%);  para  
pacientes  con  insuficiencia  aórtica  grave  y  crónica  que  se  van  a  someter  a  cirugía  cardíaca  por  otros  motivos  
(ver  pregunta  10).  Las  recomendaciones  de  clase  IIa  indican  que  AVR  está  indicado  para  la  insuficiencia  aórtica  
grave  asintomática,  con  función  del  VI  conservada  (EF  >  50  %)  si  las  dimensiones  del  VI  aumentan  o  aumentan  
de  tamaño  (diámetro  del  VI  al  final  de  la  sístole  ¼  55  mm  o  diámetro  del  VI  al  final  de  la  diastólica  ¼  75  mm) .

22.  ¿Cuáles  son  las  indicaciones  para  el  reemplazo  de  la  válvula  aórtica  en  pacientes  con  enfermedades  infecciosas?
¿endocarditis?
ICC  progresiva,  émbolos  sépticos  recurrentes,  infección  no  controlada  con  antibióticos  (a  menudo  hongos,  
bacilos  gramnegativos  o  Staphylococcus  aureus)  y  prolongación  del  intervalo  PR.  Aunque  cueste  creerlo,  la  
unión  de  las  cúspides  valvulares  aórticas  izquierda  y  no  coronaria  está  inmediatamente  adyacente  al  nódulo  
auriculoventricular  (AV).  Por  lo  tanto,  un  absceso  perivalvular  puede  enlentecer  la  conducción  AV.

23.  ¿Cuál  es  la  mortalidad  operatoria  del  reemplazo  de  válvula  aórtica?
La  mortalidad  a  los  30  días  es  del  4,3%  según  la  base  de  datos  de  la  Sociedad  de  Cirujanos  Torácicos  
(véanse  los  sitios  web  de  referencia).  En  pacientes  de  bajo  riesgo,  la  mortalidad  puede  acercarse  al  1%;  
sin  embargo,  la  mortalidad  aumenta  al  8%  si  AVR  se  combina  con  CABG.  El  reemplazo  valvular  múltiple  y  la  
CABG  aumentan  la  mortalidad  perioperatoria  al  18,8%.

24.  ¿Cuáles  son  las  complicaciones  del  reemplazo  de  la  válvula  aórtica?
  Sangrado  que  requiere  reexploración  (2%).
  Bloqueo  cardíaco  (2%),  de  nuevo,  causado  por  la  proximidad  al  nódulo  AV.
  Accidente  cerebrovascular  (1  %)  causado  por  el  aire  o  el  calcio  que  quedan  en  el  corazón  después  del  cierre  de  la  aortotomía.

  Gasto  cardíaco  bajo  (¼  5%)  en  pacientes  con  insuficiencia  VI  preoperatoria.

25.  ¿Cuáles  son  los  resultados  a  largo  plazo  del  reemplazo  de  la  válvula  aórtica?
Los  pacientes  que  sobreviven  al  período  perioperatorio  inmediato  mejoran  tanto  sintomática  como  
funcionalmente,  y  la  supervivencia  corregida  por  edad  vuelve  a  ser  casi  normal  (75%  a  los  10  años).  El  reemplazo  
de  la  válvula  aórtica  revierte  parcialmente  la  HVI  y  la  dilatación.

26.  ¿Cuáles  son  las  opciones  quirúrgicas  mínimamente  invasivas  para  AVR?
Los  abordajes  en  orden  de  uso  o  popularidad  incluyen:  abordaje  de  hemisternotomía  superior,  toracotomía  
anterior  derecha  a  través  del  segundo  espacio  intermedio,  abordaje  paraesternal  derecho  (complicado  por  
hernia  pulmonar  posoperatoria)  y  esternotomía  transversa  (menos  popular).

27.  ¿Cuáles  son  los  beneficios  potenciales  del  AVR  mínimamente  invasivo?
Los  supuestos  beneficios  de  estos  enfoques  son  una  estética  superior,  disminución  del  dolor  
posoperatorio  y  una  recuperación  posoperatoria  más  rápida  con  una  mortalidad  idéntica  a  la  de  un  
enfoque  de  esternotomía  media  completa.  Un  posible  beneficio  adicional  es  que  estos  enfoques  limitan  la  
disección  de  áreas  innecesarias  del  mediastino,  preservando  el  pericardio  intacto,  lo  que  hace  que  una  posible  
reintervención  sea  técnicamente  menos  desafiante.
Machine Translated by Google
CAPÍTULO  77  ENFERMEDAD  VALVULAR  AÓRTICA  395

28.  ¿Se  puede  utilizar  la  valvulotomía  aórtica  con  globo  para  la  estenosis  aórtica  calcificada  en  adultos?
Inicialmente,  se  esperaba  que  la  BAV  pudiera  reemplazar  la  cirugía  y  brindar  paliación  a  largo  plazo  en  pacientes  mayores  
que  tienen  un  mayor  riesgo  quirúrgico  debido  a  la  disminución  de  la  función  ventricular.  BAV  puede  aumentar  el  AVA  de  
0,6  cm2  a  0,9  cm2  pero  se  asocia  con  una  alta  incidencia  de  reincidencia  en  los  meses  siguientes;  en  otras  palabras,  el  
beneficio  es  de  corta  duración.  Desafortunadamente,  a  este  grupo  le  va  peor  después  de  la  valvulotomía  con  globo;  <50%  
están  vivos  1  año  después  de  BAV.

29.  ¿Cuáles  son  las  indicaciones  de  la  valvulotomía  con  balón?
La  valvulotomía  con  balón  es  eficaz  en  lactantes  y  niños  pequeños  con  estenosis  aórtica  congénita  y  un  anillo  aórtico  
diminuto.  Los  resultados  intermedios  son  similares  a  la  valvulotomía  quirúrgica.  En  adultos,  la  valvulotomía  con  balón  debe  
usarse  principalmente  como  un  puente  para  el  reemplazo  o  trasplante  de  la  válvula  aórtica  en  pacientes  que  están  
gravemente  enfermos.  La  mejora  temporal  de  la  función  ventricular  sugiere  que  el  paciente  se  beneficiará  de  AVR.  La  
valvulotomía  con  balón  también  puede  aliviar  los  síntomas  de  las  mujeres  con  estenosis  aórtica  severa  en  el  segundo  
trimestre  del  embarazo.

30.  ¿Es  factible  el  reemplazo  valvular  aórtico  percutáneo?
Esta  modalidad  está  realmente  en  su  infancia.  Actualmente  se  están  realizando  ensayos  en  los  Estados  Unidos  que  
analizan  esta  modalidad  para  pacientes  con  una  mortalidad  prohibitiva  para  AVR  convencional.  Los  ensayos  actuales  más  
grandes  (REVIVE  y  RECAST)  indican  viabilidad,  pero  se  complican  por  fugas  paravalvulares  y  un  aumento  de  accidentes  
cerebrovasculares.  Las  opciones  incluyen:  valvuloplastia  aórtica  con  globo,  stents  con  válvula,  válvulas  con  globo  
expandible  y  válvulas  autoexpandibles.  Uno  de  los  mayores  obstáculos  con  esta  terapia  son  los  requisitos  de  catéter  de  
gran  calibre  para  el  despliegue  (particularmente  en  pacientes  con  vasos  calcificados,  estenóticos  y  arterioescleróticos).  
Esto  ha  fomentado  los  intentos  de  un  abordaje  transapical  a  través  de  una  pequeña  toracotomía  izquierda.  Este  enfoque  
se  encuentra  actualmente  bajo  protocolo  de  estudio  en  los  Estados  Unidos.

CONTROVERSIAS

31.  ¿Se  debe  realizar  alguna  vez  el  procedimiento  de  Ross?
Para:  El  procedimiento  de  Ross  proporciona  un  excelente  alivio  hemodinámico  a  largo  plazo  (a  veces  de  por  vida)  de  
la  estenosis  aórtica  y  evita  la  necesidad  de  válvulas  mecánicas,  evitando  así  la  necesidad  de  anticoagulación.  Un  
beneficio  adicional  es  la  capacidad  regenerativa  del  autoinjerto  aórtico;  en  realidad  puede  aumentar  de  tamaño  a  
medida  que  crece  el  paciente.
En  contra:  El  procedimiento  de  Ross  es  una  operación  técnicamente  exigente  y  tiene  un  importante
curva  de  aprendizaje  con  alta  morbilidad  asociada.  El  procedimiento  destruye  una  válvula  pulmonar  normal,  lo  que  
potencialmente  le  da  al  paciente  dos  (en  lugar  de  una)  enfermedades  valvulares.

32.  ¿Se  debe  usar  una  válvula  de  tejido  en  adultos  jóvenes  entre  15  y  30  años?
Para:  La  anticoagulación  no  es  necesaria  para  las  válvulas  de  tejido  colocadas  en  posición  aórtica;  así,  se  evita  el  riesgo  
de  complicaciones  hemorrágicas  significativas  en  pacientes  activos.  Para  las  mujeres  en  edad  fértil,  las  ventajas  son  
reales.
En  contra:  la  disfunción  valvular  temprana  secundaria  a  la  calcificación  valvular  ocurre  de  manera  más  agresiva  en  
pacientes  más  jóvenes;  por  lo  tanto,  el  reemplazo  de  la  válvula  puede  ser  necesario  antes  de  los  10  años.

33.  ¿Deberían  realizarse  abordajes  mínimamente  invasivos  para  el  reemplazo  de  la  válvula  aórtica?
¿intentó?
Para:  AVR  se  puede  realizar  a  través  de  una  ministomía.  Este  abordaje  evita  una  esternotomía  completa  y  
puede  mejorar  la  estética  y  disminuir  la  pérdida  de  sangre.
En  contra:  AVR  a  través  de  esternotomía  convencional  es  sorprendentemente  bien  tolerado  y  tiene  excelentes  
resultados  a  largo  plazo.  Los  estudios  comparativos  no  han  identificado  diferencias  en  la  calidad  de  vida  entre  el  
reemplazo  de  válvula  aórtica  convencional  y  mínimamente  invasivo.  Además,  los  resultados  a  largo  plazo  del  
enfoque  mínimamente  invasivo  aún  no  están  disponibles.
Machine Translated by Google
396  CAPÍTULO  77  ENFERMEDAD  VALVULAR  AÓRTICA

SITIOS  WEB

www.acssurgery.com

www.ctsnet.org/

www.sts.org/sections/stsnationaldatabase/riskcalculator/

BIBLIOGRAFÍA

1.  Akins  CW,  Hilgenberg  AD,  Vlahakes  GJ  et  al.:  Resultados  de  la  válvula  aórtica  bioprotésica  versus  mecánica
reemplazo  realizado  con  injerto  de  derivación  de  arteria  coronaria  concomitante.  Ann  Thorac  Surg  74:1098­1106,  2002.

2.  Al­Halees  Z,  Pieters  F,  Qadoura  F  et  al.:  El  procedimiento  de  Ross  es  el  procedimiento  de  elección  para  el  aneurisma  aórtico  congénito
enfermedad  de  las  válvulas  J  Thorac  Cardiovasc  Surg  123:437­441,  2002.

3.  Bonacchi  M,  Prifti  E,  Giunti  G  et  al.:  ¿La  ministranotomía  mejora  el  resultado  posoperatorio  en  la  válvula  aórtica?
¿operación?  Un  estudio  prospectivo  aleatorizado.  Ann  Thorac  Surg  73:460­465,  2002.

4.  Bonow  RO,  Carabello  BA,  Chatterjee  K  et  al.:  Directrices  ACC/AHA  2006  para  el  tratamiento  de  pacientes  con  cardiopatía  valvular:  un  
informe  del  Grupo  de  Trabajo  sobre  Directrices  Prácticas  del  Colegio  Americano  de  Cardiología/Asociación  Americana  del  Corazón  
(Comité  de  Redacción  para  Desarrollar  pautas  para  el  manejo  de  pacientes  con  enfermedad  cardíaca  valvular).  Sitio  web  del  Colegio  
Americano  de  Cardiología.  Disponible  en:  www.acc.org/qualityandscience/clinical/guidelines/valvular/index.pdf .

5.  Borer  JS:  Reemplazo  de  válvula  aórtica  para  el  paciente  asintomático  con  insuficiencia  aórtica:  una  nueva  pieza  del  rompecabezas  
estratégico.  Circulación  106:2637­2639,  2002.

6.  Carrol  JD.  Editorial:  la  evolución  del  tratamiento  de  la  estenosis  aórtica:  ¿los  nuevos  procedimientos  ofrecen  nuevas  opciones  de  tratamiento?
para  los  pacientes  de  mayor  riesgo?  Circulación  114:533­535,  2006.

7.  Chaliki  HP,  Mohty  D,  Avierinos  JF  et  al .:  Resultados  después  del  reemplazo  de  la  válvula  aórtica  en  pacientes  con  insuficiencia  aórtica  severa  
y  función  ventricular  izquierda  marcadamente  reducida.  Circulación  106:2687­2693,  2002.

8.  Detter  C,  Fischlein  T,  Feldmeier  C  et  al .:  Valvulotomía  aórtica  para  la  estenosis  aórtica  valvular  congénita:  una  experiencia  de  37  años.  
Ann  Thorac  Surg  71:1564­1571,  2001.

9.  Lamb  HJ,  Beyerbacht  HP,  de  Roos  A  et  al .:  Remodelación  del  ventrículo  izquierdo  temprano  después  del  reemplazo  de  la  válvula  aórtica:  
efectos  diferenciales  sobre  la  función  diastólica  en  la  estenosis  de  la  válvula  aórtica  y  la  regurgitación  aórtica.  J  Am  Coll  Cardiol  
40:2182­2188,  2002.

10.  Lichtenstein  SV,  Cheung  A,  Ye  J  et  al.:  Implantación  de  válvula  aórtica  transapical  transcatéter  en  humanos:  experiencia  clínica  inicial.  
Circulación  114:591­596,  2006.

11.  Mihaljevic  T,  Cohn  LH,  Unic  D  et  al .:  Mil  operaciones  de  válvulas  mínimamente  invasivas:  resultados  tempranos  y  tardíos.
Ann  Surg  240:529­534,  2004.

12.  McCrindle  BW,  Blackstone  EH,  Williams  WG  et  al.:  ¿Son  equivalentes  los  resultados  de  la  valvulotomía  con  balón  transcatéter  frente  a  
la  quirúrgica  en  la  estenosis  aórtica  crítica  neonatal?  Circulación  104  (suplemento  I):  I152­II58,  2001.

13.  Paparella  D,  David  TE,  Armstrong  S  et  al.:  Resultados  a  medio  plazo  del  procedimiento  de  Ross.  J  Card  Surg  16:338­343,
2001.

14.  Rankin  JS,  Hammill  MS,  Ferguson  TB  et  al .:  Determinantes  de  la  mortalidad  operatoria  en  cirugía  cardíaca  valvular.
J  Thorac  Cardiovasc  Surg  131:547­557,  2006.

15.  Russo  CF,  Mazzetti  S,  Garatti  A  et  al .:  Complicaciones  aórticas  después  del  reemplazo  de  la  válvula  aórtica  bicúspide:  resultados  a  largo  
plazo.  Ann  Thorac  Surg  74:S1773–S1776,2002.

16.  Walther  T,  Simon  P,  Dewey  T  et  al .:  Implantación  de  válvula  aórtica  mínimamente  invasiva  transapical.  Circulación  116:
I­240­I­245,  2007.

17.  Webb  JG,  Chandavimol  M,  Thompson  C  et  al.:  Implantación  percutánea  de  válvula  aórtica  retrógrada  desde  la  arteria  femoral.  Circulación  
113:842­850,  2006.

18.  Webb  JG,  Pasupati  S,  Humphries  K  et  al.:  Reemplazo  transarterial  percutáneo  de  la  válvula  aórtica  en  pacientes  seleccionados
pacientes  de  riesgo  con  estenosis  aórtica.  Circulación  116:755­763,  2007.

19.  Yener  N,  Oktar  GL,  Erer  D  et  al.:  Válvula  aórtica  bicúspide.  Ann  Thorac  Cardiovasc  Surg  8:  264–267.
Machine Translated by Google

CIRUGÍA  TORÁCICA  PARA
ENFERMEDAD  NO  NEOPLÁSICA
CAPITULO  
78

Dr.  Laurence  H.  Brinckerhoff

DERRAME  PLEURAL

1.  ¿Qué  es  un  derrame  pleural?
El  líquido  pleural  se  genera  en  adultos  normales  a  razón  de  5  a  10  L  por  24  horas  en  los  hemitórax  
combinados,  pero  los  adultos  normales  tienen  sólo  20  ml  de  líquido  pleural  presente  en  cualquier  momento.  
Los  derrames  pleurales  se  desarrollan  cuando  hay  una  producción  aumentada  o  una  reabsorción  disminuida.  
Las  condiciones  patológicas  que  conducen  a  derrames  incluyen  aumento  de  la  permeabilidad  capilar  
(inflamación,  tumor),  aumento  de  la  presión  hidrostática  (p.  ej.,  en  insuficiencia  cardíaca  congestiva  [ICC]),  
disminución  del  drenaje  linfático  (tumor,  fibrosis  por  radiación),  disminución  de  la  presión  oncótica  
(hipoalbuminemia)  o  combinaciones  de  estos.

2.  ¿Cómo  se  determina  la  causa  de  un  derrame  pleural?
Se  utilizan  la  anamnesis  y  la  exploración  física,  la  radiografía  de  tórax  (de  pie  y  en  decúbito)  y  la  toracocentesis.  
La  toracocentesis  debe  utilizarse  para  evaluar  el  líquido  pleural.  El  líquido  sanguinolento  es  típico  de  
traumatismo,  embolia  pulmonar  (EP)  o  malignidad.  El  líquido  lechoso  puede  ser  evidencia  de  quilotórax  
(triglicéridos  >110)  y  el  líquido  purulento  evidencia  de  empiema.  Debe  comprobarse  el  recuento  de  células  en  
el  líquido;  citología;  equilibrio  ácido­base  (pH);  tinción  de  Gram;  cultura;  y  nivel  de  glucosa,  proteínas,  lactato  
deshidrogenasa  (LDH),  amilasa  y  triglicéridos.  Los  exudados  tienen  un  cociente  de  proteínas  >0,5  y  un  
cociente  de  LDH  >0,6.  La  causa  más  común  de  trasudado  es  CHF;  la  causa  más  común  de  exudado  es  
malignidad.  Se  observa  glucosa  <60  mg/dl  en  derrames  paraneumónicos,  derrames  reumatoides,  pleuritis  
tuberculosa  y  malignidad.

3.  ¿Cuál  es  el  manejo  de  un  derrame  pleural?
El  tratamiento  para  los  derrames  difiere  según  el  tipo,  trasudativo  o  exudativo.  Se  debe  usar  una  toracocentesis  
o  una  toracostomía  con  tubo  para  evacuar  el  derrame  y  determinar  el  tipo.  Si  el  derrame  es  transudativo,  se  
debe  corregir  el  problema  subyacente  (p.  ej.,  CHF).  Si  el  derrame  es  exudativo,  es  necesario  considerar  la  
intervención  quirúrgica  (p.  ej.,  pleurodesis  o  decorticación).
Una  decorticación  es  la  eliminación  de  un  bovino  infeccioso  de  la  superficie  pulmonar  que  permite  la  
expansión  completa  del  tejido  pulmonar,  llenando  así  un  espacio  pleural  infectado.  Una  pleurodesis  se  usa  
para  tratar  un  derrame  maligno.  Una  pleurodesis  (pegar  las  pleuras  parietal  y  visceral)  se  puede  realizar  con  
esclerosantes  (talco)  o  abrasión  mecánica.  La  sínfisis  pleural  (pleura  atascada)  da  como  resultado  una  
disminución  del  área  de  superficie  para  la  producción,  elimina  el  espacio  pleural  para  la  acumulación  y  
previene  el  colapso  y  la  compresión  pulmonar.  Los  tubos  torácicos  generalmente  se  retiran  cuando  la  
producción  es  <150  ml  por  24  horas.

4.  ¿Qué  indica  un  nivel  hidroaéreo  en  una  radiografía  de  tórax  inicial?
Un  nivel  hidroaéreo  antes  de  cualquier  procedimiento  de  drenaje  puede  representar  una  fístula  broncopleural.
Estas  fístulas  pueden  resolverse  con  un  tubo  de  drenaje  torácico  o  requerir  una  toracotomía  abierta  para  su  
reparación  definitiva.

397
Machine Translated by Google
398  CAPÍTULO  78  CIRUGÍA  TORÁCICA  PARA  ENFERMEDADES  NO  NEOPLÁSICAS

EMPIEMA

5.  ¿Qué  es  un  empiema  y  qué  lo  causa?
Un  empiema  es  un  derrame  purulento  (infectado).  El  líquido  o  la  sangre  en  el  espacio  pleural  pueden  
inocularse  directamente  con  bacterias  durante  una  cirugía  o  un  traumatismo  (33  %)  o  por  contaminación  de  
sitios  contiguos  (50  %),  como  una  infección  broncopulmonar  (la  más  común).  La  mayoría  de  los  empiemas  son  
paraneumónicos  y  los  organismos  más  comúnmente  involucrados  son  Staphylococcus  aureus,  bacilos  
gramnegativos  entéricos  y  anaerobios.  Muchas  veces,  las  infecciones  son  polimicrobianas.
A  menudo  no  hay  crecimiento  de  un  cultivo  de  empiema  debido  a  una  terapia  antibiótica  eficaz  o  técnicas  de  
cultivo  inadecuadas,  particularmente  con  anaerobios.

6.  ¿Cuáles  son  las  tres  etapas  del  desarrollo  del  empiema?
Son  la  etapa  exudativa  (líquido  de  baja  viscosidad),  la  etapa  fibrinopurulenta  (fase  de  transición  con  depósitos  
fibrinosos  pesados  y  líquido  turbio)  y  la  etapa  de  organización  (crecimiento  capilar  hacia  el  interior  con  atrapamiento  
pulmonar  por  colágeno).  Este  proceso  suele  evolucionar  a  lo  largo  de  6  semanas.

7.  ¿Cómo  se  diagnostica  un  empiema?
Se  utilizan  hallazgos  clínicos  y  radiográficos  característicos.  La  tomografía  computarizada  (TC)  es  útil  para  definir  
las  loculaciones.  La  toracocentesis  puede  revelar  pus  franco.  La  tinción  de  Gram  puede  mostrar  muchos  glóbulos  
blancos  (WBC)  y  organismos.  El  análisis  bioquímico  varía,  pero  generalmente  es  un  exudado  con  un  pH  bajo  (<7),  
LDH  alto  (>1000  UI/L)  y  glucosa  baja  (<50  mg/dl).

8.  ¿Cómo  se  debe  tratar  un  empiema?
Terapia  antibiótica  dirigida  por  tinción  de  Gram  y  cultivo.  Si  es  temprano  en  el  proceso  de  la  enfermedad,  la  
toracostomía  con  tubo  puede  ser  curativa.  La  instilación  de  enzimas  fibrinolíticas  (p.  ej.,  estreptocinasa  o  
activador  tisular  del  plasminógeno  [tPA])  puede  ser  útil  si  el  empiema  es  temprano  y  está  loculado.  Un  derrame  
infectado  loculado  (muchas  bolsas  quísticas  discontinuas)  de  menos  de  14  días  debe  someterse  a  decorticación  
mediante  cirugía  toracoscópica  asistida  por  video  (VATS,  por  sus  siglas  en  inglés)  (es  decir,  resección  de  la  piel  
adherente  y  engrosada).  La  probabilidad  de  conversión  a  toracotomía  abierta  aumenta  con  la  edad  del  derrame  o  
empiema.

9.  ¿Qué  es  una  decorticación?
La  corteza  es  la  pared  exterior  o  la  cáscara  del  empiema  (como  una  naranja).  Por  lo  tanto,  la  decorticación  es  la  
liberación  quirúrgica  del  pulmón  y  la  extirpación  de  las  paredes  de  la  cavidad  del  absceso.  La  decorticación  exitosa  
permite  que  el  pulmón  se  expanda  y  llene  todo  el  espacio  pleural;  si  no  se  produce  una  expansión  completa,  entonces  
el  derrame  puede  reaparecer  y  es  probable  que  continúe  el  atrapamiento  pulmonar.  Hay  dos  indicaciones  para  la  
decorticación:  signos  continuos  de  infección  (fiebre,  sepsis  con  glóbulos  blancos  altos)  después  del  drenaje  y  una  
hemorragia  significativa  en  el  pulmón  que  resulta  en  un  pulmón  atrapado.

10.  ¿Cuáles  son  las  complicaciones  de  un  empiema  si  no  se  trata?
La  más  común  es  la  fibrosis  pulmonar  con  atrapamiento  pulmonar  y  disnea  resultante.  Otros  incluyen  
contracción  y  deformidad  de  la  pared  torácica,  drenaje  espontáneo  a  través  de  la  pared  torácica  (empiema  
necessitans),  fístula  broncopleural,  osteomielitis,  pericarditis,  absceso  mediastínico  o  subfrénico,  sepsis  y  muerte.  
Ninguno  de  estos  resultados  es  particularmente  atractivo,  por  lo  que  en  ausencia  de  contraindicaciones  
abrumadoras,  todos  los  empiemas  ameritan  tratamiento.

INFECCIONES  Y  TUBERCULOSIS

11.  ¿Qué  es  un  absceso  pulmonar  y  cómo  se  trata?
Un  absceso  pulmonar  es  un  sitio  localizado  de  infección  ubicado  dentro  del  tejido  pulmonar  con  necrosis  tisular  
asociada.  Hay  muchas  infecciones  pulmonares  potenciales  que  pueden  producir  abscesos  pulmonares,  pero  las  
infecciones  por  anaerobios  siguen  siendo  los  tipos  de  patógenos  más  frecuentes.  A  diferencia  de  los  abscesos  en  otros
Machine Translated by Google
CAPÍTULO  78  CIRUGÍA  TORÁCICA  PARA  ENFERMEDADES  NO  NEOPLÁSICAS  399

del  cuerpo,  la  mayoría  de  los  abscesos  pulmonares  no  requieren  drenaje  y  pueden  tratarse  con  terapia  
antibiótica  sistémica.  La  cirugía  solo  se  considera  cuando  la  terapia  médica  ha  fallado.

12.  ¿Cuáles  son  las  manifestaciones  clínicas  de  la  tuberculosis  pulmonar?
Pueden  ser  casi  cualquier  cosa  o  nada  (se  ha  dicho  que  si  sabes  de  tuberculosis,  sabes  todo  de  medicina),  
pero  los  síntomas  y  signos  más  comunes  son  la  fiebre  crónica;  pérdida  de  peso;  sudores  nocturnos;  y  tos,  a  
veces  con  hemoptisis.  La  radiografía  de  tórax  típicamente  muestra  infiltrados  en  el  lóbulo  superior,  con  o  sin  
cavitación,  y  puede  diagnosticarse  erróneamente  como  un  proceso  neoplásico.
Los  pacientes  que  son  positivos  para  el  virus  de  la  inmunodeficiencia  humana  (VIH)  y  que  están  
inmunocomprometidos  suelen  tener  adenopatía  mediastínica,  derrames  pleurales  y  un  patrón  miliar.

13.  ¿Cómo  se  hace  el  diagnóstico  de  tuberculosis  pulmonar?
Frotis  de  bacilos  acidorresistentes  (BAAR)  positivo  en  muestra  de  esputo;  la  sensibilidad  mejora  con  
muestras  de  lavado  broncoalveolar  (BAL).  El  crecimiento  del  cultivo  identificará  el  organismo  específico  (es  
decir,  atípicos)  y  la  sensibilidad  al  fármaco  (cuidado  con  la  resistencia  a  múltiples  fármacos  [MDR]).

14.  ¿Cuál  es  el  tratamiento  médico  actual  para  la  tuberculosis  activa?
La  terapia  inicial  consiste  en  un  régimen  de  6  meses  con  isoniazida,  rifampicina  y  pirazinamida  durante  los  
primeros  2  meses,  y  luego  isoniazida  y  rifampicina  durante  otros  4  meses.  Con  este  programa,  el  95%  de  los  
pacientes  tienen  esputo  negativo  para  tuberculosis  al  final  de  la  terapia.  Los  pacientes  con  respuesta  parcial  
deben  recibir  terapia  durante  más  de  6  meses,  y  aquellos  con  MDR­TB  pueden  recibir  etambutol  o  
estreptomicina.

15.  ¿Cuáles  son  las  indicaciones  de  cirugía  en  pacientes  con  tuberculosis?
La  cirugía  está  indicada  para  las  complicaciones  de  la  enfermedad.  La  indicación  quirúrgica  más  común  en  
los  Estados  Unidos  es  la  MDR­TB  con  pulmón  destruido  y  enfermedad  cavitaria  persistente.  Este  tejido  
pulmonar  es  resistente  a  la  penetración  de  fármacos  y  también  puede  "derramar"  organismos  al  tejido  
pulmonar  sano.  Otras  indicaciones  incluyen  hemoptisis,  exclusión  de  cáncer  de  pulmón,  estenosis  bronquial,  
fístula  broncopleural,  síndrome  del  lóbulo  medio  o  micobacterias  distintas  del  bacilo  tuberculoso  (MOTT).

16.  ¿Qué  es  MOTT  y  cuál  es  el  papel  de  la  cirugía  con  esta  enfermedad?
Las  infecciones  por  micobacterias  atípicas,  las  infecciones  por  micobacterias  no  tuberculosas  y  las  
infecciones  por  micobacterias  distintas  de  la  tuberculosis  son  sinónimos.  El  más  común  de  estos  
organismos  es  el  complejo  Mycobacterium  avium  (MAC).  Otros  incluyen  M.  chelonae  y  abscesos,  M.  kansaii,  
M.  fortuitum  y  M.  xenopi.  MAC  típicamente  produce  enfermedad  fibrocavitaria  de  los  lóbulos  superiores  o  el  
lóbulo  medio  o  língula  de  mujeres  delgadas  y  blancas.  La  cirugía  está  indicada  para  la  enfermedad  localizada  
y,  en  combinación  con  la  farmacoterapia,  produce  conversión  de  esputo  en  95%  de  los  pacientes  con  tasas  
de  recaída  <5%.  Otras  indicaciones  para  la  cirugía  son  las  mismas  que  para  la  tuberculosis  regular.

PUNTOS  CLAVE:  CIRUGÍA  TORÁCICA  PARA
ENFERMEDAD  NO  NEOPLÁSICA
1.  Un  empiema  es  un  derrame  purulento  (infectado).  El  tratamiento  primario  es  el  drenaje.

2.  Las  tres  etapas  del  empiema  son  la  etapa  exudativa  (líquido  de  baja  viscosidad),  la  etapa  fibrinopurulenta  
(fase  de  transición  con  depósitos  fibrinosos  pesados  y  líquido  turbio)  y  la  etapa  de  organización  
(crecimiento  capilar  interno  con  atrapamiento  pulmonar  por  colágeno).

3.  La  cirugía  está  indicada  para  las  complicaciones  de  la  tuberculosis,  siendo  la  indicación  más  común  en  
los  Estados  Unidos  la  MDR­TB  con  pulmón  destruido  y  enfermedad  cavitaria  persistente.
Machine Translated by Google
400  CAPÍTULO  78  CIRUGÍA  TORÁCICA  PARA  ENFERMEDADES  NO  NEOPLÁSICAS

SITIO  WEB

www.acssurgery.com

BIBLIOGRAFÍA

1.  American  Thoracic  Society:  Diagnóstico  y  tratamiento  de  enfermedades  causadas  por  micobacterias  no  tuberculosas.
Am  J  Respir  Crit  Care  Med  156(suppl  2  pt  2):S1­S25,  1997.
2.  Colice  GL,  Curtis  A,  Deslauriers  J  et  al.:  Tratamiento  médico  y  quirúrgico  de  los  derrames  paraneumónicos:  un
guía  basada  en  la  evidencia.  Cofre  118:1158­1171,  2000.
3.  Davis  B,  Systrom  DM:  Absceso  pulmonar:  patogenia,  diagnóstico  y  tratamiento  [Revisión].  Curr  Clin  Top  Infect  Dis
18:252­273,  1998.

4.  de  Holes  A,  Sundaresan  S:  Empiema  torácico.  Surg  Clin  North  Am  82:643–671.
5.  Molnar  TF:  Tratamiento  quirúrgico  actual  del  empiema  torácico  en  adultos  [Revisión].  Eur  J  de  Cardiothorac  Surg  32
(3):422­430,  2007.
6.  Pomerantz  M,  Brown  J:  Cirugía  de  enfermedad  micobacteriana  pulmonar.  En  Kaiser  LR,  Kron  IL,  Spray  TL,  editores:
Dominio  de  la  cirugía  cardiotorácica,  Filadelfia,  1998,  Lippincott­Raven,  1998.
7.  Takeda  S,  Maeda  H,  Hayakawa  M  et  al.:  Intervención  quirúrgica  actual  para  la  tuberculosis  pulmonar.  ann  torac
Surg  79(3):959­963,  2005.
8.  Wiedeman  HP,  Rice  TW:  Absceso  pulmonar  y  empiema.  Semin  Thorac  Cardiovasc  Surg  7:119­128,  1995.
Machine Translated by Google

CÁNCER  DE  PULMÓN
CAPITULO  
79

Jamie  M.  Brown,  MD

1.  ¿Qué  tan  común  es  el  cáncer  de  pulmón?
La  incidencia  de  cáncer  de  pulmón  es  de  aproximadamente  180.000  casos  nuevos  al  año  o  54,2  por  cada  100.000  
pacientes.  Más  de  162.000  pacientes  mueren  anualmente,  por  lo  que  la  tasa  de  supervivencia  global  es  del  10  %.
Este  número  no  ha  mejorado  en  los  últimos  35  años  a  pesar  de  algunos  avances  en  el  tratamiento  debido  a:  1.  El  
tabaquismo  adolescente  y  el  tabaquismo  en  general.

2.  Mayor  incidencia  en  no  fumadores.
3.  Presentación  de  cáncer  de  pulmón  en  estadio  avanzado  en  la  mayoría  de  los  pacientes.

2.  ¿Qué  factores  de  riesgo  se  cree  que  son  importantes  en  el  desarrollo
de  cáncer  de  pulmón?
El  noventa  por  ciento  de  los  pacientes  tienen  antecedentes  de  
tabaquismo  Químicos  (hidrocarburos  aromáticos,  cloruro  de  vinilo)
Radiación  (gas  radón  y  uranio)
Amianto  

Metales  (cromo,  níquel,  plomo  y  arsénico)
Factores  ambientales  (contaminación  del  aire,  alquitrán  de  hulla,  productos  derivados  del  petróleo)

3.  ¿Los  genes  y  la  herencia  juegan  un  papel  en  el  cáncer  de  pulmón?
Sí.  Una  historia  familiar  de  cáncer  de  pulmón  probablemente  aumenta  el  riesgo  de  contraer  cáncer  de  pulmón.
Además,  se  ha  identificado  una  gran  variedad  de  biomarcadores  importantes  que  influyen  en  el  pronóstico  en  
células  de  cáncer  de  pulmón  y  tejido  de  cáncer  de  pulmón.

Pasado:  &  Evidencia  microscópica  de  invasión  vascular  &  Invasión  
linfática  &  Pleomorfismo  celular  y  figuras  mitóticas  Presente:  &  
Protooncogenes,  factores  de  crecimiento,  receptores  de  factores  de  
crecimiento  &  Factor  de  crecimiento  similar  a  la  insulina  (IGF)

&  Receptor  del  factor  de  crecimiento  epidérmico  (EGFR)
&  mutación  K­ras  (regulación  del  crecimiento  celular)
&  sobreexpresión  de  C­myc  (crecimiento  celular)  &  
subexpresión  de  bcl­2  (pérdida  de  regulación  de  la  apoptosis)
&  Pérdida  de  genes  supresores  de  tumores  &  
p53  &  Retinoblastoma  (gen  RB)

&  Pérdida  de  alelos  cromosómicos  

&  Gen  de  la  tríada  de  histidina  frágil  (FHit)
&  Receptor  de  ácido  retinoico  a  (RARa)
&  Sobreactivación  de  la  angiogénesis  &  
Factor  de  crecimiento  derivado  de  plaquetas  (PDGF)
&  factor  de  crecimiento  derivado  del  endotelio  vascular  (VEGF)

401
Machine Translated by Google
402  CAPÍTULO  79  CÁNCER  DE  PULMÓN

Futuro:

&  Terapia  génica  dirigida  a  los  enumerados  en  el  presente
&  Terapia  antiangiogénesis
&  inmunopotenciación
&  Inmunoterapia  adoptiva:  aislamiento,  expansión  y  reinfusión  de  tumores  infiltrantes
linfocitos  &  
Inmunoestimulación  no  específica  &  Vacunas  
antitumorales  &  Ningún  marcador  único  tiene  

todavía  un  significado  claro  con  respecto  al  pronóstico  en  un  paciente  determinado.

4.  ¿Cuáles  son  los  principales  tipos  histológicos  de  cáncer  de  pulmón?
La  distinción  más  importante  es  entre  carcinoma  de  células  pequeñas  y  de  células  no  pequeñas  debido  a  las  diferencias  
fundamentales  en  la  biología  tumoral  y  el  comportamiento  clínico  (cuadro  79­1).  Los  pacientes  con  cáncer  de  pulmón  de  
células  pequeñas  se  clasifican  en  enfermedad  limitada  o  extensa.  Limitado  significa  que  toda  la  enfermedad  conocida  se  limita  
a  un  hemitórax  y  a  los  ganglios  linfáticos  regionales,  incluidos  los  ganglios  mediastínicos,  hiliares  contralaterales  y  
supraclaviculares  ipsilaterales.  Extenso  describe  la  enfermedad  más  allá  de  estos  límites,  incluyendo  el  cerebro,  la  médula  
ósea  y  las  metástasis  intraabdominales.

Con  el  carcinoma  de  células  pequeñas  o  neuroendocrino,  el  tipo  de  células  pequeñas  suele  ser  extenso  en  la  
presentación  y  la  supervivencia  a  los  cinco  años  es  del  5%.  El  carcinoma  neuroendocrino,  que  está  bien  diferenciado,  se  
conoce  como  carcinoide  atípico  y  tiene  un  buen  pronóstico  pero  no  es  "benigno".

CUADRO  79­1.  MAJORHISTOLO  GI  CTYP  ES  OFL  UN  GC  AN  CER

Tipo Incidencia  Comentarios

Carcinomas  de   80%
células  no  pequeñas

Adenocarcinoma  40% Ha  aumentado  en  no  fumadores

Carcinoma  de   40% Conocido  como  epidermoide,  está  asociado  histológicamente  con  perlas  


células  escamosas de  queratina  y  es  promovido  por  el  tabaquismo  y  otros  irritantes  
inhalados.

Carcinoma  de   15%
células  grandes

Carcinoma   5% Nódulo  único,  nódulos  múltiples  o  infiltrado  que  no  se  resuelve  en  la  
broncoalveolar radiografía  de  tórax

Carcinoma  de   20% Muy  mal  pronóstico


células  pequeñas

5.  ¿Es  eficaz  el  cribado  del  cáncer  de  pulmón?
Viejo  dogma:  No.
Pensamiento  actual:  Quizás.  El  pensamiento  es  el  siguiente:  el  cáncer  de  pulmón  representa  más
muertes  por  cáncer  que  otros  tipos  de  cáncer.  El  ochenta  y  cinco  por  ciento  de  los  pacientes  presentan  cáncer  de  
pulmón  avanzado  e  incurable.  No  hemos  cambiado  la  supervivencia  para  el  cáncer  de  pulmón.  Los  cánceres  en  etapa  
temprana  que  son  asintomáticos  se  pueden  encontrar  mediante  una  radiografía  de  tórax  y  una  tomografía  computarizada  
(TC)  helicoidal.  Desafortunadamente,  la  TC  también  detecta  muchos  falsos  positivos.  Además,  la  política  de  salud  
pública  no  respalda  la  detección  del  cáncer  de  pulmón.
Machine Translated by Google
CAPÍTULO  79  CÁNCER  DE  PULMÓN  403

6.  ¿Cómo  se  presentan  los  pacientes  con  cáncer  de  pulmón?
Tos  70%  Pérdida  de  peso  10%  Dolor  óseo  
30%  Síndrome  paraneoplásico  10%  
Asintomático  10%

7.  ¿Qué  es  un  síndrome  paraneoplásico?
Los  síndromes  paraneoplásicos  del  cáncer  de  pulmón  pueden  ser  metabólicos  (p.  ej.,  hipercalcemia,  síndrome  de  
Cushing),  neurológicos  (p.  ej.,  neuropatía  periférica,  polimiositis  o  síndrome  de  Lambert­Eaton,  que  es  similar  a  la  
miastenia  grave),  esqueléticos  (p.  ej.,  acropaquias,  osteoartropatía  hipertrófica),  hematológicos  (p.  ej.,  anemia,  
trombocitosis,  coagulación  intravascular  diseminada  [CID])  o  cutáneos  (p.  ej.,  hiperqueratosis,  acantosis  nigricans,  
dermatomiositis).  De  interés,  la  presencia  de  un  síndrome  paraneoplásico  no  influye  en  la  curabilidad  final  del  cáncer  
de  pulmón.

8.  ¿El  sistema  de  estadificación  del  cáncer  de  pulmón  tiene  efectos  pronósticos  y  terapéuticos?
¿importancia?
Sí.  La  supervivencia  del  paciente  está  relacionada  con  el  estadio  de  presentación  (cuadro  79­2).

CUADRO  79­2.  ESTADIFICACIÓN  DEL  CÁNCER  DE  PULMÓN

Descripción  del  subconjunto  de  etapas

I Él Tumor  intraparenquimatoso  con  o  sin  extensión  a  la  pleura  visceral,  a  2  cm  de  
la  carina,  y  sin  metástasis  a  ganglios  linfáticos
Uno Tumor  >3  cm  o  a  través  de  la  pleura  parietal,  sin  ganglios  positivos
Yo IIa El  tumor  primario  es  similar  al  del  estadio  I  con  extensión  a  los  ganglios  
linfáticos  interbronquiales  (N1)
IIb El  tumor  invade  la  pared  torácica  sin  afectación  ganglionar  (T3N0)
tercero IIIa Extensión  del  tumor  a  los  ganglios  linfáticos  hiliares  o  mediastínicos  (N2)  o  a  la  pared  
torácica  con  ganglios  N1

IIIb Todos  los  elementos  de  IIIa  más  extensión  del  tumor  a  estructuras  mediastínicas  
(corazón  o  grandes  vasos)  o  ganglios  linfáticos  hiliares,  paratraqueales  o  
supraclaviculares  contralaterales  (N3)
IV Derrame  pleural  maligno  o  enfermedad  metastásica  (M1)

9.  Describa  el  estudio  de  un  paciente  con  una  masa  en  la  radiografía  de  tórax.
El  estudio  debe  dirigirse  hacia  el  diagnóstico,  la  estadificación  y  la  evaluación  del  riesgo.
1.  Diagnóstico  
y  TC  y  tomografía  por  emisión  de  positrones  (PET):  define  el  tamaño,  mets,  ganglios  y  riesgo  de  
malignidad  y  citología  de  esputo:  rendimiento  diagnóstico  bajo  y  broncoscopia:  rendimiento  
diagnóstico  bajo  si  el  tumor  no  es  visible  y  aspiración  con  aguja  fina  guiada  por  TC  (FNA )

  Toracoscopia  y  biopsia:  escisión  en  cuña
Machine Translated by Google
404  CAPÍTULO  79  CÁNCER  DE  PULMÓN

2.  Estadificación  
y  tomografía  computarizada  (tórax):  tumor,  evaluación  de  ganglios  linfáticos  
mediastínicos  y  PET:  90  %  sensible  y  80  %  específico  para  ganglios,  metástasis  y  
broncoscopia:  invasión  endobronquial  y  toracoscopia:  muestreo  de  ganglios  linfáticos  
y  mediastinoscopia:  muestra  de  ganglios  N2  y  N3  3  Evaluación  de  riesgos  y  
espirometría  pulmonar:  cribado  de  ventilación/perfusión  (V/Q);  si  está  en  el  límite,  debe  
dejar  al  paciente  con  aproximadamente  800  ml  de  volumen  espiratorio  forzado  (FEV1)  

después  de  la  resección  Análisis  de  gases  en  sangre  arterial  (ABG)  y  electrocardiograma  
cardíaco  (ECG)

Antecedentes  de  infarto  de  miocardio  (IM),  intervención  previa  y  
cardiopulmonar  Capaz  de  caminar  un  tramo  de  escaleras;  en  caso  
afirmativo,  tolerará  la  lobectomía  Consumo  máximo  de  oxígeno  (O2)  <15  mililitros  
por  kilogramo  por  minuto

10.  ¿Cómo  se  tratan  los  pacientes  con  cáncer  de  pulmón?
El  tratamiento  más  efectivo  para  el  cáncer  de  pulmón  es  la  resección  quirúrgica.  Desafortunadamente,  el  75%  de  los  
pacientes  presentan  enfermedad  avanzada  y  no  son  candidatos  para  la  resección.  Afortunadamente,  la  quimioterapia  
preoperatoria  con  un  régimen  que  contiene  cisplatino  ha  aumentado  el  número  de  pacientes  en  estadio  III  que  son  candidatos  
para  la  resección.  Esta  terapia  innovadora  reciente  puede  traducirse  en  mejores  tasas  de  supervivencia.  Para  el  cáncer  de  pulmón  
en  etapa  III,  varios  ensayos  clínicos  han  demostrado  una  ventaja  de  la  quimioterapia  preoperatoria  y  el  tratamiento  con  radiación  
llamado  terapia  neoadyuvante.
Incluso  la  enfermedad  en  etapa  más  baja  o  los  tumores  con  alto  riesgo  de  recurrencia  pueden  beneficiarse  de  los  
regímenes  quimioterapéuticos  más  nuevos.

PUNTOS  CLAVE:  CÁNCER  DE  PULMÓN

1.  La  tasa  de  supervivencia  general  de  los  pacientes  con  cáncer  de  pulmón  es  del  10  %.

2.  El  noventa  por  ciento  de  los  pacientes  tienen  antecedentes  de  tabaquismo.

3.  El  tratamiento  más  efectivo  para  el  cáncer  de  pulmón  es  la  resección  quirúrgica.

11.  ¿La  radioterapia  de  quimioterapia  tiene  un  lugar  en  la  terapia  de  pulmón?
¿cáncer?

La  radioterapia  es  un  tratamiento  paliativo  eficaz,  pero  no  curativo,  para  el  cáncer  de  pulmón.  Específicamente,  los  pacientes  
que  presentan  un  síndrome  de  vena  cava  superior  o  un  bronquio  bloqueado  con  neumonía  distal  con  frecuencia  pueden  
"abrirse"  con  radioterapia.  La  radiación  también  es  excelente  para  paliar  el  dolor  óseo  patológico.  Algunos  ensayos  clínicos,  pero  
no  todos,  han  mostrado  algún  beneficio  del  tratamiento  de  quimiorradioterapia  preoperatoria  en  el  cáncer  de  pulmón  en  etapa  
avanzada.  Hay  evidencia  que  sugiere  que  los  pacientes  con  cáncer  de  pulmón  en  estadio  Ib,  IIa  o  b  se  benefician  de  la  
quimioterapia  de  inducción.  El  tratamiento  de  la  enfermedad  en  etapa  III  basado  en  la  presencia  de  ganglios  linfáticos  mediastínicos  
aún  es  controvertido,  a  pesar  de  mucha  atención  y  muchos  ensayos  en  los  últimos  10  años.  Por  ahora,  la  mayoría  de  los  pacientes  
con  enfermedad  en  etapa  III  tienen  solo  un  20  %  de  supervivencia  en  el  mejor  de  los  casos.  A  la  mayoría  se  les  debe  ofrecer  
quimioterapia  preoperatoria  de  inducción.
Machine Translated by Google
CAPÍTULO  79  CÁNCER  DE  PULMÓN  405

12.  ¿Cuál  es  la  tasa  de  supervivencia  de  los  pacientes  tratados  por  cáncer  de  pulmón  de  células  no  pequeñas?
a  los  5  años?
Etapa  I:  Ia  Ib   65%  (hasta  84  %  sin  nodos  o  N1)  55%  55%  40%  
Etapa  II:  IIa   20%  10%  2%

IIb  Etapa  III:  IIIa  
IIIb  Etapa  IV:  

Tenga  en  cuenta  
que  para  la  

invasión  de  la  
pared  torácica  sin  ganglios  linfáticos,  la  supervivencia  es  del  50  %  a  los  5  años,  aunque  todavía  se  denomina  etapa  IIb.  
Además,  si  el  cáncer  en  estadio  Ia  (tumor  pequeño,  sin  ganglios  positivos)  no  se  reseca,  la  supervivencia  disminuye  del  70%  
al  7%.

13.  ¿Qué  es  la  mediastinoscopia?
La  mediastinoscopia  es  un  procedimiento  de  estadificación  en  el  que  se  toman  muestras  de  los  ganglios  linfáticos  
paratraqueales,  subcarinales  y  peribronquiales  proximales  a  partir  de  una  pequeña  incisión  realizada  en  la  escotadura  supraesternal.

14.  ¿Cuáles  son  las  indicaciones  de  la  mediastinoscopia?
La  estadificación  mediastínica  está  indicada  en  pacientes  con  cáncer  de  pulmón  aparente  o  documentado  que  tienen:

&  Cáncer  de  pulmón  conocido  con  ganglios  linfáticos  mediastínicos  >1  cm  accesibles  por  vía  cervical
exploración  mediastínica,  evaluada  por  tomografía  computarizada.
  Adenocarcinoma  de  pulmón  y  múltiples  ganglios  linfáticos  mediastínicos  <1  cm.
  Cánceres  de  pulmón  central  o  grande  (>5  cm)  con  ganglios  linfáticos  mediastínicos  <1  cm.
  Cáncer  de  pulmón  y  tienen  alto  riesgo  de  toracotomía  y  resección  pulmonar.
  TEP  altamente  sugestiva  de  metástasis  ganglionares  mediastínicas.
Si  la  mediastinoscopia  tiene  resultados  negativos,  el  cirujano  debe  proceder  con  toracotomía,  biopsia  y  resección  pulmonar  
curativa.

15.  ¿El  derrame  pleural  maligno  o  la  afectación  del  nervio  recurrente  con  el  tumor  es  una  contraindicación  absoluta  para  la  
resección  quirúrgica  del  cáncer  de  pulmón?
Un  derrame  pleural  maligno  define  el  tumor­T  como  T4  y  el  estadio  clival  es  al  menos  IIIB.  La  mayoría  de  estos  pacientes  tendrán  
enfermedad  metastásica  después  de  la  evaluación.  En  raras  ocasiones,  se  producirá  un  derrame  pleural  maligno  pequeño  en  
presencia  de  un  tumor  primario  de  base  pleural  pero  resecable.
Por  el  contrario,  tanto  el  rey  Jorge  V  como  Arthur  Godfrey  tuvieron  resecciones  quirúrgicas  exitosas  ante  la  afectación  del  
nervio  recurrente  con  el  tumor.

SITIO  WEB

www.acssurgery.com

BIBLIOGRAFÍA

1.  Arriagada  R,  Bergman  B,  Dunaunt  A  et  al.:  Quimioterapia  adyuvante  basada  en  cisplatino  en  pacientes  con
Carcinoma  de  células  no  pequeñas  resecado.  N  Engl  J  Med  350:351­360,  2004.

2.  Ginsberg  RJ,  Ruckdeschel  JC,  editores:  Cáncer  de  pulmón:  pasado,  presente  y  futuro.  Parte  I.  Clínicas  de  cirugía  torácica  de
América  del  Norte,  vol.  10,  Filadelfia,  2000,  WB  Saunders.
Machine Translated by Google
406  CAPÍTULO  79  CÁNCER  DE  PULMÓN

3.  Mountain  EF:  Revisión  en  el  sistema  internacional  de  estadificación  del  cáncer  de  pulmón.  Cofre  111:1710,  1997.

4.  Aprobar  HI:  tratamiento  adyuvante  y  alternativo  del  cáncer  de  pulmón  broncogénico.  Chest  Surg  Clin  North  Am  1:1­20,
1991.

5.  Saunders  CA,  Dussek  JE,  O'Doherty  MJ  et  al .:  Evaluación  de  flúor  18­fluorodesoxiglucosa:  imágenes  de  tomografía  por  emisión  
de  positrones  de  cuerpo  entero  en  la  estadificación  del  cáncer  de  pulmón.  Ann  Thorac  Surg  67:790­797,  1999.

6.  Sonett  JR,  Krasna  MJ,  Sunthalingam  M  et  al.:  Resección  pulmonar  segura  después  de  quimioterapia  de  dosis  alta
radiación  torácica.  Ann  Thorac  Surg  68:316­320,  1999.

7.  Strauss  GM:  Marcadores  de  pronóstico  en  el  cáncer  de  pulmón  de  células  no  pequeñas  resecable.  Hematol  Oncol  Clin  North  Am
11:409­434,  1997.

8.  Toloza  EM,  Harpole  L,  Detterbeck  F  et  al.:  Estadificación  invasiva  del  cáncer  de  pulmón  de  células  no  pequeñas.  Pecho  123:157S­166S,
2003.
Machine Translated by Google

NÓDULO  PULMONAR  SOLITARIO
CAPITULO  
80

Jamie  M.  Brown,  MD  y  Marvin  Pomerantz,  MD

1.  ¿Qué  es  un  nódulo  pulmonar  solitario?
Un  nódulo  pulmonar  solitario  o  "lesión  en  moneda"  se  encuentra  en  la  radiografía  de  tórax  o  en  la  tomografía  
computarizada  (TC)  y  mide  <3  centímetros.  Está  rodeado  completamente  por  parénquima  pulmonar.

2.  ¿Qué  causa  un  nódulo  pulmonar  solitario?
Las  causas  más  comunes  de  un  nódulo  pulmonar  son  neoplásicas  (carcinoma,  60%  a  70%  de  los  nódulos  resecados)  o  
infecciosas  (granuloma).  Los  nódulos  pulmonares  también  pueden  representar  absceso  pulmonar,  infarto  pulmonar,  
malformaciones  arteriovenosas,  neumonía  en  resolución,  secuestro  pulmonar  y  hamartoma.  Como  regla  general,  la  
probabilidad  de  malignidad  es  proporcional  al  tamaño  del  nódulo,  la  edad  del  paciente  y  los  antecedentes  de  tabaquismo.  
Por  lo  tanto,  mientras  que  el  cáncer  de  pulmón  es  raro  (aunque  ocurre)  en  personas  de  30  años,  en  fumadores  de  50  años,  
la  probabilidad  de  que  un  nódulo  pulmonar  solitario  represente  malignidad  es  del  50%  al  60%.

En  una  persona  de  70  años  con  antecedentes  de  tabaquismo  y  un  nódulo  pulmonar  de  2,9  cm,  el  riesgo  de  malignidad  es  
del  75%.

3.  ¿Cómo  se  presenta  un  nódulo  pulmonar  solitario?
Por  lo  general,  un  nódulo  solitario  se  presenta  incidentalmente  como  un  hallazgo  en  una  radiografía  de  tórax  de  rutina.
En  varias  series  grandes,  más  del  75%  de  las  lesiones  fueron  hallazgos  inesperados  en  la  radiografía  de  tórax  de  
rutina.  Menos  del  25%  de  los  pacientes  tenían  síntomas  atribuibles  al  pulmón.  Los  nódulos  solitarios  ahora  se  ven  en  otras  
pruebas  de  imagen  sensibles,  como  la  TC  helicoidal.

4.  ¿Con  qué  frecuencia  un  nódulo  pulmonar  solitario  representa  enfermedad  metastásica?
Menos  del  10%  de  los  nódulos  solitarios  representan  enfermedad  metastásica.  En  consecuencia,  no  está  indicado  un  
estudio  exhaustivo  para  un  sitio  primario  de  cáncer  que  no  sea  el  pulmón.

5.  ¿Se  puede  obtener  una  muestra  de  tejido  mediante  biopsia  con  aguja  guiada  por  fluoroscopia  o  por  TC?
Sí,  pero  los  resultados  no  modifican  el  tratamiento.  Si  el  tejido  de  la  biopsia  con  aguja  indica  cáncer,  se  debe  extirpar  el  
nódulo.  Si  la  biopsia  con  aguja  es  negativa  para  cáncer,  aún  se  debe  extirpar  el  nódulo.  La  tomografía  por  emisión  de  
positrones  (PET)  tiene  una  sensibilidad  del  90%  para  identificar  tumores  malignos.  Las  características  de  los  nódulos  en  la  
tomografía  computarizada  combinada  con  PET  pueden  tener  una  precisión  del  90%  para  descartar  malignidad.

6.  ¿Son  importantes  los  hallazgos  radiográficos?
Sólo  relativamente.  La  resolución  de  los  escáneres  de  TC  permite  la  mejor  identificación  de  las  características  que  sugieren  
cáncer:  1.  Bordes  espiculados  del  nódulo  indistintos  o  irregulares.

2.  Cuanto  más  grande  es  el  nódulo,  más  probable  es  que  sea  maligno.
3.  La  calcificación  en  el  nódulo  generalmente  se  asocia  con  una  enfermedad  benigna  (lo  contrario  de
cáncer  de  mama).  En  concreto,  mientras  que  las  calcificaciones  centrales,  difusas  o  laminadas  son  típicas  de  un  
granuloma,  las  calcificaciones  con  patrones  más  densos  e  irregulares  en  «palomitas  de  maíz»  se  asocian  con  
hamartomas.  Desafortunadamente,  en  las  lesiones  malignas  se  pueden  encontrar  focos  excéntricos  de  calcio  o  
pequeñas  motas  de  calcio.

403
Machine Translated by Google
404  CAPÍTULO  80  NÓDULO  PULMONAR  SOLITARIO

4.  Los  nódulos  se  pueden  estudiar  usando  un  escáner  CT  midiendo  su  cambio  en  relación
radiodensidad  después  de  la  inyección  de  contraste.  Esto  se  denomina  atenuación  de  Hounsefield  y  mejora  la  
precisión  de  la  predicción  de  la  presencia  de  malignidad.

PUNTOS  CLAVE:  NÓDULO  PULMONAR  SOLITARIO
1.  Un  nódulo  pulmonar  solitario  o  «lesión  en  moneda»  mide  <3  cm  y  es  discreto  en  la  radiografía  de  tórax.

2.  Las  causas  más  comunes  de  un  nódulo  pulmonar  son  neoplásicas  o  infecciosas.

3.  Si  la  lesión  resulta  ser  cancerosa,  la  lobectomía  anatómica  es  el  procedimiento  de  elección.

7.  ¿Qué  hallazgos  sociales  o  clínicos  sugieren  que  un  nódulo  es  maligno  en  lugar
que  benigno?
Desafortunadamente,  ninguno  de  los  hallazgos  es  lo  suficientemente  sensible  o  específico  para  influir  en  el  estudio.
Tanto  el  aumento  de  la  edad  como  un  largo  historial  de  tabaquismo  predisponen  a  los  pacientes  al  cáncer  de  
pulmón.  Winston  Churchill  debería  haber  tenido  cáncer  de  pulmón,  pero  no  lo  tuvo.  Así,  el  hecho  de  que  el  paciente  
sea  presidente  del  club  de  espeleología  (histoplasmosis),  tenga  una  hermana  que  cría  palomas  (criptococosis),  
creció  en  el  valle  del  río  Ohio  (histoplasmosis),  trabaja  como  sacristán  en  un  cementerio  de  perros  (blastomicosis),  
o  simplemente  hizo  un  viaje  de  senderismo  por  el  Valle  de  San  Joaquín  (coccidioidomicosis)  es  una  historia  asociada  
interesante  pero  no  afecta  el  estudio  de  un  nódulo  pulmonar  solitario.

8.  ¿Cuál  es  la  parte  más  valiosa  de  los  datos  históricos?
La  edad  del  paciente  y  el  historial  de  tabaquismo.  Más  allá  de  lo  obvio,  el  dato  más  valioso  es  una  radiografía  de  
tórax  antigua.  Si  el  nódulo  es  nuevo,  es  más  probable  que  sea  maligno,  mientras  que  si  el  nódulo  no  ha  cambiado  
en  los  últimos  2  años,  es  menos  probable  que  sea  maligno.  Desafortunadamente,  incluso  esta  observación  no  es  
absoluta.

9.  Si  un  paciente  presenta  una  neoplasia  maligna  previa  tratada  y  un  nuevo  nódulo  pulmonar  solitario,  ¿es  
seguro  asumir  que  el  nuevo  nódulo  representa  una  enfermedad  metastásica?

No.  Incluso  en  pacientes  con  neoplasias  malignas  previas  conocidas,  menos  del  50  %  de  los  nódulos  pulmonares  
nuevos  son  metastásicos.  Por  lo  tanto,  el  estudio  debe  proceder  exactamente  como  para  cualquier  otro  paciente  con  un  
nuevo  nódulo  pulmonar  solitario.

10.  ¿Cómo  debe  evaluarse  un  nódulo  pulmonar  solitario?
Un  historial  completo  de  viajes  y  ocupación  es  interesante  pero  no  afecta  la  evaluación.
Debido  a  la  ubicación  periférica  de  la  mayoría  de  los  nódulos,  la  broncoscopia  tiene  un  rendimiento  diagnóstico  de  
<50%.  Incluso  en  las  mejores  manos,  la  citología  de  esputo  tiene  un  bajo  rendimiento.  Se  recomienda  la  tomografía  
computarizada  porque  puede  definir  hasta  cierto  punto  el  nódulo  (tamaño,  calcificación,  densidad,  etc.),  identificar  
otros  nódulos  potencialmente  metastásicos  y  delinear  el  estado  de  los  ganglios  linfáticos  mediastínicos.  Como  se  
indicó  anteriormente,  la  biopsia  percutánea  con  aguja  tiene  un  rendimiento  diagnóstico  de  aproximadamente  el  80%,  
pero  rara  vez  altera  el  manejo  posterior.  La  tomografía  por  emisión  de  positrones  puede  sugerir  cáncer  con  precisión.
Más  importante  aún,  la  exploración  por  PET  puede  sugerir  la  presencia  de  enfermedad  mediastínica  
extramediastínica  o  encubierta  con  más  sensibilidad  que  la  exploración  por  TC.
El  pilar  del  manejo  en  pacientes  que  pueden  tolerar  la  cirugía  es  la  resección  del  nódulo,
por  lo  general,  mediante  lobectomía  si  se  sospecha  cáncer,  para  el  diagnóstico  mediante  un  enfoque  de  
toracoscopia  mínimamente  invasiva  o  una  toracotomía  limitada.  Las  decisiones  de  observar  los  nódulos  pulmonares  
deben  tomarse  de  manera  selectiva  (p.  ej.,  paciente  anciano  que  no  es  candidato  a  cirugía)  y  con  un  plan  de  
seguimiento  cuidadoso.
Machine Translated by Google
CAPÍTULO  80  NÓDULO  PULMONAR  SOLITARIO  405

11.  Si  la  lesión  resulta  ser  cancerosa,  ¿cuál  es  el  tratamiento  quirúrgico  adecuado?
Como  toda  toma  de  decisiones  clínicas,  eso  dependería.  Aunque  varias  series  han  sugerido  que  la  escisión  
en  cuña  del  nódulo  es  suficiente,  una  lobectomía  anatómica  sigue  siendo  el  procedimiento  de  elección  para  
un  cáncer  de  pulmón  conocido.  Esto  a  menudo  se  puede  lograr  mediante  un  enfoque  asistido  por  video.  Un  
nódulo  solitario  que  resulta  ser  cáncer  en  etapa  temprana  y  en  ausencia  de  enfermedad  metastásica  y  tiene  
una  tasa  de  supervivencia  de  hasta  80%  a  5  años.  Desafortunadamente,  la  tasa  de  recurrencia  incluso  para  
tumores  en  etapa  I  o  un  nódulo  pequeño  es  del  30  %  en  5  años.  Las  recurrencias  se  dividen  entre  locales  y  
distantes.

SITIO  WEB

http://www.acssurgery.com

BIBLIOGRAFÍA

1.  Birim  O,  Kappetein  PA,  Stijnen  T  et  al .:  Metanálisis  de  la  tomografía  por  emisión  de  positrones  y  la  tomografía  computarizada  para  
detectar  metástasis  en  los  ganglios  linfáticos  mediastínicos  en  el  cáncer  de  pulmón  de  células  no  pequeñas.  Ann  Thoracic  Surgery  
79:375­382,  2005.

2.  Davies  B,  Ghosh  S,  Hopkinson  D  et  al .:  Nódulos  pulmonares  solitarios:  resultado  patológico  de  150  consecutivamente
lesiones  resecadas.  Interact  Cardiovas  Thorac  Surg  4:18­20,  2005.

3.  Dewey  TM,  Mack  MJ:  Cáncer  de  pulmón:  abordajes  e  incisiones  quirúrgicas.  Chest  Surg  Clin  North  Am  10:  803­820,
2000.

4.  Ginsberg  RJ,  Rubinstein  LV:  Ensayo  aleatorizado  de  lobectomía  versus  resección  limitada  para  células  no  pequeñas  T1  N0
cáncer  de  pulmón.  Grupo  de  Estudio  de  Cáncer  de  Pulmón.  Ann  Thorac  Surg  60:615­622,  1995.

5.  Khouri  NF,  Meziane  MA,  Zerhouni  EA  et  al.:  El  nódulo  pulmonar  solitario:  evaluación,  diagnóstico  y
gestión.  Cofre  91:128­133,  1987.

6.  Miller  DL,  Rowland  CM,  Deschamps  C  et  al.:  Tratamiento  quirúrgico  del  cáncer  de  pulmón  de  células  no  pequeñas  de  1  cm  o  menos  de  diámetro.  
Ann  Thorac  Surg  73:1541­1545,  2002.

7.  Nesbitt  J,  Putnam  JB  Jr,  Walsh  GL  et  al .:  Supervivencia  en  el  cáncer  de  pulmón  de  células  no  pequeñas  en  etapa  temprana.  Ann  Thorac  Cirugía
60:466­472,  1995.

8.  Varoli  F,  Vergani  C,  Caminiti  R  et  al.:  Manejo  del  nódulo  pulmonar  solitario.  Eur  J  Cardiothorac  Surg  33:
461­465.  2008.

9.  Walsh  GL,  Pisters  KM,  Stevens  C:  Tratamiento  del  cáncer  de  pulmón  en  estadio  I.  Chest  Surg  Clin  North  Am  10:17­38,  2001.
Machine Translated by Google

ANEURISMA  AÓRTICO  DISSECANTE
CAPITULO  
81

Richard­Tien  V.  Ha,  MD

1.  ¿Por  qué  el  término  aneurisma  aórtico  disecante  es  realmente  incorrecto?
El  término  correcto  debería  ser  hematoma  aórtico  disecante  porque  la  lesión  no  es  un  aneurisma.  La  
sangre  pasa  a  la  media,  creando  un  hematoma  que  separa  la  íntima  de  la  media  o  adventicia.  No  está  claro  si  
el  evento  incitador  es  el  desgarro  de  la  íntima  o  la  sangre  de  los  medios  que  desgarran  la  íntima.  Por  lo  tanto,  
un  desgarro  de  la  íntima  no  es  un  requisito  previo  porque  del  5%  al  13%  de  los  pacientes  no  lo  tienen.

2.  ¿Cuándo  se  debe  considerar  el  diagnóstico?
La  sospecha  es  el  factor  más  importante  porque  ninguna  característica  es  común  a  los  pacientes  que  presentan  
disecciones  aórticas.  En  cualquier  paciente  que  se  presente  con  un  dolor  intenso  en  el  pecho  y  la  espalda,  como  
un  cuchillo,  se  debe  considerar  el  diagnóstico  de  disección  aórtica.  Otros  síntomas  incluyen  síncope  y  síntomas  
neurológicos.

3.  Después  de  considerar  el  diagnóstico,  ¿cómo  se  debe  manejar  al  paciente?
Dos  tercios  de  los  pacientes  son  hipertensos,  por  lo  que  la  presión  arterial  debe  controlarse  a  una  presión  
arterial  sistólica  (PA)  de  <100  mm  Hg.  El  dolor  también  debe  controlarse  para  reducir  el  aumento  de  
catecolaminas.  El  otro  diagnóstico  que  debe  tenerse  muy  en  cuenta  es  el  infarto  agudo  de  miocardio  (IM).
Un  electrocardiograma  (ECG)  a  menudo  descarta  un  infarto  de  miocardio,  pero  algunas  disecciones  aórticas  arrancan  una  
arteria  coronaria;  por  lo  tanto,  tanto  el  infarto  agudo  como  la  disección  aórtica  ocurren  al  mismo  tiempo  (este  grupo  de  
pacientes  tiene  una  mayor  mortalidad).

4.  ¿Cuál  es  la  pista  diagnóstica  más  significativa  en  el  examen  físico?
Un  nuevo  soplo  diastólico  valvular  aórtico,  que  indica  regurgitación  valvular  aórtica  causada  por  la  
distorsión  de  la  estructura  de  la  válvula  por  el  hematoma  mural.  Además,  el  hematoma  disecante  puede  rodear  
la  luz  o,  de  hecho,  romper  el  despegue  de  los  vasos  subclavios  o  femorales,  lo  que  da  como  resultado  la  pérdida  
de  pulsos  o  la  variación  sistólica  entre  los  brazos.  Los  hallazgos  neurológicos,  que  incluyen  paraplejía  y  
hemiplejía,  también  pueden  estar  presentes  debido  a  la  oclusión  similar  del  colgajo  de  los  grandes  vasos.

5.  ¿Qué  hallazgos  de  la  radiografía  de  tórax  son  útiles  en  el  diagnóstico?
El  mediastino  ensanchado  y  la  pérdida  de  la  silueta  del  botón  aórtico  (un  hematoma  que  rodea  la  aorta  hace  
que  el  contorno  aórtico  sea  borroso)  son  hallazgos  útiles.  En  el  15%  al  25%  de  los  pacientes,  se  presenta  un  
derrame  pleural  del  lado  izquierdo.

6.  ¿Cómo  se  confirma  el  diagnóstico?  ¿Cuáles  son  los  mejores  estudios  de  diagnóstico?
La  literatura  reporta  la  alta  precisión  de  la  ecocardiografía  transesofágica  (ETE)  y  la  angiografía  por  
tomografía  computarizada  espiral  (CTA)  en  el  diagnóstico  de  las  disecciones  aórticas.  Por  otro  lado,  a  diferencia  
de  estas  modalidades,  la  angiografía  permite  la  visualización  de  las  arterias  coronarias  o  la  estimación  de  la  
insuficiencia  valvular  aórtica.  La  decisión  de  utilizar  una  modalidad  sobre  la  otra  radica  en  la  estabilidad  del  
paciente  y  las  modalidades  disponibles  en  una  institución  determinada.  La  ETE  debe  ser  la  primera  modalidad  
si  está  disponible  en  pacientes  inestables,  seguida  de

410
Machine Translated by Google
CAPÍTULO  81  ANEURISMA  AÓRTICO  DISECTIVO  411

CTA.  La  angiografía  se  puede  utilizar  en  pacientes  estables  para  definir  la  anatomía  coronaria  y  la  
arquitectura  valvular,  aunque  los  estudios  muestran  que  la  mortalidad  interna  no  mejora  con  la  angiografía  
coronaria.

7.  ¿Cuáles  son  los  tipos  de  disección?
Hay  dos  esquemas  de  clasificación:  Debakey  y  Stanford.  El  tipo  I  de  Debakey  afecta  a  la  aorta  ascendente  y  
se  propaga  al  menos  hasta  el  cayado  aórtico.  El  tipo  II  afecta  solo  a  la  aorta  ascendente.  El  tipo  III  afecta  a  la  aorta  
descendente.  La  clasificación  de  Stanford  tiene  valor  tanto  terapéutico  como  pronóstico:

Ascendente  (tipo  A)  involucra  solo  el  ascendente  o  tanto  el  ascendente  como  el  descendente
aorta.
Descendente  (tipo  B)  implica  solo  la  aorta  descendente.  Las  disecciones  ascendentes  son  dos  veces  más  
comunes  que  las  disecciones  descendentes  y  a  menudo  comienzan  en  la  pared  lateral  derecha  e  involucran  
el  arco  aórtico  en  un  30%.

8.  ¿A  quién  le  importa  si  una  disección  involucra  la  aorta  ascendente  (tipo  A)  o  descendente  (tipo  B)?

Las  disecciones  ascendentes  requieren  corrección  quirúrgica  temprana  para  evitar  la  extensión  a  las  arterias  
coronarias  o  carótidas,  la  ruptura  hacia  el  pericardio  (taponamiento)  o  ambas.  Las  disecciones  descendentes  no  
afectan  a  la  aorta  ascendente  y  pueden  tratarse  médica  o  quirúrgicamente  (ver  Controversias).

9.  ¿Cuál  es  la  clave  del  manejo  médico?
La  PA  debe  reducirse  a  100  mm  Hg  (sistólica)  con  una  combinación  de  nitroprusiato  de  sodio  y  propranolol.  El  
propranolol  o  labetalol  es  particularmente  importante  porque  disminuye  la  contractilidad  del  miocardio  (dp/dt),  
disminuyendo  así  la  fuerza  de  cizallamiento  que  impide  la  propagación  de  la  disección  a  lo  largo  de  la  aorta.  
Conceptualmente,  la  PA  debe  reducirse  tanto  como  sea  posible,  pero  el  paciente  debe  continuar  perfundiendo  los  
órganos  terminales  (es  decir,  producir  orina).
Se  puede  agregar  nitroprusiato  de  sodio  para  un  mayor  control  de  la  PA.

10.  ¿Cuáles  son  los  principios  y  ventajas  del  manejo  quirúrgico  de  la  enfermedad  aguda?
¿disección  aórtica?
Disección  ascendente  1.  
Cerrar  el  hematoma  obliterando  el  desgarro  de  la  íntima  más  proximal.
2.  Para  restaurar  la  competencia  de  la  válvula  aórtica.
3.  Para  restaurar  el  flujo  a  cualquier  rama  de  la  aorta  que  haya  sido  cortada  y  reciba
flujo  de  sangre  de  una  luz  falsa.
4.  Para  proteger  el  corazón  durante  estas  maniobras  y  restablecer  el  flujo  sanguíneo  coronario  si  se  ha  
cortado  una  arteria  coronaria.
5.  Buscar  desgarros  en  el  arco  aórtico  transverso.
Técnica:  Actualmente  está  de  moda  el  uso  de  parada  circulatoria  por  hipotermia  profunda  con  o  sin  perfusión  
cerebral  retrógrada.  Esta  técnica  permite  inspeccionar  el  arco  y  coser  con  precisión  la  anastomosis  distal  del  injerto  
de  Dacron  a  la  aorta  ascendente  distal  de  manera  abierta.  Es  controvertido  si  reemplazar  o  reparar  la  válvula  aórtica.

Disección  descendente  1.  
Para  cerrar  el  hematoma  obliterando  el  desgarro  de  la  íntima  más  proximal.
2.  Para  restaurar  el  flujo  de  sangre  a  las  ramas  de  la  aorta  alimentadas  por  el  canal  falso.
Técnica:  la  cirugía  se  realiza  utilizando  un  bypass  cardiopulmonar  parcial,  o  la  ''pinza  y
técnica  run'',  en  la  que  se  pinza  la  aorta  y  se  sutura  el  injerto  lo  más  rápido  posible  (ver  Controversias).  La  
reparación  endovascular  con  stents  está  ganando  popularidad  y  en  algunas  situaciones  clínicas  puede  ser  la  
mejor  opción  (ver  Controversias).
Machine Translated by Google
412  CAPÍTULO  81  ANEURISMA  AÓRTICO  DISECTIVO

PUNTOS  CLAVE:  ANEURISMA  AÓRTICO  DISSECANTE
1.  El  término  correcto  debería  ser  hematoma  aórtico  disecante  porque  la  lesión  no  es  una
aneurisma.

2.  Un  nuevo  soplo  diastólico  valvular  aórtico,  que  indica  regurgitación  valvular  aórtica  causada  por  la  distorsión  de  la  
estructura  de  la  válvula  por  el  hematoma  mural.

3.  Las  disecciones  ascendentes  requieren  una  corrección  quirúrgica  temprana  para  evitar  la  extensión  a  las  arterias  
coronarias  o  carótidas,  la  ruptura  hacia  el  pericardio  o  ambas.

4.  Las  disecciones  descendentes  pueden  tratarse  médicamente;  sin  embargo,  incluso  a  los  pacientes  quirúrgicos  se  
les  debe  bajar  la  presión  arterial  a  100­110  mm  Hg  con  una  combinación  de  nitroprusiato  de  sodio  y  propranolol/
labetalol.

11.  ¿Cuáles  son  las  complicaciones  operatorias?
  Hemorragia  (20%):  bastante  frecuente  por  el  uso  de  heparina  y  la  mala  calidad  de
tejido  aórtico  (como  Kleenex  húmedo).
&  Insuficiencia  renal  (20%).
  Insuficiencia  pulmonar  (un  30%  mayor  en  reparación  de  disecciones  descendentes).
  paraplejia:  a  menudo  se  presenta  antes  de  la  operación;  como  complicación  quirúrgica,  suele  ocurrir  solo  con  
disecciones  descendentes  (11%).
  IM  agudo  o  bajo  gasto  cardíaco  (30%).
&  Infarto  intestinal  (5%).
&  Muerte  (15%):  mayor  para  disecciones  agudas  que  crónicas  y  mayor  para  reparación  de  disecciones  ascendentes
disecciones

12.  ¿Cuáles  son  los  resultados  a  largo  plazo?
De  los  pacientes  que  sobreviven  a  la  operación,  dos  tercios  mueren  dentro  de  los  7  años  debido  a  enfermedades  
cardíacas  y  cerebrovasculares  comórbidas.

CONTROVERSIAS

13.  ¿Qué  se  prefiere,  el  manejo  médico  o  quirúrgico  de  las  disecciones  descendentes?

Manejo  quirúrgico  inicial
&  Aproximadamente  el  25%  de  los  pacientes  inicialmente  tratados  médicamente  necesitan  una  operación  eventualmente.
  La  mortalidad  operatoria  es  mucho  más  baja  hoy  (20%  a  30%)  que  en  el  pasado.
  La  reparación  con  injerto  de  stent  se  muestra  prometedora  como  un  tratamiento  temprano  y  menos  morboso.
Manejo  médico  inicial
  El  manejo  médico  tiene  una  tasa  de  mortalidad  hospitalaria  más  baja  (10%  a  15%).
&  Esto  evita  la  operación  innecesaria  y  su  consiguiente  costo  y  tasa  de  complicaciones.

14.  ¿Cuál  es  el  manejo  preferido  de  la  insuficiencia  aórtica  en  las  disecciones  ascendentes?

Reemplazo  de  válvula  aórtica  y  
Easy  (conductos  valvulados  ahora  disponibles).
&  Elimina  por  completo  la  insuficiencia  aórtica.
  Debe  realizarse  en  pacientes  con  síndrome  de  Marfan.
Machine Translated by Google
CAPÍTULO  81  ANEURISMA  AÓRTICO  DISECTIVO  413

Reparación  de  válvula  aórtica

&  Con  la  reconstrucción  de  válvula  nativa,  cuando  se  hace  correctamente,  la  necesidad  de  reemplazar  la  válvula  en  un

tiempo  posterior  es  sólo  el  10%.

&  Evita  la  necesidad  de  anticoagulación,  que  es  necesaria  cuando  se  utiliza  una  válvula  mecánica  para

reemplazar  la  válvula  aórtica.

15.  ¿Cuál  es  la  reparación  preferida  de  las  disecciones  descendentes?

1.  Bypass  parcial  de  la  aurícula  izquierda  a  la  arteria  femoral  
Para:  &  Permite  la  descarga  del  corazón.

  Permite  la  perfusión  distal  para  evitar  la  isquemia  visceral.

&  Permite  tanto  tiempo  como  sea  necesario  para  completar  la  anastomosis.

En  contra:  requiere  heparinización.

2.  Pinzamiento  cruzado  aórtico  simple  Para:  
Rápido.

En  contra:  la  colocación  del  injerto  debe  realizarse  en  <30  minutos  o  la  tasa  de  complicaciones,

particularmente  paraplejía,  aumenta  significativamente.

3.  Colocación  de  una  endoprótesis  cubierta  a  través  del  desgarro  de  la  íntima.
Para:  

&  Disminución  de  la  mortalidad  hospitalaria  (10%)  en  comparación  con  la  cirugía.

&  Disminución  de  la  estancia  hospitalaria,  recuperación  más  rápida  y  disminución  del  dolor  posterior  al  procedimiento.

&  Permite  la  reexpansión  de  la  luz  verdadera  comprimida.

&  Disminución  del  riesgo  de  paraplejia.

En  contra:  

&  Puede  ocluir  ramas  arteriales  previamente  normales.

  Efectividad  en  disecciones  tipo  B  estables  bajo  investigación.

&  Los  resultados  a  largo  plazo  no  se  conocen  en  este  momento.

SITIO  WEB

www.acssurgery.com

BIBLIOGRAFÍA

1.  Barron  DJ,  Livesey  SA,  Brown  IW  et  al .:  Veinte  años  de  seguimiento  de  la  disección  aguda  de  tipo  A:  la  incidencia  y  el  alcance  
de  la  enfermedad  aórtica  distal  mediante  resonancia  magnética.  J  Card  Surg  12:147­159,  1997.

2.  Cigarroa  JE,  Isselbacher  EM,  DeSanctis  RW  et  al .:  Diagnóstico  por  imágenes  en  la  evaluación  de  la  sospecha  de  disección  
aórtica.  Viejos  estándares  y  nuevas  direcciones.  N  Engl  J  Med  328:35­43,  1993.

3.  Glower  DD,  Fann  JI,  Speier  RH  et  al.:  Comparación  de  la  terapia  médica  y  quirúrgica  para  la  disección  aórtica  
descendente  sin  complicaciones.  Circulación  82  (suplemento  IV):  39­46,  1990.

4.  Khan  IA,  Nair  CK:  Perspectivas  clínicas,  diagnósticas  y  de  manejo  de  la  disección  aórtica.  Cofre  112:  311­328,
2002.

5.  Nienaber  CA,  von  Kodolitsch  Y,  Nicolas  V  et  al .:  El  diagnóstico  de  disección  aórtica  torácica  mediante  procedimientos  de  
imagen  no  invasivos.  N  Engl  J  Med  328:1–9;

6.  Okita  Y,  Takamoto  S,  Ando  M  et  al .:  Mortalidad  y  resultado  cerebral  en  pacientes  que  se  sometieron  a  arco  aórtico
operaciones  que  utilizan  paro  circulatorio  hipotérmico  profundo  con  perfusión  cerebral  retrógrada:  no  hay  relación  entre  muerte  
prematura,  accidente  cerebrovascular  y  delirio  con  la  duración  del  paro  circulatorio.  J  Thorac  Cardiovasc  Surg  115:129­138,  1998.
Machine Translated by Google
414  CAPÍTULO  81  ANEURISMA  AÓRTICO  DISECTIVO

7.  Safi  HJ,  Miller  CC,  Reardon  MJ  et  al .:  Operación  de  disección  aórtica  aguda  y  crónica:  resultado  reciente  con  respecto  
al  déficit  neurológico  y  muerte  temprana.  Ann  Thorac  Surg  66:402­411,  1998.
8.  Wheat  MW  Jr,  Palmer  RF,  Bartley  TB  et  al.:  Tratamiento  de  los  aneurismas  disecantes  de  la  aorta  sin  cirugía.
J  Thorac  Cardiovasc  Surg  50:364­373,  1995.
9.  Hagan  PG,  Nienaber  CA,  Isselbacher  EM  et  al.:  Registro  internacional  de  disección  aórtica  aguda  (IRAD):  nuevo
ideas  sobre  una  vieja  enfermedad.  JAMA  283(7):897­903,  2000.
10.  Nienaber  CA,  Eagle  KA:  Disección  aórtica:  nuevas  fronteras  en  el  diagnóstico  y  manejo.  Parte  I:  de  la  etiología  a  las  
estrategias  diagnósticas.  Circulación  108:628­635,  2003.
11.  Nienaber  CA,  Eagle  KA:  Disección  aórtica:  nuevas  fronteras  en  el  diagnóstico  y  manejo.  Parte  II:  terapéutica
manejo  y  seguimiento.  Circulación  108:772­778,  2003.
12.  Ince  H,  Nienaber  CA:  Diagnóstico  y  manejo  de  pacientes  con  disección  aórtica  aguda.  Corazón  93:266­270,
2007.
Machine Translated by Google

IX.  CIRUGÍA  PEDIÁTRICA

ESTENOSIS  PILÓRICA  HIPERTRÓFICA
CAPITULO  
82

Denis  D.  Bensard,  MD

1.  ¿Qué  es  la  estenosis  hipertrófica  de  píloro?
La  estenosis  hipertrófica  de  píloro  (EPH)  es  un  engrosamiento  y  elongación  idiopática  del  píloro  que  produce  
obstrucción  de  la  salida  gástrica.  HPS  es  la  causa  quirúrgica  más  común  de  vómitos  no  biliosos  en  bebés.  Los  
hijos  de  un  padre  afectado  tienen  una  mayor  incidencia  de  HPS  (10%);  la  tasa  más  alta  (20%)  ocurre  en  niños  
nacidos  de  madres  afectadas.

2.  Describa  la  presentación  típica  de  HPS.
La  presentación  típica  es  un  lactante  sano  que  al  principio  se  alimenta  normalmente  pero  que  se  presenta  
entre  las  2  y  las  6  semanas  de  edad  con  antecedentes  de  vómitos  en  «proyectil».  La  emesis  no  es  biliosa.  
Después  de  vomitar,  el  bebé  parece  tener  hambre  y  se  volverá  a  alimentar  de  inmediato.  Con  el  tiempo,  el  bebé  
se  deshidrata  y,  si  se  permite  que  progrese,  sigue  la  desnutrición.

3.  ¿Cuáles  son  los  hallazgos  físicos?
Los  bebés  afectados  sufren  algún  grado  de  deshidratación.  El  abdomen  no  está  distendido  y  es  blando.  
Un  tumor  pilórico  palpable,  conocido  como  ''oliva'',  confirma  el  diagnóstico.  Una  aceituna  es  palpable  en  el  50%  
de  los  pacientes.  Los  hallazgos  asociados  son  raros,  pero  se  presenta  ictericia  leve  en  5%  de  los  lactantes  debido  
a  la  reducción  de  la  actividad  de  la  glucuronil  transferasa.

4.  ¿Cómo  se  confirma  el  diagnóstico?
Los  criterios  ultrasonográficos  incluyen  diámetro  pilórico  >1,4  cm,  ancho  de  pared  >4  mm  y  longitud  del  canal  
pilórico  >1,6  cm.  Alternativamente,  se  puede  usar  un  examen  gastrointestinal  superior  (UGI)  con  bario  para  
confirmar  el  diagnóstico  (obstrucción  de  la  salida  gástrica,  estrechamiento  del  canal  pilórico).
Los  análisis  actuales  sugieren  que  UGI  es  la  prueba  diagnóstica  radiológica  inicial  más  rentable  porque,  a  
diferencia  de  la  ecografía  (US),  se  pueden  identificar  causas  alternativas  de  vómitos  no  biliosos  (p.  ej.,  reflujo  
gastroesofágico  [GERD],  malrotación,  estenosis  duodenal).

5.  Describa  las  anomalías  electrolíticas  probables.
Los  niveles  de  electrólitos  a  menudo  son  normales,  pero  los  vómitos  de  larga  duración  finalmente  darán  
como  resultado  una  alcalosis  metabólica  hipoclorémica  e  hipopotasémica  debido  a  la  pérdida  de  ácido  gástrico  
(HCl).  La  consideración  temprana  del  diagnóstico  ha  llevado  a  una  reducción  significativa  de  esta  anomalía  
electrolítica  clásica  en  la  presentación.  La  deshidratación  se  corrige  con  cloruro  de  sodio  al  0,9%  (NaCl)  o,  en  
casos  menos  graves,  con  NaCl  al  0,5%  con  30  mEq/L  de  cloruro  de  potasio  (KCl).  Después  de  corregir  la  
deshidratación  y  los  electrolitos,  se  realiza  la  piloromiotomía.

6.  ¿Qué  procedimiento  se  recomienda  para  la  corrección  de  HPS?
Se  recomienda  la  piloromiotomía  de  Fredet­Ramstedt.  Se  hace  una  incisión  superficial  longitudinalmente  
sobre  el  músculo  pilórico  en  un  área  avascular  y  se  fracturan  las  fibras  musculares  para  exponer  la  mucosa  
subyacente.  Al  finalizar  la  piloromiotomía,  la  mucosa  gástrica  debe  sobresalir  hacia  la  hendidura  y  las  paredes  del  
músculo  pilórico  deben  moverse  independientemente  unas  de  otras.  Se  inyecta  aire  en  el  estómago  a  través  de  
la  sonda  nasogástrica  (NG)  para  identificar  la  perforación  inadvertida  de  la  mucosa.  La  piloromiotomía  se  puede  
realizar  mediante  una  incisión  transversal  en  el  cuadrante  superior  derecho  (es  decir,  un  procedimiento  abierto)  o  
mediante  tres  incisiones  pequeñas  (3  mm).

415
Machine Translated by Google
416  CAPÍTULO  82  ESTENOSIS  PILÓRICA  HIPERTRÓFICA

en  el  epigastrio  (es  decir,  un  procedimiento  laparoscópico).  Los  resultados  de  la  piloromiotomía  abierta  
y  laparoscópica  parecen  equivalentes.

7.  ¿Qué  se  debe  hacer  si  se  identifica  una  perforación?
La  mucosa  debe  cerrarse  con  varias  suturas  finas  y  cubrirse  con  un  parche  de  epiplón.  Si  la  lesión  de  la  mucosa  
es  demasiado  extensa,  la  miotomía  debe  cerrarse  con  suturas  y  debe  realizarse  una  segunda  miotomía  paralela  
a  45  a  180  grados  de  la  miotomía  original.

8.  ¿Cuándo  puede  comenzar  la  alimentación  postoperatoria?
Las  alimentaciones  de  pequeño  volumen  se  inician  después  de  que  el  lactante  se  haya  recuperado  de  la  anestesia  (2  a  3  horas)  
y  haya  avanzado  hasta  el  objetivo.  Son  frecuentes  los  vómitos  en  pequeña  cantidad  (20  %),  pero  la  mayoría  de  los  lactantes  
logran  una  alimentación  completa  dentro  de  las  24  horas  posteriores  a  la  operación.  La  piloromiotomía  incompleta  es  poco  
común  (<1%)  y  no  se  considera  a  menos  que  los  síntomas  de  obstrucción  de  la  salida  gástrica  persistan  durante  7  a  10  días  
después  de  la  cirugía.

9.  Describa  varias  hipótesis  sobre  la  patogenia  del  SPH.
Estudios  recientes  del  complejo  pilórico  anormal  demuestran  inervación  inadecuada  del  músculo  liso  del  
píloro,  contracción  excesiva  del  músculo  liso  del  píloro  circular  (reducción  de  la  sintasa  de  óxido  nítrico),  
aumento  de  las  proteínas  de  la  matriz  extracelular  (colágeno)  y  aumento  de  la  expresión  o  síntesis  local  de  
hormonas  de  crecimiento  (es  decir,  insulina).  ­similar  al  factor  de  crecimiento­1,  factor  de  crecimiento  
transformante  b­1,  factor  de  crecimiento  derivado  de  plaquetas).

BIBLIOGRAFÍA

1.  Aspelund  G,  Langer  JC:  Manejo  actual  de  la  estenosis  pilórica  hipertrófica.  Semin  Pediatr  Surg  16:27­33,
2007.

2.  Hall  NJ,  Van  Der  Zee  J,  Tan  HL  et  al.:  Metanálisis  de  piloromiotomía  laparoscópica  versus  abierta.  ann  surg
240:774­778,  2004.

3.  Leclair  MD,  Plattner  V,  Mirallie  E  et  al.:  Piloromiotomía  laparoscópica  para  la  estenosis  pilórica  hipertrófica:  una
ensayo  prospectivo,  aleatorizado  y  controlado.  J  Pediatr  Surg  42:692­698,  2007.

4.  Miozzari  HH,  Tonz  M,  von  Vigier  RO  et  al .:  Reanimación  con  líquidos  en  la  estenosis  hipertrófica  del  píloro  infantil.  Diario
Pediatría  90:511­514,  2001.

5.  Saur  D,  Vanderwinden  JM,  Seidler  B  et  al .:  El  polimorfismo  del  promotor  de  un  solo  nucleótido  altera  la  transcripción  del  exón  
1c  de  la  sintasa  de  óxido  nítrico  neuronal  en  la  estenosis  pilórica  hipertrófica  infantil.  Proc  Natl  Acad  Sci  USA  101:1662­1667,  
2004.

6.  van  der  Bilt  JD,  Kramer  WL,  van  der  Zee  DC  et  al.:  Alimentación  temprana  después  de  la  piloromiotomía  laparoscópica:  los  pros  y
contras.  Surg  Endosc  18:746­748,  2004.
Machine Translated by Google

OBSTRUCCIÓN  INTESTINAL
DE  NEONATOS  Y  LACTANTES
CAPITULO  
83

Richard  J.  Hendrickson,  MD  y  Denis  D.  Bensard,  MD

1.  ¿Qué  signos  o  síntomas  sugieren  obstrucción  intestinal  en  el  recién  nacido?
Los  signos  y  síntomas  varían  según  el  nivel  de  obstrucción.  La  obstrucción  intestinal  proximal  conduce  a  la  aparición  
temprana  de  emesis  biliosa,  generalmente  con  distensión  abdominal  mínima.  Por  el  contrario,  los  recién  nacidos  con  
obstrucción  intestinal  distal  se  presentan  después  del  primer  día  de  vida  con  vómitos  biliosos  y  distensión  abdominal  
pronunciada.  La  emesis  biliar  siempre  debe  ser  interrogada  más  a  fondo  en  bebés  y  niños  porque  el  estudio  de  contraste  
demostrará  que  una  causa  quirúrgica  para  la  emesis  biliar  estará  presente  en  aproximadamente  un  tercio  de  los  bebés.

2.  ¿Cuál  es  el  diagnóstico  diferencial  de  la  obstrucción  intestinal  en  neonatos?
Busque  una  abertura  anal,  lo  que  elimina  el  diagnóstico  de  ano  imperforado.  A  continuación,  obtenga  una  
radiografía  abdominal.  La  extensión  de  la  distensión  gaseosa  del  intestino  implica  una  obstrucción  intestinal  proximal  o  
distal.  No  se  debe  intentar  distinguir  la  obstrucción  del  intestino  delgado  de  la  del  intestino  grueso.
Proximal  (gases  intestinales  mínimos) Distal  (gas  intestinal  significativo)
Atresia  duodenal,  estenosis Atresia  ileal  

Malrotación  con  vólvulo  del  intestino  medio Íleo  meconial  o  tapón  
atresia  yeyunal Enfermedad  de  Hirschsprung

3.  ¿Cuándo  están  indicados  los  estudios  de  contraste  del  tracto  gastrointestinal?
Si  hay  peritonitis  o  neumoperitoneo,  proceda  a  la  laparotomía  exploradora  sin  demora.
La  malrotación  con  vólvulo  debe  distinguirse  de  la  otra  causa  de  obstrucción  duodenal  congénita  (atresia  duodenal).  
En  este  contexto,  el  tubo  digestivo  alto  (GI)  es  el  estudio  de  elección.  En  el  vólvulo,  el  GI  superior  demuestra  distensión  
del  duodeno  proximal,  tirabuzón  del  duodeno  distal  y  progresión  limitada  o  nula  del  contraste  hacia  el  intestino  distal.  
Por  el  contrario,  la  atresia  duodenal  aparece  como  una  bolsa  ciega  en  la  primera  o  segunda  porción  del  duodeno.  El  
enema  de  contraste  es  generalmente  el  estudio  preferido  en  todas  las  demás  formas  de  obstrucción  intestinal  neonatal.

Trastorno Enema  de  bario
atresia  ileal Microcolon;  sin  reflujo  al  íleon  terminal
íleo  meconial Microcolon;  reflujo  al  íleon  terminal  con  defectos  de  llenado
tapón  de  meconio Colon  normal;  gran  defecto  de  llenado  del  colon  izquierdo
enfermedad  de  Hirschsprung rectosigmoide  estrechado;  colon  proximal  dilatado

4.  Describa  la  atresia  intestinal.

La  atresia  puede  ocurrir  en  cualquier  parte  del  tracto  GI:  duodenal  (50%),  yeyunoileal  (45%)  o  colónica  (5%).
La  atresia  duodenal  surge  de  una  falla  en  la  recanalización  durante  la  octava  a  décima  semana  de  gestación;  Las  
atresias  yeyunoileal  y  colónica  son  causadas  por  un  accidente  vascular  mesentérico  intrauterino.

5.  Distinguir  la  atresia  duodenal  de  otras  formas  de  atresia  intestinal.
La  atresia  duodenal  se  caracteriza  por  la  aparición  de  vómitos  biliosos  (85%  de  atresia  distal  a  la  ampolla  de  Vater)  
dentro  del  primer  día  de  vida;  la  distensión  abdominal  significativa  está  ausente.

417
Machine Translated by Google
418  CAPÍTULO  83  OBSTRUCCIÓN  INTESTINAL  DE  NEONATOS  Y  LACTANTES

Aproximadamente  el  25%  de  los  niños  afectados  tienen  trisomía  21.  La  radiografía  abdominal  
muestra  una  "doble  burbuja"  causada  por  el  estómago  distendido  y  la  primera  o  segunda  porción  del  
duodeno.  La  corrección  quirúrgica  se  realiza  mediante  duodenoduodenostomía.
La  atresia  yeyunoileal  produce  vómitos  biliosos  a  los  2  o  3  días  de  vida  con  moderada  a
distensión  abdominal  severa.  La  radiografía  abdominal  muestra  asas  de  intestino  dilatadas  con  niveles  
hidroaéreos.  El  enema  de  bario  revela  un  microcolon  y  no  hay  reflujo  de  contraste  en  el  intestino  dilatado.  
Las  anomalías  asociadas  son  poco  frecuentes.  La  corrección  quirúrgica  implica  la  anastomosis  de  extremo  
a  extremo  con  o  sin  resección  intestinal  limitada.
La  atresia  colónica,  similar  a  la  atresia  yeyunoileal,  se  asocia  con  el  inicio  tardío  de  vómitos  biliosos,  
falta  de  paso  de  meconio  y  distensión  abdominal  de  moderada  a  grave.  La  radiografía  abdominal  
revela  asas  de  intestino  dilatadas  con  niveles  hidroaéreos  que  sugieren  obstrucción  intestinal  distal.  El  
enema  de  bario  demuestra  un  microcolon  con  un  corte  observado  en  un  segmento  colónico  proximal.  El  
veinte  por  ciento  de  los  bebés  afectados  sufren  una  anomalía  asociada  del  corazón,  el  sistema  
musculoesquelético,  la  pared  abdominal  o  el  tracto  gastrointestinal.  El  manejo  quirúrgico  incluye  resección  
colónica  limitada  con  anastomosis  primaria.

6.  Describa  la  malrotación  con  vólvulo  del  intestino  medio.
Durante  la  sexta  a  la  duodécima  semana  de  gestación,  el  intestino  sufre  evisceración,  crecimiento,  retorno  
a  la  cavidad  abdominal  y  rotación  en  sentido  antihorario  con  fijación.  La  malrotación  es  un  error  tanto  en  la  
rotación  como  en  la  fijación.  La  fijación  anormal  y  un  mesenterio  de  base  estrecha  predisponen  a  la  torsión  del  
intestino  medio  en  su  riego  sanguíneo  (arteria  mesentérica  superior),  oclusión  vascular  (estrangulamiento)  y  
obstrucción  (rotación  anómala  con  vólvulo  del  intestino  medio).  Por  lo  general,  un  recién  nacido  o  un  niño  sin  
antecedentes  de  cirugía  presenta  vómitos  biliosos,  distensión  abdominal  y  grados  variables  de  shock.  Si  el  
lactante  está  gravemente  enfermo,  no  se  necesitan  más  estudios  y  está  indicada  la  exploración  quirúrgica.  Si  el  
diagnóstico  es  dudoso  y  el  lactante  está  estable,  se  realiza  un  estudio  GI  superior,  no  un  enema  de  bario.  El  
tratamiento  quirúrgico  consta  de  cuatro  partes:  (1)  división  de  bandas  peritoneales  anormales,  (2)  corrección  de  
malrotación,  (3)  restauración  de  un  mesenterio  de  base  ancha  y  (4)  apendicectomía  debido  a  la  ubicación  del  
ciego  en  el  cuadrante  superior  derecho. .

7.  ¿Es  el  vólvulo  del  intestino  medio  una  emergencia  quirúrgica?
¡Sí!  El  riesgo  de  estrangulamiento  causado  por  la  anomalía  rotacional  y  las  bandas  peritoneales  anormales  
implica  una  emergencia  quirúrgica.  El  retraso  pone  al  bebé  en  riesgo  de  perder  todo  el  intestino  medio  y  
potencialmente  morir.

8.  ¿Qué  es  el  íleo  meconial?
El  íleo  meconial  (MI)  es  la  obstrucción  del  íleon  terminal  por  meconio  muy  viscoso  y  tenaz.  El  IM  es  una  
complicación  de  la  fibrosis  quística  (FQ).  El  quince  por  ciento  de  los  recién  nacidos  con  FQ  presentan  IM.  
La  combinación  de  moco  hiperviscoso  secretado  por  las  glándulas  intestinales  anormales  y  la  insuficiencia  
pancreática  produce  meconio  anormal  y  obstruye  la  luz  del  íleon  terminal.  Los  síntomas  de  intolerancia  
alimentaria,  emesis  biliosa  y  distensión  abdominal  comienzan  en  el  segundo  o  tercer  día  de  vida.  A  diferencia  
de  la  mayoría  de  las  formas  de  obstrucción  intestinal  neonatal,  la  cirugía  se  reserva  para  pacientes  refractarios  
al  tratamiento  conservador  o  MI  complejo  (atresia,  vólvulo  o  perforación).  El  sesenta  por  ciento  de  los  lactantes  
con  infarto  de  miocardio  simple  pueden  tratarse  con  éxito  con  enemas  de  Gastrografin  e  irrigación  rectal.  Si  está  
indicada  una  operación,  el  objetivo  es  eliminar  el  meconio  obstructivo  mediante  resección  limitada  o  enterostomía  
con  evacuación  del  meconio  e  irrigación  del  intestino  distal.

9.  ¿Qué  es  la  enfermedad  de  Hirschsprung?
En  esta  enfermedad,  el  intestino  está  inervado  por  células  que  se  originan  en  la  cresta  neural.  Durante  la  
quinta  a  la  duodécima  semana  de  gestación,  las  células  de  la  cresta  neural  migran  en  dirección  craneocaudal  
y  se  dispersan  dentro  de  la  pared  del  intestino  (intermuscular  al  plexo  de  Auerbach;  submucosa  al  plexo  de  
Meissner).  La  enfermedad  de  Hirschsprung  surge  de  la  falla  de  la  inervación  entérica  normal.
El  intestino  permanece  en  un  estado  contraído,  espástico  y  produce  una  función  más  que  una  verdadera
Machine Translated by Google
CAPÍTULO  83  OBSTRUCCIÓN  INTESTINAL  DE  NEONATOS  Y  LACTANTES  419

obstrucción  mecánica.  La  distensión  abdominal,  la  intolerancia  alimentaria  y  el  meconio  retrasado  o  ausente  dentro  
de  las  primeras  48  horas  de  vida  son  los  hallazgos  de  presentación  en  los  lactantes.  Los  pacientes  mayores  sufren  de  
estreñimiento  crónico,  distensión  abdominal  y  retraso  en  el  crecimiento.  Debido  a  que  la  enfermedad  siempre  afecta  el  intestino  
más  distal  (80%  a  85%  rectosigmoideo)  con  una  afectación  variable  del  intestino  proximal,  el  enema  de  bario  muestra  la  
apariencia  radiográfica  característica  de  un  recto  contraído  y  espástico  con  dilatación  del  intestino  proximal.  La  biopsia  rectal  
por  succión  que  documenta  la  ausencia  de  células  ganglionares  y  la  presencia  de  hipertrofia  nerviosa  confirma  el  diagnóstico.  
La  corrección  quirúrgica  se  realiza  mediante  la  escisión  del  segmento  agangliónico  (colorrectal  distal)  y  la  anastomosis  coloanal.

10.  ¿Qué  es  la  intususcepción?  ¿Cuáles  son  las  opciones  terapéuticas?
La  intususcepción  es  la  invaginación  del  intestino  proximal  (intussusceptum)  hacia  el  intestino  distal  (intussuscipien).  
Siguen  la  hinchazón,  el  compromiso  vascular  y  la  obstrucción.  Casi  dos  tercios  de  los  casos  ocurren  en  los  primeros  2  años  de  
vida.  Se  cree  que  la  causa  es  el  resultado  de  una  hiperplasia  linfoide  en  el  íleon  terminal  después  de  una  infección  viral.  El  
diagnóstico  debe  sospecharse  en  lactantes  previamente  sanos,  de  6  a  9  meses  de  edad,  con  vómitos,  dolor  abdominal  tipo  
cólico  y  heces  sanguinolentas.  El  enema  de  bario  o  aire  es  tanto  diagnóstico  como  terapéutico.  La  inyección  de  contraste  
demuestra  obstrucción  colónica  sin  reflujo  hacia  el  intestino  proximal.  La  reducción  hidrostática  controlada  con  bario  o  aire  tiene  
éxito  en  el  90%  de  los  casos.  El  retraso  en  el  diagnóstico  y,  por  lo  tanto,  el  retraso  en  el  intento  de  reducción  es  la  causa  
principal  del  fracaso  de  la  reducción  hidrostática  y  la  necesidad  de  una  reducción  quirúrgica.  Si  la  reducción  hidrostática  no  
tiene  éxito  o  en  niños  con  peritonitis,  está  indicada  la  reducción  quirúrgica.  El  riesgo  de  invaginación  intestinal  recurrente  es  del  
5%  para  la  reducción  radiográfica  o  quirúrgica.

11.  ¿Qué  ejemplos  de  obstrucción  neonatal  pueden  escapar  a  la  detección  temprana  y  presentar
¿Tarde  en  la  vida?

Aunque  la  mayoría  de  las  condiciones  se  identifican  dentro  de  la  primera  semana  o  mes  de  vida,  se  pueden  identificar  otras  
lesiones  además  de  la  atresia  en  niños  e  incluso  en  adultos.
Estenosis  duodenal.  A  diferencia  de  la  atresia  duodenal,  la  estenosis  produce  un  estrechamiento  pero  no  una  obstrucción  
completa  del  duodeno.  Por  lo  tanto,  los  bebés  alimentados  con  fórmula  o  alimentos  en  puré  pueden  no  volverse  
sintomáticos  hasta  la  infancia.  Los  niños  con  dolor  abdominal  intermitente  y  síntomas  de  obstrucción  de  la  salida  gástrica  
requieren  un  estudio  del  tubo  digestivo  alto,  en  particular  si  tienen  trisomía  21.
Malrotación.  Un  tercio  de  los  pacientes  con  malrotación  se  identifican  después  del  primer  mes  de  vida.
Los  niños  se  presentan  con  emesis  biliosa  y  dolor  abdominal  intermitente,  y  la  mala  rotación  generalmente  se  identifica  
mediante  una  serie  GI  superior.  Debe  sospecharse  malrotación  con  vólvulo  del  intestino  medio  en  cualquier  niño  
enfermo  con  signos  de  obstrucción  intestinal  y  sin  antecedentes  de  cirugía  abdominal.  El  análisis  de  decisión  sugiere  
que  la  corrección  quirúrgica  de  la  malrotación  asintomática  es  beneficiosa  en  niños  hasta  la  segunda  década  de  vida,  
pero  más  allá  probablemente  no  esté  indicada  debido  a  la  rara  ocurrencia  de  vólvulo  del  intestino  medio  en  adultos.

enfermedad  de  Hirschsprung.  Un  tercio  de  los  pacientes  son  diagnosticados  después  del  primer  año  de  vida.  Una  larga  
historia  de  estreñimiento  refractario  a  la  terapia  exige  una  biopsia  rectal,  particularmente  en  pacientes  con  trisomía  21.

Intususcepción.  Un  tercio  de  los  casos  ocurren  después  de  los  2  años  de  edad.  Un  punto  de  derivación  patológico
(es  decir,  pólipo,  tumor,  hematoma,  divertículo  de  Meckel)  está  presente  en  un  tercio  de  los  pacientes  mayores.

BIBLIOGRAFÍA

1.  Engum  SA,  Grosfeld  JL:  Resultados  a  largo  plazo  del  tratamiento  de  la  enfermedad  de  Hirschsprung.  Semin  Pediatr  Surg
13:273­285,  2004.

2.  Escobar  MA,  Grosfeld  JL,  Burdick  JJ  et  al.:  Consideraciones  quirúrgicas  en  la  fibrosis  quística:  una  evaluación  de  32  años  de
resultados.  Cirugía  138:560­571,  2005.

3.  Escobar  MA,  Ladd  AP,  Grosfeld  JL  et  al.:  Atresia  y  estenosis  duodenal:  seguimiento  a  largo  plazo  durante  30  años.
J  Pediatr  Surg  39:867­871,  2004.

4.  Godbole  P,  Stringer  MD:  Emesis  biliosa  en  el  recién  nacido:  con  qué  frecuencia  es  patológico.  J  Pediatr  Surg  37:909­911,
2002.
Machine Translated by Google
420  CAPÍTULO  83  OBSTRUCCIÓN  INTESTINAL  DE  NEONATOS  Y  LACTANTES

5.  Jajivassiliou  CA:  Obstrucción  intestinal  en  cirugía  neonatal/pediátrica.  Semin  Pediatr  Surg  12:241­253,  2003.

6.  Malek  MM,  Burd  RS:  El  manejo  óptimo  de  la  malrotación  diagnosticada  después  de  la  infancia:  un  análisis  de  decisión.
Am  J  Surg  191:45­51,  2006.

7.  Milar  AJ,  Rode  H,  Cywes  S:  malrotación  y  vólvulo  en  la  infancia  y  la  niñez.  Semin  Pediatr  Surg  12:229­236,
2003.

8.  Somme  S,  A  T,  Langer  JC:  Factores  que  determinan  la  necesidad  de  reducción  quirúrgica  en  niños  con
invaginación  intestinal:  un  estudio  basado  en  la  población.  J  Pediatr  Surg  41:1014­1019,  2006.
Machine Translated by Google

ANO  IMPERFORADO
CAPITULO  
84

Frederick  M.  Karrer,  MD  y  Denis  D.  Bensard,  MD

1.  ¿Qué  es  el  ano  imperforado?
Es  un  defecto  congénito  en  el  que  la  abertura  del  ano  está  ausente  o  mal  colocada,  por  lo  general  con  una  
fístula  en  la  parte  anterior  del  perineo  o  del  tracto  genitourinario  (GU).  Las  malformaciones  anorrectales  van  
desde  una  ligera  malposición  anterior  del  ano  hasta  deformidades  complejas  de  la  cloaca.  Los  niños  con  
malformaciones  anorrectales  suelen  tener  otras  anomalías  congénitas,  como  la  asociación  VACTERL.

2.  ¿Qué  es  la  asociación  VACTERL?
Defectos  vertebrales  
Malformaciones  anorrectales  
Anomalías  cardiacas  Fístula  
traqueoesofágica  Atresia  
esofágica  Anomalías  renales  
Defectos  de  las  extremidades  
La  incidencia  de  anomalías  
renales  aumenta  con  la  gravedad  del  ano  imperforado,  del  10%  con  lesiones  bajas  al  75%  con  lesiones  
altas.

3.  ¿Cómo  determina  la  gravedad  del  defecto  en  los  niños?
La  clave  es  si  el  chico  tiene  una  lesión  alta  o  baja.  Las  lesiones  bajas  se  caracterizan  por  una  fístula  al  
perineo  en  algún  lugar  a  lo  largo  del  rafe  de  la  línea  media  entre  el  ano  y  el  meato  uretral.
Después  de  24  horas,  la  mayoría  de  los  lactantes  con  lesiones  bajas  muestran  meconio  en  la  fístula.  Otros  
signos  de  una  lesión  baja  incluyen  "perlas"  blancas  a  lo  largo  del  rafe  o  un  asa  de  piel  elevada,  la  llamada  
deformidad  en  asa  de  cubo.  Los  niños  con  lesiones  altas  típicamente  tienen  glúteos  planos  sin  un  buen  
pliegue  en  los  glúteos  y  pueden  tener  meconio  en  el  meato  uretral  o  aparente  en  el  análisis  de  orina.

4.  ¿Cómo  se  valora  la  lesión  en  las  niñas?
La  mayoría  de  las  niñas  afectadas  (>90%)  tienen  una  fístula  rectovestibular  o  rectovaginal,  que  por  lo  general  
puede  determinarse  mediante  un  examen  perineal  cuidadoso.  Las  niñas  con  deformidades  de  la  cloaca  (es  
decir,  un  orificio)  tienen  una  alta  incidencia  de  obstrucción  GU,  como  hidrocolpos  u  obstrucción  de  la  vejiga.  
En  las  lesiones  bajas,  la  abertura  anal  se  desplaza  anteriormente  sobre  el  perineo.  La  ubicación  normal  del  
ano  es  a  medio  camino  entre  el  orificio  vaginal  y  el  cóccix.

5.  ¿Cómo  se  tratan  los  bebés  con  malformaciones  anorrectales?
Los  bebés  con  lesiones  altas  deben  tratarse  inicialmente  con  una  colostomía  sigmoidea  y  luego  con  un  
procedimiento  llamado  anorrectoplastia  sagital  posterior.  Los  lactantes  con  lesiones  bajas  por  lo  general  
pueden  tratarse  con  anoplastia  inmediata  o  dilatación  y  reparación  diferida.

6.  ¿Qué  es  una  anorrectoplastia  sagital  posterior  (PSARP)?
PSARP  es  un  procedimiento  realizado  a  través  de  una  incisión  longitudinal  en  la  línea  media  del  perineo,  
que  permite  la  visualización  de  la  musculatura  pélvica  y  los  esfínteres  y  la  exposición  clara  del  recto  y  la  
fístula.  Después  del  cierre  de  la  fístula,  el  recto  se  reposiciona  dentro  del  complejo  muscular  del  esfínter  y  se  
crea  un  neoano.

421
Machine Translated by Google
422  CAPÍTULO  84  ANO  IMPERFORADO

7.  ¿Cuáles  son  los  resultados  después  de  la  reconstrucción  quirúrgica?
La  continencia,  definida  como  deposiciones  voluntarias  sin  ensuciar,  depende  del  tipo  de  lesión.  La  continencia  
se  aproxima  al  100%  en  las  lesiones  bajas,  pero  es  rara  en  las  lesiones  más  altas,  como  las  deformidades  de  la  
cloaca  en  las  niñas  o  las  fístulas  del  cuello  de  la  vejiga  en  los  niños.  El  estreñimiento  está  presente  en  casi  el  
50%  de  los  pacientes,  pero  es  más  frecuente  en  los  defectos  más  simples.

PUNTOS  CLAVE:  ANO  IMPERFORADO
1.  El  ano  imperforado  es  un  defecto  congénito  en  el  que  la  abertura  del  ano  está  ausente  o
fuera  de  lugar,  por  lo  general  fistulizando  anteriormente  al  perineo  o  al  tracto  GU.

2.  Los  bebés  con  lesiones  altas  deben  manejarse  inicialmente  con  una  colostomía  sigmoidea  y  luego
con  un  procedimiento  pull­through  llamado  PSARP.

3.  Los  bebés  con  lesiones  bajas  por  lo  general  pueden  tratarse  con  anoplastia  inmediata  o  dilatación  y
reparación  retrasada.

BIBLIOGRAFÍA

1.  Kuo  MF,  Fsai  Y,  Hsu  WM  et  al.:  Médula  espinal  atada  y  asociación  VACTERL.  J  Neurocirugía  106:201­204,
2007.

2.  Levitt  MA,  Pena  A:  Resultados  de  la  corrección  de  malformaciones  anorrectales.  Curr  Opin  Pediatría  17:394­401,
2005.

3.  Niedzielski  JK:  Invertografía  versus  ultrasonografía  y  colostografía  distal  para  la  determinación  del  intestino
distancia  de  la  piel  en  niños  con  malformaciones  anorrectales.  Eur  J  Pediatr  Surg  15:262­267,  2005.
4.  Pena  A,  Levitt  MA,  Hong  A  et  al .:  Manejo  quirúrgico  de  malformaciones  cloacales:  una  revisión  de  339  pacientes.
J  Pediatr  Surg  39:470­479,  2004.
5.  Pena  A,  Hong  AR,  Midulla  P  et  al.:  Cirugía  reoperatoria  para  anomalías  anorrectales.  Semin  Pediatr  Surg
12:118­123,  2003.
6.  Rosen  NG,  Hong  AR,  Soffer  SZ  et  al .:  Fístula  rectovaginal:  un  error  de  diagnóstico  común  con  consecuencias  
significativas  en  niñas  con  malformaciones  anorrectales.  J  Pediatr  Surg  37:961­965,  2002.
Machine Translated by Google

MALFORMACIONES  TRAQUEOESOFÁGICAS
CAPITULO  
85

Denis  D.  Bensard,  MD  y  David  A.  Partrick,  MD

1.  ¿Qué  son  la  fístula  traqueoesofágica  (FTE)  y  la  atresia  esofágica  (EA)?
La  tráquea  y  el  esófago  aparecen  como  un  divertículo  ventral  que  surge  del  intestino  anterior  primitivo  
durante  la  tercera  semana  de  gestación.  La  tráquea  y  el  esófago  se  separan  por  el  crecimiento  interno  de  
las  crestas  ectodérmicas  durante  la  cuarta  semana  de  gestación.  El  fracaso  de  la  separación  da  como  
resultado  una  conexión  anómala  de  la  tráquea  con  el  esófago  (es  decir,  TEF)  con  o  sin  formación  incompleta  
del  esófago  (es  decir,  EA).

2.  Describir  las  tres  variantes  más  comunes  y  la  incidencia  relativa  de  cada  tipo.
EA  proximal  con  TEF  distal  (''bolsa  proximal  con  fístula  distal''):  85%
AE  aislado:  10%
FTE  sin  EA  (''Fístula  H''):  5%

3.  ¿Qué  otras  anomalías  ocurren  con  las  malformaciones  traqueoesofágicas?
TEF  y  EA  resultan  de  un  insulto  durante  la  fase  crítica  de  la  embriogénesis  (3  a  8  semanas  de  gestación).
Hasta  el  70%  de  los  lactantes  con  malformaciones  traqueoesofágicas  sufren  una  o  más  anomalías  
concomitantes.  Las  anomalías  cardiovasculares  son  las  más  prevalentes  (35  %),  seguidas  de  las  anomalías  
de  los  sistemas  gastrointestinal  (GI;  24  %),  genitourinario  (GU;  20  %),  esquelético  (13  %)  y  nervioso  central  
(10  %).  El  veinticinco  por  ciento  de  los  bebés  que  nacen  con  malformación  traqueoesofágica  tienen  uno  o  
más  componentes  de  la  asociación  VACTERL  (consulte  la  pregunta  2  en  el  Capítulo  84).

4.  ¿La  presencia  de  otras  anomalías  altera  el  manejo  y  el  resultado?
Los  lactantes  sanos  sin  anomalías  concomitantes  generalmente  se  someten  a  una  reparación  temprana  con  
una  tasa  de  supervivencia  de  casi  el  100  %,  mientras  que  los  lactantes  muy  prematuros  o  con  anomalías  
potencialmente  mortales  suelen  someterse  a  una  reparación  tardía.  Los  bebés  con  anomalías  letales,  como  la  
trisomía  18,  reciben  solo  cuidados  paliativos.

5.  Describir  la  presentación  clínica,  diagnóstico  y  manejo  preoperatorio  de  pacientes  con  EA  con  FTE  distal.

Temprano  en  el  período  neonatal,  los  bebés  afectados  muestran  salivación  excesiva  (es  decir,  incapacidad  para  
tragar  secreciones),  asfixia  o  regurgitación  con  la  alimentación  (es  decir,  incapacidad  para  tragar  alimentos).
La  dificultad  respiratoria  se  produce  rápidamente  debido  a  la  aspiración  de  secreciones  o  alimentos  de  la  
bolsa  esofágica  y  el  reflujo  de  ácido  gástrico  hacia  las  vías  respiratorias  y  los  pulmones  a  través  de  la  TEF  distal.
Una  sonda  nasogástrica  (NG)  no  puede  introducirse  en  el  estómago.  La  radiografía  muestra  una  bolsa  
esofágica  proximal  ciega  y  un  estómago  lleno  de  aire  causado  por  la  conexión  anómala  del  esófago  distal  
con  las  vías  respiratorias.  El  bebé  se  mantiene  en  una  posición  semi­vertical  con  drenaje  de  catéter  de  
sumidero  de  la  bolsa  esofágica  proximal  para  minimizar  la  contaminación  de  los  pulmones  ya  sea  por  
aspiración  o  reflujo.

6.  Describir  la  presentación  clínica,  el  diagnóstico  y  el  manejo  preoperatorio  de
AE  aislado.
La  EA  aislada  se  asocia  con  salivación  excesiva,  asfixia  y  regurgitación  de  alimentos.
La  incapacidad  de  pasar  una  sonda  NG  al  estómago  y  un  abdomen  sin  gas  aparente  en

423
Machine Translated by Google
424  CAPÍTULO  85  MALFORMACIONES  TRAQUEOESOFÁGICAS

la  radiografía  sugiere  el  diagnóstico.  El  manejo  preoperatorio  está  dirigido  a  la  identificación  de  anomalías  asociadas  y  la  
determinación  de  la  longitud  del  espacio.  Se  mantiene  el  drenaje  del  catéter  de  sumidero  de  la  bolsa  esofágica  proximal  
para  minimizar  la  aspiración.  La  gastrostomía  generalmente  se  realiza  dentro  de  las  primeras  24  horas  de  vida  para  permitir  
la  alimentación  y  la  evaluación  de  la  longitud  del  esófago  distal.
Por  lo  general,  los  bebés  con  EA  se  someten  a  una  reparación  tardía  para  permitir  el  crecimiento  del  esófago  distal  y  
la  reducción  de  la  distancia  del  espacio.

7.  Describir  la  presentación  clínica,  el  diagnóstico  y  el  manejo  preoperatorio  de  la  FTE  sin  EA.

Estos  bebés  muestran  asfixia  repetida  o  episodios  cianóticos  con  la  alimentación  causados  por  el  reflujo  de  alimentos  
desde  el  esófago  a  los  pulmones  a  través  de  la  conexión  traqueoesofágica  anómala.
Los  lactantes  mayores  y  los  niños  pueden  presentar  episodios  recurrentes  de  neumonía  o  enfermedad  reactiva  
inexplicable  de  las  vías  respiratorias  como  resultado  de  la  contaminación  intermitente  de  los  pulmones  a  través  de  la  fístula.
Se  utilizan  videoesofagografía  y  broncoscopia  para  demostrar  la  fístula.

8.  ¿Cómo  se  corrigen  quirúrgicamente  las  malformaciones  traqueoesofágicas?
El  tratamiento  quirúrgico  implica  la  restauración  de  la  continuidad  esofágica  y  la  eliminación  de  la  conexión  patológica  
del  esófago  con  las  vías  respiratorias.  La  corrección  de  la  EA  con  o  sin  TEF  generalmente  se  realiza  mediante  
toracotomía,  con  o  sin  ligadura  de  TEF  y  anastomosis  esofágica  terminoterminal.
En  algunos  centros,  la  reparación  ahora  se  realiza  por  toracoscopia;  aunque  factible,  los  resultados  a  largo  plazo  de  
este  enfoque  siguen  sin  estar  claros.  A  los  5  a  7  días  de  la  cirugía  se  realiza  un  esofagograma;  si  no  se  visualiza  fuga,  
se  inicia  la  alimentación  oral  y  se  retira  el  drenaje  pleural.
La  TEF  sin  EA  se  aborda  a  través  de  una  incisión  cervical,  evitando  la  toracotomía.  Se  secciona  el  trayecto  fistuloso  y  
se  interpone  tejido  sano  para  evitar  la  recurrencia.

9.  ¿Cuáles  son  las  complicaciones  tempranas  y  tardías  de  la  reparación  quirúrgica?
Complicaciones  
tempranas  Rotura  anastomótica   5%  
FTE  recurrente  Fuga   5%  
anastomótica  Traqueomalacia   15%  
Las  complicaciones  tempranas   15%

están  relacionadas  con  los  principios  quirúrgicos  básicos  de  la  cicatrización  de  heridas.  La  interrupción  de  la  anastomosis  
generalmente  se  debe  a  un  riego  sanguíneo  y  una  tensión  deficientes.
Complicaciones  tardías  
Estenosis  anastomótica   25%  

Reflujo  gastroesofágico  (RGE) 50%  

Dismotilidad  esofágica  La   100%

mayoría  de  las  estenosis  (50%)  responden  a  una  a  tres  dilataciones  realizadas  en  los  primeros  6  meses  de  vida.  Las  
estenosis  refractarias  requieren  la  identificación  del  RGE  asociado,  que  puede  empeorar  la  formación  de  estenosis.  La  
frecuencia  de  RGE  parece  estar  relacionada  con  la  longitud  del  intervalo  (es  decir,  cuanto  mayor  es  la  distancia  del  
intervalo,  mayor  es  el  riesgo  de  RGE  significativo).

PUNTOS  CLAVE:  MALFORMACIONES  TRAQUEOESOFÁGICAS

1.  Las  tres  variantes  más  comunes  son  EA  proximal  con  FTE  distal,  EA  aislada  y  FTE
sin  EE.

2.  Al  principio  del  período  neonatal,  los  bebés  afectados  muestran  salivación  excesiva,  asfixia  o  regurgitación  al  
alimentarse.

3.  El  tratamiento  quirúrgico  implica  la  restauración  de  la  continuidad  esofágica  y  la  eliminación  de  la  conexión  
patológica  del  esófago  con  las  vías  respiratorias.
Machine Translated by Google
CAPÍTULO  85  MALFORMACIONES  TRAQUEOESOFÁGICAS  425

BIBLIOGRAFÍA

1.  Atzori  P,  Iacobelli  BD,  Bottero  S  et  al.:  Traqueobroncoscopia  preoperatoria  en  recién  nacidos  con  atresia  esofágica:  
¿importa?  J  Pediatr  Surg  41:1054­1057,  2007.
2.  Keckler  SJ,  St  Peter  SD,  Valuseck  PA  et  al.:  Anomalías  de  VACTERL  en  pacientes  con  atresia  esofágica:  una  actualización
delimitación  del  espectro  y  revisión  de  la  literatura.  Pediatr  Surg  Int  23:309­313,  2007.
3.  Koivusalo  A,  Pakarinen  MP,  Turunen  P  et  al.:  Calidad  de  vida  relacionada  con  la  salud  en  pacientes  adultos  con  esófago
atresia  ­  un  estudio  de  cuestionario.  J  Pediatr  Surg  40:307­312,  2005.
4.  Konkin  DE,  O'hali  WA,  Webber  EM  et  al.:  Resultados  en  atresia  esofágica  y  fístula  traqueoesofágica.
J  Pediatr  Surg  38:1726­1729,  2003.
5.  Lopez  PJ,  Keys  C,  Pierro  A  et  al.:  Atresia  esofágica:  ¿mejor  resultado  en  grupos  de  alto  riesgo?  J  Pediatr  Surg
41:331­334,  2006.

6.  Sinha  CK,  Haider  N,  Marri  RR  et  al.:  Criterios  pronósticos  modificados  para  atresia  esofágica  y  atresia  traqueoesofágica
fístula.  Eur  J  Pediatr  Surg  17:153­157,  2007.
7.  Spitz  L:  atresia  esofágica:  lecciones  que  he  aprendido  en  una  experiencia  de  40  años.  J  Pediatr  Surg  41:1635­1640,  2006.
Machine Translated by Google

HERNIA  DIAFRAGMATICA  CONGÉNITA
CAPITULO  
86

Denis  D.  Bensard,  MD  y  Richard  J.  Hendrickson,  MD

1.  ¿Cuál  es  el  tipo  más  común  de  hernia  diafragmática  congénita?
Las  anomalías  congénitas  del  diafragma  incluyen  un  defecto  posterolateral  (hernia  de  Bochdalek),  un  
defecto  anteromedial  (hernia  de  Morgagni)  o  la  eventración  (debilitamiento  central)  del  diafragma.  La  
hernia  de  Bochdalek  es  la  variante  más  común  y  generalmente  se  presenta  en  el  lado  izquierdo  (80%).  
Aproximadamente  el  20%  ocurren  en  el  lado  derecho  y  <1%  son  bilaterales.

2.  ¿Qué  signos  y  síntomas  sugieren  CDH?
La  dificultad  respiratoria  neonatal  es  la  manifestación  más  común  de  HDC  causada  por  un  mal  
desarrollo  pulmonar  asociado.  Al  nacer  o  poco  tiempo  después,  el  lactante  desarrolla  disnea  intensa,  
retracciones  y  cianosis.  En  el  examen  físico,  los  ruidos  respiratorios  están  disminuidos  en  el  lado  
ipsilateral;  los  sonidos  del  corazón  se  escuchan  más  fácilmente  en  el  tórax  contralateral;  y  el  abdomen  
es  escafoides  debido  a  la  herniación  de  las  vísceras  abdominales  hacia  el  tórax.
El  desplazamiento  del  mediastino  puede  resultar  en  una  alteración  del  retorno  venoso  y  del  gasto  cardíaco  (CO).

3.  ¿Cómo  se  confirma  el  diagnóstico?
Una  radiografía  de  tórax  muestra  múltiples  asas  de  intestino  lleno  de  aire  en  el  tórax  ipsilateral.
Si  se  obtiene  una  radiografía  de  tórax  antes  de  la  entrada  de  cantidades  significativas  de  aire  en  el  
intestino,  se  puede  observar  un  patrón  confuso  de  desviación  del  mediastino,  desplazamiento  cardíaco  y  
opacificación  del  hemitórax.  La  inserción  de  una  sonda  nasogástrica  (NG)  seguida  de  una  nueva  radiografía  
de  tórax  a  menudo  muestra  la  sonda  (es  decir,  el  estómago)  en  el  tórax  y  confirma  el  diagnóstico.

4.  ¿Existen  otras  anomalías  asociadas  con  CDH?
El  cincuenta  por  ciento  de  los  bebés  con  CDH  tienen  anomalías  asociadas.  Menos  del  10%  de  los  pacientes  
con  múltiples  anomalías  importantes  concurrentes  sobreviven.  Excluyendo  la  malrotación  intestinal  y  la  
hipoplasia  pulmonar,  las  anomalías  cardíacas  (63%)  son  las  más  frecuentes,  seguidas  de  las  genitourinarias  
(GU;  23%),  gastrointestinales  (GI;  17%),  sistema  nervioso  central  (SNC;  14%)  y  otras  pulmonares  ( 5%)  
anomalías.

5.  ¿Qué  medidas  terapéuticas  se  deben  iniciar  antes  del  transporte  o  la  operación?
Quizás  la  intervención  paliativa  más  sencilla  y  eficaz  es  la  descompresión  del  estómago  con  una  sonda  NG,  
que  evita  una  mayor  distensión  del  intestino  y  la  compresión  pulmonar.
La  intubación  endotraqueal  permite  una  ventilación  y  oxigenación  adecuadas.  Se  emplean  estrategias  de  
protección  pulmonar  de  presiones  ventilatorias  mínimas  (<30  mm  Hg),  ventilación  rápida  (40  a  60  
respiraciones/min)  o  ventilación  oscilatoria  de  alta  frecuencia  para  evitar  el  barotrauma  en  los  pulmones,  
que  en  casos  graves  a  menudo  son  hipoplásicos.  El  acceso  venoso  y  la  reanimación  con  líquidos  completan  
la  reanimación  preliminar.

6.  ¿Qué  es  el  ''período  de  luna  de  miel''?
El  período  de  luna  de  miel  describe  el  intervalo  de  tiempo  en  el  que  un  recién  nacido  demuestra  una  
oxigenación  y  ventilación  adecuadas  en  ausencia  de  una  terapia  médica  máxima.  Independientemente  del  
deterioro  posterior,  un  período  de  luna  de  miel  sugiere  que  la  función  pulmonar  (es  decir,  el  desarrollo  
pulmonar)  es  compatible  con  la  supervivencia.

426
Machine Translated by Google
CAPÍTULO  86  HERNIA  DIAFRAGMATICA  CONGÉNITA  427

7.  Describa  el  abordaje  operativo.
La  CDH  da  como  resultado  un  trastorno  fisiológico  de  los  pulmones  que  no  se  revierte  con  la  
reconstrucción  quirúrgica  del  diafragma.  Por  lo  tanto,  la  reparación  de  CDH  no  es  una  emergencia  quirúrgica.  
El  bebé  debe  estabilizarse  antes  de  intentar  la  reparación  quirúrgica.  Un  abordaje  transabdominal  (laparotomía)  
permite  la  reducción  de  las  vísceras  abdominales  herniadas  del  tórax;  reparación  del  defecto  diafragmático  sin  
visión  obstruida  ni  tensión;  corrección  de  mala  rotación;  y  estiramiento  de  la  cavidad  abdominal;  o  creación  de  
una  hernia  ventral  con  parche  protésico  si  las  vísceras  reducidas  no  se  acomodan  fácilmente  en  el  abdomen.  En  
algunos  centros,  se  informó  la  reparación  toracoscópica  o  laparoscópica  para  la  HDC  del  lado  izquierdo,  pero  no  
se  ha  aceptado  de  manera  uniforme  debido  a  las  alteraciones  en  la  fisiología  impuestas  por  la  insuflación  de  
CO2  en  el  tórax  (neumotórax)  o  el  abdomen  (neumoperioneo)  de  los  bebés  que  ya  se  encuentran  en  estado  
crítico.  enfermo.
El  enfoque  puede  ser  útil  en  bebés  estables  que  requieren  apoyo  mínimo  o  niños  mayores  con  diagnóstico  
tardío  de  CDH.

8.  ¿Cuál  es  la  complicación  más  temida  de  la  hernia  diafragmática?
La  complicación  más  temida  es  la  circulación  fetal  persistente  (PFC).  En  la  CDH,  uno  o  ambos  pulmones  son  
hipoplásicos,  el  lecho  vascular  pulmonar  está  reducido  y  las  arterias  pulmonares  exhiben  paredes  musculares  
engrosadas  que  son  hiperreactivas.  Los  recién  nacidos  con  CDH  son  particularmente  propensos  a  desarrollar  
hipertensión  pulmonar.  La  PFC  surge  de  un  aumento  sostenido  en  la  presión  de  la  arteria  pulmonar.  La  sangre  
se  desvía  de  los  pulmones  y  la  sangre  no  oxigenada  se  desvía  a  la  circulación  sistémica  (derivación  de  derecha  
a  izquierda)  a  través  del  conducto  arterioso  permeable  y  el  agujero  oval  permeable.  La  PFC  produce  hipoxemia,  
acidosis  profunda  y  shock.  La  PFC  se  desencadena  por  acidosis,  hipercarbia  e  hipoxia,  todos  ellos  potentes  
vasoconstrictores  de  la  circulación  pulmonar.

9.  ¿Se  puede  corregir  el  PFC?  ¿Si  es  así,  cómo?
Sí.  Se  utilizan  diversas  estrategias  para  prevenir  o  revertir  la  PFC:  1.  
Monitoreo:  Oximetría  o  muestreo  arterial  (preductal  en  la  extremidad  superior  derecha;
postductal  en  la  extremidad  inferior)  permite  la  detección  temprana  de  derivación  de  sangre  no  
oxigenada  a  la  circulación  sistémica.
2.  Ventilación:  la  hipercapnia  se  corrige  mediante  ventilación  mecánica;  sedación  adecuada;  y,  si  es  necesario,  
parálisis  farmacológica.
3.  Oxigenación:  La  hipoxemia  se  corrige  con  ventilación  adecuada  y  altas  concentraciones  de
oxígeno  inspirado  (generalmente  fracción  de  oxígeno  inspirado  [FiO2]  =  100%).
4.  Reanimación:  la  acidosis  metabólica  se  trata  restaurando  la  perfusión  tisular  adecuada
(líquidos  intravenosos  [IV]  o  sangre,  inotrópicos  y  bicarbonato  de  sodio).
5.  Rescate:  Las  terapias  de  rescate  incluyen  la  administración  de  vasodilatadores  pulmonares  a  través  
del  circuito  ventilatorio  (óxido  nítrico)  o  circulación  sistémica  (priscolina,  prostaglandina  E2),  ventilación  
de  alta  frecuencia  y  oxigenación  por  membrana  extracorpórea  (ECMO).

10.  ¿Cuál  es  la  tasa  de  supervivencia  de  los  pacientes  con  HDC?
La  tasa  de  supervivencia  global  es  del  60%.  Los  principales  determinantes  de  la  supervivencia  son  el  grado  de  
hipoplasia  pulmonar  y  las  principales  anomalías  congénitas  asociadas.  Entre  los  bebés  que  sobreviven  al  período  
neonatal  temprano  sin  disfunción  pulmonar  significativa,  la  tasa  de  supervivencia  se  acerca  al  100%.

11.  ¿La  intervención  en  el  útero  tiene  un  papel  en  el  tratamiento  de  pacientes  con  HDC?
Hasta  la  fecha,  la  cirugía  fetal  para  CDH  sigue  siendo  experimental.  En  un  ensayo  prospectivo  informado  en  
1997,  los  resultados  de  la  reparación  intrauterina  de  HDC  se  compararon  con  la  cirugía  posnatal  convencional  
con  resultados  similares.  Los  investigadores  concluyeron  que  debido  a  que  la  cirugía  fetal  abierta  con  reparación  
diafragmática  no  mejora  la  supervivencia  ni  el  resultado,  la  HDC  diagnosticada  prenatalmente  debe  tratarse  
posnatalmente.  Actualmente  se  están  investigando  técnicas  de  oclusión  traqueal  endoscópica  intrauterina  que  
promueven  el  crecimiento  y  desarrollo  pulmonar  seguido  de  reparación  postnatal  de  la  hernia  diafragmática.
Machine Translated by Google
428  CAPÍTULO  86  HERNIA  DIAFRAGMATICA  CONGÉNITA

BIBLIOGRAFÍA

1.  Chiu  PP,  Sauer  C,  Mihailovic  A  et  al.:  El  precio  del  éxito  en  el  manejo  de  la  enfermedad  diafragmática  congénita
hernia:  ¿la  mejora  de  la  supervivencia  se  acompaña  de  un  aumento  de  la  morbilidad  a  largo  plazo?  J  Pediatr  Surg  41:888­892,  2006.

2.  Clugston  RD,  Greer  JJ:  Desarrollo  del  diafragma  y  hernia  diafragmática  congénita.  Semin  Pediatr  Surg
16:94­100,  2007.

3.  Kinsella  JP,  Ivy  DD,  Abman  SH:  Terapia  vasodilatadora  pulmonar  en  la  hernia  diafragmática  congénita:  hipertensión  pulmonar  aguda,  tardía  
y  crónica.  Semin  Perinatol  29:123­128,  2005.

4.  Kitano  Y:  Intervención  prenatal  por  hernia  diafragmática  congénita.  Semin  Pediatr  Surg  16:101­108,  2007.

5.  Lally  KP,  Lally  PA,  Lasky  RE  et  al .:  El  tamaño  del  defecto  determina  la  supervivencia  en  bebés  con  diagnóstico  diafragmático  congénito
hernia.  Pediatría  120:e651­e657,  2007.

6.  Logan  JW,  Rice  HE,  Goldberg  RN  et  al.:  Hernia  diafragmática  congénita:  revisión  sistemática  y  resumen  de
estrategias  de  práctica  de  la  mejor  evidencia.  JPerinatol  27:535­549,  2007.

7.  Migliazza  L,  Bellan  C,  Alberti  D  et  al.:  Estudio  retrospectivo  de  111  casos  de  hernia  diafragmática  congénita  tratados  con  ventilación  
oscilatoria  temprana  de  alta  frecuencia  y  estabilización  prequirúrgica.  J  Pediatr  Surg  42:  1526­1532,  2007.

8.  Rozmiarek  AJ,  Qureshi  FG,  Cassidy  L  et  al .:  Factores  que  influyen  en  la  supervivencia  de  los  recién  nacidos  con  hernia  
diafragmática  congénita:  el  papel  relativo  del  momento  de  la  cirugía.  J  Pediatr  Surg  39:821­824,  2004.
Machine Translated by Google

TUMORES  ABDOMINALES
CAPITULO  
87

Frederick  M.  Karrer,  MD  y  Denis  D.  Bensard,  MD

1.  ¿Cuáles  son  los  tumores  abdominales  sólidos  malignos  más  frecuentes  en  niños?
Neuroblastomas,  tumores  de  Wilms  y  hepatoblastomas,  en  ese  orden.  Los  neuroblastomas  se  derivan  del  tejido  
de  la  cresta  neural;  en  el  abdomen,  se  originan  en  las  glándulas  suprarrenales  y  los  ganglios  simpáticos  
paraespinales.  El  tumor  de  Wilms  (nefroblastoma)  deriva  del  riñón  y  los  hepatoblastomas  se  originan  en  el  hígado.

2.  ¿Es  difícil  diferenciar  clínicamente  el  tumor  de  Wilms  de  los  neuroblastomas?
Sí.  Ambos  tumores  se  presentan  como  una  masa  abdominal  asintomática.  Las  diferencias  se  resumen  en  el  cuadro  
87­1.  Además,  debido  a  que  los  neuroblastomas  producen  hormonas,  los  niños  afectados  pueden  presentar  
enrojecimiento,  hipertensión  (liberación  de  catecolaminas),  diarrea  acuosa,  equimosis  periorbitaria  y  movimientos  
oculares  anormales.

TABLA  87­1.  HAY  DOS  DIFERENCIAS  ENTRE  LM  S'  TUMOR  Y  N  EUROB  LAS  TOM  A
Tumor  de  Wilms Neuroblastoma

Edad  de  presentación 3–4  años 1–2  años

Extender  a  través  de  la  línea  media Extraño Común

Superficie  a  la  palpación Liso nudoso

Calcificaciones  de  rayos  X No Sí

3.  ¿Cómo  se  tratan  los  tumores  de  Wilms  y  los  neuroblastomas?
Consulte  la  tabla  87­2.

CUADRO  87­2.  TR  EA  TM  EN  TOFW  ILMS'  TUMOR  Y  NEUROBLASTOMA
Tumor  de  Wilms Neuroblastoma

Escisión  quirúrgica  primaria Importante  (probable) Importante  (menos  probable)

Quimioterapia Impacto  enorme Menos  receptivo

4.  ¿Cuáles  son  los  principales  factores  pronósticos  en  los  neuroblastomas  y  el  tumor  de  Wilms?
En  los  neuroblastomas,  la  edad  de  presentación  es  el  principal  factor  pronóstico.  Los  niños  menores  de  1  año  
tienen  una  tasa  de  supervivencia  general  >70  %,  mientras  que  la  tasa  de  supervivencia  de  los  niños  mayores  de  1  año  
es  <35  %.  Shimada  propuso  una  clasificación  pronóstica  basada  en  la  evaluación  de  parámetros  histológicos  
(diferenciación  tumoral,  índice  de  mitosis­cariorrexis  [MKI])  y  la  edad.  Los  tumores  aneuploides,  los  tumores  con  MKI  
bajo  y  los  tumores  con  <10  copias  del  gen  n­myc  también  tienen  mejores  resultados.

429
Machine Translated by Google
430  CAPÍTULO  87  TUMORES  ABDOMINALES

La  edad  también  es  importante  en  los  niños  con  tumores  de  Wilms,  pero  el  pronóstico  es  mejor  porque  la
los  tumores  se  extirpan  más  fácilmente  y  son  mucho  más  sensibles  a  la  quimioterapia.

5.  ¿Cuáles  son  las  diferencias  entre  hepatoblastomas  y  hepatocelulares?
carcinomas?  ¿Cómo  se  tratan  los  tumores?
Los  hepatoblastomas  generalmente  ocurren  en  bebés  y  niños  pequeños,  mientras  que  el  carcinoma  hepatocelular  
generalmente  ocurre  en  niños  mayores  de  10  años.  El  carcinoma  hepatocelular  por  lo  general  se  asocia  con  
cirrosis  y  hepatitis  B  y  es  histológicamente  idéntico  a  la  forma  adulta.  La  resección  quirúrgica  es  el  tratamiento  
primario  para  ambos  tumores.  Los  hepatoblastomas  a  menudo  responden  bien  a  la  quimioterapia  complementaria,  
mientras  que  el  carcinoma  hepatocelular  rara  vez  responde  a  la  quimioterapia.

CONTROVERSIA

6.  ¿Deberían  los  pacientes  con  hepatoblastoma  recibir  quimioterapia  preoperatoria  para
reducir  los  tumores?
La  quimioterapia  preoperatoria  reduce  los  tumores,  lo  que  facilita  la  resección  hepática  y  reduce  la  morbilidad  
quirúrgica.  Este  beneficio  debe  sopesarse  frente  a  la  considerable  toxicidad  de  los  agentes  quimioterapéuticos.

PUNTOS  CLAVE:  TUMORES  ABDOMINALES

1.  Los  tumores  abdominales  sólidos  malignos  más  frecuentes  en  niños  son  los  neuroblastomas,  el  tumor  de  
Wilms  y  los  hepatoblastomas.

2.  En  los  neuroblastomas,  la  edad  de  presentación  es  el  principal  factor  pronóstico.

3.  Los  hepatoblastomas  generalmente  ocurren  en  bebés  y  niños  pequeños,  mientras  que  los  hepatocelulares
Los  carcinomas  generalmente  ocurren  en  niños  mayores  de  10  años.

BIBLIOGRAFÍA

1.  Kim  S,  Chung  DH:  Neoplasias  malignas  sólidas  pediátricas:  neuroblastoma  y  tumor  de  Wilms.  Surg  Clin  North  Am
86:469­487,  2006.

2.  La  Quaglia  MP:  Manejo  quirúrgico  del  neuroblastoma.  Semin  Pediatr  Surg  10:132­139,  2001.
3.  Pham  TH,  Iqbal  CW,  Grams  JM  et  al .:  Resultados  del  cáncer  primario  de  hígado  en  niños:  una  evaluación  de  la  experiencia.
J  Pediatr  Surg  42:834­839,  2007.
4.  Pritchard­Jones  K:  Controversias  y  avances  en  el  manejo  del  tumor  de  Wilms.  Arco  Dis  Niño  87:
241­244,  2002.

5.  Ritchey  ML,  Shamberger  RC,  Haase  G  et  al.:  Complicaciones  quirúrgicas  después  de  la  nefrectomía  primaria  por  tumor  
de  Wilms:  informe  del  Grupo  Nacional  de  Estudio  del  Tumor  de  Wilms.  J  Am  Coll  Surg  192:63­68,  2001.
6.  Shimada  M:  Tumores  del  grupo  de  neuroblastoma.  Patología  2:43­59,  1993.
[Artículo  gratuito  de  PMC]  [PubMed]  7.  Tiao  GM,  Bobey  N,  Allen  S  et  al.
resección  convencional  y  trasplante  hepático.  J  Pediatr  148:204­211,  2006.
8.  von  Schweinitz  D:  Manejo  de  tumores  hepáticos  en  la  infancia.  Semin  Pediatr  Surg  15:17­24,  2006.
9.  Wu  HY,  Snyder  HM  3rd,  D'Angio  GJ:  manejo  del  tumor  de  Wilms.  Curr  Opin  Urol  15:273­276,  2005.
Machine Translated by Google

QUISTES  Y  SENOS  CONGÉNITOS
DEL  CUELLO
CAPITULO  
88

Frederick  M.  Karrer,  MD  y  Denis  D.  Bensard,  MD

1.  ¿Qué  son  las  anomalías  de  la  hendidura  branquial?

Quistes,  senos  paranasales  y  fístulas  que  resultan  de  la  obliteración  incompleta  de  las  hendiduras  branquiales  primera,  
segunda  o  tercera  y  están  presentes  en  el  desarrollo  fetal  temprano.

2.  ¿Qué  anomalía  es  la  más  común?
Las  anomalías  de  la  segunda  hendidura  branquial  son,  con  mucho,  las  más  comunes  y  se  presentan  cerca  del  borde  
medio  a  superior  del  músculo  esternocleidomastoideo  (ECM).  Los  restos  del  primer  branquio  son  menos  comunes  y  
las  terceras  hendiduras  son  bastante  raras  ( cuadro  88­1).

3.  ¿Cómo  se  presentan  los  pacientes  con  anomalías  de  la  hendidura  branquial?
Aquellos  con  fístulas  o  senos  paranasales  completos  presentan  drenaje  intermitente  de  un  líquido  mucoide  en  el  cuello.  
Los  pacientes  con  quistes  suelen  presentarse  más  tarde  con  una  masa  (estéril  o  infectada).  La  escisión  quirúrgica  completa  
es  el  tratamiento  de  elección.

TABLA  88­1.  ANOMALÍAS  DE  HENDIDURAS  BRANQUIALES

hendidura  branquial Apertura  Interna Apertura  Exterior Frecuencia

Primero conducto  auditivo  externo Ángulo  de  la  mandíbula 8%

Segundo fosa  amigdalina Borde  anterior  del  SCM  >90%

Tercero seno  piriforme muesca  supraesternal <1%

4.  ¿Cuáles  son  los  principales  riesgos  quirúrgicos  de  la  escisión  del  remanente  de  hendidura  branquial?
La  segunda  hendidura  branquial  discurre  a  través  de  la  bifurcación  de  la  arteria  carótida.  El  nervio  facial  está  muy  cerca  
de  la  primera  fístula  de  la  hendidura  branquial.  El  nervio  laríngeo  superior  y  el  nervio  laríngeo  recurrente  están  en  riesgo  
en  la  disección  de  una  tercera  hendidura  branquial.

5.  ¿Qué  es  un  quiste  del  conducto  tirogloso?
Un  quiste  del  conducto  tirogloso  es  el  quiste  congénito  más  común  que  se  encuentra  en  el  cuello.  Es  causado  por  la  falla  
de  la  obliteración  normal  del  tracto  de  migración  de  la  glándula  tiroides.  Embriológicamente,  la  tiroides  desciende  desde  
la  base  de  la  lengua  (agujero  ciego)  hasta  su  ubicación  normal  en  la  parte  anterior  inferior  del  cuello.

6.  ¿Cómo  se  presentan  los  pacientes  con  quistes  del  conducto  tirogloso?
Se  presentan  con  una  masa  paramidlina  en  la  parte  superior  del  cuello;  si  están  infectados,  pueden  presentar  fiebre,  
sensibilidad  y  eritema.

431
Machine Translated by Google
432  CAPÍTULO  88  QUISTES  Y  SENOS  CONGÉNITOS  DEL  CUELLO

PUNTOS  CLAVE:  QUISTES  Y  SENOS  CONGÉNITOS
DEL  CUELLO
1.  La  anomalía  de  la  hendidura  branquial  más  común  es  la  segunda  anomalía  de  la  hendidura  branquial  que  se  
presenta  cerca  del  borde  medio  a  superior  del  músculo  esternocleidomastoideo.

2.  Un  quiste  del  conducto  tirogloso  es  el  quiste  congénito  más  común  que  se  encuentra  en  el  cuello.

3.  Un  higroma  quístico  es  una  malformación  linfática  congénita  que  es  benigna  y  generalmente  se  presenta
como  una  masa  blanda  en  la  parte  lateral  del  cuello.

7.  ¿Cómo  se  tratan  los  quistes  del  conducto  tirogloso?
El  mejor  tratamiento  es  la  escisión  completa  del  quiste  junto  con  el  tracto.  Dado  que  desde  el  
punto  de  vista  embrionario  la  tiroides  desciende  antes  de  la  formación  del  cartílago  hioides,  el  tracto  puede  pasar  
directamente  a  través  del  hioides.  Por  lo  tanto,  la  extirpación  completa  del  tracto  requiere  la  escisión  de  la  porción  
central  del  hioides  y  la  disección  hasta  la  base  de  la  lengua  (es  decir,  el  procedimiento  de  Sistrunk).

8.  ¿Qué  es  un  higroma  quístico?
Un  higroma  quístico  es  una  malformación  linfática  congénita  con  predilección  por  el  cuello.  Es  una  lesión  benigna  
que  suele  presentarse  como  una  masa  blanda  en  la  parte  lateral  del  cuello.  La  escisión  suele  ser  un  desafío  
porque  los  quistes  linfáticos  no  respetan  los  planos  fasciales  y,  a  menudo,  se  entrelazan  con  las  estructuras  
neurovasculares  del  cuello.  La  escisión  casi  total  es  el  tratamiento  de  elección.

BIBLIOGRAFÍA

1.  Bloom  DC,  Perkins  JA,  Manning  SC:  Manejo  de  malformaciones  linfáticas.  Cabeza  de  otorrinolaringol  Curr  Opin
Cirugía  de  cuello  12:500­504,  2004.

2.  Foley  DS,  Fallat  ME:  Conducto  tirogloso  y  otras  anomalías  cervicales  congénitas  de  la  línea  media.  Semin  Pediatr  Surg
15:70­75,  2006.

3.  Lieberman  M,  Kay  S,  Emil  S  et  al.:  Diez  años  de  experiencia  con  remanentes  branquiales  tercero  y  cuarto.  J  Pediatr  Surg  37:685­690,  
2002.

4.  Okazaki  T,  Iwatani  S,  Yanai  T  et  al.:  Tratamiento  del  linfangioma  en  niños:  nuestra  experiencia  de  128  casos.
J  Pediatr  Surg  42:386­389,  2007.

5.  Ostlie  DJ,  Burjonrappa  SC,  Snyder  CL  et  al.:  Infecciones  del  conducto  tirogloso  y  resultados  quirúrgicos.  J  Pediatr  Surg
39:396­399,  2004.

6.  Schroeder  JW  Jr,  Moyuddin  N,  Maddalozzo  J:  anomalías  branquiales  en  la  población  pediátrica.  Cabeza  de  otorrinolaringol
Cirugía  de  cuello  137:289­295,  2007.

7.  Tracy  TFJr,  Muratore  CS:  Manejo  de  masas  comunes  de  cabeza  y  cuello.  Semin  Pediatr  Surg  16:3­13,  2007.
Machine Translated by Google

X.  TRASPLANTE

TRASPLANTE  DE  HÍGADO
CAPITULO  
89

Jeffrey  Campsen,  MD,  Michael  Zimmerman,  MD  y  Thomas  E.  Bak,  MD

1.  ¿Cuándo  y  dónde  se  realizó  el  primer  trasplante  de  hígado?
El  Dr.  Thomas  Starzl  realizó  la  primera  operación  el  1  de  marzo  de  1963  en  la  Universidad  de  Colorado  en  Denver.  El  
receptor  era  un  niño  de  3  años  con  atresia  biliar.

2.  ¿Se  considera  que  el  trasplante  de  hígado  es  una  operación  segura  y  eficaz?
Sí.  Aunque  sigue  siendo  una  operación  importante  con  riesgos  significativos,  la  supervivencia  del  paciente  y  del  injerto  
ha  mejorado  continuamente.  La  supervivencia  al  año  ya  los  5  años  debería  estar  muy  por  encima  del  90  %  y  el  70  %,  
respectivamente,  en  los  principales  centros.

3.  ¿Cuáles  son  las  indicaciones  más  comunes  para  el  trasplante  de  hígado  en  los  Estados  Unidos?
estados?

La  cirrosis  no  colestásica  caracteriza  >50%  de  los  receptores.  Este  grupo  incluye  a  aquellos  con  hepatitis  viral,  
cirrosis  alcohólica  (Laennec)  y  síndrome  de  Budd­Chiari.  La  cirrosis  colestásica  constituye  un  15  %  adicional,  y  la  
colangitis  esclerosante  primaria  (CEP),  la  cirrosis  biliar  primaria  y  la  hepatitis  autoinmune  encabezan  este  grupo.  Otras  
indicaciones  incluyen  atresia  biliar,  necrosis  hepática  aguda,  neoplasias  malignas  y  enfermedades  metabólicas.

4.  ¿La  enfermedad  más  común  que  requiere  trasplante  se  ha  desplazado  a  lo  largo  del
¿años?
Sí.  El  mayor  porcentaje  de  personas  que  ahora  reciben  trasplantes  tienen  hepatitis  C.  También  se  realizan  más  
retrasplantes  porque  algunas  enfermedades,  como  la  hepatitis  C  y  la  colangitis  esclerosante  primaria,  pueden  
reaparecer  en  los  hígados  trasplantados.  La  atresia  de  vías  biliares  es  la  principal  indicación  de  trasplante  en  niños.

5.  ¿Cómo  se  gestiona  la  lista  de  espera?
Se  han  realizado  cambios  en  la  lista  para  que  los  pacientes  más  enfermos  sean  trasplantados  primero.  Se  ha  
diseñado  un  nuevo  sistema  de  puntuación  (Modelo  para  la  puntuación  de  la  enfermedad  hepática  en  etapa  terminal  
[MELD])  para  dar  más  peso  a  los  marcadores  objetivos  de  la  enfermedad,  que  incluyen  la  creatinina  sérica,  la  
bilirrubina  y  el  índice  internacional  normalizado  (INR);  en  lugar  de  los  criterios  médicos  más  subjetivos  utilizados  en  el  
pasado.  La  puntuación  MELD  proporciona  las  mejores  estimaciones  disponibles  de  mortalidad  a  los  3  meses  para  la  
enfermedad  hepática.  Este  sistema  de  puntos  también  ha  minimizado  la  importancia  del  tiempo  pasado  en  la  lista  de  espera.
El  objetivo  de  estos  cambios  es  reducir  la  mortalidad  en  lista  de  espera.

6.  ¿Cuáles  son  algunos  de  los  avances  recientes  en  la  cirugía  de  trasplante  de  hígado?
Las  técnicas  quirúrgicas  han  mejorado  de  tal  manera  que  algunos  receptores  de  trasplante  de  hígado  no  requieren  una  
estadía  en  la  unidad  de  cuidados  intensivos  (UCI),  derivación  venovenosa  o  drenaje  biliar  externo,  y  los  tiempos  
operatorios  son  más  cortos  (4  a  5  horas).  Los  medicamentos  inmunosupresores  mejorados  han  reducido  las  tasas  de  
rechazo  y  los  efectos  secundarios.  También  se  realizan  trasplantes  de  donante  vivo  de  fragmentos  de  hígado  del  lado  
derecho  e  izquierdo.

7.  ¿Cuánto  tiempo  se  puede  mantener  un  hígado  "en  hielo"?
La  isquemia  fría  óptima  debe  ser  <12  horas.

433
Machine Translated by Google
434  CAPÍTULO  89  TRASPLANTE  DE  HÍGADO

8.  ¿Cuáles  son  algunas  de  las  complicaciones  posoperatorias  comunes  del  trasplante  de  hígado?
El  sangrado  postoperatorio,  la  infección  y  las  complicaciones  biliares  son  las  más  comunes.  La  falta  de  función  primaria  
(<5%)  y  la  trombosis  temprana  de  la  arteria  hepática  (5%)  son  menos  comunes,  pero  por  lo  general  requieren  un  nuevo  
trasplante  urgente.  La  recurrencia  a  largo  plazo  de  la  enfermedad  puede  requerir  un  nuevo  trasplante  en  un  rango  del  10%  al  
20%.

9.  ¿Qué  es  la  técnica  del  "piggy­back"?
Esta  es  una  técnica  en  la  que  el  hígado  enfermo  del  receptor  se  reseca  cuidadosamente  de  la  vena  cava,  que  se  deja  
in  situ.  A  continuación,  la  cava  superior  del  donante  se  sutura  a  un  manguito  común  de  venas  hepáticas  nativas.  Se  liga  la  cava  
inferior  del  donante.  Con  este  método,  es  posible  realizar  el  trasplante  completo  con  una  oclusión  mínima  o  nula  de  la  vena  
cava,  lo  que  genera  menos  inestabilidad  hemodinámica  intraoperatoria.

10.  ¿Es  una  opción  el  trasplante  de  hígado  de  donante  vivo?
Sí.  Utilizado  inicialmente  en  la  población  pediátrica  con  un  injerto  del  segmento  lateral  izquierdo  de  un  adulto,  este  
procedimiento  se  ha  convertido  en  una  práctica  bastante  común.  El  Lejano  Oriente  ha  tenido  una  gran  cantidad  de  series  
de  injertos  de  lóbulo  izquierdo  de  adulto  a  adulto.  En  otros  lugares,  esto  ha  sido  reemplazado  por  una  operación  de  donación  de  
lóbulo  derecho.  Tanto  los  lóbulos  del  hígado  del  donante  como  del  receptor  se  regeneran  rápidamente  a  un  tamaño  casi  normal.
Los  resultados  en  centros  experimentados  imitan  los  del  trasplante  cadavérico  con  una  supervivencia  similar  del  paciente,  
aunque  con  tasas  más  altas  de  complicaciones  y  retrasplante.  Hay  informes  de  morbilidad  y  mortalidad  tanto  del  donante  como  
del  receptor.

PUNTOS  CLAVE:  TRASPLANTE  DE  HÍGADO
1.  La  indicación  más  común  para  el  trasplante  de  hígado  en  los  Estados  Unidos  es  no  colestático
cirrosis,  generalmente  por  enfermedad  viral.

2.  El  tiempo  óptimo  de  isquemia  fría  para  el  hígado  es  <12  horas.

3.  Los  TIPS  se  pueden  usar  en  posibles  receptores  de  trasplantes  como  un  puente  hacia  el  trasplante.

11.  ¿Cómo  han  mejorado  las  derivaciones  portosistémicas  intrahepáticas  transyugulares  (TIPS)  en  este  campo  de  la  cirugía?

Los  TIPS  se  pueden  utilizar  en  posibles  receptores  de  trasplantes  como  un  puente  hacia  el  trasplante.  Este  procedimiento  es  
bastante  eficaz  para  controlar  la  hipertensión  portal  sin  necesidad  de  una  derivación  quirúrgica  abdominal  mayor.  Una  derivación  
portocava  anterior  complica  un  trasplante  de  hígado,  pero  no  es  una  contraindicación  para  el  trasplante  de  hígado.

CONTROVERSIAS

12.  ¿Se  deben  realizar  trasplantes  hepáticos  en  personas  con  enfermedad  hepática  alcohólica?
Los  centros  de  trasplantes  tienen  criterios  estrictos  de  que  los  receptores  de  trasplantes  de  hígado  inducidos  por  el  alcohol  
deben  someterse  a  pruebas  psicológicas  exhaustivas  y  abstenerse  de  consumir  alcohol  antes  de  ser  colocados  en  la  lista  de  
espera.  La  tasa  de  reincidencia  (es  decir,  pacientes  trasplantados  que  vuelven  a  beber)  sigue  siendo  baja.  Desde  el  punto  de  
vista  financiero,  el  costo  es  comparable,  si  no  menor,  que  el  tratamiento  médico  continuo  de  la  enfermedad  hepática  en  etapa  
terminal.  Actualmente  brindamos  atención  para  otros  problemas  médicos  autoinfligidos,  como  los  fumadores  de  cigarrillos.  El  
público  debe  darse  cuenta  de  que  no  se  está  sacando  a  las  personas  de  los  taburetes  de  la  barra  y  llevándolas  al  hospital  para  
su  trasplante.
Machine Translated by Google
CAPÍTULO  89  TRASPLANTE  DE  HÍGADO  435

13.  ¿Los  pacientes  con  neoplasias  malignas  hepáticas  deben  someterse  a  un  trasplante  de  hígado?
Los  pacientes  con  carcinoma  hepatocelular  (un  solo  tumor  <5  cm  o  hasta  tres  tumores  cada  uno  <3  cm)  se  pueden  
trasplantar  con  éxito  con  buenos  resultados  a  largo  plazo.  Otras  neoplasias  malignas  hepáticas,  incluido  el  colangiocarcinoma,  
por  lo  general  se  consideran  contraindicaciones  para  el  trasplante;  sin  embargo,  la  quimioterapia  neoadyuvante  y  la  
braquiterapia  junto  con  el  trasplante  en  casos  seleccionados  han  mostrado  resultados  prometedores.  Si  los  escasos  
hígados  de  donantes  deben  asignarse  a  estos  pacientes  sigue  siendo  un  tema  complejo.

14.  ¿Deberían  utilizarse  donantes  vivos  de  adulto  a  adulto?
El  campo  en  evolución  del  trasplante  de  hígado  de  donante  vivo  de  adulto  a  adulto  (ALDLTx,  por  sus  siglas  en  inglés)  
requiere  que  un  donante  sano  se  someta  a  una  operación  importante  y  potencialmente  mortal.  Los  beneficios  incluyen  un  
procedimiento  oportuno  que  salva  la  vida  de  un  ser  querido,  lo  que  reduce  el  riesgo  del  receptor  de  no  tener  un  hígado  
cadavérico  disponible  a  tiempo.  La  ética  de  someter  al  donante  a  una  cirugía  mayor  de  resección  hepática  sigue  siendo  
discutible.  Se  informa  que  la  tasa  de  complicaciones  en  los  donantes  es  de  aproximadamente  el  30%,  siendo  las  
complicaciones  biliares  el  tipo  más  común.

15.  ¿Se  deben  utilizar  donantes  en  reposo  para  el  trasplante?
Los  donantes  en  asistolia  (NHBD,  por  sus  siglas  en  inglés)  son  una  forma  relativamente  nueva  de  obtener  más  injertos  de  hígado  de  donantes.

Después  de  retirar  el  apoyo  por  parte  del  equipo  de  atención  primaria  de  un  paciente,  si  la  muerte  ocurre  rápidamente,  
la  recuperación  del  órgano  aún  puede  continuar  en  casos  seleccionados.  Los  NHBD  experimentan  un  paro  cardíaco  antes  
de  que  se  recuperen  los  órganos.  Todavía  se  están  recopilando  datos  a  largo  plazo  sobre  estos  órganos,  pero  las  tasas  de  
complicaciones  biliares  y  las  tasas  de  retrasplante  son  significativamente  más  altas  en  estos  injertos.  Sin  embargo,  con  los  
pacientes  que  mueren  en  la  lista  de  espera,  esta  técnica  amplía  el  grupo  de  donantes  y  permite  más  trasplantes.

SITIO  WEB

www.transplantation­soc.org

BIBLIOGRAFÍA

1.  Bak  T,  Wachs  M,  Trotter  JF  et  al .:  Trasplante  de  hígado  de  donante  vivo  de  adulto  a  adulto  con  injertos  de  lóbulo  derecho:  resultados  
y  lecciones  aprendidas  de  la  experiencia  de  un  solo  centro.  Trasplante  de  hígado  7:680­686,  2001.

2.  Busuttil  R,  Klintmalm,  G,  editores:  Transplantation  of  the  liver,  2.ª  ed.,  Filadelfia,  2005,  Elsevier  Saunders.

3.  Ghabril  M,  Dickson  RC,  Machicao  VI  et  al.:  El  retrasplante  hepático  de  pacientes  con  infección  por  hepatitis  C  se  asocia  con  una  
supervivencia  aceptable  del  paciente  y  del  injerto.  Liver  Transpl  13(12):1717­1727,  2007  4.  Lok  ASF,  Villamil  FG,  McDiarmid  SV,  

editores:  Liver  transplantation  for  viral  hepatitis  [tomo  completo].  Trasplante  de  hígado  8  (suplemento  1),  2002.
Machine Translated by Google

RIÑON  Y  PÁNCREAS
TRASPLANTE
CAPITULO  
90

Jeffrey  Campsen,  MD,  Michael  Zimmerman,  MD  y  Thomas  E.  Bak,  MD

1.  ¿Cuáles  son  las  indicaciones  más  comunes  para  el  trasplante  renal?
Enfermedad  renal  en  etapa  terminal  (ESRD)  causada  por  hipertensión,  diabetes,  glomerulonefritis  y  poliquistosis  
renal.

2.  ¿Por  qué  se  debe  sacar  a  los  pacientes  de  la  diálisis  y  someterse  a  trasplantes  de  riñón?
Aunque  no  es  un  trasplante  que  salve  vidas  como  el  trasplante  de  hígado  o  de  corazón,  el  trasplante  de  riñón  mejorará  
la  calidad  de  vida  de  los  pacientes.  La  supervivencia  del  paciente  a  5  años  es  mayor  después  del  trasplante  en  
comparación  con  la  continuación  de  la  diálisis.  Finalmente,  hay  un  ahorro  de  costos  con  el  trasplante  de  riñón  en  
comparación  con  la  diálisis  a  largo  plazo.

3.  ¿Cuál  es  la  supervivencia  del  injerto  renal?
Las  tasas  de  supervivencia  del  trasplante  de  riñón  cadavérico  han  mejorado  constantemente  a  lo  largo  de  los  años.  
Actualmente,  la  supervivencia  del  injerto  a  1  año  es  del  90  %,  con  una  supervivencia  del  injerto  a  10  años  >50  %.  La  supervivencia  
global  a  5  años  para  los  trasplantes  de  riñón  de  donante  vivo  es  >75  %,  que  es  mayor  que  la  del  cadáver.

4.  ¿Cuánto  tiempo  se  pueden  mantener  los  riñones  "en  hielo"?
Los  riñones  pueden  sobrevivir  y  funcionar  después  de  un  tiempo  de  isquemia  fría  más  largo  que  otros  órganos  sólidos.  
La  función  se  puede  mantener  hasta  72  horas,  aunque  la  función  óptima  se  logra  si  la  isquemia  fría  se  mantiene  por  
debajo  de  las  24  horas  (la  mayoría  de  los  centros  trasplantan  antes  de  las  36  horas).  El  "bombeo"  de  riñones  cadavéricos  
con  máquinas  de  perfusión  es  una  forma  potencial  de  aumentar  el  tiempo  de  isquemia  fría  y  disminuir  la  función  
retrasada  del  injerto.  Los  pacientes  en  la  lista  de  espera  con  frecuencia  continúan  trabajando  y  viajando  y  todavía  tienen  
mucho  tiempo  para  llegar  al  hospital  para  el  trasplante.  Además,  los  riñones  de  United  Network  of  Organ  Sharing  (UNOS)  
se  envían  con  frecuencia  a  través  de  aerolíneas  comerciales  en  todo  el  país.

5.  ¿Dónde  se  coloca  el  riñón  trasplantado?
Más  comúnmente,  el  riñón  se  coloca  en  la  fosa  ilíaca  derecha.  La  cavidad  peritoneal  se  refleja  
superiormente,  no  se  ingresa,  y  los  vasos  ilíacos  externos  quedan  expuestos.  A  continuación,  la  arteria  y  la  vena  renales  
se  anastomosan  terminolateralmente  a  los  vasos  ilíacos  y  el  uréter  se  anastomosa  directamente  en  la  vejiga.

6.  ¿Cuáles  son  las  indicaciones  de  la  nefrectomía  nativa?
Las  indicaciones  para  la  nefrectomía  incluyen  infección  crónica,  poliquistosis  renal  sintomática,  hipertensión  
intratable  y  proteinuria  abundante.  La  mayoría  de  los  receptores  de  trasplantes  no  necesitan  someterse  a  nefrectomías  
nativas.

7.  ¿Se  recomiendan  los  trasplantes  de  riñón  de  donante  vivo?
Existen  claras  ventajas  de  recibir  un  riñón  de  un  donante  vivo.  Los  tiempos  de  supervivencia  promedio  de  estos  
riñones  son  significativamente  mejores,  75  %  a  los  5  años  versus  65  %  para  los  riñones  cadavéricos.  Además,  se  
puede  evitar  el  largo  tiempo  de  espera  del  riñón  cadavérico  (normalmente  medido  en  años).  Los  donantes  son  
cuidadosamente  evaluados  para  garantizar  la  salud  y  la  ausencia  de  cualquier  tipo  de  coerción.

436
Machine Translated by Google
CAPÍTULO  90  TRASPLANTE  DE  RIÑÓN  Y  PÁNCREAS  437

8.  ¿La  donación  de  un  riñón  es  una  operación  importante  para  los  donantes  vivos?
El  estándar  de  atención  para  las  operaciones  de  donantes  se  ha  convertido  en  una  nefrectomía  de  donante  laparoscópica.
Esta  técnica  ha  demostrado  ser  segura,  sin  efectos  negativos  sobre  el  riñón.  Los  beneficios  de  esta  modificación  sobre  la  
técnica  abierta  son  una  recuperación  mucho  más  rápida  y  un  tiempo  de  regreso  al  trabajo  más  corto.  En  general,  esto  ha  
aumentado  el  número  de  personas  interesadas  en  ser  donantes  vivos.
Las  complicaciones  más  comunes  son  las  infecciones  de  heridas  y  las  hernias.

9.  ¿Cuáles  son  las  indicaciones  del  trasplante  de  riñón­páncreas?
En  general,  todos  los  diabéticos  tipo  1  que  tienen  una  diabetes  mal  controlada  a  pesar  de  un  manejo  médico  óptimo  deben  
ser  considerados  para  trasplante  de  riñón­páncreas  (KP)  siempre  que  los  riesgos  quirúrgicos  sean  aceptables.  
Desafortunadamente,  muchos  pacientes  mayores  tienen  comorbilidades  significativas,  incluso  prohibitivas.  Un  trasplante  de  
páncreas  agrega  riesgos  significativos  de  morbilidad  y  mortalidad  sobre  un  trasplante  solo  de  riñón.

10.  ¿Puede  un  paciente  someterse  a  un  trasplante  de  páncreas  antes  o  después  de  un  trasplante  de  riñón?
Sí.  Los  pacientes  pueden  recibir  un  trasplante  KP  simultáneo  (SKP).  Este  es  el  curso  más  común  y  la  operación  se  
realiza  a  través  de  una  incisión  abdominal  en  la  línea  media  con  el  páncreas  y  el  riñón  colocados  en  los  vasos  ilíacos  opuestos.  
Los  pacientes  también  pueden  recibir  un  trasplante  de  páncreas  solamente  (PTA)  o  un  trasplante  de  páncreas  después  de  un  
riñón  (PAK).  La  supervivencia  de  estos  injertos  es  de  5  años:  SKP  es  del  69%  y  PAK  y  PTA  del  58%.  El  SPK  tiene  una  mayor  
supervivencia  del  injerto  porque  los  médicos  de  trasplante  pueden  controlar  el  rechazo  del  páncreas  en  los  trasplantes  de  SPK  
a  través  del  riñón.
Algunos  centros  ahora  están  cambiando  al  drenaje  portal  del  páncreas,  con  el  drenaje  venoso  establecido  en  la  vena  
mesentérica  superior.

PUNTOS  CLAVE:  RIÑON  Y  PÁNCREAS
TRASPLANTE
1.  La  indicación  más  común  para  el  trasplante  de  riñón  es  la  ESRD  causada  por  hipertensión,
diabetes,  glomerulonefritis  y  poliquistosis  renal.

2.  Las  tasas  de  supervivencia  del  trasplante  de  riñón  cadavérico  han  mejorado  constantemente  a  lo  largo  de  los  años,  con  
tasas  actuales  de  supervivencia  del  injerto  a  1  año  del  90  %  y  una  tasa  de  supervivencia  del  injerto  a  10  años  de  >50  %.

3.  Los  trasplantes  de  riñón  de  donante  vivo  tienen  una  tasa  de  supervivencia  del  injerto  significativamente  mayor  que  
los  riñones  de  cadáver.

4.  En  general,  todos  los  diabéticos  tipo  1  con  diabetes  mal  controlada  a  pesar  de  un  tratamiento  médico  óptimo
Se  debe  considerar  el  manejo  para  el  trasplante  de  KP  siempre  que  sean  riesgos  quirúrgicos  aceptables.  SKP  tiene  una  
mayor  supervivencia  del  injerto  pancreático  a  los  5  años  que  PTA  y  PAK.
Actualmente  existe  un  debate  significativo  sobre  si  el  riesgo  de  PAK  está  justificado.

11.  ¿Cómo  se  drenan  las  enzimas  digestivas  en  un  trasplante  de  páncreas?
El  páncreas  del  donante  se  obtiene  con  un  manguito  duodenal  aún  adherido,  con  el  drenaje  enzimático  del  injerto  hacia  
este  manguito  intacto.  Luego,  el  manguito  duodenal  se  drena  en  una  porción  del  intestino  delgado  del  receptor  con  una  
anastomosis  entérica.  Una  alternativa  es  unir  el  manguito  duodenal  a  la  vejiga.  Esto  permite  hacer  un  seguimiento  de  los  

niveles  de  amilasa  en  la  orina,  pero  las  complicaciones  metabólicas  e  infecciosas  requieren  con  frecuencia  una  conversión  a  
drenaje  entérico.

12.  ¿Cuáles  son  algunas  de  las  complicaciones  que  se  observan  comúnmente  con  el  trasplante  de  páncreas?
Las  fugas  del  manguito  duodenal,  la  trombosis  venosa  del  injerto,  la  infección,  el  rechazo  y  la  pancreatitis  del  injerto  
son  posibles  complicaciones.  El  rechazo  en  PTA  es  difícil  de  detectar  temprano.  Para  el  momento
Machine Translated by Google
438  CAPÍTULO  90  TRASPLANTE  DE  RIÑON  Y  PÁNCREAS

hay  aumentos  detectables  de  glucosa,  amilasa  u  otros  marcadores,  a  veces  es  demasiado  tarde  para  salvar  el  injerto.  
SKP  puede  mejorar  la  vigilancia  porque  la  creatinina  sérica  es  un  indicador  sensible  de  rechazo.  La  incidencia  de  estas  
complicaciones  está  disminuyendo  a  medida  que  se  gana  más  experiencia,  y  la  supervivencia  del  injerto  de  páncreas  
ahora  se  acerca  a  la  supervivencia  del  injerto  de  riñón.

CONTROVERSIAS

13.  ¿Sigue  siendo  importante  la  compatibilidad  con  el  antígeno  leucocitario  humano?
Es  algo  importante.  Históricamente,  la  compatibilidad  del  antígeno  leucocitario  humano  (HLA)  era  una  
consideración  importante  al  comparar  los  riñones  de  cadáveres  con  los  receptores.  HLA­DR  es  el  más  importante  
en  general.  Con  los  agentes  inmunosupresores  mejorados  de  la  actualidad,  muchos  cirujanos  de  trasplantes  creen  
que  la  coincidencia  de  HLA  ya  no  es  crítica.  Seis  riñones  compatibles  con  antígenos  todavía  se  comparten  a  nivel  
nacional  y  disfrutan  de  cierta  mejora  en  la  supervivencia  del  injerto  a  largo  plazo.  La  calidad  del  órgano  del  donante  
sigue  siendo  el  principal  determinante  del  buen  funcionamiento  del  órgano  trasplantado.
Por  ejemplo,  un  riñón  pobremente  compatible  de  un  donante  vivo  aún  durará  más  que  un  riñón  cadavérico  bien  
compatible.  El  panel  de  anticuerpos  reactivos  (PRA)  detecta  los  anticuerpos  del  receptor  preformados  utilizando  un  
panel  de  células  de  tipificación  y  obtiene  un  porcentaje  de  células  con  las  que  reacciona  el  suero.  Este  porcentaje  ahora  
también  se  usa  para  ayudar  a  determinar  el  manejo  de  la  inmunosupresión.

14.  ¿El  trasplante  de  páncreas  detiene  la  progresión  de  la  enfermedad  diabética?
Esto  aún  no  está  probado.  Lógicamente,  esperaríamos  que  así  fuera.  Se  ha  informado  la  regresión  de  la  
neuropatía  y  la  disfunción  ocular.  Recientemente,  los  receptores  a  largo  plazo  han  mostrado  cierta  regresión  de  la  
nefropatía  microscópica.

15.  ¿Son  los  trasplantes  de  células  de  los  islotes  la  respuesta  en  el  futuro?
Probablemente,  aunque  esto  ha  sido  frustrantemente  lento  de  lograr.  Los  protocolos  recientes  que  utilizan  nuevos  
regímenes  inmunosupresores  y  nuevas  técnicas  de  aislamiento  de  células  de  los  islotes  se  han  mostrado  prometedores,  
pero  los  datos  a  largo  plazo  aún  no  están  ampliamente  disponibles.  El  proceso  requiere  que  las  células  aisladas  de  los  
islotes  se  extraigan  del  páncreas  de  un  donante.  La  mayoría  de  los  pacientes  necesitan  una  masa  total  de  células  de  
los  islotes  que  debe  prepararse  a  partir  de  dos  o  más  donantes  para  volverse  independientes  de  la  insulina.  Estas  
células  luego  se  inyectan  en  la  vena  porta,  se  alojan  en  el  hígado  y  producen  insulina.  En  teoría,  los  pacientes  obtienen  
el  beneficio  de  un  trasplante  de  páncreas  sin  el  riesgo  quirúrgico;  sin  embargo,  se  han  informado  complicaciones  que  
incluyen  hemorragia  hepática,  infarto  hepático,  hemorragia  esplénica,  hipertensión  portal  y  trombosis  de  la  vena  porta.  
Hasta  que  se  resuelvan  los  problemas  logísticos  y  de  producción  (masa)  de  células  de  los  islotes,  el  trasplante  de  
páncreas  seguirá  siendo  el  uso  más  eficiente  de  la  mayoría  de  los  órganos  de  donantes.

SITIO  WEB

www.transplantation­soc.org

BIBLIOGRAFÍA

1.  Bartlett  ST:  Nefrectomía  del  donante  laparoscópica  después  de  siete  años.  Am  J  Transpl  2:896­897,  2002.

2.  Donovitch  G:  Manual  de  trasplante  renal,  3.ª  ed.,  Filadelfia,  2001,  Lippincott  Williams  &  Wilkins.

3.  Fiorina  P,  Secchi  A:  Trasplante  de  células  de  los  islotes  pancreáticos  para  el  tratamiento  de  la  diabetes.  Endocrinol  Metab  Clin  North  Am
36(4):999­1013,  2007.

4.  Lipshutz  GS,  Wilkinson  AH:  Trasplante  de  páncreas­riñón  y  páncreas  para  el  tratamiento  de  la  diabetes  mellitus.  Endocrinol  
Metab  Clin  North  Am  36(4):1015­1038,  2007.
Machine Translated by Google

TRASPLANTE  DE  CORAZÓN CAPITULO  
91

Paul  R.  Crisostomo,  MD,  Alexandra  McMillan  y  Daniel  R.  Meldrum,  
MD,  FACS

1.  ¿Quién  realizó  el  primer  trasplante  experimental  de  corazón  y  pulmón?
Alexis  Carrel,  un  cirujano  estadounidense  nacido  en  Francia,  desarrolló  las  técnicas  vasculares  requeridas  
para  el  trasplante  de  corazón  y  pulmón  y  realizó  el  primer  trasplante  experimental  de  corazón  y  pulmón  en  
1907.  Trasplantó  los  pulmones,  el  corazón,  la  aorta  y  la  vena  cava  de  un  bebé  de  1  semana.  gato  en  el  cuello  
de  un  gato  adulto  grande.  Por  idear  la  técnica  de  la  anastomosis  vascular  y  otros  logros  destacados,  Carrel  
recibió  el  Premio  Nobel  en  1912  (el  primer  Premio  Nobel  otorgado  a  un  científico  que  trabajaba  en  un  laboratorio  
estadounidense).

2.  ¿Quién  realizó  el  primer  trasplante  ortotópico  de  corazón  y  pulmón  experimental?
Vladimir  Demikhov  realizó  el  primer  trasplante  intratorácico  de  corazón  y  pulmón  en  un  perro  en  1962.

3.  Quién  desarrolló  la  primera  estrategia  quirúrgica  requerida  para  el  corazón  humano
¿trasplante?
Norman  Shumway.

4.  ¿Quién  realizó  el  primer  trasplante  de  corazón  humano?  ¿Cuando?
Christian  Barnard  realizó  el  primer  trasplante  de  corazón  humano  en  diciembre  de  1967  (en  Ciudad  del  
Cabo,  Sudáfrica,  después  de  visitar  al  Dr.  Shumway),  aunque  el  Dr.  Shumway  sentó  las  bases  al  desarrollar  
la  técnica  en  animales.  Shumway  y  el  grupo  de  Stanford  realizaron  el  primer  trasplante  de  corazón  en  los  
Estados  Unidos  y  lograron  la  primera  serie  clínica  exitosa.

5.  ¿Quién  realizó  con  éxito  el  primer  trasplante  de  corazón  y  pulmón?  ¿Cuando?
Dr.  Bruce  Reitz  en  Stanford  en  1981  en  una  mujer  de  21  años  con  hipertensión  pulmonar  secundaria  a  un  
defecto  del  tabique  auricular.

6.  ¿Cuántos  trasplantes  de  corazón  se  realizan  anualmente?  ¿Está  aumentando  el  número
o  disminuyendo?
En  1995  se  realizaron  aproximadamente  6500  trasplantes  de  corazón  en  todo  el  mundo.  Para  2003,  el  número  
había  disminuido  a  aproximadamente  5000  y  permanece  relativamente  estable  entre  5000  y  5500.

7.  ¿Qué  anastomosis  (conexiones  quirúrgicas)  se  deben  realizar  para  una  combinación
trasplante  de  corazón  y  pulmones?
La  operación  requiere  solo  una  anastomosis  de  aurícula  derecha  a  cava  (entrada)  y  una  anastomosis  aórtica  
(salida)  con  una  conexión  en  la  tráquea.  El  trasplante  de  corazón  y  pulmón  es  menos  complicado  (menos  
anastomosis)  que  el  trasplante  de  corazón  solo,  lo  que  puede  explicar  por  qué  se  intentó  primero  el  trasplante  
de  corazón  y  pulmón.

8.  ¿Qué  anastomosis  hay  que  realizar  para  un  trasplante  de  corazón?
Aurícula  izquierda,  cava  derecha,  aórtica  y  arterial  pulmonar.

9.  ¿Cuál  es  la  técnica  quirúrgica  preferida?
El  trasplante  de  corazón  ortotópico  es  el  estándar.  La  técnica  de  Shumway  inferior  «clásica»  consistía  en  una  
anastomosis  biauricular.  Sin  embargo,  estudios  posteriores  encontraron  que  la  técnica  biauricular

439
Machine Translated by Google
440  CAPÍTULO  91  TRASPLANTE  DE  CORAZÓN

produce  distorsión  de  la  geometría  auricular  y  llenado  ventricular  anormal,  disfunción  del  ventrículo  derecho,  
insuficiencia  de  la  válvula  auriculoventricular  (AV)  y  disfunción  del  nódulo  sinusal.  Actualmente,  la  técnica  bicava  (se  
realizan  dos  anastomosis  de  la  vena  cava  por  separado  mientras  que  la  anastomosis  de  la  aurícula  izquierda  se  sigue  
realizando  en  manguito)  es  el  procedimiento  más  común.

10.  ¿Quién  es  un  donante  cardíaco  aceptable?
Los  donantes  cardíacos  aceptables  cumplen  con  los  siguientes  
criterios:  1.  Edad  <55  2.  Ausencia  de  enfermedad  cardíaca  y  
pocos  factores  de  riesgo  de  enfermedad  arterial  coronaria  (CAD).
3.  Electrocardiograma  (ECG)  y  ecocardiograma  normales.
4.  Corazón  normal  por  inspección  durante  la  recuperación  de  órganos.
Y  criterios  estándar  de  donante  de  trasplante:  1.  
Requisitos  para  muerte  encefálica.
2.  Consentimiento  del  pariente  más  cercano.

3.  Compatibilidad  del  grupo  sanguíneo  ABO  con  el  receptor.
4.  Ausencia  de  virus  de  inmunodeficiencia  humana  (VIH),  hepatitis  o  infecciones  agudas  no  tratadas.
5.  Sin  malignidad  sistémica.

11.  ¿Quién  es  un  receptor  cardíaco  aceptable?
Criterios  de  trasplante  cardíaco:  
&  Función  cardíaca  deteriorada,  cantidad  máxima  de  oxígeno  por  mililitro  por  kilogramo  de  peso  corporal  (VO2max)  
<14  ml/kg,  fracción  de  eyección  (FE)  <25%,  pronóstico  de  supervivencia  <1  año,  irremediable  New  York  Heart  
Association  (NYHA)  clase  IV.
  Sin  hipertensión  pulmonar  fija  (resistencia  vascular  pulmonar  <6  unidades  Wood).
&  Edad  <70  años.
Criterios  generales  de  trasplante:  
&  Ausencia  de  malignidad  activa,  infección  aguda,  VIH  o  trastorno  del  colágeno.
  Función  renal,  hepática  y  del  sistema  nervioso  central  (SNC)  normal.
&  Estado  fisiológico  y  estabilidad  psicosocial  razonables.
&  No  abuso  de  alcohol,  tabaco  o  drogas.

12.  ¿Qué  significa  UNOS?  ¿Cuál  es  la  diferencia  entre  el  estado  I  y  el  estado  II?

pacientes?
UNOS  significa  Red  Unida  para  el  Intercambio  de  Órganos.  Cualquier  corazón  de  donante  disponible  se  ofrecería  
primero  a  pacientes  con  estado  1A  en  la  lista  de  UNOS:  pacientes  que  están  gravemente  enfermos  y  en  la  unidad  de  
cuidados  intensivos  (UCI)  que  requieren  soporte  vital  avanzado  (dispositivo  de  asistencia  ventricular  [VAD],  bomba  
de  balón  intraaórtico  [IABP],  ionotropos,  etc.).  El  estado  1B  es  la  siguiente  prioridad  más  alta;  estos  pacientes  requieren  
apoyo  ionotrópico,  pero  es  posible  que  no  necesiten  cuidados  en  la  UCI.  Los  pacientes  de  estado  2  no  requieren  
ionotropos  y  no  son  hospitalizados  mientras  esperan.

13.  ¿Cuáles  son  las  indicaciones  más  frecuentes  de  trasplante  cardíaco  en  adultos  y  en
¿niños?

En  adultos,  la  CAD  (miocardiopatía  isquémica)  y  la  miocardiopatía  idiopática  representan  cada  una
aproximadamente  el  45%  de  los  trasplantes.
En  los  niños,  las  cardiopatías  congénitas  y  la  miocardiopatía  son  más  frecuentes,  con
el  corazón  izquierdo  hipoplásico  es  la  malformación  congénita  más  común  que  requiere  trasplante  de  corazón,  
deteriora  la  función  cardíaca  y  tiene  un  pronóstico  de  menos  de  1  año  de  vida.

14.  ¿Qué  porcentaje  de  receptores  potenciales  (en  la  lista  de  trasplantes)  muere  mientras  espera  un  trasplante  de  corazón?

Entre  el  15%  y  el  40%.
Machine Translated by Google
CAPÍTULO  91  TRASPLANTE  DE  CORAZÓN  441

15.  ¿Cuáles  son  los  criterios  de  donante  ampliados?  ¿Quién  se  beneficiaría  más?

La  persistente  escasez  de  corazones  de  donantes  ha  obligado  a  ampliar  los  criterios  de  obtención  de  órganos.  
Receptores  de  alto  riesgo  (estado  1)  que  recibieron  corazones  de  donantes  mayores  (edad  >55),  donantes  con  hepatitis  
B  (inmunoglobulina  M  central  [IgM]  negativa)  o  C  positivos,  o  donantes  con  hipertrofia  ventricular  izquierda  leve  (HVI;  grosor  
de  la  pared  ¼  13  mm)  demuestran  una  supervivencia  superior  a  la  de  los  pacientes  que  fueron  incluidos  pero  nunca  
trasplantados.

16.  ¿En  qué  momento  influye  el  tiempo  de  isquemia  del  corazón  del  donante  en  la  mortalidad?
El  tiempo  de  isquemia  del  corazón  del  donante  >6  horas  definitivamente  aumenta  la  mortalidad.  Los  tiempos  de  isquemia  entre  
4  y  6  horas  aturden  el  corazón  del  donante.  La  mayoría  de  los  equipos  de  trasplante  tratan  de  mantener  los  tiempos  de  isquemia  
(desde  la  obtención  del  donante  hasta  la  perfusión  en  el  receptor)  en  <4  horas.

17.  ¿Cuándo  es  apropiado  prolongar  el  tiempo  de  isquemia  del  donante?
El  tiempo  de  isquemia  del  corazón  del  donante  >  4  horas  es  un  predictor  significativo  de  mortalidad  en  adultos.  Sin  
embargo,  en  la  población  pediátrica,  los  tiempos  de  isquemia  del  corazón  del  donante  extendidos  hasta  8,5  horas  pueden  
ser  seguros  y  no  han  mostrado  diferencias  en  la  supervivencia.

18.  ¿Quién  fue  pionero  en  la  preservación  miocárdica  hipotérmica?
Henry  Swan  de  la  Universidad  de  Colorado.  Sumergía  a  los  niños  anestesiados  en  una  bañera  de  agua  helada  antes  de  los  
procedimientos  cardíacos.

19.  ¿Cuáles  son  las  principales  causas  de  muerte  después  de  un  trasplante  de  corazón?
Fracaso  inespecífico  del  injerto  (lesión  por  isquemia­reperfusión,  edema  de  reimplantación,  etc.),  días  a
semanas

Infección  por  no  citomegalovirus  (CMV),  meses
Rechazo  agudo,  meses
Vasculopatía  del  injerto  cardiaco,  años

20.  ¿Cuál  es  el  patrón  de  infección  típico  de  un  paciente  después  del  trasplante?
Primer  mes  postoperatorio:  patógenos  bacterianos  convencionales  encontrados  en  cirugía
pacientes
1  a  4  meses:  patógenos  oportunistas,  especialmente  CVM.  >4  meses:  
tanto  infecciones  convencionales  como  oportunistas.

21.  ¿Cómo  se  previene  el  rechazo  del  aloinjerto  cardíaco?
La  inmunosupresión  consiste  en  terapia  de  inducción  preoperatoria  o  terapia  de  mantenimiento  postoperatoria.  La  terapia  
de  inducción  consiste  principalmente  en  anticuerpos  contra  el  receptor  de  interleucina­2  (CD25)  o  globulina  antitrombocitos.  
OKT3  ya  no  se  recomienda  para  la  inducción  debido  a  una  mayor  incidencia  de  edema  pulmonar,  hipotensión  y  fiebre  alta.  
La  terapia  de  mantenimiento  por  lo  general  consiste  en  un  inhibidor  de  la  calcineurina  (tacrolimus  >  ciclosporina),  un  agente  
antiproliferativo  (micofenolato  de  mofetilo  >  rapamicina)  y  prednisona.

PUNTOS  CLAVE:  CRITERIOS  PARA  UN  CORAZÓN  ACEPTABLE
DONANTES

1.  Edad  <55.

2.  Ausencia  de  enfermedad  cardiaca  y  pocos  factores  de  riesgo  de  EAC.

3.  ECG  y  ecocardiograma  normales.

4.  Corazón  normal  por  inspección  durante  la  recuperación  de  órganos.
Machine Translated by Google
442  CAPÍTULO  91  TRASPLANTE  DE  CORAZÓN

22.  ¿Todos  los  pacientes  trasplantados  deben  recibir  terapia  de  inducción?
No.  Solo  el  50%  de  los  pacientes  que  se  someten  a  un  trasplante  de  corazón  actualmente  reciben  terapia  de  inducción.
Los  pacientes  con  anticuerpos  preformados  circulantes  (mujeres  multíparas,  esternotomía  reoperatoria,  postransfusiones,  
VAD,  etc.)  tienen  riesgo  de  rechazo  temprano  y  pueden  beneficiarse.  Los  pacientes  con  riesgo  de  disfunción  renal  también  
pueden  beneficiarse  porque  la  terapia  de  inducción  puede  permitir  el  uso  tardío  de  inhibidores  de  la  calcineurina  nefrotóxicos.

23.  ¿Influye  el  desajuste  del  antígeno  leucocitario  humano  en  la  incidencia  de  rechazo  después  de  un  trasplante  de  corazón?  
¿Se  realiza  rutinariamente  la  tipificación  del  antígeno  leucocitario  humano  antes  del  trasplante  de  corazón?

Si  y  no.  El  desajuste  del  antígeno  leucocitario  humano  (HLA)  (2  a  6  versus  0  o  1  coincidencia  de  antígeno)  se  asocia  con  
una  mayor  incidencia  de  rechazo.  Sin  embargo,  estudios  recientes  revelan  que  los  desajustes  de  HLA  ya  no  son  factores  de  
riesgo  significativos  para  la  mortalidad  a  los  5  años  como  lo  eran  en  épocas  anteriores.  La  tipificación  de  HLA  tampoco  se  
realiza  de  forma  rutinaria  antes  del  trasplante  de  corazón;  muchos  pacientes  con  enfermedades  cardíacas  tienen  un  mal  
pronóstico  y  requieren  un  trasplante  urgente,  por  lo  que  no  pueden  esperar  a  que  esté  disponible  un  órgano  con  un  alto  
grado  de  compatibilidad  HLA.

24.  ¿Es  necesaria  la  compatibilidad  ABO  para  el  trasplante  cardíaco?
No.  El  rechazo  hiperagudo  resultante  de  la  incompatibilidad  ABO  requiere  que  el  receptor  AB  preformado  y  la  activación  
del  complemento.  Los  lactantes  tienen  un  sistema  del  complemento  incompetente  y  los  títulos  séricos  de  anticuerpos  anti­
A  y  anti­B  suelen  ser  bajos  hasta  el  año  de  edad.  West  y  sus  colegas  describieron  por  primera  vez  una  serie  de  trasplantes  
cardíacos  infantiles  ABO  incompatibles  exitosos.

25.  ¿Cómo  se  diagnostica  el  rechazo  del  aloinjerto  cardíaco?
La  sospecha  clínica  surge  por  arritmia  cardíaca,  fiebre  o  hipotensión  de  nueva  aparición.
El  diagnóstico  depende  de  la  biopsia  endomiocárdica,  que  se  realiza  a  intervalos  regulares  para  detectar  evidencia  
histológica  de  rechazo  antes  de  que  aparezcan  signos  o  síntomas.  El  perfil  de  expresión  génica  de  los  linfocitos  de  sangre  
periférica  es  una  herramienta  no  invasiva  novedosa  y  cada  vez  más  aceptada  para  el  diagnóstico  del  rechazo.  Su  valor  
predictivo  negativo  es  >99%,  evitando  la  necesidad  de  biopsia  en  determinados  escenarios.

26.  ¿Cuál  es  la  complicación  más  grave  de  la  biopsia  endomiocárdica  transvenosa?
La  perforación  cardíaca  ocurre  en  el  0,5%  de  los  casos.  Esto  puede  conducir  rápidamente  a  taponamiento  y  colapso  
circulatorio.

27.  ¿Cuál  es  la  incidencia  de  la  vasculopatía  del  injerto  cardíaco?  ¿Cuáles  son  los  riesgos
factores?

La  vasculopatía  de  aloinjerto  cardíaco  (VAC;  también  conocida  como  vasculopatía  de  trasplante,  CAD  de  trasplante  o  
arteriosclerosis  de  injerto  acelerada)  ocurre  en  más  del  50  %  de  los  pacientes  5  años  después  del  trasplante  y  es  el  factor  
principal  que  limita  la  supervivencia  a  largo  plazo.  Los  factores  de  riesgo  para  CAV  incluyen  el  género  masculino  del  donante  
o  receptor,  la  edad  del  donante  mayor,  la  hipertensión  del  donante,  la  CAD  previa  al  trasplante  del  receptor  y  las  
incompatibilidades  HLA­DR.

28.  ¿Cuál  es  la  diferencia  entre  la  arteriopatía  coronaria  no  relacionada  con  el  trasplante  o  la  aterosclerosis  y  la  
vasculopatía  del  injerto  cardíaco?
A  diferencia  de  la  CAD  sin  trasplante,  la  CAV  es  un  proceso  concéntrico  difuso  que  involucra  vasos  de  tamaño  
grande  y  mediano.  La  lesión  por  conservación,  las  respuestas  aloinmunitarias  (celulares  y  humorales)  y  posiblemente  la  
infección  crónica  por  CMV  pueden  contribuir  a  su  patogenia.

29.  ¿Cómo  se  diagnostica  y  trata  la  CAV?
CAV  se  diagnostica  predominantemente  por  angiograma  y  más  recientemente  por  ultrasonido  intravascular  (IVUS).  La  
terapia  con  estatinas  diltiazem  disminuye  la  incidencia  de  desarrollar  CAV.  En  pacientes  con  CAV  establecida,  la  rapamicina  
puede  reducir  los  eventos  cardíacos  posteriores.  De  lo  contrario,  el  tratamiento
Machine Translated by Google
CAPÍTULO  91  TRASPLANTE  DE  CORAZÓN  443

ha  sido  decepcionante.  El  retrasplante  ya  no  se  recomienda  en  muchos  centros  debido  a  una  tasa  de  
supervivencia  de  1  año  de  solo  55%  y  una  incidencia  de  CAV  recurrente  de  46%.  La  colocación  de  stents  para  
angioplastia  coronaria  ha  sido  principalmente  un  paliativo  como  resultado  de  la  naturaleza  difusa  de  la  CAV.

30.  ¿Se  recomiendan  generalmente  los  inhibidores  de  la  3­hidroxi­3­metilglutaril  coenzima  A  reductasa  ("estatinas")  
para  pacientes  después  de  un  trasplante  cardíaco?
Sí.  La  hiperlipidemia  es  común  después  del  trasplante  cardíaco  y  los  inhibidores  de  la  3­hidroxi­3­metilglutaril  
coenzima  A  (HMG­CoA)  reductasa  reducen  la  incidencia  y  la  gravedad  de  la  CAV.  Además,  las  estatinas  tienen  
un  efecto  temprano  sobre  la  mortalidad,  lo  que  sugiere  que  estos  fármacos  también  pueden  tener  efectos  
inmunosupresores.

31.  ¿Qué  son  los  dispositivos  de  asistencia  ventricular?
Estos  dispositivos  están  diseñados  para  descargar  el  ventrículo  derecho  (RVAD)  o  izquierdo  (LVAD)  mientras  
apoyan  la  circulación  pulmonar  o  sistémica.  Los  pacientes  con  estos  VAD  pueden  ser  ambulatorios  y  los  dispositivos  
se  pueden  usar  durante  semanas  o  meses.  Los  VAD  se  pueden  usar  como  un  puente  para  el  trasplante  (cuando  el  
paciente  está  en  la  lista  para  el  trasplante)  o  como  terapia  de  destino  (cuando  no  se  planea  un  trasplante).

32.  ¿Cuándo  se  debe  realizar  un  trasplante  de  corazón  tras  el  implante  de  un  DAV?
El  momento  ideal  para  el  trasplante  es  entre  2  semanas  y  6  meses  después  de  la  implantación  del  VAD.  Los  pacientes  tienen  una  tasa  

de  supervivencia  de  1  año  del  92  %  si  se  trasplantan  dentro  de  este  período  de  tiempo,  en  comparación  con  el  75  %  en  caso  contrario.

33.  ¿Es  una  realidad  el  trasplante  de  células  madre  para  la  insuficiencia  cardiaca?  ¿Cuáles  son  los  mecanismos
de  su  beneficio?
Los  ensayos  clínicos  como  TOPCARE­CHF  ya  demuestran  que  el  trasplante  de  células  madre  en  pacientes  con  
insuficiencia  cardíaca  crónica  es  seguro.  El  trasplante  de  células  madre  puede  mediar  sus  efectos  a  través  de  la  
regeneración,  la  angiogénesis  y  la  remodelación  beneficiosa  del  tejido  cardíaco  lesionado.  El  trasplante  de  células  
madre  es  una  modalidad  terapéutica  emocionante  y  novedosa  que  puede  mejorar  la  función  miocárdica  en  la  
insuficiencia  cardíaca  y  permitir  la  explantación  de  DAV  sin  necesidad  de  trasplante  cardíaco.

34.  ¿Está  denervado  el  corazón  trasplantado?
Inicialmente  sí,  pero  se  cree  que  la  reinervación  parcial  comienza  en  1  año.  Debido  a  esto,  los  reflejos  anatómicos  
del  corazón  se  atenúan  (p.  ej.,  mayor  frecuencia  cardíaca  en  reposo  debido  a  la  disminución  o  ausencia  del  tono  
vagal).

35.  ¿Se  puede  trasplantar  un  corazón  dos  veces  con  éxito?
Sí.  Meiser  y  sus  colegas  trasplantaron  el  mismo  corazón  por  segunda  vez  el  19  de  marzo  de  1991,  42  horas  
después  del  trasplante  inicial.  Desde  entonces,  otros  han  informado  sobre  un  segundo  trasplante  del  mismo  
corazón.

36.  ¿Qué  es  el  ''trasplante  de  corazón  dominó''?
El  buen  corazón  de  un  receptor  de  corazón­pulmón  se  trasplanta  a  un  paciente  que  requiere  un  trasplante  de  
corazón.  Algunos  pacientes  con  disfunción  pulmonar  primaria  tienen  disfunción  cardíaca  irreversible  secundaria  
(es  decir,  síndrome  de  Eisenmenger);  otros,  sin  embargo,  como  los  pacientes  con  fibrosis  quística,  tienen  una  
buena  función  cardíaca.  Los  pacientes  con  una  buena  función  cardíaca  pueden  servir  como  donantes  y  aumentar  
el  grupo  de  donantes.

37.  ¿El  corazón  es  capaz  de  producir  factor  de  necrosis  tumoral  (TNF)?  ¿Qué  hace  TNF?
tiene  que  ver  con  el  trasplante  de  corazón?
El  factor  de  necrosis  tumoral  (TNF),  típicamente  descrito  como  una  citocina  inflamatoria  derivada  de  macrófagos  o  
monocitos,  también  se  produce  en  grandes  cantidades  en  el  corazón.  El  TNF  liberado  por  el  corazón  después  de  la  
isquemia­reperfusión  probablemente  contribuye  a  la  lesión  inmediata  (disfunción)  y  posiblemente  al  rechazo  
posterior.  Las  estrategias  anti­TNF  son  estrategias  terapéuticas  intuitivamente  prometedoras  (pero  no  documentadas).
Machine Translated by Google
444  CAPÍTULO  91  TRASPLANTE  DE  CORAZÓN

38.  ¿Cuál  es  la  tasa  de  mortalidad  general  a  los  30  días  después  de  un  trasplante  de  corazón?  ¿Cuál  es  el  
desglose  de  la  mortalidad  entre  pacientes  adultos  y  pediátricos?
El  registro  de  la  Sociedad  Internacional  de  Trasplante  de  Corazón  y  Pulmón,  que  tiene  datos  de  
aproximadamente  76.000  trasplantes  de  corazón,  ha  registrado  una  tasa  de  mortalidad  a  30  días  del  10%.
La  tasa  de  mortalidad  a  los  30  días  para  los  receptores  adultos  es  de  alrededor  del  8%;  para  los  receptores  pediátricos,  es  ligeramente  superior.

39.  ¿Cuáles  son  las  tasas  de  supervivencia  actuarial  a  5  y  10  años  para  trasplante  de  corazón?
destinatarios?
Son  75%  y  50%,  respectivamente  (y  la  calidad  de  vida  mejora  dramáticamente).

40.  ¿Qué  trabajo  queda  por  hacer  en  trasplante  cardíaco?
El  futuro  del  trasplante  de  corazón  es  brillante.  Se  está  acelerando  el  conocimiento  adquirido  en  la  protección  
y  la  lesión  por  isquemia­reperfusión  miocárdica  experimental.  Nuevas  y  emocionantes  formas  de  manipular  la  
inflamación  miocárdica  (células  madre)  y  la  inmunología  (p.  ej.,  transducción  de  señales,  terapia  génica,  
quimerismo)  mejorarán  la  función  miocárdica  postoperatoria  y  la  tolerancia  al  injerto.  En  última  instancia,  la  
alteración  genética  de  los  corazones  de  los  donantes  aumentará  el  número  de  donantes.

SITIO  WEB

www.transplantation­soc.org

BIBLIOGRAFÍA

1.  Al­khaldi  A,  Robbins  RC:  Nuevas  direcciones  en  el  trasplante  cardíaco.  Annu  Rev  Med  57:455­471,  2006.

2.  Canter  CE,  Shaddy  RE,  Bernstein  D  et  al.:  Indicaciones  para  el  trasplante  de  corazón  en  enfermedades  cardíacas  pediátricas:  una  
declaración  científica  del  Consejo  de  la  Asociación  Estadounidense  del  Corazón  sobre  Enfermedades  Cardiovasculares  en  los  Jóvenes;  
los  Consejos  de  Cardiología  Clínica,  Enfermería  Cardiovascular  y  Cirugía  Cardiovascular  y  Anestesia;  y  el  Grupo  de  trabajo  interdisciplinario  
de  investigación  sobre  la  calidad  de  la  atención  y  los  resultados.  Circulación  115(5):658­676,  2007.

3.  Crisostomo  PR,  Wang  M,  Markel  TA  et  al .:  Mecanismos  de  células  madre  y  efectos  paracrinos:  potencial  en  cirugía  cardíaca.  Choque  
28:375­383,  2007.

4.  Rahmani  M,  Cruz  RP,  Granville  DJ  et  al .:  Vasculopatía  de  aloinjerto  versus  aterosclerosis.  Circ.  Res.  99(8):801­815,
2006.

5.  Scheule  AM,  Zimmerman  GJ,  Johnston  JK  et  al.:  La  duración  de  la  isquemia  fría  del  injerto  no  afecta  los  resultados  en  los  receptores  
pediátricos  de  trasplante  de  corazón.  Circulación  106(12  Suplemento  1):I163­I167,  2002.

6.  Steinman  TI,  Becker  BN,  Frost  AE  et  al.:  Pautas  para  la  derivación  y  manejo  de  pacientes  elegibles  para  sólido
transplante  de  organo.  Trasplante  71(9):1189­1204,  2001.

7.  Taylor  DO,  Edwards  LB,  Boucek  MM  et  al.:  Registro  de  la  Sociedad  Internacional  de  Enfermedades  del  Corazón  y  los  Pulmones
Trasplante:  vigésimo  cuarto  informe  oficial  de  trasplante  de  corazón  en  adultos,  2007.  J  Heart  Lung  Transplant  26(8):  769­781,  2007.

8.  Uber  PA,  Mehra  MR.  Terapia  de  inducción  en  el  trasplante  cardíaco:  ¿tiene  algún  papel?  J  Corazón  Pulmón  Trasplante
26(3):205­209,  2007.

9.  Wang  M,  Tsai  BM,  Crisostomo  PR  et  al .:  Resistencia  de  señalización  del  receptor  1  del  factor  de  necrosis  tumoral  en  mujeres
miocardio  durante  la  isquemia.  Circulación  114  (1  suplemento):  I282­I289,  2006.

10.  West  LJ,  Pollock­Barziv  SM,  Dipchand  AI  et  al .:  trasplante  de  corazón  ABO  incompatible  en  bebés.  N  Engl  J  Med  344(11):793­800,  2001.

11.  Zaroff  JG,  Rosengard  BR,  Armstrong  WF  et  al.:  Informe  de  la  conferencia  de  consenso:  maximizar  el  uso  de  órganos
recuperado  del  donante  cadáver:  recomendaciones  cardíacas,  28­29  de  marzo  de  2001,  Crystal  City,  Va.  Circulation  106(7):836­841,  2002.
Machine Translated by Google

APOYO  CIRCULATORIO  MECÁNICO CAPITULO  
92

T.  Brett  Reece,  MD,  Anne  Cannon,  RN,  BSN  y  Joseph  C.  
Cleveland,  Jr.,  MD

1.  ¿Cuáles  son  las  indicaciones  del  dispositivo  de  asistencia  ventricular  (VAD)?
a.  Puente  al  trasplante:  Los  pacientes  necesitan  y  son  elegibles  para  un  trasplante,  pero  su
el  curso  clínico  es  tal  que  no  sobrevivirán  hasta  que  haya  un  órgano  donante  disponible.  Por  lo  tanto,  se  requiere  
apoyo  circulatorio  mecánico  para  "cerrar"  la  brecha  hasta  que  se  disponga  de  un  órgano.
El  objetivo  final  para  estos  pacientes  es  el  trasplante  de  corazón  y  deben  estar  en  la  lista  de  trasplantes  antes  
de  la  colocación  de  VAD.
b.  Terapia  de  destino:  los  pacientes  son  aquellos  con  enfermedad  cardíaca  en  etapa  terminal  que  no  son  elegibles  o  
candidatos  para  un  trasplante  de  corazón.  Aunque  no  son  candidatos  a  trasplante  en  el  momento  de  la  implantación  
del  dispositivo,  estos  pacientes  pueden  evolucionar  a  candidatos  a  trasplante  cardíaco  en  algunos  casos  en  los  que  
la  descarga  ventricular  conduce  a  otros  cambios  hemodinámicos,  por  ejemplo,  mejora  de  la  hipertensión  pulmonar.  
Los  objetivos  de  la  terapia  de  destino  son  prolongar  la  vida,  reducir  las  hospitalizaciones  y  mejorar  la  calidad  de  vida.  
La  terapia  de  destino  es  una  alternativa  al  tratamiento  médico  para  quienes  tienen  insuficiencia  cardíaca  de  clase  III  
a  IV.  Para  calificar  para  la  terapia  de  destino,  los  pacientes  deben  recibir  un  manejo  médico  óptimo  (OMM)  durante  al  
menos  60  de  los  últimos  90  días.  La  terapia  de  destino  también  se  conoce  como  "apoyo  duradero  a  largo  plazo".

C.  Recuperación:  Los  pacientes  con  situaciones  agudas  que  pueden  mejorar  entran  en  esta  categoría.  Más  
comúnmente,  esto  implica  un  shock  cardiogénico  posterior  al  infarto  o  la  falta  de  separación  del  bypass  
cardiopulmonar  (CPB).  Los  dispositivos  a  corto  plazo  pueden  permitir  la  recuperación  o  un  tratamiento  posterior  
para  dispositivos  a  más  largo  plazo  o  trasplantes.  Algunos  dispositivos  de  recuperación  utilizan  las  mismas  cánulas  
que  en  la  CEC,  lo  que  facilita  su  implantación.

2.  ¿Cuáles  son  las  contraindicaciones  del  VAD?

a.  Falta  de  apoyo  social  b.  
intelectual  c.  Infección  D.  Tamaño  
(para  Heartmate  XVE)  e.  

Gravedad  de  la  enfermedad

3.  ¿Qué  estudio  debe  realizarse  antes  de  la  colocación  del  VAD?  a.  Es  necesario  enviar  
cultivos.  Recuerde  que  estos  pacientes  suelen  tener  vías  centrales,
Foleys,  etc.,  por  lo  que  la  infección  sigue  siendo  un  gran  problema  para  ellos.
b.  Ecocardiograma  (ECHO)  en  busca  de  coágulos  en  los  ventrículos.  El  trombo  puede  no  excluir
el  paciente  solo,  pero  puede  hacer  que  avivar  sea  más  riesgoso.  ECHO  también  debe  evaluar  el  estado  de  las  válvulas  
si  es  necesario  repararlas  o  reemplazarlas  y  se  puede  hacer  al  mismo  tiempo  que  se  coloca  un  VAD.
Si  se  coloca  una  válvula  mecánica,  debe  cambiarse  por  una  válvula  bioprotésica  en  el  momento  del  implante.  C.  
Optimice  la  nutrición  porque  este  es  un  gran  indicador  del  resultado  a  corto  y  largo  plazo.  d.  Dispositivo  de  asistencia  
ventricular  izquierda  (LVAD):  el  ventrículo  derecho  debe  evaluarse  de  cerca,
porque  algunas  fallas  del  ventrículo  derecho  (VD)  pueden  ser  secundarias  a  la  falla  del  ventrículo  izquierdo  (VI).
Sin  embargo,  es  posible  que  el  ventrículo  derecho  tampoco  tolere  la  colocación  del  VAD.
Lamentablemente,  los  predictores  de  insuficiencia  del  VD  no  descartan  la  necesidad  de  soporte  biventricular.

445
Machine Translated by Google
446  CAPÍTULO  92  APOYO  CIRCULATORIO  MECÁNICO

mi.  Dispositivo  de  asistencia  del  ventrículo  derecho  (RVAD):  la  hipertensión  pulmonar  puede  ser  el  
problema  más  importante  a  definir  en  el  soporte  del  RV.  Si  los  problemas  ventriculares  surgen  de  la  
vasculatura  pulmonar,  es  posible  que  el  VAD  no  pueda  superar  la  resistencia  o  se  desgaste  prematuramente.

4.  ¿Cuál  es  el  manejo  médico  óptimo  antes  de  la  colocación  de  VAD?
Optimización  del  manejo  de  la  insuficiencia  cardíaca  con  bloqueadores  beta,  inhibidores  de  la  ACE,  hidralazina,  diuresas  y  
posiblemente  digoxina,  según  se  indique.

5.  ¿Qué  predice  los  resultados  con  la  colocación  de  VAD?
La  función  renal  preoperatoria,  el  estado  nutricional,  la  ventilación  mecánica,  la  reesternotomía,  la  presión  
venosa  central  (PVC)  elevada,  el  tiempo  de  protrombina  (PT)  y  el  índice  internacional  normalizado  (INR)  han  
demostrado  correlación  con  malos  resultados.  Se  han  desarrollado  varios  puntajes  de  riesgo  para  tratar  de  
predecir  el  resultado  que  utilizan  múltiples  factores  preoperatorios.

6.  ¿Cuánto  duran  los  dispositivos?
La  durabilidad  depende  del  dispositivo.  Los  dispositivos  a  corto  plazo  probablemente  deban  retirarse  o  
cambiarse  por  un  dispositivo  a  más  largo  plazo  dentro  de  1  a  2  semanas  debido  al  mayor  riesgo  de  infección  y  
tromboembolismo.  Los  dispositivos  a  largo  plazo  deben  durar  de  1  a  4  años  o  incluso  más.
Los  dispositivos  a  corto  plazo  que  están  destinados  a  la  recuperación  en  realidad  pueden  durar  varias  
semanas,  según  el  dispositivo.  Un  dispositivo  de  asistencia  ventricular  paracoporeo  (PVAD),  que  no  debe  
confundirse  con  un  dispositivo  de  asistencia  ventricular  percutáneo  (pVAD),  puede  tardar  semanas  o  meses  (es  
decir,  Thoratec  PVAD,  Abiomed  Ventricle).  Los  VAD  extracorpóreos  (es  decir,  CentriMag)  en  realidad  solo  están  
aprobados  por  la  Administración  de  Alimentos  y  Medicamentos  (FDA)  por  horas,  pero  la  mayoría  se  dejan  durante  
días  o  semanas.

7.  ¿Cómo  afecta  la  presencia  de  un  DAV  al  trasplante?
A  pesar  de  la  complejidad  de  la  disección  en  la  extracción  de  los  dispositivos,  los  resultados  del  trasplante  no  se  
ven  comprometidos.  Se  cree  que  esto  es  secundario  a  que  muchos  de  los  pacientes  con  DAV  están  mejor  
estabilizados  que  sus  contrapartes  con  insuficiencia  cardíaca.  Sin  embargo,  el  cirujano  que  realiza  el  explante  
debe  tener  cuidado  con  los  tractos  de  salida,  ya  sean  hacia  la  arteria  pulmonar  o  la  aorta.

8.  ¿Cuáles  son  las  clases  generales  de  dispositivos  que  se  usan  en  la  actualidad  y  cuáles  son  sus  ventajas  
y  desventajas?  a.  Interno  1.  Ventaja:  el  dispositivo  está  protegido  dentro  del  abdomen  y  el  tórax.

2.  Desventaja:  los  dispositivos  requieren  suficiente  espacio  en  el  abdomen  para  el  dispositivo  y  el
la  línea  de  conducción  sigue  siendo  un  problema  tanto  para  la  cicatrización  de  heridas  como  para  la  infección.  Además,  el  
tamaño  de  los  dispositivos  puede  limitar  la  utilización  en  pacientes  más  pequeños.  b.  Externo  1.  Ventaja:  El  tamaño  del  
dispositivo  no  excluye  a  los  pacientes  más  pequeños.

2.  Desventaja:  la  entrada  y  salida  del  dispositivo  atraviesan  la  piel,  que  puede  ser
más  difícil  prevenir  la  infección.  Además,  es  posible  que  los  dispositivos  externos  no  permitan  que  el  
paciente  abandone  realmente  el  hospital.  C.  Flujo  Axial  1.  Ventaja:  En  general  los  dispositivos  son  
mucho  más  pequeños.

2.  Desventaja:  La  pérdida  de  pulsatilidad  crea  un  ambiente  anormal  para  el  sistémico
lecho  vascular.  Además,  el  personal  médico  necesita  más  educación  sobre  cómo  tomar  y  evaluar  los  
signos  vitales  de  los  pacientes  con  estos  dispositivos.  d.  percutáneo

1.  Ventaja:  son  pequeños,  se  pueden  colocar  en  el  laboratorio  de  catéteres  a  través  de  periféricos
y  no  es  necesario  colocarlos  en  CEC  para  su  implantación.  Puede  ser  más  adecuado  para  la  recuperación  
a  corto  plazo.
Machine Translated by Google
CAPÍTULO  92  APOYO  CIRCULATORIO  MECÁNICO  447

2.  Desventaja:  Aún  se  está  evaluando  su  durabilidad.  No  son  dispositivos  a  largo  plazo.
Su  implantación  puede  no  prevenir  la  infección  a  largo  plazo.  Estos  dispositivos  no  aumentan  el  flujo  tan  
bien  como  los  otros  dispositivos.

9.  ¿Cuáles  son  los  problemas  perioperatorios  que  deben  observarse  o  abordarse?  a.  Coagulopatía:  procedimientos  
notorios  por  sangrado  postoperatorio  b.  Estado  del  volumen:  relacionado  con  el  sangrado,  pero  también  
con  el  estado  del  volumen  preoperatorio  c.  Poscarga:  afecta  directamente  la  salida  de  la  bomba,  más  
importante  en  los  dispositivos  de  flujo  axial  d.  Otro  ventrículo:  más  difícil  de  tratar  e.  Trombosis  o  embolia

10.  ¿Qué  se  debe  hacer  para  la  anticoagulación  de  estos  dispositivos?
La  anticoagulación  es  específica  del  dispositivo.  En  su  mayor  parte,  todos  los  pacientes  con  VAD  necesitan  
ser  anticoagulados  con  Coumadin  y  posiblemente  una  larga  lista  de  agentes  antiplaquetarios.  Hay  dispositivos  
que  no  requieren  anticoagulación  más  allá  de  la  aspirina.

11.  ¿Qué  problemas  de  gestión  a  largo  plazo  deben  abordarse?
a.  Enseñar  al  paciente,  la  familia  y  los  servicios  médicos  locales  cómo  hacer  frente  a  la  VAD  b.  Nutrición  
c.  Línea  de  conducción  curativa  estrechamente  relacionada  con  la  nutrición  d.  Anticoagulación  (que  es  
específica  del  dispositivo)  e.  Siga  la  ecocardiografía:  evalúe  la  función  ventricular,  la  apertura  de  la  
válvula,  el  flujo  de  entrada  y

orientación  del  flujo  de  salida  y  cualquier  indicación  de  disfunción  del  dispositivo,  incluida  la  formación  de  
trombos  o  la  degeneración  de  la  válvula  f.  Procedimientos  de  emergencia  con  educación  comunitaria  porque  
estos  temas  pueden  no  presentarse
cerca  de  la  institución  de  implantación

12.  ¿Cuándo  trasplantar  los  puentes?
El  paciente  necesita  recuperarse  de  la  colocación  del  VAD.  Esta  es  una  oportunidad  para  abordar  problemas  generales  
de  volumen  corporal,  mejorar  la  nutrición  del  paciente  y  mejorar  el  estado  de  actividad.  Aunque  los  trasplantes  se  
realizan  poco  después  de  la  colocación  del  dispositivo,  la  decisión  de  incluirlos  en  la  lista  toma  en  cuenta  muchos  
factores  diferentes  que  podrían  mejorar  el  resultado  del  trasplante.  Este  no  es  un  paciente  para  colocar  un  órgano  
marginal  de  un  donante  por  temor  a  que  se  deteriore.  Por  otro  lado,  las  complicaciones  del  dispositivo  pueden  acelerar  
la  necesidad  de  un  trasplante.

13.  ¿Qué  se  debe  evaluar  antes  de  la  explantación?
La  explantación  puede  ser  una  llamada  difícil  de  hacer.  La  función  cardíaca  puede  disminuir  temporalmente  por  el  
proceso  de  explantación,  por  lo  que  debe  haber  alguna  reserva  para  tolerar  la  extracción  en  el  período  perioperatorio.  
La  patología  puede  jugar  un  papel  importante,  especialmente  si  es  posible  la  recuperación  del  miocardio.  ECHO  juega  
un  papel  real  ya  que  el  dispositivo  de  asistencia  se  puede  apagar  para  evaluar  la  función  ventricular  subyacente.  
Finalmente,  la  decisión  de  explantación  del  dispositivo  de  asistencia  debe  ser  multidisciplinaria,  incluyendo  al  paciente  
en  estas  discusiones.

BIBLIOGRAFÍA

1.  Birks  EJ,  Tansley  PD,  Hardy  J  et  al.:  Dispositivo  de  asistencia  del  ventrículo  izquierdo  y  tratamiento  farmacológico  para  la  inversión  del  corazón
falla.  N  Engl  J  Med  355:1873,  2006.

2.  Lietz  K,  Long  JW,  Kfoury  AG:  Resultados  de  la  implantación  del  dispositivo  de  asistencia  ventricular  izquierda  como  terapia  de  destino  en
la  era  post­REMATCH:  implicaciones  para  la  selección  de  pacientes.  Circulación  1165:497­505,  2007.

3.  Rao  V,  Oz  MC,  Flannery  MA  et  al.:  Tendencias  cambiantes  en  la  asistencia  circulatoria  mecánica.  Tarjeta  J  Cirugía  194:
361­366,  2004.

4.  Rose  EA,  Gelijns  AC,  Moskowitz  AJ  et  al .:  Asistencia  ventricular  izquierda  mecánica  a  largo  plazo  para  la  insuficiencia  cardíaca  en  etapa  terminal.  
N  Engl  J  Med  34520:1435­1443,  2001.
Machine Translated by Google

TRASPLANTE  DE  PULMÓN
CAPITULO  
93

Paul  R.  Crisostomo,  MD,  Nadia  McMillan  y  Daniel  R.  Meldrum,  MD,  FACS

1.  ¿Qué  trasplante  de  órgano  humano  se  realizó  primero,  el  corazón  o  el  pulmón?
Aunque  el  trasplante  de  corazón  ha  progresado  más  rápidamente,  el  primer  trasplante  de  pulmón  humano  
precedió  al  primer  trasplante  de  corazón.

2.  ¿Quién  realizó  el  primer  trasplante  de  pulmón  humano?  ¿Cuando?
James  Hardy  realizó  el  primer  trasplante  de  pulmón  humano  en  1963;  sin  embargo,  pasaron  más  de  20  
años  antes  de  que  el  trasplante  de  pulmón  se  realizara  de  forma  rutinaria  en  la  práctica  clínica  (durante  ese  
período  de  20  años,  solo  un  paciente  se  recuperó  lo  suficiente  como  para  abandonar  el  hospital).  Este  retraso  
fue  causado  por  el  fracaso  inicial  del  injerto  secundario  a  la  conservación  inadecuada  de  órganos,  largos  tiempos  
de  isquemia,  falta  de  buenos  agentes  inmunosupresores  y  dificultades  técnicas  (principalmente  con  las  
anastomosis  bronquiales,  no  vasculares).

3.  ¿Cuáles  son  los  tipos  generales  de  trasplantes  de  pulmón?
Simple,  doble  (bilateral)  y  corazón­pulmón.

4.  ¿Cuántos  trasplantes  de  pulmón  se  realizan  anualmente?  ¿Está  aumentando  el  número  o
¿decreciente?
Aunque  se  realizó  por  primera  vez  en  1963,  no  se  realizaron  números  significativos  hasta  finales  de  la  
década  de  1980  (en  1986,  1  trasplante  de  pulmón;  en  1989,  132  trasplantes  de  pulmón).  Desde  1994,  el  
número  de  trasplantes  unipulmonares  realizados  anualmente  se  ha  mantenido  estable  (alrededor  de  700).  
Sin  embargo,  los  trasplantes  de  pulmón  bilaterales  han  aumentado  rápidamente  de  aproximadamente  100  en  
1994  a  más  de  1400  en  2005  y  continúan  aumentando  en  todo  el  mundo.

5.  ¿Por  qué  se  realizan  anualmente  el  número  de  trasplantes  combinados  de  corazón  y  pulmón?
¿decreciente?
En  1990  se  realizaron  aproximadamente  250  trasplantes  de  corazón  y  pulmón;  el  número  ha  disminuido  a  
aproximadamente  75  en  2005.  Con  la  mejora  de  los  resultados  de  los  trasplantes  de  pulmón  único  y  doble,  se  
ha  obviado  la  necesidad  de  realizar  trasplantes  de  corazón  y  pulmón  en  pacientes  con  enfermedad  pulmonar  
aislada.

6.  ¿Quién  es  candidato  para  un  trasplante  de  pulmón?
Los  candidatos  incluyen  pacientes  sin  otra  alternativa  médica  o  quirúrgica  que  probablemente  mueran  de  
enfermedad  pulmonar  dentro  de  2  a  3  años,  sean  menores  de  65  años,  no  dependan  de  un  respirador  y  no  
tengan  antecedentes  de  malignidad.  La  estabilidad  psicológica  en  el  receptor  también  es  importante.

7.  ¿Cuáles  son  las  indicaciones  más  comunes  para  el  trasplante  de  un  solo  pulmón?
Enfisema  (50%)
Fibrosis  pulmonar  idiopática  (25  %)  Déficit  
de  antitripsina  a­1  (7,5  %)
Fibrosis  quística  (FQ;  2  %)
Sarcoidosis  (2%)

448
Machine Translated by Google
CAPÍTULO  93  TRASPLANTE  DE  PULMÓN  449

8.  ¿Cuáles  son  las  indicaciones  más  comunes  para  un  doble  trasplante  de  pulmón?
FC  (30%)
Enfisema  (25%)
Fibrosis  pulmonar  idiopática  (13  %)  Déficit  
de  antitripsina  a­1  (8,5  %)
Hipertensión  pulmonar  primaria  e  hipertensión  pulmonar  secundaria  a  corregible
cardiopatía  congénita  (6%)

9.  ¿Cuáles  son  las  indicaciones  más  comunes  para  el  trasplante  de  corazón  y  pulmón?
La  hipertensión  pulmonar  primaria  (25  %)  y  la  FQ  (15  %)  son  casos  en  los  que  los  pulmones  defectuosos  
han  arruinado  un  buen  corazón.  Por  el  contrario,  con  cardiopatías  congénitas  (34%),  un  mal  corazón  ha  
destruido  buenos  pulmones.

10.  ¿Qué  se  cose  a  qué  durante  un  trasplante  de  un  solo  pulmón?  Un  doble  pulmón
¿trasplante?
Durante  un  trasplante  de  un  solo  pulmón,  se  requieren  anastomosis  entre  el  receptor  y  el  injerto  bronquial,  la  
arteria  pulmonar  y  la  vena  pulmonar  (manguito  auricular).  Las  anastomosis  para  doble  trasplante  son  las  mismas;  
sin  embargo,  el  bypass  cardiopulmonar  (CPB)  se  requiere  con  mayor  frecuencia  durante  el  trasplante  de  doble  
pulmón.  Durante  la  implantación  del  segundo  pulmón,  la  desviación  de  todo  el  gasto  cardíaco  (GC)  hacia  el  
pulmón  recientemente  isquémico  a  menudo  produce  edema  pulmonar  por  reperfusión  e  hipoxemia.

11.  ¿Qué  diagnósticos  conllevan  los  mejores  resultados  para  los  trasplantes  de  un  solo  pulmón?
A  los  pacientes  con  enfisema,  deficiencia  de  antitripsina  a­1  y  FQ  les  va  significativamente  mejor,  con  
tasas  de  supervivencia  a  1  año  de  aproximadamente  75  %.  Sin  embargo,  los  pacientes  con  FQ  o  fibrosis  pulmonar  
idiopática  pueden  obtener  un  mayor  beneficio  de  supervivencia  del  trasplante  de  pulmón  que  los  pacientes  con  
enfermedad  pulmonar  obstructiva  crónica  (EPOC)  porque  la  mortalidad  sin  trasplante  es  mayor.

12.  ¿Las  tasas  de  supervivencia  son  diferentes  para  los  trasplantes  de  un  solo  pulmón  y  de  dos  pulmones?
Sí.  Aunque  las  tasas  de  supervivencia  para  los  receptores  de  trasplantes  únicos  y  bilaterales  son  similares  durante  el  primer  
año,  en  los  años  posteriores,  los  trasplantes  de  pulmón  bilaterales  (vida  media  =  5,9  años)  han  mejorado  significativamente  
la  supervivencia  en  comparación  con  los  trasplantes  de  pulmón  único  (vida  media  =  4,4  años).

13.  ¿Cuáles  son  las  complicaciones  más  frecuentes  tras  un  trasplante  de  pulmón?
Hipertensión  (85%)
Disfunción  renal  (38%)
Hiperlipidemia  (52%)
Diabetes  (33%)
Bronquiolitis  obliterante  (33%)

14.  ¿Cuáles  son  las  principales  causas  de  muerte  después  de  un  trasplante  de  pulmón?
Disfunción  primaria  del  injerto  (días)
Infección  por  no  citomegalovirus  (CMV)  (semanas  a  años)
Bronquiolitis  obliterante  (meses  a  años)

15.  ¿Qué  es  la  disfunción  primaria  del  injerto  (DGP)?  Como  es  tratado?
El  DGP  es  una  forma  de  lesión  pulmonar  aguda  resultante  de  isquemia  o  lesión  por  reperfusión,  edema,  
preservación,  técnica  quirúrgica  y  factores  de  riesgo  del  receptor  o  del  donante.  Se  manifiesta  clínicamente  
como  mala  oxigenación,  distensibilidad  y  edema.  El  tratamiento  consiste  en  aumento  del  soporte  ventilatorio,  
diuréticos,  vasodilatación  pulmonar  (prostaglandinas  y  óxido  nítrico  inhalado),  reemplazo  de  surfactante  (sintético  
nebulizado),  oxigenación  por  membrana  extracorpórea  y  retrasplante.
Machine Translated by Google
450  CAPÍTULO  93  TRASPLANTE  DE  PULMÓN

16.  ¿Cuál  es  la  causa  no  bacteriana  más  común  de  neumonía  en  pacientes  con  trasplante  de  pulmón?

CMV,  que  generalmente  ocurre  de  4  a  8  semanas  después  de  la  operación.  La  infección  primaria  por  CMV  por  lo  general  resulta  
en  una  enfermedad  más  grave  que  la  reactivación  de  la  enfermedad.  Los  receptores  seronegativos  para  CMV  deben  recibir  solo  
hemoderivados  que  sean  negativos  desde  el  punto  de  vista  serológico.

17.  Además  de  la  terapia  inmunosupresora,  ¿qué  otros  factores  ponen  trasplantados
pulmones  en  riesgo  de  infección?
Denervación  pulmonar,  interrupción  del  aclaramiento  linfático  y  de  la  circulación  bronquial,  y  alteración  del  
aclaramiento  mucociliar.

18.  ¿Qué  es  la  bronquiolitis  obliterante?
La  bronquiolitis  obliterante,  una  de  las  principales  causas  de  morbilidad  a  largo  plazo  después  del  trasplante  de  
pulmón,  es  un  proceso  crónico  de  cicatrización  irreversible  que  provoca  la  obliteración  progresiva  de  las  vías  
respiratorias  pequeñas  del  aloinjerto  de  pulmón  y  la  enfermedad  pulmonar  obstructiva  resultante.  Clínicamente  se  
caracteriza  por  disnea  y  obstrucción  al  flujo  aéreo.

19.  ¿Cómo  se  desarrolla  la  bronquiolitis  obliterante?
Los  linfocitos  se  infiltran  en  la  submucosa  de  los  bronquiolos  y  migran  a  través  de  la  membrana  basal  
hacia  la  mucosa  de  las  vías  respiratorias.  Siguen  lesión  alorreactiva  citotóxica  y  necrosis  epitelial.
En  respuesta  a  la  ulceración,  se  forma  un  exudado  fibropurulento  dentro  de  las  vías  respiratorias  y  se  acompaña  de  
proliferación  de  fibroblastos,  células  endoteliales  y  linfocitos.  Este  tejido  mixoide  puede  ocluir  parcial  o  completamente  
las  vías  respiratorias.

20.  ¿Cuáles  son  los  factores  de  riesgo  para  el  desarrollo  de  bronquiolitis  obliterante  después  del  trasplante  de  pulmón?

Los  episodios  de  rechazo  agudo  del  injerto  son  sin  duda  el  factor  de  riesgo  más  importante.  La  infección  por  
CMV,  el  incumplimiento  de  los  medicamentos  inmunosupresores  y  la  bronquitis  linfocítica  también  son  factores  de  
riesgo  importantes.

21.  ¿Cómo  se  previene  el  rechazo  del  aloinjerto  de  pulmón?
Las  prácticas  inmunosupresoras  para  el  trasplante  de  pulmón  generalmente  han  sido  paralelas  a  las  del  trasplante  
de  corazón.  Menos  de  la  mitad  recibe  terapia  de  inducción  (anticuerpos  contra  el  receptor  de  interleucina­2  [CD25]  >  
globulina  antitrombocitos).  La  terapia  de  mantenimiento  por  lo  general  consiste  en  un  inhibidor  de  la  calcineurina  
(tacrolimus  >  ciclosporina),  un  agente  antiproliferativo  (micofenolato  de  mofetilo  >  rapamicina)  y  prednisona.

22.  ¿Cuál  es  la  incidencia  del  rechazo  agudo?  ¿Cómo  es  el  rechazo  del  trasplante  de  pulmón?
diagnosticado?
Casi  el  50%  de  los  receptores  reciben  tratamiento  por  rechazo  agudo  durante  el  primer  año  después  del  trasplante.
A  diferencia  de  los  trasplantes  de  corazón,  el  diagnóstico  de  rechazo  en  los  pulmones  trasplantados  es  impreciso  y  
puede  ser  difícil  de  distinguir  de  una  infección.  La  biopsia  transbronquial  sigue  siendo  el  estándar  de  oro,  pero  a  
menudo  requiere  tres  o  varias  biopsias  "buenas"  más.  La  broncoscopia  con  lavado  broncoalveolar  y  la  evaluación  
clínica  también  son  necesarias  para  el  diagnóstico.

23.  ¿Qué  pruebas  adicionales  pueden  ayudar  a  distinguir  entre  rechazo  agudo  y
¿infección?
Los  ensayos  de  reacción  en  cadena  de  la  polimerasa  (PCR)  para  la  neumonía  por  CMV,  Aspergillus  y  
Pneumocystis  jirovecii  y  la  PCR  múltiple  para  múltiples  agentes  infecciosos  oportunistas  y  adquiridos  en  la  
comunidad  pueden  complementar  la  tinción  de  biopsia  transbronquial  estándar  y  pueden  facilitar  la  discriminación  
entre  infección  oculta  y  rechazo  agudo.
Machine Translated by Google
CAPÍTULO  93  TRASPLANTE  DE  PULMÓN  451

24.  Describa  el  fenómeno  del  quimerismo  en  el  trasplante.
El  quimerismo  mixto  de  trasplante  es  una  afección  en  la  que  las  células  del  receptor  se  injertan  en  el  trasplante  del  
donante,  de  modo  que  el  aloinjerto  se  convierte  en  un  compuesto  genético  tanto  del  donante  como  del  receptor.  El  
quimerismo  aumenta  la  tolerancia  del  huésped  al  injerto  porque  el  receptor  no  reconoce  el  órgano  del  donante  como  
extraño.

25.  ¿Se  desarrolla  el  quimerismo  en  el  corazón  y  los  pulmones?
Sí.  La  primera  evidencia  de  quimerismo  de  trasplante  de  corazón  fue  observada  en  2002  por  Quaini  y  sus  colegas  
en  pacientes  varones  que  recibieron  corazones  de  donantes  mujeres.  En  2003,  Kleeberger  y  sus  colegas  también  
demostraron  evidencia  de  quimerismo  de  trasplante  de  pulmón  en  epitelio  pulmonar,  neumocitos  tipo  II  y  glándulas  
seromucosas.

26.  ¿Por  qué  es  emocionante  el  quimerismo?
La  naturaleza  está  tratando  de  enseñarnos  cómo  realizar  un  trasplante  sin  el  uso  de  inmunosupresores.  
Nuestro  trabajo  es  aprender  por  qué  se  induce  el  quimerismo  en  algunos  receptores  y  no  en  otros.  Es  decir,  debemos  
diseccionar  los  mecanismos  de  inducción  del  quimerismo  para  que  podamos  inducir  terapéuticamente  el  quimerismo  en  
todos  los  receptores.

27.  ¿Cuáles  son  los  principales  tipos  de  soluciones  de  preservación  para  injertos  de  corazón  y  pulmón?
Durante  casi  dos  décadas,  la  solución  Euro­Collins  (EC)  o  la  Universidad  de  Wisconsin  (UW)  se  consideraron  el  estándar  
de  oro  para  el  trasplante  de  pulmón.  Perfadex  se  acepta  cada  vez  más  para  el  trasplante  de  pulmón  y  mejora  la  función  
pulmonar  posterior  al  trasplante  y  disminuye  el  PGD  en  comparación  con  otras  soluciones.
El  estándar  de  oro  para  el  trasplante  de  corazón  es  la  cardioplejía  cristaloide  (paro)  y  la  solución  UW
(preservación).  Celsior  es  una  solución  novedosa  de  combinación  de  detención  y  conservación  para  trasplantes  
de  corazón  que  requiere  más  estudio.

28.  ¿Cuáles  son  las  principales  diferencias  en  la  composición  entre  las  soluciones  de  Euro­Collins  y  la  Universidad  de  
Wisconsin  y  Perfadex  y  Celsior?
EC  y  UW  son  soluciones  de  conservación  intracelular  con  alto  contenido  de  potasio  desarrolladas  originalmente  para  
el  trasplante  de  riñón.  En  el  trasplante  de  pulmón  provocan  una  vasoconstricción  severa.  Perfadex  es  una  solución  de  
dextrano  más  glucosa  extracelular  baja  en  potasio  que  demuestra  menos  vasoconstricción  y  disminución  de  la  formación  
de  edema  intersticial.  Celsior  también  es  extracelular.

29.  ¿Qué  porcentaje  del  flujo  sanguíneo  pulmonar  va  al  pulmón  trasplantado  después  de  un  trasplante  de  un  solo  pulmón?

Como  era  de  esperar,  casi  todo  el  flujo  sanguíneo  pulmonar  pasa  a  través  del  circuito  de  menor  resistencia  del  
pulmón  trasplantado  (dependiendo  de  la  resistencia  vascular  pulmonar  del  nativo  contralateral;  es  decir,  pulmón  
enfermo).  Si  existe  una  gammagrafía  de  perfusión  preoperatoria,  en  igualdad  de  condiciones,  se  preserva  el  pulmón  con  
la  mejor  perfusión  y  se  reemplaza  el  pulmón  dañado.

PUNTOS  CLAVE:  TRASPLANTE  DE  PULMÓN
1.  La  indicación  más  común  para  el  trasplante  de  un  solo  pulmón  es  el  enfisema.

2.  La  indicación  más  común  para  el  trasplante  de  doble  pulmón  es  la  FQ.

3.  El  quimerismo  es  una  condición  en  la  que  las  células  del  receptor  se  injertan  en  el  trasplante  del  donante,  de  modo  
que  el  aloinjerto  se  convierte  en  un  compuesto  genético  tanto  del  donante  como  del  receptor.

4.  La  bronquiolitis  obliterante,  una  de  las  principales  causas  de  morbilidad  a  largo  plazo  después  del  trasplante  de  
pulmón,  es  un  proceso  crónico  de  cicatrización  irreversible  que  produce  la  obliteración  progresiva  de  las  vías  
respiratorias  pequeñas  del  aloinjerto  de  pulmón  y  la  enfermedad  pulmonar  obstructiva  resultante.
Machine Translated by Google
452  CAPÍTULO  93  TRASPLANTE  DE  PULMÓN

30.  ¿Se  requiere  circulación  extracorpórea  para  el  trasplante  de  pulmón?
No.  Sin  embargo,  para  pacientes  con  hipertensión  pulmonar  (primaria  o  secundaria),  la  CEC  se  usa  de  forma  
rutinaria  antes  de  extirpar  el  pulmón  del  receptor.  CPB  está  siempre  en  espera.  Esta  es  una  anestesia  complicada.  Se  
extirpa  temporalmente  un  pulmón  de  un  paciente  que  vive  (apenas)  con  dos  pulmones  dañados.

31.  ¿Es  posible  el  trasplante  de  pulmón  de  pariente  vivo?
Sí.  Los  trasplantes  de  pulmón  relacionados  con  la  vida  son  un  enfoque  innovador  para  aumentar  el  grupo  de  
donantes.  Por  lo  general,  se  usa  un  lóbulo  de  cada  uno  de  dos  o  tres  donantes  para  reemplazar  un  pulmón  completo  
en  el  receptor.

32.  ¿Cómo  pueden  las  células  madre  mejorar  la  función  pulmonar  antes  y  después
¿trasplante?
Estudios  recientes  muestran  que  las  células  madre  tienen  efectos  paracrinos  agudos  que  resultan  en  la  
reparación  y  protección  del  tejido  lesionado.  Específicamente,  las  células  madre  trasplantadas  al  pulmón  
producen  factores  antiinflamatorios,  como  IL­10  y  TGF­b.  Estos  factores  antiinflamatorios  y  otros  factores  angiogénicos  
y  antiapoptóticos  pueden  mejorar  la  función  pulmonar  después  de  una  lesión  pulmonar  aguda  y  un  trasplante.

33.  ¿Qué  es  la  cirugía  de  reducción  de  volumen  pulmonar?  ¿Qué  importancia  puede  tener  para  los  pacientes  en  lista  de  
espera  de  trasplante  de  pulmón?
La  cirugía  de  reducción  de  volumen  pulmonar  ofrece  una  opción  terapéutica  para  pacientes  que  no  son  
candidatos  para  recibir  un  trasplante  de  pulmón  o  están  en  una  larga  lista  de  espera.  La  cirugía  de  reducción  de  
volumen  pulmonar  elimina  el  pulmón  no  funcional  o  destruido.  La  extracción  del  pulmón  desfuncionalizado  deja  más  
espacio  para  el  flujo  de  aire  en  el  pulmón  funcional,  lo  que  reduce  la  mortalidad  y  aumenta  la  función.

34.  ¿Quién  es  el  mejor  candidato  para  la  cirugía  de  reducción  de  volumen  pulmonar?
El  Ensayo  Nacional  de  Tratamiento  del  Enfisema  (NETT)  sugiere  que  los  mejores  candidatos  (mortalidad  más  
baja)  son  pacientes  con  un  objetivo  apical  evidente  en  el  lóbulo  superior,  distensión  torácica  marcada,  volumen  
espiratorio  forzado  (FEV1)  >20  %,  capacidad  de  difusión  del  pulmón  para  el  monóxido  de  carbono  ( DLCO)  >20%  y  
edad  <70.

35.  ¿Cuáles  son  las  contraindicaciones  de  la  cirugía  de  reducción  pulmonar?
Hipertensión  pulmonar  (presión  media  de  la  arteria  pulmonar  [PAP]  >  35  mm  Hg  o  PAP  sistólica  >  45  mm  Hg).

Cardiopatía  clínicamente  significativa.
Toracotomía  o  pleurodesis  previa  (fusión  pleural  visceral  y  parietal).
Enfermedad  difusa  sin  objetivo.
VEF  <  20  %.

Hipercarbia,  presión  parcial  de  dióxido  de  carbono  (pCO2)  >55.

36.  ¿Cuáles  son  las  tasas  de  supervivencia  actuarial  a  1,  3  y  5  años  para  los  retrasplantes  de  un  solo  pulmón?

Las  tasas  de  supervivencia  actuarial  son  del  60%,  50%  y  45%,  respectivamente.  Como  era  de  esperar,  a  estos  
pacientes  les  va  significativamente  peor.  Estos  malos  resultados  y  la  escasez  de  donantes  hacen  que  el  retrasplante  
de  pulmón  sea  un  dilema  ético.

37.  ¿Es  posible  un  trasplante  simultáneo  de  pulmón  y  páncreas?
Sí.  En  2006,  se  realizó  el  primer  trasplante  doble  simultáneo  de  pulmón  y  páncreas  en  un  paciente  con  FQ  en  el  
Methodist  Hospital,  Indiana.
Machine Translated by Google
CAPÍTULO  93  TRASPLANTE  DE  PULMÓN  453

SITIO  WEB

www.transplantation­soc.org

BIBLIOGRAFÍA

1.  Boku  N,  Tanoue  Y,  Kajihara  N  et  al.:  Un  estudio  comparativo  de  preservación  cardíaca  con  Celsior  o  University
de  solución  de  Wisconsin  con  o  sin  administración  previa  de  cardioplegia.  J  Trasplante  de  corazón  y  pulmón  25:219­225,  2006.

2.  Crisostomo  PR,  Markel  TA,  Wang  M  et  al.:  En  la  población  de  células  madre  mesenquimales  adultas,  el  género  fuente  es  un  aspecto  
biológicamente  relevante  del  poder  protector.  Cirugía  142(2):215­221,  2007.

3.  Crisostomo  PR,  Meldrum  DR:  Entrega  de  células  madre  al  corazón:  metodología  y  mecanismo  de  aclaración.  cuidado  crítico
Med  35(11):2654­2655,  2007.

4.  Fishman  A,  Martinez  F,  Naunheim  K  et  al.:  Un  ensayo  aleatorizado  que  compara  la  cirugía  de  reducción  del  volumen  pulmonar  con
tratamiento  médico  para  el  enfisema  grave.  N  Engl  J  Med  348:2059­2073,  2003.

5.  Kawut  SM,  Lederer  DJ,  Keshavjee  S  et  al .:  Resultados  después  del  retrasplante  de  pulmón  en  la  era  moderna.  Am  J  Respir  Crit  Care  
Med  177:114­120,  2008.

6.  Kleeberger  W,  Versmold  A,  Rothamel  T  et  al .:  Aumento  del  quimerismo  del  epitelio  bronquial  y  alveolar  en
aloinjertos  de  pulmón  humano  sometidos  a  lesión  crónica.  Am  J  Pathol  162:1487­1494,  2003.

7.  Oto  T,  Griffiths  AP,  Rosenfeldt  F  et  al.:  Resultados  tempranos  que  comparan  Perfadex,  Euro­Collins  y  Papworth
soluciones  en  trasplante  pulmonar.  Ann  Thorac  Surg  82:1842­1848,  2006.

8.  Quaini  F,  Urbanek  K,  Beltrami  AP  et  al.:  Quimerismo  del  corazón  trasplantado.  N  Engl  J  Med  346:5­15,  2002.

9.  Snell  GI,  Boehler  A,  Glanville  AR  et  al.:  Once  años  después:  una  actualización  clínica  de  áreas  clave  del  aloinjerto  de  pulmón  de  1996
formulación  de  trabajo  de  rechazo.  J  Trasplante  de  corazón  y  pulmón  26:423­430,  2007.

10.  Trulock  EP,  Christie  JD,  Edwards  LB  et  al.:  Registro  de  la  Sociedad  Internacional  para  el  Corazón  y  los  Pulmones
Trasplante:  vigésimo  cuarto  informe  oficial  de  trasplante  de  pulmón  y  corazón­pulmón  en  adultos—2007.  J  Heart  Lung  Transplant  
26:782­795,  2007.

11.  Wilkes  DS,  Egan  TM,  Reynolds  HY:  Trasplante  de  pulmón:  oportunidades  para  la  investigación  y  el  avance  clínico.
Am  J  Respir  Crit  Care  Med  172:944­955,  2005.
Machine Translated by Google

XI.  UROLOGÍA

EL  ENFOQUE  QUIRÚRGICO
A  LA  INFERTILIDAD
CAPITULO  
94

Randall  B.  Meacham,  MD  y  Alex  J.  Vanni,  MD

1.  ¿Qué  tan  común  es  un  problema  de  infertilidad?
La  infertilidad  es  la  incapacidad  de  establecer  un  embarazo  durante  1  año  de  relaciones  sexuales  oportunas.
Esto  afecta  al  15%  de  todas  las  parejas  en  los  Estados  Unidos.  En  el  50%  de  tales  parejas,  la  mujer  es  responsable;  en  el  
30%  de  las  parejas,  el  factor  masculino  impide  el  embarazo;  y  en  el  20%  de  las  parejas  es  una  combinación  de  ambos.

2.  ¿Cuáles  son  las  probabilidades  de  que  una  pareja  fértil  quede  embarazada  después  de  un  solo  episodio  de  relaciones  
sexuales  oportunas?
Durante  un  ciclo  ovulatorio  determinado,  el  18  %  de  las  parejas  fértiles  quedan  embarazadas  después  de  una  relación  
sexual  oportuna.

3.  ¿Cuál  es  el  mejor  momento  para  tener  relaciones  sexuales  si  una  pareja  está  tratando  de  concebir?
Los  espermatozoides  pueden  sobrevivir  en  el  moco  cervical  durante  48  horas.  Para  lograr  el  embarazo,  por  lo  tanto,  el  
momento  más  efectivo  para  tener  relaciones  sexuales  es  cada  dos  días,  comenzando  unos  días  antes  de  la  ovulación.

4.  ¿Qué  factores  ambientales  pueden  desempeñar  un  papel  en  la  infertilidad  masculina?
Aunque  la  función  reproductiva  es  relativamente  duradera,  varias  toxinas  tienen  un  impacto  negativo  en  la  fertilidad  
masculina.  El  humo  del  cigarrillo  y  el  alcohol  han  sido  implicados  como  gonadotoxinas  dependientes  de  la  dosis,  al  igual  
que  las  drogas  recreativas,  como  la  marihuana,  la  cocaína  y  la  heroína.  La  radiación  (en  cantidades  tan  bajas  como  200  rads)  
puede  influir  en  la  espermatogénesis,  al  igual  que  los  agentes  quimioterapéuticos.  Los  bloqueadores  de  los  canales  de  calcio  
pueden  interferir  con  la  capacidad  de  los  espermatozoides  para  fertilizar  óvulos.

5.  ¿Se  puede  revertir  con  éxito  una  vasectomía?
Sí,  pero  la  tasa  de  éxito  se  ve  afectada  por  la  cantidad  de  tiempo  transcurrido  desde  la  vasectomía  original.
Entre  los  pacientes  que  tienen  menos  de  3  años  desde  la  vasectomía,  la  tasa  de  concepción  después  de  la  reversión  es  de  
aproximadamente  el  75  %.  Esta  tasa  de  éxito  se  reduce  a  alrededor  del  50  %  cuando  la  reversión  se  realiza  de  3  a  8  años  
después  de  la  vasectomía  y  se  reduce  aún  más  al  30  %  cuando  han  pasado  15  años  o  más.

6.  ¿Qué  es  la  fecundación  in  vitro  (FIV)?
Con  la  FIV,  los  óvulos  se  extraen  de  una  mujer  y  se  combinan  con  esperma  en  un  entorno  de  laboratorio.
Los  embriones  resultantes  luego  se  transfieren  a  la  cavidad  uterina,  donde  maduran  hasta  convertirse  en  un  feto.  En  una  
versión  especializada  de  esta  tecnología  (es  decir,  inyección  intracitoplasmática  de  espermatozoides),  se  inyecta  un  
espermatozoide  individual  en  cada  óvulo,  lo  que  facilita  la  fertilización  y  permite  el  embarazo  incluso  en  presencia  de  una  
pequeña  cantidad  de  espermatozoides  móviles.

7.  ¿Cuál  es  el  papel  de  la  FIV  en  la  infertilidad  masculina?
Debido  a  que  el  uso  de  la  FIV  reduce  en  gran  medida  la  cantidad  de  espermatozoides  móviles  necesarios  para  generar  un  
embarazo,  puede  ser  muy  útil  en  hombres  con  mala  calidad  del  semen.  El  equipo  de  FIV  necesita  tantos  espermatozoides  
móviles  como  ovocitos  (óvulos)  para  fertilizar.

455
Machine Translated by Google
456  CAPÍTULO  94  EL  ENFOQUE  QUIRÚRGICO  DE  LA  INFERTILIDAD

8.  ¿Se  puede  utilizar  el  esperma  obtenido  directamente  del  testículo  para  generar  un  embarazo?
Durante  los  últimos  años,  se  ha  reconocido  que  la  incubación  de  tejido  testicular  generalmente  produce  
una  pequeña  cantidad  de  espermatozoides  móviles.  Mediante  el  uso  de  la  FIV,  dichos  espermatozoides  
pueden  generar  embarazos.  Incluso  entre  los  hombres  que  sufren  de  insuficiencia  testicular  grave,  puede  ser  
posible  recuperar  suficiente  esperma  para  su  uso  en  la  FIV.

9.  ¿Cuál  es  el  papel  de  la  congelación  de  semen  en  el  tratamiento  de  la  infertilidad?
El  esperma  se  puede  congelar  (criopreservar)  con  relativa  facilidad.  Después  de  que  se  criopreservan,  los  
espermatozoides  permanecen  viables  durante  períodos  prolongados  (años).  La  criopreservación  puede  ser  útil  
entre  los  hombres  que  planean  someterse  a  un  tratamiento  con  quimioterapia  o  radioterapia.

10.  ¿Usar  calzoncillos  en  lugar  de  ropa  interior  ajustada  afecta  la  fertilidad  masculina?
No.

PUNTOS  CLAVE:  ABORDAJE  QUIRÚRGICO  DE  LA  INFERTILIDAD

1.  La  infertilidad  se  define  como  la  incapacidad  de  establecer  un  embarazo  durante  1  año  de
coito.

2.  En  el  50%  de  las  parejas  infértiles  un  factor  femenino  previene  el  embarazo,  en  el  30%  de  las  parejas  un  factor  
masculino  previene  el  embarazo  y  en  el  20%  de  las  parejas  la  infertilidad  es  el  resultado  de  una  combinación  
de  factores  masculinos  y  femeninos.

3.  La  causa  más  común  de  infertilidad  masculina  es  el  varicocele.

11.  Debido  a  que  los  niveles  normales  de  testosterona  son  necesarios  para  la  producción  de  esperma,  ¿es
útil  para  dar  a  los  hombres  subfértiles  testosterona  adicional?
Aunque  la  disminución  de  los  niveles  de  testosterona  puede  causar  problemas  de  fertilidad  masculina,  administrar  
testosterona  adicional  a  los  hombres  con  niveles  normales  de  testosterona  puede  causar  una  disminución  
dramática  en  la  calidad  del  semen.  La  administración  de  testosterona  exógena  hace  que  el  paciente  deje  de  
producir  testosterona  nativa  dentro  de  los  testículos.  La  disminución  resultante  de  la  testosterona  intratesticular  
en  realidad  resulta  en  una  disminución  de  la  producción  de  espermatozoides.

12.  ¿Cuál  es  la  causa  más  común  de  infertilidad  masculina?
Varicocele,  una  colección  de  venas  dilatadas  por  encima  de  uno  o  ambos  testículos.  Entre  los  hombres  que  se  
presentan  para  el  tratamiento  de  la  infertilidad,  el  40%  tiene  varicocele.  La  corrección  del  varicocele  conduce  a  una  mejora  
en  la  calidad  del  semen  en  el  70%  de  los  pacientes.

13.  Si  podemos  clonar  a  Dolly  (una  oveja  derivada  de  la  clonación  de  un
célula  mamaria),  ¿podemos  clonar  humanos?
Aunque  por  una  serie  de  razones  éticas  críticas,  la  tecnología  de  clonación  no  se  usa  actualmente  en  
la  reproducción  humana,  teóricamente  permite  la  clonación  de  cualquier  individuo,  creando  un  duplicado  
genético.  Sin  embargo,  es  probable  que  la  clonación  no  desempeñe  un  papel  en  el  tratamiento  de  la  
infertilidad  humana.

14.  ¿La  fertilización  in  vitro  está  asociada  con  un  aumento  de  anomalías  genéticas?
Este  tema  es  controvertido,  pero  probablemente  no.  Al  menos  una  publicación  reciente  sugirió  que  los  bebés  
concebidos  mediante  inyección  intracitoplasmática  de  espermatozoides  o  FIV  tienen  el  doble  de  riesgo  de  defectos  
congénitos  importantes  en  comparación  con  los  bebés  concebidos  naturalmente.
Machine Translated by Google
CAPÍTULO  94  EL  ENFOQUE  QUIRÚRGICO  DE  LA  INFERTILIDAD  457

15.  ¿La  administración  de  suplementos  de  testosterona  mejorará  la  fertilidad  masculina?
No.  La  testosterona  exógena  induce  una  profunda  disminución  de  la  espermatogénesis  y  se  ha  explorado  
como  un  medio  de  anticoncepción  masculina.

16.  ¿Qué  es  la  clonación  en  lo  que  respecta  a  los  humanos?
Al  igual  que  la  oveja  Dolly,  la  clonación  humana  implica  el  trasplante  nuclear  del  clon  deseado  en  un  óvulo  
desprovisto  de  su  núcleo.  En  lugar  de  crear  seres  humanos  completos,  el  dilema  ético  más  controvertido  es  si  
permitir  la  clonación  de  células  u  órganos  para  su  posterior  trasplante  con  el  fin  de  curar  enfermedades  humanas.

17.  ¿Los  testículos  no  descendidos  están  asociados  con  la  infertilidad  masculina?
Sí.  La  criptorquidia  se  asocia  con  la  infertilidad  masculina.  La  disminución  de  la  fertilidad  se  correlaciona  con  
recuentos  de  células  germinales  totales  severamente  reducidos  en  testículos  no  descendidos  prepuberales.  La  
maldescencia  testicular  bilateral  sí  disminuye  la  calidad  del  semen.  Curiosamente,  la  criptorquidia  unilateral  
también  puede  afectar  la  calidad  del  semen.  Esto  sugiere  que  tanto  el  testículo  anormalmente  descendido  como  
su  contraparte  en  posición  normal  se  ven  afectados  negativamente.  El  reposicionamiento  quirúrgico  de  los  
testículos  mejora  la  calidad  del  semen;  cuanto  antes  se  haga,  mejor.

SITIO  WEB

www.auanet.org

BIBLIOGRAFÍA

1.  Cortes  D,  Thorp  JM,  Visfeldt  J:  Criptorquidia:  aspectos  de  la  fertilidad  y  neoplasias.  Un  estudio  de  1.335
niños  consecutivos  que  se  sometieron  a  biopsia  testicular  simultáneamente  con  cirugía  por  criptorquidia.  Horm  Res  55:21­27,  2001.

2.  Hansen  M,  Kurinczuk  JJ,  Bower  C  et  al.:  El  riesgo  de  defectos  congénitos  importantes  después  de  la  inyección  intracitoplasmática  de  espermatozoides
y  fecundación  in  vitro.  N  Engl  J  Med  346:725­730,  2002.

3.  Hargreave  T,  Ghosh  C:  Trastornos  de  la  fertilidad  masculina.  Endocrinol  Metab  Clin  North  Am  27:765­782,  1998.

4.  Ismail  MT,  Sedor  J,  Hirsch  IH:  ¿Los  parámetros  de  movimiento  de  los  espermatozoides  están  influenciados  por  la  ligadura  de  varicocele?  estéril  fértil
71:886­890,  1999.

5.  Johnson  MD:  Riesgos  genéticos  de  la  inyección  intracitoplasmática  de  espermatozoides  en  el  tratamiento  de  la  infertilidad  masculina:  
recomendaciones  para  el  asesoramiento  y  la  detección  genéticos.  Fertil  Steril  70:397­411,  1998.

6.  Kim  ED,  Winkel  E,  Orejuela  F  et  al.:  Obstrucción  patológica  del  epidídimo  no  relacionada  con  la  vasectomía:  resultados  con
reconstrucción  microquirúrgica.  J  Urol  160(6  pt  1):2078­2080,  1998.

7.  Meriggiola  MC,  Costantino  A,  Cerpolini  S:  Avances  recientes  en  la  anticoncepción  masculina  hormonal.  Anticoncepción
64:269­272,  2002.

8.  Naysmith  TE,  Blake  DA,  Harvey  VJ  et  al.:  ¿Los  hombres  que  se  someten  a  tratamientos  de  esterilización  contra  el  cáncer  tienen  un  futuro  fértil?
Hum  Reprod  13:3250­3255,  1998.

9.  Palermo  GD,  Schlegel  PN,  Hariprashad  JJ  et  al .:  Fertilización  y  resultado  del  embarazo  con  inyección  intracitoplasmática  de  espermatozoides  para  
hombres  azoospérmicos.  Hum  Reprod  14:741­748,  1999.

10.  Pellegrino  ED,  Kilner  JF,  Fitzgerald  KT  et  al.:  Clonación  terapéutica.  N  Engl  J  Med  347:1619­1622,  2002.

11.  Rutkowski  SB,  Geraghty  TJ,  Hagen  DL  et  al.:  Un  enfoque  integral  para  el  manejo  de  la  infertilidad  masculina
después  de  una  lesión  de  la  médula  espinal.  Médula  espinal  37:508­514,  1999.

12.  Scherr  D,  Goldstein  M:  Comparación  de  varicocelectomía  bilateral  versus  unilateral  en  hombres  con  varicocelectomía  bilateral  palpable
varicoceles.  JUrol  162:85­88,  1999.

13.  Wilmut  I:  Clonación  para  la  medicina.  Sci  Am  279:58­63,  1998.
Machine Translated by Google

ENFERMEDAD  DE  CÁLCULOS  URINARIOS
CAPITULO  
95

Dr.  Brett  B.  Abernathy

"No  cortaré  a  las  personas  que  trabajan  debajo  de  la  piedra,  sino  que  dejaré  que  lo  hagan  los  hombres  que  son  
practicantes  de  este  trabajo".  (Juramento  hipocrático)

Hipócrates  y  los  antiguos  griegos  reconocieron  que  cortar  piedra  era  un  negocio  peligroso.
Afortunadamente,  en  la  era  de  la  urología  moderna,  y  con  la  tecnología  que  ahora  está  disponible,  es  raro  que  realmente  se  
necesite  cortar  piedra.

1.  ¿Qué  tan  comunes  son  los  cálculos  en  el  tracto  urinario?
1.  En  los  Estados  Unidos,  la  prevalencia  de  la  enfermedad  de  los  cálculos  se  ha  estimado  en  un  10  %  a  un  15  %.
2.  Es  probable  que  uno  de  cada  ocho  hombres  blancos  experimente  la  enfermedad  de  cálculos  a  los  70  años.  Aunque  
los  hombres  se  ven  afectados  con  mayor  frecuencia  que  las  mujeres,  la  diferencia  parece  estar  disminuyendo.
3.  La  enfermedad  de  cálculos  del  tracto  urinario  representa  aproximadamente  600  000  visitas  al  departamento  de  
emergencias  (ED)  y  más  de  177  000  hospitalizaciones  en  los  Estados  Unidos  anualmente.
Esto  representa  una  carga  financiera  enorme  con  un  gasto  superior  a  los  2.000  millones  de  dólares  al  año.

2.  ¿Cómo  se  diagnostican  los  cálculos  en  las  vías  urinarias?
1.  Los  cálculos  sintomáticos  se  asocian  comúnmente  con  dolor  en  el  área  del  flanco  que  produce  cólicos.  El  
dolor  puede  irradiarse  a  la  ingle  ipsilateral.
2.  Los  pacientes  suelen  estar  bastante  agitados  y  tienen  dificultad  para  ponerse  en  una  posición  cómoda.
3.  La  mayoría  de  las  veces  (85%)  hay  hematuria  microscópica  o  macroscópica.
4.  Las  náuseas  y  los  vómitos  suelen  acompañar  al  cólico  ureteral  como  resultado  de  la  presión  sobre  la  cápsula  renal.

5.  Los  cálculos  asintomáticos  o  que  no  obstruyen  se  diagnostican  comúnmente  mediante  tomografías  
computarizadas  (TC)  realizadas  para  otras  afecciones.

3.  ¿Cuáles  son  los  mejores  estudios  para  diagnosticar  cálculos?
1.  La  tomografía  computarizada  helicoidal  de  secuencia  rápida  se  ha  convertido  en  el  estudio  más  común  para  el  
diagnóstico  de  la  litiasis.  Las  ventajas  de  este  estudio  incluyen  la  no  necesidad  de  contraste,  la  realización  rápida  y  la  
capacidad  de  diagnosticar  todos  los  cálculos  comunes  del  tracto  urinario,  incluidos  los  cálculos  de  ácido  úrico  y  cistina  
que  pueden  ser  radiotransparentes  en  las  imágenes  estándar.
2.  La  pielografía  intravenosa  (PIV)  y  la  urografía  por  TC  también  se  pueden  utilizar  y  proporcionan  información  adicional  
sobre  la  función  renal  y  el  grado  de  obstrucción.  Sin  embargo,  estos  estudios  requieren  contraste  intravenoso  (IV).

3.  La  ecografía  puede  ser  particularmente  útil  en  mujeres  embarazadas.  Sin  embargo,  los  cálculos  en  el  uréter  medio  
o  distal  pueden  ser  bastante  difíciles  de  identificar  con  ultrasonido  (US).

4.  ¿Cuáles  son  las  indicaciones  para  el  ingreso  hospitalario  del  paciente  con  litiasis?

1.  Cualquier  signo  de  infección  (fiebre,  leucocitosis  o  bactiuria).  La  infección  detrás  de  un  cálculo  que  obstruye  puede  
provocar  urosepsis  y  muerte.
2.  Dolor  intenso  que  requiere  analgésicos  parenterales.

458
Machine Translated by Google
CAPÍTULO  95  ENFERMEDAD  DE  CÁLCULOS  URINARIOS  459

3.  Vómitos  intratables  que  requieren  líquidos  por  vía  intravenosa.

4.  Obstrucción  en  un  riñón  solitario  o  cálculos  obstructivos  bilaterales.
5.  El  grado  de  hidronefrosis  identificado  en  las  imágenes  radiológicas  no  es  necesariamente  una  indicación  de  
ingreso.

5.  ¿Cuáles  son  los  tipos  comunes  de  cálculos  en  las  vías  urinarias  que  se  encuentran  en  el
¿Estados  Unidos?

1.  Cálculos  de  calcio  (oxalato  de  calcio,  fosfato  de  calcio  o  cálculos  mixtos  de  calcio):  80%.
2.  Cálculos  de  estruvita  o  fosfato  amónico  magnésico,  comúnmente  asociados  con
infección:  7%.
3.  Piedras  de  ácido  úrico:  7%.

4.  Cálculos  de  cistina:  1%  a  3%.

6.  ¿Cuáles  son  las  opciones  de  tratamiento  para  los  cálculos  renales?
1.  Manejo  expectante  en  cálculos  pequeños,  asintomáticos  y  no  infecciosos.  El  tratamiento  estaría  indicado  para  el  
aumento  de  la  carga  de  cálculos  o  los  síntomas.
2.  La  litotricia  por  ondas  de  choque  (SWL)  es  la  opción  de  tratamiento  menos  invasiva  y  bastante
exitoso  especialmente  en  cálculos  de  menos  de  2  cm  de  tamaño.
3.  La  ureterorrendoscopia  con  litotricia  con  láser  de  holmio  también  es  ahora  una  opción  como  resultado  de  la
mejora  en  ureteroscopios  deflectables  flexibles.
4.  La  nefrostolitotomía  percutánea  es  especialmente  útil  en  casos  de  cálculos  mayores  de  2  cm.
5.  Terapia  combinada  con  nefrostolitotomía  percutánea  y  LEOC.  Esta  combinación  se  usa  comúnmente  con  cálculos  
complejos  y  gran  carga  de  cálculos.
6.  Todavía  se  puede  cortar  en  busca  de  cálculos  utilizando  la  técnica  laparoscópica  o  de  litotomía  abierta.  ¡Los  
resultados  son  mucho  mejores  que  en  los  días  de  Hipócrates!

7.  ¿Cuáles  son  las  opciones  de  tratamiento  para  los  cálculos  ureterales?
1.  Manejo  expectante.  Estudios  de  metanálisis  recientes  demuestran  que  los  cálculos  <5  mm  tienen  un  68  %  de  
posibilidades  de  expulsión  espontánea.  Los  cálculos  entre  5  mm  y  10  mm  tienen  una  tasa  de  paso  espontáneo  del  
47  %.  Es  poco  probable  que  los  cálculos  de  más  de  10  mm  pasen  espontáneamente.  Si  el  cálculo  va  a  desaparecer,  
es  probable  que  ocurra  dentro  de  las  4  a  6  semanas  posteriores  al  diagnóstico.

2.  Terapia  médica  expulsiva  (MET).  El  tratamiento  con  bloqueadores  alfa  orales  ha  sido
Se  ha  demostrado  que  aumenta  significativamente  la  tasa  de  expulsión  de  cálculos  en  un  29  %.  El  bloqueador  alfa  
más  comúnmente  estudiado  para  MET  es  la  tamsulosina.  El  mecanismo  detrás  de  este  efecto  es  probablemente  la  
relajación  del  músculo  liso  ureteral.

3.  Ureteroscopia  (URS)  con  litotricia  láser  o  extracción  en  cesta  de  cálculos.  Esta  técnica  se  puede  utilizar  para  cálculos  
en  el  uréter  distal,  medio  y  proximal.  Si  es  necesaria  una  intervención,  la  URS  demuestra  la  tasa  libre  de  cálculos  
más  alta  para  cálculos  ureterales  distales  y  medios.  Como  resultado  del  avance  en  la  tecnología  de  los  ureteroscopios  
deflectables  flexibles,  la  URS  ahora  tiene  una  tasa  de  éxito  similar  a  la  de  la  SWL  también  en  el  uréter  proximal.

4.  SWL.  SWL  se  puede  utilizar  para  el  tratamiento  de  cálculos  en  cualquier  parte  del  uréter.  La  localización  puede  ser  
problemática  en  el  uréter  medio  debido  a  la  dificultad  para  identificar  cálculos  sobre  la  pelvis  ósea.  Aunque  es  menos  
invasivo  que  la  URS,  las  tasas  de  éxito  en  el  uréter  distal  para  la  LEOC  no  son  tan  altas  como  las  de  la  URS.  La  
LEOC  tiene  bastante  éxito  (83%  libre  de  cálculos)  para  cálculos  <10  mm  en  el  uréter  proximal.

5.  La  URS  anterógrada  percutánea  puede  estar  indicada  en  casos  seleccionados  de  cálculos  grandes  
impactados  en  el  uréter  proximal,  o  cuando  se  combina  con  nefrostolitotomía  percutánea  y  en  pacientes  
con  derivaciones  urinarias.
6.  La  cirugía  laparoscópica  y  abierta  de  cálculos  rara  vez  está  indicada,  pero  sigue  siendo  útil  en  casos  seleccionados.
casos.
Machine Translated by Google
460  CAPÍTULO  95  ENFERMEDAD  DE  CÁLCULOS  URINARIOS

8.  ¿Qué  es  un  stent  y  cuándo  se  usa?
Un  stent  es  un  pequeño  tubo  de  plástico  que  se  enrolla  en  la  pelvis  renal,  atraviesa  el  uréter  y  se  enrolla  en  la  vejiga.  Son  
útiles  para  aliviar  temporalmente  la  obstrucción  ureteral  y  pueden  facilitar  el  paso  de  los  cálculos  una  vez  retirado  el  stent.  
Los  stents  a  menudo  resultan  en  algún  grado  de  dilatación  ureteral  temporal.  No  se  recomienda  la  colocación  de  stent  de  
rutina  como  parte  de  la  LEOC  en  el  uréter.  La  colocación  de  stent  es  opcional  después  de  URS  sin  complicaciones.  Los  
stents  también  están  indicados  en  el  tratamiento  de  grandes  cálculos,  riñones  solitarios  y  tratamiento  bilateral.

9.  ¿Cuáles  son  las  posibles  complicaciones  del  tratamiento  de  la  litiasis?
1.  Sepsis.  La  bacteriuria  y  los  signos  de  infección  clínica  deben  tratarse  y  eliminarse  en  todo  caso.
posible  antes  del  tratamiento  definitivo  de  la  litiasis.  La  urosepsis  debe  tratarse  con  drenaje  de  nefrostomía  
percutánea  inmediata  o  colocación  de  stent  combinado  con  la  terapia  antibiótica  adecuada.  El  tratamiento  
definitivo  del  cálculo  puede  esperar  a  que  se  elimine  la  infección.
2.  Hematoma  perirrenal  o  daño  renal  después  de  LEOC.
3.  Rotura  del  uréter  o  estenosis  ureteral  después  de  URS  o  cirugía  abierta.
4.  Falta  de  acceso  o  identificación  del  cálculo  o  fragmentación  incompleta  del  cálculo
que  requieren  tratamiento  adicional.
5.  Steinstrasse  (calle  de  piedra  en  alemán)  es  una  acumulación  de  pequeños  fragmentos  de  piedra  en  el  uréter  que  
provocan  una  obstrucción  después  de  la  litotricia.

10.  ¿Cuál  es  el  tratamiento  médico  de  la  enfermedad  de  los  cálculos?
El  tratamiento  médico  de  la  enfermedad  de  los  cálculos  implica  identificar  el  tipo  de  cálculo  mediante  el  análisis  de  cálculos.
Luego  se  realizan  estudios  de  suero  y  recolecciones  de  orina  de  24  horas  para  identificar  los  factores  causales  
específicos  de  la  formación  de  cálculos  en  un  paciente.  Según  los  hallazgos  específicos,  el  manejo  puede  consistir  en  un  
aumento  de  la  hidratación,  cambios  en  la  dieta  y  terapia  farmacológica  específica  para  tratar  de  prevenir  la  formación  de  
más  cálculos.  Todos  los  formadores  de  cálculos  se  benefician  de  una  excelente  hidratación  para  proporcionar  una  
producción  de  orina  de  al  menos  2,0  a  2,5  L  diarios.  La  mayoría  también  se  beneficiará  de  la  terapia  con  citrato  oral  porque  
el  citrato  es  un  inhibidor  de  la  formación  de  cálculos.  El  manejo  médico  puede  reducir  el  riesgo  de  enfermedad  de  cálculos  
recurrentes  y,  potencialmente,  reducir  la  morbilidad  del  paciente  y  el  costo  del  tratamiento  de  cálculos.  La  identificación  de  
hiperparatiroidismo  primario  también  puede  tratarse  definitivamente  con  paratiroidectomía.

11.  ¿Se  pueden  disolver  las  piedras?

1.  Los  cálculos  de  ácido  úrico  a  menudo  se  pueden  disolver  usando  una  combinación  de  alcalinización  y
hidratación.
2.  Los  cálculos  a  base  de  calcio  no  se  pueden  disolver  con  tratamiento  médico.

12.  ¿Una  dieta  alta  en  calcio  aumenta  el  riesgo  de  enfermedad  de  cálculos?
Este  es  un  tema  controvertido.  Una  ingesta  dietética  adecuada  de  calcio  es  necesaria  para  proteger  contra  la  
osteoporosis.  Además,  el  calcio  ayuda  a  unir  el  oxalato  en  el  intestino,  lo  que  en  realidad  puede  reducir  el  riesgo  de  
absorción  de  oxalato  y  la  formación  de  cálculos  que  contienen  oxalato.
Sin  embargo,  hay  condiciones  en  las  que  una  mayor  absorción  de  calcio  es  un  factor  que  contribuye  a  la  formación  
de  cálculos  de  calcio.  Actualmente,  el  mejor  enfoque  probablemente  sea  recomendar  una  ingesta  normal  de  calcio  o  
una  restricción  moderada  de  calcio  en  la  dieta  y  evitar  los  alimentos  ricos  en  oxalato  para  reducir  el  riesgo  de  formación  de  
cálculos  de  oxalato  de  calcio.

PREGUNTA  EXTRA

13.  ¿Quiénes  eran  los  "litotomistas"?

Como  se  señaló,  Hipócrates  prohibió  la  extracción  de  cálculos  de  la  vejiga  y  el  perineo.
Esto  probablemente  se  debió  a  los  malos  resultados.  Hasta  el  siglo  XVI,  la  litotomía  era  vista  con  desprecio  y  
generalmente  la  realizaban  cirujanos  itinerantes  que  se  especializaban  en  este  tema.
Machine Translated by Google
CAPÍTULO  95  ENFERMEDAD  DE  CÁLCULOS  URINARIOS  461

técnica.  Se  los  conocía  como  "litotomistas".  Eventualmente,  desarrollaron  algunas  técnicas  
únicas  y  bastante  bárbaras  para  extraer  cálculos  del  tracto  urinario  inferior.  En  un  caso  famoso  
en  1651,  un  valiente  herrero  llamado  Jan  de  Doot  extrajo  su  propia  piedra  del  perineo  con  un  
cuchillo  de  cocina.  De  alguna  manera,  la  cistoscopia  bajo  anestesia  con  litotricia  láser  parece  
un  progreso.

BIBLIOGRAFÍA

[ PubMed ]  Kraft  K,  Pattaras  JG:  Manejo  médico  de  la  urolitiasis.  Serie  de  actualización  de  AUA  vol.  27  Lección  36 ,
2.  Murphy  LJT:  La  historia  de  la  urología,  Springfield,  1972,  Charles  C.  Thomas.
3.  Pearle  MS,  Lotan  Y:  Litihiasis  urinaria:  etiología,  epidemiología  y  patogénesis.  En  Wein  AJ,  Kavoussi  LR,  Novick  AC  
et  al.,  editores:  Campbell­Walsh  urology,  9.ª  ed.,  Filadelfia,  2007,  Saunders  Elsevier.
4.  Preminger  GM,  Tiselius  H,  Assimos  DG  et  al.:  2007  Guía  para  el  manejo  de  cálculos  ureterales.  J  Urol
178:2418–2434,  2007.
Machine Translated by Google

CARCINOMA  DE  CÉLULAS  RENALES
CAPITULO  
96

Fernando  J.  Kim,  MD,  FACS;

1.  ¿Qué  tan  común  es  el  carcinoma  de  células  renales  (CCR)?
En  los  Estados  Unidos,  cada  año  se  diagnostican  aproximadamente  30  000  casos  nuevos  de  carcinoma  de  células  renales,  
aproximadamente  el  3%  de  todas  las  neoplasias  malignas  en  adultos.

2.  ¿Cuál  es  la  etiología  del  CCR?
La  etiología  es  desconocida,  pero  fumar  cigarrillos  es  un  factor  de  riesgo  bien  conocido.  El  CCR  recurrente  es  
una  manifestación  común  en  pacientes  con  enfermedad  de  Von  Hippel­Lindau.

3.  ¿Cuáles  son  los  signos  y  síntomas  del  CCR?
Los  signos  y  síntomas  de  presentación  más  frecuentes  son  la  hematuria  macroscópica  o  microscópica.
La  tríada  clásica  de  hematuria,  dolor  en  el  flanco  y  una  masa  abdominal  se  encuentra  en  solo  alrededor  del  
10%  al  15%  de  los  casos  de  RCC.  Los  pacientes  con  enfermedad  metastásica  pueden  presentar  síntomas  de  
metástasis  pulmonar  o  ósea,  como  disnea,  tos  o  dolor  óseo.
Alrededor  del  20%  de  los  CCR  están  asociados  con  un  síndrome  paraneoplásico.  Muchos  tumores  
renales  sólidos  se  detectan  de  manera  incidental  mediante  una  tomografía  computarizada  (TC)  del  
abdomen  realizada  por  otro  motivo.
El  síndrome  de  Stauffer  se  diagnostica  con  pruebas  de  función  hepática  (LFT)  elevadas  en  presencia  de
RCC  que  se  normaliza  después  de  la  nefrectomía  y  la  extirpación  del  tumor;  se  cree  que  es  un  tipo  
de  síndrome  paraneoplásico.

4.  ¿Todas  las  masas  sólidas  en  el  riñón  son  carcinoma  de  células  renales?
No.  Otras  masas  sólidas  incluyen  angiomiolipomas,  oncocitomas,  sarcomas  y  lesiones  metastásicas.  Sin  
embargo,  se  debe  suponer  que  todas  las  masas  sólidas  son  RCC  hasta  que  se  demuestre  lo  contrario.

5.  ¿Cuál  es  la  relación  única  entre  el  carcinoma  de  células  renales  y  su
vasculatura?
El  CCR  tiene  tendencia  a  invadir  su  propio  drenaje  venoso.  El  trombo  tumoral  puede  extenderse  a  lo  largo  de  
la  vena  renal  hacia  la  vena  cava  inferior  (VCI)  e  incluso  hacia  la  aurícula  derecha.

6.  ¿Cómo  se  debe  evaluar  la  sospecha  de  afectación  de  la  vena  cava?
Resonancia  magnética  nuclear  (RMN)  o  venacavografía.

7.  ¿Cómo  se  trata  el  carcinoma  de  células  renales?
La  cirugía  es  el  tratamiento  óptimo  para  el  CCR  localizado.  La  operación  estándar  es  una  nefrectomía  
radical,  que  incluye  todo  lo  que  está  dentro  de  la  fascia  de  Gerota.  La  nefrectomía  radical  también  se  
puede  realizar  por  vía  laparoscópica  o  con  técnicas  laparoscópicas  asistidas  por  la  mano.

8.  ¿Cuándo  está  indicada  la  nefrectomía  conservadora  de  nefronas  en  casos  de  células  renales?
¿carcinoma?
Los  tumores  de  menos  de  4  cm  de  tamaño  se  pueden  tratar  con  cirugía  laparoscópica  o  abierta  con  conservación  de  nefronas.
La  tecnología  ablativa  (es  decir,  crioablación  y  ablación  por  radiofrecuencia)  todavía  está  bajo  investigación  
con  resultados  alentadores.

462
Machine Translated by Google
CAPÍTULO  96  CARCINOMA  DE  CÉLULAS  RENALES  463

9.  ¿Cómo  se  trata  el  carcinoma  metastásico  de  células  renales?

La  quimioterapia  ha  sido  decepcionante.  Históricamente,  se  lograron  resultados  alentadores  con  la  nefrectomía  
radical  citorreductora  y  el  tratamiento  con  interleucina­2  (IL­2).  Actualmente,  la  terapia  dirigida  con  inhibidores  de  la  
tirosina  quinasa  ofrece  alguna  evidencia  de  respuestas  definitivas  y  duraderas.
La  investigación  está  en  curso  utilizando  diferentes  estrategias  de  terapia  dirigida.

PUNTOS  CLAVE:  CARCINOMA  DE  CÉLULAS  RENALES

1.  La  tríada  clásica  es  hematuria,  dolor  en  el  flanco  y  una  masa  abdominal  generalmente  se  encuentra  en  casos  
avanzados  de  CCR.

2.  La  cirugía  es  el  tratamiento  óptimo  para  el  CCR  localizado.

3.  El  síndrome  de  Stauffer  se  diagnostica  con  pruebas  de  función  hepática  (LFT)  elevadas  en  presencia  de  RCC  que  se  
normaliza  después  de  la  nefrectomía  y  la  extirpación  del  tumor;  se  cree  que  es  un  tipo  de  síndrome  paraneoplásico.

SITIO  WEB

www.trasplantjournal.com

BIBLIOGRAFÍA

1.  Kim  FJ,  Rha  KH,  Hernandez  F  et  al.:  Nefrectomía  radical  versus  parcial  laparoscópica:  evaluación  de
complicaciones  J  Urol  170:408,  2003.
2.  Greenlee  RT,  Hill­Harmon  MB,  Murray  T  et  al.:  Estadísticas  de  cáncer  2001.  CA  Cancer  J  Clin  51:15,  2001.
3.  Figlin  RA:  Carcinoma  de  células  renales:  manejo  de  la  enfermedad  avanzada.  JUrol  161:391,  1999.
Machine Translated by Google

CÁNCER  DE  VEJIGA
CAPITULO  
97

Fernando  J.  Kim,  MD,  FACS;

1.  ¿Cuál  es  la  incidencia  del  carcinoma  de  células  de  transición  (CCT)  de  la  vejiga?
Cada  año  se  diagnostican  más  de  60  000  nuevos  casos  de  cáncer  de  vejiga  en  los  Estados  Unidos,  lo  que  
representa  aproximadamente  13  000  muertes  al  año.  Recientemente,  la  incidencia  general  de  cáncer  de  vejiga  
parece  estar  aumentando  tal  vez  como  resultado  del  envejecimiento  general  de  nuestra  población  y  los  efectos  
latentes  del  abuso  del  tabaco  y  los  carcinógenos  industriales.

2.  ¿Cuáles  son  los  factores  de  riesgo  asociados  con  el  CCT  vesical?
Edad  (incidencia  máxima  en  la  séptima  década),  tabaquismo,  exposición  ocupacional  a  colorantes  de  anilina  o  
aminas  aromáticas,  abuso  de  fenacetina  y  quimioterapia  con  ciclofosfamida.

3.  ¿Cuáles  son  los  signos  y  síntomas  del  TCC  vesical?
La  hematuria  indolora  (macroscópica  o  microscópica)  es  el  hallazgo  más  común  y  está  presente  hasta  en  el  
90%  de  los  pacientes.  La  frecuencia,  la  urgencia  y  la  disuria  también  pueden  ser  síntomas  de  presentación,  
especialmente  para  el  carcinoma  in  situ  (CIS).

4.  ¿Cuál  es  el  tipo  histológico  más  común  de  cáncer  de  vejiga?
El  TCC  constituye  >90%  de  los  cánceres  de  vejiga.  Otros  tipos  histológicos  incluyen  adenocarcinoma,  
carcinoma  de  células  escamosas  y  carcinoma  de  uraco.

5.  ¿Cómo  evalúa  a  un  paciente  con  hematuria  y  masa  vesical?
Análisis  de  orina,  cultivo  y  lavados  vesicales  para  citología;  examen  bimanual  (generalmente  realizado  bajo  
anestesia);  estudio  de  imagen  del  tracto  superior  (descartar  enfermedad  concomitante  del  tracto  superior);  ya  
sea  cistoscopia  con  biopsia  (si  la  lesión  es  pequeña  o  atípica)  o  resección  transuretral  del  tumor  vesical  (TURBT)  
de  lesiones  sospechosas.

6.  ¿Cómo  maneja  el  CCT  vesical?
El  tumor  vesical  debe  extirparse  con  resección  transuretral  endoscópica  y  fulguración.
El  tratamiento  adicional  está  determinado  por  la  etapa  patológica  de  la  enfermedad.

7.  ¿Cuál  es  la  tasa  de  recurrencia  de  TCC  después  de  la  resección  transuretral  inicial  de  la  vejiga?
¿tumor?
Aproximadamente  el  45  %  de  los  pacientes  tendrán  recurrencia  del  tumor  dentro  de  los  12  meses  posteriores  a  la  TURBT  sola.

8.  ¿Con  qué  frecuencia  espera  ver  un  TCC  vesical  con  invasión  muscular  de  alto  grado?
La  gran  mayoría  (70  %  a  75  %)  de  los  TCC  vesicales  se  presentan  como  lesiones  superficiales  (no  invasivas  
del  músculo).  Además,  la  mayor  parte  de  las  lesiones  se  clasifican  como  de  bajo  grado  y  solo  entre  el  2  %  y  el  4  
%  se  clasifican  como  de  alto  grado.

9.  ¿Con  qué  frecuencia  las  lesiones  superficiales  deben  ser  seguidas  con  cistoscopia  de  vigilancia?
y  citologia  de  orina?
Cada  3  meses  en  los  primeros  3  años  después  del  diagnóstico  inicial  seguido  de  cada  6  meses  durante  los  
siguientes  2  a  3  años  y  luego  anualmente  a  partir  de  entonces.  La  vigilancia  incluye  imágenes  periódicas  del  tracto  
superior,  especialmente  para  pacientes  de  alto  riesgo.

464
Machine Translated by Google
CAPÍTULO  97  CÁNCER  DE  VEJIGA  465

10.  ¿Existe  la  posibilidad  de  cánceres  uroteliales  concurrentes?

Alrededor  del  5%  de  los  pacientes  con  cáncer  de  vejiga  tendrán  carcinoma  urotelial  fuera  de  la  vejiga  (es  decir,  pelvis  
renal,  uréter  o  uretra).

11.  ¿Es  el  CIS  un  tipo  de  cáncer  de  vejiga  menos  agresivo?
No.  El  TCC  in  situ  es  un  tumor  plano  pero  poco  diferenciado.  Puede  hacer  metástasis  y  debe  tratarse  como  una  forma  
agresiva  de  cáncer  de  vejiga.

12.  ¿Cómo  se  maneja  el  carcinoma  de  vejiga  in  situ?
La  inmunoterapia  con  bacilo  de  Calmette­Guerin  (BCG)  intravesical  es  el  tratamiento  de  primera  línea.
Las  tasas  de  respuesta  a  BCG  se  acercan  al  70%.  Otros  agentes  intravesicales,  como  la  mitomicina  C,  por  lo  
general  son  menos  efectivos  que  la  BCG.

13.  ¿Cuáles  son  las  otras  indicaciones  de  BCG  intravesical?

La  presencia  de  tumores  superficiales  de  bajo  grado  multifocales  o  de  gran  volumen;  tumores  superficiales  de  alto  
grado;  o  en  tumores  superficiales  recurrentes.

14.  ¿Cuáles  son  los  efectos  secundarios  de  BCG?

Los  síntomas  leves  de  polaquiuria,  urgencia  y  disuria  son  comunes.  También  se  presentan  mialgias  y  febrícula  
(síntomas  parecidos  a  los  de  la  gripe).  La  fiebre  alta  o  persistente  sugiere  un  problema  más  serio  que  requiere  terapia  
antituberculosa.  En  raras  ocasiones,  se  ha  informado  de  muerte  por  BCG.

15.  ¿Cuándo  podemos  iniciar  el  tratamiento  con  BCG  intravesical?

El  inicio  de  la  terapia  con  BCG  intravesical  generalmente  se  retrasa  de  2  a  3  semanas  después  de  la  TURBT.

16.  ¿Cuál  es  el  hallazgo  patológico  más  importante  a  la  hora  de  elegir  el
¿tratamiento?

La  presencia  de  invasión  muscular.  Los  cánceres  que  no  invaden  los  músculos  se  pueden  tratar  con  vigilancia  y  TURBT  
repetida.  La  presencia  de  cáncer  con  invasión  muscular  exige  un  enfoque  más  agresivo  (es  decir,  cistectomía  radical  [o  
cistoprostatectomía  en  hombres]  con  alguna  forma  de  desviación  urinaria).

17.  ¿Qué  tipos  de  derivación  urinaria  se  utilizan  con  la  cistectomía  radical?
Las  técnicas  de  desviación  requieren  un  conducto  o  un  depósito  continente.  El  más  común  es  un  conducto  ileal.  
El  dispositivo  de  recolección  de  estoma  debe  usarse  con  un  conducto.  Los  reservorios  continentes  se  pueden  

hacer  con  intestino  delgado  o  con  intestino  grueso  y  deben  vaciarse  a  través  de  la  uretra  o  un  estoma  continente.

18.  ¿Cómo  se  trata  el  cáncer  de  vejiga  metastásico?

El  cáncer  de  vejiga  metastásico  requiere  quimioterapia.  La  mayoría  de  los  regímenes  incluyen  un  agente  basado  en  
platino.

19.  En  ciertos  países,  el  TCC  no  es  la  forma  predominante  de  cáncer  de  vejiga.  Qué  es
el  tipo  histológico  predominante?  ¿Por  qué?
En  países  como  Egipto,  donde  la  esquistosomiasis  es  endémica,  el  carcinoma  de  células  escamosas  de  la  vejiga  es  
común.

20.  ¿Hay  algún  marcador  molecular  que  pueda  usarse  para  ayudar  a  predecir  el  pronóstico  del  CCT  vesical?

La  proteína  supresora  de  tumores  p53  puede  ser  útil  para  evaluar  el  comportamiento  biológico  del  tumor  y  puede  
ayudar  con  las  decisiones  de  opciones  de  tratamiento.  El  anticuerpo  monoclonal  MIB­1  también  puede  ser  útil  para  
predecir  el  resultado  de  tumores  en  estadio  T2  o  grado  2.
Machine Translated by Google
466  CAPÍTULO  97  CÁNCER  DE  VEJIGA

PUNTOS  CLAVE:  CÁNCER  DE  VEJIGA

1.  El  cáncer  de  vejiga  se  presenta  como  hematuria  indolora.

2.  El  tipo  histológico  más  común  de  cáncer  de  vejiga  es  el  carcinoma  de  células  de  transición.

3.  El  CIS  de  la  vejiga  se  trata  con  BCG  intravesical.

SITIO  WEB

www.auanet.org/guidelines/bladcan07.cfm

BIBLIOGRAFÍA

1.  Greenlee  RT,  Hill­Harmon  MB,  Murray  T  et  al.:  Estadísticas  de  cáncer  2001.  CA  Cancer  J  Clin  51:15,  2001.

2.  Herr  HW,  Bajorn  DF,  Scher  HL:  ¿Puede  p53  ayudar  a  seleccionar  pacientes  con  cáncer  de  vejiga  invasivo  para  la  vejiga?
¿preservación?  J  Urol  161:20,  1999.

3.  Hendricksen  K,  Witjes  JA:  Estrategias  actuales  para  la  terapia  intravesical  de  primera  y  segunda  línea  para  pacientes  no  musculares
cáncer  de  vejiga  invasivo.  Curr  Opinión  Urol  17:352,  2007.

4.  Soloway  MS,  Lee  CT,  Steinberg  GD  et  al .:  Decisiones  difíciles  en  oncología  urológica:  manejo  del  carcinoma  de  células  de  transición  T1  
de  alto  grado  de  la  vejiga.  UrolOncol  25:338,  2007.

5.  Hall  MC,  Chang  SS,  Dalbagni  G  et  al.:  Guía  para  el  tratamiento  del  cáncer  de  vejiga  no  músculo  invasivo
(estadios  Ta,  T1  y  Tis):  actualización  2007.  JUrol  178:2314,  2007.
Machine Translated by Google

CANCER  DE  PROSTATA
CAPITULO  
98

Fernando  J.  Kim,  MD,  FACS;

1.  ¿Cuál  es  la  prevalencia  del  cáncer  de  próstata  en  los  Estados  Unidos?
Es  la  neoplasia  maligna  más  común  diagnosticada  en  hombres  en  los  Estados  Unidos;  Cada  año  se  diagnostican  
aproximadamente  200.000  nuevos  casos.

2.  ¿La  mayoría  de  los  hombres  mueren  con  cáncer  de  próstata  y  no  a  causa  de  él?
Sí,  pero  más  de  31  500  hombres  morirán  de  cáncer  de  próstata  anualmente  en  los  Estados  Unidos.
Por  lo  tanto,  no  debe  tratarse  como  una  enfermedad  benigna.

3.  ¿Cuáles  son  los  primeros  síntomas  del  cáncer  de  próstata?
No  hay  ninguno.  Cuando  se  desarrollan  síntomas  significativos,  es  probable  que  la  enfermedad  esté  avanzada.
Este  es  un  argumento  a  favor  del  cribado  para  detectar  el  cáncer  de  próstata.

4.  ¿Cuál  es  el  mejor  método  de  detección  del  cáncer  de  próstata?
Examen  rectal  digital  (DRE)  combinado  con  antígeno  prostático  específico  (PSA)  sérico.
Desde  que  se  introdujeron  las  pruebas  de  PSA,  ha  habido  una  migración  de  etapa  con  menos  enfermedad  metastásica  y  
más  enfermedad  local­regional  detectada.

5.  ¿Cómo  se  diagnostica  el  cáncer  de  próstata?
Se  diagnostica  con  biopsia  de  próstata,  que  es  una  biopsia  que  se  utiliza  como  guía  con  ecografía  transrectal  (US)  o  de  
manera  incidental  después  de  realizar  una  resección  transuretral  de  próstata  (RTUP)  por  hiperplasia  prostática  benigna  (HPB).

6.  ¿Cuándo  está  indicada  la  biopsia  de  próstata?
Cuando  el  resultado  de  PSA  o  DRE  es  anormal.

7.  ¿Un  nivel  elevado  de  PSA  significa  que  un  hombre  tiene  cáncer  de  próstata?
No.  El  PSA  puede  estar  elevado  con  HPB,  prostatitis  o  después  de  un  traumatismo  de  próstata.  Es  específico  de  la  próstata,  
no  específico  del  cáncer  de  próstata.

8.  ¿Qué  es  un  anuncio  de  servicio  público  gratuito?

El  PSA  libre  es  el  porcentaje  de  PSA  que  no  se  une  a  una  proteína  sérica  transportadora.  La  proporción  de  PSA  libre  
a  total  es  útil  para  determinar  cuándo  realizar  una  biopsia  de  próstata.  "Libre"  es  bueno  porque  una  proporción  más  
alta  de  PSA  libre  a  total  es  menos  probable  que  represente  cáncer  de  próstata.

9.  ¿Existen  factores  de  riesgo  conocidos  para  el  cáncer  de  próstata?
Sí.  Los  hombres  afroamericanos  y  los  hombres  con  antecedentes  familiares  de  cáncer  de  próstata  tienen  un  mayor  riesgo.
Una  dieta  rica  en  grasas  puede  desempeñar  un  papel  en  el  aumento  del  riesgo  de  muchos  tipos  de  cáncer,  incluido  el  cáncer  de  próstata.

10.  ¿Cuál  es  la  suma  de  Gleason?

Es  una  puntuación  que  el  patólogo  le  da  al  cáncer  de  próstata  para  estimar  su  agresividad.  Los  dos  patrones  predominantes  
de  cáncer  se  puntúan  del  1  al  5  y,  por  lo  tanto,  la  suma  está  entre  2  y  10.  Los  tumores  pueden  estar  bien  diferenciados  (2,  3,  
4),  moderadamente  diferenciados  (5,  6,  7)  o  pobremente  diferenciados.  (8,  9,  10).

467
Machine Translated by Google
468  CAPÍTULO  98  CÁNCER  DE  PRÓSTATA

11.  ¿Cómo  se  trata  el  cáncer  de  próstata  clínicamente  localizado?
Cirugía  (prostatectomía  radical),  radioterapia  por  haz  externo  o  implante  de  semilla  intersticial,  crioterapia,  ultrasonido  
focalizado  de  alta  intensidad  (HIFU)  o  espera  vigilante.

12.  ¿Cómo  se  trata  el  cáncer  de  próstata  metastásico  avanzado?
Terapia  de  ablación  hormonal  (orquiectomía  o  fármacos  agonistas  de  la  hormona  liberadora  de  hormona  
luteinizante)  o  quimioterapia,  pero  estos  tratamientos  son  paliativos  y  no  curativos.

13.  ¿Cuál  es  el  mejor  tratamiento  para  el  cáncer  de  próstata?
Esto  es  muy  controvertido.  Los  pacientes  deben  sopesar  factores  como  la  edad,  el  estado  general  de  salud,  el  
grado  y  la  etapa  de  la  enfermedad  y  el  riesgo  de  efectos  secundarios  frente  a  las  complicaciones  de  las  distintas  
opciones  de  tratamiento.

PUNTOS  CLAVE:  CÁNCER  DE  PRÓSTATA

1.  El  cáncer  de  próstata  es  la  neoplasia  maligna  más  común  diagnosticada  en  hombres  en  los  Estados  Unidos.

2.  El  mejor  método  de  detección  es  una  combinación  de  DRE  y  PSA  sérico.

3.  El  cáncer  de  próstata  clínicamente  localizado  se  trata  con  cirugía,  radiación,  crioterapia  o
espera  atenta.

SITIOS  WEB

www.prostatecancerfoundation.org/

www.cancer.gov/cancertopics/types/prostate

BIBLIOGRAFÍA

1.  Catalunya  WJ:  Utilidad  clínica  del  antígeno  prostático  específico  libre  y  total.  Rev  Prostate  7  (suplemento):  64,  1996.
2.  D'Amico  AV,  Whittington  R,  Malkowicz  SB  et  al.:  Resultado  bioquímico  después  de  la  prostatectomía  radical,  la  radioterapia  
de  haz  externo  o  la  radioterapia  intersticial  para  el  cáncer  de  próstata  clínicamente  localizado.  JAMA  280:969,  1998.

3.  Denberg  TD,  Kim  FJ,  Flanigan  RC  et  al .:  La  influencia  de  la  raza  del  paciente  y  la  vulnerabilidad  social  en  las  
recomendaciones  de  tratamiento  del  urólogo  en  el  carcinoma  de  próstata  localizado.  MedCare  44(12):1137,  2006.
4.  Greenlee  RT,  Hill­Harmon  MB,  Murray  T  et  al.:  Estadísticas  de  cáncer  2001.  CA  Cancer  J  Clin  51:15,  2001.
5.  Keetch  DW,  Humphrey  PA,  Smith  DS  et  al.:  Características  clínicas  y  patológicas  del  cáncer  de  próstata  hereditario.
JUrol  155:1841,  1996.

6.  Polascik  TJ,  Pound  CR,  DeWeese  TL  et  al.:  Comparación  de  prostatectomía  radical  y  tratamiento  intersticial  con  yodo­125
radioterapia  para  el  tratamiento  del  cáncer  de  próstata  clínicamente  localizado:  un  análisis  de  progresión  bioquímica  (PSA)  
de  7  años.  Urología  51:884,  1998.
Machine Translated by Google

URODINÁMICA  Y  VACÍO
DISFUNCIÓN
CAPITULO  
99

Mario  F.  Chammas,  Jr.,  MD;

1.  ¿Qué  es  la  urodinámica?
Los  estudios  urodinámicos  evalúan  los  aspectos  funcionales  de  la  capacidad  de  almacenamiento  y  vaciado  del  
tracto  urinario  inferior  (LUT).  Los  principios  de  los  estudios  urodinámicos  se  originaron  en  la  hidrodinámica.
Los  componentes  de  los  estudios  urodinámicos  son  cistometrograma,  presiones  de  punto  de  fuga,  presiones  de  
perfil  uretral,  estudios  de  flujo  de  presión,  uroflujometría  y  electromiografía.  Estos  estudios  han  evolucionado  hacia  
la  videourodinámica  con  la  adición  de  la  fluoroscopia  (es  decir,  video).

2.  ¿Qué  es  la  uroflujometría?
La  uroflujometría  es  la  medición  de  la  orina  evacuada  (en  mililitros)  por  unidad  de  tiempo  (en  segundos).
Los  elementos  importantes  de  la  prueba  son  el  volumen  anulado  (que  debe  ser  >150  ml),  el  caudal  máximo  (Qmax)  
y  la  curva  del  flujo  (que  debe  tener  forma  de  campana).  En  los  hombres,  un  Qmax  >  15  ml/seg  se  considera  normal,  
mientras  que  un  Qmax  inferior  a  10  ml/seg  se  considera  anormal.  La  asignación  de  valores  normales  en  mujeres  es  
más  difícil.  En  la  mujer,  la  uroflujometría  se  caracteriza  por  la  uretra  más  corta  y  sin  resistencia,  como  la  que  provoca  
la  glándula  prostática  en  el  varón.  Los  valores  normales  se  describen  como  un  Qmax  entre  20  y  36  ml  por  segundo.

3.  ¿Qué  es  la  hiperplasia  prostática  benigna  (HPB)?
La  HPB  es  un  agrandamiento  benigno  de  la  glándula  prostática  que  puede  provocar  síntomas  de  obstrucción  de  
la  salida  de  la  vejiga  en  los  hombres.  Estos  síntomas  se  han  denominado  síntomas  del  tracto  urinario  inferior  (STUI).

4.  ¿Qué  es  una  puntuación  de  síntomas  de  la  Asociación  Americana  de  Urología?
Es  un  cuestionario  autoadministrado  desarrollado  y  popularizado  por  la  American  Urological  Association  
(AUA)  para  la  evaluación  de  STUI  molestos  en  hombres.  Este  cuestionario  consta  de  siete  preguntas  con  una  
puntuación  máxima  de  35.  A  mayor  puntuación,  más  molesto  El  AUA  Symptom  Score  se  ha  convertido  en  un  
índice  tanto  para  el  diagnóstico  como  para  la  evaluación  del  resultado  del  tratamiento  en  pacientes  con  STUI.

5.  ¿Cuáles  son  las  principales  funciones  de  la  LUT?
El  almacenamiento  y  el  vaciado  de  la  orina  son  las  funciones  principales.  A  efectos  prácticos,  todos  los  síntomas  de  la  
disfunción  del  LUT  se  pueden  clasificar  en  el  mal  funcionamiento  de  la  capacidad  de  almacenamiento  o  de  vaciado.

6.  ¿Cuáles  son  los  mecanismos  de  control  para  la  función  LUT?
Los  mecanismos  de  control  para  la  función  LUT  se  reconocen  como  centrales  y  periféricos.  El  mecanismo  de  control  
central  consiste  en  la  porción  cortical  del  lóbulo  frontal  del  cerebro  y  el  centro  pontino  de  la  micción.  El  mecanismo  
de  control  periférico  incluye  las  inervaciones  simpática  torácica  y  parasimpática  lumbar  y  el  aparato  neuromuscular  
de  los  órganos  LUT.

7.  ¿Cuál  es  el  papel  del  sistema  nervioso  autónomo  en  la  función  de  la  LUT?
Las  fibras  simpáticas,  que  se  originan  en  la  porción  T10­L2  de  la  médula  espinal,  inervan  el  cuello  de  la  vejiga  
y  la  uretra  proximal.  Estas  fibras  controlan  principalmente  la  contracción  de

469
Machine Translated by Google
470  CAPÍTULO  99  URODINÁMICA  Y  DISFUNCIÓN  DE  LA  VACUNACIÓN

la  uretra  proximal  o  el  cuello  de  la  vejiga  y  la  relajación  de  la  vejiga,  lo  que  resulta  en  el  almacenamiento  de  
orina.  Las  fibras  parasimpáticas,  que  se  originan  principalmente  en  la  porción  S2­S4  de  la  médula  espinal,  
inervan  el  cuerpo  vesical.  La  inervación  parasimpática  permite  la  contracción  del  músculo  liso  de  la  vejiga,  lo  que  
lleva  al  vaciado  de  la  vejiga.

8.  ¿Hay  alguna  forma  mejor  de  memorizar  esta  función?
Sí.  Parasimpático:  mear;  Simpático:  Almacenamiento.

9.  ¿Cuál  es  el  papel  del  sistema  nervioso  somático  en  la  función  del  LUT?
El  sistema  nervioso  somático  controla  el  control  voluntario  del  músculo  estriado  del  esfínter  urinario  externo.  
Las  fibras  somáticas  son  transportadas  al  esfínter  por  el  nervio  pudendo.

10.  ¿Qué  es  el  reflejo  bulbocavernoso?
El  reflejo  bulbocavernoso  prueba  la  integridad  del  control  neurológico  periférico  del  LUT.
Este  reflejo  es  provocado  por  la  estimulación  del  glande  del  pene  en  los  hombres  o  del  clítoris  en  las  mujeres,  lo  
que  provoca  la  contracción  del  esfínter  anal  externo  o  del  músculo  bulbocarnoso.  Alternativamente,  el  reflejo  
puede  estimularse  tirando  del  globo  de  un  catéter  de  Foley  contra  el  cuello  de  la  vejiga.  Este  reflejo  está  presente  
en  todos  los  hombres  normales  y  en  aproximadamente  el  70%  de  las  mujeres  normales.  La  ausencia  de  este  
reflejo  en  los  hombres  sugiere  fuertemente  una  lesión  neurológica  sacra.

11.  ¿Cuál  es  la  causa  más  frecuente  de  incontinencia  urinaria  en  la  población  geriátrica?
Las  más  comunes  son  causas  transitorias,  en  su  mayoría  externas,  que  alteran  el  frágil  equilibrio  de  la  
función  del  TUI  en  pacientes  de  edad  avanzada  y  provocan  incontinencia  urinaria.  Estas  causas  se  pueden  
recordar  con  la  nemotécnica  DIAPPERS:  Delirio  Infección  Uretritis  atrófica  o  vaginitis  Farmacéutica  Psicológica  
(depresión)

Endocrino  (hipercalcemia,  hiperglucemia)
Movilidad  restringida  
Impactación  de  heces

12.  ¿Qué  es  el  shock  espinal?  ¿Qué  tipo  de  disfunción  urinaria  provoca?
El  shock  espinal  es  la  pérdida  de  contractilidad  del  músculo  liso  por  debajo  del  nivel  de  la  lesión  de  la  médula  
espinal,  lo  que  provoca  dificultad  para  vaciar  la  vejiga  o  retención  urinaria.  Este  fenómeno  puede  durar  desde  
horas  hasta  varios  meses  con  una  alta  probabilidad  de  reversibilidad  si  la  lesión  de  la  médula  espinal  no  es  
permanente.

13.  ¿Qué  es  la  disreflexia  autonómica?  Como  es  tratado?
La  disreflexia  autonómica  resulta  de  la  efusión  sistemática  de  descargas  simpáticas,  como  en  pacientes  
con  lesiones  de  la  médula  espinal  por  encima  del  nivel  T6.  Esta  disreflexia  se  desencadena  por  la  distensión  de  
la  vejiga  u  otro  estímulo  del  intestino  o  LUT.  Se  manifiesta  por  hipertensión,  bradicardia,  sofocos,  sudoración  y  
dolor  de  cabeza.  El  tratamiento  inicial  consiste  en  la  eliminación  del  estímulo,  como  vaciar  la  vejiga  y  colocar  al  
paciente  en  posición  sentada.  Los  fármacos  antihipertensivos  se  pueden  utilizar  como  profilaxis  o  como  
tratamiento  de  episodios  graves.  Esta  condición  puede  conducir  a  una  complicación  cerebrovascular  significativa  
si  no  se  trata.

14.  ¿Qué  tipo  de  disfunción  de  la  vejiga  se  observa  con  frecuencia  en  pacientes  con  diabetes?
Los  pacientes  con  diabetes  pueden  desarrollar  cistopatía  diabética,  que  es  una  complicación  crónica  de  
la  diabetes  con  una  tríada  clásica  de  síntomas:  disminución  de  la  sensación  de  la  vejiga,  aumento  de  la  
capacidad  de  la  vejiga  y  alteración  de  la  contractilidad  del  detrusor.  El  deterioro  de  la  contractilidad  del  detrusor  
puede  provocar  un  vaciado  incompleto  de  la  vejiga  y,  posteriormente,  provocar  dificultad  para  vaciar,  retención  
urinaria,  infección  crónica  del  tracto  urinario  (ITU)  y  daño  del  tracto  urinario  superior.
Machine Translated by Google
CAPÍTULO  99  URODINÁMICA  Y  DISFUNCIÓN  DE  LA  micción  471

15.  ¿Qué  tipo  de  disfunción  de  la  vejiga  se  observa  con  frecuencia  en  pacientes  con  esclerosis  múltiple  (EM)?

La  urgencia  (83  %),  la  incontinencia  de  urgencia  (75  %),  la  hiperreflexia  del  detrusor  (62  %)  y  la  disinergia  del  
detrusor  y  el  esfínter  (25  %)  se  encuentran  entre  los  síntomas  de  LUT  más  comunes  en  pacientes  con  EM.  El  
empeoramiento  de  la  disfunción  de  la  vejiga  se  correlaciona  con  el  aumento  de  la  afectación  de  la  médula  espinal  
y  los  síntomas  neurológicos  en  la  EM.  La  variación  de  los  síntomas  depende  del  sitio  de  afectación  por  la  EM.  La  
afectación  de  las  vías  pontinas  (tegmentum)  se  asocia  con  una  tasa  mucho  mayor  de  síntomas  urinarios.

16.  ¿Qué  raíces  sacras  controlan  la  fisiología  de  la  micción?
Son  S2­S4.

17.  ¿Cuáles  son  las  causas  de  la  retención  de  orina  después  de  una  cirugía  abdominal  o  pélvica?
Son  lesiones  o  disrupción  de  la  inervación  del  plexo  pélvico  del  TUI.

18.  ¿Qué  es  el  síndrome  de  Ogilvie?
La  dilatación  masiva  aguda  del  ciego  y  del  colon  ascendente  y  transverso  sin  evidencia  de  obstrucción  colónica  
distal  se  conoce  como  síndrome  de  Ogilvie.  Este  síndrome  generalmente  se  asocia  con  una  enfermedad  
médica  importante  reciente  o  un  procedimiento  quirúrgico.  Otras  asociaciones  frecuentes  son  traumatismos,  
infecciones  y  cardiopatías,  posiblemente  como  resultado  de  un  desequilibrio  en  la  estimulación  parasimpática  del  
colon.

PUNTOS  CLAVE:  URODINÁMICA  Y  FUNCIÓN  DE  VACÍO
1.  La  uroflujometría  es  la  medición  de  la  orina  evacuada  (en  mililitros)  por  unidad  de  tiempo  (en
segundos).

2.  La  HPB  es  un  agrandamiento  benigno  de  la  glándula  prostática  que  puede  provocar  síntomas  obstructivos  de  la  salida  de  
la  vejiga  en  los  hombres.

3.  Las  raíces  sacras  involucradas  en  la  fisiología  de  la  micción  son  S2­S4.

SITIO  WEB

www.icsoffice.org

BIBLIOGRAFÍA

1.  Cole  EE,  Dmochowski  RR:  Oficina  de  urodinámica.  Urol  Clin  North  Am  32:353,  2005.  2.  de  
Se`ze  M,  Ruffion  A,  Denys  P  et  al.:  La  vejiga  neurogénica  en  la  esclerosis  múltiple:  revisión  de  la  literatura  y
propuesta  de  lineamientos  de  manejo.  Mult  Scler  13:915,  2007.
3.  Gibbs  CF,  Johnson  TM  2nd,  Ouslander  JG:  Manejo  de  oficina  de  incontinencia  urinaria  geriátrica.  Soy  J  Med
120:211,  2007.
4.  Hashim  H,  Abrams  P:  Vejiga  hiperactiva:  una  actualización.  Curr  Opinión  Urol  17:231,  2007.
5.  Messelink  B,  Benson  T,  Berghmans  B  et  al.:  Estandarización  de  la  terminología  de  la  función  y  disfunción  de  los  músculos  
del  suelo  pélvico:  informe  del  grupo  de  evaluación  clínica  del  suelo  pélvico  de  la  Sociedad  Internacional  de  Continencia.
Neurourol  Urodyn  24:374,  2005.
Machine Translated by Google

UROLOGÍA  PEDIÁTRICA
CAPITULO  
100

Kirstan  K.  Meldrum,  MD  y  Mark  P.  Cain,  MD

1.  Una  niña  sana  de  3  años  desarrolla  una  infección  urinaria  febril.  Cómo  debería
ella  ser  evaluada?
Después  del  tratamiento  de  la  infección,  el  paciente  debe  someterse  a  una  evaluación  del  tracto  urinario  
con  una  ecografía  de  la  vejiga  renal  y  una  cistouretrografía  miccional  (VCUG).  Aproximadamente  el  50%  de  los  
niños  menores  de  12  años  que  presentan  una  infección  del  tracto  urinario  (ITU)  tienen  anomalías  del  tracto  
genitourinario  (GU).  Las  anomalías  más  comunes  identificadas  son  el  reflujo  vesicoureteral  (RVU),  las  uropatías  
obstructivas  y  la  vejiga  neurógena.  En  ausencia  de  anormalidades  anatómicas,  las  causas  más  comunes  de  UTI  
en  niños  son  el  estreñimiento  y  la  micción  disfuncional.

2.  ¿Qué  es  la  enfermedad  por  reflujo  vesicoureteral?
El  reflujo  de  orina  desde  la  vejiga  hacia  el  tracto  urinario  superior.  El  RVU  primario  es  causado  por  un  
mecanismo  valvular  inadecuado  en  la  unión  ureterovesical,  presumiblemente  relacionado  con  un  túnel  ureteral  
submucoso  acortado.  La  mitad  de  los  niños  con  infecciones  urinarias  documentadas  por  cultivo  tienen  RVU.

3.  ¿El  RVU  daña  el  riñón?
Es  poco  probable  que  el  reflujo  estéril  cause  daño  renal;  sin  embargo,  el  reflujo  de  la  orina  infectada  puede  
provocar  pielonefritis  y  posterior  cicatrización  renal.  Actualmente,  la  cicatrización  renal  es  la  cuarta  causa  de  
trasplante  renal  en  niños.  La  combinación  de  RVU  y  presiones  de  almacenamiento  vesical  elevadas  (p.  ej.,  
vejiga  neuropática  u  obstrucción  de  la  salida  de  la  vejiga)  es  dañina  para  el  riñón,  ¡y  una  UTI  concurrente  hace  
que  esta  situación  sea  particularmente  peligrosa!

4.  ¿Cuáles  son  las  indicaciones  para  la  corrección  quirúrgica  del  RVU?
El  reflujo  desaparece  espontáneamente  en  muchos  niños;  sin  embargo,  es  poco  probable  que  el  reflujo  de  alto  
grado,  especialmente  cuando  es  bilateral,  se  resuelva  espontáneamente.  Los  niños  con  reflujo  de  alto  grado  o  
UTI  intercurrente  a  pesar  de  la  profilaxis  antibiótica  deben  tratarse  quirúrgicamente.  El  manejo  quirúrgico  
también  es  apropiado  en  niños  con  reflujo  que  persiste  hasta  la  niñez  tardía  o  la  adolescencia.

5.  ¿Cuál  es  la  causa  más  común  de  hidronefrosis  prenatal?
Obstrucción  de  la  unión  ureteropélvica  (UPJ).  La  hidronefrosis  es  la  anomalía  más  común  detectada  en  la  
ecografía  prenatal  (US)  y  representa  el  50%  de  todas  las  lesiones  detectadas  prenatalmente.
El  50%  de  las  hidronefrosis  prenatales,  a  su  vez,  se  debe  a  la  obstrucción  de  la  UPJ.  La  obstrucción  de  la  UPJ  
es  bilateral  en  aproximadamente  el  20%  de  los  casos  y  se  asocia  con  RVU  en  el  15%  de  los  casos.

6.  ¿Cuál  es  la  causa  más  frecuente  de  obstrucción  de  la  UPJ?
Estenosis  intrínseca.  Las  causas  menos  comunes  incluyen  un  vaso  que  cruza  el  polo  inferior  (renal),  inserciones  
ureterales  anómalas  (alto  en  la  pelvis  renal)  y  fibrosis  peripélvica.

7.  ¿La  obstrucción  de  la  UPJ  puede  resolverse  espontáneamente?  ¿Cuáles  son  las  indicaciones  para
pieloplastia?
Sí.  En  última  instancia,  solo  alrededor  del  25%  de  los  niños  con  evidencia  de  obstrucción  UPJ  requieren  pieloplastia.
Las  indicaciones  para  la  intervención  quirúrgica  incluyen  empeoramiento  de  la  hidronefrosis,  función  renal  
deficiente  o  en  declive,  dolor,  infección  y  la  presencia  de  un  riñón  único  o  hidronefrosis  bilateral.

472
Machine Translated by Google
CAPÍTULO  100  UROLOGÍA  PEDIÁTRICA  473

8.  ¿Qué  es  la  ley  de  Meyer­Weigert?
Esta  ley  se  refiere  a  la  posición  de  los  orificios  ureterales  en  pacientes  con  duplicación  ureteral  completa.  
Ocasionalmente,  dos  yemas  ureterales  se  desarrollan  independientemente  del  conducto  mesonéfrico.
A  medida  que  las  yemas  ureterales  son  absorbidas  por  la  vejiga  en  desarrollo,  la  yema  ubicada  en  una  
posición  más  baja  a  lo  largo  del  conducto  (que  drena  el  polo  inferior  del  riñón)  es  llevada  a  una  posición  más  
craneal  y  lateral.  La  yema  ureteral  ubicada  en  una  posición  más  alta  a  lo  largo  del  conducto  (que  drena  el  polo  
superior  del  riñón)  se  lleva  a  una  posición  más  caudal  y  medial  dentro  de  la  vejiga.
Los  uréteres  del  polo  inferior  tienen  más  probabilidades  de  reflujo  debido  a  su  posición  lateral  dentro  de  la  vejiga,  
mientras  que  los  uréteres  del  polo  superior  se  obstruyen  con  mayor  frecuencia  y  se  asocian  más  a  menudo  con  
un  ureterocele.

9.  ¿Qué  es  un  ureterocele?
Un  ureterocele  es  una  dilatación  quística  de  la  porción  distal  del  uréter.  Los  ureteroceles  suelen  asociarse  con  
el  uréter  del  polo  superior  de  un  sistema  colector  duplicado;  sin  embargo,  también  pueden  desarrollarse  a  partir  
de  uréteres  individuales.  Por  lo  general,  son  ectópicos  (es  decir,  una  parte  del  ureterocele  se  coloca  en  el  cuello  
de  la  vejiga  o  la  uretra)  y  con  frecuencia  causan  obstrucción  ureteral.

10.  ¿Qué  es  un  uréter  ectópico?
Un  uréter  con  una  abertura  ectópica  (en  o  caudal  al  nivel  del  cuello  de  la  vejiga).

11.  ¿Cuál  es  el  síntoma  de  presentación  más  común  en  una  niña  con  un  uréter  ectópico?
Incontinencia.  En  las  mujeres,  un  uréter  ectópico  generalmente  drena  en  el  cuello  de  la  vejiga,  la  uretra  
proximal  o  el  vestíbulo.  El  orificio  también  puede  estar  ubicado  en  la  vagina  (25%)  y  ocasionalmente  en  el  útero.  
Cuando  el  orificio  ureteral  ectópico  se  coloca  debajo  del  esfínter  externo  o  dentro  del  tracto  genital  femenino,  
puede  desarrollarse  incontinencia.  La  infección  también  es  un  síntoma  común  de  presentación  de  un  uréter  
ectópico,  que  ocurre  como  consecuencia  de  una  obstrucción  ureteral.

12.  ¿Los  niños  con  uréteres  ectópicos  presentan  incontinencia?
No.  La  vía  ectópica  en  los  niños  se  extiende  desde  el  cuello  de  la  vejiga  a  través  de  la  uretra  posterior  hasta  los  
derivados  del  conducto  mesonéfrico  (es  decir,  el  conducto  deferente,  el  epidídimo  y  la  vesícula  seminal).
Por  tanto,  el  orificio  ureteral  ectópico  siempre  se  sitúa  por  encima  del  mecanismo  de  continencia.

13.  ¿Qué  porcentaje  de  bebés  nacidos  a  término  tienen  un  testículo  no  descendido?
Tres  por  ciento.  Este  número  disminuye  al  0,8%  al  año  de  edad.

14.  ¿Cuál  es  la  ubicación  más  común  de  un  testículo  no  descendido?
El  canal  inguinal  (72%  de  los  testículos  no  descendidos).  El  testículo  también  puede  estar  ubicado  en  el  
abdomen  (8%)  o  en  el  área  preescrotal  (20%).  El  veinte  por  ciento  de  los  testículos  no  descendidos  no  son  
palpables  en  la  presentación;  de  estos,  el  20%  están  completamente  ausentes.

15.  ¿Por  qué  se  debe  volver  a  llevar  el  testículo  al  escroto?
Los  pacientes  con  criptorquidia  tienen  un  riesgo  15­40  veces  mayor  de  cáncer  de  células  germinales  en  
comparación  con  la  población  normal.  Aunque  la  colocación  del  testículo  dentro  del  escroto  no  alivia  este  
riesgo,  permite  un  examen  testicular  completo  y  de  rutina.  Los  pacientes  con  criptorquidia  también  tienen  
riesgo  de  infertilidad.  Los  estudios  histológicos  han  demostrado  una  pérdida  progresiva  de  células  germinales  en  
el  testículo  no  descendido  a  partir  de  los  18  meses  de  edad.  La  orquidopexia  temprana  puede  minimizar  el  grado  
de  pérdida  de  células  germinales  y,  por  lo  tanto,  disminuir  la  posibilidad  de  infertilidad  en  el  futuro.
En  general,  cuanto  más  alto  esté  el  testículo  (es  decir,  dentro  del  abdomen),  mayor  será  el  riesgo  de  cáncer  
e  infertilidad.

16.  ¿Cuál  es  la  causa  más  común  de  obstrucción  de  la  salida  de  la  vejiga  en  los  niños?  en  chicas?
Válvulas  de  uretra  posterior  y  ureterocele,  respectivamente.
Machine Translated by Google
474  CAPÍTULO  100  UROLOGÍA  PEDIÁTRICA

17.  ¿Cuáles  son  las  manifestaciones  urinarias  de  las  válvulas  de  uretra  posterior?
Las  válvulas  de  la  uretra  posterior  son  valvas  congénitas  de  tejido  que  se  extienden  desde  el  verumontanum  
hasta  la  uretra  anterior  en  los  niños.  Ocurren  con  una  incidencia  de  1/8000  nacidos  vivos  de  varones.
Las  válvulas  uretrales  posteriores  causan  obstrucción  de  la  salida  de  la  vejiga,  lo  que  a  su  vez  conduce  a  
grados  variables  de  lesión  vesical  y  renal.  La  obstrucción  grave  puede  provocar  oligohidramnios,  hipoplasia  
pulmonar,  hipertrofia  vesical,  RVU,  hidroureteronefrosis  y  displasia  renal.
El  cincuenta  por  ciento  de  los  niños  afectados  tienen  reflujo  y  el  33%  progresan  a  enfermedad  renal  en  etapa  terminal.

18.  ¿Qué  es  un  mielomeningocele?  ¿Cuáles  son  sus  consecuencias  urológicas?
Un  mielomeningocele  es  una  protrusión  herniaria  de  la  médula  espinal  y  sus  meninges  a  través  de  un  defecto  en  
la  columna  vertebral.  La  lesión  neurológica  resultante  provoca,  entre  otros  problemas,  disfunción  de  la  vejiga.  Los  
pacientes  con  mielomeningocele  suelen  tener  incontinencia  debido  a  hiperactividad  del  detrusor,  hipoactividad  
del  detrusor,  distensibilidad  vesical  deficiente,  resistencia  de  salida  inadecuada,  falta  de  sinergia  entre  el  detrusor  
y  la  salida  o  una  combinación  de  estos  factores.  Más  importante  aún,  los  pacientes  con  vejigas  hiperactivas  y  de  
alta  presión  pueden  desarrollar  deterioro  del  tracto  urinario  superior.
Es  necesario  un  seguimiento  de  por  vida  para  estos  niños  porque  la  lesión  neurológica  puede  cambiar  con  el  
tiempo.  Los  objetivos  del  tratamiento  incluyen  el  mantenimiento  de  un  reservorio  urinario  de  baja  presión,  la  
prevención  de  infecciones  urinarias,  la  prevención  del  deterioro  del  tracto  urinario  superior  y  el  logro  de  la  continencia.

19.  ¿Cuál  es  la  causa  más  común  de  genitales  ambiguos  en  el  recién  nacido?
Hiperplasia  suprarrenal  congénita,  más  comúnmente  como  resultado  de  una  deficiencia  de  21­hidroxilasa.

20.  ¿Qué  evaluación  diagnóstica  se  debe  realizar  en  cualquier  lactante  varón  que  presente
con  hipospadias  y  criptorquidia?
La  presencia  de  criptorquidia  e  hipospadias  debe  alertar  al  médico  sobre  la  posibilidad  de  una  mujer  androgenizada.  
Siempre  se  debe  obtener  un  cariotipo  antes  de  la  intervención  quirúrgica.

21.  ¿Cuál  es  la  masa  renal  sólida  más  frecuente  en  la  infancia?  ¿En  la  niñez?
En  la  infancia:  nefroma  mesoblástico  congénito.  Este  es  un  tumor  benigno  del  riñón  que  se  puede  tratar  solo  
con  escisión  quirúrgica.  En  la  infancia:  tumor  de  Wilm.  El  tumor  de  Wilm  se  asocia  con  el  síndrome  de  
Beckwith­Wiedemann,  hemihipertrofia  aislada  y  aniridia  congénita.  Los  factores  pronósticos  más  importantes  
son  el  estadio  del  tumor  y  la  histología.  El  tratamiento  es  multimodal  y  consiste  en  cirugía,  quimioterapia  y  
radiación.

BIBLIOGRAFÍA

1.  Baker  LA,  Silver  RI,  Docimo  SG:  criptorquidia.  En  Gearhart  JP,  Rink  RC,  Mouriquand  PDE  editores:  Pediátrico
urología  Filadelfia,  2001,  WB  Saunders.
2.  Chang  SL,  Shortliffe  LD:  Infecciones  pediátricas  del  tracto  urinario.  Pediatr  Clin  North  Am  53:379­400,  2006.
3.  Cooper  CS,  Snyder  HM:  Duplicación  ureteral,  ectopia  y  ureteroceles.  En  Gearhart  JP,  Rink  RC,  Mouriquand
Editores  de  PDE:  urología  pediátrica,  Filadelfia,  2001,  WB  Saunders.
4.  Diamond  DA:  Diferenciación  sexual:  normal  y  anormal.  En  Wein  AJ,  Kavoussi  LR,  Novick  AC  et  al.,  editores:
Urología  Campbell­Walsh,  Filadelfia,  2007,  Saunders  Elsevier.
5.  Greenbaum  LA,  Mesrobian  HG:  reflujo  vesicoureteral.  Pediatr  Clin  North  Am  53:  413­427,  2006.
6.  Herndon  CD:  Hidronpehrosis  prenatal:  diagnóstico  diferencial,  evaluación  y  opciones  de  tratamiento.  mundo  cientifico
Revista,  6:2345­2365,  2006.
7.  Hutson  JM,  Clarke  MC:  Manejo  actual  del  testículo  no  descendido.  Semin  Pediatr  Surg  16:64­70,  2007.
8.  Shortliff  LM:  Infección  e  inflamación  del  tracto  genitourinario  pediátrico.  En  Wein  AJ,  Kavoussi  LR,  Novick  AC  et  al.:  
Campbell­Walsh  urology,  Filadelfia,  2007,  Saunders  Elsevier.
9.  Yeung  CK,  Sihoe  JDY,  Bauer  SB:  Disfunción  miccional  en  niños  neurogénicos  y  no  neurogénicos.  En  Wein  AJ,  
Kavoussi  LR,  Novick  AC  et  al.,  editores:  Campbell­Walsh  urology,  Filadelfia,  2007,  Saunders  Elsevier.
10.  Yohannes  P,  Hanna  M:  Tendencias  actuales  en  el  manejo  de  las  válvulas  de  uretra  posterior  en  la  pediatría
población.  Urología  60:947­953,  2002.
Machine Translated by Google

XII.  CUIDADO  DE  LA  SALUD

¿SE  PUEDE  REFORMAR  LA  ATENCIÓN  MÉDICA?
CAPITULO  
101

Dr.  Alden  H.  Harken

1.  ¿Es  la  reforma  de  salud  un  oxímoron?
Sí.

2.  ¿Qué  es  la  tarifa  por  el  servicio?
El  médico  establece  el  precio  y  el  paciente  se  compromete  a  pagarlo.  Este  sistema  tradicional  de  intercambio  
tiene  un  gran  mérito  si  ambas  partes  entienden  el  valor  del  servicio  prestado.  Si  alguna  de  las  partes  (generalmente  
el  paciente)  no  puede  estimar  el  valor  del  servicio,  es  posible  (incluso  probable)  que  el  médico  aumente  
honestamente  el  valor  del  servicio  sin  que  lo  controlen  las  percepciones  del  paciente.  Por  lo  tanto,  en  un  sistema  
de  pago  por  servicio,  los  precios  médicos  tienden  a  aumentar.

3.  ¿Qué  es  la  tarifa  con  descuento  por  el  servicio?
El  paciente  se  reúne  con  un  grupo  de  amigos,  y  acuden  al  médico  con  la  siguiente  proposición:  “Oiga,  doctor,  
puede  deslumbrarnos  con  su  elegante  charla  médica,  pero  seguimos  pensando  que  sus  precios  son  demasiado  
altos.  ¿Qué  tal  si  mis  amigos  y  yo  te  pagamos  el  80%  de  lo  que  nos  cobras?

4.  ¿Hay  alguna  diferencia  entre  los  costos  hospitalarios  y  los  cargos  hospitalarios?
Absolutamente.  El  costo  hospitalario  es  la  suma  de  los  gastos  (p.  ej.,  suturas,  salarios  de  enfermeras,  electricidad,  
esterilización  de  instrumentación,  tiritas)  que  se  gastan  en  suturar  una  laceración,  por  ejemplo.
El  hospital  generalmente  cobra  aproximadamente  el  doble  del  costo  (100%  de  margen  de  ganancia)  por  reparar  un  dedo  cortado.
Este  marcado  es  altamente  específico  de  la  industria.  Por  lo  tanto,  mientras  que  las  cadenas  de  alimentos  intensamente  
competitivas  pueden  obtener  una  ganancia  de  solo  un  centavo  por  una  barra  de  pan,  los  hospitales  y  las  licorerías  
generalmente  cobran  el  doble  del  costo.

5.  ¿Qué  son  los  costos  fijos?
Después  de  tener  en  cuenta  la  luz,  la  calefacción  y  el  personal  (enfermeras,  amas  de  llaves,  administradores)  
en  un  hospital,  pero  antes  de  ver  a  un  solo  paciente,  los  médicos  y  el  hospital  ya  han  gastado  una  gran  cantidad  
de  dinero.  Los  médicos  y  hospitales  deben  pagar  costos  fijos,  ya  sea  que  brinden  o  no  algún  servicio  médico.

6.  ¿Cuáles  son  los  costos  reales?
Estos  son  los  costos  incrementales  de  brindar  realmente  un  servicio  en  un  hospital  (además  de  los  costos  
fijos  de  luz  y  calefacción).  Por  ejemplo,  un  paciente  se  presenta  en  el  servicio  de  urgencias  a  medianoche  
quejándose  de  un  bulto  en  la  punta  de  la  nariz.  El  médico,  con  su  erudición  característica,  dice:  "Sí,  tiene  una  
verruga  en  la  nariz",  y  envía  al  paciente  a  su  casa  con  una  cuenta  de  $500.  El  costo  real  de  este  encuentro  es  
obviamente  insignificante.  El  paciente  realmente  está  pagando  los  costos  fijos  de  las  enfermeras  y  el  equipo  de  
reanimación  de  emergencia  en  caso  de  que  tenga  un  paro  cardíaco.

7.  ¿Es  la  contabilidad  hospitalaria  un  análisis  precisamente  científico  y  objetivo  de  los
¿datos?
No.

475
Machine Translated by Google
476  CAPÍTULO  101  ¿SE  PUEDE  REFORMAR  LA  ATENCIÓN  MÉDICA?

8.  ¿Qué  es  un  seguro  médico?
Tradicionalmente,  las  personas  pueden  comprar  un  seguro  que  puede  pagar  la  totalidad  o  una  parte  de  los  
cargos  del  hospital  y  del  médico  si  se  enferman.  Las  compañías  de  seguros  obtienen  ganancias,  por  lo  tanto,  
solo  si  el  paciente  se  mantiene  saludable.  Las  compañías  de  seguros  tienen  tablas  elaboradas  para  predecir  quién  se  
enfermará  y  prefieren  vender  pólizas  exclusivamente  a  personas  jóvenes  y  sanas.
Esta  práctica  se  denomina  "skimming".  La  compañía  de  seguros  asume  todo  el  riesgo,  y  les  gusta  mantenerlo  
bajo.  Por  el  contrario,  los  hospitales  deben  cubrir  los  costos  fijos,  y  cuanto  más  costosa  (y  más  frecuente)  
sea  la  atención  médica  que  brindan  los  médicos,  mejor  para  los  hospitales.

9.  ¿Qué  son  las  organizaciones  de  mantenimiento  de  la  salud?
Las  organizaciones  de  mantenimiento  de  la  salud  (HMO)  son  sistemas  complejos  compuestos,  en  su  forma  
más  completa,  por  hospitales,  médicos,  consultorios  y  una  compañía  de  seguros.  Las  HMO  contratan  a  grandes  
grupos  de  personas  (pacientes  potenciales)  para  mantener  su  salud.  Los  afiliados  pagan  una  tarifa  mensual  (al  igual  
que  el  seguro  de  salud)  para  que  todos  los  gastos  médicos  y  de  hospital  estén  cubiertos  si  los  afiliados  se  enferman.  
Sin  embargo,  a  diferencia  del  seguro  de  salud,  en  el  modelo  HMO,  los  hospitales  y  los  médicos  reciben  un  pago,  ya  
sea  que  el  afiliado  se  enferme  o  no.  Por  lo  tanto,  es  mejor  para  todos  si  los  afiliados  se  mantienen  saludables  y  fuera  
del  hospital.

10.  Inicialmente,  a  muchos  médicos  no  les  gustaban  las  HMO.  ¿Por  qué?
Porque  los  médicos  son  ferozmente  independientes.  No  querían  que  un  montón  de  gerentes  de  negocios  les  dijeran  
cómo  manejar  a  los  pacientes.

11.  ¿Por  qué  los  médicos  son  ferozmente  independientes?
Probablemente  nacimos  así.

12.  ¿Es  eso  bueno?
Probablemente  no.  Eventualmente,  todos  deberán  trabajar  juntos  y  no  golpearse  cuando  estén  enojados.

13.  ¿Los  administradores  de  HMO  realmente  dictan  cómo  los  médicos  manejan  a  sus  pacientes?
Si  y  no.  Los  médicos  han  desarrollado  estrategias  médicamente  efectivas  y  óptimamente  eficientes,  
denominadas  vías  clínicas,  para  el  cuidado  de  muchas  enfermedades  comunes.  Aunque  los  médicos  deben  
tratar  a  cada  paciente  individualmente,  cuando  nos  adherimos  a  las  pautas  de  tratamiento  predeterminadas  (como  
recomiendan  los  administradores  de  HMO),  los  pacientes  generalmente  mejoran  más  rápido  y  de  manera  más  
económica.

14.  ¿Siguen  los  médicos  estas  vías  clínicas?
Tradicionalmente,  no.

15.  ¿Qué  hacen  los  administradores  de  HMO?
Evalúan  el  uso  que  hace  cada  médico  de  recursos  costosos  (dentro  de  las  vías  clínicas  predeterminadas)  en  
relación  con  la  salud  de  los  pacientes  del  médico.

16.  ¿Acogen  con  beneplácito  los  médicos  este  tipo  de  escrutinio?
No.

17.  ¿Qué  es  una  organización  de  proveedores  preferidos  (PPO)?
Una  PPO  es  un  grupo  de  médicos  que  han  optado  por  permanecer  legalmente  independientes  de  un  hospital  y  
una  compañía  de  seguros  (si  se  unieran,  serían  una  HMO)  y,  sobre  todo,  de  los  pacientes.  Pero  las  PPO  
mantienen  su  independencia  como  médicos,  aunque  la  mayoría  de  las  PPO  requieren
Machine Translated by Google
CAPÍTULO  101  ¿SE  PUEDE  REFORMAR  LA  ATENCIÓN  MÉDICA?  477

administradores  para  coordinar  programas,  llevar  los  libros  y  evitar  que  los  médicos  se  golpeen  entre  sí.  Sin  embargo,  
las  PPO  tienen  la  percepción  de  independencia.

18.  ¿Es  costosa  la  atención  médica?
Por  desgracia  sí.  Los  médicos  argumentan  que  los  pacientes  pagan  mucho  pero  también  obtienen  mucho.  En  los  
Estados  Unidos,  los  pacientes  esperan  acceso  ilimitado  al  trasplante  de  hígado  y  a  la  resonancia  magnética  nuclear  
(RMN)  para  cada  dolor  de  cabeza.  Los  estadounidenses  creen  que  la  atención  médica  lujosa  y  costosa  no  es  solo  un  
privilegio,  es  un  derecho.

19.  Entonces,  ¿cuál  es  el  problema?
Los  directores  ejecutivos  (CEO)  de  las  grandes  corporaciones  estadounidenses  argumentan  que  el  gasto  obligatorio  en  
atención  médica  está  elevando  el  costo  de  los  productos  estadounidenses  y  haciendo  que  las  empresas  estadounidenses  
sean  menos  competitivas  en  el  mercado  global:  hay  más  atención  médica  que  acero  en  un  Chevrolet  nuevo.

20.  ¿Tienen  las  grandes  empresas  una  solución?
Ellos  piensan  que  sí.  Los  directores  ejecutivos  todavía  quieren  acceso  ilimitado  a  la  atención  médica  más  moderna  
para  ellos  y  sus  familias.  Sin  sonar  cínicos,  los  directores  ejecutivos  quieren  ahorrar  los  dólares  gastados  en  atención  
médica  para  sus  empleados  y  "las  familias  de  otras  personas".  Quieren  limitar  el  acceso  a  la  atención  médica,  pero  no  
quieren  empuñar  el  hacha  personalmente.  Entonces  desarrollaron  la  idea  de  la  capitación.

21.  ¿Qué  es  la  capitación?
Los  directores  ejecutivos  de  las  grandes  empresas  vienen  a  los  hospitales,  HMO  o  PPO  y  dicen:  "¿Por  qué  no  brindan  
atención  médica  completa  a  todos  mis  empleados  a  un  precio  fijo,  digamos,  $  180  por  mes  por  cabeza?" ).  En  este  
modelo,  los  médicos  toman  las  decisiones  sobre  quién  recibe  cuánta  atención  médica  (satisfaciendo  su  necesidad  de  
independencia),  pero  también  prometen  brindar  toda  la  atención  médica  necesaria  por  un  precio  preestablecido.  Por  lo  
tanto,  asumen  todo  el  riesgo.  A  los  directores  ejecutivos  les  gusta  este  modelo  porque  aún  pueden  ofrecer  atención  
médica  como  un  beneficio  para  los  empleados  y  presupuestar  el  costo  por  adelantado.

22.  ¿Por  qué  a  los  médicos  no  les  gusta  la  capitación?
De  repente,  los  médicos  pueden  haber  adquirido  un  poco  más  de  independencia  de  lo  que  esperaban.  
Ahora  se  les  paga  por  adelantado,  de  modo  que  todos  los  costos  de  la  atención  médica  de  los  pacientes  se  restan  
del  dinero  que  negociaron  por  adelantado.  Ahora  deben  desaconsejar  una  resonancia  magnética  para  cada  dolor  
de  cabeza  y  darle  la  noticia  a  Granny  de  que  no  se  lo  pensará  mejor  si  le  reducen  el  nitrógeno  ureico  en  sangre  (BUN)  a  
50.  Esto  es  lo  contrario  de  los  buenos  viejos  tiempos  cuando  los  médicos  eran  recompensados  si  sus  pacientes  se  
enfermaron  y  permanecieron  enfermos.  Los  médicos  podrían  surtirlos  con  una  mezcla  heterogénea  de  medicamentos  y  
tecnologías.  Ahora  los  médicos  están  tratando  de  controlar  los  costos  de  atención  médica.

23.  ¿Es  bueno  todo  este  cambio?
Absolutamente.  La  medicina  siempre  ha  cambiado,  y  cuanto  más  rápido,  mejor.  Inicialmente,  los  médicos  se  sintieron  
atraídos  por  la  medicina  como  una  disciplina  intelectualmente  estimulante  porque  la  medicina  y  la  cirugía  evolucionan  
rápidamente.

24.  ¿Pueden  los  médicos  mantenerse  al  día  con  todo  este  cambio?
Absolutamente.

25.  A  pesar  de  todos  los  Chicken  Littles  médicos  que  declaran  sonoramente  que  el  cielo  se  está  cayendo,  ¿sigue  siendo  la  
medicina  (y  más  claramente  la  cirugía)  la  profesión  más  gratificante,  estimulante  y  gratificante?

Absolutamente.
Machine Translated by Google
478  CAPÍTULO  101  ¿SE  PUEDE  REFORMAR  LA  ATENCIÓN  MÉDICA?

BIBLIOGRAFÍA

1.  Blumenthal  D:  Control  de  los  gastos  de  atención  médica.  N  Engl  J  Med  344:766­769,  2001.

2.  Dudley  RA,  Luft  HS:  Atención  administrada  en  transición.  N  Engl  J  Med  344:1087­1092,  2001.

3.  Fuchs  VR:  ¿Qué  se  avecina  para  los  seguros  de  salud  en  los  Estados  Unidos?  N  Engl  J  Med  346:1822­1824,  2002.

4.  Iglehart  JK:  Cambiando  las  tendencias  de  los  seguros  de  salud.  N  Engl  J  Med  347:956­962,  2002.

5.  Schroeder  SA:  Perspectivas  de  expansión  de  la  cobertura  de  seguros  de  salud.  N  Engl  J  Med  344:847­852,  2001.

6.  Wilensky  GR:  Reforma  de  Medicare:  ahora  es  el  momento.  N  Engl  J  Med  345:458­462,  2001.

7.  Wood  AJ:  Cuando  un  mayor  beneficio  terapéutico  conlleva  un  mayor  costo.  N  Engl  J  Med  346:1819­1821,  2002.

8.  Wright  JG:  Barreras  ocultas  para  mejorar  la  calidad  de  la  atención  médica.  N  Engl  J  Med  346:1096,  2002.
Machine Translated by Google

ÉTICA  EN  EL  INTENSIVO  QUIRÚRGICO
QUE  UNIDAD
CAPITULO  
102

Brian  P.  Callahan,  MD  y  Kathryn  Beauchamp,  MD

1.  ¿Cuáles  son  los  cuatro  principios  de  la  ética  médica?
1.  La  beneficencia  describe  el  papel  activo  de  hacer  el  bien  mediante  la  intervención.
2.  La  no  maleficencia  equivale  a  decir:  "Primero,  no  hacer  daño".  3.  La  
autonomía  da  cuenta  del  consentimiento  informado,  la  competencia  y  el  derecho  del  paciente  a  rechazar  el  
tratamiento  ya  saber  qué  está  pasando.
4.  Justicia  significa  que  todos  los  pacientes  deben  recibir  atención  justa  y  equitativa,  pero  que  la  atención  de  
un  paciente  no  debe  desperdiciar  los  recursos  limitados  para  otros.

2.  ¿Qué  es  una  orden  de  no  resucitar?
Una  orden  de  no  resucitar  (DNR,  por  sus  siglas  en  inglés)  indica  al  médico  que  no  intervenga  si  el  
paciente  no  tiene  pulso  o  tiene  apnea;  sin  embargo,  una  orden  DNR  es  mucho  más  complicada  de  lo  que  el  
acrónimo  le  haría  creer.  DNR  no  es  absoluto.  Una  orden  DNR  no  tiene  ninguna  implicación  en  ninguna  otra  
decisión  de  tratamiento.  La  Comisión  Conjunta  para  la  Acreditación  de  Organizaciones  de  Atención  Médica  
(JCAHO)  exige  que  los  hospitales  tengan  pautas  escritas  que  promuevan  la  responsabilidad  de  las  órdenes  DNR.  
Todas  las  órdenes  de  DNR  deben  documentarse  por  escrito,  al  igual  que  todas  las  demás  órdenes,  en  la  sección  
correspondiente  de  la  historia  clínica  del  paciente.  Deben  especificar  los  tratamientos  que  se  suspenderán  y  los  
tratamientos  que  el  paciente  desea  que  se  implementen.  Los  pacientes  y  las  familias  deben  participar  en  la  
decisión  de  DNR.  Además,  el  estado  de  DNR  debe  discutirse  y  revisarse  con  los  demás  miembros  del  equipo  de  
atención  médica.  Finalmente,  una  orden  DNR  no  significa  no  tratar.  Los  pacientes  con  DNR  no  deben  ser  
abandonados  médicamente.

3.  ¿Cuál  es  la  diferencia  entre  retirar  y  retener  la  manutención?
La  retirada  de  la  atención  es  cuando  se  eliminan  todas  las  medidas  de  soporte  vital,  mientras  que  la  retención  
de  la  atención  significa  que  no  se  acelera  la  atención.  Ambas  decisiones  cambian  el  objetivo  del  tratamiento  
de  mantener  la  vida  y  restaurar  la  salud  a  mantener  la  comodidad  durante  el  proceso  de  muerte.  No  existe  una  
distinción  moral  o  ética  entre  el  retiro  y  la  retención  de  apoyo.  Cualquiera  de  los  dos  permite  la  progresión  natural  
de  la  enfermedad  sin  la  interfaz  de  la  tecnología  médica.
La  decisión  de  retirar  o  retener  el  apoyo  no  equivale  a  la  muerte  del  paciente,  aunque  la  probabilidad  de  
muerte  puede  ser  mayor.  Una  vez  que  se  ha  tomado  la  decisión,  el  manejo  adecuado  debe  centrarse  en  la  
comodidad  y  el  apoyo  psicosocial  del  paciente.

4.  ¿Qué  es  una  directiva  anticipada?
Una  directiva  anticipada  es  un  método  para  delinear  los  deseos  de  aplicación  de  un  paciente  competente  en  un  
momento  en  que  ya  no  es  competente.  Maximiza  la  autonomía  del  paciente,  ayuda  a  los  médicos  en  la  toma  
de  decisiones,  optimiza  el  uso  de  los  recursos  médicos  y  brinda  protección  contra  litigios.  El  manejo  médico  o  la  
falta  del  mismo  puede  basarse  en  los  deseos  del  paciente  en  lugar  de  un  sentido  percibido  de  lo  que  es  mejor  para  
el  paciente.  Las  directivas  anticipadas  pueden  ser  un  documento  informal,  como  un  testamento  en  vida,  o  un  
documento  legal  formal,  como  un  poder  médico  duradero.  Las  leyes  de  instrucciones  anticipadas  varían  de  estado  
a  estado  y  se  activan  cuando

479
Machine Translated by Google
480  CAPÍTULO  102  ÉTICA  EN  LA  UNIDAD  DE  CUIDADOS  INTENSIVOS  QUIRÚRGICOS

un  paciente  se  encuentra  en  estado  terminal,  estado  o  inconsciencia  permanente,  o  estado  vegetativo  persistente.  A  pesar  de  
una  mayor  conciencia  de  las  directivas  anticipadas,  solo  el  25%  de  los  adultos  las  tienen.

5.  ¿Qué  es  un  poder  notarial  duradero?
Un  poder  notarial  duradero  es  un  apoderado  designado  por  el  paciente  para  tomar  decisiones.  El  tomador  de  decisiones  por  
poder  se  activa  tan  pronto  como  el  paciente  ya  no  puede  tomar  decisiones  médicas  competentes.  Por  lo  tanto,  el  poder  notarial  
duradero  debe  haberse  establecido  antes  del  deterioro  cognitivo  del  paciente.

6.  ¿Qué  es  un  testamento  en  vida?
Una  lista  de  instrucciones  hechas  por  una  persona  competente  sobre  el  tratamiento  médico  futuro.  Produce  una  guía  previa  a  la  
enfermedad  para  futuros  cuidadores  de  acuerdo  con  los  deseos  del  paciente.

7.  ¿Qué  se  incluye  en  el  consentimiento  informado?

El  consentimiento  informado  es  una  decisión  voluntaria  tomada  por  el  paciente  o  en  nombre  del  paciente  por  un  apoderado  
para  la  toma  de  decisiones.  Consiste  en  una  explicación  del  estado  del  paciente  que  se  va  a  tratar.  Explica  el  tratamiento  
propuesto  en  términos  sencillos.  Detalla  los  posibles  beneficios  y  riesgos  que  son  razonablemente  posibles.  También  incluye  una  
discusión  sobre  terapias  alternativas  y  sus  posibles  resultados.  Es  importante  destacar  que  también  debe  mencionar  el  resultado  
si  no  se  realiza  ningún  tratamiento.  Por  último,  se  deben  abordar  todas  las  preguntas  e  inquietudes  del  paciente  o  del  apoderado.

8.  ¿Qué  son  los  cuidados  inútiles  y  la  inutilidad  médica?
En  última  instancia,  la  vejez  y  la  enfermedad  nos  conquistarán  a  todos.  La  definición  de  tratamiento  médicamente  inútil  
o  inapropiado  todavía  se  debate.  No  obstante,  hay  cuatro  conceptos  principales  de  futilidad  médica:  1.  Los  profesionales  de  la  
salud  no  están  obligados  a  proporcionar  un  tratamiento  fisiológicamente  inútil.

2.  La  muerte  inminente  es  un  argumento  en  contra  del  tratamiento  si  el  paciente  no  tiene  probabilidades  de  supervivencia  para
descargar.
3.  Bajo  el  concepto  de  condición  letal,  la  atención  médica  se  considera  inútil  si  el  paciente  sobrevivirá  temporalmente  pero  
finalmente  fallecerá  como  resultado  del  proceso  de  la  enfermedad  en  curso.
4.  La  calidad  de  vida  o  futilidad  cualitativa  argumenta  en  contra  del  tratamiento  si  la  calidad  de  vida  del  paciente  es
tan  pobre  que  no  sería  razonable  prolongar  la  vida.
Sin  embargo,  se  debe  tener  cuidado  al  tomar  decisiones  médicas  basadas  en  la  inutilidad  porque  estas  decisiones  pueden  
conducir  a  profecías  autocumplidas.

9.  ¿Cuáles  son  los  determinantes  clínicos  de  la  muerte  encefálica?

Muchos  de  los  conceptos  actuales  de  muerte  cerebral  se  basan  en  el  informe  de  1968  del  comité  ad  hoc  de  la  Escuela  de  
Medicina  de  Harvard,  que  pidió  una  nueva  definición  neurológica  de  muerte  cerebral.  Pero  no  fue  hasta  1981  que  James  
Bemat  justificó  los  criterios  neurológicos  de  muerte  cerebral  al  enfatizar  la  necesidad  de  una  función  integradora  intacta  del  tronco  
encefálico  para  que  una  persona  funcione  como  un  todo.  La  Ley  de  Determinación  Uniforme  de  la  Muerte  (UDDA)  define  la  
definición  legal  de  muerte  cerebral  en  los  Estados  Unidos  como  la  determinación  de  la  muerte,  utilizando  criterios  neurológicos,  de  
un  paciente  que  ha  sufrido  un  cese  irreversible  de  todas  las  funciones  del  cerebro,  incluido  el  tronco  encefálico.  Esta  determinación  
debe  hacerse  según  los  estándares  médicos  aceptados.  Los  estándares  médicos  aceptados  varían  de  un  estado  a  otro  y  de  un  
hospital  a  otro.

El  primer  paso  es  documentar  la  causa  de  la  pérdida  permanente  de  las  funciones  cerebrales.  Esto  generalmente  se  hace  
mediante  la  obtención  de  una  tomografía  computarizada  (TC).  Esto  generalmente  muestra  edema  masivo  o  isquemia  difusa.  
Deben  descartarse  causas  reversibles  de  disminución  de  la  función  neurológica.
Esto  incluye  trastornos  metabólicos  (electrolitos  o  ácido­base)  e  intoxicación.  Se  debe  realizar  una  prueba  de  toxicidad  en  
todos  los  pacientes.  Debe  haber  una  ausencia  de  todos  los  agentes  sedantes  y  neuromusculares  durante  al  menos  5  a  7  
vidas  medias  del  fármaco.  Es  necesario  corregir  la  hipotermia,  la  hipotensión  y  la  hipoxia.  El  examen  clínico  incluye  ausencia  de  
respuesta  motora  a  estímulos  nocivos,  ausencia  de  reflejos  del  tronco  encefálico  (pupilas  fijas,  ausencia  de  córnea,  oculocefálico,
Machine Translated by Google
CAPÍTULO  102  ÉTICA  EN  LA  UNIDAD  DE  CUIDADOS  INTENSIVOS  QUIRÚRGICOS  481

respuestas  oculovestibulares,  arcadas  o  de  tos)  y  ausencia  de  impulso  respiratorio  después  de  una  prueba  
de  provocación  con  dióxido  de  carbono  (CO2)  (prueba  de  apnea).  Si  el  paciente  tiene  lesiones  que  no  permiten  
una  prueba  neurológica  completa,  como  un  traumatismo  facial  o  una  lesión  de  la  médula  espinal,  será  necesario  
realizar  pruebas  de  confirmación.  Estos  incluyen  angiografía,  Doppler  transcraneal,  electroencefalograma  (EEG)  
o  gammagrafía  con  radionucleótidos.  En  los  niños,  el  proceso  es  idéntico,  excepto  que  los  niños  entre  1  y  18  
años  deben  repetir  la  prueba  después  de  12  horas,  y  los  niños  <1  deben  tener  48  horas  de  observación  con  al  
menos  una  prueba  de  confirmación.

10.  ¿Qué  es  un  estado  vegetativo  persistente?
En  un  estado  vegetativo  persistente,  típicamente  visto  después  de  la  mejora  de  un  estado  comatoso,  el  
paciente  abre  los  ojos  pero  muestra  solo  un  comportamiento  reflejo.  El  paciente  parece  estar  despierto  pero  
no  tiene  conciencia  de  su  entorno  ni  de  actividad  mental  superior.  Otros  nombres  para  esta  entidad  son  coma  
vigil  y  mutismo  acinético.

11.  ¿Qué  es  la  eutanasia?
La  eutanasia  requiere  que  el  médico  desempeñe  un  papel  activo  ayudando  en  la  muerte  del  paciente.
Los  conceptos  de  suicidio  asistido  por  un  médico  y  eutanasia  activa  y  pasiva  son  muy  controvertidos.  En  
1992,  la  Sociedad  de  Medicina  de  Cuidados  Críticos  publicó  los  resultados  de  una  encuesta  de  especialistas  
en  cuidados  críticos;  El  87%  había  retirado  el  apoyo  para  prolongar  la  vida  de  los  pacientes.  Además,  la  ley  
estadounidense  más  reciente  relacionada  con  el  suicidio  asistido  se  aprobó  en  Oregón  en  1994.
Esta  ley  hace  que  sea  legal  que  un  médico  prescriba  medicamentos  a  pacientes  con  enfermedades  
terminales  con  el  propósito  de  suicidarse.

12.  ¿Quién  debe  dirigirse  a  las  familias  de  los  pacientes  sobre  la  donación  de  órganos?
La  organización  de  obtención  de  órganos  debe  ser  la  primera  en  acercarse  a  la  familia.  Esto  se  llama  el  principio  
de  desacoplamiento  y  elimina  cualquier  conflicto  de  intereses.  Algunos  afirman  que  el  médico  que  ha  establecido  
una  buena  relación  con  la  familia  del  paciente  debería  plantear  el  tema  de  la  donación  de  órganos.  El  mejor  
enfoque  es  probablemente  uno  combinado.  Después  de  que  la  organización  de  obtención  de  órganos  se  haya  
acercado  al  paciente,  los  médicos  deben  estar  disponibles  si  la  familia  desea  hablar  con  ellos.

13.  ¿Qué  se  les  debe  decir  a  las  familias  de  los  pacientes  cuando  la  donación  de  órganos  es  factible?
El  cirujano  debe  enfatizar  que  el  paciente  ha  muerto  a  pesar  de  que  el  corazón  late  activamente  y  que  no  sufre  
ni  siente  ningún  dolor.  Se  debe  preguntar  a  la  familia  sobre  los  deseos  del  paciente  con  respecto  a  la  donación  
de  órganos.  Todos  los  temas  deben  basarse  en  los  conceptos  de  consentimiento  informado.  Se  debe  informar  
a  la  familia  de  la  probabilidad  de  que  varios  pacientes  se  beneficien  de  los  órganos  donados.  La  familia  debe  
comprender  que  no  hay  garantía  de  que  los  órganos  sean  aptos  para  la  donación.  Se  les  debe  asegurar  que  no  
son  responsables  del  costo  de  la  atención  brindada  después  de  que  se  determine  la  muerte  cerebral  y  que  
pueden  rechazar  la  donación  de  órganos  sin  temor  a  sufrir  prejuicios.

14.  ¿Qué  es  la  donación  de  órganos  después  de  la  muerte  cardiaca?
Es  el  proceso  en  el  que  se  obtienen  órganos  de  un  paciente  que  no  ha  sido  declarado  con  muerte  
cerebral.  La  mayoría  de  estos  pacientes  han  sufrido  lesiones  cerebrales  o  de  la  médula  espinal  catastróficas,  
pero  no  cumplen  con  la  definición  clínica  de  muerte  cerebral.  Se  realiza  el  mismo  procedimiento  de  
consentimiento  informado  que  con  los  pacientes  con  muerte  cerebral.  El  paciente  es  llevado  al  quirófano  (OR)  
y  se  retira  la  atención.  Un  médico,  que  no  está  involucrado  en  la  obtención  de  los  órganos  ni  en  el  trasplante  
de  los  órganos,  está  presente  y  declara  la  muerte.  Después  de  que  se  declara  la  muerte  cardíaca,  el  equipo  de  
adquisiciones  ingresa  al  quirófano  y  continúa.

15.  ¿Cuál  es  el  papel  del  comité  de  ética  del  hospital?
El  comité  de  ética  del  hospital  educa  a  los  miembros  del  personal  del  hospital  y  proporciona  una  fuente  
de  consulta.
Machine Translated by Google
482  CAPÍTULO  102  ÉTICA  EN  LA  UNIDAD  DE  CUIDADOS  INTENSIVOS  QUIRÚRGICOS

La  función  de  la  educación  se  logra  a  través  de  grandes  rondas,  seminarios,  conferencias  
especiales  y  clubes  de  revistas.  El  comité  de  ética  del  hospital  debe  ser  visto  como  una  parte  
intrínseca  de  la  comunidad  hospitalaria.  Las  políticas  desarrolladas  deben  ser  revisadas  por  otros  
comités  y  divisiones  del  hospital  para  fomentar  un  mejor  sentido  de  cohesión  cuando  surjan  dilemas  
éticos  y  morales.
El  comité  de  ética  del  hospital  está  integrado  por  médicos  y  personal  auxiliar  con  especial  
formación  en  ética  médica.  Pueden  y  deben  proporcionar  un  escenario  para  la  colaboración  y  la  
educación  ética  general  dentro  del  hospital.

BIBLIOGRAFÍA

1.  Comité  Ad  Hoc  de  la  Escuela  de  Harvard  para  Examinar  la  Definición  de  Muerte  Cerebral:  Una  definición  de  coma  irreversible.  JAMA  
205:337­340,  1968.

2.  Bernat  JL:  ¿Cómo  podemos  conseguir  uniformidad  en  las  determinaciones  de  muerte  encefálica?  Neurología  J70(4):284­289,  2008.

3.  Harken  AH:  Ya  es  suficiente.  Arch  Surg  10:1061­1063,  1999.

4.  Laureys  S,  Boly  M:  ¿Cómo  es  ser  vegetativo  o  mínimamente  consciente?  Curr  Opin  Neurol  20(6):609­613,
2007.

5.  Luce  JM,  White  DB:  La  presión  para  retener  o  retirar  la  terapia  de  soporte  vital  de  pacientes  críticamente  enfermos  en  el
Estados  Unidos.  Am  J  Respir  Crit  Care  Med  177(1):1104­1108,  2008.

6.  Mirarchi  FL:  ¿Un  testamento  en  vida  es  igual  a  un  DNR?  ¿Los  testamentos  en  vida  comprometen  la  seguridad  del  paciente?  J  Emerg  Med  33
(3):299­305,  2007.

7.  Nishimura  A,  Mueller  PS,  Evenson  LK  et  al.:  Pacientes  que  completan  directivas  anticipadas  y  lo  que  prefieren.
Mayo  Clin  Proc  82  (12):  1480–1486.

8.  Steinbrook  R:  Donación  de  órganos  después  de  la  muerte  cardíaca.  New  Engl  J  Med  357(3):209­213,  2007.

9.  Wijdicks  EF:  10  preguntas  sobre  la  determinación  clínica  de  la  muerte  encefálica.  Neurólogo  13(6):380­381,  2007.
Machine Translated by Google

PROFESIONALISMO
CAPITULO  
103

U.  Mini  B.  Swift,  MD,  David  Altman,  MD,  MBA  y  Alden  H.  Harken,  MD

1.  ¿Qué  es  una  profesión?
Las  profesiones  son  los  medios  a  través  de  los  cuales  se  organizan  los  servicios  complejos  que  necesita  la  sociedad.
Una  profesión  ha  sido  definida  por  el  Colegio  Americano  de  Cirujanos  como:

una  ocupación  cuyo  elemento  central  es  el  trabajo  que  se  basa  en  el  dominio  de  un  conjunto  complejo  de  
conocimientos  y  habilidades.  Es  una  vocación  en  la  que  el  conocimiento  de  algún  departamento  de  la  ciencia  
o  el  aprendizaje,  o  la  práctica  de  un  arte  basado  en  él,  se  utiliza  al  servicio  de  los  demás.  Sus  miembros  se  
rigen  por  códigos  de  ética  y  profesan  un  compromiso  con  la  competencia,  la  integridad  y  la  moralidad,  el  
altruismo  y  la  promoción  del  bien  público  dentro  de  su  dominio.  Estos  compromisos  forman  la  base  de  un  
contrato  social  entre  una  profesión  y  la  sociedad  que,  a  su  vez,  otorga  a  la  profesión  el  monopolio  sobre  el  
uso  de  su  base  de  conocimientos,  el  derecho  a  una  autonomía  considerable  en  la  práctica  y  el  privilegio  de  la  
autorregulación.  Las  profesiones  y  sus  miembros  son  responsables  ante  aquellos  a  quienes  sirven  y  ante  la  sociedad.

2.  ¿Cuáles  son  los  elementos  centrales  de  una  profesión?
Todas  las  profesiones  se  caracterizan  por  cuatro  elementos  centrales:  (1)  un  monopolio  sobre  el  uso  del  
conocimiento  especializado;  (2)  a  cambio  de  ese  monopolio  que  disfrutamos,  relativa  autonomía  en  la  práctica  
y  la  responsabilidad  de  la  autorregulación;  (3)  servicio  altruista  a  los  individuos  y  la  sociedad;  y  (4)  responsabilidad  
de  mantener  y  expandir  el  conocimiento  y  las  habilidades  profesionales.

3.  ¿Qué  es  la  profesionalidad?
El  profesionalismo  describe  los  atributos  cognitivos,  morales  y  colegiados  de  un  profesional.
En  última  instancia,  son  todas  las  razones  por  las  que  su  madre  se  enorgullece  de  decir  que  usted  es  médico  y  
cirujano.

4.  ¿Por  qué  los  médicos  necesitan  un  código  de  conducta  profesional?
La  confianza  es  parte  integral  de  la  práctica  de  la  cirugía.  El  Código  de  Conducta  Profesional  aclara  la  
relación  entre  la  profesión  quirúrgica  y  la  sociedad  a  la  que  sirve.  Esto  a  menudo  se  conoce  como  un  contrato  social.  
Para  los  pacientes,  el  código  de  conducta  profesional  cristaliza  el  compromiso  de  la  comunidad  quirúrgica  con  los  
pacientes  individuales  y  sus  comunidades.  La  confianza  se  construye  ladrillo  a  ladrillo.

5.  ¿Qué  es  el  Código  de  Conducta  Profesional  del  Colegio  Estadounidense  de  Cirujanos?
El  Código  de  Conducta  Profesional  toma  los  principios  generales  del  profesionalismo  y  los  aplica  a  la  práctica  
quirúrgica.  El  código  es  la  base  sobre  la  que  ganamos  nuestros  privilegios  profesionales  y  la  confianza  de  los  
pacientes  y  el  público.  Es  nuestra  descripción  de  trabajo.

6.  ¿Cuáles  son  las  responsabilidades  de  profesionalismo  descritas  en  el  American
Código  de  Conducta  Profesional  del  Colegio  de  Cirujanos?
Durante  la  continuidad  de  la  atención  preoperatoria,  intraoperatoria  y  posoperatoria,  los  cirujanos  tienen  la  
responsabilidad  de:  1.  Servir  como  defensores  efectivos  de  las  necesidades  de  nuestros  pacientes.

2.  Revelar  opciones  terapéuticas  incluyendo  sus  riesgos  y  beneficios.
3.  Revelar  y  resolver  cualquier  conflicto  de  interés  que  pueda  influir  en  las  decisiones  de  atención.
4.  Ser  sensible  y  respetuoso  con  los  pacientes,  entendiendo  su  vulnerabilidad  durante  el
período  perioperatorio.

483
Machine Translated by Google
484  CAPÍTULO  103  PROFESIONALISMO

5.  Revelar  completamente  los  eventos  adversos  y  los  errores  médicos.
6.  Reconocer  las  necesidades  psicológicas,  sociales,  culturales  y  espirituales  de  los  pacientes.
7.  Incluir  dentro  de  nuestra  atención  quirúrgica  las  necesidades  especiales  de  los  pacientes  terminales.
8.  Reconocer  y  apoyar  las  necesidades  de  las  familias  de  los  pacientes  y  9.  
Respetar  el  conocimiento,  la  dignidad  y  la  perspectiva  de  otros  profesionales  de  la  salud.

7.  ¿Otras  sociedades  profesionales  tienen  un  código  de  conducta  profesional?
Sí.  Varios  grupos  han  creado  códigos  profesionales  y  el  Colegio  Americano  de  Cirujanos  respalda  sus  declaraciones.

8.  ¿Por  qué  los  cirujanos  necesitan  su  propio  código  de  profesionalismo?
Un  procedimiento  quirúrgico  es  una  experiencia  extrema.  Impactamos  a  nuestros  pacientes  fisiológica,  
psicológica  y  socialmente.  Cuando  los  pacientes  se  someten  a  una  experiencia  quirúrgica,  deben  confiar  en  
que  el  cirujano  pondrá  su  bienestar  por  encima  de  cualquier  otra  consideración.  El  código  escrito  ayuda  a  reforzar  estos  
valores.

9.  ¿Cuáles  son  los  principios  fundamentales  del  Código  de  Conducta  Profesional  y
los  códigos  de  otras  sociedades  profesionales?
1.  La  primacía  del  bienestar  del  paciente.
2.  Autonomía  del  paciente.
3.  Justicia  social.

10.  ¿Qué  es  la  ''primacía  del  bienestar  del  paciente''?
Esto  significa  que  los  intereses  del  paciente  siempre  son  lo  primero.  El  altruismo  es  fundamental  para  este  concepto,  y  
es  el  altruismo  del  cirujano  lo  que  fomenta  la  confianza  en  la  relación  médico­paciente.

11.  ¿Qué  es  el  "principio  de  autonomía  del  paciente"?
Los  pacientes  deben  entender  y  tomar  sus  propias  decisiones  informadas  sobre  su  tratamiento.
Esto  es  complicado.  Como  médicos,  debemos  ser  honestos  con  nuestros  pacientes  para  que  tomen  decisiones  
informadas.  Al  mismo  tiempo,  debemos  asegurarnos  de  que  sus  decisiones  sean  coherentes  con  las  prácticas  éticas  
y  no  den  lugar  a  demandas  de  atención  inadecuada.

12.  ¿Qué  es  el  ''principio  de  justicia  social''?
Como  médicos,  debemos  defender  a  nuestros  pacientes  individuales  y,  al  mismo  tiempo,  promover  la  
salud  del  sistema  de  atención  médica  en  su  conjunto.  Debemos  equilibrar  las  necesidades  de  nuestros  pacientes  
(autonomía)  y  no  desviar  los  escasos  recursos  que  benefician  a  la  sociedad  (justicia  social).

13.  ¿Cómo  puedo  aplicar  estas  nobles  ideas  a  mi  existencia  cotidiana  en  la  unidad  médico  quirúrgica?

El  Código  de  Conducta  Profesional  del  Colegio  Estadounidense  de  Cirujanos  enumera  las  responsabilidades  de  ser  
un  cirujano  específicamente.  Para  todas  las  demás  situaciones,  utilice  los  principios  rectores  para  dictar  sus  acciones.  
Algunos  ejemplos  incluyen:  La  búsqueda  de  la  ''distribución  justa  y  equitativa  de  recursos  finitos''  podría  significar  que  
usted  no
ordenar  pruebas  innecesarias  o  incluso  dar  20  unidades  de  sangre  a  un  niño  cirrótico  C.
Se  honra  el  ''compromiso  de  mejorar  la  calidad  de  la  atención''  presentando  ideas  para  mejorar  el  trabajo
flujo  en  las  salas  a  su  jefe  de  residentes,  director  de  programa  o  incluso  jefe  de  departamento.
Como  estudiante,  tiene  una  oportunidad  única  de  pasar  un  poco  más  de  tiempo  con  un  paciente  asustado  
(todos  están  asustados)  y  ayudar  a  explicar  la  naturaleza  de  la  enfermedad  y  cuáles  son  los  planes  para  hacer  algo.  
Use  palabras  sencillas,  no  palabras  de  médicos.  Los  pacientes  y  las  familias  lo  amarán  por  hacer  esto.  Y  esta  
comprensión  promueve  la  autonomía  del  paciente.
Machine Translated by Google
CAPÍTULO  103  PROFESIONALISMO  485

BIBLIOGRAFÍA

1.  Fundación  ABIM,  Fundación  ACP­ASIM,  Federación  Europea  de  Medicina  Interna:  Profesionalismo  médico  en  el  nuevo  milenio:  una  
carta  médica.  Ann  Intern  Med  136(3):243­246,  2002.

2.  Cruess  SR,  Johnston  S,  Cruess  RL:  Profesionalismo  para  la  medicina:  oportunidades  y  obligaciones.  Med  J  agosto
177:208­211,  pág.  208,  2002.

3.  Grupo  de  Trabajo  sobre  Profesionalismo  del  Colegio  Estadounidense  de  Cirujanos:  Código  de  Conducta  Profesional.  Cirugía  J  Am  Coll
197:603­604,  2003.

4.  Grupo  de  Trabajo  sobre  Profesionalismo  del  Colegio  Estadounidense  de  Cirujanos:  Código  de  Conducta  Profesional.  Cirugía  J  Am  Coll
199:734­735,  2004.

5.  Kopp  M,  Bonatti  H,  Haller  C  et  al.:  La  satisfacción  con  la  vida  y  el  estilo  de  afrontamiento  activo  son  predictores  importantes  de  la  
recuperación  de  la  cirugía.  J  Psychosom  Res  55(4):371­377,  2003.

6.  Rosenberger  PH,  Joki  P,  Ickovics  J:  factores  psicosociales  y  resultados  quirúrgicos:  una  revisión  de  la  literatura  basada  en  la  evidencia.  
J  Am  Acad  Orthop  Surg  14(7):397­405,  2006.

7.  Roth  RS,  Lowery  JC,  Davis  J  et  al .:  Los  factores  psicológicos  predicen  la  satisfacción  del  paciente  con  la  reconstrucción  mamaria  
posmastectomía.  Plast  Reconstr  Surg  119(7):2008­2015;  discusión  2016­2017,  2007.
Machine Translated by Google

ÍNDICE

Los  números  de  página  seguidos  de  t  indican  tablas;  f,  cifras.  Los  números  de  página  en  negrita  indican  
capítulos  completos.

A Absceso  (continuación)  

ABC pulmonar,  398–399,  407  
subfrénico,  398  drenaje  quirúrgico  
en  pacientes  con  hemorragia  digestiva  baja,  255  de  trauma  care
de,  82–83  tuboovárico,  188  como  
simulacro  de  apendicitis,  189
en  pacientes  con  trauma  cerebral,  99,  101  
en  pacientes  con  trauma  torácico,  119  en  
Acetaminofén,  como  tratamiento  de  la  fiebre,  65
pacientes  con  trauma  cervical,  110  en  
Acalasia,  213
pacientes  con  trauma  pélvico,  147
Trastornos  acidobásicos,  exceso  de  base  en,  48
ABCDE,  de  evaluación  inicial  del  trauma,  91
Acidosis
ABCDEF,  del  cuidado  de  quemaduras,  174–175
Absceso   como  causa  de  paro  cardíaco,  21  
Déficit  de  bases  metabólicas,  96  
abdominal  en,  76  
Relacionado  con  isquemia  
gammagrafía  con  galio  de,  83  
intestinal,  236  Enfermedad  ulcerosa  
relacionado  con  lesión  pancreática,  
140  drenaje  quirúrgico  de,  83,  84 acidopéptica.  Véase  Enfermedad  ulcerosa  péptica  Aciduria,  

agudo,  evaluación  de,  73–76   paradójica,  52  Carcinoma  de  células  acínicas,  336  Síndrome  de  

distensión  de,  74,  230,  418–419  dolor  en   inmunodeficiencia  adquirida  (SIDA).  Véase  también  Infección  por  

el  abdomen  agudo  relacionado,  73–74   el  virus  de  la  inmunodeficiencia  humana  (VIH)  asociada  con  cáncer,  326

relacionado  con  apendicitis,  187  
relacionado  con  pancreatitis  crónica,  
203  ubicación  y  duración  de,  73–74   Síndrome  medular  agudo,  107  

obstrucción  del  intestino  delgado   Síndrome  de  dificultad  respiratoria  aguda.  Ver  Síndrome  de  

relacionados,  230,  232  palpación  de,  74  lesiones   dificultad  respiratoria  del  adulto  (SDRA)

traumáticas  a  trauma  cerrado,  122–124  en  niños,  182,   Crisis  de  Addison,  307  

183  traumatismo  penetrante,  125–129  tumores  de,  429– Enfermedad  de  Addison,  306,  307  

430  en  niños,  429–430,  429t Adenocarcinoma  de  vejiga,  464  

Relacionado  con  pólipos  
colorrectales,  259,  260–261  Esofágico,  218  
pancreático,  194  Parótida,  336,  338  Tiroides,  343  
Carcinoma  quístico  adenoide,  336,  338,  340  
Compatibilidad  del  grupo  sanguíneo  ABO,  entre  donantes  de  corazón Adenoma  suprarrenal,  302,  303,  304,  306  
y  destinatarios,  442 colorrectal,  258  paratiroides,  283,  284,  286  parótida,  
Absceso 337  Poliposis  coli  adenomatosa,  260  Adhesivos,  para  
anorrectal,  248–249,  265,  266   el  cierre  de  heridas  faciales,  165–166  Adolescentes,  

intraabdominal,  76  gammagrafía  con   muerte  traumática  en,  180  Carcinoma  cortical  
galio,  83  relacionada  con  lesión   suprarrenal,  306  Adrenalectomía ,  303,  304,  306
pancreática,  140  drenaje  quirúrgico  
de,  83,  84  mediastínica,  398  
pancreática,  203  relacionada  con  
divertículo  perforado,  242  pilonidal,  267  
postoperatoria,  67

487
Machine Translated by Google
488  ÍNDICE

Glándulas  suprarrenales,  anatomía  y  productos  secretores  de, Colegio  Americano  de  Cirujanos  (Continuación)
302 Código  de  Conducta  Profesional  de,  483–484
insuficiencia  suprarrenal,  306,  307 Asociación  Americana  de  Gastroenterología,  directrices  de  detección  
Masas  suprarrenales,  bilaterales,  306 del  cáncer  colorrectal  de,  259
Tumores  suprarrenales,   Sociedad  Americana  de  Anestesiólogos,  59
302  incidentalomas,  302–303,  306   Síntoma  de  la  Asociación  Americana  de  Urología
resección  de,  306 Puntuación,  469

Hormona  adrenocorticotrópica,  en  el  síndrome  de  Cushing,  305–306 Amiodarona,  uso  en  resucitación  cardiopulmonar,  20
Ampicilina,  66
Síndrome  de  dificultad  respiratoria  del  adulto  (SDRA),  35,  122  causas   Partes  amputadas,  manejo  y  reimplantación  de,  171
de,  35  definición  de,  34  efecto  de  la  nutrición  enteral  con  aceite   Amputación  de  
de  pescado  en,  60  efecto  en  la  función  pulmonar,  37  alta  presión   las  yemas  de  los  dedos,  169–170,  170f  
versus  baja  presión,  35  baja  presión,  35  –36  ventilación  mecánica   de  extremidades  isquémicas,  351  como  

en,  43  tratamiento  de,  36,  37 tratamiento  del  melanoma,  333

Amilasa  
como  indicador  de  pancreatitis,  199,  203,  231  
como  indicador  de  obstrucción  del  intestino  delgado,  231
Canal  anal,  anatomía  de,  265
Soporte  vital  cardíaco  avanzado  (ACLS),  medicamentos Fisuras  anales,  266
usado  en,  20,  22,  23   Anafilaxia,  como  causa  de  parada  cardiaca,  22,  23
Instrucciones  anticipadas,  479–480   anastomosis

Soporte  vital  avanzado  en  trauma  (ATLS),  94  Síndrome   Billroth  I  y  II,  223,  224  en  
del  asa  aferente,  224  Pacientes  agitados,  hipoxemia   trasplante  corazón­pulmón,  439  en  
en,  46  Embolia  aérea,  broncovenosa,  120  Evaluación   trasplante  cardíaco,  439  ileorrectal,  250
de  las  vías  respiratorias,  92  en  niños,  181  ''definitivo'',  
92  orofaríngeo  o  nasofaríngeo,  19,  92  quirúrgico,  92   Aneurisma  
Manejo  de  la  vía  aérea.  Véase  también  Intubación   aórtico  abdominal,  361–365  
endotraqueal  en  el  síndrome  de  dificultad  respiratoria   definición  de,  361  factores  
del  adulto,  37  en  pacientes  quemados,  174–175   genéticos  en,  361  imágenes  
durante  la  reanimación  cardiopulmonar,  19  maniobras   de,  361,  362,  376–377  fuga,  76  ruptura,  
iniciales  en,  92  en  pacientes  traumatizados,  92   362  tratamiento  quirúrgico  de,  254,  363,  
Obstrucción  de  las  vías  respiratorias  superiores,  causas  de,  92   364–365  disección  aórtica,  410–414  
Albúmina,  53–55,  54t  Alcohol  aldosterona ,  49,  52,  303  Alcalosis ascendente  (tipo  A),  411,  412–413  controversias  
sobre,  412  descendente  (tipo  B),  411,  412,  413  manejo  
médico  de,  411,  412  manejo  quirúrgico  de,  411–412,  
413  de  la  arteria  femoral,  361  de  la  poplítea  arteria,  
361  torácica,  361  verdadero,  167

Angina  de  pecho  
bypass  de  la  arteria  coronaria  tratamiento  de,  379,  380,  381–382  
definición  de,  379  tratamiento  médico  de,  379  angioplastia  
coronaria  transluminal  percutánea

metabólica  hipopotasémica,  51   tratamiento  de,  379  
resta,  51 tratamiento  de  revascularización  transmiocárdica  de,  382–
Deficiencia  de  alfa1­antitripsina,  222 383

Desajuste  de  ventilación  alveolar/flujo  sanguíneo  (V/Q),  34 Angiografía  
Amaurosis  fugaz,  356 para  evaluación  de  aneurisma  aórtico  abdominal,  363  para  
Sociedad  Americana  del  Cáncer evaluación  de  fractura  pélvica  aguda,  148  cerebral,  357  
pautas  de  detección  de  cáncer  colorrectal  de,  259,  262  pautas  de   complicaciones  de,  163  tomografía  computarizada,  para  
mamografía  de,  309 evaluación  de  lesiones  en  extremidades,  159,  163–164
Colegio  Americano  de  Cirujanos
Curso  de  Soporte  Vital  Avanzado  en  Trauma  de,  94
Machine Translated by Google
ÍNDICE  489

Angiografía  (Continuación) Terapia  antilipídica,  para  aterosclerosis,  346
para  localización  y  tratamiento  de  hemorragia  gastrointestinal,   Antioxidantes,  efectos  cardioprotectores  de,  347
256  hepática,  130 Fármacos  antitiroideos,  como  tratamiento  del  hipertiroidismo,  288,  
289  Déficit  de  a1­antitripsina,  222

Angiomiolipoma,  renal,  462
Angioplastia   Antrectomía,  225,  227
ilíaca,  354   Ano,  imperforado,  421–422  en  niños,  
coronaria  transluminal  percutánea,  239  renal  transluminal   421  en  niñas,  421
percutánea,  299
Enzima  convertidora  de  angiotensina,  298
Inhibidores  de  la  enzima  convertidora  de  angiotensina,  299 Lesiones  de  la  
Heridas  por  mordeduras  de  animales,  65,  170–171 aorta  a  trauma  cerrado,  
Anisocoria,  100 115  relacionadas  con  lesiones  

Índice  tobillo­brazo  (ABI),  158,  350,  374 pancreáticas,  140  a  la  aorta  torácica,  
Enfermedad  anorrectal,  265–268 115  ruptura  de,  en  niños,  182

Anorectoplastia,  sagital  posterior  (PSARP),  415–416 Coartación  aórtica,  como  causa  de  hipertensión,  298,  300
Anoscopia,  255
antiácidos Disección  aórtica,  33  
como  tratamiento  de  úlcera  duodenal,  222–223  como   torácica,  105

tratamiento  de  infección  por  Helicobacter  pylori,  227  Síndrome   Insuficiencia  aórtica,  393,  394
del  cordón  anterior,  107  Datos  antropométricos,  como  componente   como  contraindicación  al  balón  intraaórtico

de  evaluación  nutricional,  53  Terapia  antiarrítmica,  26  Profilaxis   bomba,  33  
antibiótica  en  pacientes  con  estenosis  aórtica,  392  para   relacionado  con  aneurisma  aórtico  disecante,  412–413
infecciones  biliares,  192  en  pacientes  cirróticos,  209–210  para  la   Estenosis  aórtica,  391  

prevención  del  empiema,  120  para  laceraciones  faciales,  166   calcificado,  395

preoperatorias,  79  en  pacientes  con  cáncer  colorrectal,  263  en   Reemplazo  de  válvula  aórtica  en  
laparotomía  electiva,  83–84  para  infecciones  del  sitio  quirúrgico,   niños,  392  complicaciones  de,  

69,  70,  77,  81  indicación  para,  82  momento  de,  82  con   394  en  endocarditis  infecciosa,  
cefalosporinas  de  tercera  generación,  80,  82–83  Antibióticos.   394  abordajes  quirúrgicos  mínimamente  
Ver  también  antibióticos  específicos  resistencia  bacteriana  a,  80   invasivos  en,  394,  395–396
en  combinación  con  inhibidores  de  enzimas,  80  generaciones  
de,  79–80  administración  oral  versus  intravenosa  de,  81   mortalidad  operatoria  de,  394  
"limpiapipas",  79  como  "medicamentos  maravillosos",  79   percutánea,  395  con  válvulas  
Terapia  con  antibióticos  para  abscesos ,  82­83  para  pancreatitis   tisulares,  395
aguda,  201  para  quemaduras,  178  para  trauma  colorrectal,   Válvulas  aórticas,  bicúspides,  391
146  monoterapia  versus  terapia  combinada,  77  más  costosa,   Enfermedad  valvular  aórtica,  391–396  causas  

81  para  infecciones  quirúrgicas,  77  triple,  66  Terapia  anticoagulante   de,  391

para  trombosis  venosa  profunda,  367  en  pacientes  con  dispositivos   Sistema  de  puntuación  APACHE  II,  para  la  evaluación  de  la  gravedad  
de  asistencia  ventricular ,  447 de  la  pancreatitis,  200

Apendicectomía  en  
pacientes  con  enfermedad  de  Crohn,  189  
laparoscópica,  189  tasa  negativa  en,  188  
tasa  de  infección  de  herida  quirúrgica  de,  68  
Apendicitis,  187–190  condiciones  que  imitan,  189  
hallazgos  de  laboratorio  en,  187  mal  diagnosticado  
como  colecistitis  aguda,  190  como  causa  de  

mortalidad,  188  perforado,  188  Apéndice  irrigación  
sanguínea,  188  normal,  188  Arco  de  Riolano,  235  
Arritmias  cardiacas.  Ver  Arritmias  cardíacas  Gases  en  
sangre  arterial,  46–48  Conjunto  completo  de,  46  Bypass  
arterial,  como  tratamiento  de  isquemia  mesentérica,  238,  239

Agentes  antifúngicos,  sistémicos,  84
Agentes  antihipertensivos,  como  tratamiento  de  la  hipertensión  
relacionada  con  el  feocromocitoma,  300
Machine Translated by Google
490  ÍNDICE

Lesiones  arteriales,  en  las  extremidades   aleteo  auricular,  definición  de,  28
''signos  duros''  de,  158  ''signos   Patada  auricular,  pérdida  relacionada  con  estenosis  mitral  de,  385

suaves''  de,  158,  159 Presión  auricular,  izquierda,  en  regurgitación  mitral,  387
Insuficiencia  arterial,  350–355.  Ver  también  Enfermedad  oclusiva   Comunicación  interauricular,  estenosis  mitral  asociada  a,
arterial  periférica  Oclusión  arterial.  Véase  también  Enfermedad   386

oclusiva  arterial  periférica,  seis  síntomas  P  de,  158  Contenido  de   Disreflexia  autonómica,  470
oxígeno  arterial  (CaO2),  47  Índices  de  presión  arterial  (API),   Sistema  nervioso  autónomo,  papel  en  el  tracto  urinario  inferior
158,  159  Hallazgo  de  "rubor"  en  la  arteriografía,  253  para  la   función  del  tracto,  469–470

evaluación  de  lesiones  arteriales  en  las  extremidades,  159   Autonomía,  de  los  pacientes,  479,  484;
indicaciones  para,  352  para  evaluación  de  isquemia  intestinal,  236   Arteria  axilar,  lesiones  a,  162
para  evaluación  de  lesión  penetrante  de  cuello,  112  para  evaluación   Disección  de  ganglios  linfáticos  axilares,  en  cáncer  de  mama,  
de  enfermedad  vascular  periférica,  375  Asfixia,  traumática,  116   318,  324,  326–327
Aspirina  como  tratamiento  de  angina,  379  efectos  
cardioprotectores  de,  349  contraindicación  en  pacientes  con   B  

úlcera  duodenal,  222–223  como  tratamiento  de  fiebre,  65  como   Babesiosis,  86  
profilaxis  de  infarto  de  miocardio,  357  como  profilaxis  de   Bacillus  Calmette­Gue´rin  (BCG),  464,  465  Espalda,  
accidente  cerebrovascular,  357,  358  como  profilaxis  de  ataque   heridas  penetrantes  a,  127  Bacterias.  Véase  también  
isquémico  transitorio,  357  Aston,  Barry,  269  Asistolia  actividad   bacterias  beneficiosas  específicas,  77  Agentes  
eléctrica  sin  pulso,  21  Atelectasia,  como  causa  de  fiebre   bacteriostáticos,  78–79  Valvuloplastia  con  balón,  
posoperatoria,  65  Aterosclerosis,  345–349  factores  de  edad  en,   como  tratamiento  de  la  estenosis  mitral,
345  carótida,  356  como  causa  de  isquemia  mesentérica  crónica,  
238  secuelas  clínicas  de,  356  complicaciones  de,  349  como   385–386

enfermedad  inflamatoria,  349  hipótesis  de  lípidos  de,  346  como   Sistema  de  puntuación  de  Balthazar,  para  la  evaluación  de  la  gravedad  

respuesta  a  lesión  vascular,  346  factores  de  riesgo  de,  345,  346,  347   de  la  pancreatitis,  200

en  adultos  jóvenes,  345,  351  Placas  ateroscleróticas  características   Bannayan­Riley­Ruvalcaba  syndrome,  309  
histológicas  de,  356  ruptura  de,  349  Atresia Cirugía  bariátrica,  277–280
efecto  sobre  la  esperanza  de  vida,  278  
bypass  gástrico,  278–279
Enema  de  bario,  para  el  diagnóstico  de  obstrucción  intestinal  para  
la  evaluación  de  heces  con  guayaco  positivo,  262  en  obstrucción  
del  intestino  grueso,  244  en  recién  nacidos  y  lactantes,  417

Barnard,  cristiano,  439
Barotrauma,  42
esófago  de  Barrett,  218,  221
Carcinoma  de  células  basales,  pigmentado,  328
Gasto  energético  basal  (BEE),  56,  56t
Déficit  de  base,  96
Exceso  de  base,  48
Bassini,  Eduardo,  270
Reparación  de  hernia  de  Bassini,  270,  270f,  271,  275,  276
tríada  de  Beck,  119
Síndrome  de  Beckwith­Wiedeman,  474
Operación  Belsey  Mark  IV,  214,  215f,  216
biliar,  206   Beneficencia,  479.
tratamiento  de  trasplante  hepático  de,  433   Hiperplasia  prostática  benigna  (HPB),  467,  469
colónica,  417–418  duodenal,  417–418  esofágica,  423   Bloqueadores  beta­adrenérgicos
definición  de,  423  con  fístula  traqueoesofágica  distal,   como  tratamiento  de  angina,  379  
423  aislada,  423–424  como  componente  de  la   como  tratamiento  y  profilaxis  de  várices  esofágicas,  208,  210,  
asociación  VACTERL,  423  ileal,  417  intestinal,   212  como  tratamiento  de  hipertiroidismo,  288,  289  como  
417  yeyunal ,  417–418 tratamiento  de  estenosis  mitral,  385

Betacaroteno,  efectos  cardioprotectores  de,  347
Antibióticos  betalactámicos,  80
como  ''drogas  maravillosas'',  79–80
Bezoares,  como  causa  de  obstrucción  intestinal,  230
Fibrilación  auricular Bicarbonato  en  

como  contraindicación  al  balón  de  contrapulsación  intraaórtico,  33   secreciones  corporales,  50t  
definición  de,  28 contenido  de  líquido  intravenoso,  50t
Machine Translated by Google
ÍNDICE  491

Temperatura  del  dedo  gordo  del  pie,  como  indicador  del   Transfusiones  de  sangre  

estado  hemodinámico,  51  Fugas  de  los  conductos   en  pacientes  con  traumatismo  hepático,  
biliares,  133,  134–135  Conductos  biliares,  cáncer  extrahepático   132  transmisión  de  enfermedades  infecciosas,  86  

de,  194,  195  dilatados,  195  Sales  biliares,  metabolismo   durante  la  reanimación  inicial,  97  masivo,  ''ciclo  
microbiano  de,  77  Infecciones  biliares,  cobertura  antibiótica   sangriento''  de,  123  en  pacientes  con  fractura  pélvica,  
en,  192  Biliar  contenido  de  electrolitos  en  el  tracto  de,  50t   149  como  tratamiento  de  choque,  31
lesiones  a,  134  Billroth,  Albert  Theodor,  224  Billroth  I  y  II  

anastomosis,  223,  224  Biloma,  134  Análisis  de  impedancia   Disparador  de  transfusión  de  sangre,  47–48
bioeléctrica  (BIA),  53  Biopsia.  Ver  también  Biopsia  con  aguja   Volumen  de  

gruesa;  Biopsia  por  aspiración  con  aguja  fina  de  masas   sangre  en  niños,  95,  181  

mamarias,  311,  312,  314–315  de  úlceras  gástricas,  226   estimación  de,  95
de  melanoma,  328,  332,  333  de  lunares  (nevos),  329  de   ''Ciclo  vicioso  sangriento'',  123
masas  en  el  cuello,  343  próstata,  467  de  nódulos  pulmonares   Trauma  cerrado  

solitarios,  407  endomiocárdica  transvenosa,  442  defectos  de   abdominal,  122–124  en  

nacimiento.  Ver  Anomalías  congénitas  Heridas  por  mordedura niños,  182  bronquial,  
120  en  niños,  180  colónico,  
144  duodenal,  122  hepático,  
130,  132  renal,  151–154  
escrotal,  156  esplénico,  136  

torácico,  114–117

Contenido  de  
electrolitos  en  fluidos  corporales  
de,  50t  Tasa  de  exposición  de  los  cirujanos  a,  89
animal,  65,  170–171   Índice  de  masa  corporal  (IMC),  277
contaminación  bacteriana  de,  65  facial,   Gammagrafía  ósea,  de  cáncer  de  mama,  316,  318–319
166  en  la  mano,  170–171  humano,  65,   Preparaciones  intestinales  (descontaminación),  78,  79,  263
166,  170–171 Ruidos  intestinales,  57,  74,  230

Viabilidad  intestinal,  determinación  de,  233,  237
Lesiones   enfermedad  de  Bowen,  328

vesicales   ''Caja,  la''  119

relacionadas  con  histerectomía,   Arteria  braquial,  lesiones  a,  162
157  relacionadas  con  fractura   Bradicardia,  definición  de,  28
pélvica,  149  ruptura  de,  155 Determinantes  

Cáncer  de  vejiga,  464–466   clínicos  de  muerte  cerebral  de,  480–481  

metastásico,  465  carcinoma   donación  de  órganos  después,  481
de  células  de  transición,  56,  464 Imágenes  cerebrales,  en  pacientes  con  cáncer  de  mama  metastásico,
Obstrucción  de  la  salida  de  la  vejiga,  473   319

Blastomicosis,  408  Hinchazón,  relacionada   Daño  cerebral.  Ver  Lesión  cerebral  traumática  
con  la  obstrucción  del  intestino  delgado,  230  Contenido  de   Anomalías  de  la  hendidura  branquial,  431,  431  
oxígeno  en  sangre  de,  47  Viscosidad  de,  47  Enfermedad   Excisión  remanente  de,  431  Quistes  de  la  

transmitida  por  la  sangre,  riesgos  de,  86–90  Autorregulación   hendidura  branquial,  342,  431  Mama,  cambios  
del  flujo  sanguíneo  de,  30  pulmonar,  a  pulmones   fibroquísticos  en,  311  Cáncer  de  mama
trasplantados,  451  venosa,  estasis  de,  366  Nivel  de  glucosa  en  
sangre,  en  pacientes  con  trauma  pediátrico,  185  Pérdida  de   disección  de  ganglios  linfáticos  axilares  en,  318,  326–327  
sangre.  Véase  también  Hemorragia  en  niños,  181   características  de,  311  diagnóstico  de,  314  carcinoma  ductal  in  

hematocrito  en,  96  oculto,  97  ''en  la  acera'',  97  Presión   situ,  319–320  diagnóstico  temprano  de,  309  inoperable,  319  
arterial.  Véase  también  Hipertensión;  Hipotensión carcinoma  lobulillar  in  situ,  320  metastásico,  315,  315t,  316,  318  –

319  como  causa  de  mortalidad,  262  terapia  neoadyuvante  para,  
319  no  invasiva  (in  situ),  316  tasa  de  supervivencia  postratamiento  
en,  316

como  indicador  de  estado  de  volumen,  51
Machine Translated by Google
492  ÍNDICE

Cáncer  de  mama  (continuación) Quemaduras  
asociado  al  embarazo,  315  terapia   (continuación)  escisión  quirúrgica  de,  

primaria  para,  314–322  recurrente,  319,   177,  178  área  de  superficie  corporal  total  (TBSA)  de,  173,  
324  detección,  309–310  mapeo  de   174,  176,  178  zonas  de  lesión  en,  174  injertos  de  

ganglio  linfático  centinela,  316,  318   derivación.  Véase  también  Injerto  de  derivación  de  arteria  

estadificación  de,  315,  315t,  318  tratamiento  quirúrgico  de,   coronaria  (CABG)  conductos  autógenos  en,  352  extremidad  

317  como  enfermedad  sistémica,  324  marcadores  tumorales   inferior  autógena,  374  flujo  de  entrada  versus  flujo  de  salida  
para,  318–319 en,  352  oclusión  de,  351

Terapia  de  conservación  mamaria,  317–318,  319
Exploración  mamaria,  clínica,  309
Masas  mamarias,  309–313  biopsia   C

de,  311  ''tríada  diagnóstica''   Caquexia  strumipriva,  290

de,  313  dominante,  311  ''prueba   Calcio,  dietético,  como  factor  de  riesgo  de  enfermedad  por  cálculos  
triple  negativa''  para,  313 urinarios,  460
Bloqueadores  de  los  canales  de  calcio

reconstrucción  mamaria,  318 como  tratamiento  de  angina,  379  
Autoexploración  mamaria,  309 como  causa  de  infertilidad,  55

Respiración,  gasto  de  energía  en,  34 Cloruro  de  calcio,  uso  en  reanimación  cardiopulmonar,  22
Clasificación  de  Breslow  y  Clark,  de  la  invasividad  del  melanoma,  
329–330,  333 Cálculos  renales  de  calcio,  459,  460  

Bronquios,  traumatismos  cerrados  a,  120 Equilibrio  calórico,  59  Calorimetría,  indirecta,  

Bronquiolitis  obliterante,  450 56  Cáncer,  323–327.  Ver  también  tipos  

relacionados  con  el  trasplante  de  pulmón,  449 específicos  de  cáncer

Broncoscopia  para   diagnóstico  de,  323  

evaluación  de  lesión  penetrante  de  cuello,  112–113  para   como  causa  de  hipercalcemia,  281  

evaluación  de  nódulo  pulmonar  solitario,  408 respuesta  del  sistema  inmunitario  en,  325–326  

Síndrome  de  Brown­Se´quard,  107 potencialmente  mortal,  323  metastásico.  Ver  
Soplos   Tratamiento  de  metástasis  de  terapia  adyuvante,  325  

relacionados  con  lesión  arterial,  158   terapia  neoadyuvante,  325  nuevos  enfoques  de  

carótida,  357  cervical,  371 tratamiento,  325–326  terapia  paliativa,  326  
tratamiento  quirúrgico,  325  terapia  dirigida,  326  

síndrome  de  Budd­Chiari,  206,  433 tipos  de,  323  Vacunas  contra  el  cáncer,  325–326  

''Barrera  de  cuerpo  de  insecto'',  77 Colonización  por  Candida,  de  líneas  de  catéter  
Reflejo  bulbocavernoso,  470 intravenoso,  84  CA19­9,  195  Capitación,  477  
Sitios  de   Carbohidratos,  como  componente  de  fórmula  

entrada  y  salida  de  balas  de,  126,  126f   enteral,  57  Dióxido  de  carbono,  al  final  de  la  espiración,  

alojados  en  el  miocardio,  120 monitoreo  de,  22  Intoxicación  por  monóxido  de  carbono,  

Gasto  energético   175  Carcinogénesis,  323  Tumores  carcinoides,  188

de  pacientes  quemados  en,  34  
rehabilitación  de,  178
Quemaduras,  173–
179  químicas,  178  
relacionadas  con  abuso  infantil,  185

Úlceras  de  Curling  asociadas  con,  228  profundidad  
de,  177,  177t  eléctricas,  178  lesiones  por  inhalación  
asociadas  con,  175,  176  complicaciones  

potencialmente  mortales  de,  173–174  manejo  de,  174–175,   apéndice,  189,  190

176–177  por  parte  de  los  socorristas,  174  reanimación  con   Carcinoma,  definición  de,  323

líquidos,  176  apoyo  nutricional,  178  como  causa  de   Paro  cardiaco

mortalidad,  173,  173t,  178  fisiopatología  de,  174   definición  de,  19  

poblaciones  en  riesgo  de,  173  efectos  metabólicos  graves   relacionado  con  lesión  abdominal  penetrante,  126  

de,  178  estimación  del  tamaño  de,  175 perioperatorio,  en  niños,  23  relacionado  con  trauma,  23  
causas  tratables  de,  21

Gasto  cardíaco  en  
niños,  181  relación  

con  el  shock,  30
Machine Translated by Google
ÍNDICE  493

Flujo  de  iones  de  membrana  de  cardiomiocitos,  27,  27f Cefalosporinas,  tercera  generación,  79–80  Presión  de  
Fallo  de  bypass  cardiopulmonar   perfusión  cerebral  (PPC),  101  Fugas  de  líquido  
al  destete,  23,  445  en  trasplante  de   cefalorraquídeo,  100  Enfermedad  cerebrovascular.  Ver  
pulmón,  452 también  Pruebas  de  laboratorio  no  diagnósticas  para  

Reanimación  cardiopulmonar  (RCP),  19–24  compresión   accidentes  cerebrovasculares,  371
torácica  en,  19,  20  contraindicaciones  para,  23  fármacos   collares  cervicales,  105

utilizados  en,  20,  22,  23  técnica  abdominal  interpuesta  en,   lesiones  de  la  
20  apoyo  posterior  a  la  reanimación,  21  enfoque  de   columna  
"código  lento"  en ,  23  terminación  de,  23 cervical  a  lesiones  arteriales  asociadas  con,  108  
fracturas,  107  evaluación  radiológica  de,  106,  107  

estabilización  de,  en  pacientes  traumatizados,  92

Accidentes  cardiovasculares,  356,  357 enfermedad  de  Chagas,  86
Transición  de  cuidados,  13 ''Cadena  de  lagos'',  204

arteria  carótida,  356–360   Quemaduras  químicas,  178
extracraneal,  359  lesiones  a,   Quimioterapia  
108  oclusión/estenosis,  357,   adyuvante,  325  
359  asintomática,  358  velocidad  de  la   para  cáncer  de  pulmón,  404  

sangre  en,  372–373,  372t  ecografía   neoadyuvante,  325
de  diagnóstico  de,  372–373  pruebas  de   Trauma  

diagnóstico  no  invasivas  para,  371,  372–373, cerrado  de  tórax  a,  114–117  en  
niños,  182
372t   traumatismo  penetrante  para,  118­121  
posendarterectomía  carotídea,  359  disección   "regla  de  las  6  horas"  para,  118
traumática  de,  102 Compresión  torácica,  en  reanimación  cardiopulmonar,

Cuerpo  carotídeo,  funciones  de,  359 19–20  

Tumores  del  cuerpo  carotideo,  342 Tubos  torácicos,  salida  de  sangre,  118  Radiación  de  la  
Ruidos  carotídeos,  357 pared  torácica,  después  de  mastectomía,  318  Síntomas  principales,  
Endarterectomía  carotídea,  36,  358,  359   9  Abuso  infantil,  como  causa  de  lesión  y  mortalidad  pediátrica,  
complicaciones  de,  358,  359  basada  en   102,  184,  185  Niños.  Véase  también  Bebés;  Neonatos
ecografía  dúplex,  375  reoperatoria,  359  
seguimiento  de  ecografía  dúplex  de  vigilancia,  
375 tumores  abdominales  en,  429–430,  429t  anatomía  
Seno  carotídeo,  funciones  de,  359 de  las  vías  respiratorias  en,  181  tratamiento  de  la  
Síndrome  del  túnel  carpiano,  171–172 estenosis  aórtica  en,  395  reemplazo  de  la  válvula  
Carrel,  Alexis,  439 aórtica  en,  392  lesiones  ateroscleróticas  en,  345  
Catéteres pérdida  de  sangre  en,  181  volumen  sanguíneo  en,  
Candida  colonización  de,  84  cardíaca,   181  gasto  cardíaco  en,  181  compresiones  torácicas  

en  pacientes  con  estenosis  aórtica,  392  venosa   en,  19–20  colecistectomía  en,  192  hemorragia  
central,  93  uso  en  reanimación  cardiopulmonar,  20 gastrointestinal  en,  257  trasplante  de  corazón  en,  
440,  444  hemodinámica  de,  181,  185  hernia  en,  
Foley   269–270  ano  imperforado  en,  421–422  trasplante  
en  pacientes  con  trauma  pélvico,  147   de  hígado  en,  433,  434  causas  de  mortalidad  en,  
como  causa  de  infección  del  tracto  urinario,  67   182  tumores  parotídeos  en,  339  paro  cardíaco  
por  acceso  intravenoso,  93 perioperatorio  en,  23  hipertensión  portal  en,  206  
CA­29,  como  marcador  tumoral  de  cáncer  de  mama,  318–319 reanimación  en,  181  hidrocele  del  cordón  
CA­27,  como  marcador  tumoral  de  cáncer  de  mama,  318–319 espermático  en,  269  trauma  en  lesiones  
Cauda  equina  injury,  105   abdominales,  182,  183  nivel  de  glucosa  en  sangre  
Arteria  celíaca,  169 en,  185  lesiones  por  quemaduras,  173  diferencias  
oclusión  de,  238  Eje   de  género  en,  180
celíaco,  vías  colaterales  de,  235  Síndrome  de  

compresión  celíaca  (Dunbar),  239  Solución  de  preservación  
Celsior,  451  Síndrome  del  cordón  central,  107  Líneas  centrales.  
Ver  Catéteres  venosos  centrales  Lesiones  del  sistema  nervioso  
central.  Véase  también  Traumatismo  craneoencefálico  Úlceras  

de  Cushing  asociadas,  228  como  causa  de  mortalidad,  91
Machine Translated by Google
494  ÍNDICE

Niños  (Continuación)   Cirrosis  (Continuación)  
lesiones  en  la  cabeza,   como  indicación  de  trasplante  hepático,  433  como  
182  incidencia  por  región  del  cuerpo,   causa  de  hipertensión  portal,  206
180  como  causa  de  mortalidad,  180  no   Clasificación  de  Clark,  de  la  invasividad  del  melanoma,  329–
accidental  (maltrato  infantil),  102,  184,  185  lesiones   330,  333
renales,  151  lesiones  esplénicas,  184  lesiones  torácicas,   Claudicación,  350,  354
182 intermitente,  350,  374  
venoso,  368  ''Clear  the  C­
urología  de,  472–474 spine'',  92  Vías  clínicas,  476  
tumor  de  Wilm  en,  474 Clonación,  456,  457  Infecciones  
Sistema  Child­Turcott­Pugh,  de  clasificación  de   de  heridas  por  clostridios,  66  
enfermedades  hepáticas,  207,  207t Prueba  CLO,  222,  226  Coagulopatía,  
Quimerismo,  en  trasplante,  451; relacionada  con  transfusiones  de  sangre,  
Maniobra  de  elevación  del  mentón,  19,  92–93 149  Coccidioidomicosis,  408  Códigos  de  conducta  
Clamidia,  como  factor  de  riesgo  de  enfermedad  cardiovascular,   profesional,  483–484  Café,  presión  parcial  de  oxígeno  en,  
347,  348 46,  47  ''Lesión  en  moneda'',  407  Colectomía  ciega  subtotal,  
Cloruro 257  sigmoide,  262  subtotal,  233,  242  Tasa  de  infección  
en  secreciones  corporales,   de  herida  quirúrgica  de,  68  total,  233  para  colitis  ulcerosa,  
50  t  como  tratamiento  de  alcalosis  metabólica   250  Colitis  de  Crohn/granulomatosa,  248  isquémica,  238,  
hipopotasémica,  51  contenido  de   239  ulcerativa,  248,  258  como  factor  de  riesgo  de  
líquido  intravenoso,  50  t cáncer  colorrectal,  262  diferenciada  de  la  enfermedad  
Colangitis,  191 de  Crohn,  248  displasia  asociada  con,  249  tratamiento  
Colangiocarcinoma,  como  contraindicación  al  trasplante   quirúrgico  de,  249,  250  Presión  oncótica  coloide  
hepático,  435 ( COP),  35,  36  Solución  coloidal,  como  tratamiento  de  
Colangiografía,  intraoperatoria  laparoscópica,  192 choque,  31  Colon  en  la  enfermedad  de  Crohn,  248  
Colangiopancreatografía,  retrógrada  endoscópica Enfermedad  diverticular  de,  241–243  Contenido  de  

(CPRE),  134,  195,  197  inicio   electrolitos  de,  50t  Lesiones  de,  144  Contusión,  122  
de  sesión  ''cadena  de  lagos'',  204   Complicaciones  de,  145  Diagnóstico  de,  144  
inicio  de  sesión  ''doble  conducto'',   Clasificación  de,  144  manejo  quirúrgico  de,  144  
195  como  causa  de  pancreatitis,  199 obstrucción  de,  244–247  preoperatorio  ive  
Colecistectomía,  191,  192   preparación  de,  263  derecho,  suministro  de  sangre  
carcinoma  de  vesícula  biliar  descubierto  durante,  192   a,  188  Cáncer  de  colon.  Ver  Colonoscopia  de  cáncer  
laparoscópico,  191,  192  en  niños,  192  durante  el  embarazo,   colorrectal,  262  riesgos  de,  259  para  localización  de  
192  abordaje  quirúrgico  abierto  en,  192  en  pacientes   hemorragia  gastrointestinal,
con  pancreatitis,  201  profiláctico,  191  tasa  de  infección  
de  herida  quirúrgica  de,  68

Colecistitis,  191  
acalculus,  192  
apendicitis  aguda  

mal  diagnosticada  como,  190  abordaje  
quirúrgico  abierto  en,  191
Coledocolitiasis,  191
Colelitiasis,  191,  201
Colestasis,  intrahepática,  195
Churchill,  Winston,  408
El  signo  de  Chvostek,  284
Cilostazol,  350
Círculo  de  Willis,  arteria  carótida  colaterales  en,  359
Cirrosis

alcohólico,  206   256  

profilaxis  antibiótica  para,  209–210  flujo   en  colitis  ulcerosa,  249
sanguíneo  arterial,  131  como  factor  de  riesgo   Resección  de  colon,  profilaxis  antibiótica  en,  81
de  enfermedad  de  úlcera  duodenal,  222  várices   Cáncer  colorrectal,  262–264  como  
esofágicas  asociadas  con,  208  hemorragia   causa  de  obstrucción  intestinal,  245  como  
gastrointestinal  asociada  con,  208, causa  de  mortalidad,  262  terapia  postoperatoria  
209–210 en,  263–264
Machine Translated by Google
ÍNDICE  495

Cáncer  colorrectal  (Continuación)   Estreñimiento  
factores  de  riesgo  de,  262  detección   relacionado  con  cáncer  colorrectal,  
de,  259,  260,  262  tratamiento   262  como  causa  de  hemorroides,  267  
quirúrgico  de,  262,  263 relacionado  con  anorroplastia  sagital  posterior,  415–416
Colostomía,  144   Contaminación,  diferenciada  de  infección,  77,  78
permanente,  262   Contusiones  

proximal,  144 cardíacas  

Coma,  post­reanimación,  21 en  niños,  182  
Pacientes  comatosos,  100 miocárdicas,  115  
Presiones  de  compartimiento,  medición  de,  162 pulmonares,  115,  123  en  
Síndromes  compartimentales,  161,  163   niños,  182  renales,  152
abdominal,  134  definición  de,  354  
diagnóstico  de,  162  tratamiento  de,  162  no   Cooper,  Astley  Paston,  269
tratado,  162  reparación  vascular  relacionada,   ligamento  de  Cooper,  269
161 Reparación  de  hernia  del  ligamento  de  Cooper,  269,  271,  272
Temperatura  corporal  central,  64
Biopsia  con  aguja  gruesa,  de  masas  mamarias,  311,  312,
Hemograma  completo  (CBC)  en   314

abdomen  agudo,  75  en  obstrucción   Injerto  de  derivación  de  arteria  coronaria  (CABG)  
de  intestino  delgado,  231 como  tratamiento  de  angina  de  pecho,  379,  380,  381–382  
Cumplimiento,  43 comparación  con  intervenciones  coronarias  percutáneas,  381
Tomografía  computarizada  (TC)  
abdominal,  76  de  aneurismas   para  insuficiencia  cardíaca  congestiva,  
aórticos  abdominales,  362  de  apendicitis,  188   380  efecto  sobre  la  función  miocárdica,  380  
de  traumatismo  abdominal  cerrado,  122–123   permeabilidad  del  injerto  en,  352  indicaciones  
en  niños,  183  de  lesión  cerebral,  100  de  cáncer   para,  379  sin  bomba,  380  problemas  en  el  
de  mama,  316,  318–319  de  diverticulitis,  241   destete,  382  riesgos  de,  382  "tradicional",  
doble­  contraste,  76  duodenales,  141  de  lesiones   380  permeabilidad  del  vaso  en,  381
hepáticas,  130  de  obstrucción  del  intestino  grueso,  
244  de  masas  en  el  cuello,  343  de  cáncer  de  
páncreas,  195  de  necrosis  pancreática,  200  
helicoidales  de  secuencia  rápida,  de  cálculos   Enfermedad  arterial  coronaria,  379–383  
urinarios, relacionada  con  diabetes  mellitus,  383  
periodontitis  como  factor  de  riesgo  de,  348  en  
pacientes  con  enfermedad  vascular  periférica,  353
Estenosis  de  la  arteria  coronaria  principal  izquierda,  379
pulso  de  Corrigan,  393
Cortisol,  en  el  síndrome  de  Cushing,  305–306
458   Tos,  relacionada  con  el  cáncer  de  pulmón,  403

de  trauma  renal,  151  de   síndrome  de  Cowden,  309
obstrucción  del  intestino  delgado,  231  de   Proteína  C  reactiva,  54t,  346
nódulos  pulmonares  solitarios,  407,  408  de  lesiones   Crepitación  en  el  cuello,  111
esplénicas,  138 Cricotiroidotomía,  92  en  
Angiografía  por  tomografía  computarizada  (TC)  de   niños,  181
enfermedad  de  la  arteria  carótida,  371  de   Isquemia  crítica  de  extremidades,  350,  351,  354
aneurisma  aórtico  disecante,  410 enfermedad  de  Crohn,  188,  248,  258
Conmoción  cerebral,  99,  102 como  factor  de  riesgo  de  cáncer  colorrectal,  
Pacientes  confusos,  hipoxemia  en,  46 262  diferenciado  de  colitis  ulcerosa,  248  tratamiento  
Hiperplasia  suprarrenal  congénita  (CAH),  474 médico  de,  249–250  presentación  como  apendicitis,  
Anomalías  congénitas   189  tratamiento  quirúrgico  de,  248–249,  250,  251
asociadas  a  hernia  diafragmática  congénita,  426  Fecundación  
in  vitro  como  factor  de  riesgo,  456 Lesiones  por  
Insuficiencia  cardíaca  congestiva   aplastamiento  
relacionada  con  estenosis  aórtica,   avulsión,  166  en  niños,  180
391  alimentaciones  enterales  en,  57   Criptococosis,  408
relacionada  con  endocarditis  infecciosa,  394 Criptorquidia  (testículos  no  descendidos),  457,  473,
Tendones  unidos,  en  reparación  de  hernia  inguinal,  276 474

síndrome  de  Conn,  298,  300,  303 Concentraciones  de  
Conciencia,  niveles  de,  100 iones  en  solución  cristaloide,  49
Machine Translated by Google
496  ÍNDICE

Solución  cristaloide  (continuación)  como   Diabetes  mellitus  (Continuación)  
líquido  de  reanimación  en  niños,  182   insuficiencia  pancreática  asociada  con,  203  enfermedad  
en  pacientes  traumatizados,  97   arterial  periférica  asociada  con,  374  enfermedad  vascular  
como  tratamiento  de  choque,  31 asociada  con,  351
Pacientes  en  diálisis,  trasplante  renal  en,  436
Signo  de  Cullen,  199 Diafragma,  lesiones  a,  119,  182
Enfermedad  de  Cushing   PAÑALES  mnemotécnicos,  para  incontinencia  urinaria  
diferenciada  del  síndrome  de  Cushing,  305  como  causa  de   geriátrica,  437

hipertensión,  300 Diarrea,  relacionada  con  la  alimentación  enteral,  58

Síndrome  de  Cushing,  305–306   Aumento  diastólico,  32,  33
diferenciado  de  la  enfermedad  de  Cushing,  305  como   Úlceras  de  Dieulafoy,  254
causa  de  hipertensión,  298,  300 Digoxina
Envenenamiento  por  cianuro,  175 Actividad  de  bloqueo  del  nódulo  AV  de,  28  
Cistectomía,  radical,  derivación  urinaria  con,  465 como  tratamiento  de  la  estenosis  mitral,  385  

fibrosis  quística,  418 tasa  de  administración  intravenosa,  28

Cálculos  renales  de  cistina,  459 Coagulopatía  intravascular  diseminada,  102
Cistoscopia,  vigilancia,  de  lesiones  vesicales,  464 Diuréticos,  como  tratamiento  del  traumatismo  craneoencefálico,  101
Cistouretrografía   Enfermedad  diverticular  del  colon,  241–243  como  causa  

retrógrada,  155   de  obstrucción  del  intestino  grueso,  244  tratamiento  
evacuación,  en  niños,  472 quirúrgico  de,  245
Quistes   Diverticulitis,  188,  241  como  
hendidura  branquial,  342,  431   simulacro  de  apendicitis,  189  
mama,  311,  312  ovario,  ruptura,   diferenciado  de  diverticulosis,  235

188  de  la  glándula  parótida,  340   diverticulosis

conducto  tirogloso,  431–432 diferenciada  de  diverticulitis,  235  como  causa  de  

hemorragia  gastrointestinal,  242,  255
Infecciones  por  citomegalovirus,  86  en   divertículo

pacientes  trasplantados  de  pulmón,  449,  450 colónico,  241  

Cirugía  citorreductora  (''debulking''),  326 como  causa  del  dolor,  
241  perforado,  242
D   de  Meckel,  188,  189
Dacarbazina,  como  tratamiento  de  melanoma,  333   Dobutamina  

Solución  de  Dakin,  67  Davis,  GG,  188  Prueba  de  dímero   diferenciada  de  dopamina,  31  como  
D,  367,  376  Espacio  muerto,  34  Descompresión,   tratamiento  de  choque,  31,  31t

quirúrgica Órdenes  de  no  resucitar  (DNR),  23,  479
Dopamina,  diferenciada  de  dobutamina,  31
Cartel  ''Doble  conducto'',  195
gástrico,  en  pacientes  con  hernia  diafragmática  congénita, Drenaje  de  
426 lesiones  rectales  extraperitoneales,  145  pericárdico  
como  tratamiento  del  neumotórax  a  tensión,  114 percutáneo,  119  de  abscesos  pulmonares,  398
Descontaminación

del  intestino,  78   Paños  para  prevenir  la  desecación,  79
intestino  selectivo,  79   Ahogamiento,  180
de  la  piel  o  cavidades  mucosas,  78–79 Abuso  de  drogas  

Decorticación,  398 como  factor  de  riesgo  de  quemaduras,  

Desgrasado,  78–79 173  como  causa  de  infertilidad  masculina,  455

Defibriladores,  20  de   Drummond,  Hamilton,  235
Gimbernat,  Antonio,  269 Sistema  de  estadificación  de  Dukes,  para  el  cáncer  de  recto,  263
Demijov,  Vladimir,  439 ''Tumores  en  mancuerna'',  337
línea  dentada,  265 Síndrome  de  dumping,  224
Dextrosa,  5%,  en  soluciones  intravenosas,  50t,  51,  96 síndrome  de  Dunbar  (síndrome  de  compresión  celíaca),
Diabetes  insípida,  relacionada  con  lesiones  en  la  cabeza,  102 239
Diabetes  mellitus Signo  de  Dunphy,  187–188
diagnóstico  de  abdomen  agudo  en,  73   Estenosis  duodenal,  419
como  factor  de  riesgo  de  aterosclerosis,  345   Fuga  del  muñón  duodenal,  relacionada  con  la  cirugía  de  úlcera  
disfunción  vesical  asociada  a,  470  enfermedad  arterial   duodenal,  225

coronaria  asociada,  383  alimentación  enteral  en,  57   Úlceras  duodenales,  221  

relacionada  con  trasplante  de  pulmón,  449 diferenciadas  de  úlceras  gástricas,  226  múltiples,  
223
Machine Translated by Google
ÍNDICE  497

Úlceras  duodenales  (continuación)   Embolia  (Continuación)  
perforadas,  73–74,  225 diferenciada  de  trombosis,  354  paradójica,  
Cierre  de  Graham  de,  225   235  pulmonar,  108,  366,  367
recurrente,  223,  225,  226  
tratamiento  de,  222–223,  224 Embolización,  angiográfica,  123,  256
Duodenoyeyunostomía,  Roux­en­Y,  142 Servicio  de  urgencias,  pacientes  traumatizados  en,  123
Contenido  de   Enfisema,  como  indicación  de  trasplante  
electrolitos  en  el  duodeno  de,  50t   pulmonar,  449
lesiones  de,  140,  141  contundente,   Empiema,  120,  398  bilis,  
122  clasificación  de,  141,  142t   134  postraumático,  
lesión  pancreática  asociada,  140   120
penetrante,  142  obstrucción  de,  419   Empiema  necessitans,  398
anatomía  quirúrgica  de,  141 Encefalopatía,  hepática,  55
Endocarditis  infecciosa  
como  causa  de  insuficiencia  aórtica,  393  
Poder  duradero,  480; sustitución  de  válvula  aórtica  en,  394  
Disfagia,  213,  218 relacionada  con  insuficiencia  mitral,  387
Displasia  relacionada  con  colitis  ulcerosa,  249 Ligadura  endoscópica  con  banda  (LBE),  como  tratamiento  esofágico

Síndrome  del  nevus  displásico,  328 tratamiento  de  várices,  209,  209f,  210,  211,  212
Disnea   Endotelio,  papel  de,  348
relacionada  con  empiema,   Endotoxinas,  circulantes,  84–85
398  de  esfuerzo,  relacionada  con  estenosis  mitral,  384 intubación  endotraqueal

Arritmias,  cardíacas,  25–29 durante  la  reanimación  cardiopulmonar,  19  en  niños,  
relacionado  con  contusión  miocárdica,  124 181  en  pacientes  con  hernia  diafragmática  congénita,  
426
Y terapia  endovascular
Equimosis,  periorbital,  100 para  aneurismas  de  aorta  abdominal,  363,  364  para  
Ecocardiografía  de   enfermedad  oclusiva  infraguinal,  354–355
estenosis  aórtica,  391,  392  con   Gasto  energético,  durante  la  respiración,  34
ultrasonido  Doppler,  de  insuficiencia  mitral,  387  transesofágica  de   Fórmulas  enterales,  57  
aneurisma  aórtico  disecante,  410  de  estenosis  mitral,  384 ''mejorador  del  sistema  inmunitario'',  57,  
59–60  ''pulmonar'',  57,  60
Nutrición  enteral,  56–60  en  
Ectasias,  sangrado  vascular,  255 pacientes  quemados,  178  
Edema   comparación  con  nutrición  parenteral  total,  58  controversias  
relacionado  con   sobre,  59  contenido  de  agua  de,  58
quemaduras,  174  alta  presión  pulmonar,  35
Síndrome  de  Ehlers­Danlos,  como  causa  de  insuficiencia  aórtica, Ensayo  inmunoabsorbente  ligado  a  enzimas  (ELISA),  para
393 Detección  de  Helicobacter  pylori,  222
Pacientes  ancianos   Epinefrina  
quemaduras  en,  173   como  tratamiento  de  parada  cardiaca,  
incontinencia  en,  56  hernia   22  como  tratamiento  de  choque,  31,  31t  
inguinal  en,  275 uso  en  reanimación  cardiopulmonar,  20
Quemaduras  eléctricas,  178 virus  Epstein­Barr,  86
Electrocardiografía  de   Cáncer  de  esófago,  218–220
estenosis  aórtica,  391   distribución  anatómica  de,  218  historia  
correlación  con  el  flujo  iónico  de  la  membrana  de  los  cardiomiocitos,   natural  de,  219  factores  de  riesgo  de,  
27,  27f  actividad  supraventricular  activada,  26,  26f,  27,  28   218  etapas  de,  219  síntomas  de,  218  
actividad  ventricular  activada,  26,  26f,  28 tratamiento  de,  219–220

Electrolitos,  49–52  
anormalidades  de Várices  esofágicas,  208  como  
como  factor  de  riesgo  de  paro  cardíaco,   causa  de  sangrado  gastrointestinal,  208,  253  recurrente,  
22  relacionado  con  estenosis  hipertrófica  de  píloro,  415   210,  211  tratamiento  de,  208,  209–210,  209f,  210f,  
contenido  de  fluidos  corporales  de,  50t  contenido  de  fluidos   211,  211f,  212
intravenosos  de,  50,  50t
Embolia Esofagogastroduodenoscopia  (EGD),  253
aire,  broncovenoso,  120   Esofagografía,  112–113
relacionado  con  placas  ateroscleróticas,  356 Esofagoscopia,  112–113
Machine Translated by Google
498  ÍNDICE

Esófago Fiebre  (continuación)  
Barrett,  218,  221   posoperatoria,  64–67,  84  
traumatismo  penetrante  a,  120 relacionada  con  obstrucción  del  intestino  delgado,  
Ética,  en  la  unidad  de  cuidados  intensivos  quirúrgicos,  73–76 232  tratamiento  de,  65  evaluación  de,  65
Comités  de  ética,  481–482
Solución  de  conservación  Euro­Collins,  451 Fibra  dietética  
Eutanasia,  481 como  tratamiento  de  fisuras  anales,  266  
Evisceración,  71 como  profilaxis  de  diverticulitis,  241
Exanguinación,  91,  119,  140–141 Fibrinólisis,  21
Fijación  externa,  148 Fibroadenoma,  311

Extremidades,  lesiones  vasculares  a,  158–164 Fibrosis

Defectos  de  las  extremidades,  como  componente  de  la   nefrogénico  sistémico,  376  pulmonar,  
asociación  VACTERL,  421 398
Afeitado  de  cejas,  en  reparación  de  laceración  facial,  166 Mordida  de  pelea,  169
Salpicaduras  oculares,  como  factor  de  riesgo  de  infecciones  de  transmisión  sanguínea,  88 Biopsia  por  aspiración  con  aguja  fina
de  masas  mamarias,  311,  312,  314  
F diferenciadas  de  biopsia  con  aguja  gruesa,  314  de  masas  
Cara,  lesiones  por   en  el  cuello,  342,  343–344  de  cáncer  de  páncreas,  195  de  
mordeduras,  166  en   glándulas  parótidas,  340  de  nódulos  tiroideos,  292,  293
niños,  180  tasa  de  
infección  de,  69  
laceraciones,  165–168   Técnica  de  fractura  de  dedo,  133
limpias,  165  sucias,  165 Dedos,  amputados,  reimplantación  de,  169
Lesiones  en  la  punta  de  los  dedos,  169–170,  170f
Nervio  facial Fisuras,  anorrectal/ano,  248,  266

relación  anatómica  con  la  glándula  parótida,  336  lesión   fístulas

relacionada  con  endarterectomía  carotídea,  358  parálisis   anorrectal/ano,  248,  265,  266  

relacionada  con  tumor  parotídeo  de,  338  en  cirugía  de  tumor   aortoduodenal,  254  aortoentérica,  254  

parotídeo,  337,  340  parálisis  relacionada  con  cirugía  de   arteriovenosa,  158,  166  biliar­entérica,  75  

tumor  parotídeo,  339–340 broncopleural,  397,  398,  399  entero­

Adenomatosis  poliposis  familiar  (PAF),  260 enteral,  250  entero­rectal,  248–249  
Síndrome  de  melanoma  familiar  atípico  con  múltiples  lunares pancreática,  140–141 ,  203  glándula  
(FAMMM),  328,  329 salival,  339–340  traqueoesofágica,  423  

Familia,  presencia  durante  la  reanimación  cardiopulmonar, definición  de,  423  como  componente  de  
23 asociación  VACTERL,  423  sin  atresia  
Fascitis  necrotizante,  66 esofágica,  423,  424  ureterocólica,  242  
FAST  (sonografía  abdominal  enfocada  para  trauma),  91,  93 vesicocólica,  242,  243  vesicovaginal,  157

Grasa  dietética  
como  componente  de  nutrición  enteral,  57  
como  factor  de  riesgo  de  cáncer  de  próstata,  467
Emulsiones  grasas,  intravenosas,  61
Rayas  grasas,  345
Fecalitos,  apéndice,  75 Tórax  inestable,  92–93,  123  

Prueba  de  sangre  oculta  en  heces  (FOBT),  259 en  niños,  182

Derrame  fecal,  intraabdominal,  78 Flanco,  heridas  penetrantes  a,  127
Tarifa  por  servicio,  475   Golpe,  57
descontada,  475 Tendones  flexores,  laceraciones  de,  170

Aneurisma  de  la   Tenosinovitis  flexora,  169
arteria  femoral  de,  361   Resucitacion  fluida

lesiones  de,  162  uso   en  pacientes  con  abdomen  agudo,  73  
de  dispositivo  de  cierre  percutáneo  en,  163 en  pacientes  con  pancreatitis  aguda,  199  
canal  femoral,  269 déficit  de  base  en,  96  en  víctimas  de  
Fémur,  fracturas  de,  97 quemaduras,  176  en  niños,  181,  182  elección  

Cirugía  fetal,  para  reparación  de  hernia  diafragmática  congénita,   de  solución  intravenosa  para,  96  en  shock  
427 hemorrágico,  95  en  reanimación  inicial,  93  
Fiebre   inestabilidad  posterior,  97
causas  de,  64  
definición  de,  64
Machine Translated by Google
ÍNDICE  499

Reanimación  con  líquidos  (continuación) Gangrena  

en  pacientes  politraumatizados,  97 gas  clostridial,  66  no  
Fórmula  de  zonas  verdes  para,  176 clostridial,  66

Líquidos,  49–52 síndrome  de  Gardner,  260,  292
Células  de  espuma,  345 Gastrectomía,  223,  225,  227

Suplementos  de  folato,  efectos  cardioprotectores  de,  347 Bypass  gástrico,  278–279

Cuerpos  extraños,  como  causa  de  obstrucción  intestinal,  230 Roux­en­Y,  233,  278
Fracturas   Cáncer  gástrico,  221,  226

de  la  columna  cervical,  107   Obstrucción  de  la  salida  gástrica,  relacionada  con  úlcera  péptica,  222
relacionadas  con  maltrato  infantil,   Volumen  residual  gástrico  (GRV),  58
185  en  las  extremidades,  lesiones  vasculares  asociadas,  162  de   Retención  gástrica,  relacionada  con  cirugía  de  úlcera  duodenal,  225
fémur,  como  causa  de  pérdida  de  sangre  oculta,  97  de   Úlceras  gástricas,  226  

mano,  169  verdugo,  107 clasificación  de,  226  
diferenciado  de  úlceras  duodenales,  226  tratamiento  

emergente  de,  227  cáncer  gástrico  asociado  con,  227  
Jefferson,  107  abierto,   gigante,  227  perforado,  227  resangrado  en,  227  

170  pélvico,  147–150,   tratamiento  de,  227

156

angiografía  de,  148  lesiones  
vesicales  asociadas  con,  149  transfusiones  de  
sangre  en,  149  efecto  sobre  la  deambulación,  149   Gastrina,  en  el  síndrome  de  Zollinger­Ellison,  223
fijación  externa  de,  148  como  causa  de  hemorragia,   Gastritis

147 reflujo  alcalino/biliar,  224  estrés,  
228  como  causa  de  hemorragia  

Fractura  del  proceso  transverso  L5  asociada  con,  149–150 digestiva  alta,  204

Enfermedad  por  reflujo  gastroesofágico  (ERGE),  213–217
estabilización  mecánica  de,  148  como   estudio  diagnóstico  de,  213  

factor  de  riesgo  de  mortalidad,  148  lesiones   diferenciado  de  ardor  de  estómago,  213  

neurológicas,  149  abiertas,  147  prendas   tratamiento  quirúrgico  de,  214,  215f,  216,  216f  síntomas  de,  

neumáticas  antishock  y,  148  clasificación   213  procedimientos  quirúrgicos  gastrointestinales,  como  

radiográfica  de,  147  lesiones  uretrales  asociadas  a,  149   factor  de  riesgo  de  infección  de  herida,  66  estudios  de  contraste  

pene,  156,  157  de  costillas,  92–93,  122,  123 ,  130  en   del  tracto  gastrointestinal  de,  417  hemorragia  de.  Ver  

niños,  182  como  causa  de  pérdida  de  sangre  oculta,  97 Hemorragia,  gastrointestinal  inferior,  poblaciones  bacterianas  de,  

77  Fugas  gastroyeyunales,  278–279  Gastropexia,  Hill,  214,  

215f  Gastrostomía,  57  Gatorade,  49,  50t,  51  Anestesia  general,  
implicaciones  para  la  alimentación  enteral,  59  Anomalías  genéticas,  
fertilización  in  vitro­  relacionado,  456  Genitales,  ambiguos,  474  Nervio  

síndrome  de  Frey,  340 genitofemoral,  lesión  relacionada  con  el  enganche  del  psoas,  153–154

funduplicatura
Duende,  214,  215f,  216

con  laparoscopia,  217  con  
laparotomía,  217

Mechón  (parcial),  214,  216f
Furosemida  

como  sándwich  de  lasix,  36,  52  
como  tratamiento  de  estenosis  mitral,  385 Gentamicina,  66

fútil  eso,  480 Jorge  V,  Rey  de  Inglaterra,  405

Futilidad  médica,  480 Escala  de  coma  de  Glasgow,  93,  99
suma  de  Gleason,  467
GRAMO
Nervio  glosofaríngeo,  endarterectomía  carotídea

Vesícula  biliar,  lesiones  contusas  a,  122 lesión  relacionada  con,  358
Carcinoma  de  vesícula  biliar,  192 Guante,  doble,  88
Enfermedad  de  la  vesícula  biliar,  191–193 Suplementación  con  glutamina,  en  pacientes  quirúrgicos,  62

Gammagrafía  con  galio,  de  abscesos  intraabdominales,  83 Bloqueadores  de  los  receptores  de  glicoproteína  IIb/IIIa,  348
Cálculos  biliares,   Godfrey,  Arturo,  405

191  asintomáticos,  191   Bocio  

diagnóstico  diferencial  de,  194  imágenes   tóxico  multinodular,  292  tóxico  

de,  75  como  causa  de  pancreatitis,  199,   nodular,  290  (enfermedad  de  Plummer)
201 ''Hora  dorada'',  91
Machine Translated by Google
500  ÍNDICE

regla  de  Goodsall,  266 Trasplante  de  corazón  y  pulmón,  439,  448  
fórmula  de  Gorlin,  384 anastomosis  en,  439  número  anual  de,  448  

Cierre  de  Graham,  de  úlceras  pépticas  perforadas,  225 como  ''trasplante  de  corazón  dominó'',  443  

Granulomas  relacionados  con  la  enfermedad  de  Crohn,  248 indicaciones  para,  449  Soplos  cardíacos  

Enfermedad  granulomatosa,  como  causa  de  hipercalcemia, relacionados  con  estenosis  aórtica,  391  
281 relacionados  con  aneurisma  aórtico  disecante,  410  

enfermedad  de  Graves,  288,  292 relacionados  con  insuficiencia  mitral ,  387  Ritmo  

Signo  de  Turner  gris,  199 cardíaco.  Véase  también  Bradicardia;  Taquicardia  en  
Hormona  de  crecimiento,  recombinante,  62 shock  hemorrágico,  95  Ruidos  cardíacos,  relacionados  
Heridas  de  bala   con  la  tríada  de  Beck,  119  Trasplante  de  corazón,  439–444  
abdominales,  125–126,  127   anastomosis  en,  439  número  anual  de,  439  rechazo  de  
colónicas,  144  hepáticas,  130  en  la   aloinjerto  cardíaco  en,  441,  442  ''trasplantes  de  corazón  dominó'',  

línea  media  del  cerebro,  102   443  corazón  de  donante  isquémico  tiempo  en,  441  donantes  de  

miocárdicas,  120  en  el  cuello,  110,  110t   corazón  para,  440,  441  antecedentes  de,  439,  448  indicaciones  

torácicas,  118,  119 para,  440  ortotópico,  439–440  Candidatos/receptores  de  

trasplante  de  corazón  puente  para  trasplante  en,  445,  447  

aloinjerto  cardíaco  vasculopatía  en,  442–443  criterios  para,  

Intestino,  suministro  arterial  a,  235 440  terapia  de  inducción  en,  442  causas  de  mortalidad  en,  

441  tasa  de  mortalidad  en,  440  patrón  de  infección  
H   postrasplante  en,  441  tasas  de  supervivencia  en,  444  
Halo  nevo,  328  Halsted,   dispositivos  de  asistencia  ventricular  en,  445–447  Trasplantes  

William  S.,  224,  324  Hamartoma,  258,   cardíacos  (injertos)  quimerismo  en,  451  denervación  y  

407  Mano reinervación  de ,  443  soluciones  de  conservación  para,  451,  452  

segundo  trasplante  de,  443  infecciones  por  Helicobacter  
infecciones  de,  170   pylori,  228  como  causa  de  aterosclerosis,  347  como  causa  

lesiones  a,  169–172  heridas   de  esófago  de  Barrett,  221  pruebas  diagnósticas  para,  221,  

por  mordedura,  170–171  como   226  como  causa  de  úlcera  duodenal,  221,  222–223  

causa  de  deformidad,  171  lesiones   erradicación/tratamiento  de,  222–223,  227  ''ensayo  de  

en  la  punta  de  los  dedos,  169–170,  170f   curación''  en,  227  como  causa  de  carcinoma  gástrico,  221  

lesiones  por  inyección,  171  entablillado,  169   como  causa  de  úlcera  gástrica,  226–227  como  causa  de  

superficie  palmar  de,  uso  en  la  estimación  del   úlcera  péptica,  221,  348  Hematemesis,  252  relacionada  con  

tamaño  de  quemaduras, fístula  aortoduodenal,  254  laceración  hepática  relacionados,  
175   254  Hematoquecia,  252  Hematocrito  en  abdomen  agudo,  75  en  

Lesiones  del  manillar,  en  niños,  183  Hardy,   pérdida  de  sangre,  96  como  indicación  de  transfusión  de  

James,  448  Ecuación  de  Harris­Benedict  (HB),  55   sangre,  47–48  Cáncer  hematológico,  323  Hematoma  

Operación  de  Hartmann,  242  Examen  de  cabeza   disecante  aórtico,  410  epidural,  101

y  cuello,  343  Lesiones  en  la  cabeza.  Véase  
también  Lesión  cerebral  traumática  en  niños,  180,  

182,  184  Lesión  espinal  asociada  con,  105

Maniobra  de  inclinación  de  cabeza­levantamiento  de  mentón,  19

Costos  de  atención  médica,  477

Reforma  de  la  atención  de  la  salud,  475–478

Trabajadores  de  la  salud,  exposición  a  enfermedades  transmitidas  por  la  sangre  en,
86–87

Seguro  de  salud,  definición  de,  476

Organizaciones  de  mantenimiento  de  la  salud  (HMO),  476  
administradores  de,  476  gerentes  de,  476

Corazón,  relacionado  con  biopsia  endomiocárdica  transvenosa

perforación  de,  442  

Bloqueo  cardíaco  relacionado  con  el  reemplazo  de  la  válvula  aórtica,  
394  Acidez  estomacal,  213  diferenciada  de  la  enfermedad  por  reflujo  

gastroesofágico  (ERGE),  213  Insuficiencia  cardíaca.  Ver  también  

Insuficiencia  cardíaca  congestiva

como  causa  del  síndrome  de  dificultad  respiratoria  del  adulto,  35  

tratamiento  de  trasplante  de  células  madre  para,  443
Machine Translated by Google
ÍNDICE  501

Hematoma  (continuación)   Hemotórax  (Continuación)  
intraparenquimatoso,  101   relacionado  con  trauma  abdominal  penetrante,  126  como  
tumor  parotídeo  relacionado  con  cirugía,  339–340   factor  de  riesgo  de  empiema  postraumático,  120
pulsátil,  152  subdural,  101  subescapular,  132t Mecanismo  
de  acción  de  la  heparina,  367  
como  profilaxis  de  la  trombosis  venosa  profunda,  367  de  
Hematuria,  464   bajo  peso  molecular,  367
macroscópica  retardada,   Arteria  hepática,  131  
152  relacionada  con  trauma  renal,  151,   ligadura  selectiva  de,  133
152  relacionada  con  cálculos  urinarios,  458 Venas  hepáticas,  132
Hemigastrectomía,  227 Hepatitis,  diagnóstico  diferencial  de,  194
Hemobilia,  134,  254 Vacuna  contra  la  hepatitis  A,  87

Hemodinámica,  pediátrica,  181,  185 Hepatitis  B,  86,  87,  88  tasa  
Estabilidad  hemodinámica,  valoración  de,  51,  93 de  seroconversión  para,  89  
Hemoglobina,  capacidad  de  unión  de  oxígeno  de,  47 transmisión  de  cirujano  a  paciente,  88
Hemoperitoneo,  130  en   Vacuna  contra  la  hepatitis  B,  87,  88
niños,  183  como  indicación   Hepatitis  C,  86,  87,  88  
de  laparotomía  de  emergencia,  123 tratamiento  de  trasplante  hepático  de,  433  como  
hemoptisis,  385,  399 causa  de  hipertensión  portal,  206  tasa  de  
Hemorragia   seroconversión  de,  89
arterial,  158   Hepatoblastoma,  429,  430  
control  de,  160   diferenciado  de  carcinoma  hepatocelular,  430
epidural,  en  niños,  182   Carcinoma  hepatocelular  
relacionada  con  enfermedad   diferenciado  de  hepatoblastoma,  430  tratamiento  de  
diverticular  gastrointestinal,  241,  242  relacionada   trasplante  hepático,  435
con  cirugía  de  úlcera  duodenal,  225  relacionada   flujo  hepatopétalo,  206
con  úlcera  gástrica,  227  inferior,  255–257  oculta,   Síndrome  hepatopulmonar,  207
252  en  pacientes  con  pancreatitis,  204  péptica   Síndrome  hepatorrenal,  207
relacionado  con  enfermedad  ulcerosa,  222,  226   hepatotomía,  133
relacionado  con  ruptura  de  pseudoaneurisma,   Hernia  

134  resolución  espontánea  de,  257  superior,  204,   diafragmática
252–254  várices,  207,  208,  209–210,  209f,  210f,   Bochdalek,  426  
211,  211f,  212  relacionado  con  trauma  hepático,   congénita,  426–428
133  masiva,  relacionada  con  lesión  de  órgano   Morgani,  426  
sólido,  184  relacionada  con  fractura  pélvica,  147,  148,  149   femoral,  269  como  
relacionada  con  traumatismo  torácico  penetrante,  118   contraindicación  para  la  reparación  de  hernia  de  Lichtenstein,
relacionada  con  traumatismo  penetrante  de  cuello,  111,  112   272

rectal,  248,  255,  262,  265  retina,  síndrome  del  bebé  sacudido   mal  diagnosticada  como  hernia  inguinal  directa,  274  como  
­relacionado,  184  retroperitoneal,  76,  152  subaracnoideo,   contraindicación  para  la  reparación  de  la  hernia  de  Shouldice,  
relacionado  con  el  síndrome  del  bebé  sacudido, 269  reparación  quirúrgica  de,  272,  273f  ''ingle'',  269  hiatal,  
relacionada  con  la  enfermedad  por  reflujo  gastroesofágico,  213  
encarcelada

diferenciada  de  hernia  estrangulada,  274  en  bebés  o  
niños,  270  como  cirugía  de  emergencia,  276  incisional,  
184   233  en  bebés  o  niños,  269–270  inguinal,  269–276  
subdural   cuestiones  anatómicas  relacionadas,  276  directa,  269,  271,  
en  niños,  182   274,  275  en  ancianos  pacientes,  275  defecto  fascial  de,  274  
relacionado  con  el  síndrome  del  bebé  sacudido,  184 indirecta,  269,  271,  274  reparación  laparoscópica  de,  272–
Hemorroides,  267   273,  274,  276  reparación  quirúrgica  de,  270,  270f,  271,  
externas,  267   274  recurrencia  de,  271,  275  problemas  quirúrgicos  
internas,  267   relacionados  con,  276  cirugía  complicaciones  de,  275
trombosadas,  266
Hemostasia  

emergente,  111  
en  pacientes  con  trauma  hepático,  133
Hemotórax  

relacionado  con  trauma  cerrado,  
114  masivo,  92–93
Machine Translated by Google
502  ÍNDICE

Hernia  (continuación) Hipercapnia,  permisiva,  42
Peterson,  233   Síndromes  de  hipercoagulabilidad/hipercoagulabilidad,  366
deslizante,  275  como   Hiperglucemia,  relacionada  con  la  nutrición  parenteral,  61
causa  de  obstrucción  del  intestino  delgado,  230,  231   Hiperinsulinemia,  346
diafragmática  estrangulada,  119  diferenciada  de  hernia   Hiperpotasemia,  como  causa  de  parada  cardiaca,  21
encarcelada,  274  como  emergencia  quirúrgica,  276   Hiperlipidemia  
Herniorrafia,  inguinal,  tasa  de  infección  de  herida  quirúrgica   como  causa  de  aterosclerosis,  345  

de,  68  Hesselbach,  Franz  K.,  269  Triángulo  de  Hesselbach ,   relacionada  con  trasplante  de  corazón,  443  
269  Lipoproteína  de  alta  densidad  (HDL),  346  Hipócrates,  458,   relacionada  con  trasplante  de  pulmón,  449
460  Enfermedad  de  Hirschsprung,  417,  418–419   hiperparatiroidismo,  281–287,  292  síndromes  
Bloqueadores  del  receptor  de  histamina,  213–214,  222–223   de  neoplasia  endocrina  múltiple  asociados  con,  286  normocalcémico,  
Histoplasmosis,  408  Coincidencia  de  HLA  (antígeno  leucocitario   283  úlceras  pépticas  asociadas  con,  222  persistente  y  

humano),  438  HLA  (humano  incompatibilidad  de  antígeno   recurrente,  285  primario,  283  exploración  quirúrgica  para,  283  

leucocitario),  en  receptores  de  trasplante  de  corazón,  442 tratamiento  quirúrgico  de,  284,  286  secundario,  285,  286  sestamibi  
escaneos  de,  283–284  síntomas  de,  281  terciario,  285,  286  
tratamiento  de,  282

Placas  de  Hollenhorst,  356–357
Lesión  visceral  hueca,  123,  183
signo  de  Homan,  366
La  homocisteína,  como  factor  de  riesgo  de  enfermedad  cardiovascular, Relacionado  con  
347 adenoma  de  hipertensión,  303,  304  

''Período  de  luna  de  miel'',  426 como  factor  de  riesgo  de  aterosclerosis,  345  

Síndrome  de  Horner,  100 relacionado  con  aneurisma  aórtico  disecante,  410,  411  
contabilidad  hospitalaria,  475 relacionado  con  trasplante  de  pulmón,  449  relacionado  con  
Gastos  hospitalarios,  475 feocromocitoma,  304  causas  portales  de,  206  definición  de,  
Alta  hospitalaria,  13 206  postsinusoidal,  206  presinusoidal,  206  sinistral ,  207  
Comités  de  ética  hospitalaria,  481–482 sinusoidal,  206  efectos  sistémicos  de,  207  hemorragia  

Infección  por  el  virus  de  la  inmunodeficiencia  humana  (VIH),  86,  87   varicosa  asociada  con,  208  venosa,  207  portopulmonar,  
transmisión  sanguínea  de,  86,  87  exposición  de  los  trabajadores  de  la   207  postraumática  renal,  153  pulmonar,  relacionada  con  
salud  a,  88  tasa  de  seroconversión  de,  89 estenosis  mitral,  384,  385  quirúrgica,  298–301  
renovascular,  298,  299

Coincidencia/falta  de  coincidencia  del  antígeno  leucocitario  
humano  (HLA),  58,  438
Virus  linfotrópico  de  células  T  humanas  (HTLV­1),  86
Hidrocele,  de  cordón  espermático,  269
Peróxido  de  hidrógeno,  67
Hidronefrosis,  prenatal,  472
Presión  hidrostática,  intravascular,  35
Deficiencia  de  21­hidroxilasa,  474
3­hidroxi­3­metilglutaril  coenzima  A  (HMG­CoA)
inhibidores  de  la  reductasa,  349   Hipertermia  maligna,  64,  67
uso  en  receptores  de  trasplante  de  corazón,  443 Hipertiroidismo,  288–291  causas  
Higroma,  quístico,  432 más  comunes  de,  288  signos  y  síntomas  

Hiperaldosteronismo   de,  288  tratamiento  de,  289,  290
primario  (síndrome  de  Conn),  298,  300,  303  cribado,  303
Hipertrigliceridemia,  346
Hiperamilasemia  como   Hipocalcemia  post­
indicador  de  pancreatitis,  199  como   tiroidectomia,  295  tratamiento  
indicador  de  obstrucción  del  intestino  delgado,  231 de,  285

Diagnóstico   Nervio  hipogloso,  lesión  relacionada  con  endarterectomía  carotídea,  
diferencial  de  hipercalcemia,  281   358
relacionada  con  hiperparatiroidismo,  281,  282  en   Hipoglucemia,  como  causa  de  parada  cardiaca,  21
pacientes  hospitalizados,  277  en  pacientes   Hipopotasemia,  como  causa  de  parada  cardiaca,  21,  22
ambulatorios,  277  postoperatorio,  284 Hipomagnesemia,  como  causa  de  parada  cardiaca,  22
Hipoparatiroidismo,  postiroidectomía,  295
Machine Translated by Google
ÍNDICE  503

Hipospadias,  relacionado  con  criptorquidia,  474 Incidentaloma,  302–303,  306  resección  

Hipotensión de,  306  Incisiones,  efecto  quirúrgico  
Relacionado  con  la  tríada  de   sobre  los  patrones  de  respiración  
Beck,  119  relacionado  con  lesión   espontánea,  65  efecto  sobre  la  capacidad  vital,  34,  65  
cerebral,  99  como  factor  de  riesgo  de   infecciones  de,  84  Rockey­Davis,  187,  188  Incontinencia  
mortalidad,  123  como  indicador  de  pérdida  de   ectópica  relacionada  con  el  uréter,  473  en  el  geriátrico  

sangre  pediátrica,  181  relacionado  con  neumotórax   población,  437  Infantes.  Véase  también  Neonatos
a  tensión,  122  como  causa  del  tercer  espacio,  52  
relacionado  con  hemorragia  digestiva  alta,  252
Síndrome  del  martillo  hipotenar,  163,  163f  Hipotermia,  
durante  la  reanimación,  en  niños,  181  Hipotermia.  Ver  también  
Choque,  hemorrágico/hipovolémico lesiones  ateroscleróticas  en,  345  nefroma  

como  causa  de  paro  cardíaco,  21   mesoblástico  congénito  en,  474  hernia  en,  269–270  
inducción  posresucitación  de,  21  durante  la   obstrucción  intestinal  en,  417–420  muerte  traumática  en,  

reanimación,  en  niños,  181 180

Hipotiroidismo,  posoperatorio,  290
Hipoventilación,  controlada,  42 Infecciones  

Hipovolemia   diferenciadas  de

como  causa  de  parada  cardiaca,  21,  23   contaminación,  77,  78  

respuesta  fisiológica  a,  95 infestación,  79–80  de  la  mano,  

Hipoxemia   170  pulmonar,  398  quirúrgica,  

relacionada  con  lesión  cerebral,   77–85  profilaxis  antibiótica  contra,  
99  como  causa  de  confusión  mental,  46   77  barreras  para,  77  manejo  de,  
relacionada  con  pancreatitis,  199 82  Mononucleosis  infecciosa,  342  Arteria  
Hipoxia,  como  causa  de  parada  cardiaca,  21 mesentérica  inferior,  235  vías  colaterales  de,  

Histerectomía   235  oclusión  de,  238  Filtros  de  vena  cava  
como  causa  de  obstrucción  del  intestino  delgado,   inferior,  108,  367  Vena  de  vena  cava  inferior,  en  

233  como  causa  de  fístula  vesicovaginal,  157 carcinoma  de  células  renales,  462  Infertilidad,  
abordaje  quirúrgico  de,  455–457  Infestación,  
I diferenciada  de  infección,  79–80  Enfermedad  

Ibuprofeno   inflamatoria  intestinal,  248–251,  266  Ver  también  

como  tratamiento  de  quemaduras,   Colitis  ulcerosa;  enfermedad  de  Crohn
174  como  tratamiento  de  fiebre,  65

Bolsa  ileal­anal,  250  
Estenosis  ileal,  249  Ileocolitis,  
248  Anastomosis  ileorrectal,  
250  Ileostomía  Brooke,  250   como  causa  de  obstrucción  intestinal,  230  

permanente,  249  Íleon  en  la   factores  genéticos  en,  249  terapia  médica  
enfermedad  de  Crohn,  248   para,  249–250
Contenido  de  electrolitos,  50t  Íleo,   Entrada,  en  injertos  de  derivación,  352  
232  Cálculo  biliar,  75  Meconio,  417,   frente  a  salida,  352
418  paralítico,  246  Iliopectíneo   consentimiento  informado,  480;

ligamento,  269  Tracto  iliopúbico,  en   Injertos/procedimientos  de  derivación  infraguinal,  352
reparación  de  hernia  inguinal,  276   Enfermedad  oclusiva  infraguinal,  terapia  endovascular

Fallecimiento  inminente,  480   para,  354–355  
Pacientes  inmunocomprometidos.   Ligamento  inguinal,  269  
Ver  también  Síndrome  de   Lesiones  por  inhalación,  175,  176  
inmunodeficiencia  adquirida  (SIDA);   Evaluación  inicial,  de  pacientes  con  trauma,  91–94  Lesiones  
Infección  por  el  virus  de  la  inmunodeficiencia  humana  (VIH)   por  inyección,  171  Lesiones.  Véase  también  Trauma;  tipos  
cáncer  en,  326  tuberculosis  en,  399  Inmunohistoquímica,   específicos  de  lesiones
324  Inmunoterapia,  para  el  cáncer,  325–326 como  causa  de  mortalidad  pediátrica,  180
Agentes  inotrópicos,  como  tratamiento  de  choque,  31

Relación  inspiratoria/espiratoria  (I/E),  41
Resistencia  a  la  insulina,  277

Unidades  de  cuidados  intensivos  
(UCI)  tratamiento  de  la  arritmia  cardíaca  en,  25–26  ética  
en,  73–76  control  de  infecciones  quirúrgicas  en,  77

para  melanoma,  333
Machine Translated by Google
504  ÍNDICE

Interferón­gamma,  como  tratamiento  del  melanoma,  333   Yeyunostomía,  57
Interleucina­2,  como  tratamiento  del  melanoma,  333   Artículos  de  revista,  quirúrgicos,  14
Obstrucción  intestinal.  Véase  también  Colon,  obstrucción  de;   Justicia,  479
Intestino  delgado,  obstrucción  en  neonatos  y  lactantes,  417–
420  Resección  intestinal,  seguimiento  de  nutrición  parenteral, K  

Signos  de  Kanaval,  169  
62 Sarcoma  de  Kaposi,  328  
intestinos  Véase  también  Colón;  Duodeno;  Suministro  arterial  del   Fiebre  de  Katayama,  206  
intestino  delgado  a,  235  Detección  ecográfica  dúplex  con   Signo  de  Kehr,  74  Kennedy,  
colgajo  de  la  íntima,  159  en  fracturas  de  fémur,  163f  Detección   John  F.,  307  Trasplante  de  
angiográfica  por  sustracción  de,  159f  Lesión  de  la  íntima,   riñón  y  páncreas,  437  Anomalías  renales  de,  
venosa,  366  Bomba  de  balón  intraaórtico  (IABP),  32,  382   como  componente  de  la  asociación  VACTERL,
Presión  intracraneal  elevada,  101,  102  tratamiento  de,  101,  
103  seguimiento  de,  101  reducción  de,  101  Taponamiento  con   421  

balón  intrahepático,  133  Acceso  intraóseo,  en  niños,  181–182   lesiones  por  
Acceso  intravenoso traumatismo  cerrado,  151–
154  clasificación  de,  152  
lesión  pancreática  asociada,  140  lesiones  
del  pedículo  renal,  151  masas  sólidas  en,  
462  tolerancia  a  la  isquemia  caliente  en,  152

Trasplante  de  riñón,  436–438  donante  
vivo,  436,  437
en  niños,  181–182   Trasplantes  de  riñón  (injertos)
emergente,  93 tiempo  de  isquemia  fría  para,  436  
Líneas  de  catéteres  intravenosos.  Ver  Catéteres   tasas  de  supervivencia  para,  436

Intususcepción,  230,  419  Fertilización  in  vitro  (FIV),   Kilocalorías,  55,  56t,  59
455,  456  Yodo,  como  agente  bacteriostático,  79   Kocher,  Teodoro,  290–291
Irrigación  como  tratamiento  de  lesiones  rectales   maniobra  de  Kocher,  141
extraperitoneales,  145  de  heridas  infectadas,  67  
Isquemia  aguda,  354  perfusión  después,  354  5  Ps  de,  354   L  

relacionado  con  lesión  arterial,  163  miembro  crítico,  350,   Laceraciones  
351  relacionado  con  síndrome  de  martillo  hipotenar,  163   faciales,  165–168  
intestinal,  235–240 a  los  tendones  flexores,  170  al  
cuello,  110  renal,  152  al  cuero  
cabelludo,  100  Lactato,  contenido  
de  líquido  intravenoso,  50t  
Ligamento  lacunar,  269  Laparoscopia  en  
apendicectomía,  189  en  colecistectomía,  191  efecto  
sobre  el  VIH  riesgo  de  contaminación,  88  en  
determinación  de  viabilidad  intestinal  en,  237   tratamiento  de  obstrucción  de  intestino  delgado,  
embólico,  235,  236  mesentérico,  235,  237– 233  en  tratamiento  de  enfermedad  de  cálculos  
238,  239  trombótico,  235,  236,  237 urinarios,  459  Laparotomía  de  abdomen  agudo,  
76  profilaxis  antibiótica  en,  83–84  de  traumatismo  
Lesión  por  isquemia­perfusión,  como  causa  del  síndrome   abdominal  cerrado,  123  de  hemoperitoneo,  en  niños,  
compartimental,  161 183  de  lesiones  abdominales  penetrantes,  125–126  
Trasplantes  de  células  de  islotes,  438 como  tratamiento  de  obstrucción  del  intestino  delgado,  
Isoniazida,  como  tratamiento  de  la  tuberculosis,  399 233  de  lesiones  esplénicas,  137  por  etapas  o  
Isosorbide,  como  tratamiento  de  la  angina,  379 abreviadas,  123,  124  Intestino  grueso.  Ver  Colon  
Laringe  nervios  recurrentes,  295  lesiones  a,  284,  295,  
j 358
Ictericia

trabajo  de  diagnóstico  de,  197  
relacionado  con  el  cáncer  de  páncreas,  194,  195,  196  
relacionado  con  la  pancreatitis,  199
Maniobra  de  tracción  mandibular,  92–93
Fractura  de  Jefferson,  107
Testigos  de  Jehová,  47
Bypass  yeyunoileal,  278 superior,  daño  a,  295
Machine Translated by Google
ÍNDICE  505

Laringoscopia,  de  lesiones  penetrantes  del  cuello,  112–113 Pruebas  de  función  

Sándwich  de  Lasix  (furosemida),  36,  52 hepática  para  evaluación  de  ictericia,  
Lavado   197  para  evaluación  de  cáncer  de  mama  metastásico,  316  

peritoneal  de  diagnóstico,  119  de   para  evaluación  de  cáncer  de  páncreas,  195  para  evaluación  
trauma  abdominal  cerrado,  122–123  en  niños,  182   de  síndrome  de  Stauffer,  463  Trasplante  de  hígado,  433–435  
de  trauma  pélvico,  149  de  trauma  abdominal   controversias  sobre,  434  indicaciones  para,  433  donante  vivo,  

penetrante,  127  en  evaluación  de  trauma,  93   434,  435  no  donantes  a  corazón  latiendo,  435  técnica  ''piggy­
pulsátil  de  alta  presión,  70 back''  en,  434  complicaciones  postoperatorias  de,  434  

determinación  de  prioridad  para,  208  como  tratamiento  de  
hemorragia  recurrente  por  várices,  211  Trasplantes  hepáticos  
Ley  de  Laplace,  244 (injertos),  tiempo  óptimo  de  isquemia  fría  para,  433  
Postcarga  del  ventrículo  izquierdo,  insuficiencia  mitral  que   Testamentos  en  vida,  480  Lobectomía,  derecha,  133  

exacerba  el  efecto  de,  387 Lipoproteínas  de  baja  densidad  (LDL),  346  Esfínter  esofágico  
Hipertrofia  ventricular  izquierda,  391,  394 inferior,  en  enfermedad  gastroesofágica,  213  Síntomas  del  
Melanoma  maligno  léntigo,  329 tracto  urinario  inferior  (STUI),  469  Lumpectomía,  316–317,  319–
Leptina,  347 320  Diagrama  de  Lund  y  Browder,  para  estimación  del  
Condiciones  letales,  480 tamaño  de  quemaduras,  175  Lung.  Véase  también  Trastornos  
Letargo,  100 pulmonares  absceso  de,  398–399  superficie  alvéolo­capilar  de,  34  

Leucocitosis,  138,  232 contusiones  de,  115,  123  en  niños,  182  distribución  de  la  
Levotiroxina,  como  tratamiento  del  nódulo  tiroideo,  293 ventilación,  36–37  "caída",  124  flujo  de  líquido,  35  "húmedo" ,''  35,  
Reparación  de  hernia  de  Lichtenstein,  271,  272,  275,  276 36,  37  Cáncer  de  pulmón,  401–406  factores  genéticos  en,  

síndrome  de  Li­Fraumeni,  309 401–402  tipos  histológicos  de,  402,  402t  como  causa  de  mortalidad,  
Ligamento  de  Treitz,  como  sitio  de  hemorragia  gastrointestinal,  254,   262,  401  de  células  no  pequeñas,  405  presentándose  como  nódulo  
255 pulmonar  solitario,  407  factores  de  riesgo  para,  401–402  detección  
Isquemia  de  extremidades,  crítica,  350,  351,  354 de,  402  estadificación  de,  403,  403t,  404  síntomas  de,  403  

Perfusión  de  extremidades,  aislada,  como  tratamiento  de  melanoma, tratamiento  de,  404,  405  Trasplante  de  pulmón­páncreas,  452–453  
334 Perfusión  pulmonar,  36,  37  Trasplante  de  pulmón,  448–454
extremidades.  Ver  Extremidades  

Lipasa,  199,  231  Lípidos  como  
causa  de  aterosclerosis,  346  
intravenosos,  contraindicación  en  

pancreatitis,  60  Dieta  líquida,  posoperatoria,  59  ''Litotomistas'',  
460–461  Litotricia,  459,  460  Suministro  de  sangre  al  hígado,  131,  
206,  214  lesiones  a,  130–135  trauma  cerrado,  122  complicaciones  

de,  134  hemostasia  para,  133  laceraciones,  254  manejo  no  quirúrgico  
de,  132  manejo  quirúrgico  de,  132,  132t  lesión  pancreática  asociada,  

140  respuesta  a  lesiones  en,  130  anatomía  quirúrgica  de,  131,  
131f  drenaje  venoso  de,  132  Cáncer  de  hígado,  como  indicación  
para  trasplante  de  hígado,

derivación  cardiopulmonar  en,  452  
433 complicaciones  de,  449  bipulmonar,  448,  
Enfermedad  del  higado.  Ver  también  Cirrosis 449  relacionado  con  la  vida,  452  

alcohólico,  tratamiento  de  trasplante  de  hígado  de,  434  clasificación   monopulmonar,  448,  449  como  retrasplante,  
de,  207,  207t,  208 452  tipos  de,  448
insuficiencia  hepática

como  causa  del  síndrome  de  dificultad  respiratoria  del  adulto,  35  
alimentaciones  enterales  en,  57 Quimerismo  de  

Puntuación  de  Mayo  End­stage  Liver  Disease  (MELD)  de,  208,  433   trasplantes  de  pulmón  

restricción  de  proteínas  en,  55 en,  451  infección  de,  

450  soluciones  de  conservación  para,  451
Machine Translated by Google
506  ÍNDICE

Trasplantes  de  pulmón  (Continuación) Punto  de  McBurney,  187

flujo  de  sangre  pulmonar  a,  451  rechazo   McVay,  Chester,  269

de,  450 Reparación  de  hernia  de  McVay,  269,  271,  272,  275

Cirugía  de  reducción  de  volumen  pulmonar,  452 Presión  arterial  media  (PAM),  en  pacientes  con  lesión  medular,  108
síndrome  de  Lutembacher,  386
Linfedema,  334 Asistencia  circulatoria  mecánica,  445–447

Disección  de  ganglios  linfáticos,  325   Ventilación  mecánica,  39–45  en  pacientes  

en  cáncer  de  mama,  316  durante   adultos  con  síndrome  de  dificultad  respiratoria,  37,  43  ''luchar  

operaciones  de  cáncer,  324  en  melanoma,   contra  el  ventilador''  durante,  43–44  variables  límite  en,  40  

333,  334 modos  de,  39  control  asistido,  39,  40  mandatorio  intermitente,  39,  

Linfoma   40  controlado  por  presión,  39,  40  intermitente  obligatorio  

pancreático,  194  de   sincronizado,  39,  40

la  glándula  parótida,  338  que  se  
presenta  como  masa  en  el  cuello,  342,  343  
tiroides,  293

METRO
variables  de  fase  en,  40  

Magnesio,  uso  en  reanimación  cardiopulmonar,  22 posición  prona  durante,  43  variables  
de  activación  en,  40

Angiografía  por  resonancia  magnética  de  la   divertículo  de  Meckel,  188,  189

enfermedad  de  la  arteria  carótida,  371   Íleo  meconial,  417,  418

como  factor  de  riesgo  de  fibrosis  sistémica  nefrogénica,  376 Tapón  de  meconio,  417

Resonancia  magnética  como   Síndrome  del  ligamento  arqueado  mediano,  239

complemento  de  la  mamografía,  309–310  de  cáncer   Mediastinoscopia,  405
de  mama/masas,  315,  316  de  masas  en  el  cuello,  343   Mediastino  aire  

de  obstrucción  del  intestino  delgado,  231  de  lesiones   adentro,  120  

de  la  columna,  106 heridas  de  bala  a,  119
Inutilidad  médica,  480.
paludismo,  86 Historia  clínica,  como  componente  de  valoración  nutricional,  

Lágrimas  de  Mallory­Weiss,  252 53
Desnutrición   Megacolon,  tóxico,  246

como  causa  del  síndrome  de  dificultad  respiratoria  del  adulto,  35   Melanoma,  323,  328–335  definición  

primaria,  53  secundaria,  53 de,  328  disección  electiva  de  

ganglios  linfáticos,  333  incidencia  de,  328,  329  
Malrotación  intestinal  asociada   metastásico,  328,  332,  332t,  333,  334  factores  de  

a  hernia  diafragmática  congénita,  426  con  vólvulo  del  intestino   riesgo  de,  328  sitios  de,  329  regresión  espontánea  

medio,  419 de,  326,  329  sistemas  de  estadificación  para,  329–

Mamografía,  310–311,  313  diagnóstico,   330,  330f,  331,  331t  subungueal,  332  tratamiento  de,  

310  tasa  de  falsos  negativos  de,  309   333,  334  tipos  de,  329  ulceración  de,  330  signos  de  

preoperatorio,  315  de  rutina,  309   advertencia  de,  329

detección,  281,  310

Manitol,  101

Marcy,  Henry  Orlando,  274
Técnica  Marcy,  de  reparación  de  hernias,  274 Puntaje  MELD  (enfermedad  hepática  en  etapa  terminal  de  Mayo),  208,
Síndrome  de  Marfan,  como  causa  de  insuficiencia  aórtica,  393 433

Arteria  marginal  de  Drummond,  235 Melena,  252  

Malla  Marlex,  uso  en  reparación  de  hernias,  271,  272,  272f Melfalán,  como  tratamiento  del  melanoma,  334

Martell,  Capitán,  286 Mendl,  Félix,  286

Mastectomía   Meningitis,  fugas  de  líquido  cefalorraquídeo  asociadas  con,
radical  modificada,  316–317  parcial,   100  

316–317,  318  con  radioterapia,  318   Examen  del  estado  mental,  93  Vena  

segmentaria,  317  total  (simple),  316– mesentérica,  trombosis  de,  237–238  Mesenterio,  lesiones  

317  recidiva  tumoral  posterior,  324 contusas  de,  122  Síndrome  metabólico  (síndrome  X),  

277,  346–347  Metástasis,  324.  Véase  también  metastásico  en  tipos  
específicos  de  cáncer  suprarrenal,  302

Puntaje  de  Mayo  End­stage  Liver  Disease  (MELD),  208,  433
Machine Translated by Google
ÍNDICE  507

Metástasis  (continuación)   Balas  de  
definición  de,  324  renal,  462   miocardio  alojadas  en,  120  
de  tumores  sólidos,  324 contusiones  a,  115

Mioneuronecrosis,  162
meticilina,  resistencia  bacteriana  a,  80

Metimazol,  como  tratamiento  del   norte

hipertiroidismo,  289 Lesiones  en  el  lecho  ungueal,  170

Metilprednisolona,  como  tratamiento  de  lesión  medular,  107,  108 Alimentación  por  sonda  nasoduodenal,  57
Aspiración  por  sonda  nasogástrica,  para  evaluación  de  
metronidazol,  66 hemorragia  gastrointestinal,  253
ley  de  Meyer­Weigert,  473 Alimentación  por  sonda  nasogástrica,  57
Micción,  raíces  sacras  involucradas  en,  471 Programa  Nacional  de  Cirugía  Adyuvante  de  Mama  e  Intestino
Síndrome  del  lóbulo  medio,  399 (NSABP),  317,  326
Comisurotomía  mitral,  386 Cuello

Regurgitación  mitral,  387–390  causas   zonas  anatómicas  de,  111,  111f,  112  quistes  y  
de,  387  soplo  cardíaco  asociado   senos  congénitos  de,  431–432  lesiones  en  niños,  180  
con,  387  fisiopatología  de,  387  tratamiento   potencialmente  mortales,  113  penetrantes,  110–113
quirúrgico  de,  388  con  cirugía  mínimamente  
invasiva,  388–389  síntomas  de,  387

masas  en,  342–344  
Estenosis  mitral,  384–386   diagnóstico  diferencial  de,  342
intervención  mecánica  en,  385 Disección  de  cuello,  343  en  
Tamaño/área   cáncer  de  tiroides  metastásico,  294
normal  de  la  válvula  mitral  de,   Colapso  de  

384  Movimiento  anterior  sistólico  (SAM)  de,  389 las  venas  del  cuello,  
Reparación  de  válvula  mitral,  como  insuficiencia  mitral 96  distendido,  96,  119,  122
tratamiento,  388 Necrosis  

Reemplazo  de  válvula  mitral   relacionada  con  

como  tratamiento  de  insuficiencia  mitral,  388  como   quemaduras,  174  relacionada  con  el  síndrome  del  martillo  
tratamiento  de  estenosis  mitral,  386 hipotenar,  163  pancreática,  199,  200
Dolor  medio,  74 Lesiones  por  pinchazo  de  aguja,  89

Lunares  (nevos)   Dificultad  respiratoria  neonatal,  relacionada  con  hernia  diafragmática  
biopsia  de,  329   congénita,  426
escisión  de,  329  tipos   Hernia  

de,  328 diafragmática  congénita  neonatal  en,  426–428  anomalías  

MOTT  (micobacterias  con  bacilos  distintos  de  la  tuberculosis),  399–400 asociadas  con,  426  complicación  más  temida  de,  427

Carcinoma  mucoepidermoide,  336,  338,  339 obstrucción  intestinal  en,  417–420  Neoplasia.  
Mucosa,  como  barrera  frente  a  infecciones  microbianas,  77 Véase  también  Cáncer;  Definición  de  tumores,  
Cavidades  mucosas,  esterilización  preoperatoria  de,  78–79 323  Nefrectomía  nativa,  436  conservadora  de  

nefronas,  462  como  tratamiento  del  carcinoma  de  

Síndromes  de  neoplasia  endocrina  múltiple  (MEN),  222,  283,   células  renales,  462
286,  292,  304

Fallo  multiorgánico  
relacionado  con  shock  hemorrágico/hipovolémico,  97–98  como   Nefroblastoma.  Ver  Tumor  de  Wilms  Nefroma,  
causa  de  mortalidad,  77  postlesión,  en  niños,  185 mesoblástico  congénito,  474  Nefrostolitotomía,  
percutánea,  459  Neuroblastoma,  429–430  diferenciado  
Esclerosis  múltiple,  disfunción  vesical  asociada  a,  471 del  tumor  de  Wilms,  429,  429t

Complejo  Mycobacterium  avium  (MAC),  399 Tumores  neuroendocrinos,  pancreáticos,  194

Mycobacterium  con  bacilos  distintos  de  los  tuberculosos  (MOTT),  399–400 Bloqueadores  neuromusculares,  44
Nitratos,  como  tratamiento  de  la  angina,  379
Micosis  fungoide,  328 Óxido  nítrico,  37
mielomeningocele,  474 Nitrógeno,  orina  total  (TUN),  55
Mielopatía  transversa  completa,  107 Balance  de  nitrógeno,  59
Infarto  de  miocardio,  aspirina  como  profilaxis  contra, Nitroprusiato,  como  tratamiento  del  aneurisma  disecante  de  
357 aorta,  411
Preservación  miocárdica,  hipotérmica,  441 Nitrosaminas,  como  factor  de  riesgo  de  cáncer  de  esófago,  218
Machine Translated by Google
508  ÍNDICE

Nódulos Enfermedad  de  Paget,  extramamaria,  328
pulmonar  solitario,  407–409  maligno,   Dolor  

407,  408  como  enfermedad   abdominal  

metastásica,  407,  408,  409  resección  de,  408   agudo  relacionado  con  el  abdomen,  73–
tiroides,  292–297  diagnóstico  diferencial  de,  293   74  relacionado  con  la  apendicitis,  187  
múltiple,  292  solitario,  292 relacionado  con  la  pancreatitis  crónica,  203  
ubicación  y  duración  de,  73–74  relacionado  
con  la  obstrucción  del  intestino  delgado,  230,  232
relacionado  con  oclusión  arterial,  158  

NOMI  (isquemia  mesentérica  no  oclusiva),  237 epigástrico,  222  relacionado  con  
No  maleficencia,  479. pancreatitis,  199  cuadrante  inferior  
Contraindicación  de  antiinflamatorios  no   derecho,  187,  188
esteroideos  en  pacientes  con  úlcera  duodenal,  222–223  como  factor   Terapia  paliativa,  para  pacientes  con  cáncer,  326  para  
de  riesgo  de  enfermedad  ulcerosa  duodenal,  222 pacientes  con  cáncer  de  pulmón,  404  para  pacientes  
Norepinefrina,  como  tratamiento  de  choque,  31,  31t con  cáncer  de  páncreas,  196,  197
Billetes,  quirúrgico,  10,  10t Palidez,  relacionada  con  la  oclusión  arterial,  158
Enfermeras,  9–10 Páncreas

Evaluación  nutricional,  53–56 contenido  de  electrolitos  de,  50t  
Soporte  nutricional  para   lesiones  de,  140  trauma  cerrado,  
quemados,  178  para   122  complicaciones  de,  140–

pacientes  obesos,  57 141  traumatismo  penetrante,  140  
Historia  de  la  nutrición,  53 tratamiento  quirúrgico  de,  140,  142

O Páncreas  dividido,  199,  203

Pacientes  obesos   Trasplante  de  páncreas,  437–438  simultáneo  
heridas  por  arma  de  fuego  abdominales  en,  125   con  trasplante  de  riñón,  437
apoyo  nutricional  para,  57  cálculos  nutricionales   Pancreatectomía  
para,  56t,  59 distal,  196  

Obesidad   proximal,  195  
como  componente  del  síndrome  metabólico,  277   total,  196

morbilidad,  tratamiento  quirúrgico  bariátrico  de,  277–280 Cáncer  de  páncreas,  194–198

Obstilación,  relacionada  con  obstrucción  del  intestino  delgado,  230 diagnóstico  diferencial  de,  194  tipos  
Pacientes  obnubilados,  100 histológicos  de,  194  imágenes  de,  195  
Signo  del  obturador,  187–188 inoperable,  196  metastásico,  194,  196  
Octreotide,  como  tratamiento  del  sangrado  de  varices,  209 terapia  paliativa  para,  196,  197  tasas  
síndrome  de  Ogilvie,  246,  471 de  supervivencia  en,  194

OKT3,  complicaciones  de,  441
''Aceitunas'',  415

Ácidos  grasos  omega­6,  como  componente  de  la  fórmula  enteral, Insuficiencia  pancreática,  203
57,  60 Pancreaticoduodenectomía,  195
Omeprazol,  32,  214,  222–223 Pancreatitis  

Oncocitoma,  336,  337,  462 aguda,  199–202  

Oncogenes,  en  adenocarcinoma  de  colon,  260–261 tratamiento  quirúrgico  de,  200–201  
Cirugía  oncoplástica,  por  cáncer  de  mama,  318 crónica,  203–205  como  factor  de  riesgo  de  

100  secretos  principales,  1–8 enfermedad  de  úlcera  duodenal,  222  tratamiento  médico  
Estudios  de  contraste  oral,  de  obstrucción  del  intestino  delgado,  231 de,  204  relación  con  pancreatitis  aguda,  203  tratamiento  

donación  de  órganos,  481 quirúrgico  de,  204  relacionado  con  hiperparatiroidismo,  
Osteoporosis,  relacionada  con  hiperparatiroidismo,  281 281  idiopático,  199  necrosante,  199,  200
Ostomía  (estoma),  colocación  de,  249
Cálculos  renales  de  oxalato,  460
Oxigenación,  tórax  mayal  y,  124
Consumo  de  oxígeno,  34 diferenciada  de  pancreatitis  aguda,  200  nutrición  
Curva  de  disociación  de  oxihemoglobina,  47 parenteral  en,  60  inducida  por  trauma,  140

PAG
Síndromes  paraneoplásicos,  de  cáncer  de  pulmón,  403
Transfusiones  de  glóbulos  rojos  concentrados,  en  niños,  182 Complicaciones  de  
paratiroidectomía,  284  
Taponamiento   primera  exitosa,  286  

pélvico,  148  perihepático,  133 focalizada,  283
Machine Translated by Google
ÍNDICE  509

Indicaciones  de  paratiroidectomía   Úlceras  pépticas  (Continuación)  
(continuación)  para,  282,  285   antecedentes  del  paciente  de,  225  
resultado  de,  284,  285  exploraciones   perforado,  222  resangrado  en,  226  
con  sestamibi  antes  de,  283–284  subtotal,   como  causa  de  hemorragia  digestiva  
286  total,  con  autotrasplante,  286 alta,  204
Solución  de  conservación  Perfadex,  451
Glándulas  paratiroides,  embriología  y  anatomía  de,  282 Miembro  

Hormona  paratiroidea,  en  hiperparatiroidismo,  282, aislado  de  perfusión,  como  tratamiento  de  melanoma,  
285 334  de  órganos,  30
nutrición  parenteral,  60   Pericarditis,  398
complicaciones  de,  61   Periodonitis,  como  factor  de  riesgo  de  enfermedad  arterial  
costo  de,  62  definición  de,   coronaria,  348
60  seguimiento  de,  61 Arteriografía  de  enfermedad  oclusiva  arterial  
periférica  de,  352  tratamiento  de  injerto  de  
Parotidectomía,  337 derivación  de,  351  conductos  autógenos  
Relación   en,  352  falla/oclusión  del  injerto  en,  
anatómica  de  la  glándula  parótida  con  el  nervio  facial,   351,  353  flujo  de  entrada  y  salida  en,  352  
336  quistes  de,  340  ubicación  y  características  de,  336 claudicación  asociada  con,  350  isquemia  
crítica  de  miembros  asociada  con,  350,  351  
Tumores  de  parótida,  336–341,  342   diabetes  mellitus  351  pruebas  diagnósticas  no  invasivas  
quimioterapia  para,  340  clasificación   para,  374,  375  mortalidad  y  morbilidad  perioperatoria  en,  353  
de,  338–339  tinción   presión  segmentaria  de  las  extremidades  en,  351  en  
inmunohistoquímica  de,  340,  341  secciones  congeladas   pacientes  jóvenes,  351
intraoperatorias  de,  340  manejo  de,  339  de  origen  de  
unidad  de  glándula  salival,  336–337
Lavado  peritoneal  
Pars  interarticularis,  fracturas  de,  107 para  diagnóstico  de  hemoperitoneo,  130  en  
Presión  parcial  de  oxígeno  (PaO2),  46,  47  en  apnea,   pacientes  con  fractura  pélvica,  149
48 Peritonitis,  125
Presión  máxima,  43 Arterias  peroneas,  lesiones  a,  162
Enfermedad  pélvica  inflamatoria,  188   Circulación  fetal  persistente  (PFC),  427
como  mimetismo  de  apendicitis,  189 estado  vegetativo  persistente,  481;
Síndrome  de  Peutz­Jeghers,  260,  262  Gen  
Fracturas  de  pelvis  de,  147– supresor  de  tumores  p53,  465–466
150  lesiones  de  la  vejiga  asociadas  con,  156   Feocromocitoma,  292,  298,  299,  300,  302,  304,  305  ''regla  
pérdida  de  sangre  asociada  con,  147,  148,  149 de  10''  de,  304
osteomielitis  de,  145
Lesiones  penetrantes   Flemasia  alba,  368
abdominal,  125–129   Dolorosa  flegmasia  cerulea,  368
cervical,  110–113  en   Relación  médico­paciente,  9
niños,  180  colónica,  144,   Médicos,  carácter  independiente  de,  476;
145  esofágica,  120   Enfermedad  del  seno  pilonidal,  267
hepática,  130  pancreática,   Plasma

140  renal,  151  torácica,   contenido  de  electrolitos  de,  50t  
118–121  traqueobronquial,   capacidad  de  unión  de  oxígeno  de,  47
120 Músculo  platisma,  lesiones  traumáticas  a,  110,  112
Plavix,  379
Derrame  pleural,  397  
Penicilina definición  de,  397  
resistencia  bacteriana  a,  80  como   determinación  de  causa  de,  397  
"droga  maravillosa",  79–80 maligno,  405  manejo  de,  397

Lesiones  por  amputación  de  pene  a,   Espacio  pleural,  entrada  de  sangre,  118,  120
157  fracturas  de,  156,  157 sínfisis  pleural,  397
pentoxifilina,  350 Pleurodesis,  397
Úlceras  pépticas,  221–229   Síndrome  de  Plummer­Vinson,  218
Hipersecreción  de  ácido  gástrico,  221 Prendas  neumáticas  antichoque  (PASGs),  148
Relacionado  con  Helicobacter  pylori,   Neumonía,  42,  122,  450
221  relacionado  con  hiperparatiroidismo,  281 Neumoperitoneo,  126
Machine Translated by Google
510  ÍNDICE

Neumotórax,  65  en   Proctocolectomía,  249,  250,  251
niños,  182  abierto,   Proctosigmoidoscopia,  rígida,  255
92–93,  118  relacionado   Profesiones/profesionalismo,  483–486
con  trauma  abdominal  penetrante,  126  tensión,  21,  23,   códigos  de  conducta  profesional  para,  483–484  elementos  
92–93,  114 centrales  de,  483  definición  de,  483
Poiquilotermia,  relacionada  con  la  oclusión  arterial,  158
Polipectomía,  endoscópica,  259 Propiltiouracilo,  289,  290
Pólipos  colorrectales,  258–261   Cáncer  de  próstata,  467–468
adenomatosos,  262,  263  como   metastásica  avanzada,  468  como  
factor  de  riesgo  de  cáncer  colorrectal,  262   enfermedad  benigna,  467  
definición  de,  258  hamartomatosos  (síndrome   clínicamente  localizada,  468  
de  Peutz­Jeghers),  260, como  causa  de  mortalidad,  467  
262   factores  de  riesgo  de,  467  
juvenil,  259  con   tamizaje,  467
potencial  maligno,  258,  259,  260–261  detección  de,  259,   Antígeno  prostático  específico  (PSA),  467  
260  tipos  de,  258 libres,  467
Proteína,  cantidad  
Aneurisma  de  la   diaria  recomendada  en  la  dieta  (RDI)  de,  55  restricción  
arteria  poplítea  de,   en  insuficiencia  hepática  o  renal,  55
361  lesiones  de,  162 Proteína  C,  recombinante  humana  activada,  85
Vena  poplítea,  lesiones  a,  161 Inhibidores  de  la  bomba  de  
Vena  porta,  131 protones  como  tratamiento  de  la  úlcera  duodenal,  

Sistema  venoso  porta,  207 222–223  como  tratamiento  de  la  enfermedad  por  reflujo  

Presión  positiva  al  final  de  la  espiración  (PEEP),  37,  41,  42 gastrointestinal,  214,  216

automático/intrínseco  (PEEPi),  41–42 Pseudoaneurisma,  160f  
Tomografía  por  emisión  de  positrones  (PET),  318–319,  407,   relacionado  con  lesión  arterial,  158,  166  
408 infección  relacionada  con  lesión  arterial,  163  
Síndrome  posconmocional,  102 definición  de,  159  iatrogénica,  376  ruptura  de,  
Síndrome  postflebítico/postrombótico,  368,  369 134
Potasio  en  

las  secreciones  corporales,   Pseudoquistes  pancreáticos,  201,  203
50  t  de  líquido  intravenoso,  50  t Enganche  del  psoas,  153–154

Suplemento  de  potasio,  posoperatorio,  51 Signo  de  psoas,  74,  187–188
Bolsaitis  250 Presión  capilar  pulmonar  en  cuña  (PCWP),  35–36
Poupart,  François,  269 Trastornos  pulmonares,  397–400  
ligamento  de  Poupart,  269 empiema,  398  infecciones,  398  
Prealbúmina,  53–55,  54t derrame  pleural,  397  nódulos  
Organizaciones  de  proveedores  preferidos  (PPO),  476–477 pulmonares  solitarios,  407–409  
Embarazo   tuberculosis,  398
tratamiento  de  estenosis  aórtica  en,  395  
cáncer  de  mama  durante,  315  ectópico,   Insuficiencia  pulmonar,  34–38  definición  
188,  189  pielografía  intravenosa  durante,   de,  34
458  colecistectomía  laparoscópica  durante,  192 Legumbres

Corrigan,  393  
Contracciones  ventriculares  prematuras  (PVC),  aisladas, lesiones  arteriales  distales  en  las  extremidades,  162
25–26 Quincke's,  393  
Despolarizaciones  ventriculares  prematuras  (PVD),  25   martillo  de  agua,  393
Reparación  de  hernia  preperitoneal,  272–273,  273f,  274,  276  Preps.   Déficit  de  pulso,  relacionado  con  oclusión  arterial,  158
Ver  preparaciones  intestinales  (descontaminación) Actividad  eléctrica  sin  pulso  (PEA),  20,  21
Soluciones  de  preservación,  para  trasplantes  de  corazón  y  pulmón,   Volumen  de  registro  de  pulso  (PRV),  374
451,  452  ''Primacía  del  bienestar  del  paciente'',  484  Disfunción   Pulso  pequeño  y  lento,  391
primaria  del  injerto  (DGP),  449  Encuesta  primaria,  91  ''Principio  de   Estado  pupilar,  en  pacientes  con  daño  cerebral,  99,  100
autonomía  del  paciente'',  484  ''Principio  de  justicia  social ,''  484   poner,  83
Maniobra  de  Pringle,  133  Probióticos,  59  Processus  vaginalis,  abierto   Pielografía,  intravenosa,  458  de  
persistente,  269 inyección  única,  151
Estenosis  pilórica,  226  
hipertrófica,  415–416  
definición  de,  415  
perforada,  416
Machine Translated by Google
ÍNDICE  511

Tumores  pilóricos,  415 Insuficiencia  renal  (Continuación)  
Piloromiotomía,  Fredet­Ramstedt,  415–416 alimentación  enteral  en,  57  
piloroplastia restricción  proteica  en,  55  
finney,  225 Enfermedad  del  parénquima  renal,  298  
Heinecke­Mikulicz,  225 Lesiones  del  pedículo  renal,  151,  152  
Jaboulay,  225 Cálculos  renales.  Ver  Enfermedad  del  cálculo  urinario  
Pirazinamida,  como  tratamiento  de  la  tuberculosis,  399 Sistema  renina­angiotensina­aldosterona  (SRAA),
298
q Control  respiratorio,  46,  48  durante  
Cuadrantectomía,  316–317 la  reanimación  cardiopulmonar,  20
Futilidad  cualitativa,  480; impulso  respiratorio,  48
Insuficiencia  respiratoria,  34  
R aguda,  ventilación  mecánica  en,  41
''Ojos  de  mapache'',  100 Cociente  respiratorio  (RQ),  56  
Arteria  radial,  lesiones  a,  162 Frecuencia  respiratoria,  20,  46,  48  
Exposición  a  la  radiación,  como  factor  de  riesgo  de  nódulo  tiroideo,  292 Gasto  energético  en  reposo  (REE),  56,  56t  Reanimación.  
Radioterapia,  para  cáncer,  325  adyuvante,   Véase  también  Reanimación  cardiopulmonar  (RCP)  en  
325  efectos  adversos  de,  325  para  cáncer   niños,  181  Revascularización.  Ver  también  
de  mama,  319–320  para  cáncer  de   Injertos  de  derivación;  Injerto  de  derivación  de  arteria  
pulmón,  404  en  pacientes  con  mastectomía,   coronaria  (CABG)
318  para  melanoma,  333  neoadyuvante,  
325 con  todos  los  conductos  arteriales,  382  

con  injertos  de  derivación  autógenos  de  las  extremidades  inferiores,
374
Yodo  radiactivo infrainguinal,  ecografía  dúplex  de,  374–375  como  
como  tratamiento  del  hipertiroidismo,  288,  289  como   tratamiento  de  isquemia  mesentérica,  238  transmiocárdica,  
tratamiento  del  cáncer  de  tiroides,  295  como   382–383
tratamiento  del  bocio  nodular  tóxico,  290 Índice  de  riesgo  cardíaco  revisado  (RCCI),  23
Exploraciones  de  captación  de  yodo  radiactivo  (RAIU),  288,  292 Fiebre  reumática,  como  causa  de  estenosis  mitral,  384;
Índices  de  Ranson,  para  la  predicción  de  la  gravedad  de  la  pancreatitis,  200 Costillas,  fracturas  de,  92–93,  122,  123,  130  en  niños,  
Sensibilidad  de  rebote,  74 182  como  causa  de  pérdida  de  sangre  oculta,  97
Cáncer  de  recto,  262
Sistema  de  estadificación  de  Dukes  para,   Rifampicina,  como  tratamiento  de  la  tuberculosis,  399;
263  terapia  posoperatoria  para,  263 Sensibilidad  en  el  cuadrante  inferior  derecho,  244
Examen  rectal  digital   solución  de  Ringer  lactato,  49
para  detección  de   como  líquido  de  reanimación,  50t,  93,  96
cáncer  colorrectal,  262  para  detección  de   en  niños,  182  Riolan,  
cáncer  de  próstata,  467  para  evaluación  de   Jean,  235  Rockey,  AE,  188  
obstrucción  del  intestino  delgado,  230  para  evaluación  de   Incisión  de  Rockey­Davis,  
lesión  de  la  médula  espinal,  105 187  Procedimiento  de  Ross,  392,  
Bien 395  Rotaciones,  quirúrgicas,  9–18  
hemorragia  de,  248,  255,  262,  265  lesiones   Lectura  requerida  para,  14
traumáticas  de,  145  extraperitoneal,  145  
intraperitoneal,  145 Signo  de  Rovsing,  187–188
''Regla  de  10'',  304
Transfusiones  de  glóbulos  rojos,  31,  97 ''Regla  de  los  nueves'',  175
Síndrome  de  realimentación,  61
Fracción  regurgitante,  387 S

Reitz,  Bruce,  439 Solución  salina,  hipertónica,  
Carcinoma  de  células  renales,  462–463 49  como  líquido  de  reanimación,  49,  50t,  93,  96
etiología  de,  462   Saliva,  contenido  de  electrolitos  de,  50t
metastásico,  463   Glándulas  salivales  
signos  y  síntomas  de,  462  remisión   fístulas  de,  339–340  
espontánea  de,  326 tumores  de,  337,  342
Insuficiencia  renal Unidad  de  glándulas  salivales,  336–337
isquemia   sal,  49
crítica  terminal  de  extremidades  asociada  con,  351   Sarcoma  

como  indicación  de  trasplante  renal,  436  paratiroidectomía   definición  de,  323
en,  285 de  Kaposi,  328
Machine Translated by Google
512  ÍNDICE

Sarcoma  (continuación)   Shumway,  normando,  439
metastásico,  325  

pancreático,  194  renal,   Derivaciones  relacionadas  con  el  síndrome  de  dificultad  respiratoria  del  

462 adulto,  37  relacionadas  con  el  desajuste  de  la  ventilación  alveolar/flujo  sanguíneo,

Cuero  cabelludo,  laceraciones  hasta,  100 34  

Cicatrices,  faciales,  166,  167 quirúrgica  
Esquistosomiasis,  206,  258,  465 no  selectiva  y  selectiva,  211  portosistémica  

como  causa  de  hipertensión  portal,  206 intrahepática  transyugular  (TIPS),  211,  211f

SCIWORA  (lesión  de  la  médula  espinal  sin  anormalidades  

radiológicas),  106,  184 ''Signo  de  seda''  de  las  hernias  inguinales  pediátricas,  
Esclerodermia,  213 269–270

Traumatismo   ritmo  sinusal,  25
contuso  del  escroto  en,  156   Piel

Pérdida  de  piel  por,  157 en  hipovolemia,  95  como  

Lesiones  por  cinturón  de  seguridad,  144,  183 barrera  contra  infecciones  microbianas,  77  

Secretos,  Top  100,  1–8 esterilización  preoperatoria  de,  78–79

Presión  segmentaria  de  las  extremidades  (SLP),  351,  374 Cáncer  de  piel,  323

Convulsiones,  postraumáticas,  102 Injertos  de  piel/colgajos,  166,  178

Tubos  Sengstaken­Blakemore,  210,  210f,  254 Fracturas  de  cráneo,  100,  101

Bucles  centinela,  75 ''Código  lento'',  23

Ganglio(s)  linfático(s)  centinela,  definición  de,  324 Intestino  delgado

Disección  de  ganglio  linfático  centinela,  en  cáncer  de  mama,  316– lesiones  contundentes  a,  122  
317,  319 en  la  enfermedad  de  Crohn,  248  

Mapeo  de  ganglio  centinela,  de  cáncer  de  mama,  316,  318 entrada  de  gas,  75  obstrucción  
de,  230–234  asa  cerrada,  232  

Metástasis  de  ganglio  linfático  centinela,  de  melanoma,  332,  332t completa  frente  a  parcial,  231  

mecanismos  de,  230  recurrente  

Sepsis   múltiple,  233  prevención  de,  233  

como  causa  del  síndrome  de  dificultad  respiratoria  del  adulto,  35   simple,  232  estrangulado,  230,  
intraabdominal,  83 232  tratamiento  de,  232,  233  papel  

Proteínas  séricas,  evaluación  nutricional  de,  53–55, en  la  nutrición,  237  Resección  del  
54t intestino  delgado,  como  causa  del  

Gammagrafías  con  sestamibi,  para  la  localización  del  hiperparatiroidismo,   síndrome  del  intestino  corto,  62  
283–284 Fumar  como  causa  de  aterosclerosis,  

Setones,  266 345,  351  como  causa  de  enfermedad  dental,  348  como  causa  de  

Síndrome  del  niño  sacudido,  184 enfermedad  de  úlcera  duodenal,  222  como  causa  de  cáncer  

Shock,  30–33   de  esófago,  218  como  causa  de  cáncer  de  pulmón,  401,  407,  408  
relacionado  con  traumatismo  renal  cerrado,   como  causa  de  infección  del  sitio  quirúrgico,  70

151  cardiogénico,  30  definición  de,  97  post­
infarto,  445  tratamiento  de,  31,  31t

clasificación  clínica  de,  30,  96,  96t,  97  definición  de,  30  

hemorrágico/hipovolémico,  30,  95–98,  123,  382 historia  social,  53

Justicia  social,  484;
pérdida  de  volumen  de  sangre  en,   Sodio

96t  ''ciclo  vicioso  sangriento''  de,  123  en   en  secreciones  corporales,  50t  
niños,  181  definición  de,  95  como  causa  de   contenido  de  líquido  intravenoso  de,  50t  

insuficiencia  orgánica  múltiple,  97–98   milequivalentes  (mEq)  de,  49  contenido  de  

postraumática,  95–98  neurogénica,  definición  de,  97   orina,  posoperatorio,  51,  52  Suplementos  de  sodio,  

colapso  vascular  periférico,  30  relación  al  gasto   posoperatorio,  51  Lesiones  de  órganos  sólidos,  en  niños,  

cardíaco,  30  séptico,  definición  de,  97  espinal,  106,  470   183,  184  Nódulos  pulmonares  solitarios.  Ver  Nódulos  

tratamiento  de,  30–31,  31t,  32 pulmonares  solitarios  Sistema  nervioso  somático,  

papel  en  el  tracto  urinario  inferior

función,  470

Síndrome  de  intestino  corto,  62,  249 Espacio  de  Bogros,  274
Reparación  de  hernia  de  Shouldice,  271,  275 Espacio  de  Retzius,  274
Machine Translated by Google
ÍNDICE  513

Trasplante  de  células  madre,  443,  452
Criopreservación  de  esperma  de,   Stents  

456  uso  en  fertilización  in  vitro,  456   carótida,  359–360  

Hidrocele  del  cordón  espermático  de,  269   endoluminal,  245  para  

anillo  inguinal  interno  alrededor,  274   lesiones  ateroscleróticas  de  la  arteria  ilíaca,  354  ureteral,  

Lesiones  de  la  médula  espinal,  102,  105–109.   460

Véase  también  Columna  cervical,  lesiones  de;  Lesiones  de  la  columna   Tiras  Steri,  para  el  cierre  de  heridas  faciales,  165–166
torácica Esternotomía,  mediana,  118
diferenciado  de  lesiones  espinales,  105  evaluación   Esteroides  

de,  105  lesión  en  la  cabeza  asociada  con,  105  como   como  tratamiento  de  presión  intracraneal  
indicación  para  la  colocación  de  un  filtro  en  la  vena   elevada,  103  potencias  relativas  de,  

cava  inferior, 307
108   Estómago

nivel  de,  105   contenido  de  electrolitos  de,  50t  
resultado  en,  108   lesiones  a,  122,  140
evaluación  radiológica  de,  106   Estoma  (ostomía),  colocación  de,  249
tratamiento  de,  108 Heces,  guayaco  positivo,  262
Lesión  de  la  médula  espinal  sin  anormalidades  radiológicas Procedimiento  Stoppa,  272–273,  274

(SCIWORA),  106,  184 Estenosis,  ileal,  249

Lesiones  de  la   Carrera,  358,  394

columna  diferenciadas  de  las  lesiones  de  la  médula  espinal,   profilaxis  contra  aspirina,  357
105  Asociadas  a  lesiones  en  la  cabeza,  105  Resonancia   Volumen  sistólico,  en  niños,  181

magnética  de,  106 Cálculos  renales  de  estruvita,  459

Espirometría,  incentivo,  como  tratamiento  de  atelectasias,  65 Arteria  subclavia,  lesiones  a,  162
Nevus  de  Spitz,  328 Tumores  de  glándulas  sublinguales,  337
Tumores  de  las  glándulas  submandibulares,  337

Autotrasplante  de  bazo  de,  138  lesiones   Tumores  de  glándulas  submaxilares,  342
de,  136–139  contuso,  122  en  niños,   Sucralfato,  como  tratamiento  de  la  úlcera  duodenal,  222–223
184  clasificación  de,  136,  136t   Muerte  súbita  cardiaca,  19,  391
como  causa  de  ruptura  tardía,   Suicidio,  180
137  manejo  no  quirúrgico  de,  137,   Exploración  con  coloides  de  azufre,  255–256

138  manejo  quirúrgico  de,  137,  138  lesión   Nervio  laríngeo  superior,  lesión  relacionada  con  endarterectomía  
pancreática  asociada,  140  respuesta  a  lesiones  en,   carotídea,  358
130  papel  fisiológico  de,  136 Arteria  mesentérica  superior,  131,  235  vías  
colaterales  de,  235  embolia/oclusión  de,  235,  
236,  238

Superior  vena  cava  syndrome,  404  
Esplenectomía,  137,  138 Cirujanos,  Código  de  Conducta  Profesional  para,  483–484
Vena  esplénica,  trombosis  de,  204
Carcinoma  de  células  escamosas   Antecedentes  quirúrgicos,  como  componente  de  valoración  
de  vejiga,  464,  465  esofágico,   nutricional,  53
218 Lesiones  quirúrgicas,  traumáticas,  101
Heridas  de  arma  blanca Notas  quirúrgicas,  10,  10t
al  abdomen,  125,  127  como  causa   Rotaciones  quirúrgicas,  9–18
de  taponamiento  cardíaco,  119  al  tórax,  118   lectura  obligatoria  para,  14
al  colon,  144  a  los  riñones,  130  al  cuello,  110,   Infecciones  del  sitio  quirúrgico,  65,  66,  68–72  
110t  Staphylococcus  aureus,  resistente  a  la   incisión  profunda,  68,  71  espacio  de  órganos,  68  
meticilina,  80  Infecciones  por  Staphylococcus   prevención  de,  69,  70  incisión  superficial,  68,  71  
aureus,  como  causa  de  empiema ,  398  Ley   tratamiento  de,  66,  67,  71  no  tratado,  71
de  Starling,  35  Starzl,  Thomas,  433  Presión  estática/meseta,  43  
Estatinas.  Ver  inhibidores  de  la  3­hidroxi­3­metilglutaril  coenzima  
A  (HMG­CoA)  reductasa  Síndrome  de  Stauffer,  463  

Esteatorrea,  203 Ventana  pericárdica  suxifoidea,  119
Sudor  

contenido  de  electrolitos  de,  50t  
contenido  de  sodio  de,  51

Glándulas  sudoríparas,  respuesta  a  la  aldosterona,  51
"Picazón  del  nadador",  206

Síndrome  X  (síndrome  metabólico),  277,  346–347
Machine Translated by Google
514  ÍNDICE

suministro  de  oxígeno  sistémico,  47 Trombosis  (Continuación)  vena  
Sistema  venoso  sistémico,  207 esplénica,  207  vascular,  
Descarga  sistólica,  33 importancia  de,  348–349  relacionada  con  la  
reparación  venosa,  161  Tromboxano  A2,  348  
T Quistes  del  conducto  tirogloso,  431–432  Arterias  
Taquicardia   tiroideas,  294–295  Cáncer  de  tiroides,  292–297  
relacionada  con  pérdida  de  sangre,  en  niños,   anaplásico,  293,  294  folicular,  293  medular,  292,  
181  definición  de,  28  relacionada  con   293,  294  metastásico,  294,  296,  342,  343  
hipovolemia,  95  relacionada  con  estenosis   papilar,  292,  293,  294  bien  diferenciado,  294,  
mitral,  385  ventricular  sin  pulso,  20–21   296  Tiroidectomía,  294  complicaciones  de,  
relacionada  con  shock,  96t  relacionada  con   295  como  tratamiento  del  hipertiroidismo,  
obstrucción  del  intestino  delgado,  232 289,  290,  294  como  tratamiento  del  cáncer  
de  tiroides,  294  como  tratamiento  del  nódulo  
Taquipnea,  relacionada  con  el  abdomen  agudo,  74 tiroideo,  294  como  tratamiento  del  bocio  
Exploraciones  de  glóbulos  rojos  etiquetados,  255–256 nodular  tóxico,  290  Glándula  tiroides,  suministro  
Tamoxifeno,  como  profilaxis  del  cáncer  de  mama,  317,  319,  320–321 de  sangre  a,  294–295  Nódulos  tiroideos.  Ver  
Nódulos,  tiroides  Hormona  estimulante  de  la  tiroides  
Taponamiento  cardíaco,  21,  23,  92–93,  119 supresión  posterior  a  la  tiroidectomía,  296  como  indicador  
Terlipresina,  como  tratamiento  del  sangrado  variceal,  209 de  nódulo  tiroideo,  292  Prueba  de  hormona  estimulante  de  
Testículos,  no  descendidos  (criptorquidia),  457,  473 la  tiroides,  288,  295  Tormenta  tiroidea,  290  Cirugía  de  
Rotura  testicular,  156 tiroides,  293,  294  Venas  tiroideas,  294–295  Arterias  tibiales,  
Testosterona,  como  tratamiento  de  la  infertilidad  masculina,  457 lesiones  to,  162  Top  100  Secrets,  1–8  Nutrición  parenteral  total,  
Tetania,  284,  285 58,  62  Nitrógeno  urinario  total  (TUN),  55  Torniquetes,  160,  169  
Tercer  espacio,  52 Síndrome  de  shock  tóxico,  86  Toxinas  como  causa  de  paro  
Toracocentesis,  para  manejo  de  derrame  pleural,  397 cardíaco,  21  Desintoxicación  microbiana  de,  77  
Lesiones  de  la  columna  torácica,  imitación  de,  105 Toxoplasmosis ,  86  Lesiones  traqueobronquiales,  penetrantes,  
Cirugía  torácica,  por  enfermedad  no  neoplásica,  397–400 120  Malformaciones  traqueoesofágicas,  423–425.  Véase  
Cirugía  toracoscópica,  videoasistida,  398 también  Atresia  esofágica;  Fístulas  traqueosofágicas  
Toracoscopia,  127 Transferrina,  53–55,  54t  Ataques  isquémicos  transitorios  (AIT),  
Profilaxis  antibiótica   47–48,  356,  357  ''Respondedores  transitorios'',  97  Carcinoma  
de  toracostomía  con,  120  tubo,  397  en   de  células  de  transición  de  vejiga,  464  Derivaciones  
heridas  penetrantes  de  tórax,  118 portosistémicas  intrahepáticas  transyugulares  (TIPS),  211 ,  
211f,  434  Resección  transuretral
Toracotomía  en  
lesiones  torácicas  cerradas,  115  
''concha  de  almeja'',  118  abierta,  de  
fístulas  broncopleurales,  397  en  lesiones  
abdominales  penetrantes,  126  en  lesiones  
torácicas  penetrantes,  118,  122  tasa  de  infección  
de  herida  quirúrgica  de,  68
Terapia  trombolítica,  indicaciones  para,  354
Trombofilia,  366
Tromboflebitis  séptica,  67
Trombosis  

arterial  relacionada  con  lesiones,  158,  
163  relacionada  con  placas  ateroscleróticas,  
356  como  causa  de  paro  cardíaco,  21  venosa  
profunda,  366  factores  de  riesgo  clínicos  para,  
366
Análisis  de  sangre  con  dímero  D  para,   de  tumores  de  vejiga  (TURBT),  464  de  
376  diagnóstico  de,  366  pruebas   próstata  (TURP),  467
diagnósticas  no  invasivas  para,  371,  373  profilaxis   Trauma.  Véase  también  Trauma  cerrado;  Trauma  penetrante  trauma  
perioperatoria  contra,  367  síndrome  postflebítico/ craneoencefálico,  99–104  como  causa  de  paro  cardíaco,  21  
postrombótico  de,  368, colónico,  144  ''hora  dorada''  en,  91  hepática  y  biliar,  130–135  
369   evaluación  inicial  de,  91–94  como  causa  de  mortalidad,  123
relacionados  con  lesión  cerebral  traumática,  103
diferenciado  de  embolia,  354  compromiso  de  
plaquetas  en,  348–349  vena  porta,  206
Machine Translated by Google
ÍNDICE  515

Trauma  (continuación)   Ultrasonografía  (continuación)  de  
pediátrico,  180–186   trombosis  venosa  profunda,  366,  373  definición  
rectal,  145  esplénico,   de,  372  desventajas  de,  376  de  pseudoaneurismas  
136–139 iatrogénicos,  376  de  revascularización  
Lesión  cerebral  traumática,  99–104   infrainguinal,  374–375  de  enfermedad  vascular  
relacionada  con  maltrato  infantil,  102   periférica,  375  de  terapia  vascular,  374
como  factor  de  riesgo  de  trombosis  venosa  profunda,  103  
coagulopatía  intravascular  diseminada  asociada  con,  102  relacionada  
con  herida  de  bala,  102  evaluación  inicial  de,  99  como  causa   FAST  (sonografía  abdominal  enfocada  para  trauma),  91,  93  
de  morbilidad  y  mortalidad,  99  fases  de,  102 hepática,  130  intraoperatoria  laparoscópica,  192  de  cáncer  
de  páncreas,  195  de  vejiga  renal,  en  niños,  472  esplénica,  136  de  
tiroides,  292,  294  de  enfermedad  de  cálculos  urinarios,  458  de  
enfermedad  vascular,  371
Pacientes  con  lesión  cerebral  traumática,  alta  del  servicio  de  
urgencias,  102
''Prueba  de  curación'',  227
Trisomía  18,  423
Signo  de  Trousseau,  284
Tuberculosis,  342,  398 Red  Unida  para  el  Intercambio  de  Órganos  (UNOS),  440
multirresistente,  399 Precauciones  de  barrera  universales,  89
Sistema  de  estadificación  tumor,  ganglio,  metástasis  (TNM)   Criterios  de  la  Universidad  de  Washington,  para  estenosis  de  la  
para  cáncer  de  mama,  315  para  melanoma,  331,  331t arteria  carótida,  372t
Solución  de  preservación  de  la  Universidad  de  Wisconsin,  451
Linfocitos  infiltrantes  de  tumores  (TIL),  326 Carcinoma  de  uraco,  464
Factor  de  necrosis  tumoral,  en  trasplante  cardiaco,  443 Piedras  ureterales,  459
tumores ureterocele,  473
abdominal,  429–430  en   Unión  ureteropélvica,  obstrucción  de,  472
niños,  429–430,  429t  sólido,  324   Ureterorrenoscopia,  459
metastásico,  325  tratamiento  de,  325 Ureteroscopia,  459
Uréteres  

duplicados,  473  
síndrome  de  Turcot,  260 ectópicos,  473  
lesiones,  153  
reparación  quirúrgica  de,  153  
Úlceras  U.  Ver  también  úlceras  duodenales;  Úlceras  gástricas;   sin  diagnosticar,  153  
Úlceras  pépticas  aftosas,  248  Curling,  228  Cushing,  228   reimplantación  de,  con  enganche  de  psoas,  153–154
Dieulafoy,  228  marginal,  228  Arteria  cubital,  lesiones  de,  162   Uretra,  lesiones  en,  156  con  
Ultrasonografía  abdominal,  75  de  aneurismas  aórticos   fractura  pélvica  concomitante,  156  lesión  
abdominales,  361  de  apendicitis,  188  de  trauma  cerrado,   relacionada  con  fractura  pélvica,  149
122–123  en  niños,  183 Válvulas  uretrales,  posteriores,  473,  474
Cálculos  renales  de  ácido  úrico,  459,  460
Análisis  de  
orina  en  evaluación  de  abdomen  agudo,  
75  en  evaluación  de  obstrucción  de  intestino  delgado,  231
Enfermedad  de  cálculos  urinarios,  458–461  
relacionada  con  hiperparatiroidismo,  281  
imágenes  de,  75
Retención  urinaria,  relacionada  con  cirugía  abdominal/pélvica,
cabecera,  de  trauma  torácico,  119  biliar,   471

195  de  masas  mamarias,  310,  312,  315 Tracto  urinario  
inferior,  469  
Doppler,  372  de   funciones  de,  469–470  
enfermedad  de  la  arteria  carótida,  373   síntomas  de,  469  superior,  
de  enfermedad  oclusiva  arterial  periférica,  374  de   lesiones  de,  151–154
incompetencia  venosa,  373  dúplex  de  aneurisma  aórtico   Diagnóstico  de  infecciones  
abdominal,  376–377  de  enfermedad  de  la  arteria  carótida,   del  tracto  urinario,  67  
357,  371,  372–373  de  enfermedad  cerebrovascular,  371 febriles,  en  niños,  472
Relacionado  con  el  catéter  de  Foley,  67

Nitrógeno  ureico  urinario  (UUN),  55
Machine Translated by Google
516  ÍNDICE

Orina,  contenido  de  sodio  de,  posoperatorio,  51,  52 Vasopresina,  209,  257  Estado  
Citología  de  orina,  para  evaluación  de  lesiones  vesicales,  464 vegetativo,  persistente,  481  Injertos  de  
Extravasación  de  orina,  postraumática,  152 vena  conductos  autógenos  para,  352  para  
Producción  de  orina,  posoperatorio,  51 reparación  de  lesiones  vasculares  de  
Urodinámica,  469–471 las  extremidades,  161  Venas.  Véase  también  venas  
Uroflujometría,  469 profundas  específicas,  diferenciadas  de  venas  
Urografía,  tomografía  computarizada,  de  enfermedad  de  cálculos   superficiales,  373  ligadura  de,  161  vena  cava,  lesiones  
urinarios,  458 pancreáticas  asociadas  a  lesión,  140  lesiones  retrohepáticas,  
Lesiones  urológicas,  relacionadas  con  fractura  pélvica,  147 130,  133  enfermedad  venosa,  366–370  pruebas  diagnósticas  no  
Urología  pediátrica,  472–474 invasivas  para,  373  incompetencia  venosa,  diagnóstico  no  
Cáncer  urotelial,  concurrente  con  cáncer  de  vejiga,  465 invasivo  pruebas  para,

EN

asociación  VACTERL,  421,  423
Vagotomía   373

como  tratamiento  úlcera  duodenal,  223   Insuficiencia  venosa  crónica,  368  Ventilación.  
como  tratamiento  úlcera  gástrica,  225,  227   Ver  también  Ventilación  mecánica

como  tratamiento  estenosis  pilórica,  226   distribución  en  el  pulmón,  36–37  
selectiva,  224  troncal,  223,  224,  227 alteración  del  tórax  inestable  relacionada  con,  123
Desajuste  ventilación/perfusión,  como  causa  de  insuficiencia  
Valvulotomía   pulmonar,  34
aórtica,  392   Paquete  de  ventilador,  42
globo  aórtico,  395 Dispositivos  de  asistencia  ventricular  (VAD),  443,  445–447
Prótesis  valvulares,  como  recambios  valvulares  aórticos,  392, contraindicaciones  para,  445  
393 explantación  de,  447  clases  
Valvuloplastia,  balón,  como  tratamiento  de  la  estenosis  mitral,  385– generales  de,  446–447  indicaciones  
386 para,  445  izquierda  (LVAD),  445  
várices derecha  (RVAD),  446
sangrado,  253  
esofágico.  Ver  Várices  esofágicas  gástricas,   Arritmias  ventriculares,  26,  26f,  27,  28
204,  208  Varicocele,  como  causa  de  infertilidad   Fibrilación  ventricular,  19,  20–21  definición  
masculina,  456  Venas  varicosas,  369  Laboratorio  de   de,  28
diagnóstico  vascular  (VDL),  no  invasivo,  371–378   aleteo  ventricular,  definición  de,  28

para  enfermedad  cerebrovascular,  371  diferenciado  de  radiología   Lesiones  de  la  
diagnóstica  y arteria  vertebral  a,  
108  Disección  traumática  de,  102
Enfermedad  por  reflujo  vesicoureteral,  472
ecografía,  371  para   tríada  de  Virchow,  366
la  evaluación  de  la  enfermedad  oclusiva  arterial  periférica, Bypass  de  arteria  visceral,  como  tratamiento  de  isquemia  
374   mesentérica,  239
componente  terapéutico  de,  376  para   Capacidad  vital,  efecto  de  las  incisiones  quirúrgicas  en,  34,  65
evaluación  de  enfermedades  venosas,  373 Signos  vitales,  en  abdomen  agudo,  74
Lesiones  vasculares   Vitamina  C,  efectos  cardioprotectores  de,  347
aterosclerosis  como  respuesta  a,  346   Vitamina  E,  efectos  cardioprotectores  de,  347
contundente,  122  en  las  extremidades,  158– Vitamina  K,  síntesis  microbiana  de,  77
164  angiografía  de,  163–164  índices  de  presión   Disfunción  miccional,  469–471
arterial  (API)  en,  158,  159  en  extremidades   Infusión  de  volumen,  31

inferiores,  162  pulso  distal  palpable  en,  162  tratamiento   Volumen  de  lavado,  78–79
quirúrgico  de,  160–161  en  extremidad  superior,  162  oculto,   Indicadores  de  

158  asociado  a  lesiones  ortopédicas,  162 estado  de  volumen  
de,  51  en  tratamiento  de  choque,  30,  31
vólvulo

como  causa  de  obstrucción  intestinal,  230,  244  
cecal,  245  estudios  de  contraste  de,  417  definición  
Vasculopatía,  aloinjerto  cardíaco,  442–443 de,  245  intestino  medio,  con  malrotación,  418  
Vasectomía,  inversión  de,  455 sigmoide,  245  tratamiento  quirúrgico  de,  245
Vasoconstricción,  348
pulmonar  hipóxico,  36
Vasodilatación,  348
Machine Translated by Google
ÍNDICE  517

Proyectil   Heridas,  quirúrgicas.  Ver  también  Clasificación  de  infecciones  del  
de  vómito,  relacionado  con  estenosis  pilórica  hipertrófica,  415   sitio  quirúrgico  de,  69,  70  Índice  de  muñeca  braquial  (WBI),  158
relacionado  con  obstrucción  del  intestino  delgado,  230
síndrome  de  von  Hippel  Lindau,  304  factor  
de  von  Willebrand,  348  deficiencia  del  factor   X

de  von  Willebrand,  392
Radiografías  
EN abdominales  de  aneurismas  aórticos  
Warfarina,  como  tratamiento  de  la  estenosis  mitral,  385 abdominales,  361  series  de  ''cuatro  vías'',  231  
Isquemia  caliente,  en  riñones  no  perfundidos,  152 ''tres  vías  del  abdomen'',  75  de  lesiones  de  la  
Tumor  de  Warthin  (cistoadenoma  linfomatoso),  337, columna  cervical,  106,  107  niveles  hidroaéreos  en  
343 el  tórax,  397  de  aorta  estenosis,  391  de  aneurisma  

aórtico  disecante,  410  de  masas  pulmonares,  
Requerimientos  diarios  de  agua  para,   403–404  de  cáncer  de  mama  metastásico,  316  
58t  contenido  de  alimentación  enteral  de,  58 de  heridas  de  bala  miocárdicas,  120  de  lesiones  
Procedimiento  de  Whipple,  195,  196   abdominales  penetrantes,  126  para  detección  
Recuento  de  glóbulos  blancos,  en  abdomen  agudo,  75   de  sangre  en  el  espacio  pleural,  118  para  
''Gusano  blanco'',  188  Tumor  de  Wilms,  429–430,  429t,  474   evaluación  de  insuficiencia  respiratoria,  34  de  
diferenciado  de  neuroblastoma,  429,  429t  Retirada/retirada   pulmonar  solitaria  nódulos,  407  de  tuberculosis,  
de  apoyo  médico,  479  Dehiscencia  de  heridas,  71–72   399  lumbares,  de  aneurismas  de  aorta  abdominal,  
Cicatrización  de  heridas,  en  pacientes  con  obesidad  mórbida,   361
277  Infecciones  de  heridas,  65  clostridiales,  66  colónicas,  145  
definición  de,  65  no  clostridiales,  66  quirúrgicas.  Ver  Infecciones  
del  sitio  quirúrgico

CON

síndrome  de  Zollinger­Ellison,  223

También podría gustarte